Vous êtes sur la page 1sur 1000

Electrical

Dr. P.S. Bimbhra

Resultant flux
density wave

Max. bar
current

s Torque

Resultant flux
density wave

Rotor mmf
wave
Electrical Machinery
This thoroughly revised atid updated edition presents a rigorous and comprehensive treatment
of transformers and more common types of rotating electrical machines types. Each chapters
begins with rudimentary concepts and is so developed, that an average student can easily
comprehend i t The salient features of this book are:

❖ In-depth coverage of transformers, d.c. machines, 3-phase synchronous and induction


machines.
❖ Highlights that electrical machines operate on the same basic principles.
❖ Devotes a chapter on electromechanical-energy-conversion principles and another on
d.c./a.c. machine windings.
❖ Drive aspects and applications are discussed for each machine type.
❖ Clearity of presentation is enhanced by illustrative figures and examples selected from
questions-papersof important Universities, IAS, IES and GATE.
❖ Includes numerous problems, conceptual questions and objective-type questions (with
answers) to help the reader master the basic concepts.

This edition also includes: electrical machinery overview, energy efficiency and resent advances;
3-phase transformers; 3-phase induction generators; appendices on magnetic circuits, 3-phase
circuits and short-answer type questions.

All these features contribute towards making this book an ideal text for undergraduate students
of degree classes. Practising engineers, through self-study, will also find this volume useful to
them.

Dr. P.S. Bimbhra retired as Professor of Electrical and Electronics Engineering from Thapar
Institute of Engineering and Technology (Deemed University), Patiala-147004. He has over
40 years of teaching and research experience in Electrical Engineering. He has published several
papers in renowned journals and national and international conferences. Dr. Bimbhra is a fellow
of I.E. (India), a member of I.E. E. (U.K.) and a life member of I.S.T.E.

KHANNA PUBLISHERS
4575/15, Onkar House, Room No. 3-4
Darya Ganj, New Delhi-110002
Phone: 23243042 • Fax: 23243043
Contents
O v e r v ie w o f E l e c t r i c a l M a c h in e r y

T r a n s fo r m e r s 1-159
h
1 1 . Transformer Construction 2
12 Principle of Transformer Action 4
1 3 . Ideal Two-winding Transformer 5
1 .4 . Transformer Phasor Diagrams iO
1.4.1. Transform er phasor diagram at no load 10
1.4.2. Transform er phasor diagram under load 12
1.4.3. Leakage flux 14
1.5 . Rating of Transform ers 14
1.6 . Equivalent C ircuit of a Transform er 20
1.7 . Open-circuit an d Short-circuit Tests 29
1.8 . The Per U n it System 35
1.9 . Voltage Regulation of a T ransform er 40
1.10. Transform er Losses and Efficiency 49
1.10.1. Transform er losses 49
1.10.2. Transform er efficiency 50
1.10.3. S eparation of hysteresis and eddy current losses 63
1.11. Testing of Transform ers 66
1.12. Auto-transform ers 70
1.13. Parallel O peration of Single-phase Transform ers 84
1.14. Tap-Changers on Transform ers 100
1.14.1. No-Load (or off-load) tap changer 102
1.14.2. On-load tap-changer 102
1.15. Induction R egulators 106
1.15.1. Single-phase induction regulator 106
1.15.2. T hree-phase induction regulator 109
1.16. Transform er as a M agnetically Coupled Circuit 112
1.16.1. Co-efficient of coupling H7
1.16.2. M ethods of increasing the coefficient of coupling 119
1.17. Audio-Frequency T ransform ers 123
1-18. Pulse T ransform ers 128
1*19. Three-phase T ransform ers 12®
119.1. T hree-phase tran sfo rm er connections 1
1-20. Transform er Noise ^
136
J-21. Some W orked Exam ples
1-22. Sum m ary 144

2- E l e c t r o m e c h a n i c a l E n e r g y C o n v e r s io n P r i n c ip l e s 1 6 0 '222
, Principle of E nergy Conversion
■ ■ Singly Excited M agnetic System s
2'3- Reluctance M otor
(vii)

Scanned by Cam Scanner


'%V-, •
^6

(vih)

2.4. Doubly-excited M agnetic S ystem s


2.4.1. E lectrom agnetic and reluctance torques ^2
2.5. E lem en tary Synchronous M achines
2.6. Singly-excited Electric Field System s
2.7 Dynam ic E q uations ^
2.8. Some Workpd Exam ples ^

3. B a s i c C o n c e p t s o f R o t a t i n g E l e c t r i c a l M a c h in e s 233.359
3.1. Physical Concepts of Torque Production
234
3.1.1. Electrom agnetic (or interaction) torque 234
3.1.2. Reluctance (or alignm ent) torque
235
3.2. C onstructional F eatu res of R otating Electrical M achines
236
3.2.1. Polyphase Induction M achines 23­
3.2.2. Synchronous m achines 233
3.2.3. D irect C u rre n t M achines 242
3.3. Concepts of G eneral Terms P ertain in g to R otating M achines 244
3.4. G enerated emfs
250
3.4.1. G enerated e.m.f. in a full-pitched coil 250
3.4.2. G enerated e.m.f. in a short-pitched coil 252
3.4.3. A.C. M achines
253
3.4.4. G enerated e.m.f. in d.c. m achines 256
3.5. E.m.f. Polygon
263
3.5.1. D istribution, breadth or belt Factor 264
3.5.2. P itch (or coil-span) Factor 269
3.5.3. Elim ination of harm onics from altern ato r em f waveform s 272
3.6. M.m.f. Produced by D istributed W indings 285
3.6.1. M.m.f. of a coil 286
3.6.2. M.m.f. of distributed w indings 288
3.6.3. C urrent-sheet Concept 293
3.6.4. M.m.f. waveform of com m utator m achines 298
3.7. R otating M agnetic Field 301
3 . 8 . Production of Torque in Non-Salient Pole M achines 318
3.8.1. A lternative Derivation for Torque 323
3.9. Losses and Efficiency 329
3.10. M achine Ratings 333
3.10.1. Choice of power of electric m achines 339
3.11. Cooling (Loss dissipation) 344
3.12. M achine Applications 347

4. D .C . M a c h in e s 360-539
4.1. Action of Com m utator 353
4.2. E.m.f. G enerated in the A rm ature 365
4.3. Torque in D.C. M achines 359
4 .4 . Circuit Model of DC Machines 373
4.5. Methods of Excitation 376
4.6. M.m.f. and Flux Density Waveforms in d.c. M achines 379
4.6.1. A rm ature reaction 39 O
4.6.2. M ethods or lim iting the effects of „rm nturc renctiol) 385

Scanned by C am Scanner
(«)

Effect of B ru s h S h ift 386


4.7. C o m m u ta tio n P ro c e ss 390
4.8. 395
4.8.1. In terp o les
4.8.2. B ru sh es 396
C o m p e n sa tin g W in d in g s 398
4.9.
4 10 Basic Performance Equations for D.C. Machines 402
4 . 10 .1 . M agnetisation Curve 403
4 . 10 .2 . Effect of arm ature m.m.f. on d.c. machine calculations 404
4 11 Operating Characteristics of D.C. Generators 407
4.11.1. Separately-excited generators 407
4.11.2. Shunt generators 409
4.11.3. Series Generators 421
4.11.4. Compound generators 422
4.11.5. Effect of speed on external characteristics 423
4.12. Operating Characteristics of D.C. Motors 429
4.12.1. D.C. Shunt Motor , 429
4.12.2. D.C. series motor 431
4.12.3. D.C. compound motor 432
4.13. D.C. Motor Starting 444
4.13.1. Shunt and compound motor starters 445
4.13.2. Series motor starters 447
4.14. Speed Control of D.C. Motors 454
4.14.1. Speed control by varying the armature-circuit resistance 455
4.14.2. Speed control by varying the field flux 463
4.14.3. Speed control by varying the arm ature terminal voltage 477
4.15. Efficiency and Testing of d.c. Machines 489
4.15.1. Efficiency of d.c. machines 490
4.15.2. Testing of d.c. machines 495
4.16. Rotating Amplifiers 504
4.16.1. Cross-field or metadyne generators 504
4.16.2. Amplidyne
509
4.17. Perm anent Magnet DC (PMDC) Motors 519
4.18. D.C. Machine Applications
521
4.18.1. Generator applications 521
4.18.2. Motor applications
521

5.
P o ly p h a s e S y n c h r o n o u s M a c h in e s 540-702
5.1.
Excitation Systems for Synchronous Machines 540
5.2. Flux and mm f phasors in synchronous machines
543
5.2.1. Cylindrical-rotor synchronous machines 543
5.2.2. Salient-pole Machines 549
5.3.
Phasor Diagram of a Cylindrical Rotor Alternator 551
°pen-circuit and short-circuit characteristics of synchronous machines 552
5.3.2. Zero power-factor characteristic and Potier triangle 554
5.4.
Voltage Regulation of an Alternator 557
•4.1. The electromotive force (emf) method or synchronous impedance method 557
•4.2. The Magnetomotive force (m.m.f.) method 563
5.4.3. Zero power factor method 565
5.4.4. New A.S.A. (American Standards Association) method 565

. > ..
icann .am scan ner
r (*)

5.4.5. S a tu ra te d synchronous-reactance m ethod


is.

5.5.
Physical Concepts of Synchronous M achine O peration 56g
5.6.
Synchronous M otor P h aso r D iagram
5.7.
O perating C haracteristics of A ltern ato rs and th e ir R atings 575
5.7.1. E x ternal load ch aracteristics 577
5.7.2. A ltern ato r com pounding ch aracteristics 577
5.7.3. R ating of a lte rn a to rs 579
5.8. Pow er Flow Through an Inductive Im pedance 579
5.8.1. M axim um power conditions 580
5.8.2. Reactive Power 583
5.9. Circle D iagram s of Synchronous M achines 585
5.9.1. Synchronous m otor circle diagram s 604
5.10. Pow er F actor Control of Synchronous M achines 6<>4
5.10.1. Power-factor Control of Synchronous M otors
5.10.2. Power-factor control of A lternators
5.11. Two-reaction Theory of Salient-pole M achines
5.12. Pow er-angle C haracteristics of Synchronous M achines 617
5.12.1. Cylindrical-rotor synchronous m achine
5.12.2. Salient-pole Synchronous M achine 628
5.13. Synchronizing Power and Synchronizing Torque 629
5.13.1. Physical concepts of synchronizing power 637
5.14. Synchronous M achine Stability 639
5.15. H untin g and D am per W indings 643
5.15.1. H unting 647
5.15.2. D am per windings 647
5.16. M easurem ent of X d an d X 9 650
652
5.17. Efficiency of Synchronous M achines
659
5.18. O perating Lim it on Synchronous G enerators
5.18.1. O perating C hart 665
5.18.2. Capability Curves 665
667
5.19. Power F actor Correction by Synchronous Motors
669
5.19.1. Synchronous condenser
669
D uaI'Pu rP°se Synchronous Motor 071
5.20. S tartin g of Synchronous Motors rno

5.21. S o m e » T a m X hr0nOU6 ra0t° rS " ^ 679


5.22. Synchronous M achine Applications

6. P o ly p h a s e I n d u c t io n M o to r s " w iM O
6 . 1 . Induction Motor as a TYansformer
6 .2 . Principle of operation ,

el 2 :X™ ZhaS0rSandwavesin 705


6.5. Rotor e.m.f., C urrent and Power lu L
• Losses and Efficiency ' g 11
6-7. Induction Motor Phasor D iagram 720
6 .8 . Equivalent Circuit'
6.9. Analysis of the E quivalent Circuit 724
■ I- Torque-slip characteristics 727

Scanned by Cam Scanner


6.9.2. P ow er-slip c h a rac te ristics ~
6 . 10 . O p eratin g (or P erform ance) C h a r a c te r s * rt
6 . 1 0 . 1 . In d u ctio n m otor stab ility CS A u c tio n Motors 732
763
6 . 11 . D eterm in atio n of E q u iv a le n t C ircuit P a r a n w
6 . 12 . Circle D iag ram P aram eters 766
767
6.13. Pow er F acto r C ontrol of T h ree P W o i j .
775
6.13.1. S ta tic C ap acito rs InduCtl0n M otor«
786
6.14. S ta rtin g o f P o ly p h ase Induction M otors
787
fi ll'd m ! / ° f/ quirrel-caB« motors 791
6.15. Polarity Test” * ° Startmg woun<i-i-otor motors 791
6.16. Induction G e n era to r 80o
6.16.1. Self-excited or isolated induction generator 812
6 16 2 . A pplications of induction g en erato r, 813
6.17. A pplications of Polyphase Induction Motors 815
816
7. A r m a t u r e W in d in g ^ ----------- -------- --------- -----------------------------
7 .1 . Closed W indings 831-880
7-1 1 - Closed-w inding term s 833
7.2. Sim plex L ap W inding 835
7.2.1. S p lit coils 836
7.3. Sim plex W ave W inding 841
7.3.1. D um m y Coils 842
7.4. M ultiplex W indings 845
7.5. E q u alize r Rings 846
7.6. O pen W indings or A.C. A rm ature Windings “ I
7 .7 . Double L ayer W indings 853
7.7.1. In te g ra l Slot W indings 856
7.8. F ractio n al S lot W indings 887
7.9. S ingle-L ayer W indings 868
7.9.1. C oncentric W indings g7^
7.9.2. M ush w in d in g s ~

A ppendix 881-1053
A ppendix-A —M agnetic Circuits 0 gj
A ppendix-B — T hree P hase Circuits 901
A ppendix-C — M ultichoice Q uestions and Answers 918
Appendix-D — S h o rt A nswer Type Questions 1030
A ppendix-E — Table of C onstants & Conversion
F acto rs and The Greek Alphabet 1052

Index 1053-1060

Scanned by Cam Scanner


ONE
Transform ers
The tra n s fo rm e r is a device t h a t tm n c f 1 • *
another e le ctric al c irc u it th ro u g h thp 6rS e!ectn cal en ergy from one electrical circuit to
frequency. T h e e lectric c irc u T w h ic h °f GtiC fld d and w ith o u t a ch an *e * the
winding a n d th e o th e r c irc u it w h ich d elivers ^ * ? 7 m ^ S^ P? ly mainS is Called Prim ary
winding. delivers electric energy to th e load is called th e secondary

received ^ ! y ^ t h e V r i ^ a r ^ i s ^ r s t ^ electro" ia ^netic en ergy conversion device, since th e energy


useful electrical e n e rg y in th e o th e rc irc u its(sT c o S d l™ “ d reconverted to
etc ) T h u s n rim a rv a n d I (secondary w inding circuit, th ird w inding circuit
m co u rted m T ^ t i c a U v T W ^ T ” gS ° f 8 tra n sfo rm er a r* " 0 * connected electrically, b u t
are coupled m a g n e tic a lly . T h is coupling m agnetic field allows th e tra n sfe r of energy in eith er
direction, from h ig h -v o lta g e to low -voltage circuits or from low-voltage to high-voU a% circuits
If th e tra n s f e r of e n e rg y occurs a t th e sam e voltage, th e purpose of th e tra n sfo rm e r i f m erely to
isolate th e tw o electric c irc u its a n d th is u se is very ra re in pow er applications. If th e secondary
winding h a s m o re t u r n s th a n th e p rim a ry w inding, th e n th e secondary voltage is h ig h er th a n
the p rim a ry v o ltag e a n d th e tra n s fo rm e r is called a step-up tran sfo rm er. In case th e secondary
winding h a s less t u r n s th a n th e p rim a ry w inding, th e n th e secondary voltage is low er th a n th e
prim ary v o ltag e a n d th e tra n s fo rm e r is called a step-dow n tra n sfo rm er. N ote th a t a step-up
tran sfo rm er can be u s e d a s a step-dow n tra n sfo rm e r, in w hich case th e secondary of step-up
tran sfo rm er becom es th e p rim a ry of step-dow n tra n sfo rm er. A ctually, a tra n sfo rm e r can be
term ed a s te p -u p or ste p -d o w n tra n s fo rm e r only a fte r it h a s been p u t in to service. T herefore,
when re fe rrin g to th e w in d in g s o f a p a rtic u la r tra n sfo rm e r, th e te rm s high-voltage w inding and
low-voltage w in d in g sh o u ld be u se d in s te a d of p rim a ry and secondary w indings.
A tra n s fo rm e r is th e m o st w id ely u se d device in b o th low and h ig h c u rre n t circuits. As such,
tran sfo rm ers a re b u ilt in a n a m a z in g ra n g e of sizes. In electronic, m e a s u re m e n t and control
circuits, tra n s f o r m e r size m a y be so sm a ll t h a t it w eighs only a few te n s of g ram s w h ereas in
high v oltage p o w e r c irc u its , i t m a y w eig h h u n d re d s of tonnes.
In a tra n s f o rm e r , th e e le c tric a l e n erg y tra n s f e r from one circu it to a n o th e r circu it tak es
place w ith o u t th e u s e o f m o v in g p a r ts — it h a s , th erefo re, th e h ig h e st possible efficiency o u t of
all the e le ctrica l m a c h in e s a n d re q u ire s a lm o st negligible am o u n t of m a in te n a n c e an d su p e r­
vision.
In su la tio n c o n s id e ra tio n s lim it th e g e n e ra tio n of a lte r n a to r (ac g e n e ra to r or synchronous
generator) v o lta g e s fro m a b o u t 1 1 to 2 2 kV . By m e a n s of tra n s fo rm e rs , th is voltage is stepped
UP to h ig h e r eco n o m ical tr a n s m is s io n v o ltag e, 400 kV or even h ig h e r, in o rd er to red u ce th e
transm issio n lo sse s. W h e re v e r th e e le c tric a l e n erg y is re q u ire d , tra n s fo rm e rs a rc in sta lle d to
step down th e v o lta g e s u ita b le for its u tilis a tio n for m o to rs, illu m in a tio n p u rp o se s etc. T h u s the
transformer is th e m ain reason for th e w id e s p re a d p o p u la rity of a.c. system s over d.c. system s.

1^ ^ ^ ----------------------- — — ^ i i
Scanned by C am Scanner
2 Electrical Machinery [A rt 1.1

In ad d itio n to its u se in pow er system s, tra n sfo rm e rs are w idely u sed in o th e r prom inent
a re a s of electrical engineering. In com m unication sy stem s, in p u t tra n sfo rm e rs connect the
m icrophone o u tp u t to th e first stag e of an electronic am plifier. In te rs ta g e a n d o u tp u t tra n s ­
form ers are used extensively in radio and television circuits. In electronic an d control circuits,
tra n sfo rm e rs are used for im pedance m atching for m axim um pow er tra n s fe r from source to the
load. P u lse tran sfo rm ers find wide application in ra d a r, television an d d ig ita l com puters. In
power electronics, tra n sfo rm e rs are extensively used (i) for g ate-p u lse trig g e rin g an d (ii) for
synchronizing th e pulse g atin g sig n als w ith th e ac supply voltage given to th e m ain power
circuit. In general, im p o rta n t ta sk s perform ed by tra n sfo rm e rs a re :
(i) for decreasing or increasing voltage and c u rre n t levels from one circu it to a n o th e r circuit
(or circuits w hen th e re are 2 or m ore o u tp u t w indings) in low and h ig h c u rre n t circuits ;
(ii) for m atching th e im pedance of a source and its load for m ax im u m pow er tra n s fe r in
electronic an d control circuits, and
(iii) for isolating d.c. w hile p e rm ittin g th e flow of a.c. betw een tw o circu its or for isolating
one circuit from another.
T ran sfo rm er is, therefore, an essen tial piece of a p p a ra tu s both for h ig h a n d low c u rre n t
circuits.
An electromechanical energy conversion device is one which converts energy from electrical
to mechanical or from mechanical to electrical. The coupling between th e electrical and m echanical
system s is through the m agnetic field. In a transform er also, the coupling betw een th e prim ary
and secondary windings is by m eans of the magnetic field. Both in electrom echanical energy
conversion devices and transform ers, the coupling m agnetic field behaves in a like m anner.
Therefore, the fundam ental principles involved in the analysis of a tran sfo rm er are m uch m ore
common in the analysis of electromechanical energy conversion devices.
The tra n sfo rm er is a static piece of electric m achinery and concepts ab o u t its b eh av io u r
can be understood in a com paratively sim pler m anner. In view of th e above, th e an aly sis of
tran sfo rm er m u st serve as a prelude to th e stu d y of electrom echanical e n erg y conversion
devices. A t th e sam e tim e, a tra n sfo rm er is an im p o rtan t energy conversion device a n d detailed
study of its behaviour is justified.
1 . 1 . T r a n s f o r m e r C o n s tr u c tio n

T here are two general types of tran sfo rm ers, th e core type a n d th e sh ell ty p e. T h e se two
types differ from each o th er by th e m an n er in w hich th e w indings a re w ound a ro u n d th e m ag­
netic core.

The m agnetic core is a stac k of th in silicon-steel lam in atio n s a b o u t 0 35 m m th ic k for 50 Hz


tran sfo rm ers In order to reduce th e eddy c u rre n t losses, th e se la m in a tio n s a re in s u la te d from
one an o th er by th in layers of v arn ish . F or reducing th e core losses, n e a rly all tra n s fo rm e rs have
th e ir m agnetic core m ade from cold-rolled g rain -o rien ted sh eet-steel (C.R.G .O .) T h is m ateria l
w hen m agnetized in th e rolling direction, h a s low core loss an d h ig h p e rm e a b ility . ’

1 1 to) ^ T t S n SUrT0Und a r n sid e ra b le p a r t of ste e l core as show n in Fig.


1.1 (a). In th e shell-type, th e steel core su rro u n d s a m ajor p a r t of th e w in d in g s as show n in Fig.
c o n d u c to r ^ at eri al “a u * ? 6 core' ty Pe tra n s fo rm e r re q u ire s le ss iro n b u t more
conductor m a te ria l as com pared to a shell-type tra n sfo rm e r. T he v e rtic a l p o rtio n s of th e core
a re u su ally called l.m bs or egs an d th e top and bottom p ortions a re called th e yoke. T h is m eans
t h a t fo r sin g le-p h ase tra n sfo rm ers, core-type h a s tw o-legged core w h e re a s sh e ll-ty p e h a s th re e ­
legged core. Jr

v j
Scanned by Cam Scanner
Art. 1*1] ill
I!*
Transformers

" ~ 4>/i
I
H.V.

In iron-core tra n s fo rm e rs , m o st of tb „ n -
however, so m e flu x t h a t le a k s th ro u g h ^ 00 ™ ? ° " ® " '? ‘° M gh Pe r” «=aWlity core. T h e re is
he, « « T h is flu x called leakage f l ux b S £ o n e w X n °n - ? agnetic -" a te ria l su rro u n d in g
this leak ag e flu x ,s d e s ira b le as it m ^ v e s not th e o th e r- A reduction i f
C onsequently a n effo rt is alw ays made to reduce h perform “ “ considerably,
achieved by p la c in g h a l f of th e low voltage ? L V I X * C° re' tyPe f a n s f o rm e r , th is is
second leg o r 1,m b. F o r th e h ig h vo ltag e w i n d t a g X ^ h f 7 f T ® ' 6g ^ ° th e r h a lf over
the o th er h a lf o v er th e second leg, Fig. 1 1 (a) , \X W
and H.V. w in d in g o u tsid e , in o rd e r to m inim ise t h e X X n t n f ' l t d jacen t to 4116 s‘eel core
In th e s h e ll ty p e tra n s fo rm e r, th e L V a n d H V w ind reqU ir6d'
and are in te rle a v e d o r sa n d w ic h e d as show n in Ffc 1 w T T T . °Ver the centTal Umb
coils a re o f h a lf th e size o f o th e r L.V coils Shell f I' bottom an d top L.V.
low-power le v e ls, w h e r e a s c o re -tv p e c o n s tn ^ t i 5 tra n sfo r™ers are R e fe rre d for iow-voltage
transfo rm ers. YP6 c o n s tru c t^ n is u sed for h ig h -v o ltag e, h ig h -p o w er

In the sh ell-ty p e tra n s f o rm e r , th e i"n L V c T n t ^ m b d ^ ^ ^ ^ y°keS’ F lg ‘ 1 ,1 (aX


the o u ter tw o leg s a s sh o w n in Fig. 1 .1 (6 ), equally an d r e tu rn s th ro u g h

for core-type tr a n s f o r m e r s a T s h o ^ l ^ Fi * * 1 1°( t,r a n ®fori(ne^s - T he concentric coils a re used


shell-type tr a n s f o r m e r s as sh o w n in Fig. 1 1 ( 6 ) C l e a v e d (or sandw iched) coils for

1.2 (a)lL t ^ M 0 f l a m i S a tir Si 0 r thG C° re a n d She11 ty p e ° f tra n sfo rm e rs is illu s tr a te d in F ie


in i re sp e c tiv e ly . T h e ste e l core is assem bled in su ch a m a n n e r t h a t th e h u tt in' +
a v o i d r enktla y e r S 31,0 S tag g e re d a s illu s tr a ted in Fig. 1.2 (c). T he sta g g e rin g of th e b u tt io S te
At the I T ™ US a i r g a p a n d >t h e r efore, th e re lu c ta n c e of th e m ag n etic circu it is n o t in creased
t h e r e f o r ^ T ’ * C O ntm u™ s a ir Sap w ould red u ce th e m echanical s tre n g th of th e core an d
fore, th e s ta g g e r in g o f t h e b u t t jo in ts is e sse n tia l. ’

then th” 1}5 t h ° t r a n s f o r m e r c o n s tru c tio n , firs t th e p rim a ry a n d secondary w in d in g s a re w ound


is ore a m in a tio n s a r e p u s h e d th ro u g h th e coil openings, la y e r by la y e r a n d th e steel core
P p ared . T h e la m in a tio n s a r e th e n tig h te n e d by m e a n s of clam ps a n d bolts.
Low p o w e r tr a n s f o r m e r s a r e air-cooled w h e re a s la rg e pow er tra n sfo rm e rs are im m ersed
in0 \ b et t e r co oling. I n oil-cooled tra n s fo rm e rs , th e oil serv es as a coolant an d also as a n
E l a t i n g m e d iu m :
/

’ *

canned by Cam Scanner


■ —

[Art. 1.2
4 E lectrical M achinery

Butt
/ Butt
^joints
Z' joints
k V_
f/ /
--------------

(a) (b) (c)


Fig. 1.2. Two adjacent layers for (a) core and (b) shell type of transform ers
(c) arrangement o f butt joint9 in a m agnetic core.

For power frequency range of 25 to 400 Hz, tran sfo rm ers are co n stru cted w ith 0.35 m m
thick silicon-steel lam inations. For audio-frequency range of 2 0 to 20,000 Hz, iro n core w ith
suitable refinem ents is used. For high frequencies em ployed in com m unication c irc u its, core is
made up of powdered ferrom agnetic alloy. In special cases, th e m ag n etic c irc u it of a t r a n s ­
former may be m ade of non-m agnetic m aterial and in such a case, th e tra n s fo rm e r is re fe rre d
to as an air-core transform er. The air-core tran sfo rm er is p rim a rily u sed in ra d io device^ £ n d
in certain types of m easuring and testin g in stru m en ts. Cores m ad e of soft f e rrite s a r e also used
for pulse transform ers as well as for high frequency electronic tra n sfo rm e rs.
1.2. P r in c ip le o f T r a n s f o r m e r A c tio n
A transform er works on th e principle of electrom agnetic in duction b e tw e e n tw o (or m ore)
coupled circuits or coils. According to this principle, an e.m.f. is in d u ced in a coil if it lin k s a
changing flux.
In core-type transform er, h a lf of th e L.V. (and
H.V.) w inding is on one limb and th e other h a lf is
on th e second limb. In shell-type transform er, th e
L.V. and H.V. w indings are sandw iched. How­
ever, for sim plifying the draw ing and analysis of
both th ese types of transform ers, th e schem atic
d iag ram is as show n in Fig. 1.3. T he p rim a ry
w inding P is connected to an a lte rn a tin g voltage Fig. 1.3. Schem atic d iagram o f a tv/u-w inding
source, therefore, an a lte rn a tin g c u rre n t I e s ta rts transform er.

flowingthrbughA^! tu rn s. The a lte rn a tin g m m fiV ,/ s e t s u n n l w ^ , .


to th e high perm eab ility iron p a th as indicated in F ig 1 3 T he a n f n ' S C° nfined
E, in th e p rim a ry /" an d E2 in th e secondary S. If th e load is connert ‘nduces v o lta Se
load c u rre n t s ta rts flowing. ' connected acro ss th e seco n d ary , a

sam e
iron core^ T ^ e m f W u ceT in ^ eT e^ om h i^ or* wHary winding Is u s ^ d ly rrferretUo as'th
th e em f

-n
. . . - " V'
Scanned by C am b cann er
T ransform ers

, to transform er action. T hus th e transform er action requires the existence of altern atin g
Actual flux linking th e various w indings on a common magnetic core.
A transform er hav in g prim ary and secondary windings is called a two-winding transform er
vhereas a tra n sfo rm er having prim ary, secondary and tertiary windings is known as a
t h r e e - w i n d i n g tran sfo rm er. As stated before, prim ary is connected to source w hereas the
seco n d ary and te rtia ry w indings feed the load.

1 3 . Id e al T w o -w in d in g T r a n s f o r m e r
In the beginning, a tran sfo rm er is assum ed to be an ideal one, m erely for obtaining an
easier explanation of w h at happens in a transform er. For a transform er to be an ideal one,
the various assum ptions are as follows :
1. Winding resistances are negligible.
2 . All th e flux set up by th e prim ary links th e secondary windings, i.e. all the flux is confined
to the m agnetic core.
3 . The core losses (hysteresis and eddy current losses) are negligible.
4 . The core h as co n stan t perm eability, i.e. the m agnetization curve for the core is linear.

At a la te r stage, th e effect of these assum ptions will be taken up one by one.


It has been sta te d before th a t th e words prim ary and secondary should not be used with
the two windings of transform er. However, it has been found convenient to use these term s
during the tran sfo rm er analysis. B ut it should be kept in mind th a t these are arb itrary term s
as explained before. Hence forth, sub-scripts 1 and 2 would be associated respectively w ith the
primary and secondary w indings of a transform er.
Let the voltage V x applied to th e prim ary of a transform er, w ith secondary open-circuited,
be sinusoidal (or sine wave). Then th e current I e<due to applied voltage Vlf will also be a sine
wave. The m m f N xl e and, therefore, th e core flux 4 will follow the variations of Ie very closely.
That is, the flux <f>is in tim e p hase w ith the current Ie and varies sinusoidally. If Ie is zero, 4
is zero and if 7, is m axim um positive, 4 is also maximum positive and so on. Therefore, if th e
applied voltage V x h a s sine waveform, the flux $ m ust also have a sine waveform. L et the
sinusoidal v ariation of flux <{>be expressed as
4>= <lW sin cot - (L1)
where 4 m„ is th e m axim um value of the m agnetic flux in webers and co = 2 n f t is th e angular
frequency in rad/sec an d f is th e supply frequency in Hz.
The em f e 2 in volts, induced in th e prim ary N , tu rn s by the altern atin g flux 4 is given by

...( 1 .2 )
* - N ^
e i “ " yVl d t
= - N l o x Jw cos cot
' *
—N iCO<J)ma3(, sin 2

is equal to 1 .
Its m axim um value, E lmax occurs when sin 2

...(1 .3 )
and = E 1 max sin 2

IcanHecnDyiLam bcanner
6 Electrical Machinery [Art. 1.3

Rms value of em f E x induced in p rim ary w inding is given by


p E\rnax 2 jt ,
1 = ~ ^ 2 ~ = "72 f 1^max
E l = y l2 n fN 1 ^ ...(1.4)
The cu rren t Ie in the prim ary is assum ed to flow along th e p a th abcda, Fig. 1.4. T h e e.m .f.
«i induced in N x tu rn s m ust be in such a direction ------------------------- -
as to oppose th e cause, i.e. Ie\ as per Lenz’s law.
Therefore, th e direction of e^ is as shown by th e
arrows in th e prim ary N 1 tu rn s and it is seen to
oppose i>1# Since prim ary w inding resistance is
negligible, e1 a t every in stan t, m ust be equal and
opposite to
Fig. 1.4. Ideal transform er on load.
i.e.
f
or ...(1.5)
Vl = - E 1

The em f induced in th e secondary is

e2 - N 2 ~ ^ - - N 2 (o^max cos (Ot

2 ...( 1 .6 )
Rms value of emf P 2 induced in ^ ^ ^
„ Ert . •
* 2= - ^ = V2 n f N ^
From Eqs. (1.4) and ( 1 .7 ), ...(1.7)

E<i jV2
or Ei e2
. AT j x v ^
= N 2= 71'f ‘t1.
' "tax

*, air te kept * — .

ttierefore, ^ * 2 are th e re fo re " ^

♦ t t T 0 T 8' " e 5 ‘- 14 is 8een from P is , : ‘tS, eq u al a n d 0P P osite to B V COnveni(™ e an d ,


♦ th a t induces them . T h e « p plied ^ £ , a n d £ 2 la g a ta * * a*
1 lead s th e d u x 4, by 9 0 ° y 9 0 t h e m u tu a l flux

'"'f'-'ivifv-f

Scanne
Arl. 13]
Transformers
V, = -E ,

90°

le <p

E 1 ,E 2

(a)
(6 )
F ig . 1.5. Ideal transform er (a) phasor diagram and (6) tim e diagram.

h i F ig 4, i f th e sw itc h S is closed, a load im pedance Z L gets connected across the


secondary e r m in a ls . S in ce th e seco n d ary w inding resistan ce is zero, V 2 = E 2. According to
L enzs law , th e d ire c tio n o f seco n d ary c u rre n t I 2 should be such th a t th e secondary m m f f 2
(= 7 ^ 2) is o p p o site to m u tu a l flux <J> in th e core. For F 2 to be directed a g a in st 4>, th e c u rre n t
/ 2 m u st le a v e th e te r m in a l n, p a s s th ro u g h th e load and e n te r th e te rm in a l m, Fig. 1.4. The
secondary w in d in g b e h a v e s lik e a voltage source, therefore, term in al n m u st be tre a te d as
positive a n d te r m in a l m a s n e g a tiv e . T his m eans t h a t w hen term in al b is positive w ith respect
to c in Fig. 1.4, te r m in a l n is p o sitiv e w ith resp ect to m a t th e sam e tim e. T his form s th e basis
for p o la rity m a r k in g s in tra n s fo rm e rs , A rt. 1.11. If secondary w inding is w ound in a m an n er
opposite to t h a t sh o w n in Fig. 1.4, te rm in a l m would be positive w ith respect to term in al n.
This show s t h a t p o la r ity m a rk in g s o f th e w indings in tran sfo rm ers depend upon th e m an n er
in w hich th e w in d in g s a r e w o u n d a ro u n d th e legs w ith respect to each other.
In Fig. 1.4, th e se c o n d a ry m m f F 2 being opposite to <► , ten d s to reduce th e a lte rn a tin g
m utual flux 0 . A ny re d u c tio n in <)>w ould reduce E^. For an ideal tra n sfo rm er, V l = - E x. If the
applied v o lta g e Vj is c o n s ta n t, E x an d , therefo re, m u tu al flux <J> in th e core m u st rem ain
constant, a s p e r E q. 1.4. T h is can h a p p e n only if th e p rim ary draw s m ore c u rre n t I { from the
source, in o rd e r to n e u tr a lis e th e d em ag n etizin g effect of F 2. In th is m an n er, 12 causes the
prim ary to ta k e m o re c u r re n t, / / , in addition to I e such th a t

I\ N \ = I2 N 2
or C o m p en sa tin g p r im a r y m m f, F \ = Secondary m m f, F 2 ...(1.9)
Any c h a n g e in th e se c o n d a ry c u rre n t is a t once reflected by a corresponding autom atic
change in th e p r im a r y c u r r e n t so t h a t core flux rem ain s u n altered .
In th e ab o v e e x p re ss io n , is called th e load com ponent of p rim ary c u rre n t / ,. It is th u s
seen th a t core flu x in a n id e a l tra n s fo rm e r rem a in s co n sta n t and is in d ep en d en t of th e load
current.
a • , . , u l a \t hv an an ele 0 <> t h e p h a s o r diagram u n d e r load for an ideal
A ssu m m g l 2 to la g b e h m d V 2 by a n angle B, P m agnetize the
transform er c a n be d ra w n a s sh o w n in Fig. 1.6. S>ince m inis. r x a 2
“ re in o p p o site d ire c tio n s , th e y a re show n in p h ase opposition in g. . .
The total primary current / , is the phasor sum o f / , and i.e.,

“Scanned by Cam Scanner


Electrical Machinery [Art. 1.3

T h e pow er factor on th e p rim ary side of th e ideal tran sfo rm er is cos Qv


I f th e m ag n etizin g c u rre n t I. is neglected, th en Eq. (1.9)
becom es
/jA T jrr/jJV a ...( 1 .1 0 )
i.e. P rim a ry a m p ere-tu m s
= S econdary am p ere-tu m s.
T h u s for an ideal tra n sfo rm er w ith l t = 0 , we have
Vi El Ni

an d E l I 1= E2I2
or V i h = V2 I 2 ...( 1 . 1 2 )
i.e. P rim a ry volt-am peres = Secondary volt-am peres.
In Eqs. ( 1 . 1 1 ) and (1.12); V h V2, I h I 2 have rm s values.
From Eq. (1.5),

vl = N l V2 =E,= E2
dt
cfa Fig. 1.6. P hasor diagram o f an
Sim ilarly v2 = N 2 ideal transform er for inductive load.
dt
• Hi Hi
" n , =n 2 ...( 1 .1 2 a)

Eq. (1.10) in term s of in stan tan eo u s value is


i\N i = i?N2 ...( 1 .1 2 b)
M ultiplication of Eqs. ( 1 .1 2 a) and ( 1 .1 2 b) gives,
yi h = u2 h ...(1.13)
T his m eans th a t in stan tan eo u s pow er in p u t into p rim ary equals th e in s ta n ta n e o u s power
o u tp u t from th e secondary. This relatio n is a consequence of th e a ssu m p tio n s ( l ) - ( 3 ) m ad e for
a n ideal tran sfo rm er.

I f K irch h o ff 8 voltage law is applied to th e p rim ary w inding circuit abcda of F ig 1 4 th en


te rm in a l b m u st be positive w ith respect to term in al c, since c u rre n t I e can flow from a high
p o te n tia l to a low er poten tial only.
F or th e circuit abcda, th e K irchhoffs voltage law gives,
v1- e 1 = 0
vi = e i
F rom Eq. (1.5), d&
"1 = ^ 1 \T., ^
- N
ei =
dt ’ dt ...(1.14)

H e re th e e.m.f. e , is tre a te d as a voltage drop in th e directio n of c u r re n t I and its


in s ta n ta n e o u s v alu e is given by Eq. 1.14. e

flux <b is in c rp a s fre ^ th p n ' t *lro u g l1 tlle Prim a ry w inding. If 7e a n d therefore,


H , f: *> m duced In th e Prim a ry sh o u ld be in su c h a d ire ctio n th a t if
' a ‘° n e '. “ W,°U,d e; abll9h a c u rren * °P P °site to / , as p e r L enz's law . A ccordingly, th e
d irectio n o f e , is m d icated by arrow s ,n th e p rim a ry w inding, te rm in a l b is a g a in p o sitiv e w ith

Scanned by Cam Scanner


Art. 1.3]______________
-------------- .------------- -------------- Transformers 9
respect to te r m in a l c a n d e i is se e n to be a ctin g
m ath em atical fo rm a s B opposite to t/j. T his physical fact is w ritte n in

vi = - e x= N &
1 dt
where e x is tr e a te d a s a re a c tio n e m f .
a c tio n e.m .f., c o u n te r e.m .f. o r g e n e ra te d e.m.f.

The a p p ro a c h w h ic h led to th e re la tio n e , - w * : . „ ,


° n Cl - " i * 18 u su a lly re fe rre d to as th e circ u it view
point. T h e se co n d a p p ro a c h w h ic h g av e th e re la tio n , - , ,
. . , re la tio n e , _ _ .y , _ s le re fe rre d to a s th e field or dux
view-point. A n y o f th e tw o view -nointB «n ,
physical co n cep ts o f th e in te r n a l b e h a v io u r o f a t r ^ f th e field view ‘Po in t & ves b e tte r
impedance T r a n s f o r m a t i o n ^ I X m l 1 * " 7 ' “ “ ad<>Pted “ this book
Fig. 1.7 (a). A lo ad im p e d a n c e Z , is connecter! la B tam for a n ideal tra n s fo rm e r is show n in i
the p rim a ry a n d se leco
c o nnddaarv
ry w J in
j dZin gTs il nT dl i* c “t T * ?, 8e' ° n daary^ s8ida
, T P^ 8 m «a rk ed
ia e ’ Uot Points on
e d on
in stan t. F o r e x a m p le , i f d o tte d te r m in a l of nrim n "“ “ “I? of.the corresponding p o larity a t any
term in al a t a n y in s ta n t, th e n d o tte d te rm in a l nf ** W1 g “ P0sitive w ith resp ect to u n d o tted
to u n d o tte d te r m in a l a t th e sa m e in s ta n t ^ W ing ls also Po sitive w ith resp ect

o p~

bo^

Fig. 1.7. T ransfer o f im pedance in an ideal transformer.

For th e s e c o n d a ry c irc u it, = Z 2, load im pedance,


12
For a n id e a l tra n s f o rm e r , from Eq. ( 1 . 1 1 ),

Vl A T M T XT
T T1 = N ~2 aH 1 1 = 2 2

or Ml
V l -N ~ 2 V2 ....(1 .1 5 )

and N2
...(1 .1 6 )
^ - W x l2
Division o f E q. (1.15) by E q. (1.16) gives effective in p u t im pedance a t th e p rim a ry term in als
a and 6 as

Vi N i • V2 N i
Ix N2 ^2^2
v2
fw ' l Zo = Z 9' ...(1 .1 7 )
[*r
>1

[ n 2J n 2
\ /

"Scanned by C am Scanner
[Art. 1.4
10 Electrical Machinery ________ _______ ________ __________________ ________________________

This shows th a t as far as effect of Z 2 on prim ary side is concerned,


secondary circuit can be replaced by an equivalent impedance Z 2 in the p n . y

fATjf
Z ,' =
N,
This is shown in Figs. 1.7 (6 ) and (c). . , arw.0 viewpH
T hus, th e three circuits shown in Fig. 1.7 are identical as far as th e .r perform ance v.ewed
from term inals a and b is concerned.______________________________________________r_; . .
Sim ilarly, an im pedance Z x in the prim ary circuit can be tra n sfe rre d to (or r f
secondary side as
2 7 _ 7 /
N Z ,- Z i

T ransferring an impedance from one side of a transform er to the o th er is called referring


the impedance to the other side. Similarly, voltages and cu rren ts can be referred to e ith e r side
of transform er by m eans of Eq. (1.11). In this m anner, re su lta n t voltage an cu rre n on a
side can be evaluated. ,
For an ideal transform er, it m ay be sum m arized th a t (i) voltages are tra n sfe rre d in th e
direct ratio, (ii) currents in th e reverse ratio, (iii) im pedances in th e direct ra tio sq u ared and
(iv) power and volt-am peres rem ain unchanged.
Eq. (1.17) illu strates th e impedance-modifying property of a tran sfo rm er. In practice, th is
property is exploited for m atching a fixed load im pedance to th e source im pedance for th e
6 k purpose of m axim um power tran sfer from source to load. This is achieved by in terp o sin g a
W l.r ] transform er of suitable tu rn s ratio between the load and the source.
1.4. T r a n s f o r m e r P h a s o r D ia g ra m s
y The purpose of first considering an ideal transform er, i.e. a tra n sfo rm e r w ith no core losses,
no winding resistances, no m agnetic leakage and constant perm eability, is m erely to h ig h lig h t
th e m ost im portant aspects of tran sfo rm er action. Such a tran sfo rm er n ev er ex ists a n d now th e
phasor diagram s of real transform er w ith various im perfections will be considered.
M agnetization curve of th e actual tran sfo rm er core is non-linear an d its effect is to in tro ­
duce higher order harm onics in th e m agnetizing cu rren t. Since all th e q u a n titie s in a p hasor
diagram m u st be of th e sam e frequency, these h ig h er order harm onics (w hose freq u en cies are
odd m ultiples of fundam ental frequency) can’t be rep resen ted in th e p h a so r d iag ram . So a
lin e a r m agnetization curve for th e tran sfo rm er core will continue to be assu m ed .
The p h aso r diagram of a tran sfo rm er is now developed, first a t no load a n d th e n u n d e r load.
1.4.1. Transform er phasor diagram at no load. The m agnetic flux <h b ein g com m on to
both the prim ary and secondaiy, is draw n first. The induced emfs E 1 and E 2 lag <j>by 90° and are
shown accordingly in Fig. 1.8 (6 ). The voltage - E 1 is being replaced by V { ju s t for convenience. Al­
ternatively Vy m ay be treated as a voltage drop in the prim ary, in th e direction of flow of prim ary
cu rren t The various imperfections in a real transform er are now considered one by one.
(a) Effect o f transform er core loss. The core loss (or iron loss) c o n sists o f h y s te re s is loss and
eddy c u rre n t loss. These losses are alw ays p re se n t in th e ferro m ag n etic core o f th e tra n sfo rm e r,
since th e tra n sfo rm e r is an ac-operated m agnetic device. T h e h y s te re s is loss in th e core is m in i­
m ised by u sin g high g rad e m a te ria l such as cold-rolled-grain o rie n te d (CRG O ) s te e l a n d th e
eddy c u rre n t loss is m inim ised by u sin g th in la m in a tio n s for th e core.
'Hie current in the(primary is alternating, therefore, the m agnetizing force H is cyclically
varying from one positive value say H x to a corresponding negative value - H lt Fig. 1.8 (a).

v u i . i a J u r n rv^'. ■■r- . ----


Transformers ;

' e , , e 2- v2
(C)

a r ^ a S s a s ^

OP, Fig. ! . 8 (a), th e


at in sta n t w t,.It .a s e e n from F t f r ^ or lag, bemg dependent on J
♦ lags W by so m e tim e £g ^ shown leading *, „r * ,s shown ■> g
loop, is called th e h ysteretic angle. In f i g
I bv hysteretic an g le a. a.,v,nirmrrenf of the transformer and can De

" T h e n o d o a d p rim a ^ ^ ^

c u r r e n tsin c e r e function is ^ t t Z ^ T o s ^ o n e n , or power component, o

or, i/ c = —7
y x' Amp.
V { l c = Pc

...(1-18)
From Fig. 1.7 ( H il is seen ^ —y ? . current/
'« = ^ loM cu rre n t / . - 0 and therefore e x e fn g cu rren t /.
Note that in an ideal transformer, core-lo

' magnetizing current Im-

Scanned by Cam Scanner


12 Electrical Machinery [Art. 1.4

(b) E ffect o f transform er resistance. The effect of prim ary resista n ce r j can be accounted for,
by add in g to V f , a voltage drop equal to r fle, as show n in Fig. 1.8 (6 ). N ote th a t r j e is in pnase
w ith I e an d is d raw n p arallel to I e in th e phasor diagram .
(c) E ffect o f leakage flux. The existence of electric potential difference is e ssen tial for th e
e sta b lish m e n t of c u rre n t in an electric circuit. Sim ilarly th e m agnetic p o ten tial difference is
n ecessary for th e esta b lish m en t of flux in a m agnetic circuit.
F or th e direction of c u rre n t I e in th e prim ary, Fig. 1.9, th e point A is a t a h ig h e r m agnetic
p o ten tial th a n point B. T his m agnetic potential difference estab lish es : (i ) th e m u tu a l flux 0
lin k in g both th e w indings and Hi) the
p rim a ry leak ag e flux 0 f1( which links r - -------- -- "i
only th e p rim a ry w inding. The distinc­ 4 Ie ;a i . ___— -- Q
-----*
1 **
--- ~—7^ \ • / 1 >
tive b eh av io u r of th e m u tu al flux (j>and . < * 1
th e p rim a ry leakage flux 0 n m u st be T t f 1
i J 1--'--
1 —
carefully understood. The m u tu al flux © V> < * , i : t' * : K : ___ ►
•i « > / ( -1- . :--------------- o
0 exists e n tirely in th e ferrom agnetic 1

core and, therefore, involves hysteresis ;_B ------ -----------
loop. T he c u rre n t I e th a t establishes <!>
m u st lead it by some hysteretic angle. Fig. 1.9. Transformer at no load.
On th e other hand, th e prim ary leakage
flux 0 ;i exists largely in air. A lthough does pass through some iron, th e relu ctan ce offered to
0 a is m ainly due to air. Consequently 0 a does not involve any h y steresis loop and it can be ta k e n
to be in phase w ith th e cu rren t/,, th a t produces it, Fig. 1.8 (c). In th e p rim ary w inding, induces
an em f E x lagging it by 90°; sim ilarly th e prim ary leakage flux 0a induces an e.m.f. E xl in th e
prim ary winding and lagging it (i.e. 0n ) by 90°. Since Ie leads E xi by 90°, it is possible to w rite
E xi = - j l e x v T he p rim ary applied voltage Vj m u st have a com ponent j l e x h eq u al an d opposite
to E xV H e re* ! h as th e n a tu re of reactance and is referred to as th e p rim ary leak ag e reactance
in ohms. It m ay be noted th a t x x is a fictitious quantity merely introduced to rep resen t th e effects
of prim ary leakage flux.
The to tal voltage drop in prim ary a t no load is + jx f) = Ie z lt w h ere z 1 is th e p rim ary
leakage im pedance. Therefore, Fig. 1.8 (c) gives th e phasor diagram of a tra n sfo rm e r a t no load,
w here N l is assum ed to be equal to N 2. The p rim ary voltage equation a t no load can be w ritte n
as

? i = ^ i ' + 7e (ri + jx x) ...(1.19)


The p rim ary leakage im pedance drop shown in Fig. 1.8 (c), is d raw n to a la rg e r scale in
com parison w ith V f or V h ju s t for th e sake of clarity. A t no load V { an d V x a re v ery nearly
equal. Even a t full load, p rim ary leakage im pedance drop in pow er tra n sfo rm e rs is ab o u t 2 to
5% of V lt so th a t th e m agnitude of V f or E 1 (and therefore, 0 as p er Eq. (1.4)) does n o t change
appreciably from no load to full load.
It m ay be noted th a t th e to tal p rim ary flux is th e p h aso r sum of 0n a n d 0 , th erefo re, its
p haso r is a little ah ead of 0 .
1.4.2. T ra n sfo rm er p h a so r diagram u n d er load. T he secondary circuit of th e tra n s ­
form er is considered first and then the prim aiy circuit, for developing th e phasor diagram of a tra n s­
former under load.

W hen sw itch S is closed, secondary c u rre n t I 2 startB flowing from te rm in a l n to th e load, as


described in Art. 1.3 (Fig. 1.4). A ssum e th e load to hav e a lagging pow er facto r so t h a t / 2 lags

fcu.
n’itvj u_y O cji i ii iui
Art. M l
Transformers 13

v m

/:i} r !
LOAD

Fig. 1.10. Transformer under load,


secondary load v oltage V2 by an angle 0 2. At
first V2 is d ra w n w ith / 2 lagging V2 by the
secondary p.f. a n g le 02( F ig . l . n (a) The
secondary re sis ta n c e drop is accounted for
by draw ing r j 2 p a ra lle l to / 2. T he secondar^
m.m.f. I^N 2 g iv es ris e to a leak ag e flux 0/2
which lin k s only th e seco n d ary an d not th e
prim ary. T he flux 0/2 is called th e secondary
leakage flux a n d is in p h a se w ith / 2, for the
same reaso n t h a t 0 71 is in p h a se w ith I e in Fig.
1.8 (c). T he se c o n d a ry leak ag e flux induces
e.m.f. Ex2 in th e se co n d a ry w inding, lagging
0/2 by 90°. T he se co n d ary no load voltage E 2
m ust have a co m p o n en t eq u al an d opposite to
- j x 2 72. T h u s th e p h a s o r su m o f V 2 , I 2r2 and
(a) (b)
JI ^ 2 g i'^ s th e se c o n d a ry in d u ced e.m.f. E 2 as Fig. 1.11. Transformer phasor diagram for
shown in Fig. 1.11 (a). (a) lagging p.f. load and (b) leading p.f. load.

The v o ltag e e q u a tio n for th e seco ndary circuit can now be w ritte n as
Hf2 = V 2 +12 (r 2 + jx 2) = V 2 +12 z 2 ...( 1 .2 0 )
where z 2 is th e se c o n d a ry le a k a g e im pedance of th e tra n sfo rm er.
F u r th e r th e m u tu a l flux 0 is d raw n lead in g E 2 by 90° and exciting c u rre n t l e is d raw n
leading 0 by th e h y s te re tic an g le a . N ote th a t th e p h aso r V2 h a s pu rp o sely been ta k e n to th e left
of vertical lin e, so t h a t E 2 is v e rtic a lly dow nw ard and th e m u tu a l flux 0 is h o riz o n ta l.
The co m p o n e n t o f th e p r im a r y c u rre n t w hich n e u tra lis e s th e d em a g n e tiz in g effect o f / 2 is
h ' (7\ N 1 = I ^ J 2) a n d is d ra w n o p p o site to I 2. T he p h a so r sum of a n d I t gives th e to ta l p rim a ry
current / 2 ta k e n fro m th e s u p p ly m a in s. T he p rim a ry leak ag e im p ed an ce drop l l(rl + j x x) is
depicted as e x p la in e d e a rlie r. T h e v o ltag e e q u atio n for th e p rim a ry circu it u n d e r load can be
w ritten as
V, = V< + 7,(r, + > ,) = V ,'+ 7
where z 1 is the prim ary leakage impedance of the transformer. Note that the angle 0 ! between
and /j is the prim ary power-factor angle under load.
If the secondary load current / 2 leads the voltage V2 such that the load p.f. is leading, then
phasor diagram for the transform er is as shown in Fig. 1.11 (6 ). The entire procedure for
^ w in g the phasor diagram is the sam e as explained for Fig. 1.11 (a).

"Scanned by C am S canner
14 Electrical M a c h in e r y _________________ _________________________________________________ [Art. 1.5

I t m ay be seen th a t th e developm ent of tra n sfo rm er ph aso r diagram of Fig. 1.11, gives a
b e tte r physical p ictu re of w h a t h ap pens in th e prim ary and secondary w indings of a tra n s ­
form er an d its core. B ut th is p h aso r diagram is helpful only (i) w hen a tran sfo rm er is to be
stu d ied alone an d (it) w hen th e in te rn a l behaviour of the tran sfo rm er is to be understood.
W hen th e tra n sfo rm e r is a p a rt of the large power system netw ork, th e p h aso r diagram of
Fig. 1.11 should not be used. In stead , th e tran sfo rm er equivalent circuit is used.
1.4.3. L e a k a g e flu x . In a tra n sfo rm er, as secondary (or load) cu rre n t is increased, the
m ag n etic p o ten tia l of point X rises above th e m agnetic po ten tial of point Y, Fig. 1-1®- Tm s
re su lts in an in crease in th e secondary leakage flux <j>,2- H ere points X and Y are on th e limb
w h ere secondary w inding is w ound. W ith increase of secondary cu rren t, th e p rim ary cu rren t
also rise s a n d th is causes point A to a tta in a m agnetic potential h ig h er th a n t a t o pom .
As a re su lt, p rim a ry leakage flux <J>n increases. This shows th a t leakage fluxes in a tran sfo rm er
a re d ep e n d en t upon th e cu rre n ts in th e w indings.
Core flux in a tra n sfo rm e r depends upon th e em f induced in th e p rim ary w inding. W ith
in crease o f p rim ary c u rre n t, E i = V\ - 1\ ( ^ +jx{) does reduce and likewise core flux is re uce .
B u t th is reduction in £ , an d likew ise in core flux is quite sm all. T hus it m ay be sta te d th a t core
flux in a tra n sfo rm e r depends upon th e applied voltage and m ay be tre ated as co n stan t from no
load to full load.
In th e follow ing a rtic le , th e r a tin g of tra n sfo rm e rs is discussed. A fter th is, an exact
eq u iv a len t circuit of th e tra n sfo rm e r is developed first from which its approxim ate equivalent
circu it is obtained.
1.5. R a t i n g o f T r a n s f o r m e r s
T he m a n u fa c tu re r of tra n sfo rm ers fixes a nam e plate on th e tran sfo rm er, on w hich are
recorded th e ra te d o u tp u t, th e ra te d voltages, th e rated frequency etc. of a particular^trans--
form er. A ty p ical n a m e p la te ra tin g of a single p hase tra n sfo rm e r is as follows: 20 kVA,
3300/220 V, 50 Hz. H ere 20 kVA is th e rate d o u tp u t a t the secondary term in als. N ote th a t the
ra te d o u tp u t is expressed in kilo-volt-am peres (kVA) ra th e r th a n m kilow atts (kW). T his is due
to th e fact t h a t ra te d tra n sfo rm e r o u tp u t is lim ited by h eatin g and hence by th e jo sses in th e
tra n sfo rm e r T he tw o types o f losses in a tran sfo rm er are core loss and ohm ic (7 r) loss. The
core loss dep en d s on tra n sfo rm e r voltage and ohmic loss on th e tra n sfo rm er c u rre n t. As these
losses depend on tra n sfo rm e r voltage (V) and cu rre n t (7) an d are alm o st unaffected by th e load
p f th e tra n sfo rm e r ra te d o u tp u t is expressed in VA (V x 7) or in kVA an d n o t in kW. F or ex­
am ple a tra n sfo rm e r w orking on rated voltage and rated c u rre n t w ith load p f eq u al to zero has
ra te d losses an d ra te d kVA o u tp u t b u t delivers zero pow er to load. T his show s t h a t tra n sfo rm e r
ra tin g m u s t be expressed in kVA.
For an y tra n s f o rm e r :
(R ated in p u t in kVA a t (R ated o u tp u t in kVA a t
th e p rim a ry term in als) 1J th e secondary te rm in als) + Losses
(cos 0 j) (cos 0 2)
Since th e tra n sfo rm e r o p erates a t a very high efficiency, losses m ay be ignored. F u rth e r,
th e p rim a ry p.f. cos 0j an d th e secondary p.f. cos 02 a re n e a rly equal. T herefore, th e ra te d kVA
m ark e d on th e n a m e p la te of a tra n sfo rm e r, refers to both th e w indings, i.e. th e ra te d kVA of th e
p rim a ry w ind in g an d th e secondary w inding are equal.
T he voltage 3300/220 V refers to th e design voltages of th e two w indings. E ith e r of th e two
m a y serv e as p rim a ry or secondary. I f it is a ste p down tra n sfo rm e r, th e n 3300 V is th e ra te d

* A term inal is th at part o f an electrical en gineering device, w hich is intended to receive the extern al connections.

Scanned by Cam Scanner


Art- 1.51
" ” “ " Transformers 15
prim ary v o lta g e a n d re fe rs to th e vnlt a ^ i-j. , ....................
is th e r a te d s e c o n d a ry v o lta g e a n d refnfc ,a p P Pn m a r y w inding. T h e voltage of 220 V
n 0 load, w ith r a te d v o ltag e a p p lied to th e p rim V rJ t e r m f n ^ . ^ ^ 0UtPUt term in a,S *

respo n d in g r a t e d ^ o l t a g e ^ T h u s ^ CUITents a re calculated from th e ra te d kVA a n d th e cor-

R ated (or fu ll-lo ad ) p r im a r y c u r re n t = ^ > 0 0 0 - n« a


3300
R a ted (o r fu ll-lo ad ) se c o n d a ry c u rre n t = ^ ° 0 0 = 9Q ^ ^
220
w in d in g a r e ^ i g n e d '* P n m a ry a n d sec° n d a ry c u rre n ts refer to th e c u rre n ts for w hich th e

R ate d fre q u e n c y re fe rs to th e freq u en cy for w hich th e tra n s fo rm e r is designed to o p erate.


Nj
E 2 a n N 2 a re ca^ ec^ th e voltage ra tio and tu rn s ra tio respectively. T h ese two
ratios a re e q u a l a s s e e n from Eq. 1.8. A t no load, V, and E l a re n early equal in m a g n itu d e for
large tra n s f o rm e r s , th e re fo re , th e ir no-load voltage ra tio is

~1 = —1 R ated p rim a ry voltage N,


E 2 ^ 2 R ated secondary voltage N 2'
E x a m p l e 1.1. T h e e m f p e r tu rn for a single phase, 2 3 1 0 /2 2 0 V, 50 H z tra n sfo rm er is a p ­
proxim ately 13 volts. C a lcu la te (a) the n u m b er o f p rim a ry a n d secondary tu rn s a n d (b) the net
cros's-sectional a rea o f th e core, fo r a m a x im u m flu x density o f 1.4 T.
S o l u t i o n . E m f p e r tu r n E t = 13 volts.

(a) N u m b e r o f se c o n d a ry tu r n s = S econdary voltage


Et
220
N 2=~ ^ 16.92.
13
Now t h '' n u m b e r o f t u r n s c a n ’t be a fraction, therefo re, N 2 = 17 (n e a re s t w hole num ber).
For N 2 = 17,
N u m b e r o f p r im a r y tu r n s
2310^
N ,= N 2 = 17 = 178.5.
220
v ✓ ' '
T h is sh o w s t h a t N 2 c a n ’t be e q u a l to 17 tu rn s . T he o th er n e a re s t in te g e rs a re 16 or 18. It is
preferable to ta k e N 2 = 18.
... Afj = 18(10.5) = 189 tu rn s .
T h u s th e r e q u ire d v a lu e s o f a n d N 2 are 189 an d 18 tu rn s respectively.
n. 220
(b) N ew v a lu e o f e.m .f. p e r tu r n E t - volts.

T he n e t co re a r e a c a n be o b ta in e d from th e re latio n ,
' l2 n f * max = E l
220
or ^ 2 n fB mA l = E t = - j ^

Here B m = m a x i m u m value of f lu * density i n W b/m 2 or teslas and A, = Net core area.

^ o u tf iiiie u u y ^ a i i i o c d i n i e i
16 Electrical Machinery _______________ ^Arl ^ ^

••• V2ti(50) (1.4) A, = ^ = 393 cm2.


lo
I t m ay be seen th a t if N 2 is ta k e n equal to 16, th e em f p er tu rn increases and n e t core area
is m ore, w hich is n o t desirable.
E x a m p le 1 .2 . A single-phase 50 H z transform er has three w indings : a 2 2 0 -V p rim a ry, a
600-V secondary a n d a centre-tapped 11-0-11 V tertiary. For a net core area o f 75 cm 2, calculate
the num ber o f tu rn s in the three windings. The m a xim u m value o f flu x density is 1.2T.
S o lu tio n . E m f per tu rn E, = V2 rc f B mA,

= V2rc (50) (1.2) (75) x 10‘ 4 = 2.00 volts.

N um ber of tu rn s in each h a lf of th e te rtia ry winding, N 3 = -7 ^ = 5 .5 tu rn s

So ta k e Af3 = 6 tu rn s
Total n u m b er of tu rn s in te rtia ry = 2 x 6 = 1 2 .

T u rn s in th e 600 V secondary w inding = 600 x y y = 327.

and tu rn s in th e 220 V prim ary w inding = — ° * 6 = 120.

E x a m p le 1.3. (a) A 2 2 0 0 /2 2 0 V, 50 Hz, single-phase transform er has exciting current o f 0.6


A a n d a core loss o f 361 watts, when its h.v. side is energised at rated voltage. Calculate the two
components o f the exciting current.
(b) I f the transform er o f p a rt (a), supplies a load current o f 60 A at 0.8 p.f. lag on its l.v. side,
then calculate the prim ary current and its power factor. Ignore leakage im pedance drops.
S o lu tio n , (a) Exciting current Ie = 0.6 A
Supply voltage V { = 2200 V; Core loss Pc = 361 w atts.

/. Core loss com ponent Ic = ~ = = 0.164 A.

From Eq. (1.18),

M agnetising component/^, = 'JTj-

= V(0.60)* - (0.164)2 = 0.577 A.


(b ) The prim ary current component I f , required to neutralise
the effect of secondary current / 2 = 60 A, is given by

or 6 A-
V2 N«
N ote th a t — = —f
Vi ^
The c u rre n ts Ie / 2, a n d 7j are indicated in th e phasor
diag ram of Fig. 1.12.
The vertical com ponent of It = Ic = 0.164 A
The h o rizo n tal component of
A c An = 0.577 A. Fig- 1.12. Phasor diagram
.■ . ‘ . for Example 1.3.

l,j u g i 11 i c u uy w a n u t a i 11 i c i
Artj_ll£l------------------------------— _______ ___________________ __________________________Transformers 17

Vertical component of I , = / lC0s = //c o s 02 + Ic '

= 6 x 0 .8 + 0.164 = 4.964 A.
Horizontal component of l x = sin = / / sin q2 +

= 6 x 0 .6 + 0.577 = 4.177 A.
... Primary current ^ c o s O ^ ^ sin 0 ^
= V(4.964)': + (4.177)2 = 6.488 A. .

Primary power factor = cos Ql = = 1964 = Q iagging.


Ii 6.47
E x a m p le 1.4. A tra n sfo rm er is designed to have hot-rolled steel lam inations, w ith a flux
density o f 1.2 T a n d the w eight o f core a n d wire is found to he 100 kg and 80 kg respectively. I f
the transform er is redesigned w ith cold-rolled grain oriented (CRGO) steel lam inations, which
permit a h ig h er flu x d en sity o f 1.6 T ; fin d the saving in core and wire m aterials. The two types
of core m a teria ls have the sam e densities and the total flux rem ains the same. M ake suitable
assum ptions w herever required.
S o lu tio n . W ith h o t-ro lled steel lam inations:
Total flux tymax = Flux density B ml x A rea A x
j. i ci a a ty m a x 2
or 0m<ur —1.2Aj or A x — ^ 2 01

The d ia m e te r of th e circle aro u n d th e core is given by


X ,2 a ^mai
-d, =
1.2

I4
X
tm tu
1.2
L ength of th e tu r n a ro u n d th e core is _____
max
L, j = n dj j = x„ • yJ *- - KL
2
W ith CRGO la m in a tio n s :
♦ max 2
<IW = 1-6 A 2 or A 2 - 1 6 m

D iam eter of th e circle aro u n d th e core is


A fT ^ m a x
d.
2 ~ Vy nx 1 .6

L ength of th e t u r n a ro u n d th e core is
A 14 §max
L 2= n d 2= n - y n ' L6

'■*aEsa5K 3£S££.« = (AO (H eight of th e lim b) 9 ™ / 16 4

W 2 = | ( 1 0 0 ) = 75 k g '

'
iy*

j
T
18 Electrical Machinery '_________ (Art. l.S

Percentage saving in core m aterial


= -1-^ Q~ 75 x 100 = 25%.

H eight of the limbs are assum ed equal in both cases. Now for w ire,
W eight of wire when using CRGO lam inations (w 2)
W eight of wire when using h.r. lam ination (w{)
_ (L2) (Wire cross-section) (Turns)
(L.) (Wire cross-section) (Turns) <
_ aV4/« • V3
s r C i i 2
w 2 = 80 X 0.866 = 69.28 kg.
.. Percentage saving in wire m aterial
8 0 -6 9 .2 8 . . .
= ----------- x 100 = 13 .4%.

Wire cross-section and the number of turns are assumed equal in both the cases.

c u l a t e ^ e c o n d a n ' n o t l f v l T t J p e Z ^ ^ f “ design*d to operate a t 2 4 0 /1 2 0 V, 5 0 H z. Cal-


nected to : 1 S f reQuency i f the h.v. side o f the tra n sfo rm e r is con-
(a) 240 V, 40 Hz; (b) 120 V, 25 Hz- M 120 V u
(d) 480 V, 50 Hz; (e) 240 V, dc (c) 120 V’ 5 <>H* i
S o lu tio n . Prim ary voltage V, a t frequency/, is

, ^ ^ X fl+ m a x lN l
Let the prim ary voltage a t frequency f 2 be Vu so th a t
V n = ^ 2 K f2 ^m axiN l

■• _ A §max2

(a) From Eq. (j), 240 _ (40) (<t>m„T9)


or 240 (5° ) ( ^ axi)
q = 1-25 0)mail.
Secondary no load voltage a t freq u ency/j
is

and a t frequency f 2is 2


’max 2 2
E22 h <(>‘max
, 2
f\§ m a x \

= 120
This shows th a t th» ■ ' 5 0 * ‘•’" M ~ 120 volts a t 40 Hz.
'PPly frequency changes ^ i vo.tage re m ains . .
<U Prom Eq. i ? S - <25) d ’ ev en >f th<
240 <50) « W .) or < W =
From Eq. £ 22 = 120 x ( 2 5 ) ^

(5°) (4m<ui) 6 0 volts a t 25 Hz.

• ^<
h^

Icanned’b vTla m b c a n n e r
Art- 1 -5 ]________________
Transformers 19
(c) From Eq. (i). =
240 <50) ( ♦ „ „ ,) »<•
From Eq. (it), E 22 = 120 * ^2_* 0 5 4>„v..
(5 0 > *< *»,„,) = 6 0 V a t 50 Hz.
Id) From Eq. (i), — = — x <tW>)
240 50 M ^ , ) °r ♦™„z = 2 q „ „ 1.
For the sam e core area, the flux den t ’ a
large (refer to B -H curve) and the tran sform .,' m * * 1: ? e ma8I,etiz‘ng current becomes quite
<«) The direct current is alternating current I t ^ *m *g*d
change of flux. As a result, secondary induced e m f “ ro/!'eJuency - In other words, there is no
opposes the applied voltage V, is also zero Th f 22 F u rth e r th e counter emf. E u which
only by th e p r im a r y w in d in g re s is ta n c e S' ^ ° r8, ^ p rim a ry no ^oa(^ c u rre n t is lim ited
trem endously h ig h a n d tra n s f o r m e r w ill d e f in 'w y g V tV u ^ t.Sm a" ' C“ r r e n t ^ '*
E xam ple 1 . 6 . A n id ea l tra n sfo rm er hnc • ,.
turns on secondary w in d in g S a n d 60 tu rn s on tertiH Z S iC>2 ^ ™ S ° n Prim ary m inding P, 160
A to a resistive lo a d w hereas a pure-canacitanrp In n !? W lng ' g 1 13 (a>- W inding S feeds 10
a Pare capacitance load across w inding T takes 20 A

curnnt^is neglected. w U utb« transform er

J 6 ) W ith th e p o la r ity m a r k in g s on P as shorn*, m a rk the polarities on v in d tn g s S a n d T

1i ^ r J A aP lie n V 0' ta g e / 1 3 n d m U tual n u x 4 a re show n in th e p h aso r d iag ram of Fig.


U S <e). E m fe in d u c e d in w in d in g s S a n d T a re indicated as E 2 and E , respectively. C u rren t
I., - 10 A a n d th e re fo re se c o n d a ry w in d in g m m f F , - h V 2 = 10 x 160 = 1600 ATs is show n in
phase w ith E 2 b e c a u s e lo ad is re sistiv e in n a tu re . Likew ise, c u rre n t / 3 = 20 A and therefore
tertiary w in d in g m m f F 3 = I 3 N 3 = 20 x 60 = 1200 ATs m u st lead E 3 by 90“ as th e load is a pure
capacitance. R e s u lta n t o f load m m fs F 2 an d F 3 gives F23 = V1200* + 1600'2 = 2000 ATs

(a) (b)
Fig. 1.13. P ertaining to Exam ple 1.6.

Principle o f tr a n s f o r m e r a ctio n re q u ire s th a t p rim a ry w inding m u st balance th e load m m f


23 produced b y th e co m b in ed actio n of c u rre n ts in two w indings S and T.

Primary w inding mmf, F 1 = ^23 = 2000 ATs

S c a n n e d by C am Scanner
r
____ ___________

20 E lectrical M achinery___________________ (Art. 1.6

^ r F1 2000-9 0 4
C u rren t in prim ary winding, = joO ”

F * 1600
Power factor on th e prim ary side = cos 0j = y r - = 0 8 leading-

(6 ) P olarity m arkings and direction of currents are indicated on the secondary and tertiary
w indings in Fig. 1.13 (6 ).
1.6. E q u iv a le n t C ir c u it o f a T r a n s f o r m e r
The equivalent circuit for any electrical engineering device can be draw n if the equations
describing its behaviour are known. If any electrical device is to be analysed and investigated
fu rth e r for s u ita b le m odifications, its a p p ro p ria te eq u iv a le n t c irc u it is n ecessary . The
equivalent circuit for electrom agnetic devices consists of a combination of resistances, in uc-
tances, capacitances, voltages etc. Such an equivalent circuit (or circuit model) can, therefore,
be analysed and studied easily by th e direct application of electric circuit theory.
As stated above, equivalent circuit is simply a circuit rep resen tatio n of the equations
describing the performance of the device. Eqs. (1.20) and (1.21) describe th e behaviour of the
transform er under load. These equations are, therefore, helpful in arriving a t th e transform er
equivalent circuit of Fig. 1.14 (a). In this equivalent circuit, (rx + jx x) and (r2 + jx2) a re the
leakage im pedances of the prim ary and secondary windings respectively. The voltage Vx is
treated as a voltage drop in th e direction o f/j. Recall th a t the m agnitude of Vj does not change
appreciably from no load to full load in large transform ers. The m agnitude of Vx depends on f,
N x and $max, since | V{ | = \E X| .
The prim ary current I x consists of two components, see Fig. 1.11(a). One com ponent I x is
the load component and counteracts the secondary m.m.f. I 2N 2 completely. The other com­
ponent is exciting cu rren t I e which is composed of Ic and I m . The cu rren t 7Cis in phase w ith
Vx in Fig. 1.8 (6 ) and product Vx Ic gives core loss. The resistance Rc in parallel w ith V x rep re­
sents the core loss P., such th a t
(V/ ) 2
R,c ...( 1 .2 2 )
W
V I

and R c= ~r
t l C

The cu rren t 7m lags V { by 90° and this can, therefore, be represented in th e equivalent
circuit by a reactance X m such th a t
vy
-(1 2 3 )
Lm

R c and X m are shown in Fig. 1.14 (b), which is the exact equivalent circuit of a transform er.
The resistance R c and reactance X m are called core-loss resistance and m agnetizing reactance
respectively.
For m inor qhanges in supply voltage and frequency, which is common u n d er norm al
operation, R c and X m are tre ated constant.

In Fig. 1.14 (a) and (b), th e ideal transform er has been introduced to show the tran sfo rm a­
tion of voltage and cu rre n t between p rim ary and secondary w indings. Even a t th is stage, the
tran sfo rm er m agnetization curve is assum ed linear, since the effect of h ig h er order harm onics
can’t be rep resen ted in th e equivalent circuit.

Scanned by Cam Scanner


v Jr,v*V.
Art. 1 <>] %
'9*
Transformers 21

(a) Exciting current neglected.


iWT

1l-H
s

(6) Exact equivalent circuit.

rt i*i
-WVVV qrjffV. V - N, N,
tr JTinri— vwi— o— | r — n
•I II
•!

-
--------- —o— •!! f|
Ideal
transform er
(c) Referred to primary.
Nj N; r» J*l J* 2 r2
-VAV—TTOT- Avw—frnnr------o+

O- j 1 i o
Ideal
tra n s form er (j) Referred to secondary.
Fig. 1.14. Development of the exact equivalent circuit of a transformer.

In transform er analysis, it is usual to s“ ^ " e e d r o ^ T h C T ^ n s f e ^ e d to


primary q u an tities to secondary side. Seconda y
Primary side, m u st be m ultiplied by th e tu rn s ratio N x/ N 2.
Secondary resistance drop, when transferred to primary = (/ 2r 2) ^

'N i Putting/2 = /i^ r


N2

%
. •*?;*-*••

Scanned by Cam Scanner


I

(Art. 1.6
22 Electrical Machinery

(H ±\
N2

w here r 2 = r2 •
No
\ J T
If resista n ce r 2' is placed in th e prim ary circuit, then th e relation betw een voltages j ian
V 2 is uneffected. T his resistan ce r2 is called th e secondary resistance referred to prim ary. ere
fore, th e to ta l resista n ce in th e prim ary circuit is

= ri + r2 ...(1.24)
rt \ = r x + r2
N2
\ y
H ence r el is called th e tra n sfo rm er equivalent (or total) resista n ce referred to p rim ary
(N £ \
w inding. Sim ilarly th e prim ary resistance referred to secondary is r x and th e equivalent
\ /
(or total) resistance referred to secondary is

re2 = r2 + r l ...(1.25)
\2 x2 r2
-MW-----TfcT'- — vvw—
t+~ ri *1

Kig. 1.14. (e) Equivalent circuit in a general form.

Secondary leakage reactance drop I ^ , w hen tra n sferred to p rim ary is

'N A
I2x 2 = 11 x 2 ~ h x 2-
No
T h e q u a n tity x2 is c a lle d th e secondary leakage reactance referred to p rim a ry . T otal
p rim ary leak ag e reactan ce is

* 1) ...(1.26)
* e l = * l + * 2 No = x \ +x 2

w here xel is called th e eq u iv alen t or to tal leakage reactan ce referred to p rim a ry . Likew ise, the
eq u iv alen t or to tal leakage reactan ce referred to secondary is

'AT.'
*c2 = * 2 + *l = x 2 + Xi ...(1.27)
Ni
T he e q u iv a le n t (or to tal) leak ag e im pedance referred to p rim a ry is

^ e l ~ r el J*e 1

T h e e q u iv a le n t (or to tal) leak ag e im pedance re ferred to seco n d ary is

Ze2 —f e2 ~^JXe2‘

Scanned by Cam Scanner


^ i
«L*’..
fr
Art. Transformers 23

F o llo w in g t h e a b o v e p r o c e d u r e , it c a n b e s h o w n t h a t

z el = and 2e2 ~ 'IV,' cel.


ze2
. V ' V V
S im p lif ic a tio n o f t h e e x a c t e q u i v a l e n t c i r c u it. The equivalent circuit of Fig. 1.14 (6 )
can be sim plified by re fe rrin g all th e q u a n titie s to prim ary or secondary and a t th e sam e time,
moving th e ideal tra n s fo rm e r to one side. If th e secondary q u an tities are referred to prim ary.
the equivalent circ u it of Fig. 1.14 (c) is obtained. Since it is usu al to om it th e ideal transform er,
it is shown d o tted in Fig. 1.14 (c) for th e sak e of com pleteness. W hen th e p rim ary q u a n titie s are
referred to th e seco n d ary side, th e eq u iv alen t circuit of Fig. 1.14 (d) is obtained. Note th at
'JV ' (N a
Rc' and X m' = X m T he exact equivalent circuits of Fig. 1.14 (c) and (d) arc known
- = Rc H\
i »j j *i j
as T-circuits for a tra n sfo rm er, referred to prim ary and secondary windings respectively.
In the e q u iv a le n t circu its o fF ig . 1.14 (c) and id), the referred q u an tities w ith su itab le n o ta­
tion, have been u sed . A m ore g en eral eq u iv alen t circuit can be draw n as shown in Fig. 1.14 (c),
where for sim plicity, (i) a p a rtic u la r n otation for referred q u an tities has been dropped (//) the
complex n o tatio n (b ar over I , j w ith reactances etc.) h as been given up and (Hi) th e ideal tra n s ­
former is not show n. I f th e g en eral equivalent circuit refers to the prim ary, one has to keep in
mind th a t th e seco n d ary q u a n titie s have been referred to th e prim ary side. On the other hand,
if the general e q u iv a le n t circu it refers to the secondary, then the prim ary q u a n titie s m ust be
referred to th e seco n d ary side. T h u s in th e general equivalent circuit of a tran sfo rm er, one has
merely to keep in m in d ab o u t th e side to which all th e q u an tities have been referred.
It may be in te re s tin g a t th is sta g e to draw th e phasor diagram for th e equivalent circuit of
Fig. 1.14 (e) from a know ledge of th e electric circuit theory. Assume th a t the secondary load
voltage V2, load c u rre n t 12 an d angle 0 2 by which I 2 lags V2, are
known. F irst of all d raw I 2 lagging V 2 by an angle 0 2 and then add
h (r-i + j x 2) to V 2 to o b tain E 2 or V /, Fig. 1.15. It is obvious from
Fig. 1.14 (e) t h a t c u rre n t 7m d u e to voltage E 2 or V / m u st lag it by
90° and fu rth e r 7Cm u s t be in p h a se w ith E 2 or V / . The phasor sum
of7Cand I m gives 7e a n d p h a so r sum of 72 and Ie gives l v The volt­
age drop 7 1(r 1 + j x x) is now added to V f to obtain V, as shown in
Fig. 1.15. T he seco n d ary p.f. is cos 02 lagging and th e p rim ary p.f.
is cos 0 j lagging.
The voltage d ro p s 7 1(r 1 + jxi) an d 72(r 2 + jx2) have been draw n
to a much larg er scale, in com parison w ith Vj or V2 for th e sake of
clarity. Fig. 1.15. Phasor diagram for
A p p r o x im a te E q u i v a l e n t c i r c u i t . A pproxim ate equivalent equivalent circuit ofF ig. 1.14 Id.
circuit iso b tain ed from th e ex act eq u iv alen t circui t of Fig. 1.14 « ),
■fthe sh u n t b ran ch (Rna d Xin ^ lel) “ J 1^ 0 (6) r
or secondary te rm in a ls as show n in Fig. 1 . lb I , w hereas I e docs How through
I d 6 (a) th a t th e e x citin g currcnW ., does n ^ t f lo w ^ h r o u g h ^ a n d ^ i^ HUe to the

r i and x, in th e ex act e q u iv a le n t circuit. Thus t I , , R; , s 0 ncUla| | v.


exciting c u rre n t i e I ( r , + > , ) h a s been neglected in Fig. 1.1b to), m i g
e u rrc m , i.e., j , tr, jv th ro u g h r , and ,v2, w hereas 1, does not
;; It may also be seen from Fig. U 6 ‘ « h ccondnry |0„knge im pedance
through r2 a n d x 2 in th e e x act eq u iv alen t circuit. I hus tne . .

■i i .".ir-
iv/ci1 Mjf WV
_4III»/ V
_*V
_4III IWI
[Art. 1.6
24 Electrical Machinery
I r,+r2=.re
o------- ►- A v w -—W — %
f It
X,+X2=*e

‘ L

(a)
r,+r2=re x 1+x 2=xe
Y W — 'TRTiT'-
? T

r1+r2 =re X,+x2=Xe x 1+x2=xc


O- -W A V — ■W T— —o
+ +
I,=I2 It « I a
fi 1
V2
2 V,

(c) . . . (d)
Fig. 1.16. (a) and (b ) Approximate equivalent circuits of a transformer.
(c) and (4) Simplified forms of the approximate equivalent circuit.

drop due to Ie, i.e. I e (r2 +jx 2) h as been included, though Ie (r 2 + ,/x2) is actu ally zero. Since th e
exciting cu rre n t is only about 2 to 6 per cent of th e ra te d w inding c u rre n t in pow er an d d istrib u ­
tion tran sfo rm ers, th e e rro r introduced by neglecting I e (rj + jx 1) or including / e(r 2 + j x 2) is in sig ­
nificant. However, th e com putational labour involved is reduced considerably by th e use of
approxim ate equ iv alen t circuits of Fig. 1.16 (a) and (6 ). As before, one m u st k eep in m ind about
the side to which all th e equivalent-circuit q u an tities have been referred.
Still fu rth e r sim plification is achieved by neglecting th e s h u n t b ran ch R c a n d X m in Fig. 1.16
(a) and (6 ) an d th is lead s to eq u iv alen t circuit of Fig. 1.16 (c). T his sim plification is ta n ta m o u n t
to neglecting exciting c u rre n t Ie in com parison w ith ra te d c u rre n ts, w hich is a lm o st ju stifiab le
in large tra n sfo rm ers, say over 100 kVA or so. F or tra n sfo rm e rs h av in g ra tin g s n e a r 500 kVA
or more, th e eq u iv alen t resista n ce re is quite sm all as com pared w ith e q u iv a le n t leak ag e reac­
tance x,. C onsequently re m ay be neglected, leading to th e e q u iv a le n t c irc u it of Fig. 1.16 (d).
Thus, w hen a large pow er system is stu died, a tra n sfo rm e r is u su a lly rep laced by its equivalent
circuit of th e form show n in Fig. 1.16 (d).
T he eq u iv ale n t circu it of Fig. 1.14 (e) should be used only w h en th e ex citin g c u rre n t is a
large p ercen tag e of th e ra te d c u rre n t e.g., in audio-frequency tra n s fo rm e rs u sed in electronic
circuits, in tra n sfo rm e rs used for relay in g an d m e a s u re m e n t p u rp o ses etc. F o r h ig h voltage
su rg e in v estig atio n s, th e tra n sfo rm e r e q u iv alen t circ u it m u st be m odified to in clu d e th e effects
of in te r-tu rn an d tu r n to e a r th capacitances.

Scanned by Cam Scanner


,, 25 T r a n sfo rm ers
— "
The e q u iv alen t c irc u it developed in Fig. 1.14 can also be arrived a t by following th e classical
v o f m a g n etically coupled circu its. T h is is done in A rt. 1.16.
g x a m p le 1.7. For a tw o w in d in g transform er, derive an expression for the total ohm ic loss,
■ terms o f the e q u iv a le n t resistance, referred to either side.
lfl S o lu tio n . T o tal ohm ic loss in a tra n sfo rm e r
Poh = ir i + ^2% ) w a tts
Expressing I 2 in te rm s of l x gives

Ni
P o h ~ I ir i + h r2 = l \ r \ + r 2 = /? r el
N2
v yj-

If P0h *s ex p ressed in te rm s of I 2, th e n

% )
r, + r 9
p ah = r l + ^2r 2 “ h 2

Thus th e to ta l ohm ic loss in a tra n sfo rm e r = (E quivalent resistan ce referred to e ith e r side)
(Square of th e c u r re n t on t h a t side). .
E x a m p le 1.8. A 33 kV A , 2 2 0 0 /2 2 0 V, 50 H z single phase transform er has the follow ing

PaT7 r i Z Sry w in d in g (h.v. side) : resistance r , « 2 .< f t leakage reactance x , = 6.00 f t


Secondary w in d in g (l.v. side) : resistance r , =f t leakage reactance * 2 = 0.07 f t
(a) Find the p r im a ry resistance a n d leakage reactance referred to secondary.
lb)Find the secondary resistance a n d leakage reactance referred to p rim a ry
(c) F ind the eq u iva len t resistance a n d equivalent leakage reactance referred to (D p rim a ry
and (ii) secondary.
fiO C alculate the to ta l o h m ic loss a t fu ll lo a d ^ ^ ^ ^ ^ ^ ^ ^ ^

J l u } m A M T w i l n g aUnderthese conditions, fin d the pow er in p u t also.

S o lu tio n , (a) P rim a ry re sis ta n c e re ferred to secondary


2
f* Z) = (2.4) ' 2 2 0 > = 0.024 Cl.
2200 f
v /
■*
-
I

(N2 * =
„ ( 220 ) = 0.06 Cl.
6.00
= x l' = x l 2200

(6) Secondary re s is ta n c e re fe rre d to p rim a ry


(2 2 0 0 = 3 .0 0 fl.
= 0.03
= r2 = r2 N* I 220

S eco n d a ry le a k a g e r e a c t a n c e referred to p
* n
( N j ) = 0.07 (10 )2 = 7 00 Cl.
= x2 = x 2 N 2

UUUI II It^VJ uy WCJI I IOUC1I II It^l


26 Electrical Machinery (Art. 1.6

(c) (i) E quivalent resistance referred to primary


= r cl = r l + r 2' = 2.4 4 3.00 = 5.4 £2.
E q u iv a le n t le a k a g e re a c ta n c e re fe rre d to p rim a ry
= xcl = 4 x 2' - 6.00 + 7.00 = 13 £2.

(ii) E q u iv a le n t re sis ta n c e re fe rre d to secondary


= r e2 = r 2 + r { = 0.03 4 0.024 = 0.054 £2.
r*i *ei
E q u iv a le n t le a k a g e re a c ta n c e re fe rre d to secondary 4 O- '‘806s -
= xe2 = x 2 + x x = 0.07 4 0.06 = 0.13 £2. 5-4/1 13/1
33000
(d) P rim a ry full load c u rre n t I x = 2 2 0 6 ” = ^ ^ 16 A' 1

33000
S eco n d ary full load c u rre n t / 2 = = 150 A. -o -
220
O hm ic loss a t full load = f \ r eX = (15)2 x 5.4 = 1216 w a tts, Fig. 1.17. E q u ivalen t circuit
referred to h.v. sid e, Ex. 1.8.
or = t i re2 = (150)2 x 0.054 = 1216 w atts.
(e) A c u rre n t o f 160 A in th e l.v. w inding is eq u iv alen t to 16 A in th e h.v. w inding. The
e q u iv a le n t circuit of th e tra n sfo rm e r, referred to h.v. side is illu s tra te d in Fig. 1.17, from w hich
e q u iv a le n t leak ag e im pedance referred to h.v. side is
= z eX = 5.4 4 7 13
or z el = >/(5.4 )2 + (13)2 = 14.08 Q.
.-. T h e voltage to be applied to th e h.v. side, V = (16) (z fl)
= (16) (14.08) = 225.28 volts.
Pow er in p u t = 72 rel = (16)2 (5.4) = 1382.4 w atts
5.4
or Pow er in p u t = V I cos 9 = (225.28) (16) x = 1382.4 w atts.
14.08
E x a m p le 1.9. A 10 kVA, 2 5 0 0 /2 5 0 V, single phase transform er has resistances a n d leakage
reactances as follows :
rj = 4.8 £2, r2 = 0.048 £2
Xj = 11.2 £2 , x2 = 0.112 £2,
Subscrip ts 1 an d 2 denote high voltage a n d low voltage w in d in g s respectively.
W ith p rim a ry supply voltage held constant at 2500 V, calculate the secondary term inal
voltage, w hen
(a) the l.v. w in d in g is connected to a load im pedance o f 5 + j 3.5 £2.
(b) the transform er delivers its rated current a t 0 .8 p.f. lagging on the l.v. side.
S o lu tio n , (a) All th e q u an tities m ay be referred to e ith e r th e h.v. side or th e l.v. side. In
th is exam ple, l.v. w inding is th e secondary w inding since load is connected across it. W ith all
th e q u a n titie s referred to th e l.v. side, th e equivalent circuit o fF ig . 1.18 (a) is obtained, where
( N ,' r n
re2 = r2 + r x = 0.048 4 4.8 = 0.096 £2
N1 10
J
(No
and X'2 =*2 + *l = 0.112 4 11.2 = 0.224 £2.
N1 10

Scanned by Cam Scanner


Transformers 27

0 096A 0-224H
o -VWA-
+
IZ=40A
TJ
250 V (C
o

(6 )
, . . F lg ' U 8 ' E q u ivalen t circuit for Exam ple 1 9
The transformer leakage impedance r -l j i.u i ,.
fore, the total im pedance in th e secondary w i n d C c t c u it is "* SerieS’ T W
2 = 5.096 + j 3.724 = 6.31 Z 36.2°.

Hence th e lo ad c u r r e n t, I 2 = ~250 = 39<65 A


b.31
The se co n d a ry te r m in a l v o ltag e E 2 is V 2 = I 2Z [

= (39.65) [>/(5)z + (3.5)2] = (39.65) (6.1) = 242 V


(b) T he e q u iv a le n t c irc u it w ith all th e q u a n titie s referred to
l.v. side is g iv en in F ig. 1.18 (6 ). T h e r a te d c u rre n t on th e 1 v side
is
T 10,000 ^

2 250 “ A
The p h a so r d ia g ra m for Fig. 1.18 (6 ) is illu s tra te d in Fig. 1.18
(c).
From th e g e o m e try o f th is diagTam ,
O B2 = OA2 + A B 2
or (250) = >/(V 2 cos G2 + l>fe2)2 + (V 2 sin 02 + 12 x e2)2 Fig. 1.18. (c) Phasor diagram for
circuit o f Fig. 1.18 (b).
or (250) = V(0.8 V 2 + 3.84)2 + (0.6 V 2 + 8.96)2
or (250)2 = V 2 + 16.90V2 + 95.55
or V \ + 16.90 V 2 - 62,404 = 0
Tr - 1 6 .9 0 + V286 + 249,616
Vo = 1 = 241.55 V.
2“ 2
A ltern ativ ely , th e se c o n d a ry te rm in a l voltage V2 can be obtained as follows:
From th e p h aso r d iag ram , it m ay be seen th a t OD is approximately equal to OB = 250 volts.
V 2 = O D - C D = 250 - CD.
Now CD = C E + E D
= I 2re2 cos 0 2 + I ^ e 2 si 11 02
= (3.84) (0.8) + (8.96) (0.6) = 8.448 = 8.45 volts.
S econdary te r m in a l vo ltag e,
V 2 = 250 - 8.45 = 241.55 V.
The m a g n itu d e o f th e se c o n d a ry v o ltage V 2 tu rn s out to be sam e in both th e m ethods. How­
ever. the c o m p u ta tio n a l la b o u r in th e second m ethod is less th a n in th e firs t m ethod, th erefo re,
e8econd m eth o d sh o u ld be p re fe rre d .

S c a n n e d by Cam S canner
28 Electrical Machinery (Art. 1.6

02 0-7
E x a m p le 1.10. The equivalent circuit referred,
to the low -tension side o f a 25012500 V single
ph a se tra n sfo rm er is show n in Fig. 1.18 (a). The
lo a d im p e d a n c e connected to the hig h -ten sio n
te rm in a ls is 380 4 j 230 £2. For a p rim a ry voltage
o f 250 V, com pute
(a) the secondary term in a l voltage,
(b) p rim a ry current a n d power factor, a n d
(c) pow er o u tp u t a n d efficiency. Fig. 1.19. (a) Transformer equivalent circuit for
Exam ple 1.10.
S o lu tio n , (a) T he load im pedance referred to
l.t. side is o-2 a o-7n
\2 w vv-
( N j)
Z f = (380 + j 230) = (3.8 + j 2.3) £2.
~n2
T ran sfo rm er leakage im pedance 0 .2 4 y'0 .7 ohm Vf250V
and th e load im pedance 3.8 + j 2.3 £2 are in series as
show n in Fig. 1.19 (6 ). Therefore, total series im ­
p edance is 4 +./3 = 5 Z 36.9°. -o
T aking V x as th e reference phasor, Fig. 1.19. (fc) Pertaining to Exam ple 1.10.

V x = 250Z 0°.
j , 250Z0°
7> - 5Z 3.69° ~ 5° Z _ 36 9
= 50 (cos 36.9° - 7 sin 36.9°) = (40 - j 30) A.

or I f = 50 A and / 2= / , ' ^ . 5 0 x i = 5A
N. 10
.-. Secondary term in al voltage = I 2 Z L
= 5 [3802 + 2302]1/2 = 5 x 444 = 2220 V
- V, 2 5 0 / 0°
(6 ) The core loss cu rren t Ic = — = 5 Qq ^ Q0 = 0.5 Z 0° = 0.5 + j 0

- Vi 250 Z 0°
The m agnetizing c u rre n t Im = = 2 5 0 / 9 0 ° = 1Z “ 90° = 0 ~ J 1-

Exciting c u rren t Ie = Ic + I m = (0.5 - 7 I) A.


H ence to tal p rim ary c u rre n t I x - I f + I e
= (40 -730) + (0.5 - 7 I) = 40.5 - 7 31 = 51 Z - 37.4°
P rim a ry cu rre n t 7X= 51 A
and p rim ary p.f. = cos 0j = cos 37.4° = 0.794 lagging.
380
(c) Load p.f. = cos e 2 = = 0.855
[380242302]1/2
Pow er o u tp u t = V2 12 cos 0 2 = 2220 x 5 x 0.855 = 9500 w atts.
Also pow er o u tp u t = ( I f) 2 R l = (50)2 (3.8) = 9500 w atts.
V? (250)2
Core loss = 125 w a tts
Pc R. 500
or Pc = TCR C= (0.5) (500) = 125 watts.

I
Scanned by Cam Scanner
I

Art. I J i
Transformers 29

Also pow er in p u t = P ow er + l ' * ^ 0 ' 7 9 4 ^ 1 0 ' 125


= 9500 + 125 + 500 = 1 0 , 125 w a to
Efficiency = ^E ^E E t _ In p u t - I ^ RgpC
In p u t InrMii
= l _ ^ o s s e s _ i 5 0 0 + 125
In p u t 10,125 - 0.9383 p.u. or 93.83%.
1.7. O p e n -circ u it a n d S h o r t-c ir c u it T ests
These tw o te s ts on a tra n sfo rm e r hnin i j * .
circuit of Fig. 1.16, (ii) th e v o lta g e ’r e e u l a t f i tb e .Param et®>-s of th e equivalent
param eters can also be o b tain ed from tbp nh • ??• efficiency. The equivalent circuit
winding d etails. C o m p lete a n a ly sis th e f c dlm ensions «f l !>» transform er core and its
circuit p a ra m e te rs a re know n. nS orm cr can carried out, once its equivalent

in th ?tran Ifo rm eqrUired dU ring ‘W° teSts is « JuaI «>e appropriate power loss occurring

O pen c ir c u it (o r N o -lo a d ) t e s t Tbn pi™,;* r e .


a single p h ase tra n s fo rm e r is given in Fig. 1,20 fa). f a f a . s 'd i a ^ a T r f o h m e f a r w ^ m ltT r
and an am m ete r a re show n connected on th e low voltage side of the t r a n s f o r m e V X w g h
vo age side is left open circuited T he rated frequency voltage applied f a X p r l a r y ■e low
rtw l ‘V w l W1. j i P °f a variable ratio auto-transform . W hen the voltm eter
recorded ’ ' V' Winding' a" th e th re e - ^ tr u m e n t r e a d i n g s ^ j

Auto-

vr vi H .
t
(b)
Fig. 1.20. (a) Circuit diagram for open-circuit test on a transformer and
(b) approxim ate equivalent circuit at no load.
The am m eter reco rd s th e no-load c u rre n t or exciting cu rren t Ie Since Ie is quite sm all (2
to 6 % of rated c u rre n t), th e p rim a ry leakage im pedance drop is alm ost negligible, and for all
Practical p u rposes, th e ap p lied voltage Vj is equal to th e induced e.m.f. V /. Consequently, the
equivalent circu it o f Fig. 1.14 (e ) gets modified to th a t shown in Fig. 1.20 (6 ).
The in p u t pow er given by th e w a ttm e te r read in g consists of core loss and ohmic loss. The
exciting c u rre n t b ein g a b o u t 2 to 6 p er cent of th e full load current, th e ohmic loss in the
• ' ( 2 2 N
Primary ( = le2r x) v a rie s from 0.04 p er cent 100 100 x 100 to 0.36 per cent of th e full-load
Primary ohm ic loss In viow of th is fact, th e ohmic loss during open circuit te st is negligible
comparison w ith th e n o rm a l core loss (approxim ately proportional to th e sq u are of the
“PPlied voltage) H ence th e w a ttm e te r read in g can bo taken as equal to tran sfo rm er core loss.

W IIIvVVatUI II IWI
I Art. 1.7
30 Electrical Machinery

A negligible am o u n t of dielectric loss may also exist. E rror in the in stru m e n t readings may be
elim in ated if required. Let
Vj = applied rated voltage on I t. side,
Ie = exciting cu rren t (or no-load current),
and Pc = core loss.
T hen Pe = V x Ie cos 0 O ...(1.28)

No load p.f. — C08 9 , = y~J

From th e phasor diagram ofFig. 1.8 (c), it follows th a t


I c = Ic cos 0 Oand /„, = Ie sin 0 O

From Fig. 1.20 (6 ), 7C.=

Vi vx
Core loss resistance R CL - -=- = _
Lq COS

Vi2 _ Vj2 ...(1.29 a)


Vj7e cos 0O Pc
Also Ic R C L = Pc

R
K rt - —n
P‘ - ...(1.29 b)
(Ie cos 0 O)
M agnetizing reactance,
...(1.30)
Im Ic sin 0 O
The su b scrip t L w ith R c an d X m is used m erely to em phasize th a t th ese values are for th e
l.t. side.
T his m u st be k ep t in m ind th a t th e values of R c and X m, in general, refer to th e side, in
which th e in stru m e n ts are placed (the l.t. side in th e p resen t case). A v o ltm eter is som etim es
used a t th e open-circuited secondary term in als, in order to determ in e th e tu rn s ratio.
T h u s th e open-circuit te s t gives th e following inform ation : (i) core loss a t ra te d voltage and
frequency, (i i ) th e s h u n t b ran ch p a ram eters of th e equivalent circuit, i.e. R c an d X m and (i i i )
tu rn s ratio of th e tran sfo rm er.
S h o r t - c i r c u i t te s t. The low voltage side of th e tra n sfo rm er is sh o rt-circu ited a n d th e in­
s tru m e n ts a re placed on th e high voltage side, as
illu stra te d in Fig. 1.21 (a). T he applied voltage is
a d ju ste d by a u to -tra n sfo rm e r, to circu late ra te d
c u rre n t in th e h ig h voltage side. In a tra n sfo rm er,
th e p rim a ry m .m .f. is alm o st equal to th e secon­
d a ry m .m .f., th erefo re, a ra te d c u rre n t in th e h.v.
w in d in g c a u se s ra te d c u rre n t to flow in th e 1.
w inding.
A primary voltage of 2 to 12% of its rated value
is sufficient to circulate rated currents in both
H.V. L.V
primary and secondary windings. From Fig. 1.21 Fig. 1.21. (a) Connection diagram for short circuit
(b), it is clear th a t th e secondary leakage im­ test on a transform er.
pedance drop appears across the exciting branch

Scanned by Cam Scanner


Ar‘ 1.7]
Transformers 31
(Rt and Xm in p arallel). A bout h a lf ( ~
leakage im pedance
!eakage im pedance an a n dd ,, th erefo re,
th erefo re aacross
r m .. th e excitln
6 appllod
^ vo)tag-
^ gC appcars across th e secondary (9
across the exciting b ran ch an d since th e la tte r is ^ 3" ^ The “ re " u x induces th e voltage
B ( » of its ra te d value. H ence th e core loss b e W *>» “ re flux is also 1
the core flux, is 0 .0 1 p ercen t = —1— v 1 A Xlma e y proportional to th e sq u are of
j ii 100 l o o X 0.36 percent ^ ^ ^
value at rated voltage. T h e w a ttm e te r in shnri J lo o x io o x 100 of its
c irc u it test, records th e core loss an d th e « w
loss in both th e w indings. Since t h e ™ ' *2
has been proved to be alm o st negligible in com
panson w ith th e rated -v o ltag e core loss the
wattmeter can be ta k e n to re g iste r o n lj the
ohmic losses in both th e w indings.
o
. * tHe excibing cu rren t is 2 Rc' -c
to 6% of full load c u rre n t. W hen th e voltage ‘— T to
across the excitine
exciting bh ran
r a nch
4 is -i.t-.o
1 tn m~ . ag<: J?;----------------
«►
-------------------------- 1 ______ ____
voltage, the exciting c u rre n t m a y be'o ( J n e ^ ^ Equivalent circuit with short-circuit on the
( 2 1 Per- secondary side.
cent x 100
100 100 to 0.36% p e rc e n t r\ X, *2 r2
j- vm — W — -flflTIP----- WA
6
* Tlsc
*
100 x 100 x 100 of its fu ll-lo ad c u rre n t — J __
and can, th e refo re , be safely ignored As * ysc
result of this, the eq u iv alen t circuit ofFig. 1.14
o
(c), with th e secondary sh o rt-circu ited gets ■-r — jC
if)
modified to th a t shown in Fig. 1.21 (c) *
The in s tr u m e n t r e a d in g s m ay be cor­ Fig. 1.21. (C) Transformer equivalent circuit with
rected,
, if required.
-
Let Vsc, / ; and *p sc be
■sc> -sc ut= the
me
secondary short-circuited.

p e d a t e S S rhavndsM :U m eter F* ' »■» equivalent ,eakage im-

z eH =
...(1.31 (a ))

equivalent resistance referred to h.v. side, reH =


...(1.31 (6 ))
and equivalent leakage reactance referred to h.v. side,

In XeIi ~ ~ r*H ( 1 32)


h.v, side//T w ! nd 2e" ’ tIie SUbSCFipt H iS USGd t0 in d k ate th at these Quantities are referred to
In th ? P aram eters can, however, be referred to the l.v. side, if required
equivalontT 3 1 ° f tran sfo rm er equivalent circuit, the values of equivalent r e s is ta n t
pe* n c e p a a m e te ra f o r h T h " referr<!d a " ‘“ “ I Side’ ^ USed' H ° WeVcr’ if th e ' “ k “ ge ™ fa
take r, - r _a ™e te r s for b o th P n m a r y a n d seco n d ary a re req u ire d se p a ra te ly , th e n it is u s u a l to
1 2 - 2 re and x x = x 2 = -xe, referred to the same side.

and freon’ ^ 6 sho^ - c.i rcu‘t te st gives the following information : (i) ohmic loss at rated current
ncy and (u) th e equivalent resistance and equivalent leakage reactance.
^rcuit t age^ e^u,ation of a transform er can be determ ined from the data obtained from short-
Paramet 3 ° f both °Pen -circuit and short-circuit tests is necessary (i) for obtaining all the
\ _ _ _ j T s o f exact equivalent circuit and (ii) for calculating the transform er efficiency 1 1
**1^ h * - * *
orl-circuit current IIC should be, either equal to or near to the rated current. 1
iii
I
Scanned by Cam Scanner
32 Electrical Machinery

How can a w attm eter connected on the h.v. side, record the ohmic loss in the l.v. winding also?
This question m ay spring up in the mind of some readers. A little thought process provides the
explanation for this question. When rated current is made to flow in the h.v. win mg, he Lv.
winding m ust also carry rated current, because the transform er action requires j i - 2^ 2 - The
flow of rated cu rren t in the l.v. winding causes ohmic loss, which m ust be supplied from somewhere.
The only way to provide l.v. winding loss is from the input to h.v. side. B ut the entire input power
to h.v. side is recorded by the w attm eter, therefore, the ohmic losses in both the windings are given
by the w attm eter reading.
It h as alread y been stated th a t open-circuit and short-circuit tests should be performed on
th e l.v. side and h.v. side respectively only for the sake of convenience. This can e 1 us rated
by considering a 3300/220 V, 33 kVA, single-phase transform er.
F or open-circuit te s t on low voltage side, the ranges of voltm eter, am m eter and w attm eter
are 220 V (rated value), 6 A (2 to 6 % of rated cu rren t of 150 A) and 6 A, 220 V respectively. These
are th e sta n d a rd ranges for ordinary in stru m en ts and, therefore, more accurate readings can
be obtained. If th e open circuit te st is performed on th e h.v. side, a source of 3300 V may not be
readily available. At th e sam e tim e, the in stru m en t ranges are 3300 V, 0.4 A and 0.4 A, 3300 V
w hich are not w ithin th e range of ordinary instrum ents and th e resu lts obtained may not be so
accurate. Also it m ay not be safe to work on the high voltage side.
For a short-circuit test on the h.v. side, the in stru m en t ranges are 165 V (2 to 1 2 % of rated
voltage of 3300 V), 10 A (rated current) and 10 A, 165 V, which are well w ithin the range of the
o rd in ary in stru m en ts. On the other hand, in stru m en t ranges, for a short-circuit te st on l.v. side
are 11 V, 150 A, and 150 A, 11 V. In stru m en ts of such ranges and auto-transform er capable of
han d lin g 150 A, may not be readily available and a t the sam e tim e, the resu lts m ay not be so
accurate. It is for these reasons th a t the open-circuit and short-circuit te sts are conducted on
l.v. and h.v. sides respectively.
E x a m p le 1.11. A 20 kVA, 2 5 00/250 V, 50 Hz, single-phase transform er gave the following
test r e s u lt:
Open-circuit test (on l.v. side) : 250 V, 1.4 A, 105 watts.
Short-circuit test (on h.v. side) : 104 V, 8 A, 320 watts.
Compute the param eters o f the approximate equivalent circuit referred to high-voltage and
low-voltage sides. Also draw the exact equivalent circuit referred to the low-voltage side.
S o lu tio n . From open-circuit t e s t :
105
No-load power factor = cos 0O= 25Q x ^ 4 = 0.3

0O= 72.55° and sin 0O= 0.954.


Ic = Ie cos 0O= 1 .4 x 0 .3 = 0.42 A
and I m = I e sin 0O= 1.4 x 0.954 = 1.336 A.

Hence

A lternatively, th e value of R cL and X ^ can be determ ined as follows :

Scanned by C a m S ca n ner'
Transformers 33

, = v > 250 = 0.42 A


R cL ~ 595
l m = V/j -Tj? = V( 1.4 )2 - (0.42)a = 1.336 A
V i_ 250
= 187 12.
/ m 1.336
. Vsc 104
prom sh o rt-circu it t e s t : z ew - — - g - 1 3 1 2

/ , t2(8 )
. ^ H = V r,„:' - r , „ :I=V(13)2 - 5 z = 1 2 n
Equivalent circu it p a ra m e te rs referred to l.v. side are
R cL = 595 12 X mL = 187 12.
. \2
' i '2 = 0 .1212 .
^ L = 5 10 = 0.05 12; = 12
10
v /
This eq u iv alen t circu it is show n in Fig. 1 .2 2 (a).

0-121) 0 -0 5 0 o-0 6 n
0 -0 2 5 0
—7J7JTOT— VvV-----o °— VW—'TOT— —T O— w — o
006C1 0 -0 2 5 0

595 595
1870
O n

(6 )
F ie 1 22. (a) Approximate equivalent circuit referred to l.v. side and
(fc) exact equivalent circuit referred to l.v. side.

Equivalent circu it p a ra m e te rs referred to h.v. side are ■Ss'i


R cH = 595(10 )2 = 59,500 12
X mH= 187(10)2 = 18,700 Q
I--
reH = 512 ; x tH = 1212. I
An equ iv alen t circu it show ing th e se p a ra m e te rs can easily draw n.
Exact eq u iv alen t circu it p a ra m e te rs referred to . •
r u = r , 'H = i r (i = l<0.05) = 0 .0 2 5 n

/ _ 1 V r = -f 0 1 2 ) = 0.0612
XlL = * l W - " 2 * ' L ’
R cL = 595 12 and X mL = 187 12.

L o X;.e tfk ^ im O N O O 7 'I ^ r gave


following test results :
Open circuit t e s t : 400 V, 9 A, 1-50 k .
Short circuit t e s t : 3 5 0 V, ra ted current, ■ ' .rf its efficiency a t h a lf
Calculate the eq u iva len t circu it p a ra m eters referred to the n
fall load o f u n ity p o w er fa cto r.

S c a n n e d by C a m S c a n n e r
: 34 Electrical Machinery (Art. 1.7

S o lu tio n . Problem s re la tin g to 3 -phase balanced system are solved by reducing all the
q u a n titie s to per p h ase values and so is done here.
O p en -c ir c u it te st. T his circuit is perform ed on th e l.v. side, since th e applied voltage for
th is te s t is equal to th e ra ted voltage on the l.v. side, which is s ta r connected.

.\ P er p hase applied voltage Vj = = 231 V.

P er p h ase exciting c u rren t l e = 9 A.

P er p hase core loss *P„c -= ^ _P = 500 W.

Now V J e cos 0 O= Pc

Core loss cu rren t = / ,c o S e0 = / c = ^ = f f = 2.165A.

M agnetizing cu rren t I m = V/ 2 - I 2 = V92 - (2.165)2 = 8.73 A.


Vi 231
From Eq. (1.29), Rc, = -r 1 = = 106.8 Q
Ic 2.165
Vi 231
From Eq. (1.30), ‘■m.L = 26.47 £1
Im 8.73
Core loss resistance referred to h.v. side
/"Per phase voltage on h.v. sid e "
cH ~ cz. p er pbase voltage on l.v. side

= 106.8 i i f S ) =242.2 kn.

i,ooo Y
i 11,000
„ = XxL
. = (26.47) = 60.02 kO.
231 231
S h o r t- c ir c u it te s t. This te st is perform ed on h.v. side, since 350 V is a fraction of th e rated
voltage on h.v. side, which is in delta.
.*. Applied voltage/phase Vac = 350 V
200,000
C urrent/phase / „ = R ated cu rren t = ,_ . = 6.06 A.
tc 3 x 1 1 ,0 0 0

Ohmic loss p e r p hase » .3 P .T O O W

350
z eH = = 57.8 O
6.06
8C 700
= 19.06 n
A SC (6.06)'
.. xeH = V(57.8 )2 - (19.06)2 = 54.6 Cl

Efficiency a t h a lf full load 1) = 1- - r — P e r P h a s e losses -------------------


- P er phase o u tp u t + P e r p h ase losses
500 + (|jp(700)

= 1-
1 (2 0 0 (1 )
x 1000 x 1 + 500 + (700)
2[ 3 2
V

Scanned by Cam Scanner


Transformers 35

_ 675 „
" 34,008 = 9 8 0 2 P u - or 98 02%.
g The Per U n it S ystem
for electrical m ach in es or electrical m achine system s, the various calculations are com­
monly done by u sin g e p er u n it q u an tities. In this article, the method of expressing volts,
amperes, ohms etc. in p er u n it q u an tities, is explained.
For any system , th e p er u n it values are obtained by choosing a set of certain base values,
which can be chosen a rb itra rily . However, it is common and preferable to choose the rated
quantities of th e device as its base values.
For any system or device, th e p er u n it q u an tity is related to the base quantity by the follow­
ing relation.
P er u n it q u a n tity = value of the quantity in any unit
b a s e value of th a t quantity in the sam e unit
In this article, a tte n tio n is directed to single phase 2-winding transform er, for which the
base quantities a re selected as follows. The subscripts B, L and H denote base, l.v. side and h.v.
side respectively.
For high voltage side :
Base voltage VBH = R ated voltage on h.v. side
Base cu rren t IBh = R ated cu rren t on h.v. side
T ransform er rated VA -
R ated voltage on h.v. side
. , „ V BH „ V BH * V BH (rated voltage on H V side )2
Base im pedance = £2 = rated VA
v22
Dividing th e n u m e ra to r and denom inator by (1000) , we get
( V
VBH f
1000 (kV)BH
— ...(1.33a)
BH ra te d VA (.MVA)Bu
( 1 0 0 0 )2
A ctual value of reH, xeH and zeH in ohms ^
**• (reH* x eH> z eH)p.u. ~ ZBB in ohms

For low-voltage side :


Base voltage VBL = ra te d voltage on low-voltage side.
Base cu rre n t IBl = ra te d current on l.v. side
Ratpd VA of transform er
= R ated voltage on l.v. side
_ Vm . voltage on L V side)
Base im pedance Z BL = 7 — ft = r a ted VA
l BL
Zbl, as per Eq. (1.33a), can also be expressed as

(kV fBL ...(1.34a)


^J M V A U . ,
A ctual v alu e of y . »■> and z , i m ohms (1 346)
■ .•• x tL> z tL)p.u. “ Z bl ^ ohms
■ rwV' . .
•■i
A
Scanned by Cam Scanner
36 Electrical Machinery j^ r<-

_ . . . . .. R ated voltage on h.v. side _ 1


P e r u n it ra te d voltage on h.v. side = -
V BH
.. R ated c u rre n t on h.v. side _ 1
P e r u n it ra te d c u rre n t on h.v. sid e= — ; '
lbh
T h u s th e p e r u n it v alu es of th e ra te d q u a n titie s of a device are equal to u n ity . N ote also that
th e p e r u n it q u a n titie s a re dim ensionless w hereas th e base q u a n titie s hav e a im ension.
z cL in ohm s _ zeL I bl
Now {Zcl)p-u- = Z BL in ohm s " Z BLIBL
Voltage drop across zeL a t ra te d cu rre n t n ^
=' VflZ '
Eq. (1.35) show s th a t th e p.u. value of z eL (or zeH) gives inform ation about th e leakage im­
pedance drop on th e l.v. (or h.v.) side. For exam ple, in a single-phase tra n sfo rm e r of 2300/230
V h av in g z e^ = 0.03 p.u., th e voltage drop across th e h.v. leakage im pedance will be 3% of 2300,
i.e. (0.03)(2300) = 69 volts.
reL in ohms
Also (reL)P.u. = Z bl in ohm s
reL h L _ rtL JBL _ h i? reL
°r %bl I bl V bl V bl I bl
Ohmic loss a t rate d c u rre n t (136)
(reL)P.u. - R ated VA
T h u s th e p.u. value of reB or reB also furnishes th e inform ation about th e ohmic or resistance
loss a t ra te d c u rre n t occurring in th e tran sfo rm er. For exam ple, in a single-phase tra n sfo rm er
of 10 kVA, 2300/230 V h av in g reH = 0.018 p.u., th e ohmic loss a t ra te d c u rre n t is 1.8% of 10 kVA
or (0.018)(10,000) = 180 w atts.
From E qns. (1.33) an d (1.34), it can be w ritten th a t
^ Z (Q )(M V A ) b
Z ,p u )= ( * V ) |—

In case base values (kV)B and (M VA)B are modified, th en we get


_ % Z (Q )(M V A ) b old
Z (pu)cld ------ — 2
( « v ) b • old
Z (H) (M VA) b . new
Z(pu>™ = ( k V )■new
l~

. .. Z (pu)neui (MVA)Bnew (kV)2B d d


T aking th e ir ratio s, — -— = , %/nrA.-------- —t t ? -------
Z {p u )old (MVA)Bold (kV)%.new

rj t_ \ _ rj , x (MVA)fl . ^ (kV)B old n rtf}


Z (Pu )new ~ Z (pu)oid - m A ) B M ■ikV )i -< •
new

E x a m p le 1.13. Sh o w th a t the p er-u n it values o f reL, x eL a n d z eL are equal to the per-unit


values o f reH, xeI1 a n d z eH. S u b scrip ts L a n d H denote l.v. a n d h.v. w in d in g s respectively.
r.j in ohm s
S o lu tio n . (rtL)p.u.= Z m in ohm s •• (0

Scanned by Cam Scanner


Art. L2L
Transformers
But

where N L and N „ a re th e n u m b e r of tu rn s in th e 1 j
tion of the v alue of reL in Eq. (i) gives : V 3n ^ V- w*nchngs respectively. Substitu-

_ r eH 2
(rcl)p.u. = 'cH = r eH
v HJ
n h ZBn = Z BL ( n h )
(N Z bh N,
JBL
N,
= (r ew)p.u.
Similarly (xcl)p.u. = (xeH)p u and (z
=(z )
Hence th e req u ire d re s u lt h as been proved. ' P'“ "
E x a m p le 1.14. ( a ) A 2 0 k V A , 2 5 0 0 /2 5 0 V sinnlp n hn.a , /• t
impedance o f z ,H = 2 .6 + j 4.3 n w hen referred to A 1 p e‘>uiJl al ent Uaka8e
Ci) h.v. side a n d (ii) l.v. side. F in d also the total leakage im d " to
,,. jr., . r r leakage impedance drop referred to each side.

J e o n t h e ( i^ T d J n d ^ u J . t S T ^ °f 4 8 A the ^ »P*-
S o lu tio n , (a) (i) H .V. side :
Base voltage VBH= 2500 V
Base c u rre n t I BH = ^0.000 = 8 A.
2500

Base im pedance Z BH = = 3 1 2 .5 D
Ibh 8
Per u n it v alue of eq u iv ale n t leakage im pedance referred to h.v. side,
z eH in Q 2 .6 + i 4 3
^z■ ^ a ■ - si ir - 0 0 0 8 3 2 +J' ° 01376-

(ii) L.V. side : E q u iv alen t leakage im pedance referred to l.v. side


2 2
z eL z eH
(N r)
~ z eH
M
= (2.6 + j 4.3)
f 250 )

\
Nh
w \
2500
/
= 0.026 + j 0.043 a

Now VBL
' b = 250 Vl ,/flL = ^ 250
® = 80 A.

■n • 1 " ^BL _ 250 _ q iq c n


•■Base im pedance Z BL = -j— = qq
l BL

_ _ 0 ^ 2 6 + ^ M l l _ 0.00832 + ; 0.01376.
(z eDp.u. ~ g jn £i 3.125

The m a g n itu d e o f or = t / p M M ) * * (0.01S76) = 0.01607

Total le a k a g e im p e d a n c e d rop on h .v . sid e


= (0.01607) (2500) = 40.175 V
^ total le a k a g e im p e d a n c e d rop on l.v. s id e
= (0.01607) (250) = 4.0175 V.

Scanned by Cam Scanner


38 Electrical Machinery IArt. l.(»

( 6 ) (i) P.u. value of exciting cu rrent referred to h.v. side


IeH 4.8 „ -------
8 - ° ' 6 P U-

(ii) Exciting c u rre n t w hen referred to l.v. winding


f 2500>\
U =(4-8) = 48 A.
250 J'
T IeL in Amp. 48 „ „
(hOp u. = jbl in Amp. “ 80

T he p.u. value of exciting cu rren t is seen to be sam e on both th e sides.


E x a m p le 1.15. A 10 kVA, 2000/200 V, single-phase transformer has following param eters:
r t = 3.6 12 x x = 5 .2 Q
r2 - 0.04 12 x 2 = 0.056 12.
Subscrip ts 1 an d 2 denote high and low voltage w indings respectively.
F ind the p.u. values o f r el re2 xcl and x e2.
S o lu tion . H.V. side.
B ase Voltage VBH = 2000 V
. 1 0 ,0 0 0 _ A
B ase cu rre n t
7fi" = “ 2 0 0 0 ~ = 5 A
2000
/. B ase im pedance Z BH = = 40012
3.6
= 0.009
r i p “- 400
5.2
*1 p . u . - 400 = 0.013

• ohm
re\ in h s = r 1 *+ r 2 f —
M = (3.6)+ 0.04 (10) = 7.6 12

r 7-6
re l p . u . 4 0 0 = 0.019

1
r (Nff
X} + X2 >5.2 + 0.056 (lO)2! = 0 .0 2 7 .
%bh Nn 400

L.V. side. Base voltage VBL = 200 V


r 10,000 ^ A
Base current
BL 200

/. Base impedance z bl =™ =* «

4
0.056
*2 p.u. “ = 0.014.

(N rf
Pc2p.u. ~ ^ r2 + r j
BL N.

Scanned by Cam Scanner"


Art- I j l

Similarly, *e2pu. = 0.027


Note th a t in p .u . sy stem •
r. l p u . = r' 2pu = r lp u + r2 p u . sr
ii
*“ « • - * « P-«. =
This exam ple show s t h a t th e tran<!fnrr«„- • .
on both sides. equivalent param eters in p.u. system are equal B 1..
;■/ . i
The foregoing ex am p les illu strafp
summarised below : vantages of p.u. system and these are now

(i) The calculations in p er u n it au an titieo o , • ,


one need not b o th er w h e th e r th e n aram p to . 6 ^ ^ y ’ Smce ^ u rin S th e tran sfo rm er analysis,
is more significant w hen a c r u it” te o f *,l° ^
f™
V
Z °r Lv- side- This I
cutt consists of m any transform ers of different tu rn ratios.
ill) Ih e p.u. p a ra m e te rs of different r , , ■gg 1
machines) lie w ith in a n arro w range. For e x a n n le ^ m ers | a.n d other ^ e s of electrical B '
\
normally lies betw een 0 .0 2 and 0 05 n u tho Q • * e m a ^ n e tl z m 6 cu rren t of transform ers
transformers) an d 0.02 p . u . t s L l l ^ 0 005 p.u. (large
from 0.05 (large tra n sfo rm ers) to 0 1 (sm all trn n lfn th e e^ lvalent reactance usually varies
parameters of a tra n sfo rm e r (or an v other ™ US fr° m a P l e d g e of th e p.u.
and performance c h a rac teristic s T h L in f ? m achine), one can know about its design
o f ^ v e n s iK m a d e b y t^ r v a r io im m a n u f a a u r e r s i n '“ “ “ m p aris° n ° f

m s m m s ^ s s s

S o lu tio n . T otal losses in a tra n sfo rm er


( i \
- 1 (2 0 0 ,0 0 0 ) = 6185.5 W
0.97
At full load, Pc + p oft = 6185.5 IV '
.(i)
At 60% of full load, P c + (0.6 )2 • Poh = 6185.5 x 0 .6 W
0r Pc + 0.36 Poh = 3711.3 W .{ii)
From E q n s. (i) a n d (ii), ohm ic losses,

p —6IS 5.5 - 3711.3 qoce nr


. P<*~ ( 1 - 0.36) 3866 W
m core loss, Pc = 6185.5 - 3866 = = 2319.5 W

P aram eters a re re q u ire d on th e low -voltage side. From Eqn. (1.36)

' ^ S * 1 0 0 - 1-933*
Put voltage re g u la tio n is 5 = (rt2 cos 02 + xe2 sin 02) = (1.933 x 0.8 + x e2 x 0.6)
or
' x e2 = 5.756%

im -
3
icanned by C am S canner
40 Electrical Machinery----------------------------------- tA*T_U)
2 0 2870 A 0-09665 fl
rre2 in
in ohms
onms = 10Q x (1000)
2 0 0 ,0 0-0 = 0,09665 SI + r1 ------------------------ r m '--^ ----

and xe2 in ohms = 0.2878 Q


(iooo)2 ^ ,ooov ^ 1'13 ^ 111*32a
Core-loss resistance, R c = 2 3 1 9 '5 = 431.13 ft

Exciting current,
t _ c _ 2 3 1 9 .5 - Q 278 A Fig. 1.23. Pertaining to Example 1.16.
e2 V2 cos 0 O 1000 x 0 .2 5 '

and core-loss current, Ic = = 2.3195 A

M agnetizing current, I m = V9.2782 - 2.31952 = 8.9834 A

.*. M agnetizing r e a c ta n c e ,^ = .- .Qp0° ; = 111.32 ft


o.9oo4
The approxim ate equivalent circuit referred to l.v. side is show n in Fig. 1.23.
1.9. V oltage R eg u la tio n o f a T ran sform er
It is defined as the change in m agnitude of the secondary te rm in a l voltage, ex p ressed as a
percentage (or per unit) of th e secondary rated voltage, w hen load a t a given pow er factor is
reduced to zero, with prim ary applied voltage held constant.
If V2 = secondary term inal voltage a t any load,
and E 2 = secondary term inal voltage a t no load,
then at a given power factor and specified load, th e voltage regulation is given by
E —V
Voltage regulation = Secondary2rated voltage *n * » •

volta1 e7 noS ioad,“ ry V° “ aBe ° f '*trans<b™ er is *> seco n d ary te rm in a l

.-. Voltage regulation = in p.u. = x 100 in percentage.

At no load, the prim ary leakage im pedance drop is alm ost negligible, th e re fo re , th e secon­

dary no load voltage E2 =V, T he expression for voitage reg u latio n can also be w ritte n as

v ' % - v>
x 100 in percentage = x 100 fa p e rc e n ta g e
v A
N1
Here Vj is the prim ary applied voltage.
The change in secondary term inal voltage w ith l . .
sec on ary eakage im pedances of th e transform er Thp m CUlTf0I\t *s ^ u e to th e p rim a ry and
the load pf; load current, total resistan ce and to tal i e a k a a e T t ° f th ’S °h a n g e d e P e n d s on
A distribution transform er should have a 11 1 actan ce ° f t h e tra n sfo rm e r.
3 t °n )’ S° p h a t th e term in al a t th e consum er ?f V° 'ta g e re g u la tio n « •* g° ° d voltage
io lta e ' i T ’ F ° r 3 tran sfo ™ o r of large S T ' 1 r? .pre/n u ses d° “ n o t v a ry w idely as the
voltage a t th e consum ers' term in als will fal ppreeTb t t d b ^ P° ° r V° ' tagC R a t i o n ) , the
ppreciably w ith th e in c re a se in l o a d - t h i s h as a
Art.

detrimental effect on th e ° P e ra tio n ^ iffo 7 fW ~ranSf°rmerS-—


since these are designed to operate sa tis fa c tn ru !'^ tUbeS’ T V' sets*refrigerator motors etc.
transformers should be designed to have a low v l3 a constant yoltage. Thus the distribution
The voltage regulation of a transform a^ age imPedance.
circuit referred to prim ary or secondary P i p ^ o !!6 ^rom approximate equivalent
circuit of a transform er referred to the sennn^ *j *^us^ra^es the approximate equivalent
is drawn in Fig. 1.24 (6 ) for a lagging- nowpr f Si f nC* Phasor diagram for this circuit
i'JP. I !
tion, draw an arc of radius OD meeting th* I For the ca,culation of voltage regula-
1.24 (6 ) th a t O F (= E 2) is approximately equa* to 0 0 * ^ ^ ° A ^ F' 11 m3y b° SGen fr°m Fig'
E 2 = OC = OA + A B + BC (or B'C')
= OA +A B ' cos 62 + DB' sin 02
= V 2 + I 2 re2 cos Q2 + I 2xe2 sin 02 I,
Thus the voltage drop in the secondary terminal voltage
- E 2 - V 2 = I 2re2 cos 02 + / 2 xe2 sin 02 ...(1.38)

r t f E 2 ~ V “a T eqU a‘ t0 A ° ’ U ,2 *•*The cha" 6 n ^ secondary terminal


voltage is equal to th e m agnitude of £ 2 minus the magnitude of V2.

r
r<?2
-4/WW

- 6- -O-
(a)
rig. 1.Z 4. (a ) A p p r o x im a t e equivalent c ir c u it Ul a * -w in u u lg n o i u i u m r a , m sccuil
(6 ) the p h a s o r diagram o f the circuit ofFig. 1.24 (a) for a lagging power factor load.

In Eq. (1.38), p er u n it voltage regulation for any load current / 2 is

E2- V 9
'« cos e2+ ^ s i n e , ...(1.39)
E2 E
Cj2l 1 E2
In case I 2r is th e rated current, then from Eq. (1.35),
I 2 • rrn Voltage drop across re2 at ra ted current
“ Rated (= base) voltage E2
= p.u. equivalent resistance or p.u. resistance drop = er (say)
/ , / r,o nhm ic loss at rated current
I 2r r c2
Also Er = Rated VA

Scanned by Cam Scanner


[A rt. 1.9
42 Electrical Machinery ______ __________________ _________________-—

N ote t h a t ^ h e re a n d (rcL)pu in Eq. (1.36) are identical.


72/- Xe2
S im ilarly , for ra te d c u rre n t / 2, le t — p — = t x

F rom Eq. (1.39), th e p er u n it voltage regulation a t ra te d c u rre n t is given by


...(1.40 a)
e,. cos 02 + £* sin 0 2
P ercen tag e voltage reg u latio n a t rate d load
= (e,. cos 0 2 + er sin 0 2) x 100 ...(1.40 6)

I t should be noted th a t Eqs. (1.38) to (1.40) are valid for lagging pow er factors only. For
lead in g pow er factor loads, th e p hasor diagram of Fig. 1.25 (a) rev eals th a t
E 2 = OC = V2 + 12 rt2 cos 02 - 72 xe2 sin
T herefore, secondary term in al voltage drop, for any load c u r r e n t / 2, is
E 2 —V2 = I 2rc2 cos 0 2 —/ 2 xe2 sin 02 ...(1.41)

.*. p.u. voltage regulation a t any load c u r r e n t / 2 is given by


I tT.o ^ I&e 2 . ...(1.42 a)
cos 02 - sin 02

In case I 2 is th e rated (or full-load) cu rren t, th e n p.u. voltage reg u latio n is given by
er cos 0 2 - i x sin 0 2 ...(1.42 6 )
For an accu rate expression for th e voltage regulation proceed as follows :
From Fig. 1.24 (6 ), OD2 = OC2 + CD2
B ut OC = V2 + l 2rt2 cos 0 2 + l& a sin 02 and
CD = C'D - CC' = 7 ^ 2 cos 0 2 - 72r e2 sin 0 2
OD2 = E 2 = (V2 + 12 re2 cos 0 2 + / 2xt2 sin 0 2)2 + (/ 2Xe2 cos 0 2 - 12 re2 sin 02)
or [E\ - ( / 2 xe2 cos 02 - 12 re2 sin 0 2)2] 1/2
= (V2 + 12 r„2 cos 0 2 + 12 xe2 sin 02) ...(1.43)
The left h an d side expression of Eq. (1.43) can be w ritten as,
/ x2 11/2
I 2 xe2 cos 02 - 12 re2 sin 02
1-
En
. v / .
E xpanding it by binom ial theorem and nr :;1- cting h ig h er order te rm s, w e get

I 2 xe2 cos 02 - In r : sin 02\2

J
E2~ (72 xe2 cos 02 - 72 re2 sin 0o) f ....
2L2

N°w Eq. (1.43) can be re-w ritten as

E 2 •= (V2 + 12 re2 cos 02 + 12 xe2 sin 02) + g i - H2 x e2 cos 02 - / 2 re2 sin 02)2
2

/. Voltage drop in th ? secondary term in al voltage = E 2 - V 2

- (I? re2 co s 92 + h xe2 sin 02) + ~ (72 xe2 cos 02 - 12 re2 sin 02)2

V
x J U U I II I U U hJ J U U I I l v - / V y * U I II I V / I
Art. 1-9] _
, " ' T r a n sfo r m e r s 43
... p.u. voltage regu lation ---------------------------------------------------

_ &2 - V 2 12 re2 I V / 2
E T = ~ B T C0 B e 2 + sin 9, + i f e ^ c o s B 2 - ^ s i n e , '
case / , is the rated current, then p.u. voltage reg u , J ; ., ^ fcy *

£,. cos 02 + e, sin e 2 + 1 (Ei cos ^ _ £ sjn 0 y 2 ( j ^

Percentage voltage regu .ation is obtained by multiplying Eq. (1.44) by 100


In Eq. (1.44), th e term (e, cos 0, - e . i . # , ! ; . , ., „ \
for all practical purposes. Thus the voltaee r 1> • Sm 3 Ca" 'thercfore’ be neglected
for a lagging p.f. load and Eq. (1.42) for a le a t o g ap‘noadn0rmally ° btamed ^ USinB ^ <1-40)
C o n d itio n fo r z e r o v o lt a g e r e g u la tio n . It is seen from Eq (l 40) that voltage regulation
varies with load power factor. If load power factor i s varisrf „ „ . I , regulation
coenndnrv om f then , ia. lactor is varied with constant values of load current
and secondary em f, th en zero voltage regulation will occur when
er cos 02 + i x sin 02 = 0

or tan 02 = - = _ J 2 r' 2 = _ rJ l
£r E x' 2 xe2
2 Ei

M a g n itu d e o f lo ad p.f., cos 02 = —


Z e2

T he n e g a tiv e v a lu e o f t a n 02 in d ic a te s a lead in g pow er factor. T herefore, zero voltage

regulation o ccu rs w h e n lo ad p.f. is — leading. F or leading p.f.s g re a te r th a n — , th e voltage


z e2 z c2
regulation w ill be n e g a tiv e , i.e. th e v o ltag e will rise from its no load value, as th e tra n sfo rm e r
load is in c re a se d .
C o n d itio n fo r m a x im u m v o lta g e r e g u la tio n . P.u. voltage reg u latio n
= er cos 02 + ex sin 02.
The c o n d itio n for m a x im u m v o ltag e reg u la tio n is obtained by d ifferen tiatin g th e above
"'rpression w ith r e s p e c t to 02 a n d e q u a tin g th e re su lts to zero. H ere ag ain th e load c u rre n t and
secondary e m f a r e a s s u m e d to re m a in c o n sta n t.
J
-jTj- (p .u . re g u la tio n ) = - t r sin 0 2 + £x cos 0 2 = 0
d 02
£r X '2
or ta n 0 2 = - = —
Er >e2

r c2
any cos 02 = — •

H ere ta n 0 2 is p o sitiv e , th e re fo re , m ax im u m voltage reg u latio n occurs a t a lagging load

PX equal to fr . In other words, m ax im u m voltage reg u latio n occurs when load power-factor

1 w's.'le 8 , is e q u a l to th e le a k a g e im p ed an ce angle 4 of th e tra n sfo rm er.


••• T h e m a g n itu d e o f m a x im u m v o ltag e re g u latio n
r* + f . i - . V S S
= rT — + , E -i 2 ,2 E 'i
Z eo *

bcannea oy u a m b c a n n e r
(C)
(a) (b) , r
Fie 1 25 Phasor diagrams for 1-phase transformer or
(a) negative voltage r e f la t io n (V.B.) (61 tern V.R. and (c) maximum V.R.

£ Ze2 V & • 7 E2
T hus th e m agnitude of m axim um voltage regulation is equal to th e p.u. value ° ^ e^ Va1^
leakage im pedance of th e transform er. For exam ple, if a tra n sfo rm er h as a , 2 = 0.054, th en
m agnitude of m axim um possible voltage regulation is 5.4%. - rP
Phasor diagram s for a single-phase transform er for different operating power factors are
illu strated in Fig. 1.25. In Fig. 1.25 (a), E 2 < V2, voltage regulation (V. R.) is th erefo re negative.
In Fig 1 25 (6 ), E 2 = V2, V. R. is zero. Fig. 1.25 (c) is draw n u n d er th e condition of m axim um
V. R., because here load power-factor angle 02 = leakage-im pedance angle <}>of th e tra n sfo rm e r
ta n 1 (xe2/ r e2) = cos
w h e r e <}> = ( r f 2 / 2 c 2 ).

E x a m p le 1.17. Show that the per unit voltage regulation o f a transform er is given by
V'. J
-=r cos (Qsc - 0 2)
E2
>scxc 2
where Vsc is the voltage required to circulate
rated current on short-circuit, 0 *. is the p . f angle
on short-circuit a n d 02 is the load pow er factor
angle.
S o lu tio n . The transform er equivalent circuit lsc
under short-circuit is illustrated in Fig. 1.21 (c) and Iscre 2
Fig. 1.26. Transform er phasor diagram under
by re fe rrin g to it, th e p h aso r d iag ram of Fig. short circuit.
1.26 is draw n. From th ese figures, it is seen th a t
Vsc — I rated z e2 ~ sc z e2
Fig. 1.26 also reveals th a t Isc rc2 = Isc ze2 cos 6 SCand l sc x e2 = I sc z c2 sin 9SC
Division o f th ese two relatio n s by E 2 gives
/s c ^e2 I*,,
fs c rr e2
pa , z e2
II sc
~ Er r» * ~e2 _ — COS 0 ^
■“ 2 z e2 Ea *
■vsc •*c2 /sc z e2
and = £x = sin 0 sc
Er

Scanned by Cam Scanner


Transform ers 45
p.u. voltage reg ulation = ^ cos 02 + Ejc sin 0
Substitution of the values er and e,. from above, gives the per u n it voltage regulation as
I sc Ze 2

E2 ^C0S G*c cos 02 + sin 0SCSin 02J

£ 2 cos (0 SC- 0 2) ...( 1 .4 5 ) Ii

Hence the req u ired re su lt has been proved.


E x a m p le 1.18. A 20 kV A , 2 5 0 0 /5 0 0 V, single-phase transform er has the fo llo w in g
param eters:

h.v. w inding l.v. w inding


rx= 8 Q r2 = 0.3 ft
x i = 17 Cl x 2 = 0 .7 Q

Find the voltage regulation a n d the secondary term inal voltage a t fu ll load for a p.f. o f
(a) 0.8 lagging a n d (b) 0.8 leading.
The prim ary voltage is held constant at 2500 V.
Solution. E q u iv alen t resistan ce referred to l.v. or secondary winding,

re2 = 0.3 + 8 t j j S & f . O .M O


2500 I
Equivalent reactan ce referred to secondary winding,

*,2 = 0.7 + 17 ( 1 } = 1.38 a.


25
/
Full load secondary c u rre n t = 2 G?PG° = 40 A.
500 " I
(a) From Eq. (1.38), th e full-load voltage drop in the secondary term inal voltage, for a lag­
ging p.f., is
^ 2 ~ V2= h re2 C0S 0 2 + h X'2 sin 0 2
= 40 (0.62) (0.8) + 40 (1.38) (0.6) = 52.96 V.
Secondary te rm in a l voltage V2 = 500 - 52.96 = 447.04 V

and voltage regulation = E * ^ = ^ ^ = 0.1059 or 10.59%.


j&2 OUU
(6) For a lead in g p.f., th e full-load voltage drop in the secondary term in al voltage is
I
E2 —V2 —/ 2 re2 COS 02 —I2 Xe2
= 1 9 .8 4 -3 3 .1 2 = - 13.28 V
V2 = 500 + 13.28 = 513.28 V

and voltage regulation = ^ =- or - 2.656%.

Example 1.19. A 6 6 00 /4 40 V, single-phase transformer has an equivalent resistance of


0 f P ‘U. and an equivalent reactance o f 0.05 p.u. Find the full-load voltage regulation at 0.8
i
H la8, if the prim ary voltage is 6600 V. Find also the secondary terminal voltage at full load.

° ^ tio n . P .u. voltage re g u la tio n


= e,. cos 02 + Ex sin 02

Scanned by Cam Scanner


(Art. 1.9
46 Electrical Machinery

= (0.02) (0.8) + (0.05) (0.6) = 0.046


e 2- v2
= 0.046
—z
F o r a p rim ary voltage of 6600 V, the secondary no load voltage E 2 is 440 V.
/. T he change in th e secondary term inal voltage
E 2 - V 2 = 440 (0.046) = 20.25 V
an d secondary term in al voltage
Vo = 440 - 20.25 = 419.75 V.
E x a m p le 1.20. A short-circuit test, when perform ed on the h.v. side o f a 10 kVA.
V, single-phase transform er gave the following data :
60 V, 4 A, 100 watts.
I f the l.v. side is delivering fu ll load (or rated) current at 0.8 p.f. lag a n d at 400 V, fin d the
voltage applied to h.v. side.
S o lu tio n . From short circuit d ata
100
. .
= ^
r cH ~ = ^ 7 = 6 .2 5 £1
i j w
Kc 60 i f. q
sc
Z.H = = J o s ?- (6-25)“' = 13-61 n
For th e l.v. side, the p aram eters are

rci ~ 6-25 - = 0.25 Q

xeL = 13.61 | = 0.544 n

F ull load secondary cu rren t


10,000
= 25 A.
l2L 400
Now E 2 - V 2 = I 2L r eL C0S 0 2 + h L x eL sin 0 2
= (25) (0.25) (0.8) + 25 (0.544) (0.6) = 13.16 V.
For V2 = 400 V, E 2 = 400 + 13.16 = 413.16 V.
.% T he voltage applied to th e p rim ary is
2000 = 2065.8 V.
= 413.16 x
400
E x a m p le 1.21. A single-phase load is fed through a 33-kV feeder w h o s e impedance is
30 + / 120 Cl a n d a 3 3 /3 .3 k V transform er o f equivalent leakage im pedance 0.3 + jl.4 referrea
to l.v. side. For a load o f 100 k W a t p f0 .8 leading a t 3.3 k V term inals, com pute
(a) voltage a t the sending end o f the feeder,
(b) voltage a t the p rim a ry term inals o f the transform er,
(c) active a n d reactive powers a n d also the pow er factor a t the sending end o f the fee er ^
S o lu tio n , (a) For th e tra n sfo rm er and feeder, eq u iv alen t circuit referred to l.v. si e 1
show n in Fig. 1 2 7 . In th is figure, feeder im pedance referred to l.v. side is given by
3^3
(3 0 + > 1 2 0 ) x = 0 .3 0 + j 1 .2 n
33

Scanned by Cam Scanner


91
Transformers 47

,oad c u r re n t, I * - j j ^ W 37.88 A
3300 x 0.8
t is seen from Fig. 1.27 th at voltage V j . ,
is given by * sending end of the feeder and referred to l.v.

V s - V 2 + ^ ( f i c o s 9 2 - X s i n e 2)
:re
v - a sn n ! 'L =0°Q6 Q an d * = 1.2 + 1.4 = 2.6 n
* ■ 3300 + 37-83 (0.6 x 0.8 - 2.6 x 0.6)
= 3259.09 V on the l.v. side.

0-3 K
-y w — /r o s-

F ig. 1.27. E q uivalent circuit referred to l.v. side, Example 1.21.


Voltage a t th e sen d in g end of th e feeder
= 3259.09 x 10 = 32590.9 V
(i>) From Fig. 1.27, V , - [3300 + 37.88 (0.3 x 0.8 - 1.4 x 0.6)] = 3277.3 V
Voltage a t th e p rim a ry term in als of tran sfo rm er
V l = 3277.3 x 10 = 32773 V
(c) Active pow er loss = (37.88 )2 x 0.6 = 860.94 W
Reactive power loss = (37.8 8 )2 x 2 .6 = ^730.73 VAr

Complex pow er a t load end = 1 0 0 - j 1 0 0 tan (cos ' 1 0 .8 ) = (100 - j 7 5 ) kVA


Complex power a t th e sending end
= (100 - j 75) + (0.86 + j 3.730) = (100.86- j 71.27) kVA

n .2 n V A j? 4 p Cecttveaiyd reaCt‘Ve P°WerS a t th ® Sending end of the feeder are 8 6 kW and

Power factor a t th e sending end = cos t a n ” 1 7 1 2 7 = 0.8167 leading.


100.68
1 0 k V A ’ 20001200 V. 50 I'P h a tt transformer, when working on rated
on itf . ho h a d , takes an input o f 125 W at 0.15 pf. Its per cent leakage impedance, based
on its / Y is (0.5 + j 1). I f the transformer delivers 10 kW at 200 V at p f0 .8 lagging
' Sl“ e» determine the input power and power factor.
Solution. Base im pedance on H.V. side

Zt)U = ml = 40012
JBH (MVA)fl 1 0 x 1 0 - 3
a^age impedance on h.v. side,
400
Zt i = ( 0 . 5 + j l ) ^ = 2 + ; 4 a

Scanned 5y~CamScanner
[Art. 1.9
u?I 48 Electrical Machinery
iI
1 S h u n t bran ch p aram eters on th e h.v. side are calculated as u n d e r :
V? onnn2
Core-loss resistance, R c = ~p~~ 1 2 5 = ^2000 ^

No-load current, j _ 125 = 0.4167 A


c_ 2 0 0 0 x 0 .1 5
cos 6 0 = 0.15 and sin 0O= 0.98869
V, 2000 _ = 4854.4 n
M agnetizing reactance, X m = gin Q- - 0.4167 x 0.98869

The transform er equivalent circuit referred to h.v. side is show n in Fig. 1.28.

Secondary load current, / 2 = q g^jjOOO = 6 2 5 A


62.5
P rim ary load current, 7X= = 6.25 A

Load p f = cos 02 = 0.8 = cos 0 X


2 4
-vvw— <TnrYN“
?+-
u
L
Vt V'= 2 0 0 0 V O
A
D

Fig. 1.28. Pertaining to Example 1.22.

Prim ary in p u t voltage, V1 = V { + Ix (rel cos 0 X+ xel sin 0X)


= 2000 + 6.25 (2 x 0.8 + 4 x 0.6) = 2025 V
Active power loss in series resistance
• =(6.25)2 x 2 = 78.125 W
Reactive power loss in series reactance
= (6.25)2 x 4 = 156.25 VAr
Active power loss in core-loss resistance

= W m ’ 128145w
Reactive power loss in m agnetizing reactance

_ £2025]^ 72 VAr
4854.4 ' ’’ ’ ■'•‘ VAr
Complex power a t load end

= 10(1 + j ta n (cos’ 1 0.8)] = (10 + j 7.5) kV A


Complex pow er in p u t to tra n sfo rm er

= n n I 0 " 7813 + ° ’12815) +J (7 -5 + 0.15625 + 0.8447)


= (10.2073) + j (8.501) kVA

1
V. % .
Jfc
ScarTnecT b y U a m S c a n n e r
Arl- Transformers 49
... Real pow er in p u t = 10.2073 kW
8 .5 0 1
Input p f = cos ta n ' 1
V
10.2073 = 0.7684 lagging.
j 10. T ra n sfo rm er L o s s e s a n d E ffic ie n c y
In th is a r tic le , tr a n s f o r m e r lo s s e s a r e descrihpH ...u r .
d scu ssed in d e t a il u escriD ed b riefly w h e r e a s th e tr a n sfo r m e r e ffic ie n c y

„ J e ly U ) c m e T s? a r d e « i)°o C i'c T o hsesr e 3re mai"ly ‘W° kindS ° f '°SSeS in 3 transformer'


(i) Core /oss. T h e core loss Pe occurring in th e tran sfo rm er iron, consists of two com ponents,
hysteresis loss Ph a n d eddy c u rre n t loss P i.e.
Pc = Ph + P e
The h y steresis a n d eddy c u rre n t losses in th e core have been found to be given by the
expressions & '
P h = K h f B m'
and ...(1 .4 6 )
Pe = K f B m2
where Kf, p ro p o rtio n a lity c o n sta n t w hich depends upon the volum e and q u ality of th e core
m a te ria l a n d th e u n its used.
Ke ~ p ro p o rtio n a lity c o n sta n t w hose value depends on th e volum e and resistiv ity of
th e core m a te ria l, th ick n ess of lam in atio n s and th e u n its em ployed.
Bm = m ax im u m flux d en sity in th e core.
and f- freq u en cy o f th e a lte rn a tin g flux.
The value o f th e ex p o n e n t x (called S te in m e tz ’s constant) varies from 1.5 to 2 .5 , depending
upon the m ag n etic p ro p e rtie s o f th e core m a te ria l. T herefore, th e to tal core loss is
Pc = Kh f Bmx + Kef B j ...(1.47)
If it is re q u ire d to e x p re ss th e core loss in te rm s of voltage, frequency etc., th e procedure
is given below.
The applied v o ltag e V”is a lm o st eq u al to th e induced e.m.f. given by th e Eq. ( 1 .4 ), i.e.
V = V2jt f N • <J>max = V2 n f N B m A,
where A, is th e n e t core a re a a n d N is th e n u m b e r of tu rn s . S u b stitu tio n of th e v alu e of B m gives
hysteresis loss as

Ph =Kh f
^ 2 n N Aj
For a tra n s fo rm e r N a n d A, a re c o n s ta n t
•• Ph = kh V f ~ x , ...(1 .4 8 )
Thus th e h y s te re s is loss d e p e n d s on b o th th e ap p lied voltage an d its frequency.
The eddy c u r r e n t loss P e is given by
\2 f y \2
= ke V2 ...(1.49)
Pe - K' f V2 n N A i
\ y \ / t
The eddy c u r r e n t loss is, th e re fo re , p ro p o rtio n a l to th e sq u a re of th e ap p lied vo ltag e an d is
^ P e n d e n t o f fre q u e n c y f. A cco rd in g to Eq. (1.4); for a n y given vo ltag e (~ B m /), if /d e c r e a s e s
^ creases c o rre s p o n d in g ly a n d if /'in c r e a s e s B m d e c re a se s correspondingly. H ence th e eddy
rrent loss Pet a t a n y g iv e n vo lta g e, is in d e p e n d e n t of freq u en cy (
IR K . *
^ •#"
Scanned by C am Scanner
u
SO Electrical Machinery JArt^Llo

The total core loss Pc in term s of voltage and frequency is


P c - k h V x f i ~x + k t V l - ( 1 .5 0 )

(ii) O hm ic loss. W hen a transform er is loaded, ohmic loss (I R) ‘J ^ o f ^ l e c t r S


and secondary w inding resistances. Since the standard °P^ra in£ P
m achines is 75°C, th e ohmic loss should be calculated at 75°C.
In addition to the core and ohmic losses, the following two losses are also present in a

tran| f r ray " o a d lo ss. Leakage fields present in a transform er induce eddy currents in
conductors, tan k s, channels, bolts etc. and these eddy currents give rise f r __qfnrm p’ ni,
D ie le c tr ic lo ss. This loss occurs in the insulating m aterials, i.e. in e
and th e solid insulation of h.v. transform ers. . , ,
The stray load loss and dielectric loss are small and are, therefore, neglected.
1.10.2. T r a n s f o r m e r e fficien cy . The efficiency of a transform er (or any other device) is
defined as th e ratio of output power to input power. Thus
O utput power
Efficiency n = Input power
V-i h cos 82 ____ ...(1.51)
V2 / 2 cos 0 2 + Pc + T12c
w hcr 6 Pe - total core loss,
/ 22 r e2 = total ohmic losses,
V2 / 2 = output VA,
and cos 02 = load p.f.
As described before, stray load losses and dielectric losses are sm all and have been ignored.
The efficiency can also be expressed as
O utput power Input power - Losses
^ - In p u t power Input power
_ Losses ...(1.52)
Input power
The efficiencies of power and d istribution tran sfo rm ers are usually very h ig h (95 to 99%).
It is therefore, m ore accurate to d eterm ine th e efficiency from th e m e asu rem en t of losses from
Eq. (1.52) th a n from th e m easu rem en t of o u tp u t from Eq. (1.51). Reference to A rt. 3.9 is also
h e lp fu l.
Due to p rim ary leakage im pedance drop, th e e.m.f. E l (or E 2) an d th erefo re, th e m utual
flux <}>decreases for lagging pow er factor loads and m ay increase for lead in g pow er factor loads.
U n d er n orm al o p eratin g conditions, th e v ariatio n in the m u tu a l flux $ is n o t m ore th a n 2 or
3% C onsequently th e core losses can be assum ed c o n sta n t a t all loads. In o th e r w ords, it can
be s ta te d th a t tra n sfo rm e r core loss is alm o st in d ep en d en t of load c u rre n t. T h e ohmic losses
depend on th e load c u rre n t. F o r exam ple, if I 2 re is th e ohm ic loss a t full load, th e n a t h a lf full
load, the ohmic loss will be \ I 2 re.
Efficiency of a transformer can also be expressed in per unit parameters. In Eqn. (1.51).
dividing both numerator and denominator by V2 / 2, we get
cos 02
n = ----------------- P
p c— hJ 7r e2
COS 02 + . + y J
’ 2*2 v 2*2

Scanned by Cam Scanner


J
Art-
Transformers 51

— -----------------------
It can also be shown that
th a t ' eAUlva len t resistance ...(1.52 a)

n=i - p u. Pc + p .u. rc
l°ad p/+1TiP+~P~T ,
Efficiency given by Eqns. (1.52 a) and , , L " 2ft)
current 12'd
, ifferent from rated load cu rren t/,, i s ^ i v e n ^ CU,Tent °n'y ' Efficiency at >°a<*

...(1.52 c)

C ondition fo r m a x im u m e ffic ie n c y . In Ea <1 i p ,


V2 rem a in s p r a c t ic a lly c o n s t a n t . At a s p e c ifie d value nf 1 h the l°ad vo lta g e
dx\ . P ^ cos ®2> th e e ffic ie n c y w ill be
maximum
- w h en ^ = 0 . T h e r e fo r e , for Eq. (1.51) is

(V 2 12 cos 02 + Pc + / 22 ^ {v 2 CQS 02)


dr\ - V ^ 2 cos 02 (V2 cos 02 + 2 12 re2)
dl2 ~ = 0
[V2 12 cos 0 2 + Pc + / 22 r e2]2
or [V2 12 cos 0 2 + Pc + 12 re2J V 2 cos 0 2
= V 2 12 cos 0 2 (’V2 cos 0 2 + 2 I 2 r e2)
or
72 r e2 = Pc ...(1,53)
or Variable ohm ic loss, l \ re2 = C o n sta n t core loss, Pc

Hence th e m a x im u m efficiency occurs w hen th e variable ohmic loss 12 re2 is equal to the
fixed core loss P c. F ro m Eq. (1.53) th e load c u rre n t / 2 a t which m axim um efficiency occurs is
given by

...(1.53a)
■fl2 r c2
If both sides of above e q u a tio n a re m ultiplied by we get

E 2 I2 Pc
1000 1000 VF
I ull load ohmic losses
kVA load for m a x im u m q = (ra te d tra n sfo rm er kVA)
Core loss
Ohmic losses a t rated current

or ...(1.54)
0k V A )'max n
=<* v a ) V 5 , 2
.Determination o , the C S
C ™ * 1 £ ‘/The’ totaM osIe" ^ ^ t r a n s t o i n e r are known. It has ahead;
^ y be c a lc u la te d lf th e to ta l ? : * th e onen circuit te st records th e core loss P „
^ Pemted o u t t h a t th e w a ttm e te r re a d .n g ■" ‘^ e °pen < c
^ d«nng the short circuit test a t ra te d cu rre n t, it registers me « >r

Scanned by Cam Scanner


(Art. My
52 Electrical Machinery

th e to ta l lo sses (Pc + P sc) a t full load are know n from th e se tw o te s ts w ith o u t loading the
tra n s f o rm e r a n d th e efficiency can be calcu lated as follows .
F ro m Eq. (1.52)

s t full load X j-w 7 * \r a \ a j n 7 p


1 (R ated VA) cos 0 2 + Pc +
A t an y o th e r load c u rre n t, say nip, th e efficiency is given by

Pe + * P« ...(1.55)
n(R ated VA) cos 0 2 + P c + rc P*c
As before, m axim um efficiency occurs w hen v ariab le ohm ic loss - C o n s ta n t core loss, i.e.

n 2 P sc = P c
...(1.56)
or

Eq. (1.54) can now be re -w ritte n as


(k V A )max „ = (n) {kVA)nted •••(1-54)
E x a m p le 1.23. In a transform er i f the load current is kept constant, fin d the pow er factor
at. w hich the m a x im u m efficiency occurs.
S o lu tio n . For a co n sta n t load c u rren t, th e ohmic losses Psc are co n sta n t. Core loss Pc is
alread y a fixed q u a n tity . T herefore, to tal tra n sfo rm er losses are co n sta n t for a co n sta n t load
cu rren t. Now th e efficiency is
V "2 I2 COS 02
^ “ V2 12 cos 0 2 + constant losses (C)
The pow er factor a t w hich m axim um efficiency occurs can be obtained by equating
^ = 0 From th e above expression for efficiency, we get
de
dx\ [V2 12 cos 0 2 + C] V2 / 2( - sin 0 2) - (V2 12 cos 92) V2 h ( ~ sin 9 2> _ Q
dQ2 [V2 f 2 cos 0 2 + C]
or (V2 / 2)2 sin 0 2 cos 02 + C.V212 sin 02 = (V> I2f sin 0 2 cos 0 2
or sin 0 2 = 0
/. Pow er factor cos 0 2 = 1.
T h u s th e m axim um efficiency, for a c o n sta n t
load cu rre n t, occurs a t u n ity pow er factor.
It is seen from Eqn. (1.53 (a)) th a t th e load
c u rre n t a t w hich m axim um efficiency occurs does |
not depend upon th e load pow er factor because |
Pc and re2 are alm o st unaffected by a v ariatio n in '
th e load pow er factor.
A red u ctio n in th e load pow er factor reduces th e
tra n sfo rm e r o u tp u t and th erefo re th e tra n sfo rm e r
efficiency is also red u ced accordingly. Fig. 1.29
illu s tra te s th e effect of p.f. on efficiency. N ote th a t 0 5 IC
tra n sfo rm e r efficiency is m ax im u m a t th e sam e load Per unit le a d cu rren t
c u rre n t re g a rd le ss of v a ria tio n in th e load pow er Fig. 1.29. Effect o flo a d p.f. 011 tiansfornier
efficiency curves.
factor.

Scanned by Cam Scanner


Transformers 53
Energy E ffic ie n c y . P o w er tra n s fo rm e rs are used a t Hi* c a ­
. „ hieh-voltage pow er tra n sm issio n line for st^nni™ sen d in g and receiving ends of a
f o r m e r s are m an ip u lated to o p e ra te " ? VOltage
ling).T herefore, pow er tra n sfo rm e rs a re disconnected d u r i n g km ra capacity (kVA
oower tra n sfo rm er is d esig n e d to h a v e m ax im u m efficipnr 1 periods. In view of th is,
Hence the choice of a p ow er t r a n s f o r m e r ^ o7 Z t W n M M A .
should be based on full load efficiency. um bers of com peting transform ers,

to a level 8 u ,ta b le for otd.aa-


with the consum er’s te rm in a ls , w h e re a s a d istrib u tio n tr ln s T n0t C° T 6 “ dlr6Ct contact
directly connected w ith th e co n su m e r’s te rm in a ls The lnaH o n n e r m u st have ,ts secondary
Z x l wide range d u rin g a 24 h o u r day. F o T e l m p ^ a d T sth tra n sfo rm e r varies
colony may h a - e p ra c tic a lly little o r no load d u rin g ’a considerable i T T I ‘" Z reS,den^,aI
in the evenings, th e load (illu m in a tio n load etc.) m av be n e a r it *^ ° xTay tim e ’
primary of d istrib u tio n tra n s fo rm e rs a re alw ays energised and t h capa^ ty - f th a t the
lace continuously. In view of th is, th e J ' *^ '° S§ takeS
value of core loss. B u t for red u ced ^ ^ ^ '° W
one-half of its ra te d kVA, as p e r E qs (1 56) and a T ™ * ab° Ut

trans-

npriS'tn f16inCy ° f * trans/ ° rmer is defined as the ratio of total energy output for a certain
penod to the total energy input for the same period. certain
l eTgV Gf^ enCy ^ b8 L0mputed for “ V ^ n gth of load cycle, provided its load varia-
11 en6rgy efficiency 18 im p u te d for a day of 24 hours, it is called all day

•• All dayr1 = l - ^ 1y LossesinkW h n


Daily Input in kWh ...(1.57)
In a transf°rmer>the core loss is found to be 52 watts at 40 Hz and 90 watts
«Uy current losses a.t>5l^ H meaSUre<^ ^ Samepeak fluX density- Compute the hysteresis and
Solution. From Eq. (1.47), the total core loss is
Pc = K h f B mx + K e f * B m2
For constant flux density Pc = k h f + ke f
Wher* constant kh = K hBms and constant kt = K J B j
'• At 40 Hz, 52 = kh (40) + ke (40)2
* 4160 Hz, 90 = kh (60) + kt (60)2
^ 52 = 40 kh + 1600 ke
90 = 6 0 k h + 3600 k t .
°m ahove two equations, k /, and kt are found to be

k h - 10 a n d K - 100
Thus at 50 H z,

y,ter«8is log* phmkkf mJ L (5 0 ) = 4 5 w a tts.

canned by C aniScanner
Scannei
jtli
j ’ !' [A rt 1.10
■Ii
54 Electrical Machinery

. £ / 5O)2= 5 0
watts.
and eddy c u rre n t loss
e e 100 / rnpr h a s h y ste resis loss o f 3 4 0 w atts
E x a m p le 1.26. A 220 V, 60 H z, i t e r a t e d fro m 2 3 0 V 50 H z supply
a n d e d d y current loss o f 120 w a t t s j f * S te in m e tz ‘s c o n sta n t e q u a l to 1.6.
m ains, th en com pute its to tal core loss. A ssu m e o
S o lu tio n . T h e o p eratin g voltage y . , = ^ 2 n f 2 B m2 A ^
V , = V2 n f \ B m\ A i H an d 'ii
220 / 60 ml
Vi f i Bmi or 230 50 &m2
Vn /j> B m2
(60) (230) p _ t 255
" (50) (220)
From Eq. (1.46), h y steresis loss
Ph - K h f B m

Ph2 __ f/*2i BBm 2j _ 50


50 ^ 255)1 6
Phi f\B m l

(1.255)16 = (340) j( 1 .4 3 8) - 408 W.


or P h 2 - (3 4 ° ) 6

From Eq. (1.46), eddy c u rre n t loss Pe = K e f B ,

e2 m ( B *m2 '5 0 '2


(1 .2 5 5 y
B ml 60
p .i
5 (1.255)2 = 131.3 W.
or P , 2 = (120)

_ core loss
.-. Total P c2 = P h 2 + Pe2 = 4 3 3 + 131.3 - 539.3 W.
E x a m p le 1.26. The ohm ic, hysteresis a n d ed d y cu rren t losses in a tra n s fo rm e r a t 5 0 H z are
1.6%, 0.9% a n d 0.6% respectively. For a S te in m e tz ’s coefficient o f 1.6, fin d
(a) these losses a t 60 Hz, fo r the sam e system voltage a n d c u rre n t a n d
(b) the o u tp u t a t 60 H z, fo r the to ta l losses to rem a in the sa m e a s on 50 H z.
S o lu tio n , (a) S u b scrip ts 1 an d 2 will be u sed to re fe r to 50 a n d 60 H z q u a n titie s re sp e c tiv e ­
ly*
Since th e voltage an d c u rre n t a t both th e freq u en cies o f 50 H z a n d 60 H z a r e th e s a m e , the
o u tp u t also re m a in s th e sam e.
T he ohm ic loss d ep en d s on th e c u rre n t a n d h e re i t is g iv en t h a t t h e c u r r e n t a t b o th the
frequencies a re eq u al. T herefore, ohm ic losses in w a tts re m a in u n c h a n g e d a n d fo r th e same
output, th e p e rc e n ta g e ohm ic lo sses a re a g a in 1.6%. T h u s •
O hm ic loss a t 50 Hz, P ohl = O hm ic loss a t 60 H z, P ofl2 = 1.6%.

f i B ,m l 50 B ml
As before Y i or 1 = Of ^m 2 = g
V2 f2 & m 2 6 0 B m2

From Eq. (1.46), hysteresis loss, P h = K h f B m

•'** ‘ P hi fi B ml

Scanned by Cam Scanner


Transformers 55

P h2 = Pih i 50 6 (601r«Y
5 i
= P m (0-833)° 6 = 0 .8 9 6 Phl
V / «
P h2
— = 0.896 v 111 = 0.896 (0.9) = 0.806%
^ O u tp u t O u tp u t
m Eq. ( 1 4 6 ), eddy c u rre n t loss,
P e = K e f B m2
2.
( Ft m2 \
Pe2 r / i i" °
Bm
ml,
\ \ /
Sqq * ' - *
Pe2 = = P.e l
or 50 ' 6
v y
For the sam e o u tp u t, p ercen tag e Pel = percentage Pe2 = 0.6%.
Thus the ohmic, h y ste re sis a n d eddy c u rre n t losses a t 60 Hz are 1.6%, 0.806% and 0.6%

The core lo ss d e p e n d s on v o lta g e and frequency only. Therefore, Pcl = T5% ( = ° 9 + ^ ) a n d


P = ( 0.806 + 0.6)can’tbechangedforgivenvaluesofvoltageandfrequency.Forthetotal

£ * to remain th e sam e, th e ohmic loss alone can be varied.


. Tntal losses a t 50 Hz = T o tal losses a t 60 Hz
“ T ° =3 + 0 .9 + 0.6) = 1 .4 0 6 + N e w oh m ic lo sse s.
1 ( 1 . 6

■ Permissible v alu e of ohm ic losses a t 60 Hz


= 3 .1 - 1 .4 0 6 = 1694% .
Since th e ohm ic losses a re p ro portional to th e sq u are of th e current, we have
^ p erm issib le c u rre n t f N ew ohm icloeses_ 1.694
O riginal ohmic losses 160
O riginal c u rre n t
New p erm issible c u r r e n t ______
_ , / H I T , original current) = (1.028) (original current)
" V 1.60 v
For the sam e voltage, o u tp u t a t 60^Hz
= ( 1 .0 2 8 ) ( o u tp u .a t iO ) ^ ,s f0I t o same
It is, therefore, seen that output at a h ig h 9 , .
total loss. . , „un cp transformer has ohmic, hysteresis a
Example 1 .2 7 . A n 11 /0 .4 kV, 25 Hz, sin& * What do these losses become i f the trar&
eddy current losses o f 1.8, 0.8 a n d 0.3 o resp t f h e current is assum ed to remain e
fcrmeris operated from 22 KV. 50 H z s u p p ly system.
^ both the cases. . T—
Also calculate the efficiency in each case. 25 Hz and 22 kV, 50 Hz sy
Solution. S u b sc rip ts 1 a n d 2 a re u sed to den
"•Pectively. th e sam e c u rre n t, therefore, th e output P , a t 50
At 50 Hz, th e voltage is doubled - ^
u . \ d - o p .. It is given th a t
^ double th e o u tp u t P i a t 25 Hz, t e. 2 1
P r t 1 rt oat.
= 1 .8 % ; - j p = Px
Pi

Scanned by Cam Scanner


V-
ly
; S’i
56 Electrical Machinery [Art. l.io

O hm ic lo ss. Since th e cu rren t is sam e a t both the frequencies and voltages, the ohmic
losses in w a tts rem ain unaltered, i.e. P ^ 2 = P0hi.
P ercen tag e ohmic loss a t higher frequency
oh2 ohl
= - (1.8) = 0.9%.
2Pi

C ore lo ss. The voltage is related to f, B m etc. by the expression


V = y l2 K fB mA i N
\ Bml
Vi P m1 11,000
or
V2 fa Pnx2 22,000 50 B m2
/ "
1 1 Bml
or
2 2 B„ 'm2
Pm2 = Pml
The hysteresis loss P h = Kh f B mx w atts

Pk2 fa B m2
Phi fa Pml v
\ yv
Ph2~Phi

Percentage hysteresis loss a t fa, V2 is


P_h2 2 Phi
= = = 0 .8 %.
P2 2P,

The eddy cu rre n t loss Pe ~ K ef B m2 w atts


V2 f d
Pe2 (fa -°m2
P el fa V
-®ml
/

50 f
or P e2 P el 25 a r = 4 P el

P e2 4Pfil
P ercentage eddy cu rren t loss a t fa, V2 i s - p - = p -
2 1

P el
=2 = 2 (0 .3 ) = 0 .6 %

. . L osses , L o s s e s /O u t p u t
E ffic ie n c y a t / 1, 1 is - O u tp u t + L o s s e s 1 + L o s s e s /O u t p u t
P .u . lo s s e s 0 .0 1 8 + 0 .0 0 8 + 0 .0 0 3 = 1 0 .0 2 9
1 + P .u . lo s s e s 1 + 0 .0 1 8 + 0 .0 0 8 + 0 .0 0 3 1 .0 2 9

= 0 .9 7 1 8 2 or 9 7 .1 8 2 %
w tr • 0 .0 0 9 + 0 .0 0 8 + 0 .0 0 6
E ffic ie n c y a t fa, V 2 is 1 1 + Q 0Q9 + Q 0 0 g + 0 0 0 6

0 .0 2 3
= 1- = 0 .9 7 7 5 5 % or 9 7 .7 5 5 % .
1 .0 2 3

i :-:
V...
Scanned by Cam Scanner
Transform ers 57
1. W - - —— "—
- Ip 1 .2 8 . A 10 kV A , 2 5 0 0 /2 5 0 V, single-phase, tra n sfo rm e r ga ve the fo llo w in g test
£ 3 c a w lc

« ^ s: te s t:
c ir c u it 2 5 0 V, 0.8 A , 50 W
f ^ circuU t e s t : 60 V, 3A , 45 W
Calculate the efficiency a t 1 a n d l 1- o f fu ll load at 0.8 p.f.

Calculate, the load (kV A o u tp u t) a t w h ich m a x im u m efficiency occurs a n d also the value
S m u m efficiency a t 0 .8 p f.
f ) Compute the voltage reg u la tio n a n d the secondary term in a l voltage u n d e r ra ted load a t
er factors o f(i) 0.8 la g g in g a n d (ii) 0.8 leading.
^ S o lu tio n . I t m ay be n o te d t h a t
(■) open-circuit te s t h a s b een co n d u cted on l.v. side, because 250 V d u rin g th is te s t is eq u al
to t h e rated voltage on l.v. side.

(ii) Short-circuit te s t h a s b een p erfo rm ed on h.v. side, since 60 V is a fraction (2 to 12%) of


the rated voltage on h.v. side.

(a) Fulli load


w (or ra+terhd ) c u r re tn t oni h.v. m
side =10’000
^ qq =i 4t A

Ohmic loss of 45 w a tts is d u e to a sh o rt-circu it c u rre n t of 3 A.


:. Ohmic loss a t full load c u r re n t of 4 A

Psc = 45 T 2 = 80 w atts.
3
V
Fixed core loss P c = 50 w a tts.
Z
fl] rn
At 0.8 p.f. and a t | full load, th e core loss Pc = 50 W, ohmic loss = *■ SC
(80) = 5 W
4 4
\ J \ /
and output = ^(10,000) ( 0 .8 ) = 2000 W.

n at 1 full load = 1 - 2 0 0 50 V 5 o +- I - 0.9732 or 97.32%.

5 0 + i f (80)
Similarly q a t | full load = 1 - ™ = 0.9828 or 98.28%.
| (10,000) (0.8)+ 50 + —

At full load, _ 80 + 50
= 0.984 or 98.4%.
to

T]~ 8000 + 50 + 80
a>
o

5 o + ri]
---

At 1- full load, q =l- 2


= 0.9828 or 98.287c.
^5^1
- (1 0 ,0 0 0 ) (0.8)+ 50 + (80)
4 4

ieading.

^et the m axim um efficiency occur a t n tim es ,the ra te d kVA.

't>'. d ^ mic 1°88 a t m ax im u m r| = n 2(80) w atts.


Maximum q occurs w h en ohm ic loss = core loss

canned by Cam Scanner


[Art. 1.10
58 Electrical Machinery ------------ -------------------------- --------------------------------------

50 = 0.79
i.e . n 2 (80) = 50 or n " V 80

kVA o u tp u t a t m axim um
r| = n ( 1 0 ) = (0.79) (10) = 7.9 kVA.
N ote t h a t th e kVA o u tp u t a t m axim um efficiency does not depend on t e oa p. .
F o r m axim um efficiency, ohmic loss = core loss - 50 w atts. efficiency
• „ Kfi -l sn - 1 0 0 W The m ag n itu d e of m axim um em u en cy
T otal losses a t m axim um q = 5 0 + 5 0 - iu u w . m e

= 1- 10°______ 0.98443 or 98.443%.


(7900)(0.8) + 100
(c) From short-circuit test,
45
r e H ~ - - 5 £1
9
60
ZeH = Y = 2 0 Q
... xeff= 'l ( 2 0 f - ( 5 y r = 19.35 Q
_ hrJ^H 4xJ> _ 0 0 0 8 u
••• e'-" VH 2500 r
_ hrJfeH _ 4 x 19.35 = Q Q31
" VH 2500
.-. V oltage reg u latio n a t 0.8 p.f. lagging
= cos 0 2 + e* sin 0 2
= 0.008 (0.8) + 0.031 (0.6) = 0.0250 p.u. or 2.5%.
Vsc
[C h e c k . F rom Eq. (1.45), voltage reg u latio n for a lagging p.f. is ^ cos (0SC- 02) p.u.

F ull-load c u rre n t
Now Vm for full load c u rre n t = 60 x C u rre n t d u rin g sc te s t

= 60 x | = 80 V on h.v. side = 8 V on l.v. side.


o
Vsc Igc COS 0Sc = Psc = 80 W

..• sc go x 4 = 0.25
or e ,c = 75.5° a n d 0 2 = cos " 1 (0 .8 ) = 36.9°

V oltage re g u la tio n a t 0.8 p.f. lag

8 cos (75.5° - 36.9°) = ^ cos 38.6 = 0.025 p.u.l


"250 v 7 250

Now ^ = 0.025 or V 2 = 0.975 E 2.

S ec o n d a ry te rm in a l v o ltag e a t fu ll load = 0.975 E 2

or V2 = (0.975) (250) = 243.75 V.

V oltage re g u la tio n a t 0.8 p.f. le a d in g


= e,. cos 02 - ^ sin 02

Scanned by Cam Scanner


' A
Transform ers 59

= (0.008) (0 .8 )-(0 .0 3 1 ) (0.6) = - 0.0122 or - 1.22%.


[Check. P e r u n it v o lta g e re g u la tio n a t le a d in g p.f.
V Q

=— cos (0SC+ 0 2) = — cos (75.5° + 36.9°)

8
s in 22.4° = - 0.0122 p.u.)
250
£2-^2
Now = - 0 .0 1 2 2 or V 2 = 1 .0 1 2 2 E 2.
En

• Secondary te rm in a l voltage a t full load 0.8 p.f. lead,


V 2 = 1.0122 E 2 = 1.0122 (250) = 253.05 V.
E x a m p le 1.29. T h e efficien cy o f a 2 0 kV A , 2 5 0 0 /2 5 0 V, single-phase tra n sfo rm e r a t u n ity
f is 98% a t ra ted lo a d and. a lso a t h a lf rated load. D eterm ine
P-'
(a) the tra n sfo rm e r core loss a n d o h m ic losses; a n d
(b) the p.u. va lu e o f th e e q u iv a le n t resistance o f the transform er.
Solution.
, Losses
(a) T) = 1 —
O u tp u t + Losses

At full load, 0.98 = 1 -


(20,000) (1) + Pc + P4l
Pc + P sc
or = 0 .0 2
(26*000) + P C+ P 4C

or ((P
rc+
+Pr sc)) = —
o 98 = 408W
/if
p + - p
c 2 sc
V /
At ^ full load, 0.98 = 1 -
ml
| (20,000) (1) + Pc + ( i j « -

P
'e +T 4- P‘ *
or = 0.02
1 0 ,0 0 0 + P c + 1 P.
1 200 = 204 W.
or Pc + 4 p sc 0.98

But Pc + P,c = 408 w


| P „ = 204W

or = 272 W.
Psc = (204)

and = 204 - = 136 W.

(&) From Eq. (1.36)


n.. , _ O h m ic lo sse s __272_ _ q .0136.
P-u. v alue o f r , = - ^ d V t T ' 2 0 ,0 0 0

Scanned by Cam Scanner


[Art. l.io
60 Electrical Machinery

E x a m p le 1.30. The maxim um efficiency o f a 100 kVA, is 5%, find the


occurs at SOVc o f fu ll load at 0.8 p .f I f the leakage impedance o f the transform .n ne
voltage regulation at rated load o f 0.8 p.f. lagging.
O utput
S o lu tio n . Efficiency q = Q utput + Logses
O utput + Losses 1 _ ^ + Losses
or Output q Output

.*. Losses ' i - i ' O utput

O utput at maximum q = ( 100 ,0 0 0 ) (0 .8 ) (0.8) 64,000 W.


f 1
Total transform er losses = - 1 (64,000) = 1308 watts.
0.98
1308
At maximum q, ohmic losses = core loss Pc - ^ ~ ^

These ohmic losses of 654 W occur at 80% of full load current.


100
.-. Ohmic losses at full load = 654 = 1022 W.
80
1022
From Eq. (1.36), 'e.p.u. = 0.0 1 0 2 2 = er
100,000
It is given th at z e.p.u. = 0 0 5

*epu. = V(0.05)*-(0.01022)2 = 0.0489 = e,


Voltage regulation = er cos 02 + £* sin 02
= (0.01022) (0.8) + (0.0489) (0.6) = 0.0375 or 3.75%.
E x a m p le 1.31. The full-load voltage drops in a single-phase transform er are 2% and 4%
due respectively to resistance and leakage reactance. The full-load ohmic loss is equal to the iron
loss. Calculate
(a) the efficiency on fu ll load at unity p.f.
(b) the full-load p.f. at which voltage drop is m axim um and
(c) the load p.f. at which voltage drop is zero.
S o lu tio n . Percentage full-load voltage drop in resistance
h re 2
x 100 = 2
En
Percentage full-load voltage drop in leakage reactance
■^2*e2
'■x 100 = 4
Eo
l l re2
(a) From above, = 0 .02

.’. Full-load ohmic loss - 0.02 x Rated VA = Iron loss (given)

Full-load R ated VA x 1
0 = x 100
Rated VA x 1 + / 22 r e2 + Iron loss
R ated VA x 100
Rated VA + 0.02 x rated VA + 0.02 x rated VA
- vJ

Scanned by Cam Scanner


Trarufnrmer* 61

1 0 0 = 96.154%
1.04
(ft) M aximum v oltage d rop m ea n s m axim um voltage regulation. T herefore, full-load p.f. at
M ch voltage drop is m ax im u m
_ r,? _ p ercen tag e r e2 2
" “ p ercen tag e z e2 = + 4? = 0 4 4 7 2 lagging
(c) Load p f a t w hich v oltage d rop is zero
_ *c2 _ p ercen tag e x, 2 4
= 2 ,2 p ercen tag e z ~2= W 7 7 = ° ' 8 9 4 4 lcadinR
E xam p le 1 .3 2 . When a 2 0 kVA, 3,100 / 220 V, 50 Hz transformer is operated on rated voltage
nt no load, its p o w e r in p u t is 160 w a tts a t a p f o f 0.15. Under rated load conditions, the voltage
drops in the total resistance a n d total leakage reactance are, respectively, 1 a n d 3 percent o f rated
voltage. Determine in p u t p o w e r a n d p f when the transformer delivers 14.96 kW a t 22 0 V at 0.8
nflagging to a load on the L V side.
Solution. Load c u rre n t,
J _ _ 1 4 £ 6 0 _ n r »
2 ~ 2 2 0 x 0 .8 ~

Rated c u rren t on L V side


_ 2 0 ,0 0 0
" 220
R ated c u rre n t x re2
It is given t h a t ------------ ■=,---------- — = 0 .0 1
h
0 .0 1 x 2 2 0 x 2 2 0
r e2 =
= 0.0242 n
20,000
0.03 x 220 x 220
and xe2 = ***»* - = 0.0726 fi
20,000
Ohmic loss = (load c u rre n t )2 x re2 - (85)2 x 0.0242 = 174.85 W
.. Input pow er = 14960 + 174.85 + 160 = 15294.85 W
With load c u rre n t I2 as reference phasor, Fig. 1.24 (b ),

I 2 = 85 +> 0
^ ? 2 = 2 2 0 ( 0 .8 + 7 0.6) = 1 7 6 + > 1 3 2

•*. E 2 = V2 + 72 ze2 = 176 +> 132 + 85 (0.0242 +> 0.0726)


= 178.057 + > 138.171 = 225.38 /3 7 .8 1 ” V
If is taken as referen ce for convenience, th en I2 can be w ritten as

h = 85 / - 37.81 = 67.155 - J 52.11 A and E 2 = 225.38 [0 ^

From no-load values, lt = ^ * 4 848 A

,,= / ,c o » B „ - |f 0 .7 2 7 A

/ » V<4.H48)3 - (0 .7 2 7 7 * 4 793 A
(Art. l.io
62 Electrical M achinery_____________ - " . . , '
‘— n A Y in o arallel. As a re s u lt, l c is in p h a se with
Note th a t voltage E 2 ap p ears across R c and m p reference, can be w ritte n as
E , and / „ lags F.2 by 90". Therefore, no-load c u rre n t w ith E 2 as re
7„ = 0 .7 2 7 - j 4.793
= 72 + 7e =-67.155 - j 52.11 + 0.727 - j 4.793
In p u t cu rren t
= 67.882 -y'56.903 = 88.577 /-3 9 .9 7 2 _
.-.In p u t p.f. = cos (39.972°) = 0.7663 lag

Strictly speaking, / c = 0.727 x ^ 2 2 0 ” = °*7 4 5 A

and / „ = 4 .7 9 3 = 4.910 A

as £ 2 appears across R c, X m. The resu lts are modified as u n d e r :


Input cu rren t = 67.155 -752.11 + 0.745 -74.9 1 0
= 67.9 -y ‘5 7 .0 2 = 8 8 .6 6 6 /-4 0 .0 2 2 °
Input p.f. = cos (40.022°) = 0.7658 lag
Input power = 14960 + 174.85 + 160 x =15302.77 W.

E x a m p le . 1.33. The m axim um efficiency o f a 1-phase, 1 1 0 0 0 /4 0 0 V, 5 0 0 k V A tra


is 98% and occurs at 80% fu ll load, unity power factor. The percentage im p ed a n ce is 4.5%. Load
power factor is now varied while load current a n d the supply voltage are h eld c o n s ta n t a t their
rated values. Determine the load power factor at w hich the secondary term in a l voltage is m in i­
m um and fin d the value o f the later.
) S o lu tio n . Total losses in transform er
' 1 ^
- ’1 x 500,000 x 0.8 = 4858.3 W
0.988
These losses are under th e condition of m axim um efficiency. T herefore, core losses = ohm ic
losses a t 80% full load = —x 4858.3 = 2429.15 W

100
Full-load ohmic losses = 2429.15 = 3795.55 W
80
From Eqn. (1.36), r „ = x 1 00 = 0.759%

S = = (4.5 2 - 0.759 2] 1/2 = 4.4355%


I t is already proved th a t load p f a t w hich voltage reg u la tio n is m a x im u m o r ik „ i„.„i „ r
a t which secondary term in al voltage is m inim um , is given by a x lm u m . o r t h e load pf
^2 0.759
cos 0 , = — = ^ 5 - = 0.1687 lag

Now Z2 -V 2 . o .
£>2 (re2 cos 02 + sin 0 2) in p er u n it

E —V
••• x 100 = (0.759 x 0.1687 + 4.4355 x 0.98567) = 4 .5
or
V2 = E 2 ( 1 - 0.045) = 400 x 0.955 = 382 V
Second aiy term in al voltage = 382 V

Scanned b y Cam Scanner


Transformers 63

m p le 1.34. A 5 &VA, single-phase transform er has a core loss o f 40 w a tts a n d fu ll load


flic loss o f 100 w atts. The d a ily varia tion o f load on the transform er is as follow s :
7 A-M. to 1 P.M. 3 k W a t p f 0.6
I P.M. to 6 P.M. 2 k W a t p f 0.8
6 P.M. to 1 A.M . 6 k W a t p f 0.9
I A.M . to 7 A .M . N o load.
Determine the all d a y efficiency o f the transform er.
Solution. N ote t h a t core loss re m a in s fixed a t 40 W, w hereas ohm ic losses v ary as th e
square of the kVA load.

7 A.M. t o ! P.M ., kVA load = = 5 kVA

• Ohmic losses for 6 h o u rs = (100) = 100 W

1 P.M. to 6 P.M. ; kVA load = — = 2.5 kVA

2.5
Ohmic losses for 5 h o u rs = (100) = 25 W
5.0

6 P.M. to 1 A.M. ; kVA load = = ^kV A

20
.*. Ohmic losses for 7 h o u rs = (100) = 177.8 W.
3x5
1 A.M. to 7 A.M.; O hm ic losses = 0
1
Daily energy lost as ohm ic losses = ((100) (6 ) + (25) (5) + (177.8) (7)1

1969.6
= 1.97 kWh
1000
40 x 24
Daily energy lost as core loss = 0.960 kWh.
1000

•• T o tal kW h lost = 1.970 + 0.960 = 2.93 kWh.


D aily kW h o u tp u t = ( 3 x 6 + 2 x 5 + 6 x 7 + 01 = 70 kW h.
All day r) = 1________ D aily losses in kWh________
(D aily o u tp u t + D aily Losses) in kWh
2.93
=1- = 0.9598 or 95.98%.
72.93
Beth 10 ®eP a r a t >o n h y s t e r e s i s a n d e d d y c u r r e n t lo s s e s . In th is a rtic le is given a
mod for sep aratin g th e tra n sfo rm e r core loss into its two com ponents of h y ste re sis an d eddv-
^ n t losses.
For a sine flux w ave, th e tra n sfo rm e r core loss, from Eq. (1.47) is

P ' = Kh fB „ x + K ,fe B j ...(1.47)


It
wher may be seen from th is e q u atio n th a t h y steresis loss varies lin early w ith frequency
48 the eddy c u rre n t loss v a rie s as th e sq u are of th e frequency.

Div,8»on of both sid es of Eq. (1.47) by f, gives,

S ^ ^ ^ c T B y Cam Scanner
________
m
il IArl. 1,10
64 Electrical Machinery

...(1.58)
Core loss per cycle = -~ = Kh B mx + K e f B
• . , ic inrlpnendent of freq u en cy w h ereas eddy
Eq. (1.58) shows th a t hysteresis loss per cycle i P
curren t loss per cycle in proportional to frequency.
Now V = E = V2n f N ij>max

or - j = ^2n N ■tymax = ^l2 n N B m A,

For any transform er N any A ,are constant, therefore, Fo

Y or of maximum flux density B m, Eq. (1.58) can be w ritten as

...(1.59)
= K i + K2f
f
where constant K\ = Kh B xm and constant K 2 = Kc B m .
Equation (1.59) is the equation of a straig h t line. C onsequently, th e p lo t of Pc/ f against
frequency/’, results in a straig h t line, as shown in Fig. 1.30. _
c k
The values ofK x and K 2 in Eq. (1.59) can be determ ined ~ f
by performing open circuit test on the transform er. D uring
this test, the applied voltage V and frequency /'a r e varied
together so as to keep W /‘(and therefore B m) atynost con­
stant. In the laboratory, the ratio V / f can be m aintained
constant by a d.c. m otor-driven a lte rn a to r feeding th e
transform er on open circuit. An adjustm ent of the speed of
the alternator and its excitation help in keeping ratio V / f
constant. A w attm eter during the open circuit test registers
the core loss. After Pc, V and f are recorded, the core loss per
cycle is plotted against frequency f. The intercept of the
straight line on the vertical axis, gives the value of constant F re q u e n c y
K x and the slope of the line A B gives constant K2. Once K x Fig. 1.30. Plot o f core loss
and K2 are known, the hysteresis and eddy cu rren t losses per cycle again st frequency.
can be determined separately. From Eq. ( 1 .5 9 ),
Pc - + A2 / = p h
Ph = K xf \
and p .-* * ? ...(1.60)

For the separation of core loss of a three-phase transform er, th e read in g s should be changed
to per phase values,• before plotting - j against
i---£ “gaillDl /. f.

f z } : z t x , : hm performed on ,he *•»*


Terminal voltage in V
214 171.00 12S.4 85.6
Frequency in H z
50 40
Power input in W ------i 30 20
I 100
72.5 50 30

Scanned by Cam Scanner


Transformers 65

Determine the hysteresis a n d e d d y cu rrent losses a t :


(a) 6 0 Hz an d
(b)4 0 Hz-
golutio*1. F irs t of all th e re a d in g s are changed to p e r p h ase values, as given below :

214 171.00 128.40 85.6


33.3 24.2 16.67 10

It may be seen t h a t th e ra tio ^ h a s a c o n sta n t valu e of 4.28. T herefore, Eq. (1.59) can be
used for separating th e h y s te re s is a n d eddy c u rre n t losses.
1 • PC
The core loss p er cycle, i.e. — is calc u la ted in a ta b u la r form as follows :
\

0.667 0.605 0.556 0.50


,

f 50
40 30 20

pc ■ , •
In Fig. 1.31, is p lo tted a g a i n s t/ 1.T h e s tra ig h t line so obtained in te rse c ts th e v ertical axis
at the point A. The in te rc e p t OA gives th e value of K x equal to 0.39. T he slope of th e lin eA B can
be obtained at any frequency, say 50 Hz.
:. K2 (50) = 0.667 - 0.39 = 0.277
0 277
or ^ 2 = ^ i l = 0.00554.
ou

^ acanned
n by Cam Scanner
! [Art. 1.11
66 E le c tric al M a c h in e ry

From Eq. (1.59),


(а) Ph per phase = (0.39) (60) = 23.4 W
Pt per phase = (0.00554) (60)2=19.95 W ^ x 3 = 7 0 .2 W and 59.85 W
Total hysteresis and eddy-current losses a
respectively.
(б ) Ph per phase = (0.39) (40) = 15.6 W
P, per phase = (0.00554) <40)2 = 8 .8 6 W ^ W and 26.58 W respectively.
Total hysteresis and eddy-current losses a
1.11. T e s tin g o f T r a n s f o r m e r s transform er and in th is article only a few of
A v ariejy • B e s t* - 1
[ab T ra to ty^ d are helpful in gaining a E1+ E2
b etter physical insight into th e tra n s ­
form er behaviour.
(a) Polarity test. On th e p rim ary 0----------------- ,-4-
side of a two-winding transform er, one
term inal is positive w ith respect to th e
other term in al a t any one w s ta n t. A t
th e sam e in stan t, one term inal of the
seco n d ary w in d in g is P0Sltl.veT™
respect to th e o th er term in al. T hese *
relative polarities of the p n m a ry and
seco n d ary te rm in a ls a t an y in s ta n t , <*»
FiP 1 32 Polarity test on a two w inding transform er
m ust be know n if th e tran sfo rm ers are subtractivepolarity and (6) additive polarity.
to be operated in parallel or are to be
used in a polyphase circuit. fo rm er i e. A,
- e n viewed from th e h v ^ M h e — « m a ^ A , ^ ^ ^ •

being on th e extrem e n g h , p rn n n pcted to one end of th e seco n d ary w inding and


a rb itra rily in Fig. ^ t ^ f X seco n d ary w inding. A voltage of
a voltm eter ,s connected betw een v4 , a« ^ ^ ^ ^ ^ ^ induced 00 h,
s u i t a b l e v a l u e is now app :q oniial tn F - En th e n seco n d ary terminal
l „ c ir lp s r e s D e c t i v e l y .
If t h e v o l t m e t e r read in g is equal t o £ 2, tn e n s e c o n y
erted to A is positive an d is m ark ed u „ th e l.v. te rm in a l co n n ected to A 2 th ro u g h
voltm eter is n eg ativ e an d is m ark ed a 2 as show n in Fig. 1.32 (a). If v o ltm e te r re a d in g is equ^
to E l + E 2, th e n th e te rm in a ls connected to A , a n d A 2 a re n e g a tiv e a n d p o sitiv e a n d a re m ^
02 and a , respectively as show n in Fig. 1.32 ( 6 ). T h e s u b s c rip t n u m b e rs 1. 2 on th e h * . a n ^

P w indings a re so a rra n g e d t h a t w hen A 2 is n eg ativ e w ith re s p e c t to A ,, a 2 is also n eg a


respect t o n , a t th e sa m e in s ta n t. In o th e r w ords, if th e in s ta n ta n e o u s e m f is directed from .
to Aj in h.v. w inding, it is a t th e sam e tim e d ire cted from a 2 to a x in th e l.v. w in d in g . ^
W hen th e v o ltm e te r re a d s th e difference E , - E 2> th e tra n s f o r m e r is s a id to posses 5 a ^
tractiv e p o la rity Fig. 1.32 (a) a n d w h en v o ltm e te r re a d s E x + E 2, th e tr a n s f o r m e r h a s ^
p o larity Fig. 1.32 (b). In s u b tra c tiv e p o la rity , th e v o ltag e b e tw e e n A 2 a n d a 2 ( or i ^
reduced. T h e lead s connected to th e s e te r m in a ls a n d th e tw o w in d in g s a re , t h e *"e °nnI1ected
jected to h igh v oltage s tre s s . In a d d itiv e p o la rity th e tw o w in d in g s a n d th e le a d s c

ocanneu uy uarriocaririui
Transformers 67
# -----------------------------------
, and fl2 are su ^jected to high voltage stresses. On account of th ese reasons, subtractive
^ rityis Preferable to additive Polarity.
(b) Open circuit a n d (c) S h o rt circuit tests. These two te sts have already been described in
detail in A rt-1-7-
{d) load test (Back to back or S u m p n e r ’s test). A load te st on a tran sfo rm er is necessary if
maximum tem p eratu re n s e is to be determ ined. A sm all transform er can be p u t on full load
L means of a suitable load im pedance. B ut for large transform ers, full load te s t is difficult,
Ic e it involves c o n s id e ra te w aste of energy and a suitable load, capable of absorbing full load
oower, is not easily available. However, large transform ers can be p u t on full load by m eans of
Sumpner’s or back to back test. The S u m p n er’s te st can also be used for calculating th e efficien­
cyofa transformer, though th e la tte r can be determ ined accurately from open-circuit and short-
circuit tests.
The back to back te s t on single-phase transform ers requires two identical units. Fig. 1.33
illustrates the circuit for two single-phase transform ers, w here two prim aries connected in
parallel, are energised a t ra te d voltage and rated frequency. W ith secondaries open, th e w a tt­
meter Wi records th e core losses of both th e transform ers. The two secondaries are connected
in series with th e ir p o la ritie s in p h a se opposition, w hich can be checked by m ean s of a
voltmeter. The range of th is v o ltm eter connected across term inals ab, Fig. 1.33, should be
double the rated voltage of e ith e r tra n sfo rm er secondary. Zero voltm eter reading ( = 0 ) indi­
cates the secondaries are connected in opposition. Now, if th e term inals ab are short-circuited,
the current in th e secondary would be zero because Vab = 0 and th e w attm eter reading w [
remains unaltered. In case th e voltm eter reads th e sum of th e two secondary voltages, th e
secondaries are in th e sam e phase. In order to bring them in phase opposition, term in als ad
should be joined together to re su lt in zero voltage across term inals be.

Voltage
regulator
O'

V a r ia b le v o lt a g e

i
1-33. Sumpner’B (or back to back) teat on two identical aingle-phase transformers.

Scarinecn5y C am Scanner
I?
lA rt. I.]}
68 E lectrical M achinery _ -
,• „nfi th e two second
In F ig . 1.33, it is assu m ed th a t v o lta f “ ™ ^ e° s ec„ndary circuit by m eans of a voltage
p h a se op p o sitio n . N ow a voltage is injected l m aries or from a sep a ra te source. The in­
reg u la to r, fed from th e source connected to the pri < serie s-connected secondaries. By
jected v o lta g e is ad ju sted till rated current flows in t N ote th at th e full lead current
tra n sfo rm er action , prim ary w in d in gs also carry ™ . ^ars (s hown dotted) and, therefore,
in th e p rim a ries, com p letes its path through th e ma ^ ^ ^ ^ reading of voltmeter
th e rea d in g o f w a ttm e ter Wx rem ain s u n aflecte . transform ers. The low-injected
V2 is eq u al to th e su m o f leak age im pedance drops in w in d in gs, therefore, the
v o lta g e h a s g iv en rise to full load currents in th e w a ttm eter W2 (Fig. 1.33).
fu ll load ohm ic lo sse s o f both th e transform ers former then th e reading of wattmeter
I f P , and P „ are th e core and ohm ic lo sses £an now be d eterm ined by using Eq.
W} = 2 P c and th a t o f w a ttm eter W2 - 2 Psc- 1
(1 .5 2 ) or Eq. (1.55). , though th e transform ers are not supply-
It is s e e n from above th a t in S u m p n e r . te st even g ^ ^ ohm ic., occurs,
in g an y load current, y e t full iron-loss occurs m t h e ^ + 2 p j , f tem perature rise of
th e ir w in d in g s. N e t power input ^ t f e t w o ^ r a n ^ ^ . kept u n d er rated loss
th e tw o tran sform ers is to be m e ’ b| tem perature is reached,
con d ition s for sev era l hours directions o f /„ and f 2, th e prim ary current
I f 2/„ is th e no load current, then ^ prim ary curren t o f transform er 2 (sum

° [ r ar dT r ^ e r e ^ t h ^ t r s f o r m e r s do not operate under id en tical co n d ,tio n s-o n e ma,

s t s s r is r r p a = Circulating
tran sform ers can be carried out on a sing
current
u n it. For th is te s t, th e prim ary and secon ­
dary w in d in g s are connected in d elta, F g.

q u en cy. T h e w a ttm e ters W, and W, recoru


t total core loss. T h e h.v. w in .n F in op en
d eita , is con n ected to a ^ h a r y s .n g ^ p h a se
cu ddIv o f a n y c o n v e n ie n t freq u en cy,
voltage injected in the open d elta ^ a d ju sted Circulating
till full load current flows in it. T lm sto rin e Current
action s e ts up full load cu rren t in th e kv. H.V. winding
w inding also. A s before, th e readings o fw a tt-
m S e m V , and W2 are uneffected and watt-
- t ee te
m ^ r, ewc oo rr d
r rV d ss th
the ohmic losses
e ohmic l o s s eins ^all the . 1 K * )-* transfo„o«.
three phases. N ote th a t th e m agnitude of th e FiE. , . 34. Dummy load test on a three P

low -injected vo ltag e ini t h e opan h® a j th e pcr p h ase eq u iv alen t leakage "«P e cr,
equal to (full load c u rren t) (& „,), w here z „ F ^ ^ ^ # th re c .phosc Ira n .* ^
^ U ™ t T ^ I m s h " l d i n g in delta, w hereas th e lower diagram s . -

winding in open delta.

Scanned by Cam Scanner


Transformer* 69

'T ^ q /T w o single-phase transform ers o f ratings 10 kVA, 2 3 0 /4 0 0 V and 10 kVA,


p » & V le ‘e m ployed for back to back test. The l.v. side is fed from 230 V supply m ains
}l4l0 V
a/410 y are\ e reg u la to r connected
regulator connectea to
1 0 the
m e sam e 230
zju V
v m ains feeao
feedo ±5AA in
in tne
the series
seriesconneaeu
connected
*n<iO
Mf*°s a voltag . *
fo r a core loss o f 200
2 0 0 W in each transformer, calculate the readine
reading o f the w
watt-
att­
, _ the I v. side. E ach transform er has its reactance equal to three times its
^terc°nne
ti*ter
f(sista,ice', r ja(j t he voltage ra tio s of th e two tran sfo rm ers been equal, the w attm eter on the
4qq y j B u t th is is n o t th e case in the present example.
]v gide A induces th e following cu rre n ts in th e prim aries ofFig. 1.35 :
A«” re” ‘ ,. . 25
2 5 x 4400
00 1000 .
Primaryc u rre n to ftra n S m er 230 “ 23
. „ 25 x 410 1025
rimaiy cu rren t of tra n sfo rm e r 2 = 23Q - 2 3 A

two p rim ary c u rre n ts , caused by 25 A in th e secondary w i n d i n g s * shown in^Fig


These two ^ lUiJO iuuu
. ii • ._-A.*___ 17 chnu/C that. : “
• hhoffs c u rre n t law a t th e ju n ctio n K of this figure shows th a t 23 23 23
5-
!® Kircnn Thus the reading of 1
a m m e te r A , a n d th e c u rre n t coi. of w attm eter W,. T hus the reading of watt-
gjIjStllO 25 n
W is effected by an a m o u n t equal to 230 x ^ x cos 0*.
meter

F ig 1.35. P ertaining to Example 1.36.

Now

Ze
effected
■■The amount by w hich reading o f wattmeter W, is e ec
79.1 w atts.
2 5 ^ 1 = 79.1
= 230 x g s * /IQ

Scanned by Cam Scanner


lArt. I .12

70 Electrical M achinery ---------------------- ~ nte 25/23 A and I 0 flow in the


W ith te rm in a l« connected to c * * *“
same direction in the current coil o w „ , 4 7 9 .1 w atts.
... The w attm eter reading is increased to <400 + 7 9 ^ ^ ^ opposi4e t h othcr
If c is connected to 6 and d to o. itw il « e e m ^ tQ (4 0 0 _ 7 9 .!, = 320.9 w atts,
and consequently the reading of w attm eter , 3 2 0 .9 w atts, depending on the
Hence the reading of w attm eter W, may be 479.1 wa
a r r a n g e m e n t o f c o n n e c tio n s . t r a n s f o r m e r has its p rim a ry and seco n -
arrangem e A 3 0 0 k VA,
3300/400 V, t h r e e - p h a s e t n n s ^ ^ w c t/r r e n < ^
E x a m p le . • a dum m y load te st F • 3 g 0 V. Calculate the
dary windings connected in deltaJ , 0f h.v. windings, is
magnitude o f the voltage injec e ,. un^ system,
leakage i m p e d a n c e per phase m ohmsandmper J
Solution. The voltage across each phase of the h.v. w
= 3 6 £ = 1 2 0 V = VSC

On the h.v. side, v Ph - 3 3 0 0 V


3 0 0 x i 0 ! = 1 0 0 A = /sc
and V " 3 x 3300 3.3
120
r SC
= 3.96 n .
z eH *n ° ^ mS xsc 100/3.3
„ Vtese_ r a t e V ^ = _ 3 3 0 0 _ = 1 0 8 9 Q
base" hast rated h* 1 0 0 / 3 -3
ztH in Q _ 3-9§- = 0 0364 p.u.
•• z* , n p U = _ Z ^ T ' 108-9 P

1.12 A u t o - t r a n s f o r m e r s common to both th e p rim ary a n d secondary


A transform er, in -^ . c h a P a r t “ the ™ gwindi transform er, p rim ary and secondary
but in an auto-transform er th e two w indings a re n o t electn-

Ca” A S tap le arrangem ent of a step-down auto-transform er is depicted in Fig. 1.36, where W,
and N , are the num ber of tu m s between winding AC and w inding B C respectively. W hen vo -
1.

o
o
o
o
($:) o
o
o
N, B jL
+
g »i 12
o,
01
o
(*V) 2o

C I,

fa) (6 )
Fig. 1.36. Single-phase step-down auto-transformer.

r i. ' -----;~5- —• --------- --


Scanned by Cam Scanner
Transformers 71

1/ is appKe(* w*n (b n £ a n ex citin g c u rre n t s ta r ts flow ing th ro u g h th e full w in d in g


^ ,f\ h e in ternal im p ed an ce d ro p is neg lected, th e n th e voltage p e r tu r n in w in d in g A C is
f 1 1 d therefore, th e v o ltag e acro ss E C is {VX/ N X)N 2.
^ At first sight, an a u to -tra n s fo rm e r a p p e a rs to be s im ila r to a re sista n c e p o te n tia l div id er.
k this is not 80’ aS ^e8C below . A re sistiv e p o ten tial divider ca n ’t s te p u p th e voltage,
k reas it is possible “ 811 » n to -tra n sfo rm e r; th e p o ten tial divider h a s m ore losses a n d is,
w'refore, leSS efficien^ In a Po te n tla l divider, alm o st e n tire pow er to load flows by conduction,
there®9 au to -tran sfo rm er, a p a r t of th e pow er is conducted an d th e r e s t is tra n s fe rre d to load
1 tran sfo rm er action. In a p o te n tia l d ivider, th e in p u t c u rre n t, m u st alw ays be m ore th a n th e
.tout current, th is is n o t so in an au to -tra n sfo rm e r. If th e o u tp u t voltage in a u to -tra n s fo rm e r
0 less than the in p u t voltage, th e load c u rre n t is m ore th a n th e in p u t c u rre n t. Som e of th e
Elements made h ere can be a p p re c ia te d only a fte r going th ro u g h th e re m a in in g p a r t of th is

When the sw itch S is closed, a c u rre n t / 2 s ta r ts flowing th ro u g h th e load a n d c u rre n t 7j is


taken from the source. N eglecting losses,
In p u t pow er = O u tp u t power
or V x I x cosQ x = V2 / 2 c o s 02
If internal (or leakage) im p ed an ce drops and losses are neglected, th e n
cos 0 ! = cos 0 2.
Hence ^ri f i = V2 / 2
Y l 11 N2
* v . '/ r w r * ~ ( l -6 l >
Here k is less th a n u n ity . T h e re la tio n s expressed by Eq. (1.61) are identical w ith a tw o
winding transform er, see Eq. ( 1 . 1 1 ).
Let the term inal A, Fig. 1.36, be positive w ith respect to te rm in a l C, a t th e in s ta n t show n.
Then at no load, th e exciting c u rre n t flows from A to C and it esta b lish es a w orking m m f
directed vertically dow nw ard in th e core. W hen sw itch S is closed, th e c u rre n t in w inding B C
must flow from C to B, in o rd er to cre a te an m.m.f. opposing th e exciting or w orking m .m .f.,
as per Lenz s law. Since th e w orking m.m.f. in a tra n sfo rm er rem ain s su b sta n tia lly c o n sta n t
at its no-load value, th e p rim a ry m u st d raw additional c u r r e n t/! from th e source, in o rd er to
neutralise the effect of c u rre n t I CB. In w inding AC, I x flows from A to C w hile in w inding BC,
2 ows from C to B. T herefore, th e c u rre n t in w inding B C is I x from B to C a n d / 2 from C to
ere the current / 2 is g re a te r th a n I x (because V2 < V x) and th e ir m .m .fs. a re opposing each
other at every in sta n t, th erefo re,
ICB=h ~ h -
® m of w inding A B = I x (N x - N 2)

= (J 2 ~ h )^ 2 ( V I XN X= I 2 N 2
. = I Cb -^2 = m .m .f. of w inding CB.
w'adine j ^ re^ore> 8een t h a t th e tra n sfo rm e r action ta k e s place betw een w inding A B and
o th e r w o rd s, th e v o lt-a m p e re s acro ss w in d in g A B a r e tr a n s f e r r e d by
er action to th e load connected across w inding BC.
Totar " 0rm ed V A = V * b 1a b =(V , - V 2) / ,
Put VA to tra n sfo rm e r = V XI X= o u tp u t VA.
•• S g jfo n B e d W V _ ( V ,- V ^ / , V.
In p u t VA V xl x V| --.(1.62)

Scanned by Cam Scanner


lArt. 1.12
72 Electrical Machinery
r T = (V - Vo) I \ are transform ed to the
O ut of th e in p u t volt-am peres V j/j, only ab ab 1 required for th e output, are
o u tp u t by tran sfo rm er action. The rest of t e vo
conducted directly from th e input.
• Conducted VA = Total input VA - transform ed VA
' = V 1/ 1 - ( V 1 - V 2) / 1 = V2 / 1
Conducted VA Y l l l - h —(163 o)
“ In p u t VA V \h
N eglecting in tern al im pedance drops and losses, Eqs. (1.62) and <1.63, become
Transform ed power _ j _ £ ...(1.64)
In p u t power
Conducted power _ ^ ...(1.63 b)
and Input power

Eqn. (1.64) m ay be w ritten as


Inductively transferred power _ High voltage - Low voltage
------------ Total power High voltage

winding. T hus the w eight of conductor in a w inding is proportional to th e c u rre n t a n d number


of tu rn s in th e winding.
For an au to tra n sfo rm e r of Fig. 1.36, w inding A B c a rrie s a c u r r e n t o f J , a n d has
(N x - N 2) tu rn s.
... W eight of conductor for w inding A B «* {Nx - N 2) I x
W inding B C carries a cu rre n t of ( / 2 - 1\) and h a s N 2 tu rn s .
W eight of conductor for w inding B C « (/ 2 - 1\) N 2
Hence, total w eight of conductor in an au to -tran sfo rm er is
°c l x (N x - N 2) + ( / 2 - 1\) N 2
oc 2 {Ix N x - I x N 2)
oc 2 (N x - N 2) 11
If th e w eight of conductor in a n a u to -tra n sfo rm e r an d a n o rd in a ry tw o w in d in g transform er
is to be com pared, th e tw o types m u st h av e th e sam e in p u t (V^ I x), sa m e o u tp u t (V 2 Z2) and same
voltage ra tio Vx/ V 2. T herefore, an o rd in a ry tra n sfo rm e r m u s t h a v e v o ltag e V x, V2 and currents
I h I 2 for its p rim a ry an d secondary w indings respectively. I f p e a k flu x d e n s ity a n d core area are
assum ed equal, th e n e.m.f. p e r tu r n E t (= V2n f B m A,) in th e tw o ty p e s of tra n sfo rm e r, is
( \ ( \t \ .
same. Thus a two winding transformer must have N x = — and iV9 turns in its prim ®1”)
E,
V /
and secondary windings respectively.
/. Total weight of conductor in a 2-winding transformer
°c /i N x + /2 N 2
oc2Ix N x
r Transformers 73

.-^ !g co n d u c to ^ in jj^ a u to tr a n s former


• W eight of conductor in
_ 2 (N l -jV 2) / 1 n 2

W iii = l~ W r l ~ k
Weight of conductor in auto-transformer = (1 —k)
°r (Weight of conductor in 2-w inding transform er)
or C o n d u cto r weightt in ^ w in d in g tra n sfo rm er - Conductor w eight in au to -tran sfo rm er = (k)
conduct01- weight in 2 -w inding tran sfo rm er.
Q o \r i n rr n f _________________ _ • _
Saving of conductor material
j

Conductor w eight in
Thus if auto-transformer is used = k x
2 -winding tran sfo rm er
If * = 0 „1; f ' S ng o f ™ndUCtoJ raaterial is ™>y 10% and for k = 0.9, saving of conductor
material is 90%. Hence th e u se of au to tra n sfo rm er is more economical only w hen th e voltage *
ratio k is more n e a re r to u n ity . J B
D u r i n g the design, th e w indow dim ensions are decided from th e consideration of insulation
and conductor m aterial. For an au to -tran sfo rm er, a reduction in conductor m aterial m eans
3 1
lower window area and, therefore, reduced core length. Hence, for the sam e core area, the I1
weight of auto-transform er core is decreased. Therefore, saving in both conductor and core i. !
materials is effected by th e u se of an auto-transform er, in place of a tw o-w inding tran sfo rm er.
The nearer the value of k is to unity the greater is the saving in materials.
Other advantages of an auto-transformer, over a two-winding transformer are given below :
{u) Owing to th e re d u c tio n in conductor and core m aterials, th e ohmic losses in conductor
and the core loss are low ered. T herefore, an auto-transform er h as higher efficiency th a n a two-
winding transform er of th e sam e outp u t.
(m) Reduction in th e conductor m ateria l m eans lower value of ohmic resistance. A p a r t of ■
the winding being common, leakage flux and, therefore, leakage reactance is less In o th er
words, an au to -tran sfo rm er h a s low er v alue of leakage im pedance and h a s superior voltage
regulation th an a tw o-w inding tra n sfo rm er of th e sam e output.
D isad v an tag es, (i) If th e ra tio of tra n sfo rm atio n k differs far from unity, th e economic
advantages of a u to -tran sfo rm er over tw o-w inding tran sfo rm er decrease.
(li) ^ ™ain d isad v a n ta g e of an au to -tran sfo rm er is due to the direct electrical connection
ween the low -tension an d hig h -ten sio n sides. If prim ary is supplied a t high voltage, th e n an
pen circuit in th e com m on w inding BC, would re su lt in th e appearance of dangerously high
m° . a,ge oniJje l v *sid e - T h is h igh voltage m ay be d etrim en tal to the load and th e persons work-
B re. Thus a su ita b le protection m u st be provided ag ain st such an occurrence.
(«i) The short-circuit c u rre n t in an au to -tran sfo rm er is h ig h e rth a n th a t in a corresponding
w°-winding tran sfo rm er.

c i r c u i t . A n au to -tra n sfo rm er can be considered as a 2-w inding tra n sfo rm e r


g Wlnding A B of (N x - N 2) tu rn s as th e p rim ary and w inding B C of N 2 tu rn s as th e
naary. Fig 1 .3 7 (a ) T h u s, a n au to -tran sfo rm er behaves as a 2 -w inding tra n sfo rm e r w ith
out T applied voltaSe Vob (= V \ ~ v 2 ) across A 0 , p rim ary w inding c u rre n t 7lf secondary
for tV V° 6 sec° n d a ry w ind in g c u rre n t ( / 2 - / 1) as shown in Fig. 1.37 (a). T herefore,
to* transform er o fF ig . 1.37 (a),

p rim a ry w in d in g tu rn s ^1 ~ ^2 _ 1
, secondary w in d in g tu rn s N2 k ’
ereMisgiven by Eq. (1.61).

u Lry u d 11 u u d i n itM
fArt. 1.12
74 Electrical
E le c tric a l Machinery ___________________________________
r, fnr th e a u to -tra n s fo rm e r can be writ-
N eglecting th e exciting cu rren t, th e v o ltag e e q u a t.c n for th e
ten by trea tin g it as a 2-windm g tran sfo rm er r e fe r red to p rim a ry + p rim a ry leakage
P rim ary applied voltage, Vai = secondary voltage V , r e ^ d ^
im pedance drop + secondary leakage im pedance P

y ab = Y l {Nl - N 2) + h (ri +jxi) + (h - h ) (r 2 + j * 2 )


N o2
jy i • •
w here (r, + , * , ) and <r2 +J'x 2) are th e leakage im pedances of p rim a ry a n d se c o n d a ry windings
respectively.
But primary mmf, h (AT, - N2) = secondary mmf, (/2 - h ) * 2
(N i~ N z N
or (h ~ h ) ~ h ^
S ubstituting th is value of (/2 - I x) in th e above expression for Vab, we g e t

(Nt -N 2 ) N x- N 2 )
+ /i (ri + j* i) + I l (r2 + jx 2) V ,-V ,
^ 6 = ^2 N2
No

vnh= [f 2 1M -J JCi + X2 f 1 k- * ' 2 \


or = v.
V2 ^ k ' + / l \ r l + r2 k
> )
I % J b
/ v.
Fig. 1.30 (b) reveals that

....................., , . V2 w 1 - k \2
^1 = Vab + V2 = ~ ^ +h (.re l+j* e l) = ~ k +I\ Zi + 22 ...(1.65)
F
p
2
fro m
re\= rx + r2
II
+

k (^2 - j
N‘ )
»

xel = x x+ x 2 = X1 + x2 h~k1
k/
( * J \ or
z i = r i +J x i = p rim a ry leakage im pedance
and z 2 = r 2 + J X2 = secondary leakage im pedance. F
in Fij
it (JU65) givf s the e<luivalent circuit for an auto-transform er as shown in Fig 1 37 (b)
S
f ; ^ „ “ „ ;es,stance and the r“ e ■» * » •» * ~ num l
{N 2—
lo a d ^ r o & ^ X ” "6 transf0rm er' the ro ,ta * * “ * * « « - sec„ndary in F if
E2 = V2 + (I2 - I 1) (r2 + j x 2)
where E2 = e.m.f. induced in N 2 turns
V2 = load voltage
and (I2 - / , ) = current in secondary winding CB.
Phasor sum of V2 and (7, - Wr + / r \
behind V2 by load power angle 6,. E.m f F ;a • 1 3 7 ^ w here h is assum ed to lag
T
B, by 90”. Induced e.m.f. B, in N, turns is shown equal a T b efo re '^ “ Sh0W" 'eadi"e tw eer
tran s;
or V , - = | £ l | . 6’
0

fcttM
hMI *«u- --

Scanned by Cam Scanner


For the prim ary circuit, applied voltage V x h a s to balance th e induced e.m.f. E x, voltage rise
fromB to A (against th e direction of I x) an d voltage drop from C to B (in th e direction of c u rre n t
(W i)).
= E i + / i (z-! + j x j - ( / 2 - I \ ) (r 2 + j x2)
E
or V\ = + I\ (rj + jxi) - ( / 2 - 71) (r 2 + j x2)

First I x (rj + j Xj) is added to | E x \ = V x and th en (72 - 7t) (r 2 + j x 2) is su b tra c te d as show n


in Fig. 1.37 (C) to get th e applied voltage Vj for a single-phase au to -tran sfo rm er.
Step-up A u to - tr a n s f o r m e r . Fig. 1.38 shows a 1 -phase step-up au to -tra n sfo rm e r. T he
“umber of tu rn s in w in d in g A C is N 2 an d in w in d in g BC is N x so t h a t w in d in g A B h a s
(^2 ~N{j turns. H ere V2 > V x an d I x > 72. The d istrib u tio n of cu rre n ts in th e w indings is show n
mPig. 1.38. As before, assu m in g ideal conditions,

(less th a n one)
Vj
M m f. of w inding A S = / 2 (AT, - JV,)
= / , AT, - / 2 JV, = (/, - / 2) JV,
= m.m.f. of w ind in g B C
tween 8 . 8^pws t h a t tra n s f o rm e r actio n occurs be-
tran8f0^ 1C^ ng ^ C an d A B as in a step-dow n au to ­

Outpu t of step-up au to -tra n sfo rm e r Fig. 1.38. Single-phaso step-up


Put of equivalent tw o-w inding tra n sfo rm er auto-transform er.

S c a n n e a b y C am S canner
_________________ [Art. 1 .1 2
76 Electrical Machinery '

L _ = —L - .
'< v 2-v ,)/ 2 . h i - *
V 2• .
f ctpn-uD a u to -tra n sfo rm e r is also of the
Thus, th e advantage of enhanced power ratin g ° F u rth c r a n a lysis of step -u p auto­
sam e order as in th e case of step-down au o- lfn_tra n s former.
transform er can be carried out as in a step^ own a rg a n d t w o . w i n d i n g t r a n s ­
C o m p a r is o n o f c h a r a c t e r i s t i c s o f a u o- r ® , tra n sfo rm er by connecting its two
fo rm e rs A two-winding transform er can be used as an au to -tran sto rm e y
windings in series electrically. ^ considered as a tw 0.w inding
As discussed above, auto-transform er o ?• j c th secondary. R atin g s and char-
transform er with winding AB as prim ary and wind m g B C a ^the secon y *
acteristics of auto-transform ers and 2 -winding transform ers so obtainea, no ,
are discussed below : , , . ,.
<i, E a tin g s . It is seen from Fig. 1.36 th a t wm ding A B acta as ‘he p= and w tn rng
BC as the secondary of a 2 -winding transform er. As per Eq. (1.61), Fig. 1.36 and big. l.d (a),
kVA rating as an auto-transf
kVA rating as a 2 -wdg tra n sf
Prim ary input voltage Vi x prim ary input cu rre n t 11
= Prim ary voltage across wdg AB x prim ary cu rren t in AB
V i/i_________ 1 _ 1
= ( V i- V 2) I , 1 - O W j ) 1 -*
kVA rating as an auto-transform er _ Y&l2 _ 1 - __
s0 kVA rating as a 2-wdg transf. V 2 (l 2 ~ I\) (1 ~ _ ^
(ii) L o sses. When a 2-winding transform er is connected as an au to -tran sfo rm er, th e cur­
rent in different sections and voltages across them rem ain unchanged. T herefore, losses w hen
working as an auto-transform er are the sam e as the losses in a 2 -w inding tra n sfo rm e r. P e r u n it
losses, however, differ.
Per unit full-load losses as auto-transform er
Per unit full-load losses as 2-winding transform er
Full-load losses_______ kVA ratin g as 2-wdg. tra n s .
kVA rating as auto-transform er Full-load losses -
(Hi) Im p e d a n c e d ro p . When a 2-winding tran sfo rm er is used as an a u to -tra n sfo rm e r, both
l.v. and h.v. w indings are utilized completely. In addition, cu rre n t an d voltage ra tin g s of each
winding section rem ain unaltered. Therefore, im pedance drop a t full load is th e sam e in both
the transform ers. Their per u n it values are, however, different. W hen re fe rre d to h.v. side, per
unit im pedance drop as an auto-transform er is w ith respect to voltage V, a n d for a 2 -winding
transform er, it is w ith respect to (V'j - V2) = (1 - k).

Per u n it im pedance drop as an au to -tra n sf


Per u n it im pedance drop as a 2-wdg tran sf.

W ( V j “ V2) V,

drop^ V° Itage re&ulation- Regulation in transformers is proportional to per unit impedance

Regulation as an auto-transf
Regulation as a 2-wdg transf = ^ ^

Scanned by Cam Scanner


r
Transformers 77
/ \ ch o rt-circu it c u r r e n t. Per unit short-cirmif ,
^ * c e drop. U“ CUrrent 18 the rec*procal of the per unit
“"pe perunit short-circuit current as an auto-tr»n.fi.^ 1- r .
••• Per u n it s h o rt-c irc u it c u r re n t a s T ^ ^ T t ^ f b r ^ = ~
The value of k used in the above relations, Fig 1 36 (n\
• tr these relations, the value of k for step-down nr af S one In general, for
usingthe ® P down or step-up auto-transformers is = 1 v /h v
Uses Single-phase and three-phase auto-transformers are mainly employed " "
/ • \ r «for
(i) r i nin
t pterco
r c o n nn ne ectin
c t i n g gD p
o ow
w e rers vsy
s t ste
n mmo sk rh. av
.. : ____________________________________
in g voltage ratine Z a Z e mF P 7y ed> ;
• • • ui . g voltage ratios, not differing far from unitv and

^ variable ratio a u to -tra n sfo rm er (or v anac)


has a toroidal core a n d to ro id a l w in d in g . A tw h ld fo C° r#
sliding contact w ith th e w in d in g is m ad e by ^ ' 9 n Carbon
brush
carbon brush, see Fig. 1.39. T h e p osition of th e 1~~
sliding contact can be v a rie d by a h a n d w h eel /
and with this th e o u tp u t v o ltag e g ets changed. ,nput
A variac used for s ta r t i n g in d u c tio n m o to rs ^
and synchronous m o to rs a t red u ced voltages,
is commonly c a lle d a n a u t o - t r a n s f o r m e r
starter or a s ta rtin g co m p e n sa to r. .
f ig . 1.39 Single-phase variable ratio auto-transformer
Schematic d ia g ra m s for sin g le-p h a se an d
three-phase v ariab le ra tio a u to -tra n s fo rm e rs are illu stra te d in Fig. 1.40,

(a) (b)
Fig. 1.40. S ch em a tic d iagram for (a) single-phase variac and (b) three-phase variac.

When a u to -tra n sfo rm e r is u se d for su p p ly in g l.v. system , th e n its common point, such as C
*g. 1.36 in case o f sin g le -p h a se sy stem or th e n e u tra l in case of 3-phase system , m ust be
eaft ed otherw ise th e r e is a r is k of serio u s shock.
r Exam ple 1.38. A 2 0 kV A , 2 3 0 0 /2 3 0 V, tw o w in d in g transform er is to be used as an auto-
the rmer>w itfl c o n sta n t source voltage o f 2300 V. A t fu ll load o f u n ity pow er factor, calculate
Power output, p o w er tra n s fo rm e d a n d p o w er conducted. I f the efficiency o f the tw o-w inding
^form er a t 0.6 p.f. is 96%, fin d th e a u to -tra n sfo rm er efficiency a t the sam e power factor.
Solution. T he ra te d c u r re n ts o f h.v. a n d l.v. w indings are respectively

'» = H P = 8 .6 9 A and /t = ^ = 86,A .

b J ! ' t h ‘he P o la rities a s sh o w n in Fig. 1.41 (a), th e o u tp u t voltage is 2300 + 230 = 2530 V;
Wn- M th e tw » vv oo ltag
ltag es
es aa re
re in
in sa ee rie
rie ss aid in g . R ated c u rre n t of 86.9 A in th e l.v. w .nding re-
aiding.
C8- bV tra n sfo rm e r a ctio n , a r a te d c u r re n t of 8.69 A in th e 2300 V w inding acting as th e

canned by Cam Scanner


r ns~-

78 Electrical Machinery
[Art. |.l2
I ..8 6 9 A
\* 4
o
o
869A o
o
o 1
o
95 59A

(a) (b)
Fig. 1.41. Pertaining to Example 1.38.

primary . Kirchhoffs current law at point b gives the line current from th e supply line to be
equal to
(86.9+ 8.69) = 95.59 A.
/. Auto-transformer kVA rating is
2530 x 86.9
= 220 kVA
1000
2300 x 95.59
= 220 kVA.
1000
.-. The power output at full load of unity p.f. = 220 kW.
Here winding be acts as the prim ary and winding ab as the secondary.
kVA transformed 230 x 86.9 _ 2300 x 8.69
100
= 20 kVA
~ 1000
and power transformed = 20 kW.
kVA conducted = 220 - 20 = 200 kVA
and power conducted = 200 kW.
Note, how a 20 kVA two-winding transform er is capable of handling 220 kVA as an antn-
transformer. Out o f220 kVA, only 20 kVA is transform ed from prim ary ?o seco n d aA b T t r l
P former a c tm n T h e rem aining 2 00 kVA is transferred from source to Z i by d .r e c U o n d u c ta
F ie w i (LV P h 1™ “ nnf ctlon dlaeram with l.v. polarities reversed from those given in
*ig. 1.41 (a). Physically the two connection d iagram s look iden tical & it
polarities makes a marked difference in the auto-transform er o u tp u t ^
The secondary output voltage now is 2300 - 230 = 2070 V- since th e tum Q f

currents of 86^9 A a n d ^ ^ ^ r e s p ^ t h / e l y . ^ “ d ^ are ^ u al to t ” d

i M d . u r . M . f , . e 6 . 9 / caZ o t nr f . T r ^ . 1 ( l u L Cc u m n r 8 M f t f l h i al5° ^ reVeISd B

“ ,h -
Auto-transformer kVA rating
_ 2300 x 78.21 2070x86 9
1000 1000
‘ = 180 kVA
Power output a t full load = 180 kW.

Scanned by Cam Scanner


Transformers 79
___________________
XI re again w inding be acts as th e p rim ary and w inding ab as the secondary.
Transformed kVA = 230 x 86.9 = 2300 x 8.69 = 20 kVA
j nower tran sfo rm ed = 2 0 kW
andkp0” _
VA c o n d u c te d = 180 - 20 = 160 kVA
^ p ow w c o n d u c te d
= 160 kW.
O u tp u t
j^0w efficiency O u tp u t + Losses

Losses = - - 1 o u tp u t
or
Losses in 2-w inding tra n sfo rm e r
" 1 "l
= - 1 (20,000) (0.6) = 500 w atts.
< ' J . .
Since th e a u to -tra n sfo rm e r o p erates a t rated voltage and rated currents, th e losses rem ain
c o n[S
stanvaat 500
ta n t p w ------
a tts.
• Efficiency of a u to -tra n sfo rm e r for an o u tp u t of 220 kVA
L osses _ ----------- 500------------= Q 9 9 6 2 3 or 99.623%.
■= 1 “ In p u
in p u t “ 220,000 x^ 0.6 + 500
Auto-transform er efficiency, w hen o p erating for an output of 180 kVA, is
Losses _______ 500----------- = 0 .9 9 5 3 9 or 99.54%.
' “ 1_ In p u t “ 180,000
iOU,UUU xA 0.6 + UUU
V.U -r 500 I 'f

E xam ple 1.39. A 2 0 0 0 1 1 0 0 0 1 5 0 0 V


u anau todransform er
a , 35
«*pur D ra.
I
'amUabUdiagrcnn o f connections a n d fin d ,Ac e v e n t s in carious p a rts * . circait. Assume
111
the loads to have the sa m e p o w er factor. 300 a

Solution. The a rra n g e m e n t offfie three-


windings, to su it th e voltages of 3000 V, 3500
Vand 1000 V, is show n in Fig. 1.42.
The c u rre n t th ro u g h th e lo ad of 1050 •I i
kVAat 3500 V is
_ 1,050,000 = 3 0 0 A
“ 3500 3000 V
The c u rre n t th ro u g h th e lo ad o f 180 kVA
mnn \r is
at 1000 V
1 8 0 ^ 0 0 _ jg o a . 410A 480 A 300 A
1000 Fig. 1.42. Pertaining to Example 1.39.
The total kVA su p p lie d
= 1050 + 180 = 1230 kVA.

•• The total current taken from the supply main

= = 4 1 0 A< ^ ^ ap p ro p riate points gives th e distribution

F,g' 1 4 2

>canned by C am S canner
I

I A rt. 1.12
80 E lectrical M achinery-----------------___---------------------------------------------------
, * „ in e tra n sfo rm er is used as an
E x a m p le I AO. A lO kV A . 25001250 V, sinf f h % ^ ‘Z output voltage o f 2625 V. The l.v.
auto-transform er to raise the supply voltage ° f f 5° enual p a rts o f 125 V each. I f both parts
w in d in g o f the tw o-w inding transform er consists o f two equal p /
o f the low voltage w inding are used, determ ine
(a) auto-transform er kV A output and
(b) kV A transform ed a n d conducted. rnnnpcted in parallel and th en in series with
S o lu tio n . The two parts of the l.v. winding:are first ^ ^ schem e of connections, with
the h.v. winding, so th at the output voltage is 2500 + 125 - no* ■
proper polarity makings, is illustrated in Fig. 1.4 . 80A
The rate d cu rre n t of l.v. w inding is
10 ,000^1
40 A 250
T otal o u tp u t cu rre n t is 40 + 40 = 80 A.
/. A uto-transform er kVA ratin g
80 x 2625 = 210 kVA
1000
The rated cu rren t of h.v. w inding is 4 A. T h ere­
fore, th e cu rre n t draw n from th e supply is 84 A.
, , 80x125
(6 ) kVA transform ed = 1 0 0 0 = 10 kVA
and kVA conducted = 2 1 0 - 10 = 200 kVA.
Fig. 1.43. P ertain in g to E xam p le 1.40.
E x a m p le 1.41. A 3-phase star-connected auto­
transformer, supplies a balanced 3 -phase load o f 40 ,
k W a t 380 V a n d a t 0.8 p.f. lagging. I f the supply voltage is 440 V. d eterm in e the c u rre n ts m i k e
various branches o f the auto-transform er. E x citin g current a n d in te rn a l vo lta g e d ro p s are
neglected.
. . . 40,000 n A
S o lu tio n . The load c u rre n t / 2 = ~

F o r a balanced 3 -phase I,= 65-6 A


lo a d , th e p ro b le m c a n be I,=75-9 A
solved on p e r p h a se b asis.
Since ex citin g c u r re n t an d
Si
te r n a l v o lta g e d ro p s a re
leglected,
p e r p h ase kVA in p u t
P
= p er p h a se kVA o u tp u t ^ g V
440 . 380 .
or

or 11 -7 7^ x 75.9 = 65.6 A.
440
a The current flow ing 1
from neutral N to the tapping —
65-6 A
points a, b,c is
Fig. 1.44. P ertain in g to E xam ple 1.41.
(75.9 - 65.6) = 10.3 A.

The magnitude and direction of the currents in the various branches of th e 3 -phase auto­
transformer is shown in Fig. 1.44.

. i u i n io u L/j U 'c /rrfo ro n n u r


Transformers HI

' ’^ T Z ^ a U - p h a s e ideal transform er has 1000 turns a n d 400 tu ™ * J°r “ *


^ ^ t n p ,e j n r u w in d in g s respectively. The secondary w inding delivers a rests
V * Z d second? j L Z o l s A a n d C a n d a load o f 60 ♦ j 60 Q between term inals A a n d B
b e t w e e n term between B a n d C, there are 100 tu rn s so th a t between
f i n A * * a p rim a ry voltage o f 30 kV, fin d the p n m a iy

A°nd^
1 pn rd ipmrim
a ary
r y in H
p “l- 1, a i*
:re1lta * The a rra n g e m e n t of p rim a ry an
curren d secondary
and^ I f J)b >2 1 c
S°luti°m he loads ig as show n in Fig. 1.45. Voltage be-
^ tia ls A a n d C is
f
<A
£_ O S
^ o
t^een v = 3 0 ,0 0 0 x 4 0 0 = 12 0 0 0 V 3
30 kV Oo
1000 O
o o
6 0 0 ,0 0 0 -q .
^ load current 12,000
v ab = 3 0 X 3 0 0 = 9 0 0 0 V
Similarly,
, _ 9 0 0 0 _ = 150 , 45o
?lf
load current 2 ” 60 + j 60 V2 Fig. 1.45- Pertaining to Example 1.42.

IBA = l2 + 12
= 50 * j 0 + (cos 45” - J sin 45") = (125 - j 75) A

= ffl/t X 3 0 0 + Jcu x 10 0
Secondary m.m.f.
= ( 1 2 5 - j 75) x 300 + 50 x 100
= 42500 - j 22500
_ 4 2 5 0 0 - J 2 2 5 00 _ {A9 ^ _ y 2 2.5) A
Primary cu rren t 7i = 1000
= 4 8 .0 8 8 L r V f M l l .w
. . ‘“TT
= 3 0 x 4 8 .0 0 7 qq7° - 1274.991 k w
8 8 x cos 27.897
Primary in p u t o o a897°2 0.8838 lagging.
7 =

Power factor a t p rim a ry te rm m a ls = cos 27.W1 ^ ^ ^ n% positions


Example 1.43. A 1-phase secondary load resistors connected as shown
connectedto 400 V p rim a ry su p p ly f^f^Lctionsofthe winding, assum ing an ideal c .
iFig. 146 (o). F in d the currents m the three sections , [M .S , 19891
_ . . ea8jly by considering one load at a tim e and th en
Solution. This e x a m p le ca n be solved y . . . . tlli,
■perimposing th e tw o r e s u lts . , nn o is show n connected across term in a ls . n
In Fig. , 46 (6 , load ^ r e s i s t a n c e 200 0 ^ sh
?ure, secondary voltage, V2 - 0.75 x
3 0 0 = 1.5 A
Load current 200
In an auto-transform er, V \ 1 \ - ^ 2 ^ 2
_ 300_xJJ> _ 1 125 A
h ~ 400
W»th l h I 2 know n, KCL a t p o in t d gives

/" + /‘ = r2 , - , 5 - 1.125 = 0.575A


Idb = I 2 ~ l ' ~ '

beam
canned by C a m b ca nn er
lArt. i ,
82 E lectrical M achinery

Fig. 1.46. Pertaining to Example 1.43.

T hese cu rre n ts in th e two sections ab and db are shown in Fig. 1.46 (6 ).


In Fig. 1.46 (c), load of resistance 400 11 is shown connected across te rm in a ls ac. In this
figure, secondary voltage = 300 V and the secondary cu rre n t / 2' = 3 0 0 /4 0 0 = 0.75 A.
Now Vj I { = V2 1{
3 0 0 x 0 .7 5
or = 0.5625 A
400
KCL a t c gives I ca = 1{ - I f = 0.75 - 0.5625 = 0.1875 A
C u rre n ts in th e sections ac and cd are shown in Fig. 1.46 (c). S uperim posing th e results of
Figs. 1.46 (6 ) a n d (c), c u rre n ts in th e th re e sections of th e w indings are show n in Fig. 1.46 (d).
E x a m p le 1.44. A 100 kVA, 2 0 0 0 /2 0 0 V, 1-phase tw o-w inding tra n sfo rm er is to be used as
an auto-transform er for stepping up the voltage from 2000 V to 2200 V. A t rated load, the two-
w in d in g transform er has 2.5% loss, 3% voltage regulation a n d 4% im pedance. For the auto­
transform er, determ ine the fo llo w in g :
(a) voltage a n d current rating (6 ) kV A rating (c) efficiency (d ) percentage impedance (e)
regulation a n d (f) short-circuit current on each side.
S o lu tio n , (a) For au to -tran sfo rm er, ra te d voltage on l.v. side is Vj = 2000 V and on h.v. side
is V2 = 2000 + 200 = 2200 V.

R ated c u rre n t on h.v. side of au to -tran sfo rm er, / 2 = ra te d c u rre n t on 200-V side o f 2 -winding
tra n sfo rm e r = 100,0 0 0 /2 0 0 = 500 A.

Scanned by Cam Scanner


Transformers 83

* '^ T c o r r e n t on l.v. side of au to -tran sform er, I l = I 2 + rated cu rre n t on h.v. side of 2-wind-
P»«dC 1 0 0 ,0 0 0 ....
1= 5 0 0 + ~ m < r '

kVA rating as an au to -tran sfo rm er = — = 550 x 2 0 0 0 _


1100 kVA.
1000 1000
h v 2 500 x 2200
1000 1000
- = 1 1 0 0 kVA.

A ttem atively, ratin g can also be obtained as u nder :

o f transform ation of au to -tran sform er, k = =—


Ratl0u ...................................... 2200 - 11

kVA rating as an au to -tran sfo rm er = x kVA ratin g as 2-winding transform er.


1
x 100 = 1100 kVA
1 -™
11
(c) Percent full-load losses as an auto-transform er
= (1 - k) x percent full-load losses as a 2 -winding tra n s f
10 ^
x 2.5 = 0.227%

Efficiency as an auto -tran sfo rm er = 100 - 0.227 = 99.773%.


(d) Percentage im pedance as an auto-transform er
= (1 - k ) x percentage im pedance as a 2 -winding transf.
1- ^ x 4 = — = 0.364%
11
(e) Regulation as an au to -tran sfo rm er a t a certain pf
- (1 - k) x regulation as a 2 -winding tran s. a t th e sam e pf
' 1 0 ''
x 3 = 0.273%
11
(/) Short-circuit cu rre n t as an auto -transform er

1 -*
x short-circuit cu rren t as a 2 winding transf.

1 ‘x - 1
1 - 1® p.u. im pedance as a 2 -winding transf.
11

= H x — = 2 7 5 p .u .
g^ort • .
=275 x<KnlrC? ? ,CUrrent as an au to -tran sfo rm er on l.v. side w ith sh o rt circuit on e ith e r side
= 151.25 kA.
O n0^4 .

1275 * 5500
00=- °137
n 7 CUrrent
5 kA. as an a u t°-tran sfo rm er on h.v. side w ith short circuit on eith er side
^ m p le 1 A . .
a single-phase 2 5 0 /2 0 0 Vauto-transform er, w ith secondary short circuited,

m<Xry current o f 60 A a t 10 V a n d a pow er factor o f 0.24. It has ~ = k = 0.8. For an


°oltag N i1
) u Z y O f250V ’ fin d the load voltage for a load current o f 100 A a t p f o f (a) 0.8 lagging

Scanned by Cam Scanner


(Art 1.13

84 E le c tric a l M a c h in e ry

S o lu tio n . From Eqn. (1.65)


V2 . 1 -ft •*2
Zi +
Vl = T
= 60 z ei
10 = 0 + 60 'I + 1 6 22

z , = ^ = 0.1667 n
Ze\ e

r ,= z ,> * P f= b « ™ = a M a
= Vo3667!r^ a 0 4 r = 0 .1 6 1 8 £1
*,i
, v , ,_ 200_x_100 _ so A
As Vx l \ = V2 h>l \ - 250
W 250 _ Y l . + 80 ( 0 04 x 0 .8 + 0.161 x 0 .6 ]
" 0.8
V2 = 191-74 V
or

At unity pf, V , - y + / 1 ',1


y
250 = + 80 x 0.04 or V2 = 197.44 V
or
113. Parallel O peration o f Single-phase T ransform ers
W hen electric power is supplied to a locality, city or an are a , a sin g le tra n s fo rm e r, capab e
of handling the required power dem and, is installed. In som e cases, i t m a y be preferable to
install two or more transform ers in parallel, instead of one larg e u n it. T h o u g h tw o or moto
transform ers may be expensive th a n one large u n it, y et th is sch em e p o sse sse s certain ad
vantages described below.
(t) With two or more transform ers, the power system becom es m ore re lia b le . F or instance
if one transform er develops fault, it can be rem oved and th e o th e r tra n s f o rm e r s can maintain
the flow of power, though at a reduced level.
(ii) Transform ers can be switched off or on, depending u p o n th e p o w er d em and. In this
m anner, th e tran sfo rm er losses decrease and th e system becom es m ore econom ical and efficient
in operation.
(Hi) The cost of a standby (or spare) u n it is m uch less w h en tw o or m o re transform ers are
installed.
In any case, w ith th e passage of tim e, electnc pow er d e m a n d m a y becom e m ore than the
rated kVA capacity of th e alread y ex istin g tra n s fo rm e r or tr a n s f o r m e r s . U n d e r such cir­
cum stances, th e need for extra tran sfo rm er arises. Since th e su p p ly v o lta g e h a s to remain con
stan t, th e ex tra u n it m u st be connected in p arallel.
N ote th a t th e parallel operation of tra n sfo rm ers re q u ire s t h a t th e ir p rim a ry windings. 3'
well as th e ir secondary w indings, are connected in p a ra lle l. In th is a rtic le , only th e par*1" 1
operation of single-phase tran sfo rm ers is considered.
T he various conditions which m u st be fulfilled for th e s a tis fa c to ry p a r a lle l operation oft*1
or more single-phase tran sfo rm ers, are as fo llo w s: P v
(a) The tran sfo rm ers m u st have thp mu. . . , . . ■ „ „rtnnec^
to t t . s a m e voltage source, th e secondary voltages r f a U tra n s f o rm e r s s h o K c e q u a l in *

k
fv_.c_41 ii ij y Vvcii i rcrCttTTTTC.T
Transformers 85

le n t leakage im pedances in ohm s should be inversely proportional to th e ir


,i,)fhe G r a tin g s - In o th e r w ords, th e per u n it leakage im pedances of th e tran sfo rm ers
( >;ve kVA r _ tkVA\j a ra
r a tin e s m u st be
tin g De equal.
. o f e q u iv a le n t le a k a g e rea cta n ce to equivalent resistance, i.e . x t/ r t s h o u ld b e

,rif^ i r^ tra0Sf0rmerS'
tr a n sfo r m er sm u st be connected properly, so far as th e ir polarities are concerned.
the conditions listed above, condition (d) m u st be strictly fulfilled. If th e secondary
0ut e connected w ith w rong po larities, large circulating
^rmin8ls fJefloW an d th e tra n sfo rm e rs m ay g et dam aged. Condi-
T u ld be satisfied as accu rately as possible; since dif-
tion <o) °(jary voltages w ould give rise to u n d esired circulating a,, ,
^reflt Se°For conditions (6 ) an d (c), som e deviation is perm issible.
* * fulfilment of condition (d) is essen tial, w hereas th e ful-
*Thus
t ! o f other conditions is d esirable.
« 1 47 shows two sin g le-p h ase tra n sfo rm e rs in parallel,
ected to the sam e voltage source on th e p rim ary side. Ter-
conV with proper polarity m ark in g s h av e been connected both
mthe h.v. and l.v. sides. A fu rth e r check on th e polarities can be
appliedby connecting a volt-m eter V in series w ith th e two secon­ Fig. 1.47. Two single-phase
daries. Zero voltm eter re a d in g in d ic a te s p ro p e r p o larities. If transformers in parallel.
voltmeter reads th e su m o f th e tw o se c o n d a ry vo ltag es, th e
polarities are improper an d can be corrected by reversing th e secondary term in als of any one
transformer.
No-load o p e ra tio n . If th e no-load secondary voltages E a and E h for tran sfo rm ers A and B,
areequal in magnitude an d a re in tim e p h ase, th e n E a - E b = 0 and no cu rren t can circulate in
the transformer windings. I f E a an d E b a re u n eq u al or are out of phase, th e n th e re s u lta n t
voltage2a - Eb will circulate a c u rre n t I c given by th e expression,

J _ E g -E b ...( 1 .6 6 )
z ea + Zeb
hEq. (1.6 6 ), zta and z eb a re th e eq u iv alen t leakage im pedances in ohms, referred to th e
^odaries of transform ers A an d B respectively.
S in c e a n d z e6 are u su ally sm all, even a low value of (Ea - E b) m ay produce a r a th e r large
» the ■ Current- N °te t h a t d u e to tra n sfo rm e r action, th e circulating c u rre n t will flow both
iliia^mnar*f8 anc* 8ec° n d a r ie s - th is being indicated by dotted lines in Fig. 1.48 (a), w here
^hmi ) ed ** phase w ith a n d g re a te r th a n E b. T he circulating c u rre n t gives rise to addition-
Bect«d tr T ^ ^ran 8 ^o rm e rs an d , th erefo re, reduces th e efficiency of th e parallel-con-
fatedp,, 0 rmers. I t is u su a l in p ractice to keep th e circulating cu rren t less th a n 10% of th e
The t e ^ ’ C0n8equently th e tra n s fo rm e r tu rn s ratio s m u st be as nearly equal as is possible.
? n6W a R » T !l^ ltage V on th e seco n d ary side of both th e tran sfo rm ers, m u st be sam e. I t is
Vfcon* * 1 4 8 (a) th a t Jc a n d E a a re in th e sam e direction, w hereas 7Cand E b a re in opposite
herefore, if th e seco n d ary c irc u its of tra n sfo rm e rs A and B are tra v e rse d , th e n

= + 7c z eb = V ...(1.67)

Scanned by Cam Scanner


Frr
!!>
IArt. 1.13
86 E lectrica l M achin ery

—jlcxeb

• i ;-^-Icreb

jijS S ijii& A .
'C ir c u la t in g
TTW O^V /c u r r e n t s
r z ^ 'l Y >.Ea"Eh

(b)
(fl) • h i p l t h of circulating currents at no load
Kie. 1.48. g a t i n g the effect o f / , at no load.

e !?■.» i 4 ft (h) T he an g le & by w hich l c lags


E ,. (1.67) is depicted in ph aso r diagram of Pig. 1.48 <M. T he a g P y

{Ea - E b) is given by
_ l *ea *eb
P = ta n '
' ea ^cb . , ,i
Flg X48 (6 , illu stra te s th a t a t no load, th e effect of circulating c u rre n t / . is to boost the
lower voltage E„ to V an d to reduce th e h ig h er voltage E a to V.

O n -lo a d o p e r a t i o n tran sfo rm ers have equal voltage ratio s, th e m agnitudes


(a) E q u a l jvoltage are eq u al. F u rth e r if th e p rim ary leak ag e im p ed an ce drops

L “ S S T i S ? - r . ■r . - - — « ™ » « '• “ ■

A i s X m S S S it" no-load secondary voltage B . in series w ith its le a k a g e .m pedance v


T r a n s fo r m e rs is also rep resen ted in a sim ilar m an n er. T he com m on seco n d ary load ™ g
Va n d S c u rre n t I is sh ared as /„ and Ib by th e tra n sfo rm e rs a n d resp ectiv ely . The load
im pedance is Z ohm s.

Fig. 1.49. (a) and (b) Circuit m odelling o f tw o transform ers in parallel.

Scanned by Cam Scanner


Transformers 87
voltage eq u atio n for tra n s fo rm e r A is .........
E a - I a z ea = V ^ l Z

Since E a = Eb', E b - 7 a z ta = V = I Z
voltage eq u atio n for tra n s fo rm e r B is
E b - I b z eb = V = 1 2
. Eb ~ la z ea ~ E b —l b Ze[)

t , = 7‘ *"• ...(1.6 8 )

2> andt» equal. Hence th e circu it m o d e fo f 1.49 W c t T t e ^ a w ^ a s ' i l l X a ^


Pig. 1.49 (b) and th e ap p licatio n of K irch h o ffs c u rre n t law to th is circuit gives,
f T . f _ ^, . 0L z*-ea
I - I a + hb=~l1a+
. ~=---
z.eb
7 = 7* -— Zeb
*a
ZM +Z e6 ...(1.69 o)

Similarly, I h = l - Zea
z ea+ z eb ...(1.69 6 )
Multiplying both sid es of Eqs. (1.69 o) a n d (1.69 6 ) by th e term in al voltage V, we get

S„ = S = *•*
Zea + z ,b ..(1.70 a )

S„ = 5 = Z“

Here _ S„ = V / a, S t = V7t and 3 = V 7


Note that V J , S etc. a re p h a so r q u a n titie s.

t i e T s a v A S i ^ nsf 0 r T s op erated in parallel, th en voltage equation for these


ers, say A, B , C, D ..., can be w ritte n by referrin g to Fig. 1.49 (a) as u n d er :
E ° - Ia Zea = E b - I b Zeb= E c - I c Zec= E d - I d Zed = ... = V = IZ
second!^!0 3 n 8m!S ° f th 8 S 6 tra n sfo rm e rs are eclu a l> »■«■> E a = E h = E c = E d = ... = E 2 (no-load
“fy voltage), th e n from above,
The E 2 - V = I a z ea= Ib Z'b = Ic z ec = I d ztd = ... = v ...(1.71)
kVA ratW)Ve ec*u a ^ on sbow s t h a t a tra n sfo rm e r w ith lower value of full-load cu rre n t (or lower
°^er word ^ U8t ^ aV8 m ° re le a k a &e im pedance in ohm s so th a t Eqn. (1.71) is satisfied. In
ratiag8 t, ’ “ tra n sfo rm e rs in p a ra lle l are to s h a re th e total load in proportion to th e ir kVA
their kVA ^ e q u iv a le n t lea k ag e im pedances in ohm s m u st be inversely proportional to
former0f ^ atlngS‘ F ° r ex a m Ple » it* 1000 kVA tra n sfo rm e r h as z ea = 2 fi, th e n an o th er trans-
^bette ^ ^ ^ a r a ^ e ^ m u s t h av e z eb = 4 Q.
r ^ ysicai in sig h t in to th e p a ra llel operation of tran sfo rm ers can be obtained by the
c o n s id e ra tio n s :

but
r en reb
*tate
‘8 already m ade as (6) w here four conditions for the parallel operation of two or more 1-phase
er®are reported.

s d rn n e d by Cam Scanner
lA rt. l .i 3

88 E lec tric a l M a c h in e ry
lea k ag e im pedance an g le o f tra n sfo rm er A is not equal
T h e re la tio n ^ m e a n s t h a t <Ca, th e _ 1 Xeb
- r«, reb * - t a n '1 — and<>6 = tan- — .
to $6, the leakage impedance angle of transformer ^ ere . r„ ^ ^ ^ ^ ^ ^ ^

For equal voltage ratios under c o n s id e re h o m =^ ^ ^ ^^ u , The current


pedance drops are equal, re 4 z« ' ' Fjg l 5 o (o), where it is assume 6 „.
tags o by an angle ♦. and 4 ^ - hy angle ♦>. « * * 8 *

I
V r la z e a =
f IbZeb- ^ r^b* *t)
* - la * e o
^ b ^ y a / ------------►
rea la . 7, .
Ibre b ^ ^ \ /^b" 3

(a)

The secondary term inal q u a n titie s ^ a n d ^ p o w e r thcm> Fig 1.50 (M. The
order to determ ine /„ and 4 , draw „ - a q u a n tity -th erefo re, 4 and 4 can be computed
phasor s u m o f / 0 and 4 gives 1 5 0 (6) reveals t h a t transform er
o ^ ^ t e s 5^ ^ b ^ e ^ r p ^ a n ^ tr m s f m m e r B a t a ^ t o r p f as c o m p a r ^ h r th e load

and n ot th e arith m etic sum , of /. and 4- In other w ords, 4 < j an d 4 < 2 and

ou tp u t is less th a n th e sum of kVA ratin g s " h f n the sum of


reactance to resistan ce ™ «os ^ n o t e q u a “ ded t h a t a tra n s fo rm e r w ith greater
fe a T a re " n r^ ™ e; r e r e s a t a?poor p f a s com pared to th e o th e r w ith a lower leak,*

im pedance angle.
x eb
(ii) L et z ea > z eb, b u t ^ = r^
x ea _ m e a n s that
W hen Ea = Eb, 1, r „ = 4 h i a n d sin ce a „ > z,b, 4 < h- T h e r e la tio n ^ ^

= fe. T he p h aso r d iag ram of Fig. 1.50 ( 6 ) g ets m odified to t h a t shown 1 in F i g . ^


and l b have been d raw n proportional to th e reciprocal of z tn a n d z tb re sp

)y C am Scanner
Transformer* K9

operate a t th e sa m e pow er factor and


« sCerlv A is increased. Since / is th e arith m e tic
°utp,f for 4»a = <J>6, th e to tal o u tp u t kVA is equal to
S o f fkVA r a t in g s of th e in d iv id u al tra n sfo rm ers.
* < , A supplied by tra n sfo rm e rs A a n d are E 0 „
^ * 10^ r e S P e C tiV e ly H e r e > Z «*. A, < Ab<
lOJ an<J c / < Eb A,;it; m eans ^ a t sh a re d by tra n s-
than th a t sh ared by tra n sfo rm e r B.
^ erA ** t r a n s f o r m e r having g reater equivalent leakage
N°te l pre 2 ) shares less kVA (here Ea la x 10 3) and
-dance * ,
ippeo»» jower eq u iv alen t leak ag e im p ed an ce sh a re s
'^ ^ I v A In other words for parallel connected trans-
pf,t£r kVA shared by each tran sfo rm er is inversely
ai to its leakage impedance. Hence transform ers of Fig. 1.51 Parallel operation of
fV° nt kVA ratings can be o perated in p arallel provided
transformers with > z , b but — = —-
Ljr^uivalent leakage im pedances in ohm s are inversely r,a r,h
‘ to their respective kVA ratings. This statem ent
1
can b e expressed m athem atically as eq u iv alen t leakage im pedance
kV A r a t i n g

1 vt 1
z eu oc c a n d p
°ar * br
Zca _ S br
Zeb E ar
Here So, and S br are th e ra te d kVA ra tin g s of tran sfo rm ers A and B respectively.
/ . Zea ~ ^b z eb

: i l - Ear ...(1.721
h $br
The result of Eq. (1.72) can also be obtained analytically as explained below.
Prom above, /„ • * « ,= /* ■ * *

*ea pu S - i ,b ' z eb pu ' c


br
if
ea pu = zeb an d = 0 6f then
'or
...(1 72)
h ~ s
#Per*t^in>e C0nc^u<^e<i from th e foregoing an aly sis th a t tran sfo rm ers of different sizes can he
provided th e ir p er u n it im pedances are equal in m ag n itu d e and q u ality w e
(ii t v angle
^**haabee^ uo^ a£e ratios. U n eq u al secondary em fs give rise to circu latin g c u rre n t on no
N i n Pro n e * ^ a *ne(* before. W hen th e load is connected, th e tra n sfo rm e rs s h a re th e load
^ ^ resulta i0n ^ e *r ra tin g s which fu rth e r depends upon th e ir leakage im ped an-
and th Icu rren ^ *n any tra n sfo rm e r is equal to th e p h aso r sum of th e circu latin g
^°Nter °a(^ cu rre n t sh a re d by it. In Fig. 1.52, E a and E b are assu m ed in p h ase w ith
to be ha > ^ ° ac^ c u rre n ^ A logging th e term in al voltage V' by load p f an g le 0 ;< is
red equally by th e tra n sfo rm e rs. Since E a > E b, th e to tal c u rre n t sh a re d by

Scanned by Cam Scanner


i.e. L = 2+ and ^ 2 ^ 52 that the ef-
I t is s e e n fr o m t h e p h a s o r m g r * p o w e r fa c to r a n d in ­
f e c t o f c i r c u l a t i n g c u r r e n t . i s 1to r e d h a v i n g greater
T ea se the current shared by power factor of the
no-load induced e.m .f At t ^ e ^ ^ ^ ^ ern.f. is in-
second transformer decreased,
creased and the current shared y tg j and /fc can be
An analytical solution^ equations
o b t a in e d b y w r it in g d o w n t h e K ir c h n
for Fig. 1-49 (a)- _ v (1.73a)
...(1.736)
and ., to be obtained. As-
From these equations, la an“ b
s u m e Ea > Eb a s b e f o r e .
Ei
S u b t r a c t i o n o f Eq. (1.736) f r o m B q . (1.73a) g i v e usual
i£ a — E b = I q Zea ~~^b *'eb If
Fig. 1.52. On-load parallel operationof
7 L z.n-(Ea-E b) two transformers with E„ >Eb- (1.736
••• h zCt>
S u b s t i t u t i o n o f Ib i n E q . ( 1 .7 3a ) g i v e s
7a z t a Z ( Ea - E b) Z Ai
Ea - l a z m = IaZ + Zeb
z eb
Ea- E b ...(1.74a)
or /„ = B
_ Zz _ _ ^ e a Z eb

ztq+Z +^= z ea + zefc + ^


%eb
Eq ■Z'b +(Eq - Eb) ^ ...(1.74 6)
Also, zeo 2e6 + Z (zeo + zcfe) S:
Eb K -E j ...(1.74c)
S im ila r ly , Ir
_ Z 2- 1, — . — , Zea Zeb
2j(, + Z + — Zea + zefc + y tr a n s
Z ea __

E i z ea- ( E a - E b) Z forme
...(1.74 d)
Also, 0 (1.791
z ea ' z e b ^ (z e a z eb)
impe <
T h e lo a d c u r r e n t I i s g iv e n b y
/ = / „ + /( , = E q n . ( 1 .7 4 b) + E q n . ( 1 .7 4 d) E
e9 uiu
Eq • z eb + E b • z m ...(1.75' Find
1=
zea " zeb "t Z (z ea 4" z eb)
S
T e r m in a l v o lt a g e , V=IZ
(fans
_ Eq ■Zfb +Eb ■Zm ...(1.76'
Zta z tb .
2 (Zca "t z eb)

Scanned
I runs form ers 91

e x p r e s s io n s for I a a n d I h, th e sec o n d te r m r e p r e s e n ts th e c irc u la ti


ng c u rre n t u n d e r
! £ £ . in ca se and a r e n o t e q u a l T h is ' ire “ '» « n g ™ Z Z Z E q^a Z Z
|0»dCL
Ea ~ E b
/ =
'ea ...(1.77)
z ea + z eb +

nal voltage V c a n a lso b e o b ta in e d a s follow s :


Tcrnu

V = 7 2 = {Ia + I b)Z = (E ° - V \ E b_ - V '


**ea
•" Z eb
simplification th is g iv e s,
On

v' ± +± , i ) ^ +h
Z ca Z<b Z Z ea z eb
For more th a n tw o tr a n s f o r m e r s in p a ra lle l,

1 1 1 1
— + — + — + ...+ =
''ea Z eb z e 'ea z eb Z ec ...(1.78)
Eqn. (1.78) is a p p lic a b le to a n y n u m b e r o f tr a n s f o rm e r s (or g e n e ra to rs ) in n a ra llo i ft •
usually called p a ra llel-g en era to r th eo rem . p a ra lle l a n d is

If it is required to e x p re s s I a a n d I b in te r m s o f th e load c u r r e n t / th e n E o s ( 1 7 * , ) anH


(1.736)can be h a n d le d a s follow s : ’ q ’ (L 7 3 o ) a n d

E a ~ 7a z ea ~ E b — I f ^ eb

4 2ea = E a —E b + If,Zeb
Addition o f /a zeb to b o th sid e s, g iv es

4 (4 a + z eb) = ( 4 + l b) Zeb + E a ~ E h
But
4 + 4 =/
En - E *
lJa —
—7i —
— ^ eb= — + —
Z ea + z eb Z ea + z eb ...(1.79 a)
Similarly, Ea ~ Eb
...(1.79 b)
In
'^ f o r J e r s A a^ft re d u c e s to E * (L 6 9 ) from w h ich lo ad c u r r e n t I a s s h a re d by
liters A and R • ^ com P u t ed . W h e n E a > E b, th e n th e c u r re n ts s h a re d by tr a n s -
ll?9) circu lar E ? ' ( 1 ' 7 9 )' S in ° e E ° > E(” /fl ‘S m ° re th a n Ib a s is ev id en t from Eq.
^Pedance. m g c u r r e n t 1S e q u a l to th e d iffe re n c e of e.m .fs. d iv id ed by th e local c irc u it

i 46 t i
1 y Unt leakaao L tr a n s f o r m e r s h a v e e q u a l vo lta g e ra tio s. T h e m a g n itu d e o f th e ir
^ W i o o f f u / r / P T aLnCeS a re e q u a l b u t th e ir reactance t0 resista n ce ra tio s are 9 a n d 3.
> S°lution A d e liv e r e d <to tfie s u m o f th e ir in d iv id u a l k V A ra tin g s.
aflSf° ^ e r nequ-vr^ hia f x a m p le , r e f e r to F ig. 1.50. T h e d iffe re n c e b e tw e e n th e a n g le s o f th e
^ v a i e n t le a k a g e im p e d a n c e is

*b ~ 0a = ta n " 1 — - tan" 1 — = tan" 1 9 - tan" 1 3 = 12.14°.


, reb r,a

Scanned by Cam Scanner


(Art. 1.13
92 Electrical Machinery

H ere z e a ~ Zeb. b
T he load c u rre n t /, from Fig. 1.50, is given by
1 =2 Ia c o s ^ =2 h (6 0 7 ) = 2 /„ (0 994)

N u m erical sum , 2 Ia _ 1 _ 1 = 1.007.


P h aso r sum , I cos 6.07 ° 2 / 0r 2 /„ is n e a rly eq u al to their
It is th u s seen th a t th e n u m en cal sum ^ r e q u ire m e n t of keeping
p haso r sum /. T his shows th a t for p arallel c o n n e c te d tra n sfo rm e rs, in q p g
reactance to resistan ce ratio s equal is not of m uch im portance.
Full-load kVA d elivered _ V I * 10 3—
Sum of th e ir kVA ra tin g s (V7n + V Ib) 1 0' 3
_ 2 / 0 cos (6.07°) _ cQs 6 0?o _ Q 9 9 4
' ^ . .
F x a m D le 1 47. A n um ber o f transform ers w ith equal voltage ratios are o p era tin g in paral­
lel The m a g n itu d e o f their leakage im pedance voltage drops at rated cu rre" ts are not eci ual
m ltS T Z to tZ kV A output o f these transform ers, a n y one o f th e m ov
loaded ?
S o lu tio n . L et (/*,)„„ (& .)„„ (/* ,)« ■• be th e m ag n itu d e of <=9u i' ' a le n t ™ P e d a " “ ™ ltae '
drops a t ra te d c u rre n ts for tra n sfo rm ers 1, 2, 3 F u r th e r le t th e kV A r a tin g s o to * * ™ *
form ers 1, 2, 3 be (kVA)„ (kVA)2> <kVA)3... respectively. A ssu m e t h a t th e tra n sfo rm e r 1
h a s th e low est m ag n itu d e of th e eq u iv alen t im pedance voltage drop.
Since th e tra n sfo rm e rs hav e equal voltage ratio s, th e ir e q u iv a le n t im p e d a n c e s are con­
nected in p a ra lle l as in Fig. 1.43 (6 ) and th e voltage drop across z el> ze2. z ,3...m u s t be th e same.
As th e load on th e parallel-connected tra n sfo rm e rs is in creased , th e c u r r e n t s h a re d by each
tra n sfo rm e r also in creases. W hen th e voltage drop across e q u iv a le n t le a k a g e im p ed an ces be­
comes equal to (Ize)n i, th e n tra n sfo rm e r 1 is fully loaded and is c a rry in g th e r a te d c u rre n t. But
th e o th e r tra n sfo rm e rs a re n ot loaded to th e ir full-load cap acity , b ecau se (Ize)fj\ is of th e lowest
m ag n itu d e.
W hen tra n sfo rm e r 1 is fully loaded, th e voltage drop acro ss o th e r le a k a g e im pedances is
also (Ize)fi\- F or tra n sfo rm e r 2, full load voltage drop (Iz e)fl2 w ould c a u se i t to d e liv e r its rated
(kVA )2 F o r voltage d rop ( Iz jp i across tra n s fo rm e r 2 , th e load s h a re d by it is g iv en by

S s " 1™ -
(Iz )
Similarly load shared by transformer 3 = -= \ x (kVA)3

The total kVA output from the parallel-connected transform ers is alm ost equal to the
numerical sum of their individual outputs, as is shown in Ex. 1.39.

(kVA)r = (kVA), + (kVA)2 + r r - r — (kVA)3 •■d-80'


(I z e)fl3
In Eq. (1.80), (kVA)r is the total kVA output that can be delivered to the load withou'
overloading transformer 1, which has been assum ed to have m inim um full-load im p e d e
voltage drop.
The leakage impedance voltage drops may be expressed in volts, in per cent or in per

Scanned by Cam Scanner


^ Transformers 93

E%awple 1*48. Three 400 kV A, 1 1,000/6600 V, 50 H z single-phase transform ers gave the
M V g data, when operated a t rated current w ith their l.v. w indings short circuited.
' Transformer 1 : 360 V, 3025 w atts
Transformer 2 : 400 V, 3200 watts
Transformer 3 : 480 V, 3250 watts.
These three transform ers are operating in parallel w ith their p rim a ry voltage held constant
atn k v .
(a) W hat is th e g r e a t e s t lo a d a t u n ity p . f th a t ca n be p u t on th ese th ree tr a n s fo r m e r s in
parallel w ithout o v e r lo a d in g a n y o f th em ?

(b) What is th e s e c o n d a r y ter m in a l v o lta g e u n d e r th e co n d itio n o f p a r t (a).


Solution, (a) H ere (Ize)fll = 360 V, (Ize)fl2 = 400 V
and (/*e)/!3 = 480 V.
Transformer 1 is loaded first to its rated capacity, because (Ize)fll h as low est m agnitude.
Thus the greatest load th a t can be p u t on these transform ers w ithout overloading any one of
them is,

(kVA)r = (kVA), + (kVA), + (kVA), + ...


(/ 2 *)/72 ( / z e)/73 3

= 4 0 0 + 400 X 4 0 0 + 480 X 4 0 0 = 1 0 6 0 kVA


The total load operates a t u n ity p.f. and it is nearly tru e to say th a t tra n sfo rm er 1 is also
operating at unity p.f.

(b) Secondary ra te d c u rre n t = ~ ! r = 60.6 Amp


6 .6
Since transform er 1 is fully loaded, its secondary carries the rated cu rre n t of 60.6 A.

For transform er 1 , r , 2 = - - 025. = 0.825 fl


(60.6)
Full-load voltage drop for transformer 1,
E i - V 2 = I 2re2 cos 02 + I<pc.e2 sin 02
= (60.6) (0.825) (1) + 0 = 50 V
Secondary terminal voltage V 2 = 6600 - 50 = 6550 V.

■nauTTvA*! L 4 9 ‘ (<l) Transf ° rm ers w ith eQual voltage ratios are operating in parallel. Show
ne hvA load shared by a n y one transform er is given by
n n

X Z sh
S l = ----—------= --------l------------
1
Z tk X 77 z ekpu Z
, *ek ' i Zekpu
Thp
(b) used have their u su a l m eanings.
ree> tw ° w in d in g transform ers have the follow ing leakage im pedances :
Transformer 1 : W 0 k VA, 0.02 p. u.
transform er 2 : 75 k V A , 0.03 p.u.
d) H r<XnsforTner 3 •' 50 kV A, 0.025 p. u.
°W wiU they share a load o f 225 kV A ?

Scanned by Cam Scanner


IArt.
1-1J

94 Eledrical Machinery------------------ --------7 ~ T l ^ t h e g r e o t e s t kV A load th a t can be suppli<i


(HI I f no transform er ic to be overloaded, ca cu
by three transform ers in parallel. ^ s h o u td be th e o f their
(Hi) For n o rm a l loading o f each im pedance o f transform er 1 as faced,
equivalent leakage impedances? A ssum e the leag ^ ^ eqm va]ent ^

' S olu tion . <o) Since th e


im pedance voltage drops must be eq _ - I nz m = v.

I, = h=7 = ' , t is eq u al to v, i» inverse


I f . . , is sm all, 7 , m ust be large s o ^ t h a t * £ P fae expresscd m ath em atically as,
tio n al to a,i- Sam e is tru e for Is. h e tc - 1 ^ ^ v -S .

I, :— :7* : ••' I n ~z7r'*a — — *” . . c t k r


„ „ „ al cav e a u a l to E. Therefore,

T • Tn - h : : 1 ,,
1 _JL.JL.X-- .
= 2 e l ‘ Ze2 ' 2 e3 ” 2** ^
r . r . -7 t = S i : S 2 : S 3 : ... 4.
or * ........... x
. jl . ; J L ; ...: i
” 2,1 ' 2,2 ' 2 ,, — Z™
T he above relation can also be expressed a s o rS l, . , o rS „
7, oor ri h, oor ri 3...
j ^ o^r ^ ^ ouri fx„. = _ p p - -------- fp T
T
S i + S 2 + S 3 + ... + S* +
1 * .. ii V
JL or JL or — ... or — ••• or —
ze* en
2,1 z «?2

» + JL + -*. + ... + i + ... + -S


%ek en
z«l z <2 Ze3
S* t>/zek
or ~~ n
i 7* i * - i i
Y j i « i e

Since z,* V — is the ratio of impedances, the units in which they are expressed are »»»'
z ek

P° rtant' 7, S„ 1 1
71 ^ 1 V1 1
Y /* X 2e*X J ” Z ehp u. 2 - 2
Y i i eJt i ekP u-

(b) (i) Total kVA load to be shared = = 225 kVA. ^


If percentage or p.u. values are used for impedances, these m ust be referred t0
kVA base. Here, if base kVA is chosen as 100 arbitrarily, then the transformer 1 P
zel = 0.02 p.u. or 2 p.c.

ze2 = 0.03 x ^ = 0.04 p.u. or 4 p.c.

z?3 = 0.025 x ^ = 0.05 p.u. or 5 p.c.


, r t 1131
j ransformers 95
For convenience, th e p ercen tag e leakage im pedances arc used.

S ••• £ ^*ekpc% ^h x + tZ e2pc


b + 7Z e3pcr - 5M +i . »
20
I S.
from Eq- (1-81), •S'i = __ 225
1, 1 ^ - oz 'l 9 '
ekpc
20
225 V y
S o —

4 (19/20) ~ 5 9 -2 kVA
S o = —
225
«m- ’3 5 ( 1 9 7 2 0 ) = 4 7 -4 kVA.
It is, therefore, seen t h a t tra n sfo rm er 1 is o v e r l o a d u
underloaded- This is due to th e fact th a t th e ir p u 1eak** r(\as tra n sformers 2 and 3 are
Sude. P l6akage lmPedances are not equal in mag-
(ii)Since tran sfo rm er 1 h a s th e lowest l p a k ^ j .
toits rated capacity, i.e. S j = 100 kVA. impedance, it will be the first to be loaded

S j = 100 = I s*
"el pc '1 9 '
“ekpc (2 )
20
v j
or X s k = 190 kVA.
e 190
Oo —' = 50 kVA
4(19/20)
and
S;} “ 5(19/20) = 4 0 kVA>
Transformers 2 and 3 are seen to be underloaded by a considerable margin.
Alternatively, from Eq. (1.80),

(kVA)r = £ S* = 100 + f x 100 + 1 x 100


4 5

= 100 + | x 75 + ~ x 50 = 190 kVA.


o Z.5
(iu) For successful p arallel operation of transform ers, their pu leakage impedances based
100kVA°Wn k^ r a t*n &s should be equal in m agnitude. It is given th a t leakage impedance of
transform er is fixed a t 2%, therefore, 75 kVA and 50 kVA transform ers should have
4=^3 = 2%.

Check. For a common base 100 kVA,

= 2%, z, 2 = 2 x ^ =1 %
and O 100 Jm
Z(3= ~50 =
1 _ 1 3 1 9
£ J _ “ 2 +8 +4 8
* ekpc
.yr • S x= 2 2 5 - = 1 0 0 kVA, S 2 = 75 kVA and S 3 = 50kVA.
2 (9 /8 )

4 #
icanHecnDy C am S canner
96 E le c tric al M a c h i n e ^ ------------------------------ '^ T Z Z ^ s fb r m e r s g a ve the f o l l o w .

5 0 0 feVA : 4% rated volta ge ; rated cu rren a ^ dQ th e y s h a r e a lo a d o f 560 kW Qt


(a) These two transform ers are connecte i P
OS P .f l a g t . 000 v> ^ u l a t e th e s e c o n d a r y te r m tn a l voltage at *
(6) I f th eir voltage ra tin gs a re 11,
load g iv en in p a rt (a). ^
Solution. I rated ~ base ‘
V r a te d ~~ ^ b a s e = 1 *

U n d e r sh o r t circu it, Isczei ^ = Q Q3^ ^


Since / .“W 1 *1 v •
z . = 0.03
or - _n« • sin 6 « = 0.968.
Short-circuit p.f., cos 0* - . , •^ q^ = (Q Q075 + j 0 .029) p.u.
In com p lex n o ta tio n , 2,1 - • ■ q ^ = 0 0 1 2 + j 0 .0 3 8 1 . p .u .
S im ila rly z e2 • . , , , , be r e fe r r e d to t h e com m on base
(a) W hen u sin g p.u. s y ste m , th e v a lu e s o * ,i an z e2 r g u ita b le b a se kVA
kVA H ere th e common b a se kVA m a y be 2 0 0 kVA, 5 0 0 k V A y
C hoosing 500 kVA b a se arb itrarily, w e g e t
- = 500 0 0075 + j 0 .0 2 9 ) = 0 .0 1 8 7 5 + j 0 .0 7 2 5 = 0 .0 7 5 Z75.52’)
«i onn v '
200
500 (0.012 + j 0.0381) = 0.04 Z72.54"
2e2 500
2,1 + ze2 = 0-03 + j 0.11 = 0.114 Z74.74°.

S =^ = 700kVA
Total kVA,
S = 700Z - cos” 1 0.8 = 700 Z - 36.9°
— — z e2
From Eq. (1.70), s, =s - r =~
1 z el + z e2
- (700 Z - 36 9) 0 0 4 Z 72.54^_ = 2 4 5 0 z _ 3 9 v kVA
- (700 Z d b . 9 ) o n 4 Z 7 4 740

A S j = (245) (cos 39.1°) a t p.f. of cos 39.1° lag


= 190 kW a t 0.776 p.f. lag
S 2 - (700Z - 36.90, - 460 Z - 36.1“ kVA

S 2 = (460) (cos 36.1°) a t p.f. of cos 36.1° lag


= 372 kW a t p.f. of 0.808 lag.
(Check. Total power = 190 + 372 = 562 kW, alm ost equal to 560 kW]
245
(6 ) C urrent shared by transform er 1 = r — = 1.225 p.u.
^UU
Transform er 1 is, therefore, overloaded by 22.5%, i.e. 45 kVA
a an
C urrent shared by transform er 2 = = 0.92 p.u.
p,

Transform er 2 is, therefore, u n d er loaded flW,

Scanned by Cam Scanner


Transformers 97
e regulation, from Eq. (1.40), is given by t, cos e 2 + c, sin e,.
V^ l 8foriner1, th e voltage reg u latio n a t 1.225 p.u. current is
fortra = 1.225 (e,. cos e 2 + Er sin 0 2)
= 1.225 (0.0075 x 0.776 + 0.0290 x 0 631)
= 1.225 (0.024119) = 0.029546
= 0.029546
E2
V2 = (0.970454) (400) = 388.182 volts.

' Fwm p a r t >0f v rk v tr W 0bSerVed' F irstl>’' the su "> of kVA shared


. a * transformer I2 4 5 + 4 6 0 - 7 0 5 kVAi s slightly g reater th an th e load kVA ( . 700 kVA)
S » . * e two transform ers o p erate a t different power factors ; one a t a p f. greater and the
a P f 'ower th an th e load p.f. T h ese tw o points have already been d is c u .L d 7 n F .g i 50
Fvample 1.51. Two single-phase transform ers rated 1000 kVA and W ) hVA
Itakag* impedance o f (0.02 +j 0.06) a n d (0.025 + j 0.08) respectively. W hat is the largest kVA
,„d that can be delivered by the p a rallel com bination o f these two transformers w ithout over-
luadingany one ‘
Solution. For transform er 1 of 1000 kVA,
zei = (0 .0 2 + j 0.06) pu
For transformer 2 of 500 kVA,
zc2 = (0.025 0.08) pu -­
Choose a base of 1000 kVA.
z ei = 0.02 + j 0.06 = 0.06325 Z71.5650

Ze2 = "500" (° ° 2 5 + -/ 0 08) = 0 1 6 7 6 3 ^72.646°


2*1 + 2*2 = 0.07 + j 0.22 = 0.2309 Z72.350

Now Si _ _ .. s
2 el + Ze2

■■■Total kVA load, S = 1000 x = 1377.44 kVA

ta W ly , 8 . 500 1825.3 kVA

1 3818’ 2el/ = 0-06325) b ased on its own kVA mating is less th a n ze2 ( = 0.0838)
‘^sform erT 1 A r a tin S- T herefore, as th e load on th e parallel com bination is increased,
13?7.44_ 1000 —^377 f ir s t. So tra n s fo rm e r 2 s h a re s a load of
•t »44 kVA.
^ 8 exa^ P?SS’^ e ^oad on tw o tra n sfo rm e rs is 1377.44 kVA.
V d to 6 °an a^S° so^ved hy u sin g Eq. (1.80). For this, z el and zc2 should not be
^ rgest ,C°mmon base kVA. As z el < ze2, tra n sfo rm er 1 reaches its rated kVA first.
8 A load on both th e tra n sfo rm e rs, from Eq. (1.80), is

(kVA)r = 1000 + x 5 0 0 = 1377.34 kVA.


a®»ple 1.50 T 0 0838
^ ■'100 h v W° P ^ ase tra n sfo rm ers have the following data :
B : 200kVA 6 6 0 0 /2 3 0 V, z m - 1 .5 + j4 %
WA> 6 6 0 0 /2 2 0 V, z eb = 1 + j 5 %

Scanned by Cam Scanner


98 Electrical Machinery-------------- 1

These two transform ers are connected in p a ra llel a n d 6600 V is a p p lied to 6„5 ^ :
Calculate , . ,
(а) no-load circulating current a n d the ohm ic loss caused y i an
(б ) no-load term inal voltage.
S o lu tio n , (a) L eakage im pedance should be expressed in ohm s on th e secondary side.
„ . 1 0 0 ,0 0 0 .
Full-load secondary c u rre n t of tra n sfo rm er A - ggo

in ohm s = (0.015 +J 0.04) = 0.00794 +j 0.0212 £2


. „ 2 0 0 ,0 0 0 A
Full-load secondary c u rre n t of tra n sfo rm er B - 220

z eb in ohm s = (0.01 + j 0.05) 2jfo qoo = 0 00242 +J 0.0121 ft


Zea + Zeb = (0.00794 + j 0.0212) + (0.00242 + j 0.0121)
= 0.01036 + j 0.0333 = 0.03487 Z 72.720 ft
From Eqn. (1.66), circulating c u rre n t a t no-load is given by

*“t £ ' -o.os^VJi“i f ■286-78z- 72-72”A


C irculating cu rren t, Ia = 286.78 A and I b = - 286.78 A
Ohmic loss due to circu latin g c u rre n t = 7t2 (rtu + rch)
= (286.78)2 x 0.01036 = 852.04 W
No-load secondary term in al voltage, V = E a - I a zea
V = 230 - (286.78 Z - 72.72°) (0.00794 + j 0.0212)
= 230 - (286.78 Z - 72.72°) (0.02264 Z 69.450)
= 230 - 6.482 + j 0.370 = 230 - 6.493 = 223.507 V.
E xam ple 1.53. A 500 kVA, 1-phase transform er A h a vin g 0.015 p u resistance, 0.05 pu
leakage reactance is to share a load o f 750 kV A a t 400 V a n d a t 0.8 p f la g g in g w ith another 250
kVA, 1-phase transform er h a vin g 0.01 p u resistance a n d 0.05 p u leakage reactance. Their secon­
dary no-load e .m .f s are 405 V a n d 415 V respectively. F in d
(a) circulating current a t no load
(.b) current su p p lied by each tra n sform er a n d
(c) k VA, k W a n d p f o f each transform er.
S o lu tio n . F o r such n u m erical problem s, it is m ore co n v en ien t to w o rk in ohm ic values.

zc(J in ohm s = (0.015 + j 0.05) x 5Q4 = 0.0048 + j 0.016 f t = 0.167 Z 73.3 0 ft

zeb in ohm s = (0.01 + j 0.05) x ^ 0 j j 0 0 = 0.0064 + j 0.032 f t = 0.0326 Z 7 8 .6 9 0 ft

Load c u rre n t, I = - 5 0 '000 = 1375 a


400

Load impedance, Z = y = ^ = 0.2133 £1 at p f 0.8 lag

z = 0.2133 Z 3 6 .8 7 0 f t
zea + z eb ~ 0-0112 + j 0.048 = 0.0493 Z 7 6 .8 7 0 ft.

Scanned by Cam Scanner


Transformers 99

'■’’^ c u r r e n t a t n o -b ad
tg la « n 8 E _ E"
4 1 5 -4 0 5
= 202.84 Z - 76.87° A
0.0493 Z76.870
urren t a t no load is 2 0 2 .84 A a t a p f o f cos 7 6 .8 7 ° = 0 .2 2 7 lag.
. circulating^ ^ = Q q 1 6 7 ^ 73 3 o x o 0 3 6 z7 8 i6 9 o

(M ^ ' = 0 .0 0 0 5 4 4 Z 1 5 1 .9 9 0 = - 0 .0 0 0 4 8 + j 0 .0 0 0 2 5 5 5
+ z ) = (0 .2 1 3 3 Z 3 6 .8 7 0) (0 .0 4 9 3 Z 7 6 .8 7 °)
Z(Zta ‘ b = 0 .0 1 0 5 2 Z 1 1 3 .7 4 0 = - 0 .0 0 4 2 3 5 + j 0 .0 0 9 6 3 Q
E • Ztb = 4 0 5 * 0 0 3 2 6 z 7 8 6 9 ° = 13 2 0 3 ^ 7 8 .6 9 ° = 2 .5 8 9 + j 12.947
v I = 4 1 5 x 0 .0 1 6 7 Z 7 3 .3 ° = 6 .9 3 Z 7 3 .3 0 = 1.99 + / 6 .6 3 7
Ejb ' *ca
hared by tr a n sfo r m e r A , from E qn. ( 1 .7 4 (6 ) ), is
^ _ Ea z eh + (Ea - E h) Z
° ' z cb E (Zca + Zeb)
2 .5 8 9 + / 1 2 .9 4 7 + (4 0 5 - 4 1 5 ) (0 .2 1 3 3 Z 3 6 .8 7 0)
= - 0 .0 0 0 4 8 + j 0 .0 0 0 2 5 5 5 - 0 .0 0 4 2 3 5 + j 0 .0 0 9 6 3
0 .8 8 2 6 + j 1 1 .6 6 7 _ 11.7 Z 8 5 .6 7 °
" - 0 .0 0 4 7 1 5 + j 0 .0 0 9 8 6 6 " 0 .0 1 0 9 5 3 Z 1 1 5 .5 °
= 1 0 6 8 .2 Z - 2 9 .8 3 ° A
1 .9 9 + 6 .6 3 7 1 .7 0 6 4 + j 1.28 3 .6 9 6 4 + j 7 .9 17
From Eqn. (1.74 (d )), I b - 0 .0 1 0 9 5 3 Z 1 1 5 .5 0 " 0 .0 1 0 9 5 3 Z 1 1 5 .5 °
8 .7 3 7 4 Z 6 4 .9 7 0
= 797.72 Z - 50.53°
0.010953 Z115.5°
;. Currents supplied by tra n sfo rm er A and B are respectively 1068.2 A and 797.72 A.

(c)kVAshared by tran sfo rm er A = * ° 6 6 2q* = 427.28 kVA

kVAshared by transform er B = '^ 2^ 4° ° =319.1 kVA

kWshared by transform er A = 427.28 x cos 29.83°


= 370.67 kW a t p f = 0.8675 lagging
kWshared by transform er B = 319.1 x cos 50.53
= 202.85 kW a t p f - 0.6357 lagging
u4 uSllMtdifference in the n o ,o a d e.m.f.s and per u n it leakage im pedances have resulted in
,5°0~4?7 o s h a rin g a n d d iff e r e n t o p e ra tin g pfs. T ra n s fo rm e r A is u n d e rlo a d e d by
^ t h e ir^ = 7 2,72 kVA w h ereas tra n sfo rm er B is overloaded by (319.1 - 250) = 69.1 kVA
operating pfs are 0.8675 lagging and 0.6357 lagging respectively.
j Pk 1.54. Two 1-phase transform ers A a n d B have the following data :
f wer A : leakage im pedance referred to sec, z^, = 0.4 + j 2.2 fl, no-load secondary
>'510 V
Tr °
5oo ' ^ea^ aS e im pedance referred to sec, zetl = 0 .6 + j 1.7 fi, no-load secondary

c ai*r? nsf ° rm ers, o p era ted in p a ra llel, are connected to a load o f im pedance
''•harlri the reactarice to be connected in series w ith transform er B so that load
d u a lly between the two.

Scanned
nnecfbv
by Cam
Cc Scanner
100 Electrical M achinery

S o lu tio n . H ere z m = 0 .4 + j 2 .2 ft
L et th e to tal reactan ce of tran sfo rm er B be x ft. This gives
zeb = 0 6 + jx ft
From Eqn. (1.74 (b)) an d ( 1 .7 4 (d)), we get
h Ea ■Zeb + (£', ~ E b) Z
h, E b ■2CO- (Ec, - E b) Z
_ 5 1 0 (0.6 + y x) + (510 - 500) (5 + 7 3) ,
500 (0.4 + j 2.2) - (510 - 500) (5 + j 3) I
D iv id in g th e n u m era to r and d en o m in a to r by 500, w e g et I
. _ 1.02 (0.6 + j x ) + 0.02 (5 + y'3) 1

" (0.4 + j 2.2) - 0.02 (5 + j 3) 1

or 1 _ 0.712 + 7 (1.02 x + 0.06) i


0.3 + j 2.14
or 0 .7 1 2 + j (1 .0 2 x + 0.06) = 0.3 + j 2.14
0 .7 1 2 2 + (1.02 x + 0 .0 6 )2 = 0 .3 2 + (2 .1 4 )2
1.02 x + 0.06 = V4.163 = 2.0403
or x = 1.9415 ft
.•. E xtern a l reactance to be connected in serie s w ith th e tran sform er
B = x - 1.7 = 1.9415 - 1.7 = 0.2 4 1 5 ft.
1.14. T a p -C h a n g e rs o n T r a n s f o r m e r s
T he m odern eq u ip m en ts, u tilisin g electrical energy, are d esig n ed to op erate satisfactorily
at one voltage level. It is, therefore, o f p aram ou n t im portance to k eep th e c o n su m e rs’ terminal
voltage w ith in th e prescribed lim its. T he transform er ou tp ut v o lta g e and h en ce th e consumers'
term inal volta g e can be controlled by providing tap s eith er on th e prim ary or on th e secondary.
T he p rinciple o f reg u la tin g th e secondary ou tp ut voltage is based on ch a n g in g th e number
of tu rn s in th e prim ary or secondary. Let V h N, and V2, N 2 be th e p rim ary and secondary
. f v \ ! “
q u a n tities. I f TV, is d ecreased , e m f per turn on prim ary =~ in crea ses, th erefore, secondary °]

ou tp ut voltage (V,/TV,) N 2 in crea ses. On the oth er h a n d / if JV2 is in creased k eep in g TV, constant,
the secon d ary o u tp u t v o lta g e {VX/ N X) N 2 a lso in creases. In oth er w ords, d ecrea sin g primary
tu rn s TV, h as th e sa m e effect as th a t o f in crea sin g th e secon d ary tu rn s N 2
The taps which help in altering the turns ratio, m ay be placed on the prim ary or secondary side.
The choice between the two sides should be based on m aintaining the voltage per turn constant, as far
as possibie. If prim ary voltage per turn decreases, the core flux decreases and this results in poor
utilisation o f the core, though core losses are reduced. On the other hand, if prim ary voltage per turn
increases, th e core flux increases and this results in m agnetic saturation o f the core, more core losses,
increased m agnetizing current and pronounced third harmonic. In transform ers at the generating
stations, the prim ary voltage can be kept alm ost constant, consequently the taps should be provided
on the secondary side If transform er is energised from a variable voltage source, as at the receiving
end of a transm ission line, th e taps should be provided on the primary side.
O th er factors, d escrib ed below , m ay also be tak en in to co n sid era tio n , w h ile deciding upon
th e sid e to be provided w ith tap s.
(<) T ran sform ers w ith la rg e tu rn s ratio, are tapped on th e h .v. sid e sin ce th is enables a
sm o o th er control o f th e o u tp u t vo lta g e. On th e o th er h an d, ta p s on th e l.v. w in d in g , vary output
vo lta g e in large s te p s, w hich is u su a lly u n d esira b le.
(//) T a p -ch a n g in g g ea r on th e h.v. sid e w ill h a v e to h a n d le low cu rren ts, th ou gh more insula­
tion w ill h a v e to be provided.

Scanned by Cam Scanner


Transformers 101

' . difficult to ta p th e l.v. w in d in g , since it is placed next to th e core due to insulation


The h.v. w in d in g , p laced o u tsid e th e l.v. w inding, is easily accessible and can,
cansideratb°e tapped w ith o u t a n y difficulty.
th ^ ^ g ^ g r a tio n of th e foregoing p o in ts can help in deciding upon th e side of th e tra n sfo rm er
ed-
r qUestion a rise s a b o u t w h e th e r th e tra n sfo rm e r w inding should be tap p ed a t the
A mjddle. In o rd e r to in v e stig a te th is, refer to Fig. 1.53 ia), w here th e c u rre n ts in the
eI^ °r 1Uand secondary coils m u s t flow in opposite directions. T hese c u rre n ts in te ra c t with
pritnaryfl ,n betw een th e tw o coils a n d produce a rad ial force, repelling each other. This
leakage tends to com press th e in n e r coil on to th e core and b u rst th e o u ter coil aw ay from
radia* ^ repelling force m ay be re g ard ed as acting along the line joining th e centres of
thaWty8°fth e Prim ary a nd seco n d ary coils.
C o re Core R e s u lt a n t
fo r c e

V',
1 Rudiat force
1
-1 1 — 1
1 Radial force

1 11 A
Tapped part
of w in d in g , ‘
(a)
Fig. 1.53. (a) Radial force, (fc) Effect of providing tapped coils at the end of a winding

Suppose the w .nding is ta p p e d a t one end. W hen som e of th e tu rn s are cu t out by ta p


changer as shown in Fig. 1.53 (6 ); axial forces, in addition to radial forces, are also developed^
Under short circuit conditions, th e ax ial forces ten d in g to com press th e cods axia y, *
large. In order to obviate th is, th e p hysical position of th e tapped coils should ^ ‘he micMle
of winding, so th a t no axial forces a rise a fte r som e tu rn s a re cut out. ^ e c tn c a lly
changer is connected w h ere th e vo ltag e to n e u tra l is m inim um , or exa™P ' though
nected transformer, th e ta p p e d en d of th e w indings a re connected to form th e s ta r point g
Physically the tap p ed coils a re placed in th e m iddle of th e w inding, see Fig. 1.54. T h is however

Tapped p a rt
of w inding

S ta r p oint

^ connection o f the tapped coils


Scanned by Cam Scanner
lA rt. i. 14
102 Electrical Machinery
• i j frorKsfnrmers w here it is electrically essential to pro
is n o t possible in case of delta-connected tran sfo rm , rem oved from the. i-
vide th e ta p p e d coils, in th e m iddle so th a t th e tap -ch an g m g g e a r is far rem oved Irom the 1,„6
an d lig h tn in g su rg es. . . . . ,
I f th e tap -c h an g e r is designed to op erate w ith ^ p e rT te w ith th e transform eri J
o ff load (or no-load) tap-changer. A tap -ch an g er gn
circuit, is called on-load tap-changer.
1.14.1. N o -L o ad (o r off-load) ta p c h a n g e r . This ta p changer »
variations. An elementary form of n o -loadtap c l e q u a l to th e n u m b er of studs. The
mflrkpH from nr\G
m arnea irom one to
to six
six. The
m e *windings is t&ppcci
rr 3 t six p > 4 n ary stu d,s arran g edj m
m «r kpd statio * circle

tapping leads are connected to six corresponding y - tra n s fo rm e r sav on th
The face plate carrying th e six studs, can be m ounted anyw here on th e tra n sfo rm e r, say on the
yoke or on any“ Z convenient place. T he ro ta ta b le arm R can be ro ta te d by m ean s of hand,
w heel, from outside th e ta n k .
If th e w inding is tap p ed a t 2.5% in terv als, th e n w ith th e ro ta ta b le arm ,
(i) a t stu d s 1 , 2 ; full w inding is in c ir c u i t;
(ii) a t stu d s 2 , 3 ; 9 7 .5 % of th e w inding is in c ir c u i t;
(iii) a t stu d s 3 , 4 ; 9 5 % of th e w inding is in c ir c u it;
(m) a t stu d s 4 , 5 ; 92.5% of th e w inding is in c ir c u it; and
(v ) a t stu d s 5 , 6 ; 90% of th e w inding is in circuit.
S to p S fixes th e fin a l p o sitio n and
p re v e n ts th e arm R from being ro ta te d
clockwise. In th e absence of stop S, th e
arm R m ay come in contact w ith stu d s 1
and 6 . In such a case, only th e low er p a rt
of th e w inding is cut out of circuit and th is
is u n d e s ira b le from m e c h a n ic a l-s tre s s
considerations.
The tap-changing m ust be carried out
only after th e transform er is disconnected
from th e supply. Suppose arm R is a t studs
1 and 2. For bringing arm R a t studs 2 and
3 , the transform er is first de-energised and
then th e arm R is rotated to bridge studs 2
and 3. After this, transform er is switched
on to th e supply and now 97.5% of th e w ind­ Fig. 1.55. No-load tap changer.
ing rem ain s in circuit.
1.14.2. O n -lo a d t a p - c h a n g e r . T his tap -ch an g er is used for d aily or s h o rt period voltage
alteratio n s. T he o u tp u t voltage can be re g u la te d w ith th e ch a n g e r, w ith o u t a n y supply inter­
ru p tio n s. D u rin g th e o peration of an on-load ta p ch a n g e r ;
(i ) th e m ain circu it should not be opened o th erw ise d a n g e ro u s s p a rk in g will occur and
(ii) no p a r t of th e tap p ed w inding should g e t sh o rt-circu ited .
O ne form of e le m e n ta ry on load tap -c h a n g e r is illu s tra te d in Fig. 1 .56 (a). T he centre tapped
reacto r C p re v e n ts th e tap p ed w ind ing from g e ttin g sh o rt-c irc u ite d . T h e tra n sfo rm e r tapp»n^
are connected to th e correspondingly m ark ed se g m e n ts 1 to 5. Two m o v ab le fingers A and
connected to cen tre-tap p ed re ac to r via sw itches x* an d y , m a k e c o n ta c t w ith a n y one of the seg­
m en ts u n d e r n o rm al o p eration.

Scanned by Cam Scanner


T ra n s fo rm e r s 103

L.T. H.T.

O (o
fc
O o oo
a
oo -Is Supply
5
oo g=> Term inals
oJ
C
o
O
r-H
o
o 3

o
co 1h 3
a Rf
oo o |
C i------ 1
c*
o
o —' a L
oo L.
c* r—. B r
c* o 2
o
>o
o
" i 2C Z
•+-m

LI ei
“ 0 lv)
X

(b) S eq u en ce o f o p e ra tio n s from


(a) O n -lo ad ta p -c h a n g e r. ta p p in g 1 to ta p p in g 2.

F ig. 1.56.

Switches x ,y are closed O ne h a lf o th he total cu rren t flows through y , upper h a lf


and then to th e e x tern al circuit. T he ot ^ th a t currents in the u p p er and lower
of th e r e a c t o r a n d t h e n t o t h e e x t e r n a l • w h o le r e a c to r is w o u n d in t h e s a m e
h a lv es o f t h e r e a c t o r f l o w i n o p p o s i t e d i r e c t i o n s . 3 m c e ^ w h ^ r e a c ^ ^ ^ ^
direction, the m .m.f. produced by o n e - h ^ o p P ^ s ^ ^ ^ ^ . therefore, th e reactor is alm ost
These m.m.fs. a re equal a n d th e n e t.m .m . . P C onsequentiy th e voltage drop in
non-inductive an d th e im pedance offered by
the centre*tapped re ac to r is negligible. ^ ^ to segm ent 2 _ by
When a change in voltage is req u ired , th e lingers a a
adopting the following seq u en ce of operations . ^ flow through th e lower h alf
(i) Open sw itch y, Fig. 1.56 (b-i). T he e“ ^ and th ere is a large voltage drop. It should
the reactor. It, th erefo re, becom es h ig h ly i handle full load cu rren t, m om entarily.
be noted th a t th e re a c to r m u s t be designe f be moved t0 s e g m e n t 2 , w ithout any
(ii) The finger B carries no c u rre n t and can, * >
•Parking [Fig. 1.56 (b -ii)l inding between tap s 1 and 2 gets
(Hi) Close sw itch y, Fig. 1.56 [(b-iii)). The>^ a n s f o r m ^ ^ .g high for a curre t
c°nnected across th e reacto r. Since th e lm peda n t flowing through th e reactor and
°w*ng in only one d irectio n , th e local circulating cu

^ m_______
Scanned b y Cam Scanner
W

104 via
E lectrical M cminnery
ach eij —

• ■. .n Tn this manner, the reactor p rev en ts th e tapped winding frnB,


g S g lt a r t T c X d T h e terminal voltage will be m .d-w ay betw een th e potentials o f t ^

P \iv ) Open switch x. The en tire c u rre n t s ta r ts flow ing th r o u g h t h e u p p e r h a lf of the reactor,
manifested by a large voltage drop, Fig. 1.56 (b-iv).
(v) Move the finger A from segment 1 to segment 2 and then close switch x. The winding
between taps 1 and 2 is, therefore, completely out of circuit, Fig. 1.56 (b -u ). I f further change in
voltage is required, the above sequence of operations is repeated.
th e sw itcnes
For large power transform ers, tn itches x a n da y m ay be c ir cuuw
ue circ it--u
b re
r ea K
k eers
rs..
Another form of on-load tap-changer, also p rovided w ith a c e n tr e - ta p p e d reactor, is il­
lustrated in Fig. 1.57. The function of the reacto r is a g a in to p r e v e n t th e sh o rt-c irc u it of the
tapped winding. The switches 1, 2,..., 5 are connected to th e c o rre s p o n d in g ly m a rk e d taps
The switch S in Fig. 1.57 is closed d u rin g norm al C>I"T' HT
operation. With switches 2 ,3 ,4 ,5 opened and sw itch
1 closed, the entire winding is in circuit. H ere again
the two halves of the reactor, carry h a lf of th e to tal
current m opposite directions. In changing from ta p terminals
1 to tap 2 , the following sequence of operations is
earned o u t :

i °,Pu n SW“ Ch s ■Now to ta l c u r re n t flows


more voltage d ro p “ ' “ “ " “ *» -

2 is ioL c^ “ ‘^ r ctw een 1

Centre-tapped
reactor
more voltage drop. d there >s
(iv) Close switch S Thp tntoi
equally between the upper and l o n ' T ? ™ fl°WS
reactor. lower halves of the Fig. 1.57. On-load tap-changer.

For changing from tap 2 to tan 3 th k


changer^feff v a n a ^lo n d u r in g t a p - c h a n c i SG^Uence o f °P e r a ti° n s is rep ea te d .
* N C PT ° d ° n the prim ary sid e L e f ° ° n ' lo a d t a p - c h a n g e r s . A s s u m e th e tap ­
- number of t u r n s 1

^ 2 = num ber of turns between t e m in a l and ta p 1-


ATt = number " f - o n d a r ^ ^ ^ m m a U n d t a ^ . ...................
i Pn mary voltage ’ a s s um ed le s s th a n Arj and A 2.
and / = prim ary current
A , = reactor reactance when 1 n
Then, for Fig. 1.56 (a) who i th rough one h a lf o f th e reactor.
k,,ven by n 10 w hole prim ary is in circuit, th e secon d ary v o lta g e Vxl js

V -
= oa.
For Fig. 1.56 (b-i) and (b-U) « *~ -5
>^ocondnry voltoge
V V i - I X ,.
%2 = —5 - L \r
/V, iV*

Scanned by Cam Scanner


Transformers 105

For Fig th e secondary voltage


2V,
^ S3 = ■N . = oc.
n x+ n 2

For Fig. 1-56 ib-iv), secondary voltage


t, Vi ' M r
Vsi = ~ N - N s = od

For Fig. 1-56 (6 -v), secondary voltage

V M
5 = N~2 N * = 0e-
These secondary voltages Vsl to Vs5 are indicated in Fig. 1.58, It may
be seen that the m agnitude of o u tp u t voltage first decreases from oa to
ob and finally increases to oe = Vs5. The voltage changes ab, be, cd and
de in Fig. 1.58 are shown m uch larg er for clarity sake.
Example 1.55. A 100 kV A transform er is provided w ith tap-changer Fig. 1.58. Variation of
on the primary side. F in d out the tap-setting, for m aintaining rated volt­ voltage during
age on the secondary side for loads o f (a) 90 kV A a t 0 8 d f la e and (h) tap-changing.
lOOkWat 0.8p .f lag. P'h 8 ( )

The per unit leakage im pedance o f the transform er is 0.0075 + j 0.09.


Solution, (a) Taking ra te d kVA of th e transform er as the base kVA a load of 90 kVA

r ^ e ^ u to k V A tth e S S J E f t ? ^ * 9° ^ M
P.u. voltage drop or p.u. voltage regulation
= 0.9 [rp u cos Q2 + xpm sin 0 2]
= 0.9 [0.0075 x 0.8 + 0.09 x 0.6] = 0.9 [0.06] = 0.054 p.u. or 5.47,.
( l w, 6 ™ m ber tu rn s in th e p rim ary w inding should be tapped down by 5 47 ,
« A load of 100 kW or 1 00/0.8 = 125 kVA is equivalent to a load of 1.25 p u '
" U' Vo*tage drop or p.u. voltage regulation

Thu th = L 2 5 [0 ° 0 7 5 X 0 8 + 0 0 9 * 0 61 = 0 0 7 5 or 7.57,
gXa um ber of tu rn s in th e p rim ary w inding should be tapped down by 7 .57 ,.

,h'-eP ^ e s ta r lln n 1.110'433 k V tra^ fo r m e r is provided w ith a tap changer A

to

^ ' “ tio n . Per phase h.v 11,000 1 1 ,0 0 0


P er phase l.v. “ 4 3 3 /V 3 * = 44.
250
" leak;
age impedance referred to l.v. side
J
= (300 + j 1500) - (0.155 + j 0.775) II per phase.
[otai 44
Phase im pedance betw een th e tran sfo rm er secondary and th e load
= (0.155 +y‘0.775) + (0.2 + j l) + (0.6 + j 1 )
= 0.855 + j 2.775 12 - /? + jX .

Scanned by Cam Scanne?


106 Electrical Machinery 1.15

P e r p h ase load c u rre n t I L = = 14 43 ^

T o tal im p ed an ce drop per p hase


= 1L [/? cos 02 + X sin 62 )
= 14.43 10.855 x 0.8 + 2.775 x 0.6]
= 14.43 (2 .3 5 9 ) = 34.05 V.
P e r p h ase voltage to be m ain tain ed a t the load term inals

= ^ r = 2 3 iv -
P e r p h ase voltage th a t m u st be m aintained a t transform er l.v. term in als
= 231 + 34.05 = 265.05 V
At no load, th e tra n sfo rm ers l.v. term inal voltage

^ = 250 V.

T he voltage boost th a t th e tap-changer m ust provide


= 265.05 - 250 = 15.05 V. .

T ap settin g = ^ 50 ” * 1 00 = 6 0^
T ap down if th e tapped coils are on the h.v. side or tap up if the tapped coils are on the l.v.
side.

th e ^ o r " o » ^ o 'T e t a U o o n e p o l

ly, induction voltage reg u lato rs in larger sizes m ay m ovem ent and, there-

Scanned by Cam Scanner


Transformers 107

Prim ary
winding,Ni
turns
Compensating
w inding,N c
— turns
T
%
P rim a ry
win ding a x i s

Secondary
winding, N2
tu rn s
Compensating
winding axis

(a)
diagram In both the figures, secondary
Fl- J 59. (o) S in g le -p h a se ind uction regulator, (ft) Connection
' l.f. £ 2 is m axim u m and ou tp u t voltage is V ) £ 2 -
induced e.m.
• 1/ show n in F ie 1 59 <i>) If th e p rim ary
winding is m a x im u m a n d th e o u tp u t vo ltage i , 2 m axim um , b u t o f rev ersed
U rotated th ro u g h 18 0 ” Fig. 1 .6 0 to h e s e c o n d a r y ^ E ^ ^ com pensating w inding is not
polarity. T h e o u tp u t v o lta g e is t h erefo re . 2- th e ,r m agnetic ax es a re fixed
at all effected b y p r im a r y w in d in g , sm c e b o th a re on th e r
in quadrature. a n «rle 0 from its position show n in Fig.
Assume th e ro to r to be r o ta te d clockw ise ro g 1.60 ( 6 ) and th e o u tp u t v o ltag e is
1 59(b). T he m a g n itu d e o f v o ltag e is £ 2 co s« ,
s e c o n d a r y m M Condary w ,„ d in g . is
V, + E2 cos 0. W h en th e a n g le 0 is m a d e 90 , th e re bc(;n no c o m p en satin g
* U e se co n d ary w in d in g , h o w ev er c a r n e s o by a n y o th e r m m f As a
Winding, th e se c o n d a ry w in d in g m .m .f. W , a nd th e r e a c t a n c 0 offered by secon-
result of it, m .m .f. 12 ^ 2 w o uld s a tu r a te i t s a larg e v 0 „ a ge drop across th e
dary w inding w ould b e q u ite larg e. T h e re w o u |d th e ■ vo)tagc T h e co m p en satin g
secondary w in d in g , in d ic a te d by ^ t T ^ o e n ^ e n 9 = 90". th e co m p en satin g w inding C
winding, how ever, d oes n o t p e rm it th is b ' h a p p , |in g T herefore, th e secondary w ' n d ' n P
and the seco n d ary w in d in g h a v e m a x l7’u . d in g c by tra n sfo rm e r action. T he com in a 1
current / 2 in d u c es m a x im u m c u r r e n t in E. tra n sfo rm er, w ith secondary w inding
these two w in d in g s m ay be re g a rd e d a s a s w jn d jn g as th e s h o rt-circu ited seco n d ary . ^ e m ^
T) acting as th e p r im a r y a n d c o m p e n sa c o m p en satin g w inding m m f an e
h N 2 is alm o st c o m p le te ly n e u tr a lis e d by t g= and ^ th e w in d ,ng c a n d th e
offered by se co n d a ry w in d in g is F o r o th e r v alu es of 0, th is n u itu a couplm g
secondary w in d in g h a v e no m u tu a l “ “ P ^ g J ^ y o n d e = 90o th c m u tu a l coupling d ecreases
leases w ith 0 a n d b eco m es m a x im u m a
and eventually becomes zero at 0 - 180 • w ith or in phasi. opposition j ,
,k Note th at the se co n d a ry v a ria b le voltage ,<, e th e r -n ^ w u ld in g prod„ces o n iy
prim ary voltage V ,. This is d u e to the fact in

alt«rnating or p u ls a tin g flux. w ith 0 m ea s u re d from th e


^ g . 1.60 ( d ) gives the o u tp u t voltage vmu ^ p r i,lia ry and secmu .u \ " » u -
0f % 1 . 5 9 ( 6 ) . L e t ^ a n d ^ 2 b e t h e n u m b e r o f t u r

like a tra n s fo rm e r, d a ry m m f com ponent along prim ai.v w in ing axi


lOAfj PAmnnnnn^ nf n f IFT1 TV IT1IT1f ^
Scanned b y Cam Scanner
ft

108 E le c tric a l M a c h in e ry I Art. 1.15

e2
Vr-Vr £2

VE2

(o)

x p-'K&ttUTsftrtnns^—♦ V-v, ^ -V 2=V,+E2cos9


'------------ 1 90° e 2=o
------
O 5 *
ot o
o
o ! >
o V
o
o D
o V \< h
o ? *
9()° 180
Io 0 in deg.
Q —« - <
(c) <d)
Fig. 1.60. Effect of rotor m ovem ent on the output voltage.

Fig. 1.61 (a) reveals th a t


11 N , = I 2 N 2 c o s 0
12 N 2
or 7,' = - ^ - cos 0 .

H ere I { is th e load com ponent of p rim a ry c u rre n t I v T he c u rre n t 7 / e x ists in p rim ary due
to th e seco n d ary c u rre n t I2. N ote th a t seconds, v m m f com ponent I 2 N 2 sin 0 is balanced by
c o m p e n sa tin g w in d in g mmfA^. 7Cas show n in Fig. l.Cl (b).
W ith Ie a s th e p rim a ry w ind in g exciting c u rre n t, th e Aotal p rim a ry c u r re n t Ip is th e phasor
su m of Ie a n d l \ . T h e c u rre n t ta k e n from th e supply is th e p h a s o r su m of p rim a ry c u rre n t7 p and
I 2, Fig. 1.61. T h e pow er facto r o f th e load is ta k e n as cos 0 2 lagging. N ote t h a t p rim a ry winding
c u r re n t Ip is m uch s m a lle r th a n seco n d ary w in d in g c u rre n t 72.
T h e r a tin g of a sin g le -p h a se in d u ctio n re g u la to r is eq u al to th e p ro d u c t of full-load output
c u r r e n t a n d th e m ax im u m v oltage v a ria tio n from no rm al.
T h e a d v a n ta g e s o f in d u ctio n v oltage re g u la to r over th e ta p -c h a n g e r tra n s fo rm e r are (i)
sm ooth o u tp u t v o ltag e control a n d (ii) no in te rru p tio n in th e su p p ly circu it.
Its d is a d v a n ta g e s a re (i) m o re in itia l cost (ii) m ore m a g n e tiz in g c u rre n t because of the
jre se n c e o f a i r gap.

Scanned by Cam Scanner


t ijw : ,

Transformers 109

(o) (h)
Fig. 1.61. Single-phase induction regulator (a) currents and mmfs and (b ) phasor diagram for currents.

1,15.2. T h r e e - p h a s e i n d u c t i o n r e g u l a to r . A th re e -p h a se induction voltage re g u la to r


r e s e m closely a th re e -p h a se slip -rin g induction motor. The rotor carries th e p rim ary w ind­
b le s

ing and the sta to r h as th e secondary w inding as in th e case of single-phase type. T h ree-p h ase \\
primary winding is connected in s ta r, w hereas the 3-phase secondary is connected in series
with the load as show n in Fig. 1.62. In large sizes, th e induction regulator is oil-im m ersed in
a tank like an o rd in ary tra n sfo rm e r. Rotor m ovem ent is carried out in the sam e m an n er as in
a single-phase induction reg u lato r.
When the induction voltage reg u lato r is connected to 3-phase supply, th ree-p h ase cu rre n ts I
in the three phase p rim a ry w inding produce a co n stan t am plitude ro tatin g m agnetic field as in
a 3-phase induction m otor. T h is ro ta tin g m agnetic field induces e.m.fs. in th e secondary w ind­
ing whose m agnitudes depend only on th e ratio of prim ary to secondary tu rn s and are inde­
pendent of the rotor position. W hen th e rotor position is changed w ith respect to stato r, th e
magnitude of secondary e.m .fs. rem ain s co n stan t but th eir phase is altered w ith respect to
primary voltages. T he o u tp u t voltage V2 is th e phasor sum of prim ary voltage V x and th e con­
stant secondary e.m.f. E 2 induced by th e ro ta tin g m agnetic field. For fixed p rim ary voltage, the
locus of output voltage V 2 is a circle w ith centre a t the tip of phasor V x and of rad iu s E 2. T hus
maximum output voltage V2 = V X+ E 2 is obtained w hen E 2 is in phase w ith V x and m inim um I
voltage V2 = Vj - E 2 is av ailab le w hen E 2 is in phase opposition to V x. For interm ed iate values
ofoutput voltage V2, th e re is a p h ase displacem ent betw een V x and V2. It m ay be seen from Fig.
*•62(6) that
y 2 = V(V x + E 2 cos 0)2 + (E 2 sin
nd the phase d isplacem ent angle a betw een V x and V2 is

- 1 E 2 sin 0
a = ta n .. r _ n
Vj + E 2 cos 0
of ^ere ®*s the angle b etw een p rim a ry w inding axis and secondary winSm g axis. The value

and 9 = 0° w hen V2 = V X+ E 2
p . 0 = 180°
VJ —J .O V J w
W hen
i l d I \» \^ = Vrj1 - E L2.
the three-phases
**** sh * ^ ^ r efers to o n e-p h ase of th e regulator. The phasor diagram for
oltages respectively.
0Wn ^ Fig. 1.63, w h ere A B C an d abc Are th e in p u t and o u tp u t volt;

canner
110 Electrical Machinery U r t . i. IS

Note th a t th ere is phase displacem ent betw een th e supply line an d o u tp u t voltages which may
be objectionable in some applications.
I t is seen from above th a t 3-phase induction reg u lato r differs from th e single-phase induc­
tion regulator as follows :
(а) In single-phase regulator, th e secondary voltage E 2 induced by th e a lte rn a tin g flux is
always in phase with V^. The m agnitude of E 2 varies as th e ro to r position is changed with
respect to stator.

In a 3-phase induction regulator, th e m ag­


nitude of E 2 rem ains constant a t all th e rotor
positions and fu rth er E 2 is not in phase w ith Vj
except at maximum boost and m axim um buck
positions.
(б ) Single-phase induction regulator requires
compensating winding, whereas 3-phase induction
regulator requires no compensating winding.
In both th e types, th ere are no slip rings and
prim ary winding is energised through flexible
leads only. Both th e types are in a way step-
down tran sfo rm ers. In single-phase type th e
secondary induced e.m.f. depends both on the
ratio of prim ary to secondary tu rn s an d on the Fig. 1.63. Phasor diagram for 3-phase induction
rotor position with respect to stato r. In 3-phase regulator.
- i n d u c t i o n r e g u l a t o r , t h S secondary induced e T f d „
dary tu rn s and not on th e rotor position d epends only on th e ra tio of p rim ary to secon-
A . Posm on w ith respect to s ta to r.
E x a m p le 1.57. A single-phase in d n rti^ i .
p n m a ry supply voltage is 400 V, d e te rm in ^th g l? r has a n effective tu r n s ratio o f 5 : 1 • l f th‘
age. I f the load current is 10 amperes and th ° m ax}m u m a n d m in im u m values o f output volt-
calculate the prim ary w in d in g current and th l r]egulator is set ** th e m a x im u m boost posit'0*'
Qnd tke llne cu r^ t Ignore the effect o f no-load cur***

t
Scanne
Transformers 111
S o lu tio n . M ag n itu d e of m axim um secondary induced
400
6 = I T = 80 v -
• Output voltage limits = 400 ± 80 V Tl .
tages are 480 V a n d 320 V respectively. axim um and m inim um values of output vol-

Rating of secon d ary w in d in g = (Load current) (Maximum voltage variation from norm al)
10 x 80
= T o o o ~ = 0 -8 k v A-
Neglecting no-load c u rre n t, p rim ary w inding current
_ 10 x 80
400 = 2 A
... P rim ary line c u rre n t = (O u tp u t current) + (Prim ary winding current)
= 10 + 2 = 12 A.
E x a m p le 1 .5 8 .T he voltage a t the end o f a 3-phase feeder delivering 100 kVA varies between
380 and 460 volts. For how m a n y kV A m u st an induction regulator be designed to m aintain the
voltage constant a t 430 V.

Solution. P e r p h a se load c u rre n t = -l-Q0,990- A


v3 x 430
Voltage a t th e o u tp u t side of reg u lato r = 430 V.
Maximum v a ria tio n o f voltage from 430 V = 430 - 380 = 50 V.
kVA r a tin g of in d u ctio n reg u lato r
( 100,000 ^ 50x100
= V3 (50) kVA = 11.63 kVA.
73x430 430
E x a m p le 1.59. A 3-phase induction regulator is designed to vary the output voltage V2
between the lim its o f V x ± E 2- V 1 is the supply voltage and E 2 is the m agnitude o f secondary
induced e.m.f. For V 1 = 400 V, E 2 = 50 V, load = 8 kW, load p.f.= 0.8 and overall efficiency o f
0.85, calculate the ra tin g o f the p rim a ry w inding and the secondary winding.

S o lu tio n . In p u t c u r re n t = ^ x O . s t T s S x 400 = 16 9 8 A
M in im u m o u t p u t v o l t a g e = 400 - 50 = 350 V.

M a x im u m o u t p u t c u r r e n t = ^ x x 350 = 1^ ’5 °

• • R a tin g o f t h e s e c o n d a r y w i n d i n g
= V3 x 50 x 16.50 = 1.429 kVA.
The p r im a r y w i n d i n g h a s to c a r r y (i) c u r r e n t in d u c e d in it b y tr a n s fo r m e r a c tio n d u e to
secondary c u r r e n t a n d (ii) t h e d if f e r e n c e o f in p u t c u r r e n t a n d o u t p u t c u r r e n t.

Minimum o u tp u t c u r re n t
8000 = 12.83 A.
V3 x 0.8 x 450
•• M aximum p rim a ry w in d in g c u rre n t
. j 4. (Innlit cu rren t - M inim um o u tp u t current)
= P rim a ry induced c u rre n t + (in p u t curre
= 12.83 x 50 + (1 6 9g _ 12.83) = 1.60 + 4.15 = 5.75 A.
400

Scanned by Cam Scanner


(A rt. l . i 6
112 Electrical Machinery

.-. R atin g of th e p rim ary w inding


= V3 x 5 .7 5 x 4 0 0 = 3.985 kVA. '
E x a m p le 1.60. A single-phase, 50
mW 6 p er pole. The stator has 10 For a su p p ly voltage o f 230 V.
turns, then calculate the m agnitude o f m a xim u m repUla tor for a load current o f 30 A
fin d the lim it o f o u tp u t voltages. A lso fin d the rating o f the regulator for a , a.
S o lu tio n . For a phase-spread of 120°, th e d istrib u tio n factor

sin £ ( 1 2 0 °)
- = - ( 0 . 866 )
71 71
i < 120’>x 180
.-. M agnitude of m axim um secondary induced e.m.f. E 2
= V2 n f N • <t>k
N
= yl2n (50) (20) (0.0105) - x 0.866 = 38.6 volts.
71 /
/. Lim it of o u tp u t voltages
= 230 ± 38.6 = 268.6 to 191.4 volts.
R atin g of th e induction reg u lato r
= 30 x 38.6 = 1.158 kVA.
1.16. T ran sform er as a M agn etically C ou pled C ircu it
In th is article, th e eq uivalent-circuit m odel of a tw o-w inding tra n sfo rm e r is developed and
th e n com pared w ith th a t o b tained already in A rt. 1.6. H ere th e tra n s fo rm e r is tre a te d as a
m u tu ally coupled circuit in w hich th e voltage and c u rre n t re la tio n s are ex p re sse d m term s ot
resistan ces an d indu ctan ces. As before, th e m agnetic s a tu ra tio n is n eg lected , i.e. th e core is
assu m ed to h av e c o n sta n t p erm eab ility. T he p re s e n t tre a tm e n t of th e tra n s fo rm e r gives a
physical significance of th e equ iv alen t-circuit p a ra m e te rs, in te rm s of th e tra n s fo rm e r mag­
netic fields. In a w ay, th is article su p p lem en ts th e p re se n ta tio n of A rt. 1.6 a n d probably leads
to a b e tte r ap p reciatio n of th e p a ra m e te rs involved in th e tra n s fo rm e r e q u iv a le n t circuit.
In Fig. 1.64, a tw o-w inding tra n sfo rm e r is show n, w h ere r x a n d r 2 a re re s is ta n c e s of primary
a n d seco n d ary w in d in g s resp ectiv ely . T he ap p lied vo ltag e v x e s ta b lis h e s c u r r e n t ix in the
p rim a ry w inding. W hen c u rre n t i x is actin g alone, to ta l flux (j^ is produced. T h is p rim ary flux
<♦>! lin k s a ll th e p r im a r y t u r n s a n d is m ad e u p of tw o c o m p o n e n t flu x e s <}>ml a n d <t>n, i-c-

Fig. 1.64. M utual and leak age flu x es in a tw o-w in d in g transform er.

xJUUI II IVyU k_/ ’ W III vVUUI II IWI


Transiormers 113

see F ig - 1 6 4 - ^ u x ^m i p roduced by i x alone, is called th e m u tu a l flux, sin c e it lin k s


^ ? (Jr«i+ a n d s e c o n d a r y w in d in g s. F lu x 4>/i lin k s only th e p rim a ry w in d in g a n d is, t h e r e ­

* * * * * orim ary leak ag e flux. T h e m u tu a l flux j is tim e v ary in g , th e re fo re , a n e.m .f. e 2 is


fore. cft* k e s e c o n d a r y N 2 tu rn s . I f a load is connected across th e se c o n d a ry w in d in g , a cu r-
flowing.
r*® 2 S e c tio n of c u rre n t i2 iR W2 tu r n s m u st be in su ch a d irectio n a s to oppose th e flu x
r Lenz’s law. In o th e r w ords, if i2 w ere actin g alone, th e m u tu a l flu x <J>m2 p ro d u c e d by
^ 33 Id be in a direction opposite to <j>ml. W hen p rim a ry c u rre n t i x a n d s e c o n d a ry o r load
it ^ ^ acting to g e th e r, b o th <|>ml a n d <bm2 a re produced in th e m a g n e tic core a n d th e
carr* t mutual flux $ lin k in g bo th th e p rim a ry and secondary w indings is <J>mi - <j>m2 -
r 0tal flux linkage w ith th e p rim a ry w in d in g is, y j = (R e su lta n t m u tu a l flux lin k a g e s w ith
. ^ ♦ ( L e a k a g e flux lin k ag es w ith p rim ary )
** = + -Ni$/i = N X (<J>ml - <J>m2) + N X0 /i
or ¥1 = 1 + <b/i) - N x <|>m2 = ATi<t>i - N x <j>m2.
Now the KirchhofFs voltage law gives th e voltage eq u atio n for th e p rim a ry w in d in g of
transformer of Fig. 1.64 as

. ^¥1
v i = r il i + ~dT ...(1.82)
Substitution of Vt in Eq. (1.82) gives
f.
Vl = riil + d t ...(1.83)
The self inductance of a w in d in g is defined as th e to tal flux lin k ag e w ith a w in d in g p e r
ampere in the sam e w inding. T herefore, th e Belf-inductance of p rim a ry w in d in g is,
_
L x ---- ^— or

The mutual inductance is defined as th e flux lin k ag e w ith a w inding p e r a m p e re in a n o th e r


mutually-coupled w inding. T h u s th e m u tu a l in d u ctan ce of th e p rim a ry w in d in g is,
M ^l4>m2
M u = — ----
*2
Here N flm2 is th e flux lin k ag e w ith th e p rim a ry w inding due to c u rre n t i2 in th e se c o n d a ry
S-

Similarly, A/ = - 2* ” 1
h
^°r mutually-coupled circu its,
, 3 f 12 —M 2j —M.
N iK 2 = M i2 a n d N 2$ml = M i1
ubstitution of th e v alu es o f N f a a n d in Eq. (1.83), gives

v \ = r li l + ^ { L li l - M i £

. . d ix di<i
...(1.84)

canned by Cam Scanner


(Art i,16
ll4 Electrical^Machinery---------------- " T ^ ^ T T ^ s u l t a n t m u tu a l flu x linkages wi,h
T h e flu x lin k a g e w ith th e s e c o n d a r y ^ ■ ' (R* "
seco n d a ry ) + (L ea k a g e flu x lin k a g e s w ith secon d ary) tb<

. = W 24' + W - W = - W
= Ar24 'm l- W2<(tlm2 + W
or Vg2 = w 2 -N th . th flt ,s o p p o sin g th e r e s u lta n t mutual a * 71.1
T h e m in u s s ig n b efore <fo2 1S d ue to

*■ T h e v o lta g e eq u a tio n for th e seco n d a ry circu it, from F ig. 1 .6 4 , is

dy2
= r2i2 + v 2
dt

or
N 2 $2 N 2 tym\
L2 = and M -
S in ce h

(M i1 - L 2i2) = r 2i2 + y2
dt
...(1.85)
_M — + ^ + r2t2 + y2 - 0
or dt & n d ( i 8 5 ) is illu stra ted in Fig.
A m a g n etica lly coupled circu it w h ich sa trsfies E qs. (1 .8 4 , a n d (1 .8 5 ,

1 6 5 11' P (1 8 5 ) is for th e secon d ary sid e o f th e tra n sfo rm er. T h is ca n be referred to


V o lta g e Eq. (1 . ^ E q . ( 1 .8 5 ) w h en referred
p rim ary sid e b y m u ltip ly in g it w ith th e tu rn s ratio ^

to p rim ary sid e is


, d i i ^ 2t d _ \ lA + a 2r2 l2 + a v 2 = 0
dT + o i 2 d t U ya
A dd ing o M | f i j t o th e first term a n d su b tra c tin g it from th e seco n d term , w e get

l2 ...(1-86)
+ a 2r2 + av2 = 0
+ (a 2L 2 a
a
~ aM dt y y y y
T he voltage Eq. (1.84) can be w ritten as
/• \
. r d il TUTA . H
v 1= r lh + L l d t ~ o M d t

L,-aM a2L2'aM a?r2 +


... n r-m r\ qflflp. 9

" fc S I

(a) (6) jer


Fig 1.65. (a) Magnetically-coupled circuit and (A) Conductively coupled circuit of a transfer

A.

Scanned by C am Scanner
Transformers 115

•. * 1
th right h an d sid e o f above eq u ation , subtract aM from second term and add it to

^ t b ^ term
...(1-87)
v l = r 1i i + (Ll - a M ) —
and
i nfivplv coupled circu it for a tw o-w inding transform er, sa tisfy in g Eqs. (1.86)
' S t a t e d i n F i g - 1 6 5 (6).
^ Eqs. (1.86) and (1.87) should be exam ined farther.
The terni cm ^
10 w JVX N 2« W _ A M w
aM = Try ;------- — -■-----
^2 *1 ll
The mutual flu x lin k a g e (h ere N ^ ml) w ith a w inding per am pere in th e sa m e w in d in g is
ailed the magfletizing inductance, sym bol L m. ;

aM = — = L'm l

„ . ’ . WiOi , * _ *
Also L ,- a J f = L 1- L „ , = - ^ ^ --tti ♦»!>

The leakage inductance is defin ed as th e leakage flux link age (here AT,*,,) w ith a w in d in g

per ampere in th e sa m e w in d in g .
. i _ aM = i l - L ml = l l = prim ary lea k a g e inductance.
" 1 , 2f . M'l
Similarly (az L 2 - a M ) = a u - -
a
N 2 ^ l^m2
=a J • = a 2 (L2 - L m2)
L2 N X 12
. 2/
= a 2 (secondary lea k a g e inductance) = a l2.

The current flow in g th ro u g h m a g n etizin g inductance aM or L mi o f ^ i ^ e toss


(primary current ^ -seco n d a ry cu rren t referred to p n m ary, i.e. r2/o ) . S m ce th e core lo w

neglected, ( i, - ^ I m u s t be eq u a l th e tran sform er m agn etizin g current i„ , i.e. |i, - j = i„ .


_ V / . n and (1 87) can be re-w ritten a s follow s :
In view o f th e foregoin g d iscu ssio n , Eqs. (l.o b ) a n v •
From Eq. (1.86), (• \
h + av2
d i'm , 2i ^ V -I-a2r2 ...(1.88)
0 = -L ~dt a a
ml d t V \ /
^dfrom Eq. (1.87),
di di rn ...(1.89)
y i = r iii + / i + L "»* d i
'ml d t
J h e circuit of Fig. 1 .6 5 (h) can now be draw n a s show n m Fig. 1 6 6 (a),

S t lead * “ “ P‘ed e l e m e n t ^ , . In order


“^ 1 , the resultant core flux (<t>mi - *™2) h a s been taken care or ny
Vetlfy this, proceed as follows. . , = (m .m.f. due to i x) (core
pe mutual flu x <j>ml p rod u ced w ith i x a ctin g alone, is g iv e y <Pmi
J ^ ean ce A, offered to th e m u tu a l flu x). ...(1.90)
0ml = N\h A

Scanned by Cam Scanner


1
116 E le c tr ic a l M a c h in e r y

I l f ri

' • a p r e f e r r e d to p r im .r v and < « i» a gene,al form .


F ig . 1 6 6 . T r a n s fo r m e r e q u iv a le n t c ir c u it (

_ N 1 K i =th
(Nli
N ow aM = Lm 1= h ‘i 2

S im ila rly ,
... T h e r e s u l t a n t core flux * M U . - - w in d in g h ^ . W] (N , . N, ^
*2
T h u s, th e r e s u lt a n t m u tu a l flux lm k a g ^ ^ —I/ml lm
*1“ a
7 /^ P P ffultant m u tu a l flu x lin k a g e
L mi = ~i - M a g n e tiz in g c u r r e n t
iviaBuctrxxea**fo---------
or
P • , * I f
H ence th e m a g n etizin g f i a n c e ^
ta k e s care o f th e r e s u lta n t core flu x <)>. Similarly, the
^ ^ a n d ^ re sp e c tiv e ly .
le a k a g e in d u cta n ces l x and /2 t rirru it o f F ig 1 . 6 6 (a) is draw n in Fig. 1.66
For sin u so id a l applied vo lta g e, th e e q u iv a le n ^ a s b efore th e id ea l transformer
(6), w h ere th e tu rn -ratio a is a s s ^ e d fo r sm y^ ^ ^ ^ ^ ^ respectlVely
is n o t sh ow n . N o te th a t h ere le g tra n sfo rm er core lo s s can be accounted
an d th e m a g n e tiz in g reactan ce, X m = o £ ml. T h e e n
u ■ ta n ce B in p a ra llel w ith X . . H ere is th e core lo s s an d is th e core
for by a r e sista n c e i i c ,2 ^

b een referred to, m u st be k ep t in m in d . f r m .s . valuesas


For sin u so id a l a p p lied voH age Vx, E q n . (1 .8 9 ) m a y be e x p r e s s e d m te r m s o f r.m

V\ = rx1\ + j CO^i /1 + j tnLml ’ Im (1.91)


= l l ( r x + j x x) + j inLmi-Im
. ,v third term
F ir s t tw o te r m s o f E q n . (1 .9 1 ) a g r e e w ith E q n . (1 .2 1 ). L e t u s e x a m in e e
(aL mj . Im. It h a s b een proved a b o v e th a t

T _
L m\ _ T
lm
R .m .s. v a lu e o f m u tu a l flu x lin k a g e s , N x ^ - L ' m l ‘ ^m‘
and

In N x -<p,<p is th e r.m .s. v a lu e o f flux. I f ^ is th e m a x im u m v a lu e o f core flu x, th en 4> %

th e term a)Lml • Im = l can be w ritten a s

<t>n
(nNi • <j) = 2nf- N x • = 'f o n f N j • <j)maJt

Scanned by Cam Scanner


Transformers 117
t i - t f U - — ------------------------ • '

^ T ^ t h e s a m e as E q n . (1 .4 ).
^ S. r = coNi • <t>= V2tc/'TVi • <J>max = E h e.m .f. in d u ced in p rim a ry w in d in g .
.. (flLml ' i/n
1 64, if v o lta g e u2 is a p p lied to th e w in d in g h a v in g N 2 tu rn s, th e n cu rre n t i 2 is s e t up.

ln l^ ent i2 e s ta b lis h e s m u tu a l flu x <j>m2 in th e d irection o f <|>ml, th e n th e r e s u lt a n t m u tu a l


lfthe <ti o and E q s. (1 .8 4 ) a n d (1 .8 5 ) becom e
flux is ^ , . .

v x = r li 1 + L l - ^ + M ~ ...(1 .9 2 a )
at at
d i 2 ^^ di^
Vn —Tilo "*■Lto
and
It may be re c a lle d t h a t E q s. (1 .9 2 a ) a n d (1 .9 2 6 ) a r e tr u e o n ly w h e n t h e m a g n e tic s a t u r a t i o n
tected . S im ila r e q u a tio n s a r e u s e d q u ite f r e q u e n tly in g e n e r a liz e d th e o r y o f e le c tr ic a l
S dmies and in C h a p t e r 2 o n e le c tr o - m e c h a n ic a l e n e rg y c o n v e rs io n p rin c ip le s .
It has alre a d y b e e n s h o w n t h a t t h e m u t u a l in d u c ta n c e b e tw e e n tw o c irc u its is g iv e n by

M = s------- = :-------
h h
, , N 1^ 2 NiOta - <J>/2)
=— n—

Substituting th e v a lu e o f <|)ml fro m Eq. (1 .9 0 ), w e g e t

N2_i2 A _ AJ. . N xN 2
Also M = —— --------- N x N 2 A = ^

An examination of the above expressions for M reveals that the mutual inductance between
two circuits can be increased by the following methods :
(t) Increase in the number of primary turns N v
Hi) Increase in the number of secondary turns N 2.
(iii) Increase in the permeance A (or decrease in the reluctance RI) offered to the mutual
flux.

to) Decrease in the leakage flux. The leakage flux between two windings can be reduced or
emutual inductance increased, by arranging the two windings as close to each other as is
Possible.

^•16.1, C o-efficient o f c o u p lin g . In m agnetically-coupled circuits i f th ere w ere no leak age


the coupling a m o n g st th e m w ould be perfect. H ow ever, in practice, som e le a k a g e flux
ays present.
‘“ 'oupled circu its, t h e r a tio o f m u tu a l flu x to th e to ta l flu x p rod u ced b y th e s a m e cu rre n t
alone, is called th e c o u p lin g fa cto r. T h erefo re, for th e p rim ary w in d in g , th e co u p lin g
U given by

<()ml, m u tu a l flu x p roduced by ij a lon e


ki = <j>lf to ta l flu x p rod u ced b y ij a lon e

4>i-4>n ...(1.93)
4>i

ScannecTBy Cam Scanner


________ [Art

118 E le c t r ic a l M a c h i n e 0 1 _ _ ------------------- ---------- T T C ^ u C u n d ^ l f C d u c t a n c e s F o r th is

" 1 ^ ^ ^ - s e d in terms of t h e ^
Eq. ( 1 .93 ) ca n now o e - e . — ^

pose, multiply the numerator and denomm ^


*«r x \v //-1m r 1,
Jjm Ni. .M ••(1.94)

L1 2 Nl m
(W2 * m l V i l = M
h .= - r r r -r T 7 r
A lte r n a tiv e ly ,
1

.. „ fact0r Jfe2isgiven b y
Similarly for winding 2, the coup m ^ b y r .a lo n e

.(1*1

= . tj N 2 U .(1.9fj

(N ^ iV h N 1 .W 2 ,
or
. ,. J l i a definedas th e geometric mean of the coupling factors 1, *
The coefficient of coup mg

k l- k= ___________ ____ #
,..(1.97)
fN t M 'N 2 m ] M—
n 2 hx N x ' L 2)
V ...(1.981
Qmljmg
(^ 3 ilS Z S
or
L eak age coefficien t ...(1.99)
AT
L\ L
_ " (1 95) and (1-9& that l°wer value of leakage fluxes gives gnatfI

values o f coupltog (actors hj, I 2 of a winding

„ „ s . r s ? r : s — z s . , ’— ;

in te rlin k e d or coupled. r-ircuits with low value of leakage

— - —

^i = i ; from Eq. (1.94), J ^ =

Nx M
k2 E l ; from Eq. (1.9 6),-^ = 2 ^

From the above two relations, No l>2

Scanned by Cam Scanner


Transformers 119

Z mJ £
n 2 '• V l 2
for c lo sely co u p led c ir c u its
flen c e ,
^ __ a / lL 1
...(M O O )
n 2 ~ m ~ l 2 ~ y jr2

i 16 2. M e t h o d s o f i n c r e a s i n g t h e c o e f f i c i e n t o f c o u p l i n g . T h is m a y a ls o b e c a lle d a s
“methods o f r e d u c in g th e le a k a g e flu x ”.
^ Eqs (1.93). ( 1 0 5 ) a n d (1 .9 8 ) r e v e a l th a t th e co u p lin g fa cto rs an d th e c o e ffic ie n t o f c o u p lin g
be increased by r e d u c in g t h e le a k a g e flu x e s p e r ta in in g to th e tw o c ircu its. In tw o -w in d in g
^ s f o r m er3’ co efficien t o f c o u p lin g ca n b e in cr ea se d or th e le a k a g e flu x red u ced b y th e fo llo w ­
ing m ethods:
(a) By increasing the w in d o w height. In a tra n sfo rm er th e w in d o w a rea H x W is g o v e r n e d
h the co n d u cto r c r o s s -s e c tio n a l a rea o f p rim a ry and seco n d a ry w in d in g s a n d th e in s u la tio n .
For a given area i f w in d o w h e ig h t H is in c r e a se d , th e le n g th o f le a k a g e flu x p a th b e c o m e s m ore.
This has the effect o f in c r e a s in g th e r e lu c ta n c e offered to th e le a k a g e flu x p a th s a n d for a g iv e n
m m f-, the le a k a g e flu x is red u ced . U s u a lly th e ra tio H I W d oes n o t ex ce ed 4.
ib) By arran ging the p r i m a r y a n d secondary w in d in g s concentrically. F ig. 1 .6 7 (a ) s h o w s
primary w in d in g on on e lim b an d seco n d a ry w in d in g on th e o th er lim b or leg . In F ig . 1 .6 7 (6),
half of the prim ary a n d s e c o n d a r y w in d in g s are on o n e le g and th e ir o th e r h a lv e s a r e on th e
second leg. S in ce th e p r im a r y a n d seco n d a ry w in d in g s in F ig. 1.67 ( 6 ) a re m o re c lo se to ea c h
other, leakage flu x is red u ced a n d th e co efficien t o f co u p lin g in cr ea se d . R ed u c tio n o f le a k a g e
flux in Fig. 1.67 ( b ) m a y a lso b e e x p la in e d a s fo llo w s :
L .V .
W in d in g
/
1 2

« 0 ®


7
H .V
W in d in g
(o) (b)
F ig . 1 .6 7 . (a ) P r im a r y a n d s e c o n d a r y w i n d in g s o n e a c h lim b o r le g
(6 ) C o n c e n tr ic p r im a r y a n d se c o n d a r y w in d in g s .

In Fig. 1.67 (a), th e m a g n e tic p o te n tia l dif- L .V . W i n d i n g


erence betw een p o in ts A , B is N lt w h e r e a s
i
*D ^g. 1.67 (6) it s v a lu e i s * *. S in c e th e
I 1 0
0 0 0
til ^ ^ UX ^e Penc^s u Pon th e m a g n e tic p o ten -
ifference b e tw e e n p o in ts A, B it s v a lu e in
(aj* 1 (5) is a b o u t h a lf o f t h a t in F ig . 1.67

^ H.V.
^ sand w ic h in g the p r i m a r y a n d secon- W inding
i a t F i g . 1 .6 8 s h o w s t h e w in d in g 2
F ig . 1 .6 8 . W in d i n g 2 is s a n d w i c h e d b e t w e e n s e c t i o n s o f
O f o f^ tio n s and w in d in g 1 in fo u r s e c tio n s . w i n d i n g 1.
sectj&n * "E nding 2 is s a n d w ic h e d b y tw o
*indi w in d in g 1. T h e a im o f s a n d w ic h in g is a g a in to a r r a n g e th e p r im a r y a n d se c o n d a r y
***** as close to e a c h o th e r a s is p o s sib le , so t h a t th e le a k a g e flu x is r ed u ced .

Scanned by Cam Scanner


120 Electrical Machinery

T h e le a k a g e flu x c a n b e red u ced fu r th e r b y u s in g m o re s a n d w ic h e d sectio n s f0r ^


p rim a ry an d seco n d a ry w in d in g s.
id) B y u sin g shell-type construction. In sh e ll-ty p e tr a n sfo r m e r s th e l.v . a n d h.v. win^
are sa n d w ic h e d a s sh o w n in F ig . 1.1 ( b ). T h is a r r a n g e m e n t o u s in g s a n w ic ed coils cauSp
m ore c lo se n e s s o f th e p rim a ry an d seco n d a ry w in d in g s a n d th e e a g e u x is, therefore, COn.
sid era b ly red u ced .
In a d d itio n to th e red u ctio n in le a k a g e flu x b y s a n d w ic h in g in s h e ll-ty p e construction, iron
su rro u n d s b o th th e w in d in g s. In v ie w o f th is , th e flu x e s p ro d u ced y p rim a ry and secondary
w in d in g s te n d to fo llo w th e le a st-r e lu c ta n c e iron p a th a ro u n d th e m , r a th e r th a n th e high_reluC-
ta n c e a ir p a th . In t h is m a n n er th e le a k a g e flu x is fu r th e r red u ced a n d th e co efficien t of coupling
is in crea sed .
E x a m p l e 1 .6 1 . A 10 kVA, 2 3 0 0 1 2 3 0 V, s in g le - p h a s e t r a n s f o r m e r h a s the following
pa ra m eters:
r x= 1 0 n , r 2 = 0.10Cl, l x= 40 mH, l2 = 4 x 10~4 H, M ~ io u

S u b sc rip ts 1 a n d 2 indicate h.v. a n d l.v. w in d in g s respectively.


(a) F in d the self-impedances o f p r im a r y a n d secondary w in d in g s.
(b) F in d the values o f the equivalent-circuit p a r a m e te r s referred to (i) the p rim a ry and (ii)
the secondary.
(c) The p r im a r y o f this transform er is energised fro m 2 3 0 0 V, 5 0 H z source. I f its secondary
is connected to a load o f im pedance 5 + j 5 Q, f in d the seco n da ry te r m in a l voltage.
S o lu t io n , (a) P rim ary self-in d u cta n ce,

L x = L ml + l x = ~ M + l x= x 10 + 0 .0 4 = 1 0 0 .0 4 h en ry s.

S econ d ary self-in d u cta n ce,

No l
L 2 = L m2 + l 2 = M j ^ + l 2 = 10 x — + 4 x 1 0 " 4 = 1 .0 0 0 4 H

(6) P a ra m eters referred to p rim a ry w in d in g a re :

r ^ lO ft; r 2 = 0. l x = o . i x ( 1 0 ) 2 = 10 n

l x = 40 m H ; Z2 = 4 x l 0 ~ 4 = 4 x 1 0 “ 4 (1 0 )2 = 4 0 m H .
N2j
v y
^m l “ M - j —10 X 10 = 100 H.
N.

P a r a m e te r s referred to seco n d a ry w in d in g a r e :

r x = 10 x ' i f = 0 .1 0 1 2 ; r 2 = 0.1 Q
10
\ y
\2
^! = 4 0 x 10" 3
10 = 4x10 4H ; Z2 = 4 x 1 0 " 4 H .
v y
.N .
L m2 = Af ^ = 10 x ~ = i.o o H.

by
Transformers 121
^ e q u i v a l e n t circu it referred to seco n d a ry is a s sh ow n in Fig. 1 6 9 (o)
(L o Hr ^ p p ly . * • e q u iv a le n t c ircu it o f F ig. 1.6 9 („ ) g ets m odifled tQ ^ sh o w n fa
fo r

, 69 (W*
Xj = 2 n f l 1 = 2n (50) (4 x 10 4) = 0 .1 2 5 6 Q
x 2 = 0 .1 2 5 6 Q.
X m2 = 2n f L = 2n (50) (1.00) = 3 1 4 ft.

0425611

o— w w — —v M
W—o
0-1 n o-i a

o)
C l
O 1-00H 230 V
o
o
o

(a)
F ig . 1 .6 9 . E q u i v a l e n t c ir c u it r e f e r r e d to s e c o n d a r y , E x a m p l e 1 .6 1 .

Hie current flow in g th ro u g h X m2 is m u ch sm a lle r a s com pared to th a t in 5.1 ♦ , 5 1 2 5 6 a


connected in parallel w ith it.
/. Secondary term in a l v o lta g e
230
5 . 2 + j 5 . 2 5 1 2 X (5 + -/ 5 )
_ 230 i---- n-------- w-

" ^ ( 5 .2 ) z + ( 5 .2 5 1 2 ) 2 X V (5 ) X ( 5 > = 220 7 V -

S a m p le 1.62. A single-phase tw o w in d in g transform er gave the follow i

tnreni of 0 .3 5 A a n d in d u ced * ?5 f * " energised f r o m V. SO H z supp


tii)L V ' m f' across open circuited l.v. w in d in g is 110 V

^ A a tT ju c e d Y m f nr" ^ V’ " " * » * to *CS » our-


Calculate ( a cro ss open circuited h.v. w in d in g is 2 2 6 V.

w t n d T r ! i s V ) ^ o u n l Z ° f \ V' T * ^ W ^ mUtUal in du ctan ce betw een


^ f f le ie n t o f c m p lin g ™ g f 1 * 2 for h v ' a n d l v ' E n d i n g s respectively a n d

** 1core loss a n d w in d in g resistances.

^ o n . Self-inductance L = ^
Now *
Hemy E = yl2Kf ^ m a = ^ 2 n f< fmat
(aj . *vi, are m a x im u m flu x -lin k a g e s .

ax*tQUin v a lu e o f f lu x -lin k a g e s w it h h .v . w in d in g

W . .V1 . 230
V V2(x) (50)

^ h c t a n c e o f h .v . w i n d i n g ! , = ^

Scanned by Cam Scanner


122 Electrical Machinery (Art I. 1(

230 1 = 2 .0 9 2 H.
[V2(«) (50) J * R 2 x 0.35
S im ila r ly self-in d u cta n ce o f l.v. w in d in g ,
= 0 .5 0 8 4 H.
= ^ 2 (S M 5 0 j] X
( 6 ) T h e m a x im u m v a lu e o f m u tu a l flu x lin k a g e s
E2
^2n f
' e 2 ") 1 f 110 v r 1 1 = 1 H.
M u tu al in d u cta n ce M =
x ? 2 7 = V2 7t (50) J [V2 (0 .3 5 ) J
/
226
A ltern a tiv e ly , M =
>/2 n x 50 12 ; W ] = i a
Nx M 590
(c) C ou p lin g factor = 0 .9 5 6 .
N2 Lx 295 2.0 9 2
N2 M 295 1
C oup lin g factor = 0 .9 8 3 5
N t L2 590 0 .5 0 8 4
C oefficien t o f cou p lin g k = k 2 = V 0.956 x 0 .9 8 3 5 = 0 .9 6 9 6 .
C heck. M =k = 0 .9 6 9 6 V (2.092) (0 .5 0 8 4 ) = 1 .0 0 H .
E x a m p le 1 .6 3 . The s e lf a n d m u tu a l inductances o f a tw o -w in d in g tr a n s fo r m e r are
L j - 4 mH, L 2 - 6 mH, M 12 = M 21 = 1.8 mH.
Calculate the current w hich w o u ld flow in the w in d in g 1 w h en th is w in d in g is connected to
a 130-volt, (500/7t) H z su p p ly a n d the lo a d o f 0.2 m H in du ctan ce is connected across the winding
2. A ssum e p o w er losses in the w in d in g s a n d the m a g n etic c ircu it to be negligible. (I.E.S., 1982)
S o lu t io n . T h e v o lta g e eq u a tio n for th e p rim a ry w in d in g , in t e r m s o f r m s v a lu e s, can be
ob tain ed from Eq. (1 .8 4 ) as
'V1 = r x 7j + 7 ( 0 L x 7j -jo a A f I2 ...(1.101)
S im ila r ly for th e seco n d a ry w in d in g , from Eq. (1 .8 5 ),
V2 = j w A f/j - j m L 2 12 - r 2 12 ...(1.102)
S u b stitu tio n o f th e v a lu e s in E q s. (1 .1 0 1 ) a n d (1 .1 0 2 ), w it h V x a s r e fe r e n c e p h asor, gives

130 + j 0 = j 2 n ( 5 0 0 /n ) 4 x 10" 3 I x - j 2 k (5 0 0 /7 t) 1.8 x 1 0 ~ .3 12


and j 2 n (5 0 0 /n ) x 0 .2 x 10" 3 12 = j 2 x (5 0 0 /7 i) 1 .8 x 10" 3 I x - j 2n ^ ^ 6 x 10" 3 12
or 130 = 7 I x- j 1 . 8 12
4 ' 71

and 0 - j 1.8/j - j 6 .2 / 2
from above, 1 =1® /
2 6.2 1
Simultaneous solution for I x gives I x = 3 7 .3 8 4 A

1 65^6)SH e rea llh the”r iv e n ™ S° 1V<i? a 't e m a t iv e ly b* ^ ferrm g to the equivalent circuit
( } HCre a" the,glVen Parameters are referred to primary, i.e. take a = 1 in Fig. 165 <»>•
L x - a M = 4 - 1 8 = 2.2 mH
aM = 1.8 mH

Scanned by Cam Scanner


Transformers 123

fl2Z>2 " = 6 - 1 .8 - 4 .2 m H

• d u cta n ce L r efe rred to p r im a r y = a 2L = 0 .2 m H


in d u ctan ce s e e n b y t h e p r im a r y a p p lie d v o lta g e
^ota (aM) [a2L 2 - a M + a 2L]
^ 1 a M + a 2L 2 - a M + a 2L
= 2 2 x 1 0 - 3 + .C l - 8 x l 0 - 3 ) ( 4 . 2 ^ 0 . 2 ) x l 0 ^ = ^ x jq - 3 H
-3
( 1 .8 + 4 . 2 + 0 . 2 ) x 10
Total reactance a t t h e p r im a r y te r m in a ls

= 2n x — x 3 .4 7 7 4 x 1 0 ' 3 = 3 .4 7 7 4 f t

130
. C u r r e n t in th e p r im a r y w in d in g 1 = = 3 7 .3 8 4 A.

1 17. A u d io -F req u en cy T r a n s fo r m e r s
Small iron-core tr a n s fo r m e r u s e d in th e a u d io -freq u en cy ra n g e o f 2 0 to 2 0 ,0 0 0 H z a re c a lle d
udio-frequency tr a n sfo r m e r s. T h e u s e o f th e s e tr a n sfo r m e r s in ele c tr o n ic c ir c u its e m p lo y e d for
communications, m e a s u r e m e n ts a n d co n tro l is q u ite co m m o n for th e p u r p o se o f c o u p lin g load
to the voltage sou rce. P r im a r ily , th e fu n c tio n s o f a u d io -freq u en cy tr a n s fo r m e r s a re, (i ) for s t e p ­
ping up the v o lta g e in a m p lifie r s to o b ta in th e req u ired v o lta g e g a in an d (ii) for d e c r e a s in g or
increasing th e load im p e d a n c e a s s e e n by th e v o lta g e so u rce to a c h ie v e th e im p e d a n c e m a t c h ­
ing They are also e m p lo y ed s o m e tim e s for p ro v id in g a p a th for dc th r o u g h p r im a r y w h ile is o la t ­
ing it from th e seco n d a ry .
For power and d is tr ib u tio n tr a n sfo r m e r s, freq u en cy resp o n se is n o t o f a n y in t e r e s t a s t h e s e
transformers are o p era ted a t a fix e d freq u en cy o f 5 0 H z (or 6 0 H z in so m e c o u n tr ie s o f t h e
world). However, in so m e ele c tr o n ic cir c u its, th e freq u en cy o f so u rce v o lta g e m a y v a r y o v e r a
wide range. For e x a m p le , a n o u tp u t tra n sfo rm e r , w h ic h co u p les th e la s t s ta g e o f a n a u d io ­
amplifier to a lo u d sp e a k e r , m a y h a v e to o p era te ov er th e en tir e a u d io -freq u en cy r a n g e . In su c h
variable frequency a p p lic a tio n s, fr e q u e n c y r e sp o n se o f th e tra n sfo rm er is q u ite im p o r ta n t. T h e
object of th is article is to in v e s t ig a t e t h e effect o f v a r ia b le freq u en cy on t h e a m p litu d e freq u en cy
characteristic (A FC ) an d p h a s e ch a ra cte ristic o f a u d io -freq u en cy tr a n sfo r m e r s. L e t V x b e th e
source voltage on p r im a r y s id e an d V 2 be t h e load v o lta g e on th e s e c o n d a r y s id e o f a t r a n s ­
former. Then for fix ed V l a n d load , a p lo t sh o w in g th e v a r ia tio n o f V2/ V l a s a fu n c tio n o f fre­
quency is called A F C . A f la t fr e q u e n c y r e s p o n s e o v er th e e n tir e a u d io -fr e q u e n c y r a n g e is
desirable. The v a r ia tio n o f th e p h a s e a n g le o f th e load v o lta g e V2 w ith r e s p e c t to th e so u r c e
v°ltage Vj is called th e p h a s e c h a r a c te r is tic . A sm a ll v a lu e o f p h a se a n g le is d e s ir a b le w h ic h

I—vW—'TSTyi

i +

h -S O U R C E — TRANSFORMER

canned by Cam Scanner


(Art. 1.17
124 Electrical Machinery

■WV VW— W r
T
VL
R',
IS*
1 _

(C)
F ig . 1 .7 0 . E q u iv a le n t c ir c u it o f a n a u d io - f r e q u e n c y t r a n s f o r m e r (a ) C o m p le t e e q u i v a l e n t c ir c u it w i t h s o u r c e a n d
l o a d . I t s a p p r o x i m a t i o n a t ( 6 ) l o w f r e q u e n c i e s ( c ) I n t e r m e d i a t e f r e q u e n c i e s a n d ( d) h i g h f r e q u e n c i e s .

m ean s th a t ou tp u t v o lta g e follow s in p u t v o lta g e a s clo sely a s p o ssib le . B o th A F C an d phase


c h a ra cte ristic d ep en d on th e tra n sfo rm er a s w e ll a s on th e p r im a r y a n d s e c o n d a r y circuit
p aram eters.
T he com p lete audio-freq u en cy ra n g e can be d iv id ed in to th r e e p a r ts — lo w freq u en cy (LF)
ran ge, in term e d ia te freq u en cy (IF) ran ge an d h ig h freq u en cy (H F ) r a n g e. F o r o b ta in in g AFC
and p h a se ch aracteristic, con sid er an a m p lifier feed in g a load th r o u g h a c o u p lin g transform er.
T he eq u iv a le n t circu it for th is sy ste m is sh o w n in Fig. 1.70 (a). H e re a m p lifie r is rep resen ted
by sou rce v o lta g e Eg and a serie s r e sista n c e Rg; tra n sfo rm er is r e p r e s e n te d b y t h e eq u ivalen t
circu it o fF ig . 1.14 (6) w ith core-loss r e sista n c e R c n eg le cted . L oad is a s s u m e d to b e a resistan ce
Rl . For LF, IF and H F ra n g es, a sim p le a n a ly sis is a s u n d e r :
(a) L o w f r e q u e n c y . A t low freq u en cies, m a g n e tiz in g r e a c ta n c e X m d e c r e a s e s a n d a s such,
its s h u n tin g effect across th e load im p ed a n ce ca n n o t be ign ored . F or o p e r a tio n in t h e L F range,
X m is a lm o st eq u al to self-rea cta n ce = 2 n fL , o f th e tra n sfo rm er. T h e le a k a g e r e a c ta n c e s x x
and x 2 b ecom e so sm a ll a t th e s e lo w fr e q u e n c ie s th a t th e y m a y b e o m itte d le a d in g to the
eq u iv a le n t circu it o fF ig . 1.70 ( b i ). T h e seco n d a ry q u a n titie s referred to p r im a r y a re indicated
by su p erscrip t p rim es. A p p lyin g T h ev en in ’s th eo rem a t te r m in a ls a, b ; t h e c ir c u it to th e left of
th e s e term in a ls can be reduced a s fo llo w s :
Eg j X i
V oltage a t term in a ls a, b = Vab =
Rg + r 1 + j X l
j X i (R + rj)
Im pedan ce a t term in a ls a, b = Z ab =
Re + r x + j X x
The T h ev en in ’s e q u iv a le n t circu it is sh o w n in F ig . 1 .7 0 (bii). F ro m t h is c ir c u it,
Vab ' R L Eg ■jX 1 R l ' 1
Vl =
Zab + r 2 + R l ’ R g + ri + jX { ’ j X l {R +
ri)
Re + r, + jX l + ^ + ^
Its sim p lifica tio n g iv e s,

JX, R l
j X x (Rg + r l + r 2' + R l ’) + ^ + R l ') (Rg + r])

V
____________ R l
D , , (r2 + * L #) (R g + ' ’l)
* s + 7 ^ ------------

Now V t' = S V i

V, No R r'
...(1.103)
Eg N, 2 -i1/2
( r pO
1+
\x '\
R s' = R g + r i + r 2 + r l .(1 .1 0 4 )
Here
, _ (Rg + rj) (r2' + R L') .(1 .1 0 5 )
and p (R g + r x + r 2’ + R L’)
E x p r e s s io n for V L' a b o v e r e v e a ls t h a t lo a d v o lt a g e le a d s th e so u r c e v o lta g e by a p h a s e a n g le

given by
e -ta n -1^ ...(1 .1 0 6 )
<*1
ih) I n t e r m e d i a t e f r e q u e n c y . A t in t e r m e d ia t e fr e q u e n c ie s (a ro u n d 5 0 0 H zf, th e d e s ig n o f
a u d io -freq u en cy t r a n s f o r m e r s is s u c h t h a t a n d x 2 a re q u ite sm a ll and X m is s u ffic ie n tly la rg e.

Therefore c u r r e n t th r o u g h X m is n e g lig ib le a n d a s a co n seq u en ce x h x 2 a n d X can b e ignored.


T h e " n t T r u i t u n d e r t h e s e c o n d it io n s is t h a t s h o w n in F ig . 1 .7 0 (c). A n a ly s is o f th is

series circuit gives V L/ E g as

VL' Rl _ - R jL
Ej,g. R. g + r i + +Ri Rj
R [' ...(1 .1 0 7 )
• L, ~ * __ *£_
or
Eg Ni r;
* ’ i nver th e b an d o f in te r m e d ia te fre-
The v o lta g e r a t io V L/ E g r e m a in s v e r y n e a r ly c o n s t a n t o

quencies' • v a n d r can no lo n g er be n e g le c te d . T h e se
(c) H ig h f r e q u e n c y . A t v e r y h ig h f r e q u e n c ie s , i 2 g tu d ies
Should, th erefo re, b e in c lu d e d t o le a d to t h e e q m v a le n t c ir c u it o f F ig . X. >
Analysis o f t h is c ir c u it g iv e s V L/ E g a s
o ' Rl
VL' r l = tr r r z r
~E~g = R g + r , + r 2' + R l + Jx “i R * +JXfq
...(1 .1 0 8 )
or VL N2 RS 1
2 -|
Eg ^ R s'
1+
-
where

P h a se a n g le b y w h ic h lo a d v o lt a g e la g s t h e so u r c e S
...(1 .1 0 9 )
- 1 ^*2.
0 = ta n
R sf

Scanned by Cam Scanner



(Art 117 if
126 i f i aa cu hi i in i ev ri yj _ _ _ _ _ _ _ _ _ _ _ _ _ - —
LE lieac ut r i c a l M — — fi

T h e v o lta g e ra tio s oi r,q». v*-*''-' „ q


_______
in te r m e d ia te freq u en cy v „o lta
H ,„g en ratio given by
ratio given y bq. u .* “ "

R ela tiv e v o lta g e ratio for L F = ^ J Y + \ ^ 7 x ^


1 _
R ela tiv e v o lta g e ratio for H F = ^ + (X / R / y
c U tin (V / E ) as a fu n ction o f freq u en cy is sh o w n in Fig. i . 7l 0n #
T h e v a ria tio n o f v o lta g e ™ tl0 <> (1 .1 0 3 ) t h a t a s al in c r e a se s, (Rp' / X t) decrease
lo g a r ith m ic freq u en cy " wi t h freq u en cy in ’ ran ge^ F or I f range, E,
an d as a r e su lt, V j /E progres » in d ep en d en t o f freq u en cy . F or H F range, Eq. (1.108)
ie d i r e c t proportional to co, th erefo re, Vt / E , decreases accord­

in g as th e freq u en cy in crea ses in HF range.

Fig. 1.71. F req u en cy resp o n se o f an a u d io -freq u en cy tr a n sfo rm e r .

T he v a riatio n o f r ela tiv e v o lta g e ratio w ith freq u en cy is n o t sh o w n The. potato at vwhich
r ela tiv e vo lta g e ratio becom es eq u al to 1/V2 are ca lled h alf-pow er p oin ts. T h e L F range g i
low er h alf-p ow er p oin ts. T herefore, from Eq. (1 .1 1 0 ),
1 1
- 2 -i 1/2 1 2
(R '1
1+
*1
V

or
...(1.11®
or 2 kL,
T h e H F ran g e g iv e s u p p er h alf-p ow er p o in ts. T h erefo re, from Eq. (1 .1 1 1 ),
1 1
V l+ ( * ,/* /) * ^2
I or fh = 2Rid.:
. . . u . u 3)
"i
T h e b a n d w id th o f an a u d io-freq u en cy tra n sfo rm er is g iv e n b y th e ra tio
fh _ L i ...<i.i141

fl l*> R P ,ldM ,e
T h is ex p ressio n rev e a ls th a t for a ch ie v in g a w id e b a n d w id th , th e tra n sfo rm er ®^oUon(j9ry
la r g e s e lf-in d u c ta n c e an d low le a k a g e in d u cta n ce. In o th e r w o rd s, th e p rim a ry an te11^
w in d in g s sh o u ld be a s clo sely coupled a s p o ssib le, i.e. th e ir c o e ffic ie n t o f co u p lin g sho
to u n ity .

. . . .■■■ - ■. • • ■ ■■ - • • • • ••• - T . v l .

Scanned by C am Scanner
r Transformers 127
.f(, i^ i_______
m p le i4n au(R °-fre q u e n c y tra n s fo r m e r couples a 60-ohm r e s is tiv e l o a d to an
ir circuit w hich is rep re se n te d by a c o n s ta n t voltage source o f 5 V in series w ith an inter-

‘C « ° " « o f 3 0 0 0 a .
' / ) Determine the tr a n s fo r m e r tu r n s ratio so th a t m a x im u m p o w e r tr a n sfe r ta k e s p la c e fro m

J e to th e lo a d '
1 (b) the l0a d current> volta8 e a n d p o w e r u n d e r the c o n d itio n s o f m a x im u m p o w e r tra n s-

J Solution. (a) For m a x im u m p o w er tr a n sfe r , th e load r e s is ta n c e o f 6 0 f t w h e n referred to


primary side m u st be e q u a l to th e so u rce r e s is ta n c e o f 3 0 0 0 ft.
the p
6 0 IL
-'VVAV
3000 = (* 1 x 60
No
Ni r=—
^ = V50 = 7 .0 7 1
or

(b) R e fe r r in g a l l t h e q u a n titie s to lo a d s id e , th e .
equivalent circuit is a s s h o w n in F ig . 1.72. T h e sou rce
voltage on load sid e is (5 /7 .0 7 1 ) V an d th e so u rce resis- Flg 1 72 Pertaining 10 Example 1.64
tance is 60 ft.
5
• Load current, IL = —Q?1 ^ = 5 .8 9 3 m A

Load voltage, VL = 5 .8 9 3 x 10~3 x 6 0 = 0 .3 5 3 6 V


Load power = ^ l r l = (5 .8 9 3 x 10-3)2 x 60 = 2 .0 8 4 mW.

Example 1.65. A n audio-frequ en cy transform er has the follow ing p a r a m e te r s :


r, = 20 ft, lj = 1 m H , R 2 = 0.5 ft, l2 = 0.025 mH, M 12 = M 21 = 0.2 H
Iron losses are neglected.

i J o % £ anSf0rmer C° UPleS a l° a d °f 5 ° n t ° a V° ltage $0UrCe °f 5 V w h o se in tern a l resistance

fa) Find the turns ra tio for m a x im u m p o w e r transfer to load.


V Compute the lo a d voltage a t the follow in g frequencies :
<l) 100 Hz (LF), d i) 5 0 0 0 H z (IF) a n d (Hi) 15,000 H z (HF).
°lution. (a) For m a x im u m p o w er tra n sfer
I T \ 2

2000 = x 50
n 2
J
N
~ = V40 = 6 .3 2 4 5
(6) f • .. N2
i ~ 0.5 (40) = 20 ft, l2' = (0 .0 2 5 ) (40) = 1 m H,
• L * (5°) (40) = 2 0 0 0 ft, R g = 2 0 0 0 ft
R / = R g + r , + r2 + R l ' = 4 0 4 0 ft
( 2 0 2 0 ) (2 0 2 0 ) = 1 Q 1 0 fl

4040
12 = ^21 = 0.2 H, leq= l l + l 2' = 2 m H .

Scanned by Cam Scanner


128 Electrical Machinery J Arl 1.18
(i) A t 1 0 0 H z (L F ), from Eq. (1 .1 0 3 ),
VL i 2000 = 0 .0 0 9 6 6 4 3 2
1/2
Eg ^40 '
1010
4040 1+
2n x 100 x 0.2
V, = 0 .0 4 8 3 2 V.
(ii) A t 5 0 0 0 H z (IF ), from Eq. (1 .1 0 7 ),

Yk _ 1 . = 0 .0 7 8 2 7 4 2
Eg v40 4040
V i = 0 .3 9 1 4 V.
(iii) A t 1 5 ,0 0 0 H z (H F ), from Eq. (1 .1 0 8 ),
VV 1 2000 = 0 .0 7 8 2 5 2 9
1/2
Eg ^40 4 0 4 0 ' 3A
r2n x 1 5 .0 0 0 x 1 x 10
1+
4040

VL = 0 .3 9 1 3 V.

1 .1 8 . P u l s e T r a n s f o r m e r s
T h e tra n sfo rm ers w h ich h a n d le v o lta g e s and cu rren ts in th e form o f p u ls e s are called pulse
tra n sfo rm ers. T h e u s e o f su ch tra n sfo rm ers is q u ite com m on in rad ar, te le v is io n , d igital com­
p u te r an d th y risto r sy ste m s. T h e fu n ctio n s o f p u ls e tra n sfo rm ers are :
(i) for ch a n g in g th e a m p litu d e o f a v o lta g e p u lse,
(ii) for in v e r tin g th e p o la rity o f a p u lse,
(iii) for a ffectin g dc iso la tio n b e tw e e n sou rce a n d load ,
(i v ) for co u p lin g d ifferen t s ta g e s o f p u lse a m p lifiers.
T h e in p u t v o lta g e to p u lse tra n sfo rm ers is o f d isc o n tin u o u s n a tu r e a s sh o w n in Fig. 1.73 (a).
T h e m o s t im p o r ta n t r eq u ir em en t o f th e s e tr a n sfo r m e r s is t h a t in p u t p u ls e a t th e primary
s h o u ld b e rep rod u ced as a ccu ra tely as p o ssib le a t it s seco n d a ry te r m in a ls. F ig. 1.73 (6) shows a
s q u a r e w a v e p u ls e a t t h e in p u t t e r m in a ls . T h e p u ls e w id t h v a r ie s from a fra ctio n of a
m icrosecon d to a b o u t 25 m icrosecon d s. U su a lly , a su ffic ie n t tim e e la p s e s b efore th e n ex t pulse
a p p ea rs. T h e w a v efo rm o f th e o u tp u t v o lta g e a t th e seco n d a ry te r m in a ls ca n be determined
th r o u g h th e u s e o f e q u iv a le n t circu it o f th e p u lse tra n sfo rm er.
FLAT TOP
I
jL .
Ui FRONT
e> O FF PERIOD
< /E D G E


—t
o
>

3
. /
Q.
2

ON UP­ TIM E TIME


— PERIOO

(a) (b)
F i g . 1 .7 3 . ( a ) T r a i n o f i n p u t - v o l t a g e p u l s e s a n d (b) S q u a r e - w a v e i n p u t v o l t a g e p u l s e .

A ty p ic a l o u tp u t v o lta g e p u ls e is sh o w n in F ig . 1 .7 4 , w h e r e p u ls e tr a n sfo r m e r p a r a m e t e r s
a re a lso d e fin e d . T h e r ise tim e is th e tim e in te r v a l req u ired for t h e o u tp u t to r is e from 0.1 to ■

Scanned by Cam Scanner


Fig. 1.74. W aveform of o u tp u t voltage pulses for a pulse transform er.

of its final v a lu e . T h e d is t o r t io n s p r e s e n t in t h e o u tp u t p u ls e c a n b e d e t e r m in e d t h r o u g h th e
tra n sie n t a n a ly s is o f i t s e q u iv a le n t c ir c u it.
The tr a n sfo r m e r a n a ly s is is u s u a l ly c a r r ie d o u t b y d iv id in g it s s o lu tio n in to t h r e e p a r ts . T h e
first part g iv e s t h e r e s p o n s e n e a r t h e fr o n t e d g e o f t h e p u ls e , t h e s e c o n d p a r t g iv e s t h e r e s p o n s e
during th e fla t-to p a n d t h e th ir d p a r t g iv e s t h e r e s p o n s e a fte r t h e t e r m in a t io n o f t h e p u ls e .
For le a d in g e d g e o f t h e in p u t p u ls e a n a ly s is o f t h e e q u iv a le n t c ir c u it is d o n e b y in c lu d in g
stray ca p a cita n ce. In o r d e r to k e e p t h e r is e t im e w it h in lim it s , t h e le a k a g e in d u c t a n c e o f t h e
transformer s h o u ld b e k e p t to a m in im u m .
The tr a n s fo r m e r r e s p o n s e to t h e f la t-to p p o r tio n o f t h e in p u t p u ls e is c a r r ie d o u t b y u s in g
the lo w -fr e q u e n c y e q u i v a l e n t c ir c u it o f F ig . 1 .7 0 (6 ). T h e o u t p u t v o lt a g e i s s e e n to h a v e
downward tilt, or d ro p -o ff, d u r in g it s p u ls e d u r a tio n t im e . T h e o u tp u t v o lt a g e c a n n o t r e m a in
flat as th is w o u ld m e a n d c p a s s i n g t h r o u g h a tr a n s fo r m e r w h ic h is n o t p o s s ib le . T h e d r o p -o ff o f
the pulse ca n b e k e p t a s s m a ll a s p o s s ib le b y h a v in g h ig h m a g n e t iz in g in d u c ta n c e fo r t h e t r a n s ­
former.
When t h e in p u t p u ls e i s z e r o , t h e o u t p u t p u ls e d o e s n o t r e d u c e to ze r o i n s t a n t a n e o u s ly
because o f th e m a g n e t ic e n e r g y s to r e d in t h e tr a n s fo r m e r in d u c ta n c e . T h e f a ll, or d e c a y , t im e
for the o u tp u t p u ls e i s s h o w n in F ig . 1 .7 4 . T h e r e is a b a c k s w in g o f t h e o u tp u t v o lt a g e a n d
because o f tr a n s fo r m e r in d u c t a n c e a n d s t r a y c a p a c ita n c e , d a m p e d o s c illa t io n s a n d a lo n g -d u r a -
tion n egative o v e r s h o o t a r e o b s e r v e d a fte r t h e d e c a y t im e o f t h e p u ls e .
Pulse tr a n s fo r m e r s a r e q u it e s m a ll in s iz e . B o th p r im a r y a n d s e c o n d a r y w in d in g s h a v e
comparatively fe w t u r n s s o t h a t le a k a g e in d u c t a n c e is m in im u m . In o rd er t h a t tr a n s f o r m e r h a s
maximum m a g n e t iz in g in d u c t a n c e , it s co re is m a d e o f f e r r it e s or fro m h ig h p e r m e a b ilit y a llo y s
^ ch as p e r m a llo y . A s t h e o ff-p e r io d ( t im e in t e r v a l b e tw e e n s u c c e s s iv e p u ls e s ) is u s u a l ly q u it e
Iarge as co m p a red w it h o n -p e r io d , t h e t r a n s fo r m e r c a n h a n d le h ig h p u ls e -p o w e r le v e ls .

1*19. Three-ph ase T r a n s f o r m e r s


G eneration , t r a n s m is s i o n a n d d is t r ib u t io n o f e le c tr ic e n e r g y is in v a r ia b ly d o n e t h r o u g h t h e
Use of th ree-p h a se* s y s t e m s b e c a u s e o f it s s e v e r a l a d v a n t a g e s o v e r s in g le - p h a s e s y s t e m s . A s
Uc^> u large n u m b e r o f t h r e e - p h a s e tr a n s f o r m e r s a r e in d u c te d in a 3 - p h a s e e n e r g y s y s t e m for
j^Pping-up or s t e p p in g - d o w n t h e v o lt a g e a s r e q u ir e d . F o r 3 - p h a s e u p or d o w n t r a n s f o r m a t io n ,
£ree u n its o f 1 -p h a s e t r a n s f o r m e r s o r o n e u n it o f 3 - p h a s e t r a n s fo r m e r m a y b e u s e d . W h e n
cr ide n tic a l u n it s o f 1 - p h a s e t r a n s f o r m e r s a r e u s e d , F ig . 1 .7 5 (a ), t h e a r r a n g e m e n t is u s u a lly
^ g j a b a n k o f t h r e e t r a n s fo r m e r s o r a 3 - p h a s e t r a n s f o r m e r b a n k . A s in g le 3 - p h a s e t r a n s ­

Scanned by C am Scanner
(Art. 1.19 ■A:i;
130 Electrical Machinery , ■.
!:■
•■ '
i
IN.PUT

OUTPUT Cb)
F ig . 1 .7 5 . (a ) T h r e e - p h a s e t r a n s f o r m e r b a n k , b o th w i n d in g s in s t a r (6 ) t h r e e - p h a s e c o r e - t y p e t r a n s f o r m e r .

form er u n it m a y em p loy 3 -p h a se core-type co n stru ctio n , F ig . 1 .15 ( b ) or 3 -p h a s e s h e ll-ty p e con­


struction (n ot show n). A sin g le-u n it 3 -p h a se core-typ e tr a n sfo rm er u s e s a th r e e -lim b e d core,
one lim b for each p h a se w in d in g as sh o w n in Fig. 1.75 (b). A c tu a lly , e a c h lim b h a s th e l.v.
w in d in g placed adjacent to th e la m in a ted s te e l core an d th e n h .v . w in d in g is p la ce d over th e l.v.
w in d in g. A ppropriate in su la tio n is p laced in b etw een th e core a n d l.v . w in d in g and also in
b etw een th e tw o w in d in gs.
A 3-p h ase core-type tran sform er costs ab ou t 15% le s s th a n a b a n k o f th r e e 1 -p h a se trans­
form ers. A lso, a sin g le u n it occupies le s s sp a ce th a n a bank.
1. 19 . 1. T h ree-p h a se tra n sfo rm er c o n n e c tio n s
T h ree-phase tran sform ers m ay h a v e th e fo llo w in g four sta n d a rd c o n n e c tio n s :
(a) star-d elta ( 7 - A) (b) d e lta -sta r (A - Y)
(c) d elta-d elta (A - A) (<d ) sta r -sta r ( Y - Y)
T h ese connections are sh ow n in F igs. 1.76 and 1.77, w h e r e V a n d I a re ta k e n a s in p u t line
voltage and lin e current resp ectively. P rim ary and seco n d a ry w in d in g s o f o n e p h a s e are drawn
p arallel to each other. W ith p h a se tu rn s ratio from p rim a ry to s e c o n d a r y a s N x/ N 2 = a, the
voltages and currents in th e w in d in g s and lin e s are sh o w n in F ig s. 1 .7 6 a n d 1 .7 7 . T h e various
connections are now described briefly.

(a) S t a r - d e lt a (Yd) c o n n e c t io n . T h is con n ection is co m m o n ly u s e d for s te p p in g down the


voltage from a h igh lev el to a m ed iu m or low lev el. T h e in s u la tio n on th e h .v . s id e o f th e trans-
1
form er is stressed on ly to 57.74% = -j=- x 100 o f lin e to lin e v o lta g e.

For p er-p h ase m .m .f. balance, I2 N 2 = I x N x


H ere prim ary p h a se current, I x = p rim ary lin e cu rren t I
r N ,
Secondary p h a se current,

S econdary lin e current = V3 I 2 = V3" • a l


A lso, voltage per tu rn on prim ary = v o lta g e per tu rn on seco n d a ry
V _1_ Y l
73 ' N x No.

For more d eta il of3-phase transformers, see chapter 8 o f t h p h nn lr .. ..


b y th e sa m e a u th o r. ' ^ e n e r a liz e d T h e o r y o f E le c tr ic a l M a c h in e s

by Cam Scanner
Art. 1.19]
Transformers 131

S e c o n d a ry p h a s e v o lta g e , V , = - Z • i - v
Ni H 'T T H S

S e c o n d a ry line v o lta g e = s e c o n d a ry p h a s e voltage ____


V ° ' V3
Input VA = 3 ^ I = o u tp u t VA = 3 . V3 a l = V3 V7

Phase and lin e v a lu e s for v o l t a ^ a. . ,


star-delta tra n sfo rm er a re sh o w n in Fig. 1.76 (a) * °" pr‘m ary an d seco n d a ry sid e s o f

^ . « ...

Fig. 1.76. (v) Star-delta connection and (6, delta-sta, connection of 3-phase transformers.

to a high level. F o r ^ m p ^ th e s e a 'r e ^usedTn IhfblJri*'™ ‘Sf h " 1 stepping upthe volta8e
insulation is stressed to about 57.74% of line vo ltage^ "'111'* ° V' transmisslon llnes s0 that

■ Dc!ta -Sta,; t r a n s fprm era f e a lso g en era lly u sed a s d istrib u tio n tra n sfo rm ers for p rovid in g
mixed lin e to h n e v o lta g e to h ig h -p o w er eq u ip m en t an d lin e to n e u tr a U o lta g e to^1-phase fo w
power eq u ip m en t. F or e x a m p le , 11 k V /400V , d elta -sta r d istrib u tio n tra n sfo rm er is u se d to Z "

x r . i b y 3 'PhaSe l ° ™ s y ste m - T h ree' ph asa h ig h ™ „ Z er e ,u ip m e n t


231 “ o lu ^ r a u S WlreS' I_PhaSe l0W'P° W eret>UipmCTt is e n e r g ise d from

For p er-p h ase m .m .f. b a la n ce, 1 ^ 2 = I XN x

Here p rim ary p h a se cu rren t, h = ^ (prim ary lin e c u r r e n t/)

Secondary p h a s e cu rre n t, I 9 = —^ L = a -4 -
V3

Also, Yi
N2 Ni

Secondary phase voltage, v 23


2 N, a

Secondary line voltage - V3"- —


a
V
toput VA = 3 V . i = i / 3 V / = O u tp i-t VA = 3 o ^ = V3-W
a
3 , ^ ase a n d lin e v a lu e s for v o lta g e s a.’id cu rren ts on p rim a ry a s w e ll a s seco n d a ry s id e s o f a
ase d e lta -sta r tr a n s fo r m e r a re sh ow ;, in F ig. 1.76 (b).
form ^ e ^ a *^€ ^ a ( D d ) c o n n e c t i o n . T h is sc h e m e o f co n n ectio n s is u se d for la r g e l.v. tr a n s-
per ?rs< ^ is b e c a u s e a d e lta -c o n n e c te d w in d in g h a n d le s lin e v o lta g e, so it req u ires m ore tu r n s
. Phase b u t o f s m a lle r c r o s s -s e c tio n a l area. T h e a b sen ce o f s ta r p o in t m a y be a d is a d v a n ta g e
n s°me ap p lic a t i o n s

Scanned by Cam Scanner


JArt l .l9
132 Electrical Machinery
.f
_ . i 4-Upn one transformer can be removed for main-
In c a s e a b a n k o f th r e e tra n sfo r m e r s is u s , , n d ^ 0p e n -d e lta or V - con n ection ) can J;
te n a n c e p u r p o se s w h ile th e r e m a in in g tw o t [ a n sfo rm “ S„ X , e tr a n s fo r m e r b a n k . an
s till d e liv e r 58% o f th e p o w er d e liv e r ed by t h e o rig in a l 3 -p h a s e tr
F o r p e r -p h a s e m .m .f. b a la n c e , /2 N 2 —l\ N\
(p r im a r y lin e c u r r e n t I )
P r im a r y p h a s e c u r r e n t,
Nx . al
S e c o n d a r y p h a s e c u r r e n t,
al
= 73 = al
S e c o n d a r y lin e c u r r e n t 73

Y i= Y }.
A lso n 2 n x

( H e r e V j = V)
S e c o n d a r y p h a s e v o lta g e , a
V
S e c o n d a r y lin e v o lta g e , = V o= —
i a

VA = 3 V o u tp u t V A = 3 • - ■ 7 3 “V 7
In p u t
P h a s e a n d lin e v a lu e s for v o lta g e s a n d c u r r e n ts on b o th p r im a r y a n d s e c o n d a r y sid es of a
3 -p h a s e d e lta -d e lta tr a n sfo r m e r are s h o w n in F ig . 1 .7 7 (a).

(a) (b)
F ig . 1 .7 7 . (a ) D e lt a - d e lt a c o n n e c t io n a n d (6 ) S t a r - s t a r c o n n e c t io n o f t h r e e - p h a s e t r a n s f o r m e r s .

(d) S ta r-sta r (Y y) c o n n e c tio n . T h is c o n n e c tio n is u s e d fo r s m a ll h .v . tra n sfo rm ers. As


s ta te d b efore, w it h s ta r c o n n e c tio n , tu r n s p e r p h a s e a r e m in im u m a n d t h e w in d in g insulation
is s tr e s s e d to 57.74% o f lin e v o lta g e . S ta r -s ta r c o n n e c tio n i s r a r e ly u s e d in p r a c tic e because of
o s c illa to r y n e u tr a l p roblem s.*
F or p e r -p h a s e m .m .f. b a la n c e , I 2 N 2 = I x N x
P r im a r y p h a s e c u r r en t, h = p r im a r y lin e c u r r e n t, I
Nx
S e c o n d a r y p h a s e c u r r en t,
2 = a^ / i = o /
= s e c o n d a r y lin e c u r r e n t

S e c o n d a r y p h a s e v o lta g e , V -& V V

S e c o n d a r y lin e v o lta g e = V3 V2 = ^ 3 - - S - V
73 • a a
In p u t V
7 A = 3 'T 3 / = ° u t Pu t VA = 3 - ^ — a / = 73 V/

* For more details, see Chapter 8 of the h n n lt j n . „


. neralized theory of Electrical Machines" by the same a

S
am sca n ner
Art. 1.19]
Transformers 133
As before, p h a se and lin e v a lu es of voltages and
transform er are sh o w n in Fig. 1 . 7 7 (6 ) currents on both the sides of a star-star

E x a m p le 1 .6 6 . A 3-phase transform er is used m w t ,


feeder line. Per-phase turns ratio is 12 Fnr step-down the voltage o f a 3-phase, 11 kV
dary line voltage, line current a n d outnut current of 20 A, calculate the secon-
(a) star-delta (6 , d e lta -s ta r (c) delta den f f * :
. ' . I (C) d M a-dclta W> star-star. Neglect losses.
So . ( ) T h ree-p h a se transform er w ith star-delta connection is shown in Fig 1 78
(a).

I l 1=20A
I L i =20A 12:1

vPL=iio o o v T _ ii
^=nooov rlip,
l

(a )
F ig . 1 .7 8 . P e r t a i n i n g to E x a m p le 1 .6 6 .

Phase v o lta g e on prim ary, Vpl = Y j^ = -1^ ° V

Phase current on prim ary, Ipl = IL1 = 20 A


l
V p 1 V o2
Here and Ipl x 12 = /p2x l

11000
P h ase v o lta g e on secon d ary, V 52925v

Line vo lta g e on secon d ary, VL2 = Vp2 = 529.25 V


Phase current on seco n d a ry , Ip2 = 12 Ipl = 1 2 x 2 0 = 240 A
Line current on seco n d a ry , I n = 73 " /p 2 = 73" x 240 = 415.68 A

3 Vp2 11000
O u tp u t k V A = — p2 'I ‘ = 3 x 240 x = 381.04 kVA.
73~x 12 1000
(6) D elta -sta r co n n ectio n o f 3 -p h a se transform er is shown in Fig. 1.78 (6).

y lY£e± - Y m = 11000 V = 916.67 V


Phase voltage on secondary, Pi 12 12 12

V I 2 = V 3V„2 = V 3 x i ^ = 1 5 8 7 .6 7 V
Line voltage on secondary, 12
20
Phase current on primary,

/ p2= 1 2 / f l = 1 2 x ^ = 138-5 6 8 A
Phase current on secondary,

Line current on secondary, /t 2 = /,2 = 1 3 8 5 6 8 A


11000 _12x_20 _ 381 0 4 k V A .
O u tp u t k V A = 3 x 2
^ ~~ I000xv3

Scanned by Cam Scanner


134 Electrical Machinery

(c) D e lta -d e lta co n n ectio n o f 3 -p h a se tran sform er is sh ow n in F ig. 1 ./ 8 (c)


V p\ _ V l i _ 1 1 0 0 0 y _ g 3 5 i v
P h a s e v o lta g e on secon d ary, Vp2 = 12 _ 12 - 12

L in e v o lta g e on secon d ary, VL2 = Vp2 = 6351 V


• r 20 A
P h a s e cu rren t on prim ary, l p\ = ^ a

2 0 A
P h a s e cu rren t on secon d ary, Ip2 = 12 7pl- 12 x A

r- _ pr- 12 x 20 _ .
L in e cu rren t on secon d ary,

O u tp u t kVA = 3 x X = 3 8 1 .0 4 kVA.

l L, =20A I . . = 20A 12:1


12:1 *12 L2

F ig . 1 .7 8 . P e r t a in in g to E x a m p le l.b b .

(d) 3 -p h ase tran sform er w ith sta r-sta r connection is sh ow n in Fig. 1.78 (c/).
. „ vDl 110 0 0 T7
P h a se v o lta g e on secondary, V p2 = 12 = V3~x 12

„ r r 11000 11000 v
L in e v o lta g e on secondary, V L2 = vd Vp2 = S 6 12 “ 12

P h a se cu rren t on prim ary, 7pl = 7L 1 = 20 A


P h a se cu rren t on secondary, Ip2 = 12 7pl = 12 x 20 = 240 A
L in e cu rren t on secondary, IL2 = Ip2 = 240 A

O u tp u tk v A = ^ = 3 8 1 .0 4 kVA.

E xam ple 1.67. A 110001415 V, d elta -sta r tra n sfo rm er fe e d s p o w e r to a 30 kW, 415>V.
p h a s e in d u c tio n m o to r h a v in g an efficien cy o f 90% a n d fu ll-lo a d p f 0.833. C alculate t e r
f o r m e r r a tin g a n d p h a s e a n d lin e cu rren ts on both h igh a n d low v olta g e sides.
30
S o lu tio n . T ran sform er kVA ra tin g = Q 9 x Q 3 3 3 = 40 kVA

, ., rj. * T otal load in VA 4 0 ,0 0 0 _ A


L in e cu rren t on l.v. sid e o f tran sform er = ^ x H n e v o lta = " 5 5 6 5 A
1y.
F or sta r-co n n ected l.v. w in d in g, p h a se cu rren t in l.v. w in d in g = lin e c u r r e n t 0 •
= 5 5 .6 5 A

L in e cu rre n t on h .v. sid e o f tran sform er = = 2 ,1 A

A
Scanned by Cam Scanner
1.20] T ra n sfo rm e rs 135

For d e lta -c o n n e c te d h .v . w in d in g ,

phase curren t in h .v . w in d in g , = ^ (lin e c u r r e n t on h .v . s id e )

: ^ x 2.1 = 1.2 1 2 A.

E x a m p le 1 .6 8 . A n in d u s tr ia l lo a d takes 100 A a t 0.8 p f la g from a 3-ph ase 1 1 0 0 0 / 4 0 0 V, 5 0


Hz, star-delta transform er. C alculate (a) p o w e r consum ed by lo a d ( b ) kV A r a tin g o f tr a n s fo r m e r
(C)phase and line currents on both h.v. a n d l.v. sides.
S o lu tio n , (a) P o w er co n su m ed by load = y [3 V ,I, cos 02

= V3 x 4 0 0 x 100 x 0.8 x = 5 5 .4 2 4 kW .

({,) kVA ra tin g o f tra n sfo rm er = V3 x 4 0 0 x 100 x = 6 9 .2 8 kVA.

(c) Phase cu rren t or lin e cu rren t on sta r-co n n ected h .v. sid e
6 9 .2 8 „
= 7 r n r =3636A
Line current on l.v. seco n d a ry sid e It = 100 A

Phase current on d elta -co n n ected secon d ary sid e = ^ = 57 73 A

1.20. T r a n s fo r m e r N o i s e

££££££ T fr magneticcoreprodu“

ere vibrates t d h i " i ? T ^ T ‘J * d im en sio n s O r n a t e l y , th e f e r r o m a g S c

f t t s a S .'K S a t - d= ,ir .z *
(ii TV e o th e factors p rod u cin g th e n o ise in tran sform ers are th e fo llo w in g :
(«) Th ; “l Cf USe o f h u m ’ and th erefore th e n o ise, is th e m a g n eto strictio n .
"ess°fclam n?„t . t ° f ^
c ore co n stru ctio n , s iz e and g a u g e of la m in a tio n s and th e d e g r e e o f tig h t

Wd(W) J ‘ ^o'lA h a n sfo r m e r s “ “ fre<)UenCy ° f m ech a n ica l « b r a tfo n s

M ta to fth ." th e COre are a ls ° r e s P °n sib le for n o ise Production th ou gh to a le s s e r d eg ree


(a) 1 . n o ise e m issio n from a tran sform er m ay be reduced
(h h USlng *ow v a lu e o f flu x d e n sity in th e core,
^ proPer tig h te n in g o f th e core by clam p s, bolts etc.

[ ^ e r s or U latiflg th e tra n sfo rm er core from th e ta n k w a ll in c a se o f la rg e tr a n s


th e tran sform er core from w h ere it is in sta lle d in c a se o f s m a ll

^ h s io n a 1
■flK* by 1.2 x 10“ 4 percent for a flux density of IT.

Scanned by Cam Scanner


IArt. i. 2|
136 E lectrical M a c h i n e r y _________________________ _______ . .
----------------------------------T f a s s e m b ly a n d it s in s t a lla t io n helps in
I n g e n e r a l, w h a te v e r c h a n g e in t h e tr a n sfo r m t h e a u d ib i e freq u en cy range »
ta k in g th e fr e q u e n c y o f v ib r a tio n o f tn e tr a n sfo r m e r , o
m o s t w e lc o m e .
1 .2 1 . S o m e W o r k e d E x a m p le s tr a n s fo r m e r s a r e so lv e d . A few of these
In t h is a r tic le , so m e ty p ic a l ^ S t i v e e x a m in a tio n ,
h a v e a lr e a d y a p p e a r e d in o n e or th e o tn e v d im e n s io n s in c en tim eters as shown
E x a m p l e 1 .6 9 . A esapgi(h
-lnra n sfo rm er fcas f l ux d e n s ity o f 1.2 T i n the core,
in F ig . 1.79. T h e c o r e i s s q u a r e i n c r o s s - s e c t ^ n J ' o / k g a t 5 0 H ^ Core density is 7.8
the m a g n etic-field in ten sity is 600 A l m o n d the core

10
^ F o r a n a p p l i e d v o lta ge o f 1 2 0 0 V, 5 0 He a t p r i m a r y te r­
m in a ls, calculate a0
1
(a) the n u m b er o f p r im a r y tu rn s
/n . 10
(b) current,pfand p o w e r on the p r i m a l sid e u n d e r no-load 10

conditions. _4 2
10
S o l u t i o n , (a) G ross core a rea = 1 0 x 1 0 x 1 0 = . m
T
N e t core a rea = g ro ss core a rea x s ta c k in g (or iron
F ig 1 7 9 . P e r t a i n i n g to E x a m p le
A* = 0.0 1 x 0 .9 = 0 .0 0 9 m 1 .6 9 .

M axim u m v a lu e o f core flu x ,


6 =A x = 0 .0 0 9 x 1.2 = 0 .0 1 0 8 W b
Y in ax 1 m

From E q n . (1 .4 ), E x = < 2 n f N x < t w


... N u m b er o f tu r n s in p rim a ry ,
N _ _ ___ 1 2 0 0---------- _ 500 25 ~ 5 0 0 tu r n s .
" <2n x 5 0 x 0 .0 1 0 8

(6) M ea n le n g th o f flu x p a th ,
L = [(40 + 10) + (30 + 10)] x 2 = 1 8 0 cm .
M .m .f. for th e core = H L = 6 0 0 x 1 .8 0 = 1 0 8 0 A T s.
M a x im u m v a lu e o f m a g n e tiz in g cu rre n t
_ core m .m .f. _ 1 0 8 0 _ 2 16 A
" Nx 500 '
2 16
R .m .s. v a lu e o f m a g n e tiz in g cu rre n t, I m = - j g - - 1 .5 2 8 A

C ore v o lu m e = L x A , = 1.8 0 x 0 .0 0 9 = 0 .0 1 6 2 m
W
w ee ig
l g hn ti uo ifbcore
m c =—0 .0 1 6 2 x 7 .8 x 1-0­6 x 10" 3 = 1 2 6 .3 6 k g
T o ta l core (or iro n ) lo s s , P c = 1 2 6 .3 6 x 2 = 2 5 2 .7 2 W
C ore lo s s c o m p o n e n t o f p rim a ry cu rre n t,
” = 2 5 2 .7 2
• = 0 .2 1 0 6 A
C~ V' i, ~ 1200
... N o lo a d cu r r e n t, Ie = V/? + 7 ^ = V 0 .2 1 0 6 ‘ + 1.528* = 1 .5 4 2 A

/71 0 .2 1 0 6 n -,or r
N o -lo a d pf, cos 0 ° = -7 - = . CAr> = O .ld b b
/, 1 .5 4 2

Scanned by Cam Scanner


Transformers 137
P ow er a t n o-load = core lo s s = 2 5 2 .7 2 W

Also, p ow er m p u t a t n o lo a d = V , / , co s 0. = 1 2 0 0 x 1 .5 4 2 x 0 .1 3 6 6 = 2 5 2 .7 6 W.
E x a m p le 1 .7 0 . The m a g n e tic circ u it o f a 1-Dhase * r •
. fftrent configurations s h o w n in Fig. 1 8 0 bu t from th fa n s form er is energised in three

J c u d these ValUCS f0 r FigS 1 8 0 <6> a n d (C)- MagneliC « " be

d 5 r .
1 -0 O - --------»
I(T»1 V.f •
A ------------------- ,N
-r N N " E,
V.< N
1
Im^* >

(a) (b) (C)


F ig . 1 .8 0 . P e r t a i n i n g to E x a m p l e 1 .7 0 .

S o lu tio n . Fig. 1 .8 0 {a) : V = y f c n f N <J>^2

or
0ml = ~<2itfN

For flux <t>ml in th e co re, m a x im u m core flu x d e n s ity is B ml= ^ = 1 T (given ), w h e re A =


A, * '
net core a r e a .
Flux $ml is p r o d u c e d b y p r im a r y m .m .f. 7ml ■N , w h e r e m a g n e tiz in g c u r r e n t 7ml = 4 A
(given),i.e., A N <)>ml
Fig. 1.80 ( b ) : F o r e a c h co il o f N tu r n s, V = V 2 n f N 0m2
V
*m2 = ^ 2 n f N ~ <*>ml

This sh o w s t h a t e a c h coil s h o u ld p ro d u ce a flu x so th a t both th e co ils p rod u ce a com ­

bined core flux QmJ a n d t h e r e la tio n V = V2 V N <J>ml is s a tis fie d for ea ch coil. For flu x <J>ml in th e
corein Fig. 1 .8 0 (b )

d _ ^ 2 _ _ 1 J1
m2~ A , ~ A ,

The two coils are identical therefore each coil has the same magnetizing current, i.e.,

For tipper coil,

^ for lower coil,

Adding, we get (Im2/ + ^m2,,) ^ oc $ml


But 4 0m l

(1*2 + * * * " > * 4 A

Scanned by Cam Scanner


138 Electrical M a c h i n e r y --------------------------------------- --------- ---------------------------------------

M a g n e tiz in g cu rre n t o f ea ch coil, l m2 = I m 2 = 2 A.


T o ta l m a g n e tiz in g cu rre n t + An2 - 4 A-
F or core flu x e.m .f. in d u ced in seco n d a ry w in d in g is 100 a s in ig. . a.

Fig. 1.80 ( c ) : H ere V"= V2 V (2N ) 0/»3


V 0 m1

J l"
<J>m3 0m 1 _ 0.5 T

II
ii
F lu x d e n s ity in core B m3 '
" A, ' 2 A,
0m 1
/ « 3 (2 A 0 “ 0m3 or / m3 (2 N ) “ 2
A lso,

0ml-
I m3 (2 M 0ml 1 _1
4N " 2 0m 1 _ 2
4 1
or I m3 " 2 * 2 = 1 A
0ml
e.m.f.
... M a g n e tiz in g cu rren t for b oth s e r ie s co n n ected c o ils - 1 F or a core d u x o f

in d u ced in seco n d a ry - 2 _ 5 0 V.
E x a m p l e 1 .7 1 . A 2 0 kVA, 2 5 0 / 1 2 5 V, 1-phase tra n s fo r m e r h a s the fo llo w in g d a t a :
Tl = 0.15 a x j = 0 . 2 5 11, r2 = 0 .0 3 n , x 2 = 0.04 n
Subscripts 1 and 2 denote h.v. an d l.v. w indings respectively.
When th is tra n sfo rm er is fu lly loaded, the m a g n itu d e o f e.m.f. in d u c e d in p r im a ry at fu
lo a d is fo u n d to be e q u a l to the p r i m a r y te r m in a l voltage. C a lc u la te th e lo a d voltage, loa pfa
load p o w e r . N e g l e c t th e m a g n e t iz in g c u r r e n t.
S o l u t i o n . W ith se c o n d a r y p a r a m e te r s r efe rred to p r im a r y , t h e e q u iv a le n t circuit is as
s h o w n in F ig . 1.81 (a).
r 20,000 ™ A
P r im a r y fu ll lo a d c u r r e n t / x = 2gQ = 80 A

V o lta g e drop b e tw e e n p r im a r y t e r m in a l v o lta g e V x a n d p r im a r y in d u c e d e.m .f. E\ can


w r itte n b y r e fe r r in g to E q n . (1 .3 8 ) a s u n d e r :
Vj - E i = I i (r x cos 0j + Xi s in 0j)

In th e a b o v e e q u a tio n , E x is ta k e n a s r e fe r e n c e a n d 0 ! is t h e p h a s e a n g le b etw een E\ ^

/ , , s e e Fig. 1 .2 4 .
But V 1 = E j (g iv e n ), 0 = r , c o s 0 X+ x x s in 0 !

or 0, = - ta n " 1- = - ta n ~ 1 = - 3 0 .9 6 °
UI 0.25

N e g a t iv e s ig n for 0] in d ic a t e s t h a t I x le a d s E x [s im ila r to / 2 le a d in g V2 in ^
3 0 96° a s s h o w n in F ig . 1.81 (b). T h is fig u r e s h o w s t h a t
VS =E i - I (0.12+ /0.16)

Scanned by Cam Scanner


T •ansformers 139

0 15 0 25 Olf. 0-12
'SAAhr-'OW'— ---- 'UJ0"'u
U

v,» 250V E , - 2 5 0 /0 °

?:

(a) (b)
F ig . 1 .8 1 . (a ) E q u i v a l e n t c ir c u it a n d c o r r e s p o n d i n g p h a s o r d ia g r a m for e x a m p l e 1 .7 1 .

With Ei as referen ce p h asor, l x = 80 Z 3 0 .9 6 0


V2 = 250 Z 0° - 80 Z 3 0 06 fC.12 + j 0.16)
= 250 - 30 Z30.9G x 0.2 Z 5 3 .1 3 0
_ = 2 5 0 - 16 Z84..090 = 248.35 - j 15.915
V2 = 2 4 8 .8 6 Z - 3.67° j

V2 lags E x by 3 .6 7 ° a s sh o w n in Fig. 1.81 (/>'■ T his figure also reveals th a t /j leads load
voltage V { by 0! + 3.6 7 ° = 3 0 .9 6 ° + 3.6 7 ° = 3 4 .6 3 '.
.-. Load p.f. = cos 34.63° = 0.8228 leading
2 4 8 .8 6
Load voltage = 124.43 V

Secondary fu ll load cu rren t, /2 = ^ ’2 5 ^= ^

Load power = 124 .4 3 x 160 x 0 .8 2 2 8 = 16380.961 W.


E xam ple 1.72. In Fig. 1.82 fa), calculate
(a) the power d elivered by each source,
(b) the power d is s ip a te d in each resistor. [GATE, 1991 ]
Solu tion . S econ d a ry p a ra m eters 16 Q and 20 Z - 60° w hen transferred to prim ary sid e

become 16 x I = 4 Q and 20 Z - 60° x - = 10 Z - 60° V respectively. T he circuit o f Fig. 1.82 (a)

becomes the eq u iv a le n t circu it o f F ig. 1.82 ( b ).


With 5 V as referen ce, th e r e s u lta n t v o lta g e VR in th e circuit is given by

Vrt = 10 Z - 60° - 5 Z 0 C
' . \3 _ 5 = - 7 ’ 5V3 = 5 V 3 'Z -9 0 °
= 10 0 . 5 - y 2

phasor diagram o fF ig . 1.82 (c) sh o w s how VR is obtained.

^ a£nitude o f cu rren t I, in p h a se w ith VR, is given by

/ = = 0.5 V3 A
6+ 4

Scanned by Cam Scanner


(Art 1-21
140 Electrical Machinery _

(a )
F ig . 1 8 2 . P e r t a in in g to E x a m p le 1 .7 2 .

P o w e r d is s ip a te d in 6 Cl= / 2f i = (0-5V 3)2 x 6 = 4 .5 W

P o w er d is s ip a te d in 4 £1 = (0 .5 ^ 3 ) x 4 - 3 .0 W
P o w er d e liv e r e d b y 5 V so u rce „ - or Qn n w
= (5V ) (I) cos (5V , I) = 5 x 0 .5 ^ x co s 9 0 = 0 W

P o w er d eliv e r e d b y 2 0 V sou rce


= 10 x 0 .5 V(Tx co s 3 0 ° = 7 .5 W.
E x a m p le 1 7 3 F ig 2 S3 (a) shows the approxim ate equivalent circuit o f a 1.2 kVA 240/110 V
E x a m p le l . / d . r i g . nrima r v E valuate its efficiency a n d regulation when it
single-phase transformer referred to the prim ary, n v a iu u „
delivers full load a t rated voltage a n d u n i t y p o w e r factor.
1X5

J5.Q in

F ig . 1 .8 3 . P e r t a i n i n g to E x a m p l e 1 .7 3 .

S o l u t i o n . P rim a ry fu ll-lo a d cu rre n t, I x = = 5 A

W ith 2 4 0 V a s th e referen ce, p h a so r d ia g ra m to o b ta in Vin is s h o w n in F ig . 1.83 (b).

... V ^ = V (240 + 5 x l ) 2 + (5 x 5 ? = 2 4 6 .2 7 V
f*R lo ss = 5 2 x 1 = 25 W

C ore lo ss = ^ = = 2 4 .2 6 W
Kc ZuUU

E fficien cy a t fu ll load
1200 x 1 x 100
= 96.057%
1 2 0 0 x 1 + 25 + 2 4 .2 6
V o lta g e re g u la tio n

Vm ~ v i _ 2 4 6 .2 7 - 2 4 0
x 1 0 0 = 2.546% .
V,n 2 4 6 .2 7

Scanned by Cam Scanner


r Transformers 141

1.74. F i g . 1 8 4 (a) s h o w s tw o 1-phase id e a l tra n sfo rm ers T1 a n d T 2 o f tu r n s ra tio


nd 3 • 1 connected w l t th e ir P n m a n e s in p a r a lle l across a 120 V, 50 H z source. T h eir
i:1 Aaries are connected
— i in •series a s show n. The d o t-p yuoin
u uts in d ica te te
o iriuiLuit te rr m
m in
in aa ls
ls oj
o f the cor-
.1rnfluu I __ nn the nrim nnj nnri rv r* . ^ ^ .
sl nding Polarity ° n P n m a r y a n d seco n d ary sides. For R = 10 SI, d e te r m in e the cu rren t
f n frorn ^ source> P r^m a r y m p n t im p e d a n c e a n d p o w e r input.

mi

‘ 21
21

(O 120V f4 : l S
W 5 0 H r
T b 3:W +
o
o*
C3
22

J
21
(a) (b)
F ig . 1 .8 4 . P e r t a i n i n g to E x a m p l e 1 .7 4 .

Solution. For tra n sfo rm er T l , th e seco n d a ry v o lta g e is

Y li 120
or V2\ = 30 V
1
For transformer T2, th e seco n d a ry v o lta g e is
22 120
— =— or V 22 = 40 V
3
V Sf 0 n ? arl,e s a r e c ° ™ e c t e d in se r ie s w ith su ch p olarity m a rk in g s t h a t v o lta g e s
ji and V22 aid each o th er. T h erefo re, th e r e s u lta n t o u tp u t v o lta g e V 2 is g iv en by
V2 = V 2 1 + V22 = 30 + 40 = 70 V

Load current Yi = jo
h = ~K = i i : = l A

For mmf balan ce, p rim a ry c u rre n t o f T l is

/ n = 4 = 1.75 A
and
Primary current o f T 2 is

/ 21 = 4 = 2 .3 3 3 A

S°urce current =/n + / 21 = 1 .7 5 + 2 .3 3 3 = 4 .0 8 3 A

nmary input im p ed a n ce = = 2 9 .3 9 0 2 Q
Poiver 4083
EXam 1 ^PUt P rim ary = 120 x 4 .0 8 3 = 70 x 7 = 4 9 0 W a tts.
y^ ib le p0* g*7,5 ' ^ Wo id e n tic a l 2 0 0 V I 2 0 0 V, 50 H z single-phase tra n sform ers A a n d B have
iL*^rrn*r who ° SSeS a n d neSHgihle leakage fluxes. The m a g n e tizin g current taken by either
$ WninFig j ‘ed from r a te d voltage is 0.1 A. I f the w in d in g s o f A a n d B are connected as
* *4 and S ’ estim a te the currents d r a w n by PA a n d S A a n d the open circuit voltage across
^former B **** tW0 w in d in g s o f tra n sfo rm er A , w hile P B a n d S B are the tw o w in d in g s o f
11. A. S., 1991)

Scanned by Cam Scanner


M

142 Electrical Machinery lArt. 1.21

S o lu tio n . A c a r e f u l e x a m i n a t io n o f F ig . 1 -8 5 r e v e a l s t h a t
!|l
i i t | ‘ (I w in d in g SA i s d ir e c tly c o n n e c te d a c r o ss 2 0 0 V , 5 0 H z su p p ly - F o r
, ■ (* i!!
2 0 0 V a c r o s s S A, t h e m a g n e t iz in g c u r r e n t n e e d e d to e s t a b lis h t h e
ft
r e q u ir e d f lu x (a n d th e r e fo r e t h e r a te d v o lta g e 2 0 0 V ) m u s t flo w
|!S t h r o u g h t h e w in d in g SA . T h is m e a n s t h a t w in d in g SA c a r r ie s t e / ^ \ 2oov
fit m a g n e t iz in g c u r r e n t o f 0 .1 A a n d th e r e fo r e w in d in g P,* c a r r ie s n o V ^ /s o h z
■sii c u r r e n t. F u r t h e r , tr a n s fo r m e r a c tio n r e q u ir e s t h a t for 2 0 0 V a c r o ss
> 4V f
S A, v o lt a g e a c r o s s w in d in g P A m u s t b e 2 0 0 V .
I d e n t ic a l w in d in g s P A an d P B a r e s e r ie s -c o n n e c te d a c r o s s 2 0 0
V , 5 0 H z . I t s e e m s t h a t 1 0 0 V s h o u ld a p p e a r a c r o ss e a c h o f t h e

i:!•!ip w in d in g s P A a n d P B. B u t, a s d is c u s s e d a b o v e , 2 0 0 V a p p e a r s a c r o s s
w in d in g P A a n d th e r e fo r e n o v o lta g e e x is t s a c r o s s w in d in g P B. T h is
F i g . 1 .8 5 . P e r t a i n i n g to
E x a m p l e 1 .7 5 .

m s h o w s t h a t in d u c e d e m f a c r o ss w in d in g S B o f tr a n s fo r m e r B is z e r o .
T h u s cu rre n t in w in d in g S A is 0.1 A, cu rre n t in w in d in g P A is zero a n d v o lta g e across SB is
zero.
; .TV E x a m p le 1 .7 6 . T w o id e n tic a l 1 -p h a s e tr a n s fo r m e r s 1 a n d 2 , e a c h r a te d f o r 2 3 0 V/2 3 0 V, 50
H z , a re c o n n e c te d a s s h o w n in F ig . 1.86. F in d th e r e a d in g o n th e id e a l v o lt m e t e r w h en

(а ) R = z e r o o h m ib ) R = 1 1 5 Q.
(c) R = 1 0 0 0 o h m id ) R = in fin ity .
S o l u t i o n , (a ) W h en R = zero ohm , seco n d a ry w in d in g S l is
s h o r t c ir c u ite d . T h e r e fo r e , v o lta g e a c r o s s w in d in g s S x a n d P x is
zero . S o u rce v o lta g e 2 3 0 V a p p e a r s a c r o ss w in d in g P 2 a n d
th e r e fo r e v o lt m e t e r r e a d in g is 2 3 0 V. (r>^)230 v
-A Q Hz
(б) F o r i? = 115 Q, c u rre n t th ro u g h R sh o u ld b e 1 1 5 /1 1 5 = 1
A. T h is c u rre n t c a u s e s w in d in g s P 1 a n d P 2 to carry 1 A. T h u s
v o lta g e acro ss w in d in g P 2 = m a g n e tiz in g im p e d a n c e o f t r a n s ­
form er 2 x 1 A a c tin g a s it s m a g n e tiz in g c u r r e n t b e c a u s e w in d ­
in g S 2 c a rr ies no cu rren t. A s m a g n e tiz in g im p e d a n c e is la r g e ,
th e v o lta g e a cro ss P 2 r is e s m u ch a b o v e 1 1 5 V a n d t h a t a cro ss F ig . 1 .8 6 . P e r t a i n i n g to
E x a m p l e 1 .7 6 .
P j c o lla p se s. I t s h o w s t h a t v o lta g e a cro ss S 1 is m u c h le s s t h a n
1 15 V a n d lik e w is e c u r r e n t th r o u g h R is le s s th a n 1 A. H o w e v e r , t h e v o lt m e t e r r e a d in g is much
m o re th a n 1 1 5 V b u t le s s th a n 2 3 0 V. L e t th is v o ltm e te r r e a d in g b e d e n o te d b y Vb.
(c) F o r R = 1 0 0 0 Q, c u r r e n t is red u c e d in b o th S 1 a n d P v L ik e w is e , c u r r e n t in w in d in g
a ls o r e d u ced . T h erefo r e, v o lta g e a cro ss w in d in g P 2 is n o w le s s t h a n it s v a lu e in p a r t ( b). Even
th e n , th e v o ltm e te r r e a d in g is m o re th a n 1 1 5 V s a y Vc b u t V c is l e s s t h a n V b o b ta in ed in part
ib).
(d ) F o r R e q u a l to in fin ity , w in d in g S i b e h a v e s a s o p e n -c ir c u ite d w in d in g . A s expecte ,
v o ltm e te r r e a d in g is n o w 1 1 5 V.
E x a m p l e 1 .7 7 . A 4 kVA, 5 0 H z, s in g le -p h a s e tr a n s f o r m e r h a s a r a tio 2 0 0 / 4 0 0 V. The data
ta k e n on th e l.v. s id e a t th e r a te d v o lta ge s h o w t h a t the open c ir c u it i n p u t w a tta g e is 80 W.
m u t u a l in d u c ta n c e b etw een the p r i m a r y a n d s e c o n d a r y w i n d i n g s is 1.91 H. W h a t value tw
the c u r r e n t ta k e n by th e tra n s fo r m e r , i f th e n o - lo a d te st is c o n d u c te d o n th e h.v. side at ra
v o lta g e ? N e g le c t th e effect o f w i n d i n g r e s is ta n c e s a n d le a k a g e re a c ta n c e s. {GATE, I
S o l u t i o n . O p e n -c ir c u it in p u t w a t t a g e = co re lo s s in t r a n s f o r m e r = 8 0 W

o c a r ir ie u u y u a r n o c a r ir ie r
Transformers 143
loss = / cl V = 8 0 W
put core
coreloss in a tra n sfo rm er r e m a in s u n a ltered w h e th e r it is en ergised from l.v. sid e or h.v.

A
,]0SS current w h e n en e r g iz e d from h .v. sid e,
. Core

ln a transform er, E = <2 n f N <J>max = V2 n / > n


“max

, Maximum v a lu e o f flu x lin k a g e s y max w ith l.v. w in d in g =

Mutual in d u cta n ce, M = H ^ ^ n k a g e s w ith l.v. w in d in g = 1


w C u rren t in h.v. w in d in g 42 nf 42 Im2
where/m2 is th e m a g n e tiz in g cu rre n t in h .v. w inding.
, 200 1
m2 42 71 x 5 0 ' 4 2 x 1.91 ~ 0 3333 A

The current ta k en by tra n sfo rm er w h e n en ergized on h.v. side, as per Eq. (1.18), is
Je = ^ 4 + 4 > = [0.22 + 0 .3 3 3 3 211/2 = 0 .3 8 8 7 A.
Example 1.78. A 100 kVA, 11 k V / 4 0 0 V, 3-phase transformer has its maximum possible
ifficiency of 98%, when it d elivers 80 kVA a t unity p f a n d rated voltage. The maximum possible
voltage regulation o f the tra n sform er is 4%. F ind the efficiency an d voltage regulation o f the
transformer for rated kVA o u tp u t a t 0.8 p f lagging. (/./, S., 1994)
Solution. Total lo s s e s in tra n sfo rm er
A f 1 N
1
- I - i o u tp u t = - 1 x 80,000 = 1632.65 W
J\ y v0.98 y
Core losses = ohm ic lo s s e s = 1 6 3 2 .6 5 /2 = 8 1 6 .3 2 5 W

x 1 n2
Rated-load ohm ic lo s s e s x 8 1 6 .3 2 5 = 1275.51 W
0.8
1 0 0 ,0 0 0 x 0 .8
Transformer efficien cy —-------------------------- x 100 = 97.452%
y 8 0 ,0 0 0 + 8 1 6 .3 2 5 + 1275.51
kximum possible v o lta g e r e g u la tio n o f a tran sform er is equal to ze2 in pu = 0.04 p.u.

P,°® Eq. (1.36), r ,2 in p er u n it = = 0 .0 1 2 7 8

c2 = (0 .0 4 2 - 0 .0 1 2 7 8 2] 1/2 = 0.0379 p.u.


Volt
age regulation = [re2 cos 02 + xe2 sin 021
= [0 .0 1^ 2— 8 -x• 0.8
— + 0.0 3 7 9- x 0.6]- x 100 = 3.298%.
C mP!G 1,79‘ A Single-Phase a uto-transform er is to deliver a pow er o fW k\V at unity p f at
Sling WUh input v o y a g e o f V x volts. Determine the kVA rating o f each section o f the
^ W h c d C° mpare w ith the r a tin g o f a tw o-w indin g transformer assum ing an efficiency of

S o !^ ^ = 0,8 Vi> W = WO kW, p f = 1.00, T| = 0.96, determine their ratings.


: mtion t \ . . . . ..
n' 7 Wo'Winding transform er.
Sec
:ond^ w i „ d in g r a t in g = iy js J s w = w k V A

Scanned b yC am S canner
144 Electrical Machinery

. . wW i1 rr k V A
w
P rim ary w in d in g ra tin g = — x — = —

H ere p f on p rim ary sid e is a ssu m ed th e sa m e a s on th e s e c o n d a r y sid e .


Auto-transformer. A ssu m in g it a step -d o w n tra n sfo rm er, a s in F ig . 1 .3 6 , t h e v o lta g e rating
, . . . W I
o f section A S = V 1 - V2 and its cu rren t ra tin g is I i = — • y"-

kV A r a tin g o f sectio n A S = I x (V^ - V2)

1) Vj T|
y 2
w h ere k - ratio o f tra n sfo rm a tio n =
vi
W W
T h e v o lta g e across sectio n B C = V2 and it s cu rren t r a tin g is ( /2 - I x) = y - -

.-. kV A ra tin g o f sectio n B C = V2 (12 - 1{)

For tw o-w in d in g tran sform er, secon d ary r a tin g = 1 0 0 k V A a n d p r im a r y r a tin g

■ a w '■ '1 0 4 1 6 7 kV A -
For au to-tran sform er, ra tin g o f sectio n

AB = [1 - 0.81 = 2 0 .8 3 3 k V A

and ra tin g o f sectio n B C = 100 1 6 .6 6 7 kV A .


[ l - —
0.96J1 =
1.22. S u m m ary

H o v ^ C T ^ itT s61p refer a b le e le c tr o m e c h a n ic a l e n e r g y c o n v e r s io n i s in volved in il


m achines' L c e eL ctrT c^ h . L ^ r t t r ? s fo r m e r ^ n d t h e n t h e r o t a t in g electric*
r e sp ects. T h e co u p lin g b etw een °th p ^ 0 rm e^s ^ r o ta tin g m a c h in e s i s s im ila r in man
th rough t h e ? e c o nda r y w i n d i n g , o f a tran sform er i
electrical and m ech a n ica l s y ste m s (or s U t o ^ ^ ^ o t o r ^ t he co u Plin g betwee
In view o f th is, m a n y o f th e r e le v a n t eq u a tio n s ^ ^ 1S ° U gh t h e m a ^ etic fie1'
theory, are ap p licab le to th e a n a ly s is o f a c m a c h i n e d ? . n c lu s io n s d r a w n ™ t h e transform!
T h ese p oin ts o f sim ila r ity a re d isc u sse d b eio w • S° m e e x t e n t to d c< m ach in es als

th e p rim ary and s e c o n d a ^ ^ d ^ ^ s ^ w W ^ C0If ? . e d to t h e c o r e >11 th e r e fo r e Unks bo


flu x cro sses th e air-gap and lin k s b o th th e s ta to r ^ reS u lta n t mUtU
T he r e s u lta n t m u tu a l flu x in tr a n sfo r m e r i , , , gS '
and secon d a ry cu rren ts. S im ila r lv th o « u. I8.Pro t h e c o m b in e d a ctio n o f prim a
produced b y th e com b in ed a ctio n o f s ta to r ^ L ™ ^ ' g a P flUX “ r o ta tin g machine
(ii) In tra n sfo rm ers, th e su p p ly vnltn ■ i c u r r e n ts .
lin k a g es are ch a n g in g w ith tim e an d eirff! ? th e r e fo r e t h e r e s u lt a n t m utual fl
w in d in g s. In r o ta tin g m a c h in e s th e r e s u l t a n t ^ u c e d in s t a t io n a r y p r im a r y and seconds
tim e by rotor ro ta tio n an d th e n sp e e d em f* I " rW A ux lin k a g e s a r e m a d e to change w>
o f sp eed em fs d u e to m ec h a n ic a l m o tio n is ,t h e a r m a tu r e w in d in g s . T h e exist"
a l for e le c tr o m e c h a n ic a l e n e r g y conversion

" '- - y jj-

Scanned by Cam Scanner


r

1. 22) ___________________
- Transformers 145
(iii) T he r e s u l ta n t m u tu a l flux in b o th tr a n .
flux, since it ta k e s p a r t in th e enerjjy-conversion n r Z t * ™ 4 ro ta tin g m achines, is th e useful
& lin k in g one w in d in g a n d n o t th e o th er a re Z In “ n to th is u s e M flux’ ^ a k a g e
g a tin g m ac h in e s, t h e le a k a g e fiuzen t r e ^ Z Z T ? b °t h ' ^ ^ ‘r - s f o r m e r and
leakage fluxes a re la rg e ly in a ir, consequently lea k *ge rea cta n ce s- In b o th - the
magnetic s a tu r a tio n a n d a re a ssu m e d c” t ** reaC‘ anCeS * " U" effeCted by th e

j ^and”t h ^ r a « ^ e r e m f ^ A i w d * ^ 'L?ltaB e. h a s 10 o v erc o m e th e p r im a r y le a k a g e im p e d a n c e


drop a n d t h e (:o u n te r e m f m d u c e d i n t h e p r im a r y b y th e r e s u lt a n t core flu x . S im ila r ly in a.c.
w indings o f m o to r s t h e a p p lie d v o lt a g e h a s to o v erc o m e t h e le a k a g e im p e d a n c e drop an d th e
counter e m f g e n e r a t e d b y t h e r e s u lt a n t m u t u a l a ir -g a p flu x .
T he le a k a g e im p e d a n c e d r o p s a r e u s u a lly s m a ll, th erefo re in b o th tr a n sfo r m e r s a n d r o ta t­
ing m a c h in e s, t h e a p p lie d v o lt a g e i s a p p r o x im a te ly e q u a l to th e co u n ter em f. C o n se q u e n tly
constant a p p lie d v o lt a g e a n d fr e q u e n c y r e q u ir e s c o n s ta n t co u n ter e m f an d th e r e fo r e a c o n s ta n t
value o f r e s u lt a n t m u t u a l flu x . H e n c e in ac e le c tr o m a g n e tic d e v ic e s h a v in g s m a ll le a k a g e im ­
pedance d ro p s, t h e r e s u l t a n t m u t u a l flu x is d e te r m in e d b y t h e a p p lied v o lta g e a n d freq u en cy.
Since, for c o n s t a n t a p p lie d v o lt a g e a n d fr e q u e n c y th e r e s u lta n t m u tu a l flu x is c o n s ta n t, th e
currents in a ll t h e w in d in g s m u s t a d ju s t t h e m s e lv e s so a s to cre a te th is c o n s ta n t flu x .
It m a y b e c o n c lu d e d fr o m t h e a b o v e d is c u s sio n th a t th e tra n sfo rm ers, in d u c tio n m otors,
synchronous m o to r s a n d t h e a lte r n a to r s c o n n e c te d to in fin ite b u s-b a rs, m a y b e reg a rd ed a s
const a n t-flu x m a c h in e s .
(u) L o s s e s in r o t a t in g m a c h in e s a r e t h e s a m e a s in tra n sfo rm ers, ex ce p t for th e friction and
w indage lo s s e s .
(ui) An o p e n -c ir c u it, o r n o -lo a d , t e s t h e lp s in k n o w in g th e ex c ita tio n re q u ir e m e n ts an d core
loss (and fric tio n a n d w in d a g e lo s s in r o ta tin g m a c h in e s). S h o rt-circu it t e s t h e lp s in d e te r m in ­
ing th e ohm ic lo s s a n d t h e le a k a g e im p e d a n c e s o f th e w in d in g s. In ro ta tin g e le c tr ic a l m a c h in e s,
however, dc r e s is t a n c e m e a s u r e m e n t is n e e d e d . B o th op en -circu it an d sh o rt-circ u it t e s t s h e lp
in th e calcu lation o f e f f ic ie n c y o f b o th tr a n s fo r m e r s a n d r o ta tin g electric a l m a ch in es.

PROBLEMS
(a) Define a tra n s fo rm e r. H ow is th e energy tra n sfe rre d from one circuit to another? D istinguish
between prim ary a n d se c o n d a ry w indings.
(MTransformer is the main reason for the wide-spread popularity of a.c. systems over d.c. systems. Explain.
(c) Give som e tra n s fo r m e r ap p lic atio n s in electronic a n d control circuits.
M> Describe th e tw o g e n e ra l types o f tran sfo n n en .. Why is th e low-voltage w inding placed n ear th e con. ?
Why are the b u tt jo in ts in th e a d ja c e n t layers staggered?
L2. (o) W hy is iro n u s e d for th e c o n stru ctio n o f transform er-core ?
(b) Why is th e tra n s fo r m e r core la m in a te d ? .
(c) Is it possible to u s e a n y w in d in g of th e tra n sfo rm e r as its prim ary . xp _
,r. _ . , ' Annm transform ers. S ta te clearly th e q u a n titie s w hich rem ain
(d) Distinguish between stop-up and step-down trans
Unaltered or g et ste p p e d -u p o r stepped-dow n.
U . W Explain the principle of tonsferm ei sh(W ^ emf per turn in prhnsry
^ (b) Derive an expression for the e.m.f. induced in
eqjial to e.m.f. p e r tu r n in th e secondary. hafl oer tu rn e.m.f. of about 9 volts and m axim um flux
j !c) A 6300/210 V, 50 H z, sin g le -p h a se tra n sfo rm e Pf ^^ th e Qet croaB.sectional a re a of th e
£ ^ lty ° f 1-2 T. F in d th e n u m b e r of high -v o ltag e a n d o [Ati, 7 2 0 turns, 24 turns, 328.3 cm8)

. . j - crs-nm of a 1-phase transform er. D iscuss how prim ary


1 (o) D raw a n d e x p la in th e no-load p h a so r di gram
flux is a cco u n ted for in th e p h a a o r diagram .

Scanned by Cam Scanner


IP rob. 1
146 E le c tr ic a l M a c h i n e r y - '
t h p m r r e n t o n t h e s e c o n d a r y s i d e o f t h e t r a n s f o r m e r ,s
<M E x p la in h o w t h e p r im a r y c u r r e n t in c r e a s e s a s t h e c u r r e n t o

in c r e a s e d . . , - „ e . a o^oV p rim a ry , a 900V w i n d i n g w i t h a centre


(c) A s in g le - p h a s e 5 0 H z t r a n s f o r m e r h a s o u r ' T . o f g n e t c o r e a r e a 0 f 2 0 c m 2 , f r n d t h e number
ta p p in g , a 6 V w in d in g w ith a c en tr e ta p p in g a n d a n 8 V w i n d i n g ^ ^ ^ ^

o f tu r n s o n e a c h w in d in g . T h e m a x im u m v F i r s t c a l c u l a t e t h e n u m b e r o f t u r n s in
[H in t. A ce n tr e-ta p p e d w in d in g m u st h a v e e v e n n u m b e r o • ^ J g lg 0 ( ) 46Q (p rjm a ry ,
th e lo w e st-v o lta g e w in d in g , h e re 6 V w in d in g ! ... f
, ., , , D e sc r ib e its p e r fo r m a n c e w ith th e h e lp of em f
1 .5 . G iv e t h e c o n c e p t o f s in g le - p h a s e id e a l tr a n s fo r m e r .
e q u a tio n s , its p h a s o r d ia g r a m s a t n o lo a d a n d o n lo a d .
D is c u s s h o w co re flu x in a n id e a l tr a n s fo r m e r is in d e p e n d e n t o f lo a d c u r r e n t.
l J i s c u s s n o w c u r e u u a .. . tra n sferred fro m second ary t o p r i m a r y or
1 .6 . (a ) I n a n id e a l t r a n s f o r m e r , d is c u s s h o w im p e d a n c e is t i a n s l e r r e a e y

v ic e -v e r sa . H z r e s p e c tiv e ly . F o r th e sa m e em f
(b) I d e a l t r a n s f o r m e r s A a n d B a r e d e s i g n e d t o w o r k a t . (fe ) A : B = 2 0 0 : 1|
p e r tu r n a n d fo r th e s a m e c o r e flu x d e n s itie s , c o m p a r e th e ir c o r e a r e a s . ___________ _________

1 .7 . (a ) T h e p r im a r y w in d in g o f a s in g le - p h a s e t r a n s f o r m e r is « 1 [
from fix ed s in u s o id a l v o lta g e w ith se c o n d a r y o p e n -c ir c u ite d . If th e s h a d e d
p o r tio n o f th e ir o n c o r e is r e m o v e d , F ig . 1 .8 7 , d is c u s s w h a t w o u ld h a p p e n to
th e m a g n itu d e s o f its e x c itin g c u r r e n t, co re flu x a n d th e n o -lo a d p o w e r fa cto r.
(6 ) A s in g le -p h a s e , 5 0 H z tr a n s fo r m e r h a s v o lta g e r a tin g o f 1 1 0 0 0 /4 0 0
VV bu aa ss ce du Uo nl l i1 t1 s3 wW Ui nJ Ud Xi n1 I g5 tV uU rI nt i sa r a t i o . I t s —l e -----------
a k a og ----------
e a n d m a g n e t i 1z i n g i n• d u c t a• n c e s
d u r in g te s ts , a r e fo u n d to b e 0 .2 H a n d 8 0 H r e s p e c tiv e ly o n its p n m a r y
(h .v .) s id e . F o r p r im a r y a p p lie d v o lta g e o f 1 1 0 0 0 V a t 5 0 H z , c a lc u la t e t h e [
open-circuit secondary voltage.
[A n s. (a) E x c i t i n g c u r r e n t in c r e a se s, co re flu x r e m a in s c o u s ta n t a n d n o -lo a d F ig . 1.87 P e r t a i n i n g to
power factor is worsened. (6) 399 V] Problem 1.7 (a).

1.8. ( a ) A single-phase tra n sfo rm e r is designed to o p e ra te a t ra te d p rim a ry v o lta g e 230 V and at rated
frequency 50 Hz. If its p rim a ry voltage is in cre ase d by 10% on no load, d isc u ss w h a t w ould h ap p en to the
tra n sfo rm er operation.
(6) D iscuss th e a d v a n ta g e s of u sin g CRGO la m in a tio n s in tra n s fo rm e rs .
(c) A 10 kVA, 440/220 V, 400 Hz tra n sfo rm e r is d e sire d to be u se d a t a fre q u e n c y o f 60 H z. F in d th e volt
as well as th e kVA ra tin g of th e tra n s fo rm e r a t th is reduced frequency.
l A n s . ( a ) M agnetizing cu rren t, core loss and exciting cu rren t increase,
third harm onics in exciting c u rre n t is m ore pronounced, (c) 66/33 V, 1.5 kVA!
1.9. (a) E x p lain w hy tra n s fo rm e r r a tin g is e x p re sse d in kVA or VA. D e sc rib e th e sig n ific a n ce of all the
item s m entioned on th e n a m e -p la te of a sin g le -p h a se tra n s fo rm e r.
(6) A 1-phase tra n s fo rm e r h a s th e follow ing d a ta :
Peak flux density in core = 1.40 T ; N et core a re a = 0.012 m2
C urrent density in conductors = 2.5 MA/m2;
Conductor dia. = 2.0 mm ; P rim ary voltage = 230 V, 50 Hz.
C a lc u la te th e kVA ra tin g of th e tra n s fo rm e r a n d th e n u m b e r of tu r n s on th e p r im a r y w in d in g .
[Ana. (b) 1 80642 kVA. 62 turm
1.10. (a ) D evelop th e p h a s o r d ia g ra m o f a sin g le -p h a s e tr a n s f o r m e r u n d e r la g g in g -p o w e r fa c to r load
(6)
A 100 kVA, 2400/240-V , 50 H z sin g le p h a s e tra n s fo r m e r h a s a n e x c itin g c u r r e n t o f 0.64 A and a cor
loss of 700 watts, when its high-voltage side is energised at rated voltage and frequency. C a lc u la te the tv,
components of the exciting current.
If the above transformer supplies a load current of 40 am peres at 0.8 power factor lag on its l.v. side, tin
calculate the primary current and its power factor. Ignore leakage im pedance drops
IA ns. 0.292 A; 0.569 A, 4.584 A, p.f. =0.762 lagg"
1.11. (a) What current flows in the transformer primary when its secondary is open ?W hat is its fuiictu'i
Give its order of magnitude.
(b ) In a transformer, core flux depends on voltage, w hereas the leakage flu xes depend on the c
Explain.

Scanned by Cam Scanner


- r° b ------------ Transformer! 147

same num ber of *kP ° f iron’ alum inium and wood have the sam e dim ensions and are wound w ith the
of the m aterials in on j . C°reS are fed from the 8ame voltage a t a fixed frequency, then w rite the names
j j y c u rre n t l o s s j tm g t0 the m aP litudes of (i) m agnetizing current, (ii) hysteresis loss and (Hi)
eddy c u rre n t loss. t,,v e a b n e f description for each answer.
H in t. Iron is ferro-m agnetic m aterial, alum inium is param agnetic m aterial (pr = 1.002) and wood is
non m agnetic m ate ria l w ith pr = l.oo. Flux produced is sam e in all the three cores.)
Ans. (c) (i) Wood, aluminium and iron
(ii) Iron, aluminium and wood in decreasing order
(Hi) Aluminium, iron and wood
f ° f 8 tra n sform er is energised a t rated voltage of 11,000 V and a t rated frequency
0 ’ ,.es ' and 2400 w atts a t no-load. A nother transform er has all its core dim ensions >/2 times
the corresponding core dim ensions of the first transform er.
N um ber of p rim ary tu rn s, type of core m aterial and lam ination thickness are the sam e in both the
transform ers. If th e p rim ary of the second transform er is energised from 22,000 V, 50 Hz supply, calculate the
no-load c u rre n t and pow er draw n by it.
[H in t. Core loss « Core volume

pc2 = W2)3 (Pel) = (V2)3 (2400) = 6780 watts.


C o re lo ss c o m p o n e n t Ic2 = ( V 2 ) 3/ r l

/c2 = 2>/2 ( i ? 5 o i j ) = 2V2(0-2182>=0617 A

M agnetizing com ponent, 7ml

= v/»i2- ^ i * = V0.2)5*- (0.2182)3* =3.193 A


A 11,000
Now reluctance Rl2 =
12 ’ 'Pml V2n/W,
22,000
and ♦m 2 =
V2x/-N, ~ ^ ml
m.m.f. _ fm i^i
But ♦m l —R e lu c ta n c e Rlx

Also ♦m2 = 2 ^ 1 = ^ *
Rl2

^m2 ~ —?
HIj
(2/m,) = V2/ml. = V2(3.193) = 4.515 Aetc.l IAns. 4.557 A, 6780 watts.)

1.13. (a) D raw th e exact equivalent circuit of a transform er and describe briefly th e various param eters
involved in it.
(b) A 50 H z, 3-phase core-type star-d elta transform er has a line voltage ratio of 11,000/440 volts. The
cross-section of th e core is sq u are w ith a circum scribing circle of 0.4 m diam eter. D eterm ine th e num ber of
turns per p h ase on high-voltage and low-voltage windings for a m axim um flux density of 1.25 T. Assum e th a t
the insulation occupies 10% of th e to tal core area.

(H int. N et core a re a A, = (0.4 cos 45°)2 (0.9)

Low-voltage per phase turns = e m f pgr turn etC'^ (Ana. 318, 22)

1.14. A single-phase tra n sfo rm e r h a s 1000 prim ary tu rn s and 80 secondary tu rn s wound around a core of
net cross-sectional a re a 40 cm 2. T he r.m .s. value of flux density in th e core is lim ited to 1.4 T. C alculate the
maximum 50 H z r.m .s. p rim a ry voltage th a t can be applied on th e prim ary side w ithout reaching sa tu ra tio n
The core h a s a m ean len g th of 160 cm and its relative perm eability is 6000. Find th e value of exciting
current if th e tra n sfo rm e r p rim a ry is energised from th e voltage calculated above. C alculate, also, th e m ag­
netizing reactan ce as seen from p rim a ry and secondary sides.
(V 2/m Ar,)p0 Hr -A, (Ana. 1758.76 V, 0.2971 A. 5919.76 ft, 37.886 Q1
| Hint. ^

i>canne3 c5y L a m ^ c a n n e r
IProb, i
148 Electrical Machinery
r i a*rl 1 nhase tra n sfo rm er and drive from th is, the equivalent
1 .1 5 . (a ) D r a w th e p h a s o r d ia g r a m o f a lo a d e P
c ir c u it o f a tr a n s fo r m e r . Q ] o a d 0 f 8 + j ' 6 f l a n d i t s p r i m a r y i s fe d from
(6) A 400/100 V, ideal 1-phase transform er is conne ^ on both p rim a ry a n d secondary sides.
400 V, 50 Hz source. Calculate the current, power fact [Anfl 10 Af 0.8 lag, 800 W ; 2.5 A, 0.8 lag, 800 W]
Neglect m agnetizing current. tra n sfo rm e r core re m a in s fairly constant from
1.16. (a) Explain, with phasor diagram , how the flux in th e tr
no-load to full-load (assum ing lagging pfl- ^ ^ bv w in d in g resistan ces of 7 fi and
(b) A 50 kVA. 6600/230 V,single-phase t r ^ sf° ^ rQ 3 A a t a p f of 0.3 (lagging) is recorded on h.v. side
0.008 fi respectively. W ith l.v. winding open, a c _ ^ load ^ 0 g p f lagging. Determine also
with the application of full rated voltage. C alculate etiici y (Ang 96 gg% lgg ?4
the load current a t which m aximum efficiency occurs.
1.17. (a) Develop the exact equivalent circuit of a single p h ase tra n s 0rm6
(61 The constants of a single phase 50 H r. 2200/220 V tra n sfo rm er a re a s follows,
h.v. side : r, = 0.21 fi, x, = 3.84 fi, Rel = 4800 fi, Xml =3500 fi
l.v. side : r2 = 0.006 fi, x2 = 0.022 fi.
Find the equivalent circuit param eters referred to (i) h.v. side a n d («) l.v. side
[Ans. (i) = 0.81 fi, x „ = 6.04 fi, Rel = 4800 fi, Xml = 3500 0
(ii) rt2 =0.0081 fi, xt2 = 0.0604 fi, Rc2= 48 fi, Xm2 = 35 Q]

1.18. Develop the exact equivalent circuit of a 1-phase tra n sfo rm er. F ro m th is , deriv e th e approxim ate and
simplified equivalent circuits of the transform er. S ta te th e various a ssu m p tio n s m ade.
Also draw the phasor diagram for the equivalent T-circuit of th e tra n sfo rm er.
1.19. (a) Give the equivalent circuit of a tran sfo rm er an d define its v a rio u s p a ra m e te rs . C learly state the
assum ptions involved in the applicability of th is eq u iv alen t circuit.
(6) How will the equivalent circuit be modified if th e tra n sfo rm e r is a n air-co red tra n sfo rm er?
(c) A 100 kVA, 1000/400 volts, single-phase tran sfo rm er, w hen excited a t r a te d v o ltag e on h.v. side, draws
a no-load current of 3.0 A a t 0.5 lagging power factor. If it is excited from th e l.v. sid e a t ra te d voltage, determine
the no load current, power factor and power input.
[H int, (a) Assum ptions : (i) No sa tu ra tio n , (ii) E q u iv alen t circuit p a ra m e te rs re m a in c o n sta n t for minor
changes in supply voltage and frequency.) [Ans. (6) Om it Re from Fig. 1.14 (e). (c) 7.5 A, 0.5 lag, 1500 W]
1.20. (a) In open-circuit test, the ohmic losses are negligible in com parison w ith n o rm a l core loss. Explain.
(b) In short-circuit test, the core loss is negligible in com parison w ith full-load ohm ic losses. Explain.
(c) A 5 kVA, 1000/200 V, 50 Hz single-phase tra n sfo rm e r gave th e follow ing t e s t re s u lts :
Open-circuit test ( l.v. side) : 200 V, 1.2 A, 90 W
Short-circuit te st ( h.v. s id e ) : 50 V, 5A, 110 W.

e , u i v a Z t d 'r c ^ V i U. *> l v . side. Also draw the exact


[Ans- Approx. equivalent c irc u it: Rc2 = 444.44 fi, Xm2 = 179.86 fi, re2 = 0.176 fi, xt2 = 0.3592 0
Exact equivalent c irc u it: Re2 = 444.44 fi, Xm2 = 1 7 9 .8 6 fi, r2 = = 0.088 fi, x2 = = 0.1796 fll

(M A w V v T a m m n T ° f “ sil,e le -ljh a s e ‘ra n a fo rm e r s u p p ly in g a le a d in g pow er factor load.


(6) A 10 kVA, 500/250 V. s m g lc p h a s e tra n s fo rm e r g av e th e follow ing te a t r e s u l t , •
S.C. te s t ( h.v. s id e ) : 60 V, 20 A, 150 W.
The m axim um efficiency occurs a t u n ity Df an d a t l o a ,h 1 j - . .1
0.8 pf. Also calculate th e m axim um efficiency fall-lo ad c u rr e n t. D e te rm in e full-load efficiency at
1 22 WKflt ♦ r i_ [A ns. 95.625%, 96.525
l.ZZ. (a) W hat p a ra m e te rs of th e eq u iv a len t c irc u it n f a ♦*.<> r . •
and sh o rt circuit tests? E xplain. tra n s fo r m e r c a n b e d e te r m in e d from open-circui

(c) Show t h a t p e r u n i t ' *?*** Sh° U' d ^ p e rfo rm e d 0 0 a n d * > v sid es respective')
tan ce rtL or rt}h can be o b tain e d from th e following relation
rtL or rtH = ^ h m ic josses a t rated c u rre n t
Rated VA '
S u bscripts Lnad Hen
d o te l.v. a n d h.v. sid e s re sp e c tiv e ly .

S ca n n e a b y
------------- ■ ------------------------------------------ Transformers 149

1. A 5 kVA, 2200/220 V, sin g le-p h ase tra n sfo rm e r h a s th e following p a ra m e te rs :


h.v. side : r 1 = 3.4Q , x , = 7 .2 n
l.v. side : r 2 = 0.028 ft, x2 = 0.060 H

r m a(*e O liv e r ra te d c u rre n t a t 0.8 lagging pow er factor, to a load connected on


the l.v. side. I f th e load voltage is 220 V, calculate th e term in a l voltage on th e h.v. side.
(b) R e p e a t p a r t (a) for a load p f of 0.8 leading,

art W ^and^fc)^6 ^°SS ^ w a^ s ra te d voltage a n d frequency, find th e efficiency u n d e r th e conditions of

(d) T h e tra n s fo r m e r ex citin g c u rre n t is 3% of full-load cu rren t. C alculate voltm eter, a m m e te r a n d w a ttm e te r
rea inSs or op en -circu it te s t a t ra te d voltage and short-circuit te s t a t ra te d c u rre n t. T he in stru m e n ts are
connected on l.v. sid e for open-circuit te s t and on h.v. side for sh o rt circuit test.
lAns. (a) 2229.27 V (6) 2193.27 V (c) 98.47% (d) O.C. te s t: 220 V, 0.682 A 30 W. S.C. t e s t: 33.14V, 2.273 A, 32.10 W]
1.24. (a) D efine voltage reg u la tio n of a transform er.
(b) W h at c a u se s a change in secondary term in al voltage of a transform er, as it is loaded ? E n u m e ra te th e
factors w hich influence th e m a g n itu d e of th is change.
(c) A 50 Hz, sin g le -p h a se tra n sfo rm e r draw s a sh o rt circuit current of 30 A a t 0.2 p f lag w hen connected
to 16 V, 50 Hz source. W h a t will be th e short-circuit c u rren t, and its pf, when the sam e tra n sfo rm er is energised
: from 16 V, 25 H z source ?

H in t, (c) ^ = 0.2 or 24r,* = x]

3Q jg 16
Vr* + xlt 5re
re = 0.1067 D a n d xe = 0.523 fi
At 25 Hz, xe = 0.2615 Q and rt = 0.1067 H etc.l [Ans. 56.65 .
1.25. (a) F o r a lag g in g p f load, show th a t p er u n it voltage regulation of a transform er can be obtained]
the expression
*t 2pu ■ c o s (0 - 82)

where <j>= leakage im pedance angle and 02 = load power-factor angle.


(6) F ind also th e condition u n d e r w hich m axim um voltage regulation occurs.
(c) A 10 kVA, 2 5 0 0 /2 5 0 V, single-phase tra n sfo rm er h as full load ohmic loss of 300 W. T he m axim um
possible voltage drop in th e tran sfo rm er-seco n d ary voltage is 20 V. Find voltage regulation of th is tra n sfo rm er
for rated kVA o u tp u t a t 0.8 p f lagging. lAns. (c) 6.852%)
1.26. (a) V oltage reg u la tio n of a tra n sfo rm e r varies w ith power factor. V alidate th is s ta te m e n t through
suitable deriv atio n s. A t w h a t pow er factor will th e regulation be (i) m axim um and (ii) zero.
Does th e m ax im u m efficiency of a tra n sfo rm er also depend on power factor ? Discuss.
(b) A 25 kVA 2000/200 V, 50 H z tra n sfo rm e r has m axim um efficiency a t 80% of full load. Its p er unit
a resistance and im p ed an ce a re 0.012 a n d 0.05 respectively. D eterm ine its efficiency a n d voltage regulation at
• 1 . . * __ . F A n o 0 7 A Of- V O O C O W . l
j\j half of the full load a n d a t 0.8 p f lagging. (Ans- 97.4%, 1.9362%]
1.27. (a) T he full-load voltage drops in a tra n sfo rm er are 2% and 4% due respectively to resistan ce and
\* leakage reactance. T he full-load ohm ic loss is equal to iron loss. C alculate
(i) the efficiency on h a lf load a t u n ity p f
(**) th e lagging p f oil full-load o u tp u t a t w hich th e voltage drop is a m axim um and
(Hi) th e m ax im u m p e rc e n ta g e voltage drop
\ A (6) A sin gle-phase tra n s fo rm e r h a s p ercentage regulation of 4 an d 4.4 for lagging pow er factor of 0.8 and
i 0 6 respectively. T he full-load ohm ic loss is equal to iron loss. C alculate
j d) the lagging p f a t w hich full-lond reg u latio n is m axim um ,
i K U I r ,n . n- ■ i.mfvnf |A n .. (o) 95.24%, 0.4472 tag. 4.472% (6 ) 0.4472 lag, 96.154%|
(«) the full-load efficiency a t u n ity pi. v; ’
' „ ‘ -28. (a, Is th e voltage reg u la tio n a t pow er factor 0.8 lagging b e tte r or w orse th a n its value a t power factor
6 Egging for th e sa m e kVA o u tp u t ? Give ap p ro p riate reasons.

outfinieu uy odiiioudiiiiui
(Prob j
ISO Electrical M a c h i n e r y ' ~ ~ . " '
--------------------------------------------- ”k lpnkace im pedance of 4 +J 40 t l referred to its h.v 8ide
(5) A 3300/1100 V, single-phase transform er nas w ■* ^ secondary te rm in a l voltage measured
W hen unity power factor load of 20 kW is c o n n e c t^ on th e ^ ^ y te rm in tt,8.
is 1050 V. C alculate the voltage and power fac o ^ 3ig& g y q ^ ^

r th e per u n it voltage reg u la tio n of a transform er both f0r


1.29. (a) Derive an expression forcom puting tn p
laggm g «n<l leading power factors. eal)ivaie „ t re a ia ttn e e of 0.015 p.u and „ equ,valMl
(b) A 3300/230 V, single phase transform er has q t foU load a n d 0 .8 p.f. lag, for a primary
leakage reactance of 0.04 p.u. Find the secondary term in al voltage ^ ^ mry
voltage of 3300 V. logses D iscuss th e determmat10n
1.30. ( a , Deaeribe . teat on a m t h t , te a t.
o f t h e e q u i v a l e n t c i r c u i t p a r a m e t e r s which . , ,.
< 1 i nf in s , w hen delivering full load a t u n ity p f and 15% when
(b) A single-phase transform er h as a regu aml ^ reg u ia tion if th e tra n s fo rm e r is delivering halMoad
delivering the sam e load a t 0.8 pf lagging. Wh (Ans. 0.5%) (I.E.S., 1981)
a t 0 .8 pf leading ?
aV t..n .fn r m p r th a t eives its core losses. D iscuss th e determination of
1.31. Ca) Describe a te st on a 1 -phase transform er th a t gives iw
the equivalent circuit param eters which can be obtained from th is test.
. «- • r» t v A *1300/500 V 50 Hz single-phase tra n s fo rm e r is 97% and occurs
V ^ o . T ^ t T p o w « S r . If A . .m p o d .n o , is l o i . c l c u U t , l b .
0 .8 lagging.
1.32. (o) In a 1 -phase transform er, the core flux rem ain s s u b sta n tia lly c o n sta n t a n d is independent of the
load current. Explain.
(6 ) In a transform er, find the load c u rren t a t w hich m axim um efficiency occurs. S ta te th e assumptions
made.
(c) W hen a transform er w ith data given in Prob. 1.23 is loaded, it is found t h a t secondary voltage at no
load and full load rem ains unchanged. D eterm ine th e tra n sfo rm e r loading u n d e r ra te d c u rren t conditions.
Neglect the m agnetizing current. lAna. pf = 0.905 leading, 4.525 kVAl
1.3 3 . (a ) For constant values of load voltage and load pf, obtain th e condition for m axim um efficiency in a
1-phase transform er. Hence, derive therefrom th e relation

(k V A U , , = (kVAW , V m J S S L ,

(6 ) The leakage im pedance of a 10 kVA, 3300/220 V tra n s fo rm e r as given on its n am e plate is 10


Determ ine (i) th e voltage th a t m u st be applied on HV side to circ u la te ra te d c u rre n t w ith LV side shorted and
(ii) the m axim um possible voltage regulation. [Ans. (i) 330 V ; (ii) 10*1
1.34. (a) A non-sinusoidal voltage v = 150 sin 314 t - 75 sin 1 5 7 0 1 is a p p lied to th e 250 tu rn winding of a
transform er. Find th e core flux as a function of tim e.
(b) A voltage v = 200 sin 3 1 4 1is applied to th e tra n sfo rm e r w in d in g in a no-load te s t. T he resulting current
is found to be i = 3 sin (314 / —60°). D eterm ine th e core loss a n d r.m .s. v a lu e of th e exciting current.
(c) A tran sfo rm er h as the following te s t d a ta :
T est No. 1 : 100% voltage, 6 % c u rre n t, p f = 0.2
T est No. 2 : 8 % voltage, 100% c u rre n t, p f = 0.3
Identify th e tests. C alculate efficiency and p ercen tag e re g u la tio n a t full load p p f a n d a t 0 .8 pf lag g i^
[H int, (c) core loss = 0.012 Full-load ohmic loss = 0.024 * = 0.08 etc.)
[Ans. (a) ( - 1.911 cos 314 t + 0 1911 cos 1570 tll« *
(5) 150 watts, 2.122 A. (c) 96.525%, 95.694%; 2.4%.
1.35. (a) Describe briefly th e various losses in a tra n s fo rm e r a n d e x p la in how each loss varies with th
load cu rren t. r
(b) E xpress th e total core loss in te rm s of voltage an d frequency.
k !v?6f (n } A tranflfor” *®r 18 °Pe ra t®d a t ra ta d frequency b u t a t a v o ltag e h ig h e r t h a n its ra te d value.
how th e following q u a n titie s would change : a s * * «»«*• •
(i) No-load current. (ii) Hysteresis loss .n d m Eddy cu rren t loss.

Scanned by Cam Scanner


prob- H _______________ Transformers 151

(ft) A 200 V, 60 Hz single-phase tra n sfo rm er h a s h y steresis and eddy c u rre n t losses of 250 w a tts an d 90
watts respectively. If th e tra n sfo rm e r is now energised from 230 V, 50 Hz supply, calculate its core losses.
Assume Steinm etz’s c o n sta n t equal to 1.6.
[Ans. (a) (i) Increases (ii) Increases, PAoc (V)1 (iii) Increases, P t « (V)2 (ft) 487.83 watts.I
1 .3 7 . A 1 -phase, 11/66 kV, 50 MVA tra n sfo rm er h as prim ary and secondary leakage im pedances of
0 .01 + j0-0S per u n it each. The s h u n t branch im pedance in p.u is 80 + y 0 in p arallel w ith 0 +j 90. All q u a n titie s
are in Per u n *^ on tru n sfo rm e r base. C alculate, in ohms, th e p a ra m ete rs of p rim ary and secondary w indings
and the sh u n t b ran ch referred to low-voltage side.
[Ans. r, = r2' = 0.0242 D ; x, =x2' = 0.1452 Q ; Rc = 193.6 D, Xm = 217.8 fl]
1.38. Fig. 1.88 illu s tra te s th e eq u ivalent circuit 04sn om
referred to the l.v. side of a 400/3200 V, 1-phase, 50 Hz, -'7JttP* 'WvV~
16 kVA transform er. W ith p rim ary voltage held fixed at
400 V, the h.v. is loaded to its ra te d c u rre n t a t p f 0.8 lag
at its term inals. C om pute
400 V 800 n
(a) the term inal voltage a t the h.v. term inals
(ft) the l.v. input current and its pf and
(c) ihe efficiency.
[Ans. (a) 3062.4 V (ft) 40.635 A, 0.78316 lag (c) 96.229%)
1.39. (a) S uggest a su ita b le te st to predeterm ine the Fig. 1.88. Pertaining to Problem 1.38.
regulation of a tra n sfo rm e r an d explain it.
(ft) A 200 kVA, 1-phase, 50 Hz, 2300/230 V tran sfo rm er has a core loss of 300 W. At full load, I2R loss is
800 W and the leakage reactance is 0.032 p.u. C alculate the total loss and the voltage regulation for (a) 1.2
p.u. load a t p f 0.8 lagging and (6 ) 0.70 pu. load a t p f 0.6 leading.
IAns. (a) 1452 W, 0.02688 p.u. (ft) 692 W, - 0.01624 p.u.)
1.40. A single-phase load is fed through a 6 6 kV feeder of im pedance 100 + j 320 ohms and a 6 6 / 6 .6 kV
transformer of eq u iv alen t leakage im pedance 0.3 + j 1.2 ohms referred to l.v. side. For a load of 200 kW a t pf
0.8 leading a t 6 .6 kV, calculate
(a) voltage a t th e sending end of the feeder,
(ft) voltage a t th e p rim ary term in als of th e tran sfo rm er and
(c) active and reactive powers and p f a t th e sending end of the feeder.
IA ns. (a) 65394 V (ft) 65818 V (c) 201.8654 kW, 143.6865 kVAr, 0.8147 leading]
1.41. A 100 kVA, 50 Hz, 440/11000 V, 1-phase transform er has an efficiency of 98.5% w hen supplying
full-load current a t 0.8 p f a n d an efficiency of 99% when supplying half-full load c u rren t a t u nity pf. Find th e
iron losses and ohmic losses corresponding to full load current. A t w hat value of load cu rren t will th e m axim um
efficiency be a tta in e d ? [Ans. 267.37 W, 950.93 W, 120.523 A| \I.A.S., 1987]
1.42. (a) W hat a re th e approxim ations involved in obtaining th e equivalent circuit p a ra m e te rs from
open-circuit and short-circuit te s ts ? Discuss th e validity of these tests.
(ft) Discuss th e effect of frequency an d voltage on th e equivalent circuit p aram eters of a tra n sfo rm er.
(Hint, (ft) F requency effects all the p aram eters, w hereas voltage effects the exciting im pedance only.l
1-43. (a) D istinguish betw een d istrib u tio n and power transform ers
(ft) Define energy efficiency and all day efficiency.
(c) The daily v a ria tio n of load on a 1 0 0 kVA tran sfo rm er is as follows :
8.00 A.M. to 1.00 P.M. 65 kW, 45 kVAr
1.00 P.M. to 6.00 P.M. 80 kW, 50 kVAr
6.00 P.M. to 1.00 A.M. 30 kW, 30 kVAr
1.00 A.M. to 8.00 A.M. No load.
, This tran sfo rm er h a s no-load core loss of 370 w atts and a full load ohmic loss of 1 2 0 0 w a tts. D eterm ine
e aN-day efficiency of th e transform er.
,H>nt. From 8.00 A.M. to 1.00 P.M.
kVA = V( 65)*+ (45?"= 79.06
2
Ohmic loss = j x 1 2 0 0 = 750.06 W] [An*. 9 7 .9 5 9 %]

Scanned by Cam Scanner


152 Electrical Machinery

“ £ 2 S ^
are halved and the new transform er is energised from 2 2 .UUU v , oo « s s S yy^ j 5 s k S S uers have
k ? sthe
same core m aterial and the sam e lam ination thickness.
Calculate the flux density and core losses for the new tra n s orm er.
N\
(Hint. V1='l2nfi BmlAiNi and V2=<2nfxBm2 • 4A, • -5 -

. Y l 11.000 - - g g L c t c l (Ans* 12 T. 24,000 watts)


" Vt ~ 22,020 " 2 B m2 6 -1
1.45. (a) Describe th e m ethod by which the separation of th e core losses of a tra n sfo rm er is achieved.
• (b) In a te st for th e determ ination of losses of a 440 V, 50 Hz tran sfo rm er, th e to ta l iron losses were found
to be 2500 w atts a t norm al voltage and frequency. W hen th e applied voltage an d frequency were 220 V and 25
Hz, th e iron losses were found to be 850 w atts. C alculate th e eddy c u rre n t an d h y steresis losses at normal
voltage and frequency. [Ans- 1600 W atts- 90011watts]
1.46. (o) How can the polarity test be perform ed on a single-phase tw o-w inding
transform er? Explain how the subtractive polarity is preferable to additive p o la n ty in
transform ers.
' (ft) Fig. 1.89 illustrates four term inals AB and CD of a tw o-w inding tra n sfo rm er.
W hen a voltage of 100 V is applied across AB w ith BD sh o rt circuited, voltage across
term inals AC is found to be 100 V. Find the polarity and tu rn s ratio of th e tra n sfo rm er.
[Ans. A and C (or B and D) have same polarity.
Turns ratio from AB to CD is 1 : 21.
1.47. (a) In each of th e diagram s of Fig. 1.90, th e in sta n ta n e o u s p o larity of th e F ig . 1 .8 9 . P erta in in g
prim ary winding of an ideal transform er is as shown. Indicate : t o P r o b l e m 1 . 4 6 (b).
(i) the direction of flux in th e core. ,

^C- (d)
Fig. 1.90. Pertaining to Problem 1.47 (a).

(») th e p o larity of th e in sta n ta n e o u s voltage induced in th e secondary.


' (Hi) th e directions of the in sta n ta n e o u s c u rre n ts ex istin g in i k .
SSSL “ addSeC0"da,y W in d in e S w h sI1 — — * ■ » is connected S i ,# J » I

10 0 2 d V t£ ! 20° '
A, find the voltage that develops across x y . as shown I Fig’ m “
M u . fa) A (i) Clockwise (ii) d positive and c negative (iii) From s to b and
from c to d. Fig. 1.91. P e n n i n g to
Problem 1.47 (b)-

Scanned by Cam Scanner


Transformers 153

B (i) Clockwise (ii) c positive and d negative (iii) From a toband from d to c.
C (i) Anti-clockwise (ii) c positive and d negative (iii) From a toftand from d to c.
D (i) Anti-clockwise (ii) positive and c negative (iii) From a to6and from c to d. \a
(b) 20 volts.)
1.48. (a )In 10 back te s t>show to®1 one transform er may have slightly less tem perature rise than the other.
(b) In S u m p n er’s te s t, th e re a d in g of th e w a ttm e te r recording the core losses rem ains unaffected w hen
voltage is injected in th e secondary series circuit. Explain.
low
(c) In S u m p n er s te s t, th e frequency of th e voltage injected in the secondary circuit, m ay not be equal to
rated frequency. E xplain.
the
[(c) H in t. Secondary sh o rt-circu it c u rre n t Ise = H ere 7,c is made equal to th e rated cu rren t for any
^8C
value offre<Iuency m erely by a d ju stin g V. Since th e ohmic losses are equal to (Itc)2 x (Resistance), some variation
of frequency from ra te d v a lu e , will not v ary th e w inding resistance and, therefore, the ohmic losses.)
1.49. (a) T h e te m p e ra tu re of a tra n sfo rm er determ ined under either no-load condition or short-circuit
condition, does n o t give tru e te m p e ra tu re rise experienced in practice. Discuss this and explain a suitable
method of d eterm in in g th e te m p e ra tu re rise of a transform er.
(ft) Two sim ila r 40 kVA, single-phase tran sfo rm ers gave the following results w hen tested by the back to
back method : ^
lVj in the supply line : 800 w atts.
W, in the secondaries series circuit a t rated current : 1000 watts.
Calculate th e efficiency of each tra n sfo rm er a t u nity p.f. [Ans. (6) 97.8C
.
1.50. (a) Define a n a u to -tra n sfo rm e r. D istinguish clearly th e difference between a resistive potential divider
and auto transform er.
(fc) A 200/100 V, single-phase tran sfo rm er is rated 10 kVA. If this transform er is connected as an
auto-transformer across a 50-V supply, w h a t is th e m axim um output voltage th a t can be obtained from th is
transformer? If th e load c u rre n t is now 8 A, determ ine th e kVA output and the currents in th e various p a rts
of the system.
[Hint. (b) T he w in d in g designed for 100 V should be connected across 50 V supply.)
[Ans. 150 V, 1.2 kVA, input current = 24 A)

1.51. (a) D iscuss th e re la tiv e m erits an d dem erits of an auto-transform er.


(6) A 400/100 V 5 kVA. sin g le-p h ase two w inding tra n s lo n n e r is to be used os an au to-transform er to
su p p ly 400 volts from a 500 V voltage source. W hen tested as . two-winding transform er a t ra te d load a n d 0.8

Pf' ^ “ kVA” ^ t ^ - n ^ L n s f o n n e r . Also calculate the transform ed kVA an d c o n d u c e d


kVA
(ii) Find its efficiency as a n a u to -tra n sfo rm e r a t ra te d t a d 2„ kvA 98 95gft|

1.52. (a) Show t h a t in case of an au to -tran sfo rm er

<M A 10,000/5000 V sin g le-p h asc transforTO e^^rated^a^lO O ^kV A ^as^a^tw ^^^ ^ o p e ra tt it as
desired to connect th e tw o w indings of th e ira n th e sam e w h a t are the possible voltage ratios and
auto-transform er. E x p la in th e connections [An*, (ft) (0 Voltage ratio : 1/3, 150 kVA o u tp u t
the corresponding m ax im u m o u tp u ts? ((,■) Voltage ratio : 2/3, 300 kVA output]
, 41. -nw p r tra n sfe r from prim ary circuit to th e secondary circuit is p a rtly
1.53. (a) In a n a u to -tra n sfo rm e r, th e p . .. j u stify th is statem en t.
by transform er action a n d p a rtly by direc co ■ transform er is to be used as an a u to -tran sfo rm er
(ft) A 11000/2200 V, 100 kVA, single-phase t ibIebvalues of voltage ratios a n d kVA outputs,
hy connecting th e tw o w indings in series. u>v ^ 13200/2200 V or 2200/13200 V; kVA = 120
13200/11000 V or 11000/13200 V; kVA » 600)

Scanned by Cam Scanner


r
-I
I
154 Electrical Machinery IProb. l
1.54. An ideal tran sfo rm er h a s its secondary w inding labelled as a , b, c. T he p n m a r y h a s 100 turns. The
num ber of tu rn s on th e secondary side are 400 betw een a, b an d 200 b etw een b, c so t h a t to ta l tu rn s between
a, c are 600. The tran sfo rm er supplies a resistive load of 6 kW betw een a a n d c. In ad d itio n , a load of impedance
1000 Z45° Q is connected betw een a a n d b. For a p rim a ry voltage of 1000 V, com pute th e p rim a ry current and
the prim ary input. [An*. 20.691 A, 17.32 kW]
1.55. A single-phase ideal tran sfo rm er h a s 400 p rim a ry tu rn s an d 600 seco n d ary tu rn s . The prim ary i$
connected to 800 V source. The full secondary h a s a resistiv e load of 24 kW . T he seco n d ary is tapped a t 500
tu rn s across which th ere is a pure inductive load of 20 kVA. F ind th e p rim a ry c u rre n t, its pow er factor and
the power in p u t to prim ary. [Ans. 39.051 A, 0.7682 lagging, 23.999 kW]
1.56. (a) Two resistive loads R x and R 2 each of 200 fi are to
be supplied w ith voltages of 100 V and 300 V respectively, from
the available 400 V supply. To th is end, one au to -tran sfo rm er
connected as shown in Fig. 1.92 is designed.
Assum ing th e auto-transform er to be ideal, calculate th e
currents in its windings. W hat is the VA ra tin g of a two w inding
transform er which would have approxim ately th e sam e size as
the auto-transform er.
(ft) It is proposed to connect R x betw een term in als d and a
suitable tap c on th e winding, instead of across a and ft, could
it lead to a more economical design of the auto-transform er?
Explain. (I.E.S., 1979)
[Ans. (a) C urrents in the windings are as shown in Fig. 1 93
(a) .75 VAI

(6) W ith th e proposed connections, the cu rren ts in th e w ind­


ings are as shown in Fig. 1.93 (ft).

a n d HlnldCltk T e f f W0uld n o t be e c o n o m ic a l. because th e w indings h av e to h a n d le m ore currents


and would, therefore, be thicker m cross-section].
1 25A
0 5A 1 '2 5 A

0 -75A T100V 200n.

1A
400 V 1-5A

c> 300V > 200n


1 ° 20on
0 2 5 * 9
1
L J
1-25A
_ 1
1-5A

1-5A
(a)
Pig. 1.93. Pertaining to the eolution of Problem 1.56. (6)
1.57 A single-phase transformer has three w in d in g , .
supply voltage a v .i,.b ,e ie 300 V, 50 Hr, then d eterm in e V > U 2 Z $ S . “ l ^

1.53. fa) Why ie it preferable to inetall two „r m ore tr a n „f ^ ^ V’ 1M V’ 2° ° V’ and 450 V’ 600 V’ 900 ^
(4) Explain why parallel operation of transformers is n e c e s I Z " m P8raUel ‘han °" e Iarge “” it?

formers may be operated in p a r s e r " 111' Condit,on“ whlch should be satisfied before two single-phase trans-

o p e r l g Ufa the circulating current is


zero ui two single-phase transformers

Scanned by Cam Scanner


Transformers 155
prob. 11

1.59. (a) D iscuss th e effect of circulating cu rren t a t no-load, in two single phase tran sfo rm ers o p erating in
p arallel-
(b) Two tra n sfo rm ers o p e ra tin g in p arallel have different reactance to resistance ratios. Show th a t one
transformer o p erates a t a b e tte r p.f, th a n the other.
(c) Show th a t if two tra n sfo rm ers have sam e p.u. im pedances, they will sh are the load in proportion to
their kVA capacities.
1.60. (a) D iscuss how th e p a ra lle l operation of two single-phase transform ers is effected by
(i) unequal voltage ratio s and
(iii) unequal p e r u n it leakage im pedances but sam e xe/re ratio.
ii I
(6) Two single-phase 500 kVA and 400 kVA transform ers are connected in parallel to supply a load of 800
kVA at 0.8 pow er factor lagging. T he resistance and leakage reactance of the first transform er are 2.5 percent
an d 6 percent respectively an d of th e second transform er 1.6 percent and 7 percent respectively. C alculate the
kVA loading a n d pow er factor a t w hich each transform er operates.
(Ans. (6) 465.6 kVA at pf = 0.84 lag and 337.2 kVA at pf = 0.737 lag)
I
1.61. (a) D iscuss how th e p arallel operation of two single-phase transform ers is effected by
x
(j) unequal — ra tio b u t th e sam e per u n it leakage impedances,
re
(ii) w rong p olarities.
(b) Tw o-single-phase tra n sfo rm ers operating in parallel have the following data :
T ransform er A : zea = 0.2
+j 0.6 D, Ea = 208 V
Transform er B : ze&= 0.25 +j 0.55 Q, Ef, = 205 V
Load im pedance Z - 3 +j 2 Q.
Find (i) voltage across load
(ii) c u rre n t a n d pow er delivered to load and
(iii) power sh av ed by each transform er.

[H in t. (6) C alcu late i + + 1 = 3.464 Z - 65.87- ; J + g = 667.08 Z - 68.51- and use Eqn. (1.78) for
"ea ^eb
F —V _ _ _
— and If,. Load current / = /„ + If, etc.]
load voltage V. F u rth e r, use Eqn. (1.73 (a)) for Ia =
{Ans. (6) (i) 192.725 V (ii) 53.31 A, 8554.6 W (iii) 4221.58 W. 4332.78 W1
1.62. (u) T w o tra n sfo rm e rs to be operated in parallel should have the sam e capacity. Ju stify w h e th e r th is
statem ent is correct or not.
. cVmrt-rircuit tests on four 2000/200 V single-phase tran sfo rm ers :

Short-circuit data
Transformer Rating vr SC p
1 sc

125 V 48 A 1100 W
A
A
100 kVA
117 V 90 A 2050 W
B 200 kVA
147 V 140 A 3800 W
r* 300 kVA
156 V 190 A 5000 W
D 400 kVA
~ , . , . 4 would operate m ost successfully in p arallel ?
Out of th ese, w hich two tra n sfo rm e ^ Transformer3 A and fl)
Ir-M
, . , U tra n sfo rm ers w ith u n equal voltage ratios a re operatin g in parallel. Discuss,

w it h '^ M p o ^ h S a ‘ sh° red by each and their operatine powcr factors- state

“ ' T Z iT h t e . 6 6 0 0 /2 2 0 0 V tra n sfo rm ers h av e th e following short-circuit d a ta ;


Transformer A (200 kV A ): 62 V, 30.3 A, 370 W r
Transformer B (400 kV A ): 66 V, 60.6 A, 650

Scanned by Cam Scanner


156 Electrical Machinery________ _________ __________________— — -— 1

T hese transformers are put in parallel and connected to a load of 0 8 p f l»* *■«> P " m , r v v » l . . „ held ^
a l 6600 V .
(i) C alcu late th e m a „ m u m kW load th a t can be su p p h ed by th e s e tr a n s f o r m e r , w .th o u t c .u n n y rh. ^
voltage to fall below 2150 V
(ii) For th e condition of p a rt (i), find kVA sh a re d by each tra n sfo rm e r
IH in t. <fc) («) C u rre n t sh a re d by tra n sfo rm e r A = 32 765 A
10 ♦ - - x 400 U 0 8 lAna. lb) 0) 498 08 kW ( n » 216 2- k \ A. 406 33 kV'Aj
M axim um kW load =
30.3 ” 66 J .
1.64. (a) E xplain why in testin g large transform er* th e o p e n circuit te s t is e a rn e d out w ith the high.Vo|U f,
w inding open and th e short-circuit test w ith th e low-voltage w in ing s or e
(b) A 100 kVA tran sfo rm er having 1^ resistance and 4 * leakage re a c U n c e is o p e ra te d in parallel with »
200 kVA tra n sfo rm er having 1* resistance and 6 * leakage rea c ta n c e If th e to ta l load delivered is 300 k \ A
a t u nity p f . calculate the kVA load on each tra n sfo rm er a* well as th e o p e ra tin g p f of each transform er
E S . 19?s
|Ana. (h) 127.445 kVA at p f 0 9989 leading 172 776 kVA at p f 0 9994 lagg,-*

1.65. (a) Discuss th e conditions necessary for th e successful p arallel o p eratio n of sin g le-p h ase transformers
How can you check these conditions?
(5) Two single-phase tran sfo rm ers in parallel supply a load of 500 am p e res a t 0 8 p f lagging and at 40C
volts T heir equivalent im pedances referred to secondary w indings a re (2+j 3 i ohm s and (2 5 * ./5 ) ohm*
C om pute the cu rren t and kVA supplied by each tra n sfo rm e r an d th e c o rresp o n d in g pow er factors
IAns. (6 ) 304.51 A, 121 804 kVA and p.f. 0.8282 lagging ; 196 431 A. 78 572 kVA and p f 0 7524 lagging
1.66. D eterm ine the largest value of load th a t can be delivered by th e p a ra lle l com bination of two
transform ers described in Problem 1.64 (6 ) w ithout overloading any of them . !Ans. 235 56 kVAi
1.67. (a) Two single-phase tran sfo rm ers having id entical voltage ra tin g s but u n e q u a l kVA ratings are to
be operated in parallel. U nder w h at conditions can th e tw o tra n sfo rm e rs su p p ly a to ta l kVA equal to the sum
) of th eir individual kVA ratin g s w ithout any tra n sfo rm e r g e ttin g overloaded'’ D en v e th e conditions mentioned
(b) Two single-phase, 11000-440 V tra n sfo rm ers have kVA ra tin g s of 200 kVA a n d 100 kVA respectively
The equivalent resistance and reactance of th e 200 kVA tra n s fo rm e r a re resp ectiv ely 1 0 and 5 0 ohms, wher.
referred to the 11,000 V side. The equivalent reactance of 100 kVA tra n s fo rm e r re fe rre d to th e 11.000 V side
) is 9 0 ohms.
(i ) W hat should be th e equivalent resistance of 100 kVA tra n s fo rm e r if each tra n s fo rm e r is to supply
in proportion to its kVA ratin g w hen operated in p arallel?
(11) W hat is th e m axim um combined kVA th a t can be su p p lied by th e tra n s fo rm e rs in parallel w ithout
overloading any one tran sfo rm er ? (J E S 19?$
H in t, tbt fi) ~ V 3 ± H :,1 2 0
tb r *+ 9 ^ 200

(11) Since z,b = 2 r ,0, each tra n sfo rm er sh a re s load equal to its kVA r a tin g a n d b e c au se ~
the an*
1 r r

brlwveen
een /„
/„ and
...d / , IF.* 1 50 (Ml, or b rtw een 100 kVA an d 200 kVA >, |*_ - * ,) T h u , th e m u i m l l comho-J
kVA uis mvftn
given hvby u p
V|200 ♦ 100 cos ■ 4 fc)|7 ♦ llO O .in ^
* V|2<)0 * 100 cos 16.74I7 ♦ 1100 sin 16.74I3 - 297 16 kVA)

A ns. ,o ) «) . ? * a n d (U ) ^ (fe) ( |) 4 ? 9 6 o h m > ( u ) 297 16 kVAj

1.68 (a) D e te n t* th e principle of reg u la tin g th e voltage w ith th e h e lp o f ta p c h a n g e r .


b D u c u s . w h e th e r th e w inding should he tap p e d on h v side or on I v side
l Wmding 1,0 U ^ d
form ers ? ° " P ch*"ll«r be connected n e a r th e n e u tra l? W h a t a b o u t delta-connected

(h) W hat is th e difference betw een no-load an d on-load ta p -c h a n g e r. ?


<r) Describe no load ta p -c h an g e r w ith a su iU h ln d ia g ra m

Scanned by Cam Scanner


J
r __________________ Transformers 157
prflb'
© (a) W hat p recau tio n s should be observed d u rin g th e operation of on load ta p -c h a n g e rs ?
((j) Explain ^u n c^ on centre -ta p p e d rea c to r in on-load ta p changer.
((.) Describe one type of on-load tap -ch an g er, w ith proper sequence of operation for ch an g in g th e voltage.
71 A 11/33 kV, d e lta /s ta r tap -c h an g e r tra n sfo rm er supplies a star-connected load of 1200 kW a t 0.8 p.f.
p r.phase leakage im pedances for l.v. an d h.v. w indings are respectively 0.5 + j 5 ft a n d 2.0 + y l5 ft. E ach
Lductor has im pedance of 2+/5 ft. If l.v. w inding is energised a t 11 kV, find th e ta p -c h a n g e r s e ttin g to
li°e c|oa(J germinal voltage of 33 kV. |A na. Tap up by 3.5%1
pve
1 72. (a) How does the principle of operating induction regulator differ from th a t of the tap p ed -tran sfo rm er ?
(fr) In induction reg u la to rs, explain why prim ary w inding is placed on rotor and seco n d ary w in d in g on
sWtor ? . .
(C) Why is it esse n tia l to have a com pensating w inding in single-phase induction re g u la to r ? D escribe a
^le-phase induction reg u la to r w ith su itab le diagram s.
1.73. (a) For a single p h ase induction regulator, draw a phasor diagram for its cu rren ts,
tfc) Describe a th re e -p h a se induction reg u la to r w ith diagram s w herever necessary.
(c) How does a th re e -p h a se induction reg u lato r differ from a single-phase induction reg u lato r?
1.74. (a) Discuss th e rela tiv e m erits and dem erits of induction voltage regulator over tap -c h a n g e r t r a n s ­
former. _
(b) A single-phase induction reg u lato r h a s an effective tu rn s ratio of 4 : 1. If the supply voltage is 240 V,
determine the m axim um and m inim um values of output voltages.
If the load c u rre n t is 20 A and th e secondary w inding is set 60° away from the m axim um boost position,
calculate the prim ary w inding c u rre n t and th e prim ary line current. Effect of no load c u rre n t m ay be ignored.
[Ans. 300 V and 180 V; 2.5 A, 22.5 A)
1.75. Derive th e tra n sfo rm er equivalent circuit, tre a tin g it as a m agnetically coupled circuit.
1.76. (a) A single-phase tran sfo rm er, w ith its secondary w inding open, is energised from 220 V d c . I t is
assumed th at exciting c u rre n t in prim ary w inding is less th an its rated current.
If a small portion of th e iron core is rem oved by m aking appropriate cuts in the iron core, discuss w h a t
would happen to (i) exciting c u rre n t in the prim ary w inding and (ii) th e m agnitude of core flux.
(6) A two-winding 2000/200 V tran sfo rm er has the following param eters :
L, = 15 H, / , = 0 5 H, f2 = 0.01 H
(i) Calculate th e m u tu al inductance of th e transform er.
(ii) Find the self-inductance of the secondary winding.
[Ana. (a) Exciting current remains constant but core flux is reduced’
(6) (i) M = 1.45 H (ii) L2 = 0.155 HI
1.77. (a) Define m u tu al inductance and discuss th e m ethods of increasing it betw een two circuits.
(b) Show th a t if two circuits are linked w ith the sam e m agnetic circuit, the m utual in ductance is
W= *VLjL2, w here k is th e coefficient of coupling and L x, L2 are the self-inductances of the two circuits.
. 1-78. (a) Define self-inductance, m agnetizing inductance and leakage inductance. Express self-inductance
® terms of the other two.
(6) Define co-efficient of coupling. How can it be increased in two m agnetically coupled coils ?
1.79. An air-core tra n sfo rm er gave th e following d a ta for two tests conducted on i t :
Test 1. W hen 20 volts a t 400 Hz is applied to w inding 1 w ith w inding 2 open-circuited, th e c u rre n t an d
Power in winding 1 are 2A and 20 w a tts and voltage across w inding 2 is 15 V.
. Test 2. W hen 24 V a t 400 Hz is applied to w inding 2 w ith w inding 1 open-circuited, th e c u rre n t a n d pow er
10 ^ d m g 2 are 1.5 A and 18 w atts.
(a) Find the values oT r1( r2, L x, and M.
M>) Winding 1 is energised from a 30 V, 400 Hz source and w inding 2 is connected to a load of 20 ft
Find th e c u rre n t in w inding 1. lAns* (o) 5 Q, 8 Q, 1.S9 mH, 3.183 mH, 2.984 mH (6) 3.664 A)
An audio-frequency tra n sfo rm er h as p rim ary to secondary tu rn s ra tio of 30. T he in d u ctan ce m e a su re d
Primary w ith secondary open is 20 H and w ith secondary short-circuited is 0.2 H. T he tra n s fo rm e r
resistances a re 20 ft each m easured on th e prim ary side.

Scanned by Cam Scanner


_________ [Prob. 1
158 F lectn c?1 M achinery ------ --------------------------------------------
— -------- ---------------------------- ---------------------------- 77 n to a v a ria b le -fre q u e n c y v o lta g e source whose
This tra n sfo rm er is used to couple a load o resis th e follow ing :
in te rn a l resistance is 5000 0. F or th is a rra n g e m e n t, com pute
(a) Upper half-power frequency.
(b) Lower half-power frequency.
(cl Geometric mean of these frequencies frequency of p a rt (c). .
(d) The ratio of load voltage to source voltage a t t ^ (q) g30? g Hz (fc) 20.74 Hz (c) 415.1 Hz (d) 0.01724]

r em o, W Wo im pedance. I f it is connected as a 500/400 V


1.81. A 100/400 V, 2 kVA single-phase ^ centage im pedance as a n auto-transform er,
auto-transform er, deduce from fundam en s l [/.A.S., 1994] [Ans. 10 kVA, 1%]

, , j i ad m o Q from a voltage source w h ic h can be re p re se n te d by an


1.82. An ideal tran sfo rm er feeds a load of 100 12 from ;a vo g
in tern al e.m.f. of 10 V w ith an in te rn al resistance of 2000 12.
(al Find the tu rn s ratio so th a t maximum pow er is tra n s fe rre d to load.

W U nder the cond,tion of p a r t f i n d th e c u rre n t, voltage a n d ^ “ g^ 1 J lg y _^ ^

1.83. A two-winding tran sfo rm er is used as a n au to -tra n sfo rm e r. I f ^ - k, th e n d e riv e th e following

re la tio n :
V^=kV2+I1 [*! + ( * - I ) 2 Z2I
where Zj = rj +jxj and z2=r2+jx2■State the assum ptions made.
1.84. (a) For a single-phase auto-transform er, derive, from ab-initio, th e re la tio n b e tw e en in p u t power,
conducted power and transform ed power in term s of high a n d low voltages.
. (6) Discuss the advantages and disadvantages of a n a u to -tra n sfo rm e r a s c o m p a red to a two-winding
transform er.
1.85. A 2400/240 V, 200 kVA, 1-phase tran sfo rm er h a s a core loss of 1.8 kW a t r a te d voltage. Its equivalent
resistance is 1.1 percent. Find the tran sfo rm er efficiency a t p f of 0.9 a t full load.
Find also th e kVA load a t w hich m axim um efficiency occurs. C alcu late, also, th e m ax im u m efficiency at
pfs of 0.8, 0.9 and unity. [Ans. 97.826%, 181 kVA, 97.574%, 97.838%, 98.05%]
1.86. In a 1-phase transform er, zero-voltage regulation occurs a t p f o f 0.94 lea d in g , w h e re a s its maximum
possible voltage regulation is 4%. Find its voltage regulation a t p f 0.8 lagging.
If its ohmic losses are 136.5 W, find kVA of th e tran sfo rm er. [Ans. 3.348%, 10 kVA]
1.87. For a 2000/200 V, 20 kVA single-phase tran sfo rm er, th e load c u rre n t a t w h ich m ax im u m efficiency
occurs is 90 A. Its equivalent resistance is 0.015 p.u. F ind its h a lf-ra te load efficiency a t p f 0.8.
Also, calculate its voltage regulation a t full load u n ity p.f. [Ans. 96.177%, 1.5%]
1.88. For a 2300/230 V, 1-phase transform er, m axim um possible voltage re g u la tio n is 5% a n d occurs at a
p.f. of 0.25. Find its load voltage a t full-load pow er factors of 0.8 lea d a n d 0.8 lag. (Ans. 234.38 V, 220.98 V]
1.89. A single-phase transform er has eddy c u rre n t loss of 100 W a n d h y s te re sis loss of 120 W. For the
sam e supply voltage if both lam ination thickness and o p e ra tin g frequency a re re d u c e d by 10%, find the new
value of core losses. Take Steinm etz’s co n stan t = 2. 214 33 W]
1.90. (a) For a 1-phase transform er, equivalent resista n c e is 1.5%.I ts p e r u n it core loss is 0.01. Find
efficiency of th is transform er a t p.f. 0.8.

(6) A 1-phase tran sfo rm er h a s th e ratio ~ = 3 a n d eq u iv a len t re s is ta n c e is 1.5%. F in d th e full-load p.f. at


which voltage regulation is 4%.
Find th e m axim um M l-load efficiency of t h i , tra n s fo rm e r a t th e p.f. c a lc u la te d above.
. [Ans. (a) 96.97% (b) 0.7764 lag, 9 6 .28 %)

w ithA (1/ i ^ = a20ato1a b a d Uo f ^ J ^ G T ^ t m ^ ^ o L ^ o s f ° f ^ ? ° ° fl d eliv ers P °w er th ro u g h a transformer


lower half-power f l u e n c i e s of ‘ ^
(a) th e p rim a ry self-inductance and

Scanned by Cam Scanner


Transformers 159
prob. 1]

(fc) the leakage in d u ctan ce on th e p rim a ry side. (Ans. (a) 5.968 H (6) 0.0637 HI
1.92. (a) Describe four possible w ays of connections of 3-phase tra n sfo rm ers w ith re le v a n t rela tio n s am o n g st
voltages and c u rre n ts on b o th h.v. a n d l.v. sides.
(b) A ban k of th re e single-phase tra n sfo rm ers h a s its h.v. term in als connected to 3-w ire, 3-phase, 11 kV
system. Its l.v. te rm in a ls a re connected to a 3-wire, 3-phase load ra te d a t 1500 kVA, 2200 V. Specify th e voltage,
current and kVA ra tin g s of each tra n sfo rm er for both h.v. and l.v. w indings for th e following connections :
(i)Y - A (ii) A - Y (iii) A - A (in) Y - Y .
The first sym bol in d icates h.v. w inding and th e second symbol l.v. w inding.
[Ans. Each transform er rating = 500 kVA (i) 6351/2200 V, 78.37/227.27 A (ii) 11000/1270.2 V, 45.45/393.64 A
(iii) 11000/2200 V, 45.45/227.27 A (iu) 6351/1270.2 V, 78.37/393.64 Al
1.93. (a) E xplain th e reaso n s why d istrib u tio n transform ers are designed for m inim um core loss r a th e r
than ohmic loss.
(b) For th e circuit show n in Fig. 1.94, find 20/6(f Cl N ,: N,
(i) the tu rn s ra tio from p rim a ry to secondary for m ax­ O
imum power tra n s fe r (mpt ) to load,
O
(ii) the value of Xc in £1 for mpt, O )G> 2 /3 6 -8 7 ° n
O
o
@ 2°/0°
(iii) load voltage u n d e r mpt, >o
o
o<
(iv) m axim um pow er delivered to load. o
2 1
Ideal
( N l)
H in t. (b) (i) 10 = x 1 . 6 £2 e t c . tran s.
N2
\ /
Fig. 1.94. Pertaining to Problem 1.93 (6).
[Ans. (6) (i) 2.5 (ii) 3.9712 £2 (iii) 5 Z36.870 V (io) 10 W)
1.94. The efficiency a t u n ity pf, of a 6600/384 V, 200 kVA single-phase tran sfo rm er is 98%, both a t full-load
and at h alf full-load. T he pow er factor a t no-load is 0.2 lagging and th e full-load regulation a t a lag g in g p f of
0.8 is 4%. Draw th e e q u iv alen t circuit refe rre d to LV side and in sert all values. [I.E.S., 2002)
[Ans. Rc = 108.381 C l,X m = 22.124 £2, rt2 =0.01 Cl, xe2 = 0.03578 £21

1.95. (a) D iscuss th e points of sim ilarity in transform ers and ro tatin g electrical m achines.
(b) W hat is th e m ajor cause of noise in tran sfo rm ers ? Discuss th e steps which m ay help in red u cin g it.

I.

Scanned by Cam Scanner


• !
— — ■
■_____
■ '—

E le c tro m e c h a n ic a l Energy
C on version Principles
C onversion of o th e r form s of e n e rg ies in to e le c trica l e n e rg y is a com m on practice^ The main
a d v a n ta g e of th is conversion is t h a t en e rg y in e le c tric a l form c a n b e tr a n s m itte d , utilised and
c ontrolled m ore easily, re lia b ly a n d efficiently. E n e rg y c o n v ersio n dev ices a re required first for
c o n v ertin g o th e r form s of e n erg ies in to e le c trica l e n e rg y a n d th e n fo r c o n v e rtin g electrical ener­
gy in to th e re q u ire d u se fu l form s, su ch a s s o u n d , lig h t, h e a t or m e c h a n ic a l energy. In other
w ords, en erg y conversion devices a re re q u ire d a t b o th e n d s of a n e le c tric a l sy stem , since enerp
is n e ith e r a v ailab le a n d n o r re q u ire d in e le c trica l form . O ne ty p ic a l ex a m p le illustrating the
p ro cessin g of e n erg y is a n h yd ro electric p la n t w h e re e n e rg y is c o n v e rte d to electrical form. It is
th e n tra n s m itte d a n d d istrib u te d over lin e s a n d fin a lly c o n v e rte d to m e c h a n ic a l e n e r p in an
electric m ctor for final use. A n o th e r ex am p le is th e con v ersio n o f so u n d en e rg y into electrical
en erg y a t th e ta lk e r ’s end, its tra n s m is s io n over lin e s in e le c tric a l fo rm a n d its final conversion
to sound w aves a t th e lis te n e r’s end.
A n electro-m echanical en erg y conversion device is one w h ich c o n v e rts electrical energy into
m echanical en erg y or m ech an ical en erg y in to e le c tric a l en ergy. O p e ra tin g principles of energy-
conversion devices a re sim ila r, b u t th e ir s tr u c tu r a l d e ta ils differ d e p e n d in g upon th e ir function.
V arious electro m ech an ical en erg y conversion devices m a y be c a te g o rise d as u n d e r :
(i) T he firs t category of devices, involving sm a ll m otion, p ro ce sse s only low -energy signals
from electrical to m ech an ical or vice v e rsa . T h e se in c lu d e te le p h o n e rec e iv e rs, loud-speakers,
m icrophone, gram o p h o n e pick-ups a n d lo w -signal tra n s d u c e rs .
(ii) The second category consists of force-, or, torque-producing devices w ith limited mechanical
motion. Exam ples of such devices are electrom agnets, relays, m oving-iron instrum ents, moving-co
in stru m en ts, actuators etc.
(iii) T he th ird category includes continuous energy conversion devices like motors and gener
ators ; these are used prim arily for bulk energy conversion and utilization. In a machine acting ^
motor, energy is converted from electrical to m ec h a n ic a l; w hereas in a. generator, energy is conV
from m echanical to electrical. ^
E n e rg y conversion process is b a sic a lly a re v e rs ib le p ro cess. H ow ever, in practice, devic
m ay be d esig n ed a n d c o n stru c te d to s u it one p a r tic u la r m ode o f c o n v ersio n or th e other.
All th e s e e n e rg y conversion devices o p e ra te on s im ila r p h y sic a l p rin c ip le s, b u t
s tru c tio n a l d e ta ils differ. T h e coupling b e tw e en th e e le c tric a l a n d m ec h a n ic a l system s o ^
devices, is th ro u g h th e m ag n e tic or electric field. T h e object o f th is c h a p te r is to deve. j!is 0f
e le c tro m e c h a n ic a l-e n e rg y conversion p rin c ip le s w ith field e n e rg y a s th e b a sis. T h e a n a ^ ^ ,g
e n e rg y co n v ersio n devices by field -en erg y concept is m o re g e n e ra l a n d broad-based, s

Scanned by Cam Scanner


Electromechanical Energy Conversion Principles 161

^icable to 8SSing rotatioR al, linear or vibratory m otions. O th er a d v a n ta g e s o f

th
’^is c a n deal w ith both ste a d y sta te and tra n sien t an alysis o f a ll electro m ech a n ica l en e r g y
L
erters
% It glves more Ph y s,ca l ln s ‘Sh t in t° th e operation of all th ese devices, w h ich is in d isp en -

i'bl|'ii)This approach lea d s p h ysically to th e generalized theory o f electrical m a c h in e s .


)The conventional approach can be introduced at any sta g e, to stu d y th e effect o f sa tu r a -
con.metat.on etc. y

Electromechanical en ergy.“ " ^ “ d evices pertaining to categories one and tw o a s liste d


,bove are dismissednn th s chapter. C ontin uou s energy conversion d evices are treated in o th er
chapters of this booK.
R epresent chapter b egin s w ith th e principle of conservation of energy and th en e x o re s-
iims for forces and torques are developed in term s o f field variables. The essen tia l co n d itio n s
(ortheenerp conversion from electrical to m echanical form or vice verse, are clea rly b rou gh t
gutin this chapter. J 5
The energy storing cap acity o f th e m agn etic field is m uch greater (about 2 5 ,0 0 0 tim es) th a n
that of the electric
field. In view o f th is fact, electrom echanical energy conversion d evices w ith
magnetic field as th e cou p h ng m edium betw een electrical and m echanical sy ste m s are m ore
”nmon “ oommeraal ap p lication s. As su ch m ore em p h a sis is given to th ese devices. H o w e v / '
devices using electric field as th e coupling m edium are also discussed in th is chapter
2,1, Principle o f E n e r g y C o n v e r s io n

When energy is converted from one form into another, the principle o f conservation o f e Z
pcan be invoked. A ccording to th is principle, energy can n eith er be created nor destroyed i
canmerely be converted from one form into another. '

In an energy conversion device, out o f th e total input energy, som e energy is converted into
mJ p I0""1’ S° m e Gnergy 1S St° red and th e rest is dissipated. In view of this th e en erg y
equation m u st in clud e th e s e four en ergy term s and for a motor, it can be w r h i n as
Total electrical \ M ech a n ica l (n
Total e n e r g y f T o t a l energy
energy input en erg y output stored d issipated
The * * i ' ' / \ j
*»Eq (21) U wr,f, ! n e r8y conversion is based on encr«y balance Eq. (2 .1 ). It sh ould be n o ted
"Pit are treated as m V "“T * Where electrical ener® ’ inPu t m ech a n ica l en erg y
(Total P °sltlv e term s. For generator action,
0 mechanical en ergy in p u t) = (E lectrical energy output) + (Total en ergy stored )

The various f , + (T° ta l 6n ergy d issip a te d )

" S n r d V s e X T b e l o w ° 'Ved E q' ® “ electroraechanil:al con ver-


blTotal elertw . . eow-
i‘i)The en ergy in p u t from th e su pp ly m ain s is Wei.

(iii) Tot^ eChaniCal en erg y 0UtPu t is Wmo-

St°tred in a n y d evice = E nergy St0red in m a^n etic fie ,d. Wes + E n erg y
< v ni.i ' ms-
S if a i j e n e r ^ v Hi • j
Vw- as ma * ™lSsipated = E n ergy d issip a ted in electric circuit as ohm ic lo s s e s + E n erg y
^ system f f c0re lo ss (h y ste r e sis and eddy-current lo sses) + E n erg y d is s ip a te d i n
A ction and w in d a g e lo sse s etc.)

Scanned by Cam Scanner


162 E lectrical M ach in ery _________ 'l\
u , Fn (2 1) can
Z .lj can be
......w r itte- n in m o re s p e c ific te r m s as
T h u s th e e n e r g y b a la n ce E q. U - U , . . . P „ „ niimT M nnOM
+ {W + W ) + (O h m ic e n e r g y lo s s e s + C o u p lin g field energy l0sses)
Wei = Wmo + ( vves+ msi + (E n e r g y lo s s e s in mechanical
, u • 1 • , System).
. j 4-or>ri fnr p lec tr ica l, m e c h a n ic a l, in p u t, stored and nut

r e s p e c t i v e l y s u b s c r ^ t ei denotes electrical input (energy), subscript m s <

m eT t t Ct t ° o p n a t e te fm s a re grou p ed to g e th e r , th e n th e e n e r g y b a la n c e equation
ii tn e apprup + M e c h a n i c a l e n e r g y lo s s e s )
/W . _ O hm ic en erg y lo sse s) —("mo + ms _ ,, ,
tw e, w a n bj + (^ es + C o u p lin g field en erg y losses) „.(2.2a)

or w d„ = w mxh + -..(2.2H
Eq (2 21* leads to the electromechanical energy conversion model of F.g. 2.1. The varies
M , r fioiri incept; and th e fric tio n a n d w in d a g e lo sse s are irreversible
a n d t h e s e a r e t h e r e f o r e T s s ip a t e d a s h e a t . E n e r g y s to r e d in t h e c o u p lin g fie ld W„, is dealtmth
la te r in A rt 2.2 (i>). T h e e n e r g y s to r e d in t h e m e c h a n ic a l s y s t e m W m , is th e kinetic energy!
(m a ss) for a lin ear-m otion sy ste m or | J t o,2 for a ro ta ry -m o tio n s y ste m . H ere u is the linear
v elo city in m /sec and <or is th e a n g u la r v elo city in r a d ia n s p er s e c o n d N o te th at the coupling
field is a sso c ia ted w ith e.m .f. e and cu rren t i on th e e le c tr ic a l s id e a n d torque T (or force F>and
sp eed cor (or u) on th e m ech a n ica l sid e.
H e a t d u e to Heat due H e a t d u e to

Fig. 2.1. General representation of electromechanical


energy conversion device.

In Eq. (2 .2 6 ), We;ec is th e n e t electrica l e n e r g y in p u t to th e c o u p lin g field. is ^


en ergy con verted to m ech a n ica l form an d it is e q u a l to th e su m o f u s e fu l m ech an ical entf
W w m ech a n ica l en erg y sto red W,„s an d m e c h a n ic a l e n e r g y lo s s e s . Wpd is th e total energy^
sorbed by th e co u p lin g field and it is eq u a l to th e s u m o f b o th th e sto red field energy «
th e co u p lin g field en erg y lo sse s. .
It is evident from above th a t electrical losses (ohm ic and coupling-field losses) as well as m * ^
cal losses (friction and w indage losses), though a lw ays p resen t, p la y no basic role in the ene ^
version process. N eglect of th ese losses in th e rep resen ta tio n o f Pig. 2.1 leads to Fig- ;0i,
m otorin g-m o d e an d Fig. 2 .2 (6) for g e n e r a tin g m o d e. In F ig . 2 .2 , lo s s le s s energy*00

♦If torque, speed and coupling-field energy of a m achine rem a in c o n sta n t, th en it is said to o p e rj^ ^
steady s ta te conditions. U nder these conditions, th e re is no change in sto red energies We5
therefore, Eq. (2.2a) becomes
= 'hno + Energy lost as ohmic losses, coupling field losses and mechanical lo sse s

or ‘ Total input power = Total output power + Power lost as heat due to various losses. ant.beC8°5<
• Note th a t in single-phase a.c. m achines, th e m agnetic field en erg y sto red does n o t rem ain const* ■
these m achines don’t have constant air-gap flux.

Scanned by Cam Scanner


Electrom echanical Energy C onversion P rin cip les

associated w ith e lectric a l q u a n titie s e and i a t electrical term in a ls and m e c h a n ic a l


'es T and ^ r ^or ^ an d U^ m ec^ a n *ca ^ term inal. The a d v a n ta g e o f th is m o d el is th a t
I n a tte n t io n can be fo cu sse d on th e p rocess of energy conversion, so im p o rta n t in u n d er-
diffct a action and rea ctio n o f co u p lin g field on electrical and m ech a n ica l te r m in a ls. It
^ v f v v e v e r , be k ep t in m in d th a t electrical and m echanical lo sses are an in teg ra l p a r t o f
(pnst' 0 jjanical en erg y co n v ersio n d evice and m ust, therefore, be considered in d e te r m in in g
flectr°ut or output o f an en erg y -co n v ersio n system .
^ *of the lo ss le ss co n v ersio n sy ste m o f Fig. 2 .2 (a), Eq. (2 .2 ) can be w ritten in d iffere n tia l

form as
d W clcc = d W mech + dWfed ...(2 .3 )

here d We/ec = d ifferen tia l electric a l en erg y input to coupling field


d Wmech - d ifferen tia l m ech a n ica l energy output
d = d ifferen tia l ch a n g e in en ergy stored in th e coupling field.
l

L o s s le s s
+
L o s s le s s T ,u O r
C o u p lin g T ,u )r
or*
<z C o u p lin g or 1
F ie ld
F ,u F ie ld F ,u
- M «c h .
M ech a n ic a l
te rm in a l
te rm in a l le c tric a l
Ideal c o n v e rs io n te rm inals Ideal c o n v e rs io n
E le c tric a l
te rm in a ls R e g io n Region
(a) {b) .
Fig. 2.2. Representation of lossless electromechanical energy conversion device
(a) motoring mode and (6) generating mode.
From Fig. 2.1, th e d ifferen tia l electrical energy input in tim e d t is
d Wel = v, i d t
Ohmic loss in r e sista n c e r in tim e d t is i 2 r dt.
Differential electrical energy input to the coupling field,
d Welec = d Wei - ohmic loss
...(2 .4 )
= ( vt - ir) i d t = ei d t

(2.3) now becom es


...(2 .5 )
ei d t = d Wmech + d W/w
* ■ 0 balance Eq. (2 .5 ) is o b ta in ed by applying th e principle o f conservation o f e n e r g y to

^"storing m od e. T in s Eq. (2 .5 ) a lo n g w ith Faraday's law o f induced e .m .f .,. - ^ . form s

'j^ a m e n ta l b a sis for th e a n a ly s is o f energy-conversion devices .


,1 c ouni;^ « . . . x- kofui/ppn electrical and m ech an ical te r m in a ls is
, °Ugh thp g ‘ jle ,d r e a c t i o n . T h e in teraction Jn other w ords , co u p lin g field is th e
? betweenm^ ° f en e r g y St° red ? ! In order th at a m oving m em ber can ro ta te , or
\°Ve,'vith r 6 ectr^ca I and m ech a n ica l s y s te • exj8t jn b etw een th e sta to r a n d rotor.
^ e r e v !?PeCtt0 s ta tio n a r y m em b er’ an air g . P action and reaction on th e e lectric a l an d
J ^ i c a ] S ° red in th e C0UPlin £ field m pr°frnm electrical to m ech an ical (m o to rin g m od e)
% * al system s for th e co n v ersio n o f en erg y from e > « tn c a ‘ 6

iyjfthe outr ! CaI t0 el8CtnCal (&e n e r a tin g coupling field m u st react w ith th e e le c tr ic a l
111*n orH 18 m ech an ica l, a s in a m otor, m otor, th is reaction is th e co u n ter e.m .f.
°rde«- to absorb electric a l en erg y from it. In a mos ,

Scanned by Cam Scanner


164 Electrical Machinery

e, F ig . 2 .2 (a ), s im ila r to t h e e .m .f. in d u c e d in t h e p r im a r y w in d in g o f a transform er. Th


COU-
p lin g fie ld e x t r a c t s e n e r g y p r o p o r tio n a l to e ■i fro m t h e e le c t r ic a l s y s t e m , co n v erts and del'
e n e r g y p r o p o r tio n a l to T ■w r (or F ■u) to t h e m e c h a n ic a l s y s t e m . Uers

I f t h e o u tp u t is e le c tr ic a l, a s in a g e n e r a to r , t h e c o u p lin g fie ld m u s t r e a c t w ith the mech •


c a l s y s t e m s o a s to a b so r b m e c h a n ic a l e n e r g y fro m it . In a g e n e r a to r , th is reaction is th*
c o u n te r -to r q u e T , o p p o site to t h e a p p lie d m e c h a n ic a l to r q u e o f t h e p rim e-m o v e r. This counter6
to r q u e is p ro d u ced th r o u g h t h e in te r a c tio n o f c o n d u c to r c u r r e n t i a n d t h e co u p lin g field. In this
m a n n e r , t h e c o u p lin g fie ld e x tr a c ts m e c h a n ic a l e n e r g y p r o p o r tio n a l to [(reaction torque) x
(sp e e d )l from t h e m e c h a n ic a l s y s t e m , c o n v e r ts a n d d e liv e r s it a s e le c tr ic a l en erg y proportional
to e ■i to t h e e le c tr ic a l s y s te m . I t m a y b e s e e n from a b o v e t h a t c o u p lin g field serves as the
e n e r g y c o n v e r sio n reg io n .
T h u s , th e r e a c tio n o f th e c o u p lin g fie ld on t h e e le c tr ic a l or m e c h a n ic a l sy stem is essential
for e le c tr o m e c h a n ic a l e n e r g y c o n v e r sio n p r o c e ss. S in c e in d u c e d e.m .f. e an d torque Tare as­
s o c ia te d w ith th e c o u p lin g fie ld , t h e s e a re u s u a lly c a lle d e le c tr o m e c h a n ic a l coupling terms.
E le c tr o m e c h a n ic a l e n e r g y co n v e r sio n d e v ic e s a re s lo w -m o v in g d e v ic e s b ecau se of the iner­
tia a s s o c ia te d w it h m e c h a n ic a l c o m p o n e n ts. T h erefo r e, t h e c o u p lin g field necessary for the
e n e r g y co n v ersio n m u s t b e s lo w ly v a r y in g a n d a s su c h t h e n a tu r e o f t h is field is quasi-static.
E le c tr o m a g n e tic r a d ia tio n from th e c o u p lin g fie ld is a lm o s t n e g lig ib le .
2 .2 . S in g ly E x c it e d M a g n e tic S y s te m s
In th is a r tic le , t h e e x p r e s s io n s for e le c tr ic e n e r g y in p u t, m a g n e tic field energy stored,
m e c h a n ica l w o rk d o n e and th e m ec h a n ic a l force a re d ev elo p e d for s in g ly ex cited magnetic sys­
te m s.
(a) Electric energy input. C o n sid er a sim p le m a g n e tic s y s te m o f a toroid, excited by a single
coil a s sh o w n in F ig. 2.3. T h e in s ta n ta n e o u s v o lta g e e q u a tio n for th e electric circuit is written
by a p p ly in g K ir c h h o ffs v o lta g e la w .
v t = ir + e
w h ere e is th e rea ctio n e.m .f. ta k e n a s a v o lta g e drop in th e d irectio n o f cu rren t i, refer to Art,
1.3.

.-. e=^
dt
and v - i r+ ^L > ...(2.6)
‘ dt 1
H ere y is th e in s ta n ta n e o u s flu x lin k a g e s w ith th e c ircu it M u ltip ly in g both sides ofEq-
(2.6) by id t, w e g et

, vt id t = ri2 d t + i d \ j/
or (vt - ir) id t = i d\y
or e id t = i d\y
In v ie w o f Eq. (2 .4 ), d W elec = e id t = i • d y - (2
I f th e toro id a l core is m a d e o f ferro m a g n etic m a te r ia l, th e n m o st o f th e flux w ould be con­
fin ed to th e core. A ssu m in g th a t flu x $ lin k s a ll th e N tu r n s o f th e coil in Fig- 2.3, the m
lin k a g e s q/ a re eq u a l to Nty W b -tu m s. T h erefore, from Eq. (2.7),
d w chc ~ 1 • d \ i = iN dty=Fd<\>
In Eq. (2.8 ), b is th e in sta n ta n e o u s v a lu e o f th e coil flu x and F = i N is th e in stan tan eo u s <
m .m .f.

* T h e r e a d e r is a d v ise d to go th r o u g h A p p e n d ix A to kn'pw m o re a b o u t m a g n e tic circ u its.

Scanned by Cam Scanner


V 2’
(2 7) or (2 .8 ) s h o w s t h a t fo r t h e to ro id to ex tra c t
^ from th e s u p p ly s y s t e m , t h e f lu x lin k a g e s o f th e
t'c fie ld m u s t c h a n g e . T h e s e c h a n g in g flu x
cau se th e g e n e r a t io n o f r e a c tio n e.m .f. e. A s
j before, th e flo w o f c h a r g e s or c u r r e n t a g a in s t
s tion e m .f. e c a u s e s t h e e x t r a c tio n o f en erg y *
j ^ e l e c t r i c a l s y s t e m ..
ih) Magnetic fie ld e n e r g y sto re d . C o n s id e r a sim p le
!etic relay o f F ig . 2 .4 (a). I n it ia ll y t h e a rm a tu re is
p o sitio n . W h e n s w it c h S is clo sed , cu rren t F ig . 2 .3 . T o r o id a l c o r e e x c it e d fr o m s in g le
• tablished in t h e iV -tu r n c o il. T h e flu x s e t up source.
"S6ds on m .m .f. N i a n d t h e r e lu c ta n c e o f th e m ag- Open
position
f ath T he m a g n e t ic f ie ld t h u s p ro d u ced , crea tes Iro n yoke North
and sou th p o le s a s s h o w n in F ig . 2 .4 (a ), and a s a
V
suit of it, th ere is e s t a b lis h e d a m a g n e tic force ten d -
to to shorten th e a ir -g a p . I f t h e a r m a tu r e is n o t a l­
lowed to move, t h e m e c h a n ic a l w o r k d o n e, d W mech is
zero. According to E q . (2 .3 ), th e r e fo r e ,
d W e[ec = 0 + d W fld Iron
arm ature
This sh ow s t h a t w h e n t h e m o v a b le p a r t o f an y
physical sy ste m i s k e p t f ix e d , t h e e n t ir e e ec rica
energy input is s to r e d in t h e m a g n e t ic fiel . P ivo t-

dWfid = d W e[ec (a ) S im p le m a g n e t ic r e la y .

F lu x linkage
f ‘

C u rre n t.!

(c) .
m a g n e tic c irc u it-
.energy ^ a linear
(b) and co ...(2 .9 a )
to field energy
Fig. 2.4. P e rta in in g
ta b lish in g a flu x
,or"Eq. (2.8), dW lu = dW elK-ro‘ a gdV stored W p , * es
n e tic 8 e > d e n e ^
^ i a l flu x i s z e r o , t h e n t h e
...(2 .9 6 )
Ux linkage is g iv e n b y
Vi 4 f ■d §
i - d y 'J o tively.
Wfld = J o 0f y and 4>respe
te r m s
ed i° .\
i H <2-96). i a n d F m u s t b e ex p r e ss'
. . . .
Scanned by C am Scanner
2 .4 (6) an d (c).
ur - J W = f ' F ■d<\>= a r e a O A B O .
F o r F ig . 2 .4 (6 ), w fid ~ i o d W fld Jo

w _ fVl ^ WfM = f i d y = a r e a O A B O .
For F ig. 2 .4 (c), , W/w-J0 d fld Jo

In F ig. 2 .4 (6) an d (c), .

OACO = J + V *
area

T h is a rea OACO is ca lled th e co -en er g y -

w /u '= r ♦ < t f , - j 0, v <s '■■(2,i°)

In E a (2 10) * a n d ¥ m u s t b e e x p r e s s e d in t e r m s o f F a n d i r e s p e c tiv e ly . Co-energyhas


p h y ' L u i S i f i c ^ c ” i t ! s h o w e v e r u s e fu l in c a lc u la tin g t h e m a g n e ttc fo rces.

W ith no m a g n e tic sa tu r a tio n ,


A rea O A B O = a r e a O A C O
or Wfld= W fld'
and Wfld + Wfld' = a r e a O C A B O = fc F x = V i »i
In g e n e r a l, for a lin e a r m a g n e tic circu it,
/ 1 ■ 1 V rh ...(2.11)
W w = w li d = 2 ' l f ' = 2 p t>
T h e m a g n etic sto red e n e r g y a n d co -en er g y c a n a ls o b e e x p r e s s e d in te r m s o f reluctance an
p erm ea n ce o f th e m a g n e tic p a th .
N o w m .m .f. = (flux) (relu cta n ce)

or ^ ~^ ^ P erm eance, A

f R l = \* K

A lso W fu - V tfu '= L =

T h e se lf-in d u c ta n c e L is d e fin e d a s th e m a g n e tic f lu x -lin k a g e s p e r a m p ere, i.e.,

L = ^r
i

F rom E q. (2 .1 1 ), W„d = = A L i2 = | £

S u m m a r is in g th e r e s u lts o b ta in e d for a lin e a r m a g n e tic c ir c u it, t h e sto red m agnetic


Wf]d an d c o -e n e r g y Wpd ca n be w r itte n a s fo llo w s :

IF 2 1 1 , .J 1 £ . , -I*-1
= 557 = 0 ^ = 0 ^ = 5 r J°u le s

Scanned by Cam Scanner


^ rt' ^ Electromechanical Energy Conversion Principles 167

In th e s im p le m a g n e tic rela y o f Fig. 2.4 (a), p a th o f th e m a g n etic flu x is th ro u g h th e yok e,


arm atu re a n d a ir-g a p . S in c e th e relu cta n c e offered by th e a ir-gap is m u ch la rg er th a n th a t o f
the iron, a m ajor p a rt o f th e m .m .f. N i is req u ired to overcom e th e air-g a p relu cta n c e. A ccord­
ingly, m o st o f th e m a g n e tic en erg y Wpd is stored in th e air gap.

I f W/wi a n d W/M2 a re r e sp e c tiv e ly th e m a g n etic en erg ies stored in iron and air gap o f F ig.
2.4 (a), th e n to ta l en e r g y sto red Wpd = Wpdl + Wnd2. T he m a g n etic sto red -en erg y d e n s ity Wpdi
for th e iron is g iv e n by

wfidi = “f —------------------------- ^ — ___________________


(L en gth o f th e m agn etic p a th th rou gh iron)
(Iron area norm al to th e m a g n etic flux)
W/tei________ 1 ^ i<t>
(L ength) (Area) 2 (L ength) (Area)
F I 3
= ~ H 1 B j o u le s /m
2 (^Length J|^Area

H ere F i is th e m .m .f. req u ired to overcom e th e iron relu ctance and H i is th e m a g n etic-field
in te n sity in a m p ere-tu rn s p er m etre le n g th o f th e iron path. B is th e flu x d e n sity in W b/m 2 or
tesla .
But B = (ij H i

S im ila rly for th e air-gap , th e m a g n etic stored en erg y d en sity w ^ is

1 UD 1 „ 2 1 B2 . . . 33
Ufid2= 2 2 2 ^° 2 2 ^ J0uIes/m
H ere p.j and p 0 are th e p e rm ea b ilitie s o f iron and air gap resp ectively.
In g en era l, th e m a g n e tic stored en erg y d e n sity Wpd is g iven by
2
wH = \ H B = \ Mif2 = | y j o u le s /m 3.

w here H is th e m a g n e tic -fie ld in te n s ity in a m p ere-tu rn s per m etre a t p o in t w h ere th e p er­


m eab ility is p.
For a lin e a r m a g n e tic circu it,
•J*

WfU = v m = \ M #2 = 5 J = \ H B j o u le s /m 3 ...(2 .13)

H ere w ^ is th e co -en erg y d en sity .


An e x a m in a tio n o f Eq. (2 .1 2 ) rev e a ls th a t th e m a g n etic field en erg y can be ex p ressed in
term s o f field v a r ia b le s F, 0, y and p a ra m eters Rl, A or in term s o f in d u cta n ce p a ra m eter L and
current v a ria b le i. T h is fact th a t field en erg y can be exp ressed in term s o f circu it p a ra m eter L,
clears th e w a y for e le c tr ic c ir c u it ap p roach to th e a n a ly s is o f e lectric a l m a c h in e s, i.e. th e
gen eralized th eo ry o f electrica l m a ch in es. T h u s th e field -en erg y approach se r v e s a s th e p h ysical
basis for th e g e n e r a liz e d th eo ry o f electrical m a ch in es. •
(c) M echanical work done. C on sid er th e sim p le m a g n etic rela y o f F ig. 2.4 (a) a g a in . In Fig.
2 -5 (a), th e m o v a b le en d o f th e a rm a tu re is a ssu m ed to be h eld in op en p o sitio n . W h en th e
sw itch S is clo sed , th e e x c itin g cu rren t in cr ea se s from zero to t , = V ,/r and th e flu x lin k a g es

^ o u a i M i c u u y \ ^ a i i u u a i n i ci
16H E lectrical M a c h i n e r y ____
------------ — ' I ^ r e s i s t a n c e . A s e x p la in e d b efo re, th e magnetic energ>,
in c r e a s e s from zero to H ere r tc h e d a r e a O A B O .
sto r e d in th e fie ld is g iv e n b y th e cro -
Open position

Iron yoke

Pivot
Fig 2.5. (a) Arm ature held in open position.
, * ic S l i m e d in t h e clo sed position, so that
In F ig . 2 .5 (5), th e .n o v a b le ^ “U h e a rm a c u r r e n t a g a in rises from rerot,
th e a ir-g a p is zero. W h en zero to V s. H e re th e flu x lin k a g es V, m u tk
i . = V . / r w h e r e a s th e flu x lin k a g e s in c r e a s c o n s id e r a b ly reduced for the same
g r e a te r th a n sin ce th e r e lu c ta n c e o f th e> £ £ " i t c h e d area OCDO.
m .m .f. T h e en erg y s t o r e d in th e m a g n e t,c field ,s n o w g iv e n by
Closed
p o sitio n

Fig. 2.5. (5) Arm ature held in closed position. ^ agnetiC

W ith th e a rm a tu re in th e op en p o sitio n , th e e x c itin g c u r r e n t i x setsi u p f ie l d »in


cir c u it. T h is field cre a tes m a g n etic force, d u e to w h ic h t h e fr e e e n d o f e a rejuCtance0
th e c lo sed p o sitio n . D u rin g th e a rm a tu re m o v e m e n t, from it s o p e n p o siti , vaiue ofVr
th e m a g n e tic p a th d ecre a se s, th ereb y in c r e a sin g t h e flu x lin k a g e s from l s ^ ^ flQ#0of
T h e s e in c r e m e n ts in flux lin k a g e s in d u ce a c o u n te r e .m .f. in t h e co il, w h ic h opp
e x c itin g c u r r e n t i, i.e.
Vt - cou n ter e m f in d u ced in th e coil d u r in g a r m a tu r e m ovenaen_
i =
C oil im p e d a n c e ^ j the afIllS
T h e m a g n itu d e o f co u n ter e.m .f. in d u ced in th e e x c it in g co il d e p e n d s on ho\'
tu r e m o v e s . T h e tw o e x tr e m e c a se s o f a rm a tu re m o tio n a r e c o n s id e r e d b elo w . ^ ^ thf
/ S lo w m o v e m e n t . W ith th e a rm a tu re in th e o p en p o s itio n , t h e e x c itin g cur^ re pos'll°''f
flu x lin k a g e s a r e Vi and th e o p era tin g p o in t is A , [F ig. 2 .6 (a)). In t h e c lo sed armavelTient «f t '
th e flu x lin k a g e s a re y 2» cu rren t is i x and th e o p e r a tin g p o in t is C. T h e slo w mo

Scanned by Cam Scanner


A rt. 2.2) Electromechanical Energy Conversion Principles 169

y2-v i
arm atu re from o p e n p o s itio n c a u s e s a lm o s t a n e g lig ib le a m o u n t o f c o u n te r e .m .f. in
T im e
the coil. A s a r e s u lt o f it, t h e e x c itin g c u r r e n t i 1 r e m a in s s u b s t a n t ia lly c o n s ta n t d u r in g th e
arm atu re m o v e m e n t from o p en to clo sed p o sitio n an d a cco r d in g ly th e o p e r a tin g p o in t A r e a c h e s
the n ew o p e r a tin g p o in t C a lo n g th e v e r tic a l lin e A C a s sh o w n in F ig . 2 .6 (a).
N ow c h a n g e in th e sto r e d e n e r g y o f m a g n e tic field W ^ , d u r in g th e tim e a r m a tu r e m o v es
from op en (p o in t A ) to th e clo se d p o sitio n (p o in t C) is g iv e n by,
Wfu = (M a g n e tic en e r g y sto red in th e clo sed p o sitio n )
- (M a g n etic e n e r g y sto red in op en p o sitio n )
or Wfjd = a rea O A 'CD F O - a rea OAA'FO.
E lectric e n e r g y in p u t d u r in g th is ch a n g e is
f•v22
W.clcc = J i j d y = i i ( y 2 _ Nh) = area A CD F A 'A

But W e!cc = W f!d + WmccA


.'. a rea A C D F A A = a rea O A ’C D F O - a rea OAA'FO + Wmech
or Wmcch = a rea O A C D F O - area OA'CDFO
= a rea O A C A ’O ...(2 .1 4 )
Eq. ( 2. 14) sh o w s th a t th e m ech a n ica l w ork don e is eq u a l to th e area e n c lo se d b e tw e e n th e
tw o m a g n e tiz a tio n cu rv es at open and closed p o sitio n s and th e v ertica l y - i lo c u s d u r in g th e slo w
a rm a tu re m o v em en t. T h is is sh ow n by cro ss-h a tch ed area in F ig. 2 .6 (a).

(b)
F ig . 2 .6 M e c h a n ic a l w o r k d o n e (a I W ith s lo w a r m a t u r e m o v e m e n t. (b) W ith in s t a n t a n e o u s a r m a t u r e m o v e m e n t .

It is se e n from above th a t ou t o f th e total electrica l en erg y in p u t d u rin g slo w m o v e m e n t o f


th e a rm a tu re, so m e en erg y g e ts stored in th e m a g n etic field and th e r e m a in in g is o u tp u tte d a s
m ech an ical en erg y . It is sh o w n in E x a m p le 2.1 th a t w ith sa tu r a tio n ign ored , h a lf o f th e e le c tr i­
cal en erg y in p u t g e ts sto red in th e m a g n etic field and th e o th er h a lf a p p ea rs a s m ech a n ica l
en ergy o u tp u t d u rin g slo w a rm a tu re m o v em en t.
II. In sta n ta n eou s movem ent. H ere th e a rm a tu re is a ssu m ed to m ove from th e op en to closed
position in s ta n ta n e o u s ly . A ccording to th e constant flux linkage theorem, th e flu x lin k a g e s w ith
an in d u ctiv e circu it c a n ’t ch a n g e su d d en ly . So h ere also, d u rin g th e fa s t m o v e m e n t o f th e a rm a ­
*
ture, th e flu x lin k a g e s d o n ’t ch a n g e and rem a in co n sta n t a t y x. T h e o p era tin g p o in t, th erefo re,
travels h o r iz o n ta lly from A to A ’. A fter th e a rm a tu re h a s clo sed , th e o p era tin g p o in t tr a v e ls
from A ' to C a lo n g th e clo sed -p o sitio n m a g n e tiz a tio n cu rve [Fig. 2.6 (6)], sin ce th e fin a l o p era t­
in g p oin t h a s to be C.

Scanned by Cam Scanner


>

170 Electrical Machinery--------------------- [Art^.2

D urin g th e tim e in s ta n ta n e o u s m o v e m e n t o f th e a r m a tu r e o ccu rs from open (point A)


to
closed p osition (p oin t .A'), w e h a v e
(i) C h an ge in th e m a g n etic stored en erg y
Wfjd = area OA'FO - a r e a OAA'FO
f H'1
and (ii) elec = Jy, i d yT = 0

B ut WeUc ~ Wfid + w mech


O - a rea OA'FO - a rea O A A'F O + Wmec),
or Wmech = a re a 0A A > 0 -(2.15)
Eq. (2.15) sh o w s th a t th e m ech a n ica l w ork d o n e is e q u a l to th e a r e a en clo sed between the
tw o m a g n etiza tio n cu rves a t op en and closed p o sitio n s an d th e h o r iz o n ta l y - i locus during the
in sta n ta n e o u s m o v em en t o f arm a tu re. T h is is in d ic a te d by c r o ss-h a tc h e d a rea in Fig. 2.6 (6).
D u rin g fa st a rm atu re m o v em en t, it is se e n from a b o v e th a t (i) th e r e is no electrical energy
in p u t (ii) m ech a n ica l en erg y o u tp u t is at th e e x p e n s e o f field e n e r g y sto red , i.e., mechanical
en ergy ou tp u t = reduction in th e m a g n etic sto red en erg y .
III. Transient movement. T h e a rm a tu re m o v e m e n t w ill n e ith e r be to o slo w nor too fast,
w ill lie so m ew h ere in b etw een th e tw o e x tr e m e lim its d is c u s se d ab o v e. T h e flu x linkage-current
locus w ill accordin gly be in b etw een th e tw o locii sh o w n in F ig . 2 .6 (a) a n d (6). Initially the
arm atu re m o v em en t is slow and as it is n e a r in g th e clo sed p o sitio n , it s m o v e m e n t becomes fast.
The y -i locus w ill, th erefore, be A C 'C a s illu s tr a te d in F ig. 2 .7 . T h e o p e r a tin g p oin t A reaches
C d u rin g th e tim e a rm a tu re m oves from op en to clo sed p o sitio n . S in c e th e fin a l operating point
h a s to be C, th e o p era tin g p o in t tr a v e ls fu rth er from p o in t C' to C a lo n g th e closed-position
m a g n etiza tio n curve OA'C'C. T h erefore, th e a r m a tu r e w h e n a llo w e d to m o v e under the in­
flu en ce o f m a g n etic force, h a s \j/-i locu s A C 'C a s sh o w n in F ig . 2 .7 .

Fig. 2.7. F lu , linkage-current locus during transient m ovem ent of arm ature.

D u r in g th e tim e a rm a tu re m o v es from o p en (point*) to c lo se d p o s itio n (p o in t C ) . w e have


(i) C h a n g e in th e m a g n etic sto red en e r g y

C'D'FO - area OAA'FO


f* .
and (ii) W.elec = JVj id y = area AC'D'FA'A.

Scanned by Cam Scanner


Art. 2.2)
Electromechanical Energy Conversion Principles 171

But elec = w.u +w


fld^
rr f1 mech.
A r e a AC D FA A = a r e a OA'C'D'FO —a r e a OAA'FO + Wmec/l
or Wmech = (a r e a OAA'FO + a r e a AC'D'FA'A) - a r e a OA'C'D'FO
= a r e a OAC'A'O ...(2 .1 6 )
Eq. (2 .1 6 ) a g a in s h o w s t h a t t h e e n e r g y c o n v e r te d to m e c h a n ic a l (or m e c h a n ic a l w o r k d o n e)
is eq u al to t h e a r e a e n c lo s e d b e t w e e n th e tw o m a g n e tiz a tio n cu rv es a t o p en a n d c lo se d p o s itio n s
and th e y - i lo c u s d u r in g t h e t r a n s ie n t m o v e m e n t o f a rm a tu re.
id) Determ ination o f magnetic force. In F ig . 2 .5 (a), th e m a g n e tic force te n d in g to sh o r te n
th e air g a p in c r e a s e s a s t h e g a p le n g t h d e c r e a se s. In v ie w o f th is , th e m a g n itu d e o f a v e r a g e
m a g n etic fo rce fe(av) c a n b e o b ta in e d from th e r e la tio n ,

r _ M e c h a n ic a l w ork d o n e d u rin g a rm a tu re m o v e m e n t
...(2 .1 7 )
e <au) D is ta n c e tr a v e lle d d u rin g a rm a tu re m o v e m e n t
F rom F ig s . 2 .5 (a ) a n d (2 .7 ),
, A r e a O A C 'A 'O
Te (au)
G ap le n g th , g
In o rd er to o b ta in a s u ita b le e x p r e ssio n for th e m a g n etic force d ev elo p e d , a d iffe r e n tia l
arm atu re m o v e m e n t o f m a g n itu d e dx m u st be im agin ed in th e direction o f th e m a g n etic force fe.
T h is m o v e m e n t or d is p la c e m e n t dx n e e d n o t be a rea l one. For th is r ea so n , th e d is p la c e m e n t dx
is referred to a s t h e v ir tu a l d is p la c e m e n t an d th e e x p ressio n fedx a s th e v ir tu a l w o rk . T h is
m eth o d o f d e te r m in in g th e m a g n e tic force is ca lled th e principle of virtual work. A ccord in g to
th is p r in c ip le , th e m o v a b le (or r o ta ta b le) p a rt is a llo w ed a v ir tu a l d is p la c e m e n t dx (or dQ) in th e
d ire ctio n o f m a g n e tic force fe (or to rq u e Te). T h en its effect on th e e n e r g y b a la n c e E q. (2 .3 ) is
in v e s tig a te d in o rd er to o b ta in th e m a g n itu d e an d direction o f m a g n e tic force fe or m a g n e tic
to rq u e T e.
In F ig . 2 .8 (a ), a s su m e th e a rm a tu re to b e a t a d is t a n c e ^ from th e op en p o sitio n . T h e n a
v ir tu a l d is p la c e m e n t dx in th e d irectio n o f m a g n etic force fe is co n sid ered . T h e tw o m a g n e tiz a ­
tio n c u r v e s for t h e a rm a tu re p o sitio n s g 1 a n d (g1 + dx) h a v e b e e n sh o w n in F ig . 2 .8 (6).
T h e flu x lin k a g e -c u r r e n t lo c u s d u rin g th e d ifferen tia l d isp la c e m e n t dx is abc. H ere a is th e
o p e r a tin g p o in t a t p o sitio n g i, an d y x' an d i i are th e corresp on d in g v a lu e s . T h e o p e r a tin g p o in t
a t (gl + dx) is c an d th e co rresp o n d in g v a lu e s are y x' + rfy an d i v

F ig . 2 .8 . M e c h a n ic a l w o r k d o n e for d iffe r e n tia l m o v e m e n t o f a r m a t u r e .

(I II I W I
X /

172 Electrical Machinery j A r i 2 2

A s e x p la in e d b efore, th e m e c h a n ic a l w ork d o n e in th e v ir tu a l d isp la c e m e n t dx is


a r e a oabho = ftd x -12.18)
T h e a r e a o a b h o d iffers from th e a rea oaho by th e tr ia n g u la r a rea abh. S in ce the displace.
m e n t d x is in fin ite s im a lly sm a ll, th e tria n g u la r area abh m a y be ^ ^ e d a s dx approaches zero
A ltern a tiv ely , th e a rm a tu re m o v em en t over th e v irtu al d isp la cem e n t d y taken as mstan-
ta n e o u s . N o te t h a t th e fin a l o p e r a tin g p o in t a t (gx + dx) m u st b e c, a s ex p la in e d in Art. 2.2(c).

N ow d Welec = J ^ . id\\i = 0 ; d W mech - f e dx.

S u b s titu tio n o f th e v a lu e s in Eq. (2 .3 ) g iv e s


0 = f c dx + dWfjd a t c o n s ta n t y
or fe d x = - dW nd a t c o n s ta n t v .(2.19)
T h e e le c tr ic a l en e r g y flow d u rin g th e v ir tu a l d is p la c e m e n t d x is zero for constant y, as
s h o w n ab ove. T h u s th e m ech a n ica l w ork fedx is d o n e a t th e e x p e n s e o f th e field energy stored
an d t h is is in d ic a te d by a n e g a tiv e sig n b efore dW ’fld in Eq. (2 .1 9 ).
(dW M
,
F rom Eq. t2 .1 9 ), ft - ~ a t c o n s t a n t \j/
dx

d W fid
or fe = ~ dx
Aji = constant
...(2.20)
d W fld'
) A lso f< = - dx
= constant

N o te th a t Wfld m u st be e x p r e sse d in te r m s o f y a n d x or 0 an d x. In v ie w o f th is, Eq. (2.20)


le a d s to th e p a ra m etric e q u a tio n s for m a g n e tic force a s,
d Wfld (v> *) _ _ d Wfld (<t>. *) ...(2.21)
e~ dx dx
In th e ab ove e x p r e ssio n for m a g n e tic force, y or <(>a re in d e p e n d e n t v a ria b les. As voltage i.
e q u a l to th e d e r iv a tiv e o f y , th is e x p r e ssio n g iv e s fe for a voltage-controlled system.
S im ila r ly , th e a r e a oabho d iffers from th e a rea oacbho b y th e tr ia n g u la r area abc. ere
a g a in a r e a oahbo m a y be ta k e n e q u a l to th e a rea oacbho a s d x a p p ro a ch es zero. ^ ter" ^ 1t ^
s lo w m o v e m e n t o f th e a r m a tu r e m a y be a s s u m e d o v er t h e v ir tu a l d is p la c e m e n t dx, so a
o p e r a tin g p o in t m o v e s v e r tic a lly from a to c.
,vtq'+ rfvj/
H ere d W tUc = . i xd y = i j d y = a rea acdea.

F or o b ta in in g dW nd, Fig- 2 .8 (6) r e v e a ls th a t

a r e a a cdea = a rea ojcdo - a rea ojaeo. .


= ((M a g n etic sto red e n e r g y + C o -en erg y ) a t p o sitio n (gt + dx) ^
- ((M a g n etic s to r e d e n e r g y + C o -en erg y ) at po^ti1- ‘
In o th e r w o rd s, a r e a a c d ea r e p r e s e n ts th e d iffe r e n tia l in c r e a s e in field en ergy dWfla ^
c o -e n e r g y d W g j , from r e c ta n g u la r a r e a ojaeo to r e c ta n g u la r a r e a ojcdo, d u rin g th e vir
p la c e m e n t dx. In v ie w o f th is ,
a r e a acdea ** dWpj + d W ^

Scanned by Cam Scanner


1 ^-____________________________________________Electromechanical Energy Conversion Principles 173

B u t it is a lr e a d y s e e n t h a t a r e a a c d e a = d W elcc = ^ d\\i.

d Wfid + dW fu = i l d\\i = a r e a a cd ea

T h u s t h e d if f e r e n t ia l c h a n g e in fie ld e n e r g y dWfld is g iv e n by
dWfid^iidy-dWpS.
A lso dWmech = fedx.

S u b stitu tion o f th e v a lu e s o f d W elec = i , <fy, d W ^ = h d y - dW fld' and d W mech = f d x in Eq. (2.3)


gives,

i xd\\f = f e d x + i 1 d \ \ t - d Wnd
or fed x = + dW fjj a t c o n sta n t i ...(2 .2 2 )

The p ositive sig n before d W ^ in d icates th a t during th e virtual disp lacem ent dx, th e m ech an ical
work fe dx is eq u al to th e in crea se in co-energy at con stant current.

r dWnd'
fe = + at c o n sta n t i.

S in ce W ^ ' m u s t be e x p r e s s e d in te r m s o f (i, x) or (F , x ) ; th e p a ra m etric e q u a tio n s for th e


agnetic force a re
„ BWaJ dWnd'
= j£ f ( F .x ) ...(2 .2 3 )

Eqs. (2 .2 1 ) a n d (2 .2 3 ) g iv e th e m a g n itu d e o f electro -m a g n etic force fe b e c a u se th e a r m a tu r e


m ovem ent is lin e a r . F or a n g u la r m o v em en ts o f a rm atu re, th e electro m a g n etic to rq u e Te can be
obtained from p a r a m e tr ic Eq. (2 .2 1 ) as
dWfu (y , 9) _ j>, 8)
le ~~ ae ~ ae
and from p a ra m etric Eq. (2 .2 3 ) as

Tc = {i’ 9) = ~ d d ~ {F’ 9) ...(2 .2 5 )


Eq. (2.23) for m a g n e tic force and Eq. (2 .2 5 ) for m a g n etic torque are a p p lica b le to s y s te m s in
w hich cu rren t is an in d e p e n d e n t variab le. In oth er w ords, th e s e tw o force an d torq u e e x p r e s ­
sions are ap p licab le to current-con trolled system s.
It can be s ta te d th a t a n y p h y sica l d evice w ill develop electro m a g n etic force or torq u e, if its
m agn etization cu rv e is effected by a d ifferen tial d isp la cem en t o f its m ovab le (or ro ta ta b le) p art,
the other part r e m a in in g fixed.
It m u st be born in m in d th a t Eqs. (2.21) to (2.25) hold good ev en w h en th e m a g n etic p ath is
saturated. In ca se th e m a g n etic sa tu r a tio n is n eglected , i.e. th e grap h b etw een y and i or 0 and
F is considered lin e a r, th en
' Wfld = Wfld.

...(2 .2 6 )

T h erefore, from Eq. (2.21),

S canned by Cam Scanner


174 E le c tr ic a l M a c h in e r y

r 1 x2 dRl
e= ~ 2 d7

A lso W/1<i = i ^ A = | . 2 L = | ^ a n d from E q. (2 .2 6 ),

1.
L ( 2 .28 )
'* ~ 2 rfx ~ 2 dx ~ 2 dx
T h e m a g n e tic sto r e d e n e r g y is a lso e x p r e sse d a s = \ i y . I f cu rre n t i is expressed in
te r m s o f y a n d x , th e n from Eq. (2 .2 1 ),

ft = ~ \ (¥• * ) -(2 29)

In c a s e y is e x p r e s s e d in te r m s o f i a n d x, th e n from Eq. (2 .2 6 ),

f . . i ( £ < ( .* , -(2.30)

S im ila r ly for e le c tr o m a g n e tic to rq u e Te, w e h a v e


~ l „ 2 d R i _ l j ^ d A = l i2 dL
0 2 d0 2 de 2 d0

=- 2 V i (¥> 9) = 2 1l e ( i' e) ' (2'311


F rom an e x a m in a tio n o f E q s. (2 .2 1 ) to (2 .3 1 ), it ca n be s ta te d t h a t electrom agn etic force or
to rq u e d e v e lo p e d in a n y p h y sic a l s y ste m , a c ts in su c h a d ire ctio n a s to ten d to
(i) d e c r e a se t h e m a g n e tic sto red e n e r g y a t c o n s ta n t y or <J>, s e e E q s. (2 .2 1 ) and (2.24);
(ii) in c r e a s e b o th field e n e r g y sto red an d co -en er g y a t c o n s ta n t cu rre n t or m.m.f. Eq. (2.26);
(iii) d e c r e a s e th e r e lu c ta n c e , [E qs. (2 .2 7 ) a n d (2 .3 1 )] ,
(iv ) in c r e a s e t h e p e r m e a n c e a n d in d u c ta n c e , E q s. (2 .2 8 ) a n d ( 2 .3 1 ) ,
(v) d e c r e a s e c u r r e n t i a t c o n s ta n t flu x lin k a g e s v|/, E q. (2 .2 9 ) or in c r e a se y at constant i, Eq
(2 .3 0 ). .
T h e fo r e g o in g e x p r e s s io n s o b ta in e d for fe a n d T e a re e q u a lly a p p lica b le to magnetic ie
p r o d u c e d b y p e r m a n e n t m a g n e ts ; b e c a u s e /; a n d Te do n o t d e p en d u p on th e source o t e ie

T h e r e s u lt o b ta in e d in Eq. (2 .1 9 ) ca n a lso b e a rriv ed a t, by re fe r r in g to Fig. 2.8 ( ^ o r


v ir tu a l d is p la c e m e n t d x from p o sitio n g x to p o s itio n (g x + dx) in th e d irectio n ot fe
s t a n t y , t h e fie ld e n e r g y sto r e d d ecrea ses from a r e a oaheo to a r e a oheo, a s is eviden o
2 .9 (a ).
. fe d x = a rea oaho ,
= d e c r e a se in fie ld e n e r g y sto r e d a fte r v ir tu a l d is p la c e m e n

in th e d ire ctio n o f fe, a t c o n s ta n t y . ^


or f / i x = - dW fu w ith y c o n s ta n t ^

T h e n e g a t iv e s ig n b efo re d W ^ in d ic a t e s re d u c tio n in t h e fie ld e n e r g y sto red at cons a

lin k a g e s . .^ .„aj displ®ce


F o r o b t a in in g E q . ( 2 .2 2 ) a lt e r n a t iv e ly , r e fe r to F ig . 2 .8 (6 ) a g a in . W ith th e vi tfe
m e n t d x from p o s itio n g , to p o s itio n ( g , + d x ) in t h e d ir e c tio n a n d a t co n sta
c o -e n e r g y in c r e a s e s fro m a r e a ojao to a r e a ojcho a n d t h is is e v id e n t from F ig. •

Scanned by Cam Scanner


fedx = area oacho
= in cr ea se in co -en erg y after v irtu a l d isp la cem e n t dx in th e d irection
o f f v a t c o n sta n t current.
or fedx = + d W ^ a t co n sta n t current ...(2 .2 2 )
T he p o sitiv e sig n b efore dW flJ in d ic a te s an in crea se in th e co-en ergy a t c o n sta n t cu rren t.
Another m eth od for o b ta in in g th e ex p ressio n s for m agn etic force fe in te r m s o f field e n e r g y
and co-energy is a s u n d er :
Eq. (2.9) sh o w s th a t field en erg y Wfu is a fu n ction o f flux lin k a g es y or flu x <(>. In F ig. 2 .4 (a),
field energy is p rim a rily stored in air gap. If air gap v a ries, th e n x m ea su red from th e op en
position also v a r ie s and a s a co n seq u en ce, field en ergy stored ch a n g es. From th is , it can be
stated th at W ^ is a fu n ctio n o f tw o in d ep en d en t v a ria b les y and x (or <j>and x).
M ech anical w o r k d o n e in d iffe r e n tia l m o v e m e n t d x in t h e d ir e c tio n o f m a g n e tic force fe is

d W mech = fA *
From Eq. (2 .7 ), dW tlec = icty
S u b stitu tin g t h e s e v a lu e s in Eq. (2.3) g iv e s
id \f = dW flJ (v , x) + fe dx
or dW f7d (.y,x) = i d \ \ / - f e dx ...(2 .3 2 )

This eq u ation is m ore g en era l th a n Eq. (2.9). For exam p le, if arm atu re is a ssu m ed s ta tio n ­
ary, dx = 0 and th erefo re
dW fu = id \|/

T h is is th e s a m e a s Eq. (2.9).
9)
Since is d ep en d en t o n y . i ; in differential form can be ex p ressed m a th e m a tic a lly
LI* ^ terms o f its tw o p a rtia l d e r iv a tiv e s as

Since v , x are in d e p e n d e n t v a ria b les, th e coefficien ts o f th e term s a sso c ia ted w ith d V and
dx m Eqs. (2.32 ) an d (2 3 3 ) m u s t be eq u a l.
\
ii
Scanned by Cam Scanner
176 Electrical Machinery

. , _ dW nd (V, X)
" l~ 3V -(2.34,
_n J r _ jjjjjgd (V> *)
dx -.(2.35)
E q . (2 .3 5 ) is t h e s a m e a s E q. (2 .2 1 )
I t is c le a r from F ig . 2 .4 (c) t h a t c o -e n e r g y W^d' d e p e n d s on i a n d x an d is given by
Wfld\ i , x ) = i \ v - W f u t y ' X ) .>(2 36j
or dW fid (i, x) = i d y + y d i - dW pd (i|/, x)
S u b s t it u t io n o f E q . (2 .3 2 ) in th e a b o v e e q u a tio n g iv e s
d\Vfid (i, x) = i d y + y d i - id \f + f / i x
= \\id i+ f cd x ...(2.37)
S in c e W^d' is d e p e n d e n t on i, x; Wpd in d iffe r e n tia l form ca n b e ex p ressed mathematically
in te r m s o f its p a r tia l d e r iv a tiv e s a s
dWfid\i> x) dWn d (i,x )
d w ^ (i, X) = r ~ - L di + — ^ dx ...(2.38)

A s i, x are in d e p e n d e n t v a r ia b le s , th e c o e ffic ie n ts o f th e te r m s a sso c ia te d w ith di, dx must


b e in d e p e n d e n tly e q u a l in E q s. (2 .3 7 ) a n d (2 .3 8 ) a n d t h is le a d s to th e p aram etric equations as
= dW nd' (239)
dl

an d f . = 3WlU^ ’ X) •••'2-40>

Eq. (2 .4 0 ) is th e s a m e a s E q. (2 .2 3 ).
W e h a v e d e r iv e d th e e x p r e s s io n s for m e c h a n ic a l w ork d o n e, th e m a g n etic force ft etc, by
ta k in g a n e le m e n ta r y e x a m p le o f a s im p le m a g n e tic r e la y m e r e ly for convenience. The reader
s h o u ld n o t co m e to th e co n clu sio n th a t a ll th e s e e x p r e s s io n s a re ap p lica b le only to the simp e
m a g n e tic r e la y a lo n e. A c tu a lly th e v a r io u s e x p r e s s io n s o b ta in ed so far, h a v e broad significance
a n d a p p ly e q u a lly w e ll to a ll p h y sic a l s y s te m s h a v in g co u p lin g m a g n etic field as the link e
tw e e n s ta tio n a r y a n d m o v in g p a rts. I f it is req u ired to c a lc u la te fe or Te for any physical system ,
t h e n it s m o v a b le p a rt is im a g in e d to be g iv e n a v ir tu a l d is p la c e m e n t d x or dQ in the d ir e c t io
f e or Te - u s e o f E q s. (2 .2 1 ), (2 .2 3 ) or (2 .3 1 ) th e n g iv e s th e req u ired r e su lts.
W h e n E q s. (2 .2 1 ), (2 .2 3 ), (2 .2 4 ) and (2 .2 5 ) a re u se d , o n e sh o u ld k eep in mind
a lg e b r a ic s ig n a n d s e c o n d ly th e v a r ia b le s in w h ic h en e r g y a n d co -en erg y functions (id ^
Wfjd a r e e x p r e s s e d . T h e ch o ice b e tw e e n Wpd a n d Wpd d e p e n d s u p o n th e in itia l descripti°n
t h e p h y s ic a l m a g n e tic s y s te m u n d er c o n sid era tio n . ,
E x a m p l e 2 .1 . For a certa in relay, the m a g n e tiza tio n cu rves for open a n d closedposi ^ ^
th e a r m a tu r e a re lin ear. I f the a rm a tu re o f the rela y m oves fro m open to closed Poslllj? n^eiween
s ta n t c u rre n t (i.e. very slow ly), sh ow th a t the electrica l energy in p u t is sh a red equa y
f ie ld en erg y s to re d a n d th e m ech a n ica l w ork done. ^
S o l u t i o n . W ith th e r e la y in th e op en p o sitio n , a s s u m e th e o p e r a tin g p oin t to be A,
t h e c u r r e n t a n d flu x lin k a g e a re i x an d qq r e sp e c tiv e ly , F ig. 2 .1 0 . ^ ^
In t h e c lo se d p o sitio n , th e o p e r a tin g p o in t is B , so t h a t th e c u r r e n t is z'j and the fluX

is V2.

;canned by Cam Scanner


Art. 2.2]
Electromechanical Energy Conversion Principles 177

T h e a r m a t u r e m o v e m e n t from o p en to clo sed ^


p o sitio n is a t c o n s t a n t c u r r e n t, th e r e fo r e , th e vim Closed
position^
lo c u s is a lo n g t h e v e r t ic a l lin e A B .
D u r in g t h e m o v e m e n t from o p en to clo se d p o si- Y2
tio n , t h e e le c tr ic a l e n e r g y in p u t is

^ e le c ~ *1 d \ y = t’ l (\Jf2 — \|fj)

T h e m e c h a n ic a l w o r k d o n e W mech is g iv e n by th e 1
Open
c r o ss -h a tc h e d a r e a O A B C O . position
w mech = t r ia n g u la r a r e a O A B C O

= g (V 2 " Vi)*'i

M a g n e tic s to r e d e n e r g y

Wftd = t r ia n g u la r a rea O C B D E O
Fig. 2.10. Pertaining to E xam ple 2.1.
- t r ia n g u la r a r e a O A C E O
1 . 1 . 1 , .
= 2 ^ 2 * 1 “ 2 ^ 1 = 2 (V 2 - V l) *1

I t is s e e n from a b o v e t h a t Wfld = = A Welec

T h is p r o v e s t h e r e q u ir e d r e s u lt.
E x a m p l e 2 .2 . F or th e s im p le m a g n e tic rela y o f Fig. 2 .4 (a), the v a ria tio n o f flu x lin k a g e vj/
in te r m s o f c u r r e n t i a n d d is p la c e m e n t x fro m th e open p o s itio n is g iv e n b y th e r e la tio n
V = ix l/2.
O b ta in a n ex p ressio n fo r th e m a g n etic force.
S o l u t i o n . T h e e le c tr o m a g n e tic force f e ca n be o b ta in ed by ta k in g th e p a r tia l d e r iv a tiv e o f
e ith e r t h e fie ld e n e r g y fu n c tio n or co -en er g y fu n ctio n W
F rom m a g n e tic s to r e d e n e r g y c o n s id e r a tio n s , th e m a g n e tic force is
d W f l d ( v , X)
£ =- ...(2 .2 1 )
dx

From E q . (2 .9 ) x) = J0 t'dvp = JQ ^

1 _ £ f l) - 3/2
Te dx * 1/2 ‘ 2 2 2
V /
_ V2 J _ ...(2 .4 1 )
- 4 • X3/2

From th e c o -e n e r g y c o n s id e r a tio n s
...(2 .2 3 )

B u t from E q . (2 .1 0 ), x) = J0 = J0 ** ^ =x ~2

:2
vl/2 i. i! I „ - l /2
x 2 2 2

I X ...(2.41a)
4 j:
1/2

Scanned by Cam Scanner


' i f

178 Electrical Machinery


(Art. 2.2

A lso , from E q. (2 .3 0 ),
I . 1
1 . d . .1 / 2 , _ i_
1/2 •(2.416)
~2 K ( 4" x

.1/2 or i = .1/2
But y = ix

S u b s titu tio n o f th is v a lu e o f i in Eq. 2 .4 1 (a ) g iv e s,


_y2 1 1 1 ...(2.41c)
^ t ' 4 7 * 4 x 3/2
T h u s th e m a g n itu d e o f force is th e sa m e , w h e th e r it is c a lc u la te d from field-energy or *

enl S T J . relay, /h n c fio n o f rWafron ~ (Ae curne„( , « *


excitin g coil, the p ositio n o f a rm a tu re x a n d the flu x lin kages y is g iven y
i = 2 \ ? + 3 \ y { l - x + x ? ) ,x > 0 .5
F in d the force on the arm a tu re a s a function o f y .
S o lu tio n . * - 2 y 3 + 3 y ( 1 - x + x 2)

F ie ld en erg y sto red , (V, x) - £ « * ( » - j ’ |2 VS + 3 V (1 - x + x 2)l d V

=^ - +3 ^ (1 - X + X 2)

M agn etic force is g iv en by Eq. (2.21).


(y , x) _ d_ ^ + ^ ( l - x + x 2)
f'= - dx dx

0 + ^ ( 0 - l + 2 x )l = ^ ( (1
l -- 2x).

For x > 0 .5 , f, is n e g a tiv e , th erefo re f , a cts to d ecrea se th e field en erg y stored at constant

flu x linksi?6S * *
E x a m p le 2.4 . The sim ple m agnetie relay o f Fig.(a) gave the following y -i char

(i) O pen positio n , y = 0.04 i W b-turn for Wb-turns


a ll valu es o f cu rren t i, Closed position
(ii) C losed positio n , y = 0 .0 6 i 0 < i < 2 0 .
= 1.2 + 0.03 (i - 2 0 ) i >20.
For an a rm a tu re m ovem en t from open to
closed p o sitio n , fin d the m agn itu de o f average
m a g n e tic force. The a ir-g a p length is 2 cm ].2
a n d the cu rren t d u rin g arm a tu re m ovem en t
is a s su m e d to rem a in con stan t a t 40 A. ^
S o l u t i o n . T h e v a r ia tio n s o f flu x lin k a g e
w ith c u r r e n t are sk e tc h e d in F ig. 2 .1 1 , both
for op en an d clo sed p o sitio n s o f th e arm a tu re.
B is th e o p e r a tin g p o in t for open p o sitio n and
C is th e o p e r a tin g p o in t for th e closed p o si­
20 A
t i o n . T h e V * lo c u s d u r in g t h e a r m a t u r e
on Exam ple 2.4

Scanned by C am Scanner
Electromechanical Energy Conversion Principle* 179

movement is a lo n g th e v e r tic a l lin e B C , sin c e cu rren t rem a in s co n sta n t at 4 0 A


The m ech a n ica l w ork d o n e d u r in g th e a rm a tu re m ovem en t
= a rea O A B C D O = area O D CFEO - area OABFEO

|< 2 0 ) ( 1 .2 ) + | ( ] . 2 + l.8 ) ( 2 0 ) \ (40) (1.6)

= 4 2 - 3 2 = 10 jo u le s.

A verage e le c t r o m a g n e t ic force, N-turns Ar“ A

L j f r j y r
f. = — -— = 5 0 0 n ew to n s.
,t(av) 2 x 1 0 2 | { ) Movable
Mo

E x a m p le 2 .5 . For the electrom agn et show n in Fig


2 12 (a), obtain an expression for the m agn etic force fr.
J k W
-
The reluctance is a s su m e d to be offered by the a ir g a p Fig 2 12 «ii Electrom agnet Exam ple 2 5
alone. M agnetic leakage a n d frin g in g is neglected.
^ N -tu rn s g—
S o lu tio n . A ss u m e th a t th e m o v a b le part tr a v e ls a Saoo 1 1
distance x in t h e d ir e c tio n o f m a g n e tic fo rce f* a s
shown in Fig. 2 .1 2 (6). A fter th is d isp la c e m e n t o f m a g ­ jr sn sr
nitude x, th e air g a p le n g th red u ces to (g - x). (g-xT*
Three a lte r n a tiv e m e th o d s o f fu rth er a tta ck are il­ Fig 2 12 ih) Pertaining to Exam ple 2 5
lustrated for th is e x a m p le .

i) For a gap le n g th o f (g - x), th e relu cta n ce is g iven by

Rl = ^
* M
I 9 d R le 1 2 d g -x 2_L
ft = ~ 2 g dx 2^g dx = 2°*

A ir-g a p flux,
Air-gap flux d e n sity , A rea A

1 B } •A 2 1B / A
f' 2 Mo* 2 Mo

Mo-*
(ti) Air-gap p e r m ea n ce
g-x
1 -I f* * -[* £ .
fe = *
dx ~2 dx[g-x

or f . V y J I?A-
f' - r ‘ ( g - x ?

Now m m ( ,i N = H , i g - x ) = - ^ i g - x )

1 b/ a
2
Mo

s z s s . - f» * - « * * - ,*a
equation from Kqa (2.30) to <2 40)

Scanned by Cam Scanner


I Art ) i
180 E le c tr ic a l M a c h in e r y

( i n ) T h e m a g n e ttic
ic nf ie
e ild
a --ee nn e irggvy s to r e d in g a p (-g * ) *8 P *
= , F ield e n e r g y d e n s ,ty )(V o lu m e o f th e m a g n e tic field ,n the air gapl
2-1
1 *L [A (g -x )\
2 p0
1 ^ ! A(g-x) . i
/• = _____^ <<&. JC) = - y - 2 p0
!' dx 3r 2 p0
i
Sam e results • t raori h
are obtained b yv th
thep ttn
h rreem
e e m te tu.
h o d s a s i ll u s t r a t e d a b o v e .. .
li ^ flo r tr n m a e n e t is d is p la c e d a x ia lly by an amount of

, mE: z * *- * - * * - * * !
le n d in g to b rin g th em in to a x ia l a lig n m en t.
The o th er d a ta are g iven below : N-turns 1mm

Pole w id th , w p = 6 m m / / / V I /
Flux d e n s ity in the a ir-g a p = 0 .8 W b /m 2 or tesla. —
S o l u t i o n . A s s u m e t h a t a ll t h e fie ld e n e r g y is c o n t a in e d
in o v e r la p p in g s u r fa c e s o f t h e p o le f a c e s . L e t t h e o v e r la p p in g
h e ig h t b e x a s s h o w n in F ig 2 .1 3 . T h e n r e lu c t a n c e o f t h e o v e r ­
Fig 2 13. E lectrom agnet. Example 26
la p p in g regio n is
_ g
Po X U p
•it _ 11 a 2 p i _ ^ a 2 8 __
and - 22 ‘ " 2 UrXW„
VoXWp
bW 1 A'
riX 2 Mo X
N ow 0, =
1 f l , 2 u „ 2 x 2g l B / W(g ..(2.421

2 Po “ ’p x ‘
1 =5 p0

A lte r n a tiv e ly . VV^ = | F 2 \ = | (»N )‘

r d* V t 1 / Kr>2

But ■A’ = g
Po

- i j ^
*•5 Po*J g
(2.42fl>
= 1 5 l^
” 2 Po
N o te th a t ft d o e s n o t d ep en d on th e a m o u n t o f a x ia l d is p la c e m e n t.
S u b stitu tio n o f t h e values in E q. (2 .4 2 ) gives
r _ 1 (0 .8 )2 (6 x K T3) (3 x 10" 3)
I* 2 4- ------------7
x x 10'
= 4 .5 8 4 n e w to n s.

Scanned by Cam Scanner


Art- ^ 1 ____________________
Electrom echanical Energy Conversion Principles 181
E x a m p le 2.7. A coil a n d plun ger m agnet is
shown in Fig. 2.14. The coil has 1500 turns and
carries a constant current o f 3 amperes. A ll the
reluctance is assu m ed to be offered by the air-gap
g. Magnetic leakage a n d fringin g is neglected.
(a) For gap lengths o f2.0,1.50,1.00 and 0.5 cm,
find air-gap flux densities and coil inductances. ’
(b) Find the energy sto red in the m agnetic
field for different gap-lengths given in p a rt (a).
(c) Compute the electrom agnetic forces for the
corresponding gap lengths.
(d) Find the m ech an ical work done, i f the
plunger is a llo w e d to m ove slo w ly from
g = 2.0 cm to g = 0.5 cm.
(e) If the coil resistance is neglected, find the
electrical energy su p p lied by the source for the
conditions o f p a rt (d).
S o lu t io n , (a ) C o n s id e r t h e p lu n g e r to m ove
by a sm all d is p la c e m e n t x in t h e d ir e c tio n o f m a g - 2 14 Coil and Plun§er m agnet, Exam ple 2.7.
Fig
netic force, so th a t t h e n e w a ir -g a p le n g th b eco m es (g - x). T h en m.m.f. per u n it a ir-g a p le n g th

iN
g -x
A ir-gap flu x d e n s it y for a n y g a p le n g th is

HoiN

_ F lu x lin k a g e s N .. „ N m .m .f.
N ow in d u cta n ce L = — ^ ---------- — = T (h») = T
C u rren t R elu cta n ce

N f Ni
= N 2 ■P erm ea n ce
. R elu cta n ce^
gap len g th (g - x), t h e co il in d u c ta n c e L z is g iv e n by

M
Lr = N 2
(g -x )
The v a lu e s o f B g a n d L x a r e g iv e n b elo w in ta b u la r form for d ifferen t g a p le n g th s .

r in cm 0.5 1.0 1.50


0

cm (gap length) 2.00 1.50 1.00 0.50

Bi *n. Wb/m2 0.2827 0.377 0.565 1.131

0.178 0.237 0.3553 0.7106

to) E n ergy sto r e d in th e m a g n e tic field ,

Scanned by Cam Scanner


182 E le c tr ic a l M a c h in e r y
of ant at for d iffere n t gap le n g th s is
S in c e c u r r e n t r e m a in s c o n s ta n t a t J A , Wfjd
W n, : 0 .8 , 1 0 6 7 , 1.60, 3 .2 w a tt-se c .
atv„,, ,J 3 u ,ix )
(c) M a g n e tic fo rc e fe~ ^x dx ’
. i. f cp m n d rela tio n m u s t be u sed .
S in c e c u r r e n t i r e m a in s c o n s ta n t, th e sec

fe = (l>x) " 2 dx
N2
= N'
But R elu cta n ce (g -x )

r_ = 0 .1 7 8 H.
F or x = 0, coil in d u c ta n c e L - J X g
zL
(g -x )

an d d
dxx [ g - x j
2 2 ( g - x )2
i oivp n b elow in tabular form.
T h e m a g n itu d e o f force f„ for d iffe r e n t g a p le n g th s. ,s g iv e n be! _______^_______
--------------------- . A E

2 .00
{g - x) in cm ‘

f. in Newtons 40 ^ / ^

m e c h a n ic a l w o rk d o n e is
, r 1 -2 _ .g k - x d x
ft 22 (i gg --Xx) f frnm 0 = 2.0 cmto
, T o ta l m e c h a n ic a l w o rk d o n e H U , . a s th e p lu n g e r is a llo w e d to m ove
) 1.5
g = 0 .5 cm , is g iv e n by
1 ,
_ L _ dx = i i 2g i
»U * = ii W o (g -* > 2
(g -* ) 0

J _ _ 1 = 2.4 watt-sec
= i(3)2 (2) (0.178) 0.5 2
Lt
d W elec = e id t = i d y x
(e )
•r •
N o w flu x lin k a g e s V *= x= (g -x ) 1

dVx rL :
~T~ ~ . ,2
(g - x)^

Thus
.1.5
■2 . f 15 tfa
H W = ‘V J 0 ^ ) 2‘
and
nl.5
1 _ 4 8 w a tt sec
= (3)2 (2) (0.178) ( g - x )

Scanned by Cam Scanner


I
Electromechanical Energy Conversion Principles 183

vtw th e to ta l e n e r g y su p p lie d by sou rce is 4.8 w a tt-sec and m ech an ical w ork done is 2.4
c T he r e s t o f t h e e n e r g y , acco rd in g to Eq. (2.3), is stored in th e m agn etic field. T hat is
w ^ he electric e n e r g y s u p p lie d is co n v erted to m ech an ical en ergy and th e rem a in in g h a lf is
rl in th e m a g n e tic fie ld . In o th er w ord s, th e m ech an ical work done is equal to th e en efg y
1° .
St0fed in th e m a_ ^g n e tic fie ld oa ft pc on n
ncfonf nnrronf
s ta n t cu rren t (h ere U
it i is
o Q A^
3 A).
St0Iff m p art (6), it is s e e n t h a t th e field en erg y stored ROTOR LONG
length 2 .0 0 cm is 0 .8 w a tt-s e c and a t gap len g th AXIS
at its v a lu e is 3 .2 w a tt-s e c . T h erefore, in crea se in ROTOR
°'5 Cmp’t ir e n e r g y s to r e d a s g a p le n g th ch a n g es from dO
2 ooto 0.5 cm, is e q u a l to (3 .2 - 0.8) = 2.4 w att-sec.

This verifies th e a b o v e s ta te m e n t. -STATOR


E x a m p le 2 .8 . F o r th e m e c h a n ic a l co n fig u r a tio n POLE AXIS

n in F ig 2.15, a s s u m e th a t a ll the field energy is STATOR


Z l n t in the o v erla p p in g regions. T h e radius r is m uch
Z a te r than air-gap len g th g . C a lcu la te th e m agnitude o f
toraae when the m a x im u m flu x d en sity m the air-gap is
t a t a i to 2.2 T. T h e o th e r d a ta are a s follow s :
R a d iu s. r = 50m m ;
Fig. 2.15. P e r t a i n i n g to E xa m p le 2.8.
Gap len gth . g=2m m ;
Length norm al to ra d iu s r is I - 10 mm.
Solution. Let th e a n gle o f overlap be 0 radians. Then reluctance of two air-gaps in series is

Rlg ho(r0)z
d
= 4>*
T e = - ^ Vg2 ^
d& W = - h g2dQ po(r0)Z

§g = B g A = Bg (rQl)
But air-gap flu x,
(B jg r l
Te = Bg(rQD ho y
Po_rZ
2 d V o rQl - ( W
dO 2g 4g
Alternatively, 2' d0
2B j £
But m.m.f., iN =
ho
'B ggrl
r2 B g g " 2 H - -
Te =
v » I
i , > he air-gap re lu c ta n ce , ro to r lo n g ax is

fends to a lig n i t s e l f w i t h t h e s ta to r - p o le axis< e x p re s s io n d e riv e d ab o ve, g iv e s

S u b stitu tio n o f t h e n u m e r ic a l . 3.8515 N-m.


(2.2)
4n x 10 n eg lig ib le a s c o m p a r e d to
a ir-g a p length is not
E x a m p le 2 .9 . R ep ea t E xam ple 2 .8 m ca
r°tor radius.
( 1 '
S o lu tio n . A verage gap r a d iu s =

Scanned by Cam Scanner


184 E le c tr ic a l M a c h in e r y

0 I.
A r e a n o r m a l to flu x p a th , A g

R e l u c ta n c e o ffered b y tw o a ir g a p s

Mo
f o Ql
r + 2
<

- 8
2g / \
1 7----- i ) =♦2 — f 1 ^
- " 2 *g dQ 0 / Mo r + 2 * I 02
Mo r+ 2 8

0 I
B u t a ir -g a p flu x , tyg - B g A g Bg r + 2*

gl r ^ g
r 2 >2 V.
( 1 1 0/ ___
T
A C - V f 1 1I Q 2 Mo
Mo r + 2*
k /
T o r q u e c a n a ls o b e o b ta in e d from t h e d e r iv a tiv e o f c o -e n e r g y W „ / from Eq. (2.10).

m .m .f. _ i ■N 0 I
A ir-g a p flu x , AC 2g r + 2 8

iN 2 0 /
F lu x lin k a g e s , Vg = N $g = Y g Vo r+ 2 8

C o -e n e r g y sto r e d in th e a ir g a p field ,
1 . 1 i2 N 2 ' 0/
V = 2 ^ I= 2 T 7 110 r + 2 8
a n y (i - 0)
F rom E q. (2 .2 5 ), Te - -^q

1 fN 2 ( 1
Mo
4 £

But B = ^ = r ^ - Mo f 1 1Q I
5 A , |_ 2 g r+ 2 8 0/
\ -

Mo •j g
B„ =
2*?
, „ 2 g Be . . .
S u b s t i t u t i n g t h e v a l u e o f iN = — rr— *=■in E q. (i) g i v e s
Mo

B, - 2 g
T = - •Mo l =B
e 4-* L mu Mo j £ ^ r + “2 8 ) '" .veS
S u b s t it u t io n o f th e n u m e r ic a l v a lu e s in t h e to r q u e e x p r e s s io n d e r iv e d a b o \e

T , (2 2 j 2 (2 x 10~3) f 1 0 x I Q '3) (50+ D - l O ' i = 3 .9 2 9 Nm


* ‘ 4 71 x 1 0 ' 7

Scanned by Cam Scanner


185
E le c tro m e c h a n ic a l E n e rg y C o n v e rs io n P rin c ip le s

^ a lem en ta ry form o f a s in g le -p h a s e relu cta n ce m otor is illu s tr a te d in F ig . 2 1 6 - I t h a s


v f sa lie n t p o les) b o th on th e sta to r a n d rotor. T h e relu cta n ce or p e r m e a n c e o f t h e m a g ­
S C i r c u i t d ep en d s on th e r e la tiv e a n g u la r p o sitio n o f th e rotor and sta to r. S in g le -p h a s e v o -
netlCnnlipd to N -tu r n coil e s ta b lis h e s sta to r p u lsa tin g flu x, w h ich cro sses th e a ir -g a p a lo *|£ ®
ag? PPn l axis. In F ig . 2 .1 6 , th e a x is o f th e sta to r flu x is in d ica ted b y a h o r iz o n ta l d o tte d
line-this axis o f sta to r p o le s is ca lled sta to r d irect a x is (or d -a x is). A n a x is 9 0 ° a w a y from e

Ro^or position q-axis


at t= 0 Y ' ] er=OOrt-6

« - axis

Rotor position }
at any tim e,t

DUS
^>
1<P Supply N -turn
L _ 0 — I coil

(6)
Fig. 2.16. Single-phase reluctance motor.

-axis is called q u a d r a tu r e a x is. A n g le 0r in F ig. Rl


1
2.16 (b), is th e sp a ce a n g le b e tw e e n sta to r d -a x is
and the long rotor-axis.
W hen s p a c e a n g le 0r = O, i.e . w h e n lo n g
rotor-axis is co in c id en t w ith sta to r cf-axis, th e R
reluctance o ffe r e d to t h e s t a t o r flu x , b y tw o '* T ic
...
small air gaps in s e r ie s w ith h ig h p erm ea b ility
iron, is m inim um . L e t t h is m in im u m relu cta n c e
be designated by R ld. W h en t h e rotor lo n g -a x is is
J tu TT 1 TtA
[ Space
2rT nnnUQ.

along th e g -a x is, i.e . w h e n t h e s p a c e a n g le


r = 90°, th e relu cta n c e offered to th e sta to r flu x,
by very la r g e air g a p s in s e r ie s w ith h ig h -
Permeability iron, is m a x im u m - le t it b e d e sig ­
n e d by R l^ i t m a y b e s e e n from F ig . 2 .1 6 th a t
en 0r = o > 2ti, etc, th e r e lu c ta n c e offered to

tbe stator flu x is R ld a n d w h e n 0r = e tc ->

relu ctance o ffered is R lq. T h e p a r a m e te r s


d and R ^ are ca lled d ir e c t-a x is a n d q u adra-
°f re^Ucta n c e s r e sp e c tiv e ly . T h e v a ria tio n
the6 U°^ance ^ w ith 0r d e p e n d s on th e sh a p e o f
tjQ
8 8babor an d rotor p o le s a n d h e r e th is v a ria
an™,’8 a88um ed to be a s in e fu n ctio n o f sp a ce
space pj 2.17. Variation of reluctance w ith space angle Or
a* g le e
as shown in Fig. 2.17.

Scanned by Cam Scanner


186 Electrical Machinery

An ex a m in a tio n o f F ig . 2 .1 7 ( a ) r e v e a ls t h a t th e r e lu c ta n c e v a r ia tio n R l w ith sd
0r can b e th o u g h t o f a s com p osed o f F ig s. 2 .1 7 (b) a n d 2 .1 7 (c). In F ig . 2 .1 7 (6) relifT
co n sta n t, th erefo re R lx can be e x p r e sse d a s 4,106

S i , = i (Rf, + R y

In F ig. 2.1 7 (c), relu cta n c e R l 2 v a r io u s sin u so id a lly . I t ca n b e e x p r e s s e d as under •

At 0r = O,J?/2 = - | ( ^ - i ? Z £f)

0r = (R lq - R ld) COS 0°
At ' e r = 45°, R l 2 = 0
or
Rh = - 2 (R lq ~ M d) cos 2 ( n /4 )

At er= 9 0 - = n / 2 , R l, = \ - R ld) = - cos 2 („/2 )

T herefore, in g en era l, th e relu cta n c e v a r ia tio n in F ig. 2 .1 7 (c) c a n be ex p ressed as

R l2 = - \ ( R l , - RId) cos 2 0r ((j)

space M g lee’ 8' <0 !‘° ab°Ve g'VeS 1116 Va'Ue ° f reluctance R l for Fig. 2.17 (a) at any
R l = R ll + R l 2

= 2 (R lq + Rid) ~ ^ (R lq - R ld) cos 2 0,. (2 43)

I From Eq. (2.31), th e torque in term s o f relu cta n c e is


T = —— 4>2 ^Rl_
' 2 9 d e7 ...(2.44)
S u b s t itu tio n o f th e v a lu e o f R l from Eq. (2 .4 3 ), in Eq. (2 .4 4 ) g iv e s

i A ssu m e th a t th e llu x produced bv coil m m f xr •


Ti ~ ~ 2t'2
^
' ( R l , - R l d)sin 2 9 .

w in d in g resista n ce in Fig. 2 .1 6 m av hp Y Y u 1S con ^ ne(l to th e sta to r core. The stator


d evices th e w in d in g resista n ces are u s u a llv t m t n Ae c a u se in w e ll-d e sig n e d energy-conversion
m agn itu d e o f cou n ter or reaction e m f P mi,o+ 1 . COnseQu e n ce o f th e s e assum ptions, the
K irch h off s vo lta g e law . w a y s be eq u a l to th e a p p lied voltage Vt, as per

v t = E = ^2 n fN fymax

If ap p lied v o lta g e Vt is sin u so id a l, th e sta tn r r o •


ta n e o u s v a lu e <p o f th e sin u so id a l flu x be w r itte n ^ *S a ^S° 3 S*nG w a v e ' the instan-
4* = ^Wr COS COt
For a c o n sta n t m a g n itu d e o f applied v o lta g e V ,
r em a in co n sta n t, w h e th e r sp ace anplp o " m a g n itu d e o f sta to r flux <J>must ais°
- . „ r m f qU 10 “ rO a e g r e e ' 9 0 ” OT *>aS a n y other value in
F ig . 2 .1 6 . T h e ex p ressio n , flux = — J
relu cta n ce reveala th a t w h e n relu cta n c e offered to constant

Scanned 5yT ^ am 5canner


^r(t 2.3] ElectromechanicalEnergy Conversion Princi

flux is m in im u m , i.e . , R ld , t h e m a g n itu d e o f c u r r e n t in N tu r n s m u s t a lso b e m in im u m . S im ila r ­


ly w h e n t h e r e lu c t a n c e o ffe r e d is m a x im u m , i.e. R lqt th e m a g n itu d e o f c u r r e n t in N t u r n s m u s t
be m a x im u m , in o r d e r to f u r n is h a c o n s ta n t flu x .
A t tim e t = 0 , t h e ro to r p o s itio n is 8° a w a y from sta to r tf-a x is a s sh o w n in F ig. 2 .1 6 (a ) an
flux is m a x im u m a s p er E q . (2 .4 6 ). T h is m a x im u m flu x produced b y sta to r , c o m p le te s its; pa
th rou gh s ta to r ir o n , ro to r ir o n a n d tw o a ir g a p s in se r ie s. A s a r e s u lt o f th is flu x p a th , n or a
sou th p o le s p r o d u c e d on t h e s ta to r , a ttr a c t so u th and n o rth p o le s in d u c e d on th e ro ° r
torque so d e v e lo p e d t u r n s t h e ro to r in th e a s su m e d d irectio n o f r o ta tio n i.e. clockw ise^
n a tiv e w a y o f d e s c r ib in g th e a b o v e h a p p e n in g s is th a t th e sta to r flu x h a s a te n d e n c y to f o llo w a
m in im u m r e lu c ta n c e p a th (or h a s a te n d e n c y to sh o rten its m a g n etic ;p a t ).
relu cta n c e p o s itio n c a n b e a t t a in e d o n ly i f th e rotor tu r n s clo ck w ise. I f 5 is ta k e n on
stde o f s ta to r d - a x is , i.e. o p p o site to t h a t sh o w n in F ig . 2 .1 6 (a ), th e rotor w o u ld r o ta te m an

an‘ F t f h e f n s “ e o u s p o s itio n o f th e rotor a t a n y t im e f a s sh o w n in F ig . 2 .1 6 ib ). th e sp a c e


an gle 0, ca n b e e x p r e s s e d a s 47)
0r = (ci>r f - 8) '
w h ere <o is th e s h a ft a n g u la r v e lo c ity in rad /sec. A ctu a lly th e torq u e d ev elo p e d b y a s m g l y -

. . . _

sta n t a n g u la r v e lo c ity o f ov ra d /sec. „ •


T h e d iffe r e n c e b e t w e e n t h e t im e a n g u la r v e lo c ity «. (s u p p y M g »
o> = 2 nf) an d t h e s h a ft a n g u la r v e lo c ity cor ( = 2 im ) m u st b e k e p t in m in d . H e r e n

sp eed in r.p .s. an(j (2 .47) r e s p e c tiv e ly ; in Eq. (2 .4 5 ),


S u b s titu tio n o f t h e v a lu e s o f <t> a n d 0r from E qs. u . >
gives in s ta n ta n e o u s v a lu e o f to rq u e as

T. = - 2\ o L « ^ - Rld) sin l2<a' " 28)

N ow cos2 oif = ^ (1 + co s 2uX)


. = Isin ( 2 ^ - 2 5 ) + cos 2ck sin (2a>r f-2 S )l -(2.48)

B u t s in A cos B = £ [sin (A + B ) + s in (A - B )]

T h erefore Eq. (2 .4 8 ) ca n b e w r itte n a s

T — i k J W . - lsin <2“ ' ‘ - 28) + 1 Si" ^ + 2“ ‘ " 28>


+ - sin (2cor t - 2oX - 26)] ...(2 .4 9 )
2

E , . ,2 .4 9 , g iv e s t h e in s t a n t a n e o u s v a lu e o f t h e ™
be o b ta in ed b y fin d in g t h e a v e r a g e v a lu e o f E q . . ^ ^ th e n a v e r a g e torq u e over th e
a n g u la r v e lo c ity (i)r is n o t e q u a l to th e tim e gu b ra ck et are v a ry in g w ith tim e,
co m p lete cy cle is zero, s in c e a ll th e th r e e s in e te r m s w ith in th e b ra ck et .
. Te <alj) = 0
w h e n oir * w.
u th e n in s ta n ta n e o u s torq u e is
H o w ev er , i f th e rotor s p e e d is su c h th a t o),. - a), tn en
1 • i* i + —sin (—25)] ...(2 .5 0 )
T , — \ K . J (W , - (sin (2ok - 25) + 5 sm (4“ ‘ ' 2

o u a iiiic u u_y v ^ a i i l o u a i 11 i c i
188 Electrical Machinery _(Art2j

In Eq (2 50), th e first tw o te r m s w ith in t h e b r a c k e ts a r e s e e n to v a r y a t tw ic e and four times


th e lin e freq u en cy r e sp e c tiv e ly . T h erefo re, t h e ir a v e r a g e v a lu e o v e r a c o m p le te cycle is Zero
H ow ever, th e la s t term w ith in th e b ra ck et, i.e. £ s in ( - 25) d o e s n o t v a r y w ith tim e,
"l
(RL - R h) -sin ( - 25)
c lac) ~ ^ 0max

1
= + i < t W 2 ( R L - R l d ) s in 26 ...(2.51)
8
This show s th at reluctance motor can develop torque o n ly a t o n e sp eed , i.e. th e synchronous speed
Reluctance motor, as usual, is therefore not a self-startin g m otor. A w in d in g p u t in th e rotor pole-faces
serves to produce induction m otor torq u e d u rin g s ta r tin g a n d a s th e s h a ft sp eed approaches
synchronous speed, rotor pulls into step and con tin u es ru n n in g a t syn ch ron ou s speed.
M axim um av era g e torque is

e (av) m = 1 4>Lr (RlQ- R ld) a n d occu rs w h e n 6 = 4 5 c

T h e average torque g iv en by Eq. (2 .5 1 ) can a lte r n a tiv e ly b e e x p r e s s e d a s follow s :


. . . M2
D irect-axis in d u ctan ce, Ld =
R ld
4 n 2
and quadrature axis in d u ctan ce. L = ——
’ q Rl.Q
0,max 2 Ov2
e (au) sin 25
8 Ld
1 _ 1
N2 sin 25.
uLq COLd
Now
= Xd is called th e d irect-a x is rea c ta n c e
and
°*Lq = X q, th e q -a x is reactan ce.
F urther V, = V2K f N i „
or at*
9"“ TtoTf a
T e (au) ~- —L.
4to sin 25
...(2.52)

A lso r1 e (au) — % (Yl Y l


——L 1
4(0 X a ' X , sin 25
vt
or
T ' {av) = 4 (o ~ Id) s i n 2 8 ...(2.53)
H ere Id, Iq are th e currents ta k en from th * i ,
m axim u m relu ctan ce p o sitio n s resp ectiv ely y 60 r0t° r is h eld in minimurn ^
R elu ctan ces R ld and R lq depend on th e g eo m etrv n f fk
con stan t for a given m otor. I f th e m a g n itu d e o f su n 1 n m a.^n e ti c circu it an d are, therefore,
therefore, th e only v a riab le in Eq. (2.51) i8 th e an ] T a 6 ** COnstant»flu x ^ is constant,
Te(av) w ith 5 is d ep icted in Fig. 2.18. I f load ton m ® 6 fk P^0 t v a ria tio n of torque
ad justs it s e lf till relu ctan ce torque becom es eo u a l tn i j m 0t° r chan^ , th e n th e angle 6
u su ally called th e load -an gle. W ith th e in crea se o f lo a d n ?k to rq u e - in v ie w o f th is, angle 5 is
it reaches its m axim u m v a lu e o f 45°. A ny fu rth er incrp ° n . e1m o to r>i° a d a n g le 5 increases till
to sta ll (fall ou t o f step or loose sy n ch ro n ism ) Ga8e m on m o to r» w ould cause it

Scanned by Cam Scanner


2.3]
Aft
^ f S l ^ k a L f nergy Cooversion P r i-r ip ,..
The s i n g l e - p h a s e r e l u c t a n c e m a c h in e ca n h 189
i e to work as a g e n e r a to r also. For th is pur
11 e, th e m m aa cc h
h in
in ee mm ust, ~ s t as
u s t hbae d r iv e n fir pur-a e(a v )
| Mot or
ductance m otor and th e n m ech a n ica l pow er is
supplied to th e sh a ft from th e prim e-m over As G en era to r
the mechanical p ow er in p u t to th e sh a ft is in!
creased, the sp ace p o sitio n o f th e rotor advanc
. I . _ /v l A A ri A n ~ _ _ « . . :es , -4 5 °
so that angle S fir st d e c r e a se s and th e n becom eess
\ • ) cs" 'g
negative- As soon a s 8 b e c o m e s n e g a tiv e, the \ 1 /
machine sta rts w o rk in g as a gen erator convert­ \ 1 /
\ i /
ing the m e c h a n ic a l p o w er in p u t to electrical \ 1 /
\ \. 1i //
power output, (F ig. 2.18).

Single-phase r e lu c ta n c e m otors are used ex­ Fig. 2.18. Torque-load angle characteristics of
tensively in d r iv in g e le c tr ic clocks and other reluctance motor.

^ “ C°M t “ t s„p p ,y frequen-

In th e s i n g l y - e x c i t e d m o t o r , i l lu s t r a t e d in F ig . 2 .1 6 , th e r elu cta n c e torque is d ev elop ed b e ­


cause th e r e l u c t a n c e s e e n b y t h e s t a t o r f lu x v a r ie s w ith rotor m o v em en t. If th e s a lie n t p ole rotor
of Fig. 2 .1 6 i s r e p l a c e d b y a c y lin d r ic a l ro to r a s s h o w n in F ig . 2 .1 9 (a) th e relu cta n ce offered to
the stator f lu x r e m a i n s c o n s t a n t fo r a ll ro to r p o s itio n s —c o n se q u e n tly no relu cta n ce torq u e is
developed. I t m a y t h u s b e c o n c lu d e d t h a t t h e r e lu c ta n c e to rq u e is p r e se n t on ly in th o se e le c tr i­
cal m a ch in es in w h i c h t h e r e lu c t a n c e s e e n b y t h e w o r k in g flu x v a r ie s w ith rotor m ovem en t. For
example, in F ig . 2 . 1 9 (b ) ( a s y n c h r o n o u s m a c h in e ), th e r e lu c ta n c e torq u e is p resen t, b ecau se th e
reluctance o ffe r e d t o t h e s t a t o r - p r o d u c e d w o r k in g flu x is a fu n ctio n o f sp ace a n g le 0,.. In Fig.
2.19 (c), th e r e l u c t a n c e s e e n b y t h e r o to r -p r o d u c e d w o r k in g flu x is in v a r ia n t or con stant, th ere­
fore, no r e lu c t a n c e t o r q u e i s d e v e lo p e d .

d-axis

-a x is

• fcl R eluctance to rq u e is ab sen t,


ob) R e lu c ta n c e to rq u e is p re s e n t (c) Keiu w
*a) Salient pole ro to r of Fig. 2.16, is
replaced by cylindrical rotor. Fig. 2.19
, / M nnrp m otor illu str a te d in Fig. 2.16, fin d an ex­
E x a m p le 2 . 1 0 . F o r th e s ^n^ le 'Pk^ Sei ^ efJ t <
^ r^ :f reluctance, m a x im u m flu x d w , « . * a n d 5.
re$sion fo r th e in s ta n ta n e o u s c u r re n i expression fo r the average electrom agnetic
the w in d in g resistan ce is n e g le cted , e P
°rque in te r m s o f r e lu c ta n c e a n d m a x im a n g u la r p o sitio n 0, is illu str a te d in Fig.
, , S o lu t io n . T h e v a r ia t io n o f reluctance w . h roto gu ^ ^ .§ p v e n by E q . (2 .4 3 )
17: A n e x p r e s s io n g i v i n g t h e v a lu e o f r e lu c t a n c e t y
n d lt is repeated here fo r c o n v e n ie n c e . , ...(2.43)

Scanned by Cam Scanner


190 Electrical Machinery

A s b efore, le t th e in s t a n t a n e o u s v a lu e o f flu x b e g iv e n by

0 = 0mox COS 0it


(2-46)
N e g le c tin g th e m .m .f. req u ired for th e iron p a th , th e to ta l m .m .f. around the closed
mag.
n e tic p a th o f F ig . 2 .1 6 is g iv e n by
T o ta l m .m .f. = coil m .m .f., iN = tyRl
. _ <j> • R l
or i= ••(2.54)
N
S u b s titu tio n o f th e v a lu e s o f 0 a n d R l in Eq. (2 .5 4 ) g iv e s
0max COS Ut
I= i (R l, + ld)- \ (R l, ~ Rld)
R 28.
N
A s b efore, le t 0r = to,.t - 5.

._ [(Rlq + _ {Rlq m d) co s (2 wr t - 28)1

W ith rotor a n g u la r v elo city (or eq u a l to th e tim e a n g u la r v e lo c ity 0 ), th e expression for the
in s ta n ta n e o u s cu rre n t i b ecom es

i= [(f?/9 + R ld) cos c* - (R lq - R ld) cos (2co t - 25) cos otf]

N o w cos A cos B = ^ [cos (A + B ) + cos (A - 5)1

. 0n
I= (Rlq + R ld) cos otf - ^ (Rlq - R l d) (cos (3orf - 25) + cos (otf - 25))
2N
T h is is th e req u ired ex p ressio n for th e in s ta n ta n e o u s c u rre n t i.

N ow V = ir + ~ d t = 0 + ^ = ^ t^ N ^max cos = ~ N 0™« “ sin


.*. In sta n ta n e o u s p ow er in p u t

Pinst = vi

or P inst = - (oN <hmai sin otf • ^ ((Rlq + R ld)cos orf - \ ( R l q - R ld) (cos (3fi* - 25)
+ cos (ctf - 25)11

or P inst
A = - “ 0max
2 + g.n ^ cog (0f “ 2 ^C° S S*n ^
+ COS (cof-25) sin toll
B u t s in A cos B = | [sin (A + 5 ) + sin (A - B )]

P __
R la + R ld i
insl 2 —^ sin 2o )t - - (Rlg - R ld) [sin (4cof - 25)
251
+ s in ( - 2o yt + 25) + sin (2o )t - 25) + sin
S in c e s in (-2co£ + 25) + sin (2cot - 25) = 0, w e g e t

W0„
p. = _ —■
^ Rld s in 2 (0 1- ^ (Rlq - Rld) [sin ( 4 <ot - 2 6 ) + sin 25)]
•* inst

Scanned by Cam Scanner


/y

1 Electromechanical Energy Conversion Principles 191

T h e fir s t tw o s i n e t e r m s w it h in t h e b r a c k e t v a ry a t tw ic e a n d fo u r tim e s th e lin e freq u en cy


respeCrnLVe ^c 6 ir a v e r a £ e v a h ie is , th e r e fo r e , zero. T h e th ird s in e term d o es n o t v a r y w ith
time. T h e r e fo r e , a v e r a g e v a lu e o f t h e a b o v e e q u a tio n is
QXt>moot
P =-
~ 7 (Rlq ~ Rld) 8in 26 {R lq - R ld) s in 28
8

T* —R ^mar , „ , ^.
•• 1 e {av) ~ w - g ~ ( r o , - R ld) s in 28.

T h is to r q u e e x p r e s s io n a g r e e s w ith t h a t a lr e a d y o b ta in ed in Eq. (2 .5 1 ).
E x a m p l e 2 .1 1 . F o r th e e lec tro m a g n e tic d evice sh ow n in Fig. 2.20, the cro ss-sectio n a l area
norm al to th e flu x is A a n d th e relu cta n ce is offered by a ir *
gap alone. C o m p u te th e a v e ra g e force on the m o va b le m em - Movable member-^—] x f—
ber in te rm s o f N , x, A etc. w h en ; ' )
(a) i = I m cos (at
N e g lig ib le
(b) v = V m cos (tit. a ir g a p

S o l u t i o n , (a ) R e lu c ta n c e o ffered by air g a p = — —
hoA
r H 2 »oA
x r z
'• L x ~ X

T,r /.* \ l* 2 r 1 -2 ^ 2 M
Wm b , x ) = - i L x = o L — ; —
2 ' ~x 2
Fig. 2.20. Pertaining to Example 2.11
dW fld' (i, x) _ i ;2
From E q. (2 .2 6 ), fe =
dx =- 2 *

l W 2M ,2 a 1 + cos 2 tuf
= - 2 y2 Im cos oit~ ~ 2 x2
X
r2.
1 N M 2
fe (au) 4 x2
1 a2 _ _ i A2 - J —
(b) From Eq. (2 .3 1 ), f e = - \ d>2 ^ = - g ♦'
M
. ai
cfi
For th e e le c tr ic c ir c u it, v = ir + ^

For v = Vm c o s cot, it s s t e a d y s t a t e s o lu tio n is


V 1 ttiLx
; , - z - t a n - 1—
V2 V r 2 + c / L / r

m = COS (tit - ta n
t =
■^r2 + (ti2 L xl
Ni N V jI k A -ta n
N ow a - — ------- 1 n — T cos
* " /? / x V r2 +

S u b s t it u t in g t h is v a lu e o f d> in fo rce e x p r e s s io n

A/2 Vii, Mo A 2 co s' (tit - ta n

PoA [x2 (r2 + (ti2L 2))

Scanned by Cam Scanner


192 Electrical Machinery

r .
S u b s titu tio n o f Lx= — — give s

N 'V llh A
COS otf - t a n - 1
le o
2 ( A 2 + (a>W2 MoA)2]
1 O f W
f. inl
,e{av) 4 [(rx )2 + (caZV2 M ) 21
P a r t (6) o f th is e x a m p le can a lso be so lv ed a s u n d er .
N • )ioA
Lx = tjv*
an*
N 2VmPoA ' . . -i(0 L l
y = N0 = — cos to( - t a n —
r
Yl x y r + (0 L x
From Eq. (2 .1 2 ), field en erg y sto red is wh<
_ iv ^ i V -*
fld = 2 -L x ~ 2 and
_ aw Z
,M_(Mvx) _ _ i
F rom Eq. (2 .2 1 ), fe ~ d x 2 N 2P o A
ren
tive
S u b s titu tin g th e v a lu e o f \|/*» w e g et not
N 'V ^ io A toL ^
—s cos a t - ta n 1 —
ie o r
2 rV + ((oAT2MoA)2 V /
> 1 W2V2 moA issi
Its a v era g e v a lu e is fe (au) =
4 (rx)2 + (coA^PoA)2
2 .4 . D o u b l y - e x c i t e d M a g n e t ic S y s t e m s •
S in g ly excited m agn etic sy stem s have a w ide variety of applications, such as d e e t r a n ^ _
r e la y s m ovin g-iron iS t r u m e n t s , relu cta n ce m otors etc. M ost o f th e electromagnet,c energy
co n v ersio n d e v ic e s b elo n g to doubly-
and
e x c ite d or m u ltip ly -e x c ite d m agn etic
system s. A doubly-excited m agnetic sy s­ wht
tem is one w hich h as tw o independent
so u r c e s o f e x c ita tio n s, e x a m p les are
and
sy n ch ro n o u s m a ch in es, loudspeakers,
tachom eters, d.c. sh u n t m achines etc.
com
F i g . 2 .2 1 i l l u s t r a t e s a s im p le
(2.5
m o d e l o f a d o u b ly e x c ite d m a g n e tic
s y s te m . T h is m o d el c o n sists o f sta to r \3 (
Source
ir o n ro to r ir o n a n d b o th are o f th e
11 v X l y * v v
J
. . rm__________ t«r4tV> A T Of
s a lie n t p o le ty p e. T h e sta to r w ith N ,
tu r n s is e n e r g ise d from sou rce 1 and
th e rotor w ith N r tu r n s is ex cited from ■Stator

s o u r c e 2 . T h e m .m .fs . p ro d u ced b y
b o th th e s ta to r a n d rotor w in d in g s are
in t h e s a m e d ir e c tio n a n d m a g n etic
to rq u e Te is in t h e a n ticlo ck w ise direc­
Fig. 2.21. Doubly-excited m agnetic sy
tio n a s s h o w n in F ig . 2 .2 1 . F or con-

Scanned by Cam Scanner


irl l A] ________________ Electromechanical Energy Conversion Principles 193

' venience, m a g n etic sa tu r a tio n and h y s te r e s is are n eglected .


The d ifferen tia l electrica l en erg y in p u t to a sin g ly excited m a g n etic sy ste m , from Eq. (2.7),
is given by
d W cUc = M y .

T h ere fo re , th e d if f e r e n tia l e le c tric a l e n e rg y in p u t dW elec fro m tw o e n e rg y s o u rc e s 1 a n d 2,


jn Fig. 2.21, is

j dWelcc = * / * V . + ' (2' j j

Here y , and y r are th e in sta n ta n e o u s total flux lin k a g es o f sta to r and rotor w in d in g s resp ec­
tively. S in ce th e m a g n etic sa tu ra tio n is n eglected , y , and y r can be ex p ressed in term s o f s e lf
and m utual in d u cta n ces.
V i = W » + Mtr irl ...(2.56)
and y r Lfi,. + M rt it j
where L t = self-in d u cta n ce o f stator w inding
L r = self-in d u cta n ce o f rotor w inding
and A/ir = A/r, = m u tu al in d u ctan ce betw een stator and rotor w indings.
In Fig. 2.2 1 , in itia lly th e sp ace a n gle b etw een rotor and stator axes is 0r and both th e cur­
rents i, and ir are assu m ed zeros. N ow th e stator and rotor coils are sw itch ed on to th eir respec-
tivc energy sou rces, so th a t th e currents rise from zero to i, and ir resp ectively. I f th e rotor is
not allowed to m ove, th en d\Vmr(h is zero and Eq. (2.3) becom es
dWtUc = 0 + dWpj
Thus, w ith th e rotor held fixed, nil th e electric energy supplied by th e tw o su p p ly sou rces,
is stored in th e m agn etic field. . j
From Eq. (2.55). d ^ p d - d^ ei* ~ + l^ V r
= i,d ( L jt + M J r) + i A L j r + Ai r,it) ...(2.57)

P* „ , = N »* Lr
r = N *r
pjj From E xnm ple (2.7), L,

W N »N r
and from Art. 1.16, Afv = Mr, = —
nigr
"'here R lt = relu cta n ce seen by th e stator flux
R lr = relu ctan ce seen by th e rotor flux
and R lsr = relu cta n ce se e n by th e resu lta n t o f stator and rotor fluxes.
Since th e rotor is n ot allow ed to m ove, th e relu ctances and therefore th e in d u cta n ces are
constant. In v ie w o f th is , th e differential ch an ges in in d u ctances, i.e. dL t d L r and dM tr in Eq.
(2.57) are all zeros.
Therefore, from Eq. (2.57),
dWfu = i, Ljdi, + iM t A r + *rM^r+ 'M A 't
= it Lt d is + ir Lfdij- + Msrd(iJ.r)
The m agnetic field energy stored in establishing the currents from zero to i, and ir, is given by

"•Ml
^ ’ Wfid - L* Jo + Jo *r c^ r + ^ tr Jo •:

X o 1 '
i = 2 i* L , + 2 iriL r + Mr' i' ir ...(2 .5 8 )

Scanned by C am Scanner
194 E lectrical M achinery
' * lA rt^
F or o b ta in in g th e m a g n e tic to rq u e Te> a s su m e th e rotor to m o v e th rou gh a virtual d' "
m e n t dQr in t h e d ir e c tio n o f Te a s sh o w n in F ig. 2 .2 1 . W ith rotor m ovem en t, relu cta n ce^ 6
R lsr a n d th e r e fo r e in d u c ta n c e s Ls, L r, M sr m u st vary. T h erefore, th e differential elect
e n e r g y in p u t d W elec d u r in g v ir tu a l d isp la c e m e n t dQr, from Eq. (2 .5 7 ) is, n°a'
d W ^ c - is d (L s is + M sr ir\ + ir d [L r ir + M sr is]
= is L s d is + i 2 d ig + is M sr d ir + is ir d M sr + ir L r d ir + i 2d L r + ir Msr dit + ir ^ dM
T h e d iffe r e n tia l m a g n e tic en erg y sto red dWfld, d u rin g th e v irtu a l displacem ent d0 fr0
E q . (2 .5 8 ), is

dW fu = — is d L s + L sisd is + 7^ ir d L r + L r ir d ir + M sr is d ir + M sr ir d is + is ir dMsl.

T h e d iffe r e n tia l m ech a n ica l w ork d on e d.Wmech, d u rin g th e d isp la cem en t dQn is
d^mech = TedQr
S y b s titu tio n o f th e v a lu e s o f dW elec, dW mech and d W ^ in Eq. (2.3) gives
ic L A i. + i 2 d L s + i r L r d ir + i r2 d L r + i c Afc,. d i r + 1,. M cr d iQ+ 2 isi/lM sr

= (Te ■dQr) + 2 is d L s + L„ iKdig + ~ i r2 d L r + L r i,. d ir + is MSI. d iv + ir Msr d is + is ir dMsr

T h e u n d e r lin e d te r m s in th e above eq u a tio n g e t ca n celled and w e g et

| i 2 d L t + \ i 2 d L r + i, ir dM sr = Te dBr ...(2.59)

_ 1 . 9 dL. 1 . dLr . . d M sr
or T* ~ 2 1‘ W r * 2 l , } W r * ls lr dQr '
It can be s e e n from Eq. (2 .6 0 ) th a t th e d ifferen tia l ch a n g es o f cu rrents d is and dir do not
c o n tr ib u te to th e p ro d u ctio n o f m a g n etic torq u e Te. It m a y th erefo re be sta ted that torque Tt
d e p e n d s on (i) th e in s ta n ta n e o u s v a lu e s o f cu rren ts i, and ir and (ii) th e angular rate of change
o f in d u c ta n c e s .
I f E q . (2 .5 8 ) for Wf,d is d iffere n tia ted w ith resp ect to th e sp a ce a n gle 0,., but w ith constant
c u r r e n ts , th e r ig h t h a n d sid e o f Eq. (2 .6 0 ) is o b tain ed . T h erefore, m agn etic torque Te can also
b e o b ta in e d from th e sp a ce d eriv a tiv e o f field en erg y ex p ressio n , i.e.
T dWfid ...(2.61)
1 1 ~ 0Q (J*> r> Or)

H ere m u s t be e x p r e sse d in term s o f is, ir an d 0r and w h e n p a rtia l derivative is taken,


c u r r e n ts a r e k e p t co n sta n t.
I n te g r a tio n o f E q. (2 .5 7 ) for dW elec> w ith c o n sta n t cu rren ts, g iv e s ^^
Welec = i 2 L s + i 2 L r + 2 is ir M sr " '
F or a lin e a r m a g n e tic circu it, Wfjd = Wfld , th erefo re Eq. (2 .6 1 ) can be w ritten as
BWfu ...(2.62 0 )

input takes
C o m p a r iso n o f E q s. (2 .5 8 ), (2 .5 9 ) a n d (2 .6 2 ) r e v e a ls th a t i f th e electrica l energy J cbaucal
n la c e a t c o n s t a n t c u r r e n ts a s g iv e n b y Eq. (2 .6 2 ), th e n h a lf o f it is convcr cuneot5.
energy, E q . (2 .5 9 ) a n d th e r e m a in in g h a lf is sto red in th e m a g n e tic field ,t c o n s t* . ^ M lhe
E q . (2 .5 8 ). In o th e r w o rd s, th e m a g n e tic e n e r g y sto red a t c o n sta n t curr
m e c h a n ic a l w o rk d o n e.

Scanned by Cam Scanner


Art. 2.4]
Electromechanical Energy Conversion Principles 195

II 1

L
. 1
1► 11
*1 1
1 ■9 wj
lz j
Source 1 S o u rce 2
Fig. 2.22. Doubly excited linear m agnetic system

gTto eF ^ . e2 .2 ™ aBnetlC {o rc e f" ^ * d° Ub'y eX d ted m a e n e tic V * ™ “ n be ob tain ed b

T h e d iffe r e n tia l e le c tr ic a l e n e r g y in p u t d \V „ ,c,f rom tw o en erg y sou rces


d^elcc ~ t'l + l 2 d\\l2
H ere
Vl - tj + I2
and \|/2 = L i i'2 + Af2i i.
AJso M u = M 2\.
The m a g n itu d e o f m a g n e tic force ft can be o b tain ed by ad o p tin g th e procedure a s follow ed
for m a g n etic to r q u e Tt . It w ill be fou n d to h a v e th e ex p ressio n

e _ _1 . dLj
2 j1 #. .2 d lt2 . . d M in
* ~ 2 Zl d x + 2 ‘2 dx + ...(2.63 a)
dx
or f* = ~ ^ - ( h , i 7 ,x ) ...(2.63 b)

mW eL
~ t-)x ...(2.63 c)

An e x a m in a tio n o f E qs. (2 .6 1 ) an d (2 .6 3 ) rev ea ls th a t th e m a g n etic torq u es and forces act


in such a d irectio n a s to te n d to in cr ea se th e field en ergy a t co n sta n t cu rren ts.
2 .4 .1 . E l e c t r o m a g n e t i c a n d r e l u c t a n c e to r q u e s . The total torque developed by th e doub­
ly excited m a g n etic sy ste m is given by Eq. (2.60). T his torque expression w ill now be exam in ed in
detail and so m e in te r e stin g conclusion s w ill be obtained.
In Fig. 2 .2 1 , i f i r = 0 , th en from Eq. (2.60),

’ 2
T. = V . ...(2 .6 4 a)
dQr
1 . dL>r
and if
i, = ° . T' = 2 dd~r ...(2 .6 4 b )

Eq. (2 .6 4 a ) s h o w s t h a t ev en w ith ir = 0, th e torq u e can be d ev elo p ed , b e c a u se th e relu cta n ce


Rl* seen by th e sta to r-p ro d u c ed flu x ch a n g es w ith rotor p o sitio n . A ch a n g e o f R ls v a r ie s th e
stator self-in d u c ta n c e L s w ith 0^ S im ila r ly , Eq. (2 .6 4 b) sh o w s th a t w ith is = 0, th e torq u e can
be developed, b e c a u s e L r is a fu n ctio n o f rotor p o sitio n . In v ie w o f th is , th e torq u e ex p ressio n s
&ven by Eq. (2 .6 4 ) a re c a lle d r e lu c ta n c e to rq u es. T h erefore, in F ig . (2 .2 1 ), th e relu cta n c e torq u e
ls Present w ith a n y o n e o f th e c u r r e n ts a c tin g a lon e, t h e p h y sica l co n cep ts a b o u t th e d ev elo p ­
ment o f relu cta n c e to r q u e s in F ig. (2 .2 1 ) is a lso im p o rta n t. I f o n ly th e sta to r is e x cited , th e n th e
s ator flu x w o u ld h a v e a te n d e n c y to follow a m in im u m relu cta n c e p a th and for d oin g th is , th e
tu m s a n tic lo c k w ise . N o w i f o n ly th e rotor is ex cited , th e rotor flu x w o u ld h a v e a ten d en cy
follow a m in im u m r e lu c ta n c e p a th an d for a c h ie v in g th is , th e rotor a g a in tu r n s a n ticlo ck ­
wise.

banned by Cam Scanner


196 Electrical Machinery____________________________ __________________________________________ 24

In F ig. 2 .2 1 , su p p o se th e s a lie n t p ole rotor is rep la ced by cy lin d r ic a l rotor and is excited a *
b efore, b y c u r r e n t ir. T h en to rq u e e x p r e ssio n from E q. (2 .6 0 ) b eco m es
1 , d L r . . d M *r
T e ~ 2 l r l d f y .+ t a l r ~dOr —(2.65)

T h e r e lu c ta n c e torq u e term | is2 ^ d isa p p e a r s, b e c a u s e n o w th e relu cta n ce R l, Seen by

dL
th e s ta to r flu x d o es n o t v a ry w ith rotor m o v em en t. C o n se q u e n tly L s is c o n sta n t and js zero

T h u s th e fir s t tw o te r m s in Eq. (2 .6 0 ), in v o lv in g th e a n g u la r r a te o f ch a n g e o f self-inductan­


c e s L s an d L r, r e p r e se n t relu cta n c e to rq u es for th e m e c h a n ic a l str u c tu r e o f Fig. 2.21.

T h e la s t term is ir ,- *r rep resen ts th a t com p on en t o f to ta l torque, w h ich d epends on the currents


CZ0r
in both th e stator and rotor w in d in gs and also on th e an gu lar ra te o f ch an ge o f m utual inductance
M sr. T h is com ponent o f torque is com m only called th e electrom agnetic torque o f electromagnetic-

en ergy conversion d evices. An exam in ation o f i,ir ^ '- rev ea ls th a t th e electrom agnetic torque can

ex ist on ly i f (i) th e tw o w in d in gs, one on stator and th e oth er on th e rotor, are m u tu ally coupled and
(ii) both th e w in d in gs carry currents.
T h e p h y sica l co n cep t ab ou t th e d e v e lo p m e n t o f e le c tr o m a g n e tic torq u e in Fig. 2.21 is as
follow s. T h e n orth , so u th p o le s p rod u ced on s ta to r by is an d so u th , n o rth p o les produced on rotor
b y ir, a ttr a c t ea ch o th er te n d in g to a lig n th e ir field s. T h e to rq u e so d ev elo p ed by th e interaction
o f sta to r an d rotor m a g n etic fie ld s is th e e le c tr o m a g n e tic or in te r a c tio n torq u e.
1 . 2 dL, 1. 2 dLA
It sh o u ld be n oted th a t th e r e lu c ta n c e torq u e d o es n ot depend on the
2 is d 9 r ° r 2 lr d 6 r
. /
d M sr
d irection o f c u rre n ts in sta to r or rotor w in d in g s. B u t th e in te r a c tio n torq u e ls lr does
dQ r

d ep en d on th e d irectio n o f cu rren ts i, a n d ir. In F ig . 2 .2 1 , w ith th e g iv e n d irection of currents


is an d ir, th e e lectro m a g n etic , or in te r a c tio n , to rq u e is a n tic lo c k w ise a s sh o w n . If direction of
cu rren t, s a y ir, is rev e rsed in F ig. 2 .2 1 , th e in te r a c tio n to rq u e w o u ld be o p p o site to th at shown
in F ig . 2 .2 1 , i.e. in th e clo ck w ise d irection ; b u t th e d ire ctio n o f r e lu c ta n c e torq u e would still be
in th e s a m e d ire ctio n a s b efore i.e. a n tic lo c k w ise .
2.5. E le m e n ta r y S y n c h r o n o u s M ach in es
In t h is a r tic le to rq u e e x p r e s s io n s , in c a se o f e le m e n ta r y sy n c h r o n o u s m a ch in es, are derived
from a co n sid e r a tio n o f th e m a g n e tic field e n e r g y sto red .
(a ) S in g le -p h a se c y lin d r ic a l r o to r m a c h in e . T h e b a s ic form o f an elem en tary single­
p h a s e s y n c h r o n o u s m a c h in e o f cy lin d rica l ro to r ty p e is s h o w n in F ig . 2 .2 3 (a). T h e variation ot
s ta to r to rotor m u tu a l in d u c ta n c e M sr can b e d e te r m in e d b y c o n n e c tin g a sin g le-p h a se supply
to s ta to r w in d in g a n d a v o ltm e te r a cro ss rotor w in d in g . W h en 0r = 0, th e flu x lin k a g es with the
ro to r a r e m a x im u m an d th is is in d ic a te d by a m a x im u m v o ltm e te r r e a d in g , therefore, mutua
^ th e
in d u c ta n c e b e tw e e n s ta to r an d rotor is m a x im u m , i.e., M = M f tl & X for 01“ = 0. W hen 0, -
" * S f'

v o ltm e te r r e a d in g is zero , b e c a u se th e s ta to r w in d in g a x is is p e r p e n d ic u la r to th e rotor


a x is - t h e r e f o r e t h e m u tu a l in d u c ta n c e M „ is z e r o fo r 0r = 7 t/2 . W h e n 0r = K, th e r° fthe
lin k a g e s a r e a g a in m a x im u m b u t in th e r e v e r s e d d ir e c tio n . C o n s e q u e n tly th e polarit)

Scanned E ^C a m S can n er
rotor in d u ced e.m .f. is a lso rev ersed and th erefo re th e m u tu a l in d u cta n ce is m a x im u m b u t n e g a ­
tive, i.e., M „ = - M ^ for 0 r = n. T h e v a ria tio n o f M sr from 0r = 0 to 0r = n is a ssu m e d to be a
cosine fu n ction o f 0r a n d th is is sh o w n in Fig. 2 .2 3 (6).
The r e lu c ta n c e s e e n by th e sta to r flu x d oes n ot d ep en d on th e rotor m o v em en t, th erefo re
L, is con stan t. S im ila r ly L r is a lso co n sta n t, Fig. 2.2 3 (b). It is seen from Fig. 2.2 3 (b) th a t th e
mutual in d u cta n ce M sr a t a n y a n g le 0r can be ex p ressed as
M v = M max cos 0r.
Here 0r is th e sp a c e a n g le b etw een sta to r and rotor field axes.
Let th e s ta to r a n d rotor in sta n ta n e o u s cu rren ts be is and ir resp ectively, th en

cos 0r

T orque, Te — ^ (is, ir, ® r )

—0 + 0 —it ir M max sin 0r ——i, ir M max sin 0r.


N eg a tiv e sig n in d ic a te s th a t th e torque Tt ten d s to reduce th e a n gle 0r. T h is torq u e ex p res­
sion is ap p licab le w h e th e r th e rotor is rev o lv in g or station ary.
In a syn ch ron o u s m a ch in e, d.c. excita tio n is applied eith er to sta to r w in d in g or to th e rotor
winding.
Let ir = h o i, = Im cos a*
and Qr = ov* - 6.
Here = rotor an gu lar velocity in rad/sec.
6 = rotor an gu lar position a t tim e t = 0
and / - V27, is th e m axim u m v a lu e o f sta to r current
T' = - Ijc Im Mmax cos art sin (o ^ - 8)
1
= - I d c 1 m M m ax q fein ( w j + wt - 8) + sin ( ( V - 8 - utf))
2
If u r * to, th en Te (aw) = 0.

canned
5?Cc by Cam Scanner
I9N F.lectrkal Machinery

11' ro to r r u n s a t s y n c h r o n o u s s p e e d , i.e., (i)r - U), th e n


7j, M„ . gjn (2oy _ 5) + s in ( - 6)1

max
T 2 sin 8 ..(2.66)
1 c (at I

(6 i T w o - p h a s e c y l i n d r i c a l r o t o r s y n c h r o n o u s m a c h i n e . Fig. 2 .2 4 (a) illustrates an


e le m e n ta r y tw o -p h a se c y l in d r ic a l r o to r P h a se 6-a x is
sy n c h r o n o u s m a c h in e w h e r e tw o p h a se arm a tu re
t
w in d in g s are d e sig n a te d by a , P T h e sp a ce a n g le
0, is th e a n g le b etw een field w in d in g a x is an d p h a se
ot-axis. Field a x is
Field
w in d in g
T h e v a r ia tio n o f m u tu a l in d u c ta n c e A be ­
tw e e n p h a se a and field w in d in g /"and A/(y b etw een
p h a se P and field w in d in g f, can be d eterm in e d by
c o n n e c tin g a s in g le -p h a s e su p p ly to rotor w in d in g ^ — j _
and tw o v o ltm e te r s, o n e a cro ss each p h a se w in d in g phase
o f t h e s ta t o r . W h en 0, = 0, m u tu a l flu x lin k a g e s c(-oxis
w ith p h a se a are m a x im u m , but w ith p h a se P th e
flux lin k a g e s are zero. T h is is in d ica ted by m a x i­
m u m v o ltm e te r rea d in g a cro ss p h a se a an d zero
v o l t m e t e r r e a d i n g a c r o s s p h a s e p. H e n c e
M..f = A/ma, an d M w = 0 for 0r = 0 W h en 0, = n / 2. it Fig. 2.24. (a) Two-phase cylindrical rotor
synchronous machine.
can be sh o w n th a t = 0 a n d A /|y = A t max- F o r
0, = n, it can a lso be sh o w n th a t Af,y = - and *
A/(Vr = 0 . (Fig. 2 .2 4 (6)1.

) T h e r e lu c ta n c e offered to th e (lu x e s p rod u ced by c_


p h a se a and P w in d in g s do»*s n ot v a ry w ith rotor o
m o v e m e n t T h e r e fo r e , s e lf - in d u c t a n c e L,„ fo r |
UI

p h a s e s a . p r e s p e c tiv e ly a re c o n sta n t an d a re eq u a l
to e a c h o th e r . S im ila r ly , s e l f in d u c ta n c e Lf o f th e
field w in d in g is c o n s ta n t. S in c e th e a x e s o f p h a s e tx
an d p h a s e p w in d in g s a re n o rm a l to ea ch o th e r and
a ir -g a p is u n ifo rm , th e m u tu a l in d u c ta n c e A f„ p b e­
tw e e n « . p w in d in g s is zero.
F rom F ig . 2 .2 3 ( b ) . M a f = M max co s 0,
an d —A/max
L et ia . in b e th e p h a s e c u r r e n ts a n d if t h e fie ld
Fig. 2.24. (fc) Its inductance variations wl
cu r r e n t. T h e m a _g n e tic field e n e r g y s to r e d is space angle Or-

IV,M ( i0 , ip, if,8.) = ^ i l L„ * 5 ip h * \ jfiL+ i „ ip


sin#'
= ^ i« t . a + k ip ^.p + o *1 C f+ iQip + ia if M maJ c o s 8 r + ip m

The electromagnetic torque Tt is

T ,= ^ («„, ip, if. e r) = - i„ i,M m s in 8 r + ip i,M m„ cos 6r

Scanned by Cam Scanner


lf=lf (direct cu rre n t in field w in d in g)
■ = / cos CO/, ip = I m sin cat
1(1
0r = a)r / - &•
A =- lrL M sin « * i - 5) cos a t + lr l mMma cos (lM . 8) sin m
= l f l m M mrtl lsin ^ cos (to,./ - 6) - sin (a^ - 8) cos co/]

3) = lfImMmcx [sin (cof - « ,/+ 5)1


...(2.67)
$hen cor * to, Tt (ail) = 0.
n
If the rotor is ru n n in g a t sy n ch ro n o u s sp eed, th en cor = co.
. Te = h K M max [sin (to/ - to/ + 8)]
~ If Im M max sin 8. (2 68)
This shows th at torq u e Tt in a 2 -p h a se synchronous m achine is constant at all in sta n ts of
sine-
The voltage eq u ation for p h a se a can be w ritten as
"a = '’a *a + P V a
But Va = L a la + M a f If = L a 7m C O S COt + Mmax If C O S (CO/ - 8)
va ~ ra %/2 / cos co/ - coL a 7m sin co/ - coMmax If sin (co/ - 8)
For phase (}, = rp ip
But Vp = f-p ip + M [iflf= £p Im sin co/ + M max If sm (co/ - 8)
l’p = rp • v2 7 sin co/ -t- co^p 7m cos co/ + co If cos (co/ - 8).
(c) S in g le -p h a sc s a l i e n t p o le m a c h in e . An elem entary form of a single-phase sa lie n t
pole synchronous m a ch in e is illu str a ted in Fig. 2.25 (a). Its variations of self-inductances
I„irand mutual in d u cta n ce M ,r, can be d eterm ined as outlined in th e previous articles (a) and
•i). The inductance v a ria tio n s are depicted in Fig. 2.25 (6). The various inductances can be
expressed in term s o f sp a ce a n g le 0r as follow s :
Rotor self-inductance, L r = constant.

Stator self-inductance, L , = ~ (Ld + L ,) + 1 (L d - Lq) cos 20, and m utual inductance,

M sr = Mmax C0S 0r-

Stotor
field a x i s

Stator

^ ( a ) Single-phase salient pole


Fig. 2.25. (b) Its inductance variations
with space angle 0r.
synchronous machine.

S ca n n e a b y Cam Scanner
200 Electrical Machinery

M a g n etic field e n e r g y sto red , o fl, i i 21 +i i M „


wH i „ 1i l l + 2 cos26rl + ‘ + C080'-
T h e in s ta n ta n e o u s m a g n e tic to rq u e is,
T - - 1 i 2 [i 2 (Ld - L q) s in 2 0 r] - ls lr M max Sin r
i e— 2 * 2 . -
= _ 1 ts2 (L^ - Lq) sin 20r - i , b M max s in r

H ere th e rotor w in d in g m u st carry d ir e c t c u r r e n t, i.e. ir - If-

A ssu m e is = 7m cos art and 0r = d),i - 5 a s b efo re.


Te = - i Zm2 co s2 0)7 (Z* - L q) s in (2o),i - 25) - I f m co s a* s in (0 ,7 - 5).

N ow c o s2 0)7 = |( 1 + cos 2o)7)


. Tg = _ i / m* _ L?) [sin (20),/ - 25) + s in (2o)^ - 26) c o s 2o)7]
' -I flm M m a x tk ™ W + ~ 6) + Sin (©^ - 0)7 - 8)]

= - 1 7 m 2 (Ld - L q) [sin (2(0,7 - 25) + f s in (2(0,7 + 2o)7 - 26)


+ \ sin (2o)r 7 - 2o)7 - 26)1 - l f Im M max • | [sin ((0,7 + (07 - 5) + s in ((0,7 - a* - 8))

For o)r * o), Te{mv) = 0


W hen o)r - t o , r e = - i 7m2 (Ld - L q) [ s in (2(07 - 2 5 ) + \ s in (4co7 - 2 5 ) + \ s in ( - 25)]

- If I m i [ s in (2o)7 - 6) + sin ( - 8)]

••• Te{av) = - \ I m2 (Ld - L q) [j s in ( - 26)1 - If l m M max \ [sin ( - 5)1

3 = 1 7 m2 (iLd - L q) sin 26 + 1 If I m

I f field cu rrent 7yris red u ced to zero, th e n

Te („„) = \ 7m2 (Ld - L q) s in 26


s in 6 ...(2.69)

...(2.70)

Eq. (2.70) g iv e s th e relu cta n c e to rq u e o f a s a lie n t - p o le s y n c h r o n o u s m a c h in e . The other


com p o n en t o f torq u e, i.e. \ I f I m M maxs m b is t h e e le c t r o m a g n e t ic o r in t e r a c t io n torque of
sa lie n t-p o le syn ch ro n o u s m a ch in e.

/. , , E x a ® p le 2 1 2 ‘ F o rth e d o u b ly ex cited m a g n e tic s y s te m s h o w n in F ig. 2 .2 1 , p lo t the nature


o f the in du ctan ce v a ria tio n s w ith d ifferen t ro to r a n g u la r p o s itio n s .

S o l u t i o n . T h e v a ria tio n o f m u tu a l in d u c ta n c e M sr w it h d if f e r e n t ro to r a n g u la r positions,


Z i th e ProcLe d u r o o u t lin e d in A r t. 2 .5 (o ). T h e in d u ctan ce varia­
tio n s w ith sp a c e a n g le 0r a re a s su m e d to be s in u s o id a l.

(i> W h en 0 , = O, a ir -g a p le n g th a n d th e r e fo r e r e lu c ta n c e i s m in im u m . C o n s e q u e n t l y the
( AT* \ ^ „
stator self-inductance L g j (
• R l /Iand rotor self-inductance L r — n rj SiG
v are m
maximum
a x im u m Since
magnetic axis of both the stator and rot n r r m i c • r V ) • J
maximum and let it be positive so that U = ! U = °' ‘He mUtUa‘ inducta"Ce "
sr max-

t o When 0, = 5 the air-gap length and therefore the reluctance is maximum. There**
L t and L r are minimum (but not np<rsK*ra\ on. « tb
stator and rotor axes are normal to each othe ^ mUtUal ‘nduCtance M « is zer0' beCaUS

Scanned by Cam Scanner


Electromechanical Energy Conversion Principles 201

_ a ir-g a p le n g t h is a g a in lnduc ,o n «
\ therefore/,, a n d L r a r e m a x i-
.tn al in d u cta n ce is a g a in m a x i-
U M sr = - M _ a s
r* tut »<
< .; ,A r t .2 .5 ( a )
v a r ia t io n s o iL Si L r a n d M sr w it h d if­
f e r a n g u la r p o s it io n s 0 „ a r e il-
^ j Trip 2 26. T h e e x p r e s s io n s for
rttrated in W - ■
and W , are

Ls = i'.O + X-S2 C0S 2®r


L, = Lra + L r2 cos 26r
(d M g r = M m a x COS 6r.
Here I * . LA L
A and a re in d ica te d in
Pig. 2.26.
Example 2 .1 3 . The s e lf a n d m u tu a l in ­ Fig. 2.26. Inductance variations of
stances of a doubly excited m agn etic sys- Fig. 2.21, Example 2.12.
timofFig. 2.21, are as follow s.
L s = 0.6 + 0.20 cos 2Qr ■H.
L r = 0.75 + 0.30 cos 2 Qr H.
and M sr = 0 . 8 cos 0r H.
The winding resistances are negligible.
fe) For stationary rotor a t a n a n gu lar position o f 9. = fin d the m agn itu de a n d direction
oftorque when the currents in the arrow direction s o fF ig • 2 . 2 1 are .
(i) is = 20 A d.c. a n d ir = 10 A d.c.
(H) is = 2 0 A d.c. a n d ir = - 1 0 A d.c.
OH) is = 2 0 A d.c. a n d ir = 0 A.

p a n [oj, if trie ---------- -


nitud* of 314 V r.m .s. a t 50 Hz. , M u
. W ftrtt. v a lu e s o f currents g iven in p a r t ta i). w rite the sta to r a n d rotor voltage eq u a tio n s.
SUme o rotor speed o f 2 0 r a d / sec for th is p a r t only. , n .
Solution. For 0r = 60° ; L „ L n M sr and th eir d eriv a tiv es w ith resp ect to 0r h a v e t h e follow -

L g = 0.6 + 0.2 cos 120° - 0 .5 0 H


L r = 0.75 + 0.3 cos 120° = 0.60 H
M
W*sr = 0.8 cos 60° = 0.40 H
A
dLg = _ 2 x 0.2 sin 2 x 60° = - 0 .3 4 6 4
dQr
dLr = - (0.3) (2) sin 120° = - 0 .5 2
dQr
dM sr
= - 0.8 sin 60° = - 0.693
dQr

Scanned by Cam Scanner


202 Electrical Machinery

(a) (i) From Eq. (2 .6 0 )


\ n dL. . . d M sr
1 . 9 d ljr
' ~ 2 1' W r + 2 h d 9 , * 1' ' d e r
S u b s titu tio n o f th e n u m erica l v a lu e s g iv e s
Te = \ (2 0 )2 ( - 0 .3 4 6 4 ) + \ (1 0 )2 ( - 0 .5 2 ) + (2 0 ) (1 0 ) ( - 0.693)
= - 2 3 3 .8 8 N -m .
S in c e torq u e is n e g a tiv e , it a cts in su ch a d ire ctio n a s to d e c r e a s e 0,., i.e. in counter-clockwi
d irection o fF ig . 2 .2 1 .
(it) T e = \ (20)2 ( - 0 .3 4 6 4 ) + \ ( - 10)2 ( - 0 .5 2 ) + (2 0 ) ( - 1 0 ) ( - 0 .6 9 3 )
= - 6 9 .2 8 - 26 + 138.6 = 4 3 .3 2 N -m .
S in ce torq u e is p o sitiv e, it acts in clo ck w ise d irectio n .
(iii) Te = 1 (2 0 )2 ( - 0 .3 4 6 4 ) = - 6 9 .2 8 N -m . (c o u n te r -c lo c k w ise )

(6) For rotor circu it, ur = rr ir +


dt
S in ce rotor w in d in g is sh o rt-circu ited , vr = 0
d\\fr
0 = 0+ ,
dt
But Vr —L r ir + M sr is

dt = *r + ^ sr = 0
or
d t (Lr ir) = ~ d t ^ sr ^
or
I>r ir - —M sr is
i.e. i -
r L r 1* ...(2.71)
It is g iv e n th a t is = V2 (20) sin wt. T h erefo re, from Eq. (2 .7 1 ),
M ’
ir = ~
i f V2{20) 8in <•* = - ^ (V 2)(20) sin a*

-- - ~g"v2
40 In
s in got
S u b stitu tio n o f th e v a lu e s in Eq. (2 .6 0 ) g iv e s

T' ~ \ ^2 0 ^2 sin M )2 ( - 0 .3 4 6 4 ) + - (-—4 0 s-•in 03t ( - 0.52)

( - 40^2)
+ (20V2) ( - 0 .6 9 3 ) sin 2 a*

I 1 3 8 .5 6 - 9 2 .4 5 + 3 6 9 .6 0 ) s in 2 to/ = 1 3 8 .5 9 1 - cos 2w ^

■ ^ ( a .) ==69.295 N -m (clo ck w ise).


(c) N o w th e v o lta g e im p ressed on th e s ta to r w in d in g
is
v• ~ (^2) 3 1 4 sin co/
But
..- r
dt dt

S c a rin e ffiv tla m S c a n n e r


Electromechanical Energy Conversion Principles 203

—L s is + M sr ir
5tltation o f tr f r o m E q . ( 2 . 7 1 ) , p v e S

MLsr
MI s =- ^Ls '' isc - £ vsc =
I io = L s'i*s
V
L‘
cajje(j th e s h o r t-c ir c u it in d u c t a n c e o f s ta to r w in d in g .
» 1SC ( M,sr
2\ r
(0-4)z
L - 0 .5 - = 0 .2 3 H.
0 .6
\
dys d T,
v = = -j - (i U 8) = 3 1 4 V2 sin tof
'5 _ d £ vs *
I s L . = _ 3 1 4 V 2 co3M ( _ _ 314V 2 cog 3 1 4 t = _ y ] 2 cos 3 1 4 £.
CO 314
= _L 414 c o s 3 1 4 t = _ 6 .1 5 c o s 3 1 4 f
s 0 .2 3

_ Msr . 0 .4 ( 6
M 1 5 ) cos 3 1 4 f = 4.1 cos 3 1 4 1 .
From Eq. (2.71), tr ~ Lr 0 .6 v ’

r e = - ( - 6 .1 5 cos 3 1 4 0 2 ( " 0 .3 4 6 4 ) + |( 4 . 1 cos 3 1 4 t f ( - 0 .5 2 )


For 0r = 60°.
2 + ( _ 6 .1 5 cos 3 1 4 1) (4.1 cos 3 1 4 t) ( - 0 .6 9 3 )

= - 6 .5 5 c o s2 3 1 4 1 - 4.3 7 co s2 3 1 4 t + 1 7 .4 8 cos2 3 1 4 t
„ _ . f l + cos 628*
= 6 .5 6 c o s 3 1 4 t = 6 .5 6 ^
< *
_ 6 5 6 _ 3 2 8 N -m (clo ck w ise)
1 e (av) 2

id) Stator and rotor voltage equations are


. dy8
Vs = r * ls + ' d T
d^r
Vr = r r l r + dt

Now \us = L s is + M s r ir _
= (0.6 + 0.20 cos 20.) (20) + (0.8 cos 0.)10
= 12 + 4 cos 20, + 8 cos 0r

y r I ^ 5 ^ 3 0 c o s 2 e r)1 0 + (0 .8 c o a 0r)2 0

= 7 .5 + 3 cos 20r + I® cos 0r

•• Stator v o lta g e e q u a tio n is

v _ o+ (1 2 + 4 cos 20r + 8 cos 0r)


' dd . d 0r
. . no i o ■ — - —8 sin Qr j i •
= —(4 s in 2 0 r) « at

But ^ = o) = 2 0 rad /sec


dt r sin 2 0 .- 1 6 0 sin ». = - 1 6 0 (.in 0. + sin 20.)

Scanned by Cam Scanner


204 Electrical Machinery

, ,. . — /7 5 + 3 co s 2 0 r + 1 6 c o s 0 r)
R otor v o lta g e e q u a tio n is v r = 0 + ^ ( '•»

= - (3 s in 2 0 r) 2 x 2 0 - (1 6 s in 0r) 2 0
= - 1 2 0 s in 2 0 r - 3 2 0 s in 0 r = - 4 0 (8 s in 0 r + 3 sin 20r).

E x a m p l e 2 .1 4 . A d y n a m o m e te r typ e m o v in g c o il Z r e T - ^ o tWo c°«*


as show n in Fig. 2.27. The s e lf a n d m u tu a l in d u c ta n c e s o f th e tw o c o ils a re L , = 0.20 mH,

L 2 = 0.15 m H a n d 0=0
M 12 = 0 .0 8 sin 0 m H . P o in te r

The fixed a n d m o vin g coils are co n n ected in se r ie s a n d


the cu rren t flo w in g th rou gh th em is l m sin cot. T h e c o n tr o l­
lin g torqu e p r o d u c e d by th e h e lic a l r e s tr a in in g s p r in g
varies in d irec t p ro p o rtio n to the deflectio n a n g le 0 a n d its
value is 0.0144 N -m a t 0 = 90°. 0= 90
(a) F in d an expression fo r torqu e in te rm s o f th e a n ­
g u la r p o sitio n 0.
(b) F in d the tim e average torqu e in te rm s ofQ.
(c) For 1 = 5, 10 a n d 15 am peres, sk etch th e v a r ia tio n
o f average torque w ith angle 0. © Mc ° o t 9

(d) Sketch a lso a curve sh o w in g the v a r ia tio n o f c u r ­ Fig. 2.27. Dynamometer type
rent w ith 0. moving coil instrument.

(e) I f the in stru m e n t is d e sig n ed to re a d a m a x im u m o f 15 A m p s , g iv e th e range of pointer


deflection.

S o lu tio n , (a) T h e m a g n e tic to rq u e is g iv e n b y 40""'5


18x10’ I =15 A
_ 1 •2 1 - 2 dL>2 . . d A f1?
r ' - 2 l1 + 2 l2 - d e + l 'l 2 - * r
H ere i 1 = i 2 = Im sin to/, b e c a u se th e tw o c o ils a r e
con n ected in s e r ie s.

••• T , = 0 + 0 + ( /„ Sin a t ? ~ (0 .0 8 s in 6) 1 0 ' 3

= 0 . 8 x l < r ‘ / m2 s in 2 (t f C o s e N - m .
(b) Te = 0.8 x 10~4 I m2 - cos 2 (0 /"
cos 0.
2
•• (ot/) = 0 4 x 10 4/m2cos 0 N-m.
58-6'
(c) For / = 5 A rms, Fig. 2.28. (a) V ariation of torque T1*(o<>i with 0-

T . m = (0.4 x l(T ■<) (V2 x 5)2 cos 6 = 2 x 10- 3 cos 0 N-m


For / = 10 A rms. T , = 8 x 10- 3 cos e N-m .

For 7 = I5 A ™ «. T , = 18 x 10" 3 COs 6 N-m.


The variation of T , w ith Aitr .
e(ait) witn different vrIhoq ■ »
s of 0 is shown in Fig. 2.28 (a).

Scanned by Cam Scanner


■yftC
Electromechanical Energy Conversion P rin c ip le s —

^ ,Lg gp rin g c o n s ta n t. T h e n for a d eflec- I


0 U * t - be 30A

/ » f9°”’ . ,9o«) = 0.0144 N-m.

*' 90 20A
cu rren t I a n d d e f le c t io n 0, t h e to rq u e
R a t i o n is given by
Spring restraining to rq u e = d e fle c tin g torq u e

00144 _ q^x 4 / m2cos 0

90 4
0 .0 1 4 4 x 1 0 4 0
/2=
90 x 0 .4 x 2 cos 0
90 e
e 0 Fig. 2.28. (b) Variation for c u rren t
COS 0
= 1.414
4 cos 0 with angle 0.

0° 5° 10° 15° 30° 45° 60° 70° 80° 90°


8indegrees
JinA 0 3.17 4.5 5.57 8.32 11.3 15.5 20.2 30.5 OC

/\SKeu;n s n u w u ig tut? v a i i a t i u n u i t u n c u i w i u i w 10 a a d u v tth


(e) For a current o f 15 A rm s, th e torque b alan ce eq u ation is

0 .4 x 10 4 (V2 x 15)2 cos 0 = ° 0


90 I
“ 112.5
l i i . 0 C cos
0 S 00 =
= 00 ,
By trial-and-error so lu tio n , th e v a lu e o f a n gle 0 is found to be 58.6°. T h u s th e r a n g ^ S ^ ^ ^
^trument
mment deflection is from zero degree to 558.6°. 8 .b .
Example 2.15. For the m agnetic relay o f Fig. 2.29, the sprin g exerts a p u ll o f 0 . 2 kg. W hen
exciting current is zero, the relay is in the oper\ p- f
tumlP Unaand aair-eo leneth
a length
*r‘8 a-D g 0isis0.5
0.5cm.
cm.With
W iththe coil
the co Armature
Arm?,Ure / 10“
^ r?onn’ ^ reduces to 0.1 cm. The coil
. 000 turns and the m agnetic leakage a n d fring * g_V— ' I
I
J fle e te d . The m agn etization curve for the relay T h “ OtlTl • . 3cmj
S e r ia l *» —’'—
Spring \
0.20 0.40 0.60 0.80 1.00 1.20

50 100 160 225 300 400 \ v */r


The
iit$ rnean length o f m agnetic iron p a th is 2 0 cm 3 ^
area ° f Cross-section throughout is 0.2 cm2. Fig. 2.29. Magnetic relay, Example 2.15.
the relay is in open position, calculate the m agnitude o f the ex citin g current re-
0 close it.
(h) w r
80l af sflould be the m agnitude o f exciting current to keep it in the clo sed p o sitio n ?
V bai 10n*(a) The electrom agn etic force fe across th e gap len g th can be o b ta in e d from th e
equation, i.e.
fe x 6 = (0.2 x 9.81) x 3

Scanned by (CamScanner
206 Electrical M achinery

_ (0 .2 x 9 .8 1 ) x 3 _ q g g i n e w to n s . Un ,
or fe - 6
I f B , is th e a ir -g a p flu x d e n s it y , t h e n

1 B *A
f = ± ~ R— = 0 .9 8 1
U 2 p0
2 _ 2 x 0 .9 8 1 x 4 k x 10 7 _
= 0 .1 2 3 3
e ~ 0 .2 x l O " 4
or B g = m 2 3 3 = 0 .3 5 1 T
F ro m B -H c u r v e o f F ig . 2 .3 0 , A T /m . fo r a flu x
d e n s i t y o f 0 .3 5 1 T a r e 8 7 .7 . T h e r e f o r e t o t a l 0 50 100 15 0 200~ 257p5rvT -
a m p e r e -tu r n s fo r ir o n le n g t h ATs/metrc 3Sl«
ill = 0 . 2 0 x 8 7 . 7 = 1 7 .5 4
Fig. 2.30. B-H curve, Example 2
.15

• )f * A m p e r e -tu r n s fo r a ir -g a p = ^ g = (P -351) (0 .5 x 10— ^


Ho 471 x 10-7
.*. T o ta l A T s fo r t h e m a g n e t ic p a th fo r a n a ir -g a p le n g t h o f 0.5 cm
= 1 3 9 6 .5 8 + 1 7 .5 4 = 1 4 1 4 .1 2
l l
E x c it in g c u r r e n t r e q u ir e d to c lo s e t h e a r m a tu r e r e la y is
*.iii*
II
II 1 4 1 4 .1 2
= 0 .7 0 7 1 A.
Ii 2000
Ii (6 ) A ir -g a p a m p e r e -tu r n s , f o r g = 0 .1 cm , a r e

| .• _ I 0 ;3 5 1 ) (0 .1 x 10" 2) _ 2 7 9 .3 2
H 4 7t x 10~7
iI T o ta l A T s fo r t h e m a g n e tic p a th , for g = 0 .1 cm , a re
i = 2 7 9 .3 2 + 1 7 .5 4 = 2 9 6 .8 6 .
E x c it in g c u r r e n t r e q u ir e d to k e e p t h e a r m a tu r e c lo sed
2 9 6 .8 6
= 0 .1 4 8 4 A.
iI 2000
ill E x a m p le 2.16. (a) The a ir -g a p u n d e r th e p o le s o f a d.c. m achin e is not uniform, due to wear
o f th e b e a rin g s. F in d a n ex pressio n fo r the u n b a la n c e d m a g n e tic p u ll on
Ill | th e a r m a tu r e . F or th is ex a m p le, a n e le m e n ta ry tw o -p o le d.c. m ach in e o f
Fig. 2 .3 1 m a y be c o n sid ered .
(b) A 2 -p o le d.c. m a ch in e h a s its a rm a tu r e d is p la c e d by an a m o u n t
o f 0 . 1 m m . O th e r d a ta are a s follo w s.
A ir-g a p flu x d e n s ity : 0 .8 T
A rm a tu re
JP A re a u n d e r each p o le : 2 0 0 cm 2
I i I
Iii U n iform a ir -g a p le n g th : 0 . 6 cm .
I
r: F in d th e m a g n itu d e o f th e u n b a la n c e d m a g n e tic p u ll b n the a rm a - <S
|l | ture.

Solu tion , (a) L e t g be th e uniform air-gap len g th o f th e d.c. m ach in e. 1


blanch
D u e to w ear, le t th e tw o air gap s becom e (g + x) an d (g - x), as shown un Example 2
|I i in Fig. 2.31. ..

rr •
wy u en i u o u i ii i o i
Electromechanical Energy Conversion Principles 207
2.51
aD
jsjorth an(^ so u th p o le s c r e a te d n e a r ea ch air-gap , ex er t a force o f a ttr a c tio n or p u ll on th e
^attire- For u n iform a ir -g a p s u n d e r th e tw o p o les, ea ch p ole p u lls th e a r m a tu r e w ith a force

n ew to n s, th e r e fo r e th e b a la n ced m a g n etic force or p u ll on th e a r m a tu r e is zero. H e re


0 2 l*o
Bg = a ir -g a p flu x d e n s ity u n d er each pole
^ A.p = a r e a u n d e r ea ch pole.
When the tw o air g a p s b eco m e u n e q u a l, th e u n b a la n ced m a g n etic p u ll co m es in to p la y an d
magnitude can be c a lc u la te d a s fo llo w s :
At the large air-g a p o f le n g th (g + x), th e d ecrea sed force o f pull

=i ? l A
2 Mo "g + x
At the sm aller a ir-g a p o f le n g th {g - x), th e in crea sed force o f pull
1 Be 2 a
~2 Mo A p g - x
:. Net u n b alan ced m a g n e tic p u ll on th e arm a tu re o f a 2-pole d.c. m a ch in e is
i

a ri i
2 Mo pS g - x g+x

n ew to n s
^0 V - x 2
For uniform air-g a p , x is zero and th e u n b a la n ced m agn etic pull is also zero.
ib) Substitution o f th e n u m erica l v a lu e s in Eq. (2.72), g iv es th e m agn itu d e o f u n b a la n ced
magnetic pull as

= <°-8)2 . (200 x 1 0 - 4) Q? X ° n T = 169.76 n ew ton s.


471 x 10~7 36 - 0 01
Exam ple 2 .1 7 . Fig. 2 .3 2 sh ow s a slo t contain ing z s conductors a n d each conductor carries
Qcurrent o f i am peres. The iron p a r t o f the m agnetic circuit is Iron
on\j j“-u7s —»j
burned to have in fin ite p e rm e a b ility.
(o) Find the va ria tio n o f the flu x d e n sity in the p a r t o f the slot
Vobooe the conductor po rtio n a n d (ii) occupied by the conductor.
tor ^ an expression fo r the leakage flu x above the conduc-
rPortion.
Slot
^ b) Find the e le c tro m a g n e tic force fe, c re a te d b y th e s lo t leakage
°£e flux presen t above the conductor portion. flux
For i = V2 (1000) sin 314t, fin d the direction a n d m a g ■ V Current
V * °f maSnetic force fou n d in p a r t (c). There are tw o conduc- out of paper
ln tfu: slot, em b ed d ed len gth is 1 m etre a n d slo t w id th is 5 Fig. 2.32. Slot leakage flux,
*• ° Example 2.17.

*PeS°iUti011, S in ce relu cta n c e o f th e iron p a th is a ssu m ed to be n eg lig ib le, a ll th e m .m.f. is


n ed in th e slo t w id th w s only.
(l) If is th e m a g n etic field in te n s ity above conductor portion in A Ts per u n it le n g th , th en

H s w s = izs

L
Scanned by Cam Scanner
208 Electrical M achinery _______ -_______

*-2,
or t f *g = — T
ws — Bs — h2
„ Ho iz s
F lu x d e n s ity B s - p 0 ^ _________ 4 .

S in ce th e cu rre n t en clo sed is c o n s ta n t, t h e flu x $ - 4


B
as sh o w n in F ig . 2 .3 3 .
(ii) F or th e con d u ctor, le t Hy b e t h e m a g n e tic fie ld Force
Fig. 2.33. Flux density variation over
in te n s ity a t a h e ig h t y from th e b ottom o f th e c o n d u c ­
th e slot depth.
tor, th en

w = iz.T -

or w y~
y ws h
M-o iz , y_
F lu x d e n sity By = \i 0 H y =
w. hi
T h e n a tu r e o f th e flu x d e n s it y v a r ia t io n o v e r t h e s l o t d e p t h (h x + h 2) is illu strated in
F ig . 2 .3 3 .
(b) S lo t le a k a g e flu x <j>g above th e co n d u cto r re g io n is

) w8
H ere I is th e em b ed d ed le n g th o f co n d u cto r n o r m a l to t h e p la in o f p a p e r . E xpression for
rev e a ls th a t th e s lo t le a k a g e flu x is d ir e c tly p r o p o r tio n a l to t h e s lo t d e p th , s lo t len g th and in­
v e r se ly p rop ortion al to slo t w id th .
(c) C on sid er a s lo t d ep th o f h e ig h t x, j u s t a b o v e h x a s s h o w n in F ig . 2 .3 3 . T h e n th e magnetic
field en erg y stored by th e slo t le a k a g e flu x 0, for t h e s lo t d e p t h o f h e ig h t x is g iv e n by

1 B 2
rfid •"

S u b s titu tio n o f th e v a lu e o f B s g iv e s

w fid /.*
(1, x) _- n
i *2 2 *2
. x i.
2

E le ctro m a g n etic force f = ~— M _ 1 Ho i z , .


'• 31

W) The direction of thpfnrCref^e<l*>y slot leakage flux present above the conductor port*011

m encal values in the expression for fet w e get


f _ 1 4 n x 1Q-7 ( 2 \> x 1 ,
* 2 5 vx i10
rr2 ^ x 1000)2 sin 2 cof

= 32n ’

f __ 3 2 k v
«(<*«) 2 - 50.265 newtons.

A
Scannea by Cam Scanner
Electromechanical Energy Conversion Principles 209
lA
pie 2.18. (a) F o ra tw o-pole d.c. m achin e, a conductor on
* t u r e carries a cu rren t o f i a m peres, a s show n in Fig . 2.34.
egression for the electro m a g n etic force fc on the conduc-
r j*ii*aneXy

* rjse the result o b ta in e d in p a r t (a), to d eterm in e the force


t w parallel conductors, c a rry in g cu rren ts i x a n d i 2 in the
o

Medirecti°n'
Solution, (a) W hen th e co n d u cto r cu rren t is zero, th e air-gap
distribution due to th e n o rth p ole, is uniform over th e pole
5th Fig- 2 35 ^ L et th e u n ifo rm flu x d en s ity for th is stator
’ 3 . If the conductor ca rr ies cu rren t i,' w ith th e field w in d in go
------
p f m h n f i n n 1 n
excited, then th e flut • ixr rdl iistr
« i1 _
ib u tio n is as n nVv «
sh o w n in F ig. O OCT / L \
2.35 (6). Fig. 2.34. Pertaining to
Example 2.18.
tetthe flux density, d ue to flu x crea ted by cu rren t i, be B x so th a t

Here the reluctance is a ssu m e d to be offered by th e air-gap alone. N ote th a t th e conductor


fluxhas to traverse th e air-g a p tw ice a s sh ow n .
When both th e field w in d in g and conductor carry current, th en th e resu lta n t flux d istrib u ­
by su p erim p o sin g th e flu x d istrib u tio n o f F igs. 2.35 (a) and ( b). T he flux d en sity
tion is obtained

IB-B,) V M B +B ,)

(c)
(a)
Fig. 2.35. Illustrating the flux distribution, Example 2.18.

^ right of conductor is se e n to be B + B x and to th e left o f it, B - B x. I f th e conductor m oves


!.8niall displacement x to th e rig h t, th en th e ch an ge in m agn etic stored energy in th e air-gap is
““ by

\ ( B +B{f 1 (B - B x) 2
glx
2 1*0 1*0 1
'hen
is the air-gap le n g th and I is th e axial len g th o f conductor.
fc
p!x n , 2B B xglx
W »J— 5 J -

Subs
3stitution of the value o f B x gives

„ (i x ) s m * M = B i x i
w fid {l>x) p0 2g

rce °n th e cu rrent carryin g conductor is

Bi ...(2 .73)
dx

Scanned by Cam Scanner


4 *'

210 E le c tric a l M a c h i n e r y ------------- X


t
Ks
conductors can n in g current „ and , in lhe

^'^Th^m agnetkfield^rdensity W, at the conductor 2, created by current .„ i s


’ sedJ “ *i\
H x • 2nd = to ta l cu rren t en clo 2 --------- j-------J-------

H - - i-
••• " 2nd
/. F lu x d e n s ity B j a t th e con d u ctor 2 is
/! -
-tiQJr \} - ' 1
V '
TT 1^0 *1 (T<
B , = H o ^ t = 2 7id V V
... Force o f attraction betw een th e tw o conductors Fig. 2.36. Illu stratin g force between two current
carrying conductors.
n ew to n s.
= B \l l 2 = 2 nd
E x a m p le 2 .1 9 . S e l f - a n d m u lu a ,.in d u c ta n c e s in co u p

L, = 3 + L 2 = 2 + j j . M /2 = M n = 2x
ooer a certain d isp la cem en t x in m etres. The co il resista n
For con stan t currents o f I j = 10 A a n d 12 = - 5 A , co m p u te fori

(a) the m ech an ical w ork done in in crea sin g x from 0 .5 to 1 m,


(b) the energy su p p lie d by each electrica l source in p a r t (a),
( c ) c h in fie ld energy in p a r t (a).
a n g e u nnd field enern
Hence verify th a t sum o f the energies a sso c ia te d w ith m e ch a n ica l w ork ^ a n d / i e ^ V
is eq u a l to the energy su p p lie d by both sources d u r in g the m o tio n fro m x
S o l u t i o n . For a lin e a r ca se,

I
‘ 2
Wfld («»- '2 .* ) = \ l x i , 2 + \ L 2 t22 + i , *2 M
25
1 = 175 +
2 + - # 2 5 + (-5 0 ) 4x
3+i 100 + 2x

, ^Wfu (ij, t2, x ) 25


(a) From Eq. (2 .2 6 ),
^ 4?

w 10 r w - f 10 2 5 - 2 5 = _ — w a tt-sec
” nuch Jo.5 ^ J 0.5 ^2 4 4
0.5

(6) = ijd v 1 +
1 \ 5
H ere y i - L j i l + M i 2 - 13 + ^ 10 + ( - 5)
£ H 0+s
Vj/2 —L 2 *2 "*■ = ( - 5 ) t ^ (1 0 )= -1 0 +

pV2(x= 1.0)
E n e r g y s u p p lie d b y so u rce 1 , ^ i2 d Vl

[ forx 5 ^ _ f 3 0 + #5- ^
= 10 30 + # -
2x 2x
LV J x = 1.0 V J x=0.5-J

b c a n n e ffiy ^ a m 5 c a n n e r
Electromechanical Energy Conversion Principles 211

s u p p lie d b y S0UrCe 2 is
gjjoji*11" r V2 (x = 1°)
W ,fcC2= J V i(I = 0S)

( 5N
= (-5 ) - = 12.5 w a tt-se c
r 10+l
- \ ■1 0 + ^ *= i \ / i = 0.5 -

Vet electrical en erg y m p u t,


WeUc = w efec, + Wei«2 = - 25 + 12.5 = - 12.5 w atC sec
25
f c )V * i * ^ i ) = 1 7 5 + 4 x
Change in field en erg y sto red from x = 0 . 5 t o x = l m
= ( w /w a t x = 1 0 ) “ (w fid a t x = 0 -5 )
/ \ ✓ \ __
25
= 175 + m . f = - — w a tt-sec
4

Here = - 1 2 . 5 = W.elec

This verifies th e r e q u ir e d r e s u lt . ,
Example 2 .2 0 . F or f/ie t o o coifc o f E xam ple 2.19, compute the value o f average m agnetic
jm atx = 0.5 m i f
(a) both coils are connected in p a ra lle l across a voltage o f 220 cos 314 t volts,
(b) both coils are connected in series across a voltage o f 2 20 cos 314 t volts
(c) coil 2 is shorted a n d coil 1 is connected to the sam e voltage as m p a r t (a).
(d.) both coils in series carry a current o f 0.5 cos 314 t A.
Solution. For a lin e a r ca se,
Wfld (iit h, X) = \ L \ h 2+ i L 2 i? + i x i2 M
1 ■2 1 'jt
+ 2 2x
2

dW/irf (*i> *2>x )

4xl * 4a? ~ ix iib


For x ~ 0.5 m,
( lV 1 •
(a) = L i i x + M i2 — s+S W +£*
1 ■ L i2
y 2 = M i\ + L 2 i 2 = 2^ + 1 + 2x

At* a 0.5 m , = 4»x + i 2 and V2 —

. d\\f\
dV i dy 2
the coils are in p a ra llel, v = ^
dt ■;i
d ix d i2 d ix d i2

3 .
= or is = 5 >,
dt 1 dt ‘ 2

Scanned by Cam Scanner


212 Electrical Machinery
u
di\ din 11 dix
220cos314t = 4 ^ +^ = T ^

d ix _ ... . 40
or — = 4 0 co s 3 1 4 t or ix= ^ s in 3 1 4 1

• 60 •
l2 = 3 1 4 Sm
S u b s titu tin g th e v a lu e o f i lt i2, in t h e e x p r e s s io n for fe g iv e s

-1 2 .„ * .2 —
f< = [(40)2 + (6 0 )2 + 2 (6 0 ) (40)] s in 2 314
(3 1 4 )2
= - 0 .1 0 1 4 2 4 s in 2 3 1 4 t
fe(av)
iv) = - 0-------
.0 5 0--------
712 N

(6) v= + +
dt dt dt dt ' dt dt
\ J \
For s e r ie s co n n ected co ils, i x = i 2 = i

2 2 0 cos 3 1 4 = 9 ^ or i= s in 3 1 4 t
dt 9x314
/ 220 N
fe = ~ 4 s in 2 3 1 4 t
9x314
or £<«,) = - 0 .0 1 2 1 2 N
(c) W ith coil 2 sh o rted and coil 1 co n n ected to v o lta g e so u rce,
„ dis du
0 = 4 - # + —#
dt dt
) 0 -

dt
4. r> d i 2
u ~ ^ 7 7 + o - 7-
dt
or
d ix
dt
=- 3
d i2
—-
dt
220 cos 3 1 4 t = v = - . f [*2 _ U d i2
dtdt ~ dt
or • 20 .
l2 = ~ sin 3 1 4 t

and ; 60 • o
1 = 3 i 4 sin 3 1 4 I

fe = ( 3 1 4 ? 1(60)2 + (20)2 ~ 2 ( 2 0) (60)] s in 2 3 1 4 1

( 40
314 s in 2 3 1 4 t
or
•fe(ao)
r tat/j = - 0 0^0v8u1n1 N .,

in betw een them . S*p a r a te d ^ d is ta n c e g h a ve a flu x density of 1.6 ■


force betw een these tw o su rfaces for area A - 7 ^ 2 ° ” fo r fe rro m a g n e tic m aterials. Findth
S o l u t i o n , L et A be th e a rea o f th •
For a d is p la c e m e n t* in th e d irectio n j . 8U rfaces w h ic h a r e s e p a r a te d by a distanceI
(g - x ) an d t h . field e n e r g y sto red “ "by betW een t h e tW 0 SurfaCeS *

Scanned by Cam Scanner


Electromechanical Energy Conversion Principles 213

d w fid *) 1 B 2A
prom Eq- (2 .2 1 ) f< = -
dx 2 Ho
*2 ,
= /9 ^A W V/
^ 1 A4” I = — X 107 N
2 4 jc x *10

26. Singly-e x c ite d E le c tr ic F ie ld S ystem s


The electric field a s a co u p lin g m ed iu m b etw een electrica l and m ech a n ica l s y s te m s can be
treated in a m a n n er s im ila r to m a g n etic field d iscu sse d earlier. H ere a lso th e term s en erg y an d
coenergy w ill be in tro d u ced to ob ta in th e ex p ressio n s for force or torque. C h a rg e and v o lta g e
associated w ith electric field are a n a lo g o u s, resp ectiv ely , to flu x lin k a g es and cu rren t in a m a g ­
netic field.
(a) E le c t r ic f i e l d e n e r g y s t o r e d . A sin g ly -ex cited en ergy conversion d evice u sin g electric
field as th e co u p lin g m ed iu m is rep resen ted in Fig.
2 .3 7 by a p a r a lle l p la te c o n d e n se r w ith fix ed and
movable p la t e s . A c u r r e n t s o u r c e fe e d s th e co n ­
denser. L eak age cu rren t o f th e co n d en ser is repre­
sented by con d u ctan ce G. T h is m e a n s th a t electric
field betw een th e tw o p la te s is co n serv a tiv e, i.e. all
energy supplied is reco v era b le from th e device. The
energy flo w in a n e l e c t r i c f ie ld s y s t e m is a ls o
governed by Eq. (2 .3 ) a s in a m a g n etic field sy stem .
When cu rren t so u rce is con n ected , cu rren t and
charge flow in to th e d ev ice o f Fig. 2 .3 7 and th e dif­
Fig. 2.37. Singly-excited electric-field energy
ferential electrical en erg y in p u t to th e con d en ser in
conversion device.
time dt is
dW eUc = v id t = vdq ...(2 .7 4 )

The m ech an ical w ork don e in tim e d t by th e force fe is


d W mech= f e dx ...(2 .75)

where dx is th e d isp la c e m e n t o f m ovab le p la te in th e direction o f fe du rin g tim e dt.

Eq. (2.3) in te r m s o f E qs. (2 .7 4 ) and (2.75) is


vd q = d W fld+ fe dx ...(2 .76)

If the m ovab le p la te is n o t a llow ed to m ove, th en fe dx = 0 and Eq. (2 .7 6 ) b ecom es

dWfitc = dWfw = vdq ...(2.77)

Note th a t in Eq. (2 .7 7 ), v is co n sta n t and charge b u ild s up from zero to q.


’Q
fid = fo u(k>

por a lin ear electric field , C = J = cap a cita n ce o f th e device.

lf» . lg £
...(2 .7 8 a )
^ /w = c J o ^ = 2 C

AIbq ...(2 .7 8 b)

Scanned by Cam Scanner


214 Electrical Machinery
-i* ju
C a p a cita n ce C d ep en d s on th e d is ta n c e b e tw e e n t h e tw o p la te s a n d can be w ritten,
p nA
C=
(g -x )
w h ere A = p la te a rea and e<, = p e r m ittiv ity o f free sp a ce.
T h u s, th e field en erg y W * is a fu n ctio n o f tw o in d e p e n d e n t v a r ia b le s , and x, i.e.

1 q2
W/m ( 9 , * ) = 2 W —(2-79)
Eq. (2 .7 9 ) rev e a ls th a t electric field en erg y can b e c h a n g e d e le ctric a lly by changing ?0r
m ech a n ica lly by c h a n g in g x (i.e. by d rift o f th e m o v a b le p la te). ie f
T h e en erg y d e n sity in electric field is g iv en by
cD 1 D2
wfld = )o E d D ~ 2 eo ,"(2-801

w h ere D = electric field flu x d en sity , C o u lo m b s/m this


E = electric field in te n sity or p o ten tia l g ra d ie n t, v o lt s /m
D
^ '
anc
(6) C o e n e r g y . For a lin e a r ca se, th e v a ria tio n o f q w ith v is a s sh o w n in Fig. 2.38. As in
i a
m a g n etic sy ste m , coenergy Wfjd is th e a rea O ACO . »
C om pare th is figu re w ith Fig. 2.4 (c). to 1
(2.f
f u‘ f ”1
A rea OACO = J0 q ■d v = C JQ vdv ant

In gen eral, Wpd' (u, x) = ^ C(x) v 2


ere
C oenergy d en sity , Wpd = ^ eaE 2 ...(2 .8 2 )

(c) C alculation of force. T h e force produced by 0“ “ Pj ^ ^ be


electric field can be ob tain ed by th e u s e o f field en erg y i r —1 r clo
or coen ergy fu n ction s as in th e ca se o f m a g n etic sy s- F iS' 2 -38- Electric field energy and coenerg) in 0J>
terns. T h e field en erg y stored is d ep en d en t on q, x and a linear circult dii
as su ch , Eq. (2 .7 6 ) can be ex p ressed as caj
dW fu (q , x) = vdq - fe dx •••(2-83) vii
T he field en erg y Wrfld (q , x) can also be ex p ressed m a th e m a tic a lly in term s of its two partial ca]
d eriv a tiv es as

...(2.84)
dW nd (q, x) = dWM d q + ™ l i dx
" dq * dx
T h e field en erg y in E qs. (2 .8 3 ) and (2 .8 4 ) is sa m e, th erefo re th e co efficien ts associated
d q and dx m u st be eq u al in both th e s e eq u a tio n s. of

v _ <>w fid (q , x) (2.85)


Tl
(2.661 Hi
f - d w fld (q ,x )
and
dx

Scanned by Cam Scanner


Electromechanical Energy Conversion Principles 215

■ «een from F ig. 2 .3 8 th a t co en erg y is g iven by


'S w M = q i v , - W /u .
jn general, (u ,x ) = gy - (q ,x )
In differential form , d W ^ 'iv , x) = d(qv) - dW M (q, x) ...(2.87)
Substitution o f Eq. (2 .8 3 ) in Eq. (2 .8 7 ) g iv es
dW fu'iv, x) = q d v + vdq - vdq + fjdx
= q d v + f edx ...(2.88)

As Wfu Is d ep en d en t on u ,x ; it can be ex p ressed m a th em a tica lly in term s o f its p artial


derivatives as

dW lu \ v . x ) = ^ - d v * ^ - d x ...(2 .8 9 )

The coefficients o f d o , dx on th e rig h t h an d sid e o f Eqs. (2.88) and (2.89) m u st be eq u al and


{jus yields th e ch arge an d force e x p ressio n s as
dWfuXv, x)
9 “ dv
3 ^ /M
and •* dbc

Eqs. (2.86) and (2 .9 1 ) g iv e th e force ex p ressio n s for electric field sy stem ; correspondingly,
to Eqs. (2 21) an d (2 .2 3 ) for th e force in m a g n etic field sy stem . N o te th a t field en ergy in Eq. 11 • i
(2 86) m ust be ex p lic itly e x p ressed in term s o f q an d x and coenergy in Eq. (2.91) in term s o f v
and x.
Substituting v a lu e o f W ^ ' (u, x) from Eq. (2.81) in Eq. (2.91), w e g et
r _ * 2 d C (x) ...(2.92)
'* _ 2 dx
The above e x p r e ssio n for force s ta te s th a t electric force acts in su ch a direction as to in ­
crease the ca p a cita n ce o f th e co n fig u ra tio n . ^
The exp ression for e lectric force, a s g iv en by Eq. (2.86), can also
be obtained by r e fe rrin g to F ig 2 .3 9 . In th is figu re w h en sw itch is
dosed, capacitor g e ts ch a rg ed to so u rce v o lta g e V,. S w itch S is th en y s
opened. N ow th e u p p e r p la te is g iv e n a v ir tu a l d isp la cem e n t dx in th e
d irectio n o f e l e c t r i c f o r c e . D u r i n g t h i s d i s p l a c e m e n t , c h a r g e o n

capacitor,, = C V . r e m a in s co n sta n t. A s t h e s w i t c h S .s o p e n ^ d u n n g r*
Witual displacement electrical energy input, d W elec = 0 and the mecham
<*1work done, d W ^ = f e dx.
From Eq. (2.3), 0 = fe dx + d at constant q.
dWfu (<q, x) ...(2.86)
9x
In Eq. (2.86), field energy stored in the electric field must be explicitly expressed in terms
0f9an d x.
Example 2.22. T w o parallel plates, each o f area A = 1 mJ, ore separated by a distance g.
* dectric field intensity between the plates is 3 x 10s V/m. a value equal to ^ breakdown
•""Wh of air Find force between the two plates. Use both energy and coenergy method,.

Scanned by Cam Scanner


216 Electrical M achinery
" " n ln te s a p la t e m o v e m e n t x in th e directs
S o lu tio n . F or a d is ta n c e g b e tw e e n th e • e le c t r ic fie ld e n e r g y stored i8 n 0f
force, red u ces th e p la te d is ta n c e to (g - *)• F or t h is s p a ,
1 q2 l <7Z( g - * )
W /M ( ? . * ) S 2 C 2 A zo
dWftrf _ 1 -S —
From Eq. (2 .8 6 ), fe = “ d* 2 AEo

q = DA —ZqEA,
For

1 2 _ 1 2 ^*-0
T h e co en erg y is g iv e n by Wfld(v, x) = - C v = ^ v ^ _ x

a y ( M ) _i 2 AZr
From Eq. (2.91), /f" ax 2 (g -x Y

For y = E (g - x ), £ = \ E 2 ZqA.
- 9
10
fe = | ( 3 x l 0 6)2 X 1 = ~r~ X 10 N .
36n 8n
B oth th e m eth o d s g iv e th e sa m e force. It is a s e x p e c te d . I t m a y b e in t e r e s t in g a t this stage
to com pare th e force produced by m a g n e tic field w h e n B is n e a r t h e s a t u r a t io n level with the
force produced by electric field w h en E is eq u a l to its b r e a k d o w n v a lu e .
ft per u n it area in m a g n e tic field (E x a m p le 2 .2 1 )
ft per u n it area in e le c tr ic field (E x a m p le 2 .2 2 )
0 .3 2 x 1Q/ x 8 n
= 25600.

) n x 103
T his sh o w s th a t force d e n s ity in th e m a g n e tic fie ld is 2 5 ,6 0 0 t im e s it s v a lu e in the electric
field at th e m axim u m p o ssib le field s tr e n g th s a s s u m e d .

k-^r'*ijn *S Lhe rcason p ra ctic a l e n e r g y c o n v e r s io n d e v ic e s m a k e u s e o f the mag­


n etic field rath er th a n th e e lectric field a s th e c o u p lin g m e d iu m . E le c tr ic fie ld d ev ices operating
at very Iow -pow er le v e ls a re s o m e tim e s u se d fo r r e l a t i v e l y h ig h f r e q u e n c y a p p lica tion s.

V °' " " T ™ U age “ aPPlied <• fix e d p la te a n d a mowbl>

h avin g cross-sectional area of 3 0 cm2. Distance be­ 1 0 cm


8cm
tween the two plates is 1 cm. r
t
S o l u t i o n . C o -en erg y sto red b e tw e e n t h e tw o
p la tes, W „ / = | CV 2 H ere C = ^ -
Z X

w here x = d ista n c e b e tw e e n th e tw o p la te s .

'V ( v ,* ) = I v 2 ^
2 x
Force b etw een th e tw o p la te s ,
f _ dW /z/ (V, x ) j
t / 2 £ 0 A
e dx --------
2 ~~T N
x
Fig- 2.40. P e rta in in g to Example 2.23

Scanned by Cam Scanner


Electromechanical Energy Conversion Principles 217

N egative s ig n in d ic a t e s t h a t e le c tr ic fo rce f e a c ts in su c h a d ir e c tio n a s to r e d u c e t h e d is ­


tance b etw een t h e tw o p la t e s .
For th e b ar to r e m a in h o r iz o n ta l in F ig . 2 .4 0 , e le c tr o s ta tic force
f e x 8 = 5 x 10
f 50 2 5“ 25 1
or x 9 .8 1 N
/e~ T = T g m s = T x 1000

But th e m a g n itu d e o f fo rce b e tw e e n t h e tv/o p la te s is f =


2 v

f i o ~ 9 x 3 0 x 10 - 4
25 9 .8 1
4 X 1000
-iv“ 36 k (1 x 10“ 2)2
1/2
2 5 x 2 x 3 6 k x 104 x 9 .8 1
or V= = 2 1 5 0 0 .8 2 2 V
4 0 0 0 x 10" 9 x 3 0 x 10" 4
~ 2 1 .5 0 k V
Thus th e v o lta g e a p p lie d b e tw e e n th e tw o p la te s is 2 5 .1 kV.
The b asic p r in c ip le d e m o n s tr a te d in t h is e x a m p le m a y b e u s e d for th e m e a s u r e m e n t o f h ig h
voltages q u ite a c c u r a te ly .
2.7. D ynam ic E q u a tio n s
The e le c tr o m e c h a n ic a l-e n e r g y -c o n v e r sio n d e v ic e s o p era te w ith e lectric a l s y s te m on o n e
side and m e c h a n ic a l s y s t e m on th e o th e r sid e. It is th e b eh a v io u r o f th e e n tir e e le c tr o m e c h a n i­
cal system th a t is im p o r ta n t, b e c a u s e a p a rticu la r s y ste m m a y o p era te s a tis fa c to r ily u n d e r
steady s ta te c o n d itio n s b u t m a y fa il m is e r a b ly u n d er th e electro m ech a n ic a l tr a n s ie n ts . In v ie w
of this, th e o p e r a tio n o f th e e n tir e s y s te m , co m p risin g o f electric a l s y ste m , co n v ersio n d e v ic e
and m ech an ical s y s t e m , sh o u ld b e in v e s tig a te d in d e ta il d u rin g th e e lectro m ech a n ic a l s y s te m
planning. T h u s th e c o m p le te a n a ly s is o f a p h y sic a l s y ste m in v o lv e s electric circu it e q u a tio n s ,
!
energy con version r e la tio n s a n d e q u a tio n s o f m o tio n s.
Sim ple m o d e ls o f s in g ly e x c ite d ele c tr o -m e c h a n ic a l s y s te m s are illu s tr a te d in F ig s. 2 .4 1 (a)
and (6). M odel o f F ig . 2 .4 1 (a) is a p p lic a b le o n ly to restricted lin e a r m o v em en ts, b e c a u se o f th e

is .
iH i

■3£>

(a) Model o f a sin g ly excited translation al electrom echanical system , involving restricted linear m ovem ent.
1
+o--------'WW'-:-----r \ le (Or
/*t

(6) Model of a singly excited rotational electromechanical system, involving restricted rotary motion.
Fig. 2.41

canned by Cam Scanner


^can
21 8 Electrical M a c h in tn

p r e s e n c e o f a s p r in g . S im ila r ly t h e m o d e l illu s t r a t e d in P ig . 2 .4 1 (6 ) is for th e restricted rot


m o tio n s o n ly lv ,. Inf s p rnin K is r e p la
ng n ia c e da b
o y a m e c iia
h a iu
n icc u
a li lo a d to r q u e .in. . .F ig . 2.4 1 (6), it become* o?
u oecomestk'
m o d el o f a r o ta tin g e le c tr ic a l m a c h in e w ith en erg y ’ c o n v e r s io n from e le c tr ic a l to mechanical *
T h e v o lta g e e q u a tio n for th e e le c tr ic a l s y s t e m o f b o th P ig s . 2 .4 1 (a ) a n d ( b ) is,

. di . (d L \
dft,
-tr + L j + i
dO, d t

. . di . (d L \
= ir + L -r- + i ••(293)
dt dOr

T h e te r m L in Eq. (2 9 3 ) is k n o w n a s th e tr a n s fo r m e r v o lta g e , b e c a u s e it involves the

tim e d e r iv a tiv e o f c u r r e n t T h e th ird term o f E q . ( 2 .9 3 ) is th e s p e e d or ro ta tio n a l voltage term


b e c a u s e o f th e p r e se n c e o f sp eed w, in it N o te th a t it is th e s p e e d v o lta g e term th a t determines
th e (low o f e n e r g y b e tw e e n ele c tr ic a l and m e c h a n ic a l s y s t e m s .
In P*ig 2 41 (a ), th e m a g n e tic force f, is o p p o se d b y in e r tia force f u , d n m p in g force fDand
r e s tr a in in g s p n n g force fK T h erefo r e th e force b a la n c e e q u a tio n is

fr* fu * fit * ft:

« St ^ f ud t
dt *

or ..(2941

w h e r e S t ® m a s s in k g s.
D ■ co efficien t o f friction nr d a m p in g c o n s ta n t in n e w to n s | mt m e tr e p er sec.,
K « lin e a r sp rin g c o n sta n t in n ew to n * j**r m e tr e d is p la c e m e n t,

an d « m d t ' *,m ‘nr 'v lo c it y in m elm v'iM x

In P ig 2 41 <M, th e m a g n e tic to rq u e 7*f is o p p o se d by in e r tin to rq u e Tr d a m p in g torque 7p


and r e s tr a in in g sp rin g to rq u e T K T h e r e fo r e th e to r q u e -b a la n c e e q u a tio n is

Tn * T k » J 4- [hit, * k \ m, dt

_ ,d \ dQ, tr
or ...(2.95)
T' m J 7 ? + D - 3 r * x *'
w h e r e J - m o m e n t o f in e r tia in k g m :
D - d a m p in g c o n s ta n t in N -m p er ra d /sec.
K = ro ta ry s p r in g c o n s ta n t in N -m p er ra d ia n
cf0r
an d a n g u la r v e lo c ity in ra d /sec.

T h e d y n a m ic e q u a tio n for a r o ta tin g e le c tr ic m o to r, from E q. (2 .9 5 ), is

T
T , -= Jr d1 ^Fr + D
„ d_Qr, + TL

w h e r e TL is t h e lo a d to rq u e o p p o sin g t h e m a g n e tic to r q u e Tr.

Scanned by Cam Scanner


Electromechanical Energy Conversion Principles 219
^rf. 2 ^ |_

The m a g n e tic fo rce f e or to r q u e T e c a n b e c a lc u ­


lated from t h e p a r t i a l d e r i v a t i v e o f Wftd a s e x ­ A rea A

plained before.
to-*)
Thus b o th t h e E q s . ( 2 .9 3 ) a n d (2 .9 4 ) d escrib e
the total b e h a v io u r o f a lin e a r e le c tr o m e c h a n ic a l
— SE Lh
system and b o th t h e E q s . ( 2 .9 3 ) a n d (2 .9 5 ) t h a t o f a
N
rotary e le c tr o m e c h a n ic a l s y s t e m . O
/TJ0 tdP----
For i l l u s t r a t i n g t h e f o r m u la t io n o f d y n a m ic
e q u a tio n s for a s y s t e m , c o n s id e r t h e co n fig u r a tio n (9-*)
shown in F ig . 2 .4 2 . R e lu c ta n c e o f t h e tw o a ir g a p s
in series is Fig. 2.42. Typical electromechanical system .
R l = * iiL z ja
Ho A
A ssum ing th e ir o n to h a v e in f in ite p e r m e a b ility , s e lf-in d u c ta n c e o f th e co n fig u ra tio n is

L (x) =
Rl 2 (g - x)

r _ 1 & d L (x) _ 1 ,2 N Ho-A _ 2 iV2 • Ho A


From Eq. (2 .2 8 ), ,fee ~ gn I
^ ~ 2o 1 2 ( g _ xyx2 4a ( g - x..*2
)

Therefore, t h e tw o d iffe r e n tia l e q u a tio n s g o v e r n in g th e sy ste m d y n a m ic p erfo rm a n ce a re a s


under:

rT . N * Ho-A di . N • Ho-A dx ...(2 .9 7 )


From Eq. (2 .9 3 ), U t = ir + — r x -r + i
‘ 2 (g -x) dt 2 (g - x) 2 * dt
2 N* H qA d2x n dx „ 5
From Eq. (2 .9 4 ), =M ...(2 .9 8 ) I
fe = i
4 (g-x)* dtf2 + D d i + K 'X

Eqs. (2 .9 7 ) a n d (2 .9 8 ) a re n o n -lin e a r d iffe r e n tia l eq u a tio n s w h ich can be so lv e d n u m e r ic a lly


on a digital co m p u ter. T h e s e e q u a tio n s ca n , h o w e v er, be lin e a r ise d for sm a ll m o v e m e n ts a b o u t
the equilibrium , or o p e r a tin g , p o in t a n d th e n can be so lv ed for d y n a m ic or s t e a d y - s t a t e c o n d i­
tions as d esired .
E x a m p le 2 .2 4 . F or th e p lu n g e r m a g n e t shotvn in Fig. 2.43, the relu ctance o f the iron p o rtio n
•i
is n e g le c te d . T h e c o il h a s N Cylinderical
turns. The sp rin g c o n sta n t is K iron stru ctu re-^
Muitons p e r u n it d isp la c e m e n t.
N o n -m agn e tic i t
N*gkct the coil -resistan ce, m ag- m ate rial
A rea A
n*tic le a k a g e a n d f r i n g i n g .
Movement o f the iron p lu n g e r is \ J a *' K
N-turiv , ty/M-----
b u rn ed ffictio n less. The coil is c o ils S Spring
e*cited from s in u s o id a l voltage
®° that the a ir -g a p flu x a t a n y AreaA T
t V
Movable
p lu n ger
lrne 1 g iven by
^ <tWr cos (at.
The p lu n g e r is e x e c u tin g a Fig. 2.43. Plunger m agnet, Example 2.24.
^pie harm onic m otion .
(a) Derive an expression for the m a g n etic force fe a c tin g on the m o va b le iron p lu n g er, in term s
'•xu. en a n d t. '

^a n
canned by Cam Scanner
IArt. 1.7
220 Electrical Machinery

( b ) F i n d an e x p r e s s io n fo r the coil vo lta g e a s a fu n c tio n * a, a n d t.


(c) W rite the d iffe re n tia l eq u a tio n o f the iron p lu n g e r m o v e m e n t m te rm s o f ^ „

(d , S o lv e the d iffe re n tia l e q u a tio n o f p a r , (e) fo r th e s in u s o id a l ste a d y -s ta te operation

m a ^ e t k ^ f o r c e ^ ’ ' ^ s ^ i l l ' s h o r t e n ^ t h ' e ' a l r - g a ^ l e i ^ ^ 11®11^ 0" ^ 61^ ® relim tan re'of t h i g ^

le n g th (g - * ) is
Rl
1 M-o ^
R elu c ta n c e o f th e n o n -m a g n e tic m a te r ia l b e tw e e n th e ir o n p lu n g e r a n d th e cylindrical iron

s tr u c tu r e is
21
R l2 =
V(A
M +JL '
T o ta l relu cta n c e Rl =
>
1 ,2 dR l d fg-x
= -* iA x
M a g n etic force, 2 dx MoA 2 M
V

1 §m ax
(1 + co s 2 art)
For $ = 4>mox cos f e ~ 2 tymax 4 \i0 A
M
d \f _ _ A .
e = - — = - ^ ( W < t W co s coi)
(b )
. e = U)NQmax s in otf.
) (c) T h e m a g n etic force fe h a s to o v erco m e th e fo rces d u e to
( i ) s p r in g (ii) frictio n an d (iii) m a s s M.
N ow K is th e spring con stant in n ew ton s per u n it d isp la cem e n t. F or an y displacem ent x,
fK = Kx n e w to n s

an d fD = D ^ n e w to n s
dx
w h e re D is th e c o efficien t o f frictio n o f th e m o v a b le p a r t in n e w to n s p er m etre per sec. an ^

is th e v e lo c ity in m /sec.

fu = M ^ - f n e w to n s
dt

and here M is t h e m a s s o f th e m o v a b le iron p lu n g e r in k g a n d — ^ is t h e a c c e le r a tio n m


dt
2
p er se c .
T h erefo re, th e fo rce -b a la n c e e q u a tio n iBfe = f M + f D + f K

or 4 Luyl 0 + co s 2 (0 /) = M ~ A + 0 + i£ r
#1
Ainctia*1
(d) N o r m a lly th e so lu tio n o f s u c h a n e q u a tio n s h a ll c o n s is t o f a c o m p le m e n ta r y 1 jy-si tatf
p a r tic u la r in te g r a l. A C .F . g iv e s th e t r a n s ie n t p a r t o f t h e s o lu tio n a n d P I . g»v e s the *

Scanned by Cam Scanner


Electromechanical Energy Conversion Principles 221

. n of th e e q u a tio n . S in c e h e r e o n ly s t e a d y - s t a t e s o lu tio n is r eq u ir ed , th e C .F . w ill n o t be


T herefore,

A ...(2 .9 9 )

The right h a n d s id e o f E q. (2 .9 9 ) is t h e fo rcin g fu n ctio n an d it is s e e n to c o n s is t o f c o n s t a n t


d double freq u en cy c o m p o n e n ts. T h erefo r e, th e s te a d y -s ta te s o lu tio n o f th is e q u a tio n m u s t
ntain sim ilar c o m p o n e n ts. In v ie w o f t h is , le t t h e s te a d y -s ta te so lu tio n o f Eq. (2 .9 9 ) b e g iv e n

x = P + Q cos 2o )t ...(2 .1 0 0 )

3 7 = - 2 wQ sin 2 ok
then dt
d 2x
= - 4cu2Q cos 2 ok.
and
d t1
d 2x . m
The values o f P a n d Q a r e o b ta in e d by s u b s titu tin g th e v a lu e s o f x and — ^ in Eq. (2 .9 9 ).
dt
Therefore,
- 4 o>2 Q M cos 2 ok + K P + K Q co s 2 ok = C (1 + cos 2 ok) ...(2 .1 0 1 )
a2
<hm
where C =
4 Mo A
Comparison o f th e te r m s on r ig h t and le ft h a n d sid e s o f Eq. ( 2 . 1 0 1 ), g iv e s

K P = C, P =|

2 - - C
and K Q - 4o> Q M = C, Q =
K - 4oo2 M
Substitution o f th e v a lu e s o f P a n d Q in Eq. ( 2 . 1 0 0 ) g iv e s th e ste a d y -sta te so lu tio n a s
A2 K cos 2ok
C C cos 2 ok
x- — i +
K K - 4 io2 M K - 4o )2 M
If the transient p art o f th e solution is required, then additional term s like A j e 1 and A^e
•tall be added to th e stea d y -sta te solution obtained above.
*•8. Som e W o r k e d E x a m p l e s
■f
T’he object o f t h is a r tic le is to so lv e so m e ty p ica l p rob lem s p e r ta in in g to th e p r in c ip le s o f
etatromechanical e n e r g y co n v ersio n . ii•1
Example 2 .2 5 . A n in d u c to r is m a d e from m agnetic core w ith tw o a ir g a p s o f e q u a l len g th
as shown in Fig. 2.44. The excitin g coil h as 1000 turns, A = 5 cm x 5 cm a n d g = 1 cm . C ore is
Qts'un*d to have in fin ite p e rm e a b ility a n d frin g in g is neglected.
(a) Find the coil inductance.
fa case coil cu rren t is 5 A, fin d the m agn etic energy g­j _ [ 3
in the inductor. F in d a lso the force on the arm a - V CROSS-SECTIONAU
AREA A — T

( J t - With c o il c u r re n t h e ld fix e d a t 5 A, fin d the


f
I
ncal energy s u p p lie d by source i f the g a p len gth 1 1 1 ..
*ases from 1 cm to 0.5 cm.
(c) For part (b), find the mechanical work done.
Fig. 2.44. P e rta in in g to E x a m p le 2.25.

Scanned by Cam Scanner


222 Electrical Machinery

S o lu tio n , (a ) For tw o air g a p s in se r ie s, r e lu c ta n c e

Rl =
M
jV2 iV2Ho A _ a
C oil in d u cta n ce,
L = n r 2g ~ 2g
(1 0 0 0 )2 (4 ti x 10 ) (0.05)
= 2 x 0 .0 1 " ° - 15708 H
w h e re a = N 2[IqA = (1 0 0 0 )2 (4n x 1 0 " 7) (0 .0 5 )2 = n x 1 0 '3

F ield en erg y sto red in in d u cto r = ^ L I 2 = ^ x 0 .1 5 7 0 8 (5 )2 = 1 .9 6 3 5 W att-sec

For c a lc u la tin g th e force on th e a rm a tu re, le t th e a r m a tu r e m ove through a distan


th a t air-gap le n g th b ecom es (g - x). Ce 180

at2M a
N ow
2 (g-x) 2 (g-x)

F ield en erg y sto red , W - - L i 2- - - a •I


2 2 (g-x)
' dW fu (i , x) i 2 o
Force on th e a rm a tu re.
S 4' ^

= 2 (5)2 n x 10 23 = 1 9 6 .3 5 N
4 (0 .0 1 )2
(6) E lectrica l en erg y in p u t a t c o n sta n t-c u r r e n t m o v e m e n t
W elec = ( V 2 ~ V i ) I

= |[LZ w h en gap h a s red u ced to 0 .5 cm ] - [Lx w h e n gap length = 1 cm]) f

J tx lO " 3 ‘ 1 1 '
(5)2 = 3 .9 2 7 W att^ sec
0 .0 0 5 0.0 1
1 ............................. 2

f 05 l r 05 -i
= Jo
ig -x Y
0.5
= — (25) (Jtx 10" 3) = 1 .9 6 3 5 W a tt-s e c
(g-x)
E x a m p l e 2 .2 6 . The a rm a tu re o f a 2 -
p o le d c m a c h in e is in a d v e r te n tly d i s ­
p la c e d a x ia lly a s sh o w n in Fig. 2.45. F or
0 = 110°, g - 0 .4 cm , a ir-g a p flu x d e n s ity
= 0 .5 T, a r m a tu r e d ia m e te r = 3 0 cm , fin d
th e a x ia l force te n d in g to p u ll the a r m a ­
tu r e in to a lig n m e n t w ith th e-field s tru c ­
ture.
S o lu tio n . T h e m a g n e tic sto r e d
i 1 B2
e n e r g y p e r u n it v o lu m e = —— • T h e
2
F ig. 2.45. P e r ta in in g to E x am p le 2-26-

Scanned by Cam Scanner


Electrom echanical Energy C onversion P rinciples 223

agnetic energy sto red in th e tw o a ir-g a p s,

Wfld = [E n ergy d en sity] [Volum e of tw o air gaps)


= 1B ^
~Q xg2
2 Mo
■ Force th a t te n d s to p u ll th e a rm a tu re in to a lign m en t,
1B2
” dx 2 Mo
s2
, 1 (0 .5 )2 x 0 . 3 x 1 1 0 x k x 0 .4 x 1 0 " 2
Te n . . _ n _ . — 229.17 N.
471 x 10"7 x 180
Example 2 .2 7 . A solen oid o f height h a n d radius r has N turns. For a solenoid current i,
calculate
(a) the energy stored in side the solenoid
(fe) the ra dial m agn etic force ten din g to burst out the solenoid
(c) the ra dial p ressu re on the sides o f solenoid
(d) the solenoid inductance.
Assume no flux ou tside the solenoid.
S o lu tio n , (a) For th e d irection o f cu rren t i show n in Fig. 2.46, the
magnetic field in sid e th e solen oid is d irected a x ially upward.
i-N
M agnetic field in te n s ity in sid e th e solenoid, H = —7 — A T /m

/ iN 9 Fig. 2.46 Pertaining to


Flux d en sity B = \i0 H = [i0 — W b /m . Example-2.27.

Field en ergy d e n sity in sid e th e solen oid I■

___ 1 iN
J /m 3

Field en ergy stored in sid e th e solenoid,


Wfld = Wfld x volum e o f solenoid
..................... n2
i_N 2 , 1 (iN) 2
x r c r ^ = 2 M°~~hT X n r
= 2*> h

» ) Radial m a g n etic force, f. = = Mo! * * (2D = M, ^ - N

(c) Radial p ressu re on th e sid e s o f solen oid


* ft ___
su rface area o f solenoid

N /m 5
2 ti r h 2 Mo

N N 2 Mo ■nr
W) Solenoid inductance
ouienoia m a u c ia n c e ^ =
u ~ m ~= h

E x a m p le 2 .2 8 . An in du ctor o f resistance 4 11 a n d inductance 2 H is sw itch ed on to a voltage


*°Urce which varies lin early from zero to 8 V in 2 sec an d then sta ys constant. F ind the energy
Stor*d in inductor (a) du rin g the 2 -sec p erio d ( 6 ) after a ll the tran sien ts are over.

Scanned by Cam Scanner


224 Electrical Machinery
J i* . 24

• - —t = 4 t
S o l u t i o n , (a ) A p p lie d v o lt a g e a t a n y t im e g
di
4 «= ^ + L ^ " 4 l + 2 d#

I ts L a p la c e tr a n s fo r m g iv e s , - j - 7 (s) [4 + 2 s]

4
or I ( * ) = s * ( 2 » + 4) s 2 (s + 2)

2 K .1 ^o2 .
L et .2 + S + 2
s 2 (s + 2) s
C o n s ta n ts K al. K M a n d X , c a n b e d e t e r m in e d in t h e u s u a l m e t h o d .

■K„1 = l,tf< ,2 = - f a n d K i = 2
2
_L I 1
!(« ) = 2 s + 2 ’s + 2

* -2^
i (t) = 2 2
II
A t t = 2 sec, c u r r e n t 7 1 = ~ [ e 4 + 2 x 2 - l ] - 1 .5 1 A

E n erg y sto red d u r in g t h e 2 -se c p erio d

= ^ L7? = \ x 2 x 1 .5 1 2 = 2 .2 8 0 1 J

. V 8
( b ) W h en a ll th e t r a n s ie n t s a re o v er, 7 = — = — = 2 A
xv 4
1 \
E n e r g y s to r e d = - x 2 x 2 2 = 4 J .
z
E x a m p le 2.29. F o r m a g n e tic c irc u it o fF ig . 2 .4 7 , le n g th o f ir o n p a t h = 120 cms., g = 0.5
cm , area o f cross-section o f iron = 5 x 5 cm 2, m. = 1 5 0 0 ,1 = 2 A , N = 1 0 0 0 tu rn s.

C a lcu la te a n d com pare the fie ld -e n e rg y s to r e d a n d fie ld -e n e r g y imn core


d e n s ity in iron a s w e ll a s in a ir g a p . N e g le c t fr in g in g a n d le a k a g e
flux.

S o lu tio n . T o ta l r e lu c ta n c e = o f ir o n p a t h + G a p le n g t h
Ho Hr x A r e a jiq x A r e a -^9
T
1 2 0 x 10"2 I—2
0 .5 x 1 0
v-7
4 7 1 X 1 0 " x 1 5 0 0 x 2 5 x 10" 4 4 it x 10" 7 x 2 5 x 1 0
109 T 120
120 0.5 1
Y = 1 8 4 6 2 x 1 0 A 'W b 1 1 . mag"
4 7tx25 1500 Exam ple 2.29.
N i.
A_ H I 1000 X 2 ,
F lu x ,
Rl 1 .8 4 6 2 x 1 0 6 X 10 = 1 0 8 3 3 m W b

F ie ld e n e r g y sto r e d in ir o n = \ f x r e lu c t a n c e o ffe r e d b y ir o n p a t h

Scanned by Cam Scanner


Electromechanical Energy Conversion Principles 225

120 x 10r 2
= 1 .0 8 3 3 x 1 0 " 3 J = 0 .1 4 9 4 2 J
4 n x 10" 7 x 1500 x 25 x 10r t

f eld energy stored in a ir g a p = ^ <t>2 x relu cta n c e o f air gap

.-2
0.5 x 10
= ± [1 .0 8 3 3 x 10" 3l2 x — = 0 .9 3 3 8 7 J
4 7t x 10 7 x 25 x 10i- 4
_ en e r g y sto red in iron
Energy d en sity in iron
v o lu m e o f iron
0 .1 4 9 4 2
-r
4
= 4 9 .8 0 7 J / m 3
120 x 10"2 x 2 5 x 1 0 '
0 .9 3 3 8 7
Energy d en sity in air gap - r4 = 7 4 7 0 9 .6 J/m "
0 .5 x 10" 2 x 25 x 10
Energy stored in a ir g a p _ 0 .9 3 3 8 7 _ 25
E nergy sto red in iron 0 .1 4 9 4 2 '
Energy d e n sity in air g a p = 74709^ = 1499 98 ^ 1500
Energy d e n s ity in iron 4 9 .8 0 7 '
This exam ple d e m o n str a te s th a t m o st o f th e field en erg y is stored in th e air gap.
Example 2 .3 0 . For the electrom agn etic relay o f Fig. 2.48, the exciting coil has N = 1200
tum, and core area A = 6 cm x 5 cm. The relay is m ade from infinitely perm eable m agnetic
material. Neglect frin g in g effects.
Arto.A
(а) Find the coil in du ctan ce for x = 1 cm. For a coil
current of 2 A, fin d the fie ld energy stored a n d force on S 3 "
thearmature. Armotur#

(б) Find the m ech an ical w ork done (or the mechani- 1/
oi energy output) a s the a rm a tu re m oves from x = 1 cm
t°x*0.5cm w ith coil cu rren t rem ain in g constant a t 2 A.
(c) Derive an expression for the m agnetic force ft on
to armature as a function o f x a ssu m in g constant coil

* n n t of 2 A. Hence fin d ^ ft dx.

m (d) Find the m ech anical w ork done in p a r t (6) i f the


Fig. 2.48. Pertaining to Example 2.30.
'■sare m a in ta in e d con stan t a t a coil current o f
h I ^ ................................
. . (\ W fld (¥. *) 1 ^
W Repeat p a r t (d) u sin g the expression J ^
2x
Solu tion , (o) R elu cta n ce o f tw o air g a p s, R l =

rv:,. . N 2 ^ Ho A
11 inductance, L (x) = = Yr
a 2 0 0 ) 2 x 4 7t x 1 0 ~ 7 x 3 0 x 1 0 J 4 _ Q 2 7 1 4 H
Por* = l c m ,
2 x 1 x 10
field energy stored ,
w . . . i i? L, = | « 2 ax 0.2714 = 0.5428 J
rf t d- 2

Scanned by Cam Scanner


F orce on a rm a tu re f - - »2 *LLJ£).
,/e dx ~ 2 dx

1 .2 d AT2 ^ 1 .2 AT2 Up A
2 1 <fx 2 x " “ 2* 2 .t ‘

F o r x = 1 r .q f _ 1 . o 2 v (1200)2 x 4 t i x 1 0 " 7 x 3 0 x 1 0 ' 4


2 2 x 1 * 1 0 -* - = - 5 4 .2 8 7 N .
(5 ) W h e n x = 1 cm , flu x lin k a g e s ,

V , = t, = 2 x 0 .2 7 1 4 = 0 .5 4 2 8 W b t
W h e n x = 0 .5 c m , co il in d u c t a n c e ,

L 2 = 2 L , = 2 x 0 .2 7 1 4 = 0 .5 4 2 8 H
V2 = *i L 2 = 2 x 0 . 5 4 2 8 = 1 .0 8 5 6 W b t.
• • M e c h a n ic a l e n e r g y o u t p u t or m e c h a n ic a l w o r k d o n e

= 2 (^ 2 - V i) »i = ^ ( 1 .0 8 5 6 - 0 .5 4 2 8 ) x 2 = 0 .5 4 2 8 J .

(C)
OX 2 rd/xr 9 rv 2 ---------- X— -
2x2
F o r ij = 2 A, / > - ! * 22 x ^ - _ f C ^
4 =2 2*

) M e c h a n ic a l w o rk d o n e ,

1 J 001
c/x

Xo 1
1
0.005

X
0 01
1
= (1 2 0 0 )2 x 4 n x 10 " 7 x 3 0 x 10 ' 4 _ _
= 0 .5 2 4 8 J.
id ) F lu x lin k a g e s for x = 1 cm a n d i , = 2 A ur = 0 5 4 9 ft [ ° :° 0 5 ° ' 01
W h e n x r e d u c e s to 0 5 t h e flu x linkacr ' * 88 m P 3rt
• C u rren t th a t ' ’T ,in k a g e s a r e a g a in 0 .5 4 2 8 W b t ( g iv e n )
• • C u r r e n t t h a t c a n D rod uce 0 .5 4 2 8 W bt w h e n r - n * • • ,
i a A 2 = 0 .5 4 2 8 " 5 Cm 18 g lV en b *
or • 0 .5 4 2 8
2= = 1A
0 .5 4 2 8
•*. M e c h a n ic a l w o r k d o n e i f t h e flu x li n k a „
x UX *m k a ^ s a r e m a in t a in e d c o n s t a n t
= 2 v, (‘ 1 - i 2) = - X 0 .5 4 2 8 ( 2 - 1 ) = 0 .2 7 1 4 J .

(e) E le c t r o m a g n e t ic fo rce , fe - ~

But
w fld - 2 I (* ) i 2 a n d v = L (x) i ori = _m _
l(x)

= x L (x ) •
£ (X) 2 Z, (a)
H ere
Rl

j
_______ Electromechanical Energy Conversion Principles 227

W 1 V2 2 * x •y 2
fld 2 AT2 n0A \ v V o A
f (V» *) _ x ■\y2 = _ _ . V2 .
u a* = 'a * rfH o A
A JV 0A .
For * - 1 cm, vjf = L (x) • t = ~ •i

(1200)2 x 4 n x 1(T 7 x 30 x 10~4 „„


r 2 = 0.5428 Wbt
2 x 1 x 10
r .5
ft ■dx

="f 5 / • dx = — [ 0.005 - 0.01 ] = °


AT Ho A iV^ m, A A ^ A

Substituting the values, we get

= --------- 0 005 x (° f e 2*?)2-------- = 0.2714 J.


(1200) x 4 n x 10" 7 x 30 x 10" 4

PROBLEMS
2.1. (o) State the ad v antages of analysing energy-conversion devices by field-energy concept ?
(ft) Describe the principle of energy-conversion. From a consideration of the various energies involved,
develop the model of an electrom echanical energy-conversion device.
(c) Show th a t th e reaction of coupling m agnetic field on th e electrical or mechanical system is essential
forthe electromechanical-energy-conversion process.
2.2. (o) Draw and explain fully th e general block-diagram representation of an electrom echanical energy-
conversion device.
(ft) For a singly excited m agnetic system , derive the relation for the m agnetic stored energy in term s of
reluctance.
2-3. (a) Based on the principle of conservation of energy, w rite an energy balance equation for a motor.
Discuss briefly about th e four energy term s involved.
(D) Write the following expressions :
(i) magnetic field energy stored Wfa in term s of <|>and Rl
(“) W/y in term s of vg a n d i
^ W/id in term s of L a n d i
Wfa in term s of v a n d L
(u) wfu in term s of F and Rl
) Wfld in term s of F a n d 4>
W) magnetic field energy density w/u in term s of B an d H
Ufa in term s of g an d B
(tt) wfid in te rm s of p and H. ,An8- Se* Art- 2 21
C t (a) A is excited by a single coil. Discuss th e conditions under which th is toroid can ex tract energy
e "upply system. '
Write expression for m echanical work done L dx from a physical consideration of Fig. 2.49 (a) and
“‘d an expression for th e m agnetic force fr (c) R epeat p a rt (6) for Fig. 2.49 (6).

Scanned
anned by Cam Scanner
dW
[Ans. (b) fe d x ~ -d with constant y, ft = - ~ (V> *)
dW *
(c)ft dx =+d Wnd with constant i, ft =+ ^d (i, x)]
<7 X

2.5. (a) For a linear m agnetic circuit, derive the following relations for th e stored magnetic energy IV
and co-energy W ^' 1,4

= l**-: 4 F t i 4 ♦ * « 4 2 £A ”4 2 /?/
£
1 ,2 1 ^ . .
=2 = 2 L J0ule9
1B
Hence show th a t th e m agnetic stored energy density Wfjd is given by —— joules/m 3.
2 p

(6) A 10 kW , 1440 rpm d.c. sh u n t g enerator h a s a tim e constant —- of 0.2 sec for its field winding. Under
. Tf
norm al o perating conditions, th e I^rf loss in th e field w inding is 400 w atts. Compute the energy stored in the
m agnetic field produced by th e field w inding, under norm al operating conditions. lAns 40 watt-sec I
[I.E.S.. JM51
2.6. F or an electrom agnetic system , show th a t th e m echanical work done is equal to the area enclosed
betw een th e two m agnetization curves a t open and closed positions of th e a rm a tu re and the y-i locus during
th e a rm a tu re m ovem ent.
2.7. (a) Describe th e principle of v irtu a l work and hence show t h a t th e m agnetic force ft is given by the
expression
dWju dW„d
/i— j f - ( ¥ .* ) — _-a f - ( f c x ) .
(3b) 3----
T he---------------
functional--------
relationship
— of m.m.f. 1F,> flux y<)>aim
and m
th ee position
position co-ordinate
co-ordinate xx oi
of »a device is ^gi'en •
.3 3 ry . . . i . . . . .1 ... r . . r— - t e n d ' 0
F= x . C om pute th e stored energy and th e m agnetic force acting in th e x-direction. Does this force
increase or decrease x ?
.decrease*
Verify your resu lt by the use of co-energy function. Ans.

2 .8 . (a) W hat is the principle of virtual work ? Illu strate its use to show th a t m ag n e tic
in an elem entary magnetic relay is given by

dx •t*n
relation bet*
(b) A sim ple electrom agnetic relay, w hen excited from a voltage source, gave th e following
c u rre n t a n d flux linkages :

Scanned by Cam Scanner


Electromechanical Energy Conversion Principles 229

. force on the arm atu re as a function of y.


Find
1
IAug. (6) - | y
a f. -x )I; force tends to increase x for x * H
2V 2 )’
• a singiy-exci
. p erive ex p re ssio n s of field en ergy, co-energy a n d th e m agnetic force in * l *ted

M echanical unit.
f'eCf » Cvamnle 2 7 find the mechanical work done, if the plunger is allowed to move instantaneously rom
(ib) I" Example . , (A n «. (b) 0 .6 0 Joulesl
. 2.00 cm to g = U.o cm.
I' ^ ^ s ^ t e and briefly explain the various phenom ena useful for th e electrom echanical energy coriv
m in rotating m achines.
5 lb) Derive an expression for reluctance to rque in a ro tating electrical m achine.
\ H int, (t) C u rren t-carry in g conductor in a m agnetic field experiences a force. Also force exists betw een
!nt-carrying circuits, because of th e interaction of th eir m agnetic fields.
ft) A ferromagnetic m ate ria l w hen placed in a m agnetic field, experiences a force which ten d s to b n n g it
into the minimum reluctance position !
2 11. (a) Show th a t th e to rque developed in a doubly excited m agnetic system is equal to th e ra te of increase
f field energy w ith respect to displacem ent a t constant currents.
° Hence or otherw ise, derive an expression for the torque developed in a cylindrical rotor m achm e w ith one
rotor S n g excited w ith d.c. and an o th er w inding on th e sta to r connected to an a.c. source. S ta te th e
assumptions m ade for obtaining th e torque expression
2.12. A simplified reluctance m otor is illu strate d in Fig. 2.16 (b). Assume sinusoidal inductance v ariation
with 0r and designate
Ld= maximum inductance when long axis of movable iron slug is in the direct a n s. Le. when 0r = 0 and n
and Lq= minimum inductance when long axis of movable iron slug is in quadrature axis, Le.. when 0r = \ and

Note that Ld and L„ m u st rem ain positive, (a) W rite an expression for the variable inductance in term s of
Ld.Lq and 0r.
(6) Find an expression for the in sta n ta n e o u s torque on the rotating elem ent. Assum e 9. - < M - « > "d
i-Imcos cut.
(rl If the movable iron slug speed re, is m ade equal to th e tim e an g u lar speed <«. is a non-rero fm e -av e rag e
torque possible? If yes, find its expression.
(H in t: For th is problem , refer to Art. 2.5 (c)l.

lA na (a) | I L d + t , l + j O w - f.,) cos 2 9 ,

(41 T, = - I I j (Ld - L,) J ein <2to,J - 2S) + \ sin (2<«,r + 2 ter - 28) e \ sin <2o>,r - 2 eg - 26) j

- 4 ,1 “ n 2 t
2.13. Two windings, one on e te to r and th e other on rotor, h as th e following p a ra m e te r, :
r, = 2.5 ft = 0.03 H
rr = 3.00 fl £,r = 0.12 H
= 0.06 coe 0,
Whtre 6, is the space angle between atator and rotor winding axes.
The ,W„ w inding, a re connected in parallel a n d the rotor i . locked . 1 9 , - D0-. W ith th e c u rr e n t, i n i t i a l ,
* '•. the windings are sw itched on to a voltage source of 30 volte d.c a t tu n e I - 0.
1°) Find ilt ir as functions of tim e.
(!>) Find an expression for th e m agnetic torque Tt as a function of tim e.
(Ana. (o) i4 - 12(1-e ~ “ 3‘) ,ir tx 10(1 - e °-5' )

(b) T. - - 7.2(1 - e " * 1* - ,■-*5r ♦ e ‘m *))

Scanned by Cam Scanner


230 Electrical M achinery------------------------ — ^
— . .
rp niaceti one on the stator and the other on th* r..
2 14. Two sinusoidally d istrib u te d c u rre n t *i h e e s P p a ra m e te rs o f th e m achine are : ' r
of an .d e a h se a ro ta tin g e le c tn c a l m achine, w ith sm ooth a tr g p
L = 1 .0 H ,L r - 0 .3 H .A /,r = 0 S c o a e r
where 0, is the r e l a t e angle between the current s h e e . ^ ^ ^ ^ ^ ^

S ta to r a n d rotor resista n c es are consioered n g g*


1 cos OK w h i l e th e rotor is short-circuited.

" , * „ ™mDUte th e in s ta n ta n e o u s to rq u e developed as a function of 0r and /


(а) W ith th e m achine a t sta n d still, comp
<6> If th e roior is allowed to r o u te , . t w h a t an g le i. w .ll come u rest.

1(6) H in t. T, - 0.2083 f„ , . # thereli)re, torque ten d s to increase 8 ,, i.e. torque tend,


For 8, = 0". ,-o . For 8. - 1 , torque is
T • d istu rb a n c e a t 8, = 0° position will take the rule,
to tak e th e rotor aw ay from 8, = 0“ position T lius a slignc
aw ay from th is position. In view o f this, position of 8 . = 0 is u n s ta b le .
For 8 - SO° T , i = 0 For 8r = 89". torque is positive, th ere fo re , to rq u e te n d s to in c re .s e the angle
r o r b r -» U , 11 (av) 1 r > qqo qn°
89° i e th e torque h a s a tendency to increase th e angle lrom 89 to W .
- u r fnrmip ten d s to d ec re a se th e an gble from 91 to 90 , Thus, between
F o re = 91°, torque is negative, therefore, to rque te n d s to d ec re a se i
8 = 0 ° and 0r = 180°, stable rest position is 90°]. (Ana. (a) 0.2083 /,„ sin 20, (1 + cos 2<of).
(6) Between 0, = 0° and 180°, stable rest position is a t 90°. Between 0,= 180° and
360°, the stable rest position is at 270°.|

2.15. (a) Find an expression for the m agnetic force developed in a doubly excited translational magnetic
system .
(б) Two air-cored coils have th e ir m agnetic axes coincident. T h e ir p a ra m e te rs a re :
L , = 0.4 H, L2 =0.2 H
M 12 = A/2i =0.1e_4*H

) where x is the distance between the two coils.


For steady c u rre n t of / j = 10 A and / 2 = 5 A, calculate th e m ag n etic force developed. Does this force tend
to increase or decrease x ? C alculate th e m ag nitude of force for x = 2.5 cm.
IA na. ( - 20e~ **) N ; decrease x ; - 18 097 Ml

2.16. A doubiy excited ro ta tin g m achine h as following s e lf a n d m u tu a l in d u c ta n c e s :


r t = 40 £1 Lt = 0.16 H
r , = 2 fl L , = 0.04 + 0.02 cos 20,
M,r = 0.08 cos 0,
where 0, is th e space angle betw een coil axes. T he ro to r is revolving a t a speed o f 100 rad/sec.
F or ig = 10 A d.c an d i, = 2 A d.c.
(а) derive expressions for the in sta n ta n e o u s voltages a p p lie d to th e s ta to r a n d ro to r windings.
(б) obtain expressions for m agnetic to rq u e a n d th e co rresp o n d in g e le c trica l pow er.
For both th e p a rts (a) and (6), find th e ir values for 0, = 90°.
• 76 V
IA ns. (a) 400 - 16 sin 0, ; 384 V ; 4 - 8 sin 20, - 80 sin 0, i "
(b) - 0 08 sin 2 0 ,- 1.6 sin 0 , ; - 1.6 N-m ; - 8 sin 2 0 ,- 160 sin 0 , 160 Wflt
2.17. T he doubly-excited m agnetic sy stem o f Fig. 2.21 h a s th e follow ing s e lf a n d m u tu a l inductances
L, =Lr - 3 + cos 2 0, M = 2 cos 0 ^
where 0 is the angle between the axis of stator and rotor coils. The coils, connected in aeries, carry a curr°n l“ „,|
i ~ V2 / cos u*. Determ ine the tim e-average torque as a function of angle 0. [Ans. - 2 / 2 (9'n 2 0

Scanned by Cam Scanner


E lec tro m e ch a n ic a l Energy C o n v ersio n P rin cip le s 23ji

c t r o m a g n e t i c rela y show n in F ig . 2.4b is wound w ith tw o coupled co ils having the following
The ele«
2 .1 8 .
outers .
. 2 , 3
L\ — , — , M —
1 x x x
,«ain an expression for the force on the arm ature as a function of tune if ij = / m] sin oil f and i2 - Im2 s i n to 2 f . f*m
oide of a v e ra g e force in case (a) to, * to2 and (6) w, = u>2.
^ magwluu
[ A m . - \ t , sin2 (o, I -| •■ li,, sin2« , I - L , ■ “ i ""

I 2 + - 12 ( 5) 2 fm, + 2 + |
(°)
2x mi 2 * 2x
219 (a) Define field energy and co-energy. Give the significance of coenergy in the derivation ot torque
r force in an electrom echanical energy conversion device.
devices m ake use of the magnetic field as a coupling medium ra tn e r
lb) All practical energy conversion
ihan an electric field. Discuss.
linear magnetic system are given by identical expressions.
9 20 (a) Prove th a t energy and coenergy in a . .
m An electric m achine h a s cyim drical s ta to r and ealient.poie roior dusiify .he e e r,e c t„ ,.S on =therw,se
ofthe following sta te m e n ts :
Ii)Reluctance torque is produced w hen exciting winding is on rotor.
(ii) Reluctance torque ie produced when exciting winding is on stator.

« When s ta to r an d rotor hoth carry exciting ~ ^ g l a n c e torques are


(iv) When stato r and rotor both c a n y exciting W1^ sg (^ Incorrect (ii) Correct (iii) Incorrect Uv) Correct.l
producet1' ae . if N = 1000 i = 1 Amp . X - 0.5 mm,
2.21. F ig. 2.50 shows a n e l e c t r o m a g n e t used to lift an iron pi • •

ELECTROMAGNET
?
1— m r r r e *
N -TUR N S

.CROSS -
s e c tio n a l
a r e a =a
IRON PIECE
M---1 r s >—

Fig. 2.50. Pertaining to Problem 2.21.


stored m agnetic field energy and (r) ihe force F on the non piece,
* s lc m \ calculate (a) coil inductance, (b) stored [GATE, 1988
H = 4nx 10" 7 H/m) lAns. (a) 0.125664 H l6> 0.062832 W-sec (c) 125.664 N]
, , , =0 A, e 300 tu rns, coil resist,incc - 0 11 and eross-sectional
2.22. For Ihe electrom agnet show n in ig •

* * * * 20 * "’ • Wl t. lift a 120 kg .tab Of .run from »


(°) Find the coil voltage require m inim um uir gap of x =0.5 mm. C alculate
,, When the iron slab is in contact w ith electrom agnet, there is a mu ^ ^ m u y ^ ^ V|
e m,nimum voltage req u ired to lift th is s a • ^ electrom agnet shown in Fig.2 50 Coil
Vm cos tot is applied to the N- urn
2-23. A voltage
^hlance ie R t,. For air-gap length x as shown, find

r ; * - - - mr r r r r : r r : L . . y a , n , .
'5) voltage-source power o u tp u t a

Scanned by Cam Scanner


232 Electrical M achinery

[H int.. Proceed a3 in Example 2.11)


A .ill
1 Vfn JV2 p0A J cos2 ut cos e + 9 in e
A ns
' (0) 2 ( 2 i ^ + (u N2 Po ^ J (6 )^ T ^ l 7 L 2 J
1 V2
m cos 0 w here
2 a)JV2 PoA
R2 2x
caN 2 p0A
0 = tan
2 xR
2.24. Fig. 2.51 show s o singly-excited electrom echanical device h a v in g a sta tio n a ry core (stator) can*,,.

Fig. 2.51. Pertaining to Problem 2.24.


N tu rn s and an unw ound ro ta tin g core (rotor). A voltage e = 20CW2 sin (100rt£) V is applied across the stator
w in d in g w hile th e rotor is ro ta tin g a t lOOn rad /s such t h a t its position 0 a t any in sta n t is given by
0 = lOOrrt + 8, w here 5 is th e position a t t = 0. Rotor surface is sh ap ed such t h a t th e air-gap reluctance is of the
form
R „ + f l j R „ —R j
“ V ^ - - ^ cos2e
(а) W rite th e num erical expression for th e core flux due to th e applied voltage (neglect winding resistance,
leakage an d fringing flux) as a function of tim e i f N - 1000 tu rn s.
(б) C alculate th e average torque developed as a function of 5, if R j = 108/ 4 x a n d Rq =3Rj. [GATE, 19MI

[Ans. (a) 0.9002 cos (lOOn t) mWb (6) 1.61258 sin 25 Nm)
2.25. (a) D istinguish betw een singly-excited a n d doubly-excited m agnetic system s.
(6) In a doubly-excited ro ta ry m achine, th e in d u ctan ce co-efficients a re
= (1.1 + 0.4 cos 20); L>22 = (0 03 + 0.005 cos 20)
Ll2 - 0.2 cos 0.
The exciting c u rre n ts a re ij = 8 A a n d i2 = 50 A.
Ac 19# I
O btain th e to rq u e/an g u lar displacem ent relatio n . D erive th e expression u sed , if any. 11 " , ,
,A n.. (5, ( -3 ..1
2.26. D erive a n expression for th e to rq u e in a doubly-excited m ag n etic sy stem having salient of
s ta to r as well as rotor. H ence show th a t d ifferen tial changes of c u rre n ts do n o t contribute to the pr°
m agnetic torque.
2.27. F or a c e rta in relay , th e m ag n etizatio n curves for open a n d closed positions of armature a
show n in Fig. 2.52. A t a rm a tu re open, air-gap len g th is 1 cm.

Scanned by Cam Scanner


Electrom echanical Energy Conversion Principles 233
Uftki
very slow m o v em en t 01 a rm a tu re , calculate m e
(a) F°* VQrk d one in m oving th e a rm a tu re from open to
j i e c h ^ ^ o j j W here does th is energy come from ?

,«pRepeatt pp aa rrtt (a)


to; in
u i case th e -a rm a tu re moves very fast.
..................... -
(M
( P _ th e m a g n itu d e of average m agnetic force or
nf nverarre on
(c) d u rin g (i) slow a rm a tu re m ovem ent and (ii) fast 0 012
the m0vem ent.
ar,n [Ans. (a) 4 W-sec (6) 2.4 W-sec (c) 400 N, 240 N]
0 fVTL-p following term s and expressions pertain to the
field system. Symbols used hove t h a t usual m eam ng:
, .i *s>2 Wf f l w x ) , . L V ^
i j L dx
tf.i.S, n 1'*'’ 2 I- (*) ’ 2 Po ”■ Fig. 2.52. P e r ta in in g to Problem 2.27.
Wr„e the analogous te rm s a n d expressions for the eleetnc
1^
,/i —
i A n s . , . ^ . ^ 2 C (x) ’ 2 e0
1 ? th = 0 5 cm and its iron

X v n ao n et r e la y ta k e s a c u rre n position, the relay position.


2.30.a 230 v . » » j r 5 s ^ i — * - * ‘t e p
” “ ture 15 h f otor o f 0 .0 8 lagging F '» d the work done ^ 0 .7 lU !
1 A at power factor ^ ^ energy stored - 2 i
230 ft, X\ = aiQe i = ^ Rest posrt*"
(Hint. Find 2 X= 3 ........ „ in Fi*. 2.63, y Of
jArmotur*
-9
r
J 3 S S S S ' - 8 “ S » ^ t h e m agnetic force as a function Armotur*
admass, M
- — — ^ ifo rth e s y s te m s h o w n .
of time. - A —|

d
(b) W rite
l.Xns.
N Poi..SL + ------ ^ dt
(b)vt = 2 ( g ' x) dt 2 ( e ~
■ nr to P roblem 2.31.
vie. 2.53 Pc-W'"'*
_______ THREE
Basic Concepts o f Rotating
Electrical Machines
The basic principles, developed in Chapter 2 for describing the behaviour of
electromechanical energy conversion devices, apply equally well to rotating electrical
machines. The purpose of this chapter is to develop general expressions for electromagnetic
torque and generated e.m.fs. in rotating machines and to make use of the basic principles of
Chapter 2 . These basic torque and e.m.f. expressions are applicable to both d.c. and a.c
machines, because the fundamental principles underlying their operation, are the same. The
constructional features of the various types of rotating machines are also described, so that the
general torque and e.m.f. expressions are modified to suit the particular construction of a
machine. It must be emphasized at the outset that both d.c. and a.c. machines work on the same
basic principles ; they differ only in construction. The final forms of the generated e m f. and
torque expressions for the two types (a.c. and d.c.) of electrical machines differ only because
their constructional details are different.
3.1. Physical Concepts of Torque Production

« J At thf k in n in g of ‘ his cbaPter' a brief of the physical concepts of torque produc


tion in rotating electrical machines is presented.
3.1.1. E lectrom agnetic (or interaction) torque. Fie a \ (n) illnBtmfon „ „ ■■ . .
stator with 2 poles and a cylindrical rotor with one conductor. When stator coils are energised6
stator magnetic flux is set up and its path is as shown in Fip 3 l („\ « energisea,
rotor conductor. If rotor conductor carries a current in d ic t. H i .1 . 1" ° current in the
flux picture ia as depicted in Fig. 3.1 ^ ‘ he
and rotor conductor, both carry currents then the flux nrodn . h C ' ” Stat° r C°
with the stator-produced flux, giving the resultant Z n Y r0t° r CU" ent interaCtS
Fig. 3.1 (c ).Since the magnet,c flux lines behave like stretch “ lst1nbukt,on1as illustrated m
ductor experiences a force m the upward direction The cl \ ° bands. the rotor con-
interaction of stator and rotor magnetic fields is calm 1 f* °ckw‘se turciue developed due to the
’ lb t&hed interaction or electromagnetic torque.
Consider now one current-carryint/ coil o n mu ,. . ...
indicated by dot under stator-north pole and hv r a direction of current in the coil is
rotor current produces rotor flux and ^ ^ ,R g ‘ 3 1 ^ ^
attracts rotor N nnle and r(,n«ic o ..., tW0 poles 011 the rotor. The stator S pole

5carm e^^ram 5can n er


fa sic Concepts „ f Rotatm. E|Mricill
Machines 235

.Direction of Fixed
-■iC rs tor<^ue magnet ■

(d)

Fig. 3.1. Illustrating the production of interaction torque.

Thephysical understanding of interaction torque can further be highlighted by referring to


Fig, 3.1 (e). In this figure, one permanent magnet free to rotate, is placed in the field of a
stationarymagnet. The tendency of the two fields to align themselves in the same direction is
^led interaction torque. The angle between stator-field axis and rotor-field axis is called the
torqueangle 8 [Fig. 3.1 (e)]. Note that the torque angle 5 in Fig. 3.1 (d) is 90°. The magnitude of
eMtromagnetic or interaction torque in all rotating machines is given by
. ••'. Te« (Stator field strength) (Rotor field strength) sin 8. .

issh l 2' * ^ uctance (o** alignment) torque. In Fig. 3.1 (a), thepath of the magnetic flux
HamT W^6n stator coils carry current. If a ferromagnetic rotor is suitably placed in this
Weet,1C^leld. the path of stator-produced flux is effected considerably as shown in Fig. 3.2.
refocta 6 ma^ne^ c ^ ux has a tendency to follow a minimum
fotoreXl.Ce ° r ^ as a tendency to shorten its flux path, the
lanceor e,1?ences a counter-clockwise torque, called the reluc-
the st ^nment ^orQue- When the long rotor axis coincides
!ero. Note^k'1' ^°^ar ax^s>the reluctance torque is reduced to
^hced n l ^ rehJctance or alignment torque can be
^ges ^ en the reluctance seen by the working flux
Pyn e rotor movement, see Art. 2.3.

Stl t 03ri the


* * a . J » oa r°^a^ nS
. electrical machine, having 2
i ! r° ^ o^ f 0r and d
4 poles mtnr show that net
on the rotor,
t ^°luti tl° i0)(fue developed is i zero.
electrical machine with 2 -pole stator and
'JlJ Cecl- u . ?iUstrated in Fig. 3.3. The rotor poles arc equal-
Fig. 3.2. Production of reluctance
^ by 90° G a^ ernate north and south poles are space- torque.
rnechanical (or 180° electrical).

S c an n ed by C a m S c a n n e r
236 Electrical Machinery

First consider rotor poles N j and N 2. Since the torque


angle between N lt N is equal to the torque angle between
N 2,S ; the force of repulsion (clockwise) between N, N i i s N
balanced by the force of attraction (anti-clockwise) between
AT2, Similarly, the torque angles between N\, S and N ,N 2
are equal and force of attraction (clockwise) between N\, S is
balanced by force of repulsion (anti-clockwise) between N , N 2. Fig. 3.3. Pertaining to Exa i
Therefore, net electromagnetic torque due to rotor poles 1
and N 2 is zero.
Now consider the rotor poles Si and S2. The torque angles between N, S x and S, S2 are
equal, therefore the force of attraction (anti-clockwise) between N, Si is balanced by force of
repulsion (clockwise) between S, S2. Similarly the force of attraction (clockwise) between
N, S2 is balanced by force of repulsion (anti-clockwise) between S, S v Therefore, no electromag­
netic torque is developed due to rotor poles and S2.
Hence the resultant electromagnetic torque developed due to 2-stator poles and 4 rotor
poles is zero. In view of this, it is essential that in all rotating electrical machines, the number
of rotor poles be equal to the number of stator poles for the development of electromagnetic
torque.
3.2. C onstructional Features o f Rotating E lectrical
M ach in es
All the rotating electrical machines, used for generation purposes, electric drives or for
control systems, have many common essential features from the construction point of view. For
example, every rotating electrical machine must possess (i) stator (stationary member) (ii) rotor
(rotating member) (iii ) air-gap separating the stator and rotor and (iv) shaft, bearing, founda­
tion etc. In addition to it, every electrical machine usually has
(а) exciting or field winding, which produces the working flux and
(б ) armature winding in which the working e.m.f. is induced by the working flux.
The current in a winding that varies as the machine is loaded is called load current. The
current that produces only a working magnetic flux and does not vary with the load on the
machine is called magnetizing current, exciting current or field current. The winding on the
machine that carries only load current is called armature winding. The winding that handles
only exciting current is called field winding. Current in the field winding is always dc. A wind­
ing which handles both the exciting current and load current is called the primary winding of
that device. The primary winding is usually the power-input winding. The power - output wind­
ing for such machines is called the secondary winding.
The armature winding handles all the power that is being converted or transform ed. The
rating of armature winding is equal to the power rating of the machine. The field winding
rating is about \ to 2% of the rated power of the machine. The power input to dc field winding
is dissipated as I2R loss in the field winding (once the required field current is established)-
The armature windings of both the d.c. and a.c. machines have to deal with alternatJJJ
current only-this is the reason why the armature structures of all rotating machm^ ines
laminated in order to reduce the eddy current losses. Further, almost all the rotating ^ oT
have even number of alternate N and S poles (called hetropolar structure). If power is
taken from the rotor it is obvious that sliding contacts are essential. All types of large ro
machines are provided with radial and axial ventilating ducts for cooling purposes.

S c an n ed by C a m S c a n n e r
Basic Concepts o f Rotating Electrical Machines 237

tJdne^reede^ribednt COns^ruc^ ona^features of more common types o f rotating electri-

3.2. 1 * ^ ,^ C^1*n es‘ P°lyphase induction machine can work as an


^ K f f n t i o n is directed main? t 6 application>its performance is unsatisfactory. In view
Jfthis, attention is directed mainly towards polyphase induction motors.

^ i ^ ^ u t e d ^ v ^ d i ^ ^ t w n 0^ 0^ m0t° r cons^sts °1 stator frame, stator core, polyphase (3- or


2-Pha etc- 1116 stator core is a of cylindrical
steel laaial „ closelv in thp ca f ° ng .leir inner Pe r i Ph e r y for housing the; 3-phase winding.
* St3t0r frame- 1116 * " » end covers made of cast-iron and
testator t o ™chamcal support to the stator core and are not designed to cany

The essential parts of a 3-phase induction motor are illustrated in Fig. 3.4 (a) and (b ). For
simplicity, the stator n ; shown to have 6 slots, though actually the numbef of stator slots is far

®ore.t^ a of t W h r i! ' Pll aS8 Wlnding design’ Three coils aa'< w and cc' rep-
T A hv 120° electrical a n ^ ^ ^ ° respectively- Three windings are space dis­
placed by 120 electrical and may be connected in star or delta as illustrated in Fig. 3.4 (c).
Many a time, the six ends of the three phase windings are brought out to the terminal box on
(hestator frame. The six ends are suitably marked to indicate the starting and finishing ends
! ^ r e e -p h a s e windings. Note that three-phase winding in the stator slots is uniformly
Idistnbuted along the air-gap periphery.
S lator frame

-o r'UlUl
•o terminals

m
u ,

%
Ei Fig. 3.4. Constructional features of polyphase induction motors.

N l v i 8120 motors use open slots so that already prepared and properly insulated coils can
S o r t e d >n open slots. Small size induction motors use semiclosed slots so as to reduce
^eairGgap *en£th between stator and rotor.
*^ N u gap between stator and rotor should be as small as is mechanically possible; this will
^*) leaj 6 fenkage flux between stator and rotor
Qto better operating power factor of the induction motor.
I

S c an n ed by C a m S c a n n e r
238 Electrical Machinery
Uri3.2

ro to X h
on the cast-iron spider carried by the shaft. • aftt.
_ , . * „ „_ n fvmp the rotor winding consists of uninsulated conductor* •
of “ S n i r r e d d e d in the setni-Cosed slots. These s „ , ^ >
UI —------------------------ .~ j l

circuited at both . by
ends . end-rings
. •--------otSkewed
the rotor f-rt.
ho
same material. For good electrical connec- s lo t s
tion, the bars are riveted, brazed or welded
with the two end-rings (Fig. 3.5). In smaller
UmrSa,'»
sizes, say below 40 kW, the assembled rotor r0,0r COr,
core is placed in a mould and the molten
conducting material, usually aluminium, is
forced into the slots. Thus the rotor bars,
end rings and the cooling fan, are cast in
one operation. Without the rotor core, the
rotor bars and end-rings look like the cage
of a squirrel, hence the name squirrel cage
induction motor. Note that the rotor bars End rings
form a uniformly distributed winding in the Fig. 3.5. Constructional details of squirrel cagero’xr
rotor slots. As the rotor bars are short-cir­
cuited by two end-rings, no external resistance can be inserted in the rotor circuit of a squirrel
cage induction motor.
In the wound-rotor type, the rotor slots accommodate an insulated winding similar tothat
used on the stator. The rotor winding is uniformly distributed and is usually connected instar.
The three leads from the star connection are then connected to three slip rings or collector rir^s
mounted on but insulated from the shaft, Fig. 3.4 (b ). Carbon brushes pressing on the s/tp rings
allow, external resistors to be inserted in series with the rotor winding for speed and startmg-
torque control. Actually, the wound-rotor type of induction motor costs more and requires in­
creased maintenance ; it is therefore only used where (i ) the driven load requires speed contrc:
or (ii ) high starting torque is required. Since the rotor is wound with polyphase windings and
carries slip rings, it is called wound-rotor or slip-ring induction motor.
In both the types, the rotor slots are not parallel to the shaft axis, i.e., the rotor slots are
skewed for obtaining a quieter and smoother operation of the induction motor.
The squirrel cage type is simpler and more economical in construction than the wound-
rotor type. Further the cage type is more rugged and requires less maintenance than
wound-rotor type, since the former does not require slip rings and carbon brushes.
A polyphase induction motor receives electrical energy from one alternating
source ; it is, therefore, called a singly excited machine. The stator carries the field mo ‘
armature winding is on the rotor. The stator winding connected to the supply. is cn *
W,n. }n.?' ‘5' m^ ar to transformer primary winding. The rotor winding15 c , Pf»
tr'insf W1 ? BinC<i rcce*VCfl encrgy from the stator by mutual flux, as in
I T S h o 7 WOrr 8’ an indllcti°n motor may he regarded as a generalized j
a now o f t , r ° WOr ,tran8f° rrnati0n fr™ Rtatnr to rotor, along with a change in f i e * * '
a How of mechanical power, see Art,. 6 . 1 . ***

by means oHndnr t *7 ° cn,lo(* aH induction motor, because stator dcfivcr* ^ jj^ n « n,fd
t h e p a r t c u l l " lr formcr ncti0,' )- The type of the rotor used, dec.de* *
ine particular type of induction motor. . fith'r

expo rts a* nowe r7 !iv° ' U m ach ‘ n° 8* In synchronous machines, the nrmnturt> f^ h cft
exports a.c. power (synchronous generator) or imports a.c. power (synchronous mo

S c an n ed by C a m S c a n n e r
------------------------- --------------------- -- ---------------Hasic Concepts of Rotating Electrical Machines 239

dHC- ! " words, the synchronous


depends on the relative motion between fieMfi eV1C|f ‘ The ^eneratlon of e.m.f., in general,
aC generator, alternator or synchronous ar™ature winding. In view of this, an
tatfonary armature, or r o t a t i n g ^ r0tating field P° leS and
synchronous machines are invariablv f, 7 Z u field Poles- Nevertheless,
stator and low-power field winding on the rotor th £ hlgh*P° Wef armature windin? on the
reverse arrangement may also be built. ’ ° Ug Sm8 synchronous machines with the

arc ^ e n bdow^ 8 ° f Pr° Viding the fleld ^ d i n g on rotor and armature winding on the stator

(а) More Economical. It is economical to have armature winding on the stator and field
winding on the rotor. In order to illustrate this, consider a 3-phase, star connected 20o1vIVa|

II kV, synchronous machine. Its line current is = 10,500 A. If the armature winding

F thre°f l UP ringS f aCh caPable handling 10,500 A would be required.


Th f ringm ust be properly insulated from the shaft for a voltage of 1 1 /V 3 , i.e.
6.35 kV. The star-point of the 3 phase winding must also be brought out through fourth slip-
nng, in order to connect it to a grounded metal plate through a resistance.
Assume now that the low-power required for the field winding is 1 MW at 500 volts. Then
the exciting or fleld current is = 2000 A. Only two slip rings, each capable of handling

2000 A, are required. Also each slip ring should be insulated from the shaft for a voltage of 500
volts only. This shows that it is cheaper to have field winding on the rotor rather than the
armature winding. Thus the construction of synchronous machine with armature winding on
the stator and field winding on the rotor is much more economical.
(б) More Efficient. With armature winding on the stator and field winding on the rotor, only
two slip rings are required in a synchronous machine. There are, therefore, reduced slip ring
losses and a more efficient synchronous machine.
(c) Better Insulation. Stationary armature windings can be insulated satisfactorily for
higher voltages, allowing the construction of high-voltage, say 33 kV, synchronous machines.
(d) Efficient Cooling. Stationary armature winding can be cooled more efficiently, thus per­
mitting the construction of large synchronous machines, say 1000 MW or above.
(c) More Output. Low-power field winding on the rotor gives a lighter rotor and, therefore,
lowcentrifugal forces. In view of this, higher rotor speeds are permissible, thus increasing the
synchronous machine output for given dimensions.
(f) Lesser Rotor Weight and Inertia. Field winding on the rotor requires less amount of copper and
Insulation. This reduces overall weight of rotor and its inertia. Reduced rotor weight allows the use of
^ ‘Priced bearings and also their longer life because of minimal wear and tear.
l. te) Rigid and Convenient Construction. Three-phase armature winding, capable of handling
niW voltage and high current can be more easily braced against electromagnetic forces when
f 18 Placed in stator slots In addition, flexible water tube connection for water cooling can be
^tailed more conveniently on stator than on the rotor. This all results in a rigid and con-
nient construction of a synchronous machine.
m ature ToothStrength. High-current synchronous machines require more ar.
7 tuta Conner for each idol Greater amount of copper can be accommodated by making the
SO that wider and stronger tooth are prepared for the armature Armature oji
> r would have wider and stronger tacth whereas the armature on rotor would lead to nor­
and weaker teeth Strong teeth also results in less noise due to vibrnt.on and are less
i

>canned by C a m S c a n n e r
240 Electrical Machinery

likely to be damaged during fabrication and use. Therefore, armatur •


provided on the stator and field winding on the rotor. e w*nding
Synchronous machines are of two types depending upon the geometri 1
rotor, viz., (a) salient-pole type and (b ) cylindrical-rotor, round-rotor or n 3 StrUctur6 of
Salient means standing out, sticking out or projecting and in Fig. 3.6 (a ) f n'Sa^fnt p0je ^
shown on the rotor. The field winding on the salient poles is a concentratedUr.Sa,.ient poleg^6
cylindrical rotor, the field winding is a distributed winding housed in the IncaS(f e.
(6 ). A further distinction between the two types is that the salient pole synch °T^ 0tS' 3?
have non-uniform air-gap — under the pole centres it is minimum and in betwe^ ^ 8machines
air-gap is maximum, Fig. 3.6 (a). In cylindrical rotor synchronous machine^k . 1Ies,the
uniform throughout, neglecting the slot-openings. ’ ae a>r*gapjs
The stators of both the types of synchronous machines are similar to the ' d •
stator. That is the synchronous-machine stator consists of a cast iron stator fr Uct*0n
laminated and uniformly slotted stator core, a polyphase distributed winding’ ? Cylindrical
stator slots, bearings, base etc. Fig. 3.6 (a). As in a polyphase induction motor th etT * “ the
and end-covers serve merely as a mechanical support to the stator core and „ V ? fr1
carry the magnetic flux. ot designedto
Synchronous generators are usually of 3-phase type because of the several advent
sociated with 3-phase generation, transmission and high-power utilization N e v e r th ^
smgle-phase alternators are also constructed for specific requirements. For the g e n e r a l
three-phase voltage, at least three coils (one coil per phase), phase displaced by 120 electrical
degrees in apace, are required. In the elementary synchronous machine of Fig. 3.6 (a), the

^'^Concentrated
f i e l d w i n d in g

Stator
frame

Stator
core D is trib u te d
a rm a tu re
w in d in g

(a)

oa, Ao— A o----------


a,tL
V / Vt>2 <
4a, > Dl
t az
a 1 V
jr» a, J
?2

(6 )
conn*
Fig. 3.6. (a) 4-pole, 3-phase salientpole construction. (6) Schematic diagram of star and delta
with coils m series, (c) Schematic diagram of star connection with coils in pnra

S c an n ed by C a m S c a n n e r
j a s i c Concepts of Rotating Electrical Machines 241

ture has a total of 1 2 slots, i.e., 4 slots per phase ; though the stator of an actual machine
larger number of armature slots, depending on the 3-phase winding design. In Fig
< e a c h phase is shown to possess two coils. For example, phase a coils afe d e b a t e d £
36 Vandfl2. ^ tW° COlls ° f eajch phase are connected in series so that their voltages add.
I n the 3-phases may be connected in star or delta as shown in Fig. 3.6 (6 ). In this figure, the
^ terminals are designated by letters a lt &lt c,. The two coils of phase a may be connected
parallel with a x, a 2 as one terminal and a,\ a2' as the second terminal. Similarly, the coils of
lases b and c can be connected. The parallel connection of two coils will increase the current
parallel path to double the value. These parallel-connected coils with six terminals may be
Interconnected in star or delta. Star connection of parallel connected coils is shown in Fig. 3.6 (c).
The frequency of the generated e.m.f., from Eq. (3.5), is
PN
120 H Z ‘

In India and man) other countries, alternators feeding the power systems, generate a fre­
quencyof 50 Hz.
. . Number of poles,
6,000
P=
Speed in r.p.m., N ...(3.1)

Oil engines and hydraulic turbines operate best at relatively low speeds, therefore, the
alternators driven by these prime-movers, must have relatively large number of poles, Eq. (3.1).
Foraccommodating large number of poles, the rotor diameter must be comparatively increased
andfrom mechanical considerations, salient pole construction has been found to suit best for
lowspeed prime-movers. Salient pole rotor structure is also employed foi comparatively small
synchronous machines, as depicted in Fig. 3.6 (a). For larger machines, the laminated salient
poles are dove tailed to the spider keyed to the shaft, Fig. 3.7 (a). Damper bars are usually
»uit hi ‘V he P° le S^ °0, t0 damp out rotor osciMations. The pole shoe of a salient pole, is
' a y shaped so as to obtain a s^ne wave for the radial flux density along the air-gap
nr^Pj ery' ^ommercially, the salient pole synchronous generators are called hydro-alternators
" ’^generators.

S t o t o r fra m e S ta to r
Oamper c o re
S ta to r
sl° " core

, „.> R'vets

U n ifo rm
0r spmer a ir g a p
(«)
3’7- Syncihn* . (6) (c).
nous machine, (a) Rotor spider and one salient-pole lamination (b) 2-pole cylindrical-rotor
1 ^axn construction and (c) 4-pole cylindrical-rotor construction.
^ ^ r h a t o i ^ ^Ur^>^nes have best operating characteristics at relatively high speeds, there­
a t* con«stJ?V^n ^ these, must have a fewer number of poles, say 2 or 4, Eq. (3.1). If
^ ^ d th e g j. c^ on Is used for such high speeds, the rotor structure may not be able to
^ ^ ^ c t i o n ^ L 118 Cen^rihigal forces developed by the salient poles. Therefore, cylindrical
ls es* suited from mechanical considerations, for high speed primemovers.

S c an n ed by C a m S c a n n e r
142 Electrical Machinery _
.. J rntnr made of one piece, solid steel forrri«
Fig. 3.7 (6 ) illustrates a 2-pole ^ mngeveral pieces of solid steel forgings placed 8fde °r,a^
core lengths, rotor is fabricated riphery. The mam poles are created , Sidt
The axial slots are cut only in a PaTt0 as shown in Fig. 3.7 (6 ). In this figure I yth*
field current, in the « ns,ott®d " ' . cylindrical-rotor structure with 2 concentric ciiis ^ al°f6
concentric coils are shown A P nux density wave in cylindrical rotor synrk*1*1*
is shown in Fig. 3.7 (c)- wavc than in salient pole machines. Commercially, cylinrf0^
m a c h i n e is more nearer to a . turb0-alternators or turbogenerators. dncal
rotor synchronous generators are calico iu

synchronous motors.
For synchronous machines of largo sizes, adequate cooling arrangements must be int0,
porated to dissipate the heat produced by the various losses. For relatively large sues, close)
circuit cooling, using air, water, hydrogen or helium, is often employed.
A synchronous machine is usually connected to an energy system, fed by other alternators
In a synchronous generator, the frequency of the generated e.m.f. depends on the rotational
speed and poles. Thus a synchronous machine can be connected to an energy system, onlywhen
the frequency or rotational speed of the synchronous machine is equal to the frequency ofthe
supply system. In other words, for successful operation, the rotational speed of the synchronous
machine must synchronize with the frequency of the energy system and this is the reason for
naming it a synchronous machine.
A single synchronous generator may operate at any frequency or speed, since it is not tobe
synchronized with any energy system. However, a single synchronous motor fed from an energy
system must synchronize its rotational speed with the system frequency.
3.2.3. Direct Current M achines. In a d.c. machine, the field winding is on the stator
and the armature winding is on the rotor. The constructional features of a typical two-pole
d.c. machine are depicted in Fig. 3.8.
Stator. The stator consists of (t) yoke (or frame) made of unlaminated ferromagnetic
material, (ii) the salient field poles bolted to the inner periphery of the yoke and (iii) bearings,
brush-rigging carrying brush-holders, end-covers etc. The yoke rests on a supporting base and
iSkml du ° f u3St ir° n f° r SmaH machines- In larger machines, the yoke is made of fabricated steel
w lie as igher permeability than cast iron. In modern dc motors to be driven through power-
electromcs converters, the yoke is constructed from laminations in order to reduce the eddy-
current loss.
Yoke or frame
The field poles are made of a stack of
.•Concentrated
stee plates (1 to 1.5 mm thick), rivetted
field winding
together. The pole core, where the exciting
Air gap
mall' , " 1 ? ' 8 W0Und' i8 u8ual‘ y of
polv fa ss' scctlon than the pole shoe (or
Pole face), duoto the following reasons:
com rC? UCCd cro88's®ction of the pole
ing. “ 88 c°W)or for the field wind-

(f>) The large pole shoo : Lominaled


he flux per po|c cntcri ~ Br “ increases armo)urc '

10 th° rtdurti» " - nir-gnprcluctanco'0' “ UC


Fig. 3.8. Constructional features of ft 2-P°lc

Scanner .a m S c a n n e r
___________________________ Basic Concepts of Rotating Electrical Machines 243

gs provide mechanical strength and support to the field winding.


(c)P°leSn0inic considerations, the yoke and poles of smaller machines may be made of
fr0*1eC°
jt-tf011, trated field winding, when excited with direct current, produces alternate north
i f,\ bec°nce1n cailed hetropolar construction. It may be seen from Fig. 3.8 that both the
asoi>th P°e aid yoke, carry half of the flux per pole.
^ature c°r r-ggjng B, as shown in Fig. 3.9 consists of a group of brush-holders and their
The b111® yoke or end-cover. Stationary carbon brushes are housed in brush holders
(tjchU1^ °ressed on to the commutator surface by means of tension controlled springs. Thus
„dareKep. nf the armature winding to the external circuit is through the commutator and
. rf»nnecti°n 01111
thec°n
brushes Cut f o r s o l d e r i n g
- L a m in a t e d A rm a tu re Q . .
w in d in g B ru sh r ig g in g a r m a t u r e w ire *
W m a t u r e core • a t t a c h e d to end
R ise r ^ c o v e r
W edge
-^ C o m m u ta to r ' h o p e d s eg m en ts
segm ent

M ic a s h e e t

Shaft

A rm a tu re C o m m u ta to r f

Fig. 3.9. Illustrating the d.c. machine construction.

Rotor. The armature core consists of a stack of circular steel laminations about 0.4 to 0.6
mmthick The periphery of these laminations is slotted to receive the distributed armature
winding Fig. 3.8. These laminations are insulated from one another so as to decrease the eddy-
currentiosses. In case of small machines, the laminations are assembled tightly on the shaft,
butonthe cast-iron spider in case of large machines.
Inaddition to the field and armature windings, a d.c. generator must have a commutator,
toserveas a mechanical rectifier for the alternating e.m.f. generated in the armature winding
todirect e.m.f. at the brush terminals. For a d.c. motor, the commutator serves as a mechanic a
inverter to invert the direct applied voltage to alternating voltage in the armature winding.
Theserequirements of mechanical-rectifier and mechanical-inverter operations demand that
4earmature and commutator be placed on the rotor and field winding on the stator.
Thecommutator is a group of wedge-shaped copper segments, insulated from each other by
J^raicasheets. The copper segments are tapered and arranged side by side to form a cylinder.
h 3.9illuStrates a hard-drawn copper segment, with two V-shaped grooves, insulated proper-
W a n ite M, from cast-iron sleeve or hub H and from rin g *. The V-shaped construction
h t1te the “ "m utator segments from flying out due to centrifugal force. The threaded nng
totk e^s the various components together and is kept in position by nu . e u is ey
L V haft, taking care that the commutator is kept some distance away (for the armature
> 2 ^ ’ fr0m tha ^m ature core. One end of each commutator segment is Projected to form
^atu COmrautator is 80 placed that riser is facing the armature core, see Fig. 3.9. Two
re wires are soldered to each riser.
be seen that a d.c. machine is equivalent to an a.c. machine Plus a mechanical rec-
'*rlte ; “ mmutator. Since the armature winding is closed on itself, d£
V l ? o a ?elta-c°nnected three-phase alternator, if the armature winding is tapped at inter-
u Metrical degrees, from the other side of the commutator.

jL
Scanned By C am S ca n n er
244 Electrical Machinery' (Art. 3.3

3.3. C on cep ts o f G eneral T erm s P ertain in g to R otatin g M ach in es


f
Some of the terms, like pole-pitch, electrical and mechanical degrees etc., are used quite i
frequently in the literature pertaining to rotating electrical machines. The purpose of this ar­
ticle is to make the reader familiar with these terms.
Fig. 3.10 (a) shows an elementary two-pole machine, with its 2 field coils excited by direct
i
y
current. The flux density at the point a in between the 2 poles will be zero. Under the centre of I
the pole indicated by point b, the flux density would be maximum say positive ; at c it is zero 1
and at point d it is again maximum but negative. Note that the flux entering the stator or i
leaving the rotor is considered positive. In other words, south pole on the stator or north pole $
on the rotor, produces positive flux density. According to this convention, the variation of flux
density B, along the air gap periphery is depicted in Fig. 3.10 (b ). If this variation of B, along
the air-gap circumference is assumed sinusoidal as shown in Fig. 3.10 (6 ) ; then it is said that
“in the elementary machine of Fig. 3.10 (a), the flux density is sinusoidally distributed in
space”.

Fig. 3.10. (a) Elementary two pole machine, (6) Flux density variation along air-gap periphery and
(c) Pulsating flux.

Suppose the field coils of Fig. 3.10 (a) are excited with alternating current i = Im sin otf.
/
Then, at art = 0, current is zero and no field flux is produced. At (at = - i = -77 and maximum flux
6 2
Bm K
. density under the pole is — At this instant of (at = - the variation of flux density wave along
x
the air-gap periphery is also sine wave as shown in Fig. 3.10 (c). At (at = the flux density

under the poles is Bm. After (at = the flux density wave starts decreasing. At (at = n, the flux %
density is again zero. After (at = n is crossed, the direction of current in the field coils is reversed
Ow . «
and consequently field poles of reversed polarity are created. At = the flux density is

Note that the axis of .field flux remains along bd. Such a flux is called alternating or
pulsating-stationary flux.
Note that the pulsating flux at any instant is distributed sinusoidally along the air-gap
periphery.
E lectrical and m ech a n ical d egrees. The e.m.f. induced in a conductor is g i'cn ^
Faraday’s law of electro-magnetic induction, which can be expressed in many different orm

S c an n ed by C a m S c a n n e r
Art. .V,M llcslc Concepts of Rotating Electrical Machines 245

In one fbnu, (he generated o.m.f. «■in given by 1)1v volts ; where /lux density B in T, length of the
conductor I in metros and velocity t> in m/see, arc orthogonal or normal to each other. If a
conductor of constant length /, moves with a constant velocity u, the e.m.f. e, induced in the
conductor, has the waveform of flux-density variation. That is, for square flux-density wave,
e.m.t. waveform is square ; for rectangular (lux-dcnsity wave, e.m.f. waveform is rectangular
and for sine flux-density wave, o.m.f. waveform is also a sine wave.
In Fig. Jh.ll) hi), assume that one conductor rotates at a uniform angular velocity. When the
conductor is at a, conductor o.m.f. is zero (because I) is zero), at b the conductor e.m.f. is maxi­
mum (because /» is maximum) and so on. In one revolution, one cycle of conductor e.m.f. is
generated, as shown in Fig, 3 ,1 1 (a). Assume now the conductor rotating in a 4 pole machine of
0

2IT Space angle


0

(a)

jf O.InElecrad, Pole-
Pitch
Mech rad.

Time angle
tot

(6 )
Fig. 3.11. Flux-density variation with spneo nnglo 0 and corresponding e.m.f. variation with time angle col
(a) For two-polo machine nnd ( b) For 4-polo machine (c) An elementary 4-pole machine.

Fig. 3.11 (c). When the conductor is at a, e.m.f. is zero ; at 6 , e.m.f. is maximum ; at c e.m.f. is zero
Md so on. In one complete revolution, two cycles of e.m.f. are generated as shown in Fig. 3.11 (6).
Since one cycle of e.m.f. or current is equal to 360° electrical, in one revolution of 360 mechanical
degrees, 2 cycles are equivalent to 720° electrical for a 4-pole machine. In other words, 720 electrical
degrees in n 4-pole machine enn be related to 360 mechanical degrees as follows :

720 olectricnl degrees = ^ (360 mechanical degrees)

or
^clect q

For a P-polo machine, — cycles of o.m.f. will bo generated in one revolution. Thus for a

^'Pole machine,

Jclrct mrch

S c a n n ed by C a m S c a n n e r
IArt. 3J
246 Electrical Machinery ------- ----------------- -------------- ---------------------------------------------------------------

dt
p ...(3.3)

gular speed in electrical radians per second and fflm is the angular speed in
where oo is the an
mechanical radians per second. 4\
of e.m.f.
As has been shown above, for a 4-pole machine , in one revolution, y

are generated. Therefore, for a P-pole machine ; in one revolution, f cycles are generated. For

P
a P-pole machine, in one revolution per second, f cycles per second are generated. This thought
P
process leads to the conclusion that for a P-po,e machine, in n rev./sec„ * - n cycles/second are

generated. But cycles per second is referred to as the frequency f a t the e.m.f. wave.
Pn cycles per oornnrl
f -_ — second nr HftrtZ
or Hertz -(3.4)
2
If the speed N is in r.p.m., then
m Hz .-(3.5)
' 120
• ,i nfPiu 1 1 (a) or (b) rives the variation of flux density B as

rotation involves time, it may be stated that space variable quantity B is transformed to line
variable quantity e, on account of rotor rotation. equal
Pole-pitch. The peripheral distance between two adjacent poles, is called pole-pitchyPole- theti
p i t c h , s a l w a y s expressed in electrical degrees, rather than in mechanical degrees It can t o - *
fore be inferred that pole pitch is always equal to 180 electrical degrees or a electrical radians. y,
Pole pitches are indicated in Fig. 3.10 (a) and Fig. 3.11 (c). Mh
Coil. Fig. 3.12 (a) illustrates a 2-pole machine, with one coil a, a housed in^two stator s o
Its developed view, with coil laid out flat is illustrated in Fig. 3.12 (6 ). Fig. 3.12 (c) gives P 1
of the cod a, a'. Fig. 3.12 (c) reveals that coil a, a' has one turn. The e.m f. is generated m active ?
lengths AB and CD only. These active lengths are called the two coil-sides of a coil. Forth eld
of Fig. 3.12 (c), each coil-side has one conductor. Thus it is seen that one turn consu ^ ^pic
conductors and one coil is made up of two coil-sides. In Fig. 3.12 (c), the two coil-si es ^ ^ ^
CD are connected in series by end connection so that the e.m.f. at coil terminals i, *>nVl
summation of two coil-side e.m.fs. en$j
If the plan of the coil o, a' is as depicted in Fig. 3.12 (d), then this coil is seen to» jj.gide ^
It can alternatively be stated that Fig. 3.12 (d) represents single two-turn coil. ba ^ ^
has two conductors and total conductors in the coil are 4, Fig. 3.12 (d). If one c0‘ duct0rs
then it is termed as single AT-turn coil, with N conductors in each coil side ana a
in the coil. Fig. 3.12 if) illustrates one iV-turn or multi-turn coil. .g ^ Ii
A coil, with two coil-sides 180 electrical space degrees apart (or one polc-pitch‘ ^il-sides ^ c
called &full-pitch coil. A full-pitch coil can alternatively be defined as a cod whose ^ jts ,
span one pole-pitch. It may be seen from Fig. 3.12 (a) that coil a, a is a full-piior’coi|.span
two coil-sides are one pole-pitch apart. Thus a full-pitch coil is a coil, with coil-p

S c an n ed by C a m S c a n n e r
r
Ar(- 3 3 !_______________________ Basic Concepts of Rotating Electrical Machines 247

Fig. 3.12. (a) Elementary 2-pole machine with one coil. (6) Its developed view
(c) One-turn coil and (d) Two-turn coil.

equal to 180 electrical space degrees. Coil-span (or coil-pitch) is defined as the distance between
the two coil-sides of one coil. Coil-span is measured in terms of electrical degrees, coil-sides or
slots. In Fig. 3.12 (e), coil span = 180° electrical space degrees, this coil is, therefore, termed a
full-pitch coil. In Fig. 3.12 (/), coil-span is less than 180° ; this coil is, therefore, called a short-
pitch, or chorded coil. A chording angle e is defined as the angle by which coil-span departs from
180° electrical space degrees. In Fig. 3.12 if), chording angle is e ° , therefore chorded roil of this
figure has a coil-span = 180° - e.
F lux p e r p o le . For calculating the flux per pole, consider Fig. 3.13 (a) and (6 ) ; where the
field windings are taken on the rotor and stator respectively. Field winding in these figures is
depicted on the cylindrical structure and note that it does not make any difference if the field
winding were on the salient pole structure. In both the figures, the origin for space angle a, for
convenience, is taken 90° away from field winding axis. In view of this, when a = 0°, the flux
density B is zero, when a = 90°, B is maximum say Bp, when a = 180°, B is again zero. This
variation of flux density with space angle a is illustrated in Fig. 3.13 (c) and if this variation is
assumed sinusoidal, then the flux density B can be expressed as,
B = Bp sin a ...(3.6)

In order to calculate the flux per pole, consider an elemental angle da, a degreesaway from
zero of the flux density wave, Fig. 3.13. If I is the axial length of the armature core and r is its
radiu8, then the pole flux component passing through the elemental surface area Irda, near the
a‘r gap, is given by
dty = B. elemental surface area = B lrda = Bp Ir sin ad a ...(3.7)

cann ed by C a m S c a n n e r
248 Electrical Machinery (Art. 3.3

(c) Fig. 3.13. (d ) Sine distributed flux-density


Fig. 3.13. Pertaining to flux per pole. wave over one pole pitch.

Total flux per pole $ can be found by integrating Eq. (3.7) from a = 0 to a = n

<J>= Jo Bp lr sin a da

= 2Bp lr . . . ( 3 .8 )

Eq. (3.8) gives the value of flux per pole for a 2-pole machine. Its magnitude for a P-pole
machine will now be obtained. For a machine of given dimensions,
per pole area for a 2 -pole machine
2 nrl
= nrl

per pole area for a P-pole machine

2 K rl 2 / IX
= ~ p ~ = p (n r l)

This shows that pole area for a P-pole machine i


times the pole area for a 2 -pole
machine.
For a 2-pole machine, with pole area nrl, flux per pole
= 2 Bp lr.

If a is measured from the field winding axis


then B = Bp cos a
t/ 2
and
-I - k /2
B lr cos ada = 2Bn lr

v-/ V-» I II l\yVJ k_/ WW IIIv-/ uu I II IVyI


3.3 ] Basic Concepts of Rotating Electrical Machines 249
Art-
2
For a P-pole machine, with pole area — (7trl),

(2 4
flux per pole is given by 2 Bp lr p iir l = p B p lr ...(3.9)
7trZ
. )
Another method of obtaining Eq. (3.9) is as under :
For a P-pole machine, the variation of flux density B with space angle a will again be
imilar to that shown in Fig. 3.13 (c) and will be given by Eq. (3.6). For obtaining the flux per
S le the elemental angle must be expressed in mechanical degrees. So consider an elemental
sp a ce angle d a min mechanical degrees (similar to d a in Fig. 3.13 (a)). The flux passing through
the elemental mechanical surface area lrdam is given by
<f<|>= B lrdam = Bp lr sin a d a m ...(3.10)

Here subscript p is used to denote peak value.


From Eq. (3.2), electrical degrees
p
= (mechanical degrees)
a
P
a =-a m
2
p
da = 2 d a "
Substituting the value of d a m in Eq. (3.10), we get elemental flux cty as
2
d<\>= — Bp lr sin a d a

2 f* .
Total flux per pole, <J>= — Bp ■lr JQ sin a d a

m± . B p lr ...(3.11)

Alternatively, total flux per pole for a P-pole machine can be obtained as under :
Sine distributed flux density wave produced by one pole, spans one pole pitch. This sine
wave can be replaced by a rectangular wave of average magnitude B av again spanning one po e
pitch, Fig. 3.13 (d ), provided the flux per pole calculated from Bp or Bav is the same. In view of
this, the flux per pole <|>can be written as
<t>= (Average value of constant-amplitude flux density wave under one pole)
x (Area pertaining to one pole of the flux density wave)

- ( \Ba
~ R „) p

. Now recall that the average value of the positive or negative half cycle of a sine wave is
&ven by

\ (Peak value of sine wave).

, . .h view of
of this, the average value of the flux density wave over one pole-pitch. Fig- 3.13 «f)
this, the
45 -

by

2 „
= - Bp

S c a n n ed by C a m S c a n n e r
250 Electrical Machinery [Art 3.4

(2 b V 2icr/A
Total flux per pole =
K

= ^ Bp rl

3.4. G enerated em fs
In rotating electrical machines, emfs can be generated in armature windings (a) by rotating
these windings through a magnetic field (6 ) by rotating the magnetic field with respect to these
windings or (c) by designing the magnetic circuit to have variable reluctance with rotor rota­
tion.* In this article, generation of e.m.f. due to relative motion between field flux and armature
winding is only considered. Elementary forms of rotating machines, depicted in Fig. 3.14, are
used in deriving the generated e.m.f. expressions. It does not make any difference whether the
field poles are created on the cylindrical rotor of Fig. 3.14 (a) or on the salient pole rotor of Fig.
3.14 (6 ). Similarly the field poles created by the cylindrical stator of Fig. 3.14 (c), may be
produced by a salient pole stator of Fig. 3.14 id). It is the relative motion between field flux and
armature winding that matters. The distribution of the flux-density wave in the air gap is
assumed a sine wave. In Fig. 3.14 (a) and (b ), stator-coil (or armature-coil) axis is stationary
while field-winding (or field) axis rotates. In Fig. 3.14 (c) and (d ), field axis is stationary but
armature-coil axis rotates with the rotor.

N -tu rn coil

(b)
N-turn coil
N-turn coil

Field axis
Field axis J— 0=0

e=ur t

Armature Armature
coil axis coil axis
(c) (d)
Fig. 3.14. Pertaining to the generation o f e.m.f. in single N-turn full-pitched coil.
of
3.4.1. G enerated e.m .f. in a fu ll-pitch ed coil. In Fig. 3.14, the two coil-sides a, a
one coil, are diametrically opposite to each other, i.e. the coil spans 180 electrical space def reesd
For simplicity, two-pole machines, with one N-turn full pitched armature coil are consider
first in Fig. 3.14. Actually an electrical machine consists of a large number of properly conncc e

* Inductor alternators work on this principle, see the book, “Generalized Theory o f Electrical Machines *>) 111
same author.

W I I I vV \j>UI II I W I
I

Basic Concepts of Rotating Electrical Machines 251

e coils. The resultant armature voltage in any machine is obtained by adding the in-
^du al cQ-j e m.fs. in a manner decided by the armature winding design.
^,Vl p. 3 . 14 , the time origin is chosen arbitrarily, at the instant coil axis coincides with the
inding axis. That is at the time origin t = 0, time angle 0 = = 0 and the field winding
■ d coil axis are coincident. Here id,, is the angular velocity in radians per second, at which
3*'S^ r rotates. An examination of Fig. 3.14 reveals that at the instant 0 = 0, the flux passing
f° h (or linking) the coil is maximum (= 0 cos 0°). In other words, at 0 = 0, the entire flux 0
^IwTced by one pole, is passing through the coil. When time angle 0 = o),t = 90°, the field wind-
Pr UC hecomes in quadrature with the armature coil axis and, therefore, the flux passing
ing h the coil is zero ( = 0 cos 90°). For 0 = <D,i = 180°, the flux passing through the coil is
^ r°Umaximum ( = 0 cos 180°), but reversed in direction. In view of the above thought process,
a^am rpccion giving the variation of flux passing through the coil can be written as 0 cos 0 or
^ 6X^ t Therefore, the flux linkages y with the full-pitch A-turn armature coil, at any time t, are .
a cos “V ■ ,
y = N (flux passing through the coil at any time t)
= N 0 cos 0),i ...(3.12)

By Faraday’s law, the e.m.f. induced in N-turn armature coil is given by

e = - ^ = N 0 a), sin o),. t - N ^ cos w.t ...(3.13)


at ^
In Eq. (313), N $ w, sin to, I is the speed-voltage term, because it contains speed to, in it.

The second term N % cos CO, t is the transformer-voltage term, becau

derivative of the flux <M f thefield flux *

vohage’ tor’ speed e.m.f.) alone. Thus forelectrica^m achinesw.tht™ ^ux, e


generated e.m.f. is equal to the speed, rotational or motional e.m.t. alone g ^
e = N wr 0 sin <V .

In Eq. (3.14), remember that N is the coil. Note


total flux per pole and (Or is the relative ve oci y therefore applicable both to d.c. and
that Eq. (3.14) represents the general e.m.f. e(l uatl0 constant).
a.c. machines in which the field flux is maldmum value of the speed or
If the single N-turn coil belongs to a .c . machines,
generated e.m.f. E mai occurs when sin (Ort - 1.
E max = cor N 0 = 2 n fr N 0
The r.m.s. value of the generated e.m.f. in a full pitched coil is
p . .. . ...(3.15)
p_ = V2 it N 0 = 4.44 f. N0
1,2 , w frmuencv since its value depends upon the
Here f t may be called the rotational orspeed / q _’ coi| The magnitude off, is
re!ative velocity between the flux-density wave and the arma
&ven by

f - — Hz
2 Jr r n q ) between the armature coil
Wh» » » , is therelative speed in revolutions per second (i.e. P ■
an<1 nux-density wave. atcd e.m.f. in single turn full-pitch
M°te that Eq. (3.15) gives the r.m.s. value of g
ature coil of an a.c. machine.
S c an n ed by C a m S c a n n e r
252 Electrical Machinery [Art. 3.4

Eq. (3.14) can be re-written as


71
e = M or <J) cos co^ -
...(3.16)
= E m ax COS (0^ - -

An examination of Eqs. (3.12) and (3.16) reveals that the speed or generated e.m.f. lags by
90° the flux that generates it. This statement is true when the flux is time-invariant and is sine
distributed in space. Nothing has been said about whether the armature coil moves or the
flux-density wave travels. It is the relative velocity between flux-density wave and the coil that
matters.
3.4.2. G enerated e.m .f. in a short-pitched coil. Let the AT-turn armature coil be short-
pitched by e electrical space degrees, so that the coil-pitch or coil-span is (180-e) electrical space
degrees. For the short-pitched or chorded coil, Fig. 3.14 (6 ) is redrawn as shown in Fig. 3.15
(a). The chording angle £ is shown as e /2 at two places, in order that coil axis and field winding
axis coincide. A chording angle , as already defined, is that angle by which the coil pitch departs
from 180° electrical. Chording angle is also sometimes called short-pitching angle. It is obvious
that the flux passing through the fractional pitch or chorded coil of Fig. 3.15 (a) is less than
the flux passing through the full-pitched coil of Fig. 3.14 (6 ).

Fig. 3.15. Pertaining to the generated e.m.f. in a short-pitched coil.

Note that the total flux per pole is again equal to 2Bp lr for a 2-pole machine and | Bp lr for

nrder°tn o fs? ace anSl e a is aBain taken as 90° away from the field axis. In
order to determine the flux passing through or linking the coil, Eq. (3.7) should be integrated
from a = - t o a = n - | , refer to Figs. 3.15 (a) and (b ).

Flux passing through the chorded coil of Fig. 3.15 (fl)


I*-e /2
= Je/2 Ep lr sin a d a = 2 B n lr cos
But 2J3 lr = <J>

.-. Flux passing through the chorded coil = A cos —


2

mum and is equal to $ cos - = <j> C08 ~ C08 0 C


When 0 = ov< = 90°, refer to Fig. 3.15 (c), the flux

Ic a n n e d by C a m S c a n n e r
Basic Concepts of Rotating Electrical Machines 253

through the coil is zero -<J>cos ^ cos 9 0 ° , In Fig. 3.15 (c), though one flux line is
passing V J
arou n d coil aa', the flux passing through the coil is zero. This fact may further be ascer-
Sh° * 5 , 0bServing that the coil magnetic axis and the field winding axis are 90° away
ta’ne rher. F °r 0 = o^. f = 1 8 0 °, the flux linking the coil would again be maxii
(>8ch ® \
J. cos 180° , but in the reversed direction. Therefore, a general expression for the flux
' ij)cos 2
passing through the chorded coil is
<j>cos ^ cos 0 or f cos ^ cos a\t.
Li L,

■The flux-linkages with short-pitched N-turn coil are


£
\y = N<j> cos — cos (D,.f ...(3.17)

By Faraday’s law, the e.m.f. induced in N-turn short-pitched armature coil is given by

e=- ^ = Nty 0)r cos | sin <0,2 ...(3.18)

In Eq. (3.18), flux <|>is considered time-invariant.


The maximum value of the speed voltage Emax occurs when sin <0,2 = 1.

••• Emax=N<ba>r cos |


Its r.m.s. value is ,

£ = ^ £ = V2Tl/;Ar*cos| ...(3.19)

It may be seen from Eqs. (3.15) and (3.19) that the effect of short-pitched coil is to reduce
the generated e.m.f. An examination of these equations reveals that this reduction factor is

c°s - and is referred to as coil pitch factor, coil-span factor or pitch factor, symbol kp.

Pitch factor, kp = cos ^ ...(3.20)

eq (3.19) can be re-written as


E = 'I'M kp f r N t ...(3.21)

, E1 (3.21) gives r m s value of the generated e.m.f. in a short-pitched N-turn armature coil
»-c. machine.
, 3; « . A.C. M achines. In rotating electrical machines, the armature turns are usually dis-
b W in sl»te rather than concentrated in single slot. This is essential from the view-po.n
■‘'Bing the completely. The -effect
the armature periphery completely effect of
o f distributing ‘the
he turns in different
turnskin difforen t
t)je re*ults in a
^ ln a further
further reduction
reduction of
of generated
generated e.m.f. by the factor kd. This factor is called
e.m.f. by
stribution, breadth or spread factor (see Art. 3.5.1.).
hus “ >e generated e.m.f. for a short-pitched distributed winding, from Eq. (3.21) is
Of E = ^J2nkp kd fr Nty ^ 22)
% re/fe E = 'l2nklufr N<b
(~ kp kd) is called the winding factor.
n (3.22), note that N is the number of distributed turns in series.

S c an n ed by C a m S c a n n e r
254 Electrical Machinery (Art.^34

The basic expression for the generated e.m.f. in a.c. machines is given by Eq. (3.22). This ^
expression is now modified for the more common types of a.c. machines, in which the air-gap ^
flux ^ is constant in amplitude.
(t) Synchronous machines #
(а) Arm ature winding. Two or three phase (i.e. poly-phase) armature winding is dis­
tributed in the slots along the armature periphery. The field winding is excited with d.c., there­
fore, the amplitude of the field flux wave does not vary with time. If Nph is the total number of
series turns per phase of the polyphase armature winding, then the e.m.f. generated in any one 4
phase, from Eq. (3.22), is given by ^
E —V2 71 kwf r Nph <}> ...(3.23)
If time origin is taken at the instant when fluxlinkages with phase a aremaximum, then
e.m.f. ea for phase a can be written as
ea = 'l2E sin ayt
For a phase sequence a, b, c ; voltages for phases b, c can be expressed as
f
eb = V2 E sin {(at - 120°)
and ec = V2 E sin {(at - 240°) *
These expressions for ea, eb, ec are phase voltagesand can bemeasured between line ter­
minal and neutral for a star-connected alternator. For a delta-connected alternator •e e, e
jeri
represent phase as well as line e.m.fs. ’ c
3K«
h ^ t Synf ? T ° U! machine can on]y at synchronous speed. Consequently the relative velocity ftni
between the flux-density wave and the armature winding must be synchronous speed. The rota­
tional frequency/;, is equal to In a synchronous machine P ( ^ ^ o n o u s speed in r.p.s.) .
Pns 2 1 ’ 2 B
2 is equal to the frequency f in Hz, for which the synchronous machine is designed. Thus the
generated e.m.f. in any one phase of a synchronous machine is given by
Eph = '& K fk wNph§ (3 2 4 ) In

t J t ° “ rnr ■ a; e r r r 1na- **
primary and secondary coils. In a r o ta tin /m a r h W f^ T? induced in the stationary
and the flux-density wave, causes flux lfn k W th 1 ^elatlve motlon between armature coil
e.m.f. is induced in the l° with time and as a result- an Bn
maximum value of the core flux, 4U . <•
pole. In a transformer, the windings are concent' f machine>flux <t>1S the total flux per
armature winding is distributed and the reducHn f + m a Synchronous machine>the ’
sion. 6 reductl0n factor kw must appear in the e.m.f. expres-
(б) Field-winding. The field winding of a u
direct current. Under steady-state condiHnnc ^ , f ous machine is always energised with
y ’ e e^d 0r exciting current is given by

/, = 7 ...(3.25) ,.
where f t . direct voltage applied to the field winding '
and rf = field winding resistance. ,
(«) Induction Machines v*1
(a) Stator. The polyphase stator winHinrr c • 1 . ^
ture winding of a synchronous machine wru a,n lnduction machine is similar to the arina- b
machine. When balanced polyphase voltages are applied to the 1

S can n ed by C a m S c a n n e r
Basic Concepts o f Rotating Electrical Machines 255
.
^ w i n d i n g , a rotating magnetic field of constant amplitude is produced in the air-gap. The
stBt°d0f this rotating field is referred to as the synchronous speed and is given by Eq. (3.64).
^c'ncc there is relative velocity between the constant amplitude rotating field and the sta­
ler stator winding, an e.m.f. is induced in the stator winding. The magnitude of this e.m. .,
tjonary Dhase is given by Eq. (3.24), which is repeated here, for convenience as
i” an ; E ^ ^ l 2 n f k mNph,(f, - (3-26)
For induction motor, f is the frequency of the applied voltages and Nphs is the stator series
s per phase. The stator induced e.m.f. Es differs from the stator applied voltage by an
I tUrD nt pnual to the stator leakage impedance drop-this phenomenon being similar to what
P amoa primary winding of a transformer. Thus E s given by Eq. (3.26) represents e
* ^ t e r e.m.f. in the stator winding. Actually the stator winding of a polyphase induction motor
- °iay be treated like the primary winding of a transformer.
? 11,3 (b) Rotor. The rotating magnetic field in the air gap of aninduction motor, travels at a
- synchronous speed of co (= rad/sec, determined by the supply frequency f of the currents

i the number of stator* p o W P . Assume that the rotor runs at a speed <o,(= nPn,)less than
J t ti £r speed Cl). If the rotor revolves in the direction of rotating field, then the relative
* teed between the rotating fluic wave and rotor is (to - <o,) rad/sec. It has been shown before that I r
4 velocity between the coil and flux-density wave, determines the magnitude of in-
dtcedI “ m f.Tn t h f cod In view of this, the induced or generated e.m.f. in one rotor coil of
tf-turns is given by (refer to Eq. 3.14), ^
. e = N (to - (o,.) (J) sin (oi - u>r)t

The e.m.f. induced in one phase of the rotor distributed winding is


vl

A- e = Nphr (W - 0),.) k w $ sin (to - cor)t


* where Nphr is the rotor series turns per phase.
* Eq. (3.28), the relative speed between rotor and rotatingfluxw a v e , ( . - co,), is

referred to as the slip speed in rad/sec.


... Slip speed = (w - cor) rad/sec

, .fh thp rati0 since it is a dimensionless quantity.


But it is more convenient to work with the ratio ,

0 )-
Slip s = CO
S ynchronous_sgeedj^Rotorsgeed ...(3.29 a)
= Synchronous speed

ns ~ n r __ N s - N r ...(3.29 b)

~ ns Ns
o)r - o X l - 5 ) ; n r = ns ( l - s ) ...(3.30)
or Nr = Nt ( l - s )
in r n q and r p in. respectively. Further nr and
Here and N . are the synchronous s p e e d y n ■>"<* ^ P
K are the rotor speeds i n r.p.s. and r.p.m. respectively.

Eq. (3.28) can now be re-written as, .,.(3.31)


e ^ s w N ^ k , ^ Bin sa t

S c an n ed by C a m S c a n n e r
256 Electrical Machinery_______________________________________________

Its r.m.s. value is E = 1/V2 [s ( 2 k f) Nphr kw <f>1


= <2KsfkwNphrt> (
An examination of Eq. (3.31) reveals that the frequency of the e.m.f. generated in th ^
winding is am rad/sec or s f Hz. It is thus seen that the slip s effects both the magnituH r°t0r
frequency of the rotor e.m.f. The frequency of the rotor e.m.f. sf, is called the slin ^
Hz. p frequency in

If the rotor is made to revolve in a direction opposite to the rotating flux wave th
relative velocity between the rotor winding and the rotating flux wave becomes (to + oi) * j
e.m.f. induced in one phase of the rotor is given by [from Eq. (3.14)]. r 811
e = (o) + <or) Nphr kw <>sin (co + cor)t
Now co+a)r = a) + C D ( l - s ) = o ) ( 2 - s ) - • a)
Eq. (3.33) becomes
e = a) (2 - s) N Ar <j>sin (2 - s) atf
t —\o.6i)
its r.m.s. value is
E = y l2 n f( 2 -s )N phrkw$ ^
Eq- (3-34) indicates that the frequency of the e.m.f. generated in the rotor cirri.;*;
f ( 2 - s) Hz. It can thus be concluded that a polyphase induction motor of the slip-rine or vimmA
rotor type can be used as * frequency converter (or changer) for changing the supply freaurnrv
f to other frequencies s f and (2 - s) f at the slip-ring terminals.
3.4.4. G enerated e m.f. in d.c. m achines. Before deriving the e.m.f. equation for a d c
machine, it is worthwhile to know the difference in the construction and operation of an ac

S K £ £ iT ? iE s T t o f a n 3 C g“ 311113 d c - "

Active length
Fixed Fixed brushes
brush*

1 N- * r I i -
( X i __ ib t u r n __
coil
1 ( 1
' • $ r
slip rings ■Copper
segment
(a)
(Ib)
Fig. 3.16. Elementary forms o f (a) a.c. generator and (ft) d.c. generator.

ends o f t h e ^ c ^ * two'p° 1® alternator, with single AT-tum full-pitched coil. The two
Two carbon brushes staF 0nne? two S^P r*nSs mounted on, but insulated from the shaft

r a b on th e s e r o t a t , n g « * * * ° r s lip * * * • for

rmg^^sphet ^ o 1^ o Cn n ^ ne, “ Pitched * shown in Fig. 3.16 (b). One copper


which these are mouhtpd Th ’ h j Fe insu^ated from each other and also from the shaft o
on which two stationary p h u cod are connected to these two copper segmen >

S can n ed by C a m S c a n n e r
Art- 3-41
— C° " » p l s °r Rolating B e d r i d Machines
Note that alternating e m n C --------------
fig. 3.16 (a), it is directly collected'C t h e l f i n r i ^ “ 2 ° f b°*h F,« 3 1 6 <a >*"<< (A). In
UK external c.rcmt. In Pig. 3 .16 (b) > “ »''P ngs and brushes in a.c. form and delivered to
and • then collected by the fixed carton b r u s h ^ f o r Z “ h ,'" d'C' by means ° f 1* “ s t a t o r
For understanding the process o f r onward transmission to load circuit,
view of the coil of Fig. 3 .16 (6), seen from^ Catl0n by means of commutator, consider end-
Fig. 3 1 7 ^>- The variation of field flux density^™ Slde‘ Thi® end view is illustrated in
3.17 (a). For the present, only fundamental s W " r g*P PeriPhery * depicted in Fig
ental sine component of the flux-density wave is con
D

. (/)
Fig. 3.17. Rectification o f a.c. to d.c. by means o f commutator

S c an n ed by C a m S c a n n e r
____________________ IArt. 3.4
25# Kln'tricul M a c h in e r y -------------------------------------------- ' ~ ~
-------------------------- , , a r inrkwise speed of tor rad/second. For the position
sidered. The rotor is assumed to revo vc • ^ ^ ,g gince the instantaneous
of the coil shown in Fig. 3.17 (b ) the e.mX in ^ ^ ^ q{ flux cutting is zero. After 90*
velocity of the coil is parallel to the magne jg maximum, since each coil side lies
travel of the coil, the e.m.f. induced l&iven y' q-■ ^ utting is maximum. The direction of
in the peak of the flux density wave and the rate .g indicated by dot and cross respec-
e.ni.f. in coil-sides a, a', as determined y rig connected via the commutator segments, to
lively. Fig. 3.17 (c). Further, co. -sides a andla are co ^ ]oad current comes
brushes B x and B respectivey. Z Z ™ ^ B ' and *>
from brush B, and enters a b r u s h ^ ^ ^ ^ (<f) ^ ^
positive and negative polarities r p the e m f induced in the coil is maximum, Fig.
induced is zero and after a Ibrthe,■travel o fM , theeap.b ^ contact with coil
8.17 M , but of reversed po anty. Note h a t n Fig. 3.17 M ^ ^
side a which is under south pole and in t i g . J . u iet, . ... .,
coil-side o' under the south pole. In order words, brush B, is always in contact with that coil-
side which is under the south pole and brush B2is alwa
comes under the north pole. Consequently the polarities of the brushes B , and B 2 remain posi­
tive and negative respectively. The e.m.f. induced in the coil is alternating but the voltage
available across the brushes B, and B i2s unidirectional, as depicte
The e.m.f. ec induced in one coil at any time t, as given by Eq. (3.14), is
ec = N co, 4>sin a\i
The voltage appearing at the brushes is unidirectional as depicted in lower part of Fig. 3.17
(/) and its average or d.c. value is
1 f"
E = - I No),. <J>sin a\t d(u>,t)
71 J0

= - N o ) r <b -< 3-36)


71

Here ior is armature speed in electrical radians per second.


Pn
(i)r = 2 n f. = 2n — = kP h .
2
Eq. (3.36) can be re-written as
2
Ec = - N (nPn )

= 2 N Pn <(> - ( 3-37)
If z is the total number of conductors in one coil of N-turns, then

Note that one turn is made up of two conductors.


Substituting the value of N, Eq. (3.37) becomes ^
Ec = P n * z ...(3-381

E.m.f. per turn, from Eq. (3.37)

= ^ (2 N P n® \
.(3.39)
= 2Pn§

S can n e3D ^ P am 5can n ^


Art. 3.4]
------------------------- virvuidl
verage e.m.f. per conductor, from Eq. (3,38) or from Eq. (3.37)

E* = 2 (Pn<te)
P • , = Pn<t>
For a single coil of Af-turns o r n f , j ...(3.40)
the e.m.f. available across the tWo b l u s h e " ' A c t a ^ ' 3 '38’ ^ the aver^ e ° r d c value of
coils uniformly distributed at the a r m a t u r e W
' i ndi ng consist
appearing across the armature ternrinah, ! ^ r ^ thls has the effect of increasing the voltaee
(3.38) can be used to determine the terminal v o lt a v !T "g the " Pple “ the output voltage. Eq.
now replaced by the number of series c T n ^ L l t g^ ° T a practlcal d.c. machine, provided z is
of armature conductors arranged in a parallel p a th ^ th " brUSheS I tZ is the total number
are « . Thus the voltage E*aooea k Ser“ S COndUCt0rS betW
n oltage E appearing between the brushes, from Eq. (3.40), is

g emf per conductor) (number of conductors in each parallel path)

jZ n P

01 E , = Kr\n
where constant g _ ?F
c a
Now speed in mech. rad/sec. *om = 2nn.
Substitution of the value of n in Eq. (3.41) gives

F _ * Z P (om ZP
a ---------- - 7r—= —----<t) Q)„
a 2n 2 na m

where constant K =
° 2710

Constants Ke and Ka depend upon the armature winding design.

I, ' tab° ald be --emembered that for d.c. machines, the armature winding is always a double
y winding and is of two types, namely lap-connected type and wave-connected type.
^ n lap winding, the number of parallel paths a is equal to the number of field poles, i.e.

fielr/U T aVG windinS, th e number of parallel paths o is always 2 irrespective of the number of
Poles, i.e. a - 2 whatever the number of field poles may be.

for h!!flhj Uld be n0ted that Eq‘ (3 1 4 ) forms the startinS P°int for obtaining the e.m.f. expression
ocn d.c. and a.c. machines.
deQ voltage available across the brushes is unidirectional. For the same value of peak flux
du. S’ y 5 P> It is easy to see that average value of brush voltage would be more for a flat-topped
becg sity wave of Fi&- 3 1 7 than for a sinusoidal flux density wave of Fig. 3.13 (c). It is
ase of this reason that dc machines are designed with flat-topped B-wave.
Xamp^e 3 *2 * A. dc gen era tor has 24 arm ature conductors. Average e.m.f. induced in one
>fthisCtor is 2 V an d each conductor is designed to handle a current o f 5 A. Calculate the rating
generator i f num ber o f pa rallel paths in this machine are (a) 2 (b) 4 and (c) 6.

^ Example 3.2.

cann ed by C a m S c a n n e r
w *1

260 tb ie c in c a i M
Electrical i vachinery
ia c n m e r y _____________________ (An. 3.4
;

Solution. The arrangement of 24 conductors for parallel paths 2 4 and 6 is illustrated i


Fig. 3.18 (o), (6 ) and (c) respectively. Here each conductor is treated l,ke a 2 -V cell with curren
carrying capacity of 5 A.

U '5 *4- 20A la a 5 «6 -3QA


l a «5*2*10A — ?+ -? +
' + I
I
L i n ,± H i l l
12 12
r y t i T' (T T1 I1 T1 T!
' '< a*1 ™
A** A **
<i Ea- 12A2 «ri S*
< i <1 <* Ea*6«2 i<
▼ vo i ! i ‘ to
! ' ;8 V
<*
vn? “ »? -24V */>)“’ i tn,i m.■
<♦ 12V
,1 i i 1 1 1
2 -I- - 1-2 i 2i | |
1 1 1 1 l l l l l l
I 1 ’ T T TyT T .T
V

(c)
(6 )
Fig. 3.18. Pertaining to Exsn.pl. 3.2. (a. Twcparali.l paths (6. Fonr-par.Ue, paths <e, Six-parai.e. paths.

. 10f - — 1 series-connected conductors in each


(а) For two-parallel paths, there are 12 - 0 - 2 J senes c°

poi
p ath . o u C t — - — Ik
x (Average e.m.f. per conductor)
= 12 x 2 = 24 V
KC
Output current, / . = (Current per path) x (Number of parallel paths)
= 5 x 2 = 10 A
,. 7 = 24 x 10 = 240 W
-_ Ejya la * !he
.% Power rating *
(б ) For four-parallel paths, there are 6 [ = f Conductors in each parallel path.
V /
Output voltage = 6 x 2 = 12 V
Output current = 5 x 4 - 20 A
Power rating = 1 2 x 2 0 = 240W
(c) For 6 -parallel paths, Ea = 4 x 2 = 8 , Ia - 5 x 6 - 30 A,
power rating = Ea Ia = 8 x 30 = 240 W . on

This example demonstrates that (i )the power rating of a d


its n u m b e r of parallel paths and (» ) the voltage between output brushes (a
e m.f. per conductor) x (conductors per parallel path). ^ per
* p I U 4-pole machine has 60 slots and 8 conductors p er slot. I m>
F or arelativespeed o f 1500 rbetween field
C alculateth^generated artnature voltage if ,h e m achine ,s
(a) a d.c. machine with lap-connected windingand
(b) a three-phase star-connected m achine with winding factor equ
each phase arc in series. _ p _ 4

S o lu tio n , (a) For lap-connected winding, a - P - •

S c an n ed by C a m S c a n n e r
Art. 3.4]
Basic Concepts o f Rotating Electrical Machines

Total number of conductors Z = 60 x 8 = 480


Speed in r.p.s. _ 1500
= 25.
60
From Eq. (3.41), the generated e.m.f.

E - §ZnP (20 x IQ"3) (480) (25) (41


a 4 “ 240 volts.

(6) Total number of turns 480


= 240.

Number of series turns per phase,

p n n r

Generated e.m.f. frequency, f = — = 4 x =£- = 50 H z..


z z
From Eq. (3.24), the generated e.m.f. per phase is
Eph = V2 n f N ph kw <(>
= V27t (50) (80) (0.96) (20 x 10“ 3) = 341 volts.
Generated e.m.f. between line terminals
= V3 Eph = V3 (341) = 591 volts.
Example 3.4. A 4-pole, 3-phase, slip-ring induction motor is coupled mechanically with a 6
pole synchronous motor. The induction motor stator and synchronous motor are energised from
the same 50 Hz voltage source.
(a) What will be the frequency o f the e.m.fs. at the rotor slip rings if the synchronous motor
is driven (i) in a direction opposite to the rotating field produced by the induction motor stator
(ii) in the direction o f the induction motor rotating field.
(b) I f the frequency o f the 3-phase rotor terminal voltage is required to be 150 Hz, calculate
the number o f poles that the induction motor must have.
(c) I f the induction motor is assumed to have 8 poles, calculate the number o f synchronous
motor poles, for obtaining 3-phase, 150 Hz voltages at the induction motor slip rings.

Solution, (a) Synchronous motor runs always at synchronous speed o f—^ r.p.m.

•*. Synchronous-motor speed = - ^ = 10^0 r-P-m-

Speed of the rotating field, produced by induction motor stator


_ 120/; _ (120) (50) _ 150Q m
" p ' 4
G) When the induction motor rotor is driven in a direction opposite to the rotating field, the
roUi- , . inaucuu 3 _ rotor conductors and its rotating field is
relative velocity between induction motor rotor wimubiu 0
(1500 + 1000) = 2500 r.p.m.
••• Frequency of the e.m.f. at rotor slip ring terminals
(Rotor poles) (Relative velocity in r.p.m.)

_ (4) (25001 _ 83i Hz


" 120 3
. Note that the rotor poles, equal to the stator poles, are 4.

S c an n ed by C a m S c a n n e r
I .

262 Finrtrical Machiner) T T T ^ T t h T ^ c t i o n of the rotating field, then


1 (if) When the induction motor rotor »
r e liv e velocity is M iooo) = go0r p m.

, Frequency of the e.m.f. ^

' 12r i to Hs at induction-motor rotor terminals, the rotating


» , For obtaining a ^ " S o n s . 8
field and rotor must run 1 PP ^ rotating ficld
it • The relative speed bctW=e^ otor Speed + Rotating Field Speed]

120 x 50 n
1000 + — ^ •
r im

where P = number of poles on the induction mot


Frequency of the e.m.f. at rotor terminals is gtven by
" 6000n
im
1000 +
im
= 150
120
or Pirn = 12 P°leS-
(120) (50) t (120) (50]
(c) Relative speed between rotor and rotating field must be equal to " 8

Here Psnl is the number of poles on the synchronous motor.


Frequency of the rotor e.m.f. is
'iO
8 120 x + 750
* sm
= 150
120
P sm = 4 P°leS-
Example 3.5. A 3-phase, 4 pole, 50 Hz slip-ring induction m otor has 420 stator turns and
240 rotor turns. The magnitude o f the rotating flux p er pole is 30 m Wb. The winding factors for
both stator and rotor windings are 0.96. Calculate frequency and the magnitude of per phase
e.m.fs. in the stator and rotor windings when the rotor is
(a) stationary
(b) revolving in the direction o f rotating flux wave at a speed o f 1440 r.p.m. and
(c) revolving opposite to the direction o f rotating flu x wave at a speed o f 1440 r.p.m.
Solution, (a) Per phase e.m.f. induced in the stator winding is given by Eq. (3.26).
.'. Stator per phase e.m.f.,

B, = 'l2 K fk wNph,^,

= V2tt (50) (0.96) j (30 x 10"3)

= 310 volts at 50 Hz
From Eq. (3.32). the rotor per phase e.m.f. is
Er = V2n s f kw Nphr if

S can n ea by u a m b c a n n e r
Art. 3 ^
Basic Concepts of Rotating Electrical Machines 263

At standstill, slip, s = — ~ °
= 1
n.

E, - ^2 71(50 x 1) (0.96) (M g ' (30 x 10-3}


_ q in 80 ' '
140 = 177 volts at 50 ( = sf) Hz.
(6) Stator per phase e.m.f. is uneflected with the rotation of the rotor.
E* = 310 volts at 50 Hz.
The speed at which rotating flux wave travels is given by
AT 120 x 50
s p r.p .m .- - = 1500 r.p.m.

For rotor speed of 1440 r.p.m. in the direction of rotating flux wave, slip
1500 -1 4 4 0
1500 “ ° -04-
From Eq. (3.32), the rotor per phase e.m.f. is

Er = V27i (0.04) (50) (0.96) (80) (30 x 10" 3) = 0.04 x 177


= 7.08 volts at a frequency of 0.04 x 50 (= sf) i.e. 2 Hz.
(c) When the rotor revolves at 1440 r.p.m. opposite to the direction of rotating flux wave
sii 1500+ 1440
1500
From Eq. (3.32), Er = V2ti s f k w Nphr <\>

= V2rc (1.96) (50) (0.96) (80) (30 x 1(T3) = 1.96 x 177


= 346.92 volts at a frequency of 1.96 x 50, i.e. 98 Hz.
3.5. E.m.f. Polygon

This article develops the physical concepts of e.m.f. polygon and these concepts are then
extended to derive the expressions for distribution and pitch factors.
Consider Fig. 3.19 (a) in which 12 conductors housed in 12 slots are shown. The field cur­
rent produces two poles on the rotor and the field flux is assumed to be sine-distributed in
space. At the instant shown in Fig. 3.19 (a), the conductor 1 is being cut by the maximum flux,
Say ♦«» therefore, the e.m.f. induced in it is also maximum, say Em. This e.m.f. can be repre­
sented by a phasor whose projection on the vertical axis must be equal to the r.m.s. value of the
em.f. generated in conductor 1. This is shown in Fig. 3.19 (a) by a phasor E x = E„/\l2 along the
vertical axis. Conductor 2 is being cut by a flux ( = <}>m cos y) less than <J)m, accordingly the mag-
^ude of generated e.m.f. is less than E m. This e.m.f. in conductor 2 is indicated by phasor
2*7 . ' whose vertical component E 2 cos y is equal to the r.m.s. generated e.m.f. in conductor
^ ote that E 2 is displaced from E } by an angle y , equal to the angle between adjacent slots.
8 an^ e Y (gamma) is usually called the angular slot pitch and is given by

v = —— -------^ E l e c t , radians
Total number of slots

- Total n m n bcrofsiots ElecL dcgrCCS'

_ —— — - — — Elect, degrees.
Slots per pole

S c an n ed by C a m S c a n n e r
[Art 3.5
Electrical Machinery

(b)

F ig . 3 . 1 9 . ' I y r ! , ? w l S d ^ o f S e c t o r e.m.fs o r star o f slot e.m.fs.


The various phasor e.m.fs JEi, * 2, *3 .......*12 *
r Via q 19 (a) Y is equal to 30°. The e.m.f. generated in conductor 3 can similarly be re­
presen tedinFig. 3.19 (a) by phasor E 3 ( = E .) whose vertical component E cos 2 y is equaHo
the r m s generated e.m.f. in conductor 3. Note that E , is drsplaced from E , by 2y and from E,
by y At the instant shown in Fig. 3.19 (a), it may be seen that conductor 4 rs not cuttmg any
magnetic flux ; therefore, e.m.f. phasor E 4 ( = E ,) m ust have a zero vertical project,on
” E cos 3v - E cos 90°). The same thought process can further be used for conductor e.m.fs.
£~ e I E,o . Note that all phasors E „ E 2,...... E 12 are of equal lengths. Their projections on
the vertical axis, determine the magnitude of r.m.s. generated e.m.fs. in the vanous conductors.
In Fig. 3 .1 9 ( 6 ) ,£ ,( = AB) is drawn parallel to E , of Fig. 3.19 (o). At the point = E2is
drawn parallel to E 2 of Fig. 3 .1 9 (a) and so on. It will be found that conductor e.m.fs.
E £, E 12 constitute a closed e.m.f. polygon. The perpendicular bisectors of each of the
conductor e.m.fs. E „ E 2 £ 12 (i.e. A B , BC, CD etc.) will pass through a point O, indicatmg
that if a circle with O as center and OA as radius is drawn, it m ust pass through the points
A, B, C, D etc. The angle subtended by each phasor e.m.f. at the center O is seen to be equal to
the slot-angular pitch y.
Starting from the front end of conductor 1, join the back ends of conductors 1 and 2 in the
Fig. 3.19 (a). Further join conductors 2 and 3 by a connection at the front and 3 and 4 at the
back and so on. The resultant e.m.f. appearing between starting end of conductor 1 ana.the
finishing end of conductor 12 (i.e. after having gone through all the conductors once), will e
found to be zero. This is evident from Fig. 3.19 (6) by the phasor addition of e.m.f. phasors
^ 1 . &2> »^12-
3.5.1. Distribution, breadth or belt Factor. The e.m.f. generated in a full-pitched
mature coil of iV-turns is given by Eq. (3.14). In rotating electrical machines, however, J
armature winding is not concentrated in a single slot, but distributed uniformly along e
gap periphery. The advantages of distributing the winding in slots are :
(i) reduction of the harmonics in the generated e.m.f. wave, thus making it approach a sin
wave, see Prob. 3.30 (a).
(ii) full utilisation of the armature iron and copper and

(iii) adding rigidity and mechanical strength to the winding.

S c an n ed by C a m S c a n n e r
Art. 3.5]____________________________
Basic Concepts o f Rotating Electrical Machines 265

assumed to have^8 slot^In Hms^sfots1 f Stribution factor- the stator of Fi&- 3 1 9 (a) is
s, a single layer, 2-pole, 3-phase winding is placed, so that
the number of slots per pole oer dHaqp n o 1 . f 2 x 1 80 >
f p e per pnase qis 3 and the angular slot pitch y is 20° = . * . Its
developed view is illustrated in Fie 3 2 n r M a„ d M l „ , I J
under one pole, are connected in series In nrd ♦ 7 belonging to one phase and
distributing the winding, full-pitched coils are used attentlon entire‘y the effects of

Coil span = dumber of slots _ 18


.. . Number of poles 2

c■ oa ,,!,d :?• a: d,x


.. : : t f xL 7 r e 1mt‘ 'n£i^
l* th! i 1coui sidte r b„ T r . 10 ( = 1 + ^ ^
10 (similar to E ^ and E 7 of Fig, 3.19 (a)] induced
in them, since these are under tha nnlo „ j ... .
instant shown. The direction nfmtnr f *• ■ are cuttm£ ^ e maximum pole flux at the
f j • • • , otor rotation is assumed clockwise The coil-sides 1 and 10 am
connected in series in such a manner that thmr o ™ jj f , es 1 and 10 are
Fig. 3.20 (d ) by a 6 ( = E, + E 1 alnna th C ? e.m.fs. are added up ; this being indicated in

generated in it"then coil-side’ 1 0 ^ 5 4 vd* « ’ T r '^ . T ' f ** 1 ^ ^


though of opposite polarity. The
„ r Fundamental comp,
o f 0 wave

Third harmonic
comp, of B -w ave

271 <t( fu n d a m e n ta l)
II
ITT *■( 3rd h a rm o n ic ) r

1 2 3 10 11 12
j i m : ; : ...

! (

Fig. 3.20. Illustrating the derivation of distribution factor.

cann ed by C a m S c a n n e r
[Art. 3.5
7nh
266 Liecincai
Electrical w
Machinery
w v h _ __^ ^ _
• 1 r to £ and Es of Fig. 3.19 (a). The total
generated in coil-sides 2 and 11 are £ 2 and E „ , sum a .ndicated in Fjg 3 20 (<j)
e.m.f. in coil 2-11 is given by the sum of E2 an »’ s|ot-angular pitch y. The vertical
by b e , phase displaced from the niaximum e m.f p ted jn coi) 2- l l . Similarly the total
projection of b e gives the r.m.s value of « * < * ■ 8 2Q (rf) by cdi phase displaced from
e m f (£3 + £ , 2) generated in coil
tj.iii.i. \*-/3 1 Yc,t o ------------------- 3-12, is indicated in
_ r g
the maximum e.m.f. position by an angle 2y. of the three (or in general q) coils
The resultant e.m.f. appearing between the te n bc and cd and this resultant is
1-10 2-11 and 3-12 is the phasor sum ot tne e.n__ v denotes the slots per pole
seen to be equal to ad in the right hand side of Fig. 3.2U t ;
per phase. r Une.nr.anh he and cd must meet at a point o and
As indicated earlier, the right bisect.Dr: ofpha ^ draw perpedicuiars oe and 0f
the points, a, b, c, d would lie on a circle with ^ angle aob is equal to
to ab
to ana ad
ab and aa respectively. From Fig. ° d .w w , 1
the slot angular pitch 7, angle a o d = 97 and angle W = 2 -

In the right-angled triangle aoe.


y_ _ ae_
sin ± =
oa

ae = oa sin
or

... E .m .f . per coil = 06 = 6c = cd = 2(ae) = 2 oa sin

The arithmetic sum of the coil e.m.fs.


= q (e.m.f. per coil) = q c< oa sin
V
In the right-angled triangle aof,
sin 9„ l -=° L
oa

a f = oa sin 91
or
The resultant e.m.f. ad, equal to the phasor sum of emfs ab, bc and cd, is given by
91
ad = 2 (a/) = 2 oa sin

The distribution, breadth or belt, factor is defined as the ratio of the phasor sum
tl
j.m.fs. to the arithmetic sum of the coil e.m.fs. Its symbol is kd. ^
P
The distribution factor can alternatively be defined as the ratio of: a
:oils distributed in slots to the resultant e.m.f. with the coils concentrated in • h
Phasor sum of coil e.m.fs. ad nr ad _ general)
Arithmetic sum of coil e.m.fs. 3 ab q •ab
„ . qy
2oa sm ^

2g oa sin

sin 91 ...(3.46)

q sm

S c an n ed by C a m S c a n n e r
'I

A rt. 3.51___________

T h o u g h E q ~ ( 3 4 6 ) is d e r i ggf!£_ Concepts o f Rotating Electrical Machines 267

to double layer windings aTso.tw ch SimP' idty' “ applias d u ally well


Distribution factor for B(| . ‘ a r g e r machi" « -
gap periphery of rotating ° f lux-density wave along the air-
such non-sinusoidal flux density d ia tX m "n ‘ n h. " Fourier sariaa a"a ly S1s of
y isirioution can be expressed as a harmonic series

The space-field har “ + ®3 3“ + ' + S " ai" " “


the generated e.m.f. waviTout of thevarious'hf ^™ ‘ 0.uhe corresPondinB time harmonics in
so here the effect of this harmonic is InW sb„ . ° T ’ thlr<* harmomc is the most dominant,
rnomc flux density waves are shown A third hiTrm E 3 20 (a)' fundamental and third har-
be imagined as produced by 3 poles a s I I ! compon<;nt of the flux density wave, may
this is illustrated in Fig. 3.20 (a) In view of th' f° r the fundamental component-
scale, would become 3y for the third harmoiic slot' anP |lar pitch yon the fundamental
component. Thus the distributun
tioniactor lor the nth harmonic, from Eq.nZ^(3.46)
factorforT ^{° ' the
is nth h3™ 0™

s in
k d„ = ~ 2
q sin ^ - (3-47 o)

Examples will show that k w is less than b tkio u tL «•


monic e.m.f. in comparison with funds. ti m. as the effect of reducing the nth har­
ing the windings in slots. undamental e.m.f. This is certainly an advantage of distribut-

tendY!o“ e U rg e t S s t n C t m l farf“ ^ “ oach^ “ * e' eCtriCal <<>r S’ ° tS P" P° ' e per phase

sin 2„1
kd
h
= ---------- 2

sin ^
2 Q t
and k, --------± _
dn qnj ..(3.47 b)
2

thJ ? h Fig h 20 <6) ° r <C>' i( adj ° inil’ e coil-sides 1, 2, 3 (or 1 0 ,1 1 ,12) belong to the same phase
hen this phase group in adjacent slots is called a phase belt or phase band. If slots per pole oer
phase is a whole number, then all the phase belts are alike. The peripheral angular d is ta n t of
a phase belt is called Its ph ase spread o( = qy) in electrical space degrees. Now Eq. (3.46) can be
re-written to include phase spread o as

a
sin­

-------------- a
q sin£

. no
sin —
From Eq. (3.47 a), kdn = (3 48
? s m - -

ca n n ed by C a m S c a n n e r
(Art. 33

Flectrical Machinet7_

sm -

p or y < 15° e le c tr ic a l, kd = o
2
nO
s in - ...(3.48 b)

and from Eq. (3.47 b), kdn ^

: ; S T r ^ s ,~ « ^ x r : » . : i v
per pole per phase tends to be larg; , ^ ^ The phagor
tends to be small for a ^ c n p drcle whose
diagram of coil emfs now b « o » « the ar ^ The dis.
chord ab is the resultant emt as s
tribution factor is there ore ^
nhasor sum ofkoiUmis__ --------- —
^ = arithmetic sum ot c o d e i a arc Fig. 3 21. Coil emfs for uniformly
2 oa sin a /2 _ distributed winding.

i- . « s - r — ■«— « —
I„ fractional-slot windings (dtscusscd in Chapter .
5 o *m

, . Here Sis^the number of slots and m, the number of phases. If b is the


is not a whole number. 1kre be writtcn as
highest common factor between S /m and P, then q
S /m = -*

The distribution factor for fractional-slot windings is obtained by replacing by S„ » H

(3.48 a).
no
sin ...(3.48 c)

^dn no
Sk ol“
sin 9 o
* , . • „ O nhase machine, the
One pole pitch is equivalent to 180 electrical space d egreesan d in a p ln other words,
winding is so distributed as to give a phase spreadof60 e ectnca space degr . g
each phase group or each phase belt occupies 60°( = electrical, under^each p
machine and 90° ( = a) electrical under each pole for a 2-phase machin . o statoT
Example 3.6. Calculate the fundamental, third and fifth harmonic e f
having
(a) 54 slots, 3-phase, 6 poles
(b) 48 slots, 3-phase, 6 poles with narrow spread winding.

Solution, (a) Slots per pole per phase, q = =3

, P x 180 6 x 1 8 0
Slot angular pitch, Y= g = ^ - 20

DyTTam Scanner
Art. 3.5]
Basic Concepts o f Rotating Electrical Machines 269

sinn x 3x 20
From Eq. (3.47 a),
n x 20'
3 sin

. _ sin 30° _
dl 3 sin 10° ~ 0 9598
= sin 90°
d3 3 sin 30° " 0,6667
sin 150°
d5 = 3 ^ 5 ^ = 0 2 1 7 6
(6) Here 48 = 16 _ 8 _ S k
3x6 6 ~ 3 ~K
n x 60
sin
From Eq. (3.48 c),
n x 60
8 sin
2x8
, _ sin 30°
dl 8 sin 3.75° “ 0 9556
sin 90°
kd3 ~ = 0.6407
8 sin 11.25°
sin 150°
kd 5 = = 0.1944
8 sin 18.75°
This example demonstrates that kd5 < kd3 < k(di-
^.

3.5.2. Pitch (or coil-span) Factor. An expression for the pitch factor has already been
derived in Eq. (3.20). An alternative way of obtaining Eq. (3.20) is presented here, for it leads
to better physical concepts of pitch-factor.

Ic '-C
fr jE a -E

Go oh
Ea=E
(b) (c)
Fig. 3.22. E.m.f. induced in one full-pitched coil.

S c an n ed by C a m S c a n n e r
(Art. 3.5
270 Electrical Machinery

Fig. 3.22 (a, illustrates one full-piteh coil and Fig. 3 .^ i l l u s t r a t e s one fractions,.pitch.
short-pitch or chorded coil with c h o r d m g ang e eq maximum flux, these have, therefore,

m axta^m e.imTs^’^ V ^ 'l n d u c e d in thenTat the instant shown. The r.m.s. values of the e.m.fs.

, p Em . E ,= = E respectively. These e.m.fs. Ea and E0\


in coil-sides a and a are Ea = = * ana V2 ,
Jraut/n in Fie. 3 22 (c) on a larger scale for clarity,
though already shown in Fig^ 3.22 (a),.are - whgre E (^= E E^ ig the f m g
The resultant voltage ER between the coil ends u , n is ,

Va'T n F L hV 2T ( t ) " l - s i r e t “ cuttmeg the maximum flux * it has. therefore, maximum


In Fig. 3.23 (o) the c coil-side a' is cutting a flux of ♦„ cos £ and as a result of it,
e.m.f. E generated in i t m s valucs of the e.m.fs. in coil-sides o and a' are
the e.m.f. generated in it is Em cos t. in .
OR - E - - r 5- = E and OA = cos e = cos e = E cos s. E.m.fs. £ a and Ea' are shown in Fig.
? A in Via 3 23 (c) on a larger scale. The resultant e.m.f. ER across the
3.23 (a) and are re-drawn in Fig. icj on a w b
coil-ends G, H is obtained by reversing £ / and adding it to i.e.,
ER —£ a —£a
F lu x w a v e

jE a'cos C

Ata
G H
(c)
(b)
Fig. 3.23. Pertaining to pitch-factor.

In Fig. 3.23 (c), reversed Ea' is drawn as BC so that E R = OC. In order to obta’?
nitude of the resultant voltage ER, draw BD perpen^cular to OC. Now in triang e -

ZO B C = 1 8 0 - £

ZO B D = | [ZOBC 1 = 90 - 1

Since ZBDO - 90°


ZDOB = e / 2 = ZBCD

S c an n ed by C a m S c a n n e r
Art. 3.5] .
— Basic Concepts of Rotating Electrical Machines 271
The resultant voltage E „ from Fig. 3.23 (c) is given by ‘
E r = OC = OD + DC

= OB cos | + CB cos -
2
= 2 E cos ^
. 2
The pitch factor is defined tV»o »on <•
e.m.f. had the same coil been full nitrh ri° ° r®s^ tant e m f. of a chorded coil to the resultant
^ “ -Pitched, symbol kn. Thus

kn = J jg s u ltant e.m.f. of a chordpd coil 2E cos 2


Resultant e.m .f of a full-pitched coil = ~ 2 E
e
= cos­
. 2 ...(3.20)

chording angk becomts n Z e t o r i c a t S ^ e t f o r e ^ ' t T f l ^ r f o r X h a ™ 2 c " *


. l ne
pn — COS
rnu J 2 ...(3,49)
The advantages of using fractional-pitch winding are •

!‘J Z dZ T 'Vhh6 C° PPer f0r ° Verhang- This results in leas of the machine \
wJ e almost a ^ e w a ™ 0niCS " ^ thareb* -n d e n n g the output e.m.f'

more copper is essential. Since the advantage of reducing the therefore,

s K - a s s ja r r 5
as t h !S g fa c t" S i r °f diStribUU°n kd“ d the Pitch f a t o ia referred to

~ X ^p (3 50)
In case of three-phase alternators, the third harmonic e.m.f. can be suppressed by star or
the a ' on" echon’ as ln ‘ he case of 3-phase transformers. Hence at the time of winding design
; r " ' h nT ” n yf ' T n ar thea“ enUatl° n of 5th and 7th orderharmonics, by I d o p W
a suitable chording angle. A chording angle of e = 30° (coil pitch = 150“ electrical) is most useful
since it gives the following pitch factors : ’

9th ha9r6monicfUndamenta1, ^ ^ harm° niC’ 0 259 f° r 5th and 7th harmonics, 0.707 for

Three-phase alternators are invariably star-connected to eliminate 3rd and 9th (i e multi
P es of 3, called triplen) harmonics from line e.m.fs. ; though 5th and 7th harmonic e m fs of
educed magnitude are present in the lines. ^ 01
From Eq. (3.24), the fundamental e.m.f. per phase is
Ephl = < 2 n fk w lNph 0!
For third harmonic, e.m.f. per phase is
Eph3 - (3f) kw3 NphiJ)3
In genera], for nth harmonic, e.m.f. per phase is
Ephn = ^2n n f k wn Nph <J>n

Here subscripts 1, 3 and n denote fundamental, third and nth harmonics respectively.

S c an n ed by C a m S c a n n e r
r 272 Electrical M achinery
lArt 33

.. „ th harmonic and fundamental r.m.s. phase e.m.fs. are


From Ephn and £ „ * , expressions, the nt
related as
ntyn <3.51 a)
Ephn _
)hl kw\ <h
Ephl Jomental flux per pole
Also, 01 - ^AveraVgeUneu0x density) (Area under one pole)

I Peak value o f fundamental component of flux


where density wave
D = Air-gap diameter of armature
L = Core length of armature
and
2DL
B
Similarly, 4>3 — ' 3P
2DL n
In general, for nth harmonic, «>„ * ■ * *

1
On = _____
From above, <j)j n Bi
Eq. (3.51 a) in terms of B v B n becomes
...(3.51
Ephn _ ^wn B. b)

Eip/,1
r
The r.m.s. phase e.m.f. is
Eph = lE2
phi + E2
ph3 + ~~ + Ephnl
The line e.m.f. in star connection does not include triplen harmonics.

sectfon^th^methods°of efiminating'or sup™ ^ sir^ h a rm o ^ c s'fn rm ^ re 'e n if waveform of an

alternator are discussed briefly. :D,i„0tnthe


In alternators, the primary source, or cause, of harmonics in the there would
non-sinusoidal field flux waveform. In case the field flux^^vrforoi ^ haye almost
be no harmonics in the alternator generated emf. Thus, first an attempt is fw..,ef„rm
sinusoidal field flux wave and then the means to reduce the harmonics from the emi
should be adopted. . othnds:
Field flux waveform can be made as much sinusoidal as possible by the following ^
(i) Small air gap at the pole centre and large air gap towards the pole en s in
synchronous machines tends to make the field flux sinusoidal.
(ti) Skew the pole faces if possible. the s]0ts
(m) In turbo-alternators, the air gap is uniform. So field winding is distri u e
in such a manner as to make the field flux waveform almost a sine wave. ,g n0t
In spite of all these attempts, field flux waveform along the air-gap can, b°w*
sinusoidal. As a result, harmonic emfs are always generated in an alternator,
ever, be eliminated or suppressed as follows :

S c a n n e d ’F y C a m S c a n n e r
Basic Concepts of Rotating Electrical Machines 273

, (t'y) Distribution : The distribution of the armature winding along the air-gap periphery
tends to make the emf waveform sinusoidal, see Prob. 3.30.
(a) Chording : With coil-span less than pole pitch, the harmonics can be eliminated, see
gq. (3.49).
(ui) Skewing : By skewing the armature slots, only tooth harmonics or slot harmonics can
be eliminated,
/**»*■----- see
' Example* 3.6 in this chapter and
VW Art
*.Miivi« 5.17.4*
kl VAI•A
(uii) Fractional slot w in d in g : An introduction to fractional-slot windings is given in Art.
7.8 in this book. In fractional-slot windings, the space relation between teeth and slots under a
given pole face is not the same as under the next and the succeeding pole faces. It follows that
the pulsation of flux and the emf produced in the coils under one of the poles are more or less
neutralized by other out of phase pulsations in the remaining coils of the winding.
(viii) Alternator connections : Star or delta connections of alternators suppress triplen
harmonics from appearing across the lines,
see Art 8.4.2.**
Example 3.7. In an alternator, show
that the output e.m.f. wave can’t contain even
harmonics.
Solution. Fig. 3.24 illustrates one full-
pitch coil a, a'. The fundamental component
of the field flux wave induces maximum
e.m.f. in coil-sides a, a , since these are cut­
ting the maximum flux <>ml. If r.m.s. value of
the e.m.f. in each coil-side is E h then the
resultant e.m.f. of the fundamental frequen­
cy across the coil ends A, B is 2E^, as ex­
plained before.
The second harmonic component of field
(lux wave also induces maximum e.m.fs. in
coil-sides a, a' , because these are cutting the
maximum flux <\>m2 at the instant shown.
The direction of the second harmonic e.m.fs.
is indicated by arrows in the lower diagram Fig. 3.24. Pertaining to Example 3.7.
oiriE. m e rresultant
of Fig. 3.24. The of the
e s u iw u n w m o second .har-
— #
monk e.m.f. between coil terminals A .B is seen to be zero, since the two second harmonic
e.m.fs. are opposing each other in the coil circuit.
This shows that the second harmonic e.m.fs. or in general, even harmonic e.m.fs., cannot be
generated in an alternator. Consequently the output e.m.f. wave is free from even harmonic
e.m.fs. ‘
This example demonstrates that even if the field flux wave contains second harmonic com­
ponent, the output voltage if1 free from second harmonic ™ ^

contain J w U harmonics. T h u s no even


harmonks can be generated-hence the output e.m.f. is free from even harmonics.

• ‘Generalized Theory of Electrical Machines; by the same Author


M 'Generalized Theory of Electrical Machines by the same Author.

S c an n ed by C a m S c a n n e r
274 Electrical Machinery

In case even-space-harmonics are present in the field flux wave the positive
a.f cycles will not be identical, which however ,s never the case m rotating o l e c t r i ^ }
Exam ple 3.8. For a 3-phase winding with coil spar,, o f l 6 0 \ determ ine the distrib
nnding factors
winding factors in
in case
case the
the winding
winding has a .ph ase spread o f 120 . Take the three-Pbase
.. r , ,1
(a) to be uniformly distributed and (b) to have 9 slots p e r pole. « Un^
Solution, (a) When the winding is uniformly distributed, the distribution factor*
Eq. (3.48 6) is rffr°'Hi
a • 120
sin — sm 3 sin 60c
ks = = 0.827
a/2 120 n
2 X 180

Chording angle e = 1 8 0 - 160° = 20°

Coil-span factor, kp = cos ^ = cos 10° = 0.9848

/. Winding factor = kd •kp = 0.827 x 0.9848 = 0.8144

180
(6) Slot angular pitch = 20c

Slots per pole per phase =- =3


o
For a phase spread of 120°, 6 adjacent slots [6 x y = 6 x 20 = 120°) must belong to fh.
phase. Therefore, distribution factor kd, from Eq. (3.48 a) is Same

. o 120
sm - sin
______________ sin 60°
= 8.8312
9 s i n ^ ' 6 s i n ^ = 6 s in 10'
2q 2x6
/. Winding factor = 0.8312 x 0.9848 = 0.8186.

Sh° W " la‘ the


How can these higher order harmonics be reducedT frequ en cy components are equal.

Solution. Slot angular pitch v - —

Order of harmonics, n = (2 Q ± l)
Substitution of the values nf v nnw • r-,
(2(? ± l)th harmonics as n m (3 .4 7 a ) gives the distribution factor for

s i n | ( 2 Q ± l,"
qn
qn ±
kd (2Qr 1) = Q 2Q

i si« h (2<? ± i) *
Q q sin
2Q
sin qn cos 2 * + cos qK si„

sin * ~ ± cos x sin "


2Q

cann ed by C a m S c a n n e r
M jjjl Basic Concepts of Rotating Electrical Machines 275

In t^e.a^° ^5 e^ ress*0^ ’ s*n *n numerator and sin x in denominator, are always zero
any value of q (the number of slots per pole per phase). In view of this, the distribution factor
for
becomes

± cos qn sin
± 1) ^
± q cos n sin —

t [ S al.TayS equal 10 ( ■ 11 and that of cos « ” is « I ual to 1 or - l , depending


uponthe alue of q. Thus the magnitude of the distribution factor for (2Q ± l)th order of har­
monic is ^

. qn
l Sm 2 Q
kdi2Q 1 1) -
q Sin 2 Q
For the fundamental component, the distribution factor, from Eq. (3.46) is
. qn
k d1 - Sm2<? h
— ~ Kdl2Q 1 1)
q Sin 2Q
The pitch factor for (2Q ± l)th harmonic, from Eq. (3.49), is
l (2Q ± 1) e
kP (2Q 1 1) “ cos 2

Here Q is the number of slots per pole. As one pole covers an angle of 180° electrical or n
electrical radians, the angle covered by Q slots is n radians. In view of this, expression for
kp (2q ± i)can be written as
l. {2n ± 1) e
*p (2Q11) = COS ------ = COS a e ± f

E, . . £
= COS 71E COS — ± Sin 71 e sin —
m z
For any value of chording angle £ in electrical degrees, sin n e = 0 and cos tie is equal to
1 or - 1. Therefore,

*p(2Q± 1) - cos ^ = kpx

These higher order harmonics of the order of (2 Q ± 1), are referred to as the slot harmonics,
tooth harmonics, slot ripples or tooth ripples. These harmonics are attenuated by skewing the
slots.

Example 3 . 1 0 . A n alternator, with full-pitched uniformly distributed winding, is wound as


foilows :

faJ Single phase winding placed in (i) two-thirds o f the slots and (ii) all the slots.
(b) All the slots are wound with (i) single phase and (ii) 3-phase windings.
fc) All the slots are wound with (i) two-phase and (ii) 3-phase windings.
For each o f the above cases, calculate the ratio o f outputs and the amount o f copper required.

cann ed by C a m S c a n n e r
<V *

■•• a
276 Electrical Machinery

Solution, (a) When two-third of the slots are wound, the phase spread a, for single phase

winding is o = - (180) = 120°. From Eq. (3.48 b ), the distribution factor for the fundamental
3
component is
. 120
sin ——
* ,, = — = 0.827
120 _JL_
2 180
f2 \
Output voltage with two-third of the slots wound is ^ (0.827).
< ) .
When all the slots are wound, phase spread o = 180° and from Eq. (3.48 6 ),
180
sin
K = 1on - = 0.637
dl 180 n
x
2 180
Output voltage with all the slots wound is « (1) (0.637).
In all the cases, it is assumed that frequency f, flux per pole <}) and the conductor cross-sec­
tion are the same.
.. . . . [(Current) (Voltage)] with all the slots wound
.. e ra 10 o ou pu s - Current) (Voltage)] with two-third slots wound

1 x 0.637
2 . . rinn.' —1.15.
I x (0.827)

Corresponding ratio of copper required = 7^ = 1.5.

It is thus seen that for 50% increase in copper, the output is increased by 15% only. This is
the reason why approximately two-thirds of the slots are wound in case of single-phase induc­
tion machines.
(b) For a 3-phase winding, the usual phase spread a is 60° (narrow-spread). .
. 60
sm —
kdi = 6Q = 0.955
x
2 180
When all the slots are wound with single-phase winding, then kdl = 0.637, from part (a).

■ Ratio of outputs = t(Current)(Vo^ age)1 for a 3-phase winding with all slots wound
[(Current)(Voltage)] for a single phase winding with
all slots wound
_ (1) (0.955)
(1) (0.637)

Ratio of copper required = ^ = 1.

Thus for the same amount of copper, the output for a 3-phase winding is 50% more than for
a single-phase winding with all the slots wound.

S c an n ed by C a m S c a n n e r
Basic Concepts o f Rotating Electrical Machines 111

(C) For a two-phase winding, the phase spread a is 90°.


90
srn

^ i o ^ r - 0 -90
x
2 180
Distribution factor for 3-phase winding, as calculated in part (6) is 0.955.

Ratio of output = Q-utPut for a 3-phase winding _ (1) (0.955)


Output for a 2-phase winding (1) (0.90) ‘ ‘
Ratio of copper required = y = 1.

Example 3.11. (a) A 3-phase, 6 pole, 1000 r.p.m. alternator has an air-gap diameter o f 28 cm
and a core length 23 cm. A two-layer narrow-spread (o = 60°) winding is employed and the wind­
ing is accommodated in 4 slots per pole per phase, with 8 conductors per slot. The coils are
short-pitched by one slot. When the flux density wave covering two pole pitches, is analysed into
space harmonics, the fundamental is found to have an amplitude o f 0.87 T and the 3rd and 5th
harmonics to have amplitudes o f 0.24 and 0.14 T respectively. Estimate the resultant phase and
line voltages on no load if the alternator is (i) star-connected and (ii) delta-connected.
(b) I f per phase reactance o f the alternator at 50 Hz is 10 fl, calculate the circulating current
when the machine is delta connected. Neglect alternator resistance.
The effect o f slot ripples should be ignored.
Solution. Pole pitch = = -? (0-28) m
r 6
In the presentation leading to Eq. (3.51 b), it is shown that

0, = = 2 x 0 28 x 0 23 x o.87 = 0.0187 Wb

a, a 2DL n 2 x 0 .2 8 x 0 .2 3 ________
Also, 03 x 0.24 = 0.00172 Wb
2DL n 2 x 0.28 x 0.23 „ ...........................
4>s = B s= -------------- x 0.14 = 0.0006 Wb.

Slot per pole per phase, q = 4.


Slot-angular pitch or slot angle

Y= I83!_____ = m = 1 5 c
Slots per pole 4 x 3
The distribution factors from Eq. (3.47 a) are
qY .6 0
sin f z sm —
kdl = ---------- = --------- yg- = 0.9576
q sin j: 4 sin —

. 3<7Y . 3 x 60
sin sin — - —
kd3 = --------= - = r ^ T - = 0.6533
dd . 3y . . 3 x 15
q sin g 4 sin — 2 —
. 5qy .5x60
sin -?-*■ sin — z—
kd5 = ------------------------------------= 0.2053
. 5y a • 5 x 15
q sin -pf 4 sin — - —
z z

S c an n ed by C a m S c a n n e r
278 Electrical M achincn
slot. This gives a chording angle of
The coil is short-pitched by one
180
c= = 15° = Y
12
coil-span factors from Eq. (3 49) are
The 15
V — r c o s 2 = 0 -991
= cos f = COS ^ = 0.9238
2
75
kpi = cos § = COS ^ = 0.793
an d 2

... The winding factors are


kwl = kpl x = 0.991 X 0.9576 = 0.949
= '0 .6 5 3 3 x 0 .9 2 3 8 = 0 .6035
t * . * « x * „ - 0.2053 x 0.793 = 0.163.
and IVU ”v *
Subscripts 1, 3, 5 are used to denote fundamental, third and fifth harmonics respectively.
Total number of slots = 4 x 3 x 6 = 72.
72x8
Total number of turns = 288.
2
288
Series turns per phase, Nph - ^ - 96.

From Eq. (3.24), the fundamental e.m.f. per phase is


Ephi = ^2 n fk,o\NPh$\
= y/2n (50) (0.949) (96) (0.0187) = 378.4 volts.
For third harmonic component, Eq. (3.51 a) gives
K s 3 <t)3
EPh 3 - Ephi kw] 4>1
- T 7B 4 x 0-6035 x 3 x 0.00172 Q 4Q volts.
" 378 4 X 0.949 X 0.0187

Similarly, E' ,nL


* 5c —E r <(,,

= 378.4 x x5* - 10.427 volts.


0.949 0.0187
(t) For star-connected alternator, the magnitude of the p *r ph:u;e e.m.f

Eph = ^Epli i + E)
/vri i — i/,3 + —
£'*/ / no
= V(378.4? + (6 6 .4 ? + (10.427?" = 334.32 volte ^
Recall that for star-conncctcd alternator, the third frequency e.m.fs- dont *I
the lin

’ E linc= ^3 _______
. 655.63
.. = ^3 V(378.4? + (10 .4 27 ? = n/3 (378.54) t
closed^?.r ^c*ta'connected alternator, the third harmonic e.m.fs. are
dosed delta and consequently don’t appear in the lines.

’' Ei>h= EUne = >/(378.4? + (10.427?" £ 378.54 volts-

Scanne am scanner
Art* 3-51 Basic Concepts or Rotating Electrical Machines 279

(b) When the alternator is delta connected, the resultant of fundamental and 5th harmonic
e.m.fs. m the closed delta is zero, but the resultant of 3rd harmonic e.m.fs. is 3E3. This e.m.f.
3Es gives rise to a circulating current determined by the total 3rd harmonic reactance of
3 x (3 x 10) Q at 150 Hz.

3E* 3 x 6 6 .4
.\ Third harmonic circulating current = = 2.213 A.
3 x (3 x 10) 3 x (3 x 10)
Example 3.12. F c a 3-phase winding with 3 slots per pole per phase and coil span o f 8 slots,
compute the breadth and pitch factors.
The flux density wave in the air gap o f this machine is found to have 20c7( third harmonic.
Calculate percentage increase in per phase r.m.s. e.m.f. due to harmonic.
Solution. Slot per pole per phase, q = 3, Slot per pole = 3 x 3 = 9

Slot angular pitch, y= ~ = 20°

sin 91
sin 30°
kd\ = = 0.9598
_y 3 sin 10°
q sin

For a coil span of 8 slots, the coil is short-pitched by one slot and the chording angle is
e = y = 20°
kpl = cos ^ = cos 10° = 0.985
= 0.9598 x 0.985 = 0.9454
u _ sin 90 _
- 3 sin 30 - 0 667
kpj = cos 30° = 0.866
kw3 = 0.667 x 0.866 = 0.5776

From Eq. (3.51 b), EpM = BpM x 0.20 = 0.1222

.-. Eph = VF^1 + (0.1222 Ephx? = 1.00744 Ephx


Therefore, percentage increase in generated e.m.f. due to third harmonic flux density com­
ponent
1.00744- 1.0
x 100 = 0.744%.
1.0
Example 3.13. A 6-pole, 50 Hz alternator has 42 slots with two-layer winding. The flux per
pole is 0.012 Wb and each full-pitch coil has 8 turns. Determine the phase and line e.m.fs. in case
(he narrow-spread coils are connected to form (a) 2-phase winding and (b) 3-phase star-con­
nected winding.
Solution, (a) Slots per pole per phase,
S/m 4 2 /2 3 x 7 7 H
;•>
q~ P ~ 6 3x2 2 [\
As q is not an integer, the 2-phnso winding is a fractional-slot winding with 5* = 7. Its , ';1
•j
distribution factor for the fundamental from Eq. (3.48 c), is, t»
90 if
sin
r* : •i
= 0.9022
n • 1
7 B ,n -T90

• ir - ‘ |

S c an n ed by C a m S c a n n e r
IArt. j
280 Electrical Machinery

kw = 0.9022 as kp = 1
. 42 x 8 _ <ro
Per phase series turns, N ph = ^ ~ A

E nh = ^ 2 K f k wN phto
? =V2n (50) (0.9022) (168) (0.012) = 4 0 3 .9 8 V

E, = V2 x Eph = 571.23 V
4 2 /3 _ 2 x 7 _ 7
(b) 7 - c " o2vxi3t 3

60
sin
From Eq. (3.48 c), = 0.9558
. 1 60
7 sin 2 T

... Eph = V2n (50) (0.9558) (112) (0.012) = 285.322 V


E, = y/3 x 285.322 = 494.18 V.
E x am p le 3.14. A 3-phase star-connected alternator has 81 slots, 6 p oles and double-laye
narrow-spread winding with coil-span equal to 13 slot pitches. The flu x density distribution ii
the air gap is given by B(Q) = sin 8 + 0.4 sin 3Q + 0.25 sin 50.
Determine the r.m.s. value o f third and fifth harm onic ph ase e.m .fs. in term s o f fundamenta
phase e.m.f. Also, calculate the ratio o f resultant line e.m.f. to the resultant phase e.m.f.
Solu tion. Slots per pole per phase,
8 1 /3 3x9 9
6 3x2 2

6 x 180
Angular slot pitch, f40l
81 3
< /
= 13y
_ 13 x 40 f520

Chording angle, e = (one pole pitch) - (coil span) = 180 - 520 f20^

60
sin
From Eq. (3.48 c), = 0.9555,
Q . 1 60
9 sin — •—
2 9
. £ 20
kp\ = cos - = cos = 0.9988

, sin 90° .
*<*3 - ^77 = 0.6399,
Q • 60
IT
'1 0
kp2 = cos = 0.9848.
T x3

S c an n ed by C a m S c a n n e r
Basic Concepts o f Rotating Electrical Machines 281

sin 150
= 0.1937,
300
9 sin

kp3 = COS = 0.958

k wl = 0.9555 x 0.9983 = 0.9539


kw3 = 0.6399 x 0.9848 = 0.6302
kwS = 0.1937 x 0.958 = 0.1856

From Eq. (3.51 b), E phz = E s i l l * 0 4 = 0 26 4 3 £ * >

Ephs - E
>*> S :* ° - 25 = ° 0 4 86
-,1/2
Resultant phase e.m.f., EPs = [ E%hi + E2
ph3 + E,pW] = 1.0355 Ephl
1/2
Resultant line e.m.f., E ( = d3 [fipM + Epfts]' 2 = 1.734 Ephi
Ei 1.734
= 1.675
Eph 1.0355
Note that line e.m.f. is not equal to V3 times the phase e.m.f. It is because of the presence
of third harmonic e.m.f. in the phase voltage.
E xam p le 3.15. The flux density distribution along the air-gap periphery o f a salient-pole
machine is rectangular as shown in Fig. 3.25 (a). Draw one complete cycle o f the em f generated
in a single-turn coil ofcoil-span 150° electrical. The coil moves with a velocity o f 20 m l sec in a
peak flux density o f l T. The effective length o f armature conductor is 0.8 m. Calculate rms value
of the voltage in the single-turn coil.

Solution. The flux density distribution is shown in Fig. 3.25 (a). With coil-span of 180“, the
*»veform of the em f generated in a singleturn coil will bc similar to B-wavcform. For coil-span

S c an n ed by C a m S c a n n e r
282 Electrical M achinery _______ ________________ _____

, pmf waveform will be different from 5-waveform


th . coil with coil-span of 150- e.ectrical is sketched in

follows: . . )as
(0 For 0”< 6 < 30*. instant 1 , coil on' is shown movrng m zero wave. Theref„re, em
coil for this range is zero as shown. the
(ii) For 30” < 0 < 60”, instant 2, coil-side a is cutting through = l T but cofl.sid .
tine zero B - wave. The resultant emf across the single-turn coil .s ««• ,,
cutting zero
e = Blv = 1 x 0.8 x 20 = 16 V
(m) For 60°< 0 < 150°, instant 3, both coil-sides are cutting through 1 T. Therefore,
net emf
generated in conductors of single-turn coil is
e = 2Blv = 2 x 1 x 0.8 x 20 = 32 V

In this manner, emf waveform of Fig. 3.25 (b ) is sketched. RMS value of this generated
emf
in one single-turn coil
1 /2
I l 6’ x f x 2 + 32’ x § = 24.44 V

Example 3.16. A 50 Hz, salient-pole alternator has the field flux-density waveform of Fig.
3.25 (a). This alternator has the following data :
Total turns = 360, phase spread = 60*, pole-pitch = 0.6 m
stator length = 0.8 m, coil-span = 180° electrical
Calculate rms value o f the fundamental e m f per phase.
Solution. The fundamental value o f peak flux density, by Fourier scries analysis, is

By = - B „ cos e = - x 1 x cos - = 1.10266 T


1 n n 6
nD 2nr
Pole pitch = 0.6 m
P P
Fundamental air-gap flux per pole, from Eq. (3.11), is

« , = | B , lr = | B , / = \ •fl, •/ (pole pitch)

= - x 1.10266 X 0.8 x 0.6 = 0.33695 Wb


71

Taking the winding as uniformly distributed


60
sin
= 0.955, kpl - 1
kdl 60 71
2 X 180
Per-phase emf, Ephl = V2x kwl Nph
= V2ji x 50 x 0.33695 x 0.955 x 120 = 8576.7 V. ^ B jS
Example 3.17. The em f generated in a conductor at any instant is given by Blv, Tol
the air-gap flux density in a rotating machine, I is the conductor length and v is
velocity. ...

With the help o f this expression, derive e m f expressions for (a) dc machine (b)
synchronous machine.

S c an n ed by C a m S c a n n e r
Basic Concepts o f Rotating Electrical Machines 283

i tion. In c = Blv, as I and u are constants, the waveshape of emf e is the same as that
— *i -. _wave
^°flux-density _■ ■i/\ iin
m oair
i *•rr /\%ra••a rra %
gap. Average t a Ii i a at a 4 % a a aa
value wm*J■•a4aw«■ ■
of emf in one conductor .> given
is ■a. •■a* I.
by ■■
oftbe
Eal, Ba, l v - x / x nDn = P0n

0 = flux per pole


where 2
for a dc machine, there are — conductors in series between the positive and negative
CL

shes. Here Z = total armature conductors and a = number of parallel paths for armature
finding. Therefore, output or brush voltage in a dc machine is given by
Number of conductors Average value of
E= in each parallel path emf generated
between the two brushes in one conductor

a a
Here n is the armature speed in rps. The above emf expression is the required expression
for a dc machine.
(b) From above, average emf generated in one conductor,
Pn
Eav = P0n = 2/0, where f = —

rms value
Now = form factor
average value
Assuming the waveshape of flux-density wave sinusoidal, its form factor is
1
n
FF =
2V2
n B"
rms value of emf per conductor _ ^
Also
average value of emf per conductor

.-. rms value of emf generated in one conductor, E rms ~ EF X Eav 2yj2 X 2/0

n
/0
n/2

Emf per turn =2 kp = ^2nf<t>kf

where kp is the coil-span factor.


Per-phase generated emf = V2n /0N p/, kp kd = V2n /0 Nph ktl
where Nph = series turns per phase
Another way of obtaining phase-emf expression is as under :
Maximum emf per conductor, E'mr a x
For a sinusoidal flux density wave,

E,nax = \ (B„) I V

I x nDn = 710/"
nDl
P

S c an n ed by C a m S c a n n e r
284 Electrical Machinery
IA
j l m a x _____71
Rms value of emf generated in one conductor - ^ ^

Now proceed as above.


Example 3.18. A 6-pole alternator, with 54 slots, runs at a speed o f 1000 r.p.m y
coils A and B are placed in slots as under : V-turn
Coil A : coil-sides in slots 1 and 11.
Coil B : coil-sides in slots 2 and 10.
Flux per pole is 0.015 Wb. Find the resultant e.m.f. i f coils A and B are connected «>
Sehes
(a) aiding (b) opposing.
54
Solution. Pole pitch = — = 9 slots = 180°

, , 4 x 180 OAO
Slot-angular pitch, y = — ^ — = 40

A full-pitch coil would have its.one coil-side in slot 1 and its other coil-side in slot 10 B t
here coil A is overpitched by one slot because this coil has its coil-sides in slot 1 and 11 , Fig. 3 L
(a). Likewise, coil B is short-pitchecf by one slot (= 20°), because its coil-sides are placed in slots
2 and 10, Fig. 3.26 (a).
In Fig. 3.26 (a), e.m.fs generated in coil-sides 1 and 10 of a full-pitch coil would be maximum
but of opposite sign. Phasor diagram for e.m.fs generated in coil-sides 1 and 11 is shown inFig
l 10 c n m t m
O
3.26 (6 ), while thatP afor
/ L \ ...V t C ls N m /% v-k-v
e.m.fs generatedi n in/ » coil-sides 2 and1 f10
n il .P in n c ? O o n n
is showni n inI ? Fig.
i/« O O P / \
3.26(c).

—Pole pitch -
=9 Slots =180'
U J LU
Coil A-> Coil B

y
* 1
• 1 * 1 1
1 1
11 11 j
• 1 \1 1
1 ■IEI
i 2 3 I 5 6 7 8 9 10 11 E,= E

1
fa) (b)
Fig. 3.26. Pertaining to Example 3.18.
One coil-side of 80-turn coil has 80 conductors, r.m.s. value of generated e.m.f. in one con­
ductor, from Example 3.17, is /<j).

E.m.f. generated in one coil-side of coil A or coil B having 80 conductors is given by

2 fty (conductors in one coil-side)

E\ = | x 50 x 0.015 x 80 = 133.306 V

S c an n ed by C a m S c a n n e r
285
Hasic r w p p u o f Rolaling Electrical M a ch in e

^ ^ ^ T _ 7 _ 133 306 V generated in coil-side 1 of coil A is shown by vertical phasor

i t “ i i a coil * i. . h . . . - r » - ■ » » * • <” *

* js r = r - ^
(6), (as done during ihe derivation of pitch factor).
^ . Resultant e.m.f. generated in coil A is

Ea = 2 E cos * = 2 x 133.306 x cos 10° = 262.56 V

„ , 1 . p. - i.. » . ; C
, ^ng behind Similarly, e.m.f. E 10 is shown leading E n yY >
ofcoil B is 160°. As in coil A, resultant of e.m.f. in coil B is

E b = 2 E cos | = 2 x 133.306 x cos 10°= 262.56 V

r Vrre q oa th\ qnd fci reveals that resultant e.m.fs EA and EB are in
(a) An examination of Figs. 3.26 (b) and (c) reveais i connected series
pha e with each other. Therefore, resultant e.m.f., when coils A and B are connecte
aiding,ER- E A + ED= 2 x 262.56 = 525.12 V.
(5) When series opposing, ER = EA ~ E B~ 0.

i , i > . ......
is the field winding.
The pattern of the magnetic field along w t u n o f i h e m m n t flowing through
rinding, depends upon (O §uch as airJ p length,
he winding and ( i n ) & m f howcverj depends only on the winding arrangement and
he winding^u rrent A knowledge of the air-gap flux distribution in a machine helps in deter-
nining the (i) generated e.m.f. waveform and its magnitude and (u) the electrical torque.
The object of this article is to investigate the m.m.f. produced by a winding distnbuted in
he slots along the air-gap periphery. For this purpose, the attention is first directed to the
« f variation over one sbt-pitch and the conclusions drawn from it, are applied for■determm-
the distributed-winding m.m.f. For one slot-pitch-m.m.f. variation, refer to Fig 3.27 and
all the Amperes Circuital Law. According to this law, the m.m.f acting on any closed path
qual to the total current enclosed by that path. In Fig. 3.27 (a), each slot is assumed to
tain 9 conductors and each conductor carries a current i. If path 1 is traversed the ampere-
ductors enclosed are 3i and this is therefore the m.m.f. change as shown in Fig. 3.27 (6). If
h 2 is traversed the current enclosed is 6i and for the path 3, the total current enclosed is 9i.
s m m f variation is sketched in Fig. 3.27 (6) and it is seen that the m.m f. changes linearly
n zero to 9i over the slot portion or slot width. If the tooth is traversed , the enclosed current
lains 9i till one slot-pitch is covered. When the three conductors of the adjoining slot are
ered, the enclosed current is again 3i as shown in Fig. 3 27 (6). It can, therefore be concluded
t the m.m.f. changes linearly over the slot portion and does not change for the tooth portion
shown in Fig. 3.27 (6). .
In order to moke the analysis simpler, the changi in m.m.f. over the slot portion is taken as
pped at the middle of the slot width. In other words, the total change of m.m.f. li e. 9i tn Fig.

S c an n ed by C a m S c a n n e r
286 Electrical M achinery

Distance along
air-gap Periphery lc>

along
lb)
air-gap periphery

(a)

Tooth
Conductors

Fig 3 27 Variation of m.m.f. along air-gap periphery.

3 27 m over the slot portion, is taken to occur at the centre of the slot, as depicted ia Fig. 3.27

(Cl 3.6.1. of a coil. Consider

dfrection^of current irf thetwo'coil'-sides isindicat^ ^ ca l ^ u p b 7 tS

laid out flat) is illustrated in Fig. 3.28 (6). simplicity:


In order to determine the coil m.m.f. the following assumpt.ons are m ^ ^ ^
(/) The permeability of stator and rotor irons is much greater^than biWy Th.s
of this, it may be assumed here that the stator and t o t o r c o r e s h a e n r n ^ p
means that the reluctance to the magnetic flux is offered by the a g P assumpti0n■
(ii) The magnetic flux lines are assumed to cross the air'8®P|‘a ‘the pole pitch-
permissible.
•missible. because the air-gap is much smaller in comparison
compel ^ ^ ^^ cur rent
According to Ampere’s circuital law, the magnetic field intensity an
enclosed are given by the relation.
j> H dl = total current enclosed. d any one c
If any of the dotted closed path in Fig. 3.28 (a) or (b) is traverse
then
ZgH = Ni

S c an n ed by C a m S c a n n e r
Basic Concepts ol Rotating -Electrical Machines 287

N-TURN
COIL

GAP

MAGNETIC A)Ss
OF STATOR COIL

STATOR ROTOR

I I
f«*— POLE PITCH POLE PITCH—H

STATOR
SURFACE
~"f l i ? i f r f c T t t w] ’
ROTOR J Ji I LL-i-dJ
SURFACE
N-TURN
COIL'

SPACE A N G L E ,(X

Fig. 3.28. (a) F u ll-p itch coil on stator, (6) D evelop ed v iew o f Fig. (a)
and (c) m .m .f. d istrib u tion alon g a ir-g a p periph ery,

17 N l ...(3.52)
g H = ~2
Thus the magnetic potential difference gH across each gap (there are two air-gaps in series) is
!M, Since any closed path around any coil-side, satisfies Eq. (3.52), the variation of magnetic potential
difference along the air-gap periphery is of rectangular waveform and of magnitude ^ Ni. The magnetic
fluxentering the stator (or leaving the rotor) is considered positive, therefore, the variation of mag­
netic potential difference along the air-gap periphery is as shown in Fig. 3.28 (c).
It is seen that the air-gap m.m.f. wave at any instant is rectangular. If the coil current is
d.c., the magnitude of m.m.f. wave does not vary with time and space. For a.c. in the coil, the
amplitude of m.m.f. wave varies with time, but not with space, i.e. the air-gap m.m.f. wave is
time-variant but space invariant.
The rectangular m.m.f. wave of magnitude \ Ni can be resolved by Fourier series into its
fundamental and higher order harmonic components. The fundamental component of rectan­
gular wave is found to be
4 Ni ...(3.53 a)
Fa1 = - cos a = F lp cos a

"'here a is electrical space angle measured from the magnetic axis of the stator coil as shown
^ Fig. 3.28. Here F lp, the peak value of the sine m.m.f. wave for a 2-pole machine is given by

...(3.53 6)
Fin-*' ^ ATs per pole

S c an n ed by C a m S c a n n e r
288 Electrical Machinery

This peak m.m.f., F lp is always aligned along the magnetic axis of the coil.
If i is alternating current, then peak value of the fundamental sine componen(.
accordance with i. That is when i is zero, F Xp is zero, when i is maximum, F Xp is . Vafies jn
so on. When i = Imax = V21, then time maximum of the peak m.m.f. for a 2-pole machi and
4ATV2/ neis-
F'\pm — ATs per pole
71
•■•(3.54)
and for a P-pole machine,
4 NV27
F 1pm ATs per pole
71
(3-55)
3.6.2. M .m .f. o f distributed w indings. Fig. 3.29 (a) depicts three full-pitched •
distributed winding on the stator and Fig. 3.29 (6) illustrates the develoneH
developed view of_r.S?. °f
(a). Each coil has Nc turns and each turn carries i amperes.

M a g n e tic a x is M a g n e tic axis


of d is t r ib u t e d of u n ifo r m
w i n d in g c u rr e n t
sheet

(a ) (6 )

Fig. 3.29. Pertaining to the m.m.f. produced by (a) a distributed winding and (b ) an equivalent current sheet

By referring to Fig. 3.28 (6) and (c), the m.m.f. variation due to coil 1 ,1 ' alone is seen tobe
a rectangular wave of amplitude ± ±NCi as shown in Fig. 3.29 (c-i). Similarly, the m.m.f. varia­
tions due to coils 2, 2 and 3 ,3 are as shown in Fig. 3.29 ic-ii) and (c-iii) respectively, The
combined m.m.f. produced by the three coils can be obtained by summing up the ordinates of
the individual coil m.m.fs. For example to the left of ccil-side 1, the combined m.m.f. is
- 3 /2 Nc i, between the coil-sides 1 and 2 the combined m.m.f. is —^ N c i, between the coil-sides

2 and 3 the combined m .m .f is + ~ N c i, between coil-sides 3 and 1' the combined m.m.f. is
3 /2 Nc i and so on. This resultant m.m.f. variation along the air-gap periphery is depicted in
Fig. 3.29 (c-ii»). It should be noted that the effect of winding distribution has changed the shape
of the m.m.f. wave, from rectangular to stepped.
When the machine has more than three slots per pole per phase, as is usually the case, the
steps are neglected and m.m.f. variation is taken as smooth over these slots. As a consequence,
stepped m.m.f. wave changes to trapezoidal m.m.f. wave as shown in Fig. 3.29 (c-iv).
The smooth variation of m.m.f. over 3 slots per pole per phase in Fig. 3.29 (c) m a y be as­
sumed to be produced by a uniformly distributed current sheet placed on the inner periphery0]
the stator as shown in Fig. 3.29 (d). The current sheet is, therefore, a thin strip of conducting
material attached to the stator and is used here to replace the uniformly distributed wiping'
the slots ofFig. 3.29 (a). In other words, the m.m.f. produced by uniformly distributed™"
® f ° , F'g ,3 '29 (<0 'S Identical, in all respects, with the m.m.f. produced by unlfo? . nthe
tnbuted Winding of Fig. 3.29 (a). If the distributed winding in the slots is on the rotor, then t
umtormly distributed current sheet would be wrapped around the rotor accordingly.

Scanned by C am S canner
liable C oncept of Rotating Kledrlcal Machine* 289
3.61
,Af‘
S t a to r 1 2 3 l'2' 3'
surfaci l _ j ® L J © L J © l m js L M j y Dap
R o to r / ' nr
s u rfa ce

Trapezoidal
m m f w ave

- S o u t h p o le
N o rth p o le N o rth p o le -
(c )
F ig . 3 .2 9 . D evelop ed dia gram and m .m .f. w a v e o f the m a ch in e o f F ig. 3 .2 9 (a).

Actually, the m.m.f. distribution along the air-gap periphery depends on the nature of slots,
winding and the exciting current. In case the m.m.f. produced by uniformly distributed winding
is sinusoidally distributed in space, then the equivalent current sheet must also be sinusoidally
distributed in space.
In Fig. 3.29 (a), the total ampere conductors in the band covering an angle o are 3Nci and,

therefore, the angular current density is J = The trapezoidal m.m.f. wave of amplitude

—— - ( = —J a) of Fig. 3.29 (c), can therefore be obtained as illustrated in Fig. 3.30 (a) by a
2 2
Trapezoidal F u n d a m e n ta l
m m f w a ve [ / comp.
M a g n e tic •
a xis S in u s o id a l
current i. 2
\ sheet 7 i
/
j
J

-9 0 -

S in u s o id a l'
m m f w ave
M a g n e lic J
axis o f - ^ * 1 (b)
cu rren t sheet
(a)
Fig. 3.30. (a) M.m.f. from current-sheet concept for Fig. 3.29 (c).
(b) M.m.f. wave produced by a sinusoidal current sheet.

>canned by C a m S c a n n e r
290 Electrical Machinery

_ Note that the m.m.f. at the middle of.


uniform current sheet of density J a •iNU v 01 unif0rrn ^

rent sheet is zero and is maxbnun, ( - \ J *> at the end o


1 1
s linearly over the angle o from - ^ J ° to + - J o, because the CUrrent
note that m.m.f. vanes
sheet is uniformly distributed. In case the currerh sheet is Tropezoidal
sinusoidal, then the m.m.f. wave is also sinusoidally dis mmf wave Fundamentoi
Comp
tributed in space as shown in Fig. 3.30 (6). But note from "f
this figure that sinusoidal mmf wave is displaced by 90
Uniform
(electrical) from sinusoidal current sheet. cu rren t
sheet
In general, for Nph full-pitched series turns per phase,
distributed over an angle a under each pole, the angular

current density is The amplitude of magnetic


. 1 1 ., •
potential difference across each gap is 2 2 ph * 3S

shown in Fig. 3.30 (c).


The Fourier-series analysis of trapezoidal m.m.f. wave M a g n e tic
a x is of y
of Fig. 3.30 (c), gives the peak value of the fundamental c u r r e n t sheet
sine-component as, (c)
Fig. 3.30. (c ) M.m.f. wave produced by
sin o /2 ( 1 w
F ATs per pole N p h s e r ie s tu rn s distributed over an
n o /2 2 ph a n g le a .

N.ph ...(3.56)
ATs/pole
= * *■

For F Xp, the subscripts 1 and p stand for fundamental and peak respectively.
Note that the effect of distributing the turns in the various slots, has resulted in the intro­
duction of breadth factor kd in Eq. (3.56). If the winding is fractional-pitch winding, then the
coil-span factor^ must also be introduced ; modifying the peak value of the m.m.f. given b> Eq
(3.56) as,
4 N
F iP = - k d k
knp - 12— ATs/pole

k —^ —- ATs/pole.
7 1 “’
...(3.57 0)
For P-pole machine, F, = z k ATs/pole
n P
As the fundamental component of m m.f. is a sine wave as shown in Fig- 3.30 (cM
tion of m.m.f. Fol along the air gap periphery can be expressed as

F - z4 l N z_ph__ i ■ J3-57 ^
a] _n K ,i>
... pn cos a = F Xp cos a
where a is measured from the magnetic axis of current sheet.
iiiMf*P
Note that Eq. (3.53 a) gives the space fundamental distribution of m.m.f- aI<^ ft) is th‘‘
periphery for a single full-pitched Af-turn coil having 2 poles, whereas
modified form of Eq. (3.53 or) for a distributed F-polc winding having Nph acr,c8

S can n ed by C a m S c a n n e r
Basic Concepts or Rotating Electrical Machines 291

If the winding carries sinusoidal alternating current, then at the instant i = 0, F Ip = 0 and
*’hen i = Iniai>F\p *s a^so maximum. Thus the time maximum of the peak m.m.f. for a 2-pole
machine is

F \pm = £ K 2 7 ATs/poIe ...(3.58)

and for a P-pole machine,

F iPm = J ^ p V 2 / ATs/pole ...(3.59)

flote that the axis of F lp is always along the magnetic axis of distributed winding or along
the magnetic axis of the current sheet as shown in Fig. 3.30.
The field winding in a cylindrical-rotor synchronous machine is distributed in rotor slots as
shown in Fig. 3.7 (6) for a 2-pole machine. The number of turns per slot can be varied in order
tominimise the harmonics in the field flux waveform. Like the armature winding mmf given
byEq. (3.59), the peak value of fundamental mmf wave Fy produced by field winding in a P-pole
machine can be expressed as

Fv = ^ ATs/pole ...(3.59 a)t ■

where Nf = total number of field turns


If= field-winding current
kf= field-winding factor
Single-phase winding. The m.m.f. produced by single-phase winding on the stator, or
rotor, is alternating or pulsating in nature. This can be demonstrated by referring to Fig. 3.31
where
(i) a single-phase winding is represented by a uniform current sheet of spread a and
(ii) only the fundamental sine component of trapezoidal m.m.f. wave is considered.
The time origin of current in single-phase winding is taken at the instant current is positive
and maximum as shown in Fig. 3.31 (a). At a)/! = 0, i j = / max and the m.m.f. waveform along the
air-gap periphery is sinusoidal with a peak value of Fpl (= Fm) proportional to maximum cur­
rent Imaz. At cof2, the current in single-phase winding is i2 and the m.m.f. waveform is again
sinusoidal with a peak value proportional to *2, see Fig. 3.31. Note that the peaks Fpi> FP2
. . . K .
are always along the axis of single-phase winding. At otf3 = —, i3 = 0 and therefore no m.m.f. is
St
produced. At a = 7t, i4 = - Iniax and m.m.f. waveform is sinusoidal with peak - Fpi as shown. If the
8Pace angle a along the air-gap periphery is measured from the axis of single-phase winding as shown,
toenm.m.f. Fj at any space angle a can be expressed as
F, =F pi cos a

^ with current variations *,, i2, i3 ■■■ ■ But the currents i,, i2, i3 ... vary sinusoidally with
^•consequently the m.m.fs Fpl, Fp2, Fp3... must also vary sinusoidally with time.

m m
" S c a n n e d Dy C a m S c a n n e r
292 Flpftrical Machinery
S in g le p h a s e d i s t r i b u t e d
.w in d i n g

K -< r - A !

(6 )

Fig. 3.31. P ertain ing to pu lsatin g m .m .f. p rod u ced b y s in g le -p h a s e distributed winding.

In Fig. 3.31 (a), the time origin of current i is taken at the instant of its passing through
fimum nositive
maximum value Therefore, at (»f x= 0, ix = Imax-
positive value.
This gives peak value of m.m.f. at instant fj as
Fp « (single-phase winding turns) (Jmax COS (Ofx)

- NImax ■cos cofl


or FPi = Fm cos cof! = Fm cos 0 = F m
where F„ N I* n
A t (0f2, i-2 = I mux cos T h is P ves
Fp2 = Fmcos (0f2
COS (01
Similarly Fp3 ~
Fp2 Fm cos orf3
= Fm
Fpi = - F m cos (of4
Substitution of the values of Fpl, Fp2 ... as given above in Eq. (3.60 (a)) gives the pulsating
m.m.f. wave as
at (ofx, F l = F m cos (of x cos a
= Fm cos 0 cos a
at (of F 2 = F m cos cof2 cos a ...(3.60 M
at (of3, Fa = Fm cos (of3 cos a
and at a)f 4, F4 = F m cos oof4 cos a

In Eq. (3.60 (b)\ F3 is zero because at cof3 = ~ , current is zero. An examination of Eq.$-60

(6)) reveals that a general expression for the stationary (or standing) pulsating m.m.f- 'vav' ’
tirmc aS8 d*stributed winding carrying alternating current, can be expr°s
terms of both tune angle mf and space angle a as, f)

F (oc, t) - f m cos tof cos a

S c an n ed by C a m S c a n n e r
Basic Concepts o f Rotating Electrical Machines 293

In Eqs. 3.60 (6) and (c), Fm is the m.m.f. per pole corresponding to the maximum current
j ^ and is equal to F ipm as given by Eq. (3.58) for a 2-pole machine or by Eq. (3.59) for a P-pole
machine. In Eq. (3.60 (c)), the term cos oof indicates that m.m.f. varies cosinusoidally (or
sinu soid ally) with time and the term cos a denotes its cosinusoidal, or sinusoidal, distribution
jnSpace along the air-gap periphery.
In order to give physical significance to Eq. (3.60 (c)), the reader should assume himselfTherself
seated at a = 0°, i.e. on the axis of the single-phase winding in Fig 3.31 (6). Now at time t = 0,
inf= 0 and the m.m.f. seen by the reader, as per Eq. (3.60 (c)) is maximum (cos a = cos 0° = 1). As
the time progresses, ait changes and the m.m.f seen by the reader also changes. At oii = 60°, the

reader sitting at a = 0° sees — Fm ; at arf = 90°, the m.m.f. seen is zero ; at arf = 180°, the m.m.f.

seen is - F m and at (of = 270°, m.m.f. seen is zero. This shows that the reader seated at a = 0° on
the axis of single-phase winding, where cos a is always unity, comes across an m.m.f. pulsating
between + Fm and - Fm. Suppose the reader is sitting at 60° away from the phase axis, i.e. at

a = 60°, then at cof = 0°, the m .m.f seen by the reader is Fm cos 0° cos 60° = - F m and at

o 1 *
a)f = 180°, the mmf seen is - ^ Fm. Thus the m.m.f. seen by the reader at a = 60° is again pulsating

between + - Fm and - ^ Fm, but he/she can never see maximum m.m.f Fm. If the reader is sitting

at a = 90°, the m.m.f. seen by the reader is always zero for all times to come, because cos 90° is
always zero.
3.6.3. Current-sheet Concept. Concept of current sheet has already been introduced in
Fig. 3.30. It may be summed up that current sheet
(i) is a thin strip of conducting material carrying current in the dot and cross directions,
(ii) is wrapped around the stator or rotor and
(im ) develops m.m.f identical to that produced by the winding it replaces.
It is also observed from Art. 3.6.2 that
(а) uniformly distributed winding having larger number of slots per pole can be replaced by
an uniformly distributed current sheet,
(б) uniform current sheet produces trapezoidal m.m.f wave and
(c) sinusoidal current sheet gives birth to sinusoidal m.m.f wave leading it by 90°.
The object of this part of the article is to develop a relation between the current sheet and
associated m .m .f For this purpose, consider a sinusoidal current sheet producing a
sinusoidal m .m.f wave as shown in Fig. 3.32 (a). Note that sine-current sheet lags the sine-
0-m.f. wave by 90°, as per Fig. 3.30 (b).

The sinusoidal current sheet with peak value A amp per metre length of air-gap periphery
'■an be expressed as J
J = A s i n a amp/m
re a is any angle measured from the origin of current sheet.

Now consider a differential element d 0mat an angle 9 from the origin as shown in Fig. 3.32.
Peripheral width of differential element = r dQm, see Fig. 3.32 (6).

Current in differential element of width r dQm, at an angle 6 from origin, is


i = A sin 6 (r d 0m) Amp.

^ C a r ir ie u u y u a r n o c a r ir ie r
294 Electrical M achinery
.M agnetic
Axis
Sinusoidal
current sheet

Si nus o
mmf w a v

Space
angle,*

(0) , i m r wave Oroduced by sinusoidal-current sheet


F K . 3-32. ( . » S im .s .id a U y " S T 3 £ d E S ft

D
Here r = rotor radius = —

where D — rotor diameter.

But »m = f 6

or d
u e
u m = -pd e
D 2 . AD ...(3.61)
i = (A sin 0) sin 0 d0
2 P
Note that m.m.f. per pole at any angle dependsupon b“
one pole-pitch apart. This means that m.m.f. at mgl th. m.m.f
current contained in the angular spread of it rad'an^Thus for det * ated from ato
any angle a, Fig. 3.32 (a) the current expression o fE q .(3 .6 1 ) difference
a + n. As m.m.f. crosses the air-gap twice, m.m.f. t (oc) or tne magneuc p
across each air gap is
+n
/ ( a) = \ f ^ s i n 0 •d 0 = ^ | - c o s 0 |“
2 *a
...(3.62)
= cos a ATs/pole

It is seen from Fig. 3.32 (o) that peak value of m.m.f. Fp occurs at a = 0. Therefore,

( 3 -62)> jn ...(3.63)
FP = F (a) la=0 = ' p “ ATs/pole

This peak m.m.f. can also be expressed as


r 2 AD 1 ( 2 a } ( k D\
—A
p~ 2 n P 2 71 P . nitchl
1 \ J 2 n /m llP °lep
= - [uniformly distributed current sheet of linear density “ A amp

Fp = - J (pole pitch)

where J=-A am p/m .


ji

S c an n ed by C a m S c a n n e r
Art. 3.6J Basic Concepts of Rotating Electrical Machines 295

In general, peak value of m.m.f. Fp can be expressed as

where J = uniform current density in Amp/rad


and <7= spread of uniform current sheet in rad.
Example 3.19. A 3-phase, 2-pole stator has double-layer full-pitched winding with 5 slots
pole per phase. I f each coil hasN c turns and i is the conductor current, then sketch the m.m.f.
per
waveform produced by phase a alone.
For Nc - 4 and per-phase current 20 A, determine the maximum, as well, as r.m.s., value o f
the peak o f fundamental m.m.f. wave.
Solution. A 2-pole stator structure with 5 slots per pole per phase is shown in Fig. 3.33

(a). The angle subtended by 5 slots is a = 5 y = 5 x -yjy = 60°. The slots of phase a are numbered
1, 2 ,... 5 and 1 , 2 , ... 5 . For double-layer winding, each slot contains 2 coil-sides or 2 Nc con­
ductors. Slots pertaining to phases b and c are also shown. Note that sequence of phase belts is
a c b a cb clockwise and each belt has phase spread a of 60 degrees electrical. Current flow in
coil-sides is indicated by dots and crosses.
Developed view of Fig. 3.33 (a) is shown in Fig. 3.33 (b). For any closed path around slot 1, the
total current enclosed is 2 Nc i ampere conductors, therefore magnetic potential difference across each

gap is — (2 Nc ij = Nc ■i. Thus, the m.m.f. variation ffom - Nc i to + Nc i is shown at the middle of slot

1 in Fig. 3.33 (ci). Like slot 1, the m.m.f variation for slot 1' is from + Nc i to - Nc i as shown. Thus,
m.m.f variation for coil 11' is of rectangular waveform with amplitude ± Nc i. Similarly, the rectan­
gular m.m.f. waveforms of amplitude ± Nc i are sketched for the coils 22', ..., 55' in Fig. 3.33 (c ii) to
Fig. 3.33 (c v). The combined m.m.f. produced by 5 coils is obtained by adding the ordinates of the
individual coil m.m.fs. The resultant m.m.f. waveform shown in Fig. 3.33 (c vi) consists of a series

Fig. 3.33. (a) A 3-phase, 2-pole stator with double-layer winding having 5 slots per pole per phase.

STcanned by Cam Scanner ~ ~ “


296 Electrical Machinery •Art- 3.6

, o n i = (conductors per slot) (conductor current). The amplitude of th


of steps each of heig ‘ amplitude of m.m.f. can also be obtained as under ■
resultant m.m.f. wave is b ivc i. f

c b
-A -

1 2 3 4 5 1' 2' 3' W 5'


<r — •j (b)_
! Nct ~
(1)
" t
-* E T

North .. North
■South pole * pole
“ pole

(c)
Fig. 3.33. Pertaining to Example 3.19. (6) developed view o f Fig. 3.33 (a) and (c) m.m.f. wavefo

Uniform current density


. [Ampere conductors in one slot] [Number of adjacent slots inj>ng_
- Angle covered by adjacent slots (i.e. phase spread)
[2 Nc i] [5] 10 N c i
«/ =

Resultant amplitude of m.m.f. = ~ J n = ^ — o = 5 N c i A T /p ole.


2 2 a

Scanned by Cam Scanner


Basic Concepts o f Rotating Electrical Machines 297

M before, the resultant m.mT. waveform of Fig. 3.33 (c vi) can be resolved into fundamental
,„ d a senes of odd harmon.cs. The peak value of fundamental sine component is
CT
8in 2
[5 N c i] = £ . kd [5 N c i] A T /pole
7T o /2 - ' /I - -
For Ac = 4, / m = < 2 x 2 0 A. This gives the maximum value (h e re /max is used) of the peak
of resultant m.m.f. wave as
60
sin
f 1pm ~ • [5 x 4 V2 x 20] = 687.687 AT/pole
30 x
180
R.m.s. value of this peak is given by
4 3
F \P = ~ ~ • (5 x 4 x 20] = 486.342 AT/pole.

3 'pkaSe balanced wind™S * considered to consist o f


current sheet o f uniform density. Trace out the m .m .f variation due tophase-a belt alone.
n case this machine has 300 full-pitched turns with phase current o f 24 A, then calculate
[a) peak amplitude o f the m.m.f. wave
(b) peak as well as r.m.s. value o f the fundamental m .m .f wave.
3 s /T h ^ r n r r J f 6 beltS ° f a 2'Pole>3'Phase machine are shown as ac' b a' cb ’ in Fig.
Likewise * k “ as™ med towards the reader and is therefore indicated by dot.
•" a ° !? Sh° r y Cr° SS- 11,8 m m f variation due to uniform current
currenTih A T *7 refernne to FiS- 3 -30 (a). The m.m.f. is zero at the middle of
current sheet and vanes uniformly over the spread of phase-belts. In between the phase-belts
varifrionThus^btalnldhasT11 'fcU£ bp is Positive *this ^ as expected. The m.m.f.
van at ion thus obtained has trapezoidal waveform as shown in Fig. 3.34.
(a) Peak value of uniform current density

J= <21
phase spread of one belt
300
Series turns per phase, Nph = —^ = 1 00
3
Phase spread, a = 60°
... J 100 X V2~x 24 2400 x 3 x <2
AT/rad
60 x
180

Peak amplitude of trapezoidal m.m.f. wave = - J o


jfi
b
r § ~ r T~® T 1 1

.canned by Cam Scanner


798 Electrical Marhiner)

‘ 1 2400 x 3 x V2 n
= - X Q
2 n J
= 1200 x <2 = 1696.8 ATs/pole.
Peak value o f fundamental m.m.f. wave, from Eq. (3.56), is
. o
sin
• [Peak value of trapezoidal m.m.f. wave]
a/2
60
sm
2 [1200 x V2] = 2063 06 ATs/pole

30 * 180
r — F
R.m.s. value of the fundamental m.m .f. wave - ^ Xpm

= 1 x - [1200] = 1459.025 ATs/pole


n n
_ , M m f waveform of commutator m achines. The armature winding of all the com-
3.6.4. M anX wav because of the necessity of commutator action. Fig. 3.35 (a) il-
mutator machin - mutator machine of both a.c. and d.c. types. In this figure,
lustrates the cross-sec * p d d ts direction of current in any conductor is in one
ourent dtrechons are showri by ^ of current * it gets reversed, see Art.

j T l t a t o direction of current as shown under each pole, remains unaltered despite Uie rotor
movement The m.m.f. established by the armature currents is always perpendicuW tothe a.aia.Seld
Z aTrfmwn Since the current direction under each pole remains unaltered the uadoraily dis-
rtbuted armature winding of Fig. 3.35 (a), may be replaced by a uniformly distributed oarenWiert
wrapped around the rotor as shown in Fig. 3.35 (6). Now according to the current sheet concept, all
rnecoWuTtors under each pole are equivalent to a u n i f o r m l y totnbutodsuigle
the whole current under each pole. The developed diagram of Fig. 3.35 (6) is shovm in Fig. 3.35 (c),
where current sheet with cross is shown positive and that with dot is shown negative.
If the total number of armature conductors is Z, then for a P-pole machine,
under each pole are ZIP. For a conductor current it, the total current u n i f o r m ly distnnnea
under each pole is {ZIP) iz. Since the spread of each pole is equivalent to n electrica ra ians,
the magnitude of uniform current density J under each pole is given by

J=-
1 zp iz Ampere conductors per electrical rad.
Current
A rm a tu re sheet
m a g n e t ic a x is
or q- a x is

M a in field
a x is o r
D ire c t
a x is

(Q) (6 )

Fig. 3.35. Two-pole commutator machine (a) its cro98-section and


(o) current sheet replaces armature winding
Art. 3.61
i ? ± C o n c ! pt! r f R ^ , , in(! FJ(ctr.cj|
Machines 299

— 3 -r
J '" : ; U n ifo rm , ,
c u rr e n t lc >
sh e e t

— (d )

q -a x is d - a x is q -a x is d -a x is q - a x is

Fig 3.35. (c) and (d). Pertaining to the m.m.f. wave produced by
the current sheet of a commutator machine.

By referring to Fig. 3.30 (a), the m.m.f. wave produced by the constant current sheet in Fig.
3.35 (c) is seen to be a triangular m.m.f. wave*. If i is the total current at the armature ter­
minals, then for a parallel paths, i / a is the current in each conductor. Since the commutator
winding has a phase spread of o = n radians, the peak of the triangular m.m.f. wave, as per Fig.
3.30 (c) and Eq. 3.64, is given by
1Z 1
-1 Jr o = "1 n
2 2 nP a
\ —2
* I n o r d e r to s h o w t h a t t h e m .m .f. p r o d u c e d in F ig . 3 .3 5 (c) is a t r ia n g u la r m .m .f. w a v e , o n e m a y p ro c e e d
a s f o llo w s :
C o n s i d e r t h e d o t t e d c lo s e d p a t h 1 in F ig . 3 .3 5 (c), w h ic h e x t e n d s o n e p o le - p itc h fro m o n e 17- a x is to th e
a d ja c e n t 9 - a x is . S i n c e t h e c u r r e n t e n c lo s e d b y p a t h 1 is in d ic a t e d b y d o t, t h e t o t a l c u r r e n t e n c lo s e d is - - or
P a
Jn a m p e r e s . T h i s p a t h t r a v e r s e s t h e a i r g a p t w ic e , t h e r e f o r e , th e m a g n e t ic p o te n tia l d iff e r e n c e a c r o s s e a c h g a p

is ^ ^ — 0 r \ j n a n d t h i s is p lo t t e d a s s h o w n in F ig . 3 .3 5 (d ). N o te t h a t ^ — is s h o w n a s p o s itiv e u n d e r b r u s h
2 r a & 2, r a
B a n d n e g a t i v e u n d e r b r u s h A, t h is is in a c c o r d a n c e w i t h r ig h t - h a n d g r ip r u le , s e e F ig . 3 .2 8 (a ).
N o w co n sid e r a n oth er path 2 , exten din g one pole-pitch again, n /4 radians away from both the 9 -axes a s
3 Z i 3n ,
sh o w n . T h e t o t a l c u r r e n t e n c l o s e d is — — — w i t h d o t s a n d - 7 ; - |= w it h c r o s s e s . T h u s th e n e t
4 r a =T

* 1 , . Z i [3 n = — - a n d t h e m a g n e t i c p o t e n t i a l d i f f e r e n c e a c r o s s e a c h g a p is
c u r r e n t e n c lo s e d is — -
P a 4 4 2 Pa
\ Z i t k ' ; t h i s is p lo t t e d a c c o r d i n g ly in F ig . 3 .3 5 (d). F o r p a t h 3 . e x t e n d in g fr o m o n e d - a x is to th e a d ja c e n t
4 Pa - 7 v

d-axis, t h e c u r r e n t e n c l o s e d is £ w i t h d o t s a n d s a m e c u r r e n t is e n c lo s e d w i t h c r o s s e s , t h e n e t c u r r e n t

e n clo se d , t h e r e f o r e , is z e r o . C o n s e q u e n t l y t h e m a g n e t ic p o t e n t i a l d iff e r e n c e a c r o s s e a c h g a p is z e r o a s s h o w m
A lin e p a s s i n g t h r o u g h t h e s e p lo t t e d p o in t s g i v e s t r i a n g u l a r m .m .f. w a v e w i t h it s p e a k v a l u e e q u a l to

1 2 * f= - «/) B u t J - - - ~ a n d t h i s g i v e s a g a i n t h e s a m e p e a k v a l u e o f t r i a n g u l a r m .m .f. w a v e a s .
2P 2 | nP a

nJ (\ Z i } 1Z i ...(3 .6 5 o)
2 n Pa 2 Pa

S c an n ed by C a m S c a n n e r
300 Electrical M a c h in e r y .

_ AT/pole
"2 Pa F
, • a the Deak of the triangular m.m.f. wave becomes A . i
For a dc commutator ma ^ ^ ^ armature terminals. For a single phase com-

S t S S * of the triangular m.m.f. wave becomes

_Z 1 — AT per pole, where / is the r.m.s. value of the total armature current.

2P a -^2 Pa triangular m.m.f. wave, obtained by Fourier-series


The fundamental sine c0™Pon* , yalue of the triangular m.m.f. wave). Therefore, the
“ S e S ofthe fundamental* s in e component F„ for a d.c. machine [Fig. 3.35 «f)|, is

(z O __8'ELlAT/pole ..(3.66)
12P a j n2KP a
/

^ ^ p ^ f° r 3 Single' PhaSe C° mmUtat0r ^ ;


[Fig. 3.35 (d)] is
' ” 8^2 . ^
E . — AT/pole
AT/Dole -(3 67)
F -A
° n2 2P
Here also W is the total number of a r m a tu r e ih m ^ the armature
In a dc machine, armature currenti oas mude and.doesnot depend on the armature
wave for a d.c. machine remains con , ; tb armature m.m.f. wave, uneffected by
speed. In case of single-phas,^commutator machine, the^a -mat

ponent, coincide with the brush axis or g-axis. ^ two poles

In actual practice, dc machines may h*ve * d/m ach in e with four poles is shown
on the stator. In Fig. 3.35 (e), schematic diagram of ^ C ^ C ^ ^ teraate Z rth and south
These poles on the stator are produced by “ ” "8 “ belts) of slots with c u r re n ts
polarity. Armature conductors are distributed in four bandsit ^ developed view
indicated by cross under S poles and dot under N poles. Fig. . f) is etched by
of Fig. 3.35 (e), where the triangular armature m.m.f. wave f° ^ . e f° “ J L ment „f field poles
referring to Fig. 3.35 (c) and (d ). As the dc machine has symmetrical arr g

Halong
1^air_goP
N peripniU
Uniform
c u r r e n t sheet
Armature
m m f wove

(/)

>canned by C a m S c a n n e r
j£:
Basic Concepts o f Rotating Electrical Machines 301
A * ill
and armature winding around its periphery, the m.m.f. and magnetic circuit conditions under
one pole-pair repeat under the next pole-pair.
3 ,7 . Rotating Magnetic Field
When balanced polyphase currents flow in balanced polyphase windings, a rotating mag­
field is produced. In other words, all polyphase a.c. machines are associated with rotating
netic
magnetic fields in their air-gaps. Consequently, a knowledge of the rotating field produced by
a polyphase winding is essential for understanding the theory of polyphase ac machines such
as polyphase synchronous and induction machines. As three-phase machines are more com­
t .
mon, attention will primarily be focussed on the production of rotating field in these machines.
For a 3-phase machine, the three windings are displaced from each other by 120 electrical !! .
space degrees along the air-gap periphery. Fig. 3.36 (a) illustrates a 2-pole 3-phase stator wind­ *!
:
ing. Note that each phase is distributed or spread over 60° electrical (called phase-spread a),
under each pole. For convenience, the three-phase winding a, b, c is represented by three full
pitched coils, aa\ bb\ cc' as illustrated in Fig. 3.36 (6). For instance, the concentrated full-
pitched coil aa' represents phase a winding in all respects. A current in phase a winding estab­ i
■i i
lishes magnetic flux directed along the magnetic axis of coil aa'. Positive currents are assumed
to be flowing as indicated by crosses in coil-sides a', b', c'. It means that when phase a alone
carries positive current, the flux produced by this phase is directed horizontally from left to
right - if phase a current is negative, the flux produced is directed horizontally from right to
left. The three-phase currents flowing in the three-phase windings, are varying sinusoidally
with time as depicted in Fig. 3.36 (c). In order to have better physical insight into the production
of rotating magnetic field, graphical analysis involving magnetic-flux plot as well as space-
phase* b’ Phase c’ Axis of
w inding winding phase lc’

Axis of
'phase'a

P h a s e ‘a ’
w inding

I I I

A t

I 'A
'P y
- j.
- -.- I !
' • • j

a•
1
Fig. 3.36, (a) 3-phase winding space displaced by 120 degrees electrical, from each other,
(b) 3-coils aa', bb’, cc' represent three-phase winding and (c) 3-phase instantaneous currents. 1
I

1 '*** i

) u c i i i i i t : u u y v^ai i i o u c i i ii itri
302 _________________________________________________
Electrical M achinery

phasor representation will b e .t h r e e - w ^ ^ n ^ "


air gap of a 3-phase machine, is due to tne cornu... t «uxcs.
G raphical an alysis-m agn etic flux plot. At the instant 1, [Fig. 3.36 I(c)l, the current in
phase aTs^osithra and maximum, say / . and this current ,s represented by cross and d * i„
c o il-sid e sA n d a respectively in Fig. 3.37 (a). Currents * and are both negahve and equal l0

it ie ib = i = - i t . Since b nd i, are negative, crosses must be s


ia

d lls in coil-sides V.<f. Right hand grip rule gives the distribution of flux at instant 1, as shown

in Fig. 3.37 (a). Note that two poles are created on the stator. At instant 2, i„ = - * =, - = and

i =_ / Therefore, crosses most be shown in coil-sides a’ b' and c [Fig. 3^37 (b)\ and the resul-
0 .* ’ . fn„ nj tn as shown in Fig. 3.37 (b). Note that two poles are created
!“ u » " ° o nf the resultant flux has turned clockwise through 60 electrical degrees in

space. At instant 3, i_ = - ' f , i„ = K and Therefore, the crosses and dots are as shown
in Fig 3 37 (c) and the 2 poles produced by the resultant flux are seen to have turned through
r f W fin P prtrical soace degrees. At other instants of time, i.e. as the time elapses, the two
poles rotate further. In this manner, a rotating magnetic field is produced. Note that the space
anrie traversed by rotating flux is equal to the time angle traversed by currents. In one cycle,
the two piles musl come to the position indicated in Fig. 3.37 (a). Thus m a 2-pole machme, the
A x i s of v
r e s u lt a n t *“\ ^ - l n s t a n t 2

(b)
Fig. 3.37. Production o f rotating magnetic field illustrated by magnetic flux plot,

rotating field travels a distance covered by two poles (i.e. two pole pitches) m one cycle. For a
_ mm .i i t• /* 1J __ ill i. 4A/2
4-pole machine, the rotating field will travel1 a distance
J-I ni a n/nimt'Qr]
covered by ftwo
Wfl T”) fll P.S.
poles, i.e. ^a /O y

vered by
revolution in one cycle. For a 6-pole machine, the rotating field will travel a distance cov
. l f — 1 revolution in one cycle and so on. This thought process reveals
two poles, i.e., —
6/2
l . i d therefore
the rotating field speed, for a P-pole machine, is revolution in one cycle a

-zf-z revolutions in f cycles — in other wordsrevolutions in one second, because f y


p/2 P/2 t Tfn denotes tne
completed in one second. Here f is the frequency of the 3 phase currents. s
rotating field speed in revolutions per sec, then
...(3.68)

:• fm

S c an n ed by C a m S c a n n e r
M - WQf
or " “ ~p r-P-in. U 3m

The speed at which rotating magnetic field revolves is called the synchronous speed.
The graphical analysis presented above brings out clearly the production of rotating mag­
neticfield, but it fails to establish that the magnitude of rotating flux remains constant at all
instants of time. The graphical analysis given below, proves this fact.
Graphical analysis-space phasor representation. When phase a alone carries cur­
rent, an m.m.f. is established along its magnetic axis. Neglecting space harmonics, the m.m.f.
issinusoidally distributed along the air-gap periphery. Further, phase a m.m.f. wave is station­
ary but pulsating in nature. For convenience, this sinusoidally distributed m.m.f. is repre­
sented by a space phasor Fa directed along the axis of phase a. Note that the magnitude of Fa
is proportional to current ia and is directed from left to right with positive ia and from right to
left with negative ia (Fig. 3.38). In other words, the space phasor Fa is oscillating along the axis
of phase a. Same state of affairs exist, when phase b alone or phase c alone carry currents.
When currents ia, ib, ic flow in their respective phase windings, then the three stationary pul­
sating m .m .fs. Fa, F b,F c combine to give the resultant m.m.f. FR which is rotating at
synchronous speed.

©c\

'1 I ^R=2Fnri
J A

p --1 F
R2 m ..
(a) (b) C (c)
Fig. 3.38. Production of rotating magnetic field illustrated by space phasor m.m.fs.

For obtaining the resultant m.m.f., refer to Fig. 3.36 (c). At the instant 1, the cunent in phase
a is positive and mt ximum, i.e. ia = Im. This current sets up maximum m.m.f. Fnl [ = F lpm of Eq.
(3.59)] in phase a. This m.m.f. Fmis represented in Fig. 3.38 (a) by space phasor Fa = F malong the

axis of phase a. At the same instant, ih = ic = - y and these are indicated by crosses in coil-sides

b, c and dots in coil-sides b\ c'. The m.m.f. phasors Fb and Fc are along their respective magnetic
. . . I,n .
axis, but are in the negative direction as shown in Fig. 3.38 (a). Since ib = ic = - the magnitude

of both Fb and F c is equal to Fm/ 2. The resultant of m.m.fs. Fa,F b, Fc in Fig. 3.38 (a) is FR and its
magnitude is given by

2F" cos - 3 rFn


m s 60 =

,R =
= Fm
r nt+
'r ~ - n
2‘ m

The vertical component of Fb and Fc cancel each other.

Note that at instant 1, current in phase a is maximum and the resultant m.m.f. FR = ^ Fm,

air«cted along the phase a axis, is sinusoidally distributed in space

cann ed by C a m S c a n n e r
304 Klectrical Machinery------------------ J?

At insun. 2, Fig. 3.36 (c). <- ~ Th* " " f PhaS" ^

to ^ in magnitude, are along their respective magnetic axis in the positive d„ec„0„ ^

m m f. F( = F „, in phase c, is indicated in the negative direction, butalong the phase

amplitude of the resultant m m f. F „ is again found to be equal to ^ but it is seen to ^

by a space angle of 60° clockwise.

At instant 3, ia = *c = “ jjT and Ife = Inr The m m f PhaSors F°' Ff>, Fc are as shown in Fig. 3 3g
3 .
(c) and the amplitude of resultant m.m.f. FR is again ^ Fm directed along phase 6-axis. In other

words, the resultant m.m.f. has turned through a further space angle of 60° from its position
occupied at instant 2. At other instants of time, i.e. as the time passes, the resultant m mf.

revolves further, but its amplitude remains equal to ^ Fm. In this manner a constant-amplitude
rotating m.m.f. or rotating field is produced in the air-gap of a three-phase machine.

Note that at instant 3, current in phase b is maximum and the resultant m.m.f. FR= - F„
2
is directed along the axis of phase 6 . It may be summarised from above that resultant m.m.f.
3 1 > 1
FR = - F m(or peak o f rotating m.m.f. wave) is directed along the axis o f that phase which carries
£
the maximum current at that instant. Fig. 3.38 reveals that rotating m.m.f. revolves from phase
a towards phase 6 and then phase c in case the phase sequence of supply currents is abc. A
reversal of the phase sequence of supply currents will reverse the direction of rotation of the
resultant m.m.f. wave.
The speed of this rotating field, as shown above, is given by Eq. (3.68) or Eq. (3.69).
Sinusoidal rotating mmf wave creates in-phase sinusoidal rotating flux-density wave inthe
air gap. With negligible reluctance of the iron path, the peak value of B -wave is given by
to fu 3 d
8 2 gP
The resultant mmf FR crosses one air-gap length g. The expression for the flux per pole
rotating at synchronous speed, from Eq. (3.11), is given by

a = — . /. r ^ MoFm _ 6 Fm
e . . p 2 * =P ~ J ~
Substituting the value of Fm from Eq. (3.59), the expression for the fundamental flux pe-
pole is given by

6 |Vr/ 4 Nph n/27


P g n
• * » ' p —

- ^ 2 .3 7(n
M or lk,„NphI '3 ™

The nux given by Eq. <3.70) ie total (lux per pole rotating at synchronous speed.

amplitude merely Ph“ 8C m m f 'S n0t “ r° tat'ne


all the three phase m m r« ih . ? mB its own phase axis. It is the combined
c tnrte phase m.m.fs. that results in constant amplitude rotating m.m.f. * » * •

S c an n ed by C a m S c a n n e r
f B a s ic C o n c e p t s o f R o t a t in g E le c t r i c a l M a c h in e s 305
---------------------------------------------------------------
. foregoing analysis can be extended to show that a rotating m.m.f. wave of constant
| jjtude Fm is produced by two phase currents flowing in two-phase winding. In general,
^hase currents flowing in m-phase winding results in a rotating m.m.f. wave of constant
fft'P fj7
mplitude equal to F m. Note that F m is the maximum m.m.f. of any one phase when current

i3maximum in that phase.


Thus the effect of polyphase currents in polyphase windings is equivalent to the mechanical
fotation of permanent magnets, or d.c. excited field poles, at synchronous speed.
3 i
In the expression of resultant rotating m.m.f. wave FR = -^Fm, Fm is the m.m.f. per pole

mrresponding to maximum current Im and its value is given by Eq. (3.59). If this value o fF m is
3 •
substituted in FR = — Fm, then the amplitude of rotating m.m.f. wave, in terms o f series turns

per phase N p/„ r.m.s. current I etc., is given by


^ 3 „ 3 Nphl Am y ,
4^2 ,
F r = g F>» = 2 T " W~P~' ATS/P° le

= 3 ATs/pole -( 3 .7 1 a )
n r

=2
.7 k
,^A
Ts/pole -'3 .7 1 A)

For m-phase machine

FR = m kw ~^p~ ATs/pole

- 0.90 m ATs/pole ...(3.72 6)

In the foregoing analysis, only the fundamental component of the m.m.f. produced by each
phase of a 3-phase winding has been considered. If the space harmonics are also taken into
account in the single phase m.m.f. waves, then the resultant m.m.f. produced by 3-phase cur­
rents flowing in three phase windings, will consist of the following components :
(t) Constant amplitude fundamental m.m.f. wave rotating in the air-gap at synchronous
speed.
(ii) Space harmonics of the order of 3, 9, 15...... i.e. triplen harmonics are absent.
(iii) Fifth harmonic m.m.f. wave— sinusoidally distributed in space with its amplitude con­
stant and equal to one-fifth of that of the fundamental component. Constant amplitude fifth
harmonic m.m.f. wave rotates in a direction opposite to that of the fundamental component and
at a speed of one-fifth of that of the fundamental synchronous speed.
(iv) Seventh harmonic m.m.f. wave-sinusoidally distributed in space with its amp itude
constant and equal to one-seventh of that of the fundamental component. Constant amplitude
•- seventh harmonic m.m.f. wave rotates in the direction of the fundamental component and at a
speed equal to one-seventh of that of the fundamental synchronous speed.
In general it may be stated for nth harmonic m.m.f. waves that these are sinusoidally
distributed in space with their constant amplitude [refer to Eq. (3.71 6)] equal to

- 1 In n J. N p h ] _ A T s /p o le
2.7 kwn
F"~~n
I is the constant amplitude o f nth harmonic component and kwn is the winding factorfor-nth
\ harmonic. Note that n can be equal to 6k ± 1 only, where k is an integer 1, 2, 3 Space

uy u a i 11j o a i ii ic i
306 Electrical Machinery

harmonics of the order of 6* + 1 (7, 1 3 ,1 9 ....... ) have them constant amplitude equal to? ^

and rotate in the direction of fundamental component at a speed of “ I of the ^ m e n t a l

synchronous speed. Space harmonics of the order of 1 ( 5 ,1 1 ,1 7 0 have their coastam

amplitude equal to ^ F t and rotate in a direction oppos.te to that of the fundamental m.m.t

wave ; at a speed of of tire fundamental synchronous speed.

E xam nle 3 21 When single-phase winding is excited by an alternating current, a pulsating


m m f wave is produced. Show that this stationary m.m.f. wave can be resolved into two con-
U a n ta Z l t t u d e travelling m.m.f. v a o es rotating in oppostte d tr
Solution It is seen from Figs. 3.30 (c) and 3.31 (6) that when smgle-phase wmdmg having
N h turns is excited by current i. trapezoidal m.m.f. wave is produced. The fundamental com­
ponent of this m.m.f. wave is sinusoidally distributed along the a,r-gaP periphery and » gnen
by Eq. (3.57 (6)) as
4 N ph . n
= 2 ^ -< ■ cos a ...W

If single-phase winding carries sinusoidally varying current i= / maK cos CO/, then m.m f. dis­
tribution, from Eq. (i), is

Fal = - k w ^ L l max C0S C0S 01


71 tr
= F m cos a cos (£>t •••(**)

v
where rpm~K
_ 1 . Kw
b • p •Ixmax ...(u’i)

Eq. («) is the same as Eq.(3.60 c) and F m as given by Eq. iiii) is equal to F lpmof Eq. (3.59).
As stated earlier, F m is the m.m.f. per pole correspondingto maximum current 7max. As exciting
current varies, magnitude of F m also varies accordingly, but its axis remains along the magnetic
axis of single-phase winding.
In Ea. (ii), cos a indicates that m.m.f. is sinusoidally distributed in space along the air-gap
periphery and cos (tit implies that m.m.f. amplitude varies sinusoidally in time at frequency (o
as shown in Fig. 3.31 (b ).

By using the trigonometric identity, cos a cos b = ^ cos (a - b ) + ^r cos (a + b), Eq. Ui) can be
Z Z
re-written in the form
t-i 1 tx 1 " (iv)
F al = 2 m C° S ~ ^ + 2 F m C° S ((X +
= Ff + F b

Here F f= ^ F m cos (a - (tit) represents a rotating m.m.f. wave travelling in the forvva

(positive a) direction at synchronous speed (o. The other com ponen t^ = | F nl sin {(& + « ) als
a rotating m.m.f. wave but travelling in the backward (negative a) direction at synch*0,101
1 v nf the
speed co. The amplitude of both Fyand F b is ± F in which is half of the amplitude 01
z
sating m.m.f. wave.

Ic a n n e d by C a m S c a n n e r
Basic Concepts o f Rotating Electrical Machines 307

Pulsoting
mmf

F ig . 3 .3 9 . T o ta l m .m .f. o f s in g le -p h a s e w in d in g re so lv e d in to tw o c o u n te r -r o ta tin g m .m .f w aves.

A physical picture of the two counter-rotating m.m.f. waves can be gained from Fig. 3.39, where
ingle-phase winding is represented by one coil aa on stator. When i = Imax at cut - 0, amplitude ot
n.m.f. is Fm along the axis of single-phase winding as shown in Fig. 3.39 (a). The m.m.f. Fm is

esolved into Ff = F b = ^ F m as shown. Both forward and backward m.m.f. waves travel at
ynchronous speed in opposite directions. At angle wth i = •cos <of,, pulsating m.m.f.

=-F and Fb travel through angle 10^ but the


fhplitude is OA = F m cos a)tx. Both Ff 2 m
lasor sum Ff + Fb = 0 A along the magnetic axis of single-phase winding, Fig. 3.39 (6).

At otf2 = - , current is zero, therefore pulsating m.m.f. is zero.


2
( are not zero. However, F f+ F b - 0 as shown in Fig. 3.39
/ 1 x
B u tF , = - F and Fb = - F m
2 m 2

S c an n ed by C a m S c a n n e r
108 Electrical Machinery ~~
----------------------- T T T ^ s i n g l c - p h a s c winding when excited by an alternating
It must be kept in mind that sing p waves. It always produces one stationarv nt
does not produce two counter-rotating • decomposition of the m.m.f. of a s in g ly ,* 'f
wave pulsating along its magnetic a . . . thi js a very important concept whirl, u
winding into two counter-rotatmg n tn rr w a v ^ _ P wh)ch ^
in the understand,ng of a , , e l ^ t r . c a ' mach.nery „ ^ ^ ^ ^ ^

E xam ple 3.22. A sm8 le'pf ° ? * " I " ? 1''decom posed in to tw o cou n ter rotating
one pulsating m.m.f. wave w ' concept to p rove that a rotatin g magnetic field of"**

currents. • .
Solution. A three-phase rotating electrical machine is woundI with three windingsQ fc c
displaced from each other by a space angle of 120 as shown in Fig. 3.36 (&)• Consider that these
windings are excited respectively by three-phase balanced currents given by

ia = Lax C0S
ih = /««,, cos ( o y - 120°) ...Ii)
= I m ax C0S " 240O) .
These currents in phase windings a, b, c set up three pulsating m.m.f. waves in the air gap
These m.m.fs are directed along the magnetic axis of phases a, b, c as indicated in Fig. 3.36 36
(6 ). For the currents given by Eq. (i ) and with the magnetic axis of 3-phase winding displaced
in space by 120°, the three m.m.fs Fa, Fh and Fc can be expressed mathematically as
Fa = Fm cos a cos mt (CM Sf au
Fh = F,„ cos (a - 120°) cos (otf - 120°) H <*!>(<' ' V ^
Ft = F,„ cos (a - 240°), cos (i»t - 240°) k>
Note that angle a is measured from the axis of phase a as shown in Fig. 3.36 (6). As in
Example 3.21, the pulsating m.m.f. of phase a canbe expressed as

Fo = 2 F>n cos (« - orf) + | Fni cos (a + mt) ...(in

Similarly, for phases b and c,

F b = g F,„ cos (a - 120 ° - wt + 120 °) + | Fm cos (a - 120 ° + cuf - 120 c)


1 „ 1
- 2 cos (« - cof) + - Fm cos (a + cof - 240°) ...Inn

Fc = 2 F>n cos (a - 240° - on + 240°) + | Fm cos (a - 240° + aV - 240°)


_ 1 „ 1
- 2 *•* cos <a ~ w/) + — Fm cos (a + (lit - 120 °)

(i), ( i D X T m m f' F r (a’ ^ can be obtained by adding the three m.m.f.s given by Eq*

F r (a, t ) = F a + Fh + Fc

“ 2 F.„ Icos (a - CO0 + cos (a + cue) + cos (a - a t) + cos (a + c.K - 240’ )

3 + cos (a - oK) + cos (a + uX - )20 '*


2 », ls (o OK) + F ,„Icos (a + ,.X) +

-*■ rim 1(1 ♦ aX-M®'1'

's c a n n e d by C a m S c a n n e r
Basic Concepts of Rotating Electrical Machines 309

But g Fm cos (a + 2 ^ m cos + ~ 120°), ~ Fm cos (a + (at - 240°) represent three

^in.f sinusoids displaced in phase by 120°. Therefore, there sum must be zero, i.e.,

2 F m (cos (a + (at) + cos (a + art - 120) + cos (a + utf - 240)) = 0

Eq. (u). therefore, reduces to

F, (Oi t) = - Fm cos (a - (at) ...(vi)


■j
It can be shown that Eq. (vi) represents a travelling m.m.f. wave of constant amplitude -

L-
» fr-
At (at = 0°, Fr (a, t) -j

At (at = 45°, FR (u ,t) =

At (at = 90°, F r (a., t ) = | Fm cos (a - 90°) = | Fm sin a I

In the above three expressions for FR (a, t), put different values of a and plot these three i i
waves with a as the base. These waves are shown in Fig. 3.40 for different instants of time
corresponding to 0°, 45° and 90°. It is seen from this graphical plot that m.m.f. wave is of con-
3 ,
stant amplitude -= Fm and is travelling in the positive a direction at a speed determined by the
Z
time angular frequency (a.

mmf tot=0 wt*90


J u>i
Ut=45 Speed , cj

Space
ongle,«t

D i r e c t i o n of
a. travelling wave

Fig. 3.40. A graphical plot indicating that | Fmcos (a - wt) is a constant


amplitude m.m.f. wave travelling in the positive a direction.

Thus the snace anele a in Eq. (vi) means that the m.m.f. wave at any moment is sinusoidally
spare thong the air-gap penphe^. Tke hme “ e S
•sveform is travelline alone the air-gap circumference at a constant angular w - Zitf electrical
(called s£>chronous speed). For a E-pole machine, the synchronous speed <o„.

^ E q . (3.3), is
o) = -| to = ^ rad/sec
u ,m p p

o c a n r ie u uy u a m o c a r i n e i
3.7
310 Electrical M a c h i n e r y -------------------------------------------------------------------

^ros
or « . = p rPS

or N .^ r p m

m ln u n , .Eq. (0. and p e a k o f ^

m.m.f. wave ^ F m is along the axis of phase a,

(if)when oot = 120" and a = 120”, current 4 is maximum and peak oftravellmg m.m.f wave

is along the axis of phase 6 ; because, from Eq. (vi). F r = 2 F" C0S (12° ° " 12° ° ’ = 2 F"
m Similarly, when cot = 240” and a = 240”, peak of travelling m.m.f. wave ts aligned aloe*

the axis of phase c. of rotating m .m .f wave travels from phase a, to phase


6 and S o 8'phase" he clockwise in Fig. 3.36 (6)1 when phase sequence of three-phase supply

is q bc
Example 3.23. In a uniform air gap Inon-salienl pole) machine show that the space har­
monics present in the rotating m.m.f. wave, generate only fundam ental frequency voltages in Ihe
stator winding.
Solution. Flux = m.m.f. x permeance
Neglecting magnetic saturation, the permeance offered to the magnetic flux is constant for
a uniform air-gap machine. Thus the flux is proportional to the m.m.f. In view of this, nth space
harmonic m.m.f. wave will produce nth space harmonic flux wave.
Let the speed of the fundamental component of rotating flux wave be N sV Then speed of the

nth harmonic flux wave = - N sl.


n
Now number of harmonic poles, produced by nth harmonic
= n (poles P, produced by the fundamental flux wave) = nP
Frequency of the generated e.m.f. in the stator winding, due to fundamental rotating flux
wave having P poles is given by
(Relative speed) (Poles P)
N s\P
120 = 120
Frequency of the generated e.m.f. in the stator winding, due to nth space harmonic rotating
m.m.f. wave having nP poles is given by
(Relative velocity ) ( Harmonic poles)
120

(nP)
n Nt l P
~120 ~ 120
This
ABMBtJ proves
| V l V T W O the
UIIVw required result.
1VwVj U I I t u 1CO U lt.

Example 3.24. Find the amplitude o f m.m.f. produced by a squirrel cage winding' ^
Solution. A squirrel cage winding, as described before, consists of one bar in each slot
these bars are short-circuited by end rings at both the sides. If rotor has S slots, then
S bars or 8 conductors. In effect S bars are equivalent to f phases, each phase having 1

TTUzjrvrKTcrrjj ^u n roL ai n ici


Basic Concepts o f Rotating Electrical Machines 311

^ll.pitched turn. Since the winding factor of single full-pitched turn is unity, the amplitude of
^ultant m.m.f. wave, from Eq. <3.72 b) is

F r = 0.90 | ATs/pole

= 0.45 ~ ATs/pole

Here / is the r.m.s. value of the bar current.


Example 3.25. Three-phase voltages are applied to the three windings o f an electrical
machine. I f any two supply terminals are interchanged, show that the direction o f rotating
tn.nt f wave is reversed, though its amplitude remains unaltered.
Solution. For this example, either graphical or mathematical analysis may be used. For
better physical concepts, first mathematical and then graphical analysis are given here.
The m.m.f. for phase a, pulsating with time and stationary in space, can be written by
referring in Eq. '3 .6 0 c) as
Fa - Fm c o s oX c o s a .
Suppose the supply terminals to phases b and c are interchanged, then for phase b, the
space angle displacement remains at 120c, but time phase angle must now become 240° instead
o f 120°. In other words., cos o X - ~ f o r phase b m . m . f ,
must be replaced by ccs u it
3
because supply terminals 6 and c arc^interchanged. In view of tnis, phase b m.m.f. Fh should
be written as

Fh = Fm cos OX - — 2*0
cos a - "7T
3 3 J
> ^
m.m.f. as

Fc = Fm cos ( . 2nl
3”
COS f 4,0
a"T
v „ ^ /
The resultant of three pulsating m.m.f. waves is
F r ( a , t) = Fa + Fb + Fc
4n 2 jO 4n
F rr cos ayt cos a + cos 03/ -
=• cos (la - ~T
2n ] cos 03/ - cos a -

Fm
= (cos (a - oX) + cos (a + aX) + cos (a - 03/ - 120 + 240) + cos (a ♦ oX - 2x)

+ cos ( a - oX - 240 + 120) + cos ( a + 03/ - 2x)l


But cos (a + 03/ - 2rr) = cos (a + 031 ) and cos (a - 03/ + 120°) = cos (a - 03/ - 240°)
Fm
F r (a, t ) = — Icos (a - 03/) + cos (a + 01/) + cos (a - 03/ - 240°) + cos ( a + aX)

+ cos (a - 03/ - 120) + cos (a * oX)]


Now cos (a - 03/) + cos (a - oX - 120) + cos (a - oX - 240) = 0

Fr (a , / ) = g cos (a + oX) ..(i>


3
A plot of Eq. (/) shows that the resultant m.m.f. is of constant amplitude -- Fm, but it is

Celling in the negative a direction, Fig. 3 .41 (a). If the phases and c were not interchanged, b

***« the m.m.f. would travel in the positive a direction as depicted in Fig. 3.40.

S c an n ed by C a m S c a n n e r
312 Electrical Machinery

(/»
F ig. 3.41 P e r t a in in g to E x a m p le 3 .2 5 .

This proves that if any two supply terminals are interchanged, the amplitude o f rotating
field remains unaltered, but its direction of rotation is reversed.

For graphical analysis, refer to Fig. 3.36 (c) and Fig. 3.38. At instant 1, ia = /,„, ih = ic = _ J£ Since
2
t/, and ic are interchanged, ih = ic = - and the resultant m.m.f. is directed along the phase a

axis, a s shown in Fig. 3.38 (a). At instant 2, = 'f,i,, = ' f and I, = - /,„. After s u p p ly terminals

of phases b and c are interchanged, current - f nows in phase winding c (indicated by cross in
i

c' and dot in c) and current - /,„ flows in phase winding 6 (indicated by dot in 6' and cross in b),
Fig. 3.41A b) As a result of it, the resultant m.m.f. wave is as shown in Fig. 3.41 (6). Note that
the resultant m.m.f has rotated anti-clockwise through an angle of 60 electrical space degrees.
At instant 3, F R will travel further by an angle of 60 electrical space degrees. This shows that
f termi" a,T
S/ ° 3phase ™ d i n g are interchanged, the direction of rotating
rotating m m f w I v p ^ ih ^upp teri^*na^s phases b and c were not interchanged, then
•rrlnh l i \ W° U ? ,n e clockwise direction, as shown in Fig. 3.38. Thus
graphical analysis also proves the required result.

a b EeXa ^ e ! a l T h v Ft L 3* 4l $hoi“ sf hree ™ ils «. b, c ; with their m agnetic axis coincident. Coils
a, o, c are excited by three phase balanced currents

/„, sin lot, /„, sin . 2n\ 4n\


T and I... sin (Dt -

respectively. Find the magnitude o f resultant m.m.f.


wave.
a b
c

Flg 3 42 ‘ G a i n i n g to Example 3 .2 6 .

Scanned by Cam Scanner


Basic Concepts of Rotating Electrical Machines 313

Solution. The m.m.fs. produced by three coils a, b, c are proportional to their respective
rr e n ts . These can, therefore, be expressed as
Fu = Fm sin co/ cos a
2rt
Fh = F„. sin C 0 < -y cos a

4k ]
Fc = F m sin CO/ - cos a.
and
The three currents are displaced in time phase by 120° or 2 n/3 radians, but the angle of
< p a c e displacement between the magnetic axes of the three coils is zero as shown in Fig. 3.42.

It is due to this reason that cos a appears in all the three m.m.f. expressions given above. Their
resultant m.m.f. is
(a, t) = Fa + F/, + Fc (I
.’ 1

S3
r . . f . 4k^

X
= Fm sin co/ cos a + sin c o / - y cos a + sin co/ - — cos OC
u
L I J -

F r ' 2 k ] 2rc 1
= - y j^sin (co/ + a) + sin (a)/ - a) + sin co/ + a —— sin co/ - a -

471 4k
sin co/ + a - + sin o tf-n -y

{ 2k ( 47c'
sin (o)t + a) + sin co/ + a - y + sin CO/ + a - y =0
But
/

2n>
and sin (co/ c o / - a - y + sin co/, - a - y
471 =0
t-

The resultant m.m.f. is FH= - y 10 + 0) - 0.

Thus the magnitude of resultant m.m.f. wave is always zero. This example demonstrates
that rotating m m.f. wave of constant amplitude can be produced, only if the time angle dis­
placement between currents and space angle displacement between winding axes, are equal
Example 3 27 A three phase star-connected winding is fed from 3-phasc balanced supply,
with their neutrals connected together. (This arrangement results in rotating m.m.f. wave o f
constant amplitude). . .i r
I f one o f the three supply leads gets disconnected, find what happens to the m.m.f. wave.
Solution. Suppose supply phase c gets disconnected. Then in Fig. 3.36 (c), t'r - 0.
7 F i
. . . • t ■ ; - n •t:.orofnre F = F F, = - F,. = 0 -this is illustrated
At instant l,c a = /„1,ift = - - 2" . ' c - u . t‘ -'ercloro' r " r » " rh 2 ’ e
in Fig. 3.43 (a). The amplitude of the resultant m.m.f. wave is ^
p R = V(Horizontal comp.)"* + (Vertical comp.)

(F ^
A Fm + - f cos 60c
2
sin 60

'\'3 r
A 4
— F
4

V7
F... = 1.32 F„r

.■>1

S c an n ed by C a m S c a n n e r
314 Electrical Machinery

Resultant m.m.f. 1.32 F _ at instant 1, is displaced from axis of phase a, by an


angle given
by
rv3 4 )
1 f Vert, comp > = ta n "1 — x —
-
tan 4 5
Horiz. comp
V / 'v /
= tan’ 1 (0.3464) = 19.1°
yj3 . .
At an instant 30° away from instant 1, = *6 = 0, ic - 0, therefore, =
I 2
Fb = Fc = 0, Fig. 3.43 (6). Thus the amplitude of the resultant m.m.f. wave is -~ F m= 0.866 f
H
i
and it is centred along phase a-axis.

Fig. 3.43. Pertaining to Example 3.27.

From instant 1 to instant 30° away from it, the time-phase angle covered is 30°, whereas
the space angle covered is only 19.1°. This means that between these two instants, rotating
m.m.f. has changed its amplitude from 1.32 F m to 0.866 F m and travels at a speed less than the
synchronous speed given by Eq. (3.65).
I F
At instant 2, ia = ib = ~ £ ,ic = 0 and Fa = F b = Fc = 0, Fig. 3.43 (c).

F r = V(Horiz. comp.)2 + (Vert, comp.)2

m
F
1 m _ - ,Q*
— sin 60°
T _ T cos 60

(F,mY2
-V 4 *m
V
The resultant m.m.f. FRat instant 2 is displaced from phase o-axis, by a
-l N 3 ..4 n
tan = ta n "1 (V3) = 60°
4 XT
Wh tnstant 30 away from instant 1 and upto instant 2, the time-angle covered is 30',
r ™ f & 18 6 r ' TJhiS means that between these two instants, the r t *
ng m.m.f. wave has changed its amplitude from 0.866 F m to 0.5 F m and travels f a s t e r than the
synchronous speed given by Eq. (3.69).

f r o n f l r l T o ?■ thare™ 'taat m m f -wave will be found to be 1.32 Fm and it will be seen that
^ “ m o * than 60 .' ‘ lme Ph38e “ " » * *• where”as the space phase angle

produced! W8Ve ° f varying.an! P^ U


,ttnt
synchronous speed given by E q. ,3.69), but at a T p e T f l u c Z t i n ^ u n d ^ s y n ^

>canned by C a m S c a n n e r
Basic Concepts of Rotating Electrical Machines 315
in -ilU — ;------------------------------
^ (.e t^e three-phase winding is delta connected or star-connected without neutral, then
l°nection of one of the supply leads, results in single-phase operation of the machine, i.e.
^,5C° m f becomes stationary and pulsating in nature.
" Example 3.28. Show that self-inductance o f a single N-tum full-pitched coil belonging to a
nlfprm air-gap machine o f gap length g. radius r and core length I is given bx
Mo Wf 2
L=
gP
Saturation and leakage-flux are neglected.
Solution. The magnitude of rectangular m.m.f. waveform from Eq. (3.52) and ig. .
pven by

F -2 'N
■ Uniform flux density over pole pitch.
„ Mo, Mo 1 '
B = — (F) = —
g g 2 "V
Flux per p<»lc. 0 = (area under one pole) <B>
2 nrl Mo Mo nr/
iN Ni
P g gP

NO Mo * r l V 2
Y Self-inductance of coil. L - . = •

s gi ven bx
, Ih^ 1 i. u S

12S \i,/l . ,, phase machine and L„ = L,. -L , - ,


p,tched coils, is given bx L „ * ^ gp3 * ' (,r ° * R

MW
, N ‘ /or a 3 phase machine

, 6 1 , 6 , — / m d iir io n r e - i.« n - ^ 1 — W
gar phase winding with S , series turn s. given by
/f>Mc i j N I

Solution. ,o ) The peak value of ^ n d a m e n - l sineexponent of man f. produced by a


ill-pitched distributed phase winding, from Eq 13 5 6 1...

Mo - Mo 4 . iN
~kd
Now

From Eq. (3 .11>, the flux per pole 0 is 16 Pqrl


‘ 4
4 « 4 , Mo 4 . iS k .N i
-- -- nr lB
Dp = p —
g
Kk d P tigP2
P

“■jV/ai 11 i c u u y w a n u t a i 11 ic i
316 Klcctrical Machinery

r (Effective stator-series turns)J>


Stator self-inductance, L = --------------------- •

I 1 A \I
/ ^ ngP2
J ngF*
(i) For a two-phase winding, the phase spread o is 90
90
sin o /2 sm ¥ 2>/2
k., =
o /2 90 n_ 71
2 * 180
Thus the self-inductance for any one phase of a two-phase machine is

16 Por ^ 2V2 f xr2 1 2 8 Mo^ xt2


= Ln = 5- ----- • Af = — o— o— A r.
ngP2 ( n J Tt^P2
(11) For a 3-phase machine, the phase spread o is usually 60°.
60
sin
« , ________ 3
d 60 k 71
y
2 180

16Mo^ (3 f N 2 = 144 Pqt/ ^


La = Lh = Lc = TV.
n
■ -g
C P
- 2 -v S S

(6) Current is in the stator series turns N s produces a fundamental sine-flux 0, given by
16 |i0r/
> (*</* A^s) is
7 lg P

Mutual-inductance, A/ = M effective rotor turns) _ 16 p0r/ (kHs N s) js


(* rf, A ff.)
1* is ngP1
w 16por/
M = — p < ** w.) <*„, Wr).

metres. t e s t a t o r t t u t a n d laid out ^ 8 ^ hat rotT r ° ^ diameter of 12


m.m.f. wave. Find the linear velocity o f the t r a v e l l i n g 8 ^ W° Ve n° W becomes a travellin6
so lu tio n , speed of rotating m . ^ f wave “ ““ "

= \p r p .s .

But in one revolution, a peripheral Hi***™ r ^


••• Speed of travelling m.m.f. wave ' ^ metreS ‘ S traverst'd
2f „
- p (7iD) m/sec = 2f kD
m/sec.

For a diameter £> = 1 2 m, the speed o ft ||' Wavc and ‘ s equal to two pole-pitches

= 2f
g - 62.82 m /sec.

S c an n ed by C a m S c a n n e r
_________________________ Basic Concepts o f Rotating Electrical Machines 317

Example 3.31. Sketch the resultant m.m.f. wave for a 2-pole, 3-phase balanced winding
yhenphase a carries (i) maximum current l m (ii) l mcos 30° and (iii) Imcos 60°. Consider each
fcIt of the winding as a current sheet o f uniform density.
Solution. The stator shown in Fig. 3.36 (o) is cut and laid out flat as shown in Fig. 3.44 (a),
jfote that the sequence of phase belts is a, c', b, a', c, b'.
(i) When ia = Im,ib = ic = - 2. These currents are indicated in Fig. 3.44 (a) by dots and
(posses. For convenience, phase belts a, c\ b etc. are shown in the same plane. Note that dots
jnd crosses in Fig. 3.44(a) are identical to that in Fig. 3.38(a). By referring to Fig. 3.30, m.m.f.
forphase a is as shown. For 7m, maximum m.m.f. is Fmas given by Eq. (3.59). For phaser b and
f the maximum m.m.f. is Fm/2 as |i6 | = |ic| = | / m. Note that for phase belt with dot, the
inm.f. variation has positive slope and for phase belt with cross, the m.m.f. variation has nega­
tive slope. Summation of the ordinates in Fig. 3.44 (6), (c), (d ) gives the resultant m.m.f. varia­
tion as shown in Fig. 3.44 (e).

~*0)

Phase
c-axis
Fig. 3 4 4 . Pertaining to Example 3.31.

H
Scanned by Cam Scanner
318 Electrical M achinery

7 7 7 7 £ / , = 0 and 4 = ' I ™ ese currents are shown in Pig. ^


(u) When ia m cos 2 . lid iine and for phase c by dotted line in Fig. 3 44,
(/). M.m.f. variation for phase a is - o T s d t o t m 'm .* 'V a r i a t i o n is shown in Fig. (3.44 h), 44<el
Phase 6 produces no m .m .. as ih• ^ a n d ^ = _ / „ , . The m .m .fs. f
m For i„ = / , cos 60° - ^ / « . ^ for ^ reader The resu|tant m m f ^
can be drawn accordingly ; this is lett as a
ever, is shown in Fig. 3.44 (.). ^ ^ M a c h in e s

3 . 8 . P rodu ction o fT o r q u e m neral torque expression for rotating electrical


The purpose of this article is to a produced by stator currents and the other by
machines, in terms of two magnetic , • an elementary two-pole uniform air-
rotor currents. In order to obtain t o torque exp^ , ,g considered.
gap (or non-salient pole) mac ne, s o ^ which is assumed sinusoidally dis-
Currents in the stator winding p . . The peak value of the stator m.m.f. F is
tributed in space, i.e. along the air-gap the stator magnetic axis is taken to be
directed along stator magnetic: axi . similarly, currents in the rotor winding produce
horizontal, with F, directed from left fc n g h t S m.la y of rotor „ m f
rotor m.m.f. which is also assumed a spatial si n,e wsms P^ ^ ^ ^ ^ ^
directed along the rotor magnetic axis as shown J . crossmg) respectiveiy. If stator or

rotor has m o r e th ^ ^ n ^ w in trin ^th en ^ is the resultant stator man.f. per pole produced by al,
stator windings and F, is the resultant rotor m.m.f. per pole produced by all rotor windmgs.

to r q l« ^ d e v e b p e d ” tl"sh o w n bn tlris^rtlcle^m t^h em a^itu d e of^Mtromagnetictor^eis


nrnduct of stator m .m .f, rotor m.m.f., and sine of the angle between their

(a:
Fig. 3.45. Production o f torque in non-salient pole machine
(a) Elementary two-pole machine and (b ) Space phasor diagram for m.m.fs. Fs and Fr-
(jf gfatOf
In Fig. 3.45 (a), the length of the radial air-gap isg and the average radius (averag ^^
and rotor radii) is r. The effective axial length is I. For deriving a general torque expres >
following assumptions are made :
(i ) The stator and rotor iron is assumed to have infinite permeability. This also me
the saturation and hysteresis are neglected.

S c an n ed by C a m S c a n n e r
Basic Concepts of Rotating Klectrical Machines 319

(jj) Tangential (along the air-gap periphery) component of the magnetic field is negligible
pared with its radial (along the radius) component. In other words, mutual flux path
the gap length g is radial.
? (»i)The length g of the air-gap is assumed much smaller in comparison with average radius
•This effectively means that there is negligible difference between the flux density at the
rjator surface, at the rotor surface or at any radial distance in the air-gap.
(iv) Only the fundamental sine components of the stator and rotor m.m .f. waves are con­
sidered.
Since the m.m.fs. Fs and F,. are spatial sine waves, these can be represented by space
phasors Fs and F,. directed along their respective magnetic axis in Fig. 3.45 (a).
The phasor sum of Fs and F,. gives the peak value of the resultant m.m.f. wave FR acting
across one air-gap. The magnitude of FR is given by
Fp = + F ? + 2F, Fr cos X ...(3.73)
Here X is the space angle between peak values of stator m.m.f. Fs and rotor m.m.f. Fr.
Resultant field intensity H, produced by m.m.f. wave FR, is sinusoidal along the air-gap
periphery. As stated before, F R acts across one air gap, therefore, peak value of magnetic field
intensity Hp is given by

H = ^R ...(3.74)
P g .
Resultant m.m.f. F R crossing the air gap, gives rise to resultant flux, called resultant
mutual flux per pole. Some stator flux may not cross the air gap—this flux linking the stator
winding but not the rotor winding, is called the stator leakage flux. Similarly, the rotor
produced flux, not linking the stator winding, is called the rotor leakage flux. Note that leakage
fluxes do not take part in the production of torque, it is the resultant mutual flux that produces
the electromagnetic torque. The effect of leakage fluxes on the electrical charact°ristics can,
however, be accounted for by means of leakage reactances, as done in a transformer.
The co-energy density at a point in the air gap, where magnetic field intensity is H, is given

by ^ Mo H 2 as Per (213).
.-. Peak co-energy density in the air gap

1 u2 1 ,
- 2 Mo p 2
g
-1 ^ 1 ...(3.75)
” 2 rr2 R
8
Since m.m.fs F s and F , are sine waves, their resultant m.m.f. F R must also be a sine wave.
Now recall that the' average value of the square of a sine wave is equal to half the peak value of
the sine wave. Sine F R is the peak value of sine wave, the average value of the square of FR is

equal to | (F*)2.

.'. Average co-energy density in the air gap, from Eq. (3.75), is

=- ^ (average value of F 2
R)
2g
2

1 Mo 1 (F r ...13.76)
= 4M o
2 8
2 g2

S c an n ed by C a m S c a n n e r
320 Electrical Machinery

Total magnetic field co-energy stored in the air gap is


Wfid = (average co-energy density in air gap) (volume of air gap)

= 7 •m> y {2Krlg) = 2 ^ 7 ^ ‘ * * Joules


W * ' in terms of peak values of stator and rotor m.m.f. waves is given as

w f t d ' ^ l ~ ( f a. + F r2 + 2 F , F r coS X) (3n

For a 2-pole machine, the number of electrical degrees are equal to the number of m h
cal degrees. Thus, for a 2-pole machine, the electromagnetic torque, as per Ea (2 (n\ \ ^
obtained as Vl can be

^ dWfid
e = - g f - (Fs, F r, X)

In general, for a P-pole machine,

J T

relation o'fEq S a s " 1" * ' 11' 6' “ * * * ” * * * * **“ » - by th,

= ord J. = £ d > . m

• _P
dK ~ 2
From above, torque is given as

T = r . ™ i l , F , ,,
* 2 3 X ^ s’ r> ^
_P ^0^ a _o _ n
~ 2 '~% T J l [F1 + F ; + 2 Fs ■Fr . cos X)
_ P Ppnrl
2 g s ' F r ■sin X Nm ' ^ yg'

upon the number of poles, ^ electr° map etic torclue dePends


m.m.f. s and sine of the angle between thp r+u gap’ P value of stator and rotor
in the torque expression (3.78) indicates * tW° fields (or m.m.fs). The negative sign
rotor, tend to reduce the angle X between th t6 m -m.fs, one on stator and the other on
tion as to align the two m.m.fs Fs and F ^ ° ° words, the torque acts in such a direc-

Electromagnetic torque given bv En fq 7 a’i *. .


tions. As stator is fixed, it is the rotor that « i S^ ^ stator and rotor in opposite direc-
stator is free to move, then stator would t V° V6 j *S assume<l that rotor is held fixed and
rotated with stator held fixed In other w n ^ “ f dlrection opposite to that in which rotor had
torques as given by Eq. (3.78) In DracHM l * 3nd r° tor exPerience equal and opposite
mitted to ground through its foundation a e\e^ romaenetic torque acting on the stator is trans­
In Fig. 3.45 (6), OA = Ft, OB = Fr, ^ r° tateS'

Fr sin X (= BD) is n o rm a l^ o p COmP° nent F ' Sm * (= A E ) is n orm al to F r. Similarly, component

I
cann ed by C a m S c a n n e r
Basic Concepts o f Rotating Electrical Machines 321

Hence it can be stated from Eq. (3.78) that electro-magnetic torque can develop only when
0f the two m.m.fs (here F s or F ,) has a component (here Fs sin X or Fr sin X) perpendicular
tothe other m.m.f. or flux.
It is usually more convenient to express Eq. (3.78) in terms of resultant m.m.f. wave FR. In
rder to obtain this expression, refer to Fig. 3.45 (6) which gives
g:
A E - F4 •sin X = CH - FR sin 5f.
BD = F,. sin X = CG = FR sin 5S
Substituting these values in Eq. (3.78), we get
P
W ri „ „ . j.
C Cl9 8' ^ S n
2 8 i, ...(3.79)
P uanrl „ „ .
or Te = _ L . * L - . F r . FRsindr
o
An examination of Eq. (3.79) reveals that the torque is proportional to the product of one
m.m.f. (here Fs or Fr) and a normal component of the resultant m.m.f. FR.
Note the difference between the torque expressions given by Eqs. T3.78) and (3.79). In Eq.
(3.78), torque is proportional to the product of two m.m.f.s (or fields) produced by their respec­
tive currents and sine of the angle between them. In Eq. (3.79), torque is proportional to the
product of resultant m .m .f (or field), one of the two m.m.f.s and sine of the angle between them.
Torque expressions given by Eqs. (3.78) and (3.79) contain fields in terms of their peak
m.m.f. waves. When magnetic saturation is neglected, it is permissible to express fields in
terms of their peak flux density waves. For this, refer to Eq. (3.73) from which peak value of
resultant m.m.f. FR is

FM- g Hp = g ^
where Bp = peak value of flux density wave due to peak resultant m.m.f. wave FR.
Substitution of this value of FR in Eq. (3.79) gives

Te = - ^ nrl Bp Ft sin 5S
...(3.80)
Te = - ~ nrl Bp - F, sin 8r

During the design of rotating electromagnetic devices, a final check is that flux density m
teeth should not exceed 1.8 to 2.2 T. This check imposes a limit on the peak value Bp of the
roenUant flux densitv wave in Eq. (3.80). The maximum value of m.m.f in Eq. (3.80) is also
limited from a consideration of the temperature rise. Thus, Eq. (3.80) is quite suitable to the
designer of electromagnetic devices as it contains important design parameters.
An alternative form of torque expression can be obtained in terms of total flux per pole *.
For a P- pole machine, total flux per pole, from Eq. (3.11), is

0=| Bp l r
_ P± ...(3.81)
or p ~ 4 Ir

Substitution o f Bp from Eq. (3.81) in Eq. (3.80) gives

7 ’, = - | P 2 0 F l, s i n 8s Nm ...(3 .8 2 o )

S c an n ed by C a m S c a n n e r
322 Electrical M a c h i n e r y ______________________________
> 1. 34,
I.
II or Te = - J P 2 Fr sin 8,. Nm
8 -(3.826)
The interpretation of Eqs. (3.82a) and (3.826) must be clearly understood. He
number of poles. The total or resultant air-gap flux per pole 0, is produced by the^ ^ 'S
effect of stator and rotof m.m.fs Fs and Fr. The m.m.f. F a is resultant of all the stato001^ ' 06^
similarly Fr is the resultant of all the rotor m.m.fs. For example, in a 3-phase rotating e]0* ^ ^ ’
machine, Fs is the resultant stator m.m.f. due to all the three stator m.m.fs ; Fr is the r
rotor m.m.f. due to the combined effect of all the three rotor winding m.m.f.’s. esultant
The electrical space angle X between stator and rotor m.m.fs. is called the torqu
whereas the electrical space angle 8S between Fs and F r ; and 8,. between Fr and FR a r e ^ 6 ’
load angles. The various torque expressions, derived above, reveal that the electro 6 **
torque is proportional to the product of interacting fields and the sine of electrical spa a?netic
between their magnetic axis. The negative sign in the torque expressions indica^311^ 6
electromagnetic torque acts in such a direction as to reduce the space angle between tv,eS>^ at
acting fields. inter-
If load angle Sr is time-varying, say 8,. = Qt, then instantaneous electromagnetic torque f
Eq. (3.826) is, r°m

T, - f P2 $ F,. sin W

The average torque over a complete cycle is zero.


Te = 0.
This shows that for the development of electromagnetic torque Te, it is essential that space
phase angle br (8S or X) must remain constant with time. If Fr is a travelling m.m.f. wave at
some speed, then $ must also be a travelling flux wave at the same speed, so that space phase
angle 8r between Fr and <J) remains unaltered with time. Further, if Fr is a stationary m.m.f.
wave, then <{>must also be a stationary flux wave for 8,. to remain time-invariant.
T orqu e in B asic M achine T ypes. The general torque expressions derived above, apply
equally well to all types of rotating electrical machines. In order to obtain torque expressions,
-identical with those already obtained by conventional methods, different pairs of axes are con­
sidered for different machines.
DC M ach in es. For d.c. machines, use Eq. (3.78), i.e. the magnitude of torque for a d.c.
machine is (replacing Fs and Fr by Ff and Fa respectively),
m p u 0n r l
Te = j ^ - F f Fa sinX af ...(3.83)

Here Ff is the peak value of the field m.m.f. per pole and Fn is the peak value of the arma­
ture m.m.f. per pole. Angle Xaf\s the torque angle between i^-and Fa. The p e a k value of th e flux
density under each pole is

The total field flux per pole fy, from Eq. (3.11), is

,or FfSSiLLB_ ...(3-84)


f 4 p 0 lr

Scanned by C am bcanner
\rt ___________________________ Hasic Concepts of Rotating Electrical Machines 323

Substitution of the value of Ff from Eq. (3.84) in Eq. (3.83) gives


P \x0nrl QfPg
c 2 g 4M0/r a Sin °f

= 8 P\ Fc s in V Nm ...(3.85)

AC Machines. For synchronous and induction machines, Eq. (3.826) is preferable. For
polyphase a.c. machines, the basic torque expression (3.826) can be expressed in a more useful
form, by including the voltage and current in it. This is done as follows :
The rotor m.m.f. F n from Eq. (3.72 a) is

Fr= m^
f= m — k nw
w^ p—ATs/pole.

and the resultant air-gap flux per pole, from Eq. (3.24)' is

d. Erh

Substitution of the values ofFr and <t>in Eq. (3.826) gives the magnitude of electromagnetic
torque T. as

Te = \ P 2
, . ) rm2v2 z j )
sin 8r
V2n f N hk,v n w P
\ J \ /
=¥ /^ ^ sin 8f

Speed in mechanical 2 Speed in electrical


For an a.c. machine,
radians per second P radians per second
2 2 _ . 4k f
.e., “ m= p " = p (2lt/ ) = p

Te = — m Eph I sin 8,. ...(3.86)


m
Note that Eph is the per phase voltage induced in the armature by air-gap flux 0 and I is the
per phase armature current.
Load angle 8^ will be shown to be equal to (90 + 9) degrees, where 9 is the time-phase angle
between Eph and /. In view of this, Eq. (3.91) becomes,

Tc = — m Eph I cos 9

or Te (om = m Eph I cos 9.


°r Mechanical power developed
= Electrical power developed.
The power balance in case of d.c. machines is demonstrated in the chapter on d.c. machines.
3.8.1. Alternative Derivation for Torque. The object of this part of the article is to
derive expression for electromagnetic torque through the concept of rotor-current sheet.
Stator flux-density wave, in phase with stator m.m.f. wave, are shown in Fig. 3-46 (a) with
^eir respective peak values of B. and Fr Rotor m.m.f. wave, with peak value Fn is shown m
P'g 3.46 (6) lagging stator m.m.f. wave by an angle X as in F i g . Ro^ ' C_U" rfie.n,t'
Wave, with peak value A n leads rotor m.m.f. wave by 90° and is stretched in Fig. 3.4 ( ).

^ C a n n e d Dy u a m b c a n n e r
324 Electrical Machinery________
Ur-
With angle 0 measured from stator d-axis, stator flux-density wave can b e e ^ T ^
B = B , cos 0 T Pes
and rotor current density = A r sin (0 - X) A /m
Consider a differential angle dQ at an angle 0 from the origin as shown in Pi
(6) and (c). The differential angle dQ in radians is equal to rdQ in metres along the
air-p
periphery.

Fig. 3.46. (a) Stator m.m.f. and flux-density waves


(6) rotor m.m.f. and current-sheet waves (c) differential angle 00.

Current in differential element,


i = A r - sin (0 - X). (r dQ) Amp
Differential torque dTe produced by current i is obtained by using the general torque orm
Force x radius = (Bli) (r) = rl (B ) (i )
... dTe= rl (B, cos0) [Ar sin (0 - X) (r d0)l
= r2l Bt A r cos 0 sin (0 - A.) dQ

But cos 0 sin (0 - A.) = cos 0 sin 0cos A. - cos20 sin A.


sin 20 , 1 - cos 20 ,
= — - — cos X - ----------- sinA.

= - ^ sin A. - ^ sin (20 + X)

... dTe = r2/ B, •A r |^- 1 sin A. - 1 sin (20 + X) dQ

Total torque can be obtained by integrating the above dTt <C>


expression from 0 = 0 to 0 = 2n (angle covered by one pole-pair).

... - i Si n A .- i 8 i n ( 2 e + X )jd 9

“S c a n n e d b y C a m S c a n n e r
Basic Concepts o f Rotating Electrical Machines 325

r2*
value o f J sin (2 0 + X) dQ will be found to be zero. This gives the electromagnetic

torque as

Te = r2l Bt A r - - sin X 0

= - r2Z Bs A r •^ sin X •(2n)

= - nr2l B s A r sin X ...(3.87)

F
Peak value of stator flux density, B , = p0 H„ = p0 —

P ■F P •F
From Eq. (3.63), for a P-pole machine, peak value of rotor-current wave is A r = ^r '

Substituting the values of Bt and A r in Eq. (3.87), we get

rp _ 2. Mo P» P ' Pr . .
i - —— r / • • n •sin K
g 2r

P M o — F .F r 8inX ...(3.78)
g 2 ' r
This is the same expression as obtained in Eq. (3.78). One can proceed further as in Art. 3.8.
E xam p le 3.32. A 2-pole, 50 Hz cylindrical-rotor machine has the following data :
D = 1.6 m, 1 = 1.8 m, g = 12 mm
Peak value o f sinusoidal rotor and stator m.m.fs are 4000 A T Ipole and 6000 A T Ipole
respectively. Rotor m.m.f. leads stator m.m.f. by 140° and both run at synchronous speed. Cal­
culate
(а) resultant peak gap m.m.f. (b) peak gap flux density
(c) total gap energy (d) electromagnetic torque and
(e) electromagnetic power
Solu tion, (a) Resultant peak gap m.m.f. Fr , from Eq. (3.73), is given by
F r = [40002 + 60002 + 2 x 4000 x 6000 x cos 140°]1/2 = 3902.55 ATs/pole.
Fr
(б) Peak gap flux density, Bp = Ho = Mo ~

_ 4k x 10\ - 7 x 3902.55 _ q ^Qgq f


1 2 x 1 0 "3
1 M0 Url rfl
(c) Total gap energy = ^ ~

1 4k2 x10~ x 0.8 x 1.8 (3902 .55 )2 = 3607.51 Joules


2 12 x 10" 3
P Mo nrl p P \
(d) Electromagnetic torque, T( = - —— Ft t r sm K

2 4k x 1 0 "7 x tc x 0.8 x 1.8 x 4000 x 6000 x sin 140o = 7308.36 Nm


* "2 12 x 10“ 3

S c an n ed by C a m S c a n n e r
326 Electrical Machinery

_ . , 4n x 50 , ^ 3ji
(e) Synchronous speed, oom = —
p = ^ “ 100 K rad /sec \

Electromagnetic power, P = T e - o)m


= 7308.36 x lOOn = 2296.05 kW
E xam ple 3.33. A uniform air-gap machine has the following dimensions •
D = 0.8 m, I = 0.5 m, g = 5 mm
When stator and rotor are energized, the stator and rotor windings produc '
rent sheets o f peak density A s = 10,000 A/ m and A,. = 6000 A/m respectively ^ musoi(fo/cur.
stator and rotor are so displaced as to give a torque angle o f 60°. The iron in th ^
assumed to have infinite permeability. mQchine is
Determine the torque in case the machine windings are designed to produce ( ) o
and (b) a 6-pole field. Pole
' field
Solution, (a) Peak value of stator m.m.f. per pole, from Eq. (3.63), is

• Fs = — — = 2 x 10,000 x 0.8 = 4000 AT

Similarly, Fr = = | x 6000 x 0.8 = 2400 AT

m p u„ nrl
From Eq. (3.78), = sin A.
g
_ 2 4 n x 10-7 x n x 0.4 x 0.5
x 4000 x 2400 sin 60°
” 2 5 x 1 0 "3
= 1312.83 Nm

(6) When P = 6, then Fx = ^ x 10000 x 0.8 = AT


o 3

F,.= ^ x 6000 x 0.8 = 800 AT


6
rp _ 6 4n x 10r 7 x n x 0.4 x 0.5 4000 onn . cno
x —- — x 800 sin 60
e 2 5 x 10“ 3 3
= 437.61 Nm
Exam ple 3.34. In rotating electrical machines, the component o f rotor m.m.f. norm al to the
stator m.m.f. (or component o f stator m.m.f. normal to rotor m.m.f.) gives rise to the production
o f electromagnetic torque. Explain the function o f rotor m.m.f. component in phase with the
stator m.m.f. (or stator m.m.f. component in phase with rotor m.m.f.).
Solution. Refer to Fig. 3.45 (a). Stator m.m.f. Fs causes the a p p e a r a n c e ofN, S p o I e son ^
stator. The rotor m.m.f. component Fr cos \ in phase with Fs, also cause t h e appearance
N, S poles on the rotor, as illustrated in Fig. 3.47. Now stator poles
produced by Fs attract the rotor poles produced by Fr cos thus caus­
ing the appearance of radial {i.e., along the radius) forces in opposite
directions. If the two air-gaps encountered by m.m.fs. Fs and Fr cos \
are of equal radial lengths, then opposing radial forces are equal. In
case the air-gaps are different, the two radial opposing forces are un­
equal and their difference gives rise to unbalanced magnetic pull,
rapid wear of the bearings, noisy performance etc. Poor workmanship
or wear of the bearings with the passage of time, may be the causes of 3 4?. p«rwifl'ng
different air gaps around the periphery. Example 334

S c an n ed by C a m S c a n n e r
In Vl“ VVU1 v*»w vw.t.^.ivnv VI »wtui ill.III.I. m WHI1 aldiui III.111.1. \Ul LUIIlJJUIlCllt Ul
tator m.m.f. in phase with rotor m.m.f.) is detrimental to the operation of rotating electrical
Machines and should therefore be made as small as possible (and therefore F, Fs sin A. as large
as possible).
When a machine is switched on, a large thud is heard only because of the presence of
f Fs cos A. in an electrical machine.
Example 3.35. A 4-pole, 3-phase, star-connected, 50 Hz turbo alternator has the following
data:
Field winding has 12 concentric coils distributed in slots with slot-angular pitch o f 8°.
Turns per field coil = 6, Series armature turns per phase = 28
Armature radius = 0.6 m, Armature length = 4 m
Gap length = 0.06 m
Winding factor for armature winding = 0.96
Field current = 1000 A.
Calculate (a) peak value o f fundamental m m f produced by field winding,
(b) peak value o f fundamental air-gap flux-density wave,
(c) the fundamental value o f air-gap flux per pole and
(d) rms values o f phase and line emfs at no load and at rated speed.
Solution, (a) There are 12 concentric coils for 4 poles. This means there are three con­
centric coils distributed in slots to create one pole. Therefore, distribution factor for field wind­
ing,
. 3x8
sin —
kd = Y = 0" 35
3 sin -

Coil-span factor, kp = 1
Winding factor for field winding,
kf - kd x kp = 0.9935
Number of field turns, Nf = 6 x 12 = 72
From Eq. (3.59 a), the peak value of fundamental mmf produced by field winding is

F „ = - x 0.9935 x 72 * 10— = 22769.3 ATs/pole


V K 4
(6) Peak value of fundamental flux density wave is
„ HqTif _ 4n x 10 7 x 22769JS _ Q 47g9 T
5 lp = P o « - g ~ 0.06
(c) From Eq. (3.11), fundamental value of air-gap flux per pole is
. r = — x 0.4769 x 4 x 0.6 = 1.1446 Wb
<P - p D \p v 4

.p . „ E=. V 2 it x 5 0 x 1 .1 4 4 6 x 2 8 x 0 .9 6 = 6833.64 V
(d) Per phase emt, *pk
E = y l3 E h = >/3x 6833.64 = 11835.86 V.
1 , \ n Hz star-connected cylindrical-rotor alternator
Example 3.36. A ■ 3-phase 6-po e ^ has 36 siots with two-layer winding of
bvelops an open-circuit emf of 415 V. IM arm*

* ■
Scanned b y C a m S c a n n e r
_ _ _ _ _ _________________________

j 5?? raf c!riea!JV! a c h i ^ ------------- f 1* C<*


fall-pitched coils with 4turns ^ ^ I d w inding has 42 turns wUh wc
length 0.4 m and a gap length 2 mm. i n ,
length
Calculate
culate density wave in air gap,
(a) the peak value o f fundamenta t u -d uced by th e fi
(b) the peak value o f fundamental m m fw P
current. r i id a t a torque angle o f 146° and at rated
In case this alternator develops a torque o f
voltage, thenvalue
Ic) peak calculate mmf,„
o f the fundam ental arm ature m m r wa v e P
p er rpole and the resultant mmf wave

P Id) th erm s value o f armature current and Us p ow er factor.


Solution, (a) Series turns per phase,

^ = ^ = 4 8

, 36 - 9
Slots per pole per phase, q = 6 x 3 " ^

. _ 180 _ on=
Slot angular pitch, Y= g -

. 60
sin —
hd - ^ = 0.966, kp - l
2 sin —

Per-phase emf, Eph - ^2nf Npfl-^ kw

Flux per pole, = V3 x 1/2 x x x 50 x 48 x 0.966 = 0 0233 W b

p ° iP

Peak value of fundamental flux-density wave,


_ g j > _ 6 x 0 .0 2 3 3 _ Q 9 7 1 T
V Mr 4 x 0 . 4 x 0 . 0 9

(6) B » = pc Hf = ^

.*. Peak value of fundamental field mmf wave,

0 g &ip 2 x 10“ 3 x 0.971 _ _ _ .


F \ f- „ = = 1545.4 A T s per pole
4 n x 10"

From Eq. (3.59 a), lf = 1545 4 180 62 A


f 4 k fN f 4 x 0.96 x 42 iBU*w A
ic) From Eq. (3.78), the magnitude of torque is given by

T - — • ^°'nr^ p p • *
2 ~ y - \ F irF la*™&af
.*. Peak value of fundamental armature m m f is

F '° = r ~ . ,» ) 1 UX2 ~ 1n' 3 8 1 8 8 ATs/pole


6 x 4it x lO’ ’ x it x 0 .0 9 x 0 .4 "

S c an n ed by C a m S c a n n e r
Basic Concepts of Rotating Electrical Machines 329

jultant value of peak mmf per pole,


F r = [Flf2 + F la2 + 2F v ■Fla ■cos 8a/]1/2
= [1545.42 + 618.82 + 2 x 1545.4 x 618.8 x cos 146°]1/2
= 1088.84 ATs/pole
(id) 1° 3 3 -phase synchronous machine, the peak value of rotating mmf wave is given by
p _ 3 „ 34, Nph- <2Ia k
F\a 2 »»*— 2 tc ' P ATs/pole

This gives rms value of armature current as


r 618.8 x 2tc x 6 .
0 12 x 0.966 x 48V2 “
For power balance,
i Q 'T U 271X 1000 10071 j ,
y/3 x 415 Ia cos 0 = Tc com, where com= ----- — ------= —— rad/s

„ 114x 10071 „
... Power factor, 6 = 3 x ^3 x 415 x 29.65 = ° ' 5602
As Fr < Flf, the pf is 0.5602 lagging.
I
3.9. Losses and Efficiency
In electrical machines, the power input (mechanical or electrical) is always more than the
nower output. The difference between power input and power output, under steady state con­
ditions, is called power loss in watts. Thus, in accordance with the law of conservation of power
\ (or energy),
power input = power output + power loss
1 or power loss = power input - power output
Power loss in a machine does not perform any useful work, it leads only to heating of the

cfflisidemtion of the power losses in electrical machines is essential for the following three
I
' “ ©Losses influence the operating cost of electrical machines. For example, a machine with

, lower efficiency has more losses and therefore increased operating cost.
(® Losses cause heating of the machine and therefore its temperature nse Greater the
\u) uosses cause nearing ui fo0fPr the deterioration of the machine
OSS, more is the temperature rise and th®ref° re' tj through its effect on the life of
insulation. Temperature rise determmes h ^ ^ ^ ^ ^ and hence ^ ,osses
winding insulation. It can, therefore, be stated tn
iotermlne the rating, or safe power < ^ ^ hgreas current component, like core-loss
(iu) Voltage drop IR is associated with ohm Obviously, this suggests that losses
current, pertains to the iron loss m electnca ^ appropriately taken into account
associated with voltage drops or current' ^ m.P°1prtricai machihe analysis can be carried out as
mthe equivalent circuit of a machine so that ele
desired.
Machine efficiency is defined as
^ . Output ...(3.88a)
Efficiency

, 0r Input-Josses
71 =
Input
Losses ...(3.886)
_ 1 Input

‘ '
leu uy u d in o u d i ii iei
330 Electrical Machinery —

For computing the efficiency, an electrical machine may be s u i t e d I to direu load test and
its output and input measured simultaneously. Eq. (3.88a) t:hen gi es the machine efficiency
It is however much difficult to perform the actual load test, because of the cost of providing
large inputs and difficulty of dissipating the large outputs. Moreover a small error m themeas-
urement of either output or input, causes about the same amount of error in the computed
efficiency. ,
Since the losses are a small percentage of output or input these can be measured more
conveniently and economically. Even more important is the fact that a small percentage oferror
in the measurement of losses, results in a still smaller percentage of error in the computation
of efficiency This is now illustrated with an example. Assume an electrical machine to have an
input of 1000 watts and output of 900 watts, so that the total losses are 100 watts and the
efficiency is 90%. Suppose there is an error of 10% less, in the measurement of output, then
efficiency is given by

x 100 = 81%.
1000
If there is 10% error (less) in the measurement of losses, then

i = ( 1 - T § 5 o ) x 1 0 0 = 91% -
Thus a given percentage of error in the measurement of output (here 10 /fr) results in almost
the same percentage of error, in the calculation of efficiency. But a given percentage of error in
the measurement of losses (here 10%), causes about one tenth (here 1%) of that percentage
error, in the computation of efficiency. In view of this, the efficiency is computed by measuring
the losses in a given machine.
When a machine is to be selected from a large number of available machines, then their
methods of loss measurement should be same. A machine having larger efficiency and, there­
fore, less energy losses is then selected. But a highly efficient machine is more expensive and,
therefore, involves more fixed charges such as interest, taxes and insurance. It should, how­
ever, be kept in mind that a more efficient machine is likely to give better performance, wi
more reliability, less breakdowns, less running charges and minimum maintenance cost, as
compared to a less efficient machine.
The various machine losses may be classified as (a) electrical losses and (b) mechanical
losses. These are described below :
(a) Electrical L o s s e s :
(i) Resistance losses or I2R losses. When current flows through various machine vvind^
ing, I2R losses occur. The resistance R, by convention, is taken as the d.c. resistance o
machine windings at the operating temperature of 75°C. Actually, effective winding resis
Reff, which is more than its d.c. value R&, should be used in computing the I2R loss. The e e .
winding resistance depends on the operating frequency and actual flux conditions ex*stint°ray
the conductor. This increment of loss, equal to (I2Reff- f R ^ ) , is usually included in thes ^
load loss discussed further. In the field circuits of d.c. and synchronous machines,^ on 3^ ^
resistance loss in the field winding is included for computing their efficiency. The 1 R
their field rheostats and other losses in the sources supplying the field winding are,
included while calculating the system efficiency
2 j. between
In addition to I R loss in the windings, there is brush contact loss at the contacts
the brushes and commutator (d.c. machines) or between the brushes and sllpetween
(synchronous and induction machines). In d.c. machines, the conduction of current in
brushes and commutator is through the short arcs in the tiny air gaps which are inb

Scanned by CamScanner"
r
Art-*?!. Basic Concepts of Rotating Electrical Machines 331

such a contact. As the voltage drop across an arc remains substantially constant, the brush
voltage drop in a d.c. machine is taken as constant at 1 to 2 volts total. Thus, the brush contact
loss in a d.c. machine is proportional to its armature current. In case of synchronous and induc­
tion machines, the brush contact loss is usually neglected for all practical purposes.
(ii) No-load core loss. This loss, also called open-circuit core loss, consists of hysteresis
and eddy-current losses. These losses are present in rotating electrical machines— in case their
stator and rotor irons carry an alternating, or time varying, flux under the condition that only
the main field (or exciting) winding is energized. In a transformer, therefore core losses are
always present. In rotating machines, the hysteresis and eddy current losses can be determined
from Eq. (1.46), but here f should be the frequency of the magnetic flux reversals to which a
given piece of stator or rotor iron is subjected in one second. Physical understanding of
hysteresis and eddy current losses in rotating electrical machines is presented below :
Hysteresis loss. For a 2-pole machine, consider a small iron piece such as ab, subjected to
alternating flux as it rotates under N and S poles, Fig. 3.48. When the small iron piece is under
N-pole, main pole flux passes through it from a to 6, caus­
ing the appearance of S-pole at a and N-pole at b. After half
a revolution, the iron piece comes under the influence of
main S-pole and now main flux passes through it from b to
a, causing the appearance of S-pole at 6 and N-pole at a.
Thus in half a revolution, the magnetism of iron piece ab is
reversed. When the armature rotates, there are con­
tinuous magnetic reversals of such small iron pieces and Fig. 3.48. Illu stra tin g h ysteresis loss.
power required for their reversals is called hysteresis loss.
It may be seen that hysteresis loss is directly proportional to the number of magnetic reversal
per second or the speed.
Eddy current loss. Consider again a 2-pole machine with solid rotor iron. When the rotor
rotates, e.m.fs. are generated in the rotor iron in exactly the same way as they are induced in
the rotor conductors. In Fig. 3.49 (a), these e.m.fs. are indicated by dots and crosses. Another
view of this figure (plan) is given in Fig. 3.49 (6). The e.m.fs. generated in the solid iron give rise
A r e a A n o r m a l to
t h e d i r e c t i o n of
eddy c u rre n t v

I " - - * -------- 1
I 1 .---------- I \
1 1 r —i 1 *
1 N f ♦ * T*t s
1 <•— j •;

OMw
c u rre n ts
(a) (Ib)

A re a A / 4, norm al to
d ire c tio n o f eddy
, c u rre n t
T

j
TT
(C)
Fig. 3.49. Illustrating eddy current loss.

S c an n ed by C a m S c a n n e r
332 Electrical Machinery

■i^-3.9
to circulating currents which are called eddy currents. These eddy currents ar
nitude, because the resistance offered by the rotor iron is quite small. The povverT mag-
(eddy currents)2 (resistance offered to the flow of eddy currents), is referred to °SS ^ Ua|to
loss. as eddy Current
This loss can be minimised by using thin laminations for the rotor structu tu
illustrated by referring to Fig. 3.49 (c), where only 4 laminations are shown for sf i '8 °an *>e
flux linking each lamination is one-fourth of that linking with solid iron ; therefo C'ty' ^
induced in one lamination is one-fourth of that induced in solid iron. At the same f ^ e-m.f.
normal to the direction of eddy currents is reduced to one-fourth of that for solid iron rtf'6 area
resistance R lnd offered to eddy currents in laminated iron is four times the resistanc f°re’
fered in the solid iron, i.e. Rlnd = 4 Rsid. Here subscripts Ind and sld stand for laminate'** °f
solid respectively. In view of this, ^
Eddy current loss per lamination _ (e-m.f per lamination)2/i? /nd
Eddy current loss in solid iron (e.m.f in solid core)2/i?

_ Eddy current loss in laminated rotor _


Eddy current loss in solid rotor
If axial length of the solid rotor core is unity, then the lamination thickness in Fig 3 49 (C)
is j. Thus it can be concluded from above that eddy current loss is proportional to the square of
the lamination thickness. If there are more laminations for a given axial length, the lamination
thickness decreases and hence eddy current losses are also decreased. Thus the use of thin
laminations minimises the eddy current loss. The usual lamination thickness is 0.4 mm to
0.5 mm. If lamination thickness is made less than 0.4 mm, the reduction in eddy current losses
is achieved, but at the cost of additional labour charges in assembling the rotor.
The eddy current loss has been found to be given by Eq. (1.46). In case the effect of lamina­
tion thickness t is to be taken into account, then the eddy current loss Pe can be expressed as

P e~ ke(t ■f B m)2 ...(3.89)


For both the expressions for Ph and Pe, the frequency/and B m can be replaced respectively
by speed and voltage if required.
In d.c. and synchronous machines, the core loss occurs mainly in the armature iron. It is
because the armature in both these machines is subjected to alternating flux as it rotates under
the effect of main fleld poles. With the slotted armature passing across the field poles, the flux
density wave pulsates in magnitude as shown in Fig. 3.50. In Fig. 3.50 (a), slot is under the
middle of field pole and the flux density is A S . With the relative motion between armature and
POLAR AXIS

POLE FACE
TEETH ^ | ’- j SLOTS

1 ! 1 1.k. 1 I i i •
FLUX DENSITY
WAVE

Fig. 3.50. Main field-flux distribution in the air-gap aa affected by slotted armature.

S c an n ed by C a m S c a n n e r
^ ^ 9)______________________________________________ Basic Concepts o f Rotating Electrical Machines 333

field poles, tooth may come un^ er the middle of pole as shown in Fig. 3.50 (6). Under this
rtjjdition, air-gap flux density is A 'B '. The field m.m.f. is same in both these figures. Here
£ ft >AB, as the air gap length along the middle of pole is less in Fig. 3.50 (6) than in Fig. 3.50
(a). These pulsations in flux density wave arising from slot openings cause losses in the field
particularly in the pole faces (or pole shoes). This loss in the pole shoes occurring due to
relative motion between field poles and slotted armature is referred to as pulsation loss. In
order to reduce this loss, pole shoes in d.c. and synchronous machines are laminated. The pul­
sation loss is composed of hysteresis and eddy current losses in the field-pole shoes. As these
lo s s e s occur in the pole-faces of field poles, these are also called pole-face losses. In induction
machines, the stator has core loss, whereas rotor core loss is almost negligible because of
reduced frequency of the flux reversals (equal to slip frequency, sf) in the rotor.
(iii) Stray lo a d lo ss. When a machine is loaded, the load current establishes an m.m.f.
which appreciably changes the space distribution of air-gap flux density wave. This leads to an
increase in the core loss from no load to full load. This increment in core loss caused by distor­
tion of the air-gap flux plus the increment in I2R loss due to non-uniform distribution of conduc­
tor current is called stray load loss. In other words, stray load loss consists of two components,
one originating in iron parts and the other in the armature conductors. In iron parts, the stray
load loss consists of (a) the eddy current loss in the stator frame, end covers etc. caused by the
armature leakage flux under load and (6) the increased teeth loss due to distortion of the flux
density wave. In the conductors, the stray load loss is due to the circulating currents set up in
the conductors by the alternating leakage flux produced by load current. These circulating, or
eddy currents make the conductor current distribution non-uniform and as a result effective
resistance of conductor increases. This gives rise to additional conductor loss, called stray load
loss In d c machines, stray load loss also occurs in the coils undergoing commutation. This loss
is usually taken as proportional to square of the load current. Stray load loss cannot be deter­
mined accurately. In d.c. machines, by convention, it is taken as 1% of the r a t e d output for
rating above 150 kW. For synchronous and induction machines, it is taken as 0.5% of their
rated output. __ t
(6) M ech anical loss. This loss consists of bearing-friction, brush-friction and windage os-
ses. The windage (wind-friction) loss includes the power required to circulate air through the
machine and ventilating ducts and is approximately proportional to square of the speed.
Brush-friction loss occurs in machines fitted with brushes, for example d.c machines
synchronous machines, wound-rotor induction machines etc. Squirrel-cage induction motors
h ^ e no brush-friction loss. This loss depends on the brush pressure, coefficient of friction and
speed
Bearing-friction loss is approximately proportional to speed. This loss further depends on
the type of bearings, their lubrication etc.

tion, their sum is therefore referred to (the machine may be a generator or a


by running the electrical machine as an uni oa‘ voltage equai to the normal generated
motor) at rated speed or frequency an . minus a small no-load armature ohmic loss
e.m.f. The total power input to the unloaded mot magnitude of no-load rotational
(no load stator ohmic loss in case of induct^ nw dTot be split into mechanical
loss. For efficiency computations, the no-ioaa rui,
nnd open-circuit core losses.

S c an n ed by C a m S c a n n e r
^4 Electrical Machinery

p 0r the ,. k c o fa q u ic k overview

account^ the p resen ce of voltages, currents and rotation of thei rotor. This diagram a^ ° »
sents almost all the losses that occur in a transformer in case the losses associated with rotSj
rotation are neglected.
Losses in
rotating electrical m achines

Constant Losses V aria b le Losses

No-load M echanical
core (iron) loss loss
S tray load Brush contact O hm ic
loss (SLL) lOSS (oe f) loss - h

Eddy
Hysteresis C onductor
current Iron SLL
loss SLL
loss

Stator Rotor
W in d a g e Friction P loss P loss
loss (°c speed2) loss (FL)

Brush Bearing
FL (°c speed2) FL (on speed)

Fig. 3.51. Various losses in rotating electrical machines.

M axim um efficiency. It is seen from above that rotating electrical machines have con­
stant as well as variable losses. At light loads, output is low, variable loss is low, the efficiency
is also low because of the presence of constant losses. The machine efficiency, however, nses
progressively with the load. But at a particular load, depending upon the design of the machine,
the efficiency is maximum. Beyond this load, efficiency diminishes. Further, efficiency vanes
with the rating (or size) of the machine; for example, efficiency is nearly 75 7c for 1-kW machine,
90% for 35 kW, 93% for 350 kW and as high as 97% for 3500 kW machine.
The amount of conductor and iron materials required for a machine of given rating is inNer
sely proportional to its speed. This means that a low-speed machine would r e q u i r e more
material whereas a high-speed machine less material for the same rating. More iron an c
ductor would entail more losses. As a consequence, it can be inferred that for the same ratWj
efficiency of low-speed machines is lower than that ^h igh -sp eed machines, the tota -P
being 3 to 4%.

For qualitative purposes, the various losses in rotating electrical machines, operating
near y constant voltage and speed, may be grouped into three main losses : ^
)l( Constant, or fixed, losses. These losses do not depend on the load current. T h e ^
of no-load core loss, friction and windage loss (i.e. no-load rotational loss! constitute con
losses unless there is an appreciable variation in speed

S can n ed by C a m S c a n n e r
^ 3,9 ] _________________ ________ Basic Concepts of Rotating Electrical Machines 335

(2) Loss proportional to load current. Brush-contact loss occurs when current is to be
conducted to, or from, the rotating parts of the electrical machine through sliding contacts. The
voltage drop across the stationary brush and the rotating parts is essentially independent of
current.
If Vb = brush drop across the sliding contacts, then brush-contact loss or brush-drop loss is
given by Vh/, where I is the load current.
(3) Losses proportional to the square of load current. Resistance loss in various
machine windings and stray load loss are proportional to the square of load current.
In view of the above, losses in rotating electrical machines may be written as a + bl + cl2
and output or input as A VI where
a = constant losses,
bl = loss proportional to load current,
cl2 = losses proportional to square of load current,
A = constant N 3 x p f for 3-phase machines, p f for single-phase machines and unity for
d.c. machines),
V = machine voltage.
Motor operation. Power input =AVI
L z £ r2
AVI ; ■
t2i
_ „ „ dn ( A V - b - 2cl) (AVI) - [AVI - a - bl - cl2} (AV)
For constant voltage v, 77 = — : 7 72
dl (Denominator)
For efficiency to be maximum, dr\/dl must be equal to zero. S !.
(AV - b - 2cl) (AVI) = (AVI - a - b l -C l2) (AV)
. t2 (n qi 1
Its simplification gives, cl = a ‘ '
Thus, the motor efficiency is maximum at a load when variable loss cl2 is equal to constant I'
loss a.
Generator operation. Power output = AVI
... Efficiency, r\ = , ~2 ...(3.92)
AVI + a + bl + c l
' dr\ (AV) (AVI + a + bl + cl2) - (AVI) (AV + b + 2c/)
For constant V, d /= (Denominator)2

For maximum efficiency, dv\/d l - 0 and its simplification gives c l - a


i.e. losses proportional to square of current = constant losses
It is seen from above that for both motors and generators, machine efficiency is maximum
when variable losses are equal to constant losses.
The variable loss, proportional to square of load current, depends upon the current density
specific resistance and volume of conductor material. Similarly, the constant loss is dependen
on flux density, specific density and volume of iron used in a machine As variable and constant
losses must be equal to obtain maximum efficiency, the value of maximum efficiency is
governed by the amount of iron and conductor materials used in a machine It is thus seen that
maximum efficiency at a particular load (near the rated load) can be obtained by suitab y
Proportionating the amount and quality of iron and conductor materials used in a machine.

^ w
“ Canned oy u a m ^ c a n n e r
336 Electrical Machinery

M a x im u m o u tp u t. The power output P 0 for a motor can be expressed as

n = A V I- a - b l- c l 2

The loss proportional to current I (usually brush-contact loss) is quite small '
be neglected without any appreciable error. In fact, this loss is regarded as" ^ ■ a".theref«re
a.c. machines. Under these conditions, n lndustria|

P0 = A V l - a - c I 2
The motor will have maximum efficiency when
dP0
= A V - 2cl = 0
dl
or I = A V / 2c
The maximum power output Pom is, therefore, given by

(A V ) (A V f
Pom = A V -a - c _
2c
\ > K
_ {AV) (AV)2 _ (AV)2
—a - - a
2c “ Ac ~ 4c

is * v / = w V)2/2c' the cfnciency under the —

4c 200 ac
n= x 100 = 5 0 -
L (AV) /2c
— J (AV)2
)

there are nmre^mn^abmiTst^^ ^ maximum Power 0utPut is less than 50%. This means that
power input annpart n l * osses occuring in the machine. In other words, about halfof the
cause temperature of the°nS U° a F f]Ilaxin™ m Power output conditions. Such a situation would
ture rise Hence it m a v GV1Ces to be much more than the specified allowable tempera
devices used in power s y s t o m ^ ^ e ^ e r a t e d ' that electrom cchamcal “ l>versta
tice these are nnerai*»H ot o i a operated to deliver maximum power output. Inprac-
maximum. This however is nnftK so” ,ew^lat less than rated load, at which the efficiency is
power devices The electronic * * S1 uat‘on.in electronic equipments which are usually low-
put As the total power hanHW 8mS *** desi&ned to operate to deliver maximum power out-
devices. P ,S VGFy Sma11’ effi«ency is of little significance in electronic

Example 3.37. fa) 77* transformer output in VA is given by


S = K B m 5 A, A u,

“ / r : r forr rA
Solution, (a) For first transformer, the rating is
s i = K B mS A l l A U)1

S c an n ed by C a m S c a n n e r
„ nI B a s ic C o n c e p ts o f R o ta tin g E le c t r ic a l M a c h in e s 337
" ”
For second transformer, core area
A ,2 = m 2A fl

and window area A lt)2 = m A („i.


The rating of second transformer,
S 2 = K B m 5 A i2 A ,,2 = K B m 5 (m2 A (1) (m 2 A wl) = m 4 5 ,.

The core or iron loss « volume.


Second transformer core losses
= m3 (core losses in first transformer)

Now I2R = («5)2 ‘ ^

First transformer I2R loss = (ax 6)2

Second transformer I2R loss


= (m2 a 5)2 P m— = m3 (First transformer / 2i? loss).
1 m2ai

■ Total losses in second transformer


= m3 (Total losses in first transformer).
Output ______51 — -----—
Now efficiency - Output + Losses S x(pf) + Losses x

For second transformer, efficiency


m4[Sx(pf)]
m4(Si(pf)) + m Wi
S x(pf) ...(3.93)
=" “ “1 ^
s i (Pf) + ^
Eq. (3.93, shows that greater the value of or, greater is the size of transformer and .arger

is the transformer efficiency, because ^ gets reduced.


It can be shown for rotating electrics, machines also that larger them s,ze, greater ,s the,

efficiency.
(b) For transformer A , efficiency
Output __ __gijgL>—
^ A = Output + Losses S x (pf) + Wi

^ - 1 [S\ (P/» = ‘ 1 lSl (P/)1 = 0^95 lSl (P/)l


Losses =
TU
Efficiency of transformer B. from, E ,. (3 93) is giv^nby
S x (pf)
Ifl -
C. CnA +
m
= 0.9744 or 97.44%.
1 + 0.0263

S c an n ed by C a m S c a n n e r
338 Electrical Machinery __________________

-UrUio
3.10. Machine Ratings
A name-plate fixed to the outside frame of an electrical machine records the d t
to its rating. A machine rating specifies the voltage, current, speed, excitation 3 f Perta‘n-ing
power output etc. under which it can operate satisfactorily. Here satisfactory one P/ e^lc’er»cy>
that temperature rise of the machine above ambient (or si -rounding) temper t 10n llnPlies
exceed a specified temperature when machine operates in accordance with the ,re ^0es not
name-plate. For all types of a.c. motors and d.c. machines, output power rating is • °n
(kilowatt). Older practice was to specify the power output of a.c. and d.c. motors in h ^ 'n ^
(1 h.p. = 746 watts). For a.c. generators, rating is in kVA or M VA. orse-power
Electrical machines are rated on the basis of their temperature rise resulti r
power losses in iron and conductor. The temperature rise mentioned on the name^l t°^ tlle
temperature difference between the hottest part of the winding under specified cond't’ ^
load, speed, voltage, excitation, cooling and the ambient temperature. For reliable and ' r"? °f
tory operation of an electrical machine, it should be ensured that its temperature rise re
within specified limits. The temperature rise not only affects the insulation of an elMri ^
machine but also its mechanical parts ; however, the extent of damage is more detriment?
insulating materials than to the mechanical parts. 0
Deterioration of insulation depends on the temperature as well as the time It has been
found that time to failure for organic insulation is reduced to half for every 8 to 10°C mp in
machine temperature.

pi In|Ul^ lng ^ a^ f ria1^ m0,re commonly employed in industrial machines, are classified as
Class E Class B Class F and Class H. Other classes of insulation are not of much commercial
interest. These classes of insulation can withstand the following maximum temperatures as per

Insulation class : E B F H
Maximum temp. °C 120 130 155 180

a 81™ abT relater t0 20-year working life of an electrical machine under


average conditions. These classes of insulation consist of the following materials :
e^c ^ ass ^ ‘ Synthetic resin enamels, cotton and paper laminates with formaldehyde bonding,

Class B : Mica, glass fibre, asbestos etc. with suitable bonding substances.
Class F . The materials of class B with more thermally-resistant b o n d i n g m a t e r i a l s .

L“ r “ ° ns ° f m ka' glass fibre' asbestos etc' in vari“

the had c^'dUion^or^baddu^cycl^


_ J y ' Th^more^ommori1 1^ ofhtemperature
nem °ie common machine rise>is
ratings are g°VernCd *
as under.
. ^ n t i n u o u s rating. It defines the output which a machine can deliver continuously
without exceeding a specified temperature rise above the ambient (40°C as per I.S.I.).
on fnh0rnn'mC " “ " S ; The output which a machine can supply for a specified period (S. 15.
;. I minutes) without exceeding its permissible temperature limb is called short-time
rating. The machine is assumed to start from cold. lLinPcrature limit is calico m

Intermittent rating. It is the output that a machine can give continuously without cx-
rest periods BPe tcmporaturp a "'identical duty cycles c o " l X o f lo a d in g »»«

Scanned by C am S ca n ne r
,101 B a s ic C o n c e p ts o f R o ta tin g E le c t r ic a l M a c h in e s 339
' - ------------------------------------------------------------------

The continuous, short-time as well as intermittent ratings depend upon the cooling and
^ermal capability of the machine. Out of these, continuous rating is the most common. The
loading period for a continuously-rated motor is so large that all parts of the machine attain
almost a steady temperature. Continuously-rated motors are used for fans, centrifugal pumps,
lathes, conveyors etc. In short-time rated motors, the loading period is so short that machine
does not attain steady temperature while the rest period is long enough for the machine to cool
to ambient temperature. Short-time rated motors are used for opening and closing weirs, bat­
tery-charging unit, sluice gates, lock gates and bridges etc. The machines with intermittent
rating are loaded with a train of identical duty cycles so that finally the rise and fall in tempera­
ture during each duty cycle are equal. Motors used for hoisting mechanisms, trams, trolley­
buses etc. are subjected to intermittent duty.
Short-time rated motors are designed with higher values of flux and current densities in
iron and conductor respectively. As a consequence, these motors have better torque producing
capability but lower thermal capacity as compared to continuous-rated motors.
A provision is usually made in continuous rated motors that they operate successfully for
±\0% variation in rated voltage and ±5% variation in rated frequency. The combined variation
in rated voltage and frequency should, in no case, exceed ± 10%. Continuous rated motors are
expected to have ample safety margin so that these can withstand short-time overload of 25 /<
at 90% of rated voltage without any damage to the machine.
3.10.1. Choice of power of electric machines. The choice of power rating of a motor
for any type of load depends upon the load-time graph. For loads remaining substantially con­
stant with time, the power rating P can be determined from the expression,
Tun
P= kW
lOOOq
where T = load torque in Nm ; wm= operating speed in rad/sec
and x\ = product of the efficiencies of transmitting device (gear, belt etc.) and the driven
equipment (fan, conveyors etc.).
In many industrial applications, the load requirements vary periodically and over a wide
range. For example, during upward journey of a hoist, the motor is fully-loaded but during
downward journey, the motor is almost unloaded. The problem is now to choose a continuous­
rated (C.R.) motor for such widely varying load cycles of periodic nature. A crude, yet quite often
used method is based on the assumption that losses and therefore heating of the motor is
proportional to square of the kW load. This assumption overemphasises the role of / R losses
as compared to the constant core loss. According to this method, rating of C.R. Motor - average
loss, or average heating, of the motor over a periodic duty cycle.
So for using this method, obtain kW load-time curve. Then rating of C.R. motor = average
value i f (kW l i d )2 - time curve over one periodic load cycle. A little thought process would
reveal that rating of C.R. motor = rms value of kW load - time penodic curve.
As such, this method of estimating the power rating of C.R. mohir for periodically varying
loads has come to be called rms power method. Thus, according to this method,
- , -*I/2
£ (*W r x time
rms power = rating of C.R. motor I time

is the same as

Scanned by Cam Scanner


340 Electrical Machinery
• . '— — fAri
If standstill time is also a part of the duty cycle (os in a lift, crane etc.) thp k " 3.10
modified as under : a 0Ve relatj
•ionis
1/2
__________ £ (kW)2 x time _
r.m.s.-power =
running
i u n i t i n g time
u n it; +
t (standstill
\ s t u i i u a t n i tim
i n i i ue/k)
/

Here the constant k accounts for the poor ventilation during the standstill t' ^
is no forced cooling. For open-type motors, k = 4 and for closed motors, k ~ \ Wben there
note that r.m.s.-power method can be used only if period of duty cycle'is sm 11 1S 'mportantt0
the time for the motor to attain a steady temperature. as Spared
to
Though r.m.s.-power method does not give precise
results, yet it is used quite often. The error involved in
using this method is partially offset by choosing a higher
standard rating of the motor. Thus the need for precise
computations is avoided with the rounding of calculated- m
rms-power rating to a higher commercially available
motor size. For example, if r.m.s. power method gives 01 B
motor rating as 46 kW, then a motor with continuous w
rating of 50 kW should be selected.
For duty cycles with high-torque peaks, a motor
selected on the basis of r.m.s.-power method cannot fur­
nish such torques. For these types of duty cycles short­
time rated motors are better suited as they possess better F**■ 3 52- Pertaining to continuous and
torque producing capabilities. short-time ratings.

tho ^ C° ntinU0US-rated moto«- can deliver higher power output for short-time ratine Let? he
the power of a continuous-rated motor. Under rated loading P it will heh t ^ g; UtP' be
sible temperature rise 9 ,as shown in Fig 3 52 TE p tp ♦ UP
f own in t ig. 3.52, The temperature rise at any time t is given by
0 = 0^.(1 _ e~t/xi<)
where zh = Heating time constant.

as a
AB, which is much u . +k .. ' x» would attain a final temperature rise

time rated motor roofs down^o ambient tem'6 ‘ 7 * ™ ^ ^


duration the final temperature rise w o u w l k 7 1,16 short'timCloadiugP.to
motor, when used with lonH P f u aga>n be AB. This means that continuous rated
the motor, th^^mnwature^s^H i?8 ^ura^ on is underutilized. F o r full utilization of
that final temperature during t time loadin£ should follow the curve AB’ C®
short-time rating t is more t J CUFVe A B C can be flo w e d only if load during
for short-time loading is less thfl ' t * 1 Ioad be P* which is more than Pr. In case the period
for the permissible temperature Hsp J r n ^ higher than ?x Can be delivered by the ^
reduced, the short-time rating of Hm I- * Can bbus be inferred that as the loading Perl° lS
If the motor is made to dpliv u ° n muous' rated motor rises,
rise would be 0/ as shown in F i / s U ° r ^ ° l0ad continuously. the final temperature
time tx. This can be expressed as U perabure rise, in this case, wcmld be a

0/ = 0 / ( l - e ~ fA>)
or 0 ,

...(3.96)
0 ^ - 1 1 - e x p ( - t x/ x h ) ]

am scanner
Basic Concepts o f Rotating Electrical Machines 341

Here temperature rise Qf is never attained by the motor.


As heating, or temperature rise, of the motor depends on loss, Eq. (3.96) can be written as

ff = ^ = | l - e x p ( - ( , / t t )l ...(3.97)

rt.],ere losses Wr and Wx are corresponding to loads Pr and Px respectively. Loss W, at rated load
p is given by
W, = core loss, Wt. + ohmic loss, W0
Let the ratio (Wc/W 0) be a. Then
W,. = aW o + Wo = (l + a )W 0 - (3-98)
But ohmic loss is proportional to current squared. As load on motof is proportional to cur
rent (except series motor), it follows that ohmic loss is proportional to square of load, as
result, ohmic loss at load Px can be written as proportional to (Px) . Since W0 is the o mic os
at rated load P r, total losses Wx at load Px can be expressed as
Wx = Wc + W0 (Px/Pr)2 = a W0 + W0 (Px/ P f
...(3.99)
= [a + (Px/Pr)2] W0

From Eqs. (3.97) to (3.99),


Wx a + (Px/Pr)
Wr ~ 1+a 1 - exp ( - tx/W
-.1/2
1+ a
- a
P* = Pr 1 - exp ( - )

In case core loss Wc is neglected, then a = 0 and Eq. (3.100) becomes


-.1/2
p -p \ I-----------
^ r 1 - exp ( - tx/\h)
From Eqs. (3.100) and (3.101), short-time rating PMfor duration tx can be obtained for a
motor with continuous rating P .and heating time constant t*.
Example 3.38. For the periodic load-time plot shown m Fig. 3.47, ft an expression o
r.m.s. value over the periodic time T. -iu(,n hv
Solution. For any time 1 measured from <„ the ordinate ,s g,ven by
„ H2- H , H t^ H ± t

Square of this ordinate is

(Tf)

R M S value o f this e x p r e s s i o n j . .
i. from lft o t ' e from 0 to 7 , ; dividing it by penodie time
T and then taking its square root,
r.m.s. value
77 T Z "(ff, - Hi)2 <‘ d , + U h W i z M i l
1
w T
J , . H 2, dt + ----------- d l+ T,
F ig. 3 63 Pertaining to E x a m p le 3.38.

S c a n n e d by u a m b c a n n e r
342 Electrical Machinery

i \ h ,2 T, + (H22 + H ,2 - 2H, ff 2) Y + (Hi - w ,2) T,]

1 /2

i )(//? + + H , t f2) r , / 3

Exam ple 3.39. A motor driving a coal-mine equipment has to supply a load rising Uny
ly from zero to maximum o f 2000 kW in 30 seconds during the accelerating period, 1000kw/
60 seconds during the full load period. During deceleration period o f 10 seconds when reBe
tive braking takes place, the kW returned to the mains falls from an initial value of600kW°"
zero uniformly. The interval for decking before I•
LOAD ~ IN
the next load cycle starts is 20 seconds. E s­ kw
timate a suitable kW rating o f the motor based
on r.m.s.-power method. 2000
Solution. The variation of load power over
a duty cycle of 120 seconds is illustrated in Fig.
3.54. The r.m.s. value of this cycle gives the kW 10 0 0 kw I
rating of a continuous-rated motor. /

The slope of the load-time curve during ac­


celeration is 2000/30 kW per sec. and that during
1,1
deceleration is 600/10 kW per sec. At any time t f a
measured from the zero of the load-time curve, the r '4 r
-6 0 0 P \
f200 ' kW and (60 1) kW respectively
load kW is j~30 -|- 60— — |
TIM E IN SEC.-
during acceleration and deceleration periods. The
F ig . 3 .5 4 . P e r ta in in g to E xam ple 3.39.
r.m.s. value is therefore given by r.m.s.-power
* 30 / f t n n \ f 10 1/2

J dt + (1000)2 x 60 + J0 (60t) * d t + 0 x 2 0
120
1/2
\2
200
® - + 6 0 x 106 + 1200 x 103| = 918.33 kW.
120

So choose a motor of rating above 918.33 kW, say 950 kW.


E xam ple 3.40. Points on the motor-duty cycle, estimated on the basis o f the proposed rolling
schedule and previous experience with rolling mills, are given in the following table:

Time, sec 0 5 36 39 55 80
R epeat cycle
Output, kW j 150 1000 1400 300 150 150

The complete curve can be obtained by joinin g these points with straight lines.
Specify the continuous kW-rating o f the motor.
tampk'
Solution. The load-time curve for one duty cycle is shown in Fig. 3.55 for this ex
Continuous kW-rating can be obtained by taking the r.m.s. value of this load-time eye
Example 3.38, the r.m.s. power can be obtained as under :

0 —5 sec ] Hi = 150, H 2 = 1000


5 - 3 6 sec ; H x = 1000, H 2 = 14
and so on.

UUUl ll It^VJ vjy W CJ I I IO U C 1 I II I t^ l


V

Art. 3-101 Basic Concepts of Rotating Electrical Machines 343

LOAD IN
kw

36 39 55
TIME IN SEC.-
Fig. 3.55. Load time graph for Example 3.40.

.*. r.m.s.-power = I f (1502 + 10002 + 150 x 1000 + ^ (10002 + 14002 + 1000 x 1400)
80 I3 w
1/2

+ - (14002 + 3002 + 1400 x 300) + ^ (3002 + 1502 + 300 x 150) + 25 x 15021


3 « J■
r i l l 1/2
= — {1954,166.6 + 45,053,333 + 2,470,000 + 840,000 + 562,500[J
80
= 797.49606 kW
Choose a motor with continuous rating of 800 kW.
E xam ple 3.41. (a) Temperature rise o f a 200 unity
efficiency of9
,w as found to be 29°C after one hour and
%
8 on
fuU-load ohmic loss is 3 times the iron loss. What is the final steady temperature rise o f the
transformer on rated load ?
(b) I f the transformer cooling is improved by using a fan so that the effective heat dissipation
is increased by 20%, find the new kVA rating (i)for the same final temperature rise os above and
(ii) for a temperature rise o f 78°C.
Solu tion. (a ) From Eq. (3.95),
20 = 0^(1 —e ~ 1/x>)

and 34 = 0^(1 —e_2/x*)


34 l - e ~ 2/T* _ i , e-
•• 20 l - e ^T7Th~L 6

Its simplification gives T/, = 2.804 hrs.


20 r = 66.67°C
1 - exp ( - 1/2.804)
Hern 0, is the fine! steady temperature rise of the transformer on rated load
(6), ( i)F o r t h e s a m e t e m p e r a t u r e rise , with increased h eat dissipation, new allowable losses

V, are
Wx = 1.2 Wr. Here a = IVC/W 0 = 1 /3

From Eq. (3.98) and (3.99),


W\ a + (Px/Pf
1+ a

M
S c an n ed by C a m S c a n n e r
344 Electrical Machinery

1
3 + {px/pry
1.2 _ L
or =

l+l
Px = 1.1255 x 2 0 0 = 225.1 kVA
(ii) For allowable temperature rise of 78°C, the new permissible losses are
78
W = x 1.2 Wr. As in part (i ),
* 66.67

78
x 1.2 =
66.67 i "
1+i
or Px = 1.241 x 200 = 248.2 kVA.
Exam ple 3.42. The efficiency o f a 3-phase, 100 kW, 440 V, 50 Hz induction motor is 90% .
rated load. Its final temperature rise under rated load conditions is 40°C and its heatin tim
constant is 180 minutes. For the same temperature rise, calculate its one hour rating incase
(a)
constant loss is equal to the variable loss at rated load, (b) constant loss is neglected
Solution. Here xh = 3 hours,
(a) a= 1
1/2
1+ 1
From Eq. (3.100), Pz = 100 - 1 = 246 kW
1 - exp ( - 1/3)
1/2
1
(6) From Eq. (3.101), P = 100 = 187.8 kW
1 - exp ( - 1/3)
This example demonstrates that neglect of core loss gives pessimistic results for the allow­
able one-hour rating.
Exam ple 3.43. One hour rating o f a machine is twice its continuous rating. Its final steady
temperature rise, i f operated on one-hour rating, is twice o f that under rated load. Find the ratio
o f core loss to ohmic loss at rated load.
Solution. From Eqs. (3.97M 3.99),
*f W r i+ a

¥ a + (Px/Prf
1 1+ a
or or a = 2.
2 a + (2)"
Thus the ratio of core loss to ohmic loss is 2.
3.11. C ooling (Loss dissipation)
Cooling of electrical machines is essential for dissipating the heat generated by various
losses and thus to prolong the life of insulating materials. In small electrical machines, naW
cooling is adequate. In these machines, cooling by natural radiation and convection associ
with random air circulation inside the machine body is enough to keep their te* pf ef0l-
within limits. However, as the machine size increases, cooling becomes more difficult, i
lowing example illustrates this fact. ,^
Exam ple 3.44. A rotating electrical machine has its linear dimensions n\ ^,neSf 1vesatne
dimensions o f another machine. Compare their losses and cooling. Both the machines ha
flux and current densities.

S c a n n e d Dy u a m ^ c a n n e r
^ ■ —

Basic C o n c e p t o f Rotating Electrical Machines 345

«dS As thT w eilh t™1^ ! ? .! 616? " ' 31 ma<* in c’ core or ir° n ioss depends on the weight or iron
used. As the weight - volume o f iron x its density, it can be stated that
Core loss « Volume
First machine coreloss, Pcl « (volume)

Second machine coreloss, Pc2 m 3 (volume)

*'• P c2 = m 3 P cl

Now I2R loss = (a8)2 ^


a

First machine I 2 R ioss = (a,5)2 — 1


Qi
Second machine l 2R loss = (m2 a l 5)2 —™
2_
m aj
= m3 (first machine I2R loss)
/. Total losses in second machine
3 ii •
= m (total losses in first machine)
Cooling surface area of second machine = m2 (cooling surface area of first machine)
For second rotating electrical machine of bigger size, the losses per unit area
_
Losses in second machine
Surface area of second machine
or losses per unit area in second machine
_ m3(losses in first machine)
m 2 (cooling area of first machine)
= m (losses per unit area in first machine)
This shows that for second machine of bigger size (m > 1), the loss per unit area is m times
that in first machine. In other words, heat to be dissipated per unit area rises as the machine
rating, or size, increases. It may therefore be stated that larger machines are difficult to cool
than smaller ones and hence require more elaborate cooling arrangements.
Forced air cooling is used for large machines. In this scheme, air is first passed through
cleaning filter and then forced into the machine for cooling purposes. A still better method of
providing clean cooled air is the closed-circuit ventilation system. This is used for small-rating
turboalternators. In closed-circuit scheme, hot air from the machine is cooled by means of a
water-cooled heat-exchanger and then returned to the machine by a fan. In order that heat
arising from the losses is removed more efficiently, cooling ducts are provided in large machines
along their core length.
Hydrogen-cooling. Large-rating turbogenerators require compactness from the view­
point of economy and transportation. For such generators, closed air-circuit ventilation system
is not sufficient to cool the machine effectively. The fan size required to circulate air also in­
creases considerably making the air cooling of large turbogenerators an uneconomical proposi­
tion. For such machines, therefore, a better coolant should replace the air. Hydrogen has been
found a well-suited cooling medium and is more common in the totally enclosed ventilating
system of large turbogenerators. Hydrogen as a coolant has the following advantages over air :
(i) Efficiency and noise. Hydrogen density is only about 0.07 that of air at the same
temperature and pressure. There are, therefore, much less windage and ventilation losses. As
a result, machine efficiency is more.

S c a n n e d by C a m S c a n n e r
i,

346 F.lectricul Machinery

Since tho revolving pnrts rotate in low-density hydrogen, the noise produced is less
(it) Cooling. On an equal-wcight basis, specific heat of hydrogen is about 14.5 times th
of air. Heat storing capacity is given by
mass (m) x specific heat (s) x temperature rise (0)
If heat stored in air is m s 0, then heat stored in hydrogen is given by
(0.07 m) (14.5 s) (0) « ms 0.
This shows that for the same temperature and pressure, heat storing capacity per Unt
volume is same for both hydrogen and air. But heat transfer between hot parts ofthemachin
and the cooling gas is about 1.5 times with hydrogen than with air. This means that cooling
with hydrogen is more rapid.
(iii) Life. When air is used in high-voltage machines, then during corona discharge the
oxygen and nitrogen in air may form ozone, nitric acid and other chemical compounds which
may react with organic insulating material. This reaction may harm the insulation, thus reduc­
ing the life of insulating material.
When hydrogen is used as a coolant, ozone and other chemical compounds due to corona
discharge cannot be formed because of the absence of oxygen. The life of insulation is therefore
increased and the maintenance expenses are reduced.
(iv ) Fire H azard. Fire hazard inside the machine is minimised because hydrogen-air mix­
ture does not explode so long as the hydrogen content is more than 70%.
(v) Rating. Hydrogen pressure inside the machine system is maintained at a pressure
above atmospheric. With this, the air cannot leak into hydrogen circuit to form an explosive
mixture. However, hydrogen leakage can only be outward. Hydrogen cooling at 1,2 and 3 times
the atmospheric pressure can raise the rating of the machine by 15, 30 and 40% respectively
above its air-cooled rating.
The stator of hydrogen cooled machine must be gas tight and explosion proof. For this pur­
pose, oil-filled gas-seals at the rotor shaft ends are necessary.
D irect cooling. It is also referred to as conductor cooling or inner cooling. The tur­
bogenerator output can be increased by raising the hydrogen pressure inside the machine. But
beyond a certain pressure, the increase in output is not economical with the rise in hydrogen
pressure. This led to the development of direct cooling of large machines.
With direct cooling of turbogenerators, it is possible almost to double their output with the
same physical size. In other words, for the same output, the physical size of the turbogenerator
can almost be reduced to half. Here the coolant (liquid or gas) is forced through ducts inside the
conductors. Thus the thermal barrier presented by the insulating material is largely overcome
and conductor losses are very effectly absorbed by the coolant.
(a) D irect gas cooling. Large stator cores for turbogenerators are provided with bothaxial
and radial cooling ducts as shown in Fig. 3.56. For rotor of 100 M W or above, the rotor conduc or
consists of hollow rectangular tubes as shown in Fig. 3.57. The hydrogen gas is admitte
these tubes through in sulatin g s t a t o r CORE RADIAL COOLING DUCTS
flexible connections at the ends. I ______________________ jL_
D ir e c t w a te r c o o lin g . W ith A X IA L
T
ratings of 1000 M W or so, hydrogen COOLING:/
cooling is not sufficient to cope with DUCTS
the dissipation of large losses. For
such ratings, the volume of hydrogen
of
required is so large that its use be­ Fig. 3.56. Radial and axial cooling ducts in stator c
comes uneconomical. As the rotor core large machines.

"Scanned by Cam Scanner


A rt. 3.12)
-------------- B a s ic C o n c e p ts o f R o ta tin g E le c tr ic a l M a c h in e s 347
length in creases for large "WEDGE r-wcnrp
ratings, the middle portion may WtDGE rWEDGE
not be cooled efficiently even
with the use of hydrogen. These
limitations of hydrogen cooling COOLING
^CONDUCTORS DUCTS
can be overcome by employing
~7 CONDUCTORS
water cooling.

Turbogenerators of the
highest ratings are likely to
have hydrogen-cooled stator
cores and direct water cooled —COOLING
DUCT
stator and rotor windings. The (a)
advantages of using water over
Fig. 3.57. Turbo-rotor conductor cooling : (a) conventional
hydrogen are as under : (6) direct gas (c) direct water.

therefore,Tarry “ o r T h e a f “ 8eVera‘ tHa‘ of ^ r o g e n i *«.

(«) Though water speed in the cooling ducts is limited to about 2.5 m/sec for avoiding
erosion and cavitation, yet water cooling is more efficient than hydrogen.
(itt) The duct area for water may be small, this permits more space for copper in the slot.
The disadvantages of direct water cooling are that (i) cost of cooling system becomes more
and (ii ) water should be highly purified and distilled.
3.12. M achine Applications
Machine application means choice of an electrical machine well-suited to meet the demand
of a load. A judicious selection of a machine for a given load requires that machine terminal
characteristics and load characteristics are known to a fair degree of accuracy. In addition, it is
essential that machine and load characteristics are compatible with each other for stable and
efficient operation of the machine-load combination.
For many industrial applications, electric motor is usually fed from a constant voltage
source. A motor connected to such a source would have one speed-torque curve as shown in Fig.
3.58 (a) for a 3-phase induction motor. The load requires torque that may, or may not, vary with
speed. In Fig. 3.58 (a), the speed-torque curve for a fan type load is shown by a dotted curve.
The steady operating point P is obtained where the load characteristic intersects the motor
characteristic. At this point P, the torque developed by the motor electromagnetically is equal
to the torque absorbed by the load mechanically.

TORQUE TORQUE
(fl) (6)
Fig. 3.58. (a) Speed-torque characteristics of a motor-load system,
(ft) Speed-torque curves o f various types of mechanical loads.

'" S c a n n e d b y C a m S c a n n e r
348 Electrical Machinery j ArL 3 J2

The torque required by a mechanical load, in general, depends upon the type of drivo,,
equipment. The speed-torque characteristic of a load represents he change in speed as the lnaJ
torque is varied. For various driven mechanisms, the speed-torque characteristics may broadly
be classified as under :
(а) S p e e d -t o r q u e c h a r a c t e r i s t i c s i n d e p e n d e n t o f speed . For some driven
mechanisms, load torque T 1 does not depend upon the operating speed. This is indicated bv
curve 1 in Fig. 3.58 (6). Examples of such loads are cranes during hoisting and loads in which
friction is dominant like reciprocating pumps, air compressors etc.
(б) Parabolic speed-torque characteristics. This is illustrated by curve 2 in Fig. 3.55
(6). In such driven equipments, the load torque is proportional to some power of speed. Ex­
amples of such loads are fans, centrifugal pumps, propellers in ships etc. In fans, the load
torque is almost proportional to square of speed.
(c) H vperbolic speed-torque characteristics. These type of characteristics are il­
lustrated by curve 3 in Fig. 3.58 (b). For such loads, the dn\ en equipment requires a torque that
is inversely proportional to speed. In other words, the load torque required is low at high speeds
and high at low speeds. Such type of characteristics are possessed by lathes, milling machines,
machine tools, winches etc. Since these loads are associated with high torque at low speeds and
low torque at high speeds, the power required to driv e such loads remains substantially con­
stant.
(d) L in ear speed-torque characteristics. These type of characteristics are indicated by
straight line 4 in Fig. 3.58 (6). Such characteristics are possessed by viscous friction and loads
in which loading torque is directly proportional to speed. Calendering machines, eddy-current
brakes and generators feeding fixed resistors display such type of speed-torque characteristics.
In all types of mechanical loads discussed above, the motor must be able to supply the
starting as well as the running torques wdthin specified limits of current and temperature rise.
The electromagnetic torque developed by a motor may remain constant with load or may
vary wuth it. Based on this, the speed-torque characteristics of motors may be classified as
under:
(a) F la t-ty p e ch aracteristics. Some motors develop electro-magnetic torque which does
not depend on their operating speed. Such a characteristic is indicated by curve 1 in Fig. 3.59
(a). These type of characteristics are possessed by synchronous and hysteresis motors.
(b) S h u n t-ty p e ch aracteristics. In motors possessing shunt type characteristics, the
speed drops from no-load to full-load by a few percent as shown by curve 2 in Fig. 3.59 (a).
Examples of such motors are a.c. induction motors and d.c. shunt motors.

HI
o
GENERATOR /
p/

CURREnT
{1a) 01
1lb)
Fig. 3.59. (a) Various types o f speed-torque characteristics o f electric motors.
(6 ) Volt-ampere characteristics o f generator-load combination.

S c an n ed by C a m S c a n n e r
(c) Series-type characteristics. In some motors, high electromagnetic torque is as­
sociated with low speed but low electrical torque with high speed. Such characteristics, indi­
cated by curve 3 in Fig. 3.59 (a), are referred to as series-type characteristics. A.C. series (also
called universal motors in small sizes) and d.c. series motors possess these types of charac­
teristics.
The speed-torque characteristics shown in Fig. 3.59 (a) can be raised or lowered in adjus-
table-speed drives. D.C. motors are best suited for such speed-controlled drives than a.c.
motors.
Just as in a motor, the operating point in generator-load combination is obtained where
there characteristics intersect each other. In Fig. 3.59 (6), solid curve shows variation of output
voltage of a generator (d.c. shunt generator or alternator) as its load current is increased. The
voltage-current graph for a load is shown in this figure by dotted curve. When this generator
feeds power to this load, operating point P is obtained by the intersection of their V-I charac­
teristics. At point P, the power given out by the generator is equal to the power absorbed by the
load. When several generators are operated in parallel, as in power generating stations, it is
desired that terminal voltage should remain almost constant over a wide variation of load.
However, when a motor is fed from a single generator, the output voltage of the generator may
be adjusted in a particular fashion so as to obtain a wide speed-control of the motor.
It is seen from above that the most outstanding features of electrical machines are the
torque-speed characteristics for a motor and V-I characteristics for a generator. Equally impor­
tant are the limits between which these characteristics can be varied without any damage to
the electrical machine. Other relevant economic features are efficiency, pf, initial cost and effect
of losses on operating cost, heating and ratings of electrical machines. Several of these impor­
tant features are discussed for the common types of electrical machines in the present book.
Steady state performance of electrical machines has been discussed in detail in this book.
However, the role played by rotating electrical machines in modern technology has become so
wide that an understanding of their transient and dynamic behaviour is of paramount impor­
tance. For this purpose, the mechanical features such as shaft elasticity, system moment of
inertia and friction must be included in the equations governing the performance of a machine­
load system. Such an analysis, being quite complex, is beyond the scope of the present book.
This type of analysis is, however, presented in the companion volume “Generalized Theory of
Electrical Machines” written by the author of the present book.

PROBLEM S

3.1. (a) Give the physical concepts about the production o f electromagnetic and reluctance torque in rotating
electrical machines.
(b) Show that the net electromagnetic torque developed is zero if the rotating electrical machine has
different num ber o f poles on its stator and rotor.
3.2. (a) Explain the production o f torque through the concept o f interaction of magnetic fields. Using this
concept, show that no electrom agnetic torque is produced if (i) stator has 4 poles and rotor has 2 poles, (n)
stator has 6 poles and rotor has 2 poles.
0b) Explain the concept o f alignm ent torque. Using this concept discuss the condition under which
reluctance torque will be developed in an electrical machine having cylindrical stator and salient-pole rotor.
[Ans. (6 ) The exciting winding must be on statorl

3.3. fa) From the construction point of view, enumerate the common essential features of rotating electrical
machines.
(6) Describe the constructional features of both types of polyphase induction motors. Why has induction
motor come to be called so ?

^ a n n e d by C a m S c a n n e r
350 Electrical Machinery-------------------------------------------------------------------------------------------------------- — -jP rob . 3
3.4. ia)D escr.be the advantages o f providing held w inding on the rotor end arma.nre winding

in “ < » G ilX r n tC Iw o f »o*h < W " ofsyn ch ron ou s machines. W hy has sy n ch ron y ^

Z . G iv e 'fte constructional features o f * machines. W ith a suitable diagram, describe the c o n e t n * ^


details o f a com m utator also.
3.6. Explain, with suitable diagrams, the fo llo w in g :
ia 1stator o f a 3-phase induction machine and the different materials used for its construction,
6 squirrel-cage rotor o f a 3-phase induction motor and the different materials used for its co„ltniclim
(c) w ound-rotor o f a 3-phase induction m otor alongwith the various materials used,
id) 2-pole, 3-phase salient-pole stator o f alternator with arm ature having 6 s ots,
(e) 4-pole, 3-phase salient-pole stator o f alternator with arm ature having 12 slots, two parallel paths with

rotTr'spider having four salient poles with damper bars, cylindrical rotor having four-pole field .indin,
and the materials used for their construction,
<g) different parts o f a dc-m achine stator and the m aterials used for their construction,
(h) different parts o f a dc machine rotor and the m aterials used for their construction.
3.7. (a) In what respect are ac and dc generators (i) sim ilar and (ii) different.
(6) How can a dc generator be converted into a 3-phase delta-connected alternator ?
(c) What is yoke, pole core, pole-shoe in a dc m achine ? W hat purpose do they serve ?
(d) W hy is pole-shoe area more than the pole-core area in dc m achines ?
(e) W hat is a com m utator ? W hat purpose does it serve in a dc generator and motor ?
(f) Distinguish between armature w ipding and field winding.
(g ) Distinguish between load current and exciting current.
3.8. (a) W hy are some parts o f electrical m achines lam inated ? --
(b) Which parts o f the following m achines are lam inated and w hich are not ? DC machines, 3-phase
synchronous m achines, 3-phase induction machines.
(c) Why is yoke lam inated in som e dc motors ?
(d) Which type o f rotor o f a 3-phase induction motor permits the insertion of external resistance in its circuit.
(e) W hy are rotor bars skewed in a squirrel-cage rotor ?
3.9. (a) Give the physical concepts o f the follow ing :
(i) Flux-density wave is sinusoidally distributed in space.
(ii) Pulsating-stationary flux.
(b) Distinguish clearly between the electrical and m echanical degrees. Show that
P

PN
and Hz.

3.10. (a) Explain the follow ing term s with respect to rotating electrical machines :
Pole-pitch, coil, coil-side, full-pitch coil, chorded-coil, chording angle, coil span.
(6 ) Show that for a P -pole m achine, the total flux per pole is given by
A

* = p B P lr
' (c) D efine the term s : conductor, end-connection, turn, tw o-turn coi1. 0f its relat‘ve
3.11. (a) D erive an expression for the e.m.f. generated in one N -tu m full-pitch coil because ^ ^ that
motion with respect to tim e-invariant flux (j). H ence show that the generated e.m.f. lag8 ^
produces it.
if •
.
Iternator "
(b) C alculate the phase and line voltages o f a three-phase star-connected 4-pole
total flux per pole 0.30 Wb. Each phase has a concentrated w inding o f 10 full-pitched ^ 666 33 V, H54‘
velocity between arm ature w inding and field flux is 1500 r.p.m .. * D ’

S c a n n e d by C a m S c a n n e r
----------------------------------------------------------------------Basic Concepts o f Rotating Electrical Machines 351

3.12. (a) Show that e.m.f. generated in a short-pitched coil is reduced by a factor cos | where c is the
chording angle.

* - S th Sh° W aV' ? e' Value ° f e m f generated in one condnctor ia P « n where


=.vDression for the ! m f r L ^ , P J P°.r “ " = relative speed betwee" nul< <“ "> eonductor. Hence derive
an expression tor the e.m.f. generated in the armature winding o f a synchronous machine.
J l Z e o r d J l ' ° f voltage equations, as a function o f time, for the three phases o f synchronous machine
wjth phase order a. b, c. Take time on gin at the instant flux linkages with phase a are (i) zero and (ii) maximum.

» 'yrite, a set o f voltage equations as a function of time for the line e.m.fs for
VrJ? alternator. Show that line voltages are also displaced from each other by a time angle of
1u\J •
(H int, (d) Line voltage ellb = e„ - eb etc.J

[Ans. (c) (i) ea = <2 E cos cof, eb = ^12 E cos (of - 120°), ec = V2 E cos (orf - 240°)
Ui) ea = >f2 E sin tat, eb = <2 E sin (lot - 120°) etc.
(d) (i) eab = V3 Emcos (a* +30°), = V3 Ein cos (col - 90°), eea = V3 Emcos (cot + 150°); E,„ = V2 £
(« ) eab = V3' Emsin (col + 30°), = V J Emsin (col - 90°), efa = V3 E,„ sin (col + 150°)|

3.13. One A^-turn coil on cylindrical stator is short-pitched by e electrical space degrees and salient-pole
rotor has 2-pole structure which produces sinusoidal flux along the air-gap periphery. Derive general expression
for the flux linkages with the chorded coil in terms o f peak flux density and core dimensions. Find also the
ratio o f fractional-pitch coil flux linkages to that o f the full-pitch coil. |Ans. 2 fl,, l.r cos e/2, cos e/2]

3.14. O ne Af-turn full-pitched coil rotates with an angular velocity o f wf rad/sec with respect to fleld flux.
Give an expression for the flux linkages with the coil as a function o f time t and hence derive therefrom
an expression for the em f generated in the coil. Discuss nature of the em f expression so obtained.
3.15. The stator o f a 4-pole, 3-phase, 50 Hz induction machine is supplied from 3-phase, 50 Hz supply.
The slip rings o f the machine are open circuited. It is desired that frequency o f the voltage across slip rings is
(a) 50 Hz (6 ) 75 Hz (c) 100 Hz (d) 25 Hz. Find the speed of the rotor to obtain these frequencies.
(A ns. (a) Zero speed, (6 ) 750 r.p.m. against the direction o f rotating field, (c) 1500 r.p.m. against the
direction o f rotating field, (d) 750 r.p.m. in the direction o f rotating field.)
3.16. (a) Explain how a polyphase induction motor o f the slipring type can be used as a frequency changer.
(b) The frequency o f the e.m.f. in the stator circuit o f a 6-pole induction motor is 50 Hz. If rotor circuit
e.m.f. has a frequency o f 2 Hz, then find :
(*") the direction in which the rotor is revolving with respect to the rotating field.
(ii) the slip and the rotor speed. (Ans. (b) (i) in the direction of rotating field, (it) 0.04, 960 r.p.m.1
3.17. (a) Explain how a.c., generated in the armature circuit o f a d.c. machine, is rectified to d.c. by means
of a comm utator.
(b) Derive an expression for the e.m.f. generated in a d.c. machine.
3.18. (a) A 10 kW , 250 V, 8-pole, 600 r.p.m. lap-connected d.c. generator has 400 armature conductors. At
rated voltage and current, arm ature ohm ic losses are 150 watts. Compute the useful flux per pole.
(b) A 6-pole d.c. generator has a rated speed o f 800 r.p.m. Calculate frequency o f the e.m.f.
(i) generated in the arm ature conductors and
Ui) at the brushes.
[Hint, (a) = 150 w atts. Calculate E„ from the relation E„ = V, + /„ ru and then the flux per pole.l
(Ans. (a) 0.06344 Wb (b ) 40 Hz ; 0 Hz I.

3.19. (a) W hat are the advantages o f distributing the windings in slots?
(b) Define distribution factor and show that its expression for the fundamental frequency component is
given by gi-n ? Y / 2
q sin y/ 2
(c) Define the term s phase-band and phase-spread. In 3-phase a.c. machines, explain why a phase-spread
of60° is preferred to a phase spread o f 120°.

S c an n ed by C a m S c a n n e r
r
352 E le c t r ic a l M a c h in e r y

IH in t, (c) For o = 60°. k = 0.955 and for a = 120°, = 0.827. T hus the phase e.m.f. js r e .
riven arm ature current, a.c. m achine ou tpu t is also reduced. F or a narrow nk ? 3nc*^ a
. . 0.955 . Se'sPread ftr»
the phase e.m.f. and output for a given cu rren t are q ^ 7 = 1-15 tim es their corresp ^
for the w ide phase-spread o f 120°. n8valuej>
Also for o = 60, k^ = 0.637 and for o = 120°, = 0. T h ou gh o = 120° eliminates third
com pletely from the phase and line e.m .fs., yet 60° phase-spread is almost al arrnonics
because third harm onics can be suppressed from the lines by connecting the k ^ * emP'°yed
delta.) b Ie Phases in star oj
3.20. (a) E num erate the advantages o f fractional pitch w inding.
( b ) Define pitch factor and derive an expression for it.
(c) W hat should be the value o f chording angle for elim in atin g
(i) third harm onics,
(ii) 5th harm onics and
(iii) 7th harm onics from the phase e.m.fs.
(d) Define w inding factor. W hat is its value for
(i) full-pitched distributed winding and
(ii) short-pitched concentrated w inding ? (Ans. (c) 6Q0 360 ^

3.21. (a) Discuss the effect o f pitch and distribution factors in an alternator.

is 500 m P^ ! r 0Sl ar‘ ; r neC! ef alternator has a ratinS o f 810 kV A at 2.5 kV and has 12 poles. Ratedspeed
00 r.p.m. There are three slots per pole per phase and num ber o f conductors per slot is 5 For a full nitJLi
winding, com pute the flux per pole to give rated term inal voltage at no load. * “

IAns. (6 ) 0.07522 Wb!

phase spread I V w t T l W ^ ^ ^ ^ ^ phaS6’ Caku,ate the breadth * «

(b) An alternator has 3 slots per pole per phase. For a coil span o f 8 slots,find the winding factor.
IA ns. (b) 0.9598, 0.8312 (6) 0.94521
iron core and r o ta te s ^ a ^ tu w f tu.rn_co d ’ tbe en8s o f which are connected to slip-rings, is wound on a cylindrical
waveform o f the em f for one rev°\ « 6 ’ <• k p arc being 75% o f the pole pitch. Sketch and account for the

ne o n h e T ^ n T h V p e l e^C* ^ (i> ° f ‘ he “ lh' * * Uf.Ai .S


V., JiyS
wi

T he coil span is 12 slots connec^e3 a p e m a tor has 120 stator slots and each slot has 10 conductors
em f harm onics is ^ h m in a te d * P°'‘ “ ° 12 W b - ‘ he induced em f between lines. Which rffc
(I.A .S .,

fa) r X X X X a v e ’ * 2 2 5 ° “ nd the last 2 2 '5’ - For 75% ° f P” 1' P^ 11” 135’ ' “"
(b) 8400.3 V, 5th harmonic.)
3.24. A 3-phase, delta-connected, 4-pole 50 a ir: v • j w . i . a in turn
coils, each short-pitched by one slot I f stator leek induction m otor has 36 stator slots and 101
rotating flux-density w ave in case the stator o f b i d u f f ,mpedanc.e 15 neglected, find the flux pei-pole®
winding. induction m otor has (a) single-layer winding (6) d°ublt‘ /L
3 2*5 A A. , J , ‘A ” 8 - <«> 0 03295 Wb lb ) 0 016

stator' s L w ; S S , ydeer X r e t n d t , UCt,° " T t " ‘ S SUPPUed from 416 V ’ 3'Pha“ " ^ 2
air-gap radius 10 c m ^ n d aair-rg r p nien^haf m m Urn Ch° rded by ° ne slot Its sta’tor' length iS 3° Cra' "1' 8n
(a) For negligible stator im pedance finH ikr. n .
<61 Find the line current drew n by X m o r in ^ ' t ? r° ,a ti" 8 " UX
(H in t. (6 ) Use Eq. 3.701 n° ' load b s s e s ore 1400 w alts' m 1SS67J«

3.26. A 3-phase, 4-pole, 50 He, star-connected l h lAna. (o) 0.0206


and w inding factor is 0.96. Its other data is as under*rbo6enerator has distributed field winding
Arm ature series turns per phase = 42

Arm ature radius = 0.4 m, Arm ature length = 4 m. G ap length = 2 cm.

S c an n ed by C a m S c a n n e r
Prob. 3]
J r , - Concepts “ < Machines Vta
W indmg factor for arm ature winding = 0 955-------------------------------------------------------- — ------ —
For no-load arm ature em f o f 11 kV finH i

3.27. A 3-phase, 8 -pole, narrow-spread d hi “ CUm!',t reqUired lA


IA"n s'. 165.74 A|
modated in 72 slots w ith 2 coil-sides per s“ t Windi" e of ,h ‘ ‘ " "M u r e o f an alternator Iis accom-
as possible
— — h-
IHint. Order of a,o, J Z Z Z ," ? W " ,t " 0l,‘d “ ,h' irW," d' " 8 ’
IA n a . C oil sp a n = 1 4 0 ° ; = 0.9024,

3.28. (a) How do the winding a- * L ' *..5 = 0.037786 ; 17 and 19, 0.9024)

(6 ) The line to line voltage o f a 3 phase t " WaVrf° ™ 0fth' ° UtPUt ’' 0" ae' " f ‘ synchronous generetor?
of Held Hux, is observed to have no thifd or n f t t i.u T lT c o t 'T n ^ haVi“ 6 Sp‘ “ distrib" ' i» "
. W hat are the most Hkeiy reasons for the absence o f these harmonics? Why ?

in voltage w a v e ? UC “ “ th‘ fundan" " t a l component o f voltage be increased by allowing full fifth harmonic
, J ’ Star connection eliminates third harmonics ;
3.29 The total fi , • ng a n g leof36° eliminat«9 fifth harmonics, (n) 5.1526%)
distribution is given b y * *** P° 8 “ * ^ ^ ° f 3 50 Hz a c’ generator is 0.069 Wb. If the flux density

B = B l s i n 0 + B 3 sin 30 + B s sin 50

where B3 - B j and f l 5 = - fl, and 0 is the angle measured from interpolar axis, then determine the r.m.s.

value o f the e.m.f. per turn. The coil span is |th o f a pole pitch.

H int, e 180° = 36°


-M )
kPt = cos - = 0.951.
k/>3= cos 3 x 18 = 0.588

*/>5 = °'
From the presentation leading to Eq. (3.51 b ),

03 = g 0|.0S = ^3 0|-

.'. Total flux per pole 0 - 01 + 0J + 05 = 01


, + 9 + 2fil= U 5 1 *
Fundamental flux per pole
01 “ I T S " 0-06 Wb
E.m.f. per turn, = V2 it fkp^$1 = 12.674 V

From Eq. (3.51 b), E3 per turn = 12.674 x x ^ = 2.612 V


U.«7Jl «J
^5 = 0
.*. per turn
Total e.m.f. >ium E = ylE'i + E3 + E§ — 12.94 volts j

3.30. (a ) “Even though the field flux density wave o f an alternator is non-sinusoidal, the alternator voltage
can be m ade sinusoidal by distributing the armature winding in slots". Illustrate your answer with e.m.f.
waveforms.
[Hint: (a) N on-sinusoidal distribution o f field flux-density wave along the air-gap periphery is shown in
Fig. 3.60 (a). I f one full-pitch coil rotates with respect to this flux-density wave, then e.m.f. generated in that
coil has the sam e w aveform as that o f the inducing flux-density w ave-this is illustrated in Fig. 3.60 (6).
Suppose there are three slots per pole per phase, i.e. there are 3 coils in the phase-belt or phase-band.
Then w aveform s o f the e.m .fs. generated in the coils numbered 1, 2 and 3 are as shown in Fig. 3.60 (cl Note
that the zero-crossin g o f the e.m.f. w aveform s is displaced from each other by a time angle o f 20° (= slot angular
Pitch o f 20°).

S c an n ed by C a m S c a n n e r
IProb. 3
354 Electrical Machinery

(c)
Fig 3 60. Effect of distributing the winding. Problem 3.30.

The e m f waveform for each coil fa o f the same


phase belt, the coils are connected in senes, th e r e s d ta n t a si„e wave. Thu
of the three »>veform m F.g. 3.60 fc) d lu stra ^ ^ a|temator is „ on .sm u Soidal, the alternator voltage can be
shows that even though the flux aens y alone the air-gap periphery.
made sinusoidal b , distributing the arm ature w m d.ng m the slots along the g P P .
I f the winding consists o f fractional-pitch coils, the resultant e.m .f. w aveform can be mad PP
better sine wave.] x ^ .
(6) The field-flux distribution o f a 3-phase star-connected alternator is give y
100 sin 0 + 30 sin 30 + 20 sin 50.
where 0 is the angle measured from the interpolar axis. fundamental
The alternator has 9 slots per pole and the coil-span is from 1 to 9. If r.m .s. value o
phase voltage is V j, then compute :
(i) r.m.s. value o f the phase voltage and
(ii) r.m.s. value o f the line voltage.
3 03
|A n s .(o ) 1.1507 V, 161 l'7SlV|1
7 f eu'
Kul J inding E*ch
3.31. A 50-Hz, 4-pole synchronous generator with 24 stator slots is w ound with two layer * ^uce(j e.ni f
1-pitch coil has 10 tum s. The sinusoidally distributed flux per pole is 0.02 W b. Calculate e
tween the lines i f the coils are connected to form ^ 594,693V1
(a) a two-phase winding ( b) a three-phase winding. lAns. (a) 686.65 ^j^ing

3.32. A 50-H z salient-pole synchronous generator, w ith tw o-layer full-pitched winding, l'as
ta :
D = 7 n i ,f = l m , stator slots = 600, rated speed = 120 r.p.m.
iductors per slot = 2.
The sinusoidal flux density has a peak value o f 1.2 T

S c an n ed by C a m S c a n n e r
- ° b;— Basic Concepts of Rotating Electrical Machines 355

a re co n n ected to f o r m T ^ l ^ p h ^ s e w lnd T ng V° lta g e th a t C3n be o b ta in e d from th is g e n e r a to r i f th e co n d u c to rs

(W F in d th e p h a s e v o lta g e i f th e c o n d u c to rs a r e co n n ected to g iv e , b a la n c e d th r e e -p h a s e w in d in g .

c o n n ected m is e r ie s . T h e re fo fe .^’ p
p h a se
se SsPread
p re a d -*180®^etc
~180 etc.| ? tPU* V° lt a g e ’ Si" g !(Ans.
! ' p h a s(a)
e w in d in g V
28506.24 a s al1
h (6) th e tu rnV|
14294.47 s

3.33. A 4-pole oc machine has its winding housed in 48 slots with coils o f span i| ,* o f a pole-pitch.
Determine the winding factors and their relative kVA ratings in case this winding is
(a) 3-phase with a phase spread o f 60°,
( b) 3-phase with a phase spread of 120°,
(c) two phase,
(d) single phase. ,A „
IAns. W in din g factors : 0.9 4 9 5 , 0 .8 2 2 2 , 0.8 95, 0.6 33
kVA : 100, 86.593, 94.24, 66.271
3.34. The flux density distribution for a 50-Hz cylindrical-rotor alternator is
B = sin 0 + 0.3 sin 3 0 + 0.2 sin 5 0 + 0.1 sin 7 0
^ ‘I ™ ’ Alternator has pole-pitch = 40 cm and core length = 35 cm. Stator coil-
span is four-fifth o f pole-pitch. Find equation for the e.m.f. generated in one-turn coil and its r.m.s. value.
IAns. e - 26.61 sin 0 + 4.935 sin 3 0 - 1.645 sin 7 0 ; 19.1762 V|
hoc t 3? ' * * a,ternator with sin8Ie lay er winding and full pitch coils, has 12 slots per pole. Each coil
has 30 turns. The flux is sinusoidally distributed in space and its value per pole is 0.03 Wb. The machine is
af SPf , 7b0 rr p m ’ P 16 current Per conductor is 50 A. For negligible internal voltage drops, find
the kVA output available from the stator if the winding is connected as :
(а) single phase and
(б ) three phase.

IH i n t : In single layer winding, number o f coils = — (number o f slots).

.-. Total number o f turns = ^ ~ - ^ j ( 3 0 ) = 1440 turns etc.] [Ans. (a) 305.424 kVA (6) 9.465 kVA]

3.36. A 3-phase, 50 Hz, 10 pole alternator with 90 slots has a star-connected winding to give a generated
e .m .f o f 11 kV at no load. The coils are chorded by one slot. If the flux per pole is 0.110 Wb, compute the
num ber o f series turns required for each phase o f this alternator. -
[ H i n t : Conductors per slot must be a whole number.) |Ans 2701
3.37. A 3-phase, 3 kV, 50 Hz, 300 rpm alternator has the following data:
Armature diameter = 1.90 m, core length = 0.39 m
Peak air-gap flux density ’ = 0.95 T
Number o f stator slots = 180
Air-gap length = 1 cm

T h is a lt e r n a t o r h a s s in g le - la y e r w in d in g a n d tw o c ir c u its p e r p h a s e . F in d th e to t a l a r m a t u r e t u r n s p e r
p h a s e . A s s u m e s in u s o id a l flu x d is t r ib u t io n . T h e w in d in g is s h o r t p itc h e d b y o n e s lo t. C a lc u la t e a ls o th e p e a k
v a lu e o f f u n d a m e n t a l fie ld m m f.
In c a s e c u r r e n t p e r c o n d u t o r is 10 0 A , e s t im a t e k V A r a t i n g o f th e a lt e r n a to r .
IAns. 240, 7559.9 At/pole, 1039.2 kVA|
3.38. T h e y o k e o f a 6 -p o le d .c. m a c h in e h a s it s e x t e r n a l a n d in t e r n a l d ia m e te r s o f 9 0 a n d 7 6 c m s r e s p e c t iv e ly
an d a x ia l le n g t h o f 3 0 c m s . T h e a r m a t u r e is w a v e c o n n e c te d w it h 7 2 s lo ts a n d 6 c o n d u c to r s p e r s lo t. T h e flu x
d e n s ity in t h e y o k e is 0 .8 6 T . C a l c u l a t e th e g e n e r a t e d e .m .f. a t th e m a c h in e t e r m i r a l s fo r a n a r m a t u r e s p e e d
o f 900 r .p .m .
[Hint: F l u x p e r p o le = 2 ( y o k e flu x ) , s e e F ig . 3.8] ( A n s . 7 0 2 .1 7 3 V|

3 .3 9 . A 3 - p h a s e d e lt a - c o n n e c t e d in d u c t io n m o to r h a s 6 p o le s , 3 6 s lo t s , 2 - la y e r w i n d in g a n d 1 2 - t u m c o ils
s h o r t- p itc h e d b y o n e s lo t . F i n d t h e flu x p e r p o le o f t h e r o t a t in g flu x d e n s it y w a v e , w i t h s t a t o r c o n n e c t e d to 4 0 0
V, 50 H z , 3 - p h a s e s u p p ly . T h e m a g n it u d e o f v o l t a g e d r o p in s t a t o r le a k a g e im p e d a n c e is a s s u m e d to b e 1 % o f
s u p p ly v o lt a g e . lA n s . 0 .0 13 2 7 W b|

aririeu u y u c m i o c a r i n e i
356 Electrical M achinery________________________________ '
— ’ i. avera ge e.m .f. g e n e r a t e d i n o n e c o n d u c to r is 2 / > . H en ce prove t W
3.40. (a) Using " £ ven b y J , f * ■« , * • P* "* “
per-phase e.m.f. genera e star.ctirme cted alternator has 144 slots with 6 conductors per slot, P|ux
(6) A 3-phase 8-pole, .^ O r ^ m -s t a ^ ^ phase and line e.m.fs.
per pole is 0.06 Wb and coi s fs if the coils are reconnected to form a balanced 2-phase winding 7
What will be the phase and 1 • ^ lg34 g ^ 3 m g y . 2595 Q2 y ^

3.41. (a) Show that the m.m.f. changes linearly over the slot width and remains unaltered over the tooth

Wldth‘ . in s is t s of one full-pitch coil on the stator. If the coil has N turns, then for
(6) A uniform air-gap ma^ e h t the m m f . along the air-gap periphery is of rectangular waveform
a current of i amperes in each turn, show tnai m e m e>

with its magnitude equal to -Ni.

Hence introduce the concept of current sheet.


A single-phase distributed winding carries alternating current. Show that the tune-maximum of the pesk
m.m.f. for a P-pole machine is given by

' *. ATs per pole-


Ib) What would be the magnitude o f the fundamental component of m .m .f produced by Uie held[winding
of a S f b r m air-gap 2-poU machine wound with 13 concentric c o „ . and wrih a slot angle
13 turns and the conductor current is 200 A.
3 43 In a balanced 3-phase stator winding, each phase-belt o f phase a has 100 ampere-conductors at the
instant of its carrying maximum current. Sketch the m.m.f. wave for this phase in case slo s per pole per phase
(q) are 1, 2, 3 and 4. Find the amplitude of m.m.f. wave for this phase and comment upon the change in m.m.f.
as q is increased.

[ H i n t : For q = 1, m.m.f. amplitude = ^ (Ni) = 50 etc.)

[Ans. 50, 100, 150 and 200. M.m.f. wave becomes more closer to sinusoidal m.m.f. as q is increased).
3.44. A 2-pole, 3-phase a.c. winding has 24 slots. Each slot carries 10 conductors. At the instant phase a
carries maximum current of 16 A, do the following :
(a) Sketch m.m.f. wave o f phase a only in a developed view of the slots. Comment upon the m.m.f. w a v e s h a p e
and indicate its amplitude in the sketch.
(b ) Calculate the amplitude of the fundamental component o f the m.m.f. wave.
(H in t. Refer to Fig. 3.29 (c)] (Ans. (a) Stepped m.m.f. wave ; 320 ATs (b) 390.2 ATs/pokl
3.45. A 3-phase, 100 kW, 6-pole, 50 Hz, delta-connected induction motor has 20 conductors per slot arr“ ^
in 72 slots. For a line current o f 76 A, compute the peak value o f the fundamental m.m.f. wave of any p ^
Also calculate the peak value o f the resultant m.m.f. wave and its speed. The winding is short‘ pltc e i
slot. [Ans. 3000.8 AT/pole ; 4501.2 AT/pole. 1000 r.p.m-

3.46. (a) Show that the pulsating m.m.f. can be expressed in terms o f space angle a and time angle w
F (a, t) = Fm cos cot cos a
where Fm is the m.m.f. corresponding to the maximum current
• nf triangulw
(6) Show that the m.m.f. produced by the armature winding o f a commutator machine is o
1Z i
w a v e fo r m w i t h its p e a k v a lu e e q u a l t o - - - A T s / p o le .

3.47. T w o -
s u p p ly . I f t h e c u r r e n t s
p r o v e t h a t a s y n c h r o n o u s ly
th e s e r ie s t u r n s p e r p h a s e .

Scanned by Cam Scanner


Prob. 31 m
Basic Concepts o f Rotating Electrical Machines 357
Hence show ,h . « » - p h a « in m.phase synchronous|y _ mf
w a v e o f c o n s t a n t a , n p „ t u d e o f f U m e5 th „ m , x im u m ^ ^ » _ 6 f

[ H in t : Proceed as outlined in Art. 3.7]


3 48 For 3 no I* * •
and rotor m.m.fs., statm g^hetlriom t0rqULe exPression in terms of the interacting stator
only when one of the two m.m.fs has a If enceahow that the electromagnetic torque can exist
3 4 9 For m 15 nas a component normal to the other m.m.f.

per pole <>, number o f poles /^andTotor'nf’n frtlr0 ^ £enera* tonlue expression in terms of total air-gap flux
the load angle or t o r ^ e a"gk “ m S S £ 5 w S i I T p U' “ « • * » « n develop J / w h * .

air gap of a rotating electrical machin^hawf* density vary sinusoida


the flux per pole is 20 m Wb the amDlituHe^f »h° ^ * rated sPeed of 960 r.p.m. At rated load conditions,
displacement between S e two w a S r ^ is W,Ve " 53° 3mpere tUmS and the space
necessary relations ‘ ®'s^imate *he power rating of the machine deriving the
„ , U.E.S., 1979) |An s. 4 kW|
A 4‘ P°le cylindrical machine has the following data :
r o to r d ia m e t e r = 0 .4 m, / = 1 m, sta to r bore d ia m e te r = 0.41 m
p e a k v a lu e o f sin u s o id a l g a p flu x d e n s ity = 1 T

Calculate the air-gap reluctance, peak air-gap m .m .f, flux per pole and total air-gap energy.
IAns. 12665.15 ATA Vb, 1989.44 A T s, 0.2 W b 1250 J1

decomposed by ZrP0duCeS P » "> f wav. which cm, be


tw oT h a.t l h f l T ^ amplitude counter-rotating m.m.f. waves. Using this concept, prove that when
X u f u l e fs p ro d u lT *“ y tW0'PhaSe ba'anCed * travelli" ‘
halnnr5^' 7711-66 C01,s a - b’ c bave their magnetic axis coincident. These coils a, b, c are excited by three-phase
alanced currents /„, cos cot, /„, cos (o* - 1 2 0 °) and /„, cos (a* - 2 4 0 ) respectively. Find the magnitude of resul­
tant m.m.f. wave.

is 11 essential to know the power losses in electrical machines? Show that the calculation of
efficiency by the measurement of losses is more accurate than by measuring the output.
(b) What are no-load rotational losses? How can these be measured?
3.55. (a) Enumerate the various power losses in electrical machines. Which of these losses are affected by
U) rotor rotation (» ) the value of flux (Hi) the load directly and (iv) square of the load.
(b) What happens to the power losses in electrical machines? Are these reversible?
(c) What is the efficiency of a generator or motor at no load?
[Ans. (a) (i) No-load rotational loss (ii) Hysteresis and eddy-current losses
(in') Brush contact loss (iu) I2R losses and stray load loss.
(c) Efficiency is zero.]
3 .5 6 . O u t p u t o f r o t a t in g e le c t r ic a l m a c h in e s is g iv e n b y
S = K D 2L B ou r c n

w h e r e D =s t a t o r b o r e d ia m e t e r , L = c o r e le n g t h , n = r o to r s p e e d in r.p .s ., Bou = a v e r a g e flu x d e n s it y in


T ,a c= a m p e r e c o n d u c to r s p e r u n it le n g t h o f a ir - g a p p e r ip h e r y a n d i f is a c o n s ta n t.
C o m p a r e t h e r a t i n g s a n d lo s s e s o f tw o r o t a t in g e le c t r ic a l m a c h in e s , th e lin e a r d im e n s io n s o f o n e b e in g x
tim e s t h o s e o f t h e o th e r . T h e flu x a n d c u r r e n t d e n s it ie s in b o th a r e th e s a m e .
H e n c e s h o w t h a t larger t h e s iz e o f t h e m a c h in e , g r e a t e r is its e ffic ie n c y .
o 8 •^ _ _
[Hint. F o r f i r s t m a c h in e , act = — — T h e r e fo r e ac2 = x a c j e t c . I
71 L)

[Ana. x4 S lt x3 W, where 5 , and W, are the rating & losses respectively in the first rotating electrical machine.l

3 .5 7 . (a) W h a t a r e t h e v a r i o u s e l e c t r i c a l lo s s e s in r o t a t in g e le c t r ic a l m a c h in e s ? G iv e a p h y s ic a l u n d e r ­
s t a n d in g o f h y s t e r e s i s a n d e d d y - c u r r e n t lo s s e s a n d s h o w t h a t th e u s e o f th in la m in a t io n s m in im is e s th e
e d d y - c u r r e n t lo s s .
(5 ) W h a t lo s s is d e s i g n a t e d a s s t r a y lo a d lo s s ? H o w is t h is c a u s e d in e le c t r ic a l m a c h in e s ?

Scanned by Cam Scanner


[Prob. 3
358 Electrical Machinery

(c) Show that m aximum efficiency occurs when constant loss is equal to variable loss.
3.58. (a) The proposed load schedule for a sm all rolling m ill is as under.
B ille t in th e r o lls , 5 0 0 k W fo r 3 0 sec.
B ille t o u t o f th e r o lls , 8 0 kVV for 4 0 sec.
F in d a c o n t in u o u s k W r a tin g o f th e m o to r.

(fc) The operating cycle for a lift is as under :


1 L o a d g o in g u p (1 m in ) = 60 k W
L o a d in g p e r io d a t top (2 m in ) = 10 k W
L o a d g o in g d o w n (1 m in ) = - 5 0 k W (p o w e r p u m p e d b a c k to s u p p ly )
L o a d in g p e r io d a t b o t to m (2 m in ) = 10 k W
On the basis o f heating select a suitable motor out o f the follow ing available sizes : 25, 30, 35, 40, 50 and
60 kVV [Ans. (a) 350 kW (= 332.866 kW) (6 ) 35 kW ( . 32.94 kW)|

3.59. (a) W hat is m eant by one-hour rating o f an electric motor ? Discuss.


(fc) A motor driving a colliery winding equipm ent has the follow ing points on its load duty cycle :

20 60 60 70 90 R epeat
T im e , sec 0 20
cycle
1000 1 0 0 0 to 5 0 0 500 5 0 0 to - 2 0 0 0 0
O u tp u t, k W 0

The complete load cycle curve can be obtained by joining these points with straight lines.
Specify a continuous kW rating o f the motor for this load cycle. [Ans. 450 kW (= 432.045 k\V)|
3.60. (a ) A continuous-rated motor can deliver higher power outputs for short-time duty. Discuss.
(6) A 10 kW, 3-phase induction motor has a heating time constant of 2 hours and attains a final steady temperature
rise of 50° C with continuous loading. Find the time for which this motor may be safely loaded at 20 kW. The fuU-load
ohmic loss is twice the core loss and ambient temperature is 30°C. [Ans. 0.811 hr]
3.61. (a) Explain the significance o f machine ratings.
(6) Describe more common types of machine ratings based on thermal considerations.
(c) Explain the method of determining the power rating o f electric motors for driving the loads requiring
almost constant torque.
(d ) Describe the method used for the determination of electric-motor rating for a periodically varying load.
3.62. (a) An induction motor has a final steady temperature rise o f 40°C when running at its rated output
of 20 kW. Determine its half-hour rating for the same temperature rise if the ohmic losses at rated output are
1.25 times its constant losses. The heating time constant is 90 minutes.
(fc) For the motor o f part (a), the motor cooling is improved by 20% because o f its installation in the hills.
Find its new continuous power rating (i) for the same temperature rise (ii) for a temperature rise of 50°C.
[Ans. (a) 47.17 kW (b) 23.324 kW, 27.57 k\V|
3.63. ( a ) What are the more commonly employed classes of insulation in electrical machines ? Discuss these
with respect to the maximum temperature, materials etc.
(fc) An electric motor with continuous rating Pr can deliver much higher output Px for a short-time duration
tx. Develop a relation that gives the short-time rating Px in terms o f P r, tx, heating time constant etc.
3.64. (a) Differentiate between short-time rated and continuous rated motors.
(fc) Temperature rise of an electric motor at rated load of 30 kW was found to be 20°C after one-hour run
and 50°C after three-hour run. In case its full-load ohmic loss is 1.5 times the constant loss, calculate its
one-hour rating without exceeding the specified temperature rise.
(c) In case temperature rise o f 10% is allowed for motor in part (fc), find its new continuous rating.
[Ans. (o) 88.74 kW (fc) 32.4 kWI
3.65. (a) The temperature rise of an 80 kW motor is 25°C after 0.5 h and 40°C after 1 0 h on full load. The
motor I B loss on full load is twice the core loss. When this motor is fitted with a fan to improve the cooling.

b M h ^ i b t i i ’ned1w ^ Z i t “ m ” 15 t0 48°C ' EStimatC the m ° t0r raU ne f° r thc fin al te ™Peraturc riSC 10
Derive the formula used for obtaining this rating.

Scanned by Cam Scanner


Prob. 3]
J t a s i c C o n c e p ts o f M a t i n g E le ctric a l M a c h in e s 359
<6. For the m otor o f par, fa >, determine j(s ^ (j) ^ ^ ^ ^ ^ ^

3.66. (a) What is pulsation loss? Discuss how this , ■ 'T 96 44 “ '° 8 “ ^ ' 3° '95 kWI
(6 ) Describe the advantages o f usinp h H ' S cau ln rotatlng electrical machines.
(c) Discuss where direct c o o , i n i T ” “ * C° ° 'ant ‘“^ " '- t o r s .
W1 npc ., eCt C00hne 15 P a i r e d over hydrogen cooling.

electric motors. n ° US tyPCS ° f sPeed-torque characteristics as possessed by driven equipments and


(e) Answer the following :

<0 M ost outstanding feature o f a generator and a motor.


<u Econom ic features o f rotating electrical machines.
(Hi) Factors leading to the progress in electrical machinery.
lAns. (e) (i) V / and speed-torque characteristics. Ui) Efficiency, p.f„ initial cost etc.
(«») Improvements in the quality and characteristics of steel, conductor and
insulating materials and also in the cooling of machines.l

\
S c an n ed by C a m S c a n n e r
FO U R
D.C. Machines

D.C. machine is a highly versatile energy conversion device. It can meet theRemand.of
loads requiring high starting torques, high accelerating and dederating torques. e
time, d.c. machine is easily adaptable for drives requiring wide-range speed control a n d quiCK
reversals. These inherent characteristics can further be modified, if desired, by fee“ ba
cuits. In view of these outstanding features, d.c. machine possesses a high degree ot ttexibility.
These are therefore widely used in industry, particularly for tough jobs as are encountered in
steel-mill drives— inspite of their higher initial cost.
DC machines discussed in this chapter have hetropolar field system (alternate ^ a n d S
poles) and armature-commutator system. In normal dc machines, stator core is not lamina e
armature core is, however, always laminated to reduce e d d y - c u r r e n ^
machines used in control systems have their magnetic circuit completely laminated. This is
done to minimise the effect of eddy-current damping on the fast response required in d
machines employed in controlled systems. •
At present the annual production of dc machines is about 40% of the rupee volume in
electrical-machine production and sales. This is on account of the fact that most highway
vehicles use batteries for the storage of electric energy. In these vehicles and automobiles dc
motors are used as starter motors, windshield-wiper motors, fan motors and for driving other
accessories in the vehicles. For these purposes, almost millions of dc motors are built each year.
In industrial applications requiring accurate control of speed and/or torque dc motor is un­
rivalled Therefore, dc motors are almost universally employed in steel and aluminium rolling
mills. power shovels, electric elevators, railroad locomotives and large earth-movmg equip­

ment. . , . , , „
The constructional features of d.c. machines have already been described in chapter 3
where it has been stated that field winding is a concentrated winding on salient poles bolted to
the stator frame and armature winding is a distributed wmdinghoused in the slots around the
periphery of the cylindrical rotor. Basic principles underlying the torque production and e.m t
Generation in d.c machines are also outlined in chapter 3. The object of this chapter is to
present the physical concepts regarding the steady state behaviour of d.c. machines.
E x am p le 4.1. (a) Sketch a 6-pole dc machine. Indicate the flux path for the six poles.
(b) Draw neat diagram o f a dc machine. Label all its parts and mention the material used
for each part.
S o lu tio n . The object of this example is to supplement the constructional details alrcaay
described in Art 3.2.3. The reader must go through this article before studying the iol -
presentation.

S c an n ed by C a m S c a n n e r
AH.
D.C. Machines 361
10) Sketch of a 6-pole dc machine is
Yoke
shown in Fig 4.1. In this figure, iron from
the bottom of armature teeth to the shaft
diameter is the armature core. The flux
paths for the six poles are also shown. It is
observed from this figure that
(£) each pole carries a flux 0 (say),
(ii) yoke handles half of the pole flux
i.e. cj)/2 ’
(in) armature core also handles a flux 0 /2 .
Examination of Fig. 4.1 reveals that main
flux 0 starts from a north pole, crosses the air Field
gap and then travels down to the armature w in d in g

core. There, it divides into two equal (0/2) hal­


ves, each half enters the nearby south pole,
each half then passes through the yoke and
reaches the starting point of north pole so as
to complete the flux path. Each flux line cros- .
ses the air gap twice. Some flux lines may F'e 4 1' Flux paths in a 6~pole dc machine
not enter the armature ; this flux, called the leakage flux, is not shown in Fig. 4.1.
(6) Various parts of a 6-pole dc machine are shown in Fig. 4.1. Commutator forms the most
important component of a dc machine. In Fig. 4.2 (a), various parts of a 4-pole dc machine alone
with its commutator are labelled.
F ie l d p o le • In te r p o le Yoke
Field
w in d in g

Fig. 4.2. (a) Constructional features of 4-pole dc machine and commutator.

Stator of a dc machine consists of yoke (or frame), field windings, interpoles, compensating
winding, brushes and end covers. Rotor consists of armature core, armature winding, com­
mutator and shaft. Stator components are described first.
Yoke. It has two functions : (i) it provides path for the pole flux 0 and carries half of it
| 0 /2 (it) it provides mechanical support to the whole machine. Since the flux carried by yoke
is stationary (i.e. constant), it is not laminated. As stated before, case iron is used for small dc
machines and fabricated steel for large dc machines. In case dc motor is to be operated through
a power-electronics converter, the yoke is laminated to reduce the eddy-current losses

S c an n ed by C a m S c a n n e r
r

[Art.
362 Electrical M a c h in e r y _ _
/■ i „ oaIp sihnp The pole core is made from cast
Field poles. Fieldpole consists of pole core a P ^ • riatcly.
steel but the pole shoe is laminate an ixe Thus both pole core and pole

s h o e ^ L ' m ^ f ^ r t w f u m ^ r o n s o f T e e t steel to reduce the eddy-current losses. The

,;» ) For dc compound machine, both shunt (thm wire) and series (thick

arc used. . ,, +Vl0 noles of a dc machine. These


In terpoles. These arefixed to the yoke in between the m P tic saturation. The
are usually tapered with sufficient- ^ ^ 1 area at J t a r - t j ^ c U d in series with the arma-
interpole winding, consisting o a e |tQ arraature current.
ture so that its magnetomotive force is propo ^ ^ alots cut in the pole faces of
C om p en satin g windings.Thesewinding P ^ ^ armature circuit This
a dc machine. Compensating winding is also conn
winding is, however, used in large c mac me ^ ^ tQ the gtator eiid cover
B ru sh es. Brushes are housed in box-type_br rushes pressed on to the commutator
Fie 4 2 (a) or the stator yoke. A small spring; eep carefully adjusted. Too sma

hrush pressure is too high, it . • y


brUSheS' „ ade o f carbon for small dc machines, e .e c t r o g r a p h it e fo r a il dc machines an
BrU anhte f o r t w voHage high-current dc machines.
C0PR „tfr components arc^nowi described below. rf tf) to

A rm a tu re w in d in g. havjng 0ne or more turns. The depending upon the

type of w in ^ n g re q u ire d of ^ ^ are , con- /g U w jj- J '-


S e d t o t h e r is e r o f s e g m e n t s o f a c o m m u t a t o r , F .g - R ls e ^ j

4 2 (a )‘ * * ^ It is ofcylindricalstructure.lt is
C o m m u ta to r . I g e n ts 0f h ig h c o n d u c -

built up of wedSe' sh^ e to reduce its wear and v Groove


tivity hard-drawn copp ^^ ^ other by 0 .8
V -G ro o ve—
tear. Segments are in ents are tapered as
mm thick mica sheetsi The s g m ^ assembly results _____
sh ow n in F ig . 4 .2 W ^ a r e in s u la te d t .t o r s e e ™ '” 1
in c ir c u la r s h a p e . H u b H sh eet M and V - p ig 4 2 . (M O n e c o m m a

fr o m co m m u ta to r se ^ ‘ Ss (i m e n ts fr o m fly in g o u t
shaped so as to prevent the segm

S c an n ed by C a m S c a n n e r
Art. 4.1]

D.C. Machines 363


d u e to c e n t r ifu g a l fo r c e s , F ig 4 2 T o T f T v ! --------------
conductors from the armahfre w i ^ ^
o. « "u ium g are connected ' ° '" " as a riser where
S h aft. On armature sw * _____
sh^ft -»eccea.
ieQ‘
or armature core in small J mounted ^ hub# of commutator (ii) «n,vi • u-

A commutator cerfnrmo +, •
vert alternating quantities to in 3 d c ’ m achine-the first one is to con-
S e Ilte°r 0I; arraatUre m m -f stationary in Z Z ^ the S6C0nd function is to keep
the alternating quantities in the armature a ? commutator function of rectifying
of a d c generator, has already been eXZ n o d ? ‘ ° f 6Ct “ uantiti“ at the outpu brufhef
quanta,es at the brushes are con verted t al t - ^ 3A A ' CaSe ° fa d c m ote, the dfrec
armature winding. Thus a c o m m u t a t o r c a n a qUantitieS <V° ltage and “ ™ « m the
as an inverter (d.c. to ax.). both as a c°nvertor or rectifier (a.c. to d.c.) and

For duf development of e i^ tro m a ^ e h c torque ^he°^eS' 7 “ P" du“ Stati° ^ air gap flux,
ary in space. The commutator in a d c machine r* rmature wave must also be station-
rendering the armature m.m.f. wave stationary in sn T ° U f S(? 0nd imP°rtant Unction of

r ‘7 ° f 'theC0“ ° r is a™ atUrB r° tati° n In


commutator segments^umbered^lto 4 Ire totu[a^ ” t ' tched C,° U,S, housed in 4 slots. The four
centre line of the poles around which the field mil t i7 6 ° and from the sbaft- The
the machine. The shape of the c oi l nd conn ctton makes h W° Und' * “ 1,ed the “» ° f
brushes along the direct axis or field axis, as deputed t F i g T T * " * l° Pla“ ^ tW° Carb° n

Fig. 4.3 differs much from a real machine hn, E . ~ . Therefore the d.c. machine illustrated in
It may also be noted that the brushes make contactwiththe m f * ? mng,the commutator action.

c oiU id ig 4 3 ;,theT are shown inside the


cheating currents a w a y t V t o ^ - d d° * in them, in-

tion 1, lower coil-side in slot 3, commutator segment 2 nnner m i a end C0nnec*


coil-side in slot 4, commutator segment 3 and back to the tGGminal ^ T h e ^ 01 d’ 0011 n I™ ™
is from commutator segment 1, lower coil-side in slot 2 back coil end mn c P Path
m slot 4, commutator segment 4, lower c o i ] ^ £
The^current entering at the right hand carbon brush divided
paths described above. The effect of current in the armature coils is t ^ t Parallel
m.m.f. that is vertically upward along an axis 90°
displaced by an angle of 90° from the rf-axis, is called the quadrature axis At the rotor surf ‘7 *
armature m.m.f. creates N-pole at the top and S-pole at the bottom of the rotor The S Gole on
the stator, attracts rotor N-pole and repels rotor S-pole. Similarly N-pole on the stator nHmrf

^r°ernoT°le andlirT el/S f ° r N‘ POle‘ ThG t0rqUC Pr° dUCed by the interaction of field and arml-
ure poles 1S caUed electromagnetic torque. If this machine is working as a motor the rStar
In t " I m u GdirCCti° n, ° f electroma^ etic torque, i.e. in the clockwise direction In Fig 4 3
m case the machine is working as a generator, then the rotor must be driven bv the nri™7
E d i r e c r nSt fh° fdiGeCtif0n ° f elect™ma^ etic torque for proper energy conversion. In Fiig. 4 3
direction of rotation for generator operation must be anticlockwise.
I

cann ed by C a m S c a n n e r
[Art. 4.1
364 Electrical Machinery

r '

k ,
2
------N — f Ii

Fig. 4.3. Armature m.m.f. is kept stationary in space along


interpolar axis by action o f the commutator.

Fig 4 3 (6) illustrates the conditions when the rotor has turned through 45° clockwise.
There are again two parallel paths, one through coil 1 and the second through coil 3. It may be
seen that coils 1 3 (housed in slots 1, 3) have maximum e.m.fs. generated in them ; because
these coils are cutting the maximum flux. Coils 2 ,4 are short-circuitedby the brushes and carry
no currents because e.m.fs induced in these coils are zero. Currents in coil-sides pertaining to
slots 1 3 in Fig 4.3 (6), again establish armature m.m.f. that is directed vertically upward. Note
that coil current in each parallel path is again 7a/ 2 as it is in Fig. 4.3 (a). •
Fie 4 3 (c) depicts the condition when the rotor has turned through by further 45° from the
position of Fig. 4.3 (6). One parallel path is now through coils 4 ,1 and the second parallel path
is through coils 3, 2. Coils in each parallel path carry again Ia/2 as it is in Fig. 4.3 (a) and (6).
The armature m.m.f. is again directed vertically upward. Hence it can be concluded that arma­
ture m m f is always stationary in space and is directed along the interpolar or quadrature
axis even though the armature rotates. For a motor, the electromagnetic torque is always ac­
ting in the clockwise direction, (Fig. 4.3) and the rotation continues. For a generator, the arma­
ture must be driven in anticlockwise direction (Fig. 4.3), for the necessary energy conversion
from mechanical to electrical. ,
An examination of Fig. 4.3 reveals that just before the coils 2, 4 are short-circuited by e
brushes, these coils carry current Ia/2 (or in general l a/a ) in one direction, oon
short-circuit is over, these two coils carry current - Ia/2 (or in general - / a/n ) jn r ^
direction as shown in Fig. 4.3 (c). Here a is the number of parallel paths and in Fig. - . ^
equal to 2. Thus during the time of short-circuit, the currents in the short-circune .

S c an n ed by C a m S c a n n e r
Art. 4.2]

D.C. Machines 365


general, must be reduced from I /n tr.
- / „ / * . This process of «P to C om m utation
p e rio d
cuited by the brushes is called cnm / C° short' clr-
during which it takes place is called r atlon and the time
current variation with time is uniform flu
depicted in Fig. 4.4, a linear commutation is obtained "

is t h ^ C e m a ^ i ^ o f ™ t e t m X c o U ^ f"8
remains constant and equa. to ,
mtude of e.m.fs. induced in the coils under each nnle i . IT * .,or '«"tiMng
stant- coil under the pole centre has m a x im fm 'e t f l T d , P°'e ^
in rt, whereas the other coils have their e m f m a r m it i 4'4' Vanatl0n o f armature-coil
proportional to the flux density wave present there CUITent Hnear commutati°n
The coils short-circuited bv the hm eU o ■
sides in the zero flux density region so that e m f T f n r h m l T process must have their coi|-
coils undergoing commutation must have t h e i r c f l r i ^ “ ^ In ° ther WOrds' the
field poles, i.e. in the interpolar or ?t * 5 *h° maB™tlc neutral axis of the
commutation in Fig. 4 3 (6) have their coil sides' • °F examPle’ co*'s 2 ar)d 4 undergoing
The shape of the coil ° Tint6rpdar axis
or direct axis. For convenience in the schem atl 4 brush-ax,s,,s aliSned along the field pole
shown along the quadrature axis ^ r in tt!e ^ “ f " d'C machino' the brushas ara
brushes and arp nnHn • ’ ' init^Le position of the coils which are short-circuited by the
L " w „ n ? ea e, T f ^ 0,n8 C
,0mmULtatl0n- thiS schematk diaSram f°r o 2-pole d.c. machine Is
he hr, Sh sv' <o)uHence 11 may be a‘ atod that the armature m.m.f. is always directed a W
The circuit reDresent T Wi*h i n t e r p o l a r quadrature axis of the d.c. machine
poles is as illustrated to R e 4 s T h l T T r 0mp'°?'ed ,for a df machine vvith any number of
two small rectangles nr IL
. figure, circular symbol repres
g es or squares at the opposite sides of circle represent the two brushes.
q-axis
q-axis

B ru sh e s
Field
w in d in g
d - o x is
— d-axis

A rm a tu re

Fi 4 . (?) (/»
g. *i.a. s c h e m a tic d ia g ra m o f (a ) 2 -p ole (i.e. m a ch in e and (6) d.c. m ach in e w ith an y n u m ber o f poles

4-2. E.m .f. G enerated in the A rm atu re


The wave-form of the rectified voltage induced in a single armature coil of a d.c. machine is
epicted in Fig. 3.17 (/). Upper part of Fig. 4.6 (a) shows two armature coils 1 and 2. E.m.f.
generated in coil 1 is zero and maximum in coil 2 at the instant t = 0 shown in Fig. 4.6 (o). As
e rotor rotates clockwise, waveforms of generated e.m .fs in coils 1 and 2, after their rectifica-
>on, are drawn in Fig. 4.6 (or). When these two coils are connected in series in between the two
rushes as shown in Fig. 4.3, the resultant rectified voltage across the brushes is obtained bv
adding the ordinates of the individual rectified voltage waves, as illustrated in Fig. 4.6 (a).

Scanned by C a m S c a n n e r
[Art. 4.2
366 Electrical Machinery

RESULTANT
RESULTAN T BRUSH VOLTAGE
in BRUSH VOLTAGE in

o
>
"X' >'
' A / \ a M ( \ A A / v V v ^ . ^
X vy -fc#1
COIL COIL I mi
COIL rrm
COIL mu
COIL t
1 2 1 2 3
(a) (b)
Fig. 4.6. Resultant brush voltage for a d.c. machine for (a) 2-coils in series and (6) 3-coils in series.

Upper part of Fig. 4.6 (b) shows three coils 1, 2, 3 displaced from each other by 120 .
Waveforms of the rectified voltage in these three coils are shown in Fig. 4.6 (6). When these coils
are connected in series between the two brushes, the resultant rectified brush voltage as shown
in Fig. 4.6 (6) is obtained by adding the ordinates of the three rectified voltage waves. It is seen
that with three coils in series (i) the resultant brush voltage has increased (ii) the brush fre­
quency has increased and (Hi) ripple in the rectified voltage has reduced considerably as com­
pared to their corresponding values with two coils in series between the brushes. As the number
of series coils between the brushes is increased, (i) the magnitude of dc voltage increases (ii)
brush frequency becomes quite high so that (iii) ripple content in output voltage gets drastically
reduced. Actually, there are several coils in series in between the two brushes resulting in an
improved waveform for the rectified voltage at the brushes. Strictly speaking, the brush voltage
waveform can’t reach a pure direct voltage obtainable from a storage battery.
An expression for the brush voltage has already been developed in Art. 3.4.4. The same
expression can be arrived at, by a little different approach. The average e.m.f. Ec generated in
one coil of N c turns, as given by Eq. (3.37), is
Ec = 2Nc Pnty
If C is the total number of coils on the armature, then for ‘a’ parallel paths, the series coils
between the brushes are C/a. •
Therefore the brush voltage or the total e.m.f. generated in armature is,

Ea = — -E e = — (2 N c Pn 0)
° a a
If Z is the total number of armature conductors, then total armature turns arc

f - C N c.

2 CNc n
E = -P n ty
a
_ Z Pn §
a
Another approach that gives better physical concepts about the generation oi dii oct \i P
is presented here.

S c an n ed by C a m S c a n n e r
A rt. 4.2]
D.C. Machines 367

The r.m.s. value of the generated e.m.f. in a distributed winding is given by V2 n kwf r N <{>,
where kw>fn N and (J) are respectively the winding factor, rotational frequency, number of dis­
tributed winding turns and total flux per pole, Eq. (3.22). From this expression, an expression
or e o a e.m. . generated in the armature of a d.c. machine can be derived. For this purpose,
assume the d.c. machine armature winding to be tapped at two diametral points A, B from the
o er en o e commutator as shown in Fig. 4.7. When the armature winding rotates, the
commu a or rus ics pick up direct or average voltage, whereas the diametral points A, B pick
up alternating voltage through two slip rings (not shown in the figure). When the tapping
u^Wnoo’i 11V 10 P?s^ 0n illustrated in Fig. 4.7 (a), the alternating e.m.f. at A , B is zero,
if rUSf V° a^ e 1S n0t zero' Wlien -A» B are in the position shown in Fig. 4.7 (6 ), maxi-
I tw 3 X° t3ge ’S P^c^e(l UP ^ Upping points A, B. An examination of Fig. 4.7 (b )
at commutator brushes are also collecting this maximum voltage. Since the brushes
nirknH , ! ° ^ rU vo^ a&e *n d.c. machines, is always equal to the maximum voltage
Up by the tapPin& P^uts A, B in Fig. 4.7 (b ). When the tapping points are in some other

th^niaximum voltage. From this, H c a n L ^

Ea = V2 [r.m.s. value of the alternating generated e.m f J


= ^ U2n kw fr N <$>\
The e.m.fs. induced in the various armaturp u- -( 4 .1 )
ture. Therefore the phasor summation of the e.m.fs indurpd i m6 ^ a^prnating in na-
two brushes, gives the brush voltage. Actually there are aTarve Van° ^ S C0!.ls between the
machine and in view of this, the phasor diagram for the e m f in f nU™ber of C0lls in a d.c.
as shown in Fig. 4.7 (C. Therefore, the distribution factor S S . 4^

k ,= Phasor sum of coil e.m.fs.


Arithmetic sum of coil e.m.fs.
Chord 2r __ 2
Arc nr n ...(4.2)
An alternative method of obtaining the exDression fn r h nr a , .

S- - — ,s equal to the nJ mum Z £ Z £ £ £ £ ^

o u d in ie u uy o a iiio u a iiiit:!
[Art. 4.2
368 Electrical Machinery

space displaced from each other, d.c.


with phase spread o equal to ir radians. Thus, from Eq. (3.48), the distrmur a
sin c/ 2 _ sin n/2 _ 2 ...(4.2 a)
kd = a /2 = ti/2 " n
In dc. machines, the chording angle is usually very small, therefore ‘ he effect
on the brush voltage can be neglected. Thus for d.c. machmes k - K and in v.ew o ,
e.m.f. generated in the armature or brush voltage, from Eq. (4.1), is

< 2 n ~ fr Nt>
71
...(4.3)
= 4 fr N<\>
zZ
Now rotational frequency f ,^and series turns between the
2a'two br

Here Z is the total number of armature conductors. From Eq. (4.3),

£ =4 ^ 0
Eja 2 2a v
_ 0 ZnP ...(3.41)
a
In case the brushes are symmetrically displaced from the main field or direct axis, then
phase spread o is 0 radians. Therefore, the distribution factor is
s i n 0 /2 _ 2 . 0
d “ 0/ 2 0 2
For total number of Z conductors, Z /a are the conductors in series, Z/an are the conductors
in series per radian. Therefore, series conductors in between the two brushes s e p a r a t e d by 0
radians as shown in Fig. 4.7 (d\ are Z 0/a7i. In view of this, series turns in between the two
„ 1 (ZQ \ r nP - - - - ­
brushes of Fig. 4.7 (d) are IV - ^ Also fr = — . Substitution of the values of kw , fn N in Eq.
an
(4.1) gives
2 . 0 nP 1 Z 6
E „ = V2
^ 5 Sin 2 ' T ' 2 ■W *
0 Z nP . 0 ...(4.4)
= ^ ~ Sm 2
An alternative approach, usually found in some books, is based on Faraday s law, i.e.
Average e.m.f. gen erated!________ Total flux cut by the conductor
in one conductor, Ez J Time, in sec., required to cut that total flux
For a P-pole machine, with <|>as the flux per pole, the total flux cut by one armature conduc­
tor in one revolution is P$ Wb. With n revolutions per second, time required for cutting the total
flux P 0, i.e. the time for one revolution is 1/n sec.
...(3.40)
E‘ = i % = n P *
For total Z armature conductors, there are Z/a conductors in series in between the two
brushes.
Brush voltage or total e.m.f. generated in the armature of a d.c. machine is
...(3.41)

S c an n ed by C a m S c a n n e r
D.C. Machines 369
Art1 413]---------------- ----------------------------------------------------------

The above approach presented after Eq. (4.4) is quite simple, but fails to give any physical
oncepts about the generation of alternating e.m.fs. in the armature coils and its conversion o
direct e.m.f. at the brushes.
Expression for em f generated in the armature of a dc machine can also be obtained asunder.
Average value o f generated em f in one conductor, E av = Bau l v
where B = average value of flux density over one pole-pitch as shown in Fig. 4.8 (6), T
I = active conductor length, m
v = 2 nrn = peripheral velocity of conductor, m/s
u m -c thP flux Der pole, then P(J) is the total flux in the dc machine. Total peripheral air-gap
area is S f “ r - mean air-gap radius. From this, average flux dens.ty is

d L
Dao 2 nrl
From above, the average value of generated emf per conductor is

E = — ^7 •I •2 nrn = Ptyn
av 2 nrl

Now proceed as given above after Eq. (3.40).


K should be noted that in d.c. machines, the

I S i S ™ — r — ^ The aim o f— g Wghgenerated e.m.f., leads to


the field from wWch is usually a flat-topped waveform illustrated m F.g. 3.17 (o).

4.3. Torque in D.C. Machines


) The electromagnetic tirque in case of d.c. machines, has been
interaction of main field flux and armature m.m.f. c nf fipld flux and
is always directed along the ?-axis, the torque angle K f between the axes of field flux and
armature m.m.f. is 90°. Therefore, Eq. (3.85) becomes,
...(4.5)

Now the peak value of the fundamental sine component F_ for a d.c. machine is given by Eq.

(3.66), i.e.
Z_ _ ATs/pole.
F
a n2 2P a
Substitution of the value ofF„ in Eq. 4.5 gives

_ It , _8

8 2P a
PZ
,(4.6)
T - <t)Ia -^ a ^ ^ a
le 2 na v 0
i. . . jr ZP . v p to 4Z) is called the armature constant.
where constant Kn = - — , as given by Eq. (3 4oj, it>
2 na _ ..
Note that in c a s e " generator, the electromagnetic torque T. opposes the primemover
torque and in case of motor, Tc rotates the driven e q u ip m c n

From Eq. (3.44), Ea = Ku <}>u)„


370 Electrical Machinery (Art. 4.3

From Eqs. (3.44) and (4.6),


Te _ I

or T w
A
a)m = E I
e L a
...(4.7)
Power balance Eq. (4.7) states that mechanical power developed is equal to the electrical
power developed.

Electromagnetic torque Te = — Ea Ia ...(4.8)


COr
Torque expression of Eq. (4.6), can alternatively be obtained as follows.
The electromagnetic torque, from Eq. (4.8), is

T' = -A - (Ea /„)

Substitution of the values of Ea and com gives


1 U ZnP
L
27m a
V
...(4.6)

The torque given by Eq. (4.6) is also called the torque developed or total developed torque.
Physical interpretation of Eq. (4.6) reveal? that for a given d.c. machine, Ka is constant and
therefore, torque Te is directly proportional to field flux and armature current. The product
E I is called the electromagnetic power or internal armature power.
- ° Torque expression for a dc machine can also be obtained by referring to Fig. 4.8. In Fig A A

t o r q u e 'w h o s e time variation is a replica of the B- wave. Th,s .s shown m F,g. 4.8 (6).

B,F ^F o rce on conductor


/ , Fl ux density,B A verage flux
density
n
’— m / \

Torque

Torque
(b)
(a) Fig. 4.8. P roduction o f torque in a dc machine.

Scanne
Art. 4.3] D.C. Machines 371

Average force on one conductor,


fav = Bav I l
As this force is unidirectional, total force acting on Z conductors is
F ~ fav - Z - Bav I I Z
Since Bav over a pole-pitch is constant as shown in Fig. 4.8 (6), and 7, 1 and Z are also con­
stant, electromagnetic force F is also constant. For r = mean air-gap radius, the electromag­
netic, or motor, torque is given by
Te = F •r = B„„ •IlZr

But B =
av 2nd
T II 7 P Z A .T
e 2nd = 2n

Ia Nm ...(4.6)
2 na
where I = conductor current
= current in each parallel path
Armature current
a Number of parallel paths
It is thus seen that average electromagnetic torque for given flux per pole and armature
current has uniform profile as shown in Fig. 4.8 (6). Eq. (4.6) also shows that motor torque in a
dc machine is independent of waveshape of B-curve. Motor torque, however, depends upon the
flux per pole for a given armature current.
Torque expression given by Eq. (4.5) em­
phasizes the interaction of main field flux <j>
with armature m.m.f. Fa for the production of
electromagnetic torque. From this, torque ex­
pression of Eqn. (4.6) is derived. Slot Armature
Tooth conductor
Torque expression derived from Fig. 4.8
begins with a force on armature conductor.
(a)
This is a convenient way of deriving the
torque expression, Eq. (4.6). Mechanism of
torque production, however, is different in an
actual machine where the conductors are
placed in the armature slots.
In o rd er to u n d ers tan d th e physical
process of torque production, refer to Fig. 4.9. «>)
In Fig. 4.9 (a) is shown the distribution of
main field flux in the absence of any arm a­
ture current in the slots. It is seen from this
figure th a t magnetic flux produced by north
pole passes the arm a tu re m ain ly through Torque
teeth because slot portion has long air gap
and therefore high reluctance. Fig. 4.9 (6 )
shows the distribution of flux produced by ar­
mature current alone. Currents in the con­ Fig. 4.9. Magnetic flux distribution (a) due to main pole
ductors are indicated by dots. The arm ature- alone ( b) due to armature currents alone (c) when both
produced flux is upwards to the rig h t of slot main-pole flux and armature flux are present.

and downwards to the left of slot.

S c a n n e d by C a m S c a n n e r
372 Electrical Machinery [Art. 4.3

When both main-field flux and armature flux are present, main field flux distribution gets
distorted as shown in Fig. 4.9 (c). Just above the slot, the armature flux flows from right to left.
This distorts the resultant flux distribution from right to left as shown. These tilted lines tend
to shorten their path and in doing so, a tangential torque from left to right is produced. It can
therefore be stated that the interaction of the main flux with the armature current causes the
resultant flux to act on the armature teeth and in doing so, electromagnetic torque is produced
in the clockwise direction in Fig. 4.9 (c). It is indeed fortunate that there is very little force on
the armature conductors. If all the magnetic force were to act on the conductors, it would harm
the insulation between the conductors and slots.
E xam ple 4.2. The armature o f a 4-pole lap-wound dc machine has core length = 30 cm,
diameter = 40 cm, total conductors = 500, speed = 1200 r.p.m. and current = 20 A.
For an average flux density o f 0.5 T, find the electromagnetic (or gross m echanical) p ow er
developed and the internal torque.
1200
Solution. Here / = 0.3 m, r = u.zu
0.20 m, speed, n = ^ 0 = 20 rps.
60

It is known that B„„ =


av 2nrl

Flux per pole, * = M * 2 k x 0 . 2 x 0.3 = 0 04712 ^


4

Generated e.m.f., Ea = = M l 712 x 500 x 20 x 4 = y


a 4
Gross mechanical power developed = Ea l a = 471.2 x 20 = 9424 W.

Internal torque, Te = ^ = 74.994 Nm ~ 75 Nm.

Exam ple 4.3. A 6-pole dc machine has 300 conductors and each conductor is capable o f
ls d ^ L 7 n a t % 7 o r p m eXCeSSWe temperature rise' The flux per pole is 0.015 Wb and the machine

tnrn^0mf t h e ^ ^ CUrrJ nt’ e m f - Power developed in the armature and electromagnetic


torque, i f the armature conductors are : (a) wave connected and (b) lap connected
Solution. Here P = 6, Z = 300, <{>= 0.015 Wb

Speed, n = -^fjp = 30 rps

The average e.m.f. per conductor = Pmj) = 6 x 30 x 0.015 = 2.7 V


(«)_When
_ the conductors are wave connected,/ there—are ——wonly M
2 parallel
p paths i e a - 2
1otal r n r r p n f / ^ c n r r o n f n n . ___ r i „
Total current, Ia = current per conductor x a = 80 x 2 = 160 A ’ ’ u “ £j.

E.m.f. Ea = (e.m.f. per conductor) •— = (2.7) _ 4Q5 y


a 2
Power developed in the armature or electromagnetic power
= E a I a = 405 x 160 = 64,800 W = 64.8 kW

Electromagnetic torque, T, = E0 Ia/a


= 343 5 Nm

(W W ien the conductors are lap connected, there are 6 parallel paths fi
Total current, Ia = 8 0 x 6 = 480 A ’ " 6*
300
K = (2.7) = 135 V

S c a n n e d by C a m S c a n n e r
f-?- Art. 4.4] D.C. Machines 373

Power developed in the armature = En 1 = 135 x 480 = 6480 W = 64.8 kW

E a Ia 64800
Electromagnetic torque, Te = = 343.5 Nm
wm 2n x 30

This example demonstrates that dc machine rating, in terms of electromagnetic power and
internal torque remains unaltered whether the armature winding is lap connected or wave
connected.

4.4. Circuit Model of DC Machines


In dc machines, all parallel paths in the armature are symmetrical. Each path has a
generated emf E a and a resistance rp as indicated in Fig. 4.10 (a) where, for the sake of
simplicity, four parallel paths are shown. Viewing from machine terminals (use Thevenin’s
theorem), the armature circuit of Fig. 4.10 (a) can be replaced by circuit model of Fig. 4.10 (b)
where series resistance ra = rp/a and ‘a’ is the number of parallel paths.
ra

Fig. 4.10. Pertaining to the armature circuit of a d.c. machine.

The field winding of a dc machine must also be represented in the circuit model. Therefore, the
schematic representation, or circuit model, of dc machines is as shown in Fig. 4.11 (a) for a dc gene­
rator and in Fig. 4.11 (b) for a dc motor. It is seen from this figure that for a generator, electromagnetic
torque Te is opposite to the rotor rotation, i.e. Te opposes the prime-mover torque. This is essential for
the conversion of energy from mechanical to electrical. This torque Te may, therefore, be called
counter-torque in a dc generator. The magnitude of this counter-torque is given by Eq. (4.6). For
motor, the electromagnetic torque Te is in the direction of rotor rotation.
/
G enerating M ode. When armature current Ia is in the direction of generated emf Ea, dc
machine operates in the generating mode. Electromagnetic power is E J a watts and the electri­
cal power output or load power is equal to terminal power VJa watts, Fig. 4.11 (a).
For a dc generator, shaft power input = E J a + no-load rotational loss
and Vt = Ea - Iara ...( 4 .9 )
where Vt = armature terminal voltage, V
Ia = armature current, A
ra = armature circuit resistance, ohm
Brush contact drop is usually taken as constant at 1 or 2 V. This drop is independent of
armature current as the conduction process is primarily through numerous short arcs. Taking
this drop as 2 V, Eq. (4.9) can be written as
Vt —Ea —Iara —2
or VJa = E J a - r„ - 2 /„
or Electrical power output = Electromagnetic power - Ohmic losses - Brush contact loss ...(4.10)

cann ed by C a m S c a n n e r
374 Electrical Machinery [Art. 4.4

0------ — W M -------- -A \V W
u+ 71
!a
+
71
Vi
m m 't fm v
( eq
rf rf

If
Vf 1) c>~~ • (
4

(a) (b)
Fig. 4.11. Circuit model of a d.c. machine for (a) generating mode and (b) motoring mode.
M otorin g M ode. When armature current Ia flows in opposition to generated emf Ea as in
Fig. 4.11 (6), dc machine operates in motoring mode.
For a dc motor, shaft power output = EaIa - no-load rotational loss

and Vt = Ea + Iara + 2 ...(4.11)


or VJa = EJa + l\ra + 2Ja
or Electrical power input = Electromagnetic power + Ohmic losses + Brush contact loss ...(4.12)
It is stated above that for generator operation, the emf and current in a conductor are in the
same direction. This is shown in a 2-pole dc generator of Fig. 4.12 (a). In this figure, for anti­
clockwise rotation of armature, generated emfs are indicated by crosses and dots below the
circles in the slots (use right-hand rule). For assumed anticlockwise rotation, S and N poles
indicated on the rotor in Fig. 4.12 (a) must be created by armature currents. This magnetic
polarity demands the current directions in the conductors to be indicated by crosses under S
pole and by dots under N pole. Current directions in slots are indicated by crosses and dots
within the circles in Fig. 4.12 (a). This shows that generated emfs and currents in a conductor
are in the same direction for a dc machine working as a generator.
Fig. 4.12 (6) shows motoring-mode operation of a dc machine. Direction of currents in the
armature conductors is indicated by crosses and dots within the circles in the slots. These cur­
rents establish N and S poles on the rotor and their interaction with the stator N and S poles
results in clockwise rotation of armature. With the armature rotation, the conductors cut the
same flux that gives rise to motor torque. This flux cutting action thus causes the generation of
emf in the very same conductors that experience motor action. The generated emfs in Fig 4 12

S c an n ed by C a m S c a n n e r
*
Art. 4.4]
D.C. Machines 375

«,) are indicated by crosses and dots below the circles in the slots (use right hand rule). It is seen
that generate em s are in directions opposite to the flow of conductor current. Since this
generated emf opposes the flow of current, it is called counter electromotive force or counter
emf- Note that counter emf Ea can never be equal to applied voltage V,. Emf Ea must always be
less than Vt because direction of current, as per Eq. (4.11), first determines the direction of
rotation and then the direction of counter emf. The magnitude of counter emf is governed by Eq.
(3.41).
Summarising the above, emf Ea in Eqs. (4.9) and (4.11) is the generated emf. In a generator,
Ea > Vf and in a motor, Ea < Vt. When dc machine is working as a motor, the generated emf Ea
is often called the counter em f or back emf. As stated above, the word counter is used because
Ea opposes the flow of current Ia. The word back is used because Ea is the voltage at the back of
ra when viewed from the armature terminals. Therefore, the generated emf Ea for a motor may
also be called voltage behind armature circuit resistance.
Example 4.4. A 6-pole lap-wound dc generator has 240 coils o f 2 turns each. Resistance of
one turn is 0.03 ohm. The armature is 50 cm long and 40 cm diameter. Air-gap flux density of
0.6 T is uniform over pole shoe. Each pole subtends an angle o f 40° mechanical. For armature
speed o f 1200 rpm, find (a) generated em f at no load and (b) the terminal voltage at full load
armature current o f 40 A.
P 6
Solution. We know that 0rJcr = - 0nurh- Therefore, pole shoe subtends an angle 40 x - = 120°

electrical.
Flux per pole = (air-gap area under one pole shoe) (uniform flux density)
( 2ru7 120
x 0.6
P * 180

= fo. ^ 0_^ 0-5 | x o.6 = 0.042 Wb


6 3
|,
Total armature turns = 240 x 2 = 480. This gives total armature conductors,
Z = 4 8 0 x 2 = 960.
1200
Speed, n = = 20 rps, P = 6, a = 6

o ♦j c i . j e* $ Z n P 0 042 x 960 x 2 0 x_6 _ Qng A v


Generated e.m.f. at no load, Ea = x = c ” ° Ub>4 v
a a fc>

(6) Number of armature turns per path = = 80

Resistance of one path = 80 x 0.03 = 2.4 Q


For 6-pole lap-wound dc generator, there are a = 6 parallel paths.
_ . „ . . . Resistance of one path _ 2.4 _ n A n
.-. Resistance of armature circuit = Number 0f parallel paths 6

For dc generator, terminal voltage at full load, Vt = Ea - Ia ra


or Vt = 806.4 - 40 x 0.4 = 790.4 V

ou aiiiieu u_y v^ai i loccii ii itri


(A rt. 4.5
376 Electrical M a c h in e r y ----------

4.5. Methods of Excitation


A d c machine can work as an electromechanical energy converter only when ,ts field wind­
ing is excited with direct current, except for small d.c. machines employing permanent mag­
nets. .
There are, in general, two methods of exciting the field windings of d.c. machines.

(a) Separate excitation and


(b) Self-excitation.
D.C. machines may have one or more field windings and their method of excitation, deter­
mines the performance characteristics of the d.c. machines. In Fig. 4.1 , t e e an side
diagrams give the physical details of the internal connections for a 2-pole machine; whereas the
right hand side diagrams give the schematic representation of d.c. machines with the method
of excitation specified.
(а) Separate excitation. The separately excited field winding consists of several hundred
turns of fine wire and is connected to a separate or external d.c. source, Fig. 4.13 (a). The
voltage of the external d.c. source has no relation with the armature voltage, i.e. the field wind­
ing energised from a separate supply can be designed for any convenient voltage.
(б) Self-excitation. When the field winding is excited by its own armature, the machine is
said to be a self-excited d.c. machine. In these machines, the field poles must have a residual
magnetism, so that when the armature rotates, a residual voltage appears across the brushes.
This residual voltage should establish a current in the field winding so as to reinforce the
residual flux.
A self-excited d.c. machine can be sub-divided as follows :
ii) Series excitation. The field winding consists of a few turns of thick wire and is con­
nected in series with the armature, Fig. 4.13 (6). In other words, the series field current
depends on the armature current and in view of this, a series field may be called a current-
operated field.
(ii) Shunt excitation. The field winding consists of a large number of turns of fine wire
and is connected in parallel (or in shunt) with the armature, Fig. 4.13 (c). Therefore, the voltage
across the armature terminals and the shunt field is the same and it is for this reason that a
shunt field may be called a voltage-operated field.
Remember that series and shunt field windings are characterised by low and high resistan­
ces respectively.
Ciii ) C om pou n d excitation. A compound excitation involves both the series excited wind­
ing and the shunt-excited winding. In some applications, a shunt-excited winding may be
replaced by a separately-excited winding. Here a d.c. compound machine, with series and shunt
field windings will be discussed.
If the series field flux aids the shunt field flux, so that the resultant air-gap flux per pole is
increased, the machine is called a cumulatively compounded d.c. machine as shown in Fig. 4.13
(d). In this figure, direction of arrows corresponds to the direction of magnetic flux produced by
shunt and series field windings. As the two arrows are in the same direction in Fig. 4.13 (d ),
this figure is for a cumulatively compounded d.c. machine. In case the series field flux opposes
the shunt field flux, so that the resultant air-gap flux per pole is decreased, the machine is
called a differentially com pounded d.c. machine as shown in Fig. 4.14 (o). In this figure, direc­
tion of arrows indicate that the two fields are opposing each other.

Scanned by Cam Scanner


Art. 4.51 D.C. Machines 377

(b)

(c)

(a) Separately excited d.c. machine. (6) Series-excited d c machine.


(c) Shunt-excited d.c. machine. (d) Compound excited d.c. machine.
F ig . 4 .1 3 . M e th o d s o f e x c ita tio n o f d .c. m a c h in e s.

of connections a d.c. compound machine may have short-shunt con-


From the view p ^ on jn short-shunt connection of Fig. 4.14 (a), the shunt field or
nection or a long-shuntconne , th armature terminals. In long-shunt connection,
voltage-excited w m d m g ^ e h f a e or line terminals as shown in Fig. 4.14
W . In o rd e rfo remember the difference between short-shunt and long-shunt connections, one

p ."
'Scanned by Cam Scanner
378 Electrical Machinery IA rL 4.5

may examine Fig. 4.14 a little more careful­


ly. A short-shunt connection requires a wire
of short length for connecting B to A ,
whereas in long shunt connection, a wire of
long length is required for connecting B to
A. However there is no appreciable dif­
ference in the operating characteristics of
short-shunt and long-shunt connections. (a)
14. compound macm ne connect
The choice between the two types, depends
(a) short shunt and ( b ) long shunt.
on mechanical considerations of connec­
tions or of reversing switches.

An examination of the electrical circuits of Figs. 4.13 and 4.14 reveals that field and arma­
ture circuits are always drawn at 90° with respect to each other. This circuit representation is
made to agree with the physical fact that the magnetic fields of armature and field windings
are space displaced from each other by 90°. Reference to Fig. 4.5 (6) is also helpful.
S h u n t f ie ld w i n d i n g
In Fig. 4.13 (d ), each pole of the compound machine
(th in w ire )
is shown to possess shunt and series field windings. Fig.
4.14 (c) illustrates how these windings are arranged on
one pole of a d.c. machine. In this figure, shunt field coil
is placed near the yoke and series field coil near the pole
shoe just for the sake of clarity. Actual physical arrange­
H
S e r ie s fie ld w in d in g
ment of these coils is as shown in Fig. 4.14 (d). It is seen
(th ic k w ire ) from this figure that first shunt field coil is wound
<c> (d ) around the pole body and over it is then wound the
p. . . . e . senes field coil. The reasons for placing the series field
g., 4.14.
---------
Series and Diiuut
shunt neid
field winding;
windings on coil outside are ( i ) convenience in erm<5f i - i W A t
one pole
pole of
° f *a d.c.
d c compound
“ “ " " “ "I machine.
» - » . for its better cooling construction and («)

- PaT lr r ed dC - * « "m a tu r e
ju sted to g w e an open circuit cottage o f 260 V Now “when thl'Z*'1 tHe,field curT^ t is ad-
rated current, the speed o f the g
n
Z
d
motor is fou
Z
t o
voltage o f the generator under these conditions. Field flux remains u r Z l t e r e d ^
Solution. It is seen from Eq. (3.41) that generated voltage is

At no load, 260 1600 x 0


Rated armature current, 24000 ...ii)

250 ~ 9 6 A
If the generated voltage under rated load is Eol, then

Eaj « 1500 x (j>


...(ii)
From (i ) and (ii) — ~ 1500 x 0
-
260 1600 x <f>
or Eal = 243.75 V
From Eq. (4.9), the terminal voltage is

Vt = En\ ~ K ra
= 243.75 - 96 x 0.1 = 234.15 V.

S c an n ed by C a m S c a n n e r
Art. 4.6] D.C. Machines 379

Example 4.6. A 230 V d.c. shunt machine has armature circuit resistance (including
brashes) o f 0.5 Q and field circuit resistance o f 115 Q. I f this machine is connected to 230 V
supply mains, find the ratio o f speed as a generator to the speed as a motor. The line current in
each case is 40 A.
Solution. Generator operation. The circuit diagram for dc generator operation is shown
in Fig. 4-15 (a)- ^ is fi£ure> = ^ ne current. If = field current and 7ol = armature current.
It is given that line current, IL = 40 A
. 230
Field current, Ir = =2A
115
KCL at point x in Fig. 4.15 (a) gives armature current IaX = IL + If = 40 + 2 = 42 A.

... Generated e.m.f., # al = v t + l a\ ra = 230 + 42 x 0.5 = 2 5 1 V


. 251 °c ng x <]>
where ng is the generator speed and <> is the field flux per pole proportional to If.

- 6-

(a) (*>
Fig. 4.15. Pertaining to Example 4.5 (a) dc generator operation and (b) dc motor operation.

M otor operation. The circuit diagram for dc motor operation is given in Fig. 4.15 (6). In
this figure, Ia2 = armature current and field current = 2 A as before.
KCL at point x in Fig. 4.15 (6) gives dc motor armature current Ia2 = l L - l f = 40 - 2 = 38 A.

.•. Motor counter e.m.f., Ea2 = Vt - Ia2 ■ra = 230 - 38 x 0.5 = 211 V

But Ea2 = 211 °c nm x <]> ...(ii)


where nm is the motor speed.

Eai 251 _ 1ZB-


From Eqns. (i) and (ii),
Da2 211 nmx<t>

or = 1.1896.
nm
Thus the ratio of speed as a generator to the speed as a motor is 1.1896.

4.6. M.m.f. and Flux Density Waveforms in d.c. Machines


It has been shown in Art. 4.1 that the current distribution under each pole of a d.c. machine
is uniform and is equivalent to a constant current sheet of proper p o a n y. e space is n u-
tion of m.m.f. produced by the armature winding is triangular, with its peak value along the
quadrature or brush axis (or 90" away from the main field axis) as shown in F,g. 3.35. The effect
of armature m.m.f. on the main-field flux distribution in the air gap is called armofure reachon.
In this article, armature reaction and its effects are dea t wi in e a i.

\ ju u i ii iovj k_/ y w u i i iv^/uui ii iv/i


380 Electrical Machinery [A rt 4.6

4.6.1. A rm a tu re reaction . The armature m.m.f. produces two undesirable effects on th


main field flux and these are,
(i) net reduction in the main field flux per pole and
(ii) distortion of the main field flux wave along the air-gap periphery.
Reduction in the main flux per pole reduces the generated voltage and torque, whereas the
distortion of the main-field flux influences the limits of successful commutation in d.c.
machines. The purpose of this article is to investigate these effects and to discuss the methods
of minimising the problems arising from armature reaction effects.
Fig. 4.16 (a) shows the flux path for the main poles of a 2-pole dc machine at no load, i.e.
with no armature currents. This main field flux, produced by field m.m.f. IfN f, is shown by
horizontal phasor OA = <Jyon the right hand side of Fig. 4.16 (a). When the dc machine is loaded,
currents flow in the armature winding. These currents are shown in Fig. 4.16 (b), by dots under
main S pole and by crosses under main Af-pole. These armature currents set up armature flux
shown by vertical flux lines in Fig. 4.16 (6), with field winding un-excited. In this figure is also
shown armature flux 4>a by a vertical phasor OB. Flux <J)a is produced by armature m.m.f.

q -axis
gna— I
**>— MNA ( o f n o l o a d )
1 r F l u x d u e 1o f i e l d c u r r e n t
a lo n e
GNA

»— M N A (a f no lo a d )
A
0 — d -axis
Field f lu x

P o le f a c e r (a)

GNA
^q-axis

0
d-axis
A rm atu re
~ f lu x
B
(b)

GNA
•M N A (o n l o a d )

i /
i/

M o to r
ro ta tio n
y S 1 V dRiesst rui lbt ua nt ito nf l u x

Fig. 4.16. Illustrating the space distribution o f (a) main-pole flux (6 ) armature flux and
(c) resultant o f both main-field and armature fluxes.

S c an n ed by C a m S c a n n e r
D.C. Machines 381

/ y If thedc machine of Fig. 4.16 is working as a motor, then its arm ature must rotate anti*
II I ckwise>because of the fact th a t N, S poles of the main field must attract arm ature produced
c V poles. *n case mac^^ne working as a generator, then its arm ature must be driven
f clockwise by the prime-mover.
An examination of Fig. 4.16 (a) and (6 ) reveals that the path of arm ature flux 0 Ois perpen-
1 d'cular to the m ain flux path <fy\ In other words, the path of the armature flux crosses the path
(“the main-field flux. Thus the effect of arm ature flux on the main field is entirely cross-mag-
\ lizing and it is for this reason th a t the flux created by the armature m.m.f. is called cross-flux.
When current flows in both the arm ature and field windings, the resultant flux distribution
• nhtained by superimposing the two fluxes of Fig. 4.16 (a) and (6 ). This is illustrated on left-
if d side of Fig. 4.16 (c)- It is seen that arm ature flux aids the main field flux at upper end ol
Wnnle and at the lower end of S-pole, therefore, at these two pole ends (or tips), the armature
flnv strengthens the main field flux. Likewise, the armature flux weakens the main field flux
at lower end of N-pole and at upper end of S-pole. I f there is no magnetic saturation then the
mint of strengthening and weakening of the main field flux are equal and the resultant flux
T r pole remains unaltered from its no-load value. Actually, magnetic saturation does occur an
as a consequence, the strengthening effect is less as compared to the weakening effect and
r e s u l t a n t flux is decreased from its no-load value. This is called demagnetizing effect of arma-

^ t a t h e phasor diagram of Fig. 4.16 (c), phasor sum of field flux <tyand armature flux 0 gives
net flux 0*. This resultant flux 0* is seen to be more than the main-field flux at no load. This
• v. m,,Br nnt true because of magnetic saturation in one of the pole tips of each pole. A
is, howe , (ONA) is along the quadrature axis of the dc machine. Magnetic neutral
of resultant field f l u . It is seen from Fig. 4 1 6
(a) that MNA at no load coincides with the GNA or qr-axis. When the dc machine is loaded,F g_
4.16 (c) reveals that MNA is shifted from GN A. TWs shift is dependent upon.the mapi.tu
S i t h e magnitude of armature ( o r > o a ~ t £ - U r U ‘ he
shift of M NA from GNA. It may therefore be stated from above that net effect of armature flux
on the main-field flux is
(i) to distort the main-field flux thereby causing non-uniform distribution of flux under
the main poles, . .
(ii) to shift the MNA in the direction o f rotation for a generator and against t e irec 10
o f rotation for a motor and .
(iii) to reduce the main-field flux from its no-load value due to magnetic satura ion.
Graphical picture of arm ature reaction. For better understsmding•” f * e int^ “"
between main-field flux and armature flux, it is preferable an -g ...
developed diagram of armature conductorrs and polas.^ ^ % eve » P % ™ ^ rushes are nlong , ho
r Hnes m F i g ^ Co, show the
distribution of main-field flux in the air gap with no a ^ t u « R g. 4 H (ft), the
variation of main fiold-flux density
roiiauuil UI mam IIUlU-llUA utliom; along
O the air-gap periphery
- - a- fv 1 is shown by a solid line.
1.—- —m- ■aa-vrJ/x«« \/ i\ IA
T hvrints underS pole and crosses underNpole. Ihese
In Fig 4 17(c) currents are indicated Dy aois u i . _ . • t?' < i 't
hi rig. * .i < (a,, currents a* linnXCjtcd produce magnetic flux as shown in Fig. 4.1 i
a r m a tu r e c u r r e n t s , w it h fie ld w in d in g u n c x c u c u , p ; 0 n n rm n l i n t h o m a in fiolri fln v
(c) As the armature flux produced by armature currents alone is normal to the main field flux,
it;, ns me armature nux i j rreated bv armature currents in a dc machine, as
as before, it is called cross flux The "urnX crented by # solid „ nc in Fi„ 4 . „
; = C 1 n ^ t « r e and is depicted accordingly byasoHd line in Fig. 4 , 7
(d). The air-gap flux due to armature m.m.f. is given by njr finp reluctancc ^ nt*er
t
ft'

Scanned by Cam Scanner


382 Electrical Machinery (Art. 4.6

Flu x d u e to f i e l d
c u r r e n t a lo n e d -a x is

U - GNA

B ru sh

*£> \ (s); >. » / » / 03; ®.


' r --v .-f -.-.r ;- r - n - -1- — '
1 1 1 — -

Fig. 4.17. Flux distribution and flux-density waveforms respectively due to (a). (6) field current alone
(c), (d) armature current alone and (e), if) both field and armature currents.

the air gap is uniform therefore the air-gap flux variation under the poles is proportional to
arm ature m.m.f. and ,s shown m Fig. 4.17 (d). In between the poles ie ., in the to
region, the long air gap offers large reluctance, consequently the arm ature flux is much smal er
in this region, in spite of a large value of mmf. The arm ature flux density waveform created bv
arm ature currents is, therefore, saddle-shaped’ as depicted by dotted curve in F k 4 17<d, I t
is observed from this figure that arm ature flux-density waveform has (i) zero value at the
centre of the pole (ii) increases from zero to maximum value at the pole tips and then (Hi)
decreases rapidly to a m inimum value at the middle of main poles.
* Shape o f a cycle-saddle.
------------------------------------- -------------------------------------------------------------------------------D.C. Machines 383

When both armature and field windings carry currents, the resultant flux distribution is
obtained by superimposing the two fluxes, field flux of Fig. 4.17 (a) and armature cross-flux of
Flg' 1 lu * r<fsultant flux distribution, so obtained, is shown in Fig. 4.17 (e). This figure
reveals the strengthening of the resultant flux at one pole tip and weakening at the other pole
tip of each pole. As expected, this agrees with Fig. 4.16 (c).
For obtaining the resultant air-gap flux density waveform, when both field and armature
windings carry currents add the corresponding flux-density ordinates of Figs. 4.17 (6) and 4.17
(d ) at every poin a^ong the air-gap periphery. The resultant flux-density distribution in the air
gaps is shown by the solid curve in Fig. 4.17 (/). It may be seen from the resultant flux-density
distribution curve that the effect of cross-magnetizing armature mmf is to decrease the flux
density un er e eading pole tips and to increase it under the trailing pole tips for generator
operation. Actually the magnetic saturation in the iron does occur and its effect is to increase
the flux density under the trailing pole tips by a smaller amount than the decrease under the
leading pole tips. Saturation effect is indicated by cross-hatched areas in Fig. 4.17 (/). Thus,
under unsaturated conditions, the amount of flux increase under trailing pole tips is almost
equal to the decrease under leading pole tips and the total flux per pole on load remains almost
unchanged from its no-load value. Under saturated conditions, the amount of increase in flux
is less^than the decrease and therefore total flux per pole on load is less than its no-load value.
Hence, under saturated conditions, the effect of cross-magnetizing armature mmf, i.e. cross­
flux, is to demagnetize the main field. But note that the demagnetizing effect of cros-flux is due
to saturation only.
Fig. 4.17 (/) reveals that point of zero flux density has shifted through an angle 0 from
C, C to D, D' respectively. In other words, MNA has shifted from GNA by an angle 0.
For a motor, a dot under south pole and a cross under north pole results in a anti-clockwise
rotation in Fig. 4.16. Therefore, above results are also applicable to d.c. motor. Since the direc­
tion of rotation is reversed, the leading pole tips for a motor are the trailing pole tips for a
generator. Hence for a motor, the effect of cross-magnetizing armature m.m.f. is to decrease the
flux under the trailing pole tips and to increase it under the leading pole tips.
Note from above that for a generator, the effect of armature reaction is to distort the flux
and shift the zero crossing of the flux density wave in the direction of rotation. In the case of
motor, the distortion of the flux and zero crossing of the flux-density wave is shifted against the
direction of rotation. In constant flux d.c. machines, such as shunt machines, the flux distortion
is much more prominent under heavy loads. In series and compound machines, the flux distor­
tion is minimum, because with the increase of armature m.m.f., there is a corresponding in­
crease in the field m.m.f.
The effects of armature m.m.f. described above, may be summarised as follows :
(i) Armature flux path is normal to flux path of main poles. That is why armature flux is
called cross-flux or cross-magnetizing flux.
(«) The armature section distorts the main-field flux distribution along the air-gap
periphery. This distortion is in the direction of rotation for a dc generator and opposite to the
direction of rotation for a dc motor. This also means that MNA is shifted in the direction of
rotation for a generator and against the direction of rotation for a motor. This shift of M NA from
GNA depends upon the magnitude of load (or armature) current.
(iii) The demagnetizing effect of armature mmf reduces the total flux per pole. This reduc­
tion has been found to be 1 to 5% from no-load to full-load.

* If one travels along the assumed direction o f rotation (i.e. from left to right for generator operation and from
right to left for motor operation), the pole tip that comes first is called the leading pole tip. Obviously, the other
pole tip is the trailing pole tip.

S c an n ed by C a m S c a n n e r
384 Electrical M achinery fA rt. 4.6

E xam ple 4.7. Explain how the demagnetizing effect o f armature m m f on the main-field flux
can be investigated through the use o f magnetization curve o f a dc machine.
Solution. The effect of magnetic saturation on the reduction of main-field flux caused by arma­
ture reaction can be visualized with the help of Fig. 4.18. In this
figure, magnetization curve, triangular armature mmf varia­
tion and S pole of a dc machine are shown. The brushes are in
w w ^ /iy y //^
the GNA. For field current I f , the field mmf is A T f . At no load,
the flux density Ba, under the entire pole shoe is ab for a field mmf
A T f . For armature current I a , armature mmf is + A T a under pole

tip Q and - A T a under pole tip P. The net mmf under pole tip Q is
( A T f + A T a ) and that under pole tip P is ( A T f - A T a) as shown. The
mmf ( A T f + A T a ) under pole tip Q corresponds to a flux density
B x . Under pole tip P, net mmf ( A T f - A T a) corresponds to a flux

density B 2. It is observed from Fig. 4.18 that increase in flux at pole


tip Q, proportional to (B x - Ba) is less than the decrease in flux
under pole P, proportional to (Ba - B2). This shows that under
saturated conditions, the resultant flux per per pole is reduced from
its no load value. Therefore, the field excitation has to be increased
under loaded conditions if the field flux is to be kept constant at its
no-load value.
Note that the flux density at the middle of pole face is not Fig. 4.18. Demagnetizing effect of
effected by armature mmf. armature m m f on the main-field flux.

E xam ple 4.8. Discuss the detrimental effects o f armature reaction in dc machines.
Solu tion. It has already been stated that effect of armature mm f on the main field flux is
two fold, (i) distortion of the main field flux and (ii) net reduction of the main field flux.
Distortion of the main field flux gives rise to three detrimental effects and reduction in field
flux leads to one bad effect. Rise in iron losses, poor commutation and sparking are caused by
distortion of main field flux, whereas the reduction in field flux influences the cost of field
winding. These are discussed briefly below.
(i) Iron losses. These losses depend on the maximum value of flux density in teeth and in
the pole shoes. The armature reaction, by distorting the main field flux waveform, increases the
flux density considerably over its corresponding no-load value. As a result ; iron losses, par­
ticularly in teeth, are much greater on load than on no load. In addition, high degree of satura­
tion in teeth forces the flux to stray into the core-end plates, end covers etc. This all leads to
more eddy-current and hysteresis losses. Roughly, iron losses at full load is taken to be 1.5
times its value at no load.
(ii) C o m m u ta tio n . At no load, zero-crossing of the flux density wave is along the GNA such
as point C, C ’. Under loaded conditions of the dc machine, zero-crossing of the flux density wave
is shifted by an angle 0, which depends on the magnitude of armature current.
For good commutation, the coils short-circuitoa Dy the brushes should have zero e.m.f. in­
duced in them. The brushes are usually placed along the GNA. Since zero-crossing of the flux
density wave is shifted from GNA or <7-axis, the coils undergoing commutation do not have zero
e.m.f. induced in them. The induced e.m.f. in the commutated coils delays the reversal of arma­
ture current in the short-circuited coils ; this may result in detrimental sparking, or poor com­
mutation, at the brushes.

S c an n ed by C a m S c a n n e r
Art. 4.6J
D.C. Machines 385

. -Jn mF avy load>the flux density waveform is distorted considerably. If


/ %ln\ mnv hp^ 0 3 ? ! i ^Fe . ^ e maximum flux density points, a much greater voltage
pnt«pvrppH C°^‘ rotational voltage between adjacent commutator
nark mav f sPark may occur between these adjacent segments. Sometimes,
this spark may spread around the commutator in the form of a ring fire.
a " *nding. Demagnetizing effect of cross-magnetizing armature mmf is to
lu UC m 'f a rUl P0r P°re itS n°-load value due to magnetic saturation. In a generator,
the^magnitude of the e.m.f. generated in the armature decreases with increase in load. In a
motor, electromagnetic torque is decreased as the flux per pole is reduced under load. In order
t ^K^nnrpfcf. ° tk 1S ^ y ctl0^ in total flux, the field mmf must be augmented. This is possible
■ J- Um u turns *n t^ie fi^d winding or (ii) by using a thick wire for field
winding schemes entails more copper and, therefore, more cost of the field

4.6.2. M ethods o f lim iting the effects of armature reaction. The cross-magnetizing
effect of armature m m f can be minimised at the design and construction stage of a dc machine.
Various methods of mitigating the effects of armature reaction are discussed below.
(a) H igh-reluctance pole tips. I f the reluctance of the pole tips is increased, then the
magnitude of armature cross flux is reduced and the distortion of the resultant flux density
wave is minimised. The reluctance of the pole tips can
be augmented by using chamfered or eccentric pole
shoes. A machine fitted with chamfered or eccentric
pole face has short air-gap length at the pole centre chomfervj j I &n9Qp
and longer air gap lengths under the pole tips, i.e. the
profile of the pole shoe is not concentric with the arma­ Short
ture core as shown in Fig. 4.19 (a). gap
A rm a tu re
Another method of increasing the reluctance to c o re s u rfa c e
cross flux is to assemble alternatively the pole lamina­ i L A
tions depicted in Fig. 4.19 (6). That is, if the first rl2
lamination has the pole tip to the left, the second n2
=32
lamination has its pole tip to the right, the third =>2
lamination pole tip to the left and so on, until the re- (b}
a iU ■ i i i o- .L • rig. 4.19. (a) Chamfered or eccentric pole (6)
quired pole depth IS developed. Since the iron area Laminations 1 and 2 are stacked alternatively
under the pole tips is almost halved, the reluctance to give a pole-face view as shown,
under the pole tips is considerably increased.

The two constructional techniques mentioned above reduce the main field flux to some
extent. In order to maintain it constant, the main field mmf must be raised accordingly. But the
influence of increased pole-tip reluctance is more pronounced on the cross flux than on the
main-field flux.
In dc machines, the short air gap at the pole centre and longer air gaps at the pole tips are
kept only to lim it the effect of cross-magnetizing armature m m f on the main pole flux. The
distribution of the flux density wave along the air-gap periphery need not be a sine wave in dc
machines. But in synchronous machines, the air gap at the pole centre is short and at the pole
tips it is larger from the view point of obtaining sine wave for the flux density wave. In
synchronous machines of the salient-pole type, the non-uniform air gap under the pole faces has
nothing to do w ith the arm ature reaction.
(b) R e d u c tio n in a rm a tu re flu x . Another constructional technique of reducing the arm a­
ture cross flux is to create more reluctance in the path of arm ature flux w ithout reducing the
main field flux noticeably. This is achieved by using field-pole laminations having several rec-

cann ed by C a m S c a n n e r
[A rt. 4.7
386 Electrical Machinery
A rm m a g n e tic

F ie ld p o le \
laminalion-*
(6 )
(a )
Fig. 4.20. (a) One field-pole lamination with four punched holes
(6 ) Two-pole dc machine having punched field-pole laminations.

tangular holes punched in them. One such lamination having four holes or slots is shown in Fig.
4.20 ia). It is seen from Fig. 4.20 (b) that reluctance offered to armature flux is increased due to
four air-gap openings introduced in the path of cross flux. As a result armature cross flux is
reduced considerably, whereas the main-field flux remains almost uneffected.
Combination of the constructional features described in Figs. 4.19 and 4.20 may be used
most effectively in reducing the armature cross flux.
(c) Strong main-field flux. During the design of a dc machine, it should be ensured that,
the main field mmf is sufficiently strong in comparison with full-load armature mmf. Greater
the ratio of main field mmf to full-load armature mmf, less is the distortion produced by arma­
ture cross flux and predominant would be the control of field mmf over the air-gap flux. Actual­
ly, this ratio depends on the type of duty cycle the dc machine has to perform.
id) Interpoles. The effect of armature reaction in the interpolar zone can be overcome by
interpoles, placed in between the main polos. The magnetic axis of interpole winding is in line
with the quadrature axis. Interpole winding is connected in series with armature so that inter­
pole mmf is able to neutralize the effect of armature mmf in the interpolar zone at all levels of
load current not exceeding the safe limit.
(c) Com pensating w inding. The effect of armature reaction under the pole shoes can be
limited by using compensating winding. This winding is embedded in slots cut in the pole faces
r of the dc machine. This is the best, but the most expensive method. This is described in detail
in Art. 4.9.
4.7. Effect of Brush Shift
It is seen from Fig. 4.16 or Fig. 4.17 that armature reaction shifts the M NA in the direction
of rotation in a generator and against the direction of rotaton in a motor. The brushes are along
the GNA. The coils undergoing commutation have, therefore, rotational e.m.f. generated in
them. As a consequence, sparking and poor commutation occurs. If the brushes are given a shift
through an angle 9, no rotational e.m.f. would be generated in the coils undergoing commuta­
tion ; this would result in smooth commutation. This shows that for obtaining good commuta­
tion, the brushes should be given a forward or backward shift so as to ensure good
commutation. In this section, the effect of brush shift in the direction of rotation (called forward
shift) or opposite to the direction of rotation (called backward shift) is investigated.
In Fig. 4.16 (c), the brushes are at GNA. Let the brushes be given a forward shift in a
generator or backward shift in a motor, so that brush axis is now along the MNA, Fig. 4.21
(a). In other words, the brushes are given a shift through an angle 0 and occupy a position PQ
as shown. The armature flux 6a = OB must be along the brush axis as before. Flux <}>a can now
be decomposed into two components ; OC and OD as shown in Fig. 4.21 (b). The component
OC = <J>a sin 0 is opposing the main field flux <[y. Therefore, OC has a demagnetizing effect on the
main-field flux. The other component OD, being perpendicular to the main flux <Jy, is the Cross-

S c an n ed by C a m S c a n n e r
Art. 4.7)
D.C. Machines 387

GNA i /MNA

tA

(a) (fc)
F ig . 4 .2 1 . E ffe c t o f b r u s h s h ift on th e r e su lta n t flu x in a d c m ach in e.

magnetizing armature flux. Note that the resultant flux is obtained from the phasor sum of
<tyand (J)a, i.e. <J>* = «>a. It is observed from Fig. 4.21 (b) that with brush shift, the resultant
flux gets reduced from its no-load value even if there is no magnetic saturation.
Draw RS making an angle 0 with respect to GNA and on the other side of brush shift, Fig.
4.21 (a). The total armature ampere turns can now be divided into two groups as illustrated in
Fig. 4.22. The conductors shown in angle ROP and QOS or the conductors lying in angle 40 for
every 360° electrical (or 20 for every 180° electrical) are producing a flux opposite to the main
field flux., This can be verified by the right-hand grip rule. Hence the ampere turns due to the
conductors contained in 40 degrees for every 360° electrical are demagnetizing in nature.
GNA
GNA
MNA
20 MNA c
" ~^ROQ./POS.180-26
P/

o ----- ---
(a) (fc)
F ig . 4 .2 2 . E ffe c t o f b r u s h s h ift (a ) d e -m a g n e t iz in g a m p e r e tu rn3 an d (fc) c ro s s -m a g n e tiz in g a m p ere tu rns.

Demagnetizing or back ampere turns/pole


_ 49
[ATs per pole]
“ 360
40 Z_
A T ,=
360 a 2P

20 h ...(4.13)
or ATj =
180 a 2P

In the above expression for A T d, angle 0 is in electrica1 degrees. In Fig. 4.22 (6), the conduc
tors lying in angle ROQ and POS produce flux perpendicular to the fl“ * as shown-
Therefore, ampere turns produced by the conductors lying within angle (360 - 40) for every
360° electrical (or for every pole pair) produce cross-magnetizing ampere urns.

r /n i '
IArt 4.7
388 Electrical Machinery

Cross-magnetizing or distorting ampere turns per pole


' 360 - 40 " la Z '

360 a 2P

' 180 - 28 " h Z '


--(4.14)

<
h"
or

II
180 a 2P

Obviously, it brush shitt « = u, me Drusnes me u.. ---- - -


the entire armature ampere turns are cross-magnetizing in nature.
The effect of brush shift car. also be
conductors and field poles. For this purpose, let the brush triangular
for a generator or backward direction for a ^ ‘ because armature mmf axis must com-
armature mmf wave is also shifted by the same angle, bee . • . ,
cide with the brush axis. The armature flux density waveform is
the interpolar or quadrature axis, Fig. 4.23 (c). When
waveform are added to the ordinates of no-load field form, e re Armature flux densitv
with brush shift is obtained as shown with solid line in Fig. . ( )• ty
waveform in Fig. 4.23 (c) reveals that reduction in flux density at pole tips 1, han the
addition in flux density at pole tips 2, 2'. For comparison purposes the r e s ^ t ^ t flux density
waveform without any brush shift is also drawn with dotted lin e . A n examination of Fig^ 4.23
(6) shows that the ordinates of resultant flux density waveform with brush shift are smaller as
compared with the ordinates of resultant flux density waveform with zero brush shift. Conse­
quently, the flux per pole is reduced if the brushes are given a forward shift in case o f a generator

Interpolar
or q-axis d-axis
d -a x is or
or # i P o lq r a x is
1 P o ia rio x is iB ru s h 1 B ru s h
| s h if t Shift
1 A .
9 iX N
! ? 1 ' XT' ' i i
: A id i
(a )

Motor G en-

R e s u lta n t f lu x d e n s ity
w pve w ith o u t b r u s h

(b )

(c)

Fig. 4.23. Effect o f brush shift on the resultant flux density waveform .

S c an n ed by C a m S c a n n e r
Art. 4.7]

or backward shift in case o f a motor. This reduction in ,


terminal voltage or an increase in the motor speed. CaUSSS * ease in the generator
If the brushes are given a backward shia in a generator or forward shia in a motor the flux

speed'faHs m° re “ * ‘ generat° r terminal ™ltaSe risas

Prior to the invention of interpoles, brush shia was carried out to improve the commuta-
t,on-forward in a generator and backward in a motor. Now-a-days brush shift is never carried
out. However, during the assembly of a dc machine, the brushes may be displaced from the
quadrature axis unknowingly. Loose brushes in the holders or non-uniform brush pressure may
also shift the brush axis from q-axis. Then the question arises as to how to detect whether the
brushes are placed correctly along the quadrature axis or not. For this purpose, the following
procedure may be adopted.
Run the machine at rated speed as a dc generator, first in one direction and then in the
opposite direction. For the same field and armature currents, if the terminal voltages for both
the directions of rotation are the same, then the brushes are placed correctly along the quadra­
ture axis.
Alternatively, run the machine as a dc motor, first in one direction and then in opposite
direction. For the same field and armature currents, if the rotor speed turns out to be the same
for both directions of rotation, then the brushes are placed correctly along the quadrature axis.
If the brushes get shifted inadvertently from the quadrature axis, then the terminal vol­
tages in case of generator or speeds in case of motor, for both the directions of rotation, would
not be equal.
Example 4.9. A 6-pole, 148 A dc shunt generator has 480 conductors and is wave-wound.
Its field current is 2A. Find the demagnetizing and crcss-magnetizing ampere turns per pole at
full load if
(а) brushes are on GNA,
(б) brushes are shifted from GNA by 5° electrical,
(c) brushes are shifted from GNA by 5° mechanical.
Solution. Here a = 2, P = 6, Ia = l i + If = 148 + 2 = 150 A, Z = 480.
(a) With brushes on GNA, 0 = 0, therefore demagnetizing ATs/pole = 0.
The entire armature reaction is cross-magnetizing in nature.

Cross-magnetizing ATs/pole = * ^ 6 = 3° ° ° ^ Ts/po^e

(b ) Here 0 = 5° electrical.
. . / i 2 -8 jg jg .
From. Eqn. (4.13), demagnetizing ATs/pole - fl . 2p

_ 2 x 3000 = 166.67 ATs/pole


180
Cross-magnetizing ATs/pole = 3000 - 166.67 = 2833.33 ATs/pole.
P . e = f x 5 = 15” electrical
(c) Here 0«fcc= 2 ' 0m*c/p 2

. . ._ . | _ 2 x i 5 x 3000 = 500 ATs/pole


Demagnetizing ATs/pole - jgo

-magnetizing ATs/pole = 3000 - 600 = 2500 ATs/pole.


Cross

S c an n ed by C a m S c a n n e r
390 Electrical Machinery [A rt. 4.8

4.8. Commutation Process


IA
Just before the armature coil reaches the brush, it carries current in one direction.

Soon after the armature coil has traversed the brush width, the coil current gets reversed to
(~Ia/a). This reversal of current in the armature coil by means of brush and commutator bars,
is called commutation process. Good commutation means no sparking at the brushes and with
commutator surface remaining uneffected during continuous operation of the d.c. machine. A
machine is said >to have^poor commutation if there is sparking at the brushes and the com­
mutator surface gets damaged during continuous operation of the machine.
The poor commutation may be caused by mechanical or electrical conditions. The mechani­
cal conditions include uneven commutator surface, non-uniform brush pressure, vibration of
the brushes in the holders etc. The electrical conditions include an increase in the voltage be­
tween commutator segments, an increase in the current density at the trailing edge of the
brush etc.
In order to have physical concepts of the commutation process, the d.c. machine coils and
commutator bars are represented as shown in Fig. 4.24. Here the two ends of the coil are con­
nected to adjacent bars (lap-connected winding is assumed). Attention will be focused on the
reversal of current in coil 1, whose two coil-ends are connected to bars 1 and 2. For simplicity,
it is assumed that
(i ) the brush width is equal to the bars width and
(ii) the mica insulation between the bars is of negligible thickness.

In Fig. 4.24 (a), the brush is fully on bar 1 and coil 1 carries current Ic = from L (left) to

R (right). The brush delivers a current 2Ic. The direction of armature rotation is taken from left
to right.
Ir 1,

I u 2 lc
-1 3 12 l b —► R otation
-R o ta tio n
L e a d in g ■T r a i l i n g
b ru sh edge T b ru s h edge
(2Ic-L2)
2lc

(a)

Jx JcUc^tJJc Ic Ic *C Ic
ran

y
(W )
■c
i
r l
rBTYOTYCjTYJ^ rW Y r o w s

__ L J
!lc | 1
2
n
J_1____ t
m
>21 z
(C) (e)
Fig. 4.24. Pertaining to the illustration of commutation process.

As soon as the brush makes contact with bar 2, coil 1 gets short-circuited and current in it
starts decreasing from Ic. The current from bar 2 to brush is, say i2 and, therefore, the current
in coil 1 is Ic - i2 from L to R. Bar 1 delivers 2Ic - 12 to the brush so that the output current is
again 27C, Fig. 4.24 (6). If the area of copper-carbon contacts decide the distribution of current,

S c an n ed by C a m S c a n n e r
Art. 4.8] D.C. Machines 391

then i2 would increase and 27c - i2 would decrease linearly. When brush makes equal areas of
contacts with bars 1 and 2, each bar delivers Ic to the brush and coil 1 carries no current, Fig.
4 24 (c). With further rotation of the armature and bars, area of contact between bar 1 and
brush, starts decreasing and therefore the current delivered by bar 1 decreases from Ic to say
1 Now the coil 1 carries current Ic — from R to L, Fig 4.24 (d) and the current carried by bar
2 is {21c - i\) so that output current is again 21c as before. When brush breaks contact with bar
1 and is fully on bar 2, the short circuit of coil 1 is over and it carries current Ic from R to L as
shown in Fig. 4.24 (e). The time required by the coil current to change from + ICto - Ic, is called
the commutation period Tc. In other words, the commutation period may be defined as the time
measured from the instant the brush is fully on bar 1 , to the instant the brush is fully on bar 2 .
It can be computed from the relation,
^ ___________ Brush width_________
c Commutator peripheral speed
The nature of current flowing in the local circuit of the coil being commutated, depends on
the following factors :
(а) Resistance of the copper-carbon contacts.
(б) Resistance of the coil being commutated.
(c) e.m.fs. induced in the commutated or short circuited coil, due to its—
(i) self-inductance and
(ii) mutual inductance with other coils undergoing commutation simultaneously. Note that
the e.m.f. due to mutual inductance effect is present only when the brush width is more than
one bar width.
(d) e.m.f. induced in the coil due to its rotation in the armature cross flux.
Resistance commutation. In the present section, the effect of e.m.fs. in<Ju£ed in thei com­
mutated coil is ignored-however their effect is discussed qualitatively at a later' ^age. For
time being, the effect of armature coil resistance and brush contact resistance is only taken into
account. For studying the effect of these resistances on the commutation process, refer to Fig.
4.24 (6) and let
Rc = coil resistance
= resistance between bar 1 and brush
and r2 = resistance between bar 2 and brush.

With no e.m.fs. induced in the commutated coU as


law, for the local circuit consisting of brush, bar 1 , coi g
from Fig. 4.24 (6 ), is
{21c - i2)r i + {Ic - h)Rc ~ h r2 = 0
Rc + 2r,
or
In coil 1 the current is given by
" i?, + 2r,
ic = Ic - *2 _ Ic Rc + r x + r 2

r2- r x
7: or ic = Ic

I
S c an n ed by C a m S c a n n e r
392 Electrical Machinery

rx + r2 - 2rx
= /c
r x + r2 + Rc

2rx
1-
or rl + ' 2
W e
1+
rl + r2
^ If coil resistance Rc is small as compared with the copper-carbon resistances r x and r th

7X+ r2 may be neglected. With this, the coil current, from Eq. (4.15), is given by

2rl
Jc — Ic 1-
r 1 +Tn
I f A , and A 2 are the areas, between bar 1 and brush, and between bar 2 and b r u s h
tively, then smce resistance is inversaliy proportional to area, we get coil c u Z t ™

ic —Ic 1 - 2i =L 1-
2Ac
At +A<j ...(4.16)
Aj A2

^ W ith the rotation of the commutator to the right, area A , decreases and area A 2 increases

Thus, at time t = 0, the brush is fully on bar 1,


A 2 = 0 and ic = / c.
At time Tc
t - 4 >A 1 = 3A2,
••• *c = 2/c •
At time T
t = ~2> = A2 ; ■•• ic = 0.
At time f=
4 ’ 3A i = A 2, Ic = - I / C
At time t = Tc, brush is fully on bar 2,
Linear commutotion
A j = 0 and *c = - I c. — r
It is also obvious from Fig. 4.24 (e) that att = T , i e just J T l Resistance
after the commutation of coil 1 is over, i£ = - / . '
The plot of coil current variation with ft 7 is sh™ — 1— — i__.
in Fig. 4.25. Under such a condition, there vrilfbeno f 7 533
mg at the brush and the commutat on ^ ^ 7 ? ^

t =0 t«Tc
1• _. ,
(4.15) and ^ “ “ * " egleCte<i' th6" E« S Fig. 4.25. Pertaining to commutation.

2A 2
1-
Aj + A 2
W e
7 7 a :
r, + r2

S c an n ed by C a m S c a n n e r
— - ................................

Art- f j j D.C. Machines 393

At time t = 0, brush is fully on bar 1, /. ic = Ic as before.


T 1/ 2 I
At time t = — , A^ = 3A 2, ic = — i.e. ic < ~
4 i +
ri + r2
. TC A A 03 _
At time t - 0 ; A t - A 2, ic - - — , i.e. tc = 0 as before.
R,' ‘C
1+
rl + r2

.l r
3T * 2 I
At time t = —f L, 3 A i = A 2, ic = - — , i.e. t, < -7 7
4 i+
St 2
rl + ^2
and at time t = Tc, brush is fully on bar 2, .-. ic = - l c as before.
The variation of coil current ic with Rc included, is shown in Fig. 4.25. Such type of com­
mutation is referred to as the resistance commutation.
In fractional kW d.c. machines, resistance commutation provides good commutation and
this is achieved by using carbon brushes so that
(i) copper-carbon resistance is larger as compared with the coil resistance and
(ii) the brush contact drop is larger as compared with the e.m.fs. induced in the commutated
coil.
The effect of various e.m.fs. induced in the commutated coil is examined qualitatively in the
following lines. { T\
1(J
Delayed commutation. In the commutated coil, the current changes from + Ic to - Ic in
a
commutation period Tc . During this current change in small intend of time Tc (2 m-sec or less), an
e.m.f. ec is induced in the coil due to its self-inductance Lc and its magnitude is gi\ en by
dic
6c = L c ~dt
Usually the brush covers more than one bar and the commu a e c y ..
duced in it due to mutual flux produced by the neighbouring coi s. * nPiahhourine coils is
one due to self-flux of the coil and the other due to the mutual flux of the neigh g ,
called reactance voltage.
AccordinE to Lenz's law the effect opposes the cause. Here the effect is the reactance voltage
According to Lenz si law ,.theejT W voltage opposes the reversal of current
and cause is the reversal 01 current, inus me , rirmiipH mil is
in the commutated coil. As a result of it, the revers 0 would attain by the linear
delayed and consequently it lags in time, the values of current it would attain by the linear
^ , A. A l a coil current is zero with linear commutation, but be-
commutation. For example at t - 9 , the con cu
& ji
a. 'ii up 7Pr0 after t > as shown in Fig. 4.26 (b).
cause of reactance voltage, the coil current 2
„ , j ^rr,mutntion or delayed commutation. I f the current
This type of commutation is called under breaks contact with bar 1, then
in the short-circuited coil has t0 brush is broken which appears in the
(t) the coil current is (Ic 1) an (u cur in ^ 4 26 (a). Thus, the effect of reactance
form of arc at the trailing brush tip a j. brush tip and temperature rise of the corn-
voltage is to cause sparking, heating ot the ira b

mutator.

S c an n ed by C a m S c a n n e r
394 Electrical Machinery
lArt 4.8 \'
Current through
commutated coil 1
o
s
^Linear commutation

, ^ D e la y e d commutation

R o ta tio n

i a p p e a rs
a s a rc
^ r a ilin g b ru s h u p

ia p p e a rs
Qs Ore
(a) (b)
Fig. 4.26. P erta in in g to d ela y ed com m u ta tion .

Good commutation means linear commutation as shown in Figs. 4.25 and 4 26 (6) Two
lmg.t0 Pr° mote good commutation are (i) resistance commutation and (ii) voltage
m“ n- r ’ reS1StaTCe COmmutation always lends a marginal support to voltage com­
utation so as to secure good commutation. These two methods are now discussed next.

pras^ce* otfarn(. ^ ^ 0 m m utatiffln *Qualitative discussion). It is seen from above that the
tion. Therefore 'fb"'slcuri™ C° u" dcrgoin6 commutation leads to delayed commute-
value as possible It ha* h f commutation, reactance voltage must be kept to as low a
proportional to fho * 66n oun. t^ie magnitude of reactance voltage is approximately
0011 Pr ^ w in d in g and squire of the number of
by using (f) sm ^Uenrth „f reactan<* ™ '‘ ago >" the commutated coil can be minimised
t u r e c o . l s a n d ^ l S ° arma*ure“ re b>' resort.ng to multipolar design (ii) chorded-arma-

of securinggood c o m m u t^ E j^ ^

the r : ! ! age C° m T i a tr ? WU1 be S6en fr0m Fig' 4 1 7 that the armature reaction shifts
m u ta te d ' rOSSingof ^ ux density wave from C, C to D, D \ Thus, as the coil is being com­
m on ^ ^ ^ flUX Which haS the polarity of the
main pole left behind for a generator. The coil has, therefore, rotational e.m.f. generated in it
and must be indicated by dot as per the right-hand rule of emf generation. This rotational e m f
tries to m aintain the current in the same old direction i.e dot and consequently opposes the
reversal of current just like the reactance voltage. For a motor the commutated coil is cutting
the flux which has the polarity of the main pole ahead. According to the right hand rule for
e.m.f. generation, the rotational e.m.f. generated in the commutated coil at C, C is opposite
(cross) to w hat it is (dot) under the m ain pole ahead. Thus, for both generator and motor, the
rotational e.m.f. generated in the commutated coil always opposes reversal of current just like
the reactance voltage. In order to cancel the effect of rotational voltage, the flux density in the
interpolar zone should be reduced to zero. This is achieved by using interpoles, in between the
m ain poles. A ctually the flux created by the interpoles does two things, namely it (i) neutralises
the arm ature cross flux and in addition, (ii) produces some flux in the interpolar zone. This
additional flux produces rotational voltage in the commutated coil in such a direction as to
n u llify the effect of reactance voltage. This method of achieving good commutation w ith the help

S c an n ed by C a m S c a n n e r
Art. 4.8]
D.C. Machines 395
of mterpoles is callod voltage commutation All m i 1
sizes, attain good commutation by means o f interposes"1 C0Tnmutat°r machines oflarger
4.8.1. Interpoles. In Pig 4.17 if thp h m e W „ . . .
generator (or against the direction of rotation for a mnTnrW *11 direc^ on ,of rotation for a
mutated coil comes under the influence of zero flux Hpn t ’ t° p01nts D ' ! then th e com-
e.m.f. induced in the commutated T of “ s SWft' the r° tati° nal
be improved. If the brush shift is more than the 6 to fig 4 ,7 (A’ e “ m.™utat,on 7 " “
coii under the brushes would be cutting the flux X p ^ l t Z d ^ e
!
SeHmpraovedr 3 " " coil and commutatfonls fu t

The disadvantages of this method are that (i) for improving the commutation, the brushes
will have to be shifted for every change in load, because angle 0 in Fig. 4.17 (/) depends on the
armature (or load) current and (») for larger shift of the brushes in the direction of rotation for
a generator and against the direction of rotation for a motor, the demagnetizing ampere-tums
increase and the main flux is reduced considerably (see Art. 4.7). This reduced flux may jeo­
pardise the operation of the generator or motor to a noticeable extent.
This method of shifting the brushes for improving the commutation was employed before
the invention of interpoles.
The interpoles are narrow poles placed exactly midway between the main poles. The inter­
poles are fitted to the yoke and are also known as commutating poles or compoles. In order to
make the flux density zero under the brush at C in Fig. 4.17 (/), the interpole must have a north
polarity. At C \ the interpole must have south polarity. From this, it may be concluded that for
a generator, the polarity o f the interpole must be the same as that of the main pole ahead of it,
in the direction o f rotation. For a motor, the polarity o f the interpole must be the same as that of
the main pole behind it.
In practice, the interpoles of appropriate polarity are strengthened so that in interpolar
zone ; the armature cross flux is neutralized and in addition some flux is produced there. This
additional flux in the interpolar zone induces rotational e.m.f. in the commutated coil in such a
direction as to oppose the reactance voltage. If this rotational e.m.f. due to the additional inter-
polar-flux is equal and opposite to the reactance e.m.f., then the resultant e.m.f. in the com­
mutated coil would be zero and therefore zero current in
that coil would amount to sparkless commutation. This is ~j i^~ Commutating
the reason why interpoles are designed to provide more
m.m.f. than the armature m.m.f. in the commutating zone. (>
In practice, the interpole m.m.f. may be 1.2 to 1.3 times the
armature m.m.f. per pole.
If the armature current increases, the armature reac­
tion and, therefore, the rotational and reactance e.m.fs. in
the commutated coil increase. In order to enable the inter- \ /■------o
poles to do their duty faithfully with the variation of arma­
ture current; the interpole winding is connected in series
with the armature. Fig 4.27 shows the series connection of F ig. 4 .2 7 . In terp oles for d.c. m a ch in e,

armature and compoles. Note from this figure also that the
interpole m.m.f. is opposite to the armature m.m.f. in the commu a mg zone.
Resultant flux density waveform is shown by solid curve in Fig. 4.17 (fl. When interpoles of
proper polarity are fitted to the yoke, the resultant flux dens, y waveform,, ; as>
Fig 4.28. An examination of this figure reveals that the, commutation, ofthe cod now takes place
in the field, the polarity of which is the same as that ofthe main field ahead for a generator. In

>canned by C a m S c a n n e r"
[A rt. 4.8
396 Electrical Machinery

main poles ahead. This


the commutated coil and thus the commutation is improved in a dc generate .
In case of d.c. motor, the commutation of the coil takes place in the field, the polarity of
which is o p p o s i t e to the main poles ahead. In view of this, rotational e.m.f. induced in the
commutated coil by generator action (apply right hand rule for generator action) is of the same
sign (dot) as it would be under the main poles ahead. The current due to this rotational e.m.f
is^lso of the same sign (dot) as it is under the main poles ahead. This helps in the reversal of
the current in the commutated coil and this improves the commutation in a d.c. motor.
In order to avoid saturation in the interpoles, air-gaps under these poles are kept longer
than under the main poles. All modern d.c. machines fitted with interpoles, can operate over
wide range without sparking. It has, therefore, become a common practice to install the mter-
poies in modern d.c. machines except in very small d.c. machines. The number of interpoles is
equal to the number of main poles but for low-power d.c. machines, say less than 3 or 4 kW,
number of interpoles may be half of the number of main poles.
4.8.2. B rushes. The commutation process is effected considerably by the type of brush
material. The various types of brushes are carbon, electrographite, copper graphite etc. C a r b o n
brushes are used for very small d.c. machines, electrographite brushes are used more f r e q u e n t ­
ly in all d.c. machines and copper-graphite brushes are used in low voltage (up to 30 V) heavy
current d.c. machines, as used for electroplating.
Carbon, electrographite and copper-graphite brushes are self-lubricating. In view of this,
no lubrication should be applied to the commutator surface.
It has been investigated that brush contact resistance is non-linear and depends on many
factors, namely brush and commutator materials, current density, brush pressure, commutator
speed, current direction, temperature, moisture in the air and atmospheric pressure. In vievv o
this, it is quite difficult to analyse correctly the electrical behaviour of the commutated coi,
since the brush contact resistance does not remain constant.

cann ed by C a m S c a n n e r
Art. 4.8]
D.C. Machines 397

£ * The mica between the


good contact between the segments nnH t t otherwise high mica insulation will not allow
! \
surface is turned down in a lathe the mir^i spar^lnS 'v‘11 occur When the commutator
commutator surface. insulation should be undercut to a level below the

CJ : £ 2 ? r n h ^ 7 ;X l l T n 2 Z i i T T I tted interpotes is cum ulalM >


motor. I,f shunt /field is wronger
stronger man
than the
the series
J r i f field,
rTri then
/u answer ^the
f *? U° W rUn ° S comPound
following:
(a) Is the motor differentially or cumulatively compounded ?
ib) Do the interpoles have proper polarity for good ?

ic) Is the direction o f rotation the same in which it was driven as a generator ?

^ S “ )a^ S UlatiT®ly comP°unded generator, the directions of


the supply terminals unchanged, the direc­
tions of currents, when running as a motor,
are as shown in Fig. 4.29 (6). Since the
series field opposes the shunt field in Fig.
4.29 (6), it becomes a differentially com­
pounded motor.

(b) For cumulatively compounded gene­


rator, the directions of currents in the ar­
mature and interpole windings are as
shown in Fig 4.29 (a). When running as a
motor, the directions of currents, both in
the armature and interpole windings, get
reversed. Thus the interpoles have proper
polarity for good commutation.
(c) For the direction of current in the ar­
mature and main N, S poles as shown in
(C) (d)
Fig. 4.29 (c) for a generator, the armature
Fig. 4.29. Pertaining to Example 4.10.
must be driven in anticlockwise direction
Since the shunt field is stronger than the series field, the main poles polarity remains un­
changed for a motor. The direction of armature current, however, gets reversed for a motor as
shown in Fig. 4.29 (d ). Now the main N, S poles must attract S, N poles created on the arma­
ture. The rotation is thus anticlockwise, the same in which it was driven as a generator.
Exam ple 4.11. A 4-pole, 100 kW, 200 V lap-connected d.c. machine has 256 conductors.
Find the number o f turns required on each interpole, i f interpolar air-gap is 1 cm and interpolar
flux density is 0.25 T. Neglect the effect o f iron parts o f the circuit and o f the leakoge.
Solu tion. The interpolar m.m.f. per pole Fcp should be equal to the combination of arma­
ture m.m.f. per pole Fa and the air-gap m.m.f. In view of this,

B cp
Sep
p0
where B cp and g cp are the interpolar flux density and air-gap length respectively.
100,000
Full load current
° 200 ~
398 Electrical Machinery (A rt 4.9

B cpr
F =
2a P ' t o ' 8 "
cp

r 500 x 256 0.25 -2


or I IV —-----------------“I" x 1 x 10
- 7
“ e" 2 (4) (4) 4 n x 10
= 4000 + 1989 = 5989
5989
or N CP- = 12 turns per pole.
500
Thus the turns on each interpole should be equal to 12.
N ote. Since the interpoles are designed to provide more m.m.f. than the combination of
armature and air-gap m.m.fs., the number of turns on each interpole should be more than 12.

4.9. C om pen satin g W indings


The interpole winding m.m.f. is effective only in the commutating zone. In other words, the
armature reaction effects are overcome only under the interpoles and the resultant flux density
under the pole faces remains dis­
torted as illustrated in Fig. 4.28.
If a d.c. machine is subjected to
heavy overloads or if it operates with
a weak main field (as in a motor in­
tended for wide speed control), then
the resultant field waveform is ex­
tremely distorted, as shown in Fig.
4.30. Any coil, whose two coil-sides
are under the influence of these peak
flux densities will have a large rota­
tional e.m.f. induced in it. In Fig.
4.30, at any instant, the coil sides a
and a' are shown under the peak flux
densities. If the rotational or speed
voltage induced in coil aa' exceeds
Fig. 4.30. Extremely distorted flux density waveform without
about 30 V (maximum permissible compensating winding and its effect on coil, a, a'.
limit is of the order of 30 to 40 V),
the air between the adjacent commutator segments to which the coil aa' is connected, may
breakdown resulting in an arc. Such an arc may then extend to nearby commutator segments and
eventually spread rapidly around the entire commutator periphery. This is because the air near the
commutator surface is fairly ionised to result in conditions favourable for flashover. Such a flashover
is detrimental both to the commutator and the supply line, since the latter is directly short-circuited
by the flashover.
There is another factor which may cause voltage between adjacent segments to exceed 30
V (30 to 40 V). This is associated with the time-variation of armature flux caused by sudden rise
or fall of the load current. For understanding this, consider a d.c. machine subjected to rapidly
changing loads of wide range. Let us first investigate its operation as a d.c. generator. The
distribution of armature reaction flux or cross flux alone, is depicted in Fig. 4.17 (c) and also in
Fig. 4.17 (d). It may be seen that a coil under the pole centres has maximum cross flux a r o u n d
it and consequently has maximum cross flux linkages. Therefore, the effect of rapidly changing
flux caused by rapidly changing loads, is more pronounced on the coil under the pole centres
and naturally the attention will be focused on this very coil.

S c an n ed by C a m S c a n n e r
Art. 4.9]

m , j , ~ D.C. M achines 399

grows. This — *. « ^ value, the cross flux

:
the pole centres is reduced if the load is suddenly i n c r p S the resultant e.m.f. in the coil u nc„
the generator falls suddenly from a high to a low value thp gf erator- th^ load on
decay. This flux decay is opposed by an e m f indu rpH1’, It armf ture reaction flux tends to
show that this e.m.f. under S-pole must be indicated by a dot i t htt]e, consideration will
the rotational e.m.f. already existing. Therefore, the L u lta n t'e ! T ™ *****
centres is increased if the load on the generator is snHHani ^ £ ln Under the Pole
in the coil exceeds about 30 V, the arc across the air near the m k T T ° f 11 COmbined e m f -
commutator bars (to which the coil is c o n n e c t e d
positive and negative brushes. resulting in flashover between

The reader may use the same thought process and mm* tp i • ,
occur if the load is suddenly increased from a low to a
The trouble of flashover which is due to the distortinn nf fi,,v j ™ ^ ,
shoes or due to the rapidly changing loads, can be effectively overcome by n e X a h z W o rc o m 6
pensating the armature m.m.f. under the pole faces. The besi way to achi
0 P ! Wlndmg or comPensating winding embedded in slots in the pole faces’
, The armature and compensating windings are connected in series so that thmv c
proportionu, to the same current. Furthef, in order Z
^rection of currents in the compensating winding must be opposite to that in the armature
winding just below the pole faces as shown in Fie 4 3 1 H a l f n f f h n ™ a- •

7 T W side o f a pole are


the left hand side of the adjoining pole, Fig. 4.31. Consequently the pole-face winding or the
compensating winding is a concentric type winding.

v ia) ! . . . (fc)
Fig. 4.31. Compensating windings embedded in slots in the pole faces.

Fig. 4.31 (6) shows the physical arrangement of connections of armature circuit and com­
pensating winding (CW). In this figure, top brush is connected to back end of the CW conductor
1 in iV-pole face. The front end of this conductor 1 is connected to front end of CW conductor 1'
in S-pole face. The back end of 1' is connected to back end of 2 in AT-pole face and so on till
conductor 4' is reached.

* Note that this is transformer e.m.f.

fe m n e c T b y C am S canner
400 Electrical Machinery

Fig. 4.32 illustrates how the com­


pensating winding m.m.f. neutralizes
the armature m .m .f. under the pole
faces. In the interpolar region, the com­
— I
t a a ft a «~£l i fQ Q O a
¥
pensating winding m.m.f. and interpole
winding mmf, both oppose armature
mmf. The resultant of these three
m.m.fs. in the commutating zone should
not be zero, from the view point of can­
celling the reactance e.m.f. Under the winding*
pole faces, compensating winding m.m.f. mrnf- * f il
neutralizes the armature winding
m.m.f. Therefore the resultant flux den­
sity waveform under the pole faces is
seen to be independent of load current
and is of the same waveform as
produced by the main field alone. This
means that the field windings have full
control over the air-gap flux. Since the
compensating winding neutralizes the
armature m.m.f. under the main poles,
the armature flux linkages are reduced.
This has the effect of reducing the arma­
ture inductance. Thus the armature-cir-
R e s u l t a n t flux
cuit time constant is reduced and this d e n s ity w a v e fo r m
results in quicker dynamic response of
Fig. 4.32. Resultant flux density waveform
the d.c. machine.
with compensating winding and interpoles.

Since the compensating winding m.m.f. neutralizes the armature m.m.f. only under the
main pole-faces, the compensating winding m.m.f. per pole A T Cis given by
Pole arc
ATc = Pole pitch X Armature m m f- Per P°le
_ Pole arc Z
Pole pitch 2 aP ...(4.17 a)

As compensating winding carries Ia, the compensating winding conductors Z can be cal-
CIV
culated as under:
7 j
**c w Aa Pole arc Z
ATC= x I Interpole
2 Pole pitch 2aP winding
or 2 _ Pole arc Z
cw ~ Pole pitch X aP ...(4.17 6)
winding
A schematic diagram of a d.c. compound machine fitted with
interpole
,------------------ TVM,UU„„16 iruiuiugo
and compensating windings ia
is m
illustrated
u s t r a t e a iin
n rFig. 4.33. The
i g . 4.0,3. The
f A r m .) -
flux produced
flux produced by by the intemole and
the interpole and compensating
rnmnpncatir1r,m ,v j;
windings _ i___
is along Shunt
the brush, armature or quadrature axis, these are therefore shown winding w inding
along the same axis. Shunt and series windings produce flux along o__________
the direct axis, these are also shown accordingly 90° awav from r.- r
bru sh axis. Fig. 4.33. Schematic diagram of a
_ d.c. com pound machine fitted
The compensating winding increases the cost of d.c. machines wit^ interP°*e anc* compensating
these are therefore used in d.c. machines subjected to heavy over- windings‘

o c a m ie u uy uam ocaiuifc;!
A n . 4.9) _________

bads or rapidly changing loads (e.g. steel m i l l ¥ ¥ r ~ l 7 ~ M‘‘C>" lleS


shunt held control. } r m motors intended for wide speed range by
summarizing, the methods of improving mmm * *. •
(a) By increasing the brush contact resistance Thi ^ m d C machmes are as follows-
bar contacts as compared with the reactance voltaee andTvf S voltaSe droP at the brush-
method called resistance commutation is employed in c,0^ mutati°n is improved. This
<M By shifting the brushes, forward in a gen erlJ” , w l a maChin6S'
was employed for commutation improvement before the inve t * m° tor’ This method
(c) By providing interpoles. This method is e m D l o v e d V n ° f 1” terpoles-
except in very small (less than about 0.3 or 0.4 kW) d c machines ° f d'C’ machines>
id) By providing compensating winding. Since the d r
ing are more expensive, these are used in those d.c. machines onlv whidl h C°™pensating wind‘
duty cycles in industry. y which have to perform severe
Example 4.12. A commutator with a diam<>ter o f Sf) rm
width o f 1.5 cm, find the time o f commutation. ° rpm' For a brush
Solution. Peripheral velocity of commutator,

v = nDn = n x 50 x cm/s
60
But v x time of commutation = brush width

.-. Time of commutation = — — x 60— = n *573


rc x 50 x 1000 M
F , J Z T ) e 4'\3' A single1 urn coil has an inductance o f 0.02 mH in the commutatine zone

®0lUt' ° n- F° r Stra'ghtHne or linear commutation, the transformer emf E, in the coil un-
ina con C°mmU lon raust be neutralized by the speed, or rotational, em f£, in the commutat-
ing coil.

Transformer emf in coil, E, = L —


1 dt
Here change of current during commutation

=—x 2= x 2 = 60 A.
a 4
Refer to Fig. 4.4. Let dt be denoted by
t. = time of commutation.

Et = 0.02 x 10~3 ^

The speed or rotational emf in a single-turn coil is given by


Er = 2 (Bau I v)

' If * = brush width, then u = . b™ shwidtb . = *


time of commutation tc

I •' £ r = 2B0„ l f - 2 ^
’ lc cc
'vhere <>c = average value of flux in the commutating zone.

S c an n ed by C a m S c a n n e r
402 Electrical M achinery (Art. 4.10
■v:
For linear commutation, Er = Et

2 - = 0.02 x 10" 3 ^
te ' tc
or <t>c = 0.5 m Wb.
Exam ple 4.14. A 2000 kW, 400 V, 14-pole d.c. machine has a lap wound armature with
1100
j. j. Lw/tuuttu/
conductors.a. The pole arc iu
i ne pute to puie-pucri
pole-pitch ruuu
ratio is u.
0. 7.
t . Compute the number o f pole-face conduc-
tore o f the compensating winding in each pole, so as to obtain uniform air-gap flux density under
the pole faces.
Solution. For a lap-connected armature,
a = P = 14

Full load armature current Ia = = 5000 A


400
Armature ampere conductors per pole
IaZ 5000x 1100
aP 14 x 14
Ampere conductors per pole to be compensated by pole face winding
_ Pole arc 5000 x 1100 (5000) (1100)
Pole pitch X 14 x 14 ~ ■ x 14 * 14
The compensating winding carries the entire armature current of 5000 A.
.’. Compensating winding conductors per pole
0.7 x 1100x 5000
= 4.
14 x 14 x 5000
Exam ple 4.15 . A compensated d.c. machine has 15,000 armature ampere turns per pole
The ratio o f pole arc to pole pitch is 0.68. Interpolar air gap length and flux density are respec­
tively 1 cm and 0.25 teslas. For rated armature current o f 850 A, calculate the compensating
winding conductors per pole and the number o f turns on each interpole.
Solution. From Eq. (4.17), compensating winding ampere turns per pole are
ATC= 0.68 x 15000 = 10,200 ATs.
/. Compensating winding conductors per pole

M.m.f. required for the air gap under the interpole


BcP r 0.25 , 9
^ 4 ji x 1 0 “ 7 X = 1989 ATs.
W ithout compensating winding, the interpole m.m.f. must be equal to 15,000 + 1989 ATs.
! lT r a^ r mug o f T dy ncutrali* « l 10.200 ATs. In view of this, the ampere
turns that must be furnished by each interpolo are 15,000 + 1989 - 10,200 = 6790 = 6800 ATs.
Number of turns on each interpole = - = 8
850 ’
4.10. Basic P erform ance E qu ation s for D .C . M achines
For d.c. machines, the expressions for the arm ature generated e.m.f. E . and electromag­
netic torque „Thave already been derived in Eqs. (3.44) and (4.6) respectively. These expr
sions are re-w ritten below, for convenience.

S c a n n e d oy u a m b c a n n e r
Art. 4.10]
___________ D.C. Machines 403
g _ <j) ZnP
IN. C » t ^ -
° a
and J Te = Ka $ I a
60 ...(3.44)

where o)m= 2 ji n rad/sec. ...(4,6)


V-
r and K -Z L
. a 2tw
In Eqs. (3.44) and (4.6), note that 4 is the total direct axis flux per pole
A careful examination of Fig. 4.14 (a) leads to the electrical equivalent circuit of A - -
pound machine of Fig. 4.34 (a). In this equivalent circuit, armature of the d c machine W h
replaced by generated e.m.f. E . and the armature resistance r_ in series w r t h T l t e thaJ the
armature resistance ra = armature winding resistance + compensating winding resistance if
present + interpole winding resistance, if present + brush contact resistance.
Iq
p----- AVW--- - ------- +0
lL ' h I
vt rf :i vQ a

i>—■ - — -6
( 6)
Fig. 4.34. Equivalent circuits for (a) d.c. compound generator (short-shunt) and
(6) d.c. compound motor (long-shunt).

In the equivalent circuit, Ia is the armature current, 7,-the shunt field current, IL the line
current, Va the armature terminal voltage, Vt the machine terminal voltage, rt the series field
resistance and rythe shunt field resistance.
For d.c compound generator (short-shunt), the voltage and current equations can be writ­
ten by applying Kirchhoffs voltage and current laws to Fig. 4.34 (a). Therefore,
Va = Ea + I a ra ...(4.18)
V t = V a ± I L rs = Ea ± l a ra ± I L rs ...(4.19)
^ h = Ia ± I f ...(4.20)
where plus sign is used for a motor and minus sign for a generator.
For the long-shunt compound machine of Fig. 4.34 (6).
Va = Ea ± I a ra ...(4.21)
v t = Va ± I a rs = Ea ± I a (ra + rs) ...(4.22)
and h =h±If ...... .
...(4.23)
Here also plus sign is used for a motor, minus sign for a generator. From Eqs. (4.18) to (4.23),
neglect those terms which are not required for the specified d.c. machine under consideration. For
example, for a d.c. shunt machine, neglect the terms pertaining to series field winding. For a d.c.
series machine, neglect those terms which are relevant to shunt field winding.
4.10.1. M agnetisation Curve. In addition to basic performance equations given above,
the magnetisation curve is also essential for determining the performance of d.c. machines.
£ 1
The magnetisation curve is the relationship between air-gap flux <}>and the field winding
m.m.f. or field winding current. From Eq. (3.44), it is seen that for constant speed a)m', the
armature generated e.m.f. Ea at no-load, is proportional to air-gap flux 4>only. In view of this, a

S c an n ed by C a m S c a n n e r
[Art. 4.10
404 Electrical M achinery

EC------ 1
! Ea
•G ___ -3
Vt ----------------
no-load /
mag. / >/ Load
curve /
i / / if m.c.for
constant
V la

1
H If or
Ff
(a) (b)
Fig. 4.35. (a) Typical magnetization curve for a d.c. machine
(6) pertaining to the calculation of armature reaction m.m.f.

plot between air-gap flux <j>or Ka <}>o)m i.e. Ea and the field winding current or m.m.f., at a given
speed, gives the magnetisation curve. This curve is also called no-load or open circuit charac­
teristic, or saturation curve. Fig. 4.35 (a) illustrates a typical magnetisation curve at a given
speed, with only one field winding excited. The magnetisation curve for any other speed, can be
obtained from Eq. (3.44) i.e.
-a l
= Ka $
to,, oo'in
, l

) or Eai ——
* O) tOn.1

This is illustrated in Fig. 4.35 (a) for speed ooml < com. For d.c. compound machines, the magnetiza­
...(4.24)

tion curve is obtained with current in the shunt field winding only. During the d.c. compound machine
analysis, the series field m.m.f. must also be taken into account as follows:
If N f are shunt field turns per pole and N s series-field turns per pole, then
total d-axis m.m.f. per pole = N fIf ± N s Is ...(4.25)
Plus sign is used for cumulatively compounded machines and minus sign, for differentially
compounded machines. Since the magnetization curve is given with current in the shunt field
alone, the equivalent shunt field current that would produce the :-ame total m.m.f. given by Eq.
(4.25), can be obtained by dividing Eq. (4.25) by N f .
.*. Equivalent current in the shunt field winding
_ N flf - ■Ns I3
Nf

- L ^ I ...(4.26)
f ~ Nf *
The dashed straight line, tangent to the straight line portion of magnetisation curve at
speed co,,,, is called air-gap line as shown in Fig. 4.35 (a).

4.10.2. Effect of armature m.m.f. on d.c. machine calculations. It has been d e scrib e d
in Art. 4.6 that the cross magnetizing effect of armature reaction produces demagnetizing effect
on the main pole flux. Since this effect is due to saturation, it can’t be included by a simple mathe­
matical expression in the d.c. machine calculations. However, the following method may be used
to include the effect of armature m.m.f. on the performance of d.c. machines.

S c an n ed by C a m S c a n n e r
1
I
fi
Art. 4.10]
D.C. Machines 405

No-load magnetization curve is the graph between armature generated e.m.f. Ea and Held
current/,(or field m.m.f. ,)with constant armature speed ei„. Load-magnetization curve is the
F
I graph between terminal voltage V, and I, (or Fj) with constant armature speed <a„ and constant
armature current fl . n Fig. 4.35 (6), the no-load and load magnetization curves are sketched
in one diagram and are respectively marked 1 and 2. In order to obtain curve 3, add armature
resistance drop Ia ra to each ordinate of curve 2. Here ra includes the brush contact resistance
i
also. For any field excitation equal to OA, AB is the armature generated e.m.f. at no load. For
load current Ia, CD is the voltage drop equal to Ia ra as per the construction. Therefore out of
the total voltage drop BD, voltage drop BC is caused by armature reaction. However, no-load
e.m.f. HG = AC can be obtained by an excitation equal to OH = O A - AH and in this manner the
voltage drop BC due to armature reaction, has been considered in terms of If or F f. In view of
this, the armature reaction demagnetizing m.m.f. ATd is equal to AH and is expressed on the
field current or field m.m.f. scale.
The magnitude of demagnetizing m.m.f. AT* which accounts for the cross-magnetizing ar­
mature reaction, should be calculated at the anticipated terminal voltage, because saturation
effects the value ofATd considerably. However, over the normal operating range of the voltage,
the demagnetizing m.m.f. ATd, accounting for the armature reaction, is taken as proportional
to armature current. With this, the net field m.m.f. acting on each pole along the d-axis is given
by
Net m.m.f. = N flf±N sls -A T d ...(4.27)
The basic equation underlying the analysis of d.c. machines are Eqs. (3.44), (4.6), (4.18) to
(4.23), the magnetization curve and power balance Eqs. (4.9) to (4.12). The armature m.m.f. can
be accounted for as explained above in Fig. 4.35 (b).
Example 4.16. A 250 V, 10 kW d.c. shunt generator has 1400 turns on each pole. At rated
speed, a shunt field current of 2 A produces a no-load voltage of250 V, but at rated load the same
load voltage of250 V can be produced by a field current of 2.2 amp. It is required, not to change
the field current for maintaining load voltage constant, but add a series field winding. Calculate
the number of series field turns per pole required for long-shunt connection.
Solution. Total m.m.f. required at rated load
= 2.2 x 1400 = 3080 ATs.
The m.m.f. at no-load = 2 x 1400 = 2800 ATs.
The m.m.f. to be supplied by series field winding
NJS= 3080 - 2800 = 280 ATs.
But series field current at full load
j lt^ooo _ 40A
5 250
280
Series field turns = 7 turns.
40
. Example 4.17. A 230 V, 10 kW d.c. shunt generator gave the following data for the mag­
netization curve at rated speed o f 1500 r.p.m.
If, amp. 0 0.2 0.4 0.6 1.00 1.40 1.80 2.00

Ea<v°hs 6 40 80 120 194 246 269 274

The resistances o f the shunt field and armature circuit (including brushes) are 184 ohms
and 0.443 ohms respectively.

S c an n ed by C a m S c a n n e r
406 Electrical Machinery [Art. 4.10

At rated load and speed, a field current


o f 1.7 amp. is required to maintain rated ter­
minal voltage. Find the demagnetizing effect
o f arm atu re reaction at rated load and
speed.
Solution. Full load current
J 10.000 , ,
L 230 - 43*4 A -
/. Total armature current at rated load,
Ia = IL + lf = 43.4 + 1.7 = 45.1 A,
Total armature resistance drop
= Ia ra = 4 5 .1x 0 .4 43 = 20 V
For a field current of 1.7 A, point A in
Fig. 4.36, represents rated terminal voltage
of 230 V. This point A lies on the load mag­
netization curve, refer to Fig. 4.36. AB is
made equal to the total armature resistance
drop of 20 V. Through B is drawn a horizon­
tal line meeting the magnetization curve at
C. Then BC is the demagnetizing effect of
armature reaction in terms of shunt field
current.
Demagnetizing effect due to armature
reaction = B C = 0.25 equivalent shunt field
current in amperes.
Exam ple 4.18. A d.c. shunt generator
has a total o f 100 in the shunt circuit, in­
cluding the field winding as well as the regulator. Its terminal voltage is 50 Vwhen the generator
is run at 1000 r.p.m., 225 V a t 2000 r.p.m. and 405 V a t 3000 r.p.m. Draw magnetization curve
at 2000 r.p.m. and hence determine the terminal voltage at this speed, i f the resistance o f the
shunt circuit is reduced to (i) 80 Cl and (ii) 70 Cl.
E
a E
a

oC3TTTOu u_y o d i n o u d i ii it:i


A rt. 4.11]
D.C. Machines 407
, Solution. For a terminal voltage of 50 V at 1000 r
,t 2000
At iuuu r.p.m., the me terminal voltage is 100 V at tho «, .... , i“ !,=“ clu current
current isis 50/100
50/100 == 0.5 A
I i r r o n t n f an*>/1AA — A A C A . S3me field P l i r r n n f Q ; _ : l __ 1 «»
current of 405/100 = 4.05 A, the terminal voltaee of 4P4 v Similarly, for a field
405 X (2000/3000) = 270 V at 2000 r.p.m. The m a a‘ 3000 r p m' is ^ v a l e n t to
drawn. Resistance lines for 80 Q and 70 Q are seen to mppt f k ° n CUrVe at 2000 r P m- is thus
at C and P to give the terminal voltages of BC = 253 V and Q P ^ ^ V ^ 1011 CUrV6 ^ ^ 4 37
4.11. Operating Characteristics of D.C. Generators
The operating characteristics of d.c. generators Dre«sp t u- ^
tween the basic quantities relevent to generator ODeratinn ” gTaPhlcally. the relationship be-
voltage V„ armature current excitation 5C * are term” al
constant and is usually fixed by the prime-mover speed * * * ° f '“ “ geMrator is

below” ' ' mPOrtant CharaCteriSti“ ° f d c- generatOTS • " ^ r in number and these are given

<0 No-load characteristic. with speed n = constant. This characteristic is also


called saturation curve, open-circuit characteristic, magnetization curve or no-load maenetiza
tion curve. »**«Bucu£a-

(it) Load characteristic. V, = f ( l ,)with both 1. and constant. This characteristic is also
called the load magnetization curve.
(tit)External characteristic. V, = f (IL) with both / Aand n constant.
(ia) Armature characteristic or regulation curve. If = f ( l t ) with both V, and n con­
stant.
of these characteristics depends upon the method of excitingthe field windings Note
that in all the dc generator characteristics listed above, speed is held constant by the prime-mover
These characteristics are now described in detail for different types of d.c. generators.
4 .H .I . Separately-excited generators. These generators are used when a wide range
of output voltage is required.

(t) No-load characteristic. This characteristic gives the variation of armature generated
e.m.I. t a with field current If, for zero armature current and constant speed.
The connection diagram for obtaining the no-load characteristic is illustrated in Fig. 4.38
(a). The armature is driven at rated speed by the prime-mover and switch S is kept always
open. It will be seen that even though the field winding is not energised, the voltmeter indicates
a small voltage (2 to 6 volts), due to the presence of residual flux in the main poles. This residual
flux voltage is shown by OA in Fig. 4.38 (6). The field winding is now energised and the exciting
current If is increased in steps—
at each step Ea and If are
Eat
recorded. Field current /.
if is m- /, c
*
creased till Ea is about 1.1 to —T—(AJ------- T" • ■■ / occ
1.25 times the rated voltage. 7
The graph between Ea and lf,
Field
called no-load characteristic or r-TTTn ( Arm. J ® 5 / s p e e d =n
saturation curve, is shown in
Fig. 4.38 (6). If the field current
f /
I OAx re s id u a l
flu x voltagi

is now decreased from OB, E


versus If plot will not follow the
J
source

(a)
---------- . r

■ (b)
U- —

original curve AC, but will lie Pig. 4.38 Separately excited generator (a) connection
above it due to hysteresis. diagram and (6) its no-load characteristic.

S c an n ed by C a m S c a n n e r
408 Electrical Machinery IArt. 4.11

The magnetization curve for low values of 7/is a straight line, because the entire field m.m.f.
is almost spent in forcing the flux through the air gap and the m.m.f. required by the iron is
almost negligible. With increased values of//-and above a certain value of field flux, saturation
sets in and m.m.f. required by the iron increases more rapidly than the flux. The knee of the
no-load characteristic is at the point D, Fig. 4.38 (6). (
(ii) Load characteristic. This characteristic gives the relationship between the terminal
voltage V, and field current If for constant /„ and speed. In order to plot it, refer to Fig. 4.38 (a).
Run the armature at constant rated speed and close the switch S. Adjust the field current till
Ia is equal to rated armature current (or any other specified current) and take the instrument
readings. Vary the load and field currents in such a manner that armature current Ia and speed
n remain constant, but terminal voltage Vt changes. Repeat this process till sufficient number
of points for the graph, Fig. 4.39 (a), are obtained. If the load resistance is made zero (i.e. short
circuit), then Vt = 0 and point a on the load characteristic is obtained. With the help of no-load
and load magnetization curves, demagnetizing effect of armature reaction can be found out, as
explained in Fig. 4.35 (6).

Fig. 4.39. (a) Load characteristic and (6) external characteristic, of a separately excited generator.

(iii ) External characteristic. This characteristic gives the variation of armature terminal
voltage V, with load current IL for constant speed and fixed field current. External or volt-
ampere characteristic can be obtained experimently by using the connection diagram of Fig.
4.38 (a) The generator is run at rated speed and its field winding is excited to give rated ter­
minal voltage at no load. Now close the switch S, vary load resistance in steps and for each step,
note terminal voltage and load current. A typical external characteristic is shown in Fig. 4.39
(6), by curve 1. The decrease in terminal voltage with increase in load is due to the voltage drops
caused by armature reaction and armature circuit resistance. Curve 2 in Fig. 4.39 (6), obtained
by adding l a ra drop to the ordinates of curve 1, is known as internal characteristic. The internal
characteristic gives the variation of armature no-load generated e.m.f. Ea minus the voltage
drop due to armature reaction, with load current IL or Ia. In other words, internal characteristic
gives the relationship between internally generated e.m.f. and the load current for constant
speed and field current. Note that ra includes the brush contact resistance also and for
separately-excited generator Ia = IL. In Fig. 4.39 (b ), OA is the no-load terminal voltage Ea.
The performance of a dc generator is gauged by its voltage regulation. It is defined as the
change in armature terminal voltage, expressed as a percentage of full-load terminal voltage,
when full load is gradually reduced to zero with speed and field current remaining unchanged.
E -V
Percentage voltage regulation = - ~ z —- x 100
*r
where Ea = no-load generated voltage

Vr = rated (or full-load) armature voltage.


Art. 4.11]
D.C. Machines 409
(iv) Armature characteristic or reeulatinn * ,
current If with load current for constant C^ rve.‘ A plot P ving the variation of field
characteristic or regulation c u ™ . ' " al V° ltage and s» Md' is called
At no load, the field current is adjusted to rivp f • i , ^
age of the machine, Fig. 4 38 (a). The load current is now in“ e ^ ° i n
steps and at each step, the field current is adjusted to keep the ter-
mmal voltage constant. The regulation curve is then plotted as shown
m Fig. 4.40. In the beginning, the curve is almost a straight line and
as the saturation sets in, it bends upwards.
This characteristic can be employed for calculating the number of series
turns for any degree of compounding, in dc compound machines. For ex­ A IL
ample, for level compounding, the additional shunt field ampere turns Fig. 4.40. A rm a tu re
(CB) x Nf must be furnished by series field armature turns (OA) x N ch a ra cteristic o f a
sep arately ex cited generator.
Series field turns,
N- = m • */ ...(4 .2 8 )

4.11.2. Shunt generators. These generators are most frequently employed, because no
separate source for excitation is required. However the load current must be well below the
maximum current for avoiding large dips in the terminal voltage.
(i) No load characteristic. This characteristic for the self-excited shunt generator, can be
plotted with the help of connection diagram of Fig. 4.41 (a) and by keeping the switch S open.
If the field winding circuit of the shunt generator is disconnected from the armature circuit and
separately excited, then the no-load characteristic with separate excitation will not differ from
that obtained with shunt excitation. This is due to the fact that small amount of current (1 to
3% of rated current) flowing in the armature of the shunt generator, has negligible effect on the
main flux. It will rather be more convenient to run the shunt generator as a separately-excited
generator for obtaining its saturation curve. The saturation or magnetization curve so obtained
can be used for shunt generator analysis without any appreciable error.
(ii) Load characteristic. The load characteristic can be obtained in the same manner as
for the separately-excited generator. Actually the load characteristics obtained both for
separate and shunt excitation, are the same. The slight difference is due to the different arma­
ture currents, Ia = IL + If for shunt generator and Ia = l i for separately-excited generator. Dif­
ferent armature currents result in different armature reaction, giving slightly different voltage
drops in the two cases.

vt
1 ____________ G
A -• — I f ^ Internal
. charoct
<§Mf‘ 1 r \ !
E x te rn a l _i_ \ * X 11
cha ra ct. J
i ]B
Load
Pf< CD* lara drop A
O F -A R drop / i
/ i
FG * Drop due to i f i
to ll i n l o - — ^ 1 i
i
— i _____ i —
lm
(6 )
^ . . / v d ia g ra m an d (6) e x te rn a l c h a ra cte ris tic.
F ig 4 4 1 . S e lf-e x c it e d s h u n t g e n e r a t o r (a ) c o n n e ctio n a ia g ra v >

The generator is run at rated speed and the neia tun biik j »

S c an n ed by C a m S c a n n e r
410 Electrical Machinery [Art. 4.11

any other suitable voltage) at no-load. Switch S is closed, the load is gradually increased in
steps and the instrument readings at each step are recorded. A plot of the terminal voltage Vt
and load current IL, with preset value of field current and speed, gives the external charac­
teristic curve 1 in Fig. 4.41 (6). The drop in terminal voltage is due to (a) armature resistance
drop Ia ra (b ) reduction in main flux due to armature reaction and (c) further reduction in field
current lf, since the terminal voltage has fallen because of the first two effects given in parts (a)
and (6). A reduction in field current causes the flux and therefore the generated e.m.f. to
decrease. Thus the terminal voltage of the generator for a given load current, will be lower
when shunt excited than when separately excited, provided no load voltage is same in both the
cases. In Fig. 4.41 (6), CD = I ara = voltage drop due to armature resistance, D F = voltage drop
due to armature reaction and FG = voltage drop due to reduction in field current. The total
voltage drop, from no-load generated voltage OA, is given by GC and consequently the terminal
voltage at full load is OA minus GC.
As the load resistance is decreased (load current increased), the terminal voltage drops
until point B is reached. If load resistance is further decreased, the load current increases
momentarily. This momentary increase in load current, produces more armature reaction thus
causing a reduction in the terminal voltage and field current. The net reduction in terminal
voltage is so large that the load current decreases and the external characteristic turns back.
In case the machine is short circuited, the curve terminates at point H. Here OH is the load
current due to the voltage generated by the residual flux.
Over the normal operating range, the internal characteristic given by curve 2, can be ob­
tained by adding Ia ra drop to the ordinates of external characteristic, i.e. curve 1.
(iv) Arm ature characteristic. Shunt generator armature characteristic is obtained by
running it as a separately excited generator. The test is carried out as explained before in the
case of separately excited generator.
Voltage build up in shunt generators. Consider an unloaded shunt generator as il­
lustrated in Fig. 4.42 (a). In Fig. 4.42 (b ), the straight line Oa is the graphical plot of Ohm’s law
for the field circuit. In other words, the slope of the line Oa, drawn through origin, represents
the field resistance, i.e.
Voltage ba
tan ZaOb = = field resistance rf in ohms.
Current Ob

Shunt
f ie ld

Fig. 4.42. Self-ex cited shunt generator (a) connection diagram


(6) voltage build-up process and (c) critical field resistance and critical speed.

'.M en ifre c r u y u d i i iM d i ii ier


• i

Art. 4.111___________________________________ _ ____________________________________D.C. Machine 411

When the armature is driven at a speed for which the magnetisation curve is given, the
residual pole flux generates a small voltage Oc, with switch S open, i.e. with zero field current
When switch S is closed, residual flux voltage Oc produces a small field current If the flux
produced by this small field current, adds to the residual flux, still larger voltages are
generated. In order to understand this, refer to Fig. 4.42 (6), where residual flux voltage Or is
shown to produce a small field current equal to Od, which in turn raises the generated voltage
to de. This voltage de, raises the field current to Of, which further raises the generated e m f
to fg, now e.m.f. fg raises the field current to Oh, which in turn increases the generated e m f
to hj and so on, till stable point n is reached. Note that stable point n is determined by the
intersection of field resistance line Oa and the magnetisation or saturation-curve Beyond the
ooint n, the generated e.m.f. given by the magnetization curve is less than that required to
maintain the corresponding field current, therefore, point n is the stable point If the voltage
build up from Oc to np fails, owing to small field current opposing the residual flux, then reverse
the shunt field terminals to obtain the voltage build up.
If shunt field resistance is increased to OA (slope of the field resistance line increased), Fig
4 42 (c) then the field resistance line and the magnetization curve intersect at point r/,
fore the voltage will not build up further than point q . If shunt field resistance is such that
renr’esents the field resistance line, then the intersection may be anywhere between points r
and s The generated voltage in such a case, would vary between the voltage xr and ys, resulting
in unstable conditions. Note that field resistance line OB is tangent to the magnetization curve
and the field resistance represented by line OB is called the critical field * a * ven
speed If shunt field resistance is more than the critical field resistance such as i >
OA there will be no voltage build up. In order to calculate the critical fieldresistanceatany
soeed a line is drawn through origin 0 and touching the maximum straight line portion o ^the
magnetization curve. The slope of this line is the critica 'fd d resistance at a speed for which
the m agnetization cu rve has been drawn.

suitable point« on the linear portion of the normal map.et.zat.on curve at speed n,. Then, fo

the same excitation oy,


Voltage ys _ Speed n±------
Voltage yt Critical speed n2
, Voltageyf v n ...(4 29)
.*. Critical speed n2 = Voltage ys 1
, j wViirb the d c shunt generator just fails to build up with
Thus, critical speed is the speed at which the a.c. s b

no externalresistance in f i e l d .,d s fl)r the shul>t generators, also


The foregoing description about the v J f8 , n the long.8hunt compound generator.
applies
)lies to the voltage build up in compound gene
g^ . . . .•. . „ ma\\
the m.m.f. produced by the aeries field turns f ., buUd. up thc voltage at no-load.
Failure to build-up. If the self-excited generators In.. to ou p k

of the following
it may be due to any one ot ionow.„K reasons. g^ ^ (m)y .f thererosldual
(i) No-residual m a g n c U s m . The vol^J ^ r a to r In case of new machine or the one that has
magnetism in the magnetic circuit of th g . trnnHp0rtotion, self-excited generator will
lost its residual magnetism due p aging J b exciting the field winding from a
fail to build up the voltage. This difficulty cmn 1k 0^ ^ ^ ^ ^ separate d.c. source
separate d.c. source for a few seconds wi' flux ftnd V0itaR0 build up can take place,
is disconnected, the main poles possess r

S c an n ed by C a m S c a n n e r
412 Electrical Machinery [Art. 4.1]

The process of connecting the shunt-field winding to a separate dc source for creating the
necessary residual magnetism in the main poles is called field flashing. t

(ii) Field connections reversed. The small voltage due to residual magnetism, should
circulate current in the field coils in such a direction as to produce flux lines aiding the residual
flux. If the field connections are incorrect, the flux produced by small field current opposes the
residual flux and the generated voltage decreases to zero. This trouble can be remedied by
reversing the field connections with respect to armature terminals.
(iii) High field circuit resistance. In case the field resistance is more than the critical
field resistance, voltage will not build up. The voltmeter connected across the armature ter­
minals will indicate only a low voltage (2 to 6V). This trouble is caused by an open circuit in the
field or armature connections, dirty commutator or a large external resistance in the field cir­
cuit.
This drawback can be overcome by (i) checking whether field or armature circuit is open {ii)
cleaning the commutator surface and ensuring good contact between brushes and commutator
surface and {iii) by adjusting the external field rheostat to zero-ohm setting.
{iu) Speed less than critical speed. With no external resistance in the field circuit, if
self-excited generator fails to build up, it may be due to the armature speed being less than the
critical speed. This trouble can be remedied by increasing the prime-mover speed above critical
speed.
External characteristic from no-load magnetization curve. The external charac­
teristic of a shunt generator can be determined from the no-load magnetization curve or satura­
tion curve, provided the field circuit resistance, demagnetizing ampere turns due to armature
reaction and armature circuit resistance are known.
Arm ature reaction neglected. In the left of Fig. 4.43, curve 1 is the no-load magnetiza­
tion curve (Ea versus If) and OA is the field resistance line {Vt versus If). At no load, BA is the
armature generated e.m.f. Ea for a field.current equal to OB. Under steady state operation and
with armature reaction neglected, the vertical distance between the saturation curve 1 and
S l nce hnei 0A ,s ec*ual t0 armature resistance drop. For example, for field current
OC, CD is the armature generated e.m.f. Ea, DD' is the armature resistance drop I r and CD’
is the terminal voltage Vt. That is “ “
DD’ - CD - CD'
or I r =E - V .
aa “ V‘ ...(4.30 a)
Ea - V t
or
r„ ...(4,30 6)

E x te rn a l
c h a ra ct.

Fig 4.43. Determination of the external characteristic from


no-load magnetisation curve (armature reaction neglected)

S c an n ed by C a m S c a n n e r
Art. 4.11] D.C. Machines 413

Eq. (4.30 a) reveals that for a known value of Ia, Iara drop and therefore V, is known.
For plotting the external characteristic, cut OG equal to Iara for any assumed armature
current Ia. Draw GH parallel to the Field resistance line OA intersecting the magnetization
curve at points M and D. From M and D, draw vertical lines meeting the field resistance line at
M' and D' respectively. In the right hand side of Fig. 4.43, make O' x - load current = assumed
(o g \ - •
armature current ----- minus shunt field current OC. A vertical line at x and horizontal line
l r° J
through points D\ M ’ meet at points d, m. These two points d, m lie on the external charac­
teristic of the shunt generator. Similarly other points on the external characteristic can be
plotted. Noterthat maximum armature current can be found by drawing a line LN, tangent to
the magnetization curve and parallel to the field resistance line OA. The magnitude of maxi­

mum armature current will be given by and maximum load current is equal t o minus
ra ra
OC'. When the terminals are short circuited, O'K is the load current due to the e.m.f. OF
generated by the residual flux.
Arm ature reaction included. When armature reaction is to be included in plotting the
external characteristic from the magnetization curve, then the armature resistance voltage
drop DC and armature reaction demagnetizing effect GC as illustrated in Fig. 4.35 (6), must be
included. In the left-hand side of Fig. 4.44, magnetization curve and field resistance line OA are
drawn intersecting each other at A, so that no-load terminal voltage is O'a = BA. For including
the armature reaction demagnetizing effect, draw OC equal to the equivalent reduction in
shunt field current caused by the demagnetizing effect of armature current. In other words,
OC is equal to the ratio
Demagnetizing ampere turns due to armature reaction
Shunt field turns
Total armature resistance drop, for the assumed value of current 7a, is 7a ra and it is drawn as
CD perpendicular to OC. Draw a line DH parallel to the field resistance line OA, intersecting
the saturation curve at points G, H. Now draw Hh, Gg parallel to DO. In the right hand side of
Fig. 4.44, O'x is equal to the assumed armature current minus the field current OB'. The verti­
cal line a tx and the horizontal lines through the points#, h meet at the points#', K . These two
points #', h' are on the external characteristic for a load current equal to O'x. Same procedure
can be adopted for plotting other points on the external characteristic. Maximum current can

Fig. 4.44. Determination of external characteristic from no-load


magnetisation curve (armature reaction included’

S c an n ed by C a m S c a n n e r
414 Electrical Machinery [ A r t 4.11

again be found out by drawing at the point N, a tangent parallel to the field resistance line. The
• C 'T )'
magnitude of maximum armature current is given b y and maximum load current is equal
?a
C'D '
to ~z — minus field current ON'. In Fig. 4.44, O'K again gives the short circuit current due to
'a
the e.m.f. generated by the residual flux.
Exam ple 4.19. The following data pertain to the magnetization curve o f a d.c. shunt gene­
rator at 1500 r.p.m.

If, amp. 0 0.4 0.8 1.20 1.60 2.00 2.40 2.80 3.00
Ea, volts 6 60 120 172.5 202.5 221 231 237 240

For this generator, obtain


(a) the voltage on open circuit to which the machine will build up (i.e. no load e.m.f.) for a
total shunt field resistance o f 100 Cl ;
(b) the critical value o f shunt field resistance at 1500 r.p.m .;

I (c) the critical speed for the shunt field resistance o f 100 f l ;
(d) the magnetization curve at 1200 r.p.m. and therefore the open circuit voltage for a field
resistance o f 100 Cl ;
(e) the terminal voltage o f the generator i f the total armature resistance is 0.3 Cl, armature
current is 50 A and the speed is 1500 r.p.m. Neglect armature reaction.
Solution, (a) The magnetisation curve at 1500 r.p.m. is drawn in Fig. 4.45. The field resis­
tance line for 100 Cl is drawn, passing through the origin and say 240 volts, 2.4 amp. point A.
The field resistance line meets the magnetization curve at the point B, giving no load e m f of
230 volts. 1

(b ) For determining the critical field resistance at 1500 r.p.m., draw a line OF passing
through ongin and the straight line portion of the magnetization curve. The slope of this line is

found to be — = 150 Cl, which is the critical resistance of the shunt field circuit at 1500 r.p.m.

(c) For determining the critical speed, choose any suitable point S on the straight line por­
tion of the magnetization curve A vertical line from 5 , meets the field resistance line at t and
the horizontal line aty. From this vertical line and from Eq. (4.29), critical speed

n2 = — x 1500 = 1000 r.p.m.

UD The data for the magnetization curve at 1200 r.p.m. can be obtained by multiplying,
each of the voltage ordinates of 1500 r.p.m. magnetization curve by or 1 Therefore the

required magnetization curve at 1200 r.p.m. is as given below in ta b u lfr fo m ,5

If, amp. 0.0 0.4 0.8 1.20 1.60 2.00 2.40 2.80 3.00
E0, volts 4.8 48 96 138 162 177 185 189.6 192

lin e lJ 10r ^ a f S n T ^ if u aWn(in rF i! ' 4 45 a" d is Seen to t e r s e s t the field resistance


volts P °f ln t° rsection « no load generated e.m.f. of 165

Scanned by Cam Scanner


, . 1t1 D.C. Machines 415
Art. 4.11] __________________ —

(e) For an armature resistance drop 50 x 0.3 = 15 volts, cut OD equal to Iura drop i.e. 15
volts. Draw DG parallel to the field resistance line of 100 Q. From G draw a vertical line meet­
ing the field resistance line at H, which gives the terminal voltage equal to 207 volts. Note that
GH = OD = 15 volts and Ea = 207 + 15 = 222 volts.

Fig. 4.45. Pertaining to Example 4.19.

Exam ple 4.20. A separately excited generator gave Ihe following data for open circuit char­
acteristic at 1000 r.p.m.

Ip,amp 0 0.20 0.40 0.60 0.80 1.00 1.20 1.40

Ea, volts 5 50 100 140 170 190 200 205

The armature resistance, including brushes, is 0.5 il. I f the generator is now shunt connected
and is driven at 1100 r.p.m., then, for a total shunt field resistance o f 180 £2, calculate

{a) no load e.m .f.;


(6) the output current and shunt field current for a terminal voltage o f 190 V ;
(c) the maximum output current and the corresponding terminal voltage ;

(d) steady state short circuit current.


For the above parts, neglect armature reaction.
(e) In case no-load voltage o f 210 V is required, find the additional resistance that must be
inserted in the field circuit.

i i c u u_y v ^ a i i l o o a i ii i c i
mmm

[A rL 4.11
416 Electrical
E < l C L ( l l \ . d i lM
T iachinery
aw iiiiivi j — ---------- -----------------

(/) For a shunt field resistance o f 150 n ,th e ter^ n^ ^ ^ f a t o r a n d ^ h e generated e.m.f.
certain load at 1100 r.p.m. Find the load supplied by the gene
Assum e that flux is reduced by 4% due to armat“ re Bq. (4.24) is given by the table

below. 1.40
1.00 1.20
0.6 0.80
/*r amp. 0 0.2 0.4
r 220 225.5
187 209
55 110 154
Ea, volts 5.5

This curve is plotted in Fig. 4.46. The hem ~ f


to meet the saturation curve at 221 volts, whic is tho field resistance line. The vertical
(6) Terminal voltage of 190 V is indicated by point 1on th ,. a, the point B is
difference BC between the magnetization curve a n d field rests
22.5 V, which is equal to the total armature resistance drop /„
j _ 22J. _ 45 amp

The shunt field current corresponding to the terminal voltage of 190 V is given by

. r — J —1 r —
- 45 — 1.06 = 43.94 amp.
Output current ~ l a lf fanfrpnt to the magnetization curve but
(c) For maximum output current, draw a MOP
parallel to the field resistance line OA. The intercep u
entire armature resistance drop.
j = 46J5 = 93 Amp.
la 0.5

0-6 0-8 1*°


If ( A m p ) ----

F ig . 4.46. P e r t a in in g to E x a m p le 4.17 a n d 4.20 (a).

Scanne
V' .
Art. 4.11] D.C. Machines 417

The tangent point R gives a field current of 0.635 amp and the corresponding no-load e.m.f.
of 160.5 volts.

t .-. Maximum output current = 93 - 0.635 = 92.365 A


and the corresponding terminal voltage
= 160.5 - OP = 160.5 - 46.5 = 114 V.
id) cur-
Under steady state short circuit, the terminal voltage is zero and the short circuit
rent is decided by the residual flux e.m.f. of 5.5 volts and the armature resistance only.
.*. Steady state short circuit current
5.5
= 11 A.
0.5
(e) The no-load e.m.f. of 210 V meets the saturation curve at D. The field current read from
210 •
the graph is 1.015 A. Thus line OD represents a resistance of - = 206.9 fl. Therefore, addi­

tional resistance required = 206.9 - 180 = 26.9 fl.


if) Terminal voltage of 180 V gives a field current of 180/150 = 1.2 A. For this If, generated
e.m.f. Ea at 1100 r.p.m. is 220 x 0.96 = 211.2 V.

” = 211 0 5 180 = 62 4 A
h = Ia ~ If = 62.4 - 1.2 = 61.2 A
Load power = 61.2 x 180/1000 = 11.016 kW.
E xam p le 4.21. A 200 amp., 30 volt d.c. generator for aircraft has the following no-load
saturation curve at 2200 r.p.m.

If. am p 2 4 6 8 10 12

VOltS 15 27 35 40 43 45

The armature resistance (including brushes) is 0.03 ohm and field winding resistance is 2.4
ohms. Armature reaction is negligible.
Under normal operation, the speed with which the generator is driven may vary from 2200
r p m to 4500 r p m An external rheostat in the shunt field circuit is varied by a regulator to
maintain the terminal voltage at 28.0 volts over the complete range in speed (2200 to
and the complete range o f output current (0 to 200 amps).
What must be the range in ohms o f this regulating rheostat? What power mustitbe^capabk
o f dissipating ?
Solution.
At 2200 r.p.m., . , -
A no load E = 28 + 0 = 28 volts. For this voltage, the field current required, from the mag­
netization curve of Fig. 4.47 is If = 4.23 A.

,\ Total shunt field resistance = ^ 3 " ^

\ External resistance R = 6 6 2 “ 2 4 = 4,22 n


At full load, E . = 28 + 200 x 0.03 = 34.0 V and the corresponding field current is 5.67 A.

. - 2^ = 4 94 fl
/. Total field resistance - 5 67 •
R = 4 94 - 2.4 = 2.54 fl.
and external resistance •

>canned by C a m S c a n n e r
418 Electrical M achinery [Art 4.11

At 4500 r.p.m.,
At no load, Ea = 28 volts at 4500 r.p.m.

.*. Ea at 2200 r.p.m. _ 2 8 x ^ 5 5 = 13 7 V


” 28 4500
From magnetization curve at 2200 r.p.m., 13.7 V corresponds to
If= 1.833
.83; A
28
Total field resistance = = 15.27 ft
1.833
and external resistance = 15.27 - 2.4 = 12.87 ft.
At full load, Ea = 28 + 200 x 0.03 = 34.0 V at 4500 r.p.m.

.*. Ea at 2200 r.p.m.

From saturation curve at 2200 r.p.m., 16.62 V is produced by a field current of 2.17 A.
28
External resistance required = 2 1? - 2 . 4 = 10.5 ft.

It is seen from above that the minimum value of external resistance is 2.54 ft and the m*®
mum value is 12.87 ft. Thus the range of the regulating resistance is from 2.54 ft to 12.87 ft.

Maximum power that must be dissipated by regulating rheostat


= (5.67)2 (2.54) = 81.66 watts. fl/
E xam p le 4.22. A 2-pole d.c . shunt generator charges a 100-V battery o f ^ ^ f ^ l l o h r n
resistance. The armature o f the machine is made up o f 1000 conductors, each o f ~
Art. 4.11]
D.C. Machines 419
resistance. The chareine n *
1105 rpm respectively. F ind th ofi to^e
10 A. and 20 A for generator speeds o f 1055 and
armature reaction effects -circuit resistance and flux per pole o f the generator. Neglect
a ' ' (J.E.S., 1979)
Solution. Armature circuit resistance

= 500 x 2 x 10" 3 x i = 0 .5 n
Let If be the shunt field current.

The generated emf at 1055 rpm is given by

Eai = Vt + (IL +If) x 0.5


= 100 + (10 + 7^.) x 0.5
At 1105 rpm,
Ea2 = 100 + (20 + If) X 0.5
But
Eal oc field current x speed
^ If x 1055
and Ea2~ I f x 1105
If x 1055 oclOO + (10 + If) x 0.5
and If x 1105 oc 100 + (20 + If) x 0.5
. 7^x1055 100 + (10 + /,) x 0.5
IfX 1105 ~ 100 + (20 + /f) x 0.5
Its solution gives If = 1 A.

••• E ai = 100 + (10 + 1) x 0.5


= 105.5 volts at 1055 rpm
Now Eal = ^ ^
CL
r <]>x 100 x 1055 x 2
or lUo.o = — — ---------
60 x 2
105.5 x 60
or f - = 0.006 Wb
1000 x 1055

.-. Field circuit resistance = ~ r = = 100 Q


V 1
Flux per pole = 6 milliweber.
E xam ple 4.23. (a) The generator o f Example 4.20 is driven at 500 r.p.m and its field coils
are grouped in two parallel circuits. A regulating resistance is included in the field circuit and
the machine builds up to a voltage o f 90 V. What is the value o f the resistance in the regulator ?
(b) A d.c. generator is fitted with the following two field windings :
(i) A separately excited winding o f 800 turns per pole and resistance o f 160 0. supplied from
a constant voltage source o f 220 V.
Hi) A shunt winding o f 500 turns per pole and resistance o f 200 Q. The open circuit mag­
netization at 500 r.p.m. is given by the data as under .

154 302 396 458 505 538


Armature voltage, V
500 1,000 1,500 2,000 2,500 3,000
Field ATs per pole

Determine the no-load terminal voltages at 500 and 600 r.p.m., in case both windings are
connected to their respective source at both the speeds and their ATs are in the same direction.

r /I lf
uy 11 lo u d i rn er
(Art 4.11
4 20 Electrical M achinery

Solution, (a) In Fig. 4.46, magnetization curve at 500 r.pin. w ^raw n as shown. For a
generated e.m.f. of 90 V, the field current is found to be 0.89 A. Thus the total resistance in the
field winding is 90/0.89 = 101.12 ft. When field coils are grouped in two parallel circuits, the
90x90
T h e r e f o r e , t h e v a l u e o f the
shunt-winding resistance reduces from 180 ft to 45 ft = 90 + 90
\ *
resistance R^ in the regulator is given by
27,(45 + 7?i) = 90

or j? 1 = _ 9P— - 4 5 = 5.562 ft.


1 2 x 0.89
(6) ATs of separately-excited winding
= — x 800 = 1100 ATs/pole.
160
E
ATs of shunt-field winding for any generated e.m.f. E are given by ^qq x 5 0 0 -2 .5 E

ATs/pole.
.-. Total ATs due to both field windings = 1100 + 2.5 E ATs/pole.
For 500 r.p.m., the calculations in tabular form are as under:

Generated e.m.f. E, V 154 302 396 458 505 538

(1100 + 2.5 E), ATs/pole 1485 1855 2090 2245 2362.5 2445

Field ATs per pole 500 1000 1500 2000 2500 3000

Plot between field ATs/pole and E gives the magnetization curve. Plot between (1100 + 2.5
E) and E gives the variation of resultant field ATs/pole and generated voltage E. Their intersec­
tion at point P, Fig. 4.48, gives the no-load terminal voltage of 490 V at 500 r.p.m.

Field ATs / P o l e — ►
Fig. 4.48. Pertaining to Example 4.23 (A).

J b U I II ICU uy o a n r ^ j o a i 11 i c i
Art. 4.11]______________ d .C. Machine* 421

The computations at 600 r.p.m. are as below :

Emf E, V 184.8 362.4 549 6 606 645 6


475.2
(1100 + 2.5 E), ATs/pole 1562 2006 2474 2615 2714
2288
Field ATs per pole 500 1000 2000 2500 3000
1500

These values are also plotted in Fig. 4.48. The plot between E and (1100 + 2.5 E) at 600
r.p.m. is seen to be coincident with that at 500 r.p.m. The point Q gives the no-load voltage as
621 V at 600 r.p.m.
4.11.3. Series Generators. These generators arc used mainly as scries boosters con­
nected in the line, to neutralise the effect of line ohmic drop.
(i) No-load characteristic (no-load magnetization curve). In a series generator, the
armature winding, field winding and load resistance are connected in series, therefore the field
current is equal to the armature or load current. In view of this, the no-load magnetization
curve can only be obtained by separately exciting its field from a low voltage source as shown
in Fig. 4.38 (a), so that armature current is always zero. The magnetization curve at one speed
for a series generator is illustrated by curve 1 in Fig. 4.49 (a).

(ii) Load characteristic (load magnetization curve). This characteristic also, can only
be obtained by separately exciting the series field winding. The shape of this characteristic is
identical with that of the separately excited generator.
(iii) External characteristic. For obtaining the external characteristic, connections of
Fig 4 49 (b) are used With the switch S open, the small voltage due to residual flux will be
indicated bv the voltmeter. When the switch 5 is closed, field current equal to the load current,
starts flowing ^ f the current in the series field produces a flux aiding the residual flux, the
generator will build up voltage till point C is reached. Note that at point C. the field resistance
line OC meets the saturation curve. The field resistance line OC depends on the total resistance
in the series circuit i.e., the slope of line OC is determined by the sum of armature circuit
resistance, series field resistance and load resistance. If the total series resistance is more than
the critical field resistance, just like a self-excited shunt generator, the b.uld up process will not
begin.
The external characteristic is shown by curve 2 in Fig. 4.49 toV Increase the load on dc
series generator in steps and at each step, record load voltage and load current. A curve passing
through these plotted points gives the external characteristic of curve i » his figure, Alt is
the load voltage or armature-terminal voltage lor a load current equal to OA. II total resist.IIK'O

S c an n ed by C a m S c a n n e r
422 Electrical Machinery

drop Iar is added to the ordinates of curve 2, the internal characteristic shown by curve 3 is
obtained. Note that r is the sum of series field resistance and armature circuit resistance (in­
cluding brushes). Thus the voltage drop BD is equal to the total armature resistance drop/or
and the voltage drop CD is due to the armature reaction. A horizontal line through D meets the
magnetization curve at F and DF gives the demagnetizing effect caused by the armature reac­
tion for a load current equal to OA.
It is obvious from the shape of the external characteristic that the series generator is a
variable voltage generator— it is therefore never used as a voltage source.
4.11.4. Compound generators. Cumulatively compounded generators are more common
because these can furnish almost constant voltage from no load to full load.
(i ) No load characteristic. For the no load characteristic, same current flows through the
series field and shunt field turns in case of a long-shunt compound generator. Since the number
of series field turns is far less than that of the shunt field turns, the effect of series field m.m.f.
may be neglected in comparison with the shunt field m.m.f. at no load. For a short-shunt com­
pound generator, series field carries no current when obtaining its no load characteristic. Thus
the no load characteristic of a long-shunt or short shunt compound generator is the same as if
it were a shunt generator.
(ii) External characteristic. The external characteristics of a compound generator are
shown in Fig. 4.50 (a). In a cumulatively compounded generator, with the increase of load cur­
rent, the series field flux aids the shunt field flux. Depending upon the number of series field
turns, the cumulatively compounded generator may be under-compounded (terminal voltage
falls with increase of load), level or flat-compounded (terminal voltage remains practically con­
stant with increase in load) or overcompounded (terminal voltage rises with increase in load).
In a differentially compounded generator, with the increase in load, the series-field flux opposes
the shunt-field flux and consequently the terminal voltage falls more rapidly. These external
characteristics, along with shunt and separately excited generators for comparison purposes
are sketched in Fig. 4.50 (a) where no-load voltage is assumed to be the same in all the cases.

(a) (b)
Fig. 4.50. (a) External characteristics of various types of d.c. generators
and (6) circuit diagram of a compound generator with diverter.

Differentially compounded generators are not damaged by short circuit. In view of this,
these generators may be used for welding purposes, where sudden short circuit occurs
everytime the electrode touches the working part. However, modern welding generators are of
special design. Slightly overcompounded generators are used for maintaining constant v o l t a g e
at the load terminals.
The degree of compounding can be controlled by connecting a suitable low resistance called
diverter, in parallel with the series field winding as shown in Fig. 4.50 (6).

S c an n ed by C a m S c a n n e r
Art. 4.11]
D.C. Machines 423

but u n d e r ^ h e ^ s ^ m n ^ n ^ f ° f Vanous types d c- generators are also drawn in Fig. 4.50 (c)
. , f , 0 same rated terminal voltage Vtr and load current ILr. The observa­
tions made from these V-I characteristics are as under :

mmrlnJhT" ^1
eneirator»_curve 1, the terminal voltage rises rapidly with load current. At
.. ’ n vo age egins to decrease owing to saturation, armature reaction etc.
(ii) For cumulative overcompounded generator, curve 2, no-load voltage is less than Vlr.
(m) For level or flat, compounded generator, curve 3, the no-load and full-load voltages are
equa . owever, ermina voltage at light load is more than Vt but at overloads it falls below
V tr-

(iv) For cumulative undercompounded generator, curve 4, no-load voltage is more than
tr

(u) For both separately-excited (curve 5) and shunt (curve 6) generators, Vtr is less than
no-load voltage. Voltage variation from no load to full load is less in separately-excited gene­
rator than in a shunt generator.
(vi) For differential compounded generator, no load voltage is much higher than Vfr.
voltag e
'DIFFERENTIAL
S EPARATEL Y E X C I T E D
J £ ____/ y^UNDER COMF
.L IT " Z ^SHUNT
5 ____

C UR RE NT
Fig. 4.50. (c) External characteristics of d.c. generators.

Fig. 4.50 (c) reveals that voltage variation from no-load to full load is quite high in series
and differentially compounded generators.
4 .11.5. Effect of speed on external characteristics. The external characteristics of
shunt and compound generators are affected considerably by the operating speed of the driving
motor. In this section, this effect is investigated first for a dc shunt generator and then for a
cumulative compound generator.
Shunt gen era tors. In Fig. 4.51 (a), magnetisation curves for two different speeds
ft! and n2 are drawn for a dc shunt generator. For the same no-load generated emf E, shunt-
field current or mmf is OB for speed n j and OA for speed n2 where rated speed n2 > n\. It is seen
from Fig. 4.51 (a) that at lower operating speed, flux or field mmf required for generating the
same no-load voltage is much greater (here OB) than that required at higher operating speed

s c a n n e d Dy u a m ^ c a n n e r
424 Electrical Machinery [Art. 4.11

(here it is OA). As a result, the iron in the magnetic circuit gets highly saturated at lower speed
nv

When the shunt generator is loaded, armature reaction comes into play. For the same ar­
mature current, the effect of armature reaction on the highly saturated field is less than on the
weak field. Let this effect be BC for strong field at speed n\ and A D on weak field at speed n2.
These two armature reactions are indicated in Fig. 4 .5 1 (a). The n et field m m f is
OC = (OB - BC) at speed nxand is OD = (OA - AD) at speed n2. At speed nj, resultant field mmf
OC gives rise to voltage CP and resultant field mmf OD generates voltage DR at speed n2. Note
that voltage at reduced speed has higher value CP than the voltage DR generated at rated
speed n2. This shows that voltage drop at high speed is more than at lower speed. As a conse­
quence, external characteristic at high speed lies below the external characteristic at low speed
as shown in Fig. 4.51 (b).

vt

*n t ( low er
speed)
'n2(higher
speed)

R a te d
c u rre n t
D A C mmf
F ie ld , m m f Loa d c u rre n t

• ^ (A)
Fig. 4.51. Effect o f speed on the external characteristics o f a dc shunt generator.

t e r i ^ r Ul a» ^ * i,; T P0Und ®enerf tors- K « already explained that open-circuit charac-


Z X ° magnetization curve for a dc compound generator is the same as that obtained for a
c shunt generator. In Fig. 4.52 (a ) ; two magnetization curves, one at speed n , and the other
at speed n2 are drawn. Here rated speed n2 > As in a dc shunt generator, field mmf requ.red

OB) for a Bivcn " d - « *

e a u a ^ ? s e r i« fiUiM m m f CUrre" ! ” “ dctcomP°und o r a t o r , net additional mmf would be


equal to (senes-fied m m f - armature react,on effect) = A A ' at speed n2 and it would be equal to
(senes - field m m f - armature reaction effect) = at speed Series-field mmf is same at

s o le d n ’ S . ' S * m T etiC 7 ™ ? highly saturated <">«re field mmf OB) at lower


X f dd'' f j armature reaction is less at speed nx than at normal speed n2. This means
that BB at speed ^ is more that AA ' at speed n2. These net additional mmfs BB' and AA' are

w resultant ^ n i f (OA + AA') = OA' at speed n2 and


(OB + BB ) = OB at speed Resultant mmf OA' at rated speed n2 generates a voltage A'C
greater than the voltage B'D generated by resultant mmf OB' at lower speed n v Higher
generated voltage A'C further boosts the shunt field current by a greater extent than that due
to lower voltage S O . As a consequence, compounding action is more effective at high speed
than at low speed and as such, terminal voltage at a particular load current is more at n9 than

O L Q I II I C U Uy O Q I I I J L Q I II I C I
Art. 4.11]
D.C. Machines 425

A A' mmf
F ie ld m m f

(a)
g . Effect o f speed on external characteristics of a dc cumulative compound generator.

at This shows that external characteristic at high speed lies above the external charac­
teristic at low speed as illustrated by solid curves in Fig. 4.52 (b ).
It is seen from above that the effect of speed on external characteristics of a cumulative
compound generator is opposite to that in a dc shunt generator.
Effect of speed on voltage regulation. Effect of speed on the voltage regulation in both
dc shunt and compound generators can be examined by
referring to Fig. 4.51 (b ) and 4.52 (b). In Fig. 4.51 (b), if
dc shunt generator runs at rated voltage, rated load cur­
rent and rated speed n2, then external characteristics at
speeds n2 and ni < n2 can be sketched as shown in Fig.
4.53. These two characteristics at speeds n2 and in
Fig. 4.51 (b) are pushed upward bodily so as to get the
same load voltage OA in Fig. 4.53.
Similarly, the external characteristics of Fig. 4.52
(b) are pushed bodily downward so as to get the same
load voltage OA in Fig. 4.53. It is observed from these
curves in Fig. 4.53 that at reduced speeds, voltage varia­
tion from no load to full load gets reduced. In other
words, the voltage regulation at reduced speeds gets im­
proved in both dc shunt and dc cumulative (under, level Fig. 4.53. Effect of speed on external
and over) compound generators. characteristics and voltage regulation.

It is seen from above that running the dc generators


at reduced speeds and higher field currents leads to better performance so far as voltage regula­
tion is concerned ; but it is accompanied by some drawbacks. There are (i) higher field currents
cause increased field-copper losses (ii) higher field currents also lead to more iron losses and
(Hi) reduced speed operation impairs cooling by ventilation. Thus overall machine efficiency
gets reduced. It is therefore advisable to operate dc generators at their designed specifications
only.
E xternal ch aracteristic from no-load m agnetization curve. The external charac­
teristic from the no-load magnetization curve of a compounded generator can be obtained in the
same manner as obtained in the case of a shunt generator, but here the effect of series field
m.m.f. must also be included.

~ ^ > U d l II IfcJU u y O d l M O U d l II IfcJI


426 Electrical Machinery [Art. 4.11

Let ATd) proportional to armature current Iu, be the demagnetizing effect of armature reac­
tion. In a cumulatively compounded generator, the series field m.m.f., proportional to armature
current, counteracts ATd. Therefore the net m.m.f. is NJs - A T d and in terms of equivalent
shunt field current, it is given by
N Js - A T d
...(4.31)
Nr
For a cumulatively compounded generator, the effect of net equivalent shunt field current
given by Eq. (4.31) may be demagnetizing if ATd > NJ S; magnetizing if N JS> ATd, or zero if
NJs =A T d.
For a differentially compounded generator
NJS+ ATd
...(4,32)
Nf
and the equivalent shunt field current given by Eq. (4.32) is always of demagnetizing nature.
.I In order to obtain the external characteristic from OCC, draw OC equal to the equivalent
shunt field current obtained from Eq. (4.31) or Eq. (4.32), for any armature current Ia. In Fig.
4.54, note that the equivalent shunt field current has been shown as magnetizing. In case
equivalent shunt field current is demagnetizing, draw OC opposite to that shown in Fig. 4.54.
The total armature resistance drop (including brushes) is indicated by CD. Draw a line DH,
parallel to field resistance line OA, meeting the saturation curve at H. Make Hh equal to CD.
With O'x equal to load current IL (= armature current/a— field current OB), draw a vertical line
at x and a horizontal line through h. Their intersection at W, gives the required point on the
external characteristic. Other points can be plotted similarly. At no-load, shunt field resistance
line meets the open circuit characteristic at A, which gives the no load terminal voltage O'A'.
Vt

v, av n rr
magnetization curve at 1200 r.p.m.
-*
If, amp. 0.0 0.2 0.4 0.6 0.8 1.0 1.20 1.40 1.60 1.80
Ea, volts 6 53 106 160 209 241 258 272 282 28S

n s J ! % T mt inef reSi Stanl e °f! t f ries Windins and ^m ature Winding (including brushes) is
u J u ,“ S„ tUrns' At m tcd ou‘Put current, the speed is 1150 r.p.m. and
shunt field current is 1.00 A. For negligible armature reaction, calculate

S c an n ed by C a m S c a n n e r
Art. 4.11]
D.C. Machines 427
(a) the terminal voltage at rated output current iftho r u .
(b)the num ber o f series field turns oer n 7
1
rator. converting it into a flat-compounded gene-

Solution. (a) Rated output current IL = I 0000. = 43 5 a


230
.-. Armature current at rated load Ia = IL + If = 43.5 + 1.00 = 44.5 A.
For long shunt compound gene­
rator, the series field current Is = ar­
mature current, Ia = 44.5 A.
Since the compound generator is
cumulatively compounded, the total
pole m .m .f. is i.e.,
(1000x 1.00 + 4 x 4 4 .5 ) ampere
turns.
.-. The equivalent shunt field cur­

rent is given by {N flf+ N Jf) or

1.00 + 4 1QQQ5 = ^ ^ t^ S
current, the generated e.m.f., from the
magnetization curve of Fig. 4.55, is 257
volts. For a speed of 1150 r.p.m. the ac­ 02 0-4 06 08 1-0 12 1-4
tual generated e.m.f. Ea is Fig. 4.55. Pertaining to Example 4.24.

Ea= 257 x = 246 volts.

.-. Terminal voltage Vt = 246 - 44.5 x 0.5 = 223.75 volts.


(6 ) Vt = 230 volts, Ia = 44.5 A.
/. The generated e.m.f. in the armature at 1150 r.p.m.
= Vt + Iara = 230 + 44.5 x 0.5 = 252.25 V.
For using the magnetization curve, the generated e.m.f. at 1200 r.p.m. will be given by

252 25 x = 263.3 volts. From open circuit characteristic, Fig. 4.55, the field current cor-
1150
responding to 263.3 volts is 1.26 A.
The total m.m.f. must be equal to 1.26 x 1000 ATs. This total m.m.f. must be produced by
the combined action of shunt and series windings.
1.26 x 1000 = 1.00 x 1000 + Ns (44.5)
_0 .2 6x _1 0 00 = 58 2
.•. Series field turns 1’ ~ 44.5

Thus the number of series field turns should be 6.


Example 4.25. to) Repeat part M e f * a n * . 4 * 1 , V
reaction in terms o f equivalent shunt field curre a
. r- u m turns explain how the desired performance as a flat
(b) Iftlie sen es field minding has t10o , P lo be 0 05 SI.
compounded generator, can be achieved. Assu

>cannea oy u a m b c a n n e r
4 28 Electrical M achinery IArt. 4.11

Solution, (a) When the demagnetizing effect is accounted for, then from Eq. (4.27), we get
1.26 x 1000 = 1.0 x 1000 + Ns x 44.5 - 0.0022 x 44.5 x 1000
nr k, 0.3578x 1000 . .
or AT = -----------—— = 8 turns.
s 44.5
(6 ) If there are 10 series field turns, then from Eq. (4.27),
1.26 x 1000 = 1.00 x 1000 + 10 Is - 0.0022 l s x 1000

or i - ...Qffi = 33 3 A
' 0.0078
Out of total armature current of 44.5 A, only 33.3 A should pass through the series field.
This can be achieved by putting a resistor, Fig. 4.50 (b), in parallel with the series field winding.
mu r 4 4 5 X R di
Therefore, 33.3 = 5 -5 ^

OT = 0 l H 3 = 01487
Thus the resistance of the diverter Rdi should be 0.1487 Cl.
Example 4.26. A 250 V compound generator has armature, series-field and shunt-field
resistances o f 0.4 O, 0.2 Q and 125 Cl respectively. I f this generator supplies 10 kW at rated
voltage, find the e.m.f. generated in the armature when the machine is connected (a) long shunt
(b) short shunt. Ignore armature reaction and allow 1 volt per brush for contact drop.
(c) I f a diverter o f resistance 0.3 Cl is connected in parallel with series-field winding, find the
percentage decrease or increase in series field ampere turns.
Solution. For this example, refer to Fig. 4.14.

Load current, IL = -1^ ° = 40 A

(a) Long-shunt connection :

Shunt field current, If = =2A


125
.-. Armature current, Ia = IL + If = 42 A
Series-field winding also carries 42 A.
.-. Generated e.m.f. in armature, E . = V, + /„ (r. + r„) + total brush contact drop

E a = 250 + 42 (0.6) + 2 = 277.2 V


(6 ) Short-shunt connection:
Voltage across shunt-field and armature terminals
*V t + h - r ,
= 250 + 4 0 x 0 .2 = 258 V
Shunt field current, If = ||| = 2.064 A

Ic = Il + //r= 42.064 A
“ ^ ^Lr») "*■ ^ 2 x contact drop per brush
= 258 + 42.064 x 0.4 + 2 x 1 = 276.8256 V.

S c an n ed by C a m S c a n n e r
Art. 4.12]

(c) Series-field ampere-turns are proportional to series-field current 7.


Series-field current with diverter = — x 7 = o fi T
0.5
/. Series-field ampere-turns with diverter « 0.6 7
Percentage reduction in series field ampere-turns
7 -0 .6 7
= J x 100 = 40%.

4.12. Operating Characteristics of D.C. Motors


The basic equations underlying the d.c. motor analysis are, Eqs. (3.44), (4.6), (4.18) to (4.23)
the no-load magnetization curve and power balance Eqs. (4.11) and (4.12). The armature m.m.f!
can be taken into account as explained in Fig. 4.35 (6).

In a d.c. motor, e.m.f. Ea generated in the armature is called back or counter e.m.f. as ex­
plained before.

___
For d.c. motors, the supply voltage is usually constant and the quantities of common inter­
est are speed, torque etc. The following are the three important operating characteristics of d.c.
motors.
(i) Speed-armature current characteristic
(ii) Torque-armature current characteristic and
(iii) Speed-torque characteristic.
The object of this article is to describe these operating characteristics for different types of
d.c. motors.

4.12.1. D.C. Shunt Motor. For constant supply voltage, the field current is constant. At
small values of armature current the demagnetizing effect of armature reaction is almost neg­

;;;
ligible and therefore the air gap flux is uneffected. For larger values of armature (or load) cur­
rents, the demagnetizing effect of armature reaction, decreases the air gap flux slightly.
The speed of a d.c. motor, from Eq. (3.44) is given by

t»m = T7-^7 ...(4.33)

nr. ^
But Ea = V , - l , r ,
V. - I r
•• - < 4-34)
Ka §

(i) Speed-current characteristic. For constant supply voltage Vt and constant field cur­
rent If, the motor speed is affected by l ara drop and demagnetizing effect of armature reaction.
With the increase of Ia, the demagnetizing effect of armature reaction increases^hich reduces
the field flux— therefore the motor speed tends to increase. But with the increase of7a, \oltage
droP L ra increases and the numerator (Vt - Iara) decreases— therefore the motor speed tends to
decrease. With the increase of Ia, the numerator decrement is more than the denominator
decrement; in view of this, the speed of d.c. shunt motor with increase of 7„ drops only slightly
from its no-load speed wmo. Since 7„ at no-load is negligibly small, the shunt motor no-load speed
is given by

vt ...(4.35)
w"'° ~ K a Q

ht5rudinieu uy u d i i louai ii lei


(A r t 4.12
430 Electrical Machinery

In case the effect of armature reaction (AR) is neglected, t h e n thedenominator of Eq. (^34)
is constant. As a consequence, speed drops faster with /„. Fig. 4.56 (a) i us
characteristics of a shunt motor with and without AR. The curve mar e spee is
included.
(ii) Torque-current characteristic. The expression Te = Ka * l a reveals that if the flux
0 is constant as in a shunt motor, the torque would increase linear1^ . ^ ^/armature
Ia. However, for larger Ia> the net flux decreases due to the demagnetizing effect of armature
reaction. In view of this, the torque current characteristic deviates rom /Characteristic
as illustrated in Fig. 4.56 (a). In case the effect of AR is neglected, Te versus Ja characteristic
would be a straight line as shown.

/.i_

t S peed, ?
A.R.Neglected y
at
01 T o rque,A R
Neglected

(a) (b)
Fig. 4.56. D.C. shunt motor, (a) speed current and torque-current characteristics and
(6) speed torque characteristics.

(iii) Speed-torque characteristic. The speed-torque characteristic is also called the


mechanical characteristic and under steady state conditions, it can be obtained as follows.

Y t - K ra
From Eq. (4.34), m Kn 0

But
Ka *
Substituting this value of Ia in Eq. (4.34)

1 T .r .
Vt -
Ka $
Vt

= (1) —f
w mo ' a ...(4.36)

It is seen from Eq. (4.36) that w ith increase of T „ the speed drops. Note th a t for larger T„
larger /„ is required and this has the effect of reducing the a ir gap flux 4>, due to saturation and
armature reaction. Since w ith increase of „T*,<is reduced, T,/<^
the speed drops more rapidly w ith the increase of torque in a shunt motor as shown in Fig. 4.56

S c an n ed by C a m S c a n n e r
A rt4 ,1 2 j______________________________________________________________________________ D.C. Machines 431

If effect of AR is neglected, then [Ka (j))2 in Eq. (4.36) remains constant. As a result, the speed
drop with Tc is slow as shown in Fig. 4.56 (6).
4.12.2. D.C. series motor. For a series motor, the field current is equal to the armature
current.
(i)Speed-current characteristic. If saturation and armaturereaction areneglected,
then main flux 0 isdirectly proportional to armaturecurrent /„. Therefore, itmay e wri en
that $ = CIa, where C is any constant.
N ow +
Ea = Ka <t>com = V, - Ia (ra rs)
vt lg (r° + rs) ...(4.37)
or Wm = Ka <$> Ka <t>
Vt _ (ra + rs) ...(4.38)
or " Ka CIa Ka C
It is seen from Eq. (4.38) that with saturation and armature reaction neglected, the sPeed-
current characteristic of a series motor is a hyperbola, as illustrated by solid line in big. .
(a). . ...
Let us now investigate the effect of including saturation and armature reaction. At in­
creased values of Ia, the flux <)>should increase with Ia, but due to the demagnetizing effect o
armature reaction and saturation, the air gap flux * tends to remain approximately constant.

An examination of Eq. (4.37) reveals that for constant <(>, the term ^ is constant and the term
A
Ig (ra + rs) increases linearly with la. Thus the speed-current characteristic of a series motor, for

larger values o f /a, approaches a straight line, as illustrated by dotted portion of the curve in
Fig. 4.57 (a). .
At no load, the armature current is very small, because the power input to motor has to
overcome the no load losses only. Thus the voltage drop Ia (ra + rs) at no load, is almost negli­
gible as compared with Vt and from Eq. (4.37), the no load speed a)m0 is

V,
Wmo KJSf
Vt ...(4.39)
KaCIa
Since w is inversely proportional to 70, the no load speed of the series motor becomes
dangerously high due to small no load current. In view of this, the series motor must always
start and operate under load mechanically coupled with it.
(ii) T o r characteristic. With saturation and armature reaction neglected,
q u e - c u r r e n t

<\>= CIa
... Te = Kat>Ia
_ k C ll = C j/2 ...(4.40)

En (4 40) shows that the torque is proportional to the square a( l„ and, therefore, torque-
current characteristic is a parabola. But for larger /„, the net flux tends to ren.am approx,mate-
ly constant (as explained in speed-current characteristic^ Consequently he torque-current
characteristic approaches a straight line for larger values of/„, Fig. 4.o7 «■).

Scanned by Uam Scanner


r 432 Electrical Machinery
[Art. 4.12

■o
01
01
Q.
-Torque
01
D
Q*
Speed
:~ -A _
__ Speed-
( s o l & A ? neglected)

(a) w,
(6)
Fig. 4.57. D.C. series motor (a) speed current and torque-current characteristics and
(b) speed-torque characteristics.

(iii) Speed-torque characteristic. If saturation and armature reaction are neglected,


then from Eq. (4.40),

i .y f K
“ V K.C
Substituting this value of Ia, in Eq. (4.38),

„ Vt ra + rs
...(4.41)
m <KaCT' KaC

Thus with negligible saturation and armature reaction, the speed-torque characteristic is
a hyperbola, as shown by solid curve in Fig. 4.57 (fe).
With saturation and armature reaction included, large torques require large currents and
these large currents tend to make air gap flux 0 constant as explained before. This has the effect
of making Te approximately proportional to Ia (i.e. Te = K J a) and in view of this, Eq. (4.37)
becomes,

Vt Te (ra + rs)
ka

Thus, above a certain value of T „ when air-gap flux $ remains approximately constant due
to armature reaction and saturation; speed-torque characteristic approaches a straight line as
ft illustrated by dotted curve in Fig. 4.57 (6). It is s p e n frnm JTtu * r sira»gnt line as
speed drop at increased load torques is almost negligible. a or a series motor, the

fhia of c^ U tV v e * und

(i) Speed-current characteristic. For a long-shunt compound motor,


Vt = Ea+ I a ( ra + rs)
and
, - K (^ h + 1 „) oim
where ^ and are respectively the fluxes created by shunt and series field windings

= '*■<♦.* + 4>„) IV' ' <r« + r«>1 ...14.42)

S c an n ed by C a m S c a n n e r
Art. 4.12] D.C. Machines 433

With increase in Ia, 0^ increases ; as a consequence, denominator of Eq. (4.42) increases but
its numerator decreases. Thus, with increase in 70, the speed drops at a faster rate in cumula­
tive compound motor than in a shunt motor. This fact is indicated in Fig. 4.58 (a) where speed-
current characteristic of cumulative compound motor is shown below the speed-current
characteristic of. a dc shunt motor. Here no-load speed is assumed to be the same. For com­
parison purposes, speed-current curve of a dc series motor is also sketched in Fig. 4.58 (a).

(a)
Fig. 4.58. Comparison of (a) speed-current and (b) torque-current characteristics of different dc motors.

(ii) Torque-current characteristic. The electromagnetic torque Te is


Te = Ka $ I a
= Ka (0*/i + 0«) h ...(4.43)
At no load, Ia = 0, 0SC= 0 and therefore Te = 0. As the armature current Ia rises with load,
shunt field §sh remains almost constant but series field <|>se rises. As a result, motor torque Te
also rises as per Eq. (4.43). Shunt motor produces torque = Ka <j)sfc Ia ; this shows that cumula­
tive compound motor developes a torque higher than that developed in a dc shunt motor. This
is shown in Fig. 4.58 (6). For comparison purposes, dc series motor torque-current charac­
teristic is also sketched in Fig. 4.58 (6).
(iii) Speed-torque characteristic. From Eq. (4.43)

*<♦ *+♦ «>


Substitution of this value of Ia in Eq. (4.42) gives

Te (ra + rs)
V ,-
K<
, ** + ♦„) Ka (0«a + 0«)

~ (ra + rt) ...(4.44)


Ka (0sA + 4>se) Ka (<J>S/, + 4>se)2
With increase in motor torque Te, armature current rises, with this series-field flux <]>se rises.
Eq. (4.44) shows that first term gets reduced and the second term becomes somewhat more
(particularly near saturation where (<J>S/, + 0^) would tend to remain constant). Consequently,
speed drop in a cumulative compound motor is more than it is in a dc shunt motor, see Eq.
(4.36). Thus, the speed-torque characteristic in a cumulative compound motor is more drooping
than in a dc shunt motor. Fig. 4.58 (c) shows these characteristics .along with dc series motor
characteristic for the sake of comparison.

S c an n ed by C a m S c a n n e r
K

[Art. 4.12
434 Electrical Machinery

The characteristics presented in Fig. 4.58 are sketched on


the assumption of same no-load values for torque and speed.
Electrical machinery is usually specified in terms of its rated, or
full-load, values. In view of this, the curves of Fig. 4.58 are
redrawn in Fig. 4.59 with their rated values for speed, torque
and current. Characteristics compiled in this figure are suitable
in so far as comparison of dc motors of the same specification is
concerned. Fig. 4.59 (a) gives speed-current curves, Fig. 4.59 (b)
is for torque-current characteristics and Fig. 4.59 (c) compares
speed-torque characteristics of dc series, shunt and cumulative
compound motors. For obtaining the curves of Fig. 4.59, push
down or push up bodily the various curves of Fig. 4.58.
M otor torque
It is seen from Fig. 4.59 (b ) that for armature current Ia
Fig. 4.58. (c) Comparison of
oelow rated current, shunt motor develops the largest torque ; speed-torque characteristics of dc
but for l a above the rated current, series motor develops the motors.
largest torque. The maximum permissible current at the start­
ing of dc motors is limited to about 1.5 times the rated current. Thus, for loads requiring high
starting torque such as for hoists, cranes and traction-type loads, series motors are the most
suited machines.

<°> . (b) (C)


Fig. 4.59. (a) Speed-current (6) torque-current and
(c) speed-torque characteristics o fd c motors for comparison at rated load.

In a differential compound motor , series-field flux <f)sc opposes the shunt-field flux <|>h.
Therefore, Eq. (4.42) and (4.44) for this motor become

- ^ ~ rs)
" Ka (0rt - <jy .'..(4.45 a)

311(1 Te = ^ sh ~ $*) Ia ...(4.45 b)


With increase in load on this motor, Ia rises, <)>*, increases, the numerator decreases but
proportional to (<f>s/l - <j>se), decreases faster. Therefore, the motor speed rises, Eq.
(4.45 a). With decrease in (<j>j/t - <jy, the motor takes increased Ia as per Eq. (4 45 b) Therefore
«W rises and speed rises still further putting more load on the motor. This cumulative process
makes the motor speed dangerously high.

At the ume of starting differential compound motor, series field dominates the shunt field,
as a result, motor runs in one direction (say anticlockwise) due to the development of motor

r r ^
S can n ed by C a m ^ c a n n e r
Art- 4.12]________________________________ D.C. Machines 435

torque. Soon after, shunt field dominates series field, so the direction of net field gets reversed
and therefore the motor torque is also reversed. Therefore, now the motor torque is in clockwise
direction. As a consequence, motor speed falls from its anticlockwise direction, speed becomes
zero and eventually motor begins to run in clockwise direction. When the motor passes through
zero speed, armature current Ia = (V./ra) becomes dangerously high and the commutator and
armature winding may get damaged. ‘
It is seen from above that a differential compound motor possesses inherent instability
during starting as well as during running conditions. This motor is, therefore, rarely used in
industry. Through the starting problem can be overcome by short-circuiting the series-field
winding during starting, yet the instability during running conditions cannot be avoided.
Example 4.27. A 230-V, d.c. shunt motor, takes an armature current o f 3.33 A at rated
voltage and at a no-load speed o f 1000 r.p.m. The resistances o f the armature circuit and field
circuit are respectively 0.3 ft and 160 ft. The line current at full load and rated voltage is 40 A.
Calculate, at full load, the speed and the developed torque in case the armature reaction weakens
the no-load flux by 4%.
Solution. At no load, the counter e.m.f. is
£«i = V , - / o lra •
= 230 - 3.33 x 0.3 = 229 V.
Field current, If = = 1.44 A.
loU
At full load, Ia2 = IL - I f = 4 0 - 1.44 = 38.56 A.
.-. Counter e.m.f. at full load is
Ea2 = 230 - 38.56 x 0.3 = 218.43 volts.
At full load, the field flux is
<(>2= 0.96 <)>! (given).
The counter e.m.f. Ea is given by

Ea ~ Ea 0 0)m
. Eai _ Ka 0)ml _ <t)t n x
E 02 Ka 4>2 C0m2 02 n2
229 1000 x ^
or
218.43 n2 (0.96 fo)

.-. Full load speed, n2 = 993.6 r.p.m.


At full load, Ea2 = Ka <|>2 com
„ . 2 18 .4 3x 60
or *•** = 2**995
/
.\ Electromagnetic, or developed, torque at full load, Te = Ka §?Ia2

218.43 x 60
x 38.56 = 80.95 Nm.
2tt x 995
Exam ple 4.28. A 220-V d c shunt motor has an armature circuit resistance o f 0.2 Q and field
resistance o f 110 ft. A t no load the motor takes 5 A and runs at 1500 r.p.m. I f the motor draws
52 A at rated voltage and rated load, calculate the motor speed and its rated shaft torque Nm. in
The rotational losses at no load and full load are the same. Neglect armature reaction.

Scanned by Cam Scanner


436 Electrical Machinery [A r t 4.12

Solution. The shunt field current


/ 220 a
f 110
At no load, armature current
Z0l = 5 - 2 = 3 A.
Counter e.m.f. Eal = 220 - 3 x 0.2 = 219.4 V.
.*. Rotational losses = E al 4 i - 219.4 x 3 = 658.2 watts.
At full load, armature current
Za2 = —If —52 —2 = 50 A.
Counter e.m.f. Ea2 = 220 - 50 x 0.2 = 210 V.
Here 4>j (no load flux) = <j>2 (full load flux), because the field current is constant at 2 A and
effect of AR is neglected.

. Eg\ _ n \4>i
Ea2 n2 4*2
nr _ _ . . ^ 2 (1500) (210) ____
2~ ♦, 219.4 =1435rpm
Shaft power, Psh = Electromagnetic power - Rotational losses
= 210 x 50 - 658.2 = 9841.8 watts.

.‘.S h a ft torque ^ =f f = 6 5 . 4 NB.

_ ® *amP*e 4*29. A d.c. shunt machine has total armature circuit resistance o f 0 4 Q. and
fudd-circuitresistance o f2 0 0 Cl. Its open-circuit voltage at 1500 r.p.m. is 230 V fo r a field current
o f 1.1 A and 210 V for a field current o f 0.9 A.

I f thismachine is made to run as a d.c. shunt motor from 230 V supply mains at its full-load
armature current o f 24 A and at 1500 r.p.m.. find the external
in the field circuit. Neglect armature reaction. inserted

f F° r d C ShUnt m° t0r’ the C0Unter or &enerated e.m.f. Ea at 1500 r.p m and


at full load is given by F

Ea = v i~ 4 ra = 230 - 24 x 0.4 = 220.4 V.


For generated e.m.f Ea= 230 V, the field current is 1.1 A and for Ea
The change in generated e.m.f. is 20 V for a field current variation of 0 o a a « .• ’ L
linear. Therefore, for a generated e.m.f. of 220.4 V a t 1500 r.p.m., the field c u r r t n U s £ “ n by'

0.9 A for 210 V + H x 10.4 for the remaining 10.4 v ] = 1.004 A.

For a terminal voltage of 230 V across shunt field winding, a field current of 1.004 A re­
quires a total shunt field resistance o f— 0- = 29 Q nfta n
1.004 “ •

External resistance that must be inserted in shunt field circuit


= 229.084 - 200 = 29.084 Q.

b c a n n e a Dy u a m b c a n n e r
Art. 4.12] D.C. Machine* 437

pole is 0.02 Wb. Armature reaction is neglected. I f the motor draws 14 A from the mains, then
compute
(a) speed and the internal (total or gross) torque developed,
(b) the shaft power, shaft torque and efficiency with rotational losses equal to 300 watts.
Solution. P = 4, Z = 500, ra = 0.25 Cl,
7 = 1 2 5 0 = 0.02 Wb.

Constant shunt field current = — = 2 A


125
(a) Armature current Ia = 14 - 2 = 12 A.
.-. Counter e.m.f. Ea = Vt - Iara = 250 - 12 x 0.25 = 247 V.
But E - *>ZnP

or 0.02 x 500 x n x 4
247 =

247.0 x 2 247
n= r.p.s. = 741 r.p.m.
10 x 4 20
Electromagnetic power
Pe =EJa= 247 x 12 = 2964 watts.
Internal torque developed = — = ^ 64 x 60 = 38.30 Nm.
(Dm 2n X 741
(6) Shaft power P»h - P t ~ Rotational losses = 2964 - 300 = 2664 watts.
P,h 2664 x 60
Shaft torque = 34.4 Nm.
<om 2n x 741
_ Output at the shaft
Efficiency
Power input

2664 = 0.762 = 76.2%.


250 x 14
Exam ple 4.31. A fan has the following speed-power characteristics:

Speed., r.p.m. 700 800 900 1000 1100 1200


Input, kW 4.5 8.5 14.00 21.1 30.00 40.75

This fan is driven by a 4-pole, 230 Vd.c. shunt motor having an armature winding with two
parallel paths and 600 conductors. The flux p er pole is 0.01 Wb and the armature circuit resis­
tance is 0.25 £1 The no load rotational losses are 500 watts. Find the shaft pow er output, operat­
ing speed, armature current and motor efficiency.
Solution. The e.m.f. generated in the armature of d.c. motor is

is in r.p.m.
' • - i f ' -

Counter e.m.f.

Now Vt = Ea + I a ra

r
a r

o o d iiiie u u_y u d i i l o u d i ii it:i


(ArU 4.12
438 Electrical Machinery

kW

Molor
50 charoct.

40

30
Operating
point
20

10

600 700 800 900 1000 1100 1200 rpm

Fig. 4.60. Pertaining to Example 4.28.

2 3 0 - 0.2 n
= (920 - 0.8 n) Amp.
0.25
.*. Shaft power output in watts,
p .k = Ea Ia - Rotational losses
= (0.2 n) (920 - 0.8 n) - 500
= 184n - 0.16 n - 500
or ' ~~Psh = (0.184 n - 1.6 x 10" 4 n - 0.5) kW.
For different speeds, the shaft power output Psh of the motor is calculated in tabular form
as follows :

n, r.p.m. 700 800 900 1000 1100

P »k W 49.1 44.3 33.5 23.5 8.3

The shaft power output versus speed of the motor are plotted on the graph in Fig. 4.60. On
the same graph is plotted the power input versus speed characteristic of the fan. The intersec­
tion of these two curves gives the required operating point, from which it is found that the
operating speed is 1012 r.p.m. and power output of the motor or power input to the fan is 22
kW.
Armature current is Ia = (920 - 0.8 n) = (920 - 0.8 x 1012)

= 110.4 A.
Armature l\ ra loss = (110.4)2 x 0.25 = 3050 watts.
.% Power input = Power output loss + Rotational losses + i l r,
= 22000 + 500 + 3050 = 25,550 watts.

Motor efficiency
= t i i x l 0 0 = 86-2%-
Example 4.32. A 230-V d.c. series motor has an armature circuit resistance o f 0.2 O. and
field resistance o f 0.1 Q. A t rated voltage, the m otor draws a line current o f 40 amps and runs at

x j u u i ii u y w u n i x j u u i ii iVyi
Aril 4.121 D.C. Machines 439

a speed o f 1000 r.p.m. Find the speed o f the motor for a line current o f 20 A at 230 V. Assume
that the flux at 20 A line current is 60% o f the flux at 40 A line current.
Solution. For 40 A line current,
Ea\ = V t - I a (ra + r„)
= 2 3 0 - 4 0 (0.2+ 0.1) = 218 V.
For a line current of 20 A, Ea2 = 230 - 20 (0.3) = 224 volts.
Now flux at 20 A, i.e. = 0.6 times the flux at 40 A

i.e. 02 = 0.6

218 _ (1000) (0i)


or
224 n2 (0.6 0X)
1000 x 224
or n2~ 2 Oi1q w0n.6C = 1713 r.p.m.
8x
Exam ple 4.33. A 15 kW, 230 V, 80 A, 1000 r.p.m. d.c. series motor has the following full
load losses expressed in percentage o f motor in pu t:
Armature circuit ohmic loss (including brush loss) = 2.8%
Field ohmic loss = 2.6%.
Rotational loss = 2.2%.
Neglect the armature reaction and magnetic saturation and assume the rotational loss to
remain constant.
I f the motor draws h alf the rated current at rated voltage, determine
(a) speed in r.p.m. and
(b) shaft power output.
Solution. Full load input = 230 x 80 = 18400 watts.

.-. Total ohmic losses = l\ (ra + r») = (18400) -yjy

or (80)2 (r0 + r.) = 994

or
(a ) Ea, = 230 - 80 x 0.155 = 217.6 volts
and Ea2 = 230 - 40 x 0.155 = 223.8 V.
Since the magnetic saturation is neglected, 0i 80 A and 02 x 40 A.

Eal _nl 4*1


217.6 _ (1000) (80)
or
223.8 n2 (40)

or

(6) Shaft power output - Ea2 la<i - Rotational loss


__ _ ............ J 2 .2 \
= 2 2 3 .8 x 4 0 -(1 8 4 0 0 )

= 8952 - 405 = 8547 watts.

>—
' ■■■■Vyv-* K_/ J ill >—
' v-i■■■■vy■
440 Electrical Machinery_____________________ .__ ________________________”---------- -------------------

Example 4 .3 4 . A40kW , 250 Vd.c. series " T


rated current from 250 V supply mams.jTAesum o f a g characters*
resistance is 0.2 £1. The motor is coupled to drive a load navi £ h r lenstic
given by
TL = 5'fn
where TL is in~Nm and n is in r.p.m'.

Fig. 4.61. Pertaining to Example 4.34.

I f the motor is energised from a 250 V d.c. source, find


(a) the operating speed o f the motor and connected load and (b) the current drawn from the
source.
Neglect magnetic saturation, armature reaction and rotational losses.
Solution, (a) Rated current = — = 160 A
250
At rated load, Ea = 250 - 160 x 0.2 = 218 V.
The rated electromagnetic torque of the motor is
r -^ 2 ^ “ _ (218) (160)
e " «u - ^ t 5 o F x 6 0 = 2 2 2 N m -
Now the relation giving the torqucspeed characteristic of the series motor, must be develop
Since the magnetic saturation is neglected
Te = K aV = Klll '

.% K = 222
(160)2
Also,
Ea = K 2<bn=K2Ian
K ~ 218
2 '(i6 0 H 1 5 0 0 )= 0 0 0 0 9 °8-

S c an n ed by C a m S c a n n e r
D .C . M achines 4 41
Art. * -12l

Note that the values of constants K x and K2 are obtained from rated conditions.
The armature current at any speed is
~ Eg _ 250 - K2lan
r„ ~ 0.2
= 1250 - 5 (0.000908) Ian
= 1250 - 0.00454 Ian
1250
or
/q 1 + 0.00454n
222 (1250)
. . 13530
Te = K xl 2
a=
(160)2 (1 + 0.00454n)2 (1 + 0.00454n)2
The above expression gives the speed-torque characteristic of the series motor.
The various points in the series-motor characteristic are tabulated below and then plotted
in Fig. 4.61.

n, r.p.m 1400 1450 1500 1550 1600 1650 1700

T,. Nm 250 234 221 209 197.8 186.8 177.8

Speed-torque characteristic of the load, obtained from expression TL = 5>//T, is tabulated


below and then plotted in Fig. 4.61.

n, r.p.m. 1400 1450 1500 1550 1600 1650 1700

Tu Nm 186 192 193.5 196.5 200 203 206

The intersection of the series motor and load characteristics, gives the operating point, from
which it is seen that the operating speed of the motor and load is 1591 r.p.m. and the torque is
199.5 Nm.
(6) Current drawn from the source
r _________ 1250_______ _ iei o *
a 1 + 0.00454x 1591 ' '
Exam ple 4.35. A 15 kW, 230 V, d.c. shunt motor has the following data for its m agnetiza­
tion curve at 1500 r.p.m.

1/. amp 0 0.2 0.4 0.6 0.8 1.02 1.15 1.32 1.56 1.92 2.40

Ev volt 6 40 80 120 160 200 220 240 260 280 300

The armature circuit resistance is 0.2 fl and the shunt field has 1000 turns per pole. A t rated
voltage and no load, the armature current is 4 A and the speed is 1500 r.p.m. For an armature
current o f 70 A, the speed is reduced to 1200 r.p.m. by providing series field turns. For long-shunt
connection, compute the number o f these series field turns. Neglect armature reaction and a s­
sume the resistance o f the series field winding to be 0.1 fl.
Solution. At no load,
Ea = Vt - Iara = 230 - 4 x 0.2 = 229.2 V.
The field current required for 229.2 V, from O.C.C., Fig. 4.62, is 1.23 A.
At load, Ea = V ,~ l a (ra + rt) = 230 - 70 x 0.3
= 209 V at 1200 r.p.m.

Scanned by Cam Scanner


[Art. 4.12
442 Electrical Machinery

0*30 0*60 0*90 1*50 1-80 2-40


If in Amp.
Fig. 4.62. Pertaining to Example 4.32 and 4.33.

Ea at a speed of 1500 r.p.m. = 209 x = 261.25 V.


1200
The field current corresponding to 261.25 V, from Fig. 4.62, is 1.575 A.
From Eq. (4.25),
Main field m.m.f., 1.575 x 1000= 1.23 x 1000 + N , (70)
0.345 x 1000
Series field turns, N%= = 5 turns.
70
Exam ple 4.36. D C . shunt machine o f this example, has the same m agnetization curve ai
1500 r.p.m. as that given in Example 4.32.
The armature circuit resistance (including brushes) is 0.2 Q and shunt field has 2000 turm
per pole. The shunt motor speed, both at no load and rated load is 1500 r.p.m. The motor arma­
ture current at rated load is 36 A.

<0,/ i r , a o ° i n v d SpMd °J, r p m -d etermme the »*“ >»« current o f the m otor when
connected to 230 V mams. Neglect armature circuit resistance drop and armature reaction at
no-load.
ib) Determine the effective armature reaction at full load in am pere turns p er pole
(<O A t rated voltage and at rated armature current o f 36 A the speed is to be reduced to 1350
r.p.m. by providing the shunt m otorwUh series field winding. Calculate the required number of
Z m ^ n d ld .™ sen es field resistance to be 0.05 H a n d the m

rur! f n, Ifthd 'T ? P pe r <* armature


current and at rated voltage o f 230 V. Assume series field resistance to be 0.05 a.
(«) I f sen es field turns installed are 20, find the internal starting torque in case the starting
armature current is limited to 50 A and the shunt field current is 1.23 A. Assum e the demag■
netizing effect o f armature reaction to be 200 ampere turns per pole

'j
A fl 4^ 2] D .C . M achines 443

Solution, (a) At no load, Ea = V ,= 230 V, as it is given that armature circuit resistance drop
is neglected.
Therefore, constant shunt field current lf , from O.C.C. of Fig. 4.62 is 1.23 A corresponding
to Ea = 230 V.
(6) At full load, Ia = 36 A.
Ea = 230 - 3G x 0.2 = 222.8 volts.
In Fig. 4.62, point A is plotted with Ea = 222.8 V and If = 1.23 amp. The horizontal distance
between point A and the magnetization curve, gives the effective armature reaction in terms of
shunt field current. Its value is 0.06 A.
Armature reaction in ampere-turns per pole
= 0.06 x 2000 = 120
(c) At rated load, with series field winding in circuit,
Ea = 230 - 36 (0.2 + 0.05) = 221 V at 1350 rpm

Ea at 1500 r.p.m. = 221 x = 245.5 V.


looU
From Fig. 4.62, Ea = 245.5 V requires If = 1.365 A. From Eq. (4.27),
1.365 x 2000 = 1.23 x 2000 + N s (36) - 120
65
Series field turns, N s = — = 11 turns per pole.
b
(d ) If series field winding has 20 turns, then net field m.m.f. from Eq. (4.27) is
(1.23 x 2000 + 20 x 36 - 120) ATs.
The net field m.m.f. in terms of the equivalent shunt field amperes is
1.23 x 2000 -h 20 x 36 - 120
lf ~ 2000
= 1.23 + 0 .3 6 - 0 .0 6 = 1.53 A.
From Fig. 4.62, the value of Ea corresponding to If = 1.53 A is 258 V at 1500 r.p.m. But
counter e.m.f. Ea corresponding to rated voltage and rated current is 230 - 36 (0.2 + 0.05) = 221
volts. Therefore, the motor speed n corresponding to Ea = 221 V is
221 _ n
258 1500

or n = | | | x 1500 = 1285 r.p.m.

(e) The net main field m.m.f. = 1 2 3 x 2000 + 20 x 50 - 200


The net field m.m.f. in equivalent shunt field current is
1.23 x 2000 + 20 x 50 - 200 „ A
I t = ------------------------------ = 1 -D O A .
2000
From Fig. 4.62, corresponding to 1.63 A field current, Ea at 1500 r.p.m. is 264 volts.

But Ea = Ka ^ o)m
264 x 60 _ 264
a ^ ~ 2n x-4500 50n
264
Starting torque, Test = K a $ I a = n x 50 = 89 Nm.

IVyV* K_/ '


444 Electrical M achinery (Art. 4.13

Exam ple 4.37. A 230 V, 250 rpm, 100 A separately-excited dc m otor has an armature resis­
tance o f 0.5 £1 The motor is connected to 230 V d c supply and rated dc voltage applied to the field
winding. It is driving a load whose torque-speed characteristic is given by TL = 500 - 10 ^
where co is the rotational speed expressed in rad/sec and T f is the load torque in Nm. Find the
steady state speed at which the motor will drive the load and the arm ature current drawn by it
from the source. Neglect the rotational losses o f the machine. [GATE, 2002 j
Solution. At rated load, motor counter e.m.f., Ea = V ,~ Iara
or K m cor = 230 - 100 x 0.5 = 180 V
where <or = rated motor speed in rad/sec

.*. Motor constant, ” 180 x 60■V-s/rad


m 271 x 250
Armature current at any speed co is given by
V ,-E a 2 3 0 - K m co
0.5

.•. Motor torque, T' = K n l a = f ± [ 2 3 0 - K m ml

Under steady state, motor torque Tt = load torque, TL


Km
or — [230 - Km co] = 500 - 10 co

or 230 180 x 60 f 180 x 60 1


0.5 " 2ti x 250 “ 0.5 2ti x 250 •co = 500 - 10 co
or 3162.73 - 94.545 co = 500 - 10 co
or co = 3162.73 - 500 = 31.495 rad/sec
84.545
.'. Speed 3 1 .4 9 5 x 6 0
= 300.75 rpm
2 tt
From Eq. (i), armature current is

/ =— oon 1 8 0 x 6 0
2 o 0 —— x 31 = 26.913 A.
° 0.5 2 t i x 250
4.13. D.C. Motor Starting
At the time of starting, the motor speed is zero, therefore counter Pmf j? v >. • ,
zero. Consequently, for the armature circuit the voltagp f • • tz ° ° (°m^1S 3 0
motor and Vt = 0 + / (r + r ) for both ls V ‘ = 0 + ^ for shunt
i . ( « + r.) tor both series and compound motors. With rated applied voltage

the starting armature current is, therefore, ^ for shunt motor and J * for both 3eries and

compound motors. Since the resistances r and (r 4. »• \ « i_ 4 a


large starting armature current from the supply mains Fo'rT S.maUer' m0t° r dr3WS
motor may have armature circuit resistance equal to 0 2 O If 3 10 kW ’ 250*V shunt
q to 0 2 “ • switched on directlv its starting
armature current would be ^ = 1250 A « ,h » o airectiy, its starting
q , , . 0.2 1250 A >whereas the rated armature current is only 40 A.
Such heavy inrush of starting current taken by the motor may result in
(i) detrimental sparking at the commutator,

<«•> damage to the armature winding and deterioration of the insulation due to overheating.

S c an n ed by C a m S c a n n e r
j

Art- 4.131
D.C. M achines 445

th0 motOT and the\oad°and° nnd qUiCk accclcratio" - whi<* may damage the rotating parts of
(iv) large dips in the supply voltage.

In view of this, the armature current must be limited to a value that can be commutated
safe y, y mser ing a suitable external resistance in the armature circuit. As the motor ac­
celerates, coun er e.m. . Eu is generated in the armature and this decreases the armature cur­
rent to a sma 1 value. Thus the external resistance inserted in the armature circuit should be
gradually decreased, as the armature accelerates. If this additional resistance inserted is left
in the armature circuit, it would result in
(i) reduced operating speed of the motor and
(it) additional energy loss and, therefore, reduced efficiency.
A precaution should be observed while starting shunt and compound motors. In these
motors, at the time of starting the field excitation should be maximum, because of the two
reasons given below :
(i) A large field current would result in low operating speed. Consequently the time re­
quired from standstill to this low speed is less and, therefore, there is less heating of the arma­
ture during starting.
(ii) Motor torque required to overcome the friction and load torque is proportional to the
product of If and Ia. Since lf is kept at its maximum permissible value, the armature current
during starting would be minimum for a given load torque. This minimum starting current
would further result in improved commutation during motor and load acceleration. Thus the
rheostat, in series with the shunt field winding, should be at zero resistance position at the time
of starting the d.c. shunt and compound motors. In case of small d.c. motors, no starting resis­
tance in the armature circuit may be required.
4.13.1. Shunt and compound motor starters. The primary function of a starter is to
limit the starting current in the armature circuit during starting
and accelerating time of the motor. The simplest type of starter con­
sists of a rheostat inserted in series with the armature circuit as
shown in'Fig. 4.63. The simple starter is however modified to in­
clude a few protective devices, such as overcurrent release, no-volt
release etc. There are two standard types of starters for shunt and
compound motors. These are, three-point starter and four-point
starter. The four-point starter is used when wide range of speed by
shunt field control is required. When no (or a little) speed control Fig. 4.63. Starting rheostat in
is required, either type of starter may be employed. the armature circuit of a d.c.
shunt motor.
Three-point starter. A three point starter, with its electrical
connections and protective features is illustrated in Fig. 4.64, within the dotted lines. The
starter is shown connected to the supply mains and d.c. shunt motor. Since only three terminals
(L, A, F) are available from the starter, it is called a three-point starter. The starter terminals
L (line), A (armature) and F (field) must be connected respectively to the supply terminal (posi­
tive or negative), motor armature terminal (any of the two armature terminals) and shunt field
terminal (any of the two field terminals).
When the motor is at rest, starter handle H is kept in the off position by a strong spiral
spring. One stud marked “OFF” in Fig. 4.64 indicates that motor is disconnected from the sup­
ply. The starting resistance is connected between contact studs 1, 2, 3, 4, . . . 6 . For starting the
motor, the handle is rotated to come in contact with stud 1. As soon as handle H touches stud
1, the’shunt field and holding coil HC get connected in series across the supply, whereas the
armature gets connected in scries with the entire starting resistance. Since the current begins

S c an n ed by C a m S c a n n e r
[Art. 4.13
4 46 Electrical Machinery

to flow in both the field and armature windings,


the motor starts rotating. After the armature has
picked up sufficient speed, the handle H is moved
to stud 2, thereby cutting out the resistance be­
tween studs 1 and 2. Movement of the handle is
continued slowly till the soft iron keeper touches
the holding magnet. During the waiting period
on each stud, the armature current falls and
speed rises exponentially, see Fig. 4 .6 9 . This fig­
ure also reveals that waiting period on first stud
is more and it reduces progressively for other
studs in sequence. The field current flowing
through HC, produces force of attraction greater
than the spring pull. In view of this, the handle
H is held in ON position and the entire starting
resistance is cut out. The holding magnet or coil
HC is also called no-volt release or low-voltage
release and has the following functions.
(i) In case of power failure, electromagnet
HC gets demagnetized and the spiral spring
brings the handle back to its OFF position. If, Fig. 4.64. Three-point starter connected
to a shunt motor.
during the power failure, the handle fails to
return to OFF position, the motor might be damaged in case the power is restored— because
then there would be no starting resistance in the armature circuit.
(ii) If the shunt field becomes open circuited accidently, H C gets demagnetized and the
starter handle is returned by spring pull to the OFF position. In case the starter handle is not
released, the speed starts increasing with the decaying of the field flux. The small residual flux
will result in dangerously high speeds. At the same time for providing a given load torque, the
armature current, starts increasing and in case the fuse or circuit breaker does not disconnect
the motor from d.c. supply mains, the armature may get damaged.
(iii) With this arrangement, the starting resistance in the armature circuit can either be
zero or full. The handle can’t be left on any intermediate stud willingly or unwillingly.
An examination of Fig. 4 . 6 4 reveals that under running conditions, the field winding hold­
ing coil H
Cand the starting resistance are in series. Since the starting resistance is’mu
smaller in comparison with shunt field resistance, its effect on the shunt field current is negli-
gib e. But the starting resistance does influence field circuit, at the time of switching off the
w h h ih T m mT V S ^ c o n n e c te d from‘ he suPP*y the field circuit comes in series
with the holding coil, starting resistance and armature circuit. The field energy stored in the
shunt field thus gets discharged in this local series circuit, as the handle returns to the OFF
position. During the time the starter handle travels from ON position to stud 1, most of the field
energy stored ,s dissipated. Consequently, when the starter handle leaves stud 1 there is no
danger of high voltage induced in the field winding and of excessive sparking at stud 1

a r m ^ r ir c u fT O s 6 M “ S T ! * the ° " ‘ ' !oad rdease is P ™ ided ” “ ries with the


armature circuit This OR is a small electromagnet. In case the armature current exceeds a

pP" t “oeneUeen ? ^ t t f f * " Z ™ ^ ^ and attrarta the movable‘softTron M


HC get short circuited In t h i f ^ £ attrad;ed- two terminals a, 6 of the no-volt release
back to the OFF Dositinn hv / nail^ier» ^ gets demagnetized and the starter arm H is pulled
from the m a ^ s hi cafe nf nv ^ SpnnS ~ the motor is thus automatically disconnected
from the mains m case of overload. The motor current at which the overload release OR should

s c a n n e d Dy u a m b c a n n e r
.1

Art. 4.13] D.C. Machines 447

operate, can be adjusted merely by varying the distance between the electromagnet and mov­
able soft iron M.
D isadvantages. In case of a three-point starter, the field circuit and the hold coil are in
series. If speeds above the normal are to be obtained, the field current must be reduced. At a
certain value of reduced field current (therefore, increased motor speed), the electromagnetic
pull of the holding coil may become less than the spring force. In such a case, the starter handle
returns to the OFF position and the motor stops. Thus a three-point starter can’t be used where
wide range of speed control, by shunt field control (or field weakening method) is required. This
undesirable feature can be overcome in four-point starters.
F our-point starter. As the name suggests, four terminals (L, L, F, A ) are available from
this starter. In Fig. 4.65, a four-point starter is
shown connected to d.c. supply mains and a d.c.
shunt motor. Under normal running conditions
with starter handle in the ON position, the hold­
ing coil HC is in series with the starting resis­
tance and an additional resistance R as shown in
Fig. 4.65. The function of resistance R is to
prevent short circuit of the supply mains, in case
the overload release OR operates. When HC gets
short circuited by OR, the current through R is
limited by its own resistance and the starting
resistance.
The other components of the four-point
starter are the same as in a three-point starter.
The shunt field winding in series with the start­
ing resistance, now permits wide speed control Supply
by field rheostats inserted in the field circuit.
Note that four-point starter permits the change
of field current by field rheostat, without effect­ Fig. 4.65. Four-point starter connected to
ing the holding coil current. Therefore, the pull of a shunt motor.
the holding magnet is unaffected and remains
more than the spring pull for any value of speed. In view of this, four-point starters are more
popular with speed controlled motors. When little or no speed control is desired, either three-
or four-point starter may be used.
‘ 4.13.2. Series m otor starters. In series motors also, a starting resistance is inserted in
series with the armature, for limiting the starting current to a safe value. As usual, thi£ start­
ing resistance is cut out gradually as the motor accelerates. Fig. 4.66 (a) illustrates k series
motor starter used in applications, where the removal of load is remote. Holding coil in series
with protective resistance R, acts as no-volt release, like the no-volt release of three-point or
four-point starter. In view of this, starter of Fig. 4.66 (a) is referred to as the no-volt release
type o f starter.
In case there is a possibility of removal of or reduction in load, starter illustrated in Fig.
4.66 (b) may be used. In this, the load current passes through the series field, armature and
holding coil. The holding coil consists of a few turns which are capable of carrying the load
current In case of removal of load or reduction in load below a safe value, the line current
flowing through the holding coil is reduced. This* reduced value of current decreases the
strength of the holding magnet and the spring pull \brings the starter handle to OFF position.
This type of starter is, therefore, referred to as the rio-load release type o f starter.

by C a m S c a n n e r
448 Electrical M achinery (Art. 4.13

. , - . W .w cu . o c a s c

Note. A d.c. motor should not be stopped by forcing the starter handle to the OFF position
If it is done, then dangerous sparking is caused at stud 1, because here the field circuit is broken
and the entire stored magnetic field energy is dissipated in the form of heavy spark.
Autom atic Starters. Push-button type of automatic starters are used quite often in in-
ustry. Even an inexperienced operator, with the help of auto-starters, can start and stop the
motor without any difficulty. y
The operation of these automatic starters depends upon, either the time delay or the
counter e.m.f. developed across the armature terminals. Here only the basic principles of the
counter e.m.f., automatic starter are presented.
C ounter e.m .f. starter. When the motor is_ switched on, the counter e.m.f. developed
across the armature terminals is zero. Contactors 1 and 1 are normally open and there­
fore, resistances and R2 are in series with the armature at the time of starting. As the motor

v X e t A ^ V o C - s en” uivn eCreaSeS- ^ *» the operating


relay A, it operates and closes the
normally open contact 1 A, thus
cutting out the starting resis­
tance Ri- With the motor speed
still rising, the counter e.m.f. in­
creases till the operating voltage
of voltage-sensitive relay B is
reached. At this instant, relay B
operates and it closes the nor­
mally open contact 1 B. Conse­
quently the starting resistor R2 is
cut out and the armature gets Fig 4.67. Counter e.m.f. automatic .tarter connected
connected directly across the to d.c. shunt motor,
supply mains as shown in Fig. 4.67.

* A contactor is a heavy duty relay designed to open or close nn electrical power circuit.
Art. 4.13] D .C . M achines 449

The disadvantage of counter e.m.f. automatic starter is that if the motor fails to start, the
counter e.m.f. remains zero, the voltage-sensitive relays can’t operate and as a result of it, the —
starting resistance may burn. Such occurrences can be avoided by employing definite time-limit
starters but this will not be described here.
D.C. Sh un t m otor starter design. Starting resistances between the various studs of a
shunt motor starter, should be graded and not made equal. This grading of starting resistances
is essential in order to avoid the abnormal rush of armature current on the last few steps,
particularly on the last stud.
During the starting process, as the starting resistance elements are cut out manually or
automatically, the maximum armature current 70j should be such that it can be commutated
satisfactorily by the brushes and commutator. At the same time, the minimum armature cur­
rent Ia2 should be able to develop electromagnetic torque Te sufficient to accelerate the load
torque TL. In other words, the armature current during starting process is taken to fluctuate
between fixed limits / ol and Ia2. The armature reaction and the armature inductance are
neglected during the motor starter design.
Fig. 4.68 illustrates a d.c. shunt motor with n resistance elements or (n + 1) studs. There­
fore, this starter is called n-section, n-element, n-step or (n + 1) stud starter. The “off” stud has
not been included in n + 1 studs. Note that resistance R 1 = + r2 + r3 + ... ■+ rn + ra ;
R2 = r2 + r3 + ... + rn + ra and so on and Rn + i = ra, armature circuit resistance (including brushes).

Fig. 4.68. Pertaining to d.c. shunt motor starter. Studs are


numbered 1, 2, 3 n + 1 for n resistance elements.

At the instant the motor is switched on, the total armature circuit resistance R x should be
equal to
____________ Terminal voltage______________ Vj_
1 ~ Maximum permissible armature current " Ial
= rl + r 2 + r3 + ... + rn + ra ...(4.46)
With the handle on stud 1, the motor accelerates, counter e.m.f. develops and as a result of
the armature current starts decreasing from Ial. When the current has dropped to minimum
current Ia2, then the counter e.m.f., with the handle on stud 1, is given by,

Eai = Vt - Ia2R\

0r = Vt' Ea± ...(4.47)


1a2

ii ic u u y w a n u t a i 11 ici
[Art 4.13
450 Electrical Machinery

At stud 1, as soon as the current drops to I a2, the resistance r x is cut out by moving the
handle to stud 2. During the notch in g-u p p rocess (process of cutting out Resistance) from stud
1 to stud 2, the speed and, therefore EaX («= (om<t>) do not change. a ,J i is cu
out, the current shoots up again to IaX. Therefore,
V, - EaX
R 2 - — j
1a\
= r2 + r3+ ... + rn + ra ...(4.48)

From Eqs. (4.47) and (4.48),


^2 hi Minimum armature current
“ Ial ~ Maximum armature current

Following the above procedure, we get


la2
In'? R-2 _ ^ 3 _ ^ 4 _ _ Rn _ R n - t-1 _
■= a ...(4,49)
R~x - R ~ 2 - R 3 ~ ' ’ '’ ~ R
R n -- l! ~ Rn R.
la !
1, 2, n—
3................n - 1.
1, n
It is seen froih Eq. (4.49) that the ratio of minimum and maximum armature currents is
equal to the ratio of total resistances on two adjacent studs.

If ^ are multiplied with each other, then from Eq. (4.49)


R i'R o R,
Rn R 3 R a R n -f 1
“ =T
riiT x T or x K'
T rk x R n- 1 R r

R 71+1
Ri Rl
-.1/71
a= ...(4.50)
hi Rl
If rQ, R x and the ratio a are known, then the number of resistance elements n can be ob­
tained.
The magnitude of various resistance elements rlf r2, r3 rn_ lf rn ; can be obtained as fol­
lows :
From Eq. (4.49); i?2 = i?i a ; rl ~ R\~ R 2 = i?i (1 —Ot) ...(4.51)
R3 = R 2 a = R x a2 ; r2 = R 2 —R 3 = R 2 (1 —a)
= a R x (1 - a) = a r x ...(4.52)
= R 3 a = R 2 a 2 = R xa 3
r3 = R z~ R i = R$ (1 - a)
= R\ (1 - a) a 2 = a V j = ar2 ...(4.53)
Similarly,
rn - l = a rl = a ’ rn-2
rn = a " " 1 r x = a •rn - 1 ...(4.54)
Usually the maximum armature current Ial on each stud is limited by commutation considera­
tions. I f /ol is known, as is usually the case, then it can be included in Eq. (4.50) ns follows :
\l/n vl/n \l/n
(I.ai ‘ ra hl^a
a =
h iR i V,

S c a n n e d by C a m S c a n n e r
Art. 4-131 D.C. M achines 451

l/n
Max. permissible armature current x ra
Vt
l/n
a = Voltage across armature at starting (4 5 5 )
or
Applied voltage V,
If minimum armature current Ia2 is specified, then Ia2 can be included in Eq. (4.50) as
follows :
l/n l/n
a= ( V k l (ra Ial)
V,
v y V

From Eq. (4.49), 7al =


a

a2
a=
Vt a

or a Ct= ^ / a2
l/n + 1
rc/a2
a= ...(4.56)
v.
If upper (/al) and lower (I a2) limits of armature current are known during starting, then the
design of shunt motor starter can be carried out as under :
(i) Compute from Eq. (4.46).
(ii) From Eq. (4.50), compute the number of steps n choosing the nearest whole number.
(iii) Now use Eq. (4.54) for computing the magnitude of various resistance elements r1( r2,
etc.
The design of d.c. series-motor starter, being somewhat more involved, is not dealt with in
this book.
Exam ple 4.38. In a 230 V, 10 kW d.c. shunt motor, it is required that the starting armature
current should not exceed twice its rated armature current. During the starting o f the motor, the
starting resistance is cut out in steps, as soon as the armature current drops to its rated value.
The field resistance is 115 ft and the total armature circuit resistance is 0.348 ft. Neglect arm a­
ture inductance.
(a) Find the external resistance required at the time o f starting the motor.
(h) Determine the value o f the first resistance element that must be cut out, when the arma­
ture current drops to rated value.
(c) Find the external resistance to be cut out in the second step.
(d) Find the total number o f steps required and their resistance values.
230
Solution. Constant shunt field current = = 2 A.
110

Rated armature current = ^^30 “ 2 = 41.4 A.

(a) External resistance required at the time of starting


V,
_ r , + r2 + ... + ra 2 x 4 1 .4 - rn

230
- 0.348 = 2.432 fi.
2 x 4 1 .4

>canned by C a m S c a n n e r
[A rt 4.13
452 Electrical M a c h i n e r y --------------------------------------------- ------ ------------------------------------------
, A i. j at Rtud 1. then the counter e.m.f. is
(b) When the armature current drops to rated current, >
Eal = Vt - 41.4 (external resistance at stud 1 + ra)
= 230 - 41.4 (2.432 + 0.348) = 114 9 V.
At the instant the handle is moved to stud 2. armature current shoots up to 2 (41.4) A, but
the counter e.m.f. remains at 114.9 volts.
Therefore, at the second stud,
Vi Z E2 i _ 2 3 0 ^ U 4 19 = 1 3 9 n
r2 + r3+ . . . + rf, + ra - 2 x 4 1 4 - g2 g

.•. External resistance r2 + r3 + ... + rn = 1.39 - r a - 1.042 fl.


.*. The resistance that must be cut out in first step is
rj = 2.432 - 1.042 = 1.39 fl.
(c) At second stud, when armature current drops to rated current, then counter e.m.f. Ea2

is
E a2 = vt- 41.4 (r2 + r3 + ... + r„ + ra)
= 230 - 41.4 (1.39) = 172.4 V.
When the handle is moved to third stud, Ea2 remains at 172.4 V, but armature current
becomes 82.8 A.
. +r +r _ 230 ~ = 0.695 fl
" r3 + - r" “ 2 x 4 1 .4 82.8
.-. The resistance that must be cut out in second step is
r2 = 1 .3 9 -0 .6 9 5 = 0.695 fl.
id) At third stud, when armature current falls to 41.4 A, then
Ea3= V ,- 4 1 .4 (r3 + ... + r n + ra)
= 230 - 41.4 (0.695) = 201.15 V.
When the handle is moved to 4th stud, then
Vt - EaZ 28.85 n
r4 + ... + rn + r„ 2 x 41 4 g2 g 0.348 fl ra.

Therefore, when the handle is moved to fourth stud, the armature resistance ra = 0.348 O,
is sufficient to limit the current to 82.8 A.
Since there are four studs, three resistance elements are required. The resistance of the
third elements is
= 0.695 - rfl = 0.347 fl.
Thus the resistance of the three steps or elements r l t r2, r 3 are respectively 1.39 fl,
0.695 fl, 0.347 fl.
Exam ple 4.39. A 240 V, 50 A, 1500 r.p.m. d.c. shunt motor has an armature resistance of
0.2 fl. For this example, shunt field current is neglected.
(а) For a four step starter (four resistance elements and five studs), calculate the values of
the various steps, with armature current not exceeding 1.40 p.u.
(б) I f counter e.m.f. starter is employed, find the voltages at which the contactors should
close.
(c) Sketch the approximate variations o f armature current and speed, during the a c c e le r a t­
ing process o f the motor.

.J
Scanned by Cam Scanner
Art. 4.13]________________________ D .C. M achines 453

Solution. Base voltage V, = 240 V = 1 p.u.


Base armature current Ia = 50 A = 1 p.u.

Base resistance R b = j*ase voltage _ 240 _ 4 g0 ^


Base current 50
Per unit armature resistance
0.2
r“ = « 0 = 0 0 4 1 7 -
(a) At the instant the motor is switched on, the total resistance R u in the armature circuit
should be given by
Vt i
R l = T L = - ± - = 0.714 p.u. or 3.43 £1.
iai 1 4
l/n U /4
f 'r a ' f0.0417
From Eq. (4.50), a = = 0.492.
1 0.714
From Eq. (4.51) ; r1 = R 1 ( 1 - a)
= 0.714 (0.508) = 0.363 p.u. or 1.742 £1.
From Eq. (4 .5 2 ); r2 = a r l
= 0.492 (0.363) = 0.1788 p.u. or 0.858 £1.
Similarly r3 = ar2
= 0.492 (0.1788) = 0.089 p.u. or 0.427 £1.
and r4 = a r3
= 0.492 (0.089) = 0.0438 p.u. or 0.210 £1.

(6) a _ 12*2. = ^ = 0.492


i/ n a x ia l
Ia2 = 0.492 x 1.4 = 0.689 p.u.
or l a2 = 34.45 A.
Let the time be reckoned from the instant the motor is switched on. At stud 1, suppose the
armature current drops to l a2 = 0.689 p.u. after time £j. Then the counter e.m.f. at is
E al = Vt - I a2R l = l - 0.689 x 0.714 = 0.508 p.u.

Voltage across the armature terminals at instant t l is


V * ! = E al + Ia2 ra = 0.508 + 0.689 x 0.0417
= 0.537 p.u. or 129.00 volts.

Therefore, the first contactor should close at 129 volts.


At stud 2, suppose the cu r re n t falls to Ia2 after time (2. then the counter e.m.f. is

Ea2= V , ~
= 1 00 - 0.689 (0.714 - 0.363) = 0.758 p.u.
,. V ^ E ^ l ^ a = 0*758 + 0.02870
= 0.787 p.u. or 187.2 volts.
At stud 3, when the current reduces to I a2 after time £3, the counter e.m.f. is

E a3 = V t - / a2# 3 = v t ~ 4 2 ( ^ 2 “ r 2)
= 1.00 - 0.689 (0.351 - 0.179) = 0.881

’ Vat3 = + 7fl2r° = ° ' 881 + 0 02870 = 0,91 0F 218-5 V° ^ S-

S c an n ed by C a m S c a n n e r
[A rt. 4.14
454 Electrical Machinery

Similarly at stud 4, Ea4 = Vt - Ia2 R 4 = Vt - Ia2 (i?3 - r3)


= 1.00 - 0.689 (0.172 - 0.089) = 0.9428
and VoM = Ea4 + Ia2 ra = 0.972 p.u. or 233 volts.
Finally, with the motor running at full load,
Ea = V , - I ara = 1 - 1.0 x 0.0417 = 0.958 p.u.
Thus the contactors, say LA, IB, 1C and ID, should close at 129,187.2, 218.5 and 233 volts
respectively.
(c) At full load, counter e.m.f. Ea = 0.958 p.u. corresponds to rated speed of 1500 r.p.m.
Now Ea = Ka iJ>o)m
But n = 1500 r.p.m. = 1 p.u.
.-. 0.958 « Ka 0 •1
At time f1( EaX « Ka <J>nj

.-. =
= = 0.531 p.u. = 796 r.p.m.
'a 0.958
0.758
Similarly n2 = = 0.792 p.u. = 1189 r.p.m.
0.958
0.881
n, = - 0.92 p.u. = 1380 r.p.m.
0.958
0.9428
and «4 = = 0.984 p.u. = 1477 r.p.m.
0.958
Approximate shapes of armature current (and electromagnetic torque also, because
Te oc Ia for constant flux motor) and speed, with respect to time are shown in Fig. 4.69.

' J500
^ _
1477
1380 1
1 1
1189
/ 1 1 l
1 1
/ *
796 1 l
1 1
/ 1 1
1 t
/ 1 1
/ 1 1 1 1
L 1 1 J -----1---------
0 *1 \ *3 *4 TIME
(6)
Fig. 4.69. Variation of (a) armature current and (6) speed ;
with respect to the starting time o f a d.c. shunt motor o f Example 4.36.
4.14. Speed Control of D.C. Motors
The term ‘speed control’ stands for intentional speed variation, carried out manually or
automatically. Natural speed change due to load, is not included in the term ‘speed control’.
D.C. motors are most suitable for wide range speed control and are, therefore, indispen­
sable for many adjustable speed drives.
The speed of a d.c. motor is given by Eq. (4.34) and it is re-written here for convenience, i.e.
V t-Ija
- < 4 -34>

PZ
where armature constant K a = 7^ - and 0 is the field flux per pole.

S c an n ed by C a m S c a n n e r
Art. 4.14]__________________________________ D.C. Machines 455

It follows from Eq. (4.34) that for a d.c. motor, there are basically three methods of speed
control and these are :
(i) Variation of resistance in the armature circuit,
Hi) Variation of the field flux, and
(iii) Variation of the armature terminal voltage.
Before describing these methods, it is preferable to define the terms base speed, speed
regulation, speed range, constant power drive and constant torque drive.
Base Speed . It is defined as the speed at which a motor runs at rated armature voltage
and rated field current. Base speed is equal to the rated speed or nameplate speed of the motor.
Speed regulation. If the speed-change from no load to full load is Aa)m then speed regula­
tion is defined as the ratio of Aiom to rated speed (or base speed) (om.

Per cent speed regulation = x 100 ...(4.57)

Speed range. It is defined as the ratio of the maximum allowable speed to minimum al­
lowable speed of the motor. When the speed range of a motor is specified, it must be mentioned
whether this speed range is at no-load, full load or a fraction of full load.
Constant power drive. If the motor shaft power (shaft torque x speed) remains constant
over a given speed range, the system is called a constant power drive. Note that in constant
power drive, higher torques are available at lower speeds and lower torques at higher speeds.
The motor size is always decided by the highest torque requirement at the lowest speed.
Constant torque drive. If the motor shaft torque remains constant over a given speed
range, the system is called a constant torque drive. Note that in constant torque drive, shaft
power varies as the speed varies.
4.14.1. Speed control by varying the armature-circuit resistance. This method is
also called armature-circuit-resistance control method. In this method, an external resistance
is inserted in series with the armature circuit to obtain speeds below the base speed only.
Shunt motor. The scheme of connections of a shunt motor is illustrated in Fig. 4.70 (a),
where resistor Rg, called a controller, is put in series with the armature circuit. Note the dif­
ference between a starter and a controller, the former is designed to carry current only for a
short time, whereas a controller can carry current for an indefinite time, without getting exces­
sively hot.
When R is not present, then the armature current Zol, from Eq. (4.34) is
Vj —Ka if) o)mi
lal = ~
ra
When R is inserted in the armature circuit and if it is assumed that there is no change in
speed for the time being, then
/ Vf ~ Zfa<(xomi __ T ra
al ra + Rg ol ro + Rg
T k fipld flux <t>remains unchanged, therefore, with the reduction of armature
c u ^ n L t , toraue T, ,= * * / . ) decreases from t o * ^ ,.S i n c e
T. has become less than constant load torque, the ^ e ^ d ^ r e a s ^ c o u n t e r (or back) e.m.f. also
increases till it becomes
decreases. As a result of it, armature current Ia -■ -nerc

"Scanned by Cam Scanner


4 56 Electrical M achinery (A r t 4.14

* *6. uiiuuk iiiuwi lum iui uy v a i y m g u ic a i m a i u i u cixcuit i eaiaiau ce


(a) Schematic circuit diagram and (b ) speed-torque characteristics.

equal to its initial value Ial, so that the initial electromagnetic torque Ka Ial is developed
again.

Vt - I ai r„ Eal
From Eq. (4.34), “ ml -
KJ Ka<j,
When new steady state condition is reached, with Rg in the armature circuit, then

_ V I ~ I q l (ro + _ Ea2
m2 KA Ka§
“ m2 _ n 2 _ ^ a 2 _ ^ t ~ I a l ( r a + R g )
“ ml ^1 Eal
£ ol Vt ~ I a l ra ...(4.58)
Eq. (4.58) shows that tom2 is less than coml.

fo lio ™ ? ’ f0r thiS type 0f Speed C0ntr01 and With 3 Constant load tor<lue’ it can be concluded as

doesPly I T * ” A ' S° tha


P, = whether f l , is in the armature circuit or not * remai" S C° nStant 3‘
(6) Power delivered to load is P 2 = Ea2 = [V, _ / , (r , p u T » / ,
torque,X <o„2. It is thus seen that the powe P 2 de ivered t l bad f c =
decrease in speed. 2 aellVered to load decreases in proportion to the

The efficiency of this method of speed control is

1 _ r i ± RK
Vflal al
Vt
Rg i® increased to obtain lower operating snnnrla • .
results in higher operational costs. efficiency is lowered and this

reduced atove M d ^ /p o o r i p e e d r e m u " '3' operational costs at


R,in the armature circuit. For example, for a controller re sist^ c e " , 1 ^ "

1
J
Art. 4.14]
' D.C. Machines 457
®mi at a certain load torque and for the same 7? fv, au i , ,
shown in Fig. 4.70 (6) g3> the speed becomes almost Gimo at no load as

The principal advantage of this m p f W ,


speeds of only a few r.p.m. are eas'l ht ■ 1S speeds below base speed down to creeping
at reduced speeds ; this method is* ^ ° ai^a^^e’ ®ut>because of considerable waste of energy
slow downs are required. economically viable where only short time or intermittent

Exam ple 4.40. A 200 V dc shunt , i. oo


rpm. Its field resistance is 100 Cln H amperes at rated voltage and runs at 1000
Compute the value o f additional rJli armature circuit resistance (including brushes) is 0.1 Cl.
to 800 rpm ; when S ° nCe re1uired in the armature circuit to reduce the speed

m ‘the W T U eU
^dependent °fSpeed in « reciprocating pump),
(.b) the load torque is proportional to speed,
M th e load torque varies as the square ofihe speed (as in a fan-type load),
(d) the load torque increases as the cube o f the speed.

Solution. Constant field current I. = —9 a


f 10Q ^ A.

.-. Armature current 4 i = 22 - 2 = 20 A

by armature resistance — d

the speeds^.e°ad t0rQUe 1S lndePendent of speed, the electromagnetic torque is constant at both

= E a ^1 Ial ~ Ea $1 Ia2
OT Ial ~ Ia2 = 20 A.
At 1000 rpm,the counter emf, Eal = Vt - I al ra = 200 - 20 (0.1) = 198 V.
At 800 rpm, the counter e.m.f. Ea2 = Vt - 2 0 (0.1 + Rg)
where Rg is the additional resistance inserted in the armature circuit.

N° W m
. ^o2 _ K a <t>l “ m2 _ n2 ■
E al K a 4>1 “ ml ” ^1
or 200 - 20 (0.1 + Re) ^ 800
198 _ 1000
Its solutiongives Rg = 1.98 Q
Loss in Rg = (20)2 (1.98) = 792 watts.
(b ) Here load torque TL « speed n.

But electromagnetic torque Te = K a


&

Under steady state,


ii

i.e.
K <l>i h\ ~ n \
and K a 4>1 Ia2 * n 2

or Ia 2 _ * h
1
>-•
M

or
O

= 16 A
II
<N

1000

cann ed by C a m S c a n n e r
4 58 Electrical M achinery (Art. 4.14

Now £ o2 = 2 0 0 - 16 (0.1 + R g)
and Eal = 198 V.
Ea2 200 - 16 (0.1 + Re) 800
Ea 1 198 1000
Its solution gives Rg = 2.5 ft.
Loss in R„ = (16)2 •(2.5) = 640 watts.
(c) Here load torque TL « (speed)2

Now as in part (6), Ka <{>, Iai n\


and Ka ^1 I q2 ^ n2
f V2
o2 n2
or

( 800
/«2 = ( 20) = 12.8 A.
1000
Now Ea2 = 2 0 0 - 12.8 (0.1+J ty
Ea2 2 0 0 - 12.8 (0.1 + JO
Sl _ 800
Thus
1 198 1000
Its solution gives Rg = 3.15 ft.
Loss in R„ = (12.8)2 (3.15) = 517 watts.
(d ) Here load torque«« (speed)3
As in part (6 );
Ka <h Ia\ Oh)3
and K a <f>i Ia2 (n2f

or 800 Y
/ a2 = (20) = 10.24 A.
1000
V
Ea 2 _ 200 - 10.24 (0.1 + Re)
I’ i - 800
E ai ~ 198 1000
Its solution gives Rg = 3.96 ft.
Loss in R'g -(1 0 .2 4 ) (3.96) = 415 watts.
i Thl® examP,e illustrates that if the armature resistance control method is employed for
loads whose torque requirement varies as the square or cube of the speed the loss in Re is
reduced considerably resulting in higher operating efficiency at reduced speeds Even then it
can t be stated whether this method is economical enough for such type of loads if reduced
speeds for longer periods are required.

n o o X/r7 iPi e, 4 ,4 1 * ^ 24°i 800f P m dc shunt motor has arm ature circuit resistance of
0.2 i t I f load torque is reduced to 60% o f its full-load value and a resistance o f 2 ft is inserted in
series with arm ature circuit, find the motor speed. Armature reaction weakens the field flux by
4% at fu ll load and by 2% at 60% o f full load.
S olu tio n . At rated load, Eal = Vt - Ial ra = 240 - 50 x 0.2 = 230 V
Also Eal « JVj <|>j
Rated torque TLt = Ka ^ I al = K a ^ x 5 0

bcannedTyTaTnScanner
Art. 4.14]
D.C. Machines 459
New torque ^ -0 .6 ^ ,-^ * ,^

It is given that <t>! - 0.96 <t>and = 0.98 q where <>is the main field flux at no load.

-i ll = _ J _ = Ka X 0.96 <t>x 50
t L2 0.6 TLl 0.6 K0 x 0.98 0 x Ia2

/q2 = 50 x 0.6 x m = 29.39 A


New counter e.m.f.,
Ea2 = 240 - 29.39 (2 + 0.2) = 175.342 V

nr_ _ 2 3 0 _ _ 8 0 0 x 0 .9 6 0
Ea2 N 2 02 175.342 N 2 x 0.98 0
.-. Motor speed, M - 800 x 175.342 x 0 96
2 2 3 0 x 0 .9 8 =597.44 rpm.

ly caSrrieT o“ tyT h L nm ethodSeneS m° t0r’ if Tang8 ° f Speed C° ntro1 iS reiluired' !t is usual"

^ *^us^ra^es schematic diagram of a d.c. series motor for its speed control by
varying the armature circuit resistance.

. (fc)
Fig. 4.71. Series motor speed control by varying the armature circuit resistance
(a) Schematic circuit diagram and (6) speed-torque characteristics.

Before the introduction of resistor R„,


S’ ‘ —*
Vt = Ka <bwml + l a l(ra + rs)
If saturation is neglected, then field flux is proportional to the armature current. Let
<|>= CIa so that 0 ! = C IaX
Vt = [Ka Cloml + (ra + r,)) l aX = [Ktoml + (ra + rs)] Ial
where K = KaC
Vt - I ai (ra + rs)
“ ml =
K I ai
After the resistor R „ is inserted in series with the armature circuit as shown in Fig. 4.71
(0),
Vt = [Ka C (0m2 + (rfl + r3 + Rg)) Ia2

For constant load torque, K a <t>i I al = Ea 4*2 h 2


0r KaC I 2
al = K aCI2
a2
0r . =
r
[Art. 4.14
460 Electrical Machinery

Vt = [K tom2 + (r0 + rs + Rg)\ Ial


Vt - I ai (ra + rs + Rg) . ..( i i )
and tom 2
Kl,a 1
From Eqs. (i) and (ii)

. ^2 ^2 _ V t - I a i i ^ + r. + Rg)
" ®ml n j” Ff —/ ai (ra + rs)
^o2 ...(4.59)
■®al
Eq. (4.59) shows that tom2 is less than coml.
Poor speed regulation is not of much importance in case of series motors. This method of
speed control is employed chiefly for series motors driving cranes, hoists, trains etc. The resis­
tors employed for limiting the armature starting current, may be used for speed control pur­
poses also.
In order to fully utilise the motor capacity at all speeds, the armature current is kept equal
to the allowable armature current, i.e. the rated armature current. For shunt motor, the field
flux is obviously constant. For series motor, the field flux is also constant, because it is produced
by armature current which is maintained equal to its rated value. Since field flux remains
constant in both types of motors, arm ature-circuit-resistance control method is usually referred
to as a constant torque [(constant field flux) (rated armature current)] drive method.
Example 4.42. A dc series motor, running a fan at 1000 r.p.m., takes 50 A from 250 V
mains. The armature plus field resistance is 0.6 Q. I f an additional resistance o f 4.4 Q. is inserted
in series with the armature circuit, find the motor speed in case the field flux is proportional to
the armature current.
Solution. For a fan, load torque, TL « n 2.
Electromagnetic torque, Te = Ka t f a.
Since field flux r■e oc 1ra -
Under study state, Te = TL
or
and
Un nr,
^ 2 = Ial = 50
1000
Now counter e.m.f. E a l= 250

and counter e.m.f. n2


E*2 = 250 - 7 ^ (4.4 + 0.6)

/ n2>'
2 5 0 - — volts.
4
Now Eq2 _ n$ 2
Ea1 nx<J>!

fh
n2 ■
20
220 ( 1 0 0 0 ) (50)
or n 2 + H 3 7 n 2 - 11.37 x l 0 5 = 0
Its solution gives n 2 = 641 r.p.m.

JUCJI II ICU UJI 0611Iui.cjnvicri


Art. 4.14]
D.C. Machines 461
Shuntcd-arm ature m ethod. Sneed f 1 ........ ......
carried out easily, but it suffers from poo ° j by armature-circuit resistance control can be
by shunted-armature method, which is ref ulation- This disadvantage can be overcome
method. In this modified method extPm 1 . ®atlon the armature-circuit resistance control
with the armature, as shown in inserted both in series and in parallel
motor. In effect, combination of R, and 7? « f 3 Shunt m° tor and in Fig' 4 72 (b) for a series
is applied to the armature. ‘ 2 C‘ S 35 3 P° tential divider “ d tha ” * * * acr° ss

Series f ield
----
rs

X c
>Rn field
>
> (f AArm j

(a)
Fig. 4.72. Shunted armature method o f speed control (a) for shunt motor and (6) for series motor.

For a d.c. shunt motor, shunt field current is unaffected by R x and R2. Applying Thevenin’s
theorem at the armature terminals, the Thevenin’s equivalent circuit for Fig. 4.72 (a) is as
—illustrated in Fig. 4.73 (a), where the shunt field winding is not shown. From this equivalent
circuit,
R2 _ T
E a —K a§ “ m — -I, r> , D *ara
R x+ R2 XV 1 T XL2

1
(U = [AVt - A R xl a - l ara]
^a<l>

[AVt - Ia (ARj + r„)] ...(4.60)

where A =
+ R2
Electromagnetic torque
Te = K a § Ia
... Te (ARi + ra)
Wm~ K a $ A V ‘ - ~ Ka r ~

By varying both it, and R2 and therefore A, speed a „ can be controlled.


For a series motor, Thevenin’s equivalent circuit looking from the armature terminals is as
shown in Fig. 4.73 (h). From this equivalent circuit,
E„ = K J a>m = V , B - (r, + * i) B h - r j a
'a

or oim= [V, B - (r. + fil) Bla -

...(4.61)
or = _ L [ V r5 - / a ((^ + /? i ) 5 + r“11
01,ni
K a*>

bC am ieu uy uam ouaiiiitii


462 Electrical M achinery [A rt> 414

Ia
fyVWA — w w s-
k R1 R? (rs t Ri) Rj
R i+ R
rs +Ri"rR2
v .-? i y ' RZ
'R , 4 R j fj+R^+R2

=-E «

—o—

(a) (6)
Fig. 4.73. Thevenin’s equivalent circuits (a) for Fig. 4.72 (a) and (b) for Fig. 4.72 (6).

P.
where B =
rs + R 1 + ^2
By varying both and R 2 and therefore B, the speed control can be carried out as is evident
from Eq. (4.61).
By shunted-armature method, no-load sp.eed of a dc series motor is not dangerously high ;
it can be adjusted to any desired value by an appropriate choice of resistors and R 2. Further
this method of speed control gives better speed regulation than that obtained by the convention­
al method of adding external resistance in series with the armature circuit. This fact is il­
lustrated by Example 4.43.
E xam ple 4.43. A 230 V d.c. shunt motor runs at 900 r.p.m. at no load, while taking an
armature current o f 2 A. The armature resistance including brushes is 0.5 Cl. A t rated load and
rated voltage, the armature current is 20 A. Find the speed regulation if
(a) 2 Cl is placed in series with armature and 3 Cl in parallel with it.
(b) only 3 II is inserted in series with the armature.
Neglect rotational losses and armature reaction.
Solution. At no load, Ea = Vt - Iara = 230 - 2 x 0.5 = 229 volts.
27t x 900
Ea = K J 0)m = Ka* = 229 V
60
229
or K jf = = 2.43 V-s/rad.
30 n
R2
(a ) A = = 0.6
Ri + R2 2 + 3
No-load speed from Eq. (4.60) is

wm0 = g ^ \A V t — I a ( A R i + ra)l

= [0.6 x 230 - 2 (0.6 x 2 + 0.5)]

= 55.4 rad/sec.

Full load speed, com = [138 - 20 (1.7)] = 42.80 rad/sec.

COmo —
Per cent speed regulation = --------------- x 100

5 5 .4 -4 2 .8 0
x 100 = 29.44%.
42.80

o o a iiiic u u_y v ^ a i 1 l o o a i 11 i e f
Art. 4.14]
D.C. M achines 463

(b ) No-load speed, a)m0 = ^ — - a ^r<* * ^ 1) _ 230 - 2 (0.5 + 3) _


= 91.77 rad/sec.
K a* 2.43

Full load speed, ov = - 3Q ~ 20 5 + 3) _


= 65.84 rad/sec.
2.43

Per cent speed regulation = ff-1 77 "6 5 .8 4


x 100 = 39.38
65.84
This example illustrates that speed regulation in case of shunted-armature method is bet­
ter than in the case of armature-circuit-resistance control method.
4.14.2. Speed control by varying the field flux. This method of speed control, also
called field-w eakening m ethod or field-current control method, gives speeds above the base
speed only.

Shunt motor. The arrangement of connections is shown in Fig. 4.74 (a). The field flux and
hence the speed of a shunt motor, can be controlled easily by varying the field regulating resis­
tance. This is one of the simplest and economical methods and is, therefore, used extensively in
modem electric drives.

(a) Schem atic connection diagram and (b) Speed-torque characteristics.

Under steady running conditions, if field circuit resistance is increased, the field current
If and the field flux * are reduced. Since the rotor speed can’t change suddenly due to its inertia,
a decrease in field flux causes a reduction of counter emf. As a result of it, more current flows
through the armature [70 = (Vt - counter e.m .f.)/rj. The percentage increase in Ia is much more
than the percentage decrease in the field flux. In view of this, the electromagnetic torque is
increased and this being more than the load torque, the motor gets accelerated. With this, the
counter e.m.f. rises and /„ starts decreasing till electromagnetic torque becomes equal to the

constant load torque. . , , .


If armature current is / . , for flux * „ and when the flux is changed to 02l then for a

constant load torque,


r T* (= T l )
Ia1 “
K h
v,-ia
1
K ail
Tt (= Tl )
Io2 = ~
KM
a 2 ra
Y u L -
Wm2 = Ka* 2

J
i

(A rt. 4.14
464 Electrical M achinery

The above phenomenon describing the changes in speed and armature current as the field
flux is varied, can be better illustrated with an example.
Exam ple 4.44. A 200-V d.c. shunt motor, with an armature resistance ofO. 1 p , is running
at 1000 r.p.m. and takes an armature current o f 50 A. I f the field flux is suddenly reduced by
10%, obtain (a) the maximum value o f current at this instant and the corresponding torque and
(b) ultimate speed and armature current after the transients are over. Assum e constant load
torque and negligible armature inductance.
Solution, (a) Initial counter e.m.f.,
Ea = V, - 1ara = 200 - 50 (0.1) = 195 V.
With a sudden reduction in field flux by 10%, the speed remains at 1000 r.p.m., due to motor
inertia, therefore, the new counter e.m.f. becomes 195 x 0.90 = 175.5 V.
/. The maximum value of armature current
. 2 0 0 - 1 7 5 .8 . 24SA
0.1
245
(initial current of 50 A)
50
= 4.9 (initial current of 50 A)
Maximum current New flux
and the corresponding torque = --------------------------x ------------------ x (Initial torque)
Initial current Initial flux

( 2451
(0.9) (Initial torque) = 4.41 (initial torque).
50
X X
Thus with a sudden reduction of 10% in the field flux, the armature current and torque at
once shoot up to 4.90 times and 4.41 times of their respective initial values. For a constant load
torque, the increased electromagnetic torque accelerates the motor, till the armature current
decreases to Ia2 given by

fo r (
al = 50 = 55.6 A
O2 0.9
Ultimate speed after the transients are over, is given by
0)m2 n2 V t-I a 2 r 0 01
n, V , - I ai r a 02

or 194.44 0i
n2 = 1000 = 1108 r.p.m.
195.00 X 0.9

This example shows that m order to avoid objectionable armature current surges the field
resistance should be increased gradually, i.e. in small steps.

For a co" stant load torQue, power input to motor V, (/„ + /,) [approximately 200 x 50 watts
before and 200 x 55^6 watts after the change in field flux] increases almost in proportion to the

v ,r 7 t h i s UZ m It„erCo0T tet e: mi ( ! 95,V and 194 44 V) r“ approximately constant hi


view of this, the motor output, i.e. E Ja also increases in proportion to the armature current.

s X t a ^ a l l v Z Z r Z Z r a 0' ' T * ? * m0t° r ° Utput' the motor ^ c o n c y remains


time gives smooth sDeed control °At'8’ ° re’ , h e most economical method and at the same
time gives smooth speed control. At any particular setting of the field regulator the speed

z s x z z r s t s r , r . * ? - - “ ■ < • « »>■ s i

S c a n n e d by C a m b c a rin e r
4.14] D.C. M achines 465

The disadvantages o f this m ethod are as follows :


(a) Top speeds are obtained with very weak field. This weak field at top speeds causes the
armature current to increase for the development of certain load torque. With increased arma­
ture current associated with weak main field, the resultant field waveform is badly distorted as
shown in Fig. 4.30. Now consider a coil aa' with its coilsides under the peak flux densities as
shown. If rotational voltage induced in coil aa' exceeds 30 V, the air between the adjacent seg­
ments, to which the coil aa is connected, may breakdown resulting in arcing or flashover. Such
an arc may extend to nearby commutator segments giving rise to flashover between positive
and negative brushes and this leads to direct short circuit on the line. This shows that com­
mutation is very poor when high speeds are obtained with a very weak main field.
(b) The armature may get overheated at high speeds, because the increased armature current
results in more ohmic losses whereas cooling by ventilation does not improve proportionally.
(c) If the field flux is weakened considerably, the speed becomes very high and due to these
changes, the motor operation may become unstable. For example, when the field flux is
weakened heavily, a constant torque load demands an increased armature current. The demag­
netizing armature reaction m.m.f. may reduce the already weakened field flux, to such an ex­
tent that the electromagnetic torque becomes less than the load torque, inspite of increased
■ f P E l N
^ —consequently the motor slows down. Constant torque ^ 0Wer “ “ requires less l a at

reduced speed, because Ea remains substantially constant. A decrease in Ia and, therefore, a


decrease in the demagnetizing armature reaction, causes the field flux to rise from its
weakened value. As a result of it, the electromagnetic torque (<* <j)/a) starts growing, even
though Ia has become less. If motor torque becomes greater than the load torque, the motor
accelerates and its speed may again become very high ; due to this, the above happening may
re-occur resulting in periodic speed fluctuations, i.e. hunting of the motor. A weak series-field
winding, (called stabilizing winding ) assisting the shunt field may avoid this unstable opera­
tion at high speeds. A stabilizing winding allows wider speed range and is added in all the dc
motors intended for adjustable speed operation. Use of compensating winding further increases
this speed range.
Exam ple 4.45. In Example 4.44, after the transients are over, the reduced field flux is sud­
denly increased to its initial value (Jjj. Calculate the armature current before the speed starts
changing.
Solution. With a sudden increase in field flux from 0.9 <ha to the counter e.m.f. rises to

194.44 x = 216 V.
0.9 9!
, , 2 0 0 -2 1 6 .
The current taken from the supply = q ^ — toU A.

Since the armature current is now negative, the machine acts momentarily as a generator
and feeds the power to the supply, at the expense of its kinetic energy. The speed reduces
rapidly till counter e.m.f. becomes less than the supply voltage, so that motor action takes place
and torque balance is obtained.
Examples 4.44 and 4.45 conclude that the field regulator resistance must be varied in very
small steps.
Series m otor. The field flux and, therefore, the speed of a series motor can be varied (a) by
Placing a resistor, called a diverter, in parallel with the series field winding as shown in Fig.
4.75 (a ); (6) by tapping the series field winding as shown in Fig. 4.75 (6), and (c) by changing
toe field coil connections from series to parallel, Fig. 4.75 (c).

>canned by C a m S c a n n e r
[A rt. 4.14
466 Electrical Machinery

(a) D iverter field control. When the diverter resistance is varied, the current in the serie
field winding is changed, Fig. 4.75 (a) and there is, therefore, a corresponding ge in field
flux and the speed.
M o v a b le
D iv e rte r con tact i

'l (r * )
Topped W
-field J

(6)
Fig. 4.75.aSehes motor speed control (a) by a diverter and (6) by tapped field.

r— ^s/2
■a/2 X.
Ns/2
( t^p
vt Nj Ns I Ia/2
2 2

(,) («)
Fig. 4.75. (c) Series-parallel field control for a d.c. series motor.

(b) T apped-field control. When the field winding is tapped, Fig. 4.75 (b ), the number o
series field turns is changed and, therefore, the series field m.m.f. and the speed are changed.
If the series motor is to work under violently varying loads, then the diverter resistance
should be highly inductive. For example, when the series motor is used for traction purposes,
the current collector may lose contact with the overhead wire. After this, the motor continues
running due to inertia, but the current and, therefore, the series field flux may collapse. After
a short while, when the contact with overhead wire is re-established, the entire current may
pass through the resistive diverter due to the large inductance of the series field winding. Since
the current in series field is almost zero due to its high inductance, the counter e.m.f. developed
by the motor would be zero and this would result in heavy in-rush of armature current when
the contact is re-established. In view of this, the diverter resistance should also be highly induc­
tive, as stated before. This difficulty is, however, not present in tapped field control which is,
therefore, preferable for series traction motors.
(c) S eries-p ara llel field con tro l. In this method, the series field winding is divided into
two equal halves. When these two halves are in series, Fig. 4.75 (ci), then for an armature
current o f Iat total field m.m.f. Fs is,

1F s = Il a
2 2
Counter e.m.f. Eas = Vt - Ia (rs + ra)
When the two halves of field winding are connected in parallel as shown in Fig. 4.75 (c »)»
then for the s a m e /0, each parallel path shares 7a/ 2 and total field m .m .f. Fp is
LN.
Fp = ( 4 / 2 ) (Ns/2) 2 = —

S c an n ed by C a m S c a n n e r
Art. 4.14] ______ D.C. Machines 467

Counter e.m.f. Eap = V p - / a


,7 •
For no magnetic saturation,
^ap _ ^2 (^a W s/ 2 )
•®as (7a AT,)

or n2 = 2n1
as ... _

This shows that parallel connection of field coils results in higher operating speed of the
series motor.
For a constant load torque, a decrease in field flux, gives increased Ia and increased speed.
Thus power input VJa and power output (= constant load torque x speed) increase and, there­
fore, efficiency remains almost unchanged.
For both shunt and series motor control, the counter e.m.f. Ea remains substantially con­
stant, because a decrease in field flux is compensated by a corresponding increase in speed. If
the armature current Ia is kept equal to the motor rated (or nameplate) current for its full
utility, the power output IaEa remains approximately constant and for this reason, field-flux
speed-control method may be called a constant power drive method.
Since EaIa remains approximately constant, the maximum torque is obtained when the d.c.
motor runs at the lowest speed. In view of this, field-flux control method is suitable to drives
requiring large torques at low speeds. In case the field-flux control method is used to drive a
load requiring constant torque over the entire speed range, then motor rating and size are
decided by the product of constant torque and the highest possible speed. Obviously, such a
motor at low operating speeds will be underutilized.
Example 4.46. A 4-pole d.c. series fan motor takes an armature current o f 60 amperes, when
running steadily at 2000 r.p.m. on a 220 V supply. The four field coils are now connected in two
parallel groups o f two in series. Assuming that the flux produced is directly proportional to the
exciting current and the load torque increases as the square o f the speed, find the new speed and
the armature current. Neglect losses and assume constant supply voltage.
Solution. Electromagnetic torque, Te <* 4>Ia

and load torque, TL « n 2 (given)


Under steady state conditions,
TL = Te
or n2 « (J) Ia
I f /al is the armature current with four field coils in series, then

P i « <t»i I a l

a1

I f/a2 is the armature current when four field coils are connected in two parallel groups, then

or 2 o c ^ I a2 as <|>2 «
n2 2

/ \2 t2
ni la\
IW 2

b a n n e d by C a m S c a n n e r
468 Electrical Machinery (Art. 4.14

f2000^
or =2 f60)
^2 L 2
V \ J
30V2 n2
or 1 0~
a2 1000
Counter e.m.f. with all the four coils in series,
Eat = Vt - I a l{ra + r , ) « n 1$1
and with the four coils in two parallel groups,
r ,'
Eap ~ Vt ~ Ia2
.r“ * ,
Since ra and rs are not given,

220 M ai
220 n2 (fa2/2 )
30V2 n2
or = 2000x60
n<l' 2 x 1000
or n2 = 2378.4 r.p.m.
. _ 30V2 x 2378.4
= 100.9 A.
02 1000
Example 4.47. A shunt motor connected to a constant d.c. voltage source, drives a load
requiring constant electromagnetic torque. Prove that, i f counter e.m.f. E u > I Vt, the speed
decreases with an increase in flux (or vice versa) and i f Ea < I V„ the speed increases with an
increase in flux. Here Vt is the armature terminal voltage.
Solution. From Eq. (4.34),
Vt - I ara
K a iD ..,(4.34)

If an external resistan ce^ is inserted in series with armature circuit, Eq. (4.34) becomes
_ V , - I a(ra + Rg)
m Ka<\> ...(4.62)
Now electromagnetic torque Te = KaWa. Substituting the value of Ia in Eq (4 62) •
Vt 0ra + R g) T e
...(4.63
For a constant electromagnetic torque Te,
doifm

<*♦ ' ...(4.64


k W

di
With an increase in flux, if the speed decreases as is usually case, then the term ^ mu:

be treated as negative, i.e. — p- < 0.

Therefore, from Eq. (4.64), - Y l . . n (ra + R e) T e


■+ 2 —a 1 : t ^ < 0

or

S c an n ed by C a m S c a n n e r
A rt. 4.14]_________________________________________________________________ D.C. M achines 4 6 9

or Vt - 2 ( r a + R g) I a > Q
or Vt ~ I° ( ra + Rg) > I a {ra + Rg)
i.e. Counter e.m .f. Ea > (total arm ature circuit resistance drop)
Also Ea + E a > [Ia (ra + Rg) + Ea]
or 2Ea > Vt, because Vt = [Ea + Ia (ra + R g)J

or V,.

Thus the speed decreases with an increase in flux (or vice versa ) for a d.c. shunt motor only
if the counter e.m.f. Ea > total armature circuit resistance drop or Ea > | Vt.

With an increase in flux, if the speed increases then — must be positive, i.e. —, m~> 0.
a<(> a0
Therefore, from Eq. (4.64),

Ka If. O3
or - Vt + 2 (r„ + Rg) Ia > 0
or V, - Ia (ra + Rg) < Ia (ra + Rg)
or Ea < Ia (ra + Rg)
or 2Ea < I a (ra + Rg) + E a
or 2Ea < Vt

or E . < ± V,.

Thus the speed increases with an increase in field flux for a d.c. shunt motor, if the counter
e.m.f. Ea < total armature circuit resistance drop or Ea < ^ Vt.

Example 4.48. A 230-V dc shunt motor has an armature-circuit resistance o f 0.4 £1 and field
resistance o f 115 £2. This motor drives a constant torque load and takes an armature current o f
20 A at 800 rpm. I f motor speed is to be raised from 800 to 1000 rpm, find the resistance that
must be inserted in the shunt-field circuit. Assume magnetization curve to be a straight line.
Solution. At 800 rpm, Tel °c (Jq 7al
At 1000 rpm, Te2 « 02 Ia2
. '£e\ _ ^ _ <t>i x 20
Te2 4*2 ^a2

Ia2 = 20 P1 = 20 k
02

where k=—
02
At 800 rpm, Eal = 230 - 20 x 0.4 = 222 V.
At 1000 rpm, Ea2 = 230 - 20 k x 0.4 = 230 - 8 k

Now
Ea2 AT2 02
222 800 x fa
= 0.8 k
230 - 8k 1000 x 4>2
0r 222 = 230 x 0.8 k - 6.4 k2

S c a n n e d by C a m S c a n n e r
(A rt. 4.14
470 Electrical Machinery __________________ _________________ -— -

or k2- 28.75 ft+ 34.69 = 0_________


_ 28.75 ± V826.5625 - 138.76 __ 27 49 or 1.26
k-

Higher value of 27.49 is not feasible.

* = ; r = 1 -26
02
Since magnetization curve is linear,

t u k = i.2 6
02 V2
T _ 230 _ 9 *
But A “ 115 _

^ = l i e = 1587 A
New shunt-field circuit resistance

230 = 144.93 £2
In 1.587
External resistance that must be inserted in shunt-field circuit
= 1 4 4 .9 3 - 115 = 29.93 £1.
Example 4.49. A 250 V dc shunt motor has an armature resistance o f 0.5 £2 and a field
resistance o f 250 £2. When driving a constant torque load at 600 rpm, the motor draws 21 A.
What will be the new speed o f the motor if an additional 250 £2 resistance is inserted in the field
circuit. <GATE> 198®
250
Solution. Field current, 7^ = 7^ = 1 A

Armature current, 7al = 21 - 1 = 20 A


Counter e.m.f., Eai =250 - 20 x 0.5 = 240 V
T 250 n c A
If) —n«A —U.O A
p 250 + 250

Neglecting magnetic saturation, <(>, L or — = f - = ——


1 02 'f2 u.5
For constant load torque, TLl = TL2
or Tel = Te2
Ka 0i Ia i= K a (J>2 / o2

New counter e.m.f., Ea2 = 250 - 40 x 0.5 = 230 V


Eal N ifa
Now

or 240 _ 600 0i _ 600 If\ 600


230 N2 02 N2 If^ 0.5 N 2

/. Motor speed, N2 = * ^ 5 = 1150 rpm.

S c a n n e d Dy u a m s c a n n e r
A r t . 4. 14] D.C. M a c h i n e s 471

Exam ple 4.50. A d.c. shunt motor takes 50 A o n full load from 250 V mains. Its speed is to
be raised by 40% by weakening o f the field flux. I f the torque at the increased speed is 20% more
than that at the initial speed, find the percentage change in field flux. The armature resistance
(including brushes) is 0.5 Cl.

Solution. Electromagnetic torque at the initial speed is Tel « ^ / ol and at the increased
. J .T .
speed, Te2 ~x At
Ma2-

I jl 41 01
Te2 h i 02
1 50 <}>!
or
1.2 4 2 02

4 2 = (50) (1.2) ~ = 60 7 1 = 60 a
02 02
01
where a=
02
At the initial speed, Eal = 250 - 50 x 0.5 = 225 V and at the increased speed,
Ea2 = 250 - 60 a(0.5) = (250 - 30 o) V.

E a2 n2 02
Now
K i n i 0i
2 5 0 - 30 a 1.4 n x 02 1.4
225 n x (<|>2 •a) a
or 250 o - 3 0 o 2 = 315.0
or a 2 - 8 . 3 3 a + 10.5 = 0
01
Its solution gives o = 1.547 =
02
.-. Field flux at the increased speed,
01
= 0.647 <t>j.
1.547
.*. Percentage reduction in field flux
<^>2 - 0.647 <(>i
x 100 = 35.3%.
0i
E xam ple 4 .5 1 . A 250 V d.c. shunt motor has the following magnetization curve at 1200
r.p.m.
215 250 275 300
Ea in V 125 180
0.80 1.10 1.36 1.76
If in A 0.38 0.58

The shunt field winding has 1000 turns per pole, m no loaa, me mum, , un>
rnth J m L arm ature current. Find the number o f serves turns p er pole required to reduce the
weed to 900 r a m when the fu ll load armature current vs 100 A. The series field will have a
esistance of 0 f l and the arm ature resistance ^ lu d i^ brushes is OJ a Assum e that the
.2
. . • j • oniiiiinlpnt to 4% reduction m m am field m.m.f.
irmature reaction a t fu ll load is equivalent i
A* ™ the armature resistance drop is zero. Therefore, the counter e.m.f. at
>oload speed o f 1200 r.p.m. is 250 V, which requires a shunt field current of 1.1 A, Fig. 4.76, or
'hunt field ATs of 1100.

S c an n ed by C a m S c a n n e r
[Art. 4.14
4 72 Electrical Machinery

At full-load, the counter e.m.f. = 250 - 100 (0.3) - 220 V.


This counter e.m.f. of 220 V corresponds to a full-load speed of 900 r.p.m. Smcethe mag.

netization curve is given for 1200 r.p.m.. the counter e.m.f. of 220 V should be — x 1200

= 293.3 V a t 1200 r.p.m.


Now corresponding to this e.m.f. of 293.3 V, the field current from ig- • .
Since the armature reaction at full-load is equivalent to reduction in main field m.m.f,

the total field m.m.f. required is 1.62 x 1000 x ^ = 1690 ATs. Shunt field is already furnish-
96
ing 1100 ATs.
Series field ATs = 1 6 9 0 - 1100 = 590
590
/. Series field turns = 6.
100

Fig. 4.76. Magnetization curve pertaining to Example 4.51.

E xam ple 4.52. A d.c. series motor, running a friction load at 1000 r.p.m., takes 40 A from
240 V supply mains. Its field resistance is 0.2 ft and that o f the arm ature is 0.25 ft. I f a diverter
o f 0.3 ft resistance, is put in parallel with the series field winding, find the m otor speed. Assume
the field flux to be proportional to field current.
Solu tion. Note that the torque required by a friction load (e.g., a reciprocating pump) sd
different speeds remains constant.
Now electromagnetic torque Te oc <j>/a.
Since 0 has been assumed proportional to Ia,
r2
5X1L -

Te l x (40)-

S c a n n e d by C a m S c a n n e r
Art- 4141 D.C. M achines 473

With a diverter in parallel with the field winding, the field current is reduced. Therefore, if
new armature current is Ia2, then only a part of it passes through the series field. Thus the new
field current is given by

J “ dm . 0.3
° 2 Rd,v + r, = 02 0.3 + 0.2 = a2-
New value of field flux <j>2 « (0.6/q2)
Thus rc2 “ (02) 7a2
86 ( 0.6 I a2) I a2
0 .6 f o2
l

1 e2 0 6 4
- 1=
T' , (40)2
1600
or I = 51.6 A.
0.6
Now Eal = V , - l a l(ra + rs)
= 240 - 40 (0.25 + 0.2) = 222 V.
When diverter is put in parallel with rJt then

r s ' R dw
Ea2 = Vt - Ia2
r° + r. + i?

= 2 4 0 - 51.6 0.25 + M = 220.9 V

Ea2 n20-j
But

220.9 n* (0.6 x 51.6)


•• 222 (1000) (40)
n2 = 1285 r.p.m.
Exam ple 4.53. A 6 p olc, 230 V d.c. series motor has a flux per pole o f 4 mWb/Amp over the
working range o f the magnetization curve which is assumed to be .inear. The load torcjue is
proportional to speed squared and its value is 20 Nm at 800 r.p.m. There are 432 wave-connected
conductors and the total resistance o f the motor is 1.0 fl. Determine the motor speed and current
when this m otor is connected to rated supply voltage.

Solution. For motor current of = 4 x 10 la Wb

_ <j>ZnP 4 x 1 0 '3 la x 432 x n x 6


= 0.0864 nl„ ...ii)
a 60a 60x2

T = = 0 0 864 nI« J o x 60 = 0 .8 2 5 12
a ...(ii)
c a) 2;w
But Ea = Vf - / a (ra + r,) = 2 3 0 - / o ..(iii)

From (i) and (iii), 0.0864 n Ia = 230 - Ia


230
or ...(iv)
1 + 0.0864 n
47 4 Electrical M achinery —

Substituting the values of Ia in (ii),

( 230
T.€ = 0.825
1 + 0.0864 n
It is given that TL « n2 or TL = Kn2
or 20 = K (800)2
K = 20/(8 00 )2 = 3.125 x 1 0 "5 Nm/r.p.m.
Under steady state conditions, TL = Te

or 3.125 x 10" 5 n2 = 0.825


1 + 0.0864 n
, y
Its solution gives n = 651.888 = 652 r.p.m.

From (iv)
from (iv), iI a = -----------
x + 0.0864 — -----------=
x 652 4.01166 = 4.012 A.

Exam ple 4.54. A 2 3 0 V, 1000 r.p.m. dc shunt motor has field resistance o f 115 Q and arma­
ture circuit resistance o f 0.5 £2. At no load, the motor runs at 1000 r.p.m. with arm ature current
o f 4 A and with full field flux.
(a) For a load requiring 80 Nm, compute armature current and speed o f the m otor.
M (b) I f it is desired that motor develops 8 kW at 1250 r.p.m., determ ine the value o f external
■fitter resistance that must be inserted in series with the field winding. Saturation and armature reac-
9^ tion are neglected.
Solution. At no load, Ea = 230 - 4 x 0.5 = Ka <(>com

„ , 228x60 , j
or a = 2n x 1 0 0 0 = v -sec/rad
(a) Now Te = Ka <$>Ia

; " = 2 § 7 = 3 6 '7 5 A
For this armature current, Ea = 230 - 36.75 x 0.5 = K a§ com

. 211.625 2rm
•• m = " 2 177 = rad/sec = —
or n = 928.3 r.p.m.
(b) Motor developed power = Eal Ial = 8000 watts

or (230 - 7a l x 0.5) 7al = 3000

Its solution gives = 422.1 A or 37.9 A. The higher value of 1 . is redundant, so take
= 37.9 A.

230
For a field current of — = 2 A, K J = 2.177 V-s/rad.

For I f = 2 A , K a 0 = 2.177 V-s/rad

For I f= 1 A ,X ,0 = ^ V - s / r a d

For W A . ( * i « i = a Y I -/n V 4*ad

S c an n ed by C a m S c a n n e r
Ar(. 4.14)_________________________________________________________________________________ D.C. M achines 475

Now Eal = 2 3 0 - 37.9 x 0.5 = ( K » , (oml


2 i i os = 1 gn x 1250
or 2 n 60
or = 1-481 A
rn = 230/1.481 = 155.3 fl.
So, the resistance that must be inserted in series with the field winding = r ^ ~ ry = 155.3 - 115
= 40.3 fl.
Exam ple 4.55. A 250-V dc series motor has armature and series-field resistances o f 0.25
fl and 0.15 fl respectively.
(а) Calculate the current for developing a torque o f 80 Nm at 1200 rpm.
(h) Calculate the percentage reduction in flux when the motor runs at 1800 rpm at h alf the
current obtained in part (a).
Solu tion, (a) For a dc motor, developed torque,

7 > —
e w
or Te w = E J a = [V, - Ia (ra + r,)l la
Substituting the values, we get

80 x 271 * *2— = (250 - I a x 0.4] Ia


60
or 7* - 625 7a + 25132.74 = 0
Its solution gives 70 = 43.2 A
(б ) For this part, let 43.2 A be denoted by Ial.

New value of current, 7a2 = - 21-6 A


Counter emf, Eal = 250 - 43.2 x 0.4 = 232.72 V
Counter emf, Eat = 250 - 21.6 x 0.4 = 241.36 V
E a\ _
Now ' N 2(02
$2 _ 1200 241.36 _ 0 6914
or 0, " 1800 232.76

100 = (1 -0 .6 9 1 4 ) x 100 = 30.86%.


Percentage reduction in flux - ^

E xam ple 4.56 A


Z t i Z T e Z r Z Z t t L must te added to attain rate d torque (a , a , s t a r t s and i t ,

at 1000 rpm.
Solu tion . Motor torque, Te = Katyla
. • (hoc I and therefore l t = i u a.
As magnetic saturation isneglected, * ■
Rated armature current, /„ 1 - 20 A
(a) At starting, E a = 0 andV, = Ia\(ra + r* ex] circuit
. in the motor armature circuit,
.-. External resistance to be added in the mo
„ _ 230 _ (0 5 ) = 11 tl.
A e r l' go

S c an n ed by C a m S c a n n e r
476 Electrical M achinery (Arl. 4.14

(6) For developing rated torque at 1000 rpm, current remains the same, i.e. at 1000 rpm,
armature current l a2 = Ial = 20 A.
Ea2 _ <j>2 _ N?Ia2 _ N 2
Eal -Nl<t>i A^l
Eal = 230 - 2 0 x 0 . 5 = 220 V
. Eq2 1000 r, 2 -\ar* rrj T7
" 2 2 0 = 1500 0r = 3 =
Also
E q2 ~ ^ t ~ ^a2 (ra + rs Eex2)
146.67 = 2 3 0 - 2 0 (0.5 + R ex2)
or Ra2 = 4.167 - 0 . 5 = 3.667 Cl.
Exam ple 4.57. A dc series motor drives a fan at 800 rpm and takes 20 A when fed from
rated voltage o f 230 V. The motor resistance is 0.4 Q. The motor speed is to be raised to 1000 rpm
by voltage control. Find the voltage and current in case magnetic circuit is (a) saturated and fb)
unsaturated.
Solution. At 800 rpm, Eal = 230 - 20 x 0.4 = 222 V
For a fan, TL « N 2 or T2 = K XN 2
For the motor, Te = Ka <j>/a

(a) When magnetic circuit is saturated, 0 = constant and Te = K la. Under steady state, Te = TL
or KIa = K X - '

or K .N 2
E lui K,N\

^iooo ' 2
/«2 = 20 = 31.25 A
800
^ 2 = ^ 2 - 3 1 .2 5 x 0 . 5
Now ^ o 2 _ W2<t>2 N2
E a i ' N ^ - N ; as C o n s t a n t

£o2 = 222 X W = 2775 V


From (i),
Vt2 = 277.5 + 31.25 x 0.5 = 293.125 V
(6) When magnetic circuit is not saturated, 4>~ 7a and T = K I 2
^ 2 a
As
E 2 I ^ = K 1N 2
2
or (4>'|

W JVi
V /
or
'.3 = 4 . $ = 2 0 x M 0 = 2 5 A
N\ 800 A
Ea3 = Vt3 - 25 x 0.5
...(ii)
Also _ ^ 3 03 _ A^3 / a3

or F - 000 v 1000 25
“3 ' 222 X l o o " x 20 = 346 87 V
From (ii),
Vt3 = 346.87 + 25 x 0.5 = 359.375 V.

ZT
....................

Art- 4.14] _________________________________ D.C. M achines 477

4.14.3. Speed. control by varying the arm ature term inal voltage. Reference to Eq.
(4.34) shows that if the armature terminal voltage Vt is varied, counter e.m.f. (Vt - Iara) chan­
ges almost proportionally and for a constant-flux motor (e.g., a d.c. shunt motor), the speed
changes approximately in the same proportion as V,. At present, the bulk power is usually
a v a ila b le as alternating voltage. So, for driving a dc motor, ac must be converted to dc and
then only fed to dc motor armature for its speed control. DC motor speed control by varying
the arma ure ermina vo tage is obtained by (a) Ward-Leonard system (6) controlled rectifiers
a n d (c) series-parallel armature control.

An examination of Eq. (4.34) reveals that addition of external resistance in the armature
circuit is equivalent to the variation of armature terminal voltage. But the behaviour of motor
with added resistance in the armature circuit is quite different from its behaviour with arma­
ture terminal voltage control method. Consequently this method of speed control is considered
separately.

(a) W ard-L eon ard system . The schematic diagram of this system is illustrated in Fig.
4.77. In this figure, M is the separately excited d.c. motor whose speed is to be controlled and
G is the separately excited generator driven by a three-phase driving motor (usually an induc­
tion motor). The combination of a.c. driving motor and the d.c. generator is called motor-gene-
rator set and it converts a.c. into d.c., which
M a in m o to r
is fed to the main motor Af. If no supply is t Motor SgneArntor SC1
available, the three phase motor can be
replaced by some prime-mover.
For starting motor Af, its field circuit is
first energised and then the generator output \ J
voltage is adjusted to a low value by decreas- A.C.
ing its field excitation. This is done in order Driving motor
to limit the starting current to a safe value
but it should be ensured at the same time
that enough starting torque is produced to
accelerate the motor and the load. In view of
To e x c ito r
this, no starting rheostats are necessary and,
therefore, considerable amount of energy is Fig. 4.77. Schematic diagram of Ward-Leonard
saved during starting. A change in the gene­ system of speed control.
rator field current varies the voltage applied
to the motor armature and, therefore, the motor speed is changed. Thus the motor speed control
is obtained merely by changing the generator field current.
In order to achieve wider speed control range, speeds below base speed are obtained by
voltage control and above base speed, by field flux control. For better utility of motor Af, its
current Ia is maintained equal to its rated current during its speed control.

Speeds, from the lowest possible speed up to base speed, are obtained by increasing the
generator output voltage, with constant motor field flux. Since the speed control is carried out
with rated current Ia and with constant motor field flux 0, a constant torque (« <|>/ j up to base
speed is obtained. Power (= torque x speed) increases in proportion to speed. Thus constant
torque and variable pow er drive is obtained up to base speed, with armature-terminal voltage
control method as shown in Fig 4.78.
Speeds above base speed are obtained by decreasing the motor field flux with constant
generator voltage. As before, the armature current Ia is kept equal to its rated value. Under
these conditions, Vt Ia or Ea Ia remains constant and the electromagnetic torque propdrtional to

'ww\ m n m m '
S c a n n e a bv C a m S c a n n e r
478 Electrical M achinery [Art. 4.14

armatur E .M O TO R F I E L D
<{)/a decreases as the field flux is decreased. Thus, (V O L T A G E CONTROL
(C O NTRO L
weakening of the motor field flux results in con­
stant power and variable torque drive above base
speed, Fig. 4.78.
POW ER
The speed range with armature voltage control ex­ /

clusively is 10:1, the lowest speed being limited mainly T0R6iJ_E_/ / _


by the residual magnetism of the generator. The speed
range with motor field control alone, is 3 :1 to 4 :1, the
highest speed being limited by poor commutation, ar­
mature heating and unstable operation. When both /
types of speed controls are employed, the overall speed
¥t
range is 40 :1.
Speed range can still be broadened to, say 200:1, /
BASE M AXIM UM
by the use of amplidynes incorporating closed-loop SPEED SPEED
system. SPEED
Braking of motor M may be carried out by Fig. 4.78. Torque-speed and power-speed
decreasing the generator excitation so that its emf characteristics for Ward-Leonard system
of speed control.
is less than the counter emf of motor M. Under
these conditions, M begins to work momentarily as
a generator, G as a motor and a.c. driving machine as a generator. Consequently the kinetic
energy of M and its load is returned to the supply mains and braking action on main motor M
takes place.
The advantages of Ward-Leonard system of speed control are as follows :
(i) Thejnain advantage of this system is its simplicity, wide range and smooth speed con­
trol. Consequently Ward-Leonard system in its original or modified form, is used extensively in
rolling mills, colliery winders etc. .
(ii) With armature reaction ignored, the decrease in speed from no load to rated load, is
mainly due to the resistance drops in both the generator armature and motor armature. The
speed regulation is, therefore, quite good.
(Hi) The direction of main motor rotation can be changed merely by reversing the generator
field current.
(iv) Speed control is carried out through the field circuits of generator and motor. Since
these field circuits are low-power circuits, the control apparatus is not costly.
(v) The efficiency at low speeds, is higher than that obtained by other methods of speed
control.
The only disadvantage of this method is its higher initial cost, because three machines
having rating equal to the full load output, are required.
E xam p le 4.58. An adjustable speed d.c. shunt motor has speed range 4 : 1. I f motor takes
an armature current o f 60 A at speed n; then calculate the armature current at speed 4 n when
speed control is obtained by (a) field flux control and (b) armature-voltage control. The motor is
driving a load requiring
(i) constant pow er and
(ii) constant torque.
Ignore all losses and armature reaction.
Solution, (a) Field-flux control, (i) When load requires constant power, then Ea Ia or
torque x speed must remain constant.

o l q i ii ic u u y o a i i i v j u a n i i c i
D .C. M achines 479

At speed n, let the field flux be <J>, so that Ea « ^


With field-flux control, the counter emf. Ea remains approximately constant, because a
re d u c tio n in flux <j) is compensated by a corresponding increase in speed n. Thus the product
(j) x n remains constant in field-flux control. Therefore, at speed 4n, the field-flux must be
so that Ea remains unchanged.
For constant power drive (Ea Ia constant) the armature current at 4n is equal to
I =60 A.
la
(ii) For constant load torque, Te « <j>Ja must remain constant and from part (a, i), it is seen
that field flux at 4n must be <f>/4. In view of this, the armature current for constant torque
/ a
^ / a = ^ x 4 / fl must be 4/0 = 4 x 60 = 240 A.

(b) Arm ature-voltage control, (t) For load requiring constant power, Vt Ia = Ea Ia is con­
stant.
If the armature voltage is V, at speed n, then at speed 4n, the armature voltage must be
4V, (field flux is constant), since speed is approximately proportional to V„ Eq. (4.34).

.*. For constant power drive, the armature current 60


is — = = 15 A.
(ii) For constant load torque, Te« $ Ia must remain constant. At speed 4n, the armature
voltage is 4Vt i.e. the counter emf now becomes 4 Ea. But 4Ea « (4n)(4>) i.e. the field flux remains
unchanged at both the speeds n and 4n. Consequently for constant torque, the armature cur­
rent is 60 A.
Part (b, ii) can alternatively be explained as follows :
For constant torque, <j)Ia is constant. In armature-voltage control method, <j> is left un­
touched, i.e. <{>remains constant. In view of this, Ia must remain constant at 60 A in order to
furnish constant torque (OC
Example 4.59. f o r a Ward-Leonard system, two identical 220 V, 15 A d.c. machines are
used. Total armature resistance o f each machine is 0.4 fl and the magnetization curve for each
machine at 1500 r.p.m. is as given below :

Ea m V 120 160 197 210 220 228 232 236 243 248
If in A 0.3 0.4 0.5 0 .6 0.7 0 .8 0.9 1 .0 0 1 .2 0 1.45

The generator o f the Ward-Leonard system is driven at a constant speed o f 1500 r.p.m.
(a) The generator field current is varied from 0.15 A to 1.4 A, while the motor field current
is maintained constant at 0.6 A. Find the speed range (i) for full load armature current o f 15 A
and (ii) for no load. Assume the armature current to be negligible at no load.
(b) The maximum speed obtained in part (a) at full load armature current, is to be doubled
wdh the help o f field flux control. I f the generator field current is not allowed to exceed 1 A, find
he minimum motor field current required.
Solution, (a) (i ) For generator, If = 0.15 A, Ea = 60 V from magnetization curve. Full load
v°ltage drop in two armature resistances in series = 15 (0.4 + 0.4) = 12 V.
•• Actual counter em f o f the motor, when the generator field current is 0.15 A
=£o ~ /0 (2ra) = 6 0 - 12 = 48 V.

cann ed by C a m S c a n n e r
1

480 Electrical M achinery [Art. 4,14

But the motor counter e.m.f. for a field current of 0.6 A at 1500 r.p.m., from the magnetiza.
tion curve of Fig. 4.79 is 210 V.

48
Minimum motor speed = — x 1500 = 343 r.p.m. = 345 r.p.m.

For I f = 1.4 A, the generator e.m.f. E a is 247 V.


Motor counter e.m.f. = 247 - 12 = 235 V.

Maximum motor speed= x 1500 = 1680 r.p.m.

Thus the speed range is 1680 : 345 or 4.87 : 1.


(ii) For no load, armature current Ia = 0.
Motor counter e.m.f. = generator e.m.f.
When the generator field current is minimum, 0.15 A, the motor counter e.m.f. is 60 V.
Minimum motor speed at no load

= 210 X s 430 r.p.m.


/. Per cent speed drop from no load to full load
4 3 0 -3 4 5 ...
= — — = 1 9 - V8 %

When the generator field current is maximum i e 1 4 A ™ * r 947


Therefore, the motor counter e.m.f. at no load iTalTo 247 V

S can n ed by C a m S c a n n e r
Art. 4.141 D.C. Machines 481

Maximum motor speed at no load for the condition of minimum speed


247
= 210 * = *^62 r.p.m.
Thus the speed range at no-load is 1762 : 430 or 4.1: 1.
Per cent speed drop from no load to full load for the condition of maximum speed
1762 -1 6 8 0
1762 x 100 —4.65%

It is seen from above that speed regulation is poor under the condition of minimum speed
setting of motor. But under the setting of maximum speed, the speed regulation is better.
(6) For generator field current of 1 A, the e.m.f. generated in the generator armature is
E„ = 236 V.
Motor counter e.m.f. at full load = 236 - 12 = 224 V.
This motor counter e.m.f. of 224 V must correspond to 2 x 1680 = 3360 r.p.m. Therefore,
motor counter e.m.f. at 1500 r.p.m. is
224
x 1500 = 100 V
3360
From the magnetization curve of Fig. 4.79, the motor field current for Ea = 100 V is 0.25 A.
Thus minimum motor field current required is 0.25 A. Note that the overall speed range with
both armature voltage control and field flux control becomes 3360 : 345 or 9.74 : 1.
(6) Speed control with controlled rectifiers. Controlled rectifier d.c. supply can be used in
place of motor-generator set of the Ward-Leonard system. Now-a-days the silicon controlled rec­
tifiers (or thyristors) have made the SCR—d.c. motor scheme much more economical and its other
advantages are less floor space, higher efficiency and quicker control of the output voltage.
Single-phase controlled rectifiers using thyristors are used for the speed control of dc
motors below base speed. These are suitable up to about 15 kW rating.
For a single-phase full converter, the speed is given by
2Vm
cos a

“ m - <4-65>
where Vm = maximum value of 1-phase source voltage
and a = firing-angle delay.
There is usually a small voltage drop in conducting thyristors. If this voltage drop is taken
as constant and equal to un then Eq. (4.65) becomes,

2 V,m
cos a - vr
'n
J 77^-2 Te ...(4.66)
m Ka •0 (Ktt 0)
por dc motor ratings above 15 kW, 3-phase controlled rectifiers using thyristors are used.
The speed in case of 3-phase full converter is given by
3 Vml
—TT— cos a - vr _
*ia
C0„, = i? i ----------- ■Te ...(4.67)
Ka •0 (Ktt V*

I 11 I W III vVUUI II IWI


(Art. 4.14
482 Electrical M achinery

can be controlled. For detailed analysis of single-phase and 3 -phase irn


types of controlled converters, the relevant literature may « « ^ made possib,e ^
The use of single-phase and three-phase controlled con _ widespread applica-
precise speed control of dc motors and has opened up new vistas tor tneir v kk

tions in industry. nf armature voltage control requires


(c) Series-parallel arm ature control. This method o |Qad u ig usual t0 employ
two identical dc motors coupled together mechan y .hunt and compound motors, i
this method for dc series motors, though it can be used for dc shunt ana comp
n i..-h ,l op L/2

* n
-'rnpJ

vt

i Ea

lL

(a)
(6)
Fig. 4.80. Series-parallel speed control o f twoefe series motors
(a) armature in series and (b) armatures in parallel.

In Fig. 4.80, the connections for series-parallel control of two identical dc motors are shown.

When the armatures are in series as in Fig. 4.80 (a), the voltage across each armature is Eas = —

and the field flux «> is established by current / M = / L. This gives motor counter emf (= Ka $ wm) as

p
nc
-Y2l =k j w where w is the motor speed when motors are connected in series and flux <t>
S* ^
because magnetic saturation is neglected. Subscript s stands for series connection.
When the armatures are in parallel as shown in Fig. 4.80 (6 ), voltage across each armature
h r
is Vt and the field flux is established by current / ap = y This gives motor counter e.m.t.

- y t ~ K .Ik . up where o)p is the motor speed when motors are in parallel. Subscript p

stands for parallel.


IT h
'ap _ V, K -2 *p _ I0p
or
” Eas V /2 K Il los 2 o)s

Electromagnetic torque, Tt = Ka <{>la or Te = Cl2 ('•' <t>06 I a)


When in series, Tes = K I 2
L because Ias = IL

h
When in parallel, Tep = K = K •— because l aD= —
2 4 ap 2
l\
T k -4
• L ee. _ 4 1

K ir 4

S c an n ed by C a m S c a n n e r
Art. 4.14] D.C. M achines 483

This shows that for the conditions specified in Fig. 4.80, the ratio of speeds with motors in
parallel to tha wi h motor in series is 4, i.e. speed range is 4 . The ratio of torques with motors

in parallel to that when in series is — •For constant power input Vt IL from dc source, Fig. 4.80
offers constant power drive.

^ jk.ove that series-parallel armature control method offers only two discrete
speeds. This method is commonly employed for the speed control of dc series traction motors.
Example 4.60. A 4 kW, 230 V, 1000 r.p.m. separately excited d.c. motor is fed from 260 V
a.c. source through a single-phase full converter. At no load and with zero firing angle delay, the
motor draws 2 A and runs at 1100 r.p.m. The armature circuit resistance is 0.5 Cl. Voltage drop
in conducting thyristors is 2 V. For a firing angle delay o f 30° and rated armature current o f 20
A, compute
(a) the motor torque and
(b) motor speed.
Solution. With zero firing angle delay, a = 0°

= Ea + Iara+ 2

_ 2>/2 x 260 „ _ _ _
or Ka<$>u>m= ------ 1 ------- 2 x 0.5 - 2 = 231.08
71
231.08 x 60
Ka * = = 2.006 V-sec/rad
2n x 1100
(a) Motor torque, Te = K a tyIa = 2.006 x 20 = 40.12 Nm
2^2 x 260 x cos 30
_7t_____ " 0.5
(b) From Eq. (4.67), “ m= x 40.12
2.006 (2.006)
2 nn
= 95.072 rad/sec =
60
n = 907.87 r.p.m. = 908 r.p.m.
Example 4.61. The magnetization curve o f a 4-pole d.c. series motor was obtained by run­
ning the machine as a separately excited generator, field and armature currents being adjusted
to the same value. The following test data were recorded at a constant speed o f 800 r.p.m. :

Terminal voltage, V 200 375 443 485 510 518

Field current, A 15 30 45 60 75 90

vu; rioi tne speed torque and current-torque curves for mis series motor when connected to
b00 V d.c. source. The resistance o f the armature circuit is 0.3 Cl and that o f the field winding is
0.25 Cl. Find also the speed and current for a torque o f 300 Nm.
(b) In case a diverter o f resistance 0.25 fl is connected across series field winding, calculate
e sPeed and torque for armature currents o f 30 and 60 amperes.
(e) With no diverter, i f the field winding is tapped at 80% o f full series turns, compute the
sPeed and torque for an armature current o f 75 A.

"Scanned by C a m S c a n n e r
484 Electrical M achinery (A rt 4 .I 4

Fig. 4.81. Pertaining to Example 4.61.

Solution, (a) The solution of this example is given in tabular form below. The given data
is for the terminal voltage V, and the generated e.m .f. EaX, which would be given by
°i ~ Vt + Ia ra as the field and armature currents are equal. With applied voltage as 600 V
Ea2 = 600 - Ia (ra + rs). Now EaXocUl ^ and Ea2 ~ n2 $x.
. Eai 800
E'o 2 n 2

or n2 = 800 ~ 2
E..
Jul
Also. rp Ea2 Ia Ea2 Ia
1» ---------= ----------x 60
co.m2 2 Tt Hr
_ E a2Ia x 6 0 Etal
2n 800 £ o2 80 n Eal la

W « 15 30 45 60 75 90. A
V, !' 200 375 443 485 510 518, V
* 0 , = Vt + fa ra . 204.5 384 456.5 503 532.5 545, V
Ea2 = 6 0 0 -Ia 591.75 583.5 575.25 567 558.75 550.5, V
('•o+'V)

2315 1216 1008


n2 = 800 -=¥ 902 839 808,
Aal
r.p.m.
36.62 137.5 245.2 360.2 476.7
Tt = 8 0 n EalIa 585.5,
Nm.

arm currenworque curves are plotted in Fig. 4.81. For a toroue of 300
M e speed and current are found to be 940 r.p.m and 52.5 A respectively from the ^ a p h of

^ui n iut1"
Art. 4.14]
D .C. M achines 485

\
(6 ) For Ia = 30 A, If = l Diverter resistance, Rn r0.25>
= 30 = 15 A
Rd + r„ 0.50
For If= 15 A, / ^ y
Eai = 204.5 V (given)
Now Ea2 = 600 - 1 R ,r '
r + — d'8
° Rn + r.

- 600 - 30 (0.3 + 0.125) = 587.25 V

n2 = 800 x = 800 x ^87-25 „


Bui 204.5 " 2297 3 r P-m-
For If= 15 A, K <b— * 80
“ ~ 2n x 800
T -v a t _ 204.5 x 60
" 2 ^ 7 8 5 5 “ x 3 0 = 73-2 3 N m
For la = 60 A, //■= 30 A and Eol = 384 V
£ a2 = 600 - 60 (0.425) = 574.5 V

n 2 = 800 x = 1196.87 r.p.m.

r ' = H l T 5 i x 60 = 2 7 5 0 2 Nm.

current is 0.8 x 75 = 60 A m J "= 503 V


for which Eal U ° '8 7- F° r = 75 A - the t a l e n t field

For Ia = 75 A, Ea2 = 600 - 75 (0.3 + 0.8 x 0.25) = 562.5 V

n 2 = 800 x 5g 2g5 = 894.6 r.p.m.

rn 503 x 60 ^
e ~ 2n x 800 X 0 3 Nm‘
Example 4.62. A dc shunt motor, with an armature circuit resistance o f 0.3 Q, operates an
w ld o f° 3 m U e c enCy * ?° %' Calculate size ° f the dc motor for raising a load o f 800 kgs at a

speed °aSe tke SUPPly voltage f alls f rom its initial value o f 230 V to190 V, calculate thehoist

Solution. Here 800 kgs is the weight to be lifted up, therefore, the resisting force due to
gravitational pull = 800 x 9.81 = 7848 kgm /sec 2 = 7848 Newtons.
Power required for lifting the load = 7848 x 3 Nm/s or W

•■ Power rating of dc motor = input power to hoist = 7848 x 3


hoist efficiency
7848 x 3
= 33634 W = 33.634 kW
0.7
For supply voltage of 230 V, motor counter emf,

Ea = (250 - /0 x 0.3)
Ea Ia = Developed power in dc motor = 33634
or
l a [250 - 0.3 7J = 33634
or
0 .3 1\ - 250 L + 33634 = 0

V
^4 K_/ y WV_4III »_/ V_*V_4I II IVyI
(A rt. 4.14
486 Electrical Machinery

Its solution gives, lu = 196.70 A


For supply voltage of 190 V, Ea = (190 - 196.7 x 0.3) = 130.99 V
Assuming the armature current the same as before,
800 x v x 9.81
EaIa = 130.99 x 196.70 = --------

where u is the new speed of hoist.


_ 130.99 x 196.70 x 0 1 = 22gB ^
800 x 9.81
Exam ple 4.63. A dc series motor hoists a load at 6 m /sec while taking 60 A at bOO V from
dc mains. The total resistance o f the motor is 0.5 fl. Calculate the resistance to be added in the
motor circuit in order to slow the hoisting speed from 6 m l s to 4 m l s. Assume no magnetic
saturation.
Solution. Motor generated emf, Ea - (600 - 60 x 0.5) = 570 V
Let A" be the load in kgs to be lifted up. Under steady state,
motor electromagnetic power = power required for lifting up the load
Ea la = A 'x 9.81 x 6
or 5 7 0 x 6 0 = A 'x 9.81 x 6 —C«)
At lower hoisting speed, let Rtx be the external resistance to be added in the motor circuit.
Then
60 1600 - 60 (/?„ + 0.5)1 = A' x 9.81 x 4 .Mi)
From (i) and (ii),
60 1600 - 60 (Rrt + 0.5)1 _ A'x 4 x9.81
' ’ 60x570 " ~ A 'x 6 x 9 .8 1
or 600 - 60 (R „ + 0.5) = 380
or Afl = 3.167 f2.
Exam ple 4.64. A 4-polc dc motor runs at 600 rpm on full load and takes 25 A at 450 V. The
2 i0 ^
armature is lap wound with 500 conductors and flux per pole is given by <t>= {1.7 x 10' )-T
Weber, where 1 is the motor current. I f the supply voltage and torque both are halved, calculate
the speed at which the motor will run. Neglect stray losses. (I.A.S,, 1984)
Solution. Here 0 = 0.017 \7
For / = 25 A, 6 = 0.017^25 = 0 .0 8 5 Wb
r, bZnP 0 .0 8 5 x 5 0 0 x 6 0 0 x 4 ___ ___ _
=~ = ------------ 6 0 7 3 ------------- = 4 2 5 V
For dc motor, V, = Ea + IarQ
450 = 425 + 25 x ra
.-. Armature circuit resistance,
rG= in.
Motor torque, Te = Ka 6la
or Te = Ka x 0.017 < T -I = K a x 0 .0 1 7 115

^ = Ka x 0.017 7|-5

Te Ka x 0.017 (25)15 (2 5 xl 5
1.5 = 2
Te/2 Ka x 0.017 (J2)

Scanned by C am S canner
_ _ _ _ _ ------------------------- D.C. Machines 487

j 25
or h = ^273 = 15.75 A

T h e re fo re , n e w a r m a tu r e c u r r e n t, Ia2 = I2 = 15.75 A
Ea2 = 225 - 15.75 x 1 = 209.25 V
Eal = 425 V, = 600 rpm, = 0.085 Wb
$2 = 0.017 (15.75)1'5 = 0.0675 Wb
K 2 _ 4>iAT2
Eal §]Nl
209.25 0.0675 x N2
or
425 " 0.085 x 600
xr 209.25 x 0.085
Of 2 “ ~425 x 0.0675 * 600 = 372 rpm.
Exam ple 4.65. A d.c. shunt motor is required to drive (i) constant power load (ii) constant
torque load and (Hi) the load whose torque requirement varies with the speed.
Under rated conditions, the d.c. shunt motor takes rated armature current and runs at rated
speed. Find the speed and current, if the following changes are made.
(a) Armature terminal voltage Vt halved, field current If unchanged '
(b) Vt unchanged, If halved and
(c) Both Vt and If halved.
Neglect saturation and all the machine losses.
Solution, (i) Constant power drive means constant Ea Ia = V, Ia.

(a) With Vt halved and If unchanged, speed n — is reduced to half, i.e. n = \ p.u. Further
Ii
lf f x
with Vt halved, Ea is halved and, therefore, for constant power drive, armature current Ia must
be doubled, i.e. Ia = 2 p.u.

Alternatively, Vn Ia l= - £ f*c2
2
••• /„2 = 2/el = 2 P u-
(6 ) With Vt unchanged and If halved, the speed is doubled, i.e. n = 2 p.u. Further with Vt
unchanged, Ea remains constant and, therefore, Ia remains unchanged, i.e. Ia = 1 p.u.
(c) With both Vt and If halved, speed remains unchanged, i.e. n = 1 p.u. Further with both
Vt and If halved,

V , A2 a2

•• / o2 = 27al = 2 P-U-
(ii) C onstant torque drive m eans constant tyla or constant I fla, because saturation is
neglected.
As in part (/), the speeds for parts to), (6) and to) are respectively i p.u., 2 p.u. and 1 p.u.
(a) With V,halved and I , unchanged, current /„ remains unchanged because torque « /,/ „
^ust remain constant.
T h e re fo re , Ia = 1 P u -

C U uy W Q I I I J t / Q I II I d
488 Electrical Machinery [Art. 4.14

(b) With unchanged and //-halved, constant torque drive requires Ia to be doubled, i.e.
Ia = 2 p.u.
(c) With both V, and If halved, constant torque requires Ia to be equal to 2 p.u.
(iii) Load torque <*= n and electromagnetic torque Te « $ Ia ° r ^ 4 4 - Therefore l{ l a « n.
As in part (i), the speeds for parts (a), (6 ) and (c) are respectively j p.u., 2 p.u. and 1 p.u.
(a) With Vt halved and //-unchanged, speed is halved.
Ifi Iai “=n
and t i a
VI *a2 06 2

la 1
4 2 - 2 2 p U’

(b ) With Vt unchanged and //-halved, speed is doubled.

4i 4 i “ n
and 2n
2 *02

4 2 = 4 4 i = 4 p-u-
(e) With both Vt and //-halved, speed remains unchanged.

4 i4 n

and S ir
2 a20cn
4 2 = 2 4 i = 2 p.u.

Exam ple 4.66. A 4-po/e dc series motor has flux per pole d>= 4 x 10~3 Ia Wb where l a is the
armature current. The motor drives a fan requiring 40 Nm at 1000 rpm. The wave-connected
armature has 480 conductors and its resistance is 1 Q.
Find the motor speed and armature current i f it is fed from 230 V dc mains.
Solution. For a fan, load torque TL « N 2
For a speed of 1000 rpm, load torque requires 30 Nm

N
For a speed of N rpm, load torque required = 30 Nm
1000
Motor torque T —^ AT 4 * 480 . 32
- 2m a ~ "2jT)T2" x 4 x 1 ° 3 4
Under steady state, T . ‘ Tl

4x480 N
or x 4 x 10" 3 /;? = 30
27t x 2 1000
1 /2
30 2 ti x 2
or L =N = 0.007 N Ui)
1000" 4 x 4 8 0 x 4 x 1 0 "3
Counter e.m.f., Ea = V , - I a ra = ^

4 x 10 " 3 x 480 x N x 4
or 230-4x1 = J ii)
60x2

o c a n r ie u u y u a m o c a r i r i e i
D .C. M achines 489

Substituting the value o f /a = 0.007 N in Eq. (ii), we get

230 - 0 . 0 0 7 N = - 6 x 4 8 0 ^ 10 * N = 0.064 N
1ZU

230
... M o t o r s p e e d , N = = 3239.44 r p m

M o to r a r m a t u r e c u r r e n t , Ia = 0.007 x 3239.44 = 22.676 A

4 1 5 Efficiency and Testing of d.c. Machines


* It is preferable
preteraD ie 10 to rrefer
e ie r to
lu nArt. 3.9, where
i t . 0.17, w n ere losses
lubbes dnuand efficiency in electrical
c i i i u c i i t j in c ic t u machines have
. *1 __ J i n r r n n n r o l F n r H p m a p K i n o c i / a n
been described in general. For d.c. machines, the various losses are as follows : n n c I h c q p . q A T P SIS fo llO W S I

1 No-load R otation al loss. This loss is made up of two losses, namely (a) the iron loss at
orking flux and speed and (b) the mechanical loss, i.e. friction and windage losses at e
operating speed. Let the no-load rotational loss be represented by W0.

2 I2R loss, (a) Armature circuit loss 12 r0, where ra includes the resistances of brush con­
tacts, armature winding, interpole winding and compensating winding, if present.
(6) Field ohmic loss Vfly. Note that this loss includes the field-rheostat loss also. In series
machines, field ohmic loss forms a part of the armature circuit loss.
3 Stray load losses. These are produced by (a) the distortion of the air-gap flux due to the
armature Reaction and (6 ) the currents in the commutated coil It is “ n“ l 5 0 kW
stray load losses. In small machines, the losses are neglected, but for machines of abou
or more they are assumed to be 1% of the rated output.

E le c tro m a g n e tic
Power in p u t
from prim e p o w e r = Ea Ia Arm. terminal
m o ver power -
J Power output
Vnl Vt »a = Vt IL

m Series Shunt
Arm. Brush
F&W N o -lo a d Stray o h m ic contact tield ficld
lo s s e s core loss (load loss .lo ss loss 1ZR loss. IJR loss
-'v'- '
N o -lo a d ro ta tio n a l loss Arm. circuit ohmic loss
J J
L L
Rotational losses Total ohmic losses
(a)
E le c tro m a g n e tic
Pow er input p ow er - E q Iq S h a ft power
fro m m a in s ,V t l L
to Load

No-load
Shunt S eries Brushi Arm. core loss
field held contact R
]2r loss |*R loss loss loss N o -lo a d ro to tio nal
Arm. c irc u it ohmic loss
toss

Total ohmic losses Rotational losses

decompound generator and (ft) d.c. compound motor. *


Fig. 4.82. Power flow diagrams for long-shunt (o)

j u q i ii ic u u y w a i i u u a i ii ic i
• . . [A rt. 4.15
490 E le c t r ic a l M a c h in e r y ________________________
• Dv,„nt field is absent in a dc machine, it is
should be interchanged in Fig. 4.82. In case senes or shunt held
accordingly omitted in Fig. 4.82. generated emf Eu in the armature of a
The electromagnetic power is associated with m e g ^ ^ gometimes ca]led rota
dc machine. The sum of no-load rotational loss a” d fronij electromagnetic power gives
tional loss. The rotational loss when added to, or shaft-power output from dc motors.
respectively the shaft-power input to dc genera“ from> electromagnetic power give
Also, the total ohmic losses when added to or s lectrjcal power output from thedc
respectively the total electrical power input to dc motor or

^ T l t l . Efficiency o f d.c. machines. In general, the efficiency is g.ven by


Losses
^" Input
Losses
For a generator, H* = 1 ~ y ( l l + Losses

ra + v / 4 + Wo ...(4 .6 8 )
1 + V f + H ,.>
where V, and IL are the output voltage and output current respectively and W„ is the no-load

rotational loss.
_ I2
a ra + Vflf+W 0 (4 69)
For a motor, - 1” y t jL
where Vt and IL are the input quantities.
Maximum efficiency. The field winding loss V ,lf and the no-load rotational losses W.
remain approximately constant. The maximum efficiency, as before, occurs when variable los­
ses ll ra are equal to the constant losses V ,l,+ W„. This can, however, be proved as follows .

V ,h
G e n e r a t o r e ffic ie n c y , ns = T T v fr T w ,

In shunt or compound machines, assume that the field current 1, is negligible as compared
with the line current 4 . With this assumption,/L = 4 and generator efficiency becomes

VJl
V,lL + llra + V/f + Wc

For a given value of V„ the maximum efficiency occurs, when

dr\, _ [VtlL + llrg + V ft + W0] V, - V t IL [Vt 4- 2l Lra\ Q


dIL [Denominator)2
or [VJl + f r a + VfIf+W0)V t = Vt IL [Vt + 2ILra)
or l l r a = Vf If + W0
or Variable armature circuit loss = Constant loss.
Same conclusion can be arrived at, from Eq. (4.69) for a d.c. motor.
E xam ple 4.67. A 10 kW, 250 V d.c. shunt generator has total no-load rotational loss of 40^
watts. The armature circuit (including brushes) and shunt field resistances are 0.5 fl an
fl respectively. Calculate the shaft power input and the efficiency at rated load. Also ca c
the maximum efficiency and the corresponding power output.

S c an n ed by C a m S c a n n e r
D.C. Machines
Art- 4.15]

S o lu t io n . Constant shunt field current


• t _ 250 _ i .
f 250
... Constant losses = Shunt field losses + Rotational losses
= 250 x 1 + 400 = 650 watts.
Generator output current at rated output

/ _ 10»000 _ 40 a
1 250
... G e n e r a t o r a r m a t u r e c u r r e n t Ia = IL + lf = 40 + 1 = 41 A.

A r m a tu r e c ir c u it lo s s = I2 ra = (41)2 x 0.5 = 840 W.


G e n e r a to r s h a ft p o w e r in p u t = 10,000 + 650 + 840 = 11490 w a t t s .
1490 ^
At rated load, r\g = 1 - x 100 = 87.02%
11490
Maximum efficiency occurs when '
Variable losses = Constant losses = 650 W.
Total losses at maximum efficiency = 650 + 650 = 1300 watts.
But variable losses = l2
a ra = 650 W.
/. Armature current at maximum efficiency is
f650
= 36 A
0.5
Generator output current = Ia~ l f = 35 A.
.-. Generator output power = 250 x 35 = 8750 W
Generator input = 8750+ 1300 = 10050 W
1300
Maximum n = 1 - x 100 = 87.07%.
10050 /

Example 4.68. A 250 V, 15 kW, shunt motor has a maximum efficiency of 88% and a speed
of 700 r p m , when delivering 80% o f its rated output. The resistance of its shunt field is 100
a Determine the efficiency and speed when the motor draws a current o f 78 A from the mains.
Solution. Rated output = 15 kW
Output
Efficiency n = 0utput + Losses
/ 1 "i
or Total losses = — - 1 Output
n

- 1 x 0.8 x 15,000 = 1636 W.


0.88
At maximum efficiency,
= (Rotational losses + Shunt field losses)
Constant losses
= Variable armature circuit losses l a r„
1636 __ gjg w
2

Input to motor at maximum efficiency


= 0.8 x 15000+ 1636 = 13636 W.

cann ed by C a m S c a n n e r
492 Electrical Machinery ( A r t 4.15

Input line current IL1 = " g ’gQ ~ = 3 4 6 ^

Armature current Ia\= 54.6 - = 52.1 A

But / 2Xra = (52.1)2 x ra = 818 W


.*. Armature circuit resistance,
818
ra = = 0.301 a .
(52.i y
Now motor input current,
IL2 = 78 A
Ia2 = 78 - 2.5 = 75.5 A.
Now J22 ra loss = (75.5)2 x 0.301 = 1718 W
Total power input = 78 x 250 = 19,500 W
Total losses = 1718 + 818 = 2536 W
.'. Motor efficiency for a line current of 78 A is
' 2536 ^
1 - x 100 = 87%.
19,500
V ✓

Now counter e.m.f. Eal = Vt - Ial ra = 250 - 52.1 x 0.301 = 234.32 V


Ea2 = 250 - 75.5 x 0.301 = 227.28 V

But = 234 32 = O il1 = 700 x h


Ea2 227.28 n2 <J>2 n2 x <f>2
For constant field current,^ = <J>2 .
. _ 700 x 227.28 ___
•’ 234.32 = 6 7 8 r -P-m-
E xam ple 4.69. A 10 kW, 900 r.p.m., 400 V d.c. shunt motor has armature circuit resistance
(including brushes) o f 1 Cl and shunt field resistance o f 400 Cl. I f efficiency at rated load is 85%
then calculate: '
(a) the no-load armature current,
(b ) the speed when the motor draws 20 A from the mains and
tc) the armature current, when the total (or internal) torque developed is 98.5 Nm
Assume the flux to remain constant.

Solution. Total losses = f — _ 1 x rated output


0

' 1
0 .8 5 ' 1Jx l 0 '0 0 0 = 1765 w -
(a) Total input = io,000 + 1765 = 11,765 W.

/. Line current 1L = i L Z § 5 = 29.41 A.

Armature current at rated load /„ = 29 41 - — - or 41 a


400 1 A'
Armature circuit loss = l\ r Q= (28.41)2 x 1 = 810 W .
Shunt field losses = lf y f - \ x 400 = 400 W.

s
S c an n ed by C a m S c a n n e r
A rt- 4.15]____________________
D.C. Machines 493
N o -lo a d r o t a t io n a l lo s s
= Total losses - r _ jy _ _

= 1765 - 810 - 400 = 555 W. f


But - w
W0-=, VM J _r2y - V = 0 0 0 W.
° * ao l ao r
ii
555 = 40 0 1 - /2 y i
. *ao oo xA
If armature circuit loss at no load ic nnni * j
H if fe r aappreciably.
not differ n n r e c ia h lv fleeted,
s the value
a>the value of no-load armature current will

.-. No-load armature current I _ 655^_


00 400 ~
(6) At rated load, £<>1 = 400 _ 2g 4J = ^ y

For motor line current of 20 A, the armature current is


4 = 2 0 - ^ = 2 0 - 1 = 19 A.
E a2 = 400 - 19 x 1 = 381 v.
The speed at rated load is 900 r.p.m.
i’
i
A fo i = 371.59 _ M i _ 900 x
Ea2 381 n2<\>2 n2 x <J>j
381
n 2 ~ g y j g g x 900 = 924 r.p.m.

(c) At rated load, Eal = K j^ (om= 371.59 V


^ A 371.59 x 60
= 2kx900 = 3'94'
Now electromagnetic torque Te = Ka^ Ia = 98.5 Nm
98 5
Armature current /_ =— — = 25 A.
3.94
Example 4.70. A 10 kW, 240 Vdc shunt motor draws a line current of 5.2 A while running
at no-load speed o f 1200 rpm from a 240 V dc supply. It has an armature resistance of 0.25 f2
and a field resistance o f 160 fl. Estimate the efficiency of the motor when it delivers rated load.
(GATE, 1993)
Solution. Constant shunt field current
' 240
= 1.5 A
160
No-load armature current1, Iao = 5.2 - 1.5 = 3.7 A
Constant no-load rotational loss, W 0 —V f Iq o Iao r a

= 240 x 3.7 - (3.7)2 x 0.25 = 884.6 W


Electromagnetic power = E Ja = shaft power + no-load rotational loss

(240 - 4 x ° -25) 4 = 10»000 + 884 6


or o.25 72 - 240 Ia + 10884.6 = 0

Its solution gives Ia = 47.73 A


Motor input = E J a + full-load ll r Q loss + shunt field loss
= 10884.6 + (47.73)2 x 0.25 + 1.5 x 240 = 11814.14 W

. M , c~ . 10,000— , QQ _ 84.64%.
■• Motor efficiency = ^ygTTT4

S c an n ed by C a m S c a n n e r
(A rt 4.1S
494 Electrical Machinery

Exam ple 4.71. A 440 V dc shunt motor has a no-load (la = 0) speed o f 2000 rpm. It is run-
ning at 1000 rpm at full load torque, reduced armature voltage and ful fie . 00 *s
reduced to 50% o f rated value with armature voltage and field vo taSe e 0
values, the speed increases to 1050 rpm. Find the armature voltage drop at full loadNeglect
rr a c a j* \\jA1 £j, 1994)
effect o f armature reaction.
Solution. Let lal be the armature current at full-load torque.

Counter emf Eal at full-load torque is


£ al = ( 4 4 0 - / aira) « - (,)
Motor torque, Tel = Ka 0 ,/ai

Now

As the field voltage is held constant, 02 - 0i


TtX Kat>xIa,
=2
Ttl/2 Ka 0i Ia2
/ u1
-JLL
la2~ ~

Counter emf Eo2 at half rated torque is


1 ...(ii)
E„2 ~ (440 - •ra) N 2 02

^ 0 ,
From (*) and (**'), 5 -*
&a2 N j02
440 - /„ , ra 1000 x 0, 20
440 - \ l a\ra 1050x0, 21
or 440 x 21 - 21 !alra = 440 x 20 - 10 Ialra
440
or Ial ra = ^ = 40 V.

.*. Armature voltage drop at full load = 7al ra = 40 V.


E xam ple 4.72. A dc shunt motor, with armature circuit resistance o f 0.1 ft, runs at 1600
rpm while taking an armature current o f 100 A from 230 V dc source. The friction and windage
loss is 300 IV, no-load core losses are 1200 IV and the total I2R loss is 2500 W. Stray load loss
equals 1.0% o f the output.
Find the shaft torque o f the motor and its efficiency.
Solu tion . Motor counter e.m.f., Ea = 230 - 100 x 0.1 = 220 V
Electromagnetic power = E a Ia = 220 x 100 = 22,000 W
No-load rotational loss = 300 + 1200 = 1500 W
Subtracting no-load rotational loss from electromagnetic power, we get
22000 - 1500 = 20500 W .
Let Psh be the shaft powder output.

.*. Stray load loss = ft.01 Psh


... 20500 - 0.01 Psh = Psh

Shaft power, Psh = = 20297 W


1.01

\ j O U l II I U U IU J V- y UI I l x _^OUI II IV-rl
D .C . M achines 495
4.15)

20297 x 60
Shaff torque, = 121.14 Nm
2 n x 1600

power input to motor = electromagnetic power + total I2R loss


= 22000 + 2500 = 24500 W
_ shaft power 20297 = %
M o to r e ffic ie n c y
output power 24500
A 15.2. T estin g o f d.c. m ach in es. D C. machines can be tested by th ree d ifferen t
■• _ . . . . . .« i m l ______ - - h /-»n o

are now described briefly.

Direct M e th o d . This method is suitable only for small d.c.


machines. In direct method, the d.c. machine is subjected to rated load
: jI
and the entire output power is wasted. The ratio of output power to '1
input power gives the efficiency of a d.c. machine.
For a d.c. generator, the rated output power is wasted in resistors.
For a d c motor, a brake lest is carried out as illustrated in Fig. 4.83. A
belt a r o u n d the air- or water-cooled pulley has its ends attached to spring
balances Sj and S 2 The belt tightening hand wheels / / , and H2, help in
adjusting the load on the pulley and. therefore, on the motor. If spring Pulley
balances are calibrated in kilograms, the motor output is given by, pjg 4 83. Brake test on
Motor output = to (Sj - S 2) r x 9 81 watts. a ^ c' motor

where S , and S 2 are the tensions on the tight and slack sides of the belt, r is theeffective radius
of the brake-pulley in metres (= ^ outside pulley diameter + j belt thickness) and to (= 2Tin) is
the motor speed in rad/sec
If V, is the motor terminal voltage and 1L is the line current, then power input to motor
= V,/; watts, and percentage motor efficiency,
ia(Sj - S 2)r x 9.81
x 10 0 ...(4.70)
v) l

For a series motor, the brake should be sufficiently tight before the motor is switched on to
the supply.
The size of the motor that can be tested by this method, is limited from the consideration of
the heat that can be dissipated at the brake drum or pulley.
The disadvantages of this method are that <i> the spring balance readings are not steady
and Ui) the friction torque, at a particular setting of the handwheels H j and H 2, does not remain
constant.
Example 4.73. A full-load brake test on a small d.c. shunt motor, gave the following data :

Spring balance readings 25 kg and 9 kg


Outside pulley diameter ^S.5 cm
Belt thickness cm
Motor speed 1500 r.p.m.
Applied voltage 230 V
Line current 12.5 A.
Calculate the shaft torque, shaft power and the motor efficiency at rated load.

cann ed by C a m S c a n n e r
I

496 Electrical Machinery (A rt. 4.15

Solution. Shaft torque = (Sx - S2) x Effective radius


19.5 0.5
= (25 - 9)
2 2 I 3 5 k6- m
= 16 x 0.1 x 9.81 Nm = 15.7 Nm.
Shaft power = 0) x Shaft torque
2k x 1500
x 15.7 = 2470 W
60
Motor input = V,IL = 230 x 12.5 = 2875 W
/. Motor efficiency at rated load
2470
hm = 2875 x 100 = 85.6%.
Indirect method. In this method, the no-load machine losses are first measured by a
suitable test and then the additional losses on load are determined
from the machine data, in order to calculate the machine efficiency.
The simplest method of measuring the no-load machine losses is by
Swinburne’s method.
Sw inburne’s method. As this is a no-load test, it cannot be per­
formed on a d.c. series motor. In this method, the machine, whether it
is a motor or a generator, is run as a no load shunt motor at rated speed
and with rated terminal voltage Vt. If Iao and If are the no-load arma­
Fig. 4.84. Swinburne’s
ture and field currents respectively (Fig. 4.84), then the -power ab- method of measuring
sorbed by the armature (= is equal to the no-load rotational loss no-load losses of a d.c. motor.
W0 plus a small amount of armature circuit loss /£ , ra.

.*. No-load rotational loss Wa = Vt Iao —1%0 ra.


Here armature circuit resistance ra includes the brush contact resistance also.
Shunt field loss = Vt ■If
Let IL be the load current at which the machine efficiency is required.
G enerator efficiency. Generator output = V, IL
Armature current h =h + If
Armature circuit loss = I a2 r1 a

Note that here ra is the armature circuit resistance when hot.


Shunt field losses = If Vt
Total losses = W + f*r + L V
” o T 1o' a T yI

... ,, =1_ X + 4 r .+ ;,y ,


* V , h + W0 + l l r „ *
M otor efficiency. When the machine is working as a motor then I = 1 , - 1 ,
Motor input = VfIL °

„ wo + l ‘ r„ + Ir V,
^ W l---------

V, + voltage ^drop in r f ’o t rated ^ n t u r e ’ c u m n U f m M h ™ 11’ 11!? ShoU’ d be equal l ° ra lcd ,c r " ’ ilu>l vol“ Kt
under test is n m otor thp T Under tcst is a gen erator. In case the machine
a n n a t u r e current ‘ “ ‘ hC a™ “lUro ah»“ 'd ■ » - * , - voltage drop in a. rated

S c an n ed by C a m S c a n n e r
A r ^ ifl __________________________________________________D.C. Machines 497

^ ^ j i r n e s method are (a) low power required for testing even large
m*° m/,u:,T,p rnn *1° osses are to be supplied from the mains and (6) the efficiency of
the machine can be calculated at any desired load.
Since the dx. machine in Swinburne’s method is running at no load, the disadvantages of
this me • a s ray °ad losses can’t be accounted for and (6) the actual performance of
the mac me on ra e oa can t be checked. For example, Swinburne’s test gives no indication
3 °U f l a ■ ? £°m™utatl°n ° n rated load is satisfactory or whether the temperature rise
on rated load is within the specified limits.
Exam ple 4.74. A 400-Vd.c. shunt motor takes 5 A at no-load. Its armature resistance (includ-
'fhe motor takes 50 A o n f u l l v d ^ res^stance ^ Estimate the kW output and efficiency when

Find also the percentage change in speed from no load to full load.
Solution. Constant shunt field current
400
/ r ^ = 2A
200
.*. No load armature current, Iao = 5 - 2 = 3 A.
Constant no-load rotational loss,
Wo = V T - I‘ ■ao
v t *a o
2 rra
= 400 x 3 - (3)2 x 0.5 = 1194.5 W
At full load, the armature current
7o = 5 0 - 7 /- = 5 0 - 2 = 48 A.
Full load armature circuit loss = ra = (48)2 (0.5) = 1152 W
Constant shunt field losses = V t L = 400 x 2 = 800 W
.-. Total losses at full load = 1194.5 + 1152 + 800 = 3146.5 W
Motor input at full load = 50 x 400 = 20,000 W.

.’. Motor efficiency 1- 1* 100 = 84.27%.


20 , 000 ^
For a shunt motor, field flux <f>is constant, therefore, the motor counter e.m.f. Ea is propor­
tional to speed n. Thus the percentage change in speed from no load to full load is the same as
the percentage change in counter e.m.f. from no load to full load.
No load e.m.f. Eao = Vt - Iao ra
= 4 0 0 - 3 x 0.5 = 398.5 V
Full load e.m.f. Ea = 400 - 48 x 0.5 = 376 V
Percentage change in speed from no load to full load
3 9 8 .5 -3 7 6
x 100 = 5.65%.
398.5
Exam ple 4.75. A 400 V, 20 kW dc shunt motor takes 2.5 A when running light. For an
armature resistance o f 0.5 Q, field resistance o f 800 ft and brush drop o f 2 V, find the full-load
efficiency.

Solution. Field current, lf= = 0.5 A

No-load armature current, Iao = 2.5 - 0.5 = 2.0 A

S c an n ed by C a m S c a n n e r
1

498 Electrical Machinery [Art. 4.15

Constant no-load rotational loss,


wr. = V , / „ , - / L r „ - V ^
= 400 x 2.0 - 22 x 0.5 - 2 x 2
or W0 = 794 W
Total constant losses = W0 + V, •f = 794 + 400 x 0.5 = 994 W
At full load, let IL = line current = Ia +If
Power input = Power output + total losses
V, ■IL = 20,000 + 994 + I2
ara + V^
400 (/„ + 0.5) = 20,000 + 994 + 0 .5 1\ + 2Ia
or 0.51] - 398/a + 20794 = 0
_ 3 9 8 1V398* - 4 x 0.5 x 20794 _ 22 A
— 1
Armature ohmic loss = 56.222 x 0.5 = 1580.34 W
Brush-drop loss = 56.22 x 2 = 112.44 W
.-. Total losses = 994 + 1580.34 + 112.44 = 2686.78 W
Power input = 20,000 + 2686.78 = 22686.78 W

Full-load efficiency of shunt motor = x 100 = 88.157%.


2 2 b o b . /o

Exam ple 4.76. Find an expression for the maximum steady state power output of a dc shunt
motor with an armature resistance ra, if the applied voltage V, and the shunt-field current fa r e
kept constant. Neglect rotational losses and assume that the theoretical maximum power output
will not overload the motor.
Solution. Power output P of a dc shunt motor is given by
P = Power input - total losses
= VJl - fr a ~ ~ rotational losses
But Ia = f - f and it is given that rotational losses are neglected.

••• P - V t ( h - If) - - VJc

In the above expression, it is given that applied voltage V, is constant. The only variable in
above expression for power output P is Therefore, differentiating it with respect to we get
dP
57- = V , - 2 / oro - y 6 = 0

Thus, armature current under the condition of maximum power output is


j vt - v b
am- 2ru

Substituting this value of armature current in (/), we get the maximum output power.

P - V‘ {V' ~ Vt) <V< - Vk>2 Vh ( V , - V b)


1 m ax n --------------- o------- r „ ---------------- —
“ 4r2„ " 2 r„

_ V?-V|Vt - VVb + Vt
2r0 4ra

S c an n ed by C a m S c a n n e r
Art. 4.15]
D.C. Machines 499

= 2^ - 4 V ,V b +2 2V,V„
4ra

p niax = v ' + v * - 2VW (V, - Vbf


4r„ ~ 4rn
In case brush contact loss VbIa is neglected, then

' p
" " 4r0

t.nfu a^ Vu OI- ^^P ^in son 's Method. In this method, two identical d.c. machines are
coupe , o mec anica y and electrically and are tested simultaneously. One of the machines
is ma e o run as a mo or and it drives the other machine as a generator. In this section, the
regenerative test on two identical d.c. shunt machines is described.
For performing Hopkinsons test, Fig. 4.85, machine I is started as a d.c. shunt motor by a
starter (not shown) and brought up to rated speed with switch S open. Both the machines I and
II run at the same speed, because these are mechanically coupled. The field current I „ of the
second machine is now adjusted and when the voltmeter across switch S reads zero, switch S
is closed. The magnitude of the armature currents I x and 72 can be adjusted to any value by
varying the field currents In and 7^. The machine with a lower value of field current runs as a
motor, because its counter e.m.f. would be less than the e.m.f. generated in the armature of the
other machine. In Fig. 4.85, if 1^ is greater than In , then machine I acts as a motor and machine
II as a generator. Thus machine I running as a motor drives machine II as a generator. Since
both the machines are coupled electrically, the power output of generator II is fed to the motor
I and it is for this reason that Hopkinson’s method is called a regenerative method. Consequent­
ly the power drawn from the supply should only be sufficient to overcome the losses in both the
machines. The speed of the set should be equal to the rated speed. If the speed falls, it can be
corrected by decreasing motor field current ln — this has the effect of decreasing EaX and in-
• j ^ o2 —Ea 1
creasing/2 = — and, therefore, I x (= I + / 2) also increases. Increase in I x more than com­

pensates the decrease in 7^, therefore, motor torque « 7,7^ increases and speed of the set
becomes more. I f £ al is made greater than Ea2 by increasing In and decreasing 7^, the functions
of the two machines can be interchanged, i.e. machine I can be made to work as a generator and
machine II as a motor.
In Fig. 4.85, 7, and 72 are the armature currents of motor and generator respectively and
V, is the terminal voltage of both the machines, i.e., Vt = dc supply voltage.

S c an n ed by C a m S c a n n e r
500 Electrical Machinery _ ____________________________________________

.\ Input to motor armature = Vt I\


If r|m is the motor efficiency, then the motor output = r)m VJi

Generator output = VJi-


If ru is the generator efficiency,
Vt I 2
Then the generator input =——

Since the generator is driven by motor, Fig. 4.85 reveals that motor output,

* input,
Vt /i •Tl„, = generator • .

h
If the efficiencies of the two machines are assumed equal, i.e. T|m = fig = then
Generator armature current
Motor armature current ' '
Actually that efficiencies of the two machines are not equal, because of the following two
reasons :
(а) The motor armature current I x ( = l + / 2) is greater than the generator armature current
/ 2 and thus the armature circuit loss in motor is more than the armature circuit loss in gener­
ator.
* S
(б) The generator field current is greater than the motor field current. Since both machines
are running at the same speed, the generator iron losses (« speed and flux) are more than the
motor iron losses.
For calculating the efficiencies, the above two reasons can be taken into consideration as
follows :
-. If ra is the hot resistance of each armature circuit, then

' armature circuit loss in generator (II) = l\ ra (


and armature circuit loss in motor (I) = l\ ra
Power drawn from the supply (excluding the field losses in both the machines) = VtI
No-load rotational loss in two machines,
W0 = V , I - r , (/? + /!).
In case resistance of generator armature circuit is not equal to the resistance of m otor
armature circuit, then

Wo = V , I - i \ r am- t l r ag \
where ram = resistance of motor armature circuit
and rag = resistance of generator armature circuit
Assume that the no-load rotational losses for each machine are the same.
. W
No-load rotational loss for each machine = — -
2
Generator output = Vt I2
Wo
Generator loss, Wg = + Vt 1^ + l\ra

S c a n n e a by L a m b c a n n e r
Art. 4.15]
D.C. Machines 501

Generator efficiency, ' 1 _____ W


. v V ft+ W
Motor tnpiit = Vt (7 j + 7^) 7

Total motor losses, u/ _ . r7


m~ ~22 + V t In + I i r l

Motor efficiency, W,m


flm = 1 -
. , - ^ + ip) j
nZato^are^mo^p^V^811^ 011 e'!n Senerated in the armature, therefore iron losses in
ofnrp mrrpnt m ° i°r *r.on l°sses. The motor with low field current has more
arm . t • ’ f , distortion of the field flux in motor is more than in the
, • 1S’ i 8 s^ray ^oad losses in motor are more (iron losses are less) than in
/■ ^^Tn^fViP ff0rf lf osses^- Thus the assumption of equal division of rotational losses
(including the effect of stray load losses also) is quite valid.
The advantages of this method are : (a) the method can be used for large size machines
because the power drawn from the d.c. source has to furnish only the losses in the two
machines, (b ) the machines can be tested under rated load conditions and thus the temperature
rise and commutation process can be checked and (c) the efficiency is being determined under
rated load conditions, therefore, the stray load losses are included.
The main disadvantage lies in the requirements of two identical d.c. machines.
Example 4.77. Two identical d.c. shunt machines when tested by Hopkinson’s method, gave
the following data :
Line voltage 230 V ; line current excluding both the field currents 30 A ; motor armature
current 230 A, field currents 5 A and 4 A.
If the armature resistance o f each machine (including brushes) is 0.025 Q, calculate efficient
cy of both the machines.
Solution. For this example, refer to Fig. 4.85.
Since the motor armature current I x is 230 A, the generator armature current is
/ 2 = /j _ I = 230 - 30 = 200 A.
Armature circuit loss in generator
= (200)2 (0.025) = 1000 W
Armature circuit loss in motor = (230)2 (0.025) = 1322 W.
Power drawn from the supply (excluding the field loss in two machines)
= VtI = 230 x 30 = 6900 W.
No-load rotational loss in both the machines
W0 = V )
= 6900 - (1322 + 1000) = 4578 W
No-load rotational loss for each machine
W„
= — = 2289 W.
2
For generator, output = VJ2 ~ 2^ ° x 20^ 46>°°0 W
»V0 TTY t2
Total losses, Wg = - s2 " + + 72 r«
= 2289 + 230 x 5 + 1000 = 4439 W

S c an n ed by C a m S c a n n e r
502 Electrical Machinery IAn. 4.15

4439 x 100 = 91.2%


••
n*= 1 - 46,000 + 4439^

For motor, input = V f(7 , + In ) = 230 (230 + 4) = 53,8 00 W


wn
Total losses, W „ = ^ + l ] r , + V,In

= 2289 + 1322 + 230 x 4 = 4531 W


4531
0m = 1 - x 100 = 91.58%
53,800
In case both the machines are assumed to have the same efficiency, then
200
= 0.93 or 93%.
230
Field's test for series machines. In this method, two identical d.c. series machines are
required which are then mechanically and electrically coupled together. This test gives efficien­
cies of both the machines. The scheme of Motor s e rie s fie ld
connections is illustrated in Fig. 4.86.
Machine M acting as a motor drives
machine G as a generator. The series
fields of both M and G are connected in
series with the motor armature. In order
that the load is not thrown off accident­
ly, the generator armature is connected
to load directly without any switch.
The motor M is started in the usual
Fig. 4.86. Field’s test on two dc series machines.
manner and the generator output is
wasted in the variable resistive load. The voltage V2 across the motor terminals, should be
equal to its rated value. Consequently the supply voltage Vt should be equal to V2 plus voltage
drop across the generator field.

Since the motor and generator fields are in series, the iron losses in both machines are the
same. The resistance of the various windings can be measured easily.
Let Vt = supply voltage
. I = motor input current
Vl = generator terminal voltage
an^ h - generator output current.
Power input to the whole set = V, /
Power output of the generator = VlI 1

Total losses in the whole set, W = V , I - Pj/j

Total ohmic losses,’ WrC = 72


* (Vr' am+T r' sm+r rrsg)\ ++ 1
I2 r
\Fag.
Here the subscripts g and m denote generator and motor respectively. For example. ran is
the motor armature circuit resistance, rfi, is the generator series field resistance, etc.
No-load rotational loss of both the machines,
W0 = W - W C
W
/. No-load rotational loss of each machine = — .
2

b c a n n e a oy u a m b c a n n e r
Art. 4.15)______________
D.C. Machines 503
M o to r p o w e r in p u t = V J .

W0
+ (r 4 •r \
Motor efficiency, n - 1 _2
' m~ V
Generator input - u 7 ± r2 *
~2" r* + *?'Vr
W
_ _ £ + T2 j2
Generator efficiency, r) = 1 _ 2 '* 1 r°*

Vt/ t + ^ r + / % + i f r w

Note that the Field's test (known after Mr. M.B. Field) is not a regenerative one because
the generator output is wasted in load resistance and not fed to to e S

m tJ J fm m e V L r ld ™ ^ depends 0n the ac™racywith which the motor input and generator

, 0n Z UGu motors is possible in case of small machines. Swinburne’s method of


si g possi e, ecause senes motors have the tendency of attaining dangerous speeds
a no- oa . n view o is, the Field s test is quite suitable for d.c. series machines. At the same
time, there is no difficulty in obtaining two similar d.c. series machines, because these motors
used for traction purposes, are usually available in pairs.
Exam ple 4.78. A Field’s test on two similar series machines gave the following data :
Motor : armature current = 60 A
voltage across armature = 500 V
voltage across field = 40 V
Generator: terminal voltage = 450 V
output current = 46 A
voltage across field = 40 V
Armature resistance (including brushes) of each machine is 0.25 Q. Calculate efficiency o f
both the machines.
Solution. Power input to the whole set = (500 + 40 + 40) (60) = 34,800 W
Generator output = 450 x 46 = 20,700 W.
.*. Total losses in the whole set = 34,800 - 20,700 = 14,100 W
Total ohmic losses = (60)2 x 0.25 + 60 (40 + 40) + (46) x 0.25 = 6230 W
.*. No-load rotational loss of each machine
_ 14,100 - 6230 ^ 3935 w
_ 2
Motor power input = (500 + 40) (60) = 32,400 watts.
Total motor losses = Armature circuit loss + Field circuit loss + No-load rotational loss
= (60)2 x 0.25 + 60 x 40 + 3935 = 7235 W.

Motor efficiency, ^ - fl - J ! ! ) * 100 = 11


Total generator losses = Armature circuit loss ♦ Field circuit loss ♦ No-load rotational loss
= (46)2(0.25) + 60 X 40 + 3935 = 6865 watts.

Generator input = 20,700 + 6865 = 27,005 W.

S c a n n e d Dy u a m b c a n n e r
504 Electrical Machinery (A rt. 4.16

6865
•*. Generator efficiency, r|# = 1 - x 100 = 74.49%.
27,005
4.16. Rotating Amplifiers
Electronic amplifiers can give outputs economically up to a certain power level and for large
amplified power outputs, rotating amplifiers are used. These amplifiers are in common use in
modern automatic electric drives, because they possess high power gain and can work
favourably under transient conditions.
There are three types of rotating amplifiers, namely, (a) machines with self-excited tuned
fields, (6) machines with a highly saturated magnetic circuit and (c) cross field machines. Since
the rotating amplifiers of the cross-field type are more common, only these are described in this
article.
Ordinarily, a separately excited d.c. generator, driven at constant speed, is a single-stage
rotating amplifier, the power gain being ^>° W.e,r 0U*'PU^ but the term rotating amplifier is not
field power
applied to it. The power amplification of 20 : 1 to 100 : 1 is obtained in one d.Q. generator
according to its size. For larger power gain, two stages as shown in Fig. 4.87 are used, the
overall power gain being the product of the power gain of each machine. Only a small power is
required by the field of the first generator
(called exciter) and the power output of the 4>n
second generator is sufficiently large to give a
high overall power gain. In such a scheme, if _Fi«M of second
m achine
ordinary d.c. machines are used as shown in Field of
Fig. 4.87, the transient response of the system 1st machine
is very sluggish, because of the large field
winding inductances. However, the two stages
o f am plification, can be combined in one
machine as shown in Fig. 4.88 (a), in which one +6 o—
armature winding behaves as two separate ar­ OC _
matures superimposed. The time constants of G e n e ra to r Exciter
such a machine are considerably less, giving Fig. 4.87. Two d.c. generators connected electrically
in cascade.
quick response to control. Such two stage
generators are called rotating amplifiers and in these the armature reaction m m f of the first
stage is made to act as field flux for the second stage. It should be noted that one armature
winding can behave as two separate armatures only if there are two fluxes electrically in quad­
rature and two sets of brushes per pole-pair. The armature reaction m.m.f. is sometimes called
the cross-reaction of the armature and it is for this reason that the two-stage generators are
also called cross field machines.

4.16.1. Cross-field or metadyne generators. The brush axes of the exciter and d.c.
generator of Fig. 4.87, are purposely shown in quadrature, so that the superposition of the two
armatures results in the cross-field generator or metadyne generator of Fig 4 88 (a) The field
winding F produces a flux <fy along the d-axis. When the armature rotates in the flux <Jy, an
e.m.f. appears across brushes AB and this e.m.f. circulates current in the q-axis field winding,
called an ampliator winding. The flux <J>V set up by the ampliator winding, generates a second
e.m.f. across the output brushes C and D which are connected to the load.

Note that the q-axis flux is produced due to both the ampliator winding m.m.f. and the
armature reaction m.m.f. If the armature m.m.f. alone is sufficient to produce the desired flux
t q, the brushes A, B may be joined together to give the schematic representation of cross-field
________ a aa t . __ ______± _______ _ , . . . . . _
generator of Fig. 4.88 (6). In ordinary jd.c. generator, the armature reaction is a troublesome

S c an n ed by C a m S c a n n e r
Art. 4.16] D.C. Machines 505

i Quadrature
f* axis

(a)
Fig. 4.88. Cross-Held or metadyne geiieiatuis tu; wiui mnpuawi
and (6) its schematic representation.

feature but in cross field generator it is put to advantage as the field flux of the second stage.
Note also that the generation of e.m.f. across brushes A, B due to flux <|y, is the first stage of
generation and the generation of e.m.f. across brushes C, D due to the flux <j>,, forms the second
stage of generation.
Construction. The pole structure of a cross-field generator is different from that of the d.c.
machine, the pole configuration for the latter, for a 2-pole machine being shown in Fig. 4.89 (a).
If the same pole structure is used for the metadyne, Fig. 4.89 (6), then the coil (undergoing
commutation) short-circuited by the output brushes C, D would be cutting the maximum d-axis
flux ((y and would have, therefore, maximum rotational e.m.f. generated in it. As a result of it,
a large circulating current may flow in this short-circuited coil, giving rise to poor commutation.
The flux density at the position of the output brushes C, D can be reduced (and, therefore,
rotational e.m.f. reduced) by making a cut in the middle of the pole face as shown m Fig. 4.89
(c) Interpoles are normally required along the output brushes and even after this cut, Fig. 4.89
IC) there is no space to fit the interpoles. Thus, in order to provide space for interpoles, one pole
is split into two separate parts, giving the appearance of 4 poles for a two-pole cross-field
machine as shown in Fig. 4.89 id). For deciding the polarity of interpoles near the brushes C
and1 , attentn^Thou,d be given to d-axis mmf produced by armature.
indicated by inner crosses and dots on the armature, produce N pole near D and S pote near
C. In order to make the flux density zero under the brush at A the mterpole near brush must
have N polarity. Similarly, the interpole near brush C must have S polarity
T h e tw o * p o le f i e l d w i n d i n g w i t h 4 - p o la r p r o je c tio n s is s o m e tim e s r e fe r r e d to a s a o n e - c y c le
me t w o poie ueiu wii 5 f. a r m a t u r e o n e c y c le o f e .m .f. is g e n e r a t e d .
metadyne, since with one revolution of the arma , y

(c)
o fm .ta d y n . g .n e r ...r from ordinary d.c. machine.
Fig. 4.89. Development

"Scanned by C a m S c a n n e r
5U6 Klpiitlml Miuliliu<rv lAri. 4.|4

In Fig, *1.89 (d), the rotor Ini* ono cominutntor Fi«ld winding
mid ono nrmnturo winding (hmigiuul for two polos. F
Tho Hold or control winding F 1m wound on two*
polar prqjodionn, belonging to ono polo. Tho main
flux fcv loaves tho polar projection* 1 and 2 and
enters tho polar projection* 4 nnd 3. F lu *
K.m.f. Kquntlona. Tho Hold winding flux fc in <t>a
directed along tho horizontal nxi* or direct nxi*
and tho lino* of force enter tho armature radially
from pole segment* 1 nnd 2 and leave radially into
the polo segment* 3 and 4. The rotntion of tho nr-
mnturc in the main flux fc generate* an e.m.f.
Eq, between the rj-axis brusho* AB, nnd ita mag­
nitude ia given by
fc ZPn
E. Ka (o fc- F ig 4.89. Developm ent o f m otadyne generator
a from ordinary d.c. machine,
or = If, provided anturation ia ignored.
Hero Ktf= voltage gonerated across g-axis brushea/fiold ampero at speed u)m.
fc = total (/-axis field flux por pole, which is proportional to field (or control)
current fc.
Since brushes AB are short-circuited, a large armature current, shown by outer crosses and
dots in Fig. 4.89 (</), starts flowing and this establishes a stationary magnetic flux fc centred
along the brushes A, B. This flux fc is directed upward and it is seen that fc and fcare in the
same direction in polar projections 2 and 4, and are in the opposite direction in the polar projec­
tions 1 and 3. If saturation is ignored, fc remains unaltered.
Rotation of the armature in fc generates emf Ed across brushes C, D and it is given by
fc ZnP

Here Kd(f is the e.m.f. generated across the d-axis brushes/^-axis armature current at speed
a),,,. When load is connected across the output brushes C, D, the load current starts flowing in
the armature and it is indicated by the inner crosses and dots in Fig. 4.89 (d.). It must be clearly
understood that e.m.fs. and currents exist simultaneously in the same armature conductors
and the position of two pairs of brushes only separate them out. Note that the armature cur­
rents are additive between brushes B, C and A, D and they are subtractive between brushes
B ,D and A , C.
When the d-axis armature current l d flows, the inner crosses and dots establish a flux fc
due to d-axis armature mmf, opposite to the d-axis field flux fc. It is due to this reason that
metadyne is called a two-stage power amplifier, with negative current feedback with respect to
the main flux.
With the first stage of generation, ^-axis armature flux fc is established 90° away (and in
the direction o f rotation) from the field flux fc Second stage of generation establishes fc. 90°
away (and in the direction of rotation) from <f»9, i.e. 180° away, in the direction of rotation from
fc. In other words, the direct axis armature flux fc opposes the main field flux fc. If the direction
o f rotation is reversed, in the first stage fc is directed downward 90° away from fc and in the
D.C. Machines 507

secon dstage is shifted 90° away from tyq, i.e.


l8o° away from main field flux <Jy. This shows
that even with reversed direction of rotation of
the armature, the polarity o f the output
brushes C, D remains unaltered and this fact
is used in a Rosenberg generator for train
lighting purposes.
In order to obtain better visualisation of
the output voltage across brushes C, D, the
spatial phasor diagram of Fig. 4.90 is of consid­
erable help. In this figure, arrows between <Jy,
Ed etc- indicate the dependence
of former on the later. For example depends
on If, E qi on <tyand so on. Fig. 4.90. Space phasor diagram for a metadyne
generator.
As already explained, the voltage
across q-axis brushes A, B (in t h e first stage) is,
g e n e r a te d

E ql = K qflf, due to f acting alone.


The d-axis armature current Id establishes d-axis armature flux and rotation of arma­
ture in this flux generates an emf E q2 across brushes A, B . Its magnitude is given by
Eq2 ~ Eqd
where Kqd is the q-axis armature voltage/d-axis armature current at speed o)m.
Since the two emfs E ql and E q2 across brushes A, B are in opposite sense as shown in Fig.
4.90, the resultant q-axis generated voltage is,
E q = E ql - E q2 = (KqfIf - K qd Id) volts.

The q-axis armature current is


_ Eql ~ Eq2 _ Kqflf Kqd Id
A - rq ri
where rq = armature resistance across brushes A, B
The voltage equation for the second stage, from the q-axis armature circuit to d-axis arma­
ture circuit is
K(jg Kgd j
F _ K j _ Kdq
E>d ~ ™-dq *q risf
At no load, Id = 0. Therefore, the output voltage at no-load is

KdqKqfVf ...(4.72)
Edo -
rqrf
EdO _ Kdq Kqf _ ^
Voltage amplification, ~y~ -

...(4.73)
Let - dq Kqd be K h then Ed = AVf - K tId
q v 1 ic Hue t o the negativecurrent feedback.
In Eq. (4.73), the presence o f term K x Id « due to me g
When load is connected, the output voltage is
Vd ~ E d ~ ldrd

x j u u i ii i u u is y u u i i iv - z i y t u i i i i V / i
508 Electrical Machinery ______________

= AVf - I d (Kl + rd) -(4.74)


Here rd is the cf-axis armature resistance.
Also Vd = load voltage = VL = IjRl nnd hi - h-
Il^ l - A V f - Id {K\ + rd)
r r A y r

0T rd + R i + Ki
Example 4.79. A mctadyne generator (uncompensated cross-field generator) has the follow­
ing constants at speed (0,„.
Kfiom (= Kqf - q-axis armature voltage/field amp.
Kd(Om {=Kqd) = q-axis armature voltage /d-axis armature current.
/CQco„, (= Kdq) = d-axis armature voltage I q-axis armature current.
rd and rq are the d-axis and q-axis armature resistances respectively.
(a) Derive expressions for the output voltage and current and show that these are inde­
pendent of the direction o f rotation.
(b) Sketch the output current with speed, for a resistive load RL across the output terminals.
Solution. Upto Eq. (4.73), follow the treatment that has been adopted for its derivation.
Therefore,

Ed = AVf - K Jd = Z f f * V f— /„ .
q f ~q

For this example, Kq[ = KfWm, Kdq = Kqa>m and Kqd = Kdwnr Substitution of these value in
Ed gives

«, _ K/Kq •< T KdKq 0)2 7


~ — --------- I f ------- Id ...( 4 .7 5 )
Q rq
From Eq. (4.75), h ^ E

In an ordinary d.c. generator, if the direction of rotation is reversed, the polarity of th


output voltage ,s also reversed, because the speed term com appears only once in its e.m.f. ex
pressmn For a metadyne generator, the presence of in Eq. (4.75) indicates that the polarit
of the output voltage remains unaltered with the reversal of direction of rotation. Eq. (4.76) als

in Z s expression.GC ^ CUrrent “ by ‘ he 5peed reversa1' because « “ st


(b ) The load voltage Vd = Ed ~ Idrd = / dRL

■■ Jd(rd + RL) = Ed = ^ >Kf(^ i


rn f rq ^
or J, - KqpJf

KdK y ^ j ) •••(4•7,
<7 7~2
(j)„,
III

At - 0, the denominator of Erp (4.77) is cqunl to infinity „ nd. therefore, /„ is rcro. Will

the increase in te,„, the term r± Z L £ decrcascs ^ ^ ^ ^ ^

S c an n ed by C a m S c a n n e r
Art. 4.16] D.C. Machines 509

' q (rd + R L)
speeds, the term wouid tend to a negligiWe

value, showing thereby that output current Id is limited to

r Kf
KdKq *f ~K~d lf
The sketch showing the variation of Id with o>m is given
in Fig- 4.91. The cross-field-generator, which possesses
these characteristics is the Rosenberg generator and it is
employed extensively for lighting trains and other vehicles
s> Fig. 4.91. Variation o f output current
4.16.2. A m plid yne. An amplidyne is a metadyne gene with speed u>"’’ for a cross field generator
rator provided with a compensating winding, on the stat<* a" d
d-axis. The compensating winding is connected in series su>",l>'me a reslst,ve IMd
with the d-axis armature circuit. Fig. 4.92 gives the
schematic representation,^ an amplidyne. It is essential in
an amplidyne that the compensating winding flux <{>
fc
neutralizes the flux fc produced by the d-axis armature
current, at all possible load currents. It may be stated here
that amplidyne is one manufacturer’s trade name for a fully
compensated cross-field generator.
Fig. 4.90 is repeated here in Fig. 4.93, but with certain
additions.
The flux fc produced by the compensating winding, op­ Fig. 4.92. Schematic representation
o f an amplidyne.
poses the d-axis armature flux fc and, therefore, assists the
field flux fc as shown in Figs. 4.92 and 4.93. Rotation of the armature in the flux fc generates
an e.m.f. Eq3 = KqcId across the brushes A, B. Thus the rotational e.m.fs. appearing across
brushes A, B are Eql due to fc, Eq2 due to fc and Eq3 due to fc, Fig. 4.93. Here Kqc is the g-axis
armature voltage/compensating winding current equal to Id, at speed oom.
For an amplidyne fc and fc are equal, i.e.
or
Kqd I d - K p Id
or
Kqd = Kqc-

Eql fc

Afc «1 +’ Ea, ~
" "E„J
V

/ifc

E»r* fc

Ed 'fc fc '
---

'<>1
Fig. 4.93. Space phasor diagram for an ampHdyne.

am Scanner
[A r L 4.16
510 Electrical Machinery

If the compensating winding ATs are not equal to d-axis armature ATs then let
Effective compensating winding turns, N c _ N c •Id/ e _
C= Effective d-axis armature turns, N a N J d/Re\.
Compensating winding m.m.f./Rel. _ ^ c _ ^ 3
d-axis armature m.m.f./Rel. $d Etq2
or q 2)
K qc = CK,qd•
i.e.
Fig. 4.93 reveals that the sense of e.m.f. E q2 is opposite to that of e.m.fs. E qi and Eq3. So the
resultant q-axis generated e.m.f. is
Eq = E qi - E q2 + E q3
= KqfIf - KqdId + Kq(Jd = KqfI f - K qd (1 - C) Id

and
q rq rq rf rq

Also Ed = K dJq = vf- (1 - C ) Id = A V f - K , ld (1 - C)


d a<rq rq rf ' rq
where A is the overall voltage amplification factor and K j is a constant, as before.
The output voltage Vd ~ E d - Id (rd + rc) = VL
or Vd = VL = AVf - l d [Kx (1 - C) + rd + rc] ...(4.78)
Eq. (4.78) isapplicable both to amplidyne and metadyne.
If the compensation is 100%, i.e. C = 1, then the cross-field generator is known as amplidyne
and for this generator Eq. (4.78) becomes, -
Vd = VL = A V f - I d ( rd + rc) ...(4.79)
If C = 0, less than or greater than one, the cross-field generator is called a metadyne.
The nature of external characteristics of a cross-field generator, with various degrees of
compensation, are shown in Fig. 4.94. In Fig. 4.94 (a), the terminal voltage and output current
are assumed equal at full load conditions, in all the cases. In Fig. 4.94 (6), the terminal voltage
at no load is assumed same for all the cases. It can be seen from this figure that for low values
of compensation, the machine gives an approximately constant current characteristic, while for
degree of compensation of the order of unity, the machine gives approximately constant voltage

(°) (b)

Fig. 4.94. Cross-field generator characteristics for different compensations.

ScannecTby C am S can n e r
Art. 4 J 6 1 _
, . ,. ry,, . D.C. MachinesU.C.M
511
achine.
characteristics. Therefore, un-compensated m + a
source and a fully compensated metadyne U a ' 1S usually called a constant current
Comparison of voltage Eq. (4 79) f0r araPhdyne, a constant voltage source,
that the negative feedback effect K xId 0f the and Ecl- (4.74) for a metadyne, shows
reSultant d-axis flux is equal to the field flux T* °Urrent *s cancelled in amplidyne and the
control o f t h e f i e l d w i n d i n g a n d i s u n a f f e r l- p H 2 J W o r d s >t h e c f - a x i s f l u x i s u n d e r c o m p l e t e
. 1. , v e e r e d by t h e d - a x is a r m a t u r e cu rren t T
It must be understood that larger power ea' k ■ recurrent/d­
eal energy of the prime-mover. Power a[ e obtamed at the expense of the mechani-
amplidyne. Since t h e power required by the fi Id 6 ° ° f 20»°00 : 1 are possible with
fie ld winding can be fed directly from an p Ip t ^ ?°ntro1, winding is very small, amplidyne
p r o v id e d , then for the given d-axis output v o l t a ^ r ? ? a.mphfieJ- If an amPliator winding is

as explained before. Also Ed0,s proportional to co2 since both IT v . • .


don to speed It is also seen from EM expressL, d‘reCt pr0p0r'
polarity of the output voltage gets reversed. ' 6 reve of field current, the

r l ™ ^ iC1 - 0nS,fa 7 “ f‘elti generat0r iS fitted With th“ ™ winding.


k Power a m p l.fic a t.o n factor. Power absorbed by the field winding is P ,= V , / , P„wer
handled by the ,-a x .s armature circuit is P, = End
a similarl
IS i j — *d*d'

.*. Power amplification of the cross-field generators . _


_ Pd _ V dId _ E qIq Vd Id
Pf Vf lf V flf 'E„I.
•q +q
(First stage power amplification) x (Second stage power amplification)
= (PAF) j x (PAF)2

Now E„ = ^f Z n P
and Vf = I fTf
nP
(PAF) 1 -------- _9-------------- -o—
Vf !f a (If rf) I f rf

If Nf is the number of field winding turns, then

a 2P 2 nnP
(PAF) j - ^ ' 2n PNt IjNf K
f . fo j
XT tyf P N f
Now tyfPNfis the total field flux linkages and, therefore, — — is the total field inductance

L IZ
If and ~l is the field time constant xf. The term - £- represents the total ampere conductors in
rf i a

the 9-axis and, therefore Iq— = AT ' is the ATs per pole in the q-axis.
’ a ■2P 1
AT P i
■■■ (TAT),-* r j r / ^

S c an n ed by C a m S c a n n e r
[A rt 4.1(
512 Electrical Machinery

VdIj E^d co„


Similarly n
Jq *q'q v

.-. Overall PAF = (PAF)Xx (PAF)2


P2 2
n2 (0'*r/t'i ATf
' In view of the above expression, the power amplification factor of a cross-field generator can
be increased as follows : .
T ,, j o j. „„n>* Up raised bevond a certain limit because oi high
(i) Increase the speed com. But (0m can t be raisea Deyuxm
sparking at the brushes and mechanical stresses in- the armature conductors.
(ii) Decrease ATf. This can be achieved by reducing the air gap so that t e same flux iS
created with smaller ATf. In order to avoid saturation in this direction, lower flux densities are
1
used in the magnetic circuit of the d-axis field flux.
(iii) The time constants t^and xq should be increased for obtaining high PAF, but at the
same time T^-and xq should be minimum ^ n^ Fn ld/2
for quicker transient response. It has
been found that when both these
requirements are achieved.
Armature ohmic loss. For a 2-pole
machine, there are two parallel paths.
Fig. 4.95 shows the distribution of arma­
ture currents ld and Iq in the armature
circuit of a 2-pole cross-field generator. If
ra is the armature resistance, then the
resistance of each parallel path will be
2ra. If Ia is the effective armature cur­ Fig. 4.95. Simplified diagram showing the distribution
rent, then the armature ohmic loss, as of armature current, in the armature of a
seen from Fig. 4.95, is 2-pole cross-field generator.

Ilr. = ii (/„ - z,))2 X 2 r„ + [i (/„ + 1,)]2 x 2 r_

or /„ = t/HT7T

/. Armature ohmic loss = (I2 + l 2


q) x ra.
Applications of the amplidyne. An amplidyne is widely used for various purposes, par­
ticularly in feedback control systems where the regulated power requirement is in the range of
1 to 50 kW. It may sometimes be found economical to feed the field winding of an alternator
directly from the amplidyne. It can also serve as a generator in a Ward-Leonard system of speed
control provided the controlled d.c. motor rating is not high. In these feedback systems, the
controlled quantity is compared with a fixed quantity and the difference between the two (i.e.
the error signal) is made to actuate the controlled mechanism in such a manner as to tend to
eliminate the error.

In general, when the controlled power requirements are within the power range of cross­
field machines, amplidynes compete favourably with other types of controlled configurations.
Here only elementary closed loop schemes involving the amplidyne are described qualitatively-
(a) Voltage control schem es. W h e i i t h e a m p l i d y n e c i r c u i t i s d r a w n i n t h e following
s c h e m e s , its c o m p e n s a tin g w in d in g is n o t s h o w n fo r th e s a k e o f c o n v e n ie n c e .

S c an n ed by C a m S c a n n e r
16
Art. 4.16]
1
(j) D.C. gen erator v o l t a g e ^ ^ ~ 7 T - - _ _ D C. M,chines 513
amplidyne exciting the field of a d.c. generator "l g' 4'.96 are 8hm™ tw° schemes, with the
In Fig. 4.96 (a), the reference voltage V !T age is to be regulated,
required value. Feedback voltage V- is tapped ff th ^ ^ potentiometer p i and fixed at a
tional to the generator output voltage The f Hh ^ ^eGdba°k P°teiitiometer P 2 and ispropor-
referen ce voltage Vref) so that the voltapp voltage Vfb is made to oppose the fixed
in
y _ ,v applled t0 the amplidyne field winding is
Vamf-{Vref- V fb). 6
ih.

is
re

le

Fig. 4.96. Circuit diagram for d.c. generator voltage-regulation


(a) with one-field amplidyne and (b) with two-field amplidyne.

An increase of the generator output voltage increases proportionally. This has the effect
of reducing Vamf-(= Vref- V^), amplidyne field flux, amplidyne voltage output and generator
field current. Consequently the generator output voltage decreases until it is brought to its
initial value. If the generator output voltage decreases, its effect can be explained accordingly.
ir- If the amplidyne has two field windings, then one of them is made to serve as a reference field
of and the other as control field, in such a manner that their m.m.fs. oppose each other. In Fig. 4.96
;or
(b), m.m.fs, Fj and F 2 oppose each other
ed A lte rn a to r
and their resultant is responsible for A m p lid y n a field
he E xc ite r
the generation of voltage across the fie ld
.e.
output terminals of the amplidyne. Its
to
operation can be described on the same 4 rW
/
'%

lines as explained for Fig. 4.96 (a).


ss-
r

(») A ltern ator voltage-regula-


is. > +
tion. In the scheme illustrated in Fig.
iy- ■97, the d.c. exciter feeds the alter­
I V
nator field whereas its own field cur­ V/t
rent is obtained from the amplidyne
Fig 4.97. Circuit diagram for alternator voltage regulation.
°utput brushes. A linear rectifier con-

S c an n ed by C a m S c a n n e r
[Aft. 4.16
514 Electrical Machinery

verts alternating voltage to d.c. voltage Vn, which is compared with a fixed reference voltage
VrEf so that their difference V„f - V* = V „„,is applied to the amphdync field wmdmg.
Any increase or decrease in the alternator voltage, actuates the entire control scheme in the
fashion described for Fig. 4.96 (a).
(b) Speed control schemes. In Fig. 4.98, an amplidyne made to regulate the motor speed
of the Wprd-Leonard scheme is illustrated.
Motor To ch o -
A m p lid y n e field g e n e ra to r

Fig. 4.98. Circuit diagram for the motor speed regulation of Ward-Leonard scheme.

For motor speed regulation, a tachogenerator is mounted on the motor shaft whose speed
is to be regulated. The tachogenerator output voltage Vtm, proportional to the motor speed, is
compared with a fixed reference voltage Vref, Fig. 4.98. Their difference Vref-~ Vtm = Vamf, ap­
pears across the amplidyne field winding. If the motor speed decreases, Vtm decreases,
Vamf ~ Vref ~ Vtmincreases, the amplidyne output voltage increases and the field current of generator
G increases. This has the effect of increasing the voltage applied to the armature of motor M,
raising the speed thereby until initial speed is attained. .
The effect of increase in motor speed can be explained accordingly.
(c) Constant current schem e. Motor used with dredgers (or excavators), ship’s windlas­
ses etc. are likely to be stalled— in view of this it is advantageous to operate such drives from
constant current systems. One of the methods for obtaining constant current system, with the
help of an amplidyne, is illustrated in Fig. 4.99. In this figure, mmfs F 1 and F2 oppose each
other. The m.m.f. Fj is proportional to the voltage Vref and m.m.f. F2 is proportional to the
generator armature current Ia which is to be maintained constant. If the current Ia decreases,
F2 decreases and as a result of it, the net d-axis field flux of the amplidyne goes up. This has
the effect of increasing the generator field current and, therefore, the generator output voltage,
until current Ia attains its initial value.

A m p lid yn e

J
S c an n ed by C a m S c a n n e r
Art. 4.16]
____________________________________________

fieldmachdne an exPression for the short-circuit current amplification of a cross-

CUrrmt o f2 obtained with

Voltage amplification A = 5 0 ; rd =5 C
l;rc = 1a ; r
Obtain an expression for the output voltage for a field current o f 60 mA.
Solution. From Eq. (4.78), Vd = A V , - l d (K ,( 1 - C ) + ri + r j
Under short-circuit, Vd = 0

•• AV/ = ^ [ * i ( l - C ) + rd + rc]
or - Id [^1 (1 - C) + rd + rc]
{ d _ ________rfi
or
If Ky (1 - C) + rd + r~c
= short-circuit current amplification.
W h en C = 0, = 2 = - ^ ° ° > <5 0 >
’ If K 1 + rd + 0
K-y = 2500 - 5 = 2495
The output voltage is given by
Vd =AVf - I d [Ky (1 - C) + rd + rc)
/MS f 100 x 60^
- ld [2495 (1 - C) + 6]
= (50) 1000
\
or Vd = 300 - Id [2495 (1 - C) + 6]. i

For various values of compensation C, the external characteristics (output voltage versus
load current) can be obtained from the above equation.
Exam ple 4.81. A 3-kW, 300 V, 200 rad,/sec, amplidyne has the following constants:
rf = 50 ra = 5 Q, rc = 1 Q
Voltage constants, Kqf = 250 VIfield amp.

K-dq ~ 100 VI amp.


K qd = 80 VI amp.

Calculate the field current and power gain at rated output. Also obtain these values when
the compensation is zero. \
Solution. Voltage amplification
= £100112501^00
r ,r f (5) (50)

For an amplidyne, the voltage equation is


Vd =A V f - I d (rd + rc)

At rated voltage of 300 V, the rated current is

3000 = io A.
300
300 = 100 Vf - 10 (5 + 1)

S c an n ed by C a m S c a n n e r
1

(Arl- 4.16
516 Electrical Machinery

or V f- 3.6 V
3,6
and k = 50 = 72 mA.
Vd Id 3000
Power gain = ~Vf^ 9 2 “ H '58° '

For a metadyne with zero compensation,


Vd = A V f - I d [K1 + r d)

But = = (100) (801 = 1600


1 r* h
rd 5
300 = 100 Vf - 10 [1600 + 5]
or Vf = 163.5 V
163.5
and If= = 3.27 A.
50
v did 3000 watts
Power gain = = 5.61.
Vflf 535 watts

Example 4.82. A cross-field generator gave the following data for its open-circuit charac­
teristic :

Eda in V 0 40 80 120 160 180 200 220

Control or field ATs 0


11.5 23 36.5 59.5 79 no 160
The field winding has 800 turns and total d-axis
armature circuit resistance is 0.5 Q. The metadyne is
Load c u r r e n t
under compensated such that the mmfs o f d-axis ar­
mature circuit and compensating winding differ by 10 240
IL where IL is the load current. Plot the external char­
x 1
acteristic for a field winding current o f 200 mA. 200 X
X
A ■iagnetisati on
Solution. For a field current of 200 mA, the field
winding ATs are N 1/
160
200 x 800 Edo / \ 1
= 160.
1000 120 / \
\
The mmfs of d-axis armature circuit and compen­ /
Ou t p u vo t ag A
sating winding, differ by 10 IL and these ampere- 80 /
turns are opposing the field winding ATs, because the locid c u r r ent

machine is under-compensated. \
40 /
Net ATs along d-axis, which are responsible for^ 1 \
the generation of emf, for a field current of 200 mA are
T 1
= (160 - 10/L)
— *-F 1 e ld (A ts )
Maximum load current can be Fig. 4.100. Curves pertaining to Example 4.79.

= 16A

and the output voltage at this current is zero.

S c a n n e d by C am ^carT n er
D.C. Machines 517

For obtaining the magnitude of output voltage for other load currents, the calculations are
done in tabular form given below :

0 2 4 6 8 10 12 14 16
l . / t i» A
2. Net d-axis ATs = (160 - 10/L) 160 140 120 100 80 60 40 20 0

3. Open-circuit em f in V for (2), from Fig. 4.100 220 213 204.7 194 180.5 161.4 128 70 0

4. d-axis resistance drop 0 1 2 3 4 5 6 7 8

5, Output voltage = (3) - (4) 220 212 202.7 191 176.5 156.4 122 63 -8

The magnetization curve and the external characteristics for a field winding current of 200
mA are shown in Fig. 4.100.
Example 4.83. For this example, the metadyne saturation curve given in Example 4.79, is
assumed to be a straight line. Under this assumption, the metadyne gives an output voltage of
206 V, for a load current o f 8 A and a field current o f200 mA. For an open-circuit voltage o f 280
V, find the field winding current.
Solution. With saturation ignored, the output voltage Vd is given by
Vd = (800 If - 10 IL) K - ILr
where K = slope of the straight line magnetization curve in volts/AT.
For Vd = 206 V, IL = 8 A and If= 200 mA, the output voltage equation is
800 x 200
206 = - 8 0 K - 8 0 (0.5)
1000

or K = ^ volts/AT. '
O
On open-circuit, Ed0 = [d-axis ATsl (K)
21
or 280 = (800 7 , - 0 )
8
or Field current l t = 133.3 mA. V
Example 4.84 .Fig. 4.96 (a) shows a circuit for dc generator voltage-regulation. The various
constants for this scheme are given below :
Amplidyne voltage amplification 100
DC generator output voltage 200 V per field ampere
Field winding resistance o f G 125 Cl
(a) Feedback potentiometer P2 tapped to give -0 .1 Vt and P^is set to give 50 V.
Find the output voltage o f generator G.
(b) Now Vfh is reduced to zero ; find the value o f V n f to obtain the generator output voltage
found in part (a).
Solution, (a) Voltage applied to the amplidyne field winding,
W ^ -V ^ f S O -O .lV ,)

Amplidyne output voltage = (50 - 0 . 1 Vt) 100 volts.


( 5 0 - 0 .1 Vi) 100
Generator field current = 5

(50 - 0.1 Vf) 100 nnn - y


Generator output voltage = ^25 *

Its solution gives Vt - 470.6 volts.

S c an n ed by C a m S c a n n e r
(Art. 4.H
518 K lc c tr lw il M m ld n o r y —

(/>) W l i o n V/,, i s r e d u c e d to zero, tho generator o u t p u t v o l t u g c i s

YteL15? x 200 = 470.6


125
. Vri/= 2.94 volts.
’* rnl device. The gain factors of the
Example 4.85. Fig. 4.101 illustrates a vo uo/^s per field, ampere respectively.
amplifier and generator arc 1.5 amperes vo mp much should he the reference voltage
If the output voltage were to be 250 volts on no-load, how mucti snou / ge
En> with a feedback potentiometer setting at 0.2 .
, Feedback
potentiom eter
*=> t
A m plifier o

Pip. 4.101. C irc u it d ia g ra m o f E x a m p le 4.82.

What happens to the level ofER with feedback setting at zero, to give tlw same voltage at the
(.Pbi. Univ. Nov. 1973)
generator terminals ?
Solution. Generator output voltage = 250 V.
For generated voltage of 80 V, the field current = 1 Amp.
250
For the generator voltage of 250 V, its field current is given by — A.

Now for current output of 1.5 A, the amplifier input voltage - 1 volt.
25
.-. For current output of -r- A, the amplifier input voltage

25 2 25 _ _Q
-gT X 3 = 12 = 2 08 VOltS-
Feedback voltage, = 0.2 x 250 = 50 V.
•\ Reference voltage ER = (Feedback voltage) + (Amplifier input voltage)
= 50 + 2.08 = 52.08 V.
If feedbnck voltage V/lt is zero then
En = 0 + 2.08 = 2.08 V.
Example 4.86. A 4 k\V, 250 V d.c. generator has armature and field resistances of 0.25 0
and 100L1 respectively. When this generator is used as shown in Fig. 4.101, the full load voltage-
regulation is improved 20 times. Calculate the amplifier gain required. Assume the generator
gain factor to be 120 volts per field ampere and feedback potentiometer setting at (a) 0.1 and (b)
unity.
Solution. If Vt is the generator terminal voltage, then the voltage across the amplify
input terminals is (EK - k Vt), where k is the feedback potentiometer setting.

.*. The generator field current = ~ ^ nmp.

where Ap is the amplifier gain factor in volts per volt input and ry is the generator field resis­
tance.

Scanned by C a m S c a n n e r
4 .1 7 ) _________________________________ D.C. M achines 519
A rt

The generator no-load emf is -— £—£ volts, where A„ is the generator gain factor.
ry- «

For a load current o f /L amperes, the generator terminal voltage is

y — -h ra

or y ...(4.80)
rf + k A frAe
The reference voltage ER remains constant and therefore ERAfrAg can’t change. In view of
this, for small changes in Vt and lL, the term ERAfrAg} becomes zero and, therefore, Eq. (4.80)
is reduced to
. Tr 0 - A l i r a rf

Full load generator-regulation for Fig. 4.101 is


hra 1 1 6 x 0 .2 5 ____ I____ 4 _
• Vt 20 250 X 20 “ 5000 pU'

Decrease in terminal voltage of the generator, from no-load to full load is

A' ' , = ~ 5 T O 0 * 250 = - 5 Volt'


For Eq. (4.81), ML = 16 A, k = 0.1, Ag ^ 120, rf = 1 0 0 Q.

. 1 0 - 16 x 0.25 x 100
5 100 + 0.1 x A frx 120
or Ap. = 158.33
For k = 1,
Af, = 15.83.
4.17. Permanent Magnet DC (PMDC) Motors
In conventional dc (CDC) motors, stationary Field winding is excited by direct current to
produce the magnetic flux needed in the machine. In PMDC motors, the stationary electromag­
nets and their associated Field windings are replaced by permanent magnets. In other words,
the Field flux required in the air gap of PMDC motor is produced by a set of permanent magnets
Fixed to the- stator. The rotor of this motor is similar to the rotor of CDC motor, i.e. rotor of
PMDC motor consists of armature core, armature windings and commutator. Stationary carb­
on brushes are kept pressed on to the commutator surface as in a CDC motor.

PMDC motors are used extensively in automobiles for windshield wipers and washers, for
blowers used in heaters and air-conditioners, to raise and lower windows, in slot cars and
electric tooth brushes, in personal computer disc drives etc. As millions of automobiles are
manufactured each year, PMDC motors are also produced in millions. Permanent-magnet
motors have been developed up to about 150 kW for use in industry.

The major advantage of PMDC motors is that they require no Field current. So the energy
Squired in producing the Field flux is saved. As space for winding is not needed, PMDC motors
are smaller in size and may even cost less than the corresponding rated CDC motors.
The limitation of PMDC motors is that excessive current in the armature winding may
emagnetize the permanent magnets. In addition, the flux density produced in the air gap by

canned by C a m S c a n n e r
520 Electrical Machinery [Art. 4il?

permanent magnets is limited. This drawback is, however, overcome o


some extent by the development of new magnetic m ateria s 1 e
somarium cobalt and neodymium-iron-boron.
As flux per pole in PMDC motors cannot be controlled, these are not
used as dc generators. Its speed and torque are controlled throug e
adjustment of armature voltage applied to the motor terminals.
The circuit model, or equivalent circuit, of a PMDC motor is shown
in Fig. 4.102. As field flux in PMDC motor is provided by permanent
magnets, the fieldwinding is not shown in the circuit modelof this
motor. But in the circuit model of a CDC motor, the fieldwindingmust tig. 4.102. Circuit model
always be indicated as in Fig. 4.11. 0 a MDC motor-
In a CDC motor, the generated, or counter, emf is given by
Ea = —(3.44)
and electromagnetic torque is given by
Te = Kat f a ...(4.6)
In a PMDC motor, flux <j>is constant and as a result, above equations can be reduced to

Ea = Km<om ...(4.82)
Te = KmIa ...(4.83)
where Km = K a tyis known as speed-voltage con stan t or to rq u e co n sta n t. Its value depends
upon the number of field poles, armature conductors etc.
Equivalent circuit of Fig. 4.102 gives
Vt = Ea + Iara = Kmcom + Iara ...(4.84)

“ m= ...(4.85)
Kn
Equations governing the performance of PMDC motor are basically the same as for dc
shunt motor with constant field. Consequently, the speed and torque of PMDC motor can be
adjusted by armature-terminal voltage control, armature rheostat control and chopper control.
E xam ple 4.87. A PMDC motor has armature resistance o f 1 Ct. When fed from 48 V dc
source, it runs at a speed o f 2400 rpm while taking 0.8 A. Determine (a) the no-load rotational
losses o f the motor (b) the motor output when running at a speed o f 1600 rpm and with source
voltage o f 40 V dc and (c) its stall torque when the source voltage is 20 V dc.
Solu tion, (a) From Equation (4.84), the generated emf Ea is given by

Ea = 48 - 0.8 x 1 = 47.2 V

At no load, all the electromagnetic power developed is used to supply the no-load rotational
losses.
.*. No-load rotational losses

= EaIa = 47.8 x 0.8 = 37.76 W


(b) Ea = E m(Dm

Speed-voltage constant, K m = |J'x 2 4 q q = V-s/rad

2w x 1600
For a speed of 1600 rpm, rad/s
60

Scanned by C a m S c a n n e r
Art- 4' I8I________________________________________________________________________________ D.C. M achines 521

Generated emf, E„ = <o„ = 0.188 x ^U <80 = 3 1 5 V


Km
3
V —F
New armature current, I = — -----£ = 40 ~ 31.5 _
ra 1.0 ~ 0,t) A
E le c t r o m a g n e t ic p o w e r d e v e lo p e d

= E J a = 31.5 x 8.5 = 267.75 W


.-. Output, or shaft power = E J , - no-load rotational losses

= 267.75 - 37.76 = 230.0 W


(c) When motor stalls, Ea = 0 and V, = Iara
on
Stall current, 4 =— = 20 A |,

Stall torque, Te = KmIa= 0.188 x 20 = 3.76 Nm, j


4.18. D.C. M achine Applications
D.C. machine applications involve reconsideration of their operating characteristics,
together with their economic and technical evaluation as compared with other competing ener- ,
gy-conversion devices. The outstanding advantage of d.c. machines is that they offer easily
controllable characteristics. Their main disadvantage is high initial investment. In spite of this,
d.c. machines still hold a strong competitive position for industrial applications because of their
attractive features. In this article, d.c. machine applications are considered briefly.
4.18.1. G enerator applications. The advent of various types of controlled rectifiers, has
declined the importance of d.c. generators. Wherever electric energy in dc form is required, it
has been found much cheaper to convert ac to dc by means of transformer-rectifier equipment
rather than with the use of ac to dc motor-generator set. The disadvantages of transformer-
rectifier equipment are poor power factor, harmonic generation, difficulty in obtaining rever­
sing and regenerative braking. Its advantages, namely less cost, less floor-space, less noise
and minimum maintenance ; more than outweigh its disadvantages in this modern age. It does
not mean that d.c. generators are not manufactured in the present times. In some applications,
dc generators are indispensable and some of these applications are dynamometers (for measur­
ing torque etc.), dc welding generators, control-type dc generators (cross-field generators) for
closed loop systems, permanent magnet dc generators {i.e. tachogenerators) etc. From among
the dc generators, the commonly used generators are separately-excited generators for wide
output-voltage control and cumulatively compounded self excited generators for maintaining
constant terminal voltage. Now a days, the principal applications of separately-excited dc gene­
rators are (i) to serve as an excitation source for large alternators in power-generating stations,
(ii) to serve as control generator in Ward-Leonard system of speed control and (Hi) to serve as
auxiliary and emergency power supplies.
The power for electroplating can be obtained from ac supply through rectifiers or from dc
generators. When electroplating is required in expensive equipment like aircrafts, bombers
etc., an uninterrrupted and constant dc voltage is essential. This supply, in practice, is usually
taken from several dc generators running in parallel. It is because of the fact that ac supply
through rectifiers used for electroplating may suffer an inadvertent voltage dip, thereby result­
ing in defective electroplating. As a consequence, expensive equipment under treatment would
have to be discarded leading to a huge loss. Therefore, it is essential to employ dc generators in
Parallel for the electroplating of expensive equipment as listed above.
4-18.2. M otor applications. D.C. motors possess excellent torque speed characteristics
offer a wide range of speed control. Though efforts are being made to obtain wide range

/W l I
Icanned by C a m S c a n n e r
"P

[A rt. 4.18
522 Electrical Machinery

s p e e d c o n t r o l w i t h a .c . m o t o r s , y e t t h e v e r s a t i l i t y a n d f l e x i b i l i t y o f d . c . m o t o r s c a n t b e m a t c h e d
by a . c . m o t o r s . In v i e w o f t h i s , t h e d e m a n d f o r d .c . m o t o r s w o u l d c o n t i n u e u n d i m i n i s h e d e v en
i n f u t u r e . A b r i e f d i s c u s s i o n r e g a r d i n g t h e d .c . m o t o r a p p l i c a t i o n s i s g i v e n b e l o w .
Shunt motors, (i) For a given field current in a shunt motor, the speed drop from no-load
to full load is invariably less than 6 to 8%. In view of this, the shunt motor is termed a constant
speed motor. Therefore, for constant speed drives in industry, d.c shunt motors can be
employed. But this motor can’t compete with constant-speed squirre cage in uc ion motor,
because the latter is more cheaper, rugged and requires less maintenance.
(ii) When constant speed service at low speeds is required, the comparison is usually be­
tween synchronous motors and dc shunt motors. It is because the construction of high perfor­
mance polyphase induction motor with large number of poles is difficult. However, for
adjustable-speed service at low operating speed, dc shunt motor is a preferred choice.
(iii) When the driven load requires a wide range of speed control (both below base speed and
above base speed), a d.c. shunt motor is employed, e.g. in lathes etc.
(iu) In a shunt motor, if field winding is disconnected from armature and connected to an
external voltage source, it becomes a separately-excited, motor. This motor offers independent
armature control and field control. Since separately-excited dc motors are easily adaptable to
wide range speed and torque control, in high-power applications these are used extensively in
steel and aluminium rolling mills and in Ward-Leonard systems of speed control. In low-power
applications, separately excited dc motor finds wide use as a control motor.
A control motor is a low-power rating (less than a few hundred watts) motor, with field
excitation held fixed and with its torque Te proportional to armature current Ia.
Series Motors. The outstanding feature of series motor is the automatic decrease in speed
as soon as increased load torque is required. The decrease in speed with increase in load torque
or vice-versa has only a marginal effect on the power taken by the series motor.
(i) Since a series motor can withstand severe starting duties and can furnish high starting
torques, it is best suited for driving hoists, trains, excavators, cranes etc. Wound-rotor induc­
tion motors compete favourably with series motors, but the choiceds governed by the economics
However, for traction purposes, series motor is the only choice. Therefore, series motors are
widely used in all types of electric vehicles, electric trains, streetcars, battery-powered portable
tools, automotive starter motors etc.

(u) S e r i e s m o t o r s c a n b e u s e d t o d r i v e p e r m a n e n t l y c o n n e c t e d l o a d s , s u c h a s f a n l o a d , b e ­
c a u s e its to r q u e r e q u ir e m e n t in c r e a s e s w it h th e s q u a r e o f t h e s p e e d .

• * (Ua1) In,order *° avoid P°lluti°n in big cities, now battery-driven automobiles are being

s s 'i s i 'K - s “* !1

for loads, requiring heavy starting torque which S e likelv l ° f C? mp° Ufnd mot°Trs ar,eUSj
series motor, no-load speed becomes dangerouslv hmV, h \ • reduced to zero. In ordinary
speed is limited by the weak shunt field. U in a comP°und motor* the no-loa

shunt motor. Weak series fidd^auses m o ^ r o o !!^ 5 ^ ? iaracteristic approaching that of a


ordinary shunt motor. Such characteristic than with an
where load fluctuates between wide limits in t e r m it w i m I * * characteristics> are
such compound motors serve as load equalizers Dn ■ y '1 7 W1 ^S attached to the sha
energy but during peak loads, the mote^rspeed L n s hght *oads> store kinetI
its stored energy to the load. This has the effect of r e d u c in X ^ o w e ^ e m a n d fr T m thesupplf-

Scanned by C a m S c a n n e r
Prob^ 4 j ___________________. ___________________________________ D.C. Machines 523

The type of drives where such compound motors can be employed are rolling mill drive, punch­
ing press, planning or milling machine etc.
(iji) When the supply voltage across the motor terminals is likely to vary considerably, such
as in traction motors, compound motors are preferred. The series field helps in reducing the
fluctuations o e armature current drawn from the supply, because of its inductance and
because of senes-field mmf effect on the flux and therefore on the counter e.m.f.
(iv) A differential compound motor is almost never used. In this motor, shunt and series
fields oppose eac o eran it is possible that at some state of operation, there may be zero flux

in the air gap. When this occurs, motor speed (n °= ^ becomes dangerously high and armature

current increases to a very high value. This shows a differential compound motor is associated
with increased armature current at high operating speed ; this motor may also draw increased
armature curren unng its starting. This increased armature current during starting or
during high-speed operation becomes dangerously so high that it may damage the commutator
and armature windings. So, a differential compound motor is rarely used in practice.
I n g e n e r a l , m a j o r c a u s e o f u s i n g dc m a c h i n e s i n e l e c t r o m e c h a n i c a l s y s t e m s i s t h a t t h e i r
d y n a m ic c h a r a c t e r i s t i c s c a n b e c o n t r o lle d e a s i l y t h r o u g h t h e u s e o f v a r i a b l e v o l t a g e a c r o s s a r ­
m a tu re a n d / o r f ie ld w in d in g s . R e c e n t a d v a n c e s in p o w e r e le c tr o n ic s h a v e , h o w e v e r , m a d e a v a i l ­
a b le v a n a b l e - v o l t a g e - v a r i a b l e f r e q u e n c y (VW F) c o n v e r t e r s o f s u f f i c i e n t p o w e r l e v e l . T h e d r i v e s
c o n s is tin g o F c o n t r o lle r s a n d a c m a c h in e s a r e n o w b e in g c o n s id e r e d fo r s u c h a p p lic a t io n s
w h ic h w e r e O n c e d o m i n a t e d b y d c m a c h i n e s .

In the end, it must be emphasized that choice of a dc motor for adjustable speed drives
should not be based merely on its merits and demerits. In fact, it is necessary to make specific,
analytic, economic and technical comparison of all possible alternatives. Operating charac­
teristics of motor and load must be compatible with each other. At the same time, comparative
studies amongst various choices must be based on the combination of motor and its associated
control equipment. As the d.c. machines are easily adaptable to control strategies, d.c.
machines offer versatile energy conversion devices and hence their demand continues un­
dwindled.

Permanent-Magtiet DC M otors. As stated before, these motors are used in automobiles


as starter motors and for windshield wipers and washers, for blowers used in heaters and air-
conditioners, for raising and lowering windows, for electric tooth brushes etc. Literally, PMDC
motors are manufactured in millions each year to meet the demand of millions of automobiles
produced each year.
At last, it may be said about dc motor applications that the use of power-semiconductor
devices in electric drive systems has strengthened the competitive position of dc motors where
Precise control of speed and/or torque is required.

PROBLEM S

4.1. (a) Draw neat diagram o f a 4-pole dc machine. Label all its parts and mention the m aterial used for
each part.

For M i f 10 kW ’ 250 V ’ 6 ' P o I e lap-connected dc -enerator runs at 1200 rpm. Armature has 500 conductors,
load arm ature-ohm ic loss o f 200 W, find the useful flux per pole. Take 2 V as the brush drop at full
[Ans. 0.0257 Wb|
convert’ ^ parts o f a dc machine are laminated i f it is to be operated through a power-electronics
e cr ? Name these parts and give their constructional details.
Calcic A 4 poIe ,aP-connected dc generator has no-load generated e.m.f. o f 500 V when driven at 1200 rpm.
mate the flux per pole i f the armature has 120 slots with 6 conductors per slot.

icanned by C a m S c a n n e r
524 Electrical Machinery _______________________________________________________ [^ o b ^

In case each conductor has a resistance o f 0.01 ii, find the resistan ce o f the arm atu re winding.
[A ns. (6) 0.03472 Wb, 0.45Q,

4.3. (a) D escribe the constructional details o f com m u tator o f a dc m achine.


(6) A 6-pole, 1500 rpm, w ave-w ound dc generator has 47 slots w ith 4 coil-sid es per slot, each coil having
3 turns. Flux per pole is 0.02 Wb. The brushes are sym m etrically displaced from the “ ' a* is so ^hat angle between
the two brushes is 40° m echanical. Find the generated e.m .f. at rated speed at no load. (Ans. (6) 732.64 V)

4.4. (a) W hat are the tw o functions o f a com m utator in dc m achines ?


(fc) E xplain how the com m utator keeps the arm ature m m f station ary in sp ace, a lon g the interpolar axis,
even though the arm ature rotates.
4.5. (a) In a dc m achine, the electrom agnetic torque is Te = Kat f a and the arm atu re generated e.m.f. „
Eu - Knfyiam. Show that the arm ature constant K„, appearing in both Te and Ea expression s, are dimensionally
equivalent.
(fc) In a dc m achine, electrom agnetic torque is Te = K(l Ia w here Ka has the units o f N m /A . The generated
e.m.f. is Ea = Kaiom w here the units o f Ka are v olt-s/ra d . Show that Ka app earing in Te and E(l expressions are
dim ensionally equivalent.
(c) A dc shunt m achine develops an open circuit e.m.f. o f 250 V at 1500 rpm. Find its developed torque for
an arm ature current o f 20 A. [Ans. (c) 31.831 Nm|
4.6. (a) The rms value o f generated e.m.f.in a distributed w inding is given by VsT n kw ■fr N <> where
^u <fr' N ^ 4 0 are respectively the winding factor,rotational frequency, num ber o f distributed turns and total
flux per pole. From this, derive an expression for the e.m.f. generated in the arm ature o f a dc machine.
(fc) A 6-pole, 12 kW , 240 V dc m achine is w ave-connected. I f this m achine is now lap-connected, all other
things rem aining the sam e, calculate its voltage, current and pow er ratings. [Ans. (fc) 80 V, 150 A, 12 kW]
4.7. (a) E.m.f. generated in one N-tum coil, rotating in a constant field flux $, at any tim e t is given by
e( = N co, <(>sin t
where o)r = armature speed o f dc machine in rad/sec.
Use this expression o f ec to derive the following e.m.f. expression for a dc generator :

E .^ v o lu

(fc) A separately-excited dc generator, operating with fixed excitation, delivers 450 kW to a dc bus at
600 V. Estimate the percentage change in generator speed required so that 180 kW is delivered to dc bus.
Resistance between bus-bar term inals and the arm ature term inals is 0.015 ii and arm ature-circuit resistance
15 0 015 a |Ans. (fc) - 2.1687%l
4.8. (a) By using Blv concept, derive the following e.m.f. expression for a dc m achine :

Ea = ^ - volts
a

(6) A 10 kW , 6-pole dc generator develops an e .m .f o f 200 V at 1500 rpm. T he arm ature has a lap-connected
n£ g ^ ,enSAty, 8 P° le Pitch is 0 9 Tesla ^ len^ h and diam eter o f the armature are
0.25 m and 0 2 m respectively. Calculate («) the flux per pole (ii) the total num ber o f active conductors in the
arm ature and (w ) the torque developed by the machine when the arm ature supplies a current o f 50 A
[GATE, 19911
lAns. (fc) (i) 0.02356 Wb (it) 340 (iii) 63.745 Nml
4.9. (a) Derive an expression for the electrom agnetic torque developed in a dc m achine by u sin g BIl concept
(fc) A 2-pole lap-wound dc shunt m otor with 360 conductors operates at a constant flux level o f 5 0 mWb.
The m otor arm ature has a resistance o f 0.12 ii and is designed to operate at 240 V taking a current of 60 A
at full load. ’ 6
(i) D eterm ine the value o f external resistance to be inserted in the arm ature circuit so that armature
current does not exceed twice its full-load value at starting.
(ii) The external resistance is com pletely cut out w hen the m otor reaches its final speed with the armature
current at the full-load value. Calculate the m otor speed under these conditions. [GATE, 19921
IAna. (i) 1.88 O (ii) 776 rp»l

Scanned by C a m S c a n n e r
D.C. Machines 525
4 10 . (a) D e v e lo p t h e c i r c u i t m o d e l o f a d c m a c h in e vr l •
llw a rm a tu r e c o n d u c to r s o f a d c m o to r o p p o s e e a c h o t h e r ° Ut d 8 a r ly h ° W th e e m f ’ a n d c u r r e n t
in

U (ft) A d c s h u n t g e n e r a t o r g i v e s a n o p e n - c ir c u it v o lta ir e o f 9 d n v ^ , . , , , „
n20 V D e te r m in e t h e lo a d c u r r e n t in c a s e a r m a t u r e c ir c u it ? r ' ’ t e r m ,n a l v o l t a e e f a l ls to
resp ectively. N e g le c t th e efT ect o f a r m a t u r e r e a c t io n H e ld - w in d in g r e s i s t a n c e s a r e 0 .1 f l a n d 5 0 f l

4 . 1 1 . <«> P r o v e t h a t in a d c g e n e r a t o r , g e n e r a t e d <61 W 5 '6 A '


w hereas in a d c m o to r , g e n e r a t e d e .m .f. o p p o s e s th e flo w o f c u r r e n t ** UCt° r a r e in t h e s a m e d ,r e c t l 0 n -
(6 ) In a 6 -p o le d c m a c h in e , t o t a l a r m a t u r e a m n f.ro a m 3 c o n d u c to ^ •
the to tal to r q u e d e v e lo p e d in t h is m a c h in e . a u u c t o r s a r e 4 2 0 0 , flu x p e r p o le is 0 .0 3 W b . C a l c u l a t e
Ia
[H in t : ( b) — Z = 4200. U s e E q n . ( 4 . 5 )]
° lA n s . (6 ) 12 0 .3 2 N m ]
4 .12 . (a ) In a d c m a c h in e , t h e m a in fie ld flu x
slot portion has longer air gap. As a consequence fl„v rl„ armaUire m ainly through the teeth, because the
in the teeth. Extend this fact to explain the nhvci’roi S1 y/.m s >s m uch low er than its value
« A 2 3 0 V , 1 0 -p o le , l a p - c o n n e c t e d d ^ h l f ' . ^ h m 3 C h in e S
1200 A E a c h c o n d u c to r h a s a c t iv e le n g t h o f 30 IiUnS &t 120 r ? m w h lle t a k i n S a n a r m a t u r e c u r r e n t o f
stated in p a r t (a ) a b o v e , f lu x d e n s it y in th e s lo t is 0 nn d ia m e te r is 12 0 cm . B e c a u s e o f t h e fa c t s
average fo rce o n e a c h c o n d u c to r a t o t a l flu x P e r P o le o f 80 m W b . C a l c u l a t e th e
(i) i f th e c o n d u c to r is h o u s e d in th e s lo t a n d

iron surfa“ - IAns- w “ 216" ,w 25 47 *


(4) A 4 - p o l e c o m p o u n d g e n l Z T : T ' ^ * ' °f ' :
i n , 0.5 Q a n d 1 Q0 f l r e s p e c t iv e ly . T h is g e n e r a t o r d S v e r e 4 k w ’ i r ’ 1) ' ^ ^ s h u il t ' fie lli r e s i s t a n c e s o f

P" ^ iH h e g e n e r a t e d ^ 2° ° V ^ 1 V

(ffl th e ilu * p e r p o le i f t h e a r m a t u r e h a s 200 la p - c o n n e c te d c o n d u c to r s a n d is d r iv e n a t 7 5 0 rp m .

4 .14 . (a) E x p la in w h y t h e e m f e e n e r M H • ^ ^ ^ ^ ^ 235 V W> 93 64 m W b ‘ 94 m W b l


W F o u r t e r m in a ls o f a d c s h u n t m a c h “ ^ ° f a d ‘ ^ o r is c a lle d ‘b a c k e.m .f.-.
the field a n d a r m a t u r e t e r m in a ls ? ^ 318 a v a ila b le - b u t th e s e a r e u n m a r k e d . H o w w o u ld y o u id e n t if y

characteristic is g i v e n l ^ t ^ w h oseex te rn a l

‘ e n n in a l^ c m 9 T 2 r n ] S tan C e t e m i n a l s a r e m a t u r e te r m in a ls w h e r e a s h ig h - r e s is t a n c e t e r m in a ls a r e fie ld ‘

with r e s p e c A o e a c h S h e r r y ?° m a c h in e ’ fie ld w in d in & a n d its a r m a t u r e c ir c u it a r e a lw a y s d r a w n a t 9 0 °

W Howwin t u d M n lth be"w eWe n dingS ' T f 0" ‘ he P°'e °f 3 dCCOm'’° u" d ?
K s c u s s t L te rm c fm l Z s H , ° f 3 d c c o m p ° ™ d “ “ c h in e ?
4 lfi i in n . . m m utatlon and commutation period.
(b) cn, *S m e a n tJ )y a r m a t u r e r e a c tio n ?
(c) t h a t th e efT ect ° f a r m a t u r e m .m .f. on th e m a in fie ld , is e n t ir e ly c r o s s - m a m e t i* ’

d e m a 8 n e ti2 e ‘ h e -“ d E x p la i n

4 18 /^ 3 *n b r ie f ly t b e f ° u r b a d e ffe c ts o f a r m a t u r e r e a c tio n .
main field ? ) ll0 W S h ° u ld th e m a in -p o le tip s b e c o n s tr u c te d to m in im iz e th e e ffe c t o f a r m a t u r e r e a c t io n o n t h e

a r e S h if‘ e d id ‘ h e d ire C t’ ° n ° f r ° ta t io n <f° ™ “ r d s h ift ) , t h e n u x p e r p o le i s r e d u c e J in .


What would h ■
n a p p e n i f t h e b r u s h e s a r e g iv e n a b a c k w a r d s h ift in a g e n e r a t o r ?
526 Electrical Machinery________________ [Proh ^

4.19. (a) Explain how the effect o f arm ature m m f on the main field flux can be described with the he!p0f
magnetization curve o f a dc machine.
(6) How can you detect w hether the brushes in a dc m achine are placed correctly along the interpolar axi$
or not ?
(c) In dc m achines, explain why air-gap length at pole centre is short as com pared to air-gap lengths at
pole tips.
(d) W hat are the various types o f brushes and where these are em ployed?
4.20. (a ) Derive expressions for the back ampere turns per pole and cross-m agnetizing ATs per pole in case
brushes are given a lead o f 9° from GNA in case o f a dc generator.
( b ) A 6-pole, wave-wound dc generator has armature conductors 360, arm ature current 80 A, angle of lead
5° from GNA. Calculate (i) back and cross ampere turns per pole (ii) num ber o f series turns per pole to neutralise
the dem agnetization. Take leakage co-efficient = 1.2.
[ H in t : (6) series field mmf per pole = |Ans> {b) (f) 20Q mQ
back ATs per pole 1
4.21. (a) W hat is m eant by com m utation in dc m achines ? Differentiate betw een good commutation and
bad com mutation. Enum erate the m echanical and electrical conditions leading to poor commutation in dc
machines.
(6) Describe the process or com m utation in dc machines through the reversal o f current in a coil.
2 Ia/a
Hence show that e.m.f. induced in a coil undergoing com m utation is given b y ——— w here /„ is the armature
•* c
current and Tc is the comm utation period.

4.22. (a) I f coil resistance is neglected as com pared to the copper-carbon resistance, linear commutation is
obtained ; if coil resistance is not neglected, resistance com m utation is obtained. Explain.
(b) On what four factors, does the current in a com m utated coil depend ? Discuss.
4.23. (a) Give the concept o f reactance voltage in dc m achines.
Discuss how reactance voltage causes under com m utation in dc m achines.
(b) Discuss resistance com m utation qualitatively.
Explain how good com m utation is achieved by m inim ising the reactance voltage.
D escribe the role played by resistance com m utation in securing good com m utation in dc machines.
4.24. (a) Sketch the general waveform o f arm ature m m f and flux in a dc m achine. W here has the flux zero
value and m axim um value and w hy ?
(b) H ow is the voltage com m utation achieved in dc m achines ?
4.25. (a) W hat are the interpoles ?
W hy are interpoles designed to provide m m f more than the arm ature m m f in the com m utating zone ?
(6) W ith the dc m achine fitted with interpoles, draw the resultant flux den sity waveform and show
therefrom the im provem ent in the com m utation process o f both the gen erator and m otor.
(c) W hat should be the polarity o f interpoles with respect to the m ain poles in a dc m achine ?
4- k ° w flashover betw een positive and negative brush m ay occu r in dc m achines subjected
to rapidly ch an gin g loads. How is this trouble o f flashover overcom e by com pen satin g w indings ?
(6) A com pensated dc m achine h
pole-pitch is 0.7. In terpolar air-gap length and flux density are respectively 1 cm and 0 3 T For rated a r m a t u r e
current o f 100C A, calcu late the com pensating w inding conductors per pole and the n u m ber o f turns on each
,n terp0le' [Ans. (b) 26 conductors, 9 turnsl
4.27 . (a ) E xplain how an arc betw een adjacent com m utator segm ents m ay occu r in dc machines subjected
to h eavy overloads. D iscuss h ow the occu rrence o f such an arc can be avoided.
(b) Show ph ysical arran gem en t o f the connections o f arm ature circu it w ith the com pensating winding
h avin g four con du ctors in its pole-face slots. -
(c) A dc com pou n d m ach in e possesses both interpole and com p en sa tin g w indings. Draw its s c h e m a t i c
d iagram in d icatin g each part clearly. D iscuss the function o f each o f the w in d in gs sh ow n in the diagram'
4 .2 8 . (a) D raw -th e equ iva len t circu it o f a dc com pou n d m ach in e for both g en era tin g and motoring mode-'
for lon g-sh u n t as w ell as sh ort-sh u n t con n ections. W rite voltage equ ation s for th ese circuits defining a11 “
p aram eters used.

Scanned by C a m S c a n n e r
P ro b . 4 J ____
------ D.C. M achines 527
(,b) In a 220-V com pou n d generator • the arm**., • ,
0.25 f2 ,0 .15 and 50 Q respectively. The load rnnaieT r? ^ 8,01168 311(1 shunt w m din e s have resistances o f
e m.f. and arm ature current w hen the m achine 8 ° V , lamps, each rated at 60 W , 220 V. Find the total
connected for (i) long shunt and (ii) short shunt.
. OQ „ (An» - (5) (D 232.67 V, 31.673 A ( « ) 232.03 V. 31.755 A|
4.20. (a) Explain the m ethods o f im proving mm™, * *• . .
, 0, . , , ® com m utation in dc machines
(5) Sketch and com m en t on the resultant flux « ,
winding and interpoles. y waveform for a dc m achine fitted with com pen satin g

4.30. (a) Explain how the effect o f arm ature reartinn ; ^ , .


load m agnetization curves. dc m achines can be determ ined from no-load and

(b) W hat useful inform ation can be obtained from th« ♦ L . .


(c) W hy does the term inal v o l u * . fall m orc raDi. , ‘ ™ ‘ characte™ t' c ° f i c g en erators ? E xplain.
|y-cxcited dc generator ? pidly in a self-excited shunt generator than in a separate-

4.31. (o) W hy does the slope o f open-circuit rkamntn ■ .■ , .


of field current ? ractenstic o f a dc generator change after a certain value

(b) W hen a dc separately-excited or shunt • a . .


the voltage across th eir brushes is not zero ? Tn ra <°r ** r d at rated speed with zero field current, w hy
f r ) A 220 V 1 5 WW okq . se i is zero, what you would do with the dc m achine ?
current o f 8 A at rated ioad c m f l t f o ^ T f t h e ** m.0t° r ,h a® armature resistance o f 2.5 fi and it draw s a
the rated speed at rated load w hat w ilf be the n o I T ' ^ a™ frxed at the value o f
between no-load.and full-load operation. 8Peed o f the motor ? Assume losses rem ain constant

IH in t : (c) Take Ia = 0 at no load] l° ATE' 13951


a 19 (si \ wru 4 / i r [Ans. (c) 944.9 rpm 1
Explain } 6atUreS ° f 3 dC 8eriCS eenerat0r d* * W * h it from the other types o f dc generators ?

a voltage o f 100V. h T i S c S ^ a n d t h e ? V T *"!*.?1 ^ ^ deliverin& 5 kW at


and terminal voltage. ° ^ *nd the load adJusted 8 kW - r>"d the new field current

(H in t : (a , External characteristic - terminal voltage rises with increase in load - in others it falls etc.

(b)Eal = k N l Ifl,im dk = ^ V -A /r p m . New armature or field current = and new Ea2 = Vl2 + ™ ™ etc.)

is 2 v ’ )wv.Separate-y excited dc generator has armature circuit resistance o f 0.1 tl and a total dron at h r„< = w
thp a ’ runnm ^ at 1000 rpm, it delivers a current o f 100 A at 250 V to a ioad o f constant resict* n-
the generator speed drops to 700 rpm, with field current unaltered, find the current d J t Z T u !Toad

. IAns. (6 ) 69.77 A]
the fol’w 2 gener,a t0r 13 devel°Pm g rated terminal voltage at some speed. For this generator answer
lowing questions and give a brief explanation in support o f your answers : ’
la) If only the direction o f rotation is reversed, will the generator build-up?

generator bujh^up*?^° ^ reV6rSal ° f direction o f rotation- the residual magnetism is also reversed. W ill the

(c) If only the field winding connections o f the original machine are reversed, will the generator build up ?
Id) If only the residual m agnetism o f the original machine is reversed, will the generator build up ?
the ml b° th the direct>on o f rotation and field winding connections o f the original machine are reversed will
inacmne build up ?

g e n jf l y tlle brushes are m oved in the direction o f rotation o f the original machine, what will happen to the
ator operation ?

« imur/*,*’ no (b) no (c) no (d ) yes, brush polarity is reversed (e) yes, brush polarity is reversed (f) com m utation
ed and generator terminal voltage falls.]
4‘88, Explain the voltage build-up process in d.c. shunt-generators.

canned by C a m S c a n n e r
528 Electrical M achinery

(6) It is found that the voltage o f a d.c. shunt generator does not build up. E xplain the various
causes o f this failure. P°ssib|.

If, amp. 0 0.2 0.40 0.65 1.02 1.75 3.15

E„, volts 10 40 80 120 160 200 240

D eterm ine the critical field resistance at


(a) 800 r.p.m . and
(b) 900 r.p.m.
(c) If the field w inding resistance is 55 Q, find the range o f field rheostat to vary the voltage from 9nn
250 V, on open circuit at a speed o f 800 r.p.m. U^
(d) D eterm ine the terminal voltage and field current o f the generator for a total armature resistar t
0.6 fl, arm ature current o f 50 A and a speed o f 800 r.p.m. N eglect arm ature reaction. Ce
[Ans. (a) 200 fl (6) 225 fl (c) 59.29 fl to 9.52 fl (d) 220 V 4 A|
4.37. A d.c. generator with interpoles is running satisfactorily as a cum u latively compounded eenera**
The m achine is stopped and its residual m agnetism is reversed. After this, the connections aremadeaa n,
were before and the m achine is run in the same direction. they
(а) D n s the m achine build-up voltage ? Explain.
(б) I f answer to part (a) is yes, is the term inal voltage the sam e as before? Explain.
(c) Is the machine now cum ulatively or differentially com pounded ?
(d) Do the com poles have proper polarity for good com m utation ?
[Ans. (a) Yes. (6) Yes, but the polarity is reversed, (c) Cumulatively compounded generator, (d) Yes ]
4.38. (a) Explain w hy the m agnetization curve for a dc generator obtained w ith increasing values nffioU
current departs from that obtained with decreasing values o f field current c a s i n g values of field

no load f s x p W 156611 “ ° SatUrati° n “ Self-e x d ted shunt ^en erator- ^ a t w ou ld be the terminal voltage at

c h a r a c te r is tic ^ ^ b ta in e d X ^ th i^ g e n e r a to r ^ E x p la in ^ eXte™ al characterist>c- H ow can the level-compounded

the s e lf excited g e n e r a t o r ^ entire m agnetization curve, whereas


itt. . rj \ T , . an range o f the m agnetization curve. Explain.

“C 39 fa) D r “ — ^ “ dS5

’ am W °A ° h" aSS“ 'n il‘g ^


(6) r.p.m
at 1000 A s h.o :r t - s h u n t c u m u la t i v e ly c o m p o u n d e dgenerator
g e n e r a t o rhas
hawthe
t b ofollow
f ii in•g data
j • curve»
e its magnetization
for

If, amp. 0.5 1.0 1.8 3.0 4.8 14.0


6.8 9.0 12.0
Ea, volts 11 40 80 140 184 220 282
240 250 274
V. uuciua
are 20 se n e s turns per pole, 1000 shunt tu ana s e n e s field resistance is U.m
120 A is 451 A Ts per pole. per pole and arm ature reaction at rated armature

F or a sh u n t field resistance o f 45 f i (
(i) determ ine the no-load term inal voltage,
(ii) i f the speed at rated arm ature current is 95fl r « ™ i , . . ,■ the field
current the sam e as found in part (i). P m -. calcu late the term in al voltage taking th

[H in t. (b ) (ii) The equ ivalen t shunt field current Jf is

Ifeq x 1006 = 4.95 x 1000 + 115.05 x 20 - 451

Scanned by C a m S c a n n e r
prob. 4] D.C. Machines 529

V, = 228 - 120 x 0.05 - 115.05 x 0.04 = 217.4 V] [Ans. (a) 222 V, (b ) 217.4 V]
4.40. (a) E xplain the nature o f no-load, external and armature characteristics o f a dc shunt generator.
Why d°es t*'e extern a* characteristic o f this generator turn back as the generator is overloaded ?
(b) A belt driven 60 kW shunt-w ound generator running at 500 r.p.m. is supplying full load to a bus bar
at 200 V. At what speed w ill it run if the belt breaks and the machine continues to run taking 5 kW from the
bus bar ? The arm ature and field resistances are 0.1 fl and 100 fi respectively. Brush contact drop m ay be
taken as 2 V. N eglect arm ature reaction. [An*. (*,) 421.404 r.p.m.)

4.41. (a ) Draw the external characteristics o f various types o f dc generators in one figure on the assum ption
of same rated term inal voltage and the sam e rated load current. Discuss the nature o f these characteristics
and compare them.
(b) A 20 kW , 250 V dc series generator is running at 1000 rpm under full-load conditions. It is given that

field flux per pole $ « + j •r* = 0 01 n , ra = 0.015 D and armature de-magnetizing am pere turns per pole
= 6% o f the field am pere-turns per pole. Brush voltage drop is 2 V. Calculate the m otor terminal voltage in case
the motor draws 100 A at 1050 rpm.

[H in t : (6) Eal = 250 + 80 x 0.025 + 2, Net field flux = x 0.94 and $2 = — x 0.94.
loU OU + / 2

Also 1 ^ = 7 ^ x 0 923 etc.) [An*. (6 ) 284.45 VI


Eq2 1050
4.42. (a) D istinguish betw een external and internal characteristics o f a dc generator. Explain, with
appropriate diagram s, how the internal characteristics can be obtained from external characteristics o f the
following dc generators :
(i) separately-excited generator (ii) shunt generator (Hi) series generator.
(b) A 240 V, 36 kW , dc shunt generator has 500 field turns per pole. On no load, the generated voltage of
240 V is obtained w ith a field current o f 2 A. For m aintaining 240 V at full load and at the sam e speed, the
field current required is 3.2 A. Calculate the num ber o f series-field turns per pole required for level com pound­
ing [Ans. (6) 4 turns)

4.43. (a) D erive the speed-current characteristics o f dc shunt, series and cumulative compound motors.
Sketch these characteristics in one figure on the assumption o f (i) same speed at no load and (ii) rated speed
at rated current. C om m en t on the nature o f these characteristics.
(b) A 4-pole dc series m otor has w ave-connected winding with 600 conductors. Total resistance o f m otor is
0.8 fl. When fed from 250 V dc source, the m otor supplies a load o f 10 kW and takes 50 A with a flux per pole
of 3 mWb. For these operating conditions, calculate the developed torque and the shaft torque.
[An*. (b) 28.65 Nm, 27.284 Nm)

4.44. (a) D erive torqu e-cu rren t characteristics o f dc shunt, series and cumulative com pound motors. Sketch
these characteristics in one figure on the assum ption off*) same torque at no load and (»«) rated torque at rated
current.
From these torqu e-cu rren t characteristics ; find, giving reasons, the most suitable m otor for traction-type
loads.

Scanned by C a m S c a n n e r
530 Electrical Machinery

Induced e.m.f., volts 130 154 172 184 192


198
Field current, amp 25 30 35 40 45
50

At rated voltage and rated speed, the motor takes a full-load current o f 36 A. Find the internal startin*
torque, in case the starting current is limited to 60 A. Assum e the dem agnetizing effect o f armature reacti^
to be proportional to the square o f the current.
(H in t : Plot the magnetization curve at 600 rpm. The dem agnetizing effect for 36 A is 7.6 A according
( 60 ' 2
Ea = 230 - 36 x 0.25 = 221 volts. For 60 A, the dem agnetizing effect is x 7 .6 = 2 1 . 1 A. Therefore, the
36 net
v
d-axis field m m f is equivalent to a field current o f 60 - 21.1 = 38.9 A. Now see part (e) o f Example 4.33)
lAn*- 260.7 Na;
4.47. A dc shunt motor is connected to a 3-point starter. Explain w hat would happen if
(a) the starter handle is moved rapidly from OFF to the ON position,
(fe) the field circuit is open and an attempt is made to start the motor,
(c) the field circuit becomes open-circuited with the m otor running at no load,
(cf) the field circuit becomes open-circuited with the m otor running at no load, with the assumption th»;
the starter is not provided with the no-volt release and the spring,
(e) the starter handle is pulled back to stop the motor,
if) there is a sudden overload o f 100%,
(g) the field excitation is m inim um at the time o f starting.
[Ans. (a) M otor draws large current, resulting in heavy sparking at the brushes. The motor accelerate
rapidly and large starting current m ay overheat the motor.
( i ) M otor would not start and the arm ature would draw heavy current from the source.
(c) to (g) Read Art. 4.13)

4.48 A 220V, 1500 rpm dc m otor has full-load arm ature current o f 30 A. It is proposed to design a starter
which restricts the m axim um arm ature current during starting to 60 A. For design purposes, the minimus
r r z r i " 8 ' S V’ r V f o ? ? ? 30 A ' lhe SeriCS resistance in >he arm atu re circuit b e i n g s
l ,: V ‘ he °. A sfu m m g that the arm ature resistance o f the dc motor is 0.5 C
t h e f i r s t tw o 't e t i s S' n “ r e S ,s t a n c e u s o d ,n t h e 5 t a r t c r ° " d t h e a m o u n t o r r e s i s t a n c e c u t d u rin g eachd
P ' |I A S ., 1 9 Z

IA n s. 3.167 fi, 1.834 0. 0.9166 G

sta rte r* ' ^ E X P ki" ^ W° “ ' d haPPen * the dC m otor is d irecll>’ sw itch ed on to the supply without W

(6) Explain the function o f no-volt release in a three-point starter


(c) Explain the advantages o f four-point starter over the th ree-poin t starter
Id) W hat is the difference betw een a starter and a controller ?
« W hat would happen if the external resistance for starting the dc m otor is left in the nrmatutc circuit?

M T he irnZZJ ' r I thC WOrkinB ° f a th ree-P“ ” ‘ sta rter used for a dc shun. nwW
losses. T he norm al field current is ’ l o m p c r c ^ i n d t h ^ m a m f t ’ Th e fa rm o tu rc oh m ic '» s s c s are hnlf of the tc“_
for this m otor. m agnitude o f resista n ce for each step o f a 6 stud sts

T he m axim um arm ature current is lim ited to tw ice the l i


u u c u 10 t w ic e t h e f u ll lo a d a r m a t u r e c u r r e n t .
( H i n t : ib) Total losses = I — - i j x 7500 = 833 W

L in e cu rren t - - 37.9 A , „.l 36.9 A . r„ . e lc ,


(36.9)

4 5 11.
4.0 ia)P Ev xplain
n l e i r l l thef follow
II • in g term s : |A m ,‘ ^ ’ 0901 0 6915 0 4386 0 2782

IJasc speed, speed regulation, speed range constant j •


’ p o w e r d r i v e n n d c o n s t n n t t o r q u e d r iv e .

Scanned by C a m S c a n n e r
P ro b ^ fl _ _ _ _ _ _ _ _ _ _ _ _ _ _ _ _ _ _ D .C . M a c h in e s 531

(fc) Describe and com pare the various methods o f speed control o f dc m otors,
4.52. (a) For o dc motor, the armaturc-circuit-resistance method of speed control is called a constant torque
drive method, explain. r

(fc) A dc shunt m otor runs at 7o0 rpm from 250 V supply and is taking a full-load line current o f 60 amperes.
Its armature and field resistance are 0.4 n and 125 D respectively.
Assuming 2 V brush drop and negligible armature reaction effect, calculate
(i) the no-load speed for a no-load line current o f 6 amperes,
(ii) the resistance to be added in series with the armature circuit to reduce the full-load speed to 600 rpm,
(in) the percentage reduction in flux per pole needed, in order that the speed may be 900 rpm when the
armature current is 30 am peres, with no-added resistance in the armature circuit.
IAns. (fc) <i) 822 rpm ; (ii) 0.7752 D ; («'//') 12.5%|
4.53. A dc shunt motor takes an armature current o f 50 A at its rated voltage o f 240 V. Its arm ature-circuit
resistance is . . an ex ernal resistance o f 1 Q is inserted in series with the armature and the field fiux
remains unchanged, then calculate :
(a) percentage decrease or increase in speed for the same load torque,
(fc) percentage decrease or increase in speed for half of the load torque.
[Ans. (a) 21.739% decrease (fc) 8.7% decrease)
4.54. (a) For a d.c. motor, the field-flux-speed control method is called a constant power drive method.
Explain.
(fc) The e.m.f. developed in the armature o f a shunt generator at 1155 r.p.m. is 240 volts for a field current
of 4.5 amperes and 255 volts for a field current o f 5 amperes. The generator is now used as a motor on a 260
V supply and takes an arm ature current o f 75 amperes. Find the motor speed, when the field current is adjusted
to 4.8 A. Armature resistance (including brushes) is 0.12 D. (Ranchi Univ.)

(Hint. For a field current o f 4.8 A, the e.m.f. generated in the generator armature is 240 + x 0.3, i.e.
0.5
£ ol = 249 V. For a motor field current of 4.8 A, the e.m.f. generated in the armature is
Ea2 = 260 - 7 5 x 0.12 = 251 V
£ oI 1155 x 4.8
and £ u l ~ n 2 x4.8etc.| (Ans. 1164.3 r.p.m.)
4.55. (a) What are the advantages o f field-flux control method over the armature-circuit-resistance control
method employed for the speed adjustment o f d.c. motors ?
(fc) A d.c. series m otor has the following data for its magnetization curve:

Current, p.u. 0.40 0.60 0.80 1.00


Flux, p.u. 0.62 0.80 0.92 1.00

The per unit values are expressed in terms o f their respective full-load values.
A diverter is used to raise its speed to 1.2 p.u. from full-load speed o f 1.00 p.u., at constant full-load torque.
Neglecting losses, calculate diverter resistance in terms o f series field resistance.
[Hint: V, = Eai « nl <J>t
Also V/ = £ o2« n2 $ 2
Now n,$i = n2$2
or
10, = 1.2 <>2
O2 = 0-833 Ch
<i2 = 0.833 p.u., because Oi is full load flux.

For constant full load torque I<,1 $1 = Ai2 $2


or
l x 1 = / fl2x 0.833
••• / o2= 1.2 p.u.

A flux o f 0.833 p.u. requires a field current Ip (from magnetization-curve) o f 0.65 p.u.

S c an n ed by C a m S c a n n e r
5 32 Electrical Machinery lprob.,

But /„2 = 1.2 p.u.


R.rfi
h2 = Au2
+ rs
or Rdi
0.65= 1.2 etc.) [Ans. Rtlj = 1.182 rj
R<n+ rs
4.56. A 4-kW d.c. series motor has four field coils. The m otor runs at 900 r.p.m . and takes 20 A from a
230 V d.c. source when field coils are in series under normal operation. E stim ate the speed and current taken
by the m otor if field coils are reconnected in two parallel groups o f two in series. A ssum e that the flux is directly
proportional to the current and all losses are neglected. Take the load torque as proportional to
(а) square o f speed (b) speed
(c) speed raised to power zero. [Ans. (a) 1070.4 r.p.m., 33.63 A (6 ) 1134 r.p.m., 31.75 A

(c) 1273 r.p.m., 28.28 A)


4.57. A 4-pole dc series motor runs at a speed o f N rpm on a fixed dc supply w ith all field coils in series
Estimate the speed at which the motor would run i f fed from the sam e supply w ith the field coils re-connected
in two parallel groups o f two coils each. Assume that the load torque varies as the square o f the speed, that
all losses are negligible and that the magnetization characteristic is linear. (/.^t 5 1989)
4.58. A 460-V series motor has its field coils split into two equal halves and takes 40 A when running at
600 rpm with the field coils in series. Determine the speed when the two halves o f the field w inding are connected
in parallel, assuming the load torque to remain constant and the m agnetic circuit to be unsaturated. The
resistance o f the armature circuit is 0.5 Q and that o f the field coils in series is 0.2 Q. ( IAS 1980)
[Ans. 842.55 rpm]
4.59. (a) Describe the disadvantages o f field-flux control method for the speed control o f a d.c. shunt motor.
(б ) A d.c. series motor gave the following open circuit curve at a speed o f 600 r.p.m. :

Field current, A 10 20 30 40 60 80
Open circuit
voltage, V 103.5 158 206 230 259 282

Total resistance o f the armature and field circuit is 1 Q Pint tho j


o f this series motor when connected to 250 V supply speed-torque end current-torque curves

Nm. F ," d a' “ the SP° C<I ° f the m0t° r Wl>en (,) arm alure cur" nt ^ 40 A and (ii) the developed torque is 120
*[Ans.
— (■ b)r (i) 548 r.p,in• (ii)/ui/u
*— 590 ».
r.p.m.)

100 l.If an additional r S a M e ' o f M n ^ i n f c r t e d h i 0 the°lrm atand


C 800 r p m Its fleld resistance is
is
line current in case load torque varies as the square o f the speed '

arm ature current o f 125 A f r o m 'a lu o v T u p r ly ' N th fe x c ita r i ° ' 2- ° <!)nVCj a load at 1245 r P m -. drawing at
total torque developed by the arm ature rem ains unaltered. J c u i a t e n e w s p e e r T " ° f ' a"' ‘
(GATE, 1981)
[Ans. 1626.68 r.p.m.l

------ t
If, A 0.25 0.50 0.75 1.00 1 50 2.00
EU.V 71 133 170 195 220 232

field has 1000 turns and rated arm ature c u r fe n H s 50 A a ? ^ t ^ S^ Unt flGld resistance is 110 a 1116 shUn[
by field-flux control, then, neglecting arm ature reaction, calculate g e ’ i f the sPeed contro1 is carried°
(o ) the range o f external field circuit rp*?iQfnnm r__ . .a
r.p.m. on rated lo a d -a s s u m e negligible arm ature cu rren ra ^ n o-load " 81’ 011 ° f 100° r ‘P m ° n n° ‘ l0ad t0 ^
(h\ f j _______ a .
(b) the series field am peres-turns reauired tn
_
j
at rated load. e sPeed from 1000 r.p.m . at no-load to 913 r.pi.m

Scanned by C a m S c a n n e r
Prob. 4]
D.C. Machines 533
(c) The speed at rated load with the series winHi™ ■ • .
rp.nv at no-load. £ ln circuit and with the shunt field set to give 2000
[Hint. Plot the O.C.C.

,„) A. no-load, E. , 220 V and , , rrom o .c .C . i, A


At rated load. E, = 220 - 50 x 0.2 , 210 V a . 2000 r.p.m..

Therefore, at 1000 r.p.m ., E „ = 210 x — - me


2000 ~ v
and the corresponding field current from O.C.C. is 0 38 A etc
(6) At no-load, £ a = 220 V and 7^= 1.5 A.
At rated load, E(l = 210 V at 913 r.p.m .’

Therefore, at 1000 rpm, Ea = 210 x — ° - = 230 V


913
and the corresponding field current from O.C.C. is 1 857 A *s.m i t * 1 j
1875. Therefore, ATs to be provided by series field winding no-load are 1500 and at rated load,
= 1875 - 1500 = 375 ATs.
(c) At no-load speed o f 2000 r.p.m., Eu = 220 V
Therefore, at 1000 r.p.m ., Ea = 110 V
and field current is 0.3875 A.
At rated load, total field ATs are 0.3875 x 1000 + 375

Equivalent shunt field current = 0 3875 x 1000 + 375


1000 U.7625 Amp.
Corresponding 0.7625 A. R . from O.C.C. is 155.5 V at 1000 r.p.m. But at rated load.
Ea = 220 - 50 x 0.2 = 210 volts,
Speed at rated load = x 1000 . n , n . nm
155.5 00 " 1350 r-P-m [Ans. (a) 36.67 fl to 468.95 fl)

* .i2 £ 2 2 2 : z 2 &
operating from a 200 V supply.
“ J *°rMtak“ s10 Arandr runs
1“ ,h ecube°fspMd
a l WOO T>m when

to SCO™™4" ‘ he Va‘ “ e ° f reSi5tance t0 be inserted in « « ■ “ >e armature to reduce the operating speed
(/.A.S., 1988) [Ans. 11.751 fl)
r 220 V d° cHunt m ° t0r has *** armature resistance o f 0.5 f2 and field circuit resistance o f 290 o It
“ h t h e t a V . ™ 1 ° f 41 A w he" d« K™™>* «• »*»• I f it is desired raise^.h^pee“ by 60 percen!

W m a g n e tis X n T a 'a c T e r is 0" '■ “ reSiS' a" “ 10 b ' " T ”* As5U m e


U -A*., 1994) [Ans. 159.5 fl]
Ttc c i V S t m otor’ w hen fed from 200 V dc source, delivers full-load torque to a load at lOfin
constant c l l l ^ t reduCed t0 10? V > tbe motor sPeed becomes 800 rpm with the load torque rem aining
Calculate the arm ature voltage drop at rated torque. Assume no magnetic saturation. [Ans 12.5 V]

of a dr* k Mf ke aPProxim ate estim ates, giving b rief reasons, o f the changes in the armature current and speed
c snunt m otor w hen operating conditions are changed as described below :
(a) With the load torque and field current held constant, the armature voltage is halved.

ofthespeed1 CUITent held constant- the terminal voltage is halved ; load torque is proportional to square

k) With the arm ature voltage and load torque held constant, the field flux is halved.
d) With the torque held constant, both the armature voltage and field flux are halved. (I.A S 1987)

IA ns. With n, and / ol as the initial values : (a) y , /„ , (6) ^ (c) 2/i,. 2/ a, (d) 2/a,|

load t 67 A 250*V, dc shunt m otor has an arm ature current o f 20 A when running at 1000 rpm against some
•The arm ature resistance is 0.5 fl and brush contact drop is one volt per brush. By how m uch must
the m a T S
armat. m x be reduced to raise the speed by 50% if the developed torque is constant ? Ignore effects o f
re reaction and m agnetic saturation. (GATE, 1990) |An». 34.81%)

Scanned by C a m S c a n n e r
1
534 Electrical Machinery l p rob.4

4.68. Two adjustable-speed dc shunt motors have speed-control range o f 1500 to 500 rpm. In both th
motors, speed adjustm ent is obtained by field-flux control. M otor A drives a load requ iring constant power over
the entire speed range ; m otor D drive a load requiring constant torque over the entire speed range. Ignore all
losses and arm ature reaction in both the motors.
(a) If power outputs are equal at 1500 rpm and arm ature currents are each 90 A , find their armature
currents at 500 rpm.
( b ) If pow er outputs are equal at 500 rpm and arm ature currents are each 90 A, find their armature
currents at 1500 rpm.
IH int. Read Example 4.55 carefully before solving this problem.] [An9. (a) 90 A, 30 A (6) 90 A, 270 A|
4.69. Repeat parts (a) and (b ) o f Problem 4.68 in case speed adjustm ent is obtained by armature-voltage
control, other conditions rem aining unchanged. (Ans. (a) 270 A, 90 A (6) 30 A, 90 A|
4.70. (a) Describe in detail the methods o f speed control o f d.c. shunt m otors. W hile w orking with a very
weak field, a shunt m otor shows tendency (i) to flashover between brushes and (u ) to hunt. Explain clearly the
reasons for these and the methods o f avoiding them.
(6) For the speed control o f a d.c. shunt motor by varying the field flux, show that the field resistanc
should be changed in small steps in order to avoid objectional arm ature current surges.
4.71. (a) W hat is meant by constant power drive and constant torque drive ? E xplain how a d.c. motor can
be adopted for these types o f drives.
(6) How is the rating o f a d.c. motor affected by a decrease in speed ?
(c) W hat is the effect o f brush lead in a d.c. m otor ?
(d) W hich losses o f a d.c. shunt m otor are constant ?
4.72. A d.c. shunt motor is running at 1500 r.p.m. at rated load torque. D iscuss w hat would happen to the
motor operation, i f the following changes are made :
(а) Field terminal are reversed.
(б) Supply wires are reversed.
(c) Brushes are shifted against the direction o f rotation.
(d) Brushes are shifted in the direction o f rotation.
(e ) The arm ature is rewound with a fewer number o f turns o f thick wire.
(f) Some o f the field-tum s are short-circuited.
(Ans. (a) Direction o f rotation is reversed, com m utation unaffected.
(6) Direction o f rotation is unchanged, com m utation unaffected.
(c) M otor speed increases and com m utation is improved.
(d) M otor speed decreases and com m utation is worsened.

or output. maCh' ne in part is ™ k i” f as a generator, determ ine the sh a ft pow er input and the e le c t*

fc) I f the m achine in part to ) is w orktop as a m otor, determ ine electric pow er input and the shaft power

J
Scanned by C a m S c a n n e r
p r o b j^ j_____________________________
" D .C . M a c h in e s 535
[Hint,
IH ini. (d) Vt -= V(l
ya, Vt E(l 1- +2
I(lru +
+ Iuru = ^50
^= 250 +
+ :30 x 0.2 + 2 = 95R v n
to remain at 30 A. Constant torque and constant flux require current

• F V' o.
•• a2 °-2 - 2 = 121 volts.
. _ 1500 x 121
2~ 250 rpm -
(e) K Ia = constant = 7500 W
But Ea = 258 - 2 - 0 .2 1 '

Eu = 256 - 0.2 x 2500


E„a
?2
or Eu - 256 Ea + 1500 = 0
Eu = 250.00 V.
j _ 1500 250
and ~ ~250~ * l u f r P m l
[Ans. (a) 47.75 Nm, (6) 8100 W, 7260 W ; (c) 7740 W, 6900 W, (d) 726 r.p.m. (e) 2500 r.p.m.].

frnmtt’ero to ^O O ^r^m ^ar^oTtai'npH [JSed for adJustable-speed drive over the range o f 0 to 2000 r.p.m. Speed
r m a * *. o btained by adjusting the armature terminal voltage from zero to 230 V with the
field current kept constant. Speeds from 1000 to 2000 r.p.m. are obtained by d e cre a s in g S I field JSh
the armature term m al voltage held constant at 230 V. Armature reaction effects aidTosses may^te & o r e d

S m ! r p ed by t f C l0j d remains constant over the entire speed range. Show the general form
of the curve for arm ature current and power versus speed, over the entire speed range.
(b) Instead o f keeping the load torque constant, suppose that the armature current is not to exceed a
specified value. Show the general form o f the curve for allowable load torque and power versus speed over the
entire speed range. r
t
[Ans. (a) From 0 to 1000 rpm : armature current remains constant at 7 (say) but power rises linearly from
O to V I .
.1
t a
From 1000 to 2000 rpm : armature current and power, both increase linearly from 7 to 21 and
a a
from V 1 to 2 V I respectively.
t a t a r J
(b) From 0 to 1000 rpm : allowable torque remains constant say T but power rises linearly from 0 to
V 7 at 1000 rpm.
t a r

From 1000 to 2000 rpm : allowable torque decreases inversely with speed from
T1
to —

.e. allow able torque , and power remains constant a tV I .


(' speedJ <o
4.75. A d.c. shunt m otor is required to supply a load requiring a constant torque of 120 N-m, over a speed
of 500 to 2000 r.p.m.
(а) Explain the three methods o f obtaining the required speed range.
(б) For each o f the methods o f part (a), specify the base speed and the kW rating of the motor.
(c) Compare the merits and demerits associated with each method of speed control.
(Ans. (6) For armature-circuit-resistance control and armature-terminal voltage control methods, the base
speed is 2000 r.p.m. For field-flux control method, the base speed is 500 r.p.m. For each method,
the power rating is 25.133 kW.]
4.76. (a) Enumerate the various losses in a dc machine. Which of these losses are constant ? Derive
expressions for the efficiency o f a dc generator and a dc motor.
:r
„ <tt A 6 kW, 230 V, 4-pole wave-connected dc motor hat 400 armature: conductors. At fullI load[theruseful
» w per pole is 0.02 Wb and rotational losses are 100 W. F,nd the full-load speed. |Ane. (fc) 876.86 rpml
4„ , . „ a a, , .AurriniT in a dc shunt motor and state how each loss varies as the load
on tK ’ t ■^ numera^e tbe l° s®®s 0 , j p»erminjn(r the efficiency of a shunt motor at various loads
he machine is altered. Describe a method o f d eterm ining ^ e euici y
d wiU»outactu^ly p ^ ttta ^ ,M d on the rnoWr. Critically examine the assumptions made ia the method described.

S c an n ed by C a m S c a n n e r
536 Electrical Machinery

(b) The speed o f a 500 V dc series motor coupled to a fan is reduced to one h a lf o f full speed by a
resistance. At full speed, the current is 100 A and the load torque is proportional to the square of the
how that the power input is proportional to the cube o f the speed and calculate the resistance re
assum ing that the field is unsaturated and motor losses do not vary. (Ans fAi
,7-50O]
4.78. Discuss how power input and motor torque get adjusted autom atically as load on the shaft of rnii ^
dc m otor types is varied :
(a) shunt m otor (6) series motor (c) cumulative compound motor.
(H inM a) Shaft load increases, speed falls, counter e.m.f. falls, arm ature current rises etc. (fc) Speed fa||
= "rr ' rises etc.
K “ * + (ra + r,)

(c) Speed falls, Iu = Vf rises etc.]


Kum+ lra + r,)- K * *
4.79. A dc shunt motor is running at a certain speed. Discuss the effect on the speed o f this motor if t
(i) line voltage is reduced to half
(ii) armature-term inal voltage is reduced to h a lf but its field current is kept constant
(Hi) field current is reduced to h alf but its armature terminal voltage is held fixed.
Neglect armature reaction and all losses. Answer the three parts listed above under the assumrninn r
m agnetic saturation (a) neglected (fc) included. assumption of

(iii) s '£ ”ed rises] N ° Chanee SP“ d<“ >‘ Peed h“ 1VCd W ) Speed d°” blcd (W (i)Speed l e a s e s ® ) speed hahrtd
4.80. (a) What is meant by torque ?
(fc) W hat is the difference between speed regulation and speed control ?
(c) Distinguish between constant-speed, variable-speed and adjustable-speed motors.
(d) Discuss the significance o f back e.m.f. in a dc motor.
[H in t: (a) Tendency o f m otor to produce shaft rotation.

just a s s g j i K o t s andm limilsthearraature» *-to*-


4.81. A dc shunt m achine gave the following data for its OCC at 1500 rpm :

0.6 0.9 1.2 1.5 1.8 2.10 2.4 2.7


120 180 229 260 282 298 306 310

lS O O ^ m . ^ T h e ^ o - lo a d ^ u r r e n t ^ n e g l^ b u '^ e ^ s h 'u n t ^ f ie ld 'w in d 'in ^ lf a s '^ O O ^ t u r n s ^ e r ^ p o le ^ m 0 t°r ' ru n sa l


C alculate the num ber o f series-field turns per Dole rem.irpH ,
an arm ature current o f 90 A from the m ains Thi«s arm afi reduce the speed to 1200 rpm while taking
series-field w in din g loss o f 450 W. rC cu rren l cau ses an arm ature loss o f 810 W and

[H in t : At 1200 rpm , En2 = 230 - — - - 218 V and />


, 90 90 and thls e m f- at 1500 rpm is 270 V which needs a
field cu rren t o f 1.635 A etc.]
. Qf. * p - . nn IA n s . 10 turns)
4 .8 J . i n tig. 4.80, c u r r e n t i n p u t to e a c h m o t o r w h e t h e r in . .
v o l t a g e i s Vt. C a l c u l a t e t h e r a t i o o f t o ) s p e e d s a n d « ,) t o r q u e s w t h m o to n ' W h C rC aS
__ • ... , „ / t o r q u e s w i t n m o t o r s in p a r a l l e l to t h o 1 * w it h m otors in
s e n e s . D o e s t h t s s c h e m e o ff e r a c o n s t a n t - p o w o r o r a c o n s t a n t - ,o r q u o d r i v e u n d e r t h e I d i t m ^ p e l d 7

. 0_ ... (Ans. (a) 2 (fc) 1. C onstr.nt-iorque drivel


4.83. (a) D i s c u s s t h e effect o f s p e e d a n d s i z e o n t h e e f f i c ie n c y o f d c m a c h in e s .

(6 ) F o r a 4 0 0 V lo n g - s h u n t c o m p o u n d g e n e r a t o r • t h e c o n s t a n t l r . e e ^ • r • • j in-id
r o t a t i o n a l l o s s , i s l O k W . T h e r e s i s t a n c e s o f t h e 'armature a V r f a s o n T T " ® . ° [ e ” ,lo l ,u n 0n d
0 .1 Cl, 0.05 Cl a n d 5 0 Cl. F i n d t h e m a x i m u m e f f i c ie n c y a n d t h e o u t p u t n l w h i c h i t o c c u r s 'L S r ' SP l
IH i n t : (b) l l (ra + r,) = 1 0 k W e tc .]
IA n s. (6 ) 83.34%, 100.08 kWI

Scanned by C a m S c a n n e r
D.C. M achines 537
|
4.84. fa) !oss®s *n a dc machine. Which of these losses arc (0 constant (ii)
to current and (t») proportional to current squared.
p r o p o r t io n a l

< » In a,t h T cr t h e ht o t o T l o « « nw ed,d/ , ; “ rr“ t |M5CS at 1000 rpm are 200 W and 100 W respectively. Find
the speed at which the total losses would bo reduced to half on the assumption of constant field flux.
|Ans. (b) 581.015 rpml

4-85- (A mnS ai n°ul0ad 3t 600 r p m- ^ resistance of the field coils is 25 C2. Find
what resistance must be placed in series with the field coils to increase the speed to 700 r.p.m. at no load. Neglect
armature reaction an rmature resistance drop. The magnetization curve for the machine is as under :

If.A -* 1 2 3 4 5 6
0. Wb —» 0.0044 0.008 0.0102 0.0115 0.0121 0.0124

where If= Field current and


<t>= Field flux per pipe (Ans. 21.15 fil
4.86. In a test on a d.c. shunt generator, whose full load output is 200 kW at 250 V, the following figures
were obtained.
(a) With the machine at rest, a p.d. o f 8 volts produced an armature current of 400 A.
(b) With the motor running at no-load and at rated speed, the line current was 36 A, the field current 12
amperes and the supply voltage 250 V.
Obtain the generator efficiency at full load and half full load. !■
Hi !
8
[Hint.
= 400 = 0 02
For motor at no-load, Ia = 36 - 12 = 24 A.
1
/ 2 ra at no load = (24)2 x 0.02 = 11.52 W ft
.-. Constant losses = 36 x 2 5 0 - 11.52 = 8988.48 W.
Generator on full load,

••• —
ill
l2
a ra = (812)2 (0.02) = 13200 W.
Total losses = 8988.48 + 13,200 = 22,188.48 W r
,
22,188.48
Tw= (1- 200,000+ 22,188.48 x 100 etc.] |Ans. 90.019% ; 88.981%]

4.87. (a) W hat is the effect o f excitation, speed and load on the losses of a d.c. machine ?
(b) A 230-V d.c., shunt m otor is taking 5 A when running light (i.e. at no load). The armature resistance
(including brushes) is 0.2 Q and field circuit resistance is 115 D. For an input current o f 72 A, calculate the
shaft output and efficiency. Also calculate the armature current at which the efficiency is maximum.
[Ans. 14.432 kW, 87.15%, 75.77 A]

4.88. A 600 V dc shunt m otor drives a 60 kW load at 1000 rpm. The field resistance is 100 D and armature
resistance is 0.15 Q. Stray-load loss is negligible. In case motor efficiency is 85%, determine
(a) the rotational losses
(b) the speed at no load and the speed regulation. [Ans. (a) 5118.3 W (b) 1026.52 rpm, 2.652%1

4.89. Hopkinson’s test on two sim ilar dc shunt machines gave the following data :
Line voltage 230 V • Line current, excluding both the field currents, 40 A ; motor armature current 350 A ;
ne]d currents 5 A and 4.2 A.
Calculate the efficiency o f each machine. Armature resistance of each machine is 0.02 fi.
[Ans. Generator efficiency = 92.855% ; Motor efficiency = 92.844%!

lh» -4'90, (o) Justify correctness or otherwise o f the statem ent: “For increasing the speed of dc series motor,
Ie armature diverter’ connection is preferred to the ‘field diverter’ connection.” ( /A S ., 19S9)

n. In electric drive schem es ‘field forcing1 is recommended for reducing starting time o f driving motors.
,Sci« s . ’ (l.A.S., 1988)

c i i 11 j o a i ii i c i
-3 8 Electrical Machinery____________________________________________________________________ ____[Prob 4

^ ^'xP^a*n what happens when the field current o f a shunt wound m otor is reduced suddenly by ab0
( / A .S „ jg 8Q)
IH int : (o) Arm ature diverter or shunted arm ature method is used only for decreasing the snpori /> ,
sen es m otor below the base speed.] speedo fdc

f,A
4.91. (a) “The m agnetom otive force due to armature reaction in a dc generator can be effectively utili j
0 result in an equivalent two generators in cascade in one frame and yieldin g larger power amplificati «
Justify this statement. 0 on'
„ 1982)
(o) A cross-field m achine may behave as a constant voltage or a constant current generator depend
upon the degree o f com pensation.” Justify on the correctness or otherwise o f this statem ent. (I.A.S 199^
Cc) W ard-Leonard method o f speed control o f a dc m otor provides a sm ooth control in both the direction ■
Justify on the correctness or otherwise o f this statement. ( /^ 5

[H in t : (a) The two generators in cascade in one frame yield a cross-field generator.]
4.92. (a) In cross-field machines, one pole consists o f two polar projections. Explain the reason.
(b) In cross-field generators, the polarity o f the output brushes rem ains unaltered even with revered
direction o f rotation. Explain. reversed
(c) A 4-kW , 250 V, 3000 rpm amplidyne has the following constants :
rf = 60 fi, ra = 5 f2, rc = 1 fl
Voltage constants, Kqf = 3 0 0 V /fie ld amp
K(jq = 120 V/am p, Kqd - 50 V/am p

is zerCoalCU,ate the fieM CUITent 3nd P° Wer g3in 3t rat6d ° UtpUt MS° ° btain these vaIues when the compensation
(Ans. (c) 48.056 mA, 28868.39, 27.125 A, 9.061]
tW p fh Derr e “ } e" preSSIOn for the outPut volta&e o f a partially compensated cross-field machine Obtain
therefrom the external characteristics o f this machine for different degrees o f compensation.
... am plidyne J as an input field w inding o f 100 O resistance and 500 turns. It has a two-pole armature

" f J i is
circuit s s5 fi
T including
ta r c r r tors-
the com T.he winding. The speed is 3000 rpm and an excitation
pensating o fr1 rr
e tk c l' iance
AT on eitherof ,he

is 20Pn°ancTsa^uration is neglected031™ 13*8 ^ V° ltag0 am pllflcatl0n and Pow er am plification. Load resistance

[ H i n t : (6) <]y= ^ x 500 ( 6 x l O " 5) W b

ugjjP Eq = 6 Vf, $q = 4.5 x 10“ 3 Vf

Power output = J | x V / J x 2 0 etc., (Ans. (6) 90. 25920]

machines’. D i s c ^ t h e f e c t o T o n T h i l ^ t h e y d ^ p t n d ^ P° W8r am plification factor as applied to cross-field

external c h a r a c t e r is t i^ o f a L p h ^ e " ^ V° Uage am phflcation factor (ft) po ver am plification factor and (in)

upon w h eth er*th l’fp e e d is r a is e d ^ flo w e r e d " *6Xtenial characten stic gets bodily shifted up or down depending

4.95. W hat is a cross-field m achine ? Give its rnnQtrnr'fim.oi , . . . . .


expression for the output voltage o f an am plidyne. UF6S ex Plam its working. Denve a

G i v e a n y tw o a p p lic a t io n s o f a m p lid y n e .

,enet dc raotors- Discuss ,hcir

CirCUil ° f “ PM D C m 0l0r' H “ W d “ S “ d ilfcr equivalent circuit »f*

D e r iv e e .m .f . e n d to r q u e e x p r e s s io n s fo r n P M D C m o to r fr o m th o c o r r e s p o n d in g e x p r e s s io n s used in
c o n v e n t io n a l d c m o to r s . * r

ScannecTby C a m S c a n n e r
Prob- 4]______________________________________
, ' ' D.C. Machines 539
4.97. A small perm anent-m agnet dc motor runs at i ---------------------------
fed from 6 V dc source. Its armature circuit resistanceis ° f 10’600 rpm and takes 1° mA when
(fl) no-load rotational losses its
(fc) stall torque
(c) speed to achieve a shaft power 1W
(d) efficiency under the conditions o f part (c).

4.98. (a) Compare the application o f dc geiltTatore w T lT W ^ 5^ ^ ^ ^ 8401 6 ^ W) 12*im


Are the dc generators being built in the present ace ? TV l ° transformer-rectifier equipment.
is indispensable. P Cnl age * Dlscuss applications where the use o f dc generators
(,b) Describe the applications o f dc shunt series anH fc,,™ i *
„ . j * , . , ’ cumulative compound motors
How do dc motors com pete with ac motors ? Discuss motors.
4.99. A 500 V, 25 HP (= 18.65 kW) dc shim ! ♦>
and armature resistances are 650 ohm’s and 0 57 o h n /r e s r f5 ?■ Cu.rreat of 2-4 A whilerunning light. The field
brush drop of 2 V. &‘ ° hm’ resp«tiv ely. Calculate full-loadefficiency, assuming a
(I.E.S., 2002) (Ans. 89.297%!

S c an n ed by C a m S c a n n e r
\

Polyphase Synchronous Machines


A three-phase synchronous machine is a doubly-excited ore machine, because its field wind­
ing is energised from a dc source and its arm ature winding is connected to an ac source. When
working as a motor, the synchronous machine takes in active power from an ac source. Durin?
its working as a generator, synchronous machine delivers or exports ac power. However, the
field winding of a synchronous machine always absorbs or takes in power from a dc source.
Since a synchronous generator delivers ac output, it is also known as an alternator.
Under steady-state conditions, operating speed of a synchronous machine depends on the fre­
quency of armature currents and the number of field poles and is given by Eq. (3.68) as under:
, P n s P ■N s
f = - 2 - = - 5 6 ~ Hz
where P = number of field poles
ns = rotor speed in rps (called synchronous speed)
Ns = rotor speed in rpm (called synchronous speed)
and f = frequency of armature currents.
A synchronous generator is universally employed for the generation of three-phase power
at all generating stations. The largest-sized electrical machines are polyphase synchronous
generators. The highest rating so far for 3-phase alternators, as reported in the literature ri
1700 MW.
The constructional features, of the two types of synchronous m achines, are already
described in Art. 3.2. The cylindrical-rotor construction of Fig. 3.7 is used for two or four pol'e
steam-turbine generators. That is why cylindrical-rotor synchronous g e n e ra to r are called
turbo-generators or turbo-alternators. A cylindrical-rotor synchronous machine is characterised
by long core length and small diameter so as to limit the centrifugal forces developed in the
high-speed rotor. The salient-pole construction shown in Fig. 3.6 is the most suitable for multi­
polar slow-speed water-turbine generators. That is why salient-pole synchronous generators
are called hydro-generators. Most of the synchronous motors are of the salient-pole type. A
salient-pole synchronous machine has small core length and large diam eter so that large num
ber of field poles can be accommodated on the rotor periphery.
The expression h r the generated emf in a synchronous machine is derived in Art. 3.4.3. It has also
>een concluded in Art. 3.8 that for the production of electromagnetic torque the relative vekvity
between the interacting stator and rotor fields must be zero. The object of this chapter is to develop
the techniques for the steady-state analysis of polyphase synchronous machines.
5.1. E x c ita tio n S y stem s fo r S y n ch ro n o u s M ach in es
In large synchronous machines, the field winding is always provided on the rotor, because
ol certain advantages described in Art. 3.2. In the present article is given a brief account of the

canned by CamScanner
13
Polyphase S yn ch ron ous M achines 541

rjous schemes employed for supplying dc excitation to the field winding of large synchronous
va
machines- Some of the more important excitation systems are given below :
(a) D-C- E x c ite rs . This is an old conventional method of exciting the field windings of
synchronous generators, in this method ; three machines, namely pilot exciter, main exciter
and the main 3-phase alternator are mechanically coupled and are therefore driven by the same
shaft- The pilot exciter is a dc shunt generator feeding the field winding of a main exciter. The
main exciter is a separately-excited dc generator. The dc output from the main exciter is given
to the field winding ot the main alternator through brushes and slip rings as shown in Fig. 5.1.

The conventional method of excitation suffers from cooling and maintenance problems as­
sociated with slip rings, brushes and commutators as the alternator ratings rise. The trend
toward modern excitation systems has been to decrease these problems by minimising the
number of sliding contacts and brushes. This trend has led to the development of static-excita-
tion and brushless-excitation systems.
(6 ) S ta tic E x c ita tio n . In this method, the excitation power for the main alternator field is
drawn from output term inals of the main 3-phase alternator. For this purpose, a 3-phase tran s­
former TR steps down the alternator voltage to the desired value. This 3-phase voltage is fed to
the 3-phase full-converter bridge using thyristors. The firing angle of these thyristors is con­
trolled by means of a regulator which picks up the signal from alternator terminals through
potential transform er PT and current transformer CT as shown in Fig. 5.2. The controlled
power output from thyristor unit is delivered to the field winding of main alternator through
brushes and slip rings as shown.

125V Balfery bank


for field flashing

Fig. 5-2. Static excitation for a synchronous generator.

Scanned by CamScanner
542 Electrical M achinery

For initiating the process of static excitation, first of all, field winding is switched on to the
station inttery bank to establish the field current in alternator The alternator speed is a,J
justed to rated speed. After the output voltage from alternator has built up sufficiently, ^
alternator field winding is disconnected from battery bank and is switched on to the thyristor
bridge output. , , J. .... ,
A good number of protection devices are installed in the static excitation scheme f0r any
possible fault in the excitation system.
The advantages of static excitation are as under :
( 1) The excitation system, with the use of reliable and high-power SCRs, is simple in design
and provides fast response characteristics as needed in modern power systems.
(2) Since there is no separate rotating-type exciter, the system is free from friction, windage
and commutator loss occurring in the exciter. This makes the system more efficient and in
addition, maintenance is reduced.
(3) As excitation energy is taken directly from the alternator terminals, the excitation volt­
age is proportional to alternator speed. This improves the overall system performance consi­
derably.
(c) Brushless excitation. This method of excitation is illustrated in Fig. 5.3. In this
scheme, main shaft of prime-mover drives pilot exciter, main exciter and the main alternate.
Silicon diode rectifiers are also mounted on the main shaft.

terminal
icrmi
Fig. 5.3. Brushless-excitation for a synchronous generator.
Pilot exciter is a permanent-magnet alternator with perm anent-m agnct poles on the rotor
thvr ^ o ^ r o h T h / e W1,ndln/ ° ni he f at0r' Three'P^ase power from pilot exciter is fed to
sunnlied to station "r T lP °n r After rectification, the controlled dc output is
supplied to stationary field winding of main exciter. The three-phase power developed in the
n ^ a T s h a f t The d t *** the r ° tati" B silicon-diode’rectifiermounted
shaft to the t i f 2 ero tPo T u fr0n: dl° t reCiiflCr bride e is delivered, along the main hollow
shaft, to the main alternator field without brushes and slip rings.
A signal, picked from alternator terminals throuph PT anri pt* * i i.u r • rrionf
thyristor bridge. This enables the control of field current I f th P T '.C° n tr°’S th? fir' nB '
governs the alternator output voltage. Since the « r W I m“ “ 2,xclter whlch «ventuall>
any sliding contacts and brushes this arrangement r ° Z J S 5 3 d°eS n0t rcq“
come to be called as brushless excitation system mg synchronous machmc 1,85

quirldby Z SyStCmS' tha direCt CUrrCPt rC‘


the brush-gear design br>rmr>pC ncreases considerably (up to 10 kA or so). In such cases,
e dCSlgn beCOmes more im p lica te d and the reliability of the turbogenerator
] — ---------------------------------------------------------------------- --------------------Polyphase Synchronous M achines 543

e ope
“i,erati0ntodrtCIfe ^ b rbrushless
^ M e w excitation
« d ta IH«*in® St° 'Uti0" °f foedinB the ficld windin8 »f larSO tur­
bogene system. In view of its many advantages the brushless
Xtition system is employed in almost all large tnrhmro™ \many advantages, tne orusniest
d these days turbogenerators being designed and manufac-

g 2i flu x and mmf p h asors in synchronous machines

,f wa^e° For a c v l i n d r i c a h ^ ° ° interaction between field mmf wave and arma­


ture m , seDarafeiv anj r synchronous machine, space and time phasor diagrams
are deve P . p salip^001? *n 0ne *'1£ure f°r the purpose of analysing the
machine p ' 1 . n -pole, however, the effect of orientation of armature mmf
wave with respect to held poles is examined.
5.2. 1.C y ^ d r i c a l ^ °to r synchronous m achines. A physical picture of the interaction
between ie rma ure mm waves can be obtained by (i) first developing the space-
phasor diagram involving field mmf, armature mmf and their resultant mmf and then (ii)
developing e ime p asor mgr am involving excitation voltage and armature current. Space
and time-phasor mgrams are then combined in one phasor diagram. Only the space-fun-
damenta componen s o ie -mmf and armature-mmf waves are considered in this article and
their space-harmonics are neglected.
For a better understanding of the internal happenings in a synchronous machine, the alternator
operation is considered under different operating power factors and loads as follows:
(i) No-load operation (ii) unity-pf load (Hi) zero-pf lagging load (iv) zero-pf leading load and
(v) lagging-pf load. Alter this, operation of cylindrical-rotor synchronous motor is also exa­
mined.
In the presentation that follows, the magnetic saturation is neglected. This allows the re­
presentation of mmf wave or phasor by their corresponding flux wave or phasor.
Case I : N o-load o p eratio n . The synchronous generator is brought up to synchronous
speed by adjusting the speed of its prime-mover. The alternator is now separately-excited from
a dc source. The alternator terminal voltage at no load is made equal to its rated value by
adjusting its field current. The per-phase generated emf, as per Eq. (3.24), is given by
Ef = < 2 n fN phkw -ty ...(5 .2 )
where Ef = no-load voltage, excitation voltage or excitation emf
P n
f - —- — = frequency of generated emf

Nph = series turns per phase


kw = winding factor
<|y= flux per pole produced by dc current in the field winding.
It has a lread y been shown in Eqs. (3 .1 2 ) and (3.16) th at
generated emf lags by 90° the flux that generates it. This is indicated Axis of
in Fig. 5.4, where Ef is shown lagging <{y by 90°. In this figure, field held o
g0<
mmf per pole F f is equal to IfN f. As saturation is ignored, field flux —
rf
phasor i(yis also indicated in phase with field mmfF/. Axis of field is
along <jy. As field winding m mf F f and therefore field flux ty- are FlBd^ ra^ea{1at0°[oa^ sor
sinusoidally distributed along the air-gap periphery, these are repre­
sented by phasors /'/•and <fyin Fig. 5.4.
Case I I : U n ity p f lo ad . In order to study the effect of armature mmf wave on the field mmf
Wave, consider a two-pole cylindrical-rotor alternator of Fig. 5.5 (a). The field winding on the
r°tor is fed dc through brushes and slip rings and the stator carries 3-phase distributed arma-
tl»e winding. As in Art. 3.7, the concentrated full-pitch coils a a , b b } c c on the stator, repre­
Sent three-phase windings a , b, c in all respects.

S ca n n e d by CamScanner
544 E lectrical M ach in ery

The field current indicated by crosses and dots in the field winding on rotor, creates fi 1
mmf Ff and field flux <Jywhich are sinusoidally distributed along the air-gap periphery. As such
both Ff and <Jyare represented by space phasors Ff and <{y as shown in Fig. 5.5 (a), (6 ) ancj ’
Note that (Jyis in phase with Ff as saturation is neglected. The field flux <tycreates N and Sp0]e
on the rotor. Recall that the emf induced (= Blv) in a coil is maximum when its coil-sides **
lying in the maximum flux density position. As the instant shown in Fig. 5.5 (a), let phase1^
have its coil-side a, a' in the maximum flux density position, facing N, S poles of the rotor. Th
e.m.f. induced in coil-sides a, a will, therefore, be maximum. For anticlockwise rotation of
rotor, emf generated in coil-side a is indicated by dot and in a' by cross. As the coil-sides 6' -
are under the influence of field pole N, emf generated in these two coil-sides must be indicated
by dot. The magnitude of this e.m.f. would, however, be less than the maximum value The
e.m.f. generated by <|yalone is called the excitation voltage and this is indicated in Fig. 5 .5(G)
dots and crosses in the armature winding on the stator as discussed above. The magnitude of
this emf is given by Eq. (5.2).

■A x is of

A x is o f
phase a
w CO

. ^Hav.c-Fu3iuun 01 neia, armature and resultant u .


for unity pf load, (c) space-phasor diagram, (d) time-nhasor tW r mmfPh.a*ors m a cylindrical-rotor alternator
’ diagram and (e) combined space and time phasor.
When alternator is connected to 3-phase load 3 •
rise to 3-phase balanced currents. Let us assume That t w ! . m arm atureW '11,£"
with the excitation emf E,.In other words altem.t 1 j ™ CUrrent
Unite nf n, * . ’ alternator load is at unity pf with respect to Ef

.5 (o),
. S t f^r inpTTT uTTu rrent Iaandexcitationvoltageare maximumat thasame
E ,is maximum in phase ‘o' as indicated. At the same instant,
1

_______________________________ Polyphase Synchronous Machines 545

ature current Ia is also maximum in phase coil-sides a, a ' ; this is shown in Fig. 5.5 (b ).
^mature current in coil-sides b, b and c, c' as indicated in Fig. 5.5 (6) is less than maximum.
Irlp mrnf sef UP ky ff16 armature current is called the armature-reaction mmf. Recall that for
lanced polyphase currents flowing in a polyphase winding, the peak value of the resultant
k3 f wav®'s a^on£ that phase-axis which carries the maximum current, see Art. 3.7. In view
^this, the resultant armature reaction mmf Fa (due to the combined action of 3-phase mmfs),
° set up al° n6 °f P^ase a because this phase carries the maximum current. It is seen
from Fig- 5-5 (5) that synchronously rotating armature mmf Fa acts vertically upward
. kt-hand grip rule) at the instant considered. Since the rotor is also being driven at
vnchronous speed, the relative velocity between Ff and Fa is zero. Note that for the reference
direction of rotation chosen, the field mmf Ff is ahead of F a by 90° in space and this is depicted
cordingly m the space-phasor diagram of Fig. 5.5 (c). In order to draw the space phasor
diagram, the reader should imagine oneself seated on the stator tooth or standing in the air
L-now one would first see rotating mmf Ff passing by oneself and after rotor travel of 90°
in space, the reader would see rotating mmf Fa. Accordingly, Fa is-shown lagging Ff by a space
angle of 90° in Fig. 5.5 (c). The phasor sum of Ff and Fa gives the resultant air-gap mmf F r.
The two poles N, S created on the stator by F a tend to produce an electromagnetic torque by
ttracting rotor N, S poles. For generator operation, the prime-mover torque must be opposite
to this electromagnetic torque and this is indicated in Fig. 5.5 (ft) by prime-mover torque
rotating the rotor anticlockwise.
Armature rotating mmf F a, given by Eq. (3.71), is proportional to armature current l a and
is therefore in phase with‘7 a. In Fig. 5.5 (c), F a is shown vertical and along the axis of phase
‘a’ Therefore, in Fig. 5.5 (d), Ia must be drawn parallel to Fa, i.e. ', along the axis of phase ‘a ’. But
I is in phase’ with E f, therefore Ef must also be drawn along the axis of phase ‘a ’ in the time-
ohasor diagram of Fig. 5.5 (d). Space-phasor diagram 5.5 (c) and time-phasor diagram can be
superimposed to give the combined space and time phasor diagram of Fig. 5.5 (e). Note from
$ 5 5(c) that arm ature mmf F a is perpendicular to field flux <ty, therefore armature reaction

" Z e S ™ t e d i n p h a s e - a is maximum at the i n field poles N S are m the position


s t a n t

iheem igeneraieum pj nrmahire current in phase-a coil sides a, a


shown in Fig. 5.5 (a). For zero pfilagg|«g 1 > lled to a new position 90° electrical (= 90°
would be maximum after the field p o ^ maximum emf position of Fig. 5.5 (a). In other
mechanical in a 2 -pole machine) ahead oi reaches its maximum value with the same
words, by the time current in phase-a coll’s >g h&ve travelled forward in the direc-
polarity (indicated by dot in a), the rotor p , The currents jn
tion of rotation by 90° electrical, or-half a i n d i c a t e d by dots in 6 ', c' and
phases b and c are, however, less than rr-ultant mmf produced by 3-phase balanced
crosses in 6 , c. As stated before, peak value * hage which carries the maximum cur-
currents in a 3 -phase winding is along the axis rmTent at the instant considered, the resul-
rent. Since phase-a carries the maximum arm . „f Dhase-a in Fig. 5.6 (a). A careful study
tent mmf F a is seen to be vertically up a ong ^ ^ direc^ y 0pp0ses the field mmf Ff.
°f Fig- 5.6 (a) reveals th a t flux created by arma a load where Fa is shown to oppose
% 5.6 (6 ) illustrates the phasor diagram for zer p J ^ ^ ^ 9Q0 Therefore
Ff- In this figure, axis of field is taken horizon f therefore be stated that for zero pf
f« lags tyor F fb y 180", i.e., K °PPoses armature mmf is entirely demagnetizing
jagging load on the 3 -phase alternator, the nature o
te nature.

Scanned by CamScanner
546 E lectrical M achinery

Ef

u
Max. orm .
current
^ _o
N on stator-^
Axis__ % Ff I90 ^a
o? field Fa * *Fr Fa *a

(6)
Fig. 5.6. For zero pf lagging load (a) Space-phasors Ff, Fa and F r ^|Jh ar^ aturC CUrrent
I lagging E f by 90° and (b ) space and tim e-phasor diagram .
Case IV : Zero pf leading load. The three-phase alternator is developing balanced 3-
phase emfs and is connected to a load whose power factor is zero lead.ng with respect to E,. As
before phase-a is considered here as well.
In Fig 5 5 (a) is shown that emf generated in phase-a is maximum because field poles
N S face coil-sides a and a' respectively. For zero pf leading load, the current m phase-a coil-
sides a a' would be maximum when the field poles are 90° electrical before the maximum emf
position of Fig 5.5 (a). This is indicated in Fig. 5.7 (a) where rotor poles N, S are shown 90 prior
to the maximum emf position of Fig. 5.5 (a). Since phase coil-sides a, a' carry maximum current,
the resultant of rotating armature mmf F a is directed vertically up along the axis of phase-a.
Phasor diagram 5.7 (b) illustrates the happenings in Fig. 5.7 (a) where E f is shown lagging
by 90°, I a leads Ef by 90° and F a is in phase with F f so that resultant mmf F = algebraic sum of
Ff andF°a. An examination of Fig. 5.7 reveals that flux created by arm ature mmf F a directly aids
the field'mmf.Fyor the field flux (Jy. It can, therefore, be stated th at for zero pf leading load on a
3 -phase alternator, the armature mmf is entirely m agnetizing in nature.

a)
/

90
Axis o< Fr Ff JJ
field <Pf, h % Fa

• . <b)
fig. Z.7. For zero pf lending lond (n) spneo-phnsors Ff, F„ and Fr with nnnaturc current
!„ lending /i/by 90° and (/;) space and time-phasor diagram.

Scanned by CamScanner
Polyphase Synchronous Machines 547

r e V : Lagging p f load. Lagging power-factor loads, having pf other than zero pf lag-
re more common. So let us consider a general case of armature current Ia lagging the
ili
ging’ ton voltage by a time-phase angle vy° electrical. This means that load p f with respect to
ejccitatlg jagging. For a two-pole machine, \y° electrical = \y° mechanical. In Fig. 5.5 (a), emf,
£flS C d ^ phase-a coil-sides a, a' is maximum because of the maximum flux-cutting action.
Sen^ra 2ingp/’l°a^ having p f angle \|/° electrical, the current in coil a, a’ would attain maximum
p0r *a^ er geid poles have moved to a new position \j/° electrical ahead of the maximum emf
value a .p j 5 5 Xn other words, by the time armature current in coil a, a' attains maxi­
position tg e same polarity (dot in coil-side a), the rotor poles N, S would have moved
fhulT1 v, ^ eiectrical as shown in Fig. 5.8 (a). As before, the resultant of rotating armature
fopvar ^ directed vertically upward along the axis of phase-a, because this phase carries the
^ armature current at the instant considered. A careful study of Fig. 5.8 (a) indicates
maXinJmature reaction mmf Fa lags behind the field mmf Ff by a space angle of (90 + v 0). Resul-
thatar m^s ^ gives mmf F rt this is shown in the space-phasor diagram of Fig. 5.8 (a)
^Afh) In Fig 5 8 (c), Ia is drawn parallel to Fa and Ef is shown leading Ia by an angle \\i°. This
t, in time-phasor diagram of Fig. 5.8 (c). As stated before, excitation emf lags the field mmf
r S flu * V by a time-phase angle of 90”. Also I „ lags E , by v “ because load p f . s cos y,
f f’ ng Recognition of these facts leads to the combined space and time phasor diagram of t ig.
iTid) for a lagging p f - cos y°.

L— A*'S ol

• a rvlindrical-rotor alternator for a


and resultant ^
5.8. (a) Space-orientation of field, a™?^'irg_phasor diagram and (d) com
^ging pf load (b) space-phasor diagram (c

Scanned by CamScanner
[Art s>2
548 E lectrical M ach in ery _______________________________________________ .. . . . „

In Figs. 5.8 (6 ) and (d), phasor of 5-8 <d). '


Figs. 5.5, 5.6 and 5.7. Axis of p ase a is a f ^ ig made to rotate in clockwise
If direction of rotor rotation is reversea i. -, regults are obtained. With armature
direction as shown in Fig. 5.9 (a). angle as shown in Fig. 5.8 (c), the
current Ia lagging the excitation emt / y « . 9 ,^ The combined phasor diagram of
orientation of phasors F , F and F is ^
Fig. 5.8 (d) is again obtained and it is left to t ^ ^ Qf
A ~ * phase s

A xis of
field

Fie. 5.9. With clockwise rotor rotation (a) a t no load an ^


(5) orientation of space-phasors Ff, Fa and Fr for a lagging p f ■

The resultant, or air-gap, mmf F r is seen to be made up of the phasor sum of field mmf F,
and the armature reaction mmf F 0. Mathematically, it can be written as
...(5.3)
F r = F f+ F a
For a uniform air-gap machine, the reluctance is constant at all angular position of the
rotor. If the iron part of the magnetic circuit is assumed to have infinite permea n ty , 1. .
saturation is neglected, then _
Fr
Reluctance “ Reluctance Reluctance
...(5.4)
$,• = <{>/•+0a
Thus, the waveforms of the various fluxes per pole, i.e. <>f, <}yand <{>a are also sine waves an
can, therefore, be represented by phasors as illustrated in Figs. 5.5 (e), 5.6 (6 ), 5.7 (b ), 5.
and 5.9 (6 ).
The above treatment does not hold good for a salient-pole synchronous machine, becaus
air gap is not uniform.
Space-phasor and time-phasor quantities, as usual, are taken to rotate counter'C,°r g(&),
and this is indicated by an arrow marked co in Figs. 5.5 (c), (d), ( c ) ; 5.6 ( 6 ), 5.7 (6 ) an l • •
(c), (d ). The field mmf wave or field poles have always a tendency to align themselves a
resultant air-gap flux <J>r or along the armature-reaction flux <j>n ; this is shown by an ^
marked Tc in Figs. 5.5 (e) and 5.8 (d ). Since the electromagnetic torque Te opposes the vrc ^^
of rotation co, the machine must act as a generator. It can also be stated by referring to
(6 ), 5.8 (a) and 5.9 (6 ) that the field poles must be driven (by the prime-mover), ahea
resultant air-gap flux, for generator operation. *ure
For zero p f lnnd, alternator output = 3 (per-phase excitation emf, Ef) (per-phase a
— ----- , -Iu)
current, u. (pf).— ro. —
is---------
zero. So —
no gprime-mover
---------— -- torque
— g— — is ------------
needed to run the alternat
^ .01 (neg g (5(b,
friction, windage and no-load core losses). This can be ascertained by referring to b--

Scanned by CamScanner
Polyphase Synchronous M achines 549

7 (b) where field mmf or the field poles are seen to be in line with resultant mmf Fr. As
- tjj poles are already aligned with the resultant mmf F r, no electromagnetic torque is
the fiel
^ p vijndrical-rotor sy n ch ro n o u s m otor. The combined space and time phasor diagram
W vlindrical-rotor
, . rn tn r m n t.n r w
motor n r k in c r aatt a
working lo irm n rr p f can
q lagging

for Gained from Fig. 5.8 (d) which is for an alternator


be obt-ng at a laggingp/'load. This figure is illustrated in Axis Of
o fra lQ where, in order to maintain the conventions* of phase 0
Ef
■ 5 5 to 5-9> the current ~ fa (and not 7a) lags Ef by
■ Fig- 5 1 0 - So’ in this figure» is shown opposite to
’O *Rotating armature mmf phasor F a is in phase with 'Ia

Armature-reaction flux <t>a is also in phase with Fa as


k fj^e resultant, mmf F r_is obtained by the phasor
A x is of
bC° a fF and Ff, i.e. F r = F a + F f, Fig. 5.10. It is seen from fie ld .
suW_ “e the field poles lag behind the resultant / ^
/
. ~fi.iv and the electromagnetic torque Te acts in i r
t 1 :
direction of rotation co, i.e. the machine now must e /
ate as a cylindrical-rotor synchronous motor. In t/ ____
/
°?her words it may be stated that for motor operation,
Z field poles must be dragged behind the resultant air- * Vo, rh
Dflux by the retarding shaft-load torque,
if ^mature current Ia lags Ef by 90° in a three-phase Fig. 5.10. Combined space and time phasor
, 1 i- m n f F icr S 1 1 is ob- diagram for a cylindrical-rotor synchronous
synchronous motor, the phasor diagram of g. . motor /o lagging Ef.
lined It is seen from this figure that armature mmf

“ ayThus be concluded from the foregoing that if armature current f„ lags the excitation
emfEj-by 90°, the nature of arm ature mmf, or armature-reac 10 „
(а) demagnetizing in an alternator and
(б) magnetizing in case of s y n ^ o t i s rnotor.
In case arm ature current Ia leads E f y .
nature of armature mmf, or armature-reaction mm
F«is
(a) magnetizing in case of alternator an
(b) demagnetizing in a s y n c h r o n o u s mo or.
In this article, the c o m b i n e d :s pac®,^|ne^ for
Phasor diagram of Fig. 5.8 id) has been Dh ase
° 3-phase altern ato r and F ig . 5 .1 0 f o r . 3 -phase
synchronous motor. These diagram s, 1^ ^ g ,
^ time-phase angle between E f and l a d for cylmdncal rotor.
diagram
^ s p j p o s i t i o n of F a and therefore air-gap mmf ^ onousinotorwith/0 laggmg^by 90 .

PrWith r e s p e c t to field poles. - v n c h r o n o u s m achines, the flux


y n ciu u W ith
5.2.2 . S a lie n t-p o le M a c h in e s In c y t o d r i c a l - r ^ r ^ ^ orientation o{

induced by arm ature m m f wave is


current ia input current 4a*
cu rrent^ and for a motor.
a generator, current is output
.fASU’- ••

Scanned by CamScanner
(Art. 53
550 E le ctrica l M ach in ery

respect to the field poles. It is because £ &


mg slots) and therefore constant ^ ^ ^ along the interpolar, or quadrature,
field-pole axis, or direct axis, is mu produced by an mmf is noticeably greatei
axis. Therefore, in salient-pole machmes the p line with the
when peak of mmf wave is in salient-pole synchronot
interpolar axis. In other words, the p armature mmf wave with respect to the
machines depends on the spatial orientation of the armature mmi f me

fleldIn 0safient-pole synchronous generator, if the armature c u rr e n t/la g s the ™ltage


E bv90" the armature mmf F . (in phase with and proportional to I.) rs along he field-pole
A^byyu , tnearm
and it directly ar
opposes “ flux^ty, Fig. 5.12
the field ( ) as in
_ 19 la) : aa cyiimu
cylindrical-rotor alternator.
. ,In other
words the armature reaction flux 4>a for V = 90° has a demagnetizing effect on the fie c flinc^
and consequently the resultant air-gap flux $r is reduced-th is is illustrated in Fig. . (W. In
this figure the fundamental sine component * , of the armature reaction flux is shown opposing
the fundamental sine component *,for y . 90° and their
bv dashed-dot sine wave. When the armature current l a leads Ef by 90 , the armature m t a
is aga n along the field-pole axis but it is now directly aiding the field flux ^ n o t shown ,n an,
figure,. In other words, the armature reaction flux an.

J Ef

Axis of
e
■€

°o
03
u

d-axis> . ,Q ,

Field flux.<fy Resultont


air gap flux <Pr Armofure
A rm a ju re s u rfa c e

q -a x rs
q-axis
d -o x is
d -a x is
Fig. 5 .1 2 . Salient-pole synchronous g en erato r air-g ap fluxes
along d irect axis. The flux <|>0 opposes for y = 9 0 ° lagging.

In case the arm ature current is in phase with Ef , the arm ature mmf F a would act on an^ ^
90° away from field, or direct, axis, i.e. along the interpolar axis, Fig. 5.13 (a). In Fig. ■ 0
are illustrated the fundamental sine component of field flux and sine wave ol t a i ^
The actual arm ature flux wave <{>„' is obtained by dividing F a by the air-gap reluctanCmature-
the reluctance along the interpolar axis is quite large than at the pole tips, the actua a

Scanned by CamScanner
Polyphase Synchronous Machines 551

+. n flux wave 4)a' is saddle-shaped


rtion i|UA *
as shown in
--------- " 111
3^ J (f %rn1--'* ^ ,1V tiroirn A ; ic cAam i-» ? of. ai a q-axls
r 8 l3 (ft). This flux wave <$>„' is seen to consist
fig'^ e n t a l component <J)a and a predominant third
^ en
fun component. The resultant oftjyand <J)a' can
t,<irm°n*c ' , • ■' Ta
n ■led by adding their corresponding ordinates
. a __ i-la n ^ rt _* rm
d it Will o e s e e n wtcxu i w o u i i a u i cU I-gdp I1UX
Id be considerably distorted (not shown). Such a
'V°torted flux wave would contain third harmonic la
d lS and
nnv anu as a resu
-------lt,
-- third harm -onic ---------emfs
- are
Fa
eratedi in
• J-Ua r* **»y*o f i i r o nV iocnc U n f 1-U« — ___ /*_
the armature phases but these emfs A xis of
fre zero across the lines. Therefore, when Ia is in fie ld pole •4>q
ahase with Ef, the effect of armature mmf Fa is to
90‘
\ tort the field flux wave without appreciably d-oxis-
^djfying its strength. From above, it can be stated
i _- mmf F
that armature JTa along
o ln n fr fthe
nn rtir n c f ak
direct, or polar, axis (a)
Field flux
4>r A rm ature Fundamental
mmf ,F q a rm a tu re flu x ,0 Q

A ctual a rm a tu re
flu x ,£ a Arm ature
• surface
SSI'S

q-axis

( 6) .
Fig 5 13 Salient-pole synchronous machine air-gap fluxes along
‘ ‘ q u ad ratu re axis. The flux is cross-magnetizing.
is magnetizing or demagnetizing in nature in case time-phase angle between E , and /„ i s 9 0 .
But when F fs along quadrature axis, it distorts the main field in case t.me-phase ngie be_
h, V “a T ■ Tn artnfll nractice I lags Ef because of the nature of industrial and
hveen Ef and / . is zero. In actual p « f ^ both on the direct axis and
domestic loads, consequently arm ature mm aP
quadrature axis. . arrn..nt bv reSorting to two-reaction theory.
The effect of salient poles can be taken into account by resorti g
This is discussed in Art. 5.11.
5.3. Phasor D iag ram o f a C y lin d rica l to the resultant m.m.f. of all
The flux actually e x is tin g in t h e air-gap o armature reacti0n m.m.f. F a have been com­
, ladings. In Art. 5.2, the field m.m. - f lindrical-rotorsynchronous machine.The
[aed together to give the r e s u l t a n t m.m. . r, iblc because of the fact t h a t:
Phasor addition of the two m.m.fs. F f and „ V ., gap periphery and
fa) these two m.m.fs. are distributed sinusoidal,ly alo,i(5 f t specdi, , lhc stat0r
the relative velocity between the two i otber<
^ rotor m.m.fs. are stationary with resp

Scanned by CamScanner
[A rt S j
552 E lectrical M achinery

In order to draw the general phasor diagram, it is a s s u m e d th a tth e ^ r phase qu antity


i.e.V, the terminal voltage, /„ the armature current, r„ the armature resistance x , the arma.
ture leakage reactance and cos 9 the load power-factor, are known. Firs o e terminal
voltage VT, with armature current f„ lagging behind V, by the load power-factor angle 9, are
drawn. To V„ are added the armature resistance drop Ia ra in phase with / an he armature
leakage reactance drop /„ xa, perpendicular to/„ as illustrated in Fig. 5.14. The phasor sum of
/„ r and 7„gives the perphase emf. E r,which is ca
behind leakage impedance. _
The air-gap voltage E,. must be induced by the resultant air-gap flux 0,. leading it by 90°.
For a uniform air-gap machine, flux 0,. will be in phase with the resultant m.m.f. F n if the effect
of hysteresis is neglected. The armature reaction m.m.f. F a is in phase with and is proportional
to, the armature current/,,. The field m.m.f. ^produced by the direct current, after overcoming
Fa, must be able to produce the resultant m.m.f. Fr, such that F r = F f + F a, Eq. (5.3).
If the armature current is zero (i.e. on no-load) Fa = 0 and F r = Ff. In other words, at no load
the resultant m.m.f. F r is equal to the field m.m.f. F f and
air-gap flux 0,. is aligned along F{, Fig. 5.15 (c). The emf
generated under no-load conditions must lag Ff by 90
and this is indicated by no-load voltage Ef. This no load
voltage or open-circuit voltage E f is called the excitation
e.m.f. or the excitation voltage. In Fig. 5.14, excitation
voltage Ef is drawn lagging Ff by 90°. The angle y be­
tw e e n ^ and Ia is known as the inner displacement angle
or the internal power-factor angle. The angle 5 between
E f and Vt is called the load angle or the pow er angle. As
E f and E r lag by 90° the respective m.m.fs. Ff and F r that Fig. 5 .1 4 . G eneral phasor diagram of a
produce them, Fig. 5.14, the angle between 2^ and E,. is cylin d rical-rotor alternator.

the same as that between Ff and F r.

5.3.1. The open -circuit and sh o rt-circu it ch a ra c te ris tic s of syn ch ron ou s machines.
These characteristics are useful for finding out the parameters (or constants) of the synchronous
machines and for determining their performance.
For obtaining the open-circuit characteristic (O.C.C.), the alternator is driven at constant
rated speed and the open circuit terminal voltage is noted as the field current is gradually
increased from zero. Thus the OCC is a graph between the field current If or field m.m.f. Ff and
the generated emf E f. For OCC, the final value of /^should be about 125% of the rated voltage.
Fig. 5.15 (a) illustrates the circuit diagram for obtaining O.C.C. and Fig. 5.15 (6 ) shows the
O.C.C. of the alternator with field current along the abscissa and E f along the ordinate. The
O.C.C. (also called the no-load, saturation, or magnetization ch aracteristic) will not be a
straight line, because of saturation in the iron part of the m agnetic circuit. At small values of
field current or Ff, the air gap requires almost the whole of F f and m.m.f. required by the iron
*Some students do ask in the class as to why V, is not taken vertical or horizontal. Here V, is drawn behind
excitation voltage so that (V, + synchronous impedance) = Ef turns out to be vertical. This makes field fiuxfy*
or axis of field, horizontal. When drawing complete phasor diagram involving both e.m.fs and m.m.fs at different
operatingpfs, it seems to be convienient to take field-winding axis horizontal.
Now the question arises, why not take field-axis vertical ? I must say, it all depends upon the vim a
whims of the reader.

Scanned by CamScanner
Polyphase Synchronous M achines 553

Fig. 5.15. (a) C ircuit diagram for obtaining O.C.C. (fc) O.C.C. and S.C.C. of an altern ator and
(c) phasor diagram of an altern ator on open circuit.

is almost negligible. But when the m.m.f. has exceeded a certain value, the iron parts require a
good amount of m.m.f. and the saturation sets in. In Fig. 5.15 (6 ), for E f equal to oa, the m.m.f.
for the air-gap is ab and that for the iron part is be. If it is assumed that the iron part of the
magnetic circuit requires zero m.m.f., the relation between E f and If is called the air-gap line.
Zero m.m.f. for the iron part means that the reluctance is offered by the air-gap alone and that
offered by the iron part is zero. The air-gap line is obtained, if the initial straight line portion
of O.C.C. is extended as shown in Fig. 5.15 (6 ). The O.C.C. may be plotted in per unit values,
where unit voltage refers to rated voltage and unit field current is the excitation corresponding
to unit voltage on the air-gap line.
For obtaining the short-circuit characteristic, the machine is driven at rated synchronous
speed and the arm ature terminals are short-circuited through an ammeter, as shown in Fig.
5-16 (a). Now the field current If is gradually increased from zero, until the short-circuit arm a­
ture current has reached its maximum safe value, equal to about 125 to 150% of the rated
‘torrent. Latter readings may be taken in a short time, in order to avoid armature overheating.
Under short-circuit conditions, the terminal voltage Vt is zero and phasor diagram of Fig.
5-H reduces to th at shown in Fig. 5.16 (6 ). The air-gap e.m.f. E r generated by the resultant
to-m.f. Fr or air-gap flux <}),. is sufficient to overcome the leakage impedance drop /„ (r 0 + jxai),
^ 6- 5.16 (6 ). It can, therefore, be expressed as

E r =7a (ra + jx al) - (5-5)

Scanned by CamScanner
E le c tric a l M ach in ery [Art. «

Fig. 5.16. (a) Circuit diagram for obtaining S.C .C. and
(fe) phasor diagram of the altern ator for short-circuit conditions.

Generally xai is much greater than ru and, therefore, the armature current Ia lags Er by an
angle of nearly 90°. Phasor diagram of Fig. 5.16 (6 ) reveals that F a is almost opposite to
therefore, F a is almost entirely demagnetizing in nature. The resultant m.m.f. F n almost equal
to F f —F a, is reduced and consequently the resultant air-gap flux is decreased, showing thereby
that the saturation under short-circuit conditions does not occur. Therefore, short-circuit char­
acteristic is a straight line through the origin.
Another way of explaining why see is a straight line is as under:
In a synchronous machine, the value oixai is in between 0.1 to 0.2 per unit and ra is usually
negligible. Taking an average value ofx^ as 0.15 pu andra = 0.0, the magnitude of air-gap e.m.f.
E r at rated armature current, from Eq. (5.5), is 0.15 pu, i.e. E r = 0.15 pu. Flux <)),. that generates
E r must also be 0.15 pu. In other words, the resultant air-gap flux <$>r during sc test is only about
0.15 of its value under normal-voltage conditions. Such a low value of <J)r does not saturate the
synchronous machine. Thus, alternator during sc test operates under unsaturated conditions
and as a result see is a straight line.
5.3.2. Zero p o w er-facto r c h a ra c te ris tic and P o tie r tria n g le . Zero power factor char­
acteristic (z.p.f.c.) of an alternator is a plot between armature terminal voltage and its field
current for constant values of armature current and speed. Zero-power-factor characteristic
(z.p.f.c.), in conjunction with O.C.C., is useful in obtaining the armature leakage reactance
x^ and armature reaction m.m.f. Fa. For an alternator, z.p.f.c. is obtained as follows :
(i) The synchronous machine is run at rated synchronous speed by the prime-mover.
(«) A purely inductive load is connected across the arm ature terminals and field current is
increased till full load armature current is flowing.
(Hi) The load is varied in steps and the field current at each step is adjusted to maintain
full-load armature current. The plot of armature terminal voltage and field current recorded at
each step, gives the zero-power-factor characteristic at full-load arm ature current.
The phasor diagram of Fig. 5.14, under zero-power-factor over excited conditions (described
above), takes the form of Fig. 5.17 (a). From this figure, it can be seen th at the terminal voltage
Vt and the air-gap voltage E n are very nearly in phase and are, therefore, related by the simple
algebraic equation

v ‘ = E ' ~ I° x°‘ ed
The resultant m.m.f. F r and the field m.m.f. F f are also very nearly in phase and are rt-la
by the simple algebraic equation
Ff = F r + F a

Scanned by CamScanner
A rt. 5-2 1
Polyphase S y n ch ro n o u s M ach in es 555

at^ero^poww-f^tor over-excited*^I)^<^c^fz.pXcC^nrd PoUer^angie

- resultant

The O.C.C. and z.p.f.c. are shown in Fie 5 17 I’M -x. a.-
equal to OP, the open-circuit voltage is P K With the field e ^ & °a °F ^ CUrrent Jf’
' changed, the arm ature terminals are connected to a purelv ind^F* ®peed. re" lainin&un*
armature current flows. An examination of Fig. 5.17 (a) and (6 ) rev ells t h t t ^ ^ l0ad

resultant
resultant m m f OF,
m.m.f. O p the f a ^ ^ voltage
T h air-gap ^ '***ifthan
E r is FC“ and CB =°Ia
P r( =. ^
is }subtracted
by * - ^ , from
^ T Ft h
= Fer
the terminal voltage F B = PA=,V, is obtained. Since z.p.f.c is a plot between the terminal volt­
age and field current l f or F f, which has not changed from its no load value of OP the point A
lies on the z.p.f.c. The triangle ABC so obtained is called the Potier triangle, where
80 , A ~ F °’ Thus>from the Potier triangle, the armature leakage reactance xul and armature
reaction m.m.f. F a can be determined. rmacure

If the arm ature resistance is assumed zero and the armature current is kept constant then
e size of Potier triangle A BC remains constant and can be shifted parallel to itself with its
ornerC remaining on the O.C.C. and its corner A, tracing the z.p.f.c. Th\is the z.p.f.c. has the
ame shape as the O.C.C. and is shifted vertically downward by an amount equal to Ia xal (i.e.
^akage reactance voltage drop) and horizontally to the right by an amount equal to the arma-
re reaction m.m.f. F a or the field current equivalent to armature reaction m.m.f.
The z.p.f.c. can also be obtained experimentally by connecting an under-excited synchronous
, °r ovar-excited alternator terminals. The excitation of the alternator and motor are
^ged in steps so as to keep the alternator armature current constant at its rated value. The
P giving the variation of alternator terminal voltage with its field current, gives the z.p.f.c.
For determining xal and F a experimentally, it is not necessary to plot the entire z.p.f.c. Only
0ut° Points A and F shown in Fig. 5.17 (b) are sufficient. The point A (PA = rated voltage) is
tamed by actually loading the over-excited alternator by an under-excited synchronous
r* so that the rated arm ature current flows in the alternator. The other point F on the

Scanned by CamScanner
556 Electrical M achinery
A ft
z.p.f.c., corresponds to the zero terminal voltage and can, therefore, be obtained by perform^
short-circuit test. So here O F is the field current required to circulate short-circuit curren? 5A
equal to the armature current (generally rated current) at which the point A is determined in
the zero (near zero) power factor test. rat
Now draw a horizontal line AD, parallel and equal to FO . Through point D, draw a straigU rer
line parallel to the air-gap line, intersecting the O.C.C. at C. Draw CB perpendicular to AD
Then ABC is the Potier triangle from which
BC = Ia xal
and AB = Fa
Since the armature current Ia at which the point A is obtained, is known, xal can be calcu­
lated. sf
The O.C.C. is not exactly the same as the curve between the air-gap voltage Er and the
resultant m.m.f. Fr, assumed before. This difference crops up due to the field-leakage flux being E
different under load from its value at no-load. The effects of this inequality are more important
in salient pole synchronous machines than in cylindrical rotor synchronous machines. In case tl
the field leakage flux at no-load and under z.p.f. load, is the same, then O.C.C. represents c
exactly the relation between Er and Fr — this however, is not the case. At z.p.f. over-excited, the
field current is larger for a given air-gap voltage than for the same voltage on open circuit —
e.g., in Fig. 5.17 (6 ), for voltage equal to FC, the field current or excitation is OP at z.p.f. load
and OF on open circuit. More lf or more Ff causes more leakage flux on load than on no-load
This increment in leakage flux on load from its value at no-load, passes through the field poles
thus field m.m.f. F f must produce this component of leakage flux. Consequently, for a given
voltage on O.C.C. and for the same voltage on voltage
z.p.f. load, the air-gap m.m.f. F r under load a-g line
r- Loaa
Load --
(= Fr + component of field m.m.f. required to v . magnetisation
curve
produce the increment in field leakage flux —
F a) must be more. In view of the above, a point zpfc, Ia--const
on the O.C.C. is shifted to the right, by an
amount depending upon the component of field
m.m.f. required to produce the increment in
field leakage flux. Thus load magnetization
curve at z.p.f. over-excited, i.e. the curve be­
tween Er and F n is as shown by the dotted
curve, below the O.C.C. in Fig. 5.18. This load-
saturation curve intersects DC a t c'. The
f Field c u rre n t or field mmf
Potier triangle is ABC and CB is the reactance
voltage drop Ia xp, due to Potier reactance xp Fig. 5.18. O.C.C., z.p.f.c. and load-magnetization
curve of a synchronous machine.
and c'b' is the actual armature leakage reac­
tance drop Ia xa[. Thus xp is found to be greater than xai.

The O.C.C. and load-magnetization curve at z.p.f. over-excited may come closer or may even
overlap for larger values of field excitations. In view of this, can be determined with fair
accuracy experimentally, by drawing the Potier triangle at higher values of excitations.
In cylindrical-rotor machine, xp and xal are approxim ately equal; but in salient-pole
machines, particularly those having long and thin poles, the difference between x„ and xd may
be substantial. ;

scan n ed Dy u a m b c a n n e r
Polyphase Synchronous M achines 557

^ ^ T ^ g u l a t i o n of an A ltern ator
5.4. ® , g t^e change in terminal voltage, expressed as a percentage (or p.u.) of the
It is de 1 joad at a given power factor is removed, with speed and field curren
^ i ^ u n c b a n g e d . Therefore,
lining uncnai»ecu.................
remaininb Ef - V t .
V oltage regulation = m p.u.

- f y — x 100 in percentage.
V, ‘ “
s* the no-load excitation voltage and Vt is the full-load terminal voltage at the same
Here£'/,lsulc .
a nd field excitation. ,
spee 3 logging power-factor load, Ef always increases and for a leading power-factor oa ,
d consequently the voltage regulation may be positive or negative.
e c r e a s e —

Eftaay of automatic voltage regulators have curtailed the importance of computing


he voltage regulation of synchronous' machines, it is still worth-while to know its value, be-

“ “^V C entheToad "th ro w n off, the voltage rise must be known, since the winding insula-
i. ,iri he able to withstand this increased voltage.
“""(if) Voltage regulation determines the type of automatic voltage-control eqmpment to be

U8et -1 qteadv state short-circuit conditions and stability are affected by the voltage regulation.
% p t r l C e r a t i o n of one aiternator, with other alternators, is affected considerably by

its voltage regulation. , . b obtained by actually loading it. In


In case of small machines, the volt age r gu regulation by actual loading, be-
large machines, it may not be possible to obta ^ roviding the large input. Certain
cause of the cost of d,ss,Patmg * * of power are conducted and from these, the machine
simple tests, involving only small ami>un s p ■ A few methods, for computing the

method, though gives inconsistent r This procedure can be applied to cyhndncal-


introduces the concept of ^ ch™n0" S resultant air-gap flux * r is not affected by the
rotor synchronous machines only, magnetic circuit is assumed to have constant
angular position of the rotor. The iroi» the replacement of mmfs by their cor-
permeability, i.e. saturation is neg^ec ondjng emfs. In short the mmfs can e rep
responding fluxes and, therefore, th e c j r e s p o n t o g ^ e ^ ^ ^ ^ of determining
by their corresponding emfs and it 1
voltage regulation is called the em me ■ resultant mmf F r generates air-gap voltage
Field mmf F f generates E f lagging it by , generate armature-reaction em
lagging it by 90°, similarly arm ature reaction m . ^ ^ ^ o fF f and Fa gives
^ar lagging F a by 9 0 ° - t h i s is illustrated in ig. • ath6matically it can be written as
K similarly the phasor sum o f E f and £ „ must be Er (5 6
f r = F f Jr P a .,.(5.6 6 )

and E r = E f+ E a r

cannecfby CamScanner
i

558 Electrical M achinery [Art S.4

(6 )
Fig. 5.19. (a) and (6) Phasor diagram of component mmfs and the
corresponding emfs for Em f or synchronous-impedance method.

Note that triangle with sides Ff, Fa and Fr is similar to the triangle whose sides are
Ef, Ea, and Er.
It is seen from Fig. 5.15 (b ) that slope of the air-gap line may be expressed as Ef/Ff or
Ef/If. If E f/F f is taken as K, then Ef = KFf. As E( lags Ff (or <|y) by 90°, it is possible to express
the relation between £^and Ff in phasor form as
-E ^ -jK T f
Similarly. Vt — j K F , ...(5.7)
and * „ — JK T .
Note that here K is the slope of the air-gap line.
The armature-rcnction mmfFa is in phase with, and proportional to, armature current/0.
K =c la
where C is a constant, refer to Eq. (3.71)
Substitution of Fa in Eq. (5.7) gives

Eor = - j C K la ...(5,8)
Further substitution of Ear in Eq. (5.6) gives,

E, = E f- jC K la
or Ef = E r + j CK 7a.
Fig. 5.19 (a) reveals that phasor sum of Vt, l a ra and Ia xal gives air-gap emf E r. As an equa­
tion, this can be expressed as
E r = V ,+ Ia (ra + j x al)
Ef =V t + l a ra + j 7a (Xai + CK) ...(5.9)
The term C K in Eq. (5.9) has the dimension of a reactance, because it appears in bracket
with xd . Writing
C K = X ,or-

Scanned by CamScanner
Polyphase Synchronous Machines 559 I :
gq (5.8) can be re-written as
' , K r = - j K F a = - j C K la = ~ jX arl a ...(5.10)

^ 15 ^ t0 the PreS6nCe °f armature reactions


xai + Xar = Xs ...(5.U) 1
where Xs is called synchrotious reactance of the cylindrical-rotor synchronous machine. The 1
reactance Xan due to arma ure reaction mmf is called the armature-reaction reactance or the m
M’
magnetizing reactance, o e at ^ is a fictitious reactance and it accounts for the voltage
vj<jr generated by armature reaction mmf Fa. The term (ra +jXs) = ZS) is called the synchronous
° ,ed a n ce of the cylindrical-rotor synchronous machine.
imp
It is seen from Fig. 5.19 (a) that air-gap voltage Er is given by
r.i
Er = Vt + \ { r a + jXal) ...(5.12 a)
E x a m in a tion of Fig. 5.19 (6 ) reveals that
E f= E r + jIa Xar
~ ^ar a + j l a (x al ^ar)
or Ef =V t + 7a (ra +jXs) ...(5.12 b)
Eqs. 5.12 (a) and (6 ) give the equivalent circuit of a cylindrical-rotor synchronous generator
as shown in Fig. 5.20 (a). Finally, the equivalent circuit reduces to that shown in Fig. 5.20 (fa).
*ar x al *S
-^DTinr- -\w -
la
+
($ )Ef e

(a) (*)
Fig . 5 .2 0 . E q u iv alen t circu it for a cylindrical-rotor synchronous generator.

An examination of Fig. 5.19 (6 ) reveals that 7a xai accounts for armature leakage flux,
laXar accounts for the arm ature reaction mmf F a (or armature reaction flux) and Ef accounts
for the field mmf Ff (or field flux). In short, the synchronous reactance X s takes into account the
entire flux set up by the balanced polyphase armature currents.
For an alternator, the power and, therefore, I a flows out of the machine. For a synchronous
■»otor, the power and, therefore, /„ flows into the machine. Thus the voltage equation for a
synchronous motor can be obtained from alternator voltage equation (5 12) simply by writing
M.) in place of Therefore, the synchronous motor voltage equat

Vt = E f + l a (ra + j X s)
...(5.13)
or Vt = E f + l a Zs
. , . , „„ofnr the cylindrical rotor synchronous motor.
As before Z is the synchronous impedanc
.nrf short-circuit characteristics are required foi
Measurement o f Z g a n d X g. Open-circuit
the determination of Z. and X ,. R 5 15.
r t iq zero and vt - 1
h open-circuit test, the arm atu re current l a

y CamScanner
560 E le c trical M ach in ery lA rt^

In the short-circuit test, entire emf E f is consumed in circulating the short-circuit current
Isc, through the synchronous impedance Zs, Fig. 5.21 (a) and (6 ). Therefore,
E f Open-circuit terminal voltage for a certain field current
s~ L ~ Short-circuit current for the same field current

fa *s
-W W r -'fflfiRT'—tv
Lsc
*r
v+=o
© E f

_v

(a) . . .
Fig. 5.21. (a) Eq uivalent circuit under sh ort-circu it te st and
(6) the phasor diagram under sh ort-circu it test, for a cylindrical rotor a ltern ator.

An open circuit characteristic OCC and short-circuit characteristic see are shown in Fig.
5.22 (a). It is seen from this figure that for a field current of OC amperes,

Zs in ohms = CD = in volts
5 BC in amperes
If there were no saturation, Zs would be constant. Actually Zs is variable and it decreases
with the onset of saturation in the O.C.C., see Fig. 5.22 (a).

\ __-— -
Zs J

ro* ri
££■-£C
E
> a o
.£ .£ £ B
* VI & /
uj — rvi V

If °r Ff —*~
(a) (6)
Fig. 5.22. (a) Determination of synchronous impedance of an alternator and
(b) its voltage-phnsor diagram for a lagging power factor load.

For calculating the voltage regulation, only one value of Z , can be used and since it is vari­
able with the excitation, considerable care should be used in computing Zs. Generally the lowest
value ofZ s, obtained from the largest possible short-circuit current, is used for determining thf
voltage regulation. Now

Z, = W + X ' f

____
Seanried byXam Scanner
The dc resistance r * ot one phase is measured, as usual, by voltmeter-ammeter method. If
the armature winding is star-connected and neutral is not available, then
_ 1 voltmeter reading
Tdc ~ 2 ammeter reading
In case the armature winding is delta-connected, then
_ 3 voltmeter reading
rdc ~ 2 ammeter reading a
The effective armature resistance per phase ra is then calculated from the relation
ra = (1.2 to 1.3) rdc
An accurate method of determining the effective armature resistance is described in Art.
5.17.
After r is calculated, X s can be determined from Eq. (5.14). With the knowledge of
ra and the voltage phasor diagram of Fig. 5.22 (b) is drawn. With Vt as the reference phasor,
the excitation voltage Ef ior any value of 7a, is given by
E f = V t + Ia Z s _
= Vt (1 + j 0.00) + Ia (cos e - j sin 6) (ra + j X s)
= (Vt + L rn cos G + Ia X s sin 0) +J (/„ X s cos 0 - Ia ra sin 0)

. E{ =
If Ia is taken as the reference phasor, then
Ef = Vt (cos 0 + j sin 0) + l a (1 +./ 0 00) (ro X s)
. Ef = V(Vf cos~Q + Ia raf + (Vt sin 0 + IaX tj .
The above voltage eouation can also be obtained by referring to Fig. 5.23 (a), which is self-
I t is seen from this figure that OBD is a right-angled triangle. Therefore
e x p la n a to r y

o d 2= o b 2+ b d 2
= (OA + A B f 4- (BC + CD)2
or E 2 = Cv t c o s 0 + /a raf + (V, sin 0 + Ia X sf

(c)
(a) (b)
Fie 5 23. Voltage phasor diagram for an alternator at
(o) lagging p f load (b) unity p f load and (c) leading p f load.
562 E le c tr ic a l M a c h in e ry ______________ ___________________________________ —

For a leading pf load, the phasor diagram for alternator is shown in Fig. 5.23 (c). It i8' Se^
from this diagram that
OD2 = OB2 + BD 2
or E 2 = (Vt cos 0 + Ia raf + (V, sin 0 - I a Xsf
For upf load, it is seen from the phasor diagram of Fig. 5.23 ( b ) th at
E 2 = (Vt + Ia ra)2 + {Ia X f
Now the voltage regulation in percentage can be obtained from the relation
E f-V .
f y - - x 100

Fig. 5.22 (a) shows that for low values of field current or for low values of flux density, the
synchronous impedance is large and as the saturation sets in, Zs sta rts decreasing. During the
short-circuit test, F a is almost opposite to F f— consequently the resu ltan t m.m.f. F r, responsible
for the resultant air-gap flux density, is very small, Fig. 5.21 ( 6 ). This results in low value of
flux density which is much less than the flux density under actual working conditions—conse­
quently an unsaturated value of Zs is obtained. As unsaturated value of Zs is more than the
saturated value, voltage regulation computed by emf method is much higher than the actual
value. It is because of this reason that the emf method is called a p essim istic method. This
procedure, however, tells us th at the voltage regulation would always be less than that com­
puted by this method.
Under actual operating conditions, the air-gap flux density is quite large, iron is consider­
ably saturated and Zs should, therefore, be small. In order to compute the voltage regulation
under the operating conditions, Zs should be calculated at largest possible short-circuit current
as stated before.
R a n g e o f S y n ch ro n o u s Im p e d a n ce . It may be worth-while at this stage to know the
normal range of the values of synchronous impedance and its components. It is only in the per
unit system that values of these param eters lie in the narrow range. The value of armature
resistance ra is usually around 0.01 pu. It means th at voltage drop in the arm ature resistance
at rated arm ature current is about 1% of the rated voltage. Arm ature leakage reactance is in
the range of 0.1 to 0.2 pu and synchronous reactance is of the order of 1.0 pu. The value of ra is
so low th at it can be neglected for all practical purposes for machine ratings above a few
hundredkVA. It should, however, be included for the computation of losses, temperature inse
and efficiency. It has been found that as the machine size decreases, arm ature resistance in-
creases whereas synchronous reactance decreases.
The magnitude of synchronous reactance is influenced by air-gap length as under:
Magnetizing reactance, Xar = oiL ar
where L ar = magnetizing inductance

But L■‘-'ar _______



air-gap reluctance
A synchronous machine with large air gap has low value of L „ and In other words, Hie
magnitude o fX is inversely proportional to air-gap length. A rm ature leakage reactance .v„, is
usually treated as constant. As and X „ are of the order of 0.15 pu and 0.85 pu respectively,
length 3 m ant r ° le ° " X' S ° 11 can be stated t h a t* . is inversely proportional to air-gaP

Scanned by CamScanner
M a B n e t o m o t i ^ ^ T ^ T n ------------ M«HnK 563
> ■ WCTe int°.their co~ " / e hm t \n ,Vhe e m f' »•«>•<. »H the
procedure is adopted, i.e., each emf is replaced kv „ « r i m m f method, the reverse
sumPti0I1S of e'm f' method are mv°ked here (unifom !? Provided a11 the as-
The voltage equation of a synchronous marKi . g&P 3nd neBlect of saturation).
E r-V +7 y ne’ WOr^ ng as an alternator is
V‘ + Ia ra + j7 a X,
for using m.m.f. method, the voltage Eq (5 121 f -(5.12)
involving m.m.fs. only. Examination of Eq (5 71 « I T \ v T ! . be converted to a new equation
gives the value of the corresponding m.m f T h e r » » f « dlvision of e.m .f. in volts by - j K
into an m.m.f. equation as follows. ’ dlvisi0n of Eq. (5.12) by - j K converts it

_ -J« -jK
or = \ *ar
0 ~ JK ~ jK K k ~ ...(5.15)
As stated before, K is the slope of air-gap line
Ef _ -

From Eq. (5.7), — — - F f and similarlv let - u_ , , -=


~ JK -jK eclual to F rl. The field m.m.f. Ff induces,
in the armature winding, an e.m.f. fla g g in g
it by 90°, similarly the m.m.f. F rl must in­
duce an e.m.f. E' lagging F rl by 90° as shown
in Fig. 5.24.
From Eqs. (5.8) and (5.10), it is seen that
w . Ia CK _
- Ia C - F a = armature reaction
K K
m.m.f. in phase with Ia. Here armature reaction . Fq + FqI
reactance drop I0 Xar has been transformed into
m.m.f. Fa. Similarly, armature leakage reac­
tance drop Ia xal can be transform ed into
Fig. 5.24. Phasor diagram for m.m.f. method.

equivalent m.m.f. F al, such that ~ Xal Vial.


f L- F
K
As Fa is in phase with Iat similarly F^ is also in phase with Ia.
Introducing these changes, Eq. (5.15) becomes,

^ = ^ rl-(^ o i+ ^ c ) -(5 .1 6 )
Note that the m.m.f. (F al + F a) is in phase with the armature current 7a, as illustrated in Fig.
5-24. It is seen from Fig. 5.24 that a is the angle (i) by which Ia lags E ’ and (ii) between the
aormal line to F rl and (Fa + F d ).
Eq* (5.16) can be interpreted by saying that the actual armature reaction m.m.f. F a, has
taen increased by an amount F d . This increase in m.m.f. i.e. F^, takes into account the effect
°f armature leakage reactance xa!.
For a purely reactive load, the phasor diagram of Fig. 5.24 gets modified to that shown in
5.25 (a). From this figure, F rl is seen to be nearly equal to the numerical difference between
f and (Fa + F ai)t i.e. they can be related by simple algebraic equation,
g| F rl = Ff - ( F a + F al) .-(5.17)

, % -
k.-, . ■

Scanned by CamScanner
564 Electrical Machinery

Terminal voltage V, is nearly the same as E' in Fig. 5.25 (a). The O.C.C. is again assume
to represent the relation between E' (voltage behind resistance r„) and m.m.f. Frl. In Fig. 5 2g
(6), OP (= Ff) is any field excitation, from which PF = Fa + Fal has been deducted to give the
resultant m.m.f. Frl = OF. This m.m.f. F rl induces 90° lagging e.m.f. E' equal to Vt = FM = pA
The point A, therefore, lies on zpfc, because it corresponds to the terminal voltage V, and field
m.m.f. Ff or field current If. Here zpfc is seen to be shifted horizontally to the right of O.C.C. by
an amount equal to (Fa + Fa/). Thus (F 0 + Fai) can be obtained by measuring the horizontal dis­
placement between O.C.C. and zpfc, such as MA = O F . At point F , V t = E = 0, consequently
Frl is zero and Ff = Fa + Fal = OF. Therefore, (Fa + F a/) is the m.m.f. or field current, required to
circulate full-load armature current under short circuit test.

m.m.f. method (6) Determination of (F0 + Fal) by m.m.f. method.

Hence, in order to obtain voltage regulation by m.m.f. method:


(i) Plot O.C.C. and S.C.C.
(it) Find E' = Vt + la ra and obtain the corresponding value of F rl from O.C.C.
(iii) Find (Fa + Fal) from S.C.C.
(iu) Calculate Ff = Fn + (F 0 + Fal) as shown in Fig. 5.24. Field mmf F^can also be obtained
from Fig. 5.26 (a), where mmf components in Fig. 5.24 are redrawn by taking F rl horizontal. So,
in this figure, take AB = F rl. Now draw BC = (Fa + F al) at an angle of (90 + a )0 with respect to
AB as shown. Then AC gives the required value of field mmf Ff which is given by the relation
Ff = V(AS + BC sin a )2 + (BC cos a )2 '

(a)
(b)
Fig. 5.26, Pertaining to the calculation of field m m f Fr for (a) mmf method and (6) zpf method.

Scanned by CamScanner
jf f ip h a s e Synchronous M achines 565
Mow, c o r r e s p o n d i n g to f i e ld minffy, obtain Fh-from OCC an d i
^ ’ (or
0,0 nltcinflioi. f mUUOand th«s the voltage regulation of
k 4 3- Zero power factor method. ThiB is nkn ^oii i ^
(or trinnglc) method of obtaining the voltage rpmil y ^ neral method, Potier reac-
^ ' iinvolving
involving voltages is used,
used. whereas
w h e rL . V„
in g*
m m r !n the
. ^ method, the hphasor
««*»»«*, 1" 1®*'*

>s u9cdl For the zpf mcthod. the e.m.fs, are £ haS°r diagram involviug
fold ampcre-turns or field amperes. as v°hages and the m.m.fs. as
The procedure for z.p.f. method requires thp nno ^ ^ ^
F irst of all, determinethe air-gap voltage E r by the relation 801 diagram o fF iS- 5 1 4 -
® r - V , + 7„(>■<,+j*'i) . „
According to the magnitude of E o
nbtain F r fr„m O.C.C. and draw it leading E by 90" The
armature react,™ and‘ armature leakage reactance can be determined from the
Pohcr triangle, ns explamed before. Now F„ ,s drawn in phase with as shown in Fig 5 14
Then
P rK -F .
is Obtained and corresponding to F , excitation voltage is recorded from O.C.C. and the volt­
age regulation obtained.

Field mmf can also be obtained by referring to Fig. 5.26 (6 ), where mmf components in
Fig. 5.14 are redrawn by takmg F r horizontal. So, in this figure, and angle 6
is the angle between E, and Then AC = Ff can be calculated as in Fig. 5.26 (a).
Z.p.f. method requires O.C.C. and z.p.f.c., and gives quite accurate results. Actually only
two points on z.p.f.c. are sufficient, as explained in Art. 5.3.2. ’
5>4*4* New A.SJV. (Aanerican S tan d ard s Association) method. This method is essen­
tially a modification of the m.m.f. method and gives satisfactory results both for cylindrical-
rotor and sahent-pole synchronous machines.
New A.S.A. method requires O.C.C. and z.p.f.c., though the latter may not be known com­
pletely. Only two points A and F , are sufficient to be known on the z.p.f.c., Fig. 5.27. The point
Ais obtained by loading the over-excited alternator by an underexcited synchronous motor till
°ad armature current at rated voltage is flowing. The point F is obtained by noting field
Air

k i^ a l A ir-g a p Ef
^ v o lta g e , Er

7 '
Rated
voMage

- ~0 Field
-0F=Ff *0M- excitation
Fig. 5.27. Determination of voltage regulation by new A.S.A. method.
. ■■■■■■i■■ ..
irieu uy u a m o c a r in e i
\r
566 E lectrical M achinery

excitation (Fa + F al), required to circulate full-load


e.'
armature current when the alternator is s or ci
st
cuited. The armature leakage reactance xa[ is aeter-
lii
mined from the Potier reactance drop BC.
ze
In well-designed sy n c h r o n o u s machines the
cc
ohmic resistance is very small, and in rig. • O
voltage drop Iara is neglected, then angle di
9 = angle a. Fig. 5.28. D eterm ination of field excitation
0 'M , for new A.S.A. method. fi
The m.m.f. phasor diagram of Fig. 5.24, is
redrawn in Fig. 5.28, in which F / = O'G is taken from F
the air-gap line of Fig. 5.27 at rated voltage 0 0 . The m.m.f. ( „ J ^ q in F,g. C
5.27 and this is drawn as GH in Fig. 5.28 making an angle 90 + 9 with F r . The resultant of?;
and (Fa + Fa[) is O'H in Fig. 5.28.
Now determine Er =Vt + l a (rG+jx al) and use the magnitude of E r in obtaining the satura­
tion effects. A horizontal line is drawn through K, so that OK - E Tins line intersects the
air-gap line at H and the O.C.C. at M. The distance HM, on the field excitation scale, gives the f
nHditional excitation that must be added to the unsaturated excitation 0 H, to determine the r
total excitation 0'M = Ff. Now corresponding to O'M = Ff =O F, excitation voltage F P -E f is
read from O.C.C. and the voltage regulation obtained. x
5.4.5. S atu rated sy n ch ron ou s-reactan ce m ethod. In e.m.f. and m.m.f. methods of cal­ c
culating the voltage regulation, the saturation 1
is neglected. However, under actual operating L IN E A R MAG. i
conditions, the magnetic circuit is always in a CURVE

saturated state. The extent of this saturation


under load, can be taken into account by intro­
ducing saturation factor k. As in Art. 5.3.2, as­ MAGNETIZATION
CURVE
sume that the flux-condition in the magnetic
circuit of the machine under load, is decided by
the air-gap voltage E r and the corresponding
resultant m.m.f. F r as read from O.C.C.
With the help of Eq. (5.18), air-gap voltage
E r is calculated for a certain value of Ia and
power-factor at which the voltage regulation is
required. For this value of E r, from Fig. 5.29, the CB K*
field amperes- -
resultant m.m.f. read from O.C.C. is OB and
Fig. 5.29. Pertaining to the saturated synchronous
from the air-gap line resultant m.m.f. read is
reactance.
OC. Now the saturation factor k is defined as
OB ...(5.19'
k =
OC
Had there been no saturation, the resultant m.m.f. (or field current) OB, would g e n e r a te
air-gap voltage E rag. Since two triangles OCF and OBD are similar, the saturation factor c* •
also be written as
OB BD E mR E r from air-gap line
k -
OC BA Er E r from O.C.C.
for the same field m.m.f. or field current.

Scanned by CamScanner
Polyphase Synchronous M achines 567

‘T ° « * » *■ associated
^ r e s p o n d i n g to the rcu ita n t ' ~ d* ^
f r a r magnetization curve OAa. Under this assum r he ™achme operation now follows the
cro, armature m.m.f. F u would be zero and as per E o ’ ^ ^ ature c^ re n t is reduced to
ns’cquence, operating point would move from A to a n In « ! f m' F f~ F r ~ ° K ^ &
nAu The point a is located by the intersection nfii g ar curve
£ £ » i t point K. With linear ?S ? ^ ^
^ ,oad e.m.f. fin - Since it does not really hannen p u the,fiel.d m m f- 0 K would generate a
J -(,nt OK is a real mmntif u- u • ’ A^asnophysical significance. However, the
field current UK is a real quantity which is to be calculatpH in fa,.+ t-w *.
L 5.29 is introduced only for the determination « f r u fac*’rthe construction given in
QAB ®nd OaK, current OK. From similar triangles
OK OB
aK AB
or field current, OK = ^ ( <OB) (5 20)

The required field current is thus calculated from Fn « om j- *


field current of magnitude OK, the excitation e.m.f. Er Kb is found from OCC and voltage
regulation obtained. '
The synchronous reactance X . has two components, see Eq. (5.11). The leakage reactance
* romams constant, because the leakage flux path is mainlyin air and is almost unaffected by
saturation. But the armature reaction reactance X „ is affected by the magnetic saturation
because the path of the armature reaction flux is mainly through iron. In view of this, the
saturation factor k, should be applied to the magnetizing reactance X „ only. The unsaturated
synchronous reactance Xsag is calculated as follows :
2 _ e.m.f. from air-gap line
sa£ armature current from S.C.C.
for the same field current. Note that the unsaturated synchronous impedance Zs remains
constant and may be calculated for any value of field current.
Unsaturated synchronous reactance

X$ag ~ ^ Z ’sag ~ r a ...(5 .2 1 a )


Armature reaction reactance
Xar ~ Xsug —Xd .
Since saturation factor should be applied to Xar only, the saturated synchronous reactance
is given by

Xss = xd + Xmsk ~ al fl ...( 5.21 b)

In order to use the saturated-synchronous-reactance method for voltage-regulation :


(i) First calculate air-gap voltage E r = Vt +7a (ra + jxa[).
(»*) Mark E r = BA on the O.C.C. and find the corresponding voltage BD, on the air-gap line,
% 5.29.
(w’0 Obtain the saturation factor as
. BD
~BA

w:-

Scanned by CamScanner
[Art. 5 4
568 E lectrical M achinery

(iv) Calculate Xsag from Eq. (5.21 a).


(u) Find saturated synchronous reactance Xss from Eq. (5.216).
0») Draw phasor diagram as illustrated in Fig. 5.22 (6), but with X s replaced by Xs, _ now
calculate E^and thus the voltage regulation.
If the field excitation is to be determined for given values of terminal voltage, load current
and power factor, the foregoing five methods described forvoltage-regulaticn computations, are
also helpful. For example, in e.m.f. and saturated synchronous reactance methods, calculate
Ef and then the required field excitation Ff, is read from O.C.C. In e.m.f., zpf and new A.S.A.
methods, the required field excitation under load is calculated directly under the specified load
conditions. , _
E xam p le 5.1. A 220- V, 50 Hz, 6-pole star-connected, alternator with °hm ic resistance of 0.06
ohm per phase, gave the following data for open-circuit, short-circuit an d full-load zero-power-
factor ch aracteristics:

0 .40 0.60 0 .8 0 1.00 1.20 1.40 1 .80 1 2 .2 0 1 2 .6 0 3.00 3.40


Field current, A 0.20

Open-circuit voltage, 146 172 194 232 2 6 1 .5 284 300


29.0 58.0 87.0 116 310
Ef in V

Short-circuit current, 4 0 .0 4 6 .3 5 9 .0
6 .6 13.2 20.0 2 6 .5 32.4 —
— —
h e ln A

Z.p.f. — 0 29 88 140 177 208


— — - — 230
terminal voltage in V

Find the percentage voltage regulation at fu ll-load current o f 40 am p s at power-factorof0.8


lag by (a) e.m.f. m ethod (b) m .m .f m ethod (c) z.p.f. m ethod (d) new A.S.A. m ethod and (e)
saturated-synchronous reactance method.
C om pare the results so obtained.
220
Solution. Rated per phase voltage Vt = = 127 V.

Per phase values for O.C.C. and z.p.f.c. are tabulated below and O.C.C., S.C.C. and z.p.f.c.
are plotted in Fig. 5.30.
' '
If in A 0.2 0.4 0.6 0.8 1.0 1.20 1.40 1.80 2.20 2.60 1 3.00 3.40
Ef in V 16.73 33.5 50.2 67.0 84.3 99.3 112 134 151 164 173.2 179.0
z p f term inal voltage in V —
— — — —
0 16.73 50.8 80.8 102 120 132.7

(a) E.m .f. m eth o d . The values of the synchronous impedance Zs and synchronous reac­
tance Xs) are tabulated below for different values of excitations (taking E^from O.C.C. and I*
from S.C.C. for the same field c u rre n t):

If in A 0.20 0.40 0.60 1.80


0.80 1 .0 0 1.20 1.40
Ef in V 16.73 33.50 50.2 67 84.3 99.3 112 134
I sc «* A 6.6 13.2 20.0 26.5 59.0
32.4 40.0 46.3
Zs in fl 2.535 2.535 2.51 2 27
2.53 2.51 2.48 2.42
X t in £1 2.53 2.53 2.51 2.53 2.42 2.27
2.51 2.48

Scanned by CamScanner
HI
in:
Polyphase Synchronous Machines 569
i
■: i

Fig. 5.30. Determination of alternator voltage regulation. Example 5.1.

HereX s = Zs, since ra is quite small.


For full load and power factor of 0.8 lagging, the phasor diagram is similar to that given in
Fig. 5.22 (6 ). With Vt as the reference phasor,
Vt = 127 + >0.00
l a = 40 (0.8 ->0.6) = (32 ->24).
••• Ef = V t + I a (ra +jXs)
= 127 + (32 - >24) (0.06 + >2.27) = 182.92 +>70.16
or Ef = V(182.92)z + (70.16)* = 195.5 volts.
Percentage voltage regulation
1 9 5 .5 -1 2 7
x 100 = 53.9%.
“ 127
Excitation voltage E,-can also be calculated by referring to Fig. 5.23 (a).
Note that minimum value of Zs, corresponding to maximum short-circuit current, has been
used here.
to) M.m.f. m eth o d . Voltage behind armature resistance ra,

E' = Vt + l ara.
For convenience, take I a as the reference phasor.

E ' = 127 (0.8 + >0.6) + 40 (0.06)

>cannea Dy uambcanner
570 E lectrical M achinery

= 104 -k>76.2
or E' = V(104)* + (7G.2)2 = 129.0 V.
For E' = 129.0 V, the field excitation F rI from O.C.C. is equal to 1.69 A.
From S.C.C., Fa + Fal = The field current required to circulate full-load short circuit current
= 1.20 A.
From Eq. (5.22), the angle a in Fig. 5.24, is given by
' 7 6 .2^
a = tan - 1 = 36.2°.
104
In phasor form, F f - 1.69 icos (90 + a) + j sin (90 + a)l
= 1.69 I- sin a + j cos al
= 1.69 (- 0.591 +j 0.807] = - 1 + j 1.365.
A P . + F->
= - 1 + j 1.365 - 1.20 = - 2.20 + j 1.365
Ff = 2.59 A.
Field mmf Ff can also be computed by referring to Fig. 5.26 (a) where AB = F rl = 1.69 A •
BC = (Fa+Fal) = 1.20 A and a = 36.2C
.-. Ff = V( 1.69+1.20 sin 36.2)2+<1.20 cos 36.2)r = 2.5868 = 2.59 A
Corresponding to Ff = 2.59 A, Ef from O.C.C. is 163.5 V.
1 6 3 5 - 127
.•. Percentage voltage regulation = x 100 = 28.75%.
127
(c) Zero pow er-fnctor m ethod. First of all, the Potier triangle ABC is drawn as described
before Point A corresponds to the rated voltage of 127 V on the z.p.f.c. The line AD is drawn
parallel and equal to F ( ) = 1.2 A. Then DC is drawn parallel to the air-gap line, meeting the
O.C.C. at point C. Perpendicular Cli on AD, gives drop equal to 30 volts.
Armature leakage reactance
30
*•/ = 40 = 0.75 n

The air-gap voltage E,, from Eq. (5.18), is

= V, + 7a (r .+ > x al)
With Ia as the reference phasor,

E r = 127 (0.8 +7 0.6) + 40 (0.06 + j 0.75)


= 104 +j 106.2 ...(5.23)
••• Er = \'(104)2 + (106.2)2 = 148.6 volts.
Correspondmg to E r = 14S.6 V. the field current F r from O.C.C. is 2.134 A. The a rm a tu re
m.m.f. F a, from Potier triangle is AB = 0.84 A.

Now the angle between E r and I , is, say f, then from Eq. (5.23),
- l ( 1 0 6.2^
(3 = tan = 45.6°.
104.0
F r = 2.134 [cos (90 + 45.6) + j sin (90 + 45.6)]
= ( - 1.524 + j 1.494) Amp.
Fa = 0.84 Amp.

j
Scanned by CamScanner
5^4]__________ ___________________ ___________________ Polyphase Synchronous Machines 571

... Ff = F r - F a = [(- 1.524 + j 1.494) - (0.84))


= - 2 .3 6 4 + > 1.494
Ff = 2.797 A.
or
Field mmf F f can also be calculated by referring to Fig. 5.26 (6 ) where AB = 2.134 A,
~ r = 0.84 A and p = 45.6°.
pt' " ____________ ____________
... */■ = V(2.1m + U.B4 sin 45.6°)^ + (0.84 cos 45.6°)*
= 2.7963 A = 2.797 A
For Fy= 2.797 A, the excitation voltage from O.C.C. is 169.0 volts.
... percentage voltage regulation
169 - 127
= — ^ — x 100 = 33.1%.

(id ) New A.S.A. M ethod. The armature leakage reactance xal and the air-gap voltage E r
needed for this method, have already been calculated in part (c).
Corresponding to E r = 148.6 volts, the difference in the field current between the O.C.C. and
the air-gap line, in Fig. 5.30, is HM = 0.366 A.
The field current, corresponding to the rated per phase voltage of 127 volts on the air-gap
line, is O'G = 1.507 A = F / in Fig. 5.30.
For Fig. 5.28, 9 = cos" 1 (0.8) = 36.9°
and GH = (Fa + F^) = 1.2 A.
Now by referring to Fig. 5.28,
0 H = 0'G + (Fa + Fal) sin 9 + j (Fa + F^) cos 9
= 1.507 + (1.2) (0.6) + j (1.2) (0.8)
= 2.227 + j 0.96
or O'H = 2.425 Amp.
Adding the saturation effect HM = 0.366 A to O'H, we get
O'M = O'H + HM = 2.791 Amp.
The excitation voltage ^ f ro m O.C.C., corresponding to O'M = Ff = 2.791 A, is 169 V.
169 —127
/. Percentage voltage regulation = — —rz— x 100 = 33.1%.
1Z I
(e) S a tu r a te d s y n c h r o n o u s -r e a c ta n c e m ethod. From p a rt (c), air-gap voltage
Er = 148.6 volts. In Fig. 5.30, E r is marked as KM and corresponding value of E lxtg from air-gap
line is
K L = 179.5 V.

•■Saturation factor k = 7777 = = 1.207


_ KM 148.6
From part (c), xa /= 0.75 O.
Unsaturated synchronous reactance Xsag, from air-gap line and S.C.C. is, say for a field
Currentof 1.2 A, 1

’ •Saturated synchronous reactance


2.50 —0.75 _ o on n
* . = 0.78 + - t .207 ~

i
Scannea by CamScanner
572 E le c tric a l M a ch in e ry

With the help of part (a), E f= V t + I a (ra + j Xss)


With Vt as a reference phasor,
E n = 127 + (32 - j 24) (0.06 + j 2.20)
= 179.8 + j 68.48
or E^ = 192.1 volts.
From Eq. (5.20), the required field current is

I , = ^ - O | 4 x 2 . 1 5 = 2.78 A
1 Er 148.6
In Fig. 5.30, OB is shown as OK.
For If = 2.78 A, from O.C.C. is found to be about 169 V.

.*. Percentage voltage regulation = x 100 = 33.1%.


127
As already stated, z.p.f. method gives quite accurate results and here the voltage regulation
with this method is 33.1%. The voltage regulation by new A.S.A. and saturated synchronous-
reactance methods is also 33.1% i.e., equal to the accurate figure. Therefore, these two methods
can also be termed as accurate methods.
The voltage regulation by e.m.f. method is 53.9%. This value is much higher than the ac­
curate value of 33.1% and in view of this, this method may be called pessimistic method. The
voltage regulation by m.m.f. method is 28.75%, less than the accurate value of 33.1%, therefore,
this method can be termed as an optimistic method.
5.5. P h y sica l C o n cep ts of S y n ch ro n o u s M ach in e O p eratio n
In well-designed synchronous machines, the arm ature resistance ra is quite small. The ar­
mature leakage flux is also small as compared with the resultant air-gap flux <(»,.. In view of this,
the armature leakage impedance (ra +jxal) may be neglected, under normal operating condi­
tions.
Reference to Fig. 5.19 (6 ) reveals that with the neglect of ra and xal, air-gap voltage E,
becomes equal to the terminal voltage Vt. If the synchronous machine is connected to an infinite
bus , then the air-gap voltage E,. remains equal to the constant bus bar voltage Vt. Under the
conditions stated above, the air-gap flux is given by Eq. (5.2), i.e.
A_ Vt
' = 'V 2n A „A r- ...(5.24)
In the above expression, V „ f,k a Npk, are all constant, therefore, air-gap flux A remains
constant and ,s unaffected by the nature of synchronous machine loatL

illustrated iifnha = ° ’/ “ = ° t ° ad angk &rf b6tWeen F t and F ' is * e r o -th is condition is


illustrated m phasor diagram of Fig. 5.31 (a). With the neglect of r and x , toroue angle is
equal to load angle^For Fa = 0, the machine is neither generating nor motoring- under such a
condition, the synchronous machine is said to be floating on the in fw * T j *

orientation of does n otalter as V^th^infinitetejs v^ltege^is helcf fixetf^°r ° r 3 m0t°r'

* An infinite bus is a constant-frequency constant-voltage source.

M
Scanned by CamScanner
Polyphase S y n ch ron o u s M achines 573

Fig. 5.3 1 . Space-phasor and time-phasor diagrams for aa synchronous


synchronous mac
machine
(a) a t no-load (b) working as a generator and (c) working as a motor.

When the machine shaft is given power from the prime-mover, the field poles are driven
ahead of the resultant air-gap flux <|>,. or resultant m.m.f. Fr. As the field poles are pushed ahead
of Fr by the prime-mover torque, the field m.m.f. Ff also occupies a space position ahead of F,
by an angle 8,y as shown in Fig. 5.31 (6 ). As F^-lags Ff by 90°, E f occupies a position ahead of
Vt by angle 5,.f. It is seen from Fig. 5.8 (d ) that with field poles ahead of F r, the machine works
as an alternator. The appearance of load angle 8,.^between Fr and Ff, gives rise to the develop­
ment of electromagnetic torque, Eq. (3.82),

T' = 8 ^ F f ^ sin V (5.24 a)

When torque Te becomes equal to the applied prime-mover torque, the generator works
under steady state conditions. It is seen from Fig. 5.31 ( b ) thatF^equal toF,. under the assump­
tion) is no longer in phase with F,.. Since the m.m.f. relation Fr = Ff + F a must hold good, arm a­
ture reaction m.m.f. F a comes into existence. For developing Fa, a current in phase with F a,
must flow in the armature winding as shown in Fig. 5.31 (6 ). Excitation emf E{ lags Ff by 90°'
Also, E,. = Vt lags F r by 90°. As F r lags F^rby torque angle 8(/, E, (or Vt) lags Ef by the same angle
y ^ince 4 has a component in phase with Ef or Vt, the machine is acting as a generator deliver­
ing electrical power to the bus bars. In the manner explained above, the mechanical power
lnPut to the synchronous m achine is delivered as electrical powers ouyout equal to
m 14 cos 0 . Here 0 is the pf angle of Ia with respect to Vt or E r Also, E f= V t + j l a Xar.
s^ow^ng that the mechanical power input is equal to the electrical power output
i a cos 0; the torque expression including the load angle 8ra, i.e., the load angle between <{)
and4,,m ust be used.

•• T

But armature m.m.f., from Eq. 3.72(a) is

...(3.72 a)

j
Scanned by CamScanner
574 E lectrical M a c h i n e r y __________________________ _______ ___________ _____________________ ______

. of the values
Substitution i ofca<t>r f,nm Eq. (H 24) and
from Fn a F„a from ^Eq. (3.72 a), gives
V
V, 2 V2 , N ph 4 sin 8ra •w, chr0r
ft d 2
r<0)* 8 P M 2 nfNph k U) m ~1T w p Ff £e
/
= ■Vf/a sin 8ro ^ltuC
s 4n f m ad<

From Art 3.8, P°ne


Synchronous speed in mech. rad per sec., e
9 , . ture
o)s = | (Synchronous speed in elect, rad per sec). the .

. 2 A 4ii£
i.e. cos = -p (271/) = -p'-

From Fig. 5.31 (8), it is seen that 8ro = 9 0 - 0 .


or
Substitution of these values, gives mechanical power input

= Tetos = — •cds • mVJa sin (90 - 0)


e s~ w seri
= m 7 /a cos 0 = electrical power output.
sta
When external load is gradually put on the machine shaft, the field poles slip back in space,
with respect to the air-gap flux <J>r which is fixed in space by V*. When field poles are behind If-
^
F r, the machine works as a motor, see Fig. 5.10. Note that the relative velocity between field
poles and resultant flux (})r is always zero. The effect of field poles lagging behind the space- in
phase position of 0r, causes the appearance of load angle 8r/-between and F f as shown in Fig,
5.31 (c). Since the m.m.f. relation Fr = F f+ F a must hold good again, the armature reaction Vt
m.m.f. Fa comes into existence as before. Consequently the armature current I0 must flow in
the armature winding, in order to establish F a. As before, Ey-andEr lag by 90° the mmf
F f and Fr respectively. The machine now acts as a motor, because Ia has a component in phase ri
opposition to Ef or Vt. If all the losses are ignored, the electrical power input cos 0 can be
shown to be equal to Te ■tos, as before.
In the Figs. 5.31 (6) and (c), t
ab = Ff sin 8r/-= F a cos 0 = CIa cos 0
f
Vt ■F.
or Power P = VJa cos 0 = — sin brf ...(5 .2 5 ) 1

In Eq. (5.25), left hand side Vt Ia cos 0 gives the active power present in both generating and
VE
motoring modes of a synchronous machine. The right hand side of this equation, - 7^ sin
as per Eq. 5.24 (a), is proportional to torque. Thus, Eq. (5.25) demonstrates that active power
existing in both generating and motoring modes is proportional to the mechanical torque as
expected.

In Eq. (5.25), C and Vt are constant. Therefore, for constant Fr, the electrical power varia­
tion with load angle 8r/-is as shown in Fig. 5.32.

It is also seen from Fig. 5.31 (6) and (c) that


ob + be - oc
or Ff cos 8rf+ F a sin 0 = Fr ...(5.26 0)

Scanned by CamScanner
Polyphase Synchronous M achines 575

Under no load, Ff F n Fig. 5.31 (a). But as the syn- iPower


umnous machine is loaded electrically or mechanically n
y gets displaced from F r in space, though their magi P o w -lo o d i: . »
angle curve ’.'Vt.
„{t»des remain unaltered. The resultant m.m.f. F , is now
ade up of the combined action of armature m.m.f. com-
m
F «n
p0nent Fa sin 90 and field m.m.f.
ana neia comnonent Ff
m.m.t. component F.cno sr/, The
cos 8
other component of armature m.m.f. Fa cos 0, or the arma­
ture current component Ia cos 0 ,must adjust itself to meet
the torque requirement Te as per Eq. (5.25). i;__________j__________ \ ort
prom Eq. (5.26 a), 90° 180°
Fig. 5.32. Power-load angle
Ff COS h,f+ CIa s in 0 = F characteristic of a synchronous machine. I

or
V fFf
— K .4. V T a
V F
Vt r
C cos 6’f + V „ s i n e = - L x ...(5.26 6 )
SI
|jK
Since Ia sin 0 has a time-phase angle of 90° with respect to Vf, the product V, Ia sin 0 repre­ fl!
Shi
sents the reactive power flow in the armature of a synchronous machine. ,'K
1i'I
Eq. (5.25) shows that for constant electrical power V, la cos 0, Ff sin 8(/ must remain con­
stant as Vi and C are constant. Let us discuss the effect of varying field mmf Ff or field current liL
,„P;
l<- . . iSBi
IfF/ris increased, 8;/ decreases, sin 8^ decreases so that F/rsin hrf remainsconstant. With
V •F
is
11«fa
1!'$!
increases in Ff,h,.f decreases, cos 6,.^ rises, * ^cos 5rf rises and therefore

V t •F r Vt „
la sin 0 - ^ F f cos 5,.f - reactive power falls with increase in Ff or If. '

IfFyis decreased, Senses, sin 8,yrises so that Ff sin 8,f is constant. So, with increase m F f, 8,f
Vt F f V ■F V ■F
rises, cos 8,y decreases, — cos 8( /falls. Therefore, Vt Ia sin 0 = ~ c ~ - ~ c f cos 8(/ = reactive
power rises with decrease in Ff or If.
It is seen from above that reactive power flow in a synchronous machine can be controlled
byvarying the field excitation F f or the field current If. |[ j
A careful examination of Fig. 5.31 (b ) and (c) reveals that as synchronous machine is loaded
from floating conditions, alternator begins operating at a leading pf whereas the synchronous
motor at a lagging pf.
5.6. Synchronous M otor P h a so r D iagram
For gaining physical insight into the operation of a synchronous machine, the m.m.f. and
flux phasors are very useful. Since these m.m.fs. or fluxes manifest themselves as generated
v°ltages in the armature winding, the voltage phasor diagram of a synchronous-machine may
0Ifly be considered for simplicity. The analysis of the synchronous machine can thus be carried
°utwith the help of its voltage phasor diagram.
to order to draw the synchronous motor phasor diagram, refer to Fig. 5.31. Alternator volt-
aSe Phasor diagram of Fig. 5.31 (b), is redrawn in Fig. 5.33 (a), but for the lagging power factor
anAby including the arm ature leakage impedance (ra + j x j . Note that Fig. 5.33 (a) agrees with
Pis- 5.22 (6 ). Here /„ has a component in phase with and the machine is, therefore, operating
asa generator. If this machine is made to work as a synchronous motor at the same terminal
^ aSe V„ the current / gets reversed and Fig. 5.33 (o) gets mod.fied to that shown m Frg. 5.33
(6)' Note that Fig. 5,33 (6 ) corresponds to the conventions adopted in Fig. 5.10 or in Fig. 5.31

>cannea Dy uambcanner
576 Electrical M achinery

(c). Here Ia has a component in phase opposition to Ef, the machine is, therefore, acting as a
synchronous motor. Voltage equation,
Ef =V t + l a (ra + jX s)
is applicable to both the Figs. 5.33 (a) and (b ). Although synchronous motor phasor diagram of
Fig. 5.33 (6 ) is exact, it is usually customary and simpler, to omit the negative sign with Ia and
draw its phasor diagram as shown in Fig. 5.33 (c). This figure reveals th at the voltage equation
for a synchronous motor is
Vt = Ef + l a (ra +jX s)
which agrees with Eq. (5.13) already obtained for a synchronous motor.
It should, however, be remembered that for an alternator, p h a so r E f is always ahead of
phasor Vt, just as field poles are ahead of <f>r. For a synchronous motor, p h asor Ef is always
behind phasor Vt, just as the field poles are behind <[),..
nno !tfWl11 f6 £ t lpful 3t lhlS Stag® t0 draw synchronous motor phasor diagrams at different
operating pfs. These are shown in F.gs. 5.34 (a) to (c) for the same V, and /„ cos 0. It is seen from
these figures that for lagging pf, Fig. 5.34 (a ),
E j = (Vt cos 0 - I a r a)2 + (V, sin 0 - I a X f

Scanned by CamScanner
Polyphase Synchronous Machines

, , upf, Fie - 534 (6)’ E f = (V‘ ' Iar°)2 + (/“ *«>2


f /for leading pf, Fig. 5.34 (c),
an E) = ( y t C0S 0 - I a r a)2 + (V t sin 9 + I a X t f
Example 5.2..An is connected to an infinite bus. I f its O.C.C.
andS-C.C. are available, explain how its leakage reactance can be determined.
Solution. The zpfc of an alternator, at
ted armature current, can be obtained by
over-exciting it. For the determination of «!
leakage reactance, only two points need be
^own on the z.p.f.c. In Fig. 5.17 (6 ), points F
can be obtained from S.C.C. The other point A
can be obtained by over-exciting the alternator
connected to the under-excited synchronous
m0tor—the alternator phasor diagram per­
taining to this operation is illustrated in Fig.
5.35 (a). In this figure, Vt represents terminal
voltage of the alternator or of the synchronous
motor and Ef is the excitation voltage of the
alternator. k !•
In this example, the synchronous motor is
connected to a constant-voltage source Vt. If I
the field current of synchronous motor is in­ . (o) •• !■ i.
Fig. 5.35. Phasor diagram for cylindrical rotor (a)
creased, Ef increases but Vt remains constant. over-excited alternator, operating at near z.p.f. and (b)
So, now increase the field current, till the over-excited synchronous motor operating at near z.p.f.
II
synchronous motor draws rated armature cur- .
rent—under this condition the phasor diagram of Fig. 5.35 (6) is obtained. It is seen from Fig.
5.35 {b) that the time-phase angle between Ia and Vt is nearly 90°, as it is in Fig. 5.35 (a).
It is seen from above that if an over-excited alternator working at near z.p.f. lagging gives
A as the operating point in Fig. 5.17 ( 6), then the same machine working as an over excited i.
synchronous motor at near z.p.f. leading would give the same operating point.
This is because the small amount of synchronous machine losses have almost negligible
effect on the operating point A. Once the point A in Fig. 5.17 (fc) is plotted, the leakage reactance
*ai can be obtained as explained in Art. 5.3.2.
1
5.7. Operating C h a ra c te ris tics of A lternators and their Ratings
The steady-state operating characteristics of a cylindrical rotor alternator can be obtained
fromits equivalent circuit shown in Fig. 5.20. Only the main characteristics of practical impor­
tance are presented here.
5.7.1. E x te rn a l load ch a ra c te ris tics. The external load characteristics or the alter­
nator volt-ampere characteristics, represent the variation of armature terminal voltage Vt with
armature current / a', for a constant field current.
Consider the alternator to be running at constant speed and with constant excitation, i.e.
e ^temator has constant E f. At no load, Ia = 0, therefore, Ef =Vt = 1.0, Fig. 5.36(a). The nr-
m«ure resistance drop, as usual, is neglected for convenience. For a lagging p.f. load the
pl,as°r diagram is shown in Fig. 5.36 ( 6 ) and V, is less than Bf. For unity power-factor load, V,
18W n less than E .b u t it is more than its value at a lagging power factor load For a leading
^ ■ f a c t o r load V is more than Ef. Thus it is seen that the terminal voltage V, falls on lagging

k_/ y w i i ivVuui ii iw i
578 E lectrical M achinery

vt >vo
v.=10 VH
. Ef-1-0 Ia*s
E ^ io r ■ Vf<10 \
^ a X s E,=1 \ E.-VO f||£
>vt < io \ iIQXS
>,<10

la \
>1. *a\ \
/ \
V l

\ 'o >o
/ \
... (c) (d) («) if)
'-F ig . 5 .3 6 . A lternator p h aa.r diagram s for constant a , at no load , 6 , for lagging power facto,
(c) at unity p.f. (d) for leading p f (e) zpf lag and (/) zpt leaa.

power factor loads and may rise on to


leading power factor loads—this in­ // • 1
1
ference follows from Fig. 5.36. The 1
1
variation of V, with Ia for constant g, 1■0-8 i
1 i\
Ef, is illustrated in Fig. 5.37 (a) for
different power-factor loads. _ \° 9 ' 1
The operation of alternator at j L\\ \\1 1 IJ
X\ \M
\t1 //
zero power factor lagging and zpf £ /0> f /
1
leading is shown in Fig. 5.36 (c) and 1
Armature Currcnt(p.u) Armature current ( p.u.) —
(f ) respectively. For zpf loads, it is seen
. ----
load in kVA(p.uJ
°r ,
rciau u n V, = Ef ± I a X,
th a t sca la r relation (Q) (6)
holds good. As V. = Ef ± l a Xs is the Fig. 5.37. A ltern ato r ex tern a l c h a ra cte ristic s w ith If held constant
. . . . . . . . rn (a) a t its no-load value and (fr) a t its rated V, and rated l„.
equation of a straight line, the varia- w Ul 1
tion of V, with Ia are straight lines for ^ i axs
both z p f lagging and z p f leading loads jT ,Vt=V0
as shown in Fig. 5.37 (a). For zpf lag- ^ vt=io
Er>1u
ging, th e a lg e b ra ic equation is
Vf = Ef —I(jXs and for zpf leading, the Er<|0
algebraic equation is V, = E f+ IaX s. For
zpf lag, when V, = 0, Ia = Ef/X r For zpf
lead, for the same short-circuit current

as during the zpf lag, the terminal


Xs (C)
Fig . 5 .3 8 . A lte rn a to r p h asor d iag ram s for constant terminal
voltage is V, = £ , + * * > = 2Ef = 2.00. v oltage and a t differen t power factors of
. , , j (a) lagging (6) u n ity and (c) leading.
With alternator running a t rated , .r i J . . reducedto
terminal voltage Vt and at rated current Ia (= 1 . 0 ), what happens if load current s r ^
zero ? No-load voltage, or excitation voltage, E f may rise or fall depending upon the loa P­
is discussed below : , „ mnrp than 1°
If term inal voltage at rated current is 1.0, then no-load voltage E f must be m gJg
for a lagging pf load ; this is illustrated in Fig. 5.38 (a). At unity pf also, it is seen irom

Scanned by CamScanner
_ _ _ _________Polyphase Synchronous Machines 579

hat Ef 'xs more ^ an however for leading pf load, Ef is less than V, at rated load. The
(l>) ences drawn from Fig. 5.38 are shown in Fig. 5.37 (b ) in the form of V, - Ia characteristics
'nldifferent load pfs under the condition that alternator delivers rated Vt at rated Ia.
IqXs
5 7.2- Alternator compounding characteristics. These characteristics show the varia-
'f =io . nf field current required to maintain rated terminal voltage, as its load at a specified power
is increased. In practice, the alternator terminal voltage is held constant by controlling its
Id current manually or by automatic voltage
Lmlators.
. order to illustrate the variation of Ef or Ift for
taining constant voltage Vt, refer to Fig. 5.38. At
ma,n p.f., Ef is more than 1.00 if Vt = 1.00. For
a lagging
Ef is a little more than 1.00, but for leading
f in Fig- 5.38 (c), Ef < 1.00. In other words, for main­
taining constant terminal voltage Vt< the excitation
voltage Ef or the field current If should be increased for
laggingpower-factor loads and should be decreased for
leading power-factor loads. For unity power-factor
loads, the increase in l{ with the increase in Ia is small Armature current in p.u
or kVA in p u *■ .
as compared with 0.8 p.f. lagging loads. Fig. 5.39 il­
lustrates these compounding curves for alternators at Fig. 5.39. Alternator compounding curves for
maintaining rated terminal voltage Vt.
0.8 p.f- lag. unity P-f- and at 0.8 p.f. lead.
For zero power-factor lag or lead, the phasor diagrams are already drawn in Figs. 5.36 (e)
and if)respectively. As Ef= Vt ± IaXs is an algebraic equation for zpf loads, the graph between
EfM&la is a straight line for maintaining Vt constant. For zpf lag, the algebraic equation is
Ef= Vt + IJC, fcnd for zpf lead, the algebraic equation is Ef =Vt - IJCS.
Also, for zpf lag, the armature mmf is direct demagnetizing ; therefore the field excitation
or Ef must be increased in order to maintain armature terminal voltage V, constant. For zero
pf lead, the armature mmf is direct magnetizing, the field current and therefore Ef must be
reduced for maintaining Vt constant. This is shown in Fig. 5.39 for zpf loads also. For zpf lead,
y
whenEf = 0, / . = Vt/X s. For zpf lag, for the same short-circuit current as during zpf lead, the
1 I
y
excitation voltage is Ef = Vt + t t •Xs = 2Vt = 2.00 pu.

5.7.3. R ating of a lte rn a to rs. The rating of a.c. machinery, such as alternators, trans­
formers (and cables) is determined by their heating and hence by losses in them. These losses
are made up of I 2R losses, core losses and a small amount of friction and windage losses. The
I2R losses depend on current and the core losses on voltage, therefore, these losses are almost
unaffected by the load power factor. In view of this, rating of a.c. machinery to supply a given
Ioad, is determined by the volt-amperes of that load and not by the load power alone. Thus,
the alternators are rated in terms of kVA or MVA. However, the boiler and turbine sizes and
fuel requirements in a thermal station or the turbine size and water requirements in a
hydroelectric station, are dependent on the power output and are independent of the load volt
amperes. For example, for 100 MW load at 0.8 p.f., the alternator (transformer and transmis­
sion line also) rating is 125 MVA, while the size of the boiler, turbine and fuel (or water) re­
tirement are dfecided by 100 MW alone.

Scanned by CamScanner
580 E le c tric a l M ach in ery

While stating the kVA or MVA rating of alternators, the power-factor (usually from 0.8 1
0.9 lagging) for which they are designed to operate under steady state conditions, must U
stated. In modern alternators, the terminal voltage of the alternator is kept almost constant
(within ± 5% of rated voltage) by automatic voltage regulators. The need for mentioning the
power factor arises, because an alternator designed to operate at 0.9 p.f. at rated load, would
require more field current, more Ef, when operated at 0.80 p.f. and at rated load, see Fig. 5 39
More field current would result in over-heating of the field system, which is not desirable. If the
alternator power factor given on the name-plate, is not mentioned whether it is leading or
lagging one, then it should be taken as a lagging one. This is because, for constant terminal
voltage, lagging power factors require more field current than the leading power factors, see
Fig. 5.39. Thus a lagging power factor places a limitation on exciter output and current in the
field coils.
Under rated power and voltage conditions, reactive power flow handled by an alternator is
limited by its armature heating, for operating power factors near the rated value. When the
) operating power factor is away (usually lower) from its designed value, the reactive power flow
is limited both by the field heating as well as armature heating.
5.8. P o w e r Flow T h rou gh an In d u ctive Im p ed an ce
A more general problem of power flow through an inductive impedance is considered here
since the problems associated with the steady-state power flow in many systems, can be studied
with its help. Fig. 5.40 (a) shows two a.c. voitage-sources E x and E 2 interconnected through an
impedance Z ZQz. With the current I flowing from E x to E 2, the phasor diagram is as shown in
Fig. 5.40 (6 ), from which
e x= e 2+iz
■j _ E \ - E 2
or
z
E,
...(5.27)

Eq. (5.27) shows that current / is the difference of two currents y and lagging behind their

respective voltages by angle 0, as shown in Fig. 5.40 (6 ). Here impedance angle 9Zis given by
0,, = tan - 1
R

Z /9 z
-AW/ TTWtf'- *
R —p - X

t-
Source Load
(a) (b)
F ;g . 5 .4 0 . (a ) C ircu it for inductive im pedance betw een two v o ltag es and (6) its phnsor diagram

J
Scanned by CamScanner
-

&
(
Polyphase Synchronous M achines 581

It ia seen from Fig. 5.40 (6 ) that ctz + 8, = 90°. , Vf5‘.::.i .


J
-

The power Pi at the source end E x of the impedance Z, is given by


Pi = (component of I in phase with E^.
'
prom E<1- (5.27) and Fig. 5.40 (6 ), the component of 1 in phase with E l is seen to be
Ei E " ■ ■1
~2 cos 0* “ ~z cos (5 + 9*)
m ,
P 1- E l E l r, Eo 1
i,;:b
Y cos e* - 2 cos (5 +
•i t".-
El E ,E ii
2 cos 0Z— cos (6 + 02) ...(5.28) im
m
Now 0Z= R
cos Q 2, m
'J,!itil!
.

! j ,ij j»:ii
and if 0, = (90 - “*)is substituted in Eq. (5.28), we get
If
E\ R E ,E 2 li
P, =' “ ~ ~ Y ~ icos “ a *) + 90°M ill

Ei E 2 E\R $
sin (6 - az) + ...(5.29) ill
The power P 2 at the load end E 2 and flowing through the impedance Z, is given by
P 2 = E 2 (component of I in phase with E 2).
From Eq. (5.27) and Fig. 5.40 (6 ), the component of I in phase with E 2 is
'E i Eo '
— cos (0Z- 5) - y cos 0Z

E 2Ei E o 1
R2= — cos (0Z- 8) - — cos 0Z

E 2Ei r Eo
= —^— C0S “ (“ r + &)1 “ Y C0S
ipH
E2E 1 E]
sin (6 + oq) - —| J?
Z2
...(5.30) II
The power flow in a cylindrical-rotor synchronous machine is a special case of the above
ore general problem of power flow through an inductive impedance—this is because the
Va«nt circuit of this machine is identical with the circuit of Fig. 5.40 (a).
and 5 orf’ Undying the power flow in a cylindrical-rotor alternator, compare Figs. 5.40 (a)
•20 (8). It is seen from these figures that
E i = Ef, E 2 = Vt,
Z = Zs = ra +jXs.
’' ovver input to generator, from Eq. (5.29) is
E fV . Ej
p ig " - T 1 sin (8 " <**)+ r* ~ < 5-3D
P
Wer 0u^put of generator, from Eq. (5.30) is
E V V?
Pog = y y sin (&+ a x) - 2 ^ ra ...(5.32)

Icanned by CamScanner
IP

[Art. 5.8
582 E lectrical M achinery _________________________________________________

Fig. 5.20 (6 ) also gives the equivalent circuit for a cylindrical-rotor synchronous motor,
provided E , and V,are interchanged and the load is removed. In vrew of this, when working as
a motor,
E x = Vt, E 2 = E f
Z = Zs = ra +jXs
.*. Power input to motor, from Eq. (5.29) is
vf
Pim = sin (5 - “OLz)
z) ++ ^ ra .-(5.33)
*

Power output of motor, from Eq. (5.30) is

Pom= ^ S i n <8 + O r ) - § r a ...(5.34)

The subscripts i, o ,g and m stand for input, output, generator and motor respectively.
Note that the power at the shaft for a synchronous motor is Pom minus the rotational losses
(friction, windage and core losses). Here Pom is called the mechanical power developed or gross
power developed. Similarly the mechanical power input to generator is Pig plus the rotational
losses.
The difference between inputs (Pig or Pim) and outputs (Pog or Pom), for either a generator or
a motor, must be equal to ohmic loss 7 ^ . This can be proved from Eqs. (5.31) and (5.32) or from
Eqs. (5.33) and (5.34) as follows. From Eqs. (5.31) and (5.32),
EfVt r
Pig ~ Pog = - j — [sin 5 cos cq - cos 8 sin cq - sin 8 cos oq - cos 8 sin cq] + (E f + Vf)
4 Zs
ra p.2 , ^EjVt .
= ~2 (Ef + Vt) 7?— cos 8 sm oq

= JjL ,
o \P*f W) y yF 2 V
* 2! -
COS 0
s s

= ^p2 [Ef + V f - 2Ef Vt cos 8).

From AABC of Fig. 5.40 (b ) and with appropriate changes


(AC)2+ (AB)2 - 2 (AC) (AB) cos 6 = (BC )2
or B 2 + Vf - 2E/V c, os 5 = / 2 Z f
r.
Pig Pog - _2 (la Zs) - l \ r a - ohmic loss.
Zs
Similarly, P im- P 0m = & a.
Generally, arm ature resistance ra is negligible and this makes
oq = 0
and Z .= X S.
With these changes, the Eqs. (5.31, 5.32, 5.33 and 5.34), all reduce to
p ig ~ Pog = Pim - P om= P (say)
E fV ,
= sin 8 ...( 5 .35)
t

Scanned by CamScanner
Polyph ase S y n ch ro n o u s M ach in es 583

flVflm inationofEqs.(5.31), (5.34) and Fie 5 dn


f iterminal^
n a l s where
wueic the
w.c voltage
v isag e is a,,-and power nut™,* E fJ !|nput to generator
isat. . -cIs The powers P ig and P
Ptg om should therefore
Pom therpfh™ k
be pn l . a, 6 m om is
1 also at the same
^ l a c e d by ( - 8). This can be verified as under : opposite to each other but with
5r6P Pom = ~ [Pig from Eq. (5.31)]
EfV t P2
Z sin ^ ~ a *) ~ —4 r

EfV t . El
Zs ‘ sm (5 + a z )~ ~_ 2 r, ...(5.34)
Similarly, Pjm = —Pog.
5.8.1. Maximum power conditions. In this section nf , .
in both alternators and synchronous motors is studied article, maximum power flow
(a) Maximum power output. With constant E f V and 2 thp ™ w r

^ r e X t o l " g6nerat0r " " m° t0r " ° btained ^ differentiating E q . ^ V o r Eq“


^ _ d P ^ _ E f Vt
d b ~ db ~ c°s (6 + oq) = 0
or b + oq = 90°
6 = 90 - a z = 0r.
.-. For maximum power output, load angle 8 = impedance angle 0
From Eq. (5.32), maximum power output from a generator is *’
P _ B fV , V f
°g <max) Z r° ...(5.36)
and from Eq. (5.34), maximum power out (or maximum power developed) in case of a motor is
F .M S’?
P . E fV<
(max) - ~ £ 2 ra
...(5.37)

fro l F i ^ r ^ j h f r ^ T e t r t t V r ^ f f r o m 5T 4 H T “ “ “ be °btai- d
8=0. = 90° - a.. These are drawn in Fig. ^ 4 H o )T n d (fc). fc> 3 m ° t 0 r ' by m a k in «
(e2-e)

D A D A
pjg g _(®) (fc)
■ •rhasor diagrams under maximum power output conditions for (o) generator and (6) motor.

>canned oy cam scanner


584 E lectrical M achinery _____________________

In Fig. 5.41 (a) for alternator, with line AB extended,


[CBA = k —0Z- 0
and in A ABC, [BCA + {CBA + 0Z= 71
... /BCA = n - 9, - / CBA = 7C—fl, —(tt —9, —6)
.v /BCA = 0, pf angle.
For synchronous motor in Fig. 5.41 (b), with line AB extended,
/ CBA = tc - 9, - (8 - 9)
In A ABC, /BCA + / CBA + 02 = n
... /BCA = 7i - 0, - /CBA = rc - 02 - (7i - 6, - 5 + 0)
/BCA = 5 - 9 = 9, - 9.
Now draw BD perpendicular to AC in Fig. 5.41 (a) and (6 ). Then pf at which alternator is
operating under maximum power output conditions is given by
_ ____ CD C A -A D
cos 0 = cos /BCA = —
Ef - y , cos e, B /Z .- y .r „
/„ z . Ic z ] •'•<5,38)
In case 0r is known, the motor pf under maximum power output conditions can be obtained
from the relation
V. - E f cos 0.
cos /BCA = -----------------
la
V, Zt - Ef ra ,
or cos (0, - 0) = / ...(5.39)

Fig. 5.41 (a) and (6 ) also gives


Ia Z, = 'lEf + Vf - 2 E f V, cos 0, ...(5.40)
In case Z, is known, the magnitude of armature current under maximum power output
conditions can be obtained from Eq. (5.40).
(6 ) M axim um pow er input. With constant Ef, Vt and Zs, the condition for maximum
power input to generator or motor can be obtained by differentiating Eqs. (5.31) or (5.33) with
respect to 8.
d P ^ d P ^ EjfV,
" d 6 dbs ~ Zs c o s (5 - a ’) - °
or 5 = 90 + a z = 8Z+ 2a* = 180 - 0Z.
For maximum power input,
load angle, 5 = 90 + a 2 = 180 - impedance angle 0Z.
Maximum power input to generator, from Eq. (5.31), is

p. (5.41) ,
■* ig (max) - y 72 a I
s As
and maximum power input to motor, from Eq. (5.33), is

xP.im(nuu) -~ M
^ + ^ ra
s s

Scanned by Cam bcanner


Polyphase Synchronous M achines 585

hasor diagrams, under the conditions of maximum power input, can be obtained from
! P . . for
^ oo(b) <•___an alforn atn r or
alternator or from
from Fip.
Fig. 5.3d
5.34 frl
(c) fnr
for qa mntnr
motor, Ktr
by molrmn
making R
6— 90 j.+ n
= QO a,. TThese
h e se are
are
\o. 5-2* yu> _ in n„A (h\
Fig' D‘ n Fie 5.42 (a) and (6 )
gbo 1.6
1q rOs
I
// M\
TI I ‘
/ 1
A s > /J \°*
1 I
i 1
QS 1 J i
i
S '/ i

. (b)
pig, 5.42. Phasor diagram s under maximum power input conditions for (a) generator and (b) motor.

In Fig. 5 42 (fl) ^or alternator, with line AB extended,


/C B A = 7t - 02 - 6
. /B C A =re—5 —/CBA = T t - 8 - 7 t + e2+ 0 = - 8 + 02+ 8
But 5 = 02 + 2 <x2
... /B C A = - 02 - 2 a 2 + 02 + 0 = 0 - 2a2

F o r m o to r , f r o m F i g . 5 . 4 2 (b ),
[C B A = 7 t - 0 2 - 8 + 0
... /BCA = 7t - 8 - /CBA = 7 t - 8 - 7 t + 8 , + 8 - 6 = 6, - 8.

The A ABC in Fig. 5.42 (a) and (6 ) can be analysed for determining the performance of
sy n ch ro n o u s machine under maximum power input conditions.
From an examination of the phasor diagrams of Fig. 5.23 for an alternator, it is seen that
when
(j) {Ef cos 8 - Iara cos 0) > V„ pf is lagging'
(ti) (Ef cos 5 - Ia ra) = Vt, pf is unity ...(5.43a)
(iii) (Ef cos 5 - Ia ra cos 0) < pf is leading
Further, phasor diagrams of Fig. 5.34 for a synchronous motor reveal that when
(i) {Ef cos 5 + I ara cos 0) > Vt, pf is leading
{ii) (Ef cos 8 + I ara) = Vt, pf is unity - ...(5.43b)
(Hi) (Ef cos 8 + Iara cos 0) < Vt, pf is lagging
5.8.2. R e a ctiv e P o w e r. So far as the reactive power flowin a synchronous machine is
concerned, reactive power is of interest only at the output terminals of a generator and at the
Jnput terminals of a motor.
Reactive power at the generator output terminals, from Fig. 5.40, is
Qog = E 2 (component of I in quadrature lagging E 2)

= V, 'E ■ (~0Z- 5)
—l1- sin - - v ‘ sin 02'

Scanned by CamScanner
586 E lectrical M achinery jArt. S.fj
VE V?
= sin [90 - (cq + 8)] - sin 0Z
"S S
V, E f c ^ v?
— —^ — COS (6 + C^) — 2 s •(5.44 a)
A*

Output terminals for the generator are the same as the input terminals for-the motor.
Qim = - Qog with 8 replaced by ( - 8)
Vt E, Vf
— cos ( 8 + cq) + n Xs
"5 A

= - f ^ X - ^ c o s (8 —cq) -.(5.44 b)
z r s z
For a generator, the condition for maximum reactive power is obtained from Eq. (5.44 a).

ig s = ^ s i n ( 8 + a j) = 0

or 8 = - a, or 8 + a, = 0
V tE f Vf Vt
a Og (max)
Zs Z] s zs
Ef - \ X ,

Vt
= ± [ Bf - V t ]ifr. « 0

When is maximum, 8 = 0Z= 90 - cq, the reactive power under this condition is given by

Vf v
Qog - - -~2

For a motor, the condition for maximum reactive power is obtained from Eq. (5.44 6).
dQim Vt E f
- ^ = - - ^ 8 . 1 ( 8 - 0 , 1 = 0

or 8 = cq or 5 - oq = 0

Qim (max)
=y±x _Y&=Yi
z] s Zs zs
Vt
= - ^ [ V , - E /] i f r a = 0

When Pim is maximum, 8 = 90 + a z, the reactive power under this condition is

V2
O - —
„2
Xs

G e n e ra tin g m od e. If ra = 0, then from Eq. (5.44 a),

„ VtEf ' Vf

...( 5 .4 ^
= Y (E f cos 5 - Vt)

^ c a n n e c rtT ^ ^ ^ c a fln e r
Polyphase Synchronous M achines 587

J ' L .............................. ' ...... .......................... ' ""


”” orator, positive values of active and reactive powers indicate their flow out of the
For a flow of reactive power in a generator for different excitations is as under :
gofl°rBt°r £ Co8 6 > V( (overexcited), Qog is positive and therefore reactive power flow is out-
(j) When ^ator delivers reactive power just as it delivers active power. Under this condi-
w»rd’ i-e- ia
prd, [V ^ to r operates
ooerates at a lagging pf.
ti0n’ When cos 6 = Vt (normally excited), Qog = 0. The generator neither delivers nor ab-
V
,
ft
( ^ ' . p0Wer. Under this condition, synchronous generator pf is unity.
0°rb8. rC W h e n £ / cos 8 < Vt (under excited), Qog is negative, therefore reactive power flow is
^ ■ generator is absorbing reactive power from the infinite bus. Under this condition,
inWftrator operates at a leading pf.
°'tern the relations derived above, it can be stated for an alternator that
under the conditions of maximum reactive power output, Qog{mun 18 negative. This i
^ ^hat alternator absorbs reactive power under this condition and
me8/n9i mder the condition of maximum active power output when 8 = 0Z= 90 - a z, reactive
o u tp u t Qog is negative. This means that alternator absorbs reactive power under this \
I

COndMo°toring M ode. If r„ = 0, then from Eq. (5.44 b),


V ,E f . V?
o = ---- ^ cos 5 +
xs xs
=^Vt-{V t - E f cos 8) - (5 4 6 )
X
rt tnntnr nositive values of active and reactive powers indicate their flow into the motor.
For am >P . , ,, 0f reactive power in a motor is as under :
F or different excitation negative and therefore reactive power flow is I
(i) When Ef cos 8 > V, (overexcited), g ^ ^ ^ ^ ^ motor is
outward, i.e. motor_i£I
motor is acting
act ng as a g ^ fa
—operating
„ a^ g at a leading pf.
pf.
delivering reactive power o neither delivering nor ab-
« , When E f cos 6 = V, (norma l y mot or operates at unity pf.
sorbing reactive power. Un er l • itiye and reactive power flow is inward,
(iii) When E f cos 8 < V, (under excite ), f U active power. Under this condition,
ie. motor is absorbing reactive power ju st as it is absormng
synchronous motor pf is lagging. „„„ stated for a synchronous motor th at
From the relations already derive , l . ut tj,e reactive power Q'im (max) 1S
(i) under the condition of power and
positive, the synchronous motor is there ore a ^ reactive power Q,mis positive. This
(it) under the condition of m axim um ac iv ^ ^ condition also,
means that synchronous motor absor s reac 1 maximum values of active or reactive
It can therefore be stated th a t under tne c f ,.
------------------ «x/»V\inp always
•ower, a synchronous m achine always absorbs
absorbs reatw
re f hronous m achine delivers delivers
reac- reac-
Further, it can be stated , in general, p i e r . Under norm al e x c . f t * * U
to power w h ereas a n u n d erex cited one a b
■either absorbs n or d eliv ers reactive pow er. variation of active and reac ive

ovvers can be plotted in case values ot Ef, i

Scanned by CamScanner
5

lA rt- 5.8
588 E lectrical M ach in ery

A ctive power
I

Load a n g le
(ra d .)

G enerator

Stable region J
p f lead fo r motorl . Synch. m/c(overexcited)
and lag for A lt. | R eactive power(p.M)
iD e liv e rs Q

N orm ally
e xcite d

-ir
-t “ 7 Load angle
Unity pf (ra d .)
fo r both

p f lag fo r motor S ,n c h r t c ( u n g r - d )
a nd lead fo r Alt. A b so rb s Q

Fig. 5.43. Active and reactive power versu s load angle ch a ra cte ristics.

suming V, = 1.0, Xs = 1.25, ra = 0 and Ef = 2.0, the active power versus load angle curves are
plotted in Fig. 5.43 with the help of Eq. (5.35). For generator, the active, or synchronous, power
P is shown positive and for synchronous motor as negative.
S y n ch ro n o u s g e n e ra to r. For generator, the reactive power can be obtained from Eq.
(5.45) as under : t

Qcs = 1 2 5 (2 “ S 8 _ 11 = 0,8 (2 “ S 5 " X)


For 6 = 0°, Qog = 0.8. As this reactive power is positive, the alternator delivers this reactive
power and operates at a lagging pf.
For 8 = n, Qog = - 2.4. As this reactive power is negative, the alternator absorbs reactive
power and operates at a leading pf.
For 5 = 60°, Qog = 0. Therefore, alternator neither delivers nor absorbs reactive power and
it operates at unity pf.
S y n ch ro n o u s m o to r. For motor, from Eq. (5.46),

Qim = ( 1 - 2 cos 5) = 0.8 ( 1 - 2 cos 8 )

For 5 = 0°, Qim = - 0.8. As reactive power is negative, the motor delivers this reactive po'ver
to infinite bus and operates at a leading pf. As a result, this reactive power is plotted above t e
reference line in Fig. 5.43 indicating th at motor is delivering this reactive power.

Scanned by CamScanner
1
I ; •’
Polyphase Synchronous Machines 589 1 ;*•;rl-/*-
\t .>

For 5 = *, Q,j» t e C f t 18 Pf°SHiVe' the 8^ chr'>"'>“ ™*-or absorbs this ,v'\


reactiva power. It to therefore plotted below the reference line. The motor operates at a lagging v-; i

Pf> . . . 5 = 60°, Qim = °- The synchronous motor neither d e liv e r. „u ,


■t operates at unity power factor. 1 6rS n0r absDrbs react,ve P°wer

“^ E x a m p l e 5
.3A 400V ^sloVhs'ZT^synchronous motor runs at rated voltage
* d; ! ! hJ l ^ T i r o n fir J m ’ S S ia - is 0 5 +j4 n and
U p l a n d efficiency for (a)maximum potueroutput and
S o lu tio n . Here Vt = 400 V ,E f = 510 V
Zs = Vo.52 + 4^= 4.031
0Z= tan (4/0.5) = 82.87°, az = 90 - 02 = 7.13°
(a) For maximum power output in case of motor, refer to Fig, 5.41 (6 ) From Eq (5 37 ) in
maximum power output is s i ; , rrom aq. u>.cw j, :|

„ E fVt 5? _ 510x400 510


Zs -r2 a —
Zy a ~ 4.031 x 0.5
4.031

= 42604.2 watts per phase
Shaft power output ll
= P„ F.W. and iron losses
t (max) ~ if
h;
= 42604.2 x 3 - 900 = 126912.6 W
.2 . Jnm2
From Eq. (5.40), Ia Zs = [5102 + 4002 - 2 x 510 x 400 x cos 82.87 ]1/2
or Ia = 150.79 A and line current = V3 x 150.79 = 261.17 A
i
From Eq. (5.39), 1

cos (9, - 9) = = ^ - l 4; 1!,!.1 : . 5! 0. ? - 5 = 0.554 1U’:l


•*a^s (150.79) (4.031)2
or 0Z- 0 = 56.36° or 6 = 26.51° Hi

pf = cos 0 = cos 26.51°= 0.895 lag I


Check. Power input to motor is given by II
V3 x 400 x 261.17 x cos 0 = 42604.2 x 3 + (150.79)2 x 0.5 x 3 = 161919.03 W llit
or cos 0 = 0.895 lag

Efficiency = * = = 78.38%.
J input 161919.03 |
(b) For maximum power input, refer to Fig. 5.42 (b ).
From Eq. (5.42), maximum power input to motor is

510 x 400 400


P.ini (max) x 0.5
4.031 4.031
= 55531.2 watts per phase
From Fig. 5.42 (b)
1/2
I aZ s = [5102 + 4002 - 2 x 510 x 400 x cos 97.13°]
or
/„ = 170.21 A per phase
Lin16 c u rre n t = 1 7 0 .2 1 x ^ 3 = 294.8 A

Icanned by CamScanner
if!
♦ji?
590 E lectrical M achinery ____________ _________________________________ _ -—
-•'•M
Shaft power output = 55531.2 x 3 - (170.21)2 x 0.5 x 3 - 900 = 122236.43 W
From Fig. 5.42 ( 6),
Vt + Et cos 0r 4 0 0 + 5 1 0 cos82j Z _ 0.675
ill! cos (02 —0 ) = ----- - ( 170 .21) (4.031)
; - ... (02 - 6) = 47.53° or 9 = 35.34°
... pf = cos 0 = cos 35.34° = 0.816 lag
Check. Power input = 400 x 170.21 x cos 0 = 55531.2
or cos 0 = 0.816 lag
Efficiency
ranciency = 55531
122236-
2 x^ 3- x 100 = 73.37%.
Exam ple 5 4 A 3300 V, star-connected synchronous motor has synchronous impedance of
0.4 + j5 “ p 'r PL , For an « citation e.m.f. o f 4000 V an d m otor input pow er o f 1000 kW at
rated voltage, compute the line current and pf.
Solution. Here Vt = 3300/^3 = 1905.3 V,
Ef = 4000/V3 = 2309.5 V
Zs = V0.42 + 5 2 = 5.016,
az = tan- 1 (0.4/5) = 4.57°
Per phase power input, from Eq. (5.33), is given by
1000,000 1905.3 x 2309.5 . - _ . 1905.3 \2
x 0.4
3 = 5.016 S ( z) 5.016
V,
or sin ( 5 - oc*) = 0.314
or 5 = 18.31 + 4.57 = 22.88°
It is seen from Fig. 5.34 (c) that
IaZs = [1905.32 + 2309.52 - 2 x 1905.3 x 2309.5 x cos 22.88 °]1/2
Ia = 184.43 A
Now 3 V Ja cos 0 = 1000,000 W
„ 1000,000
cos 0 = o— x VoTTo
3 x 1905.3o—
long 184.43 - 0-9486
Here Ef cos 8 + Ia ra cos 0 = 2309.5 x cos 22.88° + 184.43 x 0.4 x 0.9486
= 2197.85 > V, (= 1905.3).
Therefore, motor is operating at a pf, cos 0 = 0.9486 leading.
E x a m p le 5 .5 . A 2 3 0 V, 4 p o le , 50 H z, s t a r - c o n n e c te d s y n c h r o n o u s m otor has
ra +jXs = 0.6 +j3.0 H p er phase. Its field current is so ad ju sted that m otor draw s 10 A at upf
from rated voltage source. Now with the field current unchanged, the loa d on the motor is in ­
creased till it draw s 40 A from the supply. F in d the torque developed a n d the new pf.
230
Solution. Here V, = ^ = 132.8 V,

Z = Vo.62 + 3 2 = 3 .0 6 ft
a 2 = tan -1 (0 .6 /3 )= 11.31c
From the synchronous motor phasor diagram at upf, Fig. 5.44 (a),
V W - W 2 + ( « ) 2] ^
= [(132.8 - 6)2 + (10 x 3 )2]1/2 = 130.3 V

Scanned by CamScanner
-fg jy g h a s e S y n c h ro n o u s M a ch in e s 591

LOCUS OF
=fAS LOAD
, IS
6 = y / INCREASED

(a)

g' 5‘44- Pertaining to Example 5 5


When load torque is increased with E f held fixpH w r n ,
shown in Fig. 5.44 ( 6 ) and the motor begins ODeraH™ I i load angle 5 “ t e a s e s as
seen from this phasor diagram th a t ? ^ * 3 lagging Pf / a = 40 A. It is also
- (Ef + Vf — 2Ef Vt cos 8 ) 1 /2
or
or
Power input from Eq. (5.33) is

p _ 130.3 x 132.8 .
3.06 Sln ( 5 5 . 4 4 - 11.3i) + f 132.8 x 0.6
3.06
= 5 067.48 W = V ,/a cos 9
pf = cos 9 = 5067.48
132.8 x 40 = 0.954 lag -
Power developed = 3 (P r . [5067.48 - 40* x 0.6) x 3 = 12322.44 W
Now < o . n = />, H e r e = M O = 1500rRm _

.'. Gross torque, T ,= = 78 45 Nm


2 ti x 1500 ......... .........
V e n o u s motor at constant
Power is 1000 kW,, the PI
p f is.
is 0 8 leadingf i n d the p f when input is increased to
u.a leaning,

e i .. fifino ' H E .S ., 1984)


Solution. Here V, = = 3 8 1 0 .6 V,

pf = 0.8 leading
j _ 1000,000
0 V3 x 6 6 0 0 x 0.8 " 1 0 9 3 5 A
zs= V l.52 + 122 = 12.09
^ clz = tan (1 .5 /1 2 ) = 7.13°
e Phasor diagram for a leading pf load is shown in Fig. 5.34 (c) from which it is seen that
f y = l(Vt c°s 9 - I araf + (V, sin 0 + Ia JJQ2] 1/2
= [(3810.6 x 0.8 - 109.35 x 1.5 )2 + (3810.6 x 0.6 + 109.35 x 12 )2]1/2
= 4 6 1 1 .9 1 V

Scanned by CamScanner
_ J A t l si t
592 E le ctrica l M a c h i n e r y ' -
7"! TITTi=nn kW load angle 5 must increase. From Eq. (5.33),
When the input is increased to 1500 k W .loa B
i^nninno 4611.91 x 3810,1 w.p { b _ az) + ' 3 8 ! 0 £ T x 1 .5
12.09
3 = 12.09
Its simplification gives
5 = 21.1°
Fig. 5.34 (c) also shows that
/„ = i IE? + V® - 2 V, c°s 6!1

or ;„ = (4611 .912 + 3810.62 - 2 x 4611.91 x 3810.6 x cos 21.1°)

= 143.23 A
Now, input power per phase
= 3810.6 x 143.23 x cos 0 = 500,000 W
or cos 0 = 0.9161
As Ef cos 5 + Iara = 4611.91 x cos 21.1° + 143.23 x 1.5 x 0.9161
= 4808.73 > Vt (= 3810.6 V), therefore
synchronous motor is operating at a pf = 0.9161 leading.
Exam ple 5.7. A 3-phase, 6-pole, 2.3 kV, 200 kVA, star-connected synchronous motor has
synchronous reactance o f 12 H per phase and negligible resistance. The m otor is initially operat­
ing at a load o f 120 kW with the field current adjusted such that the arm ature current is mini­
mum. The field current is now increased such that the arm ature current is increased by 50%.
With this field current, the load is reduced to 60 kW. Calculate the new values o f armature
current and power factor. (.GATE, 1988)
2300
Solution. V, = = 1327.95 V
Minimum armature current corresponds to unity p.f.
.*. 3V, Ia cos 0 = 120,000 W

or = 30.122 A
0 3 x 1327.95 x 1
The phasor diagram under upf is as shown in Fig. 5.45 (a). With the increase in field cur­
rent, Ef rises to En and Ia to 7al such that 7ol = 1.5 70 = 1.5 x 30.122 = 45.183 A as shown. Under
this condition, pf cos 0j is given by
7al cos 0! = Ia

or co sO ^ — = 0.667
1.0
The magnitude of En can be obtained from Fig. 5.45 (a) as
En = [(Vt cos 0O2 + ( Vt sin 0! + 7al X f ) l/2
= 1(1327.95 x 0.667)2 + (1327.95 x 0.745 + 45 183 x 12)2]1/2
= 1769.21 V
When load is reduced to 60 kW with E ^ remaining constant, the phasor diagram is as
shown in Fig. 5.45 ( b ). Excitation emf E^ shifts to new orientation OC and load angle reduces
from to 52. The current is seen to become Ia2 and load angle 52.

Now sin 62 = w
a* 3
or 62 = 5.863°

Scanned by CamScanner
p o ly p h ase Synchronous M achines 593

L OCUS OF

Fig. 5.4 5 . Pertaining to Example 5.7. (6)


From AOAC, Ia2 Xs = [1769.212 + 1327 9 5 2 _ 2 x i7 « a o t ™
„r 7a2 = 39.02 A 2 x 1769 2 * 1327.95 x cos 5.863]>'2
Also 3Vt I a cos 02 = 60,000

C0S 02 " 3 x 1327.95 x 39.02 = 0,386 lead-

o f t z z ™ kh z i: p arar m th i~ i ^
resistance o f 1%, delivers a pow er output in kW equal to y ^ ncf ronousJ eact™ ce o f 50% an d a
unit equals 1.2 times the term inal voltage fin d out t L r ™ A Ifth e emf ° f t h i s
operating. ^ n n d ° Ut the Power f ^ o r at which the machine is
j (I. A. S., 1989)
Solution. It is given that ^ . 1% . and 1 £ . 5Q% = 0 g0 pjg ^ (fl) shows ^

E f = (V i c o s 9 + h r f + (V , s i n 9 + / a X f

f l r \2
cos 0 + 00 • a Ia X /
-V ? sin 0 + ° s
.V Vt Vt
Here
= 0 .7 I nud.
( 1.2 V,)2 = Vf[(cos 9 + 0.01 x 0.7 )2 + (sin 0 + 0.5 x 0 .7)2]
or 1.44 = 0.014 cos 0 + 0.7 sin 0 + 1.12255
0.014 cos 0 + 0.7 sin 0 = 1.44 - 1.12255 = 0.31745
M )
lt “ known ^ a t (A cos 0 + 5 sin 0) = sin 0 + tan ' 1 —
B
^ can be written as
0r 0.70014 sin (0 + 1.14577°) = 0.31745
or sin (0 + 1 .1 4 5 7 7 °) = 0.4535
pow f 0 = 25.824°
actor = cos 25.824° = 0.900 lagging.

Scanned by CamScanner
[Art.
594 E le ctrica l M ach in ery

/ x a o l ann v inn hVA star-con n ected syn chron ou s m achin e is used


Exam p le 5.9. (o) A 3-phase, 400 V, 100 k V A , s t a r e 4 ( m w T he ^
as a motor. The stray loss o f the m achine, C alculate the p f a n d efficiency when lh,
impedance p er p hase o f the m achine ,s 0 13 4 j l . 3 9 U ^ ^ ^ ^ ^ *
excitation is so adjusted a s to take a line cu <7 otb at under an d over excitation ans
delivered constant at 75 k W .D eterm ine the ex ed a tw n .em fsb o th at „0
the respective lo a d angles. Give the p h a so r d iag ram s, w n a i w in h mal
excitation ? „ . . . ■
(b)For the motor in part (a), calculate the load angle “
output and the value o f the possible m axim um output when un der excite . ( . .S., 1987)
Solution, (a) Line current = rated current/phase
_ f f l 0 Q-9- = 144.342 A
“ V3 x 400

Per-phase voltage, Vph = ^ = 230.95 V .


Stray losses = 4000 W, Power delivered = 75 kW
Power developed = 75000 + 4000 = 79000 W
O h m ic losses in armature = 3J 2 ra = 3 (144.342) x 0.13 = 8125.5 W
Total losses in motor = 4000 + 8125.5 = 12125.5 W
Power input = 75000 + 12125.5 = 87125.5 W
Also, ViT Vt Ia cos 8 = Power input
87125.5
Power factor = cos 0 = = 0.8713
100,000
or 0 = 29.39°
losses 12125.5
Efficiency = 1 - 1 - x 100 = 86.083%
input 87125.5
V. ✓ \ /

For Fig. 5.34 (a), Vt cos 0 = 230.95 x 0.8713 = 201.23 V


V, sin 0 = 230.95 x 0.491 = 113.396 V
Ia ra = 18.764 V and Ia Xs = 144.342 x 1.3 = 187.64 V
As IaXs > Vt sin 0, the phasor diagram for lagging pf is as shown in Fig. 5.46. In this figure,
Ef can be obtained from A ABC.
E f = (V, cos 0 - I a raf + {IJC, - V, sin 0 )2
= (201.23 - 18.764)2 + (187.64 - 113.396)2
= (182.466)2 + (74.244)2
or E f = 196.9924 = 197.0 V
B C 74.244
tan (6 - 0) =
AB 182.46
or 6 = 2 2 .1 4 + 29.39° = 51.53°
For leading pf load, it is seen from Fig. 5.34 (c) th at

E f = (182.466)2 + (187.64 + 113.396)2


= (182.466)2 + (301.036)2
or Ef = 352.02 V.
f .c. n. 301.036 „
tan (5 + 0 ) = Yg 2.466 or 8 = 58.78° - 29.39° = 29.39° F ig . 5 .4 6 . Pertaining to
E x a m p le 5.9.

Scanned by CamScanner
Polyphase Synchronous M achines 595

'^ ^ \ j r r e n t at normal excitation


IIXPU _ S T l^ S , A at unity pf.
"7^ x400x 1
For maximum power output,
(b)
- I ra _ 1 0.13
5 = 0Z= COS
'
y£js = COS
V0.132 + 1.3Z
. Load angle 8 = 84.23°
From Eq- (5-37)» maximum Power outPut 1S give11
Ef Vt gf
om
= 2S Z\ *
/ — \2
197.0 x 230.95 197
x 0.13
1.3065 1.3065
= 31866.5 W/phase = 95599.5 Watts (total).
Example 5.10. A 500 kVA, 3-phase, 6.6 kV, star-connected synchronous m otor has
synchronous reactance ofO +j20 Cl per phase.

(a) For a load angle o f 10°, the motor takes rated current. Find the excitation emfs both at
lagging and leading pfs.
(b) Find the mechanical power developed and p f in part (a).
(c) Find the minimum excitation voltage for delivering 200 kW at rated voltage without
falling out o f step.
Solution. Terminal voltage,

Vt = = 3810.6 V

Rated armature current,

L = J ° -°— = 43.74 A
° v3 x 6.6
(a) Phasor diagram in Fig. 5.47 is drawn with Ia Xs same for
both leading and lagging pfs. It is seen from this figure that
(Ja Xt)2 = E f + V f - 2EfVi cos 8
(43.74 x 20 )2 = E f + 3810.62
- 2EfX 3810.6 cos 10P
0r E ) - 7 5 0 5 .4 2 ^ + 13755397 = 0 Fig. 5.47. Pertaining to
Exam ple 5.10.
Its solution gives E f - 3180.5 V and 4324.9 V

Excitation emfs are En = 3180.5 V per phase for lagging pf and Eft = 4324.9 V per phase
R eading pf. Their corresponding line values are En = <3 x 3180.5 = 5508.63 V and En
s W x 4324.9 = 7490.73 V.
(b) For lagging pf, the mechanical power developed is
3810.6 x 3180.5 gin 1Q0 = 315682.3 W
20

Scanned by Cam bcanner


I Art.
596 E lectrical M ach in ery

3 1 5 0 8 2 ,3 --------- , 0 .6 3 i 4 l a g
Power factor \f,'l y OGOO x 43.74
For leading pf, the mechanical power developed is
p = 3 3810.6 x 43 2 4 3 sin 10° = 429270.4 W

_ ^29270^4------- = o 8 r) 8 5 |c a ( i
Power factor
\3x6G 00 x 43.74
(c) The synchronous motor will fall out of step when 6 - 90 .
E/X 3810.6 .
200,000 = 3 sin 90’
20
or Ef - 349.90 V per phase or 606 V (line).
Exam ple 5.11. A 3-phase, 415 V, 6-polc, 50 Hz, star-connected synchronous motor has an
em f o f 520 V (L - L). The stotor winding has a synchronous reactance o f 2 ohm s / phase, and the
motor develops a torque of220N m . The motor is operating o ff 415 V, 50 Hz bus.
(a) Calculate the current drawn from the supply and its power factor.
(b) Draw the phasor diagram showing all the relevant quantities. (GATE, 1992)
415
Solution. Per phase V, = = 239.6 V
520
Per phase Ef = = 300.2 V

„ , , 4 nf 4 n x 5 0 lOOn ,.
Synchronous speed, o)4. = -jf- ---- = —- — rad/s

Ef V, .
3
Torque — --------
sin 8
. — I c -1

A's
™ 3 520x415 . P
or 220 = —~ " •------ sin 6
100n
Its solution gives 5 = 12.33°.
From the phasor diagram of Fig. 5.48, it is seen that
( I J C f = E j + V f - 2Ef Vt-cos 8
E xam p le 5 11
1 tonA r>2 . non r.2
or Ia = - (300.22 + 230.92 - 2 x 300.2 x 239.6 x cos 12.33 °]1/2 = 41.8 A
Current drawn from the supply = 41.8 A
r- ErV,
Power = v3 V, x 41.8 cos 0 = sin 8
A.
n r 520 ,
cos 0 - p.f. - 2 x 41 § x V?T X 8,n ‘^° = 0.7669 lending.
E xam p le 5.12. A 3-phase, 50 llz, 415 V, synchronous m achin e operates at rated voltage an and
at a leading pow er factor o f 0.9. S haft pow er is 15 k\V an d the excitation e m f is 400 V l(p< r
p hase resistance is 0.5 ohm, fin d the synchronous reactance. N eglect m ech an ical losses oOf t f
system. ~ - —•% •»***
(GATE, I&V
S olution. Assuming the synchronous machine to he star connected.
400
V, - ~ j~ - 239.6 V, Ef tx = 230.95 V

S in n e d by (jam Scanner
-------------------------------------------------------Polyph ase S y n ch ro n o u s M a ch in e s 597

As Er leSS ^ an ^ anc* ^ *s leadin&> ^ must work as an alternator. For this operation,


or diagram of Fig. 5.23 (c) is applicable.
phaSp°0wer input = V3 x 415 x I a x 0.9 W
... Shaft power = >/3 x 415 x I a x 0.9 - 372 x 0.5 = 15000 W
72 - 431.27 Ia + 10,000 = 0
or . . .
Its solution gives I a - 24.585 A and 406.685 A. The higher value of armature current is not
-kIp • ■
feasible, I'o = 24.585 A.
From the phasor diagram,
E f = [(V, cos 0 + Iara)2 + (Vt sin 0 - Z ^ ) 2]
or 230.952 = (239.6 x 0.9 + 24.585 x 0.5 )2 + (239.6 x 0.436 - 24.585 x Xs)2
Its solution gives Xs = 2.736 ft.
Example 5.13. A 3-phase synchronous motor has 80% synchronous reactance an d negli­
gible resistance. When connected to bus bars at rated voltage and the excitation adju sted fo r an
e m f of 120%, the m achine draw s an input kVA o f 100%. Find the m echanical pow er delivered by

the motor, neglecting all losses. I f for the sam e load on the motor, the excitation is reduced to
gen era te an em f o f 100%, what would be the input kVA ? (I.A.S.,1994)
Solution. Three-phase problems can be solved in per unit by treating them as single-phase
problems.
Here Xs = 0.8 pu, ra = 0 , E f = 1.2 pu, Vt = 1.00 pu. For an input kVA of 100% at Vt = 1.00,
V, Ia = 1.00 and therefore Ia = 1.00 p.u.
As Ef= 1.2 pu is more that Vt = 1.00, synchronous motor is working at a leading pf. From
Fig. 5.34 (c),
E f = (Vt cos 0 - I a r J 2 + (V, sin 0 - I J C f
1 .22 = cos2 0 + (sin 0 - 0 .8)2
1.44 = cos2 0 + sin2 0 + 0.64 - 1.6 sin 0
0r sin 0 = f f = 0.125
1.6
Power factor, cos 0 = 0.9922 leading.
Mechanical power developed by the motor
= Vt Ia cos 0 = 1 x 1 x 0.9922 = 0.9922 pu.
When excitation emf is reduced to 1.00 pu for the same load, then
ErVt
sin 5 = 0.9922

lx l
~0 g- sin 5 = 0.9922 or 8 = 52.54°

Prom Fig. 5.34 (c), w ithra = 0,

( I J C f = E f + V f - 2Ef Vt cos 8
Qr (0.8 7a)2 = l 2 + l 2 - 2 x 1 x 1 x cos 52.54°
7a = 1.1065 p.u.
hput kVA =VtI a = 1 x 1.1065 = 1.1065 p.u. or 110.65% kVA.

Scanned by CamScanner
598 E le c tric a l M ach in ery

E xam p le 5.14. An alternator, with synchronous reactance o f 0.8 pu, is connected to on ^


finite bus at rated voltage. With its excitation em f adjusted to 1.3 pu, the alternator delivers an
output o f 0.5 pu. Neglect all losses.
(a) Determine the load angle, arm ature current in p er unit and the pow er factor o f the alter,
nator.
(b) The alternator is now m ade to operate at another value o f excitation em f which results in
the sam e values o f pow er output and arm ature current. Under these conditions, find the excita.
tion voltage, load angle and the pow er factor.
(c) O f the excitations in (a) and that obtained in (b), which is more likely to be used in prac.
tical situations an d why ? ■
Solution, (a) It is given th a tX s = 0.8 p.u., V, = 1.0, Ef - 1.3 p.u., P - 0.5 p.u.
Er Vj .
sin 5 - P Ef =!•3 p.u
X
- 1 0.5 x 0.8
5 = sin = 17.92c
1 x 1..3
It is seen from the phasor diagram of Fig. 5.49 that
{ I J C f = E f + V] - 2EfV, cos 5
1/2
or 4 = 7TF f1-32 + l 2 - 2 x 1.3 x 1 x cos 17.92°]
0.8
= 0.581 p.u.
Also, Vt Iu cos 0 = P
0.5
or pf = cos 0 = = 0.8606 lagging.
1 x 0 .5 8 1
(b) Another value of excitation E f is possible as shown in
Fig. 5.49. As l a, Vt and power are unchanged, the magnitude
of pf remains the same but it is now 0.8606 leading. Fig. 5.49
shows that
Fig. 5 .4 9 . Pertaining to
N\ E fi = (V t cos e)2+ (V t s i n 3~ 4X s) 2 Exam p le 5.14.

w = 0.86062 + (0.5093 - 0.581 x 0.8)2


or Efx = 0.86175 p.u.
- 1 0.5 x 0.8
Load angle, 8j = sin = 27.66°.
0.86175 x 1.0
(c) E f = 1.30 p.u. is more likely to be used in practical situations because the alternator, tn
addition to supplying the active power, must deliver reactive power to the system.
E x a m p le 5.15. A star-connected, 11 kV turbogenerator, with synchronous impedance of
1 + jlO Q p er p h a se is connected to an infinite bus at rated voltage. The alternator delivers on
arm ature current o f 100 A at unity pow er factor to the bus bar.
(a) With the altern ator output rem aining constant, the altern ator excitation is in crea sed )
15%. F in d the new values o f arm atu re current, loa d angle a n d pf.
(b) With the excitation o f (a), discuss how altern ator can be m ad e to operate at unit\ pf-
Under this condition, fin d the arm atu re current, lo a d angle a n d p ow er delivered to the bus.

S o lu tion . Per phase V, = = 6351 V

Scanned by CamScanner
i r
_ f o|yphase Synchronous M achines 599
rrbp phasor diagram a t unity pf is drawn in Fig 5 50 « •
Tbe P g- 0 '50* ^ gives the excitation emf
as
E f = (V, + Iara) + (Ia Xs)2 = (6351 +
100 x l)2 + (100 x 10)2
E f = 6 5 2 8 .0 5 V
or
Z , = Vl 1 + 10" = 10.0512, a , = tan - 1 f
= 5.71°
m 10
1 s_ 1 100 x 10 ' <
Load angle, 8 -ta n — j + — = 8 .8 P

prom Eq. (5.32), per phase power delivered to bus bar is


6528.05 x 6351
^ 10.05 sin (8.81 + 5.71°) - '6 3 5 1 '2 x 1 = 634947.65 W.
10.05
(a) New value of excitation emf, E n = 6 5 2 8 .0 5 x 1.15 = 7507.3 V. With this value of En
irmature current lags bus-bar voltage V, by an angle 8 „ Fig. 5.50. As alternator output is
onstant, the power delivered to bus remains unchanged. From Eq. (5.32),

7507.3 x 6351 6351 ^


634947.65 = sin (8 i + 5.71°) - xl
10.05 10.05
Its simplification gives load angle
8 j = 1 2 .5 9 ° - 5 .7 1 ° = 6 .88 °.

Also, (/alZ s)2 = 75 0 7 .3 2 + 63512 - 2 x 7507.3 x 6351 x cos 6 .88 °


1
or hi = (1422.5611) = 141.55 A

Power output of alternator = Power input to busbar


r a. ry 634947.65 , .
or power factor = cos 0 = x ^4 ^ 55 = 0 7063 lagging.

(c) By increasing the steam input to the turbine, the


turbogenerator can be made to work at unity power factor.
Voltage Vt remains fixed as it is an infinite bus voltage. For
more steam input, E ^ traces a locus as shown in Fig. 5.50
until unity pf is reached. It is seen that

E^ = (6351 + Iai x l )2 + (107o2)2


or 7507.32 = (6351 + / a2)2 + (l° /« 2)2
or 101 4 + 2 x 6351 I a2 + 63512 " 7507 32 = 0
or 722 + 125.76 Iai ~ 158657 = 0
Its solution gives arm ature current l ai = 3 4 °-3 A

Power delivered to infinite bus


= Vt I a cos 0 = 6 3 5 1 x 3 4 0 .3 x 1
= 2 161245.3 W per phase
= 6 4 8 3 7 3 5 .9 W for three-phases

canned by CamScanner
■ *'
Hi
. in1
600 Electrical Machinery

Exam ple 5 .1 6 . A synchronous machine is synchronized with an infinite bus at rat i


age. Now the steam input to prime mover is increased till synchronous machine starts on °°k
at rated kkVA.
CL t r a t ^ d V A
The machine
'T h o m n n h i rt o
has synchronous rimpedance
h n c o m r t o r l n n n o Zs
7.
= 0.02 -i-
— f) 0 0
+j0.8p.u.
i O Q r * •>
Deterin'^^
A ..

operating p f o f the alternator and its load angle. ^


Solution. For the working conditions described, the phasor diagram is shown in Fie c
At the time of synchronizing, E^= V, and 5 = 0. As the steam input is increased, Er is pu
ahead till rated kVA is delivered to bus by the alternator. The right-angled trianglerMft l ^
that sh°ws
Ofi2 = OA2 + AB2
E} = (Vt cos 0 + Iara)2 + (IJC, - Vt sin 0)2
2 2
L rA (IX N
12 = 1 2 cos 0 + ■- sin 0
Vt
1 = [(cos 0 + 0.02)2 + (0.8 - sin 0)2]
cos2 0 + 0.0004 + 0.04 cos 0 + 0.64 + sin2 0 - 1.6 sin 0 = 1
1.6 sin 0 - 0.04 cos 0 = 0.6404 „.(i)
/ •
But A sin 0 + B cos 0 = VA2 + B 2 sin 0 + ta n "1 —

From Eq. (i), we get


- i 0.04 0.6404
sin 0 - t a n 7—7-t -^ = 0.400125
1.6 Vl~6* + 0.04
or 0 = 23.586 + 1.432 = 25.01830 Fig. 5.51. Pertaining to
Example 5.16,
Power factor = cos 25.018° = 0.9062 leading

Zs = V0 .82 + 0.022 = 0.8002, ot, = tan~1 0.02 = 1.432c


0.8
Power output of generator, from Eq. (5.32), is

1x 1 \2
sin (5 + 1.432°) -
0.8002 / 10.8002 x 0.02 = VtIa cos 0
= 1 x 1 x cos 0 = 0.9062 p.u. ;
or 8 = 4 8 .1 7 1 - 1.432 = 46.74°
Load angle = 46.74°.

5 1 7* A 4'p ° l*> star:connected, 50 Hz, 11 kV, 40 MVA turbogenerator, withe


, , i reC!Cj ° l ' p 'u'>ls. c° nnected to a power network. This power network can be
Z 7T i u l!i! ?' f aSerieSreactance
° f J 0.5 a
A voltage regulator adjusts
the field current such that alternator terminal voltage rem ains constant at 11 kV.
The generator delivers an output o f 40 MVA.
(a) Draw the p h a sor diagram under the conditions specified above.
(b) F in d the arm ature current and the alternator pow er factor.
(c) F in d also the m agnitude o f excitation emf.

Solution, (a) The equivalent circuit for the conditions specified is shown in Fig. 5.52 («'
Phasor diagram is drawn in Fig. 5.52 (6) where V„ = per-phase bus bar voltage = V, = per P***

it alternator terminal voltage . A i g S = 6 3 5 1 V, excitation em f = Ef and C D = /„ X,. |

V;» i j •
i;. t: ■
Scanned by CamScanner
X f ij
Polyphase Synchronous M achines 601

Xs .2-42fl xe=0.5
— “Tnnnr— —
+
1q >

@ E f vb

(a)
Fig. 5.52. (o) Equivalent circuit and (6) phase, diasnu. U 7.
(b) Per phase arm ature current,
r 40,000
h = = 2 0 9 9 -52 A

X. in ohms = . X,i n pu Xf f = 0.8„ x x11 = 2 42 £1


l7n
0 . V3
V3 40,000
40 non w
Voltage drop A C = I a -Xe = 2099.52 x 0.5 = 1049.76 V

Power-factor angle, 0 = sin" 1 ^ = sin" 1 = 4.74

Alternator power factor


= cos 4.74° = 0.9966 lagging ! .

(c) It is seen from the phasor diagram that


E ) = OB + (BC + C D f
1049.76 . (i
= (6351 x 0.9966)2 + + 2099.52 x 2.42
if
1?-
or E f = 8454.9 V line to neutral
= 14643.9 V (line voltage).
Example 5.18. A 3 -p h ase star-connected 400 V synchronous motor takes a power input o f .. r
5472 watts a t ra ted voltage. Its synchronous reactance is 10 Cl per phase and resistance is neg­
ligible. I f its excitation voltage is ad ju sted equ al to the rated voltage o f 400 V, compute the load
angle, power fa c to r a n d the arm atu re current.
° c - iQxs
Solution. W ith Vt = E f, the synchronous motor phase diagram
is as shown in Fig. 5.53.

Here 230.95 V

E f Vt
Now sin 6 = P

400 X 400 s _ 5472


or — - — - r — sin 0 - „
3 x 10 3 Fig. 5.53. Pertaining to
or Example 5.18.
6 = 20c

Scanned by CamScanner
(A rt. 5.8
602 E lectrical M achinery

Thus the motor operates with a load angle of 20°.


Fig. 5.53 reveals that I a lags Vt and Vt = Ef and ra = 0, 0 = \ 5 = 10°.
.\ Motor pf, cos 0 = cos 10° = 0.9848 lag
It is also seen from Fig. 5.53 that ab = Vi sin 0
/. I a Xs = ac = a b + be = 2 a b = 2 V, sin 0
_ 2 x 230.95 x sin 10° _ 7 gg A
or
I a ~ 10
Exam ple 5.19. A 2000 V, 3-phase, star-connected cylindrical rotor synchronous motor with
synchronous reactance Xsm = 2 Cl is connected to a turbogenerator with Xsg = 3 £2, through trans­
mission line o f reactance Xtr = 1.5 Cl. The synchronous m otor is draw ing 100 A at rated terminal
voltage and at unity power factor at its terminals.
(а) Compute the alternator and motor excitation voltages.
(б) Find the power transfer between alternator an d motor.
(c) With the excitations fixed as obtained in part (a), fin d the m axim um pow er transfer be­
tween the two machines. Also fin d the arm ature current, term inal voltage o f the synchronous
motor and its pow er factor under these conditions.
Solution, (a) The equivalent circuit for this example is shown in Fig. 5.54 (a) and its phasor
diagram is drawn in Fig. 5.54 ( b ).
.

jiaXSg
XSg = 3fl Xsm =2fl

T . „ Vm iiaxtr

9m ^axsm

Ef„\
(a) (6)

Rated motor terminal voltage per phase


Tf 2000
m = ~ J g ~ = H 54 V
Ia = 100 A.
From phasor diagram of 5.54 (6),

Efm = ^Vrn + ( 4 Xsm f = V(1154)2 + (100 X 2)2 = 1170 V.


and
Efe = W t + UaiXsg+Xtr))2 = V(1154)iJ + (1 0 0 x 4.5)2 = 1240 V.
(■b) Angle between Vm and Efm is

= cos- 1
m 1154
= cos 1
E,fm 1170 = cos 1 (0.987) = 9=
Angle 82 between Vm and Efg is

m
52 = cos- 1 = cos- 1 f ^
E,fg 1240 = co s"1 (0.929) = 21.8°.
The power angle between Efg and Efl
Jfm I®
bgm = 21.8° + 9° = 30.8°.

Scanned by CamScanner
Polyph ase S y n ch ro n o u s M a ch in e s 603

tra n sfe rbetween alternator and motor


? 0 *CT p y
b fe x f'» j g
= AXsm + X, + X'■SB
+ A tr T '*■«. gm
.M g
=» , » 3 0 . 8 f9
6.5
/io
^ 112.1 kW per phase = 336.3 kW for three-phases. W /
/.) fo r maximum power transfer between alternator and X d

c h r o n o u s motor, the angle between Efg and Efm must be 90°, as ^ y J a xsm
1\
9J L n in Fig' 5,54 (c)‘ From this Phasor diagram. i* w CS. r
Sh0W 4 (Xam + Xtr + Xsg) = JE fm+ EJg 8 Efm
r ... V(1170)^ + (1240)^ Fig. 5.54. (c) Phasor diagram for
6^5-------- ^60 A. maximum power transfer
Example 5.19.
P f"1ErA' _ (1170) (1240)
max = A,

xr
6.5
- 223.0 kW per phase = 669.0 kW for 3-phases
E
From phasor diagram p = cos"1 'El
■fa (Xam+ Xlr + XSg)
^1170^
= cos,-1 = 46.2°.
1690
A B - h Xsm sin P = (260) (2) (0.721) = 375 V.
and B C = 4 Xsm cos P = (260) (2) (0.6925) = 360 V.
Vm = i(O B ? + (A B f = yl(Efm~BC)z + (A B f
= V(810)2 + (375)2 = 892.6 V line to neutral
= 1546 V line to line
Synchronous motor terminal voltage Vm can also be obtained from AOAC as under :

K = E}m+ - 2 Efm (IJC 'J cos 3


= 11702 + (2 x 260)2 - 2 x 1170 x 520 x cos 46.2°
= 892.8 V line to neutral
It is seen from Fig. 5.54 (c) that power-factor angle for synchronous motor is I D O A . In
A0AC, --------
AC (= / a Xsm) _ OA (= Vm)
sin IAOC sin P
260 x 2 _ 892.6
sin IAOC sin 46.2°
•' /AOC = 24.86°
In right-angled triangle DOC, /DOC = 90 - p.
Power factor angle /DOA = / DOC - IAOC
= (90 - P) - IAOC = 90 - 46.2 - 24.86° = 18.94°
••Synchronous motor power factor = cos I D O A
= cos 18.94° = 0.9459 leading

>cannea Dy Cam bcanner


604 E lectrical M a ch in ery ____________________ __________________________________________________________

The maximum power transfer of 669 kW represents the maximum load that can be put
the synchronous motor and alternator for the two machine system under consideration. A l0ai
greater than 669 kW would cause the synchronous motor to loose synchronism.
Exam ple 5.20. A 20 MVA, 3 p h a se star-connected altern ator with an im pedance o f 5 Qand
a resistance o f 0.5 Q, is operating in p a ra llel with co n stan t-v otlag e 11 kV bus bars. I f its f ^
current is adju sted to give an excitation voltage o f 12 kV, then calcu late .
(а ) the m axim um pow er output from the altern ator an d
(б) the arm atu re current an d pow er factor under m axim um p ow er conditions.
Solution, (a) Alternator terminal voltage per phase,

Vt = 11^ ° ° = 6350 V.

Alternator excitation voltage per phase, E f - ^ = 6930 V,

Maximum power output from the alternator, from Eq. (5.36) is given by

P - M Z L '
* og (max) 2 °

= (6930) (6350) _ .(6350)! x 1 = 6350 Q_


5 (5) 2 5
= 8 MW per phase = 24 MW for 3 phases. '
fa_ _ (b5
(6) cos02 = 7r = - ^ i = O.l
Z, 5

From Eq. (5.40), Ia Zs = + V f - 2 E f Vt cos Qz

= V(6930)* + (6350)2 - 2 ^ 6 9 3 0 7 (6350H 0.10) = 8920.


Ia = = 1784 A.

From Eq. (5.38), the power-factor under maximum power conditions is given by
E fZ . - V , r 6930 x 5 - 6350 x -
C°S 9 " =~ 1784 (5)2 = ° '7° 6 leading'
5.9. Circle Diagrams of Synchronous M achines
The steady state behaviour of a synchronous machine can easily be obtained from its circle
diagrams. These diagrams offer quick graphical solution to many synchronous m a c h in e
problems, though the results are a little less accurate from those obtained analytically.
In this article, the circle diagrams of cylindrical-rotor synchronous motor have only been
discussed.
5.9.1. Synchronous m otor circle diagram s. H ere th e excitatio n -circle and power
circle diagrams for a cylindrical-rotor synchronous motor are developed. With the help o f these
two circle diagrams, synchronous motor V-curves are obtained.
(a) The excitation circles. The excitation circle diagram gives the locus of armature cur­
rent / a, as the excitation voltage Ef and load angle 6 are varied. This circle diagram f°ra
synchronous motor is based on its voltage equation,

Vt = E f+ I a Z„

Scanned bv CamScanner
Polyph ase S y n ch ro n o u s M a ch in e s 605

...(5.47)

Vt _ Ef
rrent phasors = — (= OC) and (= OB) lag behind their corresponding voltage
The cu
angle 9Z and arm atu re cu rrent
by ”2 * , . 1-rr
phas°r htained by taking their difference
I -0A>1S ................• ............ 1- L- L-------
Eq <5 47^' N° te th at the angle betW8en
asPer
h is the power angle 8, as shown in
5 and
I
* c k Phasor CA is parallel to OB and in
figij, o.o° - 1
> g
C A - O B - -£• In F ig . 5 .5 3 , the
magnitude
orientation of phasor V, is deliberate so that

^becomes horizontal.
zs
Alternatively, the same result can also be
Fig. 5.55. Excitation-circle diagram ; illustrating
obtained as follows. W ith V, as reference the locus of arm ature current as E f and 5
phasor, Eq. (5.47) can be re-written as are varied in a synchronous motor.

Vt ZO Ef z ~ 5 _ Yi'i Ef o
1 = 7 = /2 - e ,- y z - s - 0 ,
z sze2 Zfc tz ljz
"V E V E
In expanded form, Ia = Y cos ez - y cos (5 + ez) + J - -=r sin 0Z+ -={■ sin (5 + 0Z)
s s As As
The magnitude of /* is
V, Ef v E
rt2 = — cos 0Z- y cos (8 + 02) - sin 0Z+ f f sin (8 + 0Z)
As s " s S

vt %
V E n
cos (8 + 0Z) cos 0Z+ sin (8 + 0Z) sin 0Z j
v y
( v ty (E f \ v t Ef ...(5.48)
ILa2 = • 2 ■ ■f cos 8
vz . y S

t Eq. (5.48) states that I a (= OA) is one side of a triangle (AOCA), whose other two sides,
ndude between them a variable angle 8 (= ZACO), the two sides being of magnitude.
y Ef A
Y (= o c ) and
both
°f which are of the nature of currents.

IfVr, . y
i ls assumed constant, f = OC
ZQ
18Co
For a fixed excitation voltage E , the extremities of phasors Ef/Z , and I„ follow the
« load is changed on the motor. This locus,_known as the “ ^ f t T a d s
he magnitude and power factor o f f and the load angle S, for different shaft loads.

canned by Cam scanner


606 E lectrical M achinery

For Ef > Vt, the armature current I a, for the same load angle 5, is equal to OF Fig. 5.55, an<}
it leads Vt. Thus for Ef <Vt>the motor operates at a lagging p.f. and for E f > „ the motor may

° PeNoteaLVuhem ^cifrnuni value of load angle ACO can be equal to 0Z, as proved in Art. 5.8.
(b) The pow er circles. A power circle gives the locus of the arm ature current 4 as
mechanical power developed and power-factor angle 0 are varied.
The power output per phase in case of a synchronous motor is
P - V t I a cos Q - I a ra -(5.49)
where Vt Ia cos 0 is the pei phase power input to asynchronous motor and P is the mechanical
power developed including both the iron and mechanical losses. In other words,
P = Shaft power + Iron and mechanical losses.
Eq. (5.49) can be re-written as
V, • P
4t2 - — cos 0 + — = 0 ...(5.50)
a ra a ra

or J 2 cos2 0 + 72 sin2 0 - — 7a cos 0 + — = 0 ...(5.51)


ra ” ra
Let x = Ia sin 0 and y = Ia cos 0. With this substitution, Eq. (5.51) becomes,

x2 + y2- — y + — = 0 -(5.52)
ra ra
Eq, (5.52) is the equation of a circle* with its
centre at
f TT \
0,
2 r„
Zero-power
r t t \2 circle
and radius A Vt
2r„
as shown in Fig. 5.56. The co-ordinates of any point
on the circle, such as point A, are {x,y) or
( 4 sin 0 , 4 cos 0). It is seen from Fig. 5.56 that ro

AO = V(4 sin 0)* + ( 4 cos 0)2 = I a and ACO A = 0.


T herefore a line join in g th e origin
O [x —0, y = 0] and any point on the power circle,
gives the armature current I a and its power-factor
angle 0 with Vt, as shown in Fig. 5.56.
Eq. (5.50) can be treated in an alternative man-
x = l Q s in 0
V,
ner also. Addition of to each side of Eq. (5.50) Fig. 5 .5 6 . Power-circle d iagram illustrating
2r„ the locus of arm ature current for constant
gives m echanical power developed.

X l ')
-2 - •4 cos 0 = (X l
2r. 2 r„ 2 r„

"The general equation of a circle is

* 2 + y 2 + 2gx + 2/y + c = 0,
with its centre a t a n d radius =

Scanned by CamScanner
Polyphase Synchronous M achines 607

2
This equation shows that f v 1] P is one side of a triangle whose other two sides are
w ” ra

JjL with angle 0 in between them, see Fig. 5.56. Note that ^ and V. are in the same
»»i 2t.' 2r°
phase-
,1
When P - °>the radlus °f the power circle = —
2ra
howing thereby that the zero-power circle passes through origin, marked O in Fig. 5.56. As the
S vver developed P , goes on increasing, the radius of the power circle goes on reducing and
^axiroum power would occur when the radius of power circle is zero, i.e. when
P max _ q
*

Pmax _
°r ra ~4%
V2
or rp max -= —
a
™a

Corresponding to the maximum power Pmax the power circle of zero radius is the point C
itself and the armature current is
Vt . .
ty ■in phase with Vt, i.e. power factor is unity.

v, v2
Maximum power input = V, /_ cos 0 = V, —- •1 = —
2 ra 2ra
V2
and maximum power output, Pmax = —L -

Efficiency at maximum output = 50%.


An efficiency of 50% is too low a value for a synchronous motor. At this efficiency, losses
would be about half of the input and temperature rise would be far above the permissible
temperature of the motor. As such, maximum power output of V f/4ra can never be obtained in
practice from a synchronous motor.
Eq. (5.50) when solved for Ia, for given values of power and power factor gives two currents,
w >ch are also indicated as OA and OB in the power-circle diagram of Fig. 5.56.
^ (c) V*curves. Figs. 5.55 and 5.56 show separately, the locus of armature current Ia as a
10n °f excitation voltage and power. These two circle diagrams may be superimposed as

u®trated in Fig. 5 .57 . Here OC' is taken equal to — and with C as centre, a zero-power circle

°f i*adi ^ *
US 2 *s drawn. Now make angle COD equal to 02 and join the point C with point D. The

^ g*e ODC' must be a right angle, since the point D lies on the circle with O C as its diameter.

°mnght-angled triangle ODC, it is seen that OD = O C cos 0t = y y = ^ Si» ce v t >s al°ng

Scanned by CamScanner
[Art. s.9
608 E lectrical M achinery

the Lne OCC', the centre of the ex­


citation circles must be at a distance
V
of (= OD) from O. In view of the

above, the point D on the zero power Excitation


circle must be the centre of the ex­ circles
citation circles.
For any specific power output P
and excitation Ef, the operating point is
found by the intersection of the cor­
responding circles. For instance, for the
excitation represented by the heavy
line, there are two values of armature
currents, l uo’ lagging Vt and Ia0 leading
Vtl for power P = 0. For the given ex­
citation Ef/Z s, these two values
Iao and Ia0' are plotted as shown in the
lower diagram of Fig. 5.57. in this man­
ner, the armature current values are
plotted for various values of excitations
for fixed powers. The various plotted
points so obtained are joined together
as illustrated in the lower diagram of
Fig. 5.57. Since the shape of the lower Field Ef
excitation Zj
portions of these curves resemble the
Fig. 5.57. Construction of V- and O-curves from
letter V, these are called “V-curves” of a
power and excitation circles.
synchronous motor. When con­
sidered as a whole, they are called
“O-curves.” .
The maximum theoretical excitation is
p
DD' = "iax)
Zs
and here the excitation circle is tangent to the zero-power circle.
But DD' = OC'
E,/ (max) Vt

or E f(m
t a x) - * z .

nr-n' o-mi-?lnIUnIuan maximum excitations for any power correspond to the points on line
. Similarly, the minimum and maximum values of arm ature currents for any power P,
correspond to the line O C C . Since the terminal voltage Vt is also along OCC’ the minimum and
maximum currents occur at unity power factor.

In Fig. 5.57, the curve A QB , i.e. H 2, joins all the points corresponding to minimum and
maximum excitations for the different powers considered. In Fig 5 58 (a) for any power P. tlJc
minimum and maximum excitations are DH and D H, and the corresponding armature currents
are OH and OHx respectively.

canned by Cam scanner


¥

\Vhen excitation is zero, the current, l u jB o n . m


-pig, 5.57, for P = 0. Now as the excitation is . m 8<58 (a) and O'A' in fK i
% (s - h min) with excitati™ equal to '"creased ,/0 first decreases to itaminim.,(Iia5![f.n'

pG Fig-5 58 (o) and when Y becomGS


ater than DG, /„ also starts increas-
a This explains for the bend in the
compounding curve H 2 for minimum
and maximum excitations in Fig. 5.57.
In rFig-
in ig- 5.58 (a),^ the current u OG
u is
*■ the* nnwpr nirrlo fnr.
ta n g e n t to the power circle for which
the radius is CG and the power factor
is found to be minimum. In general, it
may be stated th at minimum power to)
factor for any load-power, occurs when
the current line is tangent to the power
circle for that load. and minimum arm ature currents.

In Fig. 5.58 (8), D F is drawn normal to OC. When the excitation f i . equal to (= in

the lower diagram of Fig. 5JS7), / . = 0 (point A itself in Fig. 5.57) and'when excitation is equal
to its minimum excitat.on D F toss than O'A in Fig. 5.57, current is OF in Fig. 5.58 (6) I d

in Fig. 5.57. If the excitation ^ is made greater than DF, current is also more than O'F, Fig.

5.57. This explains for the bend in the unity power-factor curve ff, or AQB which has been
obtained by joining all the points pertaining to unity p.f.
The compounding curve for unity power factor H x i.e. AQB, corresponds to the line OCC’
and it will be observed th a t to the left of curve H lt p.f. is lagging and to the right of it, the p.f.
is leading. An exam ination of Fig. 5.57 also reveals that the compounding curve ff2, i.e. A'Q3'
corresponds to the line DCD'. The point Q in the lower diagram, corresponds to the point C in
the upper diagram of Fig. 5.57.
E
In Fig. 5.57, arm atu re current Ia versus excitation -=f have been plotted to obtain V-curves

°f a synchronous motor. If l a versus field current If, is to be plotted, then for each value of
A
calculate^. Now from OCC, find I f f o r each value of Ef and plot/a versus If to get synchronous
rootor V-curves— this is illustrated in Example 5.21.
Exam ple 5.21. A 1500 kW , 2 2 0 0 V, 3-phase, 50 Hz, star-connected cylindrical-rotor
synchronous m otor h a s a rm a tu re resistance o f 0.32 Cl per phase. Its open circuit data are given
mom;

15.00 20.00 25.00 30.00


—------ Field current. A 5.00 10.00
2140 2650 3040 3340
^^Pen-circuit terminal voltage, Ef 760 1500

Core loss - 60 kW

Scanned by CamScanner
610 K lcclrical M achinery

Friction an d w indage loss = 40 kW . .


Draw the V-curves at half-fu ll load, for the synchronous m otor running at rated voltage^
rated frequency. Plot also the variation o f p . f with field current.
Solution. For an exciting current of 15 A, the open-circuit per phase voltage is

Ef = = 1235 V.
V3
_ E f _ 1235 _
Synchronous impedance, Z = - x = ^ 7 f = 1 .6 4 8 a
he 750

Per phase terminal voltage V, = = 1269 V


\'3
1 1269
Now = 770 A.
Z. 1.648
The following table gives field current, per phase excitation voltage island E j/Z s

in A 5.00 1 0 .0 0 15.00 2 0 .0 0 2 5 .0 0 30.00


Ef, in V 438 866 1235 1529 1755 1929
Ef/Zs, in A 266 526 750 928 1065 1170

Vt 1269
Also = 1982 A.
2 r„ " 0.64
For half-lull load output of 750 kW, total mechanical power developer per phase is
750 + 60 + 40 850
P= kW
3 3
Radius of power circle corresponding to 8 5 0 /3 kW power
CM ______
i

V / ra

(1982)2 - -85Q,QQQ = 1748 A.


•V ? 3 x 0.32
Scale 100 A = 1 cm.
Two circles with CO = 1982 A and CG = 1748 A as radii
are drawn. Cut OD = Vt/Z s = 770 A and with D as centre
draw different curves with Ef/Z s as radii (given in table) so
that they intersect the power circle at a h a 2, .., a G, see Fig.
5.59. Now O a , is the armature current for an excitation circle
of radius D a , = Ef/Z , = 266 A. But this excitation D a , cor­
responds to a field current of 5 A. Thus one point on the V-
curve representingIa = Oal = 640 A and If = 5.00 A is located.
Similarly other points on the F-curve are obtained, tabulated
and plotted in Fig. 5.60. Further, for I{ =. 5.0 A, the pf is
cos tCoa^ ; for If = 10.0 A, the pf is cos /Coo., and so on.
Fig. 5 .5 9 . Pow er and excitation circle
d iag ram . Exam ple 5.21.

Scanned by CamScanner
F ie ld c u r r e n t, I f

Fig. 5.60. Variation of arm ature current and p.f. with field current, Exam ple 5.21.

5.00 1 0 .0 0 15.00 2 0 .0 0 2 5 .0 0 3 0 .0 0
If, »■"A
640 345 235 295 400 496
*« A

0.507 lag 0 .7 2 3 lag 0 .9 9 9 lag 0 .8 1 9 lead 0 .6 5 lead 0 .5 6 6 lead


p.f.

Example 5.22. Repeat Exam ple 5.21, i f the arm ature resistance ra is neglected.
Solution. Since armature resistance ra is neglected, synchronous reactance,

x ~ £_1235
= 1.648 Q
s Isc 750
Radius of power circle corresponding to 850/3 kW of power is equal to infinity.
Also Vt/2 ■ra —> oo (infinity). Thus the power circles become straight lines' with centre at
infinity.
Now P = V, Ia cos 0 - 0
or 850^00-Q
-= 1269 Zc cos e

Working component of armature current


„ „ 850,000 noo »
= I- c o s e = 3 ^ m 223A-
Scale 100 A = 1 cm.
Taking Vt = 1269 V along y-axis, draw OG = In cos 0 = 223 A in phase with V,. Then a line
^fawn through G parallel to x-axis, represents the power of 283.33 kW per phase and note that
is also the locus of armature current for different values of excitations.

icanned by CamScanner
612 E lectrical M achinery

f vt

Fig. 5.61. Construction of V-curvea, Exam ple 5.22.

Take OD = Vt/X s = 770 A, 90° away from OG ( V 0, = 90°) and with D as centre draw dif­
ferent curves with E f/X s as radii, (given in Example 5.21 as E f/Z s = E f/X s) intersecting the
power lines at a u a 2, ..., a 6, Fig. 5.61. As before, O a1 is the armature current corresponding to
the excitation D au i.e. 5 A. In this manner, other points of the V-curve are obtained, tabulated
-ind can be plotted as in Example 5.21. Values of power factors are calculated as cos IVfig^
cos / V,oa . f'tc. and tabulated as in the previous example.

If, in A 5 .00 1 0 .0 0 15.00 2 0 .0 0 2 5 .0 0 30.00

I a, in A 664 365 230 258 350 440

p.f. 0 .3 3 4 lag 0.602 lag 0 .9 7 4 lag 0 .8 6 1 lead 0 .6 4 lead 0.515 lead

Exam ple 5.23. A 1100-V, 50 Hz, 3-phase star-connected cylindrical-rotor synchronous


m otor h as its synchronous im pedance o f 0.7 + j3.2 ft p er phase. It is working at rated voltage,
rated frequency with an input o f 350 kW. The field current is adju sted to give an electromotive
force o f 1650 V. Calculate the arm ature current, pow er factor an d load angle.
Solution. Terminal voltage per phase,
V, = = 635 V.
V3
E.m.f. per phase,
V3
Synchronous impedance, Power input
tinoc Zs = V(0.7)* + (3.2)z = 3.28 ft.
Vt 635
= 193.7 A
3.28

—L 952
and = 290 A.
3.28 vt Us
Now Fig. 5 .6 2 . P ertaining to Example 5.23.
V/ / o co s0 = ^ k W .

.*. Working component of current,

)n‘S ca le 50 A = 1 cm.
: fak in g OG = 183.6 A and a line through G and parallel to x-axis, represents per phase po'^r
input o f 116.7 kW. Draw a line through O, making an angle 0, = 77.68° with Vt, see Fig. 5.62, an

Scanned by Cam5canneP
A r t Polyphase S y n ch ron o u s M ach in es 613

V E
cut OD ~ 2, s ~ ^ ^ anC^ ~ Zs ~ ^ Then OA = armature current, Ia = 194.5 A and p.f.
5 cos {ZAOG) = cos 19.5 = 0.942 leading. The load angle ZODA = 41.4°.
5. 10 . Power F a c to r Control of Synchronous Machines

excitation- [ " S h T S *• of " f “


synchronous raachi„es is examined first £ £ £ &
infinite bus. r■ iI
l-'i
5,10.1. P ow er-factor C ontrol o f Synchronous Motors. In Art. 5.9, the synchronous I- i
motor V'-cutves were obtained from the superposition of excitation and power circles. The object
10 I t ia vt,■ ^slc Picture of what happens in a synchronous motor, when
its field current is varied. The conclusions arrived from this physical understanding, are fur­
ther supported by synchronous motor phasor diagrams for different field currents.
Is1M
It should be remembered at the outset that a.c. electromagnetic devices must draw a mag­ I.. >
netizing current from the a.c. source in order to establish the working flux. This magnetizing !:!’!
current lags the applied voltage by almost 90°. In other words, the function of the magnetizing
current or laSS‘n« reactive VA.drawn by a.c. electromagnetic device (or an a.c. motor), ia to set
up the flux in the magnetic circuit of the device.
A synchronous motor is a doubly-excited machine, its armature winding is energised from
an a.c. source and its field winding from a d.c. source. When synchronous motor is working at
constant applied voltage, the resultant air gap flux as demanded by constant V„ remains sub­
stantially constant as per Eq^ (5.24). This resultant air-gap flux is established by the co-opera­
tion of both a.c. in the armature winding and d.c. in the field winding. If the field current is Ill
sufficient enough to set up the air-gap flux, as demanded by constant Vt then magnetizing
current or lagging reactive VA required from the a.c. source is zero and, therefore, the motor
operates at unity power factor. This field current, which causes unity power factor operation of
the synchronous motor, is called norm al excitation or normal field current. If the field current
is made less than the normal excitation, i.e. the motor is unde, excited, then the deficiency in
flux (= Constant air gap flu x -F lu x set up by d.c. in the field winding) must be made up by the
armature winding m.m.f. In order to do the needful, the armature winding draws a magnetizing
current or lagging reactive VA from the a.c. source and as a result of it, the motor operates at
a lagging power factor. r
In case the field current is made more than its normal excitation, i.e. the motor is over­
excited^ then the excess flux (= Flux set up by d.c. in the field winding—resultant air-gap flux)
must be neutralized by the armature winding. The armature can do so only if it draws a demag­
netizing component of current from the a.c. source. Since in a motor, the magnetizing current
lags the applied voltage by about 90°, the demagnetizing component of current must lead the
applied voltage by about 90°. In view of this, the excess flux can be counterbalanced only if the
armature takes a demagnetizing current or leading reactive VA from the a.c. source conse­
quently the synchronous motor operates at a leading power factor. It can, therefore, be inferred
from above that a synchronous motor operates at a lagging power factor when underexcited and
« leading power factor when overexcited. The extent of power factor lead or power factor lag,
ePends upon the degree of over excitation or under-excitation.
The effect of field current on the synchronous-motor power factor can also be explained with
he help of its phasor diagram . For simplicity, arm ature resistance ra is neglected and
synchronous reactance X8 and terminal voltage V, are assumed to remain constant. From Eq.
■35), it is seen th at the per phase power is given by

p = sin 5 = V(Ia cos 0.


Xt . ...j , \o.> iUi.ijL.inu> irujiftinuu irli tie;

c u uy o a i iu u a i ii i c i
614 E lectrical M achinery

Fig. 5.63. Effect of field current on synchronous


motor power factor. Fig. 5.64. Synchronous motor (a) V-curves and
- current curves.
(fc) power factor versus field

V, J I Z T c lT Z . °TUhfsU^ “ at' t V h '* C° S * " I ”8* rm a in b“ a“


varies, but component of E ,normal to V
and therefore armature cu rren t/ alsovaries b u t T n J h 54 remam const ant- As E , varies, IaX,
, ° anes>butm such a manner as to k eep / cosGconstant

Tbi : “ byr r 7t he fie,d ^ "


armature current mus^clmnge to” ^ the phasor of
power factor. When the excitation is increased to E the hTd * eref°re’ operates at unity
6a so that E sin £ - j? • x ^ ^oad angle must decrease from 62 to
a g a i n ,th e p S a s e o f ^ t u 'r e '
fore, the motor now operates at a leading power factor Note th aU h " " T Flg’ 5 6 3 ^
of arm ature currents are equal i e I cns ft - t n r active power components
the arm ature current is m i n i m a at unhy p f C° S F ‘f ' 5 63 als° reVeals “
tion of arm ature current with the changes in field * .admg or Egging P fs- This var,a‘
fixed powers. Since the plot resembles the letter V these are c a lfd 1 5 ‘64 (a) for d^ eTent
motor. Note that V-curves show the relation between 2 V-curves o f a sy n c h ro n o u s
constant shaft load and fn r cnncfQr.f f • 1 1 armature current and field current with
Sn inff all the m in im n l , . Grminal V° ltaee* In Fig' 5 64 (a>the ™ ity p.f. dotted curve,
joining all the mmimum-armature current points, is called unity power factor compounding

Scanned by CamScanner
• Polyphase S y n ch ro n o u s M achines 615

t" I
Similarly compounding curves for 0.8 p.f. lag and 0.8 p.f. lead, are shown by dotted curves
clirm 5.64 (a). The C°™P0™ ^ u °u u p fs' Can also be drawn if required. These f
f i n d i n g curves * which the field current should be varied in order to rP

lain13’11 c0flS.fn ^Vhe svnchronmi«! a & s-Note that the synchronous motor compounding
,rves are similar to the synchronous generator compounding curves of Fig. 5.39.
c For c°nstanfit4 ?M arfobta^ne^Nofp r ^ f ^ 8 5 ? ° , ^ 38 3 function of the field current, the
curves of Fig. 5f * (6 )a ” ob 3‘ fold ° the field current for u.p.f. at full load is more than
the field current for U- P ^ * ™ :l° ad; It may also be seen from Fig. 5.64 that if the synchronous
niotor at fullk®?.*® nPr T h e rn ^ P C Kf / ! m0Val °f the shaft load causes the motor to ;i:F
operate at a leacbnS p. • urves obtained by plotting power factor versus field currfent
resemble inverted 7 , these curves are, therefore, sometimes called inverted V-curves of a
synchronous motor.

y The ability of a synchronous motor to operate at leading power factors when overexcited, is
'I
utilised for ^ P ™ ^ , ^ 1 1 1 / °f many supply systems. For example, an industrial

im
employ a synchronous motor to
the power factor of the entire load and thus save the organisation from the low-power
p r o v e
i!;:
factor penalty clause. * ¥II I
It should be remembered that an induction motor must take lagging kVAr from the suddIv
system, in order to set up its working air-gap flux. A single line diagram of a 3-phase induction
m o t o r connected to an ac source is given ,n Fig. 5.65 (a). When a synchronous motor is con­
nected in parallel with it , then this motor, when overexcited, supplies lagging kVAr to the
induction motor locally .A s a result distribution lines are relieved completely or partially from
supping the lagging kVAr needed by the induction motor. Now the supply lines have to LndT e
reduced kVA and reduced cu rren t; this leads to better operation of the overall system see Art
iV w J Vi,” u" mf ! ° r iS Sti11 °PeratlnS at same lagging pf, absorbing saine
kVAr and kW. It is the combined load, consisting of over-excited synchronous motor and induc-
kW
LAGGING kVAr

■'li

"III
. I

fy - 5.665. (q>q p, . _ (6)


' nase Induction motor (b) An overexcited synchronous motor in parallel with induction motor.

o o a i i i i c u u_y v a 111“
IArt. s.
616 E lectrical M achinery 10
tion motor, that is working now at an improved pf. In case the deman o agging r required
by induction motor is completely met by the overexcited synchronous mo or oca y, then the
combined load operates at unity pf.
When the primary function of a synchronous motor is to improve the power factor of thc
system or to control the flow of kVAr, then the synchronous motor carries no mechanical load
on its shaft. Such an unloaded synchronous motor with no shaft extension, is called a
synchronous condenser or synchronous compensator and is used in large power systems, where
static capacitors are uneconomical.
Under ideal conditions, the synchronous motor phasor diagrams for zpf leading and zpf
lagging are drawn in Fig. 5.66 (a) and (6) respectively. As Ef and Vt are in phase, load angles
is zero and power P - 0. From these phasor diagrams, it is observed as under :
(а) As 7a leads Vt by 90° just like a capacitor, synchronous motor behaves like a capacitor
bank under zpt leading operation.
(б) As Ia lags Vt by 90° just like an inductor, synchronous motor behaves like an inductor
bank under zpf lagging operation.
Fig. 5.66 (c) shows the V-curve of an ideal synchronous motor. .
Ef
j'a x s
'o 1
V «‘ Vt
j'c * s
E ( > Vt
6=0 £| E r v,
P=0 6*0
(Overexcited) P*0
( U n d e r -e x c it e d )
In d u c to r C a p a c it o r
(la g g in g pf)' ( le a d i n g p f )
-}90°
"f1
Ia
u n it p f
(a)
lb) W
Fig. 5.66. Synchronous m otor (a) zpf leading (6 ) zpf lagging (c) V -curve.

5 10.2. Pow er-factor co n tro l of A lte rn a to rs. For a single alternator, or an isolated al­
ternator, delivering power to a load, the relevant characteristics are already described in Art.

fn fi^ t s T s T s c u lS V n ^ t' behaVi° Ur ° f 'CyHndrical-rot° r ““ or connected to an

The n h « tT ‘° r’ ‘ herVc f qUati°n iS = V‘ + flJC ’ With zero arm ature resistance,


alternator s T ™ , fu '*dFaWn Fig' 5 6 3 '
The "fore T “ V T armatUre CUrrent is laadiaS V, = V„ = bus-bar voltage,
f r o ! infinite b S ' ere* Clted alte™ to r operates at a leading pf and absorbs reactive power

current / ‘ i s to n h a , ” T V eXCitati°D em f rfSeS- Fo r armature


m aUycJ ted L ' t T e i T ! J ' y f Unity pf “P o t i o n , the alternator is said to be

a synchronous motor, plot of In versus L is rall«i v r , g' ,\ „


infinite bus TKp nnumr f f t- I V-curve of an alternator connected to an
d r a w n in R g V 67 ( f t ! T C rS l‘S C U rr6 n t C U rV e ' fa l0 w n a s in ™ r t e d V - c u r v e is also

b can n ecn D ^ fafrS can n eP


Xihu

fLr
>6e<d
k e<,6

> f
glej

cHor

lctor

(a)
F%. 5.67. Effect . f field cucceet „„ an , l ter„ a . . r e j e c t e d * i n f i l bu,

can notv be summarised as given'below' Fo^conv8™8™101! fr°m that connected to infinite bus
and the one connected to infinite bus as A2 enience, let us call isolated generator as A l
(i) Speed. When speed ofA l is u .. ,.
Its output frequency also increases with increase in 9^ ° ° e,mf and termilial voltage both rise,
reactance also rises. In other words onprati™ f “ !? 3S 9 consecluence its synchronous
trolled above or below that g i v e n ! l £ n ! ! K r ,,eir °f iS° 'ated ge" erator ca" b<=
Speed of\A2 csnnot be slterpH ac ife r
by bus-bar frequency. Further, t e r m t a a l S o f e r i t ? " ? 0r<i' Spe<=d are governed
<«) Field excitation . If field current of Al is a * S'bar V°ltage‘
al- voltage both rise. Its operating power factor dependsupon'the n ^ tT ^ n * ° Y ^ f and termmal
Jt. heater load, operating pf of isolated generator is always unhy example, for
an
ready operating at a leading pfiTet’ R g . T s T w fading to la^ n g if al­
ce. - urves can be obtained only for A2 and not for alternator A l onstant power, alternator
he
re.
er

v0'ta£e and operat-


mcrease » l„ad angle as £/shlfls away fr0£ infln“ ™ ™ b e c a u s e of the

•11. Two-reaction Theory of Salient-pole Machines

o T rns : r : nmer uw ft :: ehmv ^ r ; ; 7; r.^ s ™ ^ : : er cour d for by


" 'S n e l w h e r e 'f t ^ 006 **•“ eVf° IVed' T,.hiS “ permissiblc ™ ease o^n o^sllienT pot
hally c o n s f ! ^ air'gaP 1S uniform and the reluctance of the magnetic circuit i s « k f
may be. s*ant-whatever orientation of the field poles with respect to the armature mmf wave

Scanned by CamScanner
JA rt- S.l|
618 E le ctrica l M ach in ery

In salient-pole machines, the armature mmf cannot be «"*


equivalent reactance. This is because the air gap is no unl internolar or quadrature g
polar or direct axis is considerably smaller than that along *>».
Therefore, the cylindrical-rotor theory, based on constant air-gap *elu^ app,led
to a salient-pole machine which has different air-gap reluctances along its periphery.
In a salient-pole synchronous machine, the location of armature-winding mmf relative to
the field-winding mmf is similar to that obtained in a cylindrical-rotor synchronous machine.
In order to recapitulate this, Fig. 5.68 (a) is drawn by referring to Fig. 5.5 (a) but with round
rotor replaced by salient-pole rotor. In Fig. 5.68 (a), field poles N, S on rotor are shown facing
coil-sides a, a . Therefore, maximum emf is generated in coil a, a For the anticlockwise direc­
tion of rotor rotation, the direction of enif is indicated by dots in the coil-sides (a b , c ) under
the influence of AT-pole and by crosses in the coil-sides (a', b, c) under the e f ect of S-pole.
q -axis

(b) spatial orientation of F„ relative to Ff.

As before, the armature current is assumed to lag excitation emf E f-by a time-phase anglt
\\i° so that internal p f is cos \\i° lagging. Therefore, current in coil a, a' would attain maximum
value after the field poles have moved to a new position \j/° electrical ahead of the maximum emf
position of Fig. 5.68 (a). In other words, by the time arm ature current in coil o, a' reaches
maximum value, the rotor poles N, S would have moved forward by vy° electrical from its maxi­
mum emf position of Fig. 5.68 (a). This is shown in Fig. 5.68 (6), since coil a, a' representing
phase ‘a ’ carries maximum current, peak of the resultant rotating arm ature mmf Fa is directed
vertically upward along the axis of phase ‘a ’. Fig. 5.68 Lb) reveals th at arm ature mmf Fa lags
field mmf Ff by a space angle of (90 + y°) as in a cylindrical-rotor alternator of Fig. 5.8, With
changes in internal p f angle /, the reluctance offered to arm ature mmf F varies because of a
change in its spatial orientation relative to the field-pole axis or direct axis. This shows that the
magnitude of armature reaction flux per pole cj)„ (= F a/reluctance) varies with a change in angle
y. This difficulty of encountering different reluctances by arm ature reaction mmf Fa, as the
angle y changes, can be overcome by resorting to two-reaction theory suggested by Blondel.
According to two-reaction theory , the sinusoidal arm ature m m f F is resolved into two
sinusoidal components, one F(l(i along d-axis and the other F m/ along q- axis. It is seen from Fig-
5.68 ( b ) that Fud experiences minimum but constant reluctance along d - axis. Similarly, com­
ponent F aq encounters maximum but constant reluctance along q-axis. Thc rf-axis compon^
F„d is seen to be demagnetizing in nature in Fig. 5.68 (6). In general the rf-axis compopent
Flld is magnetizing in nature if y is an angle oflead and is demagnetizing in nature if
angle of lag, i.e. F ad produces only a change in the field strength. The q-axis component hi

Scanned by CamScanner
v wr

~ . . . !^}yP}iase Synchronous Machine aio


juCes only a distortion of the field-flux wave. T h earn lT —
Sought of, as produced by those components o f armature K d 311(1 F"‘ ma* be v. i
,iadrature and in time-phase with Ef, i.e., Fud and F are 1 2
'l h &re resPectively in time-
8 •i:
01
Sely, see Fig. 5.69. From this figure, it is s e e n C t ^ r 311(1 V«*Pec- [l>1
COS(5 + 0) = C0S v - In °ther words>the armature current 7 *'* t 0) = h V and 7<= h
j and /,_the component Id is in time-quadrature with E wh 18 reS°lved into ^ components
I InFig- 5.69, axis offield is taken horizontal along the field 2 * 1 “ mponent7<;is ^ time-phase with
E 'lags <tyby 90°. Terminal voltage Vt is taken to lag E( J Exdtation voltage S'
v by internal p f angle y as assumed. 1 y UdQ -angk b and armature current Iu lags
Efby “— *’ . ~ lags I
Note that a direct-axis quantity is one whose mafmptin er ■
11
here Fflrf is a direct-axis quantity. Since component of armat “ al°ng the field pole axis-e^-
with Ef, produces Fad, Id is also a d-axis quantity Simila 1 UFG CUrren^ *n bime-quadrature
whose magnetic effect is along the interpolar axis and her" h f, q” adrature-axis quantity is one
It should also be noted that bothFod and F are .v. both/% and7«j are q -axis quantities,
time-phasor quantities. q P as°r quantities, whereas Id and Iq are
In a salient pole synchronous machine the relu n ^ i ,
less than that along the <7-axis. In view of this the comnn /n field~pole axis i.e. d-axis is
byreluctance along d-axis) and <bao (equal to F dividr n , S *ad (equal to F“d P itie d
thesame ratio as are the component m.m fs , i l by re' UCtance al™B "-“ is) are not in
jW F ad
U Kq

q -axis

ScanneaD yuam bcanner


li

620 E lectrical M achinery (Art. 5 jj

This is depicted in Fig. 5.69 by cj>„ (= §ad + 4>„7) and Fa {=Fad + F aq) not being in phase wi^
each other.
In Art. 5.4.1, armature m.m.f. Fa is replaced by time-quadrature lagging induced e.mf
Ean see Fig. 5.19 (a). In a similar way, the armature m.m.f. components in Fig. 5.69, can be
replaced by the induced voltages Bad and E aq, lagging in time by 90°, their respective m.m.f
components Fud and Faq (or ld and Iq). For the sake of clarity in the figure, only Ead is shown
lagging F ad by 90°. By neglecting saturation and by referring to Eq. (5.10); Ead the voltage
induced in the armature winding due to d-axis armature m.m.f. component F ud, can be written
as
F ad ~ ~ jK d Fad = -jC K d l d = - jX ad I d.
Similarly Eaq, the voltage induced in the armature winding due to qr-axis armature reaction
component Faqt is given by
Eaq = - j K q Fuq = - j C Kq l q = - jX aq 7q.
, Here Xad, X^ are proportionality constants similar to the magnetizing reactance Xar of the
cylindrical rotor machine, see Eq. (5.10). Thus here Xad and Xaq are respectively the d-axis and
q-axis magnetizing reactances (or armature-reaction reactances) o f a salient pole synchronous
machine.
The phasor sum of Ef, Ead and E aq gives the air-gap voltage F,r just as in a cylindrical-rotor
machine, see Eq. (5.6), i.e.
F r ~ F f + Ead Fatj ...(5.53)
If voltage drops 7ara and j7a xal are subtracted from En the terminal voltage V, is obtained,
see Fig. 5.69.
The leakage reactance drop in the armature winding may be resolved into two components
as given below.

ce = ~ J xoi = - J Qd + Tq) xai = - j Iq •*ai - j l d Xal = cd + de


Hence the armature leakage reactance drop can be combined with the equivalent arma­
ture-reaction emfs Ead = - j XadId and Eaq = - j Xaq 7q. Therefore, from Fig. 5.69,
Vt = oa + ab + be + cd + de + e f

^ ^ a<t ~ jld X(id ~ J IqXal ~ jld ' xal ~ Iara


=jf &aq + Xal) - j l d (Xad + x^) - 7ara
= E f - j l q x q - j l d x d - Iara
Here Xad + xal = Xd, is the d-axis synchronous reactance and Xan+ x , = X is the q-axis
Synchronous reactance. q
Note that the arm ature leakage reactance x„, has been assumed to be the same for both d■
and q-axis currents.

The phasor diagram of Fig. 5.69 is redrawn in Fig. 5.70 (a) by introducing Xd and X,. In its

rator is gfvenhi £ 0,00,7 PhaS° r for 3

t io n ? h !f lffereTK ^.etwce,n ‘ he e m _f ” ethod (or cylindrical-rotor theory method) and two-rcac-


.on theory method must be carefully noted. The e.m.f. method introduces the concept of
synchronous reactance X„, which accounts for both the arm ature leakage and armaturc-reac-

Scanned by CamScanner
Polyphase S y n ch ron o u s M ach in es 621
r
jlq X q

dAd
I i dx
d Ad

•I*
(o I
ijlj I
F,C 5 7^ h r l r reacl,0n lhe° ° ' phaSOr d,a^ m o ra . a l , e n t i l e
wnchronous generator and ih) its S1mplif,ed version
ill:
unit fluxes ; the two-reaction-theory method introduces the concept of two reactances namelv
ft and .V, The d-axts synchronous reactance X„ accounts for the armature leakage T u x and the II-'
|1* .
d - a x i s component of arma.ure-reactton dux. whereas X , accounts for the armature leakage flux

*nd9 component of armature-reaction flux I n c m f m p th n ri <x * • ?


r L f or / bv 90 wh, n -,s ,n , .T d' the armature-reacti°n e.m.f.
. I, ' ,w<’ re»>:‘'°n ,h™r-v method, armature reaction e.m.f.
=v£urf ♦ Aov lags r , or la by an angle other than 90

F , 5701*1 reveals that the phasor sum of and>7A gives the excitation
voltage E,.Thus the voltage equaUon for each phase o f , salient-pole synchronous generator is
E/ - V, ♦ t / . , « A , / , - V A , ...(5.54)
Analysis of P h a so r D iagram . In order to use the phasur diagram of Fig 5 70 (6) the
armature current /.. must he ved „« <f. and components Usually V, / ™
factor angle 0. Xd an <Awarc given in a ' power
problem, but for calci luting / , and I . „*
thr internal power-fact t angle (6 •+flj *' * *
must be known Korthisj irpose.draw
^normal to/u as shown in Fig 5 71 i a )
Since ab is 90 away from / ,. it must be !l:
I'.
■'reactance drop, say / i7A Draw at per­ 'it.
pendicular too6. It is seen from Fig 5 71
0 00 ^ and acb are right-angled ■ r
angles and. therefore, E bac = Z 6 - 0
ac = ah cos (5 + 9)
= -YZUcos (5 + 0) = de \\
’r

[ = Ia cos <5 - 6)] •ai Ib)


or
-V= x . Fig. 5.71. Resolving of I„ into its d- and (/-axes components
Id and E 1 0 1 f°r a synchronous generator and
<b) for a synchronous motor.

bcannea oy uam bcanner


622 K lectrical M u c liin m

Thus in Fig. 5.71 (a), ab = /„ Xq. 1fo b = E f, then


F f = V, + 7„r„ +JI(fXq
Note that phasors Ef and are directed along the same line.
Therefore, the phasor sum V, + 7aru +j7(l Xq gives E f and the angular position of the •
tion voltage Ef with respect to V,, i.e. angle 5 is known. From this, the angle (5 + 0) and ^
fore, IHand Iq can be obtained—this is illustrated in Example 5.24, Fig. 5.71 (q) also reveal K
bd = cd - cb at
= Xdld ~ XqIu sin (5 + 0)
= XdI d - X ql d
= (Xd - X q) I d
Eq. (5.54) can now be re-written as
_ E ,= V,+ l ara + +j l d (Xd - X , ) (5 56)
Salien t-p ole sy n ch ron o u s m otor p h aso r diagram . For a synchronous generator a
VO tages are generated and the currents are output currents. For a synchronous motor’
voltages are applied and the currents are input currents. Thus the motor voltage equation ca6
be obtained from the generator voltage Eq. (5.54) by writing ~7a in place of 7a. Therefore th"
voltage equation for a salient-pole synchronous motor, from Eq. (5.54) is
~ E f+ raIa t j I dXd +j7q Xq
The voltage phasor diagram as per Eq. (5.57), is given in Fig. 5.71 (6). In order to compute
om V„ Iai ra, Xdt Xq and power factor angle 0, draw ab perpendicular to I u. As before, ab is
90° away from 7a and therefore, a b must be a reactance drop, say Ia X. Fig. 5.71 (6) reveals that
Zabc = 90 - (0 - 5)
lara
Idia VfsinS
V h

Vf cosS

(6)
Pig. 5 .7 2 . P h asor d iag ram s a t lagging pf for salient-pole (a) altern ator and (b ) motor.

Scanned by Cam scanner


Polyphase S ynch ron ou s M ach in es A2 3
Zbac = 0 - 5 .

°r x =x„.
Therefore, the angular position of£/-can be ohtainor) k„ a- —
wis permits the calculation of angle ,6 1 8 ) . / , and , Her0 °b ' * / - -jU r
^ re-written as ' e 11 uu d (Ar/ ~ *,,) ^9- <5-57) can
jjqW 1
V<= Ef+ jld (Xd - xq) + J I j c + r J

? ,s beh,nd v - for * —
» * " > f « a t,lindrt*al-n)tor mathin. ^ I T ’^

r,eld-winding slots on <7-axis reluctance-consequentlv /°d tffe h t T T * ^ l° **“ effeCt ° f


rotor machines. sequently Xd differs slightly from Xq in cylindrical

at lagging as well as leading poweffMtors.'rhi’s wufcertainly leTdtor° T t t maChdne W° rk‘ng

cos 0. tailS' Thus' for alternator working at a lagging pf


od = o f + fc + cd
°r Ef= V / C O s 6 + V a + ^ d ^
...(5.59 a)
/Xq = Vj sin 8 + /j r
n'h
a 6 ab + " ...(5.59 6)
From Aoa'6, tan \\i = ^ + aaaa '
oa' a'x + ox
_ IgX,, + Vt sin 6
. h ra + V, cos 0 ...(5.59 c)
whprp •
V = internal pf angle by which Ia lags Ef
= 5 + 0.
For a synchronous motor working at a lagging pf, Fig. 5.72 (6),

W ^ sin 5 ♦/,. : ; 5^ M
From Aoa'6, tan y = ^ 4 = l a ~ ab
oa' ox* - a'x
_ Vt sin 0 - lgXq
where V ,c o s 0 - / o ro ...(5.60 c)
For 1 ■ V = 0 - 5.
nat°r and^oads’ the phasor diagrams are drawn in Fig. 5.73 (a) and (6) for an alter-
yucnronous motor respectively.
ls seen from FiS- 5.73 (a) that
Ef= V, cos 5 + l q ra - Id Xd ...(5.61 a)
IqXu = V, sin 5 - l dra ...(5.61 /,)
tan Q ^ . ViSine-/^
...(5.01 c)
oa' V, cos 0 + l a ru

Scanned by CamScanner
624 E lectrical M achinery

For Fig. 5.73 (6), Ef = Vt cos 5 - Iqra + 7^Yrf


IgXq = V, sin 5 - l d ra
a'b a ’a + a b _ Vt sin 9 + 4 Xq
tan V - ^ 7 ~ _ a 'x V, cos 9 - /„ ra
Vf sinS-

Vt cos S

Vt cosS

10)
Fig. 5.73. Phasor diagrams a t leading pf for salient-pole (a) altern ator and ( 6 ) motor.

Computation of internal pf angle y facilitates the determination of l d and 7 as


Id = Ia sin y
and Iq = Ia cos \|7 ...(5.63)

Example 5 2 4 . A salient-pole synchronous generator has the follow ing per-unit parameters :
Xa = 12, Xq = 0.8, ra = 0.025.

k V A ° a ? r Z T o Z ? e ? 7 7 Uage 7 °" ^ ^
ated voltage an d at pow er factor o f (a) 0.8 lagging a n d (b) 0.8 lead in g
Solution. For th,s example, refer to Fig. 5.71 (o). With V, as a reference phasor,
Vt = 1.00 + >0.00
4 = 0.8 -> 0.60 = l.oo / - 36.9°
J 4 x q - j (0.8 -> 0.60) (0.80) = 0.48 + > 0.64
4 ra = 0.020 -> 0.015
Thus
Z / = V ,+ j l a Xq +7ara
= (1 +>0.00) + (0.48 +> 0.64) + (0.020 - ; 0 0151
= 1-50 +j 0.625 = 1.625 /2 2 62- ' ’
8 - 22.62- and / 5 + 0 = 22.62 + 36.9 = 5 9 .52 °.

Scanned by CamScanner
Polyphase S yn ch ron o u s M ach in es 625

R e s o lv in g the arm ature current Ia into its d-q components, we

get Id = 1.00 sin 59.52° = 0.862 )i<j xd


/ = 1.00 cos 59.52° = 0.507.
and
u . ' - (a)
Rig, 5.71 E >f and Eff are alongo the
- reveals that same lUie
- v oaiuc od.
line Ofl, o 'o
an j^0 qinoH Kv QrlrlinfT fVtn 1 i_r » ■ -
Thus Ef canbe obtained by adding the length bd = Id (Xd - X ( )
1'nu° *7 ° v,
-ically to E f.
1numerically 7
E f= E f + Id {Xd - X q)
= 1.625 + (0.862) (0.40) = 1.9698
£ ,= 1.9698 /2 2 .6 2 ° .
and
The above solution can also be obtained by referring to Fitr
5 72 (a). From Eq. (5.59 c), '
Fig. 5.74. Phasor diagram for a
Ia X + Vt sin 0 0.8 + 0.6
leading p.f. for the salient-pole
tan y = -T— 1— — — synchronous generator
Ia ra + V, cos 0 0.025 + 0.8 of Example 5.24.

orr y
T = 59.49°.
Id = Ia sin V = 0.862 and Iq = Ia cos y = 0.507
Loadangle, 6 = y - 0 = 59.49° - cos" 10.8 = 22.59°.
From Eq.(5.59 a), ^ = l x cos22.59 + 0.507 x 0.025 + 0.862 x 1.2 = 1.9701.
(6) For the salient-pole synchronous generator, the phasor diagram for a leading power-fac­
tor is illustrated in Fig. 5.74. Here again Vt is taken as the reference phasor.
,\ Vt = 100 + y'0.00
and l a = 0.8 +y'0.60 = 1.00 Z 36.90
j l a Xq = j (0.8 +j0.6) (0.8) = - 0.48 + j 0.64
l aru = 0 .0 2 0 + j 0.015
Thus Ef = Vt + j I a Xq + Iara
= 1 + j 0.00 - 0.48 + j 0.64 + 0.020 + j 0.015
= 0.54 + j 0.655 = 0.849 Z50.50°
•• 5 = 50.50°.
For a leading power factor, the internal power factor angle between Ia and Ef is given by
y = Z5-0
Z5 - 0 = 50.50 - 36.9 = 13.60°
Resolution o f/a into its dq components, gives
Ia = 1.00 sin 13.6° = 0.235
and
*I q
q= 1.00
— l.U cos J.O.U
U CUB 13.6°..
The excitation voltage E f is again the numerical sum of E f and Id (Xd - Xq).
E f = E f + Id (Xd - Xq)
= 0.849 + 0.235 (0.4) = 0.943 p.u.
or
E f - 0 .9 43Z 50.50.
^ternatively, from Eq. (5.59 c) and Fig. 5.74,
0 ab c b - c a cb - y x Ia Xq -V ,s \ n d
a n y - tan ( ^ ~ oa ~ ox + xa ox + yc Vt cos 0 + /„ ru

Scanned by CamScanner
626 E le c tric a l M ach in ery lA rt S.ij

Here \j/ = 8 - 0 is positive as E f leads Ia, but 9 is negative.


_ 1 1 X 0.8 —1 X 0.6 , q £.qo
V = tan j x 0.8 + i x 0.025 “ 1 3 M
Id = 0.235, Iq = 0.972
From Fig. 5.74, 8 = v + 0 = 13.63 + 36.9° = 50.53°. From Eq. (5.59 a),
Ef = 1 x cos 50.53 + 0.972 x 0.025 + 0.235 x 1.2 = 0.942
E f- V A
Now the per unit voltage regulation can be obtained by using the relation For;

leading power factor, the voltage regulation is seen to be negative.


If required, the field currents, corresponding to the excitation voltages can be obtained from
OCC of the salient-pole generator.
E xam p le 5.25. F or a salient-pole synchronous m achine, prove th at the d-axis synchronous
reactance Xd, can be obtained from its OCC an d SCC. N eglect arm atu re resistance.
Solution. Phasor diagram of Fig. 5.70 (6) for a salient-pole
synchronous generator reveals that with ra = 0.
' Vt s in b = Iq XQ
Under steady state short-circuit conditions, terminal volt­
age V, = 0.
Iq Xq = Vt sin 5 = 0. ,idxd
or Iq = 0.
Now the short-circuit current 90
a
h e - h + h = h- lsc~ld
Thus the phasor diagram of Fig. 5.70 (6) under short-circuit Fig. 5 .7 5 . Two reaction theory
conditions and with ra = 0, gets modified to that shown in Fig. p h asor diagram , under
sh o rt-circu it condition
5.75. The short-circuit arm ature current I s c becomes equal to
and with ra = 0 .
Id, because l q = 0.

From the phasor diagram of Fig. 5.75, it is seen that


Ef = Id Xd
or
" h Isc

1,6 ° bt“ ed fr° m Eq' < 554' f0r “ al“ r' V, = 0, = 0 and

E f = j I d Xd
or „ .
aEfr h- Xd - - I _s c Xd
a
The excitation voltage Ef is obtained from o.c.c and -4. r
a given field current. Thus " lrcui^ cu rront l sc from s.s.

a given fieid current


T. . h h o rlcircu .t current for the same f o ld current---------
The exc.tat.on voltage of the synchronous machine, as given by E q“
Ef ='l2nfkwN ph$f.

‘S canned by Cam scanner


P oly p h ase S y n c h ro n o u s M a ch in e s 627

flux <f>^-is proportional to field m.m.f. F^-or field current If, excitation voltage can
SinC
itten as
be 1
E f = ^ ( 2 n f ) k tvNph (klf)

Ef = K w If
or X a = O) Lr
and r _ E { K a l f K If
...(5.64)
Isc~ T d ~ i n .7 ‘ T 7
X is a constant and L d is the d-axis synchronous inductance of the synchronous
^ere gq (5.64) shows th at for a given field current If, the armature short-circuit current
,ubstantially constant over a wide range of frequency or alternator speed. In view of
• s substantially
re"“T
remT ring ine the short-circuit test, it is not necessary
n ecessai.,for the alternator
........ speed to be equal
r ------------ . to the
this, u
this, during Speed. However, at considerably lower speeds, the resistance becomes comparable
reactance an(l change in arm ature current is manifested.
V1 ff mple 5-26. From the p h a so r diagram o f an over-excited salient-pole synchronous m otor
■ oith armature current leadin g the term inal voltage), prove the follow ing relations :
4 (Xq cos 0 + ra sin 0)
tan 5 = jfqXq
V, + I a (Xq sin Q - r a cos 0)
Vt sin 5 - Iara sin (5 + 0)
and 4 cos (5 + 0)
where 8 and 0 are load an d pow er-factor angles respectively.
Solution. Phasor diagram for an over-excited synchronous
motor is drawn in Fig. 5.76.
From the tip of the voltage phasor Vt, i.e. point a, draw ab
perpendicular to od. Now bc = d e —Iq Xq. Since the resistance
dropac is parallel to Id, ac = Idra.
a b = ac + cb
= Idra + IqXq-
Fig. 5.76. Phasor diagram of an
From the phasor diagram, it is seen that overexcited synchronous motor.
. _ ab ac + cb
sin o = — =
oa oa
+ Iq X Q
Id r a ...(5.65)
Vt
But Id = 4 sin (5 + 0)
and 4 = 4 cos (5 + 0).
•••From Eq. (5.65),
Vt sin 5 = ra 4 sin (5 + 0) +Xq 4 cos (5 + 0)
or
Vt sin 5 = rB4 (sin 5 cos 0 + cos 5 sin 0)
‘ + Xq 4 (cos 5 cos 0 - sin 5 sin 0).
Collecting sin 5 terms on the right hand side and cos 5 terms on the left hand side, we get
sin 5 [V, + Xq 4 sin 0 - ra 4 cos 0] = cos 5 [ra 4 sin 0 + 4 x q cos 0]
sin 5 4 (Xq cos 0 + ra sin 0)
= tan 5 =
cos 5 V, + 4 (* 9sin0" r“ cos 9)

Scanned by CamScanner
1:
I

Eq. (5.65) readily yields the value oiX q as


V. sin 5 - Idra Vt sin 5 - ljr a sin (6 + g)
I g — = --------J ^ s I s T i j
From above, X , can be calculated by measuring Vt, Ia, r a, power input (for computing ang|e
9) and 5. The load angle 6 is usually measured with the help of a stroboscope.
5.12. P o w e r-a n g le C h a r a c te r is tic s o f S y n c h ro n o u s M a c h in e s
In this article, the power expressions are derived in terms of power ank|e &>^ om Phasor
diagrams of cylindrical-rotor and salient-pole machines. The power flow through a non-salient
pole synchronous machine is already discussed in Art. 5.8 an t e presen erivation merely
supplements the previous analysis.
Armature resistances of synchronous machines are usually small, these are, thereto^
neglected in this article.
5.1 2 .1 . C ylindrical-rotor synchron ou s m ach in e. Consider that the synchronous machine
is acting as a generator and is feeding power to an infinite bus of constant voltage Vt> as shown in
the single line diagram of Fig. 5.77 (a). Its phasor diagram for a lagging power factor and with zero
armature resistance, is illustrated in Fig. 5.77 (b).

(a)
Fig. 5.77. Cylindrical-rotor alternator connected to infinite bus, (a) its single line diagram
(b) its phasot diagram for a lagging power factor and (c) its power angle characteristic.

The per phase power delivered to the infinite bus is given by


P = Vt Ia cos 0 ...(5.66)
It is seen from Fig. 5.77 (6) that Zoba = 9 0 - 0 and Zobc = 180 - (90 - 0) = 90 + 0. The tri­
angle obc reveals that
be oc .
sin Z6oc sin Zobc
or xsia
sin 6 sin (90 + 0)
or X J a sin (90 + 0) = E f sin 8
or Xs Ia cos 0 = £y-sin 8

or Ef
Ia cos 0 = ^ s i n 8.

Substitution of value of / cos 0 in Eq. (5 66) gives


D Ef Vt . ' • ’
P = ~ t t ~ sin 8 ...(5.67)

Scanned by CamScanner
Polyphase Synchronous Machines 629
(5 .67) agrees with Eq. (5.35) already derived for n j . .
lor a cylmdrical-rotor synchronous
mach in e
With the help of Eq. (5.67), the variation of power P with ™ , c • .
77 (c). This power versus load-angle curve has a sinncmd i p°wer' anSle 5. 1S plotted in Fig.
5 „er angle characteristic of the cylindrical-rotorsvnchm shaPe and 1S usually called the
C i s taken as positive and therefore, for The P°WerP’ f°r ^ene‘
f svnchronous machine, running as a generator nr t V8‘ P°wer angle characteristics !
f0r X “ d, see Eq. (5.35). 6 ^ °r motor’ are if its armature resistance
is
ti
5. 12.2. Salient-pole Synchronous Machine. Consider that n,„ = u . .

Hi i r ‘

^ 1%e components of V in phase wi?™ , and W v /™ T X p tc U v e ly


Since these two components of Vt are in phase with I and 7 u respectively,
to the bus is d q’ he per phase power delivered tej:

P ~ Id ('Vt sin 5) + Ig (V, cos 5) 1;


1;
...(5 .6 8 )

jlqXq
1
FJ
R e s u lt a n t
/ pow er

I n fin ite
bus
0 9 d O l 8 0 o^ S

I, I:
O-
* d - Xq r ^ i ls in 26
Vt MOTOR generator
(a)
0b) (C)
ik\ u T. *7 generator to) single line diagram
(Ib) phasor diagram for a lagging p.f. and (c) power-angle characteristic.

From Fig. 5.78 (6), V, sin b = a b = d c = InXn

Vt sin 8
hi
and
vt cos b = o a = o d ~ a d = o d - b c = Ef - I dXd
E f - V t cos 8
d --------- v -------

Instituting the values o iI d and Iq in Eq. (5.68) and simplifying, we get

p Ef V, V?(l 1
E = —hr— sin 8 + — — - — sin 28 ...(5.69)
X, Xn X ,

C B Trn e r
630 E le c tr ic a l M a ch in e ry

Eq. (5.69) gives the power versus load-angle characteristic of a salient jxile machine

shown in Fig. 5.76 (c). The total power consists of a fundam ental component sin 5 and

. vf sin 25.
second harmonic component —
Xd
The first term in Eq. (5.69) is identical with th at obtained for a cylindrical rotor machini
This component of power is called the e l e c t r o m a g n e t i c p o w e r , b ec a u s e its existence depends 0
the existence of both the arm ature winding (with Vt) and the field winding (with If). The secon
term of Eq. (5.69) exists even when the field current is zero, i.e. E f is zero. This second com
ponent of power is present because the arm ature-reaction flux has a tendency to pass throug)
the field structure along its minimum reluctance path, i.e. along the field-pole axis or direct
V f( i
axis. Since in 26 exists because of the different reluctances along d- and <7-axes
Xd
1 V,
it is called the reluctance pow er and the term sin 26 is called the reluctance
'Xd
torque. Here to, = 2tw, and ns is the synchronous speed in rps. In view of the above, a salient-
pole synchronous motor connected to an infinite bus, would continue running as a reluctance
motor and at synchronous speed if its field current is reduced to zero. For a cylindrical rotor
machine, X(i = Xq = Xt and the reluctance power is zero, therefore, it can’t run as a reluctance
motor.
In Eq 5.59 (c), positive values of 6 correspond to generator operation where p lead s V,. For
motor operation, the power-angle characteristics are of the same shape when armature resis­
tance is neglected, except that now Ef lags V, and 5 is taken as negative.
If the synchronous machine is connected to an infinite bus of constant voltage Vh through
a transformer, a transmission line or some other reactance X, then the power for a cylindrical
rotor machine, from Eq. (5.67), is
Ef V,
P= sin 5, ...(5.70)
xk+x
For a reactance X between a salient-pole machine and an infinite bus of voltage V„ the
power, from Eq. (5.69), is given by
Ef v t . . v; 1 1 ...(5.71)
P = sin 26!
Xd + X S,n , + 2 xv+x x .+ x
Here the load-angle 6j is the time-phase angle between the bus bar voltage V, and the
excitation voltage Ef.
dp
At constant V, and Ef, the condition for maximum power is obtained by putting = ®-
Therefore, for a cylindrical-rotor machine, from Eq. (5.67),
dP E f V.
= ~lv cos 6 = 0 or 6 = 90°
do Xs
This shows that the maximum power in a cylindrical-rotor synchronous machine occurs at
6 = 90°, as is obvious from Fig. (5.77 c).
For a salient-pole synchronous machine, from Eq. (5.69),
dP VtEf a( 1 i) oK
cos 6 + V? I — - — cos 26 = 0

Scanned by CamScanner
Polyphase Synchronous M achines 631

In is equation, substitute cos 28 = 2 cos2 5 - 1 and then its solution gives,


17

cos 5 = -
W t ,Xd - X a)

KIN
* Vt(Xd - X a) ...(5.72)
Out of the ± signs, only positive sign should be used b ecan i •
lts The value of load angle 5, from Eq. (5 72) is spph ®e ne* a^vesi8n gives ambiguous
[or a sahent-pole machine, the maximum power occurs at 8 < 90° J 9°°' ThiS Sh°WS th&t a
n In order to calculate the maximum power for a salipnt ^ l ’ aSuCan Seen from Fig‘ 5/78
(5 72) and substitute it in Eq. (5.69). salient-pole machine, find angle 8 from Eq.
Example 5.27. A 3-phase, 400 V, star-connectprl «m„i, .
n infinite bus at rated voltage. The synchronous mnrhi r°n° US machine is synchronized with r
If 9 5 kW. The machine resistance is negligible an d X - 5 0 ^ ^ made t0 delwer a shaft load e
S . * — - - loss tetei fooLn, £ £ & £ :Z * C 2
s
,„) power angle, arm ature current, p fa n d draw the phasor and
(Wthe maximum power output an d the corresponding power annle
Solution, (a) Shaft power = 9500 W 8
Friction, windage and core loss = 500 W
Power developed in the synchronous motor,
P = 9500 + 500 = 10,000 W
~
Also P = 3 EfXt sin 8 + \ Vf f \ l)
sin 28
Xq Xd
v _ a)
4002 . s 1
o

10,000 = 3 ' l V
T^T5S m 5 + 2 I s sin 28
3.2 5
or \ > \
By trial and error, 8 = 11.623°

IdX d = Ef - V t cos 8 _ I(l xq


or r _ 230.95 - 230.95 x cos 11 fi9.3°
ld ~ ^-------------------- = 0.947 A

Also Vt sin 5 _ 230.95 sin 11.623°


U- Xn = 14.541 A
3.2
'1 Armature current, / . = V / J + / 2 = V0.947z + 14.5412 = 14.572 A
ig' 5,73 (b) also shows that
= h cos (8 + 0)
or
S + 0 = cos"1 ( O = cos"1 (14.541'I = 3.738°
14.572
V /
Q ^ ~1 1.0Zo = — / .OOD
^ 3 lb) nlrnS ° U^ ne&ative for a leading pf phasor diagram of Fig
’ wer-factor angle 0 is actually lagging.
Fig. 5.79. Pertaining to
The » Pf= cos 7.885 = 0.9905 lagging. Example 5.27.

P asor diagram for this part is as shown in Fig. 5.79.


r-
nil
ill;
it ;! 632 Flcctriciil M achinery
J * rt- 5.1

(b ) For maximum power developed, from Eq. (5.72),


230.95x3
cos 5 = - + (0.444)5
, ' 4 x 230.95 x 1.8 “ V 2
= - 0 . 4 4 4 4 ± 0 .8352
or 5 = 67°
.*. Maximum power developed
» 4002 4002 f 1.8
sin 67° + 3 sin 134° = 35930.2 W
3x5 3 x 2 5 x 3.2
\
/. Maximum power output
= 35930.2 - 500 = 35430.2 W.
E x a m p le 5.28. Show that for a salient-pole synchronous generator, the per-phase reactive
power in terms o f power angle 8 and for a lagging pow er factor is given by
^ Ef" "V, 9 "Vf2 2'1 1\
Q = ~y ~ cos 5 - — - V f sin2 5.
a X„*1 x drf

Also, determine the maximum reactive pow er Q, the generator can deliver with fixed excita
tion. Neglect arm ature resistance. *
Solution. The product of voltage and quadrature lagging component of armature current
gives the reactive power Q. ’
From the phasor diagram of Fig. 5.78 (6), it is seen th at Id lags oa = Vt cos6 by 90°, whereas
Iq leads ab = V, sin 8 by 90°. Therefore, the reactive power per phase is givenby
Q = Vt cos b I d - V , sin 8 I q ...
Also Vt sin b = ab = dc = Iq Xq
Vt sin 8
I* 9 =

and Er - V, cos b = a d = be =Xd Id


E f - Vt cos 8
Id - ------- ----------

Substituting these values o f/rf and I q in Eq. (i) gives,


n _ Vt cos 6 (Ef - V, cos 5) Vf sin2 5

R k . . ^
On simplification, this gives

w r, Er v t v f
Q = ~ Y ~ cos 8 - — - v f
0
■ 2 c
sin 8
Ad xd '

TheJ et ore !afromeEPq° ' 7 * > , we g e f ' W° U‘d be maitimum when *»■» * * °-
E f V, vt
V? V
max = x ^ ~ c o s 0 ~ x ~ .~ 0 = x ' l Ef - V) ...(Hi)

W° rkinB a t " l3gging Pf> the reaCtive P°wer « is obtaincd


_ EfV, , V?
<? = COS 5 - —r
X

= TT (E f COS b - V , ) „.(iv)

Scanned by CamScanner
Polyphase Synchronous Machines 633 ■I
:;i
“ *s f
«be» “" n d e ^ n Z a T ^ n V ” " ^ ^ Thia 1 “ * » * " » * *
* at u.p f. In case
,■ j i•E , cos 5 > V, T the Jl 7 * ™ P°W “ Q = ° a" d the alternator :
nperat c * - ■* ^ " he alternator is ov
altem atorlsover-excited, Q is positive-conse- ',ri
Terfy * e alternator del,vers react,ve power to the bus-bars.
If#/cos 5 < Vt, i.e. the alternator is underexcited O is now i
bsorbs or consumes reactive power. ’ 1Ve’ conseQuently the alternator
rn general, an overexcited generator or motor Drodurpc .
cystem network, whereas under-excited QvnJi, dellvers or exports reactive power
;°orts reactive power from the system network. ronous machine absorbs, consumes or im­
p E x a m p l e 5,29. A synchronous generator is running overexcited with This
;actiUe crhine with a synchronous reactance o f 1 90 n j i- . • p
to the bus. P ' dehvenn§ a synchronous power o f 0.50
P If the prime-mover torque is increased by 1% bv hm„ rrt,„u . n ~
and reactive power Q change ? ' y mUch wdl the synchronous power P
zxcita. So!U^ “" : !°vr a CylindriCal' r0tor synchconous machine thc synchronous power or the real 'iii
power is given Dy

'Trent, p = 5 lv ‘■sin 5
X. ;i;
!i(
lereas A C 1-4 x 1 . 11
i.e. U.o = ~ - - sin 5
1.2
or 6 = 25.4°
—(t)
Y/c increase in torque means 1% increase in real power.
dP = 1% of its previous value
... 1
i.e.
d p = 100 ( 0 , 5 ) = 0005 p u -
i
For cylindrical-rotor machine, reactive power from Eq. (5.45) or Eq. (ii) is,

cos 5 -
Vf
X.
dQ Ef Vt
sin 8
db
dP ErVt
But
db X . C0S ’
...(H )
= _ tan 8 = - tan 25.4° = - 0.475.
dP
5 = 0. •• dQ = - 0.475 (1%) = - 0.475%.
This shows that with 1% increase in prime-mover torque, the active power P is increased
y of its previous value, but the reactive power Q is decreased by 0.475% of its previous
...(Hi) value,
lined Example 5.30. A synchronous machine has been synchronised with an infinite bus. Now,
Without changing the field current, the machine is made to deliver real power to the bus. Will it,
e sarne time, generate or consume reactive power ?
Solution. For proper synchronization, Ef = Vt
2 V? Vf y/**
Reactive power, Q = - r r c o s S - y - = 3f (cos 5 - 1).
X. x s

Scanned by CamScanner
1
I A rt. S.
634 E le c tric a l M a ch in ery 12

Since the real power P ia delivered to the bus, theload


than one. This means Q is negative, whic s arrived at from the synchronoi^
or consuming reactive power. The same conclusion can be arrive y nronOUs
generator phasor diagram when E f= Vt.
Exam p le 5 31. A 400 V, 3 p h a se s t a r -connected synchronous m otor w ith Xd 6 n and Xq ,
n x a m p n - o .o i ./i w , y current is redu ced to zero, find th*
4 Q, is running in p a ra llel with arc 6 •f A/so com p ute the arm ature current
m axim um load that can be put ° ^ ^ esynp {he f orm ula used fo r relu ctan ce power. Neglect
an d pow er factor at the m axim um power, Prove j t

the field current is reduced to aero, excitation voltage F becomes z, 0.


Therefore the phasor diagram of Fig. 5.71 (6), for E ,= 0, gets. modified to th at shown in Fig.
5 80 Note’that the arm ature resistance r„ is neglected. In this diagram V, cos 5 is in phase with
7, and V, sin 5 is opposite to I d.T herefore, the reluctance power is give
P = V, cos 6 l (l - vt sin 5 Id jTqXq
It isalso seen from the figure that Vt sin 8 = IQXq.
V. sin 6
q X
and Vt cos 5 = Id Xd
Vt cos 5
i ,=

Substitution of the values of I d and Iq in P gives Fig. 5 .8 0 . Reluctance motor


Vt cos 8 V( sin 8 Vt sin 8 Vt cos 8 phasor diagram with ru = 0 ,
P=

1
"q XdJ
For maximum reluctance power, sin 28 = 1, i.e. 8 = 45
7-2 / , , •\ A
v [(± _ ± ( 400^1 ( I 1)
*P max = l =
CO

2 X , Xd 4 6
\ * V /
= 2220 watts per phase
= 6660 watts for 3-phases.
The component currents I q and Id, corresponding to maximum power and 8 = 45°, are

j V' sin 5 _ (231) (0-707)


X,
Vt cos 8 ^ (231) (0.707)
and
Xd 6
Armature current corresponding to maximum power
= / a = V 7 J7 7 f
2 2
fl>
(231) (0.707) V1(1)

\ ++ £ = 4 9 . 1 Amp.
4 6
Also, 'feVtla cos 9 = Pmax
or V3 x 400 x 49.1 cos 0 = 6660 W
or cos 0 = 0.1958
Therefore, power factor at maximum power is 0.1958 lagging.

Scanned by CamScanner
Art 5.12]
~ ^ £ l ^ se S yn ch ron o u s M ach in es 635
Example 5.32. The synchronous motor o f Example 5 ? /
x— Gold c u r r e n t is d p crv n v o d 1~— i ^ O .J i,
Jfthe m o to r field current is decreased, load angle 8 must i t r e T J ? * “ ^ W to 27 kW.
I t mxnimum excitation voltage an d the maximum s t a b k f a ^ ^ % de™ nd ° f the W . Find
Solution. Per phase load = — •= 7 k\y ^°0^'

±__±\
sin 28
Xq Xd
p _ v< * « -* ,
2 ' v y sin 28
or
£ ,= -

xvr' sin
■ 8*

Since P and V, are constan ts, E , shall be minimum when . „ The va|ue

Eq. (0 is d d5 ’
V, ' Vf x , - x „
TT sin 8 2 ^ 2 cos 28
Ad

V? X d - X , V,
P -~ - sin 28 T T COS 8
_

dE f 2 W
db =0
'V '
t t1 sin
• 8c

or - y - sin 8 cos 28 — * ?)- = V' x


Ad A,, P ~ 2 • ' ^ x f Sm25 77- cos 8 .
Its simplification gives
V ? * ,-* , . 3
cos 8 = — Sin 8
...(«)
For minimum Ef, Eq. (ii) gives the maximum possible value of 8 for given P and V,. The
value of 8 obtained from Eq. (ii), when substituted in Eq. (i), gives the minimum excitation
voltage Ef min.
Substituting th e valu e of P, Vt, Xd in Eq. (ii) gives,

„ (231)2 (2) . 3 K ncQ, • 3 X


C0S 5 = ~700Cr ' 24 Sm 5 = 0 634 Sm 6
its trial and error solution give 8 = 63.2° and this is the maximum stable load-angle. Sub­
stitution of this value of 8 (= 63.2°) in Eq. (i), gives the minimum excitation voltage as

7 0 0 0 - (2311 A c sin
i , (126.4°)
24
= 199 V.
Jf, nun
231
sin 63.2°

E x a m p le 5 .3 3 . F o r a 3 p h a se salient-pole synchronous motor ru n n in g as a reluctance


or. derive the follow ing relation u n d e r m axim um power conditions.
X j + X,
tan 0 =
X , - Xa
tohere 0 = pow er-factor angle of reluctance motor.

, a i ii i c u uy o a i iu u a i ii i c i
1
636 E lectrical M ach in ery __________________________________________________ —

S o lu tion . It is seen from the phasor diagram of a reluctance motor, Fig. S.80 that
Id = Ia sin (0 - 6) and I q = 4 cos (6 - 5)
4 _ cos (9 - 5) ^
or sin (0 - 5)
Phasor diagram of Fig. 5.80 also gives
V, sin b = Iq Xq
and Vt cos 8 = IdXd
x _ Iq
or tan oc — ■v
id Ad

or ~ tan 5 ...(ii)
id Aq
From Eqs. (i) and (ii), we get
Xd . K cos (9 - 5)
y tsn o . cv
Xq sin (0 - 5)
Under maximum power conditions in a reluctance motor, 6 = 45c
/ Xd _ cos 0 x 0.707 + sin 0 x 0.707
*'• Xq ' sin 0 x 0.707 - cos 0 x 0.707
Xd _ sin 0 + cos 0
or
Xq sin 0 - cos 0
(sin 0 + cos 0) + (sin 9 - cos 9)_ Xd +X q
or
(sin 0 + cos 0) - (sin 0 - cos 0) Xd - X{q
sin 0 , A Xd + Xq
■= tan 0 = v
cos 0 Xd - X q
Under maximum power conditions, reluctance motor operates at a lagging power factor.
E xam p le 5.34. A 3-phase synchronous gen erator is deliverin g a pow er o f 0.9 p.u. to an
in fin ite bus at ra ted voltage a n d a t p f 0.8 laggin g. T he g e n e r a to r h a s Xd = 1.0p.u and
Xq = 0 .6 p.u. Determine the load angle an d the excitation voltage.
In case loss o f excitation takes place, w ill the gen erator rem ain in synchronism ?
Solution. In per unit system,
Vt I a cos 0 = Power
or 1 x Ia x 0.8 = 0.9
4 = 1.125 p.u.
It is seen from the phasor diagram of Fig. 5.71 (a) with ra = 0 th at

tan (6 + 0) = + s*n e _ 1.125 x 0.6 + 1 x 0.6


v t cos 0 i x 0.8
or (5 + 0) = 57.894°
5 = 57.894° - c o s '1 (0.8) = 57.894° - 3 6.87° = 21.024°.
S° = Ia sin (8 + 0) - 1.125 sin (57.894°) = 0.9 5 3 p.u.
*‘ . Ef - V{ cos S + = 1 x cos 21.024° + 0.9 5 3 x 1.0 = 1.8864 p.u.
When loss of excitation takes place, B , . 0 and the m axim um power is then given by
i ,,2 r i n
sin 28
Xd

J
Scanned by CamScanner
_____________ * Polyphase Synchronous M achines 637

1
sin 90° = 0.333 p.u.
0.6
tx/ th loss of excitation, the maximum power that the reluctance generator can deliver to
ite bus is 0.333 p.u. As this is less than 0.9 p.u., the generator will lose synchronism.
Synchronizing P o w e r and Synchronizing Torque
5 l^The rate at which synchronous power P varies with 5 is called the synchronizing-power
ff rient P s r lt *s a^SO kn°wn as stiffness of coupling, rigidity factor or stability factor. For a
^ical-rotor machine, from Eq. (5.67),
cyl p dP Ef Vt
P™ = ~ 77 =
db
COS 8 ...(5.73)

For a salient pole machine,


P = ^ = M . cos5+v*
cos 28 ...(5.74)
'o ' db Xd ‘ Xd
The coefficient P^ is equal to the slope of the power angle curve and its variation with 8 is
illustrated in Fig. 5.81 for both types of machines.
The synchronizing-power coefficient is a measure of the stiffness of electromagnetic cou­
ntingbetween stator and rotor fields. Too large stiffness of coupling means that the motor tends
to follow closely, the variation of speed caused by the disturbance in electric power supply. In
case there is no power-supply disturbance, then too much stiffness of coupling would cause the
motor speed to remain practically constant, regardless of the mechanical load fluctuations. In
view of this, too rigid electromagnetic coupling causes undue mechanical shocks, whenever
there are fluctuations in the mechanical load or the supply. Examination of Eqs. (5.73) and
(5.74) reveals that Psy is inversely proportional to Xs or Xd and is directly proportional to the
excitation voltage Ef. Consequently an overexcited synchronous machine is more rigidly
coupled than the one which is underexcited. Large air-gap decreases the value ofXs or Xd, thus
asynchronous machine with longer air-gap is more stiffer than the one with smaller air-gap.
Note that the units of synchronizing-power coefficient are watts per electrical radian. Now
one electrical radian = 180/rc electrical degrees.
K E r Vt
p
1 sy = cos 8 watts/electrical degree.
180

sy

S ta b le . i Unstable^

Power]

Fie. 5.81. Sy rch ro n o u i-p o w e r coefficient variation w ith 8 for


(o) cy lin d rical rotor m achine and (b) aalient-pole m achi .

>canned by CamScanner
638 E le c tric a l M a ch in e ry _________________ ______ ________________________

It should be noted that one electrical degree in space is smaller than one mechainical degree
in space, for machines with more than two poles. For examp e, in a p , one electri­
cal degree in space is equal to 1 of one mechanical degree in space, n gener ,

One electrical degree in space


= ~ (one mechanical degree in space).

p F V
... p , =—- . cos 8 watts/mech. deg. ...(5.75)
360 X8
Synchronizing-pow er coefficient gives rise to synchron izin g torq u e coefficients at
chronous speed. If is the synchronizing torque coefficient, then
l dP .
Tsv = — m “ 77 Nm/elect. rad.
ay o)s db

^ = ■ 3 M Nm/meCh deB- ■■,5 -76»


where m = number of phases
and cds= 2n ns and ns is the synchronous speed in rps.
Fig. 5.81 shows that Pay is positive for stable operating region and is negative for unstable
region. Thus the criteria of steady-state stability of a synchronous m achine is that the
synchronizing-power coefficient Psy or Tay should remain positive. The magnitude of Psy or Tsy
indicates the degree of steady-state stability ; e.g., when 8 is near to zero degree at no-load,
Psy has a large value and, therefore, the degree of stability is high. As the value of 8 increases,
Psy decreases and, therefore, the degree of stability is reduced.
Now consider a synchronous machine in which the load-angle has changed from 8 to
8 + AS due to some transient disturbance. The variation of synchronous power associated with
the change of load angle A8, is called the synchronizing power P s. From Eq. (5.73), the
synchronizing power is given by
_ dP PE fVt
P s= db' X ~ cos 5 A5 ...(5.77)
E fVt ( i
„m i \
Also P. = ^ - c o s 8 + V ? [ — - — cos 28 A8 ...(5.78)

Eqs. (5.77) and (5.78) give the synchronizing power for a cylindrical-rotor and salient-pole
machines respectively, for a phase displacement of angle A8.
At no load, 8 = 0 and V, = Ef. Therefore, the synchronizing power at no load, from Eq. (5.77),
is
vf
P* = Y s * b ...(5.79
n . Ec Vt
cu t x = ^ ' = steady-state short circuit I K.

= v t he A8 watts (5 79 b)

i n t o t ^ r ^ 0" reVeaiS; ha‘ S? ch™ni2mg P O '"* is tran sien t in nature, i e. i t comes


case Y f Ud,den d,sturbance in the steady-state operating conditions. In
to How from ^ in the h" dl3tUrbr C! m pnm e' T Ver tor<>ue causes the synchronizing power
£ « h ? L n c ; th . ' ° rest0re the rotor t0 its previous position. A sudden
disturbance in the generator or motor field current, also causes the synchronizing power to

Scanned by CamScanner
Art . 5 .U I P olyph ase S y n ch ro n o u s M a ch in e s 639

come into play, so as to maintain the synchronism. In case of motor, a sudden disturbance in
shaft-load torque causes the synchronizing power to flow from or to the motor, so that rotor is
brought back to its initial steady position. In short, the synchronizing power comes into play
only when the steady-state operating conditions are disturbed. Once the steady state condition
is reached after the disturbance, the synchronizing power reduces to zero. Thus the synchroniz­
ing p°w e r*s t ransient in nature and exists only for the time during which the disturbance
persists. The synchronizing power flows from, or to, the bus, in order to maintain the relative
velocity between interacting stator and rotor fields zero— once this equality is attained, the
synchronizing power vanishes.
The synchronizing power gives rise to synchronizing torque Ts and its magnitude for a
small displacement A8 is given by
rr 1_ p _ 1 dP _

= T,y ■46 ...(5.80)


It may be seen from Eqs. (5.73) and (5.77) that synchronizing power coefficient Psv is equal
to the synchronizing power per unit of phase displacement, i.e., = P sV Ab. Similarly
Ts = Ti7 •Ab.
If armature resistance is not neglected for a cylindrical-rotor machine, then from Eqs.
(5 32) and (5.34), the synchronizing power is given by

*P e “. Mr-m cos (5 + a,) AS ...(5.81)

where a 2 = t a n '1

5.13.1. Physical con cep ts of synchronizing power. Consider a synchronous motor


connected to an infinite bus with terminal voltage V,, armature current /„, excitation voltage
Ef and steady-state load angle 8. The angle 8 depends on the load on the motor shaft.
Due to some transient disturbance, let the load angle increase from 8 to 8 + A8. This distur­
bance causes the rotor to shift back by an angle A8from its previous position. Since the phasor
depends on the rotor position, this small departure A8 causes the phasor E ,to shift back from
its steady-state position OA to new position OB. The voltage
phasor V, is not affected by transient disturbance, because V,
is the terminal voltage of infinite bus. The small departure A8
BA=ES
from OA to OB, causes the appearance of synchronizing volt­
age Es = BA, see Fig. 5.82.

E s = BA = 2 Ef sin y
When E^shifts back, 7 ^ drop increases to7a, Xs, i.e. arma­
ture current increases from Iu to Ia\ and this gives rise to
synchronizing current 7S, such t hat 7„i = 7„ + 7t. Remember
that it is the synchronizing voltage E s that causes the ap­
pearance of synchronizing current 7S.
Es ^ 1 . A8

/. - 3 F - 2* - 5 “ n T
Note th a t /, lags E, by 90° because only X, is considered. Fig. 5 .8 2 . P ertaining to the physical
concept of synchronizing power.

‘ In case th e load an g le ch a n g e s from 5 to 5 A6 very rapidly, X, should be rep laced by X,{ or X,(\ d ep ending
upon how fast A8 c h a n g e s w ith tim e.
640 Electrical Machinery

s AS'l
Now the angle between Vt and synchronizing current Is is equal to 6 + t Therefor,^ ^

synchronizing power flow from bus to the motor is


fc A8^
p a = y t i a cos 5 + t

2E f 8A
or P„ = Vt' sm — •cos

For small disturbance A8,


A8 A8
sm

AS^
and cos = cos 5.
Y
E f Vt dP
:. Synchronizing power, Ps = ^ ■cos 8 •A8 = A6 - P sy ■A8.

In case load angle decreases from 8 to 5 - A5 due to some disturbance, then synchronizing
power would flow from motor to infinite bus.
E x a m p le 5 .3 5 . A 2 0 MVA, 3 -p h ase star-con n ected 11 kV, 12 p o le, 50 Hz sallmt-pok
synchronous m otor, w ith negligible arm atu re resistance, h a s reactan ces o f = 5 0 . and Xq = 3
O. At fu ll load, unity pow er-factor a n d rated voltage, com pute :
(а) The excitation voltage.
(б) P ow er (and check the result).
(c) Synchronizing pow er p e r electrical degree a n d the correspon din g torque.
(d) Synchronizing pow er p er m ech an ical degree a n d the correspon din g torque.
(e) M axim um value o f pow er-angle 8 a n d the correspon din g pow er.
S o lu tio n . Per phase terminal voltage,

V, = = 6,350 V.

P er phase arm ature current,


j _ 2 0 ,j )_0 0 2049 A
a V3 (11) 9 A-

is drawn in Fig. 5.83. With Vt as reference phasor,


Vt = 6350 + j 0.00
Ia = 1049 + j 0.00
/. j I a Xq = j (1049 + j 0.00) (3) = 0 + j 3147.
From the phasor diagram,
oa = E f = Vt - j l a Xq
= 6350 - j 3147 = 7090 / - 2 6 .4 °.
5.35.
Load angle 8 = 26.4°.
Now *d = Ia sin 5 = (1049) (0.444) = 466 A.
The excitation voltage Ef is the arithm etic sum of oa and a b
E f = oa + ab

Scanned by CamScanner
Polyphase Synchronous M achines 641

= E f + Id (-%d “ Xq) = 7090 + 466 (5 - 3)


= 3022 volts to neutral.
EfVt . _ v- t2t(xd-xA
P = X ~s ~2 XdXQ sin 28.

5 (2) (5) (3) sm sz -8


= 6670 kW per phase.
For three phases, P = 6670 x 3 = 20,010 kW, which is almost equal to the specified 20,000
w The small difference springs from the rounding off the calculated values.
(c) Synchronizing power per electrical radian is given by
dP Ef v t o ^ ( X d -X „ )
y ~ c°s 8 + Vf — - q- cos 28
db = Xd X#,
(8022) (6350)
cos 26.4° + (6350)2 cos 52.8°
15
= 12,370 kW per phase = 37,110 kW for 3-phases.
• Synchronizing power per electrical degree
= (37,110) (n)
180 /KW
and the corresponding synchronizing torque,
647 x IQ3 _ 647 x IQ3
T =
cp, 27m,
But ng= Synchronous speed in rps
_ 2 f _ 100
P 12
„ _ 647 x IQ3 x 12 XT
" s 2n(100) - 12,360 Nm.
(d) Synchronizing power per mechanical degree
P 12
= 647 x - = 647 x y = 3882 kW.

and the corresponding synchronizing torque


= (3882) (10)3 (12)
2 x (1 0 0 ) ’
(e) At constant Vt and Ef, the maximum value of power angle is given by Eq. (5.72), i.e.

cos 8. = E fL—
Xo2-----
4Vt (Xd - X q)
+c 4Vt (Xd - X q)

= - (8022) (3) JV ~ (8022) (3)


= 0.3775.
4 (6350) (2) 4 (6350) (2)
Thus the maximum value of power angle is cos 1 (0.3775) = 67.85°.

m (8022U63501 gin 6? 85o + ^ ^ sin 135.70°

= 11,325 kW per phase


= 33,975 kW for 3-phases.

Scanned by CamScanner
IP

642 E lectrical M achinery [Art^s^jj

Exam ple 5 .3 6 . A 3000 kVA, 3-phase, star-connected, 6,600 volt, d-pole, 50 Hz alternator ha
a synchronous reactance o f 20% an d is running in p a r a lle l w ith infin ite bus. Calculate th
synchronizing p ow er a n d the corresponding synchronizing torqu e p e r m ech an ical degree \
p h a se displacem en t (a) a t n o-load a n d (b) a t fu ll load, 0.8 p ow er-factor lagging.

Solution. Per phase voltage, Vt = - 3810 V.

Per phase armature current, Ia = ^ 3^ 5500^ = A'


Xs in ohms
Percentage reactance, Xs = ---- y~/j ----- x

„ . . 20 3810 OQn
- ^ inohms = l 0 0 X '2 6 2 ’ = 2 -90-
(a) At no load, 8 = 0 and Vt = Ef.
Synchronizing power per mechanical degree,
D_ d P 11P „ Vt Ef R nP
s ~ m ' d b ' 360 X, 008 ' 360
= 3 I3810)2 g(82
2.90 360 1U4H KW-
The corresponding synchronizing torque is
rp _ ^0 D _ P&
8 2*AT, ' “ 2wi,

= 2 ^ 0 0 0 ) (1048) (1° 3) “ 13'340 Nm '

° Perati” g ** 3 P° Wer faCtor is “ %

Ef = 3810 + j 262 (0.8 - j 0.6) (2.9) = 4 3 1 0 Z 8 .1 1 °


The synchronizing power per mechanical degree is
p _q kP Er Vt
360 X ^ 005 8
_ Q 7t x 8 (4310) (3810)
" 360 29 Z cos 8.11 = 1173 kW .
The corresponding synchronizing torque is
m _ 1 n ______ 8
5 2nns 8 ~ 2 k (100) ^1173 x 10 ) = 14,930 Nm .
Example 5.37. A star-connected 8 pole 50 •*.». u
ice of 11 Q per phase. On no-load both the ■* Qbronous motor has synchronous reac-
>kV. Neglect all losses. ’ ^ 6XCltatwn and th* terminal voltages are equal to
(a) For a sudden disturbance in the shaft-1nnd ,
-hanical degree. Calculate the synchronizer, e rotor fa^s back in space by one
torque tending to restore the rotor to it* nr*gCUrrent\synchronizing power and synchrony
fb) With gradual increase in the shaft .
ee. Calculate the armature current Fnr * * rotor slides back in space by one mechanical
lanical degrees, through which the rotor s l f a b a c k ™ °f 3 ° A ftnd the ^ *

Scanned by CamScanner
Polyphase Synchronous Machines 643

For this example, refer to Fig. 5.82.


S o lu tio n s
{ ) At no load, 6 _ .
. ^andV, are in phase.
0 n e m echanical degree in space i .

= | electrical degrees in space

. sy n ch ron izin g v o lta g e

BA = E, = 2E{ sin ^ = 2 (3300) sin | = 230 V.


Z Z

Synchronizing current in the armature,

Synchronizing power, Ps —m Vt Is cos R A5'


8+T
= 3 (3300) (20.91) cos (0 + 2°) = 206.9 kW.
2x50
Synchronous sp e e d , ws = 27tn8 = 2n x — — = 25 it rad/sec

S y n ch ro n izin g torque,
(206,900) = 2634 Nm.
T =
25n
(6) When the rotor slides back by one mechanical degree, then l a X3 drop becomes equal to
230 V and the armature current increases from zero to 20.91 amperes.
For an armature current of 30 A,
o2Ef
r sin
■ —A5
2 x 3 3 0 0 . A8
4 = 30 =
11 Sm 2
A8 330
or sin — = •=0.05.
2 2 x 3300
A5 = 5.732 electrical degrees.
5.732 x 1 = 1.433 mechanical degrees.
:. The rotor slips back by

5.14. Synchronous Machine Stability


Any machine system is said to be stable if, under steady state, it is operating in equilibrium
and a sight disturbance does not effect the stable operation. The term equilibrium denotes
perfect power balance between input and output powers.
A synchronous machine connected to an infinite bus is said to be working in a stable condition,
if it is in synchronism or in step with the bus. Unstable operation denotes loss of synchronism or
falling out of step.
v. t

Stability. The tendency of a synchronous machine to develop forces so as to maintain


synchronism and equilibrium, is called stability.
Stability lim it. A stability limit represents the maximum power flow possible, when the
synchronous machine is operating with stability. .

Scanned by CamScanner
644 E lectrical M ach in ery_______________________________________________________________

The terms stability and stability limit are applicable both to steady
conditions.
Steady state stability limit. It pertains to the maximum power flow possible throupw
particular point without loss of stability, when the power is increased very gradually.
Assume that the load on synchronous motor is increased gradually, until point C o,- ^
mum power is reached. If an additional increment of shaft load is put on the motor, the )0J
angle 5 would increase as demanded by the load. But with an increase in 5 beyond thatgiv*
by point C, the electric power input decreases, as is obvious from the shape of the power-ariei‘
curves in Fig. 5.84. Since, with the increase of shaft load, the electrical power input h3;
decreased, the motor gets retarded more quickly. As a consequence of it, the angle 8 increase
further and the motor pulls out of step or loses synchronism. Thus Pm given by the point C*
the steady-state (or static) stability limit—the region to the left of point C is stable one.

(a)

Fig. 5.84. Power-angle ch aracteristics of (a) salient pole and ( 6 ) cylindrical


rotor synchronous m achines showing stable and unstable regions.
The steady-state limit can be improved upon as follows :
(i) Synchronous machine excitation should be increased, so that becomes more.
(ii) Reactance should be decreased, e.g. two transmission lines in parallel would reduce the
line reactance to half.
(m) Series capacitors may be used to reduce the line reactance.
T ransient stability limit. It refers to the maximum power flow possible through a point
without loss of stability, when a sudden disturbance occurs. Sometimes a transient distur­
bance, such as a sudden increment of load, may cause the loss of stability because of its sudden
application, but may be carried without loss of synchronism if it were applied gradually. Usual­
ly the disturbances for which the transient stability limit is desired to be known, are caused by
a fault, or by switching in or out some circuits in a supply system, or by a sudden increment of
shaft load.
Transient stability limit is more important since
(i) it is lower than steady state stability limit and
(ii) the transient disturbances are more common.
The object of studying the transient stability of a synchronous machine is to find whcthj
or not the synchronism is maintained, i.e. whether or not the load angle 5 settles down 0
steady operating value, after the machine has been subjected to a large disturbance.
The method of determining the transient stability of a system is known as the eqmd
criterion and has been dealt with in detail in the companion volume, in the chaPte
synchronous machines.

Scanned by CamScanner
P o ly p h a se S y n c h ro n o u s M a c h in e s 645

ct a d y -sta te s ta b ility lim it on p o w er c irc le s . The


j state stability limit can be shown on the power circle
s am also. In Fig. 5.85, two circles representing powers P ,
are drawn. F irst consider the points to the left of line
n'Vor any one value of excitation, say
vCV' Ef
BD = 2 ^,

the armature current increases from Ial to 7fll' as the shaft load
is in cre a sed from P x to P 2 (P2 is smaller radius). Now 7al' cos 0,'
jg greater than 7fll cos 0!. Thus, with an increase in shaft load
from?\ p 2>the electrical power input and load angle increase
respectively from Vt 7ol cos Ql to Vt 7al' cos 0 / and from 8, to 5 /
tiiis indicates a stable operation.
On the right-hand side of line DCD', with increase in load
from Pi to P 2, the arm ature current decreases from Ia2 to Ia{ . Fig. 5 .8 5 . S te a d y -s ta te sta b ility
lim it on pow er circle s
Since electrical power input has decreased from Vt Ia2 cos 02 to
Vt Ia2 cos and the load angle has also decreased with the increase in shaft load from P x to
P2t the motor oDeration
d motor operation is unstahlo The
is unstable. Tho lline n r r t ’ thus
i n o DCD' fk.ic. corresponds to *the i ___stability line— the

region to the left of it is stable and to the right of it, unstable.

In Fig. 5.57 for V-curves, the curve A'QB' corresponds to the line DCD’ of Fig. 5.85. In Fig.
5.57 also, the portion of the O-curves above A'QB' represents unstable operation and can’t be
obtained experimentally. In Fig. 5.57, if the load increases from P l to P 2, the arm ature current
increases from LM to L N in the region below A'QB' whereas it decreases from L S to L K in the
region above A'QB', thus verifying the results obtained from power circles.
For a given excitation, the maximum power output of a synchronous motor can also be
obtained from Fig. 5.85,

where CD = C B + B D = CB +

Here CB is the radius corresponding to the maximum power output for a given excitation
6CJU3.1 to

Ef
b d - ±
Radius for m axim um power circle

Ef f yt max
= CB = CD w J 2r

z.
V
V 2r, max Er fv ,
= CD~ z r
.E i
z.
V " >
f Vt * p max f yt
2r„ ra 2 r0
T V v
simplification gives
Ef Vt
p max = -fir
Z] "*

canned by CamScanner
(Art.
646 E lectrical M achinery 5.14
Thr» result is the same as obtained earlier in Eq. (5.37). . - . , .
lhc result is p Qnd ra, can be obtained, again by ref6,
hi rin g ^ o V T g T srifC B ^ th e'rad iu s for any pow’er P, then the minimum excitation Possib,eis
DB, i.e.
d b =d c - bc
2
v, •> _P
or E f nun ~ •(5.82)

to
^1
ra

I----
Q
K
Example 5.38. A 15 kW, 400 V, 50 Hz, 3-phase, Y-connected synchronous motor, with per pha#
values o f resistance 1 Cl and reactance 5 Cl, is working at rated voltage and at rated load :
(а) Find the load angle, arm ature current a n d power factor o f the motor, when the excitation
voltage is adjusted to 480 V.
(б) With the excitation voltage unchanged, determine the m axim um load the synchronous
motor can carry and the corresponding values o f load angle and the arm ature current.
(c) For a total load o f 12 kW, find the minimum excitation voltage required.
15
Solution, (a) Per phase power, P = — - 5.00 kW.
3
400
Per phase terminal voltage, Vt = ZJ j f = 231 V.
V, 231
Radius of zero power circle = 115.5 A.
2ra 2x1

Radius of 5 kW power circle =V E T F = 9 i.A A.


\
480
Per phase excitation voltage, E f = —rj- = 277 V.

Synchronous impedance Zs = Vl2 + 52 = 5.1 Q.


Vt 231
= 45.3 A
5.1
277
and = 54.3 A.
Zs 5.1
Scale 10 A = 1 cm.

Draw the zero-power circle with radius CO = 115.5 A and cut OD = — = 45.3 A. This gives
S
ZCOD = 02 automatically. Now draw a circle with CQ = 91.4 A as radius, representing 5 kW.
• E
With D as centre, draw an excitation circle with D F = = 54.3 A as radius, intersecting the 5
8

kW power circle in F . This gives arm ature current Ia = OF = 2.6 x 10 = 26 A, p.f. = cos ZFOC
= cos 17.2° = 0.955 leading, load-angle = ZODF = 28.4°.

C h e ck . Power developed = Vt Ia cos 0 - / 2ra

= (231) (26) (0.955) - (26)2 x 1


= 5050 W per phase.

j
Scanned by CamScanner
Polyphase Synchronous M achines 647

With the excitation fixed, the circle


maximum power, has a radius of
-ving in it
>
CG-
CG = 6.10 cm = 61.0 A
.
p Iv
/(O 2
max i i
61 , A
w
This gives P ^ = 9620 W per phase kv
= 28.860 kW for 3-phases. P= i'. ii
5-00kW
for the development of maximum power,
the armature current = OG = 63.5 A, load angle 1.V
ODG = 78.7°. Note here that 0Z= 8 = 78.7°.
power factor is cos ZCOG = cos 21.4° = 0.931 P=0
lag, see Fig- 5.86. f’i
Check. From Eq. (5.37),
Fig. 5.86. Construction of power-circle diagram ,
Example 5.38.
p
r mas
- M£ 2 ra -jl'i '
*»!:'
j
■iili
_ (277) (231) '211)
x 1 = 9580 W per phase.
5.1 5.1 Vr,
;V' .

12
(c) Per phase load P = — = 4 kW I
From Eq. (5.82), the minimum excitation voltage is
2
V, \ fv .l P Ii-
ICM
1 U5
1

E f mm = E ,
ra
l 2S !!•
-
i

= 5.1 115 . 5 - ^ < 1 1 5 . 5 ) 2 - ^ P = 96.14 V.

5.15. H unting and D a m p e r W indings


In this article, physical concepts of hunting in a synchronous machine are described first.
Then the use of damper bars in damping out these rotor oscillations, or hunting, is presented.
5.15.1. H u n tin g . A synchronous machine operates satisfactorily, if the mechanical speed
of the rotor is equal to the stator field speed, i.e. if the relative speed between the rotor and
stator fields is zero. Any departure from these conditions, gives rise to synchronizing forces,
which tend to maintain this equality.
Unloaded synchronous motor operation is illustrated in Fig. 5.87 (a) and (6), where all the
losses are neglected and load angle 5 is assumed zero at no load. The rotor structure in Fig. 5.87
(a) is shown different from its actual construction, merely for simplicity in showing the rotor
filia tio n s . In the phasor diagram of Fig. 5.87 (6), Xd andX , are assumed equal for con­
venience,
^ the shaft load is put on the synchronous motor in small steps, the load angle would in ­
crease gradually from zero. For any shaft load Plt the load angle would be 5! (say), arm ature
current would be Ial, this is illustrated in Fig. 5.88 (a), (6) and (c). U nder steady state,

Scanned by CamScanner
648 E le ctrica l M ach in ery

4vt
r S ta to r j'a Xs-
Ef
Rototing
a ir-g a p flu x

5 -0

(6 )
W Fig. 5 .8 7 . Unloaded synchronous motor operation (a) its physical in te rp re ta tio n and (b ) its phasor diagram.

P l = ?fx Vi sin 8j = Pm sin b, and the operating point is ‘a 5 as indicated in Fig. 5.88 (a), (b) m

(c). This operating point travels from ‘o’ to ‘a ’ as the load is gradually increased to P h Fig. 5 gg
(c).
Now suppose the load P x is applied suddenly to the unloaded motor shaft, instead of
gradually. Now the motor must slow down momentarily {i.e. the rotor speed must become less
than synchronous speed) in order to supply the load P^ As a result, load angle starts building
up from zero degree. As soon as is first reached during its forward swing, electrical power
developed P,„ sin becomes equal to shaft load P 1( but equilibrium is not established, since the
D i r e c t io n o f
ro ta tio n

F irst f o r w a r d
.-swing s ’ sw in g
nLn M
N 1 I Nr - N
/£ 1
J
/Nr <Ns f 1 D eceleration

0 S, 2 6)

(°) id)
Fig. 5.88. Illustrating the rotor hunting in a synchronous motor.

Scanned by CamScanner
— ________________ Polyphase Synchronous Machines 649
■is
peed is less than synchronous speed, i.e. Nr < Ns. In order to boost the rotor speed to Ns,
^ o t o r swings further. As soon as load angle exceeds 8X, Pm sin 8 > P x. In other words, now
^ectrical power input Pmsin 8Xhas exceeded the shaft load Plt therefore the rotor gets
,orated. At some angle 8 3 - 2 81, the rotor attains synchronous speed and the current increases

tfilafc jXs
-^e operation at load angle 2 8t is indicated by point ‘6’ in Fig. 5.88 (a), (6) and (c). Note that for
rotor travel from o to a , the rotor decelerates ('.' Pm sin 8 < P{] and from a to 6 , the rotor ac­
celerates (V Pm sin 6 > Pi). At load angle 2 8 b Nr =NSbut Pmsin 8 is still greater than Plt the rotor T.
therefore continues to accelerate even above synchronous speed. The effect of rotor speeding up above
synchronous speed causes the load angle to decrease from 2 5^ After some time, load angle decreases
§j, thoughPmsin §1 = P 1at this angle, the equilibrium is not yet established, because now the rotor
speed is more than synchronous speed, i.e. Nr > Ns. This operating point at angle 8Xduring the first
backward swing is indicated by point ‘a ’ in Fig. 5.88 (a), (b ) and (d). As the rotor speed is above Ns>
the rotor continues its first backward swing below 8V As soon as load angle becomes less than
gif p1> pmsin 8, rotor therefore gets decelerated. Under the assumption of no losses and no damp­
ing, the rotor would attain synchronous speed during its first backwardswing only at 8 = 0 as
— V —E
indicated by point ‘o’ in Fig. 5.88 (a), (6) and (d). At zero angle, Ia = —. f as shown in Fig. 5.87
J As
(b) and 5.88 (6).
At zero load-angle, P msin 8 is zero, as a result shaft load Px slows down the rotor, the load
angle begins to rise during its second forward swing from zero to 8Xand then from to 2 as
before if there were no damping. In this manner, the rotor swings or oscillates first to one side
and then to the other side of the new equilibrium position or new space-phase position of 8Xas
shown in Fig. 5.88 (a). Note that the new equilibrium position of load angle is given by
= sin" 1 (Pi/Pm). This phenomenon, involving tne oscillations of the rotor about its final equi­
librium position, is called hunting. Fig. 5.88 (6) reveals that during the rotor oscillations or
hunting, the orientation of phasor E f changes relative to fixed voltage Vt and because of this
reason, hunting is also called phase-swinging. Fig. 5.88(a) depicts the internal happenings of
howrotor hunting occurs and how load angle 8 varies from zero degree to 2 and back. In Fig.
5.89 (a) is shown the variation of load angle 6 with time in case the motor system has no damp­
ing. Fig. 5.89 (6 ) shows the variation of load angle 8 with time when damping is present in the
system. A real physical system does possess inherent damping. As a result, the rotor of
synchronous motor eventually settles down to stable operating point with a load angle 5^ The
w°rd “hunting” has been used here, because after sudden application of load, the rotor has to
search for, or hunt for, its new equilibrium space position.

T im e
Time

(a) ^
pig- 5.89. V ariation of load angle 6 , after sudden loading of an unloaded synchronous m otor with
(a) no damping and ( 6 ) damping present.

it 11
>cannea oy uam bcanner
650 E le c tric a l M ach in ery
[Art 5.1

From the phasor diagram of Fig. 5.88 (6), it is seen that hunting is associated with nou,
E V ^
Vt Ia cos 0 or sin 6 and current pulsations, which can be observed in the laboratory with

the help of wattmeter and ammeter. The rotor hunting can also be observed in the laboratorv
by means of a stroboscope light falling on the rotor shaft. At normal synchronous speed th
rotor appears stationary.
In an alternator synchronized with infinite bus, if the gate-opening in case of hydroelectric
power stations (or steam-valve opening in case of thermal power stations) is decreased sudden
ly, the alternator will slow down momentarily thereby decreasing the load angle. Rotor oscilla
tions or hunting will follow before the Final equilibrium space position is reached.
Hunting is objectionable, particularly when the synchronous machine is coupled with a
system whose torque variations contain harmonics, e.g. air-compressor, reciprocating engine
etc. If frequency of the torque component happens to be equal to that of the frequency of free
oscillations of synchronous machine the latter may fall out of step. The other bad effects of
hunting are as follows :
(i) It produces severe mechanical stress and fatigue in the shaft.
(it) It causes great surges in current and power flow.
(tit) It increases machine losses and thus the temperature rise of the machine.
The various causes of hunting may be
(а) a sudden change in load,
(б) a fault in the supply system,
(c) a sudden change in field current,
(d ) a load or drive containing harmonic torques.
The undesirable phenomenon of hunting, can be guarded against in three ways :
(i) by using a flywheel,
(ii) by designing the synchronous machine with suitable synchronizing power coefficient or
stiffness factor and
(m ) by the employment of damper or amortisseur windings and this is discussed below in
detail.
5.15.2. D am per windings. Damper windings consist of low-resistance copper, brass or
aluminium bars, embedded in slots in the pole-faces of salient-pole machines. The projecting
ends of the bars are connected to short-circuiting strips of the same m aterial as used for the
bars. Sometimes interpolar connectors are omitted to form incom plete type of damper winding
as shown in Fig. 5.90 (a). When strips on both sides of the pole shoes are joined by interpolar
connectors as in Fig. 5.90 (6), com plete type of damper winding is obtained Fie 5 90 (c) shows
.D a m p e r b
bars-?
Damper
bars-

(a) <b) (c)


Fig. 5 .9 0 . D am p er w indings (a ) incom plete type. ( 6 ) and (c) com p lete type.

Scanned by CamScanner
A rt 5.15]
Polyphase Synchronous M achines 651
\ how strips are interconnected by intemnlar ,
windings are of two types : onnectors. It is seen from above that damper
N
(i) Incomplete, non-connected, or open type, Fig. 5.90 (o).
(u) Complete, or connected type, Fig. 5 90 (h)

K am° rtiSSeUr CirCUit- * * « " Of this, damper winding is also


Ici\
nrhin^rnrovide^ulthfn n°r,t ,USed on turbo-generators. But the solid-steel rotor cores of such
>tk r* m an pnui valent rntn 8 K Cl!rrents>especially in the quadrature axis, where the iron may
5 circuit, thus producing the same effects as those of damper bars.
amper a^s niay orm the starting winding for synchronous motors of the salient-pole
free
^ e' 0 r ^00 s ar m 2 orque, damper bars should have high resistance. But for large damping
ts of
effect near synchronous speed, the damper bars should have low resistance. Therefore, a com­
promise be ween good starting torque and good damping effect should be made, for obtaining
satisfactory operation of the synchronous motor. However, such a problem does not exist in
alternators, because the purpose of damper bars in them is merely to damp out rotor oscilla­
tions, i.e. hunting. Therefore, low-resistance damper winding can be used in alternators.
It should be noted that when the rotor is running at synchronous speed, the relative speed
between damper bars and rotating air-gap flux is zero. Because of zero relative speed, no flux-
cutting action takes place and e.m.f. generated in damper bars is zero-consequently no damping
torque is developed. The damper winding comes into play only during rotor hunting, when rotor
speed departs from synchronous speed.
In order to understand how the damper bars damp out hunting, assume the rotor speed to
become 10 r.p.m . less than the synchronous speed of 1500 r.p.m. (say), Fig 5.91 (a). Under this
assumption, the relative velocity between rotor and air-gap field is 10 r.p.m.. Fig. 5.91 (6 ). If S
pole is assumed on the stator near the damper bar, an e.m.f. shown by a dot is induced in the
bar, Fig. 5.91 ( 6 ). This bar current sets up its own flux lines which interact with the S pole to
or produce a torque in the direction of rotation as shown in Fig. 5.91 (c).
The effect of this torque is to accelerate the rotor, tending to make the two speeds equal.
This torque in the direction of rotation, is known as induction motor torque and is produced
whenever the rotor speed falls below the synchronous speed.
1500 rpm

lOrpm STATOR
surfaced
t STATOR
SURFACE

— y
ROTOR DAMPER L ROTOR
SURFACE BAR DIRECTION*) ^ ^ ) SURFACE
nc
OF
E0RCE

(6 ) (c)
(a) Fig. 5.91. Pertaining to the production of torque due to damper bars.

/ „ ti.ni rotating 5 pole becomes stationary) from left (L) to right (R) on both
--------------------------------------------- *
Superimpose a velocity of 10 r.p.m s condition, S pole comes to a standstill but damper bar moves
S pole and damper bar in Fig. 5.91 (H U n to «h«eo determinjnE ^ djrecti„„ f f the daraper bar
from Lot Rta 10 r.p.m. Now apply n f j Ikon fmger (near lhe thumb » in the direction of
According to this rule, thumb is in the dnectic ^ ^ f Kote lhat thumb, first finger are m the same plane but
flux lines and middle finger glve3 tt\ . h finger is normal to the plane of thumb and first finger,
normal (ie. perpendicular) to each other , in

J I II IVyI
1

652 E le c tric a l M a ch in e ry

Likewise, when the rotor speed becomes g reater than synchronous speed, induction
rator torque (against the direction of rotation) is produced, which tends to retard the rotor 1*
make the two speeds equal. In other words, when the rotor speed falls below the synchro*. ri‘
speed, the slip becomes temporarily positive, induction m otor action comes into play and ^
is accelerated—when the rotor speed exceeds synchronous speed, the slip becomes temporar,
negative, induction generator action takes place and rotor is retarded.
Thus, when the rotor speed departs from the synchronous speed, the damping torn
(motor or generating torques) are brought into play to m ake the relative speed between rot!'
and stator fields as zero. The magnitude of these damping torques is approxi mately proportio^
al to the slip speed, provided the slip is small.
5.16. M easurem ent of Xd and Xq
The d-axis synchronous reactance is determined from o.c. and s.c. tests, as illustrated h
Ex. 5.25. The q-axis synchronous reactance can be m easured by the following three methods
(a) Slip te st. From this test, the values of Xd and Xq can be determined. The synchronou
machine is driven by a separate prime-mover (or motor) a t a speed slightly different fro
synchronous speed. The field winding is left open and positive sequence balanced voltages "f
reduced magnitude (around 25% of rated value) and of rated frequency are impressed acro°
the arm ature terminals. Under these conditions, the relative velocity between the field do]S
and the rotating arm ature m.m.f. wave is equal to the difference between synchronous soepd
and the rotor speed, i.e. the slip speed. A small a.c. voltage across the open field winding indi
cates that the field poles and rotating m.m.f. wave, are revolving in the sam e direction-and
is is what is required in slip test. If field poles revolve in a direction opposite to the rotating
m.m.f. wave, negative sequence reactance would be m easured

Reduced
d-axis

Ja=ld
ita

q-axis
(a) d-axis
Fie- 5.92. Pertaining to the physical (h)
con cep ts of ( a ) X d an d (6 ) A'w.

Scanned by CamScanner
1

Polyphase Synchronous Machines 653


\ After one-quarter of slip cycle, the peak of armature m.m.f. wave acts on the interpolar or
s
q.axis of the magnetic circuit, Fig. 5.92 (6) and the reluctance offered by long air gap is maxi­
\ mum. At this instant, the ratio of armature terminal voltage per phase to the corresponding
'S
armature current per phase, gives q-axis synchronous reactance Xq. Oscillograms of armature
Hy
current, terminal voltage and the e.m.f. induced in the open-circuited field winding are shown
in Fig- 5-93. A much larger slip than would be used in practice, has been shown in Fig. 5.93,
merely for the sake of clarity. When the armature m.m.f. wave is along the direct axis, the
%
armature flux passing through open field winding is maximum, therefore, the induced field
e.m.f ) “ 4)a ^ 1S zero. The d-axis can, therefore, be located on the oscillogram of Fig. 5.93
where the induced field e.m.f. is zero. When armature m.m.f. wave is along g-axis, the armature

flux linking the field winding is zero, therefore, the induced field e.m.f. — is maximum. Thus
• i « dt
the q-axis can also be located on the oscillogram. Waveforms of voltage across open field and
'Us the armature current in Fig. 5.93 reveal that armature current varies cyclically at twice the slip
frequency.

of
ss
2S
id
i*
d
S

Fig. 5.93. Typical oscillograms in slip test.

If oscillograms can’t be taken, then an ammeter and a voltmeter are used as shown in the
connection diagram of Fig. 5.94. The prime-mover (or d.c. motor) speed is adjusted till ammeter
and voltmeter pointers swing slowly between maximum and minimum positions. Under this
condition, maximum and minimum readings of both ammeter and voltmeter are recorded in
order to determine Xd and Xq. Since the applied voltage is constant, the air-gap flux would be
c°nstant. When crest of the rotating m.m.f. wave is in line with the field-pole axis, Fig. 5.92 (a)
j^mmum air-gap offers minimum reluctance—consequently the armature current, required
the establishment of constant air-gap flux, must be minimum. Constant applied voltage
minus the minimum impedance voltage drop (armature current being minimum) in the leads
and 3-phase variac gives maximum armature-terminal voltage. Thus the d-axis synchronous
rGactance is given by
Maximum armature terminal voltage per phase
A~ Minimum armature current per phase

nr i
^!anneci by CamScanner
[Art, s,
654 Electrical M achinery

Pr im e -m o v e r

■.!,

■Vi1; i

Field winding
open
jU'l
iff-. Fig. 5.94. Slip-test connection diagram for obtaining Xd and Xq.

By a similar thought process,


•f Minimum armature terminal voltage per phase
Maximum armature current per phase
During slip test, it would be observed that swing of the ammeter pointer is very wide,
whereas the voltmeter has only small swing because of the low impedance voltage drop in the
leads and 3-phase variac. Since low armature terminal voltages are used, values of reactances
obtained are unsaturated values.
When performing this test, the slip should be made as small as possible, otherwise the
currents induced in the amortisseur circuits would cause large errors in the measurement of
Xd and Xq (lower value of reactances for larger slips). It is however quite difficult to maintain
very small slips, as the reluctance torque due to saliency tends to bring the rotor into
synchronism with the rotating armature m.m.f. wave. It is because of this reason that tjhe slip
test must be conducted at low values of armature terminal voltage so that reluctance torque
due to saliency is low. .
The advantages of oscillographic method over voltmeter-ammeter method are (i) elimina­
tion of the inertia effects of voltmeter and ammeter and ( i i ) the possibility of large slip-speed,
which in turn allows higher armature-terminal voltages to be applied.
In practice, there may be error in reading the oscillograms. At the same time voltmeter
ammeter readings are not very reliable because of their inertia effect. In view of these
shortcomings, slip test is conducted only to determine the ratio oiX q/X d so that any error in the
measurement ofX d and A, gets cancelled. Now, using the value o fX d computed from o.c. and
s.c. tests, Xq can be determined as follows :

, from slip test •(Xd, from o.c. and s.c. tests).

(6J M axim u m -lagging-curren t test. In this test, the salient pole synchronous machine is
made to run as an unloaded synchronous motor, with balanced rated voltage across its ter-
mmals. Its field current is then reduced to zero gradually. Reference to Eq. (5.69) shows that
for zero field current, Ef is zero, but the reluctance power is not zero. In view of this, even for

at svnchrmmus^need m0t0r COntinues ■ reluctance motor


t h e S T S 1 f Ct; 0n ° f th6 field Current is now reversed by a suitable switch. If
the re\en?ea ficid current, i.e. the negative excitation is inrm oe.^ n __

the rotor falls out of step, i.e. loses synchronism


rotor is again pulled mto step with the next pole of opposite polarity. Ju st bffore the rotor

Scanned by CamScanner
ll
m
t. 5.16] Polyphase Synchronous M achines 655
x (

begins to lose synchronism momentarily, armature terminal voltage and maximum stable ar­
mature current are recorded. Then the g-axis synchronous reactance Xq is given by
Vt (armature terminal voltage per phase)
...(5.83)
^ 7a (maximum stable armature current per phase)
The advantage of this method is thatXq is obtained under approximately normal operating
conditions of saturation. However, for synchronous machines having high ratios of Xd/X q or
large armature rated currents, the maximum-lagging current test may be conducted at reduced
armature voltage, otherwise the maximum stable armature current may become dangerously
high- , i
(c) Reluctance-m otor test. For this test, the synchronous machine is run as a reluctance
motor and its load is then gradually increased, till it steps out of synchronism. When the reluc­
tance motor is on the verge of losing synchronism, i.e. under maximum reluctance power con­
ditions of 5 = 45°, the maximum stable armature current, armature terminal voltage and
maximum stable armature power are recorded.
If damper winding is present, then the reluctance motor, just after losing synchronism, will
continue running as an induction motor at a speed less than synchronous speed and the current
drawn will be quite large. In view of this, care should be taken in noting the instrument read­
ings at maximum reluctance power conditions, i.e. just before the rotor beings to lose
synchronism. .
The reluctance power, for Ef = 0 in Eq. (5.69), is given by
V? ( l _ L sin 25
P = ~P
Xn
‘■q ^d

For maximum reluctance power, 6 = 45'

P
vUi
1 max =
xn
d -TT
Assume y” =
Q
v? v?
...(5.84 a)
l-J
1

H
n

Pmax - 2 X n K
\ /
S'

From Fig. 5.80, Ex. 5.31 and for 5 - 45°,


Vt sin 8 = Xq Iq,
Vt
7* ~ l 2 X n
and Vt cos 5 = X d h
Vt
4=

Now armatur^ current l a - 'fr'd + 7;

Vt J 1

...(5.84 6)
1
(ArL S.■K
656 Electrical M achinery

Dividing Eq. (5.84 a) by Eq. (5.84 b), we get


Vt [ K - 1 ••(5.85)
V2 WFTT
“ _ V\o rW rmined from Eq. (5.85). The value oftf
Since Pm„ , 7„ and V, are measured, K can ^ calculated from Eq. 5.84. Then ft,
greater than one is retained and consequently , ^ advantage of this method is that
; h t : ; i u t " ^ - T e 1 o r d l T r a p p r o b a t e * normal operating conditions,

S“ p.e 5.39. Find on expression for the reactive p a v er as a * * * » £ £ « « * * 1


(a) for a salient-pole synchronous motor working at a lagging power f ,
(a) jor a saiieru current reduced to zero.
(b) for a salient-pole synchronous-generator with its peia cu
Neglect armature resistance for both parts (a) and (b).
Solution. As stated before, the product of voltage and quadrature component of armature
current gives the reactive power.
(a) For a salient-pole motor working at a lagging power factor, refer to Fig. 5.71 (b) for
finding the reactive power.
In this figure, when ra = 0, oa becomes equal to Vt.
... Q = Vt cos 5 Id + Vt sin 8 Iq ...(5.86)
■ r Vt sin 8
Also Vt sin b = Xq Iq, •*• X„
t Vt cos 8 - E f
and E f + Xd Id - Vt cos 8,

Substituting the values of Id and I q in Eq. (5.86), we get


Vt cos 8 (Vt cos 8 - E f) Vt sin 8 Vt sin 8
Q = ------------------—--------------------- 1--------------------------------
xd X,
V} E,V,
» ...(5.811
=-
cos 8 + — + V) sm 2 8
Ad Ad x, xd
(6) The phasor diagram for a salient-pole synchronous generator with zero E r, is showni
Fig. 5.95. The armature current is seen to be leading Vt, therefore, the generator is operatin
at a leading power factor.
Its reactii 2 power Q is given by
Q = - V t c o s 5 I d - Vt sin 8 Iq
Vt sin 8
Now V, sin 8 = X I , I =
X„
Vt cos 8
and Vt cos 5 = X d Id, Id = XA
r2
v?
Q = - Y l cos2 8 - -zj- sin2 8
xd Xn
V: 1 Fig. 5.95. SaU w JJ^
= - — _ v2 sin2 8. synchronous 5 ' “ ™“ / , , . *
Xd diagram with I f -
Art. 5.16]
n

ZXZ t elt n a L k
Z !t T ! h: q'aXiS $ynChr0nOUS be determined from the
maximum-lagging current test. Armature resistance and the rotational losses are to be ignored.
Solution. For negligible armature resistance in a salient-pole synchronous motor the ac­
tive and reactive powers are respectively given bv Eos rn aSw JI *
_ frxr
.here for. convenience.
n i o n r n Therefore, y B n Dy ^qs.(5-69>and (5.87). These are re-wntten

Ef Vt . n fi
P=
sin 25

to
^ Ef v t V?
and Q- - y cos 5 + + v? sin2 5.
Arf Aj Xd Xq
For negative excitation during the maximum-lagging current test, P and Q are given by s

p E,V
f ‘ -• V? r i n
sin 25
2
5
and C OS
^ + X~ + | ^ “Isin2 5

When the motor is just on the verge of falling out of step,

^ =0
db
dP EEl r .a d cos 25 = 0.
d T - ^ c o s S + Vf Xq xd
At no-load, angle 5 is zero.
dP_ _ _ EfVt
= 0
db x„ X,

or Ef =V t — - 1 ...(5.88)
*0
Since the rotational losses are ignored, the power input is zero, i.e. cos 0 = 0 or
0 = 90° lag. Thus .1
Q = Vt Ia sin 6 = Vt Ia.
For negative excitation and for 5 = 0,

i ...(5.89)
°r i a = - jr ( E

Substitution of E f from Eq. (5.88) in Eq. (5.89) gives

X j_ 1
Vt + v,
V
or
la
This resu lt agrees with Eq. (5.83). . . . .

o u a i ii icu uy w a iiiju a iin c i


658 E le ctrica l M ach in ery

mltage and c u r Z ^ e A T J lp ^ Z
6.5 A. Armature resistance is negligible. The P w
machine is 20 MVA, 3-phase, star connected, 11 kV, 5U ti /
unitypf and rated voltage, calculate
(а) the excitation voltage and
(б) the reluctance power developed by the machine.
a . 1 2 0 / _ 1 2 0 x 5 0 = 1500
Solution . Synchronous speed - p - 4 r
The synchronous machine is running at 1490 rpm, less than, the
synchronous speed and the rotor winding, U the£eld v™dmg “^ ;
age readings indicate that the stator is energized from 3-phase, 50 Hz source
at reduced voltage. A careful study of the question reveals that this question
pertains to slip test on a salient-pole synchronous machine.
F ig . 5 .9 6 . Pertaining to
Therefore, d-axis synchronous reactance, E x a m p le 5.41.
Maximum voltmeter reading per phase _ 30. _ ^M n
d " Minimum ammeter reading per phase 6.5
Minimum voltmeter reading per phase _ _25_ _ 0 p
* " Maximum ammeter reading per phase 10.0
Taking the machine to be working as a synchronous generator, the phasor diagram at unity
pf is as shown in Fig. 5.96.
, y 20,000 K
(a) Full-load armature current, Ia = = 1049.7b A

„ .. 11000 v
Per-phase armature voltage, Vt = —^ — = buol v

Taking V, as reference, we get V, = 6351 /0-°


Ia = 1049.76 /0 0
... E f = Vt + j l a Xq
a = 6351 + j 0 + j (1049.76) x 2.5
= 6351 + j 2624.4 V
£ / = 6871.88 722.45 °
Therefore load angle, 5 = 22.45°
D -axis current, Id = / a sin 6 = 1049.76 sin 22.45° = 400.88 A

Per-phase excitation emf, E f = E f + Id (Xd - Xq)


or Ef - 6871.88 + 400.88 (4.62 - 2.5) = 7720.866 V
Also, E f - Vt cos b + Id Xd = 6351 x cos 22.45° + 400.88 x 4.62 = 7721.74 V.
Line value of excitation voltage = V(T x 7720.866 = 13372.54 V
(6) Reluctance power developed by the machine
V ]f
sin 2 5
Xa X,
/
_ (6351) 1
sin (2 x 22.45°)
2.5 4.62
= 2612966.2 W per phase

Scanned by CamScanner
A rh £ £ ]__________
Polyphase Synchronous M achines 659
, Total reluctance power
= ^261296^2
1000 = 7838.899 kW
Example 5.42. A salient-pole synchro

S S m T mfmite
clrfenr 'bU
SEXPtain Whal ^aZ
n1f' ,S
ctr^ i« ge/s open-circuited ac-
Solution. Under normal operatinn * u
power M l sin 8 and r e l . rp Vf r 1 r° n S generat0r SUPPlieS both electromagnetic
X sm 28, to the infinite bus.
^Xn "d “uo. When open cir-
ttuciiupencir-
cult occurs in the field circuit, Bf and, therefore ^ .• a
does not become zero. In view of this th r Mr° ' tUt reluctance Ptm’er
“glo
winding, operates as a reluctance g e n e r l t o X ^ h r o n o u . ! " ' 1 f t * °pen circuit in the
icWfV, d-u • hronous speed. If pnme-mover power input
is Igss than tho maximum reltirtannn i V ( i -i
mum reluctance power developed = - M - L u ^ .
, ., . 2 I " L by reluctance gene­
rator, then this generator runs stablv at tjvripV. ' ^
to the infinite bus. In case the mechanicafpow™ inmu ^ a" d “ ntinues feeding active power
inity power, the reluctance generator loses synchronism » >S m° r6 v tbe maximum reluctance
generator gets open-circuited. After the generator’h a s n 't ” u fidd C'rCuit of synchronous

However, the synchronous generator should nnt h / P°WGr the buS'


hon generator for a long time. This is becausp 3S * reluctance generator or induc-
mechanical power input, both the reluctance and induct °f eXcitation and for the same

mechanical power input. Since cylindrical rotor margin • ync^ronous speed because
Eenp0tt 1 SP8ed has exceeded synchronous speed, the machine t0 lnflnite bus and
stat ^ ° r at asyncbr°nous speed. Large currents are set un in the as an induction
absorhair’8aP nUX t0 Produce the necessary electrical torque The indi ^ *° interact "dth
general P° Wer from the bus b“‘ d e f e r s active power to W h genCrat0r
^ ? L ™ dhamaging and sh°uid not be pp™ itted
to be quitl rr0n° j S ° peratlon as induction generator at about 10% of the rated dow o r "a
H bi Pt o S ? „ n d : r , h
„ operation may,be permiued at the™ ai
conies inf
c°rnes in, 1 ?as.
abee few-power
low -P ° we: asynchronous operation is not required, the governor aft on
^ to play which prevents «.,* rotor
vents the 1UW1 speed
OFi;i;U from reaching dangerously high values
Efficiency of Synchronous
h l ’A n m i e Machines
e various losses in synchronous machines are :
odoad rotational losses, (a) friction and windage loss and (b ) open-circuit core loss.
1 Field circuit loss, (c) I 2R loss in the field winding, where R is its resistance
D ^ c t load loss. (d ) I2R loss in armature winding.
660 Electrical M achinery

«„) Stray load losses. («) in iron and ( f I ini the t0 as shorl.ci
The combination of direct load loss and stray load losses .s cm, S

Friction and windage loss can be measurec | d ri^ f


speed with Us held wind,ng unexcited. The mechan ca ^ « U*
synchronous machine gives its friction ana wi (W.+ W2) becomes equal to the sum t
field winding is excited, the m e c h a n . c a l p w e r mput W ,+ J sum,
friction, windage and open-circuit core losses ot tne syncx ie open.
circuit core loss is given by W2.
The open-circuit core loss, a combination of eddy-current and hysteresis losses is caused I,
changing flux densities in the machine iron when only the field winding is energised- Smcethe
edd^urrent and hysteresis losses are approximately proport,onalto square of the voltage,
open-circuit core loss also vanes approximately as the square of the open-c.rcmt voltage-^
variation is illustrated in Fig. 5.97 (a). The variation of no-load rotational losses with fe]t
ccurrent
u r r e n t is
is aas
s sshown in rFig.
how n m i g . 5.97 (6). Note that
tucac the ------------
intercept of no-load rotational losses withthe
# 1 .* 1 t /* * i? •— d •tunrlnxTA ln c c n n /'ni icr\ unf Vi r» %.
vertical axis in Fig. 5.97 (6), gives the constantconsi friction and windage loss, because with zero field
rent open-circuit core loss is also zero.
current
The field circuit loss can be determined from Vf Ift where V^is the applied voltage to the field

. (a) . . tf>)
Fig. 5.97. Variation of (a) open circuit core loss w ith open circu it voltage
and ( 6 ) rotational losses with field cu rre n t.

winding and If is the field current. Alternatively field circuit loss = l}rf.
Here /y is the field winding d.c. resistance calculated at 75°C
The short-circuit load loss is a combination of ohmic
loss in the armature winding and the stray load loss The
armature dc resistance is measured and corrected for the
working temperature rise, usually 75°C. Then the stray
load loss is obtained by the following relation :
Stray load loss = (Short-circuit load lossWArmQi,
o mil: l o s s , calculated by u s i n g h o t v a l u e o f d.c. r e s i s t a n c e )
the variation of short-circuit lnad ’ nor. j ,
loss, will, the armature current is shown p*. 5 90*3 . f t 1

F ig . 5 .9 8 . Short-circuit load1035
j iriH inds cun’es

Scanned by CamScanner
Polyphase Synchronous Machines 661

The stray load loss consists of two components, namely :


(j) iron loss or core loss due to armature leakage flux and
(ii) arm atu re ohmic loss due to skin effect and eddy currents in the armature conductors.
In iron Par^s>^ e. stray l°ss caused by the armature leakage flux, is due to the eddy
' -i 1 r* 1non ap in i-Un i i .
,ent and hysteresis losses in the teeth, core end-plates, end-covers etc.
« I corr1
5 f in them. These induced emfs set up circulating currents or eddy currents in the
e in d u c e d
If '
inductors and make the conductor current distribution non-uniform which gives rise to addi­ 1
tional ohmic loss. This additional ohmic loss forms a considerable percentage of stray load loss.
In a.c. electromagnetic devices, the short-circuit load loss divided by the square of the short-
circuitarmature current gives its effective resistance rac^- Thus for a synchronous machine,
II
„ _ Short-circuit load loss
r a e f f .-~ z r : ------------------------------- ~ ...(5 .9 0 )
(Short-circuit armature current)
The stray load loss and effective armature resistance once computed at rated current are
co n sid ere d to remain constant under normal operating conditions.

The power flow diagrams for a 3-phase alternator as well as for a 3-phase synchronous
motor are shown in Fig. 5.99 (a) and (b) respectively. It is seen from this figure that no-load
rotational loss,
Pr = friction and windage loss + open-circuit core loss
Pr + V flf = constant losses
Q
Short-circuit load loss = 3 Ia ■ra + stray-load loss.

lacosy
Mech. power Electrical power
In p u t - o - output,3Vtl acos0

F &W Open circuit Field circuit Arm.circuit Stray load


, Loss core loss ^ loss.VfJf loss,3 la ra ,oss

No-load Rotational loss Short circuit load loss


(a)

3EflaCosy Mech. power


Elect, power input
3Vt Iacos e - o output, T.uls

r
Arm .circuit Stray load Field Open circuit F s,W
lo s s , 3 l i r a loss circuit core loss ^oss
\________L _ L _________ ^ loss, v *
Short-circuit load loss vj ' f No 'ood R o ta tio n at
(6 )
Fig. 5.99. Power flow diagrams for (a) a 3-phase alternator and (b) a 3-phase synchronous motor.

As in a dc machine, maximum efficiency in a synchronous machine occurs when


variable losses = constant losses
0r 3 I 2am'r a = Pr + Vf ‘ If
Where C is the armature current at which maximum efficiency occurs in the synchronous
Machine.

Scanned by CamScanner
662 E lectrical M achinery_________________________________ ____________________________________ IAtl^

A lte rn a to r efficiency in the laboratory. The efficiency of an alternator can be deter.


mined in the laboratory by using a d.c. shunt motor as its prime mover and by Performing^
following tests :
(i) Uncouple the dc motor and alternator. Run the dc motor alone at rated (= synchro,,*,,,
speed of the alternator. For the dc motor, record armature terminal vo age a and annat^
current Ia. For known value of armature-circuit resistance ra.
Va Ia = Friction, windage (F.W.) and core loss of dc motor,
or Wx = Va Ia - l l r a
Note that Va Ia is the total input to motor armature only ; it is not the total input to dc

m<> Couple the alternator mechanically ^ t h dc m'rt Run the dc motor again „
synchronous speed, with alternator field unexcited. Record Vv Ia again. Mo ,
Vj a = W1+ l l r a + F.W. loss of alternator, Wf
or Wf - V J a - (Wi + llr a)
In step (.ii), h will be different from its value in step
m R ^ e a t step (ii) but with alternator excited with normal field current and with am,,
ture circuit open.
v I = Wj + Iara + Wf + open-circuit core loss of alternator, W2
or W2 + Wf = V J a - ( W 1 + I2ara)
nr W2 = V J a - ( W 1 + & a + Wf)

If (W2 + ^ is Plotted as a " n " s s \ i


curve on the vertical axis gives Wf . It is because wiui i f , 2

) o b t a i n e d , see Fig. 5 .9 7 (6). e,rmmptriral short-circuit test a


current of alternator. Record

dc motor. Now,
vj = vr, + l\ra + Wf + short-circuit load loss, VV3

Wq = VJa - (Wi + Pja + Wf) In

" Since field cu rre n t is ^

(app^ z : s t : : : : the ^ * * * . — « . « — “ d- -

m alT otal a lte rn a to r losses = F.W . loss + open-circuit core loss ♦ s h o r t - c i r c m U o a d j ^ ^

= Wf + W 2 + W 3 + VfIf (5.1

to tal losses
A lte rn a to r efficiency= 1 - input ^ cQre loSS sho'

Scanned 6y~CamScanner
Art-_5.17]_____________________________________________________ Polyphase Synchronous M achines 6 6 3

Exam ple 5.43. A 100 kVA, 440 V, 3 -p h a s e , s ta r -c o n n e c te d a lt e r n a t o r h a s th e following d a t a .*


F r ic tio n a n d w i n d a g e l o s s _ 3 4 0 w a tts
O p e n - c i r c u it c o r e l o s s = 4 8 0 watlSt
75°C
F ie ld w i n d i n g r e s i s t a n c e a t = i s o Q.
Effective arm ature resistance per phase =0.02 ft
The voltage applied to field winding is 220 V. Calculate the alternator efficiency at 0.8 p.f.
And at
(a) h alf full load and
(b) full load.
'ic Solution. Full-load armature current per phase,
r im o o o ,
at ° ^3 x 440 " 131>22 A-
(a) Short-circuit load loss at half-full load

(Ila\
=3 (Effective armature resistance per phase)
2
f 131.22
=3 (0.02) = 258.28 watts.
2

Field-circuit loss
= * u s o “ = 2 6 8 8 9 wattSl
Total losses = 340 + 480 + 258.28 + 268.89 = 1347.17 watts.
Efficiency at half full load, from Eq. (3.91), is
f 1ry \
1347.17
1- x 100 = 96.742%.
40,000 + 1347.17
(6) Short-circuit load loss at full load
= 3 (131.22)2 (0.02) = 1033.12 watts.
Total losses = 340 + 480 +1033.12 + 268.89 = 2122.01 watts.
/. Efficiency at full load, from Eq. (3.91), is
2122.01
1- x 100 = 97.416%.
80,000 + 2122.01
V J
Exam ple 5.44. A 40 kVA, 400 V, star-connected synchronous machine has short-circuit load
loss o f 1.5 kW at rated armature current and at 30°C. At this temperature, the dc resistance o f
the armature is 0.118 Q per phase. Calculate the effective armature resistance in p er unit and
also in ohms p er p h ase at 30°C. Find also the ratio o f ac to dc resistance.
Solution. Rated armature current,

I = f f l ^ = 57.74A
ar V3 x 400
Short-circuit load loss in per unit
= i | = 0.0375
40
Effective armature resistance, from Eq. (5.90), is
S.C. load loss in p.u. 0.0372 .........
Y ~ 0.0375 p.u.
r“ 'eff (armature current in p.u.)2 (1.0)

I I IvVUUI II IWI
664 Electrical M achinery

, 1.5 X 1000 _ cfin W


Short-circuit load loss per phase = g - vv

Effective armature resistance, raeff= —“ “ “ 2 = 0 1 5 a per Phase‘


( 5 #. / 4 )

The ratio of ac to dc resistance is given as under :


r° eff _ JL2JL = \ 2712.
radc ~ 0-118 •
Example 5.45. A 500 kVA, 11 kV, 3-phase star-connected alternator has the following data ;
Friction and windage loss - 1500 W
Open-circuit core loss = 2500 W
Effective armature resistance per phase = 40 ohm
Field copper loss = 1000 W
Find (a) alternator efficiency o f half-full load and at 0.8 pow er factor lagging,
(b) maximum efficiency o f the alternator. (I.A.S., 199S)
Solution. Full-load armature current per phase,

/a = ^ n : = 26-244A
Short-circuit load loss at half-full load

( Ia "
=3 x ra + stray-load loss, which is zero here
2
26.244 ^
=3 x 4 = 2066.24 W

Total loss at half-full load = 1500 + 2500 + 2066.24 + 1000 = 7066.24 W


Efficiency at half-full load

1 7066.24
-
x 100 = 96.587%.
500,000 x | x 0.8 + 7066.24

(b) For maximum efficiency,

variable losses, 3 12am • ra = rotational loss + field-circuit loss


3 ^Im ■4 = 1500 + 2500 + 1000 = 5000 W
The current Iam at which maximum efficiency occurs is given by

5000
12 = 20.412 A
‘ am

Output maximum efficiency = 3 Vt ■I . Cos 0

o 11000
~ * ~ 1 T X 20 412 x 0 8 = 311,111.54 W
Total losses at maximum efficiency = 2 x 5000 = 10,000 W

Maximum efficiency = 1- 10 ,0 0 0
311,111.54 + 10,000 x 100 = 96.886%.
s ArtJJfl _
Polyphase Synchronous M achines 665
5.18. Operating L im it on Synchronous Generators
Ml electrical devices are designed to nnprato
,imum performance, these limits should not be ' T i 1* F °r ‘ heir ° P'
an infinite bus, the operating limits are active cower (Hen a f lterna^ors delivering power to
reactive power, power factor, stator and rotor L r l ! S ' 1 U^°?. Pnme-mover output),
ing the working limits of various ODeratinu , an he stat>dity limit. A diagram show-
Z r t of a synchronous g e n e r a t e alterna‘ °r ‘S Ca" ed a" °p era‘^
nected to infinite bus is to know the activn S!-° operatlng chart of aa alternator con-

the 7 dpower an6le at whS,c


nator for fixed value” of actife^ower a n T m l t a s e ^ ' ] ^ !°adin
During the planning of a power system r a r i ' l t reactive capability chart or curve.
synchronous generators so far as their contribution *n the selection of
cerned. These curves are also helpful durin* the nJ , allowable reactive power is con-
allowable reactive power that can be taken from t v n ope *°n t°hnow about the maximum
19)
power factors at rated voltage In this ar[ L r for different power loadings and
curves of cylindrical-rotor alternator are presented ^ ^ ^ then the capablllty

devdqm^eiU of iU^operating d ia i^ Hg *5* otfm) show T V “ 7 ™ th° baSi* f° r the


for lagging power factor. In this figure, armature c,rcu,t r J s f a n c e L n e S e X d 0^ " ^ ™ " ^
small m large synchronous generators. Now each side of Fig. 5.100 (o) is & !d ed by X s So we
get GO = ^ , OM = / „ and GM= S in Fig. 5 100 (()). Extend G0 to N and draw Mp

Wf WandrP 0 perpendicular t0 G 0' Here ON = PM = / . sin 0, therefore,


V /Wl e 1!° v dr,OW
VI 2 11 v ’ =2 P° Wer alternator- A180 NM = OP = 1. «® 0 and therefore,
A- “i , active power of alternator. It is, therefore, seen that line OP perpen-

t p i t 0?aTtem atorS P° W6r ““ ° N “ ^ WHh G° rep“ tha - a c -

For constant Ia, locus of armature current Ia is a circle with radius OM and centre on thP
point 0 . For constant field current, ex­
citation circle is drawn with G as the
centre and GM as the radius. Load
i . f V '
angle 5 is in between GO ‘ and
Xc
GM Power-factor angle 0 is

^ I ween OM and OP or between


A and MO. Based on Fig. 5.100 (6),
G operating chart of a cylindrical-
r°tor alternator is drawn as under :

First of all, draw GO = on some

Scale in Fig. 5.101. When Ef = Vt, the


(a) (b)
c*rcle w ith G as ce n tre and v / as Fig. 5 .1 0 0 . Pertaining to the development of operating ch art
* of a cylindrical-rotor altern ator.
a ius, passes through the point O.

Scanned byTJamScanner
666 E le ctric a l M a ch in ery ________

This circle may be termed as


per-unit excitation circle. In 7 c, pu E*t
0-7 pi 4 -rv r v s?
this manner, more excitation
circles, with G as centre, are
drawn as shown in Fig. 5.101.
Now the rated power of al­
ternator is drawn as AB paral­
lel to GO and at a horizontal
distance of
QP _ per-phase rated power
per-phase voltage
Note that the horizontal distance LQ9ginQ
raociiv*
scale is the sam e as th at of P0 w*r
Ef/X s. If the a lte rn a to r is
designed for, say, 0.85 lag, then a
Unity pi
line lagging V, or OP (as in Fig.
5.100) by an angle of cos” 1 0.85 Leading
is drawn. This power-factor line reactive
meets the power line AB at the power
point M. The line joining M with
O gives the magnitude of arma­
ture current. Now a circle with
0 as centre and OM as radius
gives the current-limit curve for
power = GA and p f - 0.85 lag. If
alternator is designed for, say Fig. 5 .1 0 1 . O peratin g c h a r t o f a cy lin d rica l-ro to r altern ator.
Ef = 2.5 V„ then the excitation-
1unit curve with G J as radius is drawn. The excitation-limit and current-lim it curves meet at
current is the i t ^ T h * ^ J R ' evA^ on is the limit and *>r the curve RM, armatur
current s the limit. The maximum power limit line is given by AM The horizontal line Gl

s e t s :
permit some additional loading of the alternator6 In^racticif Umit which shoul
usually about 10% less than the theoretical stability luEit P Umit 18
For drawing the practical stability curve consider anv . .
on the line GK. Cut up = 10% of ra te r ! MW n * ' Say “ or Per' unit excitatioi
excitation circle in / Th i p o i n t ^ D! ! nt T ^ ParaHel t0 GP The ™
points on the practical ^ * ' = V‘ ° * *
the minimum possible excitation for this alternator T to ir? h ar1?'' Here ° N
cylindrical-rotor alternator is JR M S N O I An„o J ’ com plete operating area o
tells the MW, MVA, MVAr, current n f evrit«,« peratl,ng P01nt placed within this area, at one
a operating point T as shown in Fig. 5 lo i « 1/ ° ° ^ ang^e of alternator. For instance, fo
equal to OQ, p f equal to cos /G O T and aroint,?,8 through T and parallel to GO gives powe
by (OT) (operating voltage), W ^T utput is giten bv (TO w eqUa‘ l ° ° The V’A outPut is pVf'
the excitation. The load angle is equal to TGO g r a t i n g voltage) and GT represent

Scanned by CamScanner
Art. 5.181
— Polyphase Synchronous Machines 667
5. 18.2 . C ap ability C u rves. As alreaH t- + a
indicate the maximum reactive-power lo a rW 3 u Capability curves for large turbogenerators
ings at rated voltage. These curves are derived as foil nous valuesofactive-powerload-
If P = active power and Q = reactivp '
is given by P r of a synchronous generator, the apparent power

apparent power = = 3 yf

writtenas PhaS8 V° ltage and 7° = phase current of a 3-phase alternator. Eq. (5.92) can be re-

P2 + Q2 ~ (3 Vt Ia)2 = 0
If x = P and y = Q, the above equation becomes
+y - (3 vtia)2= 0 ...(5 .9 3 )
Eq. (5.93) is the equation of a circle, with centre A at
(0, 0) and radius Ab = 3 V, I a as shown in x-y plane or P-
Lagging pf
Q plain in Fig. 5.102. Any point on the circle represents o p e ra tio n
the apparent power, active power, reactive power and pf.
For e x a m p le , for p o in t b .A b * a p p a re n t power
= 3 Vt l a,A f = a c tiv e pow er = 3 Vt Ia cos 0, bf= reactiv e
power = 3 V, 7a sin 0 for operating p f - cos £bA f lagging.
Since radius of semicircle = ZVJa = total VA of the alter­ x

nator, the semicircle gives the locus of constant apparent T


power. Operation of alternator may lie on the semicircle Leading pt

abede indicating rated VA loading, or within the semicircle operation


abede indicating VA loading less than rated VA.
The first quadrant operation abc of the semicircle cor­
responds to alternator operation at lagging pf, because lagging
reactive power is taken positive by convention. Therefore, alter­
nator operation in the first-quadrant pertains to lagging p f Fig. 5 . 1 0 2 . Pertaining to the locus of
whereas fourth-quadrant operation gives alternator opera- constant apparent power,
tion at leading power factor. Point c in Fig. 5.102 pertains to
unity p/* operation. As the alternator operation moves from c towards a ; p f becomes more lag­
ging progressively and at the point a, the p f is zero lagging. Similarly, p f becomes more leading as
the alternator operation moves from c to e and its value is zero leading at point e. Alternator opera­
tion at lagging p f needs more field current, whereas for leading p f operation less field current is
required. An increased value of field current would entail more field copper loss and therefore more
heating of rotor. This means that field current cannot be permitted to increase at one’s whims.
There is, therefore, an upper safe limit for the value of field current in the first quadrant operation
°f alternator. For obtaining these boundary points, consider alternator operation with constant
terminal voltage and variable field current (or variable emf Ef). The field current is limited by a
consideration of the rotor-field heating. Three-phase complex power output of a synchronous gene-
fator is given by
S = P + jQ = 3 v j a ...(5.94)

With ra = 0 ,E f = V t + j l a Xs for an alternator.


- « r E[ Z Vl 1
or 7
h jx,1

Scanned by CamScanner
668 E lectrical M achinery lArt. s I
T.
E ,- V t
n=j

P +j Q = j 3 V,
r Ef- v t' ■3 Ef Vt 3 V?"
-J x, xs J
Xs

or
^ 3 V? • 3 Er Vt
P +JQ + J ~ x~ =J
x*
3V f 3 Er Vt
P+j Q+ =J
Xs

3 Vf 3 Er Vt
P2 + Q +
X X.
<N,

1
V

.J
CO
V2
CO

___
3
or P 2 + Q2 + 2 Q
tl

xs ■J L J
2 _2 3 V2 ( 3 V,
P 2 + Q2 + 2 + * [V * -E * ] = 0 ...(5.95 a)
X,
Let x = P and y = Q,

2 2 „3v? ( 3 V, "j
x +y + 2 •y + [V ? -£ ? ]= 0 ...(5.95 b)

3 V?
This is the equation of a circle with centre at 0 ,- and
X
2 2 M/2
3 Vt ] 3 Er Vt
radius = 02 + (V ? -E ? )
X ~ X
Eq. (5.95), the equation of a circle, is represented in
. ( 3V ^
PQ plane in Fig. 5.103 with centre at 0 , - 1 and
S
3 E •V
radius = ^ ‘ Here only a segment of this circle is

3 Er Vt
shown as fgbh. In X , V, and Xs are constant, there-
S

fore radius of the circle is governed by Ef or lf or the rotor


field heating. This means that curve fgbh represents the
limit of field winding heating. So the alternator operation
must fall within this circle so as to limit the heating of rotor Arm ature

field winding. heating Wn't

The point of intersection of the two circles, one with


• ■ 3 E •V
radius 3 Vt l a and the other with radius — £ — - represents
Xg "5
the machine rating ; in Fig. 5.103, this point is 6. This F iS - 5 . 103 . P e rta in in g to the construction
shows that operation of alternator for lagging p f operation of caPabi,ity curves ^ a synchronous

Scanned by CamScanner
Arl. 5191
Polyphase Synchronous M achines 6f>9
must be constrained below the field-heating
limit curve fgb. Altowoble
rotor heating

In actual practice, an alternator must I P


deliver reactive power to the system. In
other words, the alternator must always
work in first quadrant of PQ plane so that
it can operate at lagging p f and deliver ac­
tive power as well as reactive power to the
infinite bus as desired. In order to illustrate
the alternator operation in first quadrant
Fig. 5.104 is drawn by referring to Fig!
5.103. In this figure, AB is the rotor-heating
limit curve and B C is the armature-heating
P0 Rated kVA or
limit curve. Point B defines the machine Allowable arm heating

rating at rated pf. In Fig. 5.104, rated p f is Fig. 5.104. Capability curves of a cylindrical rotor
taken 0.85 lagging. synchronous generator.

In Fig. 5.104, the operating region is OABCO. This region can, however, be augmented to
operating region O A B C O provided alternator is better cooled by external means. For
machine-rating point B , OD = acting power loading, OE = reactive power loading so that
<OD2 + OE2 = 1.00 (rated apparent power).
From Fig. 5.104, the maximum reactive-power loading can be obtained for any value of
power loading of alternator at constant voltage. For example, for half the rated power output
= OF, the maximum reactive power loadihg is FG under normal cooling. This reactive-power
capability can, however, be increased to FH with improved cooling of alternator.
An examination of capabilities curves of Fig. 5.104 reveals that (a) for pf above rated value
(more than 0.85), field current required is less, therefore armature heating is the limiting factor
and (6) for p f less than rated value (<0.85), field current will have to be increased, therefore
rotor-field heating is the limiting factor.
5.19. P ow er F a c to r C orrection by Synchronous Motors
For the transmission of a certain amount of a.c. power to a load centre, a low power factor
would mean greater load current. Such a current would result in greater I R loss in the trans­
mission lines, transformers and alternators. As a consequence, the entire energy system would
operate at a reduced efficiency. Further, a greater load current at a low power factor would
mean poor voltage regulation of the transmission lines, transformers etc. and this would result
in an unsatisfactory operation of the utilization devices like induction motors, fluorescent tubes
etc. Thus from the viewpoint of efficiency, voltage regulation and better operation of the
utilization devices, load p.f. should be as close to unity as is economically viable.
It has already been discussed in Art. 5.10 that an outstanding advantage of a synchronous
/ L . looHinfy n fs when overexcited. An examination of Fig. 5.64
motor is its ability to °Pera e ' d' tQ operate at unity and leading p fs. merely by
reveals th at a synchronousm otorcanb ^ ofUs field current Tho advantagc „r
increasing
. . . v o u M n g iits
ls
excitation voltage E/W1 „lo.„ („mhvMunteraC
tern by counteractingting aa part
part,or the entire
this fact is taken in improving the pf o . primary function is to improve the pf of
Jagging kVAr KVAr of ui the
Uie load.
iuau. A
XX.s>™ chron0?, •„!
........ . i |]oacj
’oad on0n it is called a ssynchronous
y n e n r o n o u s rcondenser.
o n u tn w . «A
an electrical system and with no mecna ^ 0f a supply line, is used only when static
synchronous condenser, installed at tn
capacitors are found uneconomical. (a) 8hows an elementary energy system net-
5.19.1. S y n c h ro n o u s co n d en ser. Hg- • alternator excitation voltage, I is the
work. If V, is the term inal voltage at pom , f

Ju a in ie u u_y v ^ a i i u c a i ii i c i
y?:1 i
■? I i
670 E le ctrica l M ach in ery
T r a n s fo r m e r s

- '.j
. ; . ft
■i

Fig. 5.1 0 5 . (a) An elem en tary energy system netw ork


(b) its voltage-phasor diagram and (c) com ponents of the load kVA.

load current at pf cos 0 and X is the total reactance between E f and Vt, then phasor diagram
for this system is as shown in Fig. 5.105 (b ). At the point A, the load is oa kW proportional to
I cos 0, the apparent power is ob kVA proportional to 7 and load kVAr is ab proportional to
/s in 0. With the installation of a synchronous condenser at A, Fig. 5.106 (o), suppose the load
p.f. at A is improved to unity. Then, lagging kVAr required by the load and equal to ab are
locally supplied by the synchronous condenser as og = a b as shown in Fig. 5.106. In this man­
ner, the energy system network bletween alternator and point A is relieved of lagging kVAr,
the current is reduced from ob to oa ^nd the voltage at point A rises from oc to od for the same
E f as shown in Fig. 5.106. Since the system current has reduced, it would result in decreased
I 2R loss and therefore better system efficiency. Thus, with the help of a synchronous condenser,

© ­
Al t e r n a t o r

(a)

Vt A

-II-
(b)
Fig. 5 .1 0 6 . (a) Total lagging kVAr of the load a re supplied locally
by synchronous condenser, (6) and (c) P h a so r d ia g ra m s

Scanned by CamScanner
Polyphase Synchronous M achines 671

niorepo^er ^ sin 5jcan be delivered to load because V, has increased ; p.f. of the system
and its efficiency are unproved and consequently the general operation of the load apparatus
becomes b

" v t T f r T T b e t r : supt s iagging kvAi somewhat •«» than ab in Fig. 5.106,


the p.f- at P°mt d be near to unity and even then the efficiency and voltaee reeulation
*„„ld improve. In pract.ce, the p.f. is improved up to about 0.9 or 0^5 S g b S a u S a b o ”
this value the cost of the synchronous condenser far outweighs its a d v a n c e s

T i T i e w T t h i r i a r r denSeV S a motor but designed with a large Held

so n d ethe r is — bot h

‘" T of
area f ccross
L t ^section
n T M d p„Toge
of field field
poles as cu rrT torequires
compared f same
an ordinary synchronous motor of the f i
rating.

va) the kVA rating o f the synchronous condenser and


(b) total kVA o f the factory.

r T T " - T thf solution° f s“ch an eI£ample, it is convenient to draw its phasor diagram
first and then solve it accordingly. For this example, refer to Fig. 5.107. aiagram

Load kVA = ob = = 3000


U.b
la o b = cos"1 0.6 - 53.13°
/qoc = cos"1 0.95 = 18.195°
Load kVAr = ab = ob sin 53.13°
= 3000 x 0.8 = 2400 kVAr
Combined kVAr = ac = oa tan 18.95°
= 1800 v 0.329 = 592.2 kVAr
(a) Synchronous condenser rating
od = be = ab - ac
= 2400 - 592.2 = 1807.8 kVA.
Fig. 5.107, Pertaining to Exam ple 5.46.
Remember that synchronous condenser rating is in kVA
and not in kVAr.
(b) Total resultant kVA of the factory,
oc = -{(o a f + (ac)2 = ^(1800)^ + (592.2)^ = 1894.91 kVA.
Note that kVA required by the factory has reduced from 3000 kVA to 1894.91 kVA but at
ajt8 exPense of a synchronous condenser of 1807.8 kVA rating. If the factory has to purchase an
Withrnat° r’ trans^ormer and cables, their ratings are now only 1849.91 kVA instead of 3000 kVA
sav‘ °Ut-Synchronous condenser. At the same time, improved p/w ill affect good amount of
lngs in the energy bill.

Usu5]'l19‘2 * ^ u a l‘Pu rP ose Synchronous Motor. A synchronous motor is used in a factory


an t l / ^°r sendn6 ^wo purposes ; one is to drive a mechanical load and the other is to correct
tub° ,erwise low lagging p.f. of the electrical load. The use of induction motors and fluorescent
es m a factory is more common, but with their use the overall p.f. of the factory becomes

Scanned by Cam scanner


672 E le ctrica l M a ch in ery ___________________________________________________

low. For a fixed factory load in kW, a low p.f. would mean more
kVA demand and as a result the factory has to pay more, in
such factories, it is advantageous to install a synchronous motor
for correcting the p.f. of the factory and at the same time, or
driving a constant-speed load such as a large pump, a -c- gen e
rator etc. #
In Fig. 5.108, ob is the load kVA, oa is the load kW and a b is
the load kVAr. Further, oc = bd is the synchronous motor kVA,
be = og is the motor load in kW, ed = eg is the leading sup
plied locally by the motor to the factory, o f is the combined load
in kW, od is the combined load kVA and fd is the combined load
kVAr. The power factor of the plant or factory is improved trom
cos Zaob to cos Zfod. Fig. 5 .1 0 8 . P ertaining to the use of
E x a m p le 5 .4 7 . A factory has an average load o f 300 kW at a dual-purpose synchronous motor.
o f . o f 0 6 lagging. A synchronous motor, with an efficiency o f
88%, 'is used to raise the com bined p.f. to 0.90 lagging an d at the sam e tim e supply a mechanical
load o f 60 kW. Calculate
(a) total load kVA,
(b) kVA capacity o f the synchronous m otor an d
(c) synchronous motor operating pow er factor.
Solution. For the solution of this example, refer to Fig. 5.108.
. . - 60
Synchronous motor input = og = af = 68.18 kW
0.88
Combined load, o f —oa + a f= 300 + 68.18 = 368.18 kW.
of_ 368.18
(a) Total load kVA od = ■= 409.09 kVA.
Combined p.f. 0.90
(b ) /a o b = cos~ 1 (0.6) = 53.13°
Load kVAr, ab = fe = oa tan /a o b = 300 tan 53.13°= 400
r /fo d = cos~1 (0.9) = 25.84°
Combined load kVAr, fd = o f tan /fo d = 368.18 tan 25.84° = 178.30
.-. Leading kVAr supplied by synchronous motor
= cg = d e = f e - f d = 400 - 178.30 = 221.70
Synchronous motor input = og - be = a f = 68.18 kW
kVA capacity of synchronous motor,
oc = db = y!(og)2 + ( c g f
= V(68.18)^ + (221.70)2 = 231.95 kVA
(cl Synchronous motor operating p.f.

= cos /cog = s e = = 0.294 leading.

It is seen from this example that installation of 231.95 kVA synchronous motor reduces t e
overall load kVA from 5 0 0 1= 409.09.

E x a m p le 5.48. A substation operating at its fu ll lo a d o f 1000 kVA supplies a load ofP°l*


factor 0.71 lagging. C alculate the p erm issible a d d ition al lo a d at this p.f. an d the rating0'
ideal synchronous condenser to raise the substation pow er fa c to r to 0.87 lagging.

Scanned by CamScanner
T
Polyphase Synchronous M achines 673

Solution. Refer to Fig. 5.109 for this example.


Load kVA, oa = 1000
Load kW, ob = 10 0 0 x 0 .7 1 = 710
Load kVAr, ab = V(1000)*- ( 7 1 0 )T = 704.2
When synchronous condenser is installed, the operat­
ing p.f- is improved from cos [a o b = 0.71 lag to cos /cod
- 0.87 lag- But note that the load kVA delivered by substa­
tion remains
o a = o c = 1000 kVA.
Mew power delivered to load,
oa = oc
od = oc cos /cod =1000k'V\
o f = ce
= 1000 x 0.87 = 870 kW.
% Note that the load p.f. remains 0.71 lag. But the com­
bined p f o f the load and synchronous condenser at the sub- Fig. 5.109. Pertaining to Exam ple 5 .48 .
vital sta tio n terminals is 0.87 lag.

870
New load kVA, oe = - p p = 1225.352 . *

This means that with the installation of synchronous condenser, total load that can be
su p p lied at p.f. 0.71 lag is 1225.352 kVA. Therefore, permissible additional load at p .f. 0.71 lag
is given by
a e = o e - o a = 1225.352 - 1000 = 225.352 kVA.
New load kVAr, de = yloe2 - o d 2 = V( 1225.352)* - (870)* = 862.895 •
Load kVAr with the use of synchronous condenser,
cd = V(1000)2 - (870)* = 493.052.
.*. Leading kVAr supplied by synchronous condenser,
o f - ce = d e - c d = 862.295 - 493.052 = 369.843
.*. Rating of synchronous condenser = 369.843 kVA.
Exam ple 5.49. An industrial plant takes a load o f 4 MW at p f 0.8 lagging from a power
system network. It h as been decided to replace one 400 kW induction motor by a synchronous
motor o f the sam e rating in order to raise the plant pf. The induction motor and synchronous
motor both h a v e effic ien cy o f 90%. The induction m otor operates at p f 0.9 lagging but
synchronous m otor is designed to operate at p f0 .8 leading. Find the new p fo ft h e plant. Calcu­
late also the percentage reduction in line current.
Solution. Term inal voltage is taken as the reference phasor.

Complex power of plant = 4000 - j 4000 tan (cos 0.8)


A = 4000 - j 3000 kVA
Power input to induction motor

A w
0.9
Complex power requirement of induction motor
_400 4 0 0 tan(coa- 1 0.9)
0.9 0.9 •
B = 444.44 - j 215.254 kVA

% j u u i ii i l . u uy w u i i i x j u u i ii i u i
674 E le c tr ic a l M a ch in e ry

Complex power requirement of synchronous motor


400 . 400 L , __ - 1 n q\
= M +J- 5 J tan(cos °'8)
= 444.44 + j 333.33 kVA
Net requirement of plant
= A - B + C = 4 0 0 0 - j 3000 - 444.44 + j 215.254 + 444.44 +j 333.33
= 4000 - j 2451.42 kVA
= 4691.424 / - 31.5° kVA
v. New pf = cos 31.5° = 0.8526 lag
44000
0 0 0 x 10°
10 3 5000 , , A
Original line current = ^ xVr x 0 g = ^ y t

Here Vt = line voltage

New line current 4m ^4kA


V,
Percentage reduction in line current
500 0 - 4 6 9 1 .4 2 4 x 100 = 6.172%.
5000
E x a m p le 5 .5 0 . Sketch the p h asor diagram s o f a 3-phase synchr
(а) a t the m om en t o f synchronizing
(б ) when working as a motor an d
M mhenworking a s a generator, m arking the reference directions for voltages and
in the associated circuit diagram s. Indicate, on the p h asor the
each case. . - .. .

sumed to remain constant.


(а) At the moment of synchronizing, magnitude magnitude o fV , Both E gm iV , m ^

be in phase, because no power flow takes place and load angle 6 is zero, so that
zero Armature current / . is, therefore, also zero. The phasor and circuit diagrams, at the >»
ment of synchronizing, are as shown in Fig. 5.110 (a). in Fi? 5 110(»
(б ) When working as a synchronous motor, E ,m u st lag behind V, ^ s*°'™ g f , *ith

e th at V, remains vertical, since v, is uw ^ shown


( 0 When it is made to work as a s ^ c h r o n o u s generator ^ . m s t ^ „,

in n o (C. The arm ature ^ ^ diagroms - *


leading p.f. with vo tage equ f taIon„ vertic

Scanne
Polyphase Synchronous M achines 675
ln-0
Yt °“----- —_o

6- 0 '
* (a )

(6 )

Xs
(C)

Fig. 5.110. Synchronous machine phasor and circuit diagrams (a) at the moment of synchronizing
(o) when working as a motor and (c) when working as a generator.

Exam ple 5.51. An alternator is running at synchronous speed. Its field is now energised
from an ac source at rated frequency. Discuss the nature o f the generated voltage and the alter­
nator performance.
Solution. When field winding on the rotor is excited with alternating current, an alternat­
ing (or pulsating) field is produced. As per Example 3.21, this field can be resolved into two
components, one rotating forward at synchronous speed to with respect to rotor and the other
rotating backward ^t speed cowith respect to rotor. The relative speed between backward rotat­
ing field and stationary armature, given by the sum of forward rotor speed to and backward
rotating field speed ( - co) is zero. This component of field, therefore, generates no voltage in the
armature.
The relative speed between forward rotating field component and standstill armature is
double the synchronous speed, i.e. 2to. Thus, the frequency of the e.m.f. generated in the arma­
ture is 2/" and the magnitude of per-phase generated emf is given by

V2* (2f) Nph k w ■ = <2nf Nph ■A„ ■

where <f>max is the amplitude of pulsating, stationary field flux produced by alternating current
in the field winding of alternator.
Core losses in the armature at double frequency 2 f would be more. Field iron would also
have core losses. Alternator efficiency would therefore be reduced considerably.
Exam ple 5.52. Discuss what you understand by the statement, "Synchronous machine has
lost synchronism or synchronous machine has fallen out o f step.”
Solution. Steady-state operation of an alternator or a synchronous motor demands that
the relative speed between stator and rotor fields is zero. In other words, rotor must run at
synchronous speed when the synchronous machine is connected to an infinite bus. In such
Cases ; the rotor speed, equal to synchronous speed, is determined by bus-bar frequency and
the machine poles. However, an isolated alternator may run at any speed. The statement,
“synchronous machine has fallen out of step” applies to sync’ ronous machines connected to an
infinite bus. / * . 'j

Scanned by CamScanner
Aif] * •
m
676 E lectrical M achinery
iM l. • 1 to a cylindrical-rotorsynchronous machi,
,ad on
When load on theshaft, or mechanical input to a cyi .cm,
! •
■ ::I E, V the load anglebecomes more 4than qn° , -*■—
-u o r, 90 ^ - sin 5 now beginsto decreasee
ovrppH ^ —■— . l i i c iu a u uiJk*v — --a
siiiiif
:! . ! t-t
s k ..jq Sneed It is then said that a cylindrical-rot0r
therefore rotor no-longer runs at s^ chr0" si {]ariy when load on the shaft, or mechaniCa,
synchronous machine has lost synchronysm. itg maximUm stable limit (8 approximate
input to a salient-pole synchronous machi than synchronous speed
ly between 60° to 85°), the rotor begins to run at a sp^ ^ ^
.« « »

con t^ues runningS^syncl?ro^ous>specd as reluctance motor. Now if load on the motor exceed,
0 5 y* f J _ _ _L ] the reluctance motor loses synchronism and begins to run at sub-synchronous

sDeed as"a"7squirrel cage induction motor with increased line current and poorp/'. The operation of
reluctance motor as a squirrel cage induction motor is possible because of the presence of damper
bars in the pole faces o f salient-pole synchronous machine. In case field current is made zero ma
cylindrical-rotor synchronous motor, it may run at reduced speed with motor torque developed dlu re
.to a combination o f eddy-current and hysteresis torques.
h . The asynchronous operation of a synchronous motor at subsynchronous speed is damaging
and therefore the motor must at once be disconnected from the supply mains.
5.20. Starting of Synchronous Motors
When three phase currents are flowing in the 3-phase winding on the stator of a
synchronous motor, a synchronously rotating magnetic field is set up. If the rotor is stationary,
then relative speed between standstill rotor field and rotating stator field is equal to the
synchronous speed. Consequently the torque angle between rotating stator poles and station­
ary rotor poles is a function of time. An examination of Eq. (3.78) reveals that the electromag­
netic torque varies sinusoidally with time— it reverses during each half cycle and therefore, the
average value of electromagnetic torque over a complete cycle is zero. Hence a synchronous motor,
on its own, has no net starting torque. . ( *y . /

u Pun-m.
ft ?■]•ii1 jn.rr

iw nor; b n i i . e d .' iif


. . ■ 1■

\A•,r*. t IV*!'i'Vi-V
*0 'h ^
. . . (“) W ■
F ig . 5 .1 1 1 . P e rta in in g to th e developm ent of s ta r tin g to rq u e in a sy n ch ro n o u s m otor.

refer ^
For another way of describing that a synchronous motor has no net startin g torque,
two pole synchronous motor of Fig. 5.111. W ith three phase currents in the three phase ar
ture w in ding , statorN, S poles rotate at synchronous speed. A t the instant shown in Fig-
(a), statorN, S poles attract stationary rotor S, N poles— thus producing a clockwise torqu

S ca nned by CamScanner
Polyphase S yn ch ron o u s M ach in es 677

. i l l
tor After half a cycle, i.e. after 0.01 sec = — for a 50 Hz supply system, stator poles
tne r° _ \ '
the position shown in Fig. 5.111 (6). Now the stator N, S poles repel stationary rotor
0C°c coles, thus producing a counter-clockwise torque. In Fig. 5.111 (a), the rotor is urged to
te clockwise, but soon after, i.e. after 0.01 sec, the rotor is urged to rotate anti-clockwise.
rf rotor due to its inertia can’t respond to such quick reversals of electromagnetic torque,
equently the rotor remains at standstill showing thereby that in a synchronous motor, the
net starting torque is zero.
Steady state electromagnetic torque is developed in a synchronous motor only when mag-
tic locking between stator and rotor fields takes place. This can happen only when relative
^eed between stator and rotor fields is zero. In Fig. 5.111 (a), if stator and rotor fields rotate
^the same direction as shown and at the same speed, then rotor S pole is magnetically locked
with stator N pole and rotor N pole with stator S pole and in this manner steady state torque
s developed. This means that rotor must be brought up to a speed equal to the rotating stator-
field speed for the production of steady-state electromagnetic torque. This can be accomplished
by two methods, namely
(a) auxiliary motor starting and
(b) induction motor starting.
These two methods of starting synchronous motors are now described below. ,
(a) A u xiliary-m otor sta rtin g . The purpose of the auxiliary motor is to bring the
synchronous motor speed, near to its synchronous speed. The auxiliary motor may be an induc­
tion motor or a d.c. motor.
If 3-phase induction motor is used as an auxiliary motor, then it is mechanically coupled
with synchronous motor. Both the motors have the same number of poles and are energised
from the same 3-phase supply. The auxiliary 3-phase induction motor brings the main motor
speed almost equal to its synchronous speed. At this time, the arm ature winding of
synchronous motor is also energised from 3-phase supply. Now when the field winding of main
motor is connected to d.c. source, the field poles get locked with those produced by armature
winding. As a result of this, main motor starts running as a synchronous motor at synchronous
speed. The auxiliary induction motor can now be disconnected from three-phase supply.
Sometimes an induction motor with two poles, fewer than the synchronous-motor poles, is
used as an auxiliary motor. This induction motor runs the main motor above its synchronous
speed. After this, the induction motor is switched off and the synchronous motor armature is
switched on to 3-phase a.c. supply. When the speed of the set is just above synchronous speed
of the main motor, the field winding is energised from d.c. supply. By the time the field current
rises to its constant value, the set attains synchronous speed.
If the synchronous motor is coupled with a d.c. machine, as it is usual in the laboratories,
then dc machine is first run as a d.c. motor. The main motor, now made to operate as a
synchronous generator, is synchronized with the 3-phase supply system in the usual manner.
If the d.c. motor is now switched off, the main motor starts running as a synchronous motor.
The disadvantage of this method of starting is that the motor can’t be started under load ;
in case it is desired to do so, the auxiliary-motor rating will be large, thus increasing the cost of
the set. In view of this, the auxiliary-motor starting is used only for unloaded synchronous
motors. At the same time, the auxiliary motor has to overcome primarily the inertia of the
unloaded synchronous motor, its rating is therefore much lower than the rating of synchronous
motor.

sS rTned by CamScanner
678 E lectrical M ach in ery ------------------------ ------------------- ------------- ------------.----------------------------------------------

(6) S tartin g by d am p er-w in d in g (o r in d u ctio n -m o to r sta rtin g ). In order to make the


motor a self-starting synchronous motor, the amortisseur or damper win ing is embedded jn
the slots in the rotor pole faces. This winding is short-circuited at o en s y metal rings
Thus the damper winding is exactly similar to the squirrel cage winding of 3-phase induction
motors.
When armature is excited from 3-phase supply, a rotating magnetic field is established.
This rotating field and the damper winding develop induction motor torque-rotor is, thereforej
accelerated up to about synchronous speed. Note that the synchronous motor with damper bars
in its rotor pole shoes, runs as a squirrel cage induction motor, from standstill up to near its
synchronous speed. If the field winding is now energised from a d.c. source, the rotor and stator
poles will lock together provided the rotor poles just approaching the stator poles are of opposite
polarity as shown in Fig. 5.112. In all other cases (e.g. rotor S pole ahead of stator N pole, rotor
N pole near stator N pole etc.), the rotor and stator poles will not lock together immediately
after the instant of closing the field circuit. In view of this, the synchronous motor torque is not
constant but consists of induction motor torque plus a sinusoidally time-varying torque of very
low frequency, equal to the slip frequency. This pulsating torque causes violent disturbances in
the supply current. In case the load torque required is not too great, the positive half cycle of
the synchronous motor torque pulls the rotor into synchronism under favourable conditions.
Actually pull-in process of a salient-pole synchronous motor is quite complex and will not be
discussed here.

Fig. 5 .1 1 2 . P ertain in g to the startin g of synchronous m otors.

Instead of direct-on-line starting, the synchronous motor is sometimes started by star delta
starting, reactor starting or auto-transformer starting, in order to limit the starting current.
The field winding has usually a large number of turns as compared to the stator turns. At
the time of starting very high voltage may be induced in the field winding. The stator may be
thought of as the primary winding and the field winding as the secondary winding of a trans­
former. On starting, rotating magnetic field cuts the field winding at synchronous speed and,
therefore, high values of e.m.fs. are induced in it. This high value of induced e.m.f. may cause
breakdown of the field winding insulation. The voltage induced in the field winding can be
limited by short-circuiting the field winding or by connecting it to a resistance whose value is
about 7 to 10 times the field winding resistance itself. As rotor speeds up, the induced e.m.f. in
the field winding decreases, therefore, the external resistance in the field circuit should be
gradually reduced. When the rotor reaches normal speed, external resistance in the field circuit
is reduced to zero and field winding is opened ; after this the field winding is connected to a d.c.
source. Another advantage of short-circuiting the field winding or of connecting it in series with
external resistance during synchronous motor starting is that additional torque is developed
due to interaction between rotating field and field-circuit m.m.fs. This additional torque adds
to the induction motor torque developed by damper bars and in this manner, starting torque is
increased. Star-delta starting, reactor-starting or the auto transform er starting also helps in
reducing the voltage induced in the field winding.

Scanned by CamScanner
Polyphase Synchronous M achines 679

5 20.1* Starting of synchronous motors against high torques. Auxiliary-motor starting


, ^auction-motor starting for synchronous motors are employed, when the load requires low rotatr
torque- For loads requiring high starting torques, the synchronous motor may be started as fol­
lows- . . .
(o) Synchronous-induction motor. The rotor of synchronous induction motor is similar to
tor ofwound-rotor induction motor. At the time of starting, high resistance is inserted in
he rotor circuit in order to develop high starting torque as in a wound rotor induction motor.
as the motor working as slip-ring induction motor speeds up, the external resistance is gradual-
reduced to zero. Zero external rotor resistance would cause the rotor speed to be very near to
vnchronous speed. At this time, rotor short-circuit is removed and rotor winding is switched
over to d.c. supply as shown in Fig. 5.113. The rotor poles thus created would be attracted by
stator poles and synchronism is thus achieved.
Rotor

resistance
Fig. 5.113. Synchronous-induction motor circuit diagram .

Synchronous induction motor has a larger air gap than that of a slip-ring induction motor,
because the machine must operate as a synchronous motor at normal loads with high stability
limit (proportional to air-gap length).
(b) Super-synchronous motor. The stator of this motor is not bolted to the bed plate, but is
suspended in bearings so that it can rotate. The rotor is mechanically coupled with load requir­
ing high-starting torque. When three phase supply is given to the stator through slip rings,
rotating magnetic field running at synchronous speed with respect to stator is set up. Since the
stator structure has less inertia than the rotor plus load inertia, unloaded stator structure
starts speeding up in a direction opposite to the rotating magnetic field while the rotor remains
stationary When the stator speed is near the synchronous speed, the rotor is excited by direct
current, thus converting the induction motor into a synchronous motor Under these conditions,
stator is running at synchronous speed n5 (say anticlockwise), stator field is rotating clockwise
at speed n and rotor is stationary. Now band brake is gradually applied to the stator structure
to decrease its speed. I f the stator speed decreases by nr.p.m . then the rotor must ru n at r
r p m. clockwise m the direction of rotating field so that the r e t o e p p e e d b e ^ e e n stator an.
rotor fields is zero. When the stator structure is brought to rest by band brake, the rotor start
running clockwise at synchronous speed in the direction of rotating field. In this manner, max
mum synchronous motor torque equal to the pull-out torque, isavafiable for starting the heai
torque load on the rotor shaft. In an ordinary synchronous motor, the pull-out torque is ava
able only when the rotor speed is synchronous. Super-synchronous motors are used in pap
and cement m ill drives where starting duty is very severe.

Scanned by CamScanner
ii:£

680 Electrical Machinery


1 ? •r:
5.21. Som e W orked E xam p les
I - In this article, some typical problems pertaining to polyphase synchronous machines
solved. Most of these problems have appeared in one or the other competitive examination ^
i}[5
.!** E x a m p le 5.53. A 3-phase, 2-pole, 4 MVA, 50 Hz, star-connected turbogenerator hnc n
follow ing d a t a : th?
I Arm ature slots = 72, conductors p er arm ature slot = 4
Rotor slots = 24, rotor slot angular pitch = 10°
ii
Conductors p er rotor slot = 20 ■
Armature leakage impedance per phase = 0.1 + j 2 CL Its O.C.C. at rated speed is given below ■
tip
i\
If, A 40 80 120 160 200 240 280
Ii! 320 360
ipi
I !I Er, V 2490 4980 7470 9390 10620 11460 12030 12450 12660
111
>ri< ------- -------
m 1
ilf ■
Calculate the voltage regulation at fu ll load at 0.8 p f lagging.
Solution. Terminal voltage per-phase,
v _ 11000
Vt = ~ T Js ~ = 6 3 5 1 V

Full-load armature current,

/o = ^ T I I = 2 0 9 9 5 A
Phase values of E f are tabulated below. Open-circuit characteristic is obtained by plotting
per-phase E f and field current/^ as in Fig. 5.114.

If. A 40 80 120 160 200 240 280 320 360


Ef, V 1437 .6 4 2 8 7 5 .3 4313 5 4 2 1 .5 6 1 3 1 .6 4 6 6 1 6 .6 6 9 4 5 .7 7 1 8 8 .2 7309

From Fig. 5.14, air-gap voltage E r is given by


E r =V t + Ia (ra + jxa[)
Taking Vt as reference phasor,

Vt = 6351 +./0, Ia = 209.95 (0.8 Tj0 .6 )


E r = 6351 + 209.95 (0.8 - j 0.6) (0.1 + j 2)
= 6351 + (167.96 - j 125.97) (0.1 + j 2)
= 6619.74 + j 323.32 = 6627.63 Z2.8°

c 1 J he ! f ld mmf m term s of field current, corresponding to E r = 6627.63 V, from OCC of Fig.


5.114 is E r = 242 A.

Fundamental field mmf per pole, from Eqn. (3.59 (a)) is


n 4 , Nf - I f
F i f = n kf ~ P A T s/Pole
For field winding, r = 10° and rotor slots per pole

9 =f = 12 '

Scanned by CamScanner
Polyphase Synchronous Machines 68£
Art- 5.2*1 “
. Distribution factor for field winding,
s in 91s in 1 2 x 1 0

kd = ~ J = ~ T ~ l ^ ^ 0S28 i
q sin £ 12 sin —

T a k in g f u ll-p itc h fie ld c o ils , kp = 1.0


.. W i n d i n g f a c t o r f o r f i e l d w i n d i n g , k w = kd x k p = 0.828
24 x 20
Total field turns, Nf = — - — = 240

4 240 L
F u = - x 0.828 ——1 = 126.51 If ATs/pole.
71 ‘ 2
F u n d a m e n ta l a r m a tu r e re a c tio n m m f F a, f r o m Eqn. (3.72 ( 6 ) ) , is

F„ = 0.9 ^ k t ATs/pole

o xr 72x4 .. 72 2 x 180 eo
Here m - 3, Nph 2x3 48, q gx3 »Y 72

qv c m • -----------
12 x 5
sin _i_L
" sin — -—
* .= — = --------------- = 0.9552
7 . 5
q sin ^ 1 2 s in —

Assuming full-pitch armature coils, kp = 1, k w - 0.9552.


p a = 0.9 3 x 48 x 20&95 x 0.9552 _ 12995 3 A Ts/poie

Since we are working in terms of field current, the armature mmf F a, in term s of equivalent
field current is given by
armature mmf Fa in ATs/pole 12995.3
= 102.72 A
field mmf F lf per pole 126.51
An examination of Fig. 5.14 reveais that the angle between F r and Ia or Fa is 90 + (5, where
angle P = p f angle + angle by which E r leads Vt.
... p = cos" 1 0.8 + 2.8° = 36.87 + 2.8 = 39.67°

From Fig. 5.24 (5), F ) = Ff. + F 2 - 2 F r ■Fa cos (90 + P)


= 2422 + 102.722 - 2 x 242 x 102.72 cos 129.67°
F f= 317.57 A
Excitation em f E f, corresponding to equivalent field curren t If = 317.57 A from OCC is
^168 V per phase.

_ E r Y l ^ inn = 7168 - 6 — x 100 = 12.864%.


Voltage regulation = ~ y — * !®0 - 635j

Scanned by CamScanner
If— *
Fig. 5 .1 1 4 . O pen-circuit ch a ra cte ristic for E x a m p le 5 .5 3 .

E x a m p le 5.54. A 2000 kVA, 11 kV, 3-phase star-connected altern ator h a s synchronous im­
p ed a n ce o f Zs - 0.3 + j 5 D. p er phase. It delivers fu ll lo a d current a t a p f o f 0.8 lagging and
norm al rated voltage. Compute the term inal voltage fo r the sam e excitation a n d current at 0.8
P f tending. (J.A.S., 1997)

S o lu tio n . Terminal voltage per phase, Vt = = 6351 V

Full-load arm ature current per phase, I a = - p Q0° = 104.976 A


^ V3 x 11
With Vt as reference phasor, Vt = 6351 + j 0
4 = 104.976 (0.8 - j 0.6)
.•. Excitation emf, E f= V t + Ia Zx
= (6351 +y'0) + 104.976 (0.8 - j 0.6) (0.3 + j5 )
_ = 6351 + (83.98 - 62.986) (0.3 + j 5)
E f = 6 691.124 + j 401.004 = 67 0 3 .1 3 /3 .4 3 °
n o Nr°,W’ ^ itH the magnitude of excitation voltage held fixed, the nature of load pf is altered to
0.8 pf leading. The new terminal voltage Vt) with load cu rrent rem aining the same, can be
computed as under :

Scanned by CamScanner
Polyphase S yn ch ron ou s M ach in es 683

Take Vt as reference phasor again


Ef = Vt + 104.976 (0.8 + j 0.6) (0.3 +j 5)
E f= (V, - 289.74) + j 438.8
E f = (Vt ~ 2S9.74)2 + 438.82 = 6703.132 '
or
Vt = [6703.132 - 438.82]1/2 + 289.74 = 6978.5 V
Line terminal voltage = V3 x 6978.5 = 12086.762 V.
Exam ple 5.55. A 30 MVA, 11 kV, 50 Hz, 6-pole, star-con n ected a lte r n a to r h a s
2 = 0.005 + j 0.70 p.u.. Find the load angle and power factor in case the alternator is delivering
rated current at rated voltage with excitation em f 1.5 pu.
Solution. Here Ia = 1.0 pu, Vt = 1.0 pu, r0 = 0.005 pu, Xs = 0.70 pu, Ef - 1.5 pu.
From the phasor diagram of Fig. 5.23 (a), we get
E) = (V, cos 0 + Ia ra)2 + (V, sin 0 + Ja •X f

or 1.52 = [cos 0 + 0.005]2 + [sin 0 + 0.70]2


or 2.25 = cos2 0 + 0.0052 + 0.01 cos 0 + sin2 0 + 0.49 + 1.4 sin 0
or 0.01 cos 0 + 1.4 sin 0 = 0.759975 ~ 0.76
As per Example 5.8, the above expression can be written as
1.4000357 sin (0 + 0.40925) = 0.76
or 0 = 32.4682°
Power factor = cos 32.4682 = 0.8437 lagging
It is also seen from Fig. 5.23 (a) that
£? + V? - 2 £ r V, ■cos 5 = (7„ Z,)2
1.52 + l 2 - 2 x 1.5 cos 5 = (0.0052 + 0.7)2) = 0.49 0 025
„ -1 3.25 -0.490025 nr,rn
:. 5 = cos 1 ---------- ---------- = 23.075°.
O
Example 5.56. A salient-pole synchronous generator, with Xd = 1.2 pu and X(/ = 0.8 pu, is
connected through an external reactance o f 0.10 pu, to an infinite bus o f voltage 1.0 p.u. The
generator delivers rated VA at p f 0.9 lagging at its terminals.
(а) Calculate the generator terminal voltage and its armature current.
(б) Calculate the generator load angle
and the excitation voltage. JXqlq
Solution. For the system shown in Fig.
5.115 (a), the phasor diagram is shown in
Fig. 5.115 (6) where Vb = bus-bar voltage
= 10 pu, V, = generator terminal voltage,
K ~ generator current, Ef = excitation voltage
and 5 = generator load angle.
At the generator terminals,
Vt Ia = 1.00 p.u. V 1
Xd.Xq
'a XgtO-1 <
It is seen from the phasor di; (Q
5.115 (6) that
Ef ' \
^7t —Vb + j 1„ •xc (a) (h)
or Vi Fig. 5.115. Pertaining to Example 5.56.

Scanned by CamScanner
l A r t 5.2i
684 E lectrical M achinery

With V, as reference phasor, Vt = Vt, Ia = (0.9 - j 0.436) a


. Vb = Vt - j (0.9 - 0.436) l a x 0.1
= V, - j 0.09 Ia - 0.0436 Ia
1
or Vb = (V, - 0.0436 I a) - 0.09 Ia |
I
. V2b = (Vt - 0 .0436 l af - (0.09 IaY
= Vf + 0.0019 J2 - 0.0872 V, l\ - 0.0081 I a 1.00 r

Vf - 0.0872 (1) - 0.0062 i f = 1.00. But. Ia


r
—1 0
or

.2 0.0062 X 1
V ?- - 1.0872 = 0

or - 1.0872 Vf - 0.0062 = 0
Its solution gives Vf = 1.092873 and Vt = 1.0454 p.u.
= 0.9566 p.u.
Ia Vt 1.0454
(b) Also E f = V, + j Ia Xq
With Vt as reference, E f = 1.0454 + j (0.9 - j 0.436) x 0.9566 x 0.8
= 1.3791 + j 0.6888 = 1.5415 /2 6 .5 4 °
.•. Load angle 6 = 26.54°. Power-factor angle 0 = cos 1 0.9 = 25.842
Id = l a sin (8 + 0) = 0.9566 sin (26.54 + 25.842) = 0.7577 pu
.-. Excitation voltage, Ef = E / + Id (Xd - Xq)
= 1.5415 + 0.7577 x 0.4 = 1.8446 pu.
E xam p le 5.57. A salient-pole synchronous motor has Xd = 0.85 pu an d Xq = 0.55 pu. It is
connected to bus bars o f 1.0 pu voltage, while its excitation is adju sted to 1.2 pu. Calculate the
maximum power output that the motor can supply without loss o f synchronism . Compute the
minimum pu excitation that is necessary for the m achine to stay in synchronism w hile supplying
the full-load torque. (I.A.S., 1996)
Solution. The power output in a salient-pole synchronous motor is given by
~ E f' V . Vf
P = - t y — sin 5 + —
<?
1 .2 x 1 . 0 1 1
= - sin o + — sin 2 6
0.85 2 0.55 0.85
v
or P = 1.412 sin 6 + 0.321 sin 2 8
dP
Power P is maximum when ~ = 0
do
dP
•; = 1-412 cos 8 + 0.321 x 2 cos 2 8 = 0

z 1-412 cos 8 + 0.642 [2 cos2 8 - 1] = 0


or 1.284 cos2 8 + 1.412 cos 8 - 0.642 = 0
From this, we get 8 = 69.764°
Maximum power output, J>„„ = 1.412 sin 69.764 + 0.321 sin 2 x 6 9 .7 6 4 = 1.5332 pu

Scanned by CamScanner
fI

Polyphase Synchronous M achines 685

The excitation would be minimum when = 0. In Example 5.32, it is shown that the load
ie 5 at which excitation emf is minimum is obtained from
joS1 v2 v v
sin3 5
°-30
1 0.85x0.55
cos 5 = 0.6417 sin3 5.
°r Its trial and error solution gives 8 = 63°
P V? xd- x q .

V V V sm 2 0
E1 _ *^0
•. tofmm -
V' sin
• 8X
Ad
! _1 0.30
sin 2 x 63°
2 0.85 x 0.55
= 0.70635 pu.
sin 63°
0.85
Example 5.58. A given 3 MVA, 50 Hz, 11 kV, 3-phase, Y-connected alternator, when sup­
plying 100 A at zero p f leading has line to line voltage of 12370 V; when load is removed the
term in a l voltage falls down to 11000 V. Predict the regulation o f the alternator when supplying
full-load at 0.8 p fla g . Assume an effective resistance of 0.4 Q per phase.
12370
Solution. Per-phase terminal voltage, V, = = 7142 V

Pu-phase excitation voltage, " - H000 = 6351V


Bf
Is
. 3000
Full-load phase current,
“* W 7 u =
For zero p f leading, the phasor diagram is shown in Fig. 5.116. It is seen
from this diagram that
Ef + V f - 2 Ef V, cos b = (Ia Z f
Here load angle 8 ~ 0.
.-. (V, - E ft = <i0 Z ,f
_ V ,- E f 7142 - 6351 „ Fig. 5 .1 1 6 .
or
Z» = _ L 100 A ~ 7'9 1 a P ertain in g to
E xam p le 5 .5 8 .
Synchronous reactance, Xs = V7.912 - 0.42 - 7.90 Q
For 0.8 pf lagging, phasor diagram of Fig. 5.23 (a) gives
E j = (6351 x 0.8 + 157.46 x 0.4) + (6351 x 0.6 + 157.46 x 7.90)2
= 5143.7842 + 5045.5342
Excitation voltage, E f= 7211.58 V
Ef - V , 7 2 1 1 .5 8 -6 3 5 1
Voltage regulation x 100 = - x 100 = 13.55%.
V, 6351
Exam ple 5.59. A star-connected alternator is synchronized with an infinite bus o f H I
lts steam input is then increased till its output power is 15 MW. Now, when its excitation en
C rea sed to 130%, the synchronous machine starts operating at a p f o f 0.8 lagging. Compute
synchronous reactance o f the m achine. Neglect armature resistance.

Scannea byuarnScanner
686 E le c trical M a ch in e ry ______________________________

Determine the pow er factor, loa d angle a n d the arm atu re current o f the m achine before the
excitation e m f is increased. ' ' ’• *000]
Solution. For star-connected alternator,
g
V3 V, ■Ia cos 0 = Power E»
V3 x 11000 x Ia x 0.8 = 15 x 106 W =13V„\
xVb - 6 3 5 1 V
Per-phase armature current, 6351V

/ _ 15 x 106 = 984.15 A
° ^3 x 1 1 0 0 0 x 0 .8
Per-phase bus voltage, 90°6a
11000
= = 6351 V

Under the condition of increased steam


input and increased field excitation, the phasor
diagram is shown in Fig. 5 .1 1 7 (a). This
diagram gives (6)
Fig. 5 .1 1 7 . P e rta in in g to E x a m p le 5.59.
(OA)2 + (AB)2 = OB2
^2
(6351 x 0.8)2 + (6351 x 0.6 + 984.15 Xs)2 = (1.3 x 63 5 1 )J
or 0.82 + (0.6 + 0.15496 •Xs)2 = 1.32 o r X s = 2 .7 4 Q
Under the condition of increased steam input, the phasor diagram is shown in Fig. 5.117
E •V
(6). Here 3 ^ b sin 5 = 15 x 106

or 3 sin 5 = 15 x 106 or 8 = 19.856°.


2 .7 4

It is seen from the phasor diagram th at pf = cos ^ = cos 856 = 0.985 leading.
2 2

Also, 19.856
/„ X, = 2 x 6351 sin

Arm ature current, Ia = - x 6 3 5 1 sin 9.9325 _ ?9 9 2g4 A

5.22. S yn ch ronous M achine A pplications


Synchronous machines have wide variety of applications as listed below :
S yn ch ron ou s G en erato rs. Synchronous generators or altern ato rs are used universally
for the generation of three-phase power in the whole world.

t Se cyliln drica‘-r° tor altern at°rs <UP to 1000 M W or so) in thermal


,n a h Z a n d rrS T hydro' statlons Synchronous generators are also used
in ships and air-crafts. The range of ratings and speeds of synchronous generators is very wide.
All synchronous generators are invariably three-phase star-connected m achines

o v e ^ ^ = ^ ? er ° rS- T h T 3re alS° Ca" ed condensers These are


comnensators are ™ . 0U* mach,nes ln ratings up to about 100 M Vr. Synchronous
situations where s t a f ® nara e reactive power and thus im prove the system power factor in
situations where static capacitors become more expensive.

s m a K thev a S MOt° rS- These are USUally made in sizcs above about 5 0 kW. because in
sm all sizes they are more expensive as compared to induction motors.

Scanned by CamScanner
,, Polyphase Synchronous M achines 687
prob^il— " ” ' ’
The main disadvantages of synchronous motors are :
(.) the requirement of d c supply for field excitation,
(it) the starting, synchronizing and control devices are more expensive, and
(iii) the motor is more sensitive to system disturbances.
Its principal advantages are as follows :
(j) An overexcited synchronous motor can generate reactive power to improve the system
0wer factor and at the same time drive a constant-speed load.
(ii) It has high operating efficiency and constant speed.
(iii) For operating speeds less than about 500 rpm and for power requirements from
35 kW up to about 2500 kW ; size, weight and cost of synchronous motors are much less than
those of induction motors of the same speed and kW rating. It is because of these reasons that
synchronous-motor applications are on the rise for such speed and power requirements.
In view of the above, synchronous motors are preferred for driving the loads requiring high
powers at low speeds ; for example, large low-head pumps, reciprocating pumps and compres­
sors, rolling mills, ball mills, crushers, pulp grinders etc.
The use of synchronous motor for a specific purpose requires the considerations of the fol­
lowing specifications :
(i) Starting torque. It pertains to the ability of the motor to accelerate the load.
(ii) Pull-in torque. It refers to the ability to pull into synchronism when changing from
induction to synchronous motor operation.
Pull-out torque. It pertains to the ability of the motor to remain in synchronism under
(iii)
rated lo a d conditions.

PROBLEMS
5.1. (a) Give th e co n stru ctio n al d etails of rotor of both salient-pole and cy lin d n ca l-ro to r sy n ch ro n o u s
machines.
(b) D escribe th e v ario u s sch em es used for exciting large synchronous m achin es. W h ich sch e m e is b ein g
preferred th ese d ays for e x citin g v ery larg e turbo-gen erators and why ?
5.2. (a) D iscuss th e following giving reason s :
(i) C ylin d rical-roto r a lte rn a to rs h av e sm all d iam eter and large core length.
(ii) Salient-pole a lte rn a to r s h av e larg e d iam eter and sm all core length.
(b) A 4 1 5 V, sta r-co n n e cte d , 3 -p h a se , 4-pole synchronous g en erato r h a s 3 6 sta to r slo ts ca r ry in g double layet
3-phase w indings form ed th rou g h uniform ly distributed coils each short-p itch ed by one slot. T h e d ia m e te r at
the air gap is 2 0 cm and th e effective ax ia l length is 6 0 cm. T h e ro to r driven a t 1 5 0 0 rp m is e x cite d to resu l'
in a sinusoidally v ary in g air-g ap flux d en sity distribution in sp ace w ith a p eak v alu e of 0 5 T. C a lc u la te th«
number of series tu r n s p er p h ase to g e n e ra te rated no-load voltage. (I.A.S., 1996
lA n s. tfe) 3 8 tu rn ;

5 .3 . D erive an em f exp ression for an a lte rn a to r from fund am en tals show ing cle a rly th e e x p re ss io n s fc
Pitch and d istrib u tio n fa cto rs. F in d , th erefro m , th e ra tio of induced em fs of n th h a rm o n ic to fu n d a m e n ta l. C a
adopt sh o rt-ch o rd in g for sin g le-lay er w inding ?
5.4. (a) E x p la in th e se v e ra l m ean s by w hich an a tte m p t is m ade to obtain th e w av efo rm of th e g e n e r a l.
enrf in a 3 -p h ase a lte rn a to r sinu soid al.
, (6) The p h ase e m f of a 3 -p h a s e , 5 0 Hz a lte rn a to r consists of a fu n d am en tal, a 2 0 % th ird h a rm o n ic and
10% fifth h arm o n ic. T h e am p litu d e of th e fu n d am en tal voltage is 1 0 0 0 V. C a lcu la te th e rm s line v o lta g e w h
tlle a lte rn a to r w in dings a re in (i) s t a r an d (ii) delta.
If tho is 12 fl calcu late th e circu la tin g c u rr e n t w h en th e m a ch in e is de
to-uiMUd “ per P IAn*-(l” 1,1 1230 85 V 710141 V •3 929

Scanned by UamScanner
688 Electrical M achinery

5 .5 . The field-form of a 3 -p h ia e 6 0 H a ft.


£ £ 3 f c w S '^ t h ^ r * U * e ca n s being c o n n e d J *
groups. The a rm a tu re d iam eter is 125 cm and core length 4 5 cm . D eterm m
(a) the expression for th e instantan eou s em f per conductor,
(6) the expression for the instantaneous em f per coil and ,
(c) the rm s phase and line voltages. [Ans. (a) 17.675 sin wt + 5 .3 3 sm 3 wt + 3 .5 3 5 s.n 5 wt + 1.77 sir>7 ^
(6) 67.61 sin wt + 22.624 sin 3 wt + 5.5 sin 5 wt - 2 .7 5 2 sin 7 u,C (c) 4200 V, 7195.4 V|

5 .6 . (a) W h at are th e cau ses of harm onics in th e em f w aveform s of sy n ch ro n ou s g e n e ra to rs and what means
are adopted to minimise! th em ? ’
(.b) A 3-phase a lte rn a to r h as 2 slots per pole p er p h ase an d coil sp a n of 5 slo t p itch es. The flux density
wave of a lte rn a to r consists of a fundam ental and a 25% th ird h a rm o n ic. C a lc u la te th e p e rcen tag e increase in
th e phase voltage due to harm onic. s ‘ * ^0-893<J|
5 .7 . (a) W h at do you u n d erstan d by th e te rm sp ace-p h ase an gle ?
(b) In a cylin d rical-rotor alte rn a to r, a rm a tu re c u rre n t is in p h ase w ith th e e x cita tio n voltage. Develop the
sp ace and tim e-p h asor d iag ram s for this a lte rn a to r. D raw th e se tw o d ia g ra m s in one p h a so r d iag ram and show
th a t a rm a tu re reaction m m f a t unity p f is cro ss-m agn etizin g in n a tu re .
D iscuss w h eth er th e field poles a re ah ead o r behind th e re s u lta n t a ir-g a p field.
5 .8 . Develop and draw' space and tim e-p h aso r d iag ram s for a cy lin d rica l-ro to r m a ch in e in ca se the alternator
o p erates a t an in tern al pow er facto r of (o) zero lagging (b ) zero lead in g an d (c) cos y lagg in g .
D iscuss th e n a tu re of a rm a tu re reactio n m m f an d th e d evelopm ent of e le ctro m a g n e tic torque in each of
th e th ree cases listed above.
5 .9 . (a) D raw th e combined space and tim e-p h aso r d ia g ra m for a cy lin d rica l-ro to r a lte r n a to r w ith armature
cu rre n t lagging the excitation emf. D iscuss about th e v ario u s p a ra m e te rs involved in it.
(6) E xp lain how th e d iag ram of p a rt (a ) ca n be applied to a sy n ch ro n o u s m oto r.
(c) D iscuss th e location of field poles rela tiv e to th e r e s u lta n t a ir-g a p m m f in p a r ts (a) an d (b).
5 .1 0 . (a ) In a salien t-p ole synchronous m ach in e, exp lain how th e a r m a t u r e flu x d ep en d s on the spatial
orien tatio n of th e a rm a tu re m m f w ave relativ e to th e field poles.
(6) ■
D iscuss th e n a tu re of a rm a tu re re a ctio n flux in a sa lie n t-p o le sy n ch ro n o u s m a c h in e in case armature
cu rre n t (i) lags th e excita tio n em f E f by 9 0 ° (ii) lead s E f by 9 0 ° an d (Hi) is in p h a se w ith Ef.

5.11. (a ) In case a rm a tu re te rm in a l v o ltag e, a r m a tu r e c u rr e n t, p o w e r-fa cto r a n g le e tc. a re known, draw


and d iscu ss th e g en e ra l p h a so r d iag ram for a cy lin d rica l-ro to r a lte r n a to r sh o w in g th e v a rio u s m m fs as well.
(b) E xp lain how o p en -circu it an d sh o rt-c irc u it te s ts a re co n d u cted on a sy n ch ro n o u s m a ch in e .
(c) W h a t is an air-g a p lin e ? D iscu ss its sig n ifican ce w ith re s p e c t to th e m a g n e tiz a tio n ch a ra cte ristic of an
a lte rn a to r.

5 .1 2 . A sy n ch ro n ou s g e n e ra to r on load ta k e s a r m a tu r e c u r r e n t a t a la g g in g p o w e r-fa c to r an gle G. Its leakage


im p ed an ce is assu m ed negligible. F o r a n in te rn a l p o w e r-fa cto r an g le y , find o u t th e a n g le by w hich
(a) a ir-g a p e x cita tio n m m f lead s o r lag s a r m a tu r e mmf
(b) r e s u lta n t m m f lead s o r lag s a r m a tu r e m m f
(c) e x cita tio n e m f lead s or lag s r e s u lta n t m m f an d
(d) a ir-g a p e m f lead s o r lag s a ir-g a p e x cita tio n m m f.

IA n s. (q) leads by (9 0 + y ) (b) leads by (9 0 + 0) (c) lags by (9 0 - 8) Id) lags by (90 + 0)1

5 .1 3 . A 3 -p h a s e sy n ch ro n o u s m o to r, co n n e cte d to an in fin ite b u s, is su p p ly in g a co n sta n t-to rq u e load. Its


field c u r r e n t is a d ju sted to o p e ra te it a t u n ity p.f. N e g le ct le a k a g e im p e d a n c e .

Now if its field current is increased, explain what happens to the following ; *
(a) The magnitude of resultant flux wave.
(b) The magnitude of armature current and its pf.
(c) The magnitude of armature current in phase with V,.
‘ id) Rotor movement away from resultant mmf wave or towards it.
[/ fi'.-U I 'f : * . 1
(e) The space-phase angle between armaturo mmf wave E„ and the resultant air-gap mmf wave Fr

"Scanned by CamScanner
L—- ' ” y ~ " ~ jfol> phase Synchronous Machines 689
[H in t. R cfe r to F l B 5 3 1 ( 0 1 ----------------------------------

|Ans. ( o ) R e m a in s c o n s t a n t . ( b ) I(I i n c r e a s e s a n d p f h i
up re s u lta n t m m f w a v e , i.e. in th e d ir e c tio n o f ro to ti e ad in g (c) R e m a in s u n ch an g ed . Id) T o w ard s
JJJ ‘ e f w h e re 0 = p f a n g le ) ° i a i ,0 n (e) b etw een F „ an d F r in cre a s e s from 9 0 ° to

" 5 .1 4 . A r o u n d - r o to r s y n c h r o n o u s m o to r on load A
inlpedance is assum ed negligible. Find out the angle bywhfch CUrFent * a lead in g p f an g le 0. Its lea k a g e

(fl) a ir-g a p e x c i t a t i o n m m f le a d s o r la g s a r m a t u r e m m f!"


(/,) r e s u l t a n t m m f le a d s o r la g s a r m a t u r e m m f,
(r) e x c ita tio n m m f l e a d s o r la g s r e s u l t a n t m m f,

id) a ir-g a p e x c i t a t i o n m m f le a d s o r lag s th e a ir-g a p e m f and


uO a ir-g a p e m f le a d s o r la g s th e a r m a t u r e m m f. ■

itKX laes by ,9° + v ’ (<” lags by (90” t e ’ lcl ,aes by ,90" +si >d) ieads b>' ,9° - si <ei ,aes by
^ *\ h e*effect o f l a c t r i n ^ c u r r o n t ^ 0 *. CUJrren t b a s tb e e B ®c t o f w eakening th e m ain field ; but in a sy n ch ro n ou s
motor, th e e tle c t ol la g g in g c u r r e n t is to s tre n g th e n th e m ain field. E x p la in .

^ th r^ n rh n p "m n v p r^ f^ V ^ 0 .?n.^ n ^ n ^ e b u s, >s su p p lyin g som e pow er a t p f 0 .9 lag. F o r co n sta n t pow er


input from th e p r im e -m o v e r f th e field c u r r e n t is in cre a se d , exp lain w h at h appens to th e load an gle. Will th e
rotor m ove o w a r s e r e s u a n t a ir-g a p m m f o r a w ay from it ? W h a t h ap p en s to th e m ag n itu d e o f a rm a tu re
current an d p o w e r f a c to r ? N e g le c t le a k a g e im p ed an ce.
[H in t, (o ) In F ig . 5 .1 4 , a r m a t u r e m m f F u for laggin g c u rre n t, h a s a com ponent in p h ase opposition to Ff.
For a s y n c h ro n o u s m o to r. F ig . 5 .1 0 show's t h a t a r m a tu r e m m f Fu for a laggin g cu rre n t, h as a com ponent in
phase w ith F^-etc.)
IH in t. (6 ) R e fe r to F ig . 5 .3 1 (6)|
|Ans. (b ) L o a d a n g le d e c r e a s e s a n d th e ro to r m oves to w ard s the re s u lta n t air-g ap mmf. A rm a tu re cu rre n t
increases a n d p o w e r f a c to r b eco m es m o re laggin g.)
5 .1 6 . ( a ) D ra w p h a s o r d ia g ra m o f a ro u n d -ro to r a lte rn a to r on (;) open -circuit and (ii) sh o rt-circu it. D iscuss
why s h o r t-c ir c u it c h a r a c t e r i s t i c is a s t r a i g h t line w h e re a s o p en -circu it c h a ra c te ris tic is a curve.
(b) A 3 0 0 k V A , 4 0 0 V a l te r n a t o r co n n e cte d in d e lta is now recon n ected in s ta r. C a lcu la te tbe new voltage,
cu rrent an d kV A r a tin g s .
lAns. (6) 6 9 2 .8 V, 2 5 0 A. 300 kVAl

5 .1 7 . A n id eal s y n c h ro n o u s m a c h in e , w ith negligible leak ag e im p edan ce, is connected to an infinite bus.


At no load, th e m a c h in e is n e ith e r g e n e r a tin g n or m otoring.
(a) E x p la in how th e m e c h a n ic a l p ow er in p u t T w s to th e synchronous m achin e is delivered a s electrical
power o u tp u t m V , /„ co s 0.
(6) E x p la in how th e a p p lica tio n o f s h a ft load ca u se s a synchronous m achin e to o p erate as a m otor.
Also sh ow t h a t th e r e a c tiv e p o w er flow in a syn ch ron ou s m achin e can be controlled by v aryin g its field
current *
5 .1 8 . ( a ) E x p la i n how th e P o tie r tria n g le ca n be draw n w ith th e help of OCC and an y tw o points on the

(b ) W h a t is load m a g n e tiz a tio n cu rv e of a sy n ch ron ou s m achin e ?


E x p la in th e d iffe re n ce b etw een P o tie r re a c ta n c e and a rm a tu re -le a k a g e re a cta n ce
5 .1 9 . ( a ) D is c u s s th e fa c to rs t h a t g o v ern th e ch an g e in term in al voltage of an isolated syn ch ron ou s g en erato i

when feed ing its ow n load . deliverin g 2 0 MW and 8 MVAr to an infinite bus at 11 kV The
(6) A 3 -p h a s e s ta r -c o n n e c te d a lte r n a to r D eterm in e the load an gle and the e x cita tio n e m f of the
a lte rn ato r h a s s y n c h ro n o u s im p ed an ce ot u m. |Ans (/>) 22 4g<, 14265 3 V(
altern ato r.
5 .2 0 . (a ) D efin e th e v o lta g e re g u la tio n o f an OTeatcr than the no-load term in al voltage ? E xp lain
b p o ssib le to h a v e th e full-load a lte rn a to r i . load ed -
(6 ) W hy d oes th e a r m a t u r e te rm in a l 0 f a lte rn a to rs ?
(c) W h a t is th e n e c e s s ity o f co m p u tin g vnu a c e regulation and hence show th a t synchronous
«f> D evelop th e e m f m eth o d of d ^ j n . n g ^ e voltag ,
reactan ce c o n s is ts o f tw o co m p o n en ts of

Scanned by CamScanner
j a u u e o s i u e o Aq p a u u e o s

690 Electrical M a c h in e ry ____________ _________________________- — —


5 .2 1 . (a) E xp lain the m ethod of d eterm in in g th e voltag e re g u la tio n of a n a l te r n a t o r by new ASA method
(b ) D escribe how sa tu ra te d synchronous re a c ta n c e o f a n a lte r n a to r is ca lc u la te d . H e n ce , exp lain the meth0cj
of determ ining the voltage regulation by u sing this s a tu ra te d re a c ta n c e .
5 .2 2 . (a) C om pare the resu lts obtained for voltage re g u la tio n by em f, m m f, zpf, n ew A SA and saturated

synchronous-reactance m ethods.
(b) A 3 -phase, 17.32 kVA, 4 0 0 V, star-co n n ected a lte rn a to r is d e liv e rin g r a te d load a t 4 0 0 V and at pf o e
lag. Its synchronous im pedance is 0 .2 + J2 Q p er p h ase. Fin d th e load an g le a t w h ich it is operatin g.
Now, with th e m agnitude of excitation v oltage held co n sta n t, th e n a tu r e o f r a te d load p f is altered to pf
0 .8 leading. D eterm ine the new value of term in a l voltag e an d load an gle.
[H in t. (b ) F irs t calcu late E f ( = 2 6 7 .5 7 V) a t 0 .8 p f lag an d th e n find o u t Vt a t p f 0 .8 leading.)
[A n s. (b ) 7 .9 5 ° ; 5 0 2 .4 4 V, 9 .2 5 =]

5 .2 3 . ia) E xp lain why synchronous im pedance m eth od o f co m p u tin g th e v o lta g e re g u la tio n , leads to
pessim istic value a t lagging p.f. loads.
(b) A three-p h ase, 5 0 Hz, 2 0 0 0 kVA, 1 1 kV, sta r-co n n e cte d a lte r n a to r h a s a fu ll-lo ad v oltag e regulation f
1 0 % a t 0 .8 p.f. lag. Now the speed of th e a lte rn a to r p rim e-m o v er d e cre a s e s to giv e a freq u en cy of 48 H
D eterm ine its effect on a lte rn a to r ra tin g and also on th e v oltag e re g u la tio n a t 0 .8 p.f. lag, T h e field and armatu
cu rren ts a re assum ed unchanged. N eglect a rm a tu re re s is ta n c e . e
[(b) H in t. E xcitatio n voltage S in ce I f is co n sta n t, n ew e x c ita tio n v o lta g e E ^ = 0 ,9 6 E and
X si = 0.9 6 X s l. F o r negligible a rm a tu re re sista n ce , F ig . 5 .2 2 (b ) show s t h a t V l2 = 0 .9 6 , Vn = 1 0 .5 6 kV etc]
[A n s. 1 0 .5 6 kV, 1920 kVA, 10%)

( A , 6; i6 . kV; 3 Ph a s e > 5 0 H z > star-co n n ected a lte rn a to r g a v e th e follow ing d a ta fo r open circuit
circu it and full-load zero-pow er facto r tests : circuit, short

If in A 3.2 5 .00 7.5 0 1 0 .0 0 14.00


Ef in k V 3.10 4 .9 0 6 .60 7 .5 0 8.24

Isc in A 500 778 11 7 0 — --


z.p.f. terminal voltage in kV — 1.85 4 .2 4 5 .7 8 7.00
. J...W IU V w * » iim u u j.v * V h JJu Jl/L U iV t iO \J.£* t.
Jk U d i l

0 .8 p.f. lagging by the following m ethods :


(a) e.m .f. m ethod
(b) m.m.f. m ethod
(c) z.p.f. m ethod
(d) new ASA m ethod and

(e) satu rated -syn ch ro n ou s re a c ta n c e m ethod.

5 .2 5 . (a ) An a lte rn a to r h a? u ^ ^ 3 1 '?7 % ’ ^ 1 3 ’52% I (c) 17'3 2 % - (d) 1 7 -18% - « 17-32^


1 a lte rn a to r h as a syn ch ronous re a c ta n rp nforKP <■ ,
regulation w hen w orking a t full-load (i) 0 .8 p.f. la g (t7) u n ity p f a n d r ^ ! r e s i s t a n c e - C a lc u la te its voltage
ib) The m ag n etizatio n curve of a 10 kVA 2 3 0 V e;n u .. ^ P ‘^ ea<*'
Its synchronous reactance is 1.2 f! per phase'and r e k i s t a n ^ n e S f '* a lte m a t° r is « shown in Fig. 5.30.
The terminal voltage of the alternator u i ® g ie-
load a t 0 .8 p.f. lag. C om pute th e ra n g e of field c u r r e ^ a E t m ^ t I *1°™ n ° ‘ l0 a d t 0 1 A tim es the rated
o u t- a d ju s tm e n t w h tc h t h e v o lta g e r e g u la to r should carry

[Hint, fa) From Fig. S .23 (a ), the excitation voltage E with


phasor, can be written as f’ lth ra equal to zero and with Ia as reference

E f = (V, cos 0 )2 + (Vt sin 0+ Ja X s)2

= Vf (cos 0)2 + [sin 0 + ^ 1 _u 2 f m Q ,2


[ { Vt J “ f(0.8) + (0.6 + 0.2)2] = 1.28 V?
Er = 1.13 V. etc.i ,A
(a) 13% ; 1 .9 8 % ; - 1 0 .6 % {b ) 1.76 A to 2.26 Al
Polyphase Synchronous M achines 691

5 ,2 6 . A 11 kV, 5 0 Hz, star-con nected cylindrical rotor altern ator gave the following d ata on open c i r c u it :

0 10 15 20 31 55
IAn A 39 50 '
-
0 5.00 7.60 9.00 11.00 12.80 13.40
Efin hV 12.00

^ excitation ui w was lu circulate a tuU-load cu rren t of 160 A on sh ort-circu it. F o r zero-power


factor test, an excitation of 66 A w as required to circulate 1.25 times the full load cu rren t a t 11 kV. E stim a te
voltage regulation a t full load 0 .8 p.f. lagging, by z.p.f. method. Neglect arm atu re resistan ce.
[Hint. Draw o.c.c. and s.c.c. in th e usual m anner. Plot point A corresponding to excitation of 6 6 A and
voltage of 11 kV. Since point A corresponds to 1.25 tim es the full-load cu rren t, i.e. 1.25 x 1 6 0 = 2 0 0 A, O F
should be equal to th e excitation required to circulate 2 0 0 A on short circuit, refer to Fig. 5 .1 7 (fa). H ere
0f = 25 A. F rom A draw a line A D parallel and equal to O F . Through D draw DC p arallel to air-g ap line.

Drop CB perpendicular to DA. Th en CB = 1.25 x 160 (*„,) and full-load arm atu re m.m.f. = In this problem ,

Xal = 3.90 £1 and full load a rm a tu re m .m .f. is equivalent to 1 8 .0 0 A.] [Ans. 15.45%)

5.27. A 3-ph ase star-co n n ected a ltern ato r is rated 1600 kVA, 1 3 ,5 0 0 V. The a rm a tu re effective re sista n ce
and synchronous re a cta n ce a re 1.5 Cl and 3 0 Q respectively per phase. C alculate th e p ercen tage reg u latio n for
a load of 1280 kW a t pow er factors of (a) 0 .8 leading, (fa) unity and (c) 0 .8 lagging. (I.E.S., 1986)
[Ans. (a) - 11.991% (fa) 3.227% (c) 18.6%)
5.28. (a) E xp lain th e m m f m ethod of determ ining the voltage regulation of an a lte rn a to r.
(fa) A 3 0 kVA, 4 4 0 V, 5 0 H z, 3-ph ase star-con n ected alte rn a to r gave the following te s t d a ta :

Field current
2 4 6 7 8 10 12 14
(A)

Terminal
155 287 395 440 475 530 570 592
voltage (V) I-
S.C. Current
11 22 34 40 46 57 69 80
(A)

Resistance between any two terminals is 0.3 ohm. Find the regulation at full load, 0.8 pf lagging by (i)
synchronous-impedance method and (ii) mmf method. [Ans. (fa) (i) 50.66% ; (ii) 31.496% )
5.29. (a) Describe the procedure for determining the voltage regulation of an alternator by saturated
synchronous-reactance method.
(fa) For an armature leakage reactance of 0.8 £1 in Prob. 5.28 (b), compute the following at rated voltage,
rated load and 0.8 pf lagging :
(i) Unsaturated synchronous reactance
(ii) Saturation factor
(iii) Saturated-synchronous reactance and
(io) the voltage regulation. [Ans. (b) 7.372 £1, 1.246, 6.074 £1, 33.07%)
5.30. (a) Explain the method of determining the voltage regulation of an alternator by zpf and new ASA methods.
(fa) The following data relate to a 14860 kVA, 3-phase, 50 Hz, 40-pole, star-connected hydroelectric generator:

If in Amp 10 15 20.5 25 30 35 1 40 45

Ef in kV 6.45 9.0 11.0 12.2 13.25 14.0 14.5 15.0

S.C. test. lf * 18 A, IIC= 780 A.


Zpf test. If = 52.5 A, = 780 A, Vt = 11 kV. _
Find the voltage regulation at rated armature current of 780 A at 0.8 pf lag by zpf and new ASA methods.
Take armature circuit resistance of 0.2 ohm per phase. lAns. (fa) 33.84%, 34.07%)
5.31. (a) Why is the e.m.f. method so called ?
(6) What are three factors that cause a change of the alternator terminal voltage as it is loaded ?

Scanned by Cam scanner


ijlC
ii;. . id . . I
692 E lectrical M a c h i n e r y _ - -------------------------------- ' ------|

t t Develop th e eq u ivalen t eircu it of a cy lin d rica l-ro to r , \


i-i (d) W h at h appens to th e valu e of sy n ch ro n o u s re a c ta n c e X s if a ir g^p
.'iii
ie) W h at is the excitatio n voltage in c a s e of sy n ch ro n o u s m a ch in e s ?
f if, F o r zero-pow er facto r laggin g load on an a lte rn a to r , a r m a t u r e m .m .f. F . o p p o ses m ain -C eld m .m.f.
•fi but for zero-pow er facto r lead in g load, F „ a ss ists Ff . E x p la in . .
[A n s. (b ) ru, xal and X ,ir (d ) X 5 d e cre a se s b e ca u se a ir-g a p r e lu c ta n c e in c r e a s e s (e) Ef . ts m ag n itu d e depends j
on speed (or frequency), field cu rre n t, n u m b e r of a r m a tu r e tu r n s p e r p a s e e c.

5 .3 2 . (a) I f an a lte rn a to r is not ru n n in g a t sy n ch ro n ou s ®p e ®d ' d ‘^ f re s is ta n c e 'fo r° p a rts ( K f r " '


(ii) sh o rt-circu it cu rre n t and (iii) o p en -circu it c h a r a c te ris tic . N e g le ct a r m a t u r e r e s is ta n c e lo r p a rts (0 and
(,b) In an a lte rn a to r, less field c u rre n t is req u ired to re a c h r a te d v o lta g e for le a d in g p op eration , whereas
m ore field cu rre n t for laggin g p f o peration . E x p la in .
(c) S yn ch ron o u s-im p ed an ce m ethod does n o t giv e a c c u r a te v a lu e o f v o lta g e re g u la tio n o f a n alternator.
D espite th is, w hy it is still used.
[A n s. (a ) (i) No efTect (ii) No effect (iii) O CC sh ifts bodily up or down d e p e n d in g on w h e th e r speed is more
or less th a n synch ron ou s speed. (b ) for lead in g pf, E f is less ...]
5 .3 3 . A 3-p h ase a lte rn a to r is ra te d a t 5 kVA, 1 1 0 V, 2 6 .3 A, 5 0 H z a n d 1 0 0 0 rp m . T h e s ta to r resistance
betw een term in als as m easu red w ith dc is 0 .2 fi. W ith no-load a n d r a te d sp e e d , th e s t a t o r line voltage is
1 6 0 V for a field c u rre n t of 4 A. A t ra te d speed, th e sh o rt-c irc u it s t a t o r c u r r e n t p e r te r m in a l is 6 0 A for a field
cu rre n t of 4 A. C om pute (a ) syn ch ron ou s im p ed an ce p e r p h a se (b ) v o lta g e re g u la tio n a t 0 .8 p f laggin g at rated
load. . . .
wL T h e a lte rn a to r is star-co n n ected .
'J f [H in t. T ak e r„ = 1 .2 5 rt/c) [A n s. (a) 1.54 fl (6) 50.08%]

■pr 5 .3 4 . (a) Show how th e p h a so r d ia g ra m ca n be used to d e m o n s tr a te th e co n d itio n s o f m o to rin g or generating


in a syn chron ou s m achin e.
(b) D raw a lte rn a to r p h aso r d ia g ra m s for lagg in g , u n ity an d le a d in g p o w er fa c to r s on th e assumption of
sa m e (i) term in al voltage and (ii) e x cita tio n v oltag e.
5 .3 5 . (a ) E x p la in how one can d istin g u ish w h e th e r a p h a so r d ia g ra m is fo r a n a l t e r n a t o r o r a synchronous
m otor.
(b) D raw p h aso r d ia g ra m s for cy lin d rica l-ro to r sy n ch ro n o u s m o to r w h en w o rk in g u n d e r lag g in g , unity and
leadin g pow er facto rs on th e assu m p tio n o f sa m e (i) te rm in a l v o lta g e a n d (ii) e x c ita tio n v o lta g e .
5 .3 6 . D raw th e v o lta g e p h a so r d ia g ra m o f a n a l te r n a t o r a n d a s y n c h ro n o u s m o to r, so t h a t th e sam e voltage
& eq u ation, n am ely E f= Vt + Ia (ra + j X s), is ap p licab le to both th e s e m a c h in e s . H e n c e d ra w th e comm only used
voltag e p h a so r d iag ram for a cy lin d rical ro to r sy n ch ro n o u s m o to r.

How can one d istin g u ish w h e th e r th e v o ltag e p h a s o r d ia g ra m is for a n a l t e r n a t o r o r for a m otor ?

* • f ‘3 7 ' T * 6,™ ?1 a n d jin te rn a I c h a r a c te r is tic s o f a n a l te r n a t o r . E x p la i n th e s h a p e of th ese charac­


te ris tic s w ith th e help o f p h a so r d ia g ra m s.

p l a t e d ? E X P lam ^ a ,te rn a t0 rS a re r a te d in k V A ‘ W h a t is thB n e c e s s ity o f m e n tio n in g th e p.fs. a t their name

a’lem at0r " ° S5' E * plain ‘ " a altern ator is designed J.


B^ [A n s, (b) Lagging p l l

input*and ^ T u t ^ t ^ ^ " 1 alternator, derive an expression for its power

ata ™ „ * » n P,1
th e p ow er o u tp u t, a r m a t u r e c u r r e n t an d Df u n d e r t h o r n
1 7 ^ °f 1+J'100
,1 * ? ,
Wfi
a n e x c i t a ti o n v o lta g e of 6 .4 kV. Find

annature * 66^

amount to 10 kw- d“ t h ". i t “ r t r r X r : aP« a ^( G °A aT E.»- 1 9 o i i


[A n s. 1431.816 kWl

Scanned by CamScanner
Polyphase Synchronous M ach in es 693
* 4 0 . A 3 3 0 0 -V , s ta r - c o n n e c t e d s y n ch ro n o u s m o to r w nrkc .
, inn Its s y n c h ro n o u s im p e d a n c e is 1 + j in O nPr t. co n sla n t te rm in a l v o ltag e an d co n sta n t
th e ^ m a in s. F in d f t . p f w h en t h ^ p ^ " " ~ -

5 4 ^"cert a i n ^ a d °theUi n p u ^ is *90^5*kW^at,3^o ° f 3 '^ h a se s t a r 'Connected 6 6 0 0 V sy n ch ro n ou s m o to r is 2 0


For a l l ™ ! ™ ' V° lta g e an d th e in d u <*d e.m .f. is 8 9 4 2 V. D e te rm in e th e
fin*cu rrent a n d P g a r m a t u r e re s is ta n c e . [Am> g? Q55 A 0 g 247 lca d ,

5 ’t 2o d e t e r m i n e 4 t h e ^ c i L t ^ e m ^ ^ 0^ ? ^ 16'1 syn ch ron ou s m o to r h a s a syn ch ron ou s im p ed an ce of


leading a iid w ith a n efFicien cy o f ^ “ * * *• — w orks a t ^ d at 0 , pT

5 .4 3 . A 2 0 0 0 V 3 -p h a s e s ta r-c o n n e c te d sy n ch ro n o u s m otor h a s syn ch ron ou s im p ed an ce of 0 .5 + j 5 i l p er


phase. F o r a n e x c ita tio n v o lta g e o . 3 0 0 0 V , th e m o to r ta k e s an inp u t of 9 0 0 kW a t ra te d voltage. C om p u te the
line cu rre n t an d p.i. [Ans <260 71 A 0 .9 9 6 5 lead!

5 .4 4 . A 1 1 -k V , 3 - p h a s e , m e s h -c o n n e cte d sy n ch ro n o u s m otor h a s synchronous re a c ta n c e of 10 O p er p h ase.


lts friction, w in d a g e a n d iro n lo sses a r e 1 0 0 0 kW .

(а) Th e m o to r e x c ita tio n is so a d ju s te d t h a t it ta k e s u.p.f. cu rre n t a t a sh aft load of 10 M W . C a lc u la te th e


excitation v o lta g e .
(б) Now th e e x c ita tio n e .m .f. is in c re a s e d to 1 2 .5 kV and th e sh aft load is ad ju sted so t h a t m o to r ag a in
operates a t u .p .f. C a lc u la te th e s h a ft load o u tp u t. (A ns. (a) 1 1 4 9 3 .9 5 V (6) 1 8 5 9 2 .7 6 kW|

5 .4 5 . A 3 -p h a s e s ta r -c o n n e c te d a lte r n a to r w ith sy n ch ron ou s im pedance of 0 + j 5 Cl p e r p h a se is co n n ected


to an 11-kV s y s te m . T h e a l te r n a t o r p ow er o u tp u t is found to be 10 M W and re a ctiv e pow er o u tp u t a s 3 M VA.
Compute th e m a g n itu d e o f e x c ita tio n v o lta g e , load an gle, line cu rre n t and power facto r.
[Ans. 13172.54 V, 2 0 .1 8 5 °, 5 4 7 .9 6 A, 0 .9 5 7 8 leadl
5 .4 6 . A 2 3 0 0 - V , 3 -p h a s e , s ta r-c o n n e c te d , 5 0 H z syn ch ron ou s m otor o peratin g a t n o rm a l v o ltag e is e x cite d
to give an e x c ita tio n v o lta g e o f 2 4 0 0 V. D ete rm in e th e m a xim u m pow er developed, a rm a tu re c u rr e n t an d pow er
factor u n d er th is e x c ita tio n . P e r -p h a s e sy n ch ro n o u s im p edan ce is 1 .5 + j 21 Q.
• [A ns. 2 4 2 .7 kW, 8 7 .8 5 3 A. 0 .7 9 2 7 lag!

5 .4 7 . F ro m th e e q u iv a le n t c irc u it of a cy lin d rical ro to r synchronous m otor, derive exp re ssio n s for th e pow er
input and p ow er o u tp u t in te r m s of load an g le , syn ch ron ou s im p edan ce, excitatio n v o ltag e etc.
• * 2
Show t h a t th e d iffe re n ce in p ow er in p u t an d pow er ou tp u t is equal to ohm ic loss I„r„.
5 .4 8 . A 1 0 k V A , 3 8 0 V , 4-p o le, 5 0 H z, sta r-co n n e cte d cylindrical ro to r a lte rn a to r h a s a s t a t o r re s is ta n c e
and sy n ch ro n ou s r e a c ta n c e of 1 ohm an d 15 ohm s resp ectiv ely . It supplies a load of 8 kW a t ra te d v o lta g e and
0.8 power fa c to r la g g in g .
(а) D raw a p h a s o r d ia g ra m of o p eratio n .
(б) E x p re s s th e r e s is ta n c e a n d sy n ch ro n o u s re a c ta n c e in p er u n it v alu es w ith th e m a ch in e r a tin g a s th e
base.
(c) C a lc u la te th e p e r c e n ta g e reg u la tio n .
W) W h at is th e te rm in a l voltage if th e load is suddenly removed (with the speed and excitation unaltered) ?
[A ns. (a) See Fig. 5.23 (a), (b) 0 .0 6 9 3 pu, 1 .0 3 9 p.u. (c) 8 5 .5 3 ^ (d ) 7 0 5 V|

5 .4 9 . A 3 -p h a s e , 4 0 0 V, 6 -p o le , 5 0 H
0.5 +, 8 n p e r p h a se . Its m p u l c u r r e n t ,s 1 0 A a t u m ty p o w j* f a ^ ^ ^ ^ ^ ^
cu rrent re m a in in g c o n s t a n t, th e load to rq u e is 1 [Ans n g gg Nm 0 7 7 6 iagging(
the developed to rq u e a n d th e n ew p ow er la cto r.
• r riovolnned in a cv lin d rical-rotor a lte rn a to r in te rm s of p ow er an gle
5 .5 0 . (a ) D e riv e a n e x p re ss io n for p ow er developed in a cym

and sy n ch ro n o u s im p e d a n c e . ,0 4 , 1 .2 5) p .u .. d eliv ers ro le d c u r r e n t to in fm ile


(b) An a l te r n a t o r , w ith a synchronous im p edan v ^ f fn ctor j u s t before faUinR out of
busbars a t p f 0 .8 la g g in g . F o r th e s a m e e x cita tio n , (A n s. <f>) 1.80 p.u.. 0 .8 9 5 8 leading)
step. , . « r
„ . 4 „„r.n Prted syn ch ron ou s m otor h a s sy n ch ro n o u s im p e d a n ce ot
5 .5 1 . A 3 -p h a s e , 2 2 0 0 V , 2 0 0 k W , s ta r - ou tp u t of 1 0 0 k W , th e m o to r e x c ita tio n is so a d ju ste d
F 2 + j l 2 D p e r p h a s e . I ts rotntionu![ loss is 2 t • e x cita tio n vultage an d th e load angle.
as to give th e o p e r a tin g p f 0 .8 lead in g , a cu c . ^ moloI. s(in ft is in cre a se d to ra te d load.
„ (6) W ith th e e x c ita tio n fixed us found in p a r t to ,^ ^
bar this load , find th e a r m a t u r e c u r r e n t and pow

Scanned by CamScanner
^ —— ■ ■ ——

~~ + 20333.3 . 0 * 1 ’
I H i n t . (o ) 3 ( V „ ci>»8 - 4 ' ’o) - 102I< °r “ 2 7 1 5 .5 3 V, 1 5 .3 1 3 ° (frl 5 8 .7 2 1 A. 0.9584 leading

4 , h s. , 0 0 0 aw. 3 .3 kV, 5 0 H z, 4 - p o l ,
5 .6 2 . A . . o o u y 8-pole s ia r-c o n n e c te d sy n ^ g q ^ re sp e ctiv e ly . Neglect
from . 3-p h ase, 5 0 0 kVA, W ^ « I» ^ th e g e n e r a to > , ‘Z o u s Z rm
c a n d e liv e r A ssum e

S S S k s M S S a ss—
p f conditions a t th e ir te rm in a ls.
*- a-
1AM. 11621.3 \ m;
5

, , - : n a c v lin d ric a l-r o to r sy n c h ro n o u s m otor in terms of


5 .5 3 . to) D erive an exp ressio n for p ow er developed m a cylrn
load angle an d syn ch ronou s im pedan ce. a r m a t u r e c u r r e n t o f 2 0 0 A a t u n ity power fan ,,
(fa) A tu rb o -a lte rn a to r h avin g a r e a c ta n c e of i u w n a im e . m o v e r in p u t is n o t ch an g ed but the emf
when n a m in g on 1 1 0 0 0 V, co n sta n t freq u en cy ^ " 8d ‘ f “ ' rPfa c l0 r. ( I A * . J99,.
is raised by 20% . find th e valu e of a rm a tu re c u rr e n t a p [A ng 2 4 3 .3 A, 0.822 lag'

, „ „ a . fflnce o f 1 0 f l p e r p h a s e a n d negligible armature


5 .5 4 . A 3-p h ase, tu rb o -a lte rn a to r w ith 8 . ^ e c o w U n t-f r e q u e n c y b u s -b a rs a n d su p p lie s 1 0 0 A at unity pf
re sistan ce is con n ected to 11 kV co n sta n t v o lta g e con ^ c ita tio n 0 f th e a l t e r n a t o r is in creased by 25%.
to th e system . If th e tu rb in e pow er is k ep t c o n s ta n t an d tn e ex c
w h at would be th e new cu rre n t an d p f ! (7.A.S., 1994,

Make appropriate assumptions. [AnS- ig o .6 5 A, 0.5245 lag!

, •. . 1 1 lfV 1 6 0 0 kVA. h a s negligible a rm a tu re resistance and a

5 .5 5 . A 3-phase delivering full-load cu rre n t a t a ce rta in pf, th e voltage regulation of


synchronous reactan ce of 3 0 fi per phaw . W h « g delivered by th e m achin e. (/ A S.. 199V,
the m achine is zero. E stim ate the pf of the load an po (A n a. 0 .9 8 0 1 leading, 1568.2422 kW|

5 .5 6 . F o r a sy n ch ro n o u s m a c h in e , e x c ita tio n e m f = 2 .3 p .u ., t e r m in a l v o lta g e = 1 .0 0 p.u., r„ = 0.


X = 1 5 pu P lot th e v ariatio n s of a ctiv e and re a c tiv e p ow ers. F in d th e m a x im u m v a lu e s of a ctiv e and reactive
powers and the corresp on d in g load an gles. In th e re a c tiv e -p o w e r d ia g r a m , in d ic a te th e r e a c tiv e power flow to
or from the syn chron ou s m ach in e. A t w h at load an g le is th e p ow er f a c to r u n ity .
[Ans. P max = 1.533 a t 8 = 9 0° ; (max) = 0 .8 6 6 a t 8 = 0°. Load angle = 64.23°!

5 .5 7 . (a) Develop th e e x cita tio n circles for a cy lin d rica l ro to r s y n ch ro n o u s m o to r. H ow a re these circles
helpful in stu d yin g th e ste a d y s ta le b eh av io u r of sy n ch ro n o u s m o to rs ?
(fa) E x p la in th e d evelopm ent of pow er circle s for a cy lin d rica l ro to r s y n c h ro n o u s m o to r.
Show th a t :

) (i) zero-pow er circle p a sse s throu g h origin,

(**) Pm ax ~ a
V?
- a n d
.
a
(iii) efficiency a t m axim u m pow er o u tp u t = 50% .
5 .5 8 . E x p la in how th e excita tio n an d p ow er circle s ca n be su p e rim p o s e d o b ta in V -c u rv e s o f a cylindrical
ro to r sy n ch ro n ou s m otor.
H ence show t h a t :
(i) m in im u m an d m axim u m c u r r e n ts far a n y p ow er o c c u r a t u .p .f.
(ii) m in im u m p.f. for a n y load pow er o ccu rs w h en th e c u r r e n t lin e is t a n g e n t to th e p o w er circle for that
load.
5 .5 9 . W ith re g a rd to sy n ch ro n o u s m o to r V -cu rv e s, e x p la in th e follow ing:
(a) T h e re is a bend in th e com p oun d in g cu rv e o b tain ed by jo in in g the m in im u m a n d m a x im u m excitation
points.
(fa) U n ity p.f. com p oun d in g cu rv e h a s also a bend in it.
5 .6 0 . A 4 3 3 - V, 3 -p h a s e , s ta r-c o n n e c te d sy n ch ro n o u s m o to r h a s a sy n c h ro n o u s r e a c ta n c e o f 5 fJ per phase
F o r a p ow er o u tp u t o f 15 k W , find its m in im u m a r m a t u r e c u r r e n t , e x c ita tio n v o lta g e und ’ he power angle
A rm a tu re re s is ta n c e is negligible.

[H in t First calculate Ia, then Ef and 5 by using its voltage phasor diagram). [Ans. 20 A 4 6 6 4 V, 21 83s!

Scanned by CamScanner
Polyphase S y n ch ro n o u s M a ch in e s 695

5 .6 1 . E ^ lain/ 7 ^ v ^ C fv\v,°nSv,d e r a tl0 n ^ hoW a s y " 0111,000115 m o to r ca n be m a d e to o p e ra te a t le a d in g


or lagfinS P ° w e r faCt° TS- V e n f y th e ab0Ve co n c^u s l° n s w ith su ita b le p h aso r d ia g ra m s .
5 .6 2 . (o) W h a t do you u n d e r s ta n d by th e te r m s , n o rm a l e x cita tio n , u n d e r-e x c ita tio n an d o v e re x c ita tio n in
onnection w ith s y n c h ro n o u s m o to r o p e ra tio n ?

(f>) A sy n ch ro n o u s m o to r is ru n n in g a t a ce rta in load. W h en its field c u r r e n t is in c re a s e d , its a r m a t u r e


current d e cre a s e s. B e fo re th is c h a n g e , e x p la in w h e th e r th e m otor w as d eliv erin g or a b so rb in g th e la g g in g
reactive kVA.
(c) R ep eat p a r t (b ), in c a s e a n in c re a s e in field c u rr e n t ca u s e s a corresp on d in g in c re a s e in a r m a t u r e c u r r e n t.
[A ns. (b) O p e ra tin g a t la g g in g p.f. i.e. m o to r is ab so rb in g laggin g re a c tiv e kVA from th e b u s. (c) D e liv e rin g
lagging re a c tiv e k V A to th e bus.]

5 .6 3 . It is se e n from th e V -c u rv e s of a sy n ch ro n o u s m otor th a t for one v a lu e of a r m a t u r e c u r r e n t , th e r e


are two d ifferen t v a lu e s o f field c u r r e n t. -

(a) Which of th e s e tw o v a lu e s of field c u r r e n t w ould give g re a te r m a rg in of s ta b ility ?


(b) O ut of th e s e tw o field c u r r e n t s , w h ich w ould give g r e a te r efficiency.
(c) In p ra c tic e w h ich of th e s e tw o field c u rr e n ts w ould be u sed for sy n ch ro n o u s m o to r o p e ra tio n ?
[A n s. (o) an d (c) L a r g e r v a lu e o f field c u rr e n t. (6 ) L ow er v alu e of field c u r r e n t (b e c a u se it g iv e s lo w e r
field-circuit losses.)]

5 .6 4 . (n) A s y n ch ro n o u s m o to r is o p e ra tin g a t 0 .2 p.f. la g a t half-full load. W ith E f, V, a n d * , r e m a in in g


constant, exp lain w h e th e r its p ow er fa cto r is w o rsen ed or im proved w hen it is m ad e to o p e r a te a t full load .
(6) A sy n ch ro n o u s m o to r is o p e ra tin g a t a c e r ta in load. E x p la in how you w ill find o u t in th e la b o r a to ry
whether it is o p e ra tin g a t a le a d in g o r lag g in g pow er facto r.
(c) Th e field c u r r e n t of a sy n ch ro n o u s m o to r is ad ju sted to o p erate it a t u n ity p.f. a t no load . I f th e load is
increased, k eep in g Ef, V, c o n s ta n t, will th e sy n ch ro n ou s m otor be d eliverin g or ab so rb in g r e a c tiv e p ow er ?

|Ans. (a ) p.f. im p ro v e s (b ) D e cre a s e field c u r r e n t ; if l u d e cre a se s p.f. is le a d in g an d if l u in c r e a s e s p.f. is


lagging (c) A b so rb in g la g g in g re a c tiv e kV A o r ab so rb in g re a ctiv e power.]
5 .6 5 . (a ) In a d.c. s h u n t m o to r, c o u n te r e.m .f. (or g e n e ra te d e.m .f. E(l) m u st alw a y s be le ss th a n th e te r m in a l
voltage V,. It is p o ssib le in a sy n ch ro n o u s m o to r to h a v e co u n te r e m f or e x cita tio n e m f E f g r e a te r th a n te r m in a l
voltage V, ? E x p la in .

(b) A sy n ch ro n o u s m a c h in e is ru n n in g u n d e r ste a d y -s ta te conditions a t ra te d v o lta g e a n d r a t e d fre q u e n cy ,


At an e x cita tio n v o lta g e o f 1 .5 p .u ., th e ra te d c u rr e n t flows a t 0 .8 p.f. lead in g. Is th e sy n ch ro n o u s m a c h in e
operating as n m o to r o r a g e n e r a to r ? E x p la in .
(c) A v a ria b le r a tio th r e e p h a se tra n s fo rm e r is in terp o sed b etw een a sy n ch ro n o u s m o to r a n d th e b u s b a rs .
If the sy n ch ro n o u s m o to r te rm in a l v o ltag e is red u ced w ith th e help of v a ria b le -ra tio tr a n s f o r m e r , th e a r m a t u r e
current in c re a s e s . B e fo re th is ch a n g e , w as th e sy n ch ro n ou s m otor w ork in g a t a le a d in g o r a la g g in g p.f. ?
Explain. (A ns. (a) Possible (6) Synchronous m otor (c) L ead in g pow er factor.]

5 .6 6 . (a ) C o m p a re th e p e rfo rm a n ce of a sy n ch ro n ou s g e n e ra to r co n n ected to a n in fin ite b u s w ith t h a t of


an isolated a l te r n a t o r o p e r a tin g on its own load.
(b) A n a l te r n a t o r , co n n e cte d to in fin ite b u s, is o p e ra tin g a t u n ity p ow er fa c to r a t h a lf-fu ll lo a d . W ith field
current re m a in in g c o n s ta n t, s t e a m in p u t is in c re a s e d till a lte rn a to r begins to o p e ra te a t full lo ad . U n d e r th is
condition, d iscu ss w h a t h a p p e n s to p ow er fa c to r an d re a c tiv e pow er flow.
(c) An a l te r n a t o r is sy n ch ro n iz e d to an in fin ite b u s th ro u g h a v a ria b le -ra tio t r a n s f o r m e r . I f th e a l te r n a t o r
term inal v o lta g e is in c r e a s e d w ith th e h elp of v a ria b le -ra tio tra n s fo rm e r, th e a r m a t u r e c u r r e n t in c r e a s e s . B e fo re
this ch an g e, w a s th e a l t e r n a t o r w o rk in g a t a le a d in g or lag g in g p ow er fa c to r ? E x p la in .
[A n s. (6 ) P o w e r fa c to r b eco m es le a d in g an d a lte r n a to r ab sorb s re a c tiv e p ow er (c) L e a d in g pf]
5 .6 7 . (a ) E x p la in w h y cy lin d rica l ro to r th e o ry c a n ’t be ap p lied to a sa lie n t-p o le sy n ch ro n o u s m a c h in e .

(b) Show' t h a t in a n a l te r n a t o r ,
(i) if a r m a t u r e c u r r e n t la g s E f by 9 0 e, th e a r m a tu r e re a c tio n m m f is co m p le te ly d e m a g n e tiz in g an d

(‘0 if a r m a t u r e c u r r e n t le a d s E y b y 9 0 c, th e a r m a tu r e re a c tio n m m f is co m p le te ly m a g n e tiz in g .

(c) Show t h a t w h en a r m a t u r e c u r r e n t is in p h a se w ith E f , th e effect o f a r m a t u r e m m f is to d is to r t th e


fidd-flux w a v e , w ith o u t a p p re c ia b ly m o difyin g its s tre n g th .

I V* K_#'
_____________________ IProh.S
696 E lectrical M ach inery
— tn sa lie n t-p o le sy n ch ro n o u s m a ch in e s and draw its
5.68. (a) Explain the two-reaction theory as applied to salient p yn
p h a so r d iag ram for a lagg in g p.f. loa . th a n d tw o . r e a ctio n th e o ry .
(b) E x p la in the difference b etw een cy c e n e ra to r v o lta g e eq u atio n for a salient-pole
(c) How can m otor v o ltag e a t la g g in g p.f.
synchronous machine ? Draw the voltage p .- lia b le resistance, has the following per unit
5.69. (a) A salient-pole synchronous generator with negligible resistan f
parameters :
X^O.8

„ g e n e ra to r U f l y i n g r a ^ V A a . r a te d v o lta g e an d a t 0 .8 p.f. la g g in g , co n tp n te th e load a„gle

th eo ry is applied to
one re a c ta n c e equal to X ,. C om p u te th e load an gle an d e x c .ta t.o n v o ltag e by u g y

" T H“ r ,g . s ,l « . S, is th e e x c ita tio n v o lta g e w .th cy lin d ric a l r o t o r th e o r y su ch th a ,

F - V + il X, an d th e angle betw een p h aso r En an d V, is th e load an g le w ith th is th e o ry .


A- d * .. . x P are almost equal but the load angles with the two theories
I t w i l l be seen that the magnitudes o / /i £ . and power over the normal operating range
are considerably different from each other. 1*1 view of this, Vt Ia, E and power
can be com pu ted by sim ple cy lin d rical-ro to r th e o iy w ith su fficien t .accuracy^ ^ ^ ^ ^ ^ ^

5 .7 0 (a) F o r a cy lin d rical-ro to r a lte rn a to r w ork in g a t lag g in g p.f. show t h a t


In (Xs cos G- \ sin 9)
tan 8 =-
: V, + /„ (Xs sin 9 + r„ cos 0)

(b ) F o r a salien t-p ole synch ron ou s m oto r, w orking a t lag g in g p .f., show t h a t
Ia (X0 cos d - r a sin 0)__
ta n S = v ; - / a ( X ; s i n 0 + ra cos 0)

(c) F in d an exp ression for pow er in te rm s o f load an gle 5, for a sa lie n t-p o le sy n ch ro n o u s m o to r w orking at
a lagg in g p.f. A rm a tu re re s is ta n c e m ay be n eglected .
5 7 1 A 3 -p h a s e , 3 0 k W , 4 3 3 v olts, 5 0 H z, sta r-co n n e cte d sa lie n t-p o le s y n ch ro n o u s m o to r is o peratin g at
0 8 d f. leadin g an d tak in g 4 0 A from th e m ain s. If th e d ire ct an d q u a d ra tu re a x e s r e a c ta n c e s of th e machine
a re 5 Q an d 3 Q re sp ectiv ely , ca lcu la te th e m a x im u m p ow er th e m o to r c a n d evelop i f its e x c ita tio n is m am tain e
co n sta n t. ’ ; __,
[H in t. Ef = 4 0 0 .2 8 4 V, hmax = 70.875°]. [A*18- 64 4673 W
5 7 2 (a ) A salien t-p o le sy n ch ro n ou s m o to r is co n n ected to in fin ite b u s. I f its field c u r r e n t is reduced to
zero, w ill'it sto p o r co n tin u e ru n n in g ? I f th e l a tt e r , w h a t w ill be its sp e e d ? A t w h a t lo a d a n g le w ill it fall out
of ste p w ith zero field c u r r e n t ?
(6) A sa lie n t-p o le sy n ch ro n o u s m o to r w ith d a m p e r b a rs is co n n e cte d to a n in fin ite b u s sy ste m . Its field
c u rr e n t is red u ced to zero an d th e load on th e sy n ch ro n o u s m o to r is g ra d u a lly in c re a s e d . I t h a s been found m
p ra ctice t h a t a f te r th e m o to r h a s fallen o u t of ste p , it co n tin u e s ru n n in g a t s u b -sy n ch ro n o u s speed. Explain
how i t h ap p en s. W h a t w ill h ap p en to th e m a g n itu d e of a r m a tu r e c u r r e n t a n d its p.f. ?
(A n s. (a ) S y n ch ro n o u s sp eed , 8 = 4 5 ° (6 ) A fte r fallin g o u t o f s te p , it w o rk s a s a n in d u ctio n m otor at
su b -sy n ch ro n o u s sp eed . T h e m o to r d ra w s la rg e c u rr e n t a t a poor p.f.]
5 .7 3 . A sa lie n t-p o le sy n ch ro n o u s g e n e r a to r on load d ra w s a n a r m a t u r e c u r r e n t a t a la g g in g pow er-factor
an gle 0. F in d o u t th e an g le by w hich
(a) a ir-g a p e x c ita tio n m m f lag s or le a d s th e a r m a tu r e m m f F (l,

(b ) g -a x is co m p o n en t of a r m a tu r e m m f lag s or le a d s F a ,

(c) d -a x is co m p o n en t o f a r m a t u r e m m f lag s or le a d s F (l an d

(d ) a ir-g a p e x c ita tio n m m f lag s o r lead s a r m a t u r e te rm in a l v o lta g e .


(A n s. (a) leads by (90 + 0 + 8) (b ) leads by (8 + 0) (c) lags by 1(90 - (0 + 5)] (d) lends by (90 +

Scanned by CamScanner
______________ _____________________ Polyphase Synchronous Machines 697
— —'
(n) D raw th e p h a so r d ia g ra m of a salien t-p ole syn ch ron ou s g en erato r show ing also th e field com*
W rite w h eth er th e s e salien t-p o le sy n chron ou s g e n e ra to rs a rc low-speed or high-speed syn ch ron ou s
ponei»ts-_
p hines*
A salient-pole sy n ch ro n o u s g e n e ra to r h as th e following per u n it p a ra m e te rs :
(t>> S X ,,= 1.00, Xq = 0 .6 0 , r (l = 0.02

. . .,ener a to r is d eliv erin g ra te d kVA a t ra te d voltage and a t 0 .8 pf leading, com p u te th e pow er angle
h excitation em f. D raw th e p h aso r d ia g ra m also. |Ans. <8 ) 36 .8 7 °, 0.821

nl* 5 75 Draw th e follow ing p h a so r d ia g ra m s for salient-pole synchronous m achin es clearly m en tion in g th e
arameters involved .
p ‘ A ltern ato r o p e ra tin g a t lead in g p.f. w ith p ow er-factor angle 0 less th a n pow er angle 6 .

I,b) A ltern ato r o p e ra tin g a t lead in g pf w ith 0 > 6.


(c) Synchronous m o to r o p e ra tin g a t laggin g pf w ith 0 < 8.
(d) S ynchronous m o to r o p e ra tin g a t laggin g p f w ith 0 > 8 .
_ A 6 -pole 5 0 H z sy n ch ro n o u s m otor is supplied from a 4 0 0 0 V , 3-p h ase m ain s. T h e m otor r ®a ^t a n ^ s
' ‘^q o t l X = 7 0 0 ft. L o sse s a re negligible. Th e excitation of th e loaded m otor is so ad ju sted t h a t th e
g e n erated v o lta g e p er p h ase is 2 0 0 0 V a t a pow er angle of 2 0 °. C a lcu la te th e developed to rq u e , th e
T c u r r e n t and th e pf (in d ica tin g lag or lead). D raw th e com plete p h asor d ia g ra m , show ing 1,, an d w ith
!hpir m agnitudes. T h e re a r e th r e e su g gestio n s reg ard in g th e valu e of th e power angle a t w hich m a x im u m pow er
% . A o p e d : (.) 2 0 - < 8 < 9 0 * (ii) 5 = exacU y 9 0 - (iii. . > lag. <Z

5 7 7 A salien t-p o le sy n ch ro n o u s m otor w ith r„ = 0 . X d = 1 .0 p.u. and X„ = 0 .5 p.u. is o p e ra te d on infin ite


bus-bar of 1 0 0 p.u. v o lta g e . Show t h a t for 1 .0 0 synchronous pow er, th e excitation v oltage is
Ef = cosec 8 - cos 8

Also d erive th e con d ition for load an gle w hen syn ch ron ou s power is m axim u m .

c Ef Xi
lAns. cos 8 = - 4V< {X>{ _ X j ± y 2 ± ^4V( {X(i
£r x « —T

5 .7 8 . F o r a 3 -p h a s e sa lien t-p o le sy n ch ro n ou s m ach in e o p eratin g as re lu cta n ce g e n e ra to r u n d e r m a x im u m


power conditions, d e riv e th e follow ing rela tio n .
X,, + X ,
tan 0 = -r? y-

where 0 = pf angle of relu ctan ce generator.


5 .7 9 . A 3 0 kV A , 5 0 H z, 4 pole a lte r n a to r c o n n o t e d to infinite b u s a t ra te d v o lta g e h a s X „ - 1 .0 0 p.u . an d

X = 0 .6 p.u. .
. r- J .U , 1 in tn rm <5 of ra te d pow er t h a t th e m a ch in e ca n d e liv e r w ith o u t loss
(a) F o r zero e x c ita tio n , find th e o u tp u t in te rm s 0 1 r a te a powei
of synchronism .
(b ) U n d er th e co n d itio n s of ( a ), e s tim a te th e c u rr e n t and pf.

(c) Fin d th e m a x im u m a v a ila b le to rq u e in Nm .

(H in t, (c) T o rq u e in N m = ~ (p u. p ow er) (H atin g in VA)]

lA ns. (a) 0 .3 3 3 p.u. (8) 1.374 p.u.. 0 .2 4 2 4 lag (c) 6 3 .6 6 2 Nml

. • j a n in fin ite bus sy ste m is su p p lyin g its r a te d p ow er. W h a t


5 .8 0 . (a ) A tu r b o -g e n e r a to r sy n ch ro n iz e d w ith an i n f i n i t e M ay ?
happens to the a ltern ato r if its field c irc u it g els open iiv o f I f its field c u r r e n t ia re d u ce d
, • j infin ite bus-bar o p e ra te s a t u n ity p.f. I f its tield c u r r e n t is r e a u c e a
(/>) A tu r b o -g e n e r a to r sy n ch ro n iz e d w ith infin rpnrtiv e D o w e r7 •
to 80% of its p re v io u s v a lu e will it be ab so rb in g or d eliv erin g re a c tiv e p ow er
ui u s p re v io u s v a iu e , w u „ „ r n a tnr sunDhes no a ctiv e p ow er to th e b u s b a r s , co n se q u e n tly
IAns. (a ) W ith £y=0, P = 0, th e re fo re t e a specd is defined as the sp e e d w h ich a p rim e -m o v e r
he a lte rn a to r sp e e d s up to ru n -a w a y sp eed . i1 . rpdlicect to zero). (8 ) T h e a l te r n a t o r a b so rb s re a c tiv e
w°uld a tta in if full load on th e p rim e -m o v e r w ere sudd y
power.)

Scanned by CamScanner
L i a u Kai ififlvuiiivij ________ .- ____________ _ ____ —— ■—^

5 .8 1 . A 3 -p h a s e salien t-p o le sy n ch ro n o u s m o to r, w ith X < - 1 .0 0 p u . A , = 0 .8 p u , is c o n n e c te d to an in„ „ ,u


bus o f ra te d v o ltag e 1 0 pu. C a lc u la te th e p e rc e n ta g e o f rn te d o u tp u t t h a t th is s y n c h r o n o u s m o to r can deliv,,
“ l l g s A r o n L w „ h zero Held c u r r e n t. C a lc u la te t h e p .tr .
conditions. ’ laSi
5 .8 2 . (a ) D erive a n exp ressio n for th e re a c tiv e p ow er o u tp u t from th e t e r m i n a l s of a cy 1indrica|-rotor
a lte rn a to r.
(6) A 3 -p h a s e , 2 0 MVA, 11 kV, sta r-c o n n e c te d a l te r n a t o r h a s Z , = l + . / 8 f J p e r p h a s e . D eterm in e the
m axim u m re a ctiv e pow er th a t can be d eliv ered by th is a l t e r n a t o r for a n e x c i t a ti o n v o lta g e o f 14 kV.
(c) R ep eat p a rts (a ) an d (b) in c a s e a r m a tu r e -c ir c u it r e s is ta n c e o f th is a l t e r n a t o r is n e g le c te d .
(A n s. (b) 4 .2 0 9 1 M VAr (c) 4 .125 MVAr|

. 5 .8 3 . A cy lin d rica l-ro to r sy n ch ro n o u s m o to r is ru n n in g w ith c o n s ta n t field c u r r e n t a t a la g g in g pf. Armature


circu it re s is ta n c e is negligible. D iscu ss, by m e a n s of p h a s o r d ia g ra m s , t h a t h a p p e n s to th e m a g n itu d e s of load
an gle, a rm a tu re c u rr e n t and pow er fa c to r if th e follow ing c h a n g e s in its o p e r a tin g co n d itio n s a r e m ade.
(а) F re q u e n cy red u ced by 10% , both ap p lied v o lta g e an d load p o w e r c o n s ta n t.
(б) B o th applied v oltag e an d freq u en cy red u ce d by 10% , load to rq u e c o n s ta n t.
E V
[H in t : U se th e relatio n P = —{rr1 sin 8 an d th e v o lta g e p h a so r d ia g ra m .)

- [A ns. (a) 8 rem ain s co n stan t, Ia is in creased an d p f becom es more lagging

(6) All the th ree 8, l u and pf rem ain unaltered |

5 .8 4 . A sy n ch ro n ou s m o to r ru n s a t a load an gle of 2 0 ° a t r a te d v o lta g e a n d a t r a t e d fre q u e n cy . A rm ature


circu it re s is ta n c e is neglected . F o r co n s ta n t field c u r r e n t, co m p u te th e v a lu e o f load a n g le w ith th e following
ch an g es in its o p eratin g conditions.
(а ) F req u en cy in creased by 10% , load pow er an d ap p lied v o lta g e c o n s ta n t.
(б) F req u en cy red u ced by 10% , load to rq u e an d ap p lied v o lta g e c o n s ta n t.
(c) B oth applied v oltag e an d frequ ency red u ced by 10% , load p o w e r c o n s ta n t.
W) B oth applied voltag e an d freq u en cy red u ced by 10%>, load to rq u e c o n s ta n t,

[A n s. (a) 2 0 ° (8) 1 7 .9 3 ° (c) 2 2 .3 3 5 ° (d ) 20’]

° raW ^ P r aSOr diagTam ° f a s a lie n t-P ° le sy n ch ro n o u s m o to r w o rk in g a t le a d in g p f an d obtain


th erefrom an e x p ress,o n for pow er in te rm s of load an g le 5. N e g le ct a r m a t u r e r e s is ta n c e .
(6) A salien t-p ole sy n ch ro n ou s m o to r h a s th e follow ing p e r u n it c o n s ta n ts :
X d = 1.25, X q = 1.00.

■ p.u . m 0 t° r takCS r a t e d C U m m t (le a d i"e > “ V f * . d eliverin g 0.5


[H in t. V, sin 8 = X q Iq,

'■ l q = sin 8- Also V, Iu cos 8 = 0 .5


cos 0 = 0.5 or 0 = 60°.
Now Ja cos (0 + 8) == sin 8 = cos (9 0 —8)
60 + 8 = 9 0 - 8 or 8 = 15°
/ rf = / a sin (60 + 15) = ...)

s y n ^ l r mDn & Chr0" U i" e -P ° W er “ ™ , . U d w ith s t e a d .v s t n t e , , a b i l i , ! ^ h

are disturbed. * Chr0mz,nfe |,owtr' Show ,h al “ “ ™ s into piny only when steady-state opernting conditions
5.87. fa) Give the physical concepts of synchronizing power

rotor synchronous t S X Z flg ' lT ' ° 'CClriC" ' r“d“ n ° f >h“ “ >* * « « « for m-phase cylindrical
\/j
(0 m — COS u on no load and /;.■) !h X i E
1,1 > m2 ~ C08 S C09 a i on load.

Scanned by CamScanner
Polyphase Synchronous Machines f>9*)
---------------------
Here £/■, anc* a re pCr p*ia s e v a tues opexcitation voltage, term inal voltage and synchronous im pedance
f-
pectively. 6 is th e load -an g le and a , = t a n ' 1 — ■
re
(Hint- (8) A t no load 8 = 0 and E f = V,. Also sin a , = 0.1

5 88. A 5 MVA, 11 kV , 5 0 H z, 4 pole, star-con n ected synchronous generator with synchronous re a cta n ce
‘ u con nected to an infinite bus. Find synchronizing power and the corresponding torque per unit of
mechanical angle d isp lacem en t
(а) at no load and
(б) at full load of 0 .8 p.f. lag. [Ans. (a) 249 347 kW, 1587.4 Nm ; (b) 354 073 kW, 2254 1 Nm.

<t 8 9 . On n0 lo a d ’ em f a 2 ,2 star-con n ected, 3-phase, 4-pole, synchronous m otor m ay be considered


1 to and in p h ase w ith th e term in al voltage. If, on the application of load, the em f induced is retard ed y
^ c c h a n i c a l d egree, ca lcu la te th c arm q tu re cu rren t and power developed by the m otor, takin g the 5yn cj)ro!V ^ 5J
reactance to be 3 ft p er p h ase an d the s ta to r resistan ce to be negligible. ^ ^ ^ ' g ^ ' 13 W |

5 9 0 . A 15 M VA, 11 kV, 5 0 H z, 3-p h ase, 4-pole, star-connected cylindrical-rotor synchronous g en erato r


ni:'pc r a ted o u tp u t a t pf 0 .9 lagging to an infinite busbar. Its synchronous reactan ce is 0 .3 5 pu. eterm i t
the synchronizing to rq u e for a sh aft displacem ent of 0.4° (m echanical). Neglect losses and s a tu r a l’™ 0 2 _ ^

5 .9 1 . (a ) W h en will you call any m achine system to be operating under stable conditions 0
(8) Define th e te rm s stab ility and stab ility limit.
(c) Explain stead y-state sta b ility limit. How can it be improved for synchronous m achines ?

5 .9 2 . (a ) E x p la in th e tra n s ie n t stab ility limit. W h at is the object of studying the tra n sie n t stab ility limi ^
of synchronous m ach in es ?
(6) E xp lain how ste a d y -s ta te stab ility lim it can be depicted on the power circle diagrams^
5 .9 3 . (4 ) D escrib e, w ith physical concepts, the hunting phenomenon in synch.ronous m a c h in e .
E xp lain w h y h u n tin g is objectionable. W h at a re the ca n o n s causes of h n n .m g ? How can . t be redu
(8) E x p la in th e action of d am p er bars in dam ping out the ro tor-oscllation s.
(c) W h at is th e effect of d am p er b a rs, under stead y-state operating conditions

5 .9 4 ^ (a ) W h a t inform ation do the ^ J ^ j ^ g x p l a i n ^ h y t h e ^ a ^ n t-im le m achine is able to


they look like for th e tw o ty pes o f synch o n .u s m achm es J9 S 3 ,
rem ain in s y n c h r o n y on low load even though h er for m axlm u m
(8) E xp lain th e te rm 'synch ron ous im pedance and how it attects m e ^ ^
power delivered w hen th e m achin e acts as a mo o r a lte rn a to r and in d u ctio n -start
(c) S ta te and exp lain th e difference betw een the dam per wino g ^
synchronous m otor. m s th e efTect of v aryin g
W) In th e ca se o f a syn ch ronou s m otor, describe w ith m e n p p

(i) th e e x cita tio n a t co n sta n t m echan ical load, i972>


(li) the m e ch a n ica l load a t co n stan t value of - iom ^ ^ and ^

5 , 5 . Desc r i be how slip , s can b — ^ ^ durfn6 , he „ , p le s l ,


(8) E x p la in w hy th e p oin ters of am m ^ ^ ^ ^

(c) W hy should th e slip be k ep t as sm a < jQad w jt^ term inal voltage V', = 1. Now th e e x cita tio n
(d) A s a lie n t pole syn ch ron ou s m otor is w or Y
; P , mnture c u r r e n t s = 0.5 p.u. Thc ratio y -o b ta in e d from slip
., • - n f i o u . and a rm a tu re c u r r tiu i„ i \
of the m otor ie ad ju sted to give E f - U.b p.u. </
1 r v nnd A' N eglect all losses.
t e s t i s 1.6. D e t e r m i n e th e v alu es of A, / a <i diag ram for 5 = 0 (at no load> and w ith
M i n t , (d ) F i r s , d r . * U » nalient-polo synchronous m otor phn. ^ ^ ^ „ p # . 0 * pu,

* 01 .. „ M i, , nd o u . d r .t u r . a xU synrhrnnous r e .c t .n c e s . How a m th e se
5 .9 0 . E x p la in d o u rly th e te rm s d .ri
determ ined in th e la b o ra to ry ■

\ \ '.

Scanned by Cam Scanner


1

_____________________ JFrolt. s
700 Electrical M achinery -

The re su lts of slip te s t on a slar-co n n cctcd a lte rn a to r a r e given below


V ,„„v=100V , ViniH = 9 6 V
f|mrv= 1 ° A . Alim = 7 A.

All are line values. N eglecting re sista n ce , ca lcu la te X (, and X „ in o h m s. |Ans. 8 .2 4 8 n , 5.543 ft|

5 .9 7 . (a) E xp lain m axim u m -lagging c u rr e n t te s t for d e te rm in in g , - a x i s sy n ch ro n o u s r e a c ta n c e Shew

th a t X is given by j in S!. H ere 1i„s th e m axim u m stab le a rm a tu r e c u rr e n t p e r p h a se .


‘(I
(b) E x p la in th e d e term in a tio n o { X d a n d X q from re lu cta n c e -m o to r te s t.

5 .9 8 . A 3-p h ase star-co n n ected ^ tu re S u l

m ain tain ed a t th e earlier-v alu e, w h at would be th e pow er fa cto r a t w hich th e m a c h in e o p e ra te s ^ e g l e c t a l l


in tern al losses in the m otor. ' ' ’’
06
IH in t. Load = 2Q0 = ° 6 pU' V' = 1-° P M inim um a rm a tu re c u rre n t = f x T = ° ^ PU‘ *n creased arm a'
tu re cu rren t = 0 .9 pu etc. See E xam p le 5.7.1 |Ans- 0 3 8 7 6 leadingl
5 .9 9 . (a ) E n u m e ra te th e various losses in synchronous m ach in es. E x p la in how th e s e lo sse s can be deter­
mined.
(b) E xp erim en ts conducted on a se t of dc m otor and a 10 kVA 3 -p h a s e a lte r n a to r g a v e th e following data
for the pow er-input to d.c. m otor a rm a tu re :
(i) d.c. m otor ru n n in g alone : 2 4 0 w atts
(ii) d.c. m otor a lte rn a to r coupled to g eth er w ith la te r u n e x cite d : 3 6 0 w a tts
(Hi) S am e as in (ii) but w ith a lte rn a to r excited : 4 6 0 w a tts
(to) A ltern ato r s.c. te st a t ra te d c u r r e n t : 6 0 0 w a tts.
C alcu late the a lte rn a to r full-load efficiency a t 0 .8 p.f. lead in g w ith its field c u r r e n t an d field term inal
voltage equal to 0 .7 4 A and 2 2 0 V respectively. (A ns. 92.777%]

5 .1 0 0 . A 3-p h ase, 2 0 0 0 kVA, 11 kV star-co n n e cte d a lte rn a to r h a s th e follow ing loss d a ta :


Open circuit core loss at 11 kV = 14 k\V
Short circuit load loss at 90 A, 75°C = 10 kW
Friction and windage loss = 16 kW
Field winding resistance a t 75°C = 0.31 O
F o r a field cu rre n t of 11 0 A, com pute th e a lte r n a to r efficiency a t ra te d load 0 .8 p f la g g in g .
C om pute also the effective a rm a tu re re s is ta n c e in p er u n it. IA n s . 9 7 .1 2 5 % , 0 .0 0 6 8 p.u.l

5 .1 0 1 . A 2 5 kW , 1 0 0 0 V, 3-p h ase sta r-co n n e cte d sy n ch ro n o u s m o to r is co n n ected to a n in fin ite bus a t rated
voltage. A t no load, th e m otor ta k e s a line c u rre n t of 6 A an d 1 k W , w hen field c u r r e n t is 3 A a t 2 5 0 V dc.
A rm a tu re circu it re s is ta n c e is 1 Q p er p h ase. C a lcu la te
(b) no-load ro tatio n al loss of th e m ach in e
(b) m ach in e efficiency a t ra te d load a t p f 0 .9 lag and *
(c) m axim u m efficiency a t p f 0 .9 lag.

IH in t. (a) N o-load ro ta tio n a l loss = 1 0 0 0 - 3 x 6 2 x 1 = ...| IA n s , <„) 8 9 2 W (6 ) 9 1 . 6 8 * (e> 9 1 .7 4 *1

5 .1 0 2 . (o) F o r th e sam e kVA load, efficiency o f a n a l te r n a t o r a t le a d in g p.f. is m o re t h a n a t laggin g p.f.


e x p la in .

(b) A sy n ch ro n o u s m o to r is su p p lyin g a c o n s ta n t load. W ill its efficien cy be g r e a t e r a t 0 8 p f lead or 0.8


p.f. lag ? E x p la in . H

(c) An a lte r n a to r h a s an efficiency o f 9 0 p e r ce n t w h en o p e ra tin g a t u n ity p.f. a n d a t r a te d kVA. E xp lain


w h a t h ap p ens to its efficien cy a t ra te d kV A b u t a t 0 .8 p.f. lag g in g .

IA n s . (a) An a lte rn a to r o p e ra tin g a t lead in g p.f. re q u ire s le ss field c u r r e n t, th e r e fo re , le ss f.e ld -circu it loss
and g r e a te r efficiency, for th e sa m e kV A load, (b) E fficien cy is g r e a te r a t 0 .8 p f la g (c) A t 0 8 p f. output
d e cre a se s (8 0 p er ce n t of its previou s v alu e), n o-load ro ta tio n a l lo sse s an d s .c . lo ad lo s se s re m a in unchanged

Scanned by CamScanner
Polyphase Synchronous M achines 701

0 f lag req u ires m ore field c u rre n t— th ere a re , th erefore, more field cu rren t losses and thus efficiency

5 5 103* Wh®*1 *s an ° P e r a ^ n 6 c h a r t ? W h a t is th e basis for the developm ent of operating c h a rt for an


alternator ^ . .
Describe how an o p eratin g c h a r t of a rou nd -rotor a lte rn a to r is developed from its p h asor d iagram . E xp lain
p oin t w ith in i ts o p e r a tin g a r e a r e n d e r s u sefu l in fo rm atio n ab ou t th e a l te r n a t o r o u tp u t in
JS MVA, p f > ad a n Ble e tc -
5 104. W h a t a re re a c tiv e cap ab ility cu rves of an a lte rn a to r ? Develop the capability cu rves sta rtin g from
the locus of co n sta n t a p p a re n t pow er of a cy lin d rical-rotor altern ator.
Discuss th e q u a d ra n t in w hich a lte rn a to r operation is more common so far as capability curves are
ncerned. Is it possible to au g m e n t th e activ e an d reactive power capabilities of an a lte rn a to r ? E xp lain .
5 105. (a) E x p la in w hy th e co n stru ction of synchronous condensers differs som ew hat from those of stan d ard
synchronous m otors.
(b) W hen is it d esirab le to use dual-purpose synchronous m otors ?
(c) An in d u strial load of 2 0 0 kW is supplied a t 1 1 0 0 0 V, the p.f. being 0 .8 lagging. A synchronous m otor
• reauired to m eet an ad d ition al load of 5 0 kW and a t the sam e tim e to raise th e re s u lta n t p.f. to 0 .9 lagging
Find the kVA cap acity of th e m otor and p.f. a t which it m ust operate. (Ans. (c) 57.75 kVA, 0.8 6 5 8 leadingl

5 .1 0 6 . (a) W h a t is a sy n ch ro n ou s condenser ?
[b) W h at a re th e a d v a n ta g e s of in stallin g a synchronous condenser in an electrical sy stem ? Illu s tra te you r
answer with an exam p le.
(c) An in d u strial p la n t h a s an a v erag e load dem and of 8 0 0 kW a t a pf of 0 .71 lagging. A synchronous m otor
of 400 kVA is in stalled for driving an additional load and for improving th e p lant power factor. Th e synchronous
ioad is 1 6 0 kW a t an efficiency of 90% . F o r synchronous m otor operation a t ra te d kVA, ca lcu la te th e
S b a d kVA an d th e re s u lta n t pf. I A » . <r> >070.24 kVA. 0 .9 1 3 6 loggmgl
5 .1 0 7 . (a) Show t h a t th e u se of a synchronous condenser im proves th e efficiency and regu lation of a sy stem .
(b) A co n su m er h a s a to ta l load of 2 0 0 0 kW a t a pf of 0 .8 lagging. If it is required to im prove th e pf to 0 .9 5
lagging d eterm in e th e kVA ra tin g of th e synchronous condenser for the purpose. U -A .i.. jy y si
* lA ns. (b ) 8 4 2 kVAl

5 .1 0 8 . (a ) In F ig . 5 .1 0 5 (A) an d (B ), explain
whether the sy n ch ro n o u s m ach in e is w orking as a
generator or a m o to r, w ith field w inding on th e rotor
and with the d irectio n of ro tatio n show n. The phasor
diagram for each figu re sh ou ld also be draw n in d icat­
ing pf angle, in te rn a l pf an gle, excitatio n voltage, te r­
minal voltag e, a r m a tu r e c u rr e n t etc. for lagging as
well as leading pf o p eratio n s. A rm a tu re resistan ce
may be n eglected .
(8) In ca se sy n ch ro n o u s m ach in e of p a rt (a ) is
operating a t no load, show th e o rien tatio n of field poles
(A) (B)
in relevan t d ia g ra m s . D raw p h a so r d iag ram un er Fig. 5.105. Pertaining to Problem 5.108.
these con d ition s also a ss u m in g e x cita tio n voltag e
greater th a n te rm in a l v o ltag e. N eg lect all losses.
lA ns. (a) (A) A lternator (B) Motorl

• •* c W t - r i r c u i t ch a ra c te ris tics is availab le for a 3 -p h a se sy n ch ro n ou s


5 ,1 0 9 . (a ) D a ta for o p en -circu it an infinite bus is w orking as a syn ch ron ou s m otor a t no load ,
machine. In c a s e th is m a ch in e , co n n ected to an infinite bus, is vo g y
describe how its P o tie r r e a c ta n c e ca n be obtained.
1 , . . evnchronized w ith an infinite bus a t ra te d v o lta g e , s te a m input
(8) A salien t-p o le sy n ch ro n o u s m ach in e yn ,. deiivers r a ted kVA to th e infinite bus at a load
to prim e-m over is now in c re a se d till th e sy " c . m a ch ;n e is 1.2 pu, find th e v a lu e of A ', p ow er fa cto r and
angle of 4 0 °. I f d -a x is sy n ch ro n o u s re a c ta n c e F in d a ls0 th c ^ a c t i v e pow er d elivered to th e
Power delivered to in fin ite bus u n d er th e con ^ ^ 0.6 5 4 4 pig 0 .8 7 6 6 leading. 0 .8 7 6 6 pu, - 0 .4 8 1 pul
bus. ’

5 . U 0 . (a ) S how t h a t a sy n ch ro n o u s m otor h as no no „ Rain st light-lootl torq u e.


<6, D escrib e th e n teth od s o t ^
(c) E x p la in th c m eth od s of s la ttin g syn •

icanned by CamScanner
702 E le ctrica l M ach in ery _________ _____________ ____ __________________________
_ . ., m otor sta rtin g of a sy n ch ro n ou s m o to r, exp lain w hy it is necessary (j)
* • field winding al the t ™
(M D urin g the induction-m otor s ta rtin g , w hen th e Held windrng
in th e supply cu rre n t. E x p la in how th is occurs. G .v e a m ethod of o vercom in g th ts o ccu rren ce^

(c) F o r a synchronous m ach in e, th e th re e -p h a se w inding is on th e ro to r an d field w in d ing is on the stato,.


Th e revolving field produced by th e 3 -p h ase a rm a tu re cu rr e n t revolves clockw ise. E x p la in th e direction in which
ro to r m u st ro tate lAns. tii) It can be overcome by increasing the field current groduan,.
ro to r m u st ro tate. (c) Direction 0f rotor rotation m ust be anti-clockwise.)

5 .1 1 2 . (a) A synchronous m otor is supplying a ce rta in load. In o rd e r to stop th e m o to r, it is essential to


sw itch-off first th e ac supply an d th e n its field cu rre n t. E x p la in th e re a so n . ^ ^
(6) An a lte rn a to r connected to infinite bus is o p eratin g a t h a i f f u i U o a d i n c r e a s e of its i field current
cau ses a d ecrease in th e a rm a tu re cu rren t. Is th e a rm a tu re cu rre n t lea in g o ggi ge .
(c) E xp lain how th e g rap h betw een p f and field cu rre n t ca n be o b tain ed form th e V -cu rv e of a synchronous
m otor.
[A n s. (a ) V -curves in d icate t h a t if field cu rr e n t is first re d u ce d to zero, a r m a tu r e c u r r e n t increases to a
valu e m u ch h ig h er th a n its ra te d cu rre n t w hich is d e trim e n ta l to both th e sy n ch ro n o u s m o to r and the supply
sy stem .
(b) A rm a tu re cu rre n t is lead in g the term in a l voltage.
(c) F o r m inim um a rm a tu re cu rre n t, p f is unity. F o r an y o th e r field c u r r e n t, Iu cos 0 = Since
a rm a tu re cu rre n t I„ is know n from th e V -curve, p f for an y v alu e of field c u rr e n t ca n be calcu lated .]

5 .1 1 3 . (a) D iscuss the ap p lication s of synchronous g e n e ra to rs and sy n ch ro n o u s co n d en sers.


(b ) E n u m e ra te th e ad v an tag es and d isad v an tag es of u sin g sy n ch ro n ou s m o to rs in in d u s tria l applications.
(c) D iscuss th e in d u strial ap p lication s w h ere syn ch ron ou s m o to rs a re p re fe rre d to in d u ctio n motors,
5 .1 1 4 . Th e tra n sfo rm e r an d cable feeding pow er to an in d u stria l p la n t h a v e a r a tin g of 1 0 0 0 kVA and are
fully loaded by th e p lan t w hich d raw s 1 0 0 0 kVA a t a p f of 0 .7 laggin g. A 1 0 0 k W sy n ch ro n o u s m otor having
an efficiency of 90% is to be added to the p la n t load w ith ou t in cre a sin g th e r a tin g of tra n s fo rm e r and cable. At
w h at p f m u st th e syn chron ou s m otor o p erate ? [A ns. 0 .652 leading)

5 .1 1 5 . A star-co n n ected synch ron ou s m ach in e, w ith Z s = 1 + j 10 Q p e r p h a se , is sy n ch ro n ized with an


infinite bus of 11 kV. The m ach in e is m ad e to o p erate a t a le a d in g p f of 0 .8 w ith a n a r m a tu r e current of
5 0 A and w ith te rm in a l voltag e a h e a d of th e e x cita tio n emf. C a lc u la te th e m a g n itu d e of e x cita tio n emf.
How can this m achin e be m ad e to o p erate a t u n ity p f w ith o u t a lte rin g its field e x c ita tio n ? U nder this
condition, find th e a rm a tu re c u rr e n t and load angle. [I.A.S., 2000)
[A n s. 6 6 2 4 .9 7 V ; 2 6 0 .7 5 1 A ; 23.176°|
5 .1 1 6 . A cy lin d rical ro to r a lte rn a to r is o p e ra tin g a t a la g g in g pf. S how t h a t E f I„ • cos \jI is equal to
E r V,
y sin 5, w here vf = in te rn a l p ow er-facto r an gle an d 8 = load an gle. A r m a tu r e -c ir c u it re s is ta n c e is neglected.

5 .1 1 7 . A cy lin d rical-ro to r sy n ch ro n ou s m ach in e is sy n ch ro n ise d w ith a n in fin ite b u s a t r a te d voltage. Now


th e s te a m inp u t to th e m ach in e is in cre a se d till th e sy n ch ro n ou s m a ch in e b egin s to o p e r a te a t its ra te d current.
T h e sy n ch ro n ou s m ach in e h a s Z s = 0 + j 1.2 pu. C a lc u la te th e pf, load an g le a n d th e a c tiv e a n d reactiv e power
d elivered to th e infin ite bus u n d er th ese conditions.

W ith o u t a n y ch an g e in th e s te a m in p u t, how can th is a lte r n a to r be m a d e to d e liv e r no re a c tiv e power to


th e bus ? F in d th e e x cita tio n v o ltag e an d load an gle u n d e r th is con d ition .
[A n s. 0 .8 lead in g, 6 = 7 3 .7 4 °, P = 0 .8 pu, Q = - 0 .6 0 p u , E f = 1 .3 8 6 2 p u , 8 = 5 9 .9 6 ° )

5 .1 1 8 . A salien t-p o le sy n ch ro n o u s m ach in e is sy n ch ro n ized w ith a n in fin ite b u s a t r a t e d v o ltag e. Its steam
input is th e n in c re a se d till th e m ach in e d eliv ers r a te d c u r r e n t to th e b u s. T h e m a c h in e h a s X (l = 1.0 pm
X q = 0 .8 pu an d n egligib le re s is ta n c e . F in d th e load an g le , p ow er fa c to r a n d a c tiv e a n d re a c tiv e powers under
the conditions s ta te d h ere.

If th e m a ch in e is m ad e to o p e ra te a t u n ity p f w ith o u t a lte rin g its s t e a m in p u t, c a lc u la te th e new excitation


voltage an d load an gle. [A ns. 4 8 .7 7 °, 0 .8 7 6 1 leading, 0 .6 2 8 0 7 pu, - 0 .4 8 2 3 2 pu ; 1 .3 2 1 6 pu, 35.03°i

Scanned by CamScanner
_________________________________________ S I X

Polyphase Induction Motors

Introduction
A polyphase induction motor is a singly-excited a.c. machine in the sense that it is supplied
0wer from a single ac source. Its stator winding is directly connected to a.c. source, whereas
its rotor winding receives its energy from stator by means of induction {i.e. transformer action).
Balanced polyphase currents in polyphase windings produce a constant-amplitude rotating
mmf wave. The stator-produced m.m.f. wave and rotor-produced m.m.f. wave, both rotate in
the air gap in the same direction at synchronous speed. These two m.m.f. waves are thus sta­
tionary with respect to each other, consequently the development of steady electromagnetic
torque is possible at all speeds but not at synchronous speed. The stator and rotor m .m .f . waves
com bine to give the resultant air-gap flux density wave of constant amplitude and rotating at
synchrdhous speed. Since an induction motor can’t run at synchronous speed, it is called
asynchronous m achine. /
The stator of a 3-phase induction motor is essentially similar to that of a synchronous /
» M“ e reader fs advised to go

maintenance, it is used where higher start­

ing torque and speed control is desired.


o b je rt^ ^ s ’ chapter i^ to ^ v elo p t h ^ t ^ in iq u e s fo r t ^ a r S ^ is of ^l^phaselnduction machines.

sider an induction motor with both itssta ^ ^ ^ e,ectromagnetic torque is developed.


assumed open-circuited so that rotor cur freauencv U to the stator winding, causes the
Application of three-phase balanced voltages at line frequency h ______
production of a rotating magnetic field. This rota. g Ao-
r “i
flux cuts both the stator and stationary rotor conduc El I
tors at synchronous speed, consequen y"e. ■ ■
line frequecy f x are induced in them. Per phase value
the e.m.f. E x induced in stato r winding is give
Eq. (3.26) as „ B<^ _ stqtor 1
E x = V 2nA *u ;i^i ^ ° 6.1. 3-phase wound-rotor induction motor
F ie
Here N x = stato r series turns per p ase with both stator and roto. in star.

and kwl = stato r winding factor.

Scanned by CamScanner
704 E le c tric a l M ach in ery

Similarly per phase value of e.m.f. E 2 induced in the standstill rotor winding js
E 2 = '\2nfykw2N 2 ty
Here N2 = rotor series turns per phase
and k w2 = rotor winding factor.
Line frequency fy appears in the expression for E 2 also, because rotor is at standstill he
4>is the resultant air-gap flux per pole.
The e.m.f. or voltage ratio for the induction machine from Eqs. (6.1) and (6.2) is
Ey N xk wl N x'
E 2 ~N 2 kM N2 ' -(6.3) j
Here N f and N2’ are called the effective number of stator-series turns per phase and roto,.
series turns per phase respectively.
Eq. (6.3) is similar to the voltage ratio of a transformer, see Eq. (1.11). Thus, a wound-rotor
induction motor at standstill is similar to a transformer at no load. Further, the resultant
mutual flux in a transformer is due to the combined action of primary and secondary m.m.fg.
similarly in induction machines, synchronously rotating air-gap flux (or mutual flux) is due to
the combined action of both stator and rotor m.m.fs. In induction machine, the rotating air-gap
flux generates counter e.m.f. Ey in the stator winding similar to the counter e.m.f. induced (by
the mutual flux) in the primary winding of a transformer. As the transformer is loaded, the
m.m.f. of the secondary current reacts on the primary in order to draw more power from a.c.
source, similarly with the increase in shaft load, the rotor m.m.f. reacts on the stator winding
in order to extract more power from the a.c. source. A 3-phase induction motor with its rotor
blocked behaves similar to a transformer under short circuit. In addition, stator and rotorwind-
ings of an induction machine possess resistances and leakage reactances just like the resistan­
ces and leakage reactances of the primary and secondary windings of a transformer. In viewof
these similarities, an induction machine is called a gen eralized transform er and it is for this
reason that the stator winding of an induction machine is treated as its primary and the rotor
winding as its secondary. Consequently, the phasor diagram and equivalent circuit of a 3-phase
induction motor are almost similar to those in a transformer. The subscripts 1 and 2 are used
to denote the stator and rotor quantities respectively, as in a transformer.
The voltage ratio of the induction machine includes winding factors &u l and k^, because
the stator and rotor windings are distributed along the air-gap periphery . [The transformer
primary and secondary windings, being concentrated windings, require no^induig factors. In
Eqs. (6 .1) and (5TZ)7<Rs the average value of the rotating flux per pole. In tran&former-emfEqs.
(l~?Tan5^(1.7), maximum value of the core flux is used.
Another difference between induction motors and transformers is that the no-load current
in induction motors varies from about 30 to 50% of full-load current, whereas in transformers,
no-load current varies from 2 to 6% of full-load current. This is because of the fact that the
mutual flux in a transformer completes its path through low-reluctance iron core^whereas the
mutual flux in an induction motor has to cross the air-gap between stator anHrotonj^orj
constant applied voltage, the mutual flux remains substantially constant. Since thereluctance
offered to the mutual flux in an induction motor is more due to the presence of air gap, a con-
c nj m f ) a.
stan t mutual flux — . ' — requires more magnetizing m.m.f. and, therefore, more ma0
reluctance #,
netizing current. In induction motors, the magnetizing current (lagging nearly 90° behin
applied voltage) forms a considerable portion of no-load current, th at is why induction m0
operate a t low power factors a t no loads. Usually the no-load power factors of induction m ^
are in the neighbourhood of 0.15 or so. The effect of low value of no-load power fac or -

Scanned by CamScanner
Polyphase Induction M otors 705

^R the full-load operating power factor of the induction motor. Thus, for keeping the in-
deCf n rTT^2Lmagnetizing-CUrrea^ PW therefore, a better no-load and full-load power
motors are kept as small as are mechanically possible.
f^ p rin cip le o f operation
^ The stator winding of a 3-phase induction motor is connected to 3-phase balanced supply.
^nn, flow of 3-phase currents in the 3-phase stator winding produces a rotating magnetic field
Tne or
s discussed in Art. 3.7. The speed of rotating field is the synchronous speed, ns = - jf- rps. The
rotating flux wave cuts the stationary rotor conductors P ■na times per sec and therefore emfs
induced in the rotor conductors. As the rotor circuit is short-circuited, these induced emfs
ve rise to current in the rotor conductors. The interaction of these rotor currents with rotating
flux wave produces torque in the rotor of a 3-phase induction motor and as a consequence, rotor
begins to rotate.
According to Lenz’s law, e f f e c t o p p o s e s t h e c a u s e . Here, effect is the developed torque and
cause is the flux-cutting by the rotor conductors. Therefore, as per Lenz’s law, the developed
torque must oppose (or minimise) the cause, that is flux-cutting action. This is possible only if
the developed torque forces the rotor to rotate in the direction of rotating field. When this hap­
pens, the relative speed between rotating flux and rotor conductors is reduced and therefore
flux-cutting action (times per sec) also gets reduced. For example, if rotor speed is n r rps m the
direction of rotating flux wave, the relative speed between rotating flux wave and rotor conduc­
tors becomes (ns - n r) rps and the flux-cutting action reduces from P ■n s times per sec to
P ■(ns - n r) times per sec as demanded by Lenz’s law. This shows that r o t o r m u s t r o t a t e in t h e
d irection o f r o t a t i n g m a g n e t i c f i e l d when 3-phase supply is given to stator of a 3-phase induction
motor. If rotor is assumed to run at synchronous speed n s in the direction of rotating field, then
there would be no flux-cutting action, no emf in rotor conductors, no current in rotor bars and
therefore no developed torque. Thus, the r o t o r o f 3 - p h a s e i n d u c t i o n m o t o r c a n n e v e r a t t a i n
synchronous speed.
6.3. Flux and m m f phasors and w aves in Induction Motors N
As stated before, when stator of a 3-phase induction motor is excited from a balanced 3-
phase source, a constant-amplitude rotating mmf is produce . otormm so ®
same direction at synchronous speed. Therefore, stator rotating mmf wave and ro taro U tm g
mmf wave are stationery with respect to each other; these can be combined together togetaie
resultant air-gap mmf wave or resultant flux density wave rotating at synchronous s ^ e i
^nce the relahve speed between rotor mntf and the Ksultantfluxdensitywaveiszerm a steady
torque is developed by their interaction. Tlie_purp ^ torque is partly depend.
a the direction of rotating magnetic field m d induction motor is considered and
ent upon the rotor circuit parameters. First ot a , P e
(hen the squirrel-cage induction motor.
_, . . „ ,0 o ^ o c e slip-ring induction motor, with its rotor
Fig. 6.2 (a) illustrates a simple 2-pole, P with coil8 a a% b fc', c c' representing
stationary. The rotor is shown to carry 3-p h iE d u ced bv the resultant flux-density wave are
Phases a, 6, c respectively. Two poles N lt i P . direction. These two rotating poles
shown rotating at synchronous speed in the an d phase a has maximum emf
Cut the stationary 3-phase rotor winding a hes under the pole centres of poles
generated in it, because coil aa' represen in£ P ’ emf generated in phase a is indi-
K 8 X; where the resultant flux density is peak. Ma«mum. * ^ ^ ^ ..
cated by larger cross and dot in coll:sl^ ‘ flf tw^coil-sides is less than the maximum
afluence of pole N i, emf generated in t

Scanned by CamScanner
•frf
706 Electrical Machinery jA rt

: M ax amf
^ and current
R e su ltan t
s flux dens<ty
w a ve

•fi

Resultant
rotor mmf
w ave

Resultant f lu x
d e n s i t y wav*

P T
90

h
e2
Rotation
(b)
tx '
Fig. 6.2. Slip-ring induction m otor : F o r zero rotor leakage re a c ta n c e (a) flux d en sity an d ro to r m m f phasors
(6) rotor developed view with flux-density and ro to r m m f w av es and (c) p h a so r d iag ram .

indicated by crosses of lower size. Sim ilarly, dots are shown in coil-sides b, c. I f the rotor circuit
is assumed purely resistive, then current in any coil would be in phase w ith the emf generated
L lt t e T - ‘ ' V I Cated in F ig - 6 2 (0) In other WOTds’ coiI has maximum emf
b b ' T / ' h l V and als0 tbe “ “ “ “ “ current in it at the instant, w hereas other phase cods
b , c c has current less than the maximum as shown in Fig. 6.2 (a) R ecall that the neak of

c“ emm L : r mprc t : i in > 3; phase “ g ai» g * • ^


™ ‘ he " ' “ “ uum current m rt. In vrew of this, the resu ltan t rotor m m f F 2 due to the

phase has ^ u ™ ^ I t T O ” 7 * ® 2 <0)' 18 along tha <* H — a , because tins

stator pohs A^ S anH T T 'a 6 2 ^ Remember ^ a t the relative velocity betweeu


tracts S thin^n i r0t”r",nd1UCed P0les S 2 l» rero. R otating pole N , repels N2 but at-

...i . a s “ r r . i r d 5™ ." ? - '" ! " " ;'''

in g m a g n e ti c fie ld . T h i s s h o w s t h a t a ^ h M e i ’ O r b e g *n S t 0 r 0 t a t e i n t h e d i r e c t i o n o f r o U t

tu a liy th e r o to r a t t a i n s a s p e e d n , le s s th a n s y n c h ro n o u s J J S u .‘ 8 e lf' S ta rtin K m 0 t° r 1

Scanned by CamScanner
P oly p h ase In d u ctio n M o to rs 707

The sp&ce an^ e between F 2 and <{>is called the load an gle , symbol 5. In Fig. 6.2 (a), 6 is seen
ual to 90°. According to Eq. (3.826), the magnitude of electromagnetic torque is
to beeq
T = - P2 4>F2 sin 90 ° = ^ P 2 <pF2 ...(6.4)
e 8
'phis shows that if rotor circuit is purely resistive, load angle has an optimum value of 90°.
ultant flux density wave, sinusoidally distributed along the air-gap periphery, is shown in Fig.
jj^a) and (6). The rotor mmf wave, also sinusoidally distributed in space, lags the flux- density
' ve by S = 9A°> Fig- 6.2 (a) and (6). As in a synchronous machine, rotor mmf wave, in a 3-phase
* ductiou motor, has a tendency to align itself with the resultant flux density wave ; the torque is
therefore directed to the right in Fig. 6.2 (6). In other words, the rotor moves in the direction of
rotating magnetic field. Fig. 6.2 (6) compliments the presentation given in Fig. 6.2 (a).
phasor diagram for the condition illustrated in Fig. 6.2 (a) and (6) is shown in Fig. 6.2 (c).
Here, air-gap flux is taken horizontal for convenience. Rotor generated emf E 2 lags the air-gap
flux <(>by 90° as in any rotating electrical machine. Rotor current I 2 is in phase with E 2, because
rotor circuit is assumed to be purely resistive. Rotor mmf F 2 must be in phase with and propor­
tional to rotor current I 2 th at produces it.
Actually, the polyphase rotor winding is housed in slots distributed along the rotor
periphery. Therefore, rotor winding possesses considerable amount of leakage reactance x2 at
standstill. Accordingly, rotor current in each phase lags behind the generated emf in that phase

by rotor pf angle 02 = tan" 1 — •For example, coil-sides a, a'under stator AT1( poles have max-
r2
imum generated emf as shown in Fig. 6 .2 (a). On account of rotor leakage reactance, currents
in coil-sides a, a' would attain maximum value only when the poles N X, S x have travelled
through an angle 02 from its maximum emf position shown in Fig. 6.2 (a). This is illustrated in
Figs. 6.3 (a) and (6). As before, the peak of rotor rotating mmf F 2 is along the axis of phase ‘a ’,

ip-ring induction m otor :


ib) rotor developed view with f i u x - a e n s ,

Scanned by CamScanner
J*rt ll
7l\m
08 tiecirn-ai
E I achinery
lectrical M _________ _____ .
— f . Fies 6 3 (a) and (b ). Examtnatjon of p,^
because this phase carries the maximum curren ^ dispia ced from the peak of flux * b>
6.3 (a) and (h) indicates that peak of rotor m 2 flux density wave or air-gap flU|
a load angle of (90 + 9). Poles N x, \, p r o u electromagnetic torque is developed in the
4 , interact with rotor poles N2, S 2 as betore 3 } and {6).
direction of rotating magnetic fiel as s own i ^ ^ pig 6 2 (c). Here flux 4 is taker,
Phasor diagram of Fig. 6.3 (c) is draw n by rotor CUrrent / 2 lags rotor emf £ 2by
horizontal as before. The generated emf A, lags f J 5 = 90 + 0. The electromag.
rotor power-factor angle 8, and rotor m m fF 2 lag s* by a load B
netic torque is now given by
Tt - 5 p 2 4 F 2 sin (90 + 02) ~

= a P2 4 F 2 cos 02 " (6-5)


8
The rotor power-factor angle 02 is equal to tan - | Here x2 is the per phase rotor leakage mac

tance at standstill and r2 is the per phase ^

z r Ar sr s u T ^
s Z n g torque. 82 should be as small as possible, so that load angle becomes nearer to optunum value
of 90° this can be achieved by either reducing x2 or by increasing r 2.
For explaining the development of torque in a squirrel cage induction motor, refer to 2-pole
3-phase induction motor of Fig. 6.4 (a). Two poles W,, S , produced by the rotating air-gap flux cut
the stationary rotor bars at synchronous speed. The bars a. just under the pole centres haw
maximum emfs generated in them and this is indicated by larger cross and dot m bare a b mlFig
6 4 (o) Bars away from the pole centres have reduced magnitudes of generated emfs and these are
indicated by varying sizes of crosses and dots. As the bars are embedded in the rotor iron, the rotor
circuit possesses considerable leakage reactance. Consequently, rotor currents in each bar lags

behind the generated emf in that bar by rotor pf angle 02 = tan 1 ^7 - For example, bars a. b under

the poles have maximum generated emfs as shown in Fig. 6.4 (a). On account of rotor leakage
reactance, currents in bars a, b would attain maximum value only when the poles NX,S Xhau
travelled through an angle 0 2 from its position of maximum emf in Fig. 6.4 (a). This is illustrate
in Fig. 6.4 (6 ). Rotor currents establish synchronously rotating m m fF 2 and it is seen from Fi£ 6 4
1 if c
(b ) that F 2 is space-displaced from air-gap flux <J>by a load angle 6 = 90 + 9>. Stator poles ^ m
produced by resultant flux density wave interact with rotor-induced poles N2, S > and thervtore
electromagnetic torque is developed in the direction of rotating magnetic field as shown in Fi£ ^
(6 ). The phasor diagram is again the same as in Fig. 6.3 (c).
Reaction of the rotor on stator of a squirrel cage induction motor is also depicted in Fi.c ^ ,
In Fig. 6.5 (a) are shown instantaneous magnitudes of bar voltages. Flux density at bars 4
11 (or a and b) is maximum, these bars have, therefore, maximum em f generated in them •

the flux density wave has travelled through power-factor angle 0a = tan 1
, c u r r e n t s ia •'
ri ^
4 and II attain maximum values as Hhown in Fig. 6.5 (5). Profile of the instantaneous
currents is sinusoidal. In other words, bar currents are equivalent to a sinusoidal c u r r e n t s * ^
Sinusoidal bai currents produce a sinusoidal mmf wave lagging sinusoidal current s 1* ^
90" as per Fig. 3.30 (/>). It is seen from Fig, 6.5 (b) th at load angle 6 is equal to (90 +

Scanned by CamScanner
Polyphase Induction M o to rs 709

Resultant flu*
density wave

Mo* emf

(a) (b)
Fig. 6.4. Squirrel cage induction motor (a) direction of generated emf in rotor b a r s
(b) direction of cu rren ts in rotor bars, rotor mmf wave and flux-density wave.

m m f wave has a tendency to align with the resultant flux density wave, the torque is therefore
directed towards the right. In other words, rotor moves in the direction of rotating magnetic
flux density wave.
The above presentation brings out the similarity between cage and wound rotors, in so far
as their reactions on the stator are concerned. The rotor mmf is also called the armature mmf

-6*90 +V R e su lto n t flux


density wave
Rotor mmf I
wave

Instantaneous
bar currents

Rotat i o n

o f Fig. 6.4. to) profile of rotor generated em f and


P,g' 6 5 P ««*aining on flux-density wave.

’ 7! ■w yp-
Scanned by CamScanner
710 E le c tric a l M a ch in e ry [^ U 3

or armature-reaction mmf, because in an induction motor, stator and rotor windings are caUed
the field and arm ature windings respectively. 1
Examination of Figs. 6.4 and 6.5 reveals that number of rotor poles in a squirrel cage indu[.
tion motor is decided by the number of inducing flux-density poles. In Figs. 6.4 and 6.5, the
inducing flux density has two poles, the rotor mduced poles are therefore, two_ In case lluj
density wave has four poles, number of poles induced in rotor would also be four, hi other
words, a squirrel-cage rotor winding can adjust itself to any number of stator poles. But» „
wound-rotor induction motor, both the stator and rotor w in d in g s m ust be wound for the same
number of poles, though the number of stator and rotor phases may be different.
Squirrel-cage winding is short-circuited on itself, therefore, it is not accessible and no ei.
tem al resistance can be inserted in the rotor circuit. The wound rotor is however accessible
through its slip rings, therefore, external resistance can Im inserted in the rotor circuit. This
means that load angle 90 + 02 is under control only m case of wound-rotor induction motor. For
example, the starting torque can be controlled in case of wound-rotor induction motor but not
in a squirrel-cage induction motor.
It may thus be inferred from above that a starting torque is developed in a 3-phase induc­
tion motor in the direction of rotating magnetic field. Eq. (6.5) shows that torque m a 3-phase
induction motor depends upon (i) number of poles squared (it) air-gap flux per pole, <|>(m) rotor
mmf per pole F 2 and (iv) rotor p f angle 02. Rotor of a 3-phase induction motor eventually attains
steady speed nr < ns. Under this condition, relative speed between rotating flux and rotor con­
ductors is (n - n ) The development of electromagnetic torque with rotor speed nr can also be
explained in a manner similar to that in Figs. 6.2 to 6.5. However, the angle of lag 02 would now

be tan- 1 i f 2 where s = slip as defined in Eq. (3.29) and magnitude of rotor mmf would depend on
r2 ’
load on the shaft of induction motor.
It has already been discussed why the rotor of a 3-phase induction can never attain
synchronous speed.
E x a m p l e 6 .1 . A th ree-p h a se, star-con n ected,
wound-rotor induction m otor is fed from d.c. sources as
show n in Fig. 6.6. The currents in both stator an d rotor
windings are kept within limits. I f the d.c. supply ter­
m in al is ch an g ed from a p h a se winding to b p h a se
w inding o f the stator, fin d the num ber o f m echanical
degrees through which the rotor would move, in case o c supply
the m otor h a s (a) two poles (b) fou r poles an d (c) six Fig. 6.6. C ircu it diagram , Example 6.1.
poles.
* o f

S o lu tio n , (a) In Fig. 6.6 (a), coil a, a' represents phase a winding. When the direc on^
direct current in phase a is as shown by cross in a and dot in a', then two stator poles n g ^
produced. The d.c. in the rotor windings also produces two poles N 2, S 2 as shown. In B
it is assumed th at pole S 2 is near N 1 and pole N 2 is near S h because poles of ^ P 051^ hase b
attra ct each other. When the supply terminal is changed from phase a wmdlI^f,own in Fig
winding, the stator produced poles JV, rotate through !20» elect clockw.se^ dbyN.
6 6 ( b ) In this figure, N 2 is attracted by S x but repelled by N lt similarly S 2 is at ^
but repelled by S v As a result of this interaction, rotor structure moves throug
clockwise and occupies the final steady position as illustrated in Fig. 6.6 (c).

Scanned by CamScanner
Polyphase Induction Motors 711 Ii

/ /* f/ /u\> -

(a) „• (fc) (c)


frig. 6.6. Pertaining to Exam ple 6.1.

For a 2-pole machine, 9c/ec = 9mec/,, therefore, the rotor revolves 120 mechanical degrees, i.e.
^ough one-third of a revolution.

(fc) For a P -pole machine, 9e,ee = ^ Qmech

For a 4-pole machine, as the supply is changed from phase a winding to phase b winding,
the rotor again revolves through 120° elect.

Kech = ^ (120°) = 60°.

Thus in a 4-pole machine, the rotor rotates through 60 mechanical degrees (i.e. l/6th of a
revolution) as the d.c. supply is changed from one phase to the next phase in sequence.
2
(c) For a 6-pole machine, Qmech = — (120°) = 40°.
o
Thus in a 6-pole machine, the rotor revolves through 40 mechanical degrees (i.e. l/9th of a
revolution) as the d.c. supply is changed from one phase winding to the adjacent phase winding.
6.4. Rotor Frequency
It has been shown th at rotor starts running in the direction of rotating magnetic field. At
standstill, rotor conductors are being cut by rotating flux wave at synchronous speed nt, there­
fore frequency f 2 of the rotor e.m.f. and current is equal to the line frequency f x. When rotor
revolvesat a speed of nr r.p.s. in the direction of rotating flux wave, the relative speedbetween
synchronously-rotating flux and rotor conductors becomes (ns - n r) rps.
•\Frequency of rotor e.m.f.
.p , . [relative speed between rotating flux and)
( o es) j rotor structure in r.p.s
...(6.6 a)

P (*, ~ n r) ...(6.6 6)

The slip has already been defined by Eq. (3.29 b ) as


ns —nr synchronous speed-rotor speed
s= n ~ synchronous speed
ns ~ n r = sn ,
P ■sn, Pn„ ...(6.7)
•*. Rotor frequency, f 2 = ■= s = s fj
2 2 "

Thus the frequency of the rotor e.m.f. and current in an induction motor is given by the
Product of slip s and the line frequency f v It is for this reason th a t f2 is also called the slip
flu en cy .

canned by CamScanner
712 Electrical Machinery _____________________________

At standstill, rotor frequency is f\ and the field produced by rotor currents revolves at a

speed equal to 2 (rotor fre(\uencl l = n ) with respect to rotor structure. When the rotor i*
rotor poles
i uLui jjuica iPp 6
revolving at a speed nr, the rotor frequency is sf\ and the rotor-produced field revolves at a

speed of ^ = sn, with respect to rotor structure. But the rotor is already revolving at a

mechanical speed of nr rps with respect to stator. Therefore, the speed of rotor field with respect
to stator is equal to the sum of mechanical rotor speed nr and rotor-field speed sn, with respect
to rotor. Thus the speed of rotor field with respect to stato r is given by
nr + sn, = n, (1 - s) + sn, = n, r.p.s.
But the stator-field speed with respect to stator is synchronous speed n, rps. This concludes
th at the stator and rotor fields are stationary with respect to each other at all possible rotor
speeds. Since the relative speed between stator and rotor fields is zero, a steady torque is
produced by their interaction and rotation is maintained.
The rotor of an induction motor can never attain synchronous speed. In case rotor speed is
synchronous, the rotor conductors would be stationary with respect to the synchronously-rotat­
ing magnetic field. As a result of it, no e.m.fs. would be generated in rotor conductors and,
therefore, rotor m.m.f. would be zero. But the existence of rotor m.m.f. as per Eq. (6.5), is essen­
tial for production of torque. This shows that the operating speed of the rotor of an induction
motor can never be equal to the synchronous speed.
E x a m p le 6 .2 ,A3-phase, 50 Hz induction motor has a full-load speed o f 1440 r.p.m. For this
motor, calculate the following:
(a) number of poles
(b) full-load slip and rotor frequency
(c) speed of stator field with respect to (i) stator structure and (ii) rotor structure and
(d) speed of rotor field with respect to (i) rotor structure (ii) stator structure and (iii) stator
field.
For parts (c) and (d), answer should be given in rpm and rad I sec.
S o lu tio n , (a) The use of full-load speed of 1440 r.p.m. in Eq. (3.69) gives
m n
1440 120 h
----- 1 2 0- x 5 0

„ n120x50 ,1 ,
OT ~ I440- 6 P
Since the number of poles must be even and a whole number, the induction motor must
have 4 po«es. Note that an induction motor runs at a speed, a little less than synchronous speed.
(6) Synchronous speed,
xr 120 x 50 ____
N, = ----- ------ = 1500 r.p.m.

•• ollD> _ N‘ _~_N r —
c i:— 5„ — _ 1500 - 1440
■ “ 0„ Q
„A.
H N, 1500 UU4-
Rotor frequency, f2 = s f = 0.04 x 50 = 2 Hz.
(c) (i) Speed of stator field with respect to stator structure
= N , = 1500 r.p.m.
2*Nf 2n x 1500 ....................
= gQ = ----- — ------= 150.08 rad /sec

Scanned by Cam Scanner


. -— ----Polyphase Induction Motors 713

/••) Speed of s ta to r field w .r.t. revolving rotor structure


= 1 5 0 0 — 1 4 4 0 = 60 r.p.m .
2k x 60 __
- gQ - 6 .2 8 3 ra d /se c

(d) (i) Speed of rotor field w.r.t. rotor structure


_ 120 (rotor frequency)
poles
120 x 2
= — ^— = 60 r.p.m. = 6.283 rad /sec

(ii) Speed of rotor field w.r.t. stator structure


= (M echanical speed of rotor) + (Speed of rotor field w.r.t rotor structure)
= 1 4 4 0 + 6 0 = 1500 r.p.m . = 150.08 rad/sec.
(iii) Since both th e sta to r and rotor fields are rotating at synchronous speed of 1500 rpm
th respect to sta to r stru ctu re, speed of rotor field with respect to stator field is zero. Thus the
stator and rotor fields are station ary with respect to each other.
Exam ple 6.3 .A properly shunted centre-zero galvanometer is connected in the rotor circuit
ofa 6-pole, 50 Hz wound-rotor induction motor. I f the galvanometer makes 90 complete oscilla­
tions in one minute, calculate the rotor speed.
Solution. One com plete oscillation of galvanometer corresponds to one cycle of rotor fre­
quency.
90 ' „
Rotor frequency, f 2 = sfx - 6 q _ m

or Slip, s = ^ = 7 j§ = 0 03

Rotor speed = (1 _ s) = (1 - 0.03) = 970 r.p.m.

Exam p le 6 .4 . A

(а) Atwhat two speeds the pnme-m over s o ^ ^ ^ ^


(б) Find the ratio o f the two voltages ava ^ remain the same at the two
(c) Find out whether the phase sequence of 3-ph.

SPeed$' , * « id «nth respect to stator structure


Solution, (a) Speed of rotor field with respe
120 fi _ 120 x 50 _ 15qo r pm

^ 120 h _ 120 x 20 _ in a 3-
Speed of rotor field with respect to rotor structure - p 4

Phase induction motor, , (w r t ) rotor = speed of stator field w.r.t.


rotor speed, n r ± speed of rotor field with respect to (w.r.t.)
*tator, nt
°r nr ± 600 = ^ ^ " r e c t i o n of stator field at a speed,
For positive sign, rotor m ust

Scanned by Cam Scanner


lArt. 6.5
714 Electrical Machinery

For negative sign, rotor must be driven against the direction of stator field at a speed,
nr = 1500 + 600 = 2100 rpm.
( b ) Rotor emf at any slip s is given by E ^ = n (sf i) ^2 ^w2
_ .. 1 5 0 0 -9 0 0
h or nr = 900 rpm, slip sx = - = 0.4
1500
.*. = ViT n (0.4 f x) N 2 <f>k w2 -ii)
_ ____ .. 1 5 0 0 -2 1 0 0 a a
For nr = 2100 rpm, slip s2 = J^qO =”
... f i 2 f / ' = V 2ic(-O .4/i)A r2 0AIB2 ...(ii)
From Eqs. (i) and (ii), we get
V2n ( - 0.4 f{) N 2 <f>k w2
= -1 .
£ 2/ V2 71 (0.4 A) N 2 0 k w2
(c) For rotor speed nr = 900 rpm clockwise (say), stator field is running at 600 rpm clockwis
with respect to rotor. Let the phase sequence of the emfs generated at slip rings be abc.
For rotor speed nr = 2100 rpm clockwise, stator field is running a t 600 rpm anticlockwise
with respect to rotor. Therefore, phase sequence of the emfs generated at slip rings is reversed,
i.e. it is now acb.
6.5. R otor e.m.f., C u rren t and Pow er
At standstill, the relative speed between rotating magnetic field and rotor conductors is
synchronous speed ns ; under this condition let the per phase generated e.m.f. in rotor circuit
be E 2. When the rotor speed is, say 0.4ns, slip is 0.6 and the relative speed between rotating field
and rotor conductors is 0.6 ns ; under this condition, the per phase generated e.m.f. becomes
E2 .
0.6 ns x — = 0.6 E 2. At a rotor speed of 0.8 n4, slip is 0.2 and the relative speed between rotating

field and rotor conductors is 0.2 ns, therefore, the per phase generated e.m.f. under this condi-
E2
tion becomes 0.2 ns x — = 0.2 E 2. In general, for any value of slip s, the per phase generated

e.m.f. in rotor conductors is equal to sE 2 = s [<2nfxkw2 N 2 (j)] = V2n (s/i) k w2 N 2 <J>.


Now rotor leakage reactance = 2 n (Rotor frequency) (Rotor leakage inductance l2)
Rotor leakage reactance at standstill = 2n/1/2 = x2 fl.
Rotor leakage reactance at any slip s = 2nf2 l2 = 2 k (sfx ) l 2 = s;c2 Q.
Rotor leakage impedance at standstill = Vr2 + x2.
At any slip s, rotor leakage impedance = Vr2 + (s*2)2 .

phasPe ^ T u r gren„etrated diVided * phaSe r °t0 r leaka8e ™ pedance gives the per

Per phase rotor current at standstill = -r i f 2


• + 4'
Per phase rotor current at any slip s is given by

I - _ E2
^2 + (sx2f l ( r 2/ s f + xl ’

Scanned by Cam Scanner


Polyphase In d u ction M o to rs

current / 2 lags the rotor voltage E 2 by rotor power factor


fhe r°tor °
_ l sx2
02 = tan

a\ can
//v Q\ an be
DC represented
i c p cocuvvru by ^a simple --------
series~ circuit as shown in -_ ------
Eq-I. '(0.0/
f ;. n rfrom
____i.U'Ir.
this circuit fVtof
that nper
o r r»Viaco
phase npower
n w o r iinput
n n n f t.n
to rrotor,
n tn r . Fig. 7 Rotor equivalent-
6.7.
S<r. 6 - 7 *l _ ▼ n circuit of an induction
Fig P = Ea/a cos 02. motor.
Per phase rotor resistance
But, cos 2 _ per phase rotor impedance
r2/ s
V(r2/ s f + (x2)
E2
r r2 T2 r 2 ...(6.9)
K' ■lo — - 1 2
+ (*2) S S
An examination of Fig. 6.7 also reveals that per phase power input to rotor is equal to

as the reactance x2 consumes no power.


p is the power transferred from stator to rotor across the air gap. In view of this, Pg is
called the air-gap power. The expression for Pg may be written as
1—
Pe =l\ — = l\ r2 + 2
o ( S J
= (Rotor ohmic loss) + Internal mechanical power developed in rotor ( P J
- « P f + (l - S ) P S - (6'1 0 a )
„ t2 ( l - s \ ...(6.10 b)

r s x ...(6.10 c)
Rotor ohmic loss =
1 - s Em~ s Pg
Eq. (6.9) reveals that rotor ohmic loss
= l\r2 = sPg
...(6.11)
= s (power input to rotor)
Internal (or gross) torque developed per phase is given by
Internal mechanical power developed in rotor
e“ Rotor speed in mechanical rad. per sec.
P m ( l - s ) P g _P l ...(6.12)
T= m
CO,. (1 - S) C0S ®s

Here u. ( - 2 m ,) = ^ is the synchronous speed in mechanical radians per second.

Also electromagnetic torque Te is


^ i l\r2 Rotor ohmic loss
” co ~ o)s s (w„) Slip
...(6.13)
~Z 1 1j2
0 rA
271 nc s

Scanned by Cam Scanner


716 E le c tric a l M a ch in e ry ________________ IA rt. 6.d

The power available at the shaft can be obtained from Pg as follows :


Output or shaft power, P s/, = P m —Mechanical losses (friction, and windage losses),
or Psh = P g - Rotor ohmic loss - Friction and windage losses.
= Net mechanical power output or net power output
P,h Psh
Output or shaft torque, T *= "'<« Hi

If stator input is known, then air-gap power Pg is given by


Pg = stator power input - stator T2/? loss - stator core loss.
6.6. L o sse s and E fficien cy
The losses in a 3-phase induction motor are (i) fixed losses and (ii) variable losses.
F ix e d losses : These losses are composed of (a) core loss (6) bearing friction loss (c) brush
friction loss in wound-rotor induction motors (WRIMs) only and (d ) windage loss. These losses
are taken to be constant over the normal working range of the induction motor. Friction loss
may vary slightly with load and speed. But this loss is also considered constant as the speed
varies only by a small amount over the normal working range of the motor.
A 3-phase induction motor under its usual working conditions has very low-frequency cur­
rents in the rotor circuit. As a result, rotor core losses, almost proportional to frequency
squared, are negligible. The fixed losses or rotational losses (friction, windage and core losses)
can be obtained directly by performing no-load test on the induction motor.
Fixed losses = Power input at no load - (stator I 2R loss at no load)
V ariable lo s s e s : These losses are composed of (a) stator ohmic loss (6) rotor ohmic loss (c)
brush contact loss for WRIMs only and (d) stray load losses. The stator and rotor ohmic losses
can be calculated directly in case ac resistances of stator and rotor windings are known. Total
ohmic osses can be obtained by performing blocked rotor test on the induction motor The rotor
ohmic loss in a squirrel cage induction motor can only be obtained by subtracting stator ohmic
loss from the total ohmic loss measured during the blocked rotor test.
| The brush contact loss for WRIM = slip-ring current x 1 volt.
Stray load loss occurs in iron as well as in conductors. Their m easurem ent is very complex

bad and Pr rau y ‘aking °'5% ‘eSS ‘han itS calculated

powertaput bXtributtd b a ” -phLtMuctionmot0” T T ' •‘ indiCateS dearly h°W


power, differs from stator input power by total stator ^ m? P° Wer’ ° F air'g3P
rotor core loss is neglected. The power at the rotor ^ SS®S ,Hjlder normal running conditions,
developed by friction and windage losses. differs from the mechanical power

to7tato?Pfrdm R^ inP u * L A ir-gap power


imams P°w « r J M echanical power
d e v e lo p e d , P m

ro to r sh a ft
Psh
1- m c o re °ln « P ^'c ,l 0 ,• W in d a g e
'« • < * * .» “ & * „ £ , loss
at small sl.ps) and sTlp rm g sf.f a n y )

Fig. 6.8 Power-flow diagram for a 3-pha.e induction motor.

Scanned by Cam Scanner


Polyphase Induction Motors 717
A *"'
efficiency of a 3-ph ase induction motor is given by
*•*- p
^ =
rp sh +. rp f +, Pp oh x 100

e Fgh = shaft power


* p f ~ fixed losses = core loss + friction and windage losses
p h = stator and rotor ohmic losses + brush contact loss (if any).
Exam ple 6*5. A 3 -p h ase, 4-pole, 1440 rpm , 50 Hz induction motor has star-connected rotor
•ding, having a resista n ce o f 0.2 Cl p er p h a se an d a standstill leakage reactance o f 1 Cl per
^ When the sta tor is en erg ised at rated voltage an d frequency, the rotor induced e.m.f. at
Itondstill is 120 V p e r p h a s e .
[a) C alcu late th e rotor current, rotor p ow er factor an d torque both at starting an d at fu ll load

and compare these results.


(b) I f an extern al resistan ce o f 1 Cl p e r p h a se is inserted in rotor circuit, calculate rotor
cu rren t rotor p o w er fa c to r a n d torque a t the time o f starting.

„ , , 120 x 50 , _
Solution. Synchronous speed = ----- = 1500 r.p.m.

.. r E2 120
Rotor current at starting, l 2st - ^ + x'l ~ ^ Q2f +

120 = 117.67 A
1.0198
• ^2 0-2 0 1 Qfi
Rotor power factor a t startin g = = yJ(0 .2 f + (l)J ~

Synchronous speed in radians per second,


_ 2ft _ 2 n (1500) _ n ra(jySec.
“ s - 60 " 60
_ 3 _ lT \2 _r l
Starting torque, T e st - a I1**) i

= J L n 17.67)2 (0.2) = 52.9 Nm.


50n
Nameplate speed of 1440 r.p.m. is the full-load (or rated) speed of the motor.
1500 - 1440 = 0 Q4
Full load slip, s“ 1500
sE 2 (0 04) (120)
Rotor current at full load, l 2fi ~ \[f^+ (sx2)£ >T(0.2)^ + (0.04 x 1)

_ J A - = 23.53 A.
" 0.204
r2 = A A - = 0.98.
Rotor power factor at full load - ^r 2 2 + (SX^ 0.204

1 2 ^"2
Full-load torque, Tef j - w
o), ^ s
=^
J - 3 (2 3 .53)2 H = ^87N m .
50n

Scanned by Cam Scanner


\
i

718 E le c tric a l M a ch in e ry [A rt^ U

Ratio of starting to full-load current


117.67
23.53 " 5 0° ‘
Ratio of starting to full-load torque
52JI0 i oo.
52.87
(b ) When external resistance is inserted in the rotor circuit, the total rotor circuit resistance
becomes 0.2 + 1 = 1.2 ft per phase.
. t - 1^0 _ 120 _ go A
" 2,1 V(1.2)2 + (l) 1 562 '

Rotor p.f. at starting " ^( 1.2 ) + (1)* " 1 ^ 2 * 0 '?68

T'« = 5 t a 3 <76'82)2 ¥ = 135 25 Nm-


This example demonstrates that with the insertion of external resistance in rotor circuit,
/ 2 s7 has decreased, Test has increased and rotor power factor at startin g has improved. These
are the main advantages of wound-rotor induction motor over squirrel-cage induction motor.
E x a m p le 6.6. A 3-phase, 6-pole, 50 Hz induction m otor h a s a fu ll-lo a d sp eed o f 960 r.p.m.
with its slip rings short-circuited. The m otor drives a constant-torque load.
I f the rotor speed is reduced to (a) 800 r.p.m. an d (b) 400 r.p.m . by in serting external resis­
tance in the rotor circuit, com pare the rotor ohm ic losses at these two red u ced speeds with that
at full load.
Solution. Constant torque load requires constant electromagnetic torque Te. Eq. (6.12)
shows that for constant torque Te, air-gap power Pg remains constant because synchronous
speed (dependent on poles and frequency) is constant. Eq. (6.11) shows th a t for constant P ,
rotor ohmic losses are proportional to slip, i.e. *’
Rotor ohmic loss « slip.

Now slip at full load = — 0 ~ 960 = 0 04


1000 '
Slip at 800 r.p.m. = ^ - QQ~ 800 = o 20
K 1000
Slip at 400 r.p.m. = 1 0 0 0 - 40°- = 0 60
F 1000 UbU
•• at 800 r.p.m. = Slip at 800 r.p .m. _ 0.2
I 2 r2 at full load Full-load slip 0.04 = 5.
Rotor ohmic loss at 400 r.p.m. 0.6
Rotor ohmic loss at full load = 0 0 4 = 15>
Exam ple 6.7. A 10 kW, 400 V, 3-phase 4-pole 50 H , , • ■
running at no loa d with a lin e current o f 8 A con n ected induction motor is
line current is 18 A an d the input,pow er is i i M M °f 6 6 ° ^ At * * ^ ^
ft an d friction, w indage loss is 420 w atts Fnr 1 : f >tator effectiv e resista n ce p e r p h a se is 1.2
(a) stator core loss ; n eg h g ^ U rotor o W /osses calculate,
(b) total rotor losses at fu ll lo a d ;
(c) total rotor ohm ic losses at fu ll lo a d •
(d) fu ll lo a d speed ;
Ie) internal torque, shaft torque a n d m otor efficiency.

Scanned by Cam Scanner


Polyphase Induction Motors 719

. (a) At no load, total power input is equal to the sum of stator core loss, friction
S°lutl° nirtCc stator no-load I 2R loss and negligible rotor core loss,
j>yinda£e ’ .
** Stator core loss
■' - power input at no-load - Friction and windage loss - Stator I R loss at no load
f 8 \z
= 660 - 420 - 3 (1.2) = 163.2 W.
^3
,MAir-gaP power atfu llload ,
p = Stator input at full load - Stator core loss - Stator full load ohmic loss
9
f ia Y
= 11200 - 163.2 - 3 H g (1 2 ) = 10,648 watts.

. Total rotor loss = Rotor input power, Pg - Shaft power


= 10,648 - 10,000 = 648 watts.
(c) Total rotor loss consists of rotor ohmic loss and friction, windage loss.

• Rotor ohmic loss = 3 /2 r2 —648 —420 = 228 watts.

id) Now, 31\ - j = Pg

or 3o \(l\ i 2) -=< sPg


L 2 r2) >sg

or (228) = Sfl Pg —Sfl (10,648)


228
.-. Full-load slip, sfl = 64g = 0.0212

Full-load speed of rotor,


Nr = N , (1 - s) = 1500 (1 - 0.0212) = 1468.2 r.p.m.

(e) Internal torque, Te = ^ = 67.79 Nm.

Shaft torque, T,„ = ^ £ * _ s) = ^ = 65 04 Nm’

EfBd“ * = i w t a S S t = n j o o = 0 892 or 8 92% -


Example 6.8. An induction motor has an efficiency o f 0.9 when the shaft load is 45 kW. At
this load, stator ohm ic loss and rotor ohmic loss each is equal to the iron loss. The m echanical
h>ss is one-third o f the no-load losses. Neglect ohmic losses at no load. Calculate the slip.
Solution. Total losses for an output of 45 kW are

x 45000 = 5000 W
0.9
„ 2 * *
Total losses = stator P R loss + stator core loss + rotor I R loss + mechanical loss ...(i)
At no load, the losses include mechanical (friction and windage) loss, stator core loss and a
8,nall amount of T2/? loss in stator and rotor. As I2R losses are neglected at no load, we have,
no-load losses = stator core loss + mechanical loss ...(ii)
mech i8given that mechanical loss is one-third of no load loss or no-load loss is 3 times the
^ical loss. Therefore, from (ii), we have,
3 (mechanical loss) = stator core loss + mechanical loss

Scanned by Cam Scanner


720 Electrical Machinery
(Art 6.7

or mechanical loss = ± (stator core loss)

Let stator I 2R loss (= rotor I 2R loss = stator core loss) be A. Then, from (i),

Total losses - A + A + A-h —


IA
or 5000 = y
Rotor ohmic loss = A = 1428.57 W
1428 57
Air-gap power, Pg - 45000 + 1428.57 +•------- — = 47142.86 W
2
Rotor I2R loss 1428.57
Slip = 0.0303.
47142.86

6.7. Induction M otor P h asor Diagram


Fig. 6.3 (c) shows rotor m.m.f. F 2 lagging behind air-gap flux by a space angle of 90 + 02. The
rotor m.m.f. reacts on the stator and calls for a compensating load component of stator current
12 such that
load component of stator m.m.f. = rotor m.m.f. F 2
or
No No2 kk w 2
12 = fo X TFT
x r{/ =
N - * 22 x N
» rl-ki rwl
^ - (615)

Here AY and N 2 are the effective number of stator and rotor turns respectively. In Fig. 6.9,
load component of m.m.f. = I 2 N {, of the total stator m.m.f.
Fi, is shown opposing rotor m.m.f. F 2. Similarly, load-com­
ponent current I 2 , of the total stator current I lt is shown op­
posite to rotor current / 2. In Fig. 6.9, per-phase rotor induced
e.m.f. E 2 lags 4>by a time-phase angle of 90° as shown in Fig.
6.3 (c).
If hysterisis is neglected, then air-gap flux is in phase
with the resultant air-gap m.m.f. Fr. As in a transformer, the
no-load magnetizing m.m.f. of the motor does not differ from
resultant air-gap m.m.f. F r. It is because of this reason that
motor magnetizing current Im is shown in phase with F r in Fig.
6.9. The phasor sum of F ,' and F r gives the total stator m.m.f.
F v Similarly the phasor sum of stator-load component of cur- ^ g g Pertaining t0 the induction
rent In and motor magnetizing current I mgives the total stator motor phasor diagram (bare over
\ 11 E\ etc not 9hown)-
cu rre n t/, as illustrated in Fig. 6.9. The stator (or the primary) 11 '
induced e.m.f. F , and rotor (or secondary) induced e.m.f. E 2 are shown lagging $ by 90 as i
transformer. c m (a) where
Complete induction motor phasor diagram at standstill is drawn in Fig 6 1 “ >“'•
m.m.fs. are not shown for the sake of clarity. At standstill, E 2 is shown equal to h J
core-loss component of stator current, i.e.1, is in phase w
and windage loss is zero. The stator no-load current is 7„ = 7,„ + 7„ and the stator loa
7, = 7', + 70. The stator applied voltage V, must balance the stator counter ean.l.
and the stator leakage impedance drop 7, (r, + > ,) as shown in Fig. 6.10 (a). The p

Scanned by Cam Scanner


Polyphase Induction M otors 721

(o) (b)
Fig 6.10. Induction motor phasor diagram at (a) standstill and (b ) at a full-load slip s.

angle 0, (between and /,) at the stator terminals is very high, i.e. stator power factor is very
poor at the time of starting a 3-phase induction motor.
At normal operating speed, slip s is small. The rotor voltage equation now becomes,
72(r2
s £2= +;sx2) and this is illustrated in the phasor diagram of Fig. 6.10 (6 ). In this figure,
?0=7
m+ 7/c, where Ifc is the friction, windage and core-loss component of stator current. The rest
ofthe phasor diagram is drawn in the same manner as illustrated in Fig 6.10 (a). Fig. 6.10 (b)
reveals that full load power factor at the stator terminals has considerably improved (0.8 to 0.9
lagging) from its power factor at starting.
In the phasor diagrams of Figs. 6.9 and 6.10, all quantities have per-phase values.
6.8. Equivalent C irc u it
An induction motor has been shown equivalent to a transformer, naturally induction motor
equivalent circuit should be similar to the transformer equivalent circuit The cnty differ
is on account of the fact that the secondary winding (i.e. rotor winding) of an inaction motor
rotates and, therefore, involves the development of mechanical power. e fnrmpr All
tion motor equivalent circuit proceeds in the same manner as in the case of a transformer. All
the equivalent circuit parameters have per phase values.
We shall develop here first the circuit model for stator and then Ior|the rotor Thesetaro
circuit models are then combined to obtain the equivalent circuit of a 3-phase induction motor,
stator-circuit model. When a voltage V, is applied to the stator terminals of a 3-phase
“ duction motor, a rotating air-gap flux 4 is established. ^ “ rmfnaTvoltage V, has to over-
aU the 3 phases of stator (or the primary) winding. Th _
»me counter e m f V , ( - - E ,)and the stator leakage impedance drop Oh +/*!
« like a transformer, the stator or primary voltage is given by,
_ - -7 , • h ...(6.16)
V ^ V S + h iri+ JX i)
As in the case of a transformer, the stator cu rrent/, cons.stsof twocomponen^Onecom-
is the load component and counteract the reto m a m f / ^
(6-15>- The other component is the exciting current Z„ whose lunction
'

SH I I V I l z s E n i h i
Scanned by Cam Scanner
722 Electrical Machinery fA rt 6.8

air-gap flux <J>and to provide the core loss. As in a transform er, the exciting current Ie can be
resolved into two components, core-loss component I c in phase with Vy and a magnetizing com­
ponent I m lagging Vy by 90°. In a transformer, the exciting current is also called the no-load
c u rre n t; but in a 3-phase induction motor, it is not
so. It is because the no-load current I0 in a 3-phase
induction motor, as shown in Fig. 6.10, has to ere-
ate resultant air-gap flux (j) and to supply the no H
load losses (= core loss + friction, windage loss + small
stator and rotor I 2R loss), whereas function of It in
a transformer is to create <f>and to provide only the
core loss.
In a 3-phase induction motor, product V { l c )------------ / F re q u e n c y ,^ -------------------------------------------
gives core loss (= norload loss - friction, windage
loss - negligible stator and rotor 12R loss at no Fig. 6.11. S ta to r equ ivalen t circu it for a
polyphase induction motor.
load). In the equivalent circuit, Ic and I m are ac­
counted for by a shunt branch, consisting of core-
vA V i ) as illustrated in Fig.
loss resistance /?„ in parallel with magnetizing reactance

6.11. Once Rc and Xm are calculated under normal operating conditions, these are then treated
constant, even though the counter e.m.f. Vy or E l (across Rc and Xm) may depart slightly over
the normal operating region. Param eters Rc and Xm simulate respectively the core loss (= no­
load loss-friction and windage loss-small I2R loss at no load) and air-gap flux as in a trans­
former. .
R o to r c irc u it m odel. The per phase rotor current is given by Eq. (6.8). It is repeated here
for convenience in phasor form as
sE <j
/,= ...(6.17)
r2 + jsx2
Dividing the denominator and numerator by s gives
E,
Io = ...(6.18)
'+ J*2

Note that the magnitude and phase o f/2 obtained from the Eqs. (6.17) and (6.18) are the same.
There is, however, a significant difference between these two equations. In Eq. (6.17), slip-frequen-
cy generated e.m.f. sE 2, when divided by r2 + jsx 2 gives slip-frequency current, but in Eq. (6.18)

line-frequency e.m.f. E 2 when divided by — + jx 2 gives line-frequency current. Equivalent circuits


of Fig. 6.12 (a) and (b) are obtained from Eqs. (6.17) and (6.18) respectively.
In Fig. 6.12 (a), the rotor circuit consists of (i) a constant resistance r2 (ii) a variable leakage
reactance sx2 (iii) slip frequency emf s E 2 and (iv) rotor current I 2 at slip frequency sfv This
interpretation is in line with the actual physical facts existing in the rotor circuit of a 3-phase
induction motor.
In Fig. 6.12 (b), the rotor circuit consists of (i) a constant leakage reactance x2 (ii) a v a r ia b le
resistance r2/ s (iii) line-frequency emf E 2 and (iv) rotor current / 2 at line frequency. This inter-

Scanned by Cam Scanner


Polyphase Induction M o to rs 723

J*2

Ea

b' /Frequency, f

(b)

1*1
— •— -qip—
■Jtw r- r -aw .—
*rr h X 1U h
L
ijX m J A
r
'i V €, 1 :*c
EfS'Ei H 1c Tim

M ------------
. u - £ ---------
/Frequency, f
Trequency.f
(d)

PS f ° n differentlp'eratingconditions.

Fie6.12 (6) differs from the magnitude of E , or v , oy


Therefore, rotor voltage E 2 must be referred to stator or primary. .

Rotor voltage when referred to stator

- N ’ - E]

f r2 • ' (N ^
— +J*2 S
I5 )K J
md *2
/, xre = / 2'a s in Eq. (6.15).
I 1 J • i a nf Fi<r. 6.12 (c) is of the line frequency and has
The voltage appearing across term in s a ^ These terminals can, therefore, be
e same magnitude as at the terminals ab oi r g. - ^ of an induction motor as shown in
nnected together to give the complete equivaie ^ t0 stator have been pnmed^A
8 6 12 W). Note that in this figure, rotor redrawn in Fig. 6.13 fo) where, for
« general equivalent circuit of induction motor has bee
tnplicity j j

(») the complex notation (bar over I j ^ the rotor quantities have been referred
®ut one must keep in mind that in Fig- 1q /a \are at stator frequency.
st*tor side. Note that all the quantities in Fig. 6 . « I '

Scanned by Cam Scanner


724 E le c tric a l M ach in ery

'1 X1
< \*A\— ''DOTfV
-1+

(a) (6)
Pig. 6.13. (a) Equivnlcnt circuit for n polyphase induction motor
(/>) modified form of induction motor equivalent circuit.

Another form of equivalent circuit is drawn in Fig. 6.13 (6), where the resistance — has

been split into r2 and r2 I 1 - 5)Li.e.



\ I s ;
”2 (1
— = r2 + r2
s *
This equivalent circuit emphasises the similarity between transformer and induction motor
equivalent circuits. For example, at standstill s = 1, the circuit of Fig. 6.13 (6) becomes the
equivalent circuit of a short-circuited two-winding transformer. In case s = 0 (at synchronous
speed), the circuit becomes the equivalent circuit of an open-circuited transformer.
In Fig. 6.13 (6), r2 is the actual rotor resistance (referred to stator), whereas the variable
resistance r2 ^ - jis the electrical analogue of the variable mechanical load.

Use of th e eq u ivalen t c irc u it. At the time of starting, large currents flow in stator and
rotor windings. These heavy inrush currents cause magnetic saturation and as a result of it,
leakage reactances are less than their values at full-load operating conditions The rotor fre­
quency varies from line frequency at the time of starting to a low value of slip frequency at
lull-Ioad speed. Therefore, the distribution of current in the rotor conductors is different at high
and low rotor frequencies, consequently the rotor resistance varies significantly over this speed
range. In view of this the equivalent circuit parameters should be determined at those values
of rotor frequency and current, at which the performance calculations are to be carried out by
using the equivalent circuit.
) Note that Rc in the induction motor equivalent circuit does not represent the no-load loss,
it represents only the core loss of the induction motor.
6.9. A nalysis of th e E q u iv alen t C ircu it
frnrrfiL^n imp° rt.a nt Perf° r mance characteristics of an induction motor can be determined
from its equivalent circuit. Expressions for the air-gap power Pg, internal mechanical power
m developed in rotor have already been obtained in Art. 6.4

tin n l™ USl ° f thC 0XKCt of Pig- 6.13 (6) is quite laborious. Some simplifica-
turns can however, be made in this equivalent circuit. In transformers, an analysis of the
equ,valent circuit is carried out either by neglecting the shunt branch consisting of Rr and .Y„,
in parallel, or by moving the shunt branch across the primary terminals. This simplification is
however not permissible in case of induction motor equivalent circuits. This is due to the fact
that the exciting current in transformers varies from 2 to 6% of full-load current and per unit
primary leakage reactances are small ; but in case of induction motors, the exciting current
vanes from 30 to 50 per cent of full-load current and per unit stator leakage reactances are

Scanned by Cam Scanner


Polyphase Induction M otors 725

^ r a t i v c l y higher. On account of this, large errors M *1 Q


— vw— —t * -TJtJff'-----
c”-o ^
V .^ d u c e d in
‘•ntrociu*;c“ *** th
—e d eterm
........ in —
.......... atio n of power
power and
and
ai"e 1 in case the shunt branch is neglected, or con-
t°rqUj ’across the stator term inals in a 3-phase induc-
nected V, zf
tion mo^o r ' ,
Under normal operating conditions of constant volt-
and frequency, core loss in induction motors is
constant. In view of this fact, core-loss resis­
L V
Fig. 6.14. Induction motor equivalent
tance^ representing the motor core loss, can be omitted circuit without core loss.
t^e equivalent circuit of Fig. 6.13 (6). But, for
determining the shaft power or shaft torque, the constant core loss must be taken into con-
deration, along with the friction, windage and stray load losses. With this simplification,
equivalent circuit of Fig. 6.14 is obtained with negligible loss of accuracy.
The equivalent circuit of Fig. 6.13 (ft) is redrawn in Fig. 6.15 where the various losses in a
3.phase induction motor are indicated. The shaft power is equal to mechanical power developed
in (r2 (1 - s) / s1 minus the mechanical loss (friction and windage losses). In Fig. 6.14, Rc is ig­
nored, but loss represented by Rct i.e. core loss, must be accounted for along with friction and
windage losses while utilizing this equivalent circuit for computation of machine performance.

Rotor M echanical
Statori Stator S t o t o r core ^
U R loss power
I*R Loss loss developed in
Input i
power1 Rotor r ot or , Pm
input
power
Fig 6 15 Power distribution in a 3-phase induction motor.

Note that all the e n t i t i e s used in the e q u i v a l e n t ^ are per phase ,u a n « « e s. ^


Steady-state perform ance Pa ra ™ ter ivalent circuit of Fig. 6.14. In this figure, Z,v s
the^p^r-phase hnpedancl offered to the stator by the rotating air-gap field. Note that Z^includes

the reflected effect of rotor leakage impedance 1 + j*2 and the magnetizing reactance Xm. From

Fig. 6.14, / N
r2 ■

C
+JX 2
1 1 — -i— --------- --

Zf = R f + jX f=
— + j(X2+ Xm)

Impedance Z , as seen by the stator applied voltage V, .s


Z , = r , + j * i + Z!

Stator current,

Ira n
Scanned by Cam Scanner
726 E le c tric a l M a c h i n e r y (Art ^ o

The total power lost in the parallel combination of - j + jx 2 and jX m is — . Since Zf = R ^ j^

the power lost in Zf is l\ Rf and it must be equal to l\ —


s
•\ Per-phase air-gap power,

o V -(6.19)'
Once air-gap power is known, shaft output, torque, efficiency etc. can be determined by
referring to Art. 6.5.
E xam p le 6.9. A 3-phase, star-connected, 400 volts, 50 Hz, 4-pole induction motor has the
following p er p h a se constants in ohm s referred to s ta to r :
rj = 0.15, x2 = 0.45, r2 = 0.12, x2 = 0.45, Xm = 28.5.
Fixed losses (core, friction an d windage losses) = 400 watts.
Compute the stator current, rotor speed, output torque a n d efficiency when the motor is
operated at rated voltage an d frequency a t a slip o f 4 percent.
Solution. From the equivalent circuit of the induction motor as shown in Fig. 6 16 the
impedance offered to the voltage source is ’
„ 015 045
^"2 .
7 + 7 *2
(JX ) •+9" ---- 'titi
z ab ~ r 1+ j x i + J.
7 + j (x2 + X J
= r i + j * i + R f+ jX f
f0 1 2 .
0 ^ 4 + J 0 45 ° 28-5)
= 0.15 + j 0.45 + ^-------7 ~ 9 o L Fig' 6 1 6 E d u ctio n motor equ.valent
- m n a* 00^ 5 circu it, Exam ple 6.9
= 0.15 + j 0.45 + 2.87 + j 0.738
= 3.02 -t-j 1.188 = 3.25 / 21.470.
From above, Rf = 2.87 Q

Stator current, =
I x -------400 - 71 o c / 01 n o a

^3 (3.25) /2 1 .4 7 ° “ 71-06 L r 21.47°


Power factor - cos 21.47° = 0.93 lag
■ = V3 <400 ) <7 1 0 6 )(°-9 3 ) = 45.784 kW.
rrom Eq. (6.19), total air-gap power,

Pg - m I xR f = 3 (71.06)2 (2.87) w atts = 43.4 7 6 kW.


Synchronous speed, ns = ^ = —x 50 = 25 rps

Rotor speed, " r = (1 - s) n, = 0.96 x 25 = 24 rps, or 1440 rpm


—2tc (25) —50 7i rad/sec
Mechanical power developed,
Pm = (1 - s) P g * (0.96) (43.476) = 4 1 .737 kw
Power output, Psh = P ^ - fixed losses = 41.737 - 0.40 = 41.337 kW

Output torque - - sh 41337 41337


<*V (1 - s) co, _ 0.96 x 50tc = 274 12 Nm -

Scanned by Cam Scanner


^ 1
P oly p h ase In d u ctio n M o to rs 727

various losses in the induction motor are as under :

Stator ohmic losses = 3 i] r l= 3 (71.06)2 (0.15) = 2272.3 W


Rotor ohmic loss = sPg = (0.04) (43476) = 1739 W
Fixed losses = 400 W
Total losses = 4411.3 W
p ow er output, P sh = 41.337 kW
Power input = P sh + total losses = 41.337 + 4.411 = 45.748 kW
. , losses , 4.411
... Effic.ency . 1- _ . i - _ _ = 0.9036 or 90.36%.

6.9.1. T o rq u e-slip c h a r a c te r is tic s . When torque-slip or power-slip characteristics are


required, application of Thevenin’s theorem to the induction motor equivalent circuit of Fig.
6.14 reduces the computation labour considerably.
For applying Thevenin’s theorem to Fig. 6.14, consider two points a, b as shown. From
these two points view towards the voltage source Vj. As per Thevenin’s theorem, the circuit
consisting of r v x lt X m and source voltage Vx can be replaced by an equivalent voltage source
V, and an equivalent impedance Ze = R e + jX e, as shown in Fig. 6.17, where

Vx {jXm)
<3

...(6.20)
n

r l + j { x l + X n)
...(6.21)
IN

and
ii

rl + j ( x l + Xm)

Re Xe a Re Xe ■x2 ri
+jo—- w
™—qfifr- e - W - -AWr- -AW—I t ---
T
ve -{

i _________ i
(a) . ^
Fig. 6.17. Thevenin’s equivalent circuits of an inductiou motor.

Here Ve is the voltage appearing across terminals a, b with the rotor circuit disconnected
from these two points. Equivalent stator impedance Zt is the impedance viewed from terminals
a, b towards the voltage source and with the source voltage short circuited.
For most induction motors, ( i , + X J is much greater than r,. In view of this, r , can be
neglected in the denominator of both the Eqs. (6.20) and (6.21), without causing any noticeable
error. -
v
■xr
VJCm tr v
V,Xm ...(6.22)
Thus, xx + Xt
Xi
r\Xn jx i X„
and Z<= R<+j X'~ Xi + Xm + Xy+X„
r 1X,n. *1 X„ ...(6.23)
+7
XT *1
X x = Xj + X m, is the stator self-reactance per phase.
Here

Scanned by Cam Scanner


IAn. 6.9
728 Electrical M achinery

. , 4 • > of ritr fi 17 (a) rotor current is given by


From the equivalent circuit of big. o.W W , r

•••(6.24 al
/z = (I r2'
Re + - + j (*2 + %e)
s j
and from Eq. (6.13), the total to. que is
V;i -------- — Nm ...(6.24 6)
(0, ' r2 ^
Re + —
e s
Hence m is the number of stator phases.
For convenience, Eq. (6.24 h) is re-written as,
Kt______ ■— Nm ...(6.25)
s
' r2^
Re + - ■X2
c s
m Vl m nV1,
where K( =
cos 2nns
and X = x2 + X e.
The variation of torque with slip, or speed, of an induction motor can be plotted from Eq. (6.24) for
different values of slips and with the motor connected to constant-frequency voltage source. A general
shape of the torque-speed or torque-slip curve is shown in Fig. 6.18. Depending upon the value of slip
an induction motor can have the following operating regions or modes :
(a) Motoring Mode : 0 < s < 1.
Under normal operation, rotor revolves in the direction of rotating field produced by the
stator currents. As such, the slip varies from 1 at standstill to zero a t synchronous speed, i.e.
1 < s > 0. The corresponding speed values are zero (s = 1.0) and synchronous speed (s = 0.0).

----------------• - S P E E D

2 0 1-5 10 05 0 -0 -5 -1 0
j L1P -------------------------- 1
Fig. 6.18. Torquo-slip curve for nn induction m nclnnc showing
its braking, motor and gen erator regions.

Scanned by Cam Scanner


P olyph ase In d u ction M o to rs 729

kiGcneratm g m o d e :s < 0
.g 0perating mode, slip ts negative, i.e. s < 0. An induction motor will operate in this
for ^hen its stator term inals are connected to constant-frequency voltage source and
gofl °°y j nVCn above synchronous speed by a prime mover. The connection of stator ter-
' > r 15 '■oltage source is e sse n tia l in order to establish the rotating air-gap field at
s 8peed. In case stator is disconnected from voltage source and rotor is driven above
,yndir°n° US by the prime mover, no generating action would take place.
mode ; * > I ­
f h mode, slip is greater than 1. A slip more than one can be obtained by driving the
^°r 1 h*8 Drime mover, opposite to the direction of rotating field But such an use in practice
jotor. * 7 * a C.tlcai utility of slip more than 1 is obtained by bringing the rotor to a quick stop
|Sr*re .tlf,n called plugging F o r o b ta in in g s > 1. or for obtaining plugging, any two stator
fcvbraking ’ . Wlth thls the phase sequence is reversed and, therefore, the direction
are in_ _ etic field becomes suddenly opposite to that of the rotor rotation The- l*le^tr0'
of rotating nQW artin|, opposite to rotor rotation, produces the braking action. Thus the
l"aenl " C be quickly brought to rest by plugging, but the stator must be disconnected from the
" “'“(.before the rotor can start tn the other direction
t i l e three regions of operation .braking region, s = 2.0 to r = 1.0 ; motor region, a - 1-0 to
A . . O r a t o r region, s = 0 to a = - 11 arc illustrated in Fig. 6 18.
, =0an g' ■ l to ru u e This is also referred to as stalling to
Maximum ' " “ rn ^ ‘ 3 , etjon for maxjmum internal torque can be obtained by using the
s fe r theorem of circuit theory As per Eq (6 13). torque is maximum when

7 > l maximum.. e when power delivered to r ,/a ,s maximum. Applying this theorem to Fig.
* . , . 1 , . W1-; maximum when impedance r2/s becomes equa to i
617 lot, power absorbed by r . , . ^ B„urce V,. when
.agmtude of impedance seen by rt s tow arc
m
r* - vk ; ^ x: r =
*«r

^ T h u s the slip » „ r »< "'•'x,nUJm ,0 r'1Ut' UCC“ rS “ K,VCn ^ „ 26)

‘ ra F n ( 6 2 5 ) g iv e s m a x im u m to rq u e Trm a s
Substitution of r , s„,r ■" " f '* ‘" J * ' *
K, I K * x ‘ l _ _
T,m “ i/t + v7.r^ A r iJ - x ‘

K, w L + x\
= (6 2 7 ,

_ _ _ _ _ _ ’

T’«■ = 2 IRe + _ irs is directly proportional to


. v at which maximum t o r q u e ^ independent of r2.
Eq. (6.26) shows that the s * a^ icates that maXamUm Q
the rotor resistance r 2, but Eq- ( •

Scanned by Cam Scanner


730 E le c tric a l M a ch in e ry [ 4 rt ^

This means that if r2 is increased by inserting external resistance in the rotor circuit of
wound-rotor induction motor, the magnitude of maximum internal torque is unaffected but the
slip at which it occurs is affected proportionally.
Now K t (oc Vf) is proportional to the square of stator voltage Vlt R e is proportional to stator
resistance rx and X is proportional to a n d x 2. In view of these facts, an examination of Eq
(6.27) shows that
(i ) Tem is directly proportional to the square of the stator voltage
(“ ) Tem is reduced by an increase in stator resistance r x {i.e. R e) and
(*“ ) Tem is reduced by an increase in stator leakage reactance x 1 and rotor leakage reactance x2
For obtaining higher value of maximum torque, the air-gap is kept as small as is possible
A small air gap allows more flux to be mutual between stator and rotor windings. As a conse­
quence, leakage fluxes and therefore leakage reactances are reduced and the magnitude of
maximum torque becomes more. •
Typical torque-slip curves for an induction motor with variable rotor-circuit resistances arP
shown m Fig. 6.19 (a). For the load torque curve TL as shown, the speed is n x for a rotor-circuit
resistance r2, speed is reduced to n2 for r2 > r2 etc. For rotor-circuit resistance r2/", Fig. 6.19 (a)
reveals that it is not possible to obtain maximum torque during motoring mode of the three-
phase induction machine. This value of rotor resistance can, however, give maximum torque
only during braking mode of the induction machine. From the torque-slip curves of Fig 6 19 (a)
it can be concluded for a wound-rotor induction motor th at *
torque rernaiifs^unaffected3” ^ COntrobec*by vary*n£ the rotor-circuit resistance but maximum

(ii) the starting torque can be varied by changing the rotor-circuit resistance
fi , q ‘A\ u6 l CUrLent ^ ken by the motor sha11 als° vary with rotor-circuit resistance Fie
slip andOf rotor-circuit resistance on the stator input current as a function of

(iv) the power factor at starting is also affected by rotor-circuit resistance


In order to get better performance of the induction motor starting tnrm il • ■ j u

T el f
Testa ^Mox. Torque 3

' n .'
" L2 SC c
u
u
D
'e *t2
u
L.

^ X \ ' s
o
v'Z > ri 'y r < <s>
L — \ p -

10 ■SLIP-
00 r Slip
SPEED n,
F ig y $ 4 9 . (a) Induction m otor torque-slip curves with
0 ns/2 Hg
d ifferent values o f rotor-circuit resistan ces. ig. 6.19. (6) Effect of rotor resistance on stator current
versus slip characteristics of a 3 -phase induction motor.

Scanned by Cam Scanner


Polyphase induction M otors 731
6 . 9 ] -------------------- .. ---------------------------------
Torque. At starting, slip s = 1.00. From Eq. (6.25), starting torque Tt lt is given
817

by Kt ng.
Te st ~
(R, + r tf+ tf'2 '"(6'28>
se-
o r q u e Test
r t i. n S ttorque -X e s l can be controlled by
---------------------------------------- J varying
•— rotor
& » circuit
vw * w x x -u n / resistance
i c o i o v a i « , c aas illustrated
o u ii1
iu o u h .w .j U
i a
^ _ order
jn dor to get
pet maximum starting torque,
toraue. the Tntnr-rirmit. reKistanr.e must be in
rotor-circuit resistance in-
^ ‘ edto y ffi+ X ? , refer to Eq. (6.26) where smT = 1.00. This new value of rotor-circuit resis-
As
cr0aS be obtained by inserting external resistance equal to (Vl?2 + X2 - r2) in the rotor
‘"S u T th e time of starting. :or
° fC totor
tor operaww“
o p eratio n n e a r synchronous
* ------------ speed.
“ When slip s is small, • r2/s
i- becomes quite * ge
^ oared to other denominator terms of Eq. (6.24 a) and consequently these can be
1 #4/ \m o m i « f o r m o T?n ( d O A n \ o n r l n n n o n n n o n t l v tllPRP. f.HTl D0 is
large as comp ^ 0yt ^ • •e,
^ored Thus from Eq. (6.24 a),
d-
sVe ...(6.29)
In —
r2 n-
Kt 2 _ sK t ...(6.30 a)
am^from Eq. (6.25), r2
rl
s ig
....(6.30 b) le
= sK ir
g
K = — is a constant
Here is
An examination of Fig. 6.19 (with rotor-circuit resistance r.L) also reveals that under normal
operating region of small slips, the torque-slip characteristic is almost a linear one.
If terminal voltage is variable, then from Eq. (6.30 a),
m■V? ...(6.31)
Te = s e . — = KV\s
co, r2

Here IC is also a constant. expression of an induction motor can also be


Motor to rq u e in te rm s of T em. lhe torque f
_ . tnrnue T and the dimensionless ratio . In order to get a
expressed in terms of maximum torque em mT
■ „r„t„r resistance r„ or the stator equivalent resistance
simple and approximate expression, st
Re, is neglected.
Division of Eq. (6.24 b) by Eq. (6.27) gives
Te 2 [ReJ^ ± £ l .T
-1
■em
Re +

Since rl or Re is neglected,
...(6.32)
Tg 2X T
S
T.em / >\
•2 -X2
S
V /

From Eq. (6.26), r2 = smTX ( v R‘ = 0)

canned by Cam Scanner


732 E le c tric a l M a ch in e ry

Substitution of this value of r2 in Eq. (6.32) gives


2X shitX
2 s
(smTx] +x2
s
or 2
s mT , s
s smT
or T0 =
Wem
SmT s - (6.33)
S SmT
6.9.2. Pow er-slip c h a ra cte ristics. The total internal mechanical power developed is

Pm = m (1 - s) P ' = m 1%r. f \ - s w aits


s
\
m V? fl- s
P... =
•••(6.34)
M * -s)
R e + r2 + ■X2

Maximum power transfer theorem is invoked again to obtain maximum value of internal
mechanical power developed. Since F m per phase is the power delivered to r ( l ~ s
, internal
mechanical power developed is maximum, refer to Fig. 6.17 (6), when

SmP) = + r 2f + X2
~ — ...(6.35 a)
or 2
SmP =
_V(/2e + r2)* + <X2 + r 2 ...(6.35 b)

In order to get maximum power P „ , substitute from E q . (6 35 a) ^ ^


r2 ( l - S ) . _ mp
in Eq. (6.34).

P mm =
[Re + r2 + V(/ee + r2) + X?]'2, + X2
or Pmm = mV2
________ ■
, 2 We + r2
* + ^(Re
v e + • rr2)'z
2/ ~ + ^x 2
J] ...(6.36)

if R e is replaced by (R e ^ E q' (6 M ) Can be obtained from the denominator of Eq. (6.27)


It is observed from Eq. (6.36) that P j ■
From Eqs. (6.26) and (6.35) it is s e e T th -T " 8 °n reS‘StanCe r 2 whe™»s T,m does not.
Eq. (6.34) shows that at starting, slip . . , ^ " 7 “ ^ ^
zero. The variation of mechanical power developed w i t h T mechanical Power developed P„ is
At starting, though the internal mechanical n o w A T * ,S ' llustrated in FiS 6.20.
ut is not zero. This is due to the fact that when slin*^ 1S zero, the starting torque
, the air-gap power transferred from

Scanned by Cam Scanner


Polyphase Induction M otors 733

rotor is l\ r<i and this all appears as ohmic loss in the rotor circuit and, therefore, the
tor «
,ical pow«r developed P m is zero. However, T ,a = ± . is not zero.
•bam1
p-e 6.20 shows th at power P m during braking region is negative. In other words, power
fig
ring plu£&tn£ *s re^urned ^he supply. As slip s is more than 1 during plugging, Eq.
h*24) shows that P m is negative. Note, however, that Te is positive during plugging or braking
(6
regi°n‘

i i I ___ _i _x — —t- — — — —i _ —
2-0 _ ~ 1-5 10 0-5 0 -0 -5 -1 -0
-• S L IP ----------------------

Fig 6.20. Power-slip curve for an induction machine


showing its braking, motor and generator regions.

In order to get maximum power output from an induction generator, reference to Fig. 6.20
shows that the rotor m ust be driven at a speed given by

nr

Example 6.10. F or the m otor o f Exam ple 6.9, determine (a)


torque occurs, the m axim um torque an d the correspon ing power o p , ^ to produce
and the torque at s ta rtin g ; (c) the external resistance to e inser e . Q^ ^ ^ j t/
maximum torque at starting ; (d) the internal pow er developed for a slip o f 0.04 and (ej t
maximum internal pow er developed a n d the correspon ing s ip.
Stator to rotor effective turns ratio is 1.2. ^ ^ Thevenin>s
Solution, (a) Reduce induction motor equiva
equivalent circuit of Fig. 6.17 (a), in which

(0 .1 5 + J2 8 .9 5 ) J 28-95
(0.15) (/28j l _ n 1476 12
Ke ~ j'2 8 .9 5

and _ (0^511/28,51 = 0 443 Q


Ae ” y '2 8 .9 5

Scanned by Cam Scanner


734 Electrical Machinery

From Fig. 6.17 (a), torque is maximum when


r2
s mT
0.12 = 0.1476 +j'0.893
or
SmT
0.12 ‘= 0.1326
or s mT -

Vi 28.5 = 400 _28Jl. _ 22? 4 v


Ve~ -l3 ' 28.95' V3 28.95
4r£ 4rt x_50 = 5Q n radysec
Synchronous speed, p 4
From Eq. (6.27), maximum internal torque is
______________ 3 (227.4j----------
(227.4)2 = 5- = 469.1 Nm
em (50 7i) 2 [0.1476 + Vo.1476 + 0.893 ]
Corresponding mechanical power developed .
= 469.1 (1 - 0.1326) (50 ti) = 63915.3 W
Output, or shaft, power = 63915.3 - 400 = 63515.3 W
63515.3
Maximum shaft torque - (50 7i) (0.8674) = 466.2 Nm
(6) At starting s = 1. From Thevenin’s equivalent circuit, the rotor current at starting is
given by
/ = 22J A —r = 243.94 A
2st Vo.2676 + 0.893
0.12
= 3 (243.94)2 —7—= 136.4 Nm
Starting torque, 50n
(c) From Fig. 6.17 (a), for maximum torque at starting, we have
r2 + R
= 0.1476 + y’0.893

or r2 + R = Vo.14762 + 0.8932 = 0.9051 O


E xtern al resistance (referred to stator) which m ust be inserted in rotor circuit,
R = 0 .9 0 5 1 - 0.12 = 0.7851Q .
. • ., 0.7851 n e^Ko n
Actual external resistance that must be inserted in the rotor circuit = ^2 - u.

(d) From Eq. (6 .34 ), the internal power developed for a slip of 0.04 is
3 x (227.4)' 0.96
x 0.12
m r- 0.04
0.12x0.96
0.1476 + 0.12 + + .8932
0.04
= 41736.36 W = 41.736 kW
(e) The slip at which internal power developed is maximum is given by Eq. (6.35) as
0.12 = 0.114
SmP~ [(0.1476 + 0.12 )2 + 0.8932]1/s + 0.12

Scanned by Cam Scanner


Polyphase Induction Motors 735

(6 36 ), m axim um intern al power developed is


' „ ________________ 3_x (227.4,2
2 [0.1476 + 0.12 + -Jo.2676a + 0.893Jf
= 64647.44 W = 64.647 kW.
ple 6 .1 1 . F o r the th ree-p h a se induction m otor o f E xam ple 6.9, fin d the maximum
^ H orq u e d ev elop ed u n d er the follow in g con ditions :
l O V Iphase, 50 H z (b) 115 V Ip h a se, 50 Hz
. H 5 V Iphase, 25 Hz (d) 23 V Ip h a se, 5 Hz.
° lution. The Thevenin’s equivalent circuit param eters at 50 Hz from Example 6.10 are .
° R e = 0 .1 4 7 6 Cl, Xe = 0.443 Cl, r2 = 0.12 Cl, x2 = 0.45 Cl
28.5
V = Vi = 0.9845 Vl
e 28.95

Synchronous speed U3r = ^ f = n f with P = 4.

FromEq. (6.27), for f = 50 Hz,


3 V?
tm 7i x 50 2 [0.1476 + >/0.14762 + 0.893z]
= 9.071 x 10" 3 V?

(а) When supply voltage is 230 V at 50 Hz,


Ve = 0.9845 x 230 = 226.44 V
.
••
T e.m = 9 071 x 10" 3 x (226.44)2 = 465.12 Nm.
(б) For Vt = 115 V at 50 Hz, Ve = 0.9845 x 115 = 113.22 V

••• T
* e.m = 9' 071 x 10" 3 x (113.22)2 = 116.28 Nm

As expected, maximum torque is —of that obtained in part (a).

(c) For f= 25 Hz, Ve = 0.9845 x 115 = 113.22 V


Y i ^
- x 28.5
2
= 0.2215 n
Re = 0.1476 Cl

*2 = 0.225 Cl
113.22 - = 396.24 Nm.
“ Tem = 7 ix 2 5 2 [0.1476 + 'T o.U ltf + 0.4465 1
W) When f = 5 Hz, Ve = 0.9845 x 23 = 22.644 V

K = ~ x 0.443 = 0.0443 Cl, x2 = ^ * 0 45 = 0 045


, _„ 644
__________________ 9.9 - 1 5 9 Q6 Nm
7\ =
*m~ n
it x 5
2 [0.1476
0.1476 + V0.1476y
^ 0 .1 4 /0 + 0.0893 ] iower
TV . x i f V / f i a kept constant, the maximum torque at lower
This example dem onstrates th at even if V / f i P
Nuencies is reduced due to the effect of winding resistance.

Scanned by Cam Scanner


736 E lectrical M achinery _ _ _ _______ ______________________________[A rtT j

Exam ple 6.12. From the equivalent circuit o f a polyphase induction motor, obtain theft,
lowing relation s:
~~7~.—2—
(a) h s t _ -\l s + s mT
U2 ~ v S2 (! +Smf)

/ \
hm T SmT
(b ) i +
I2 '2
Here I2st is the stator load-component o f current at starting and Tg-mT Is the stator load-com­
ponent o f current at maximum torque. Neglect stator resistance.
Solution, (a) From the equivalent circuit of Fig. 6.17 (a), the stator load-component of
starting current I 2st is

h “ -J(Rl + r2f + (X, + x2?


The stator load current I 2 at any slip s is
V.
/,=
2 V(72e + r2/ s f + (Xe + x2f
With negligible stator resistance r lt Re = 0 and the ratio of 7 ^ and I 2 gives

hst J l(r2/s)*+
(r2/ s f X ?
...(6.37 a)
h ~ V rl++.X 2
ro
From Eq. (6.26) with Re = 0,

r2~ SmT X
Substitution of the value of r2 (= smTX) in Eq. (6.37 a) gives

_ 'l(s „ r /s f F t g
h (SmTX f + t f

=v ...(6.37)
+ SmT)
(6) Stator load cu rren t/2 with rj = 0 is

V.
7o —

Current / 2 mr corresponding to maximum torque is given by

2" T
h-mT _ ~(r2/ s f + X 2
Io ” V'
2 ' (r2/ SmT f + ^ 2
But r2 = smTX,

hm T -v/ ( S n T / s f t f + X2
/o ■
(SmT-7SmT1)2 ^ + X 2

Scanned by Cam Scanner


Polyphase Induction Motors 737

*17
_ J 1 + (smr/sVr _ \ | l
...(6.38)
> 1+ 1 2 ig.
le 6.13. A 10 kW, 400 V, 3-phase, 4-pole, 50 Hz slipring induction motor develops
t (i e 10 at rated voltage an d frequency and with its slip rings short-circuited. se-
oU torque equal to twice the full-load torque, occurs at a slip o f 10% with zero exter- 1a
;n the rotor circuit. Stator resistance and rotational loss are neglected. Determine
I resistan
0 slip and rotor speed at full-load torque,
(ii) rotor ohmic loss at full-load torque, ^
(iii) starting torque at rated voltage and frequency, ^
(iv) starting current in terms o f full-load current, .g
(v) stator current at m axim um torque, in terms o f full-load current and e
(Vi) full-load efficiency. d-
The rotor resistance is now trebled by inserting external resistance in the rotor-circuit. For
lhe some full-load torque, find n-
(uii) the slip at maximum torque,
(viii) full-load slip and rotor speed,
(is) starting torque, S
(x) starting current in terms o f full-load current, ie
»r
(xi) rotor 12R loss at full-load torque and
(xii) developed power at full-load torque and efficiency. is
Solution, (i) Maximum torque,
Tem = 2 (full-load torque, Te fl)
Ttm = 2Tefl and smT= 0.10

From Eq. (6.33), -=£-= ------------- or^ = “ or - 0.4 s + 0.01 = 0


T em s mT t sfi 2 M + i2 _
Sp smT Sfi 0.1

Its solution gives full-load slip,


sfl = 0.0268 or 2.68%.
Rotor speed, Nr = 1500 (1 - 0.0268) = 1459.8 = 1460 r.p.m.
(ii) Since the rotational losses (i.e. fixed losses) are neglected, the mechanical power
developed is Pm = shaft power of 10,000 watts.

Air-gap power, f»s = = 10275.4 W.

Rotor ohmic loss at full-load torque


= sPs = 0.0268 x 10275.4 = 275.4 watts.

Alternatively, rotor ohmic loss


=P -P = 10 275.4-10,000 = 275.4 W.
g m ’
738 Electrical Machinery (A rt. 59

(m ) Full-load torque, Ttfl = - p - = 1<,°>2. ^ = 65.42 Nm.


M efl 2 n n s 2n (25)
•e-st
Now
He m ^m T ^ 1

1 s mT

T e .st ~
- 27g»— = 2 (2 ^ (65.42) = 25.91 Nm.
00.1
1 JL 1 10.1 10.1 v '
1 + 0.1

^ is t _ '\ j Sf l + SmT
(iv) From Eq. (6.37), T ~ ’ 0 .. 0
s fl ( 1 + s m T )
^ 2—
(0.0268) + ( 0-1/ =3g44
-4(0.0268)2 (1 + 0.01)
h st = 3.844 I2fi

(u) From Eq. (6.38),

= 2.623
*** ^2 mT - 2.623 I2fl.
Note that current I2 mT corresponding to maximum torque is less than the current I2tt at
starting.

(vi) Efficiency = x 100 = ~ ^0QQ x 100 = 97 32%


Input 10275.4
(vii) Before the external resistance is inserted in the rotor circuit, we have from Eq (6 26)
with rj = 0, M

^ ~ = yJ(Xe + x2)2 = y l x * = x

or = smT = 0.10.
When the rotor-circuit resistance is trebled, then
3 r2
~ s m T l'

.-. New value of slip at which maximum torque occurs is


smn = 3 (0.10) = 0.3 or 30%.
<«■«) With increase in rotor circuit resistance, maximum torque T,m rem ains unchanged
As full load torque remains same, Tem = 2 T ^ . From Eq. ( 6.33)
1_ 2
2 i l +M
0.3 Sl
Its solution gives full-load slip
$i = 0.0804 = 3 times the previous full-load slip (= 0.0268).

Scanned by Cam Scanner


Polyphase Induction Motors 739

j \r - 1500 (1 - 0.0804) = 1379.4 r.p.m., less than previous speed of 1460 rpm.
speed,
r , „ _ t
~T 1 0-3
em —~ + ~~z
0.3 1

r . . . = 5763 (2T" ,) = s i s (6 5 -42) = 7 2 0 9 Nm'

It is more than previous T „ , = 25.91 Nm.

,, hn= ± (0-3) - - 3 7
(,) From Eq. <6-3 7 )> / 2/I V (0.0804)2 (1 + 0.09)
/ = 3 - 7 12fi, less than previous I 2 5st = 3 844 V
\ k „ chan„ i„ the rotor circuit influences the stator through the resistance r ^ s . With
<*° A”5' d “ " f to 3 r , and with new full-load slip a, = 3s, resistance r 2/s remams constant
rj being increased “ ^ “ j e n t circuit parameters are constant and the stator ,s therefore
This implies that^afl q ^ ^ ^ means that stator current and power facto ,
unaware ol any b r
dr-gap power and torque remain unchanged so long as f ts constan .

Rotor ohmic loss at full-load torque = *,/>,


= 0.0804 x 10275.4 = 826.14 W.

(xii) Power develope^t fufl-load torque,^ ^ (10275.4) = 9449.26 W.

9449,26 x 100 = 91.96%.


Efficiency ( 10275.4 mtor^ir-
This example illustrates that the ^ *
ait is (a) to decrease the rotor speed ^ ^ ^ starting torque
ncrease the slip at which maximum q WOUnd rotor induction motor has a
Example 6.14. A 60 kW, 400 V, 3 frequency with rotor winding short
ull-load slip o f 0.04 when operating at rated t g ^ Stator resistance and rotational loss
ircuited at slip rings. The slip at m axim a , ^ full-load rotor ohmic losses.
,re neglected. Determine (a) the maximum resistance in each r o t o r phase. F o r
Rotor resistance is now doubled Zrque (d) full-load slip and <e) full-load
crated power output, determ ine (c) slip at maximum a
irque.
Solution. Synchronous speed,
2ii x 1000 _ J04.72 rad/s
03*s _ 60
ou j/
Rotor speed, to, = 104.72 (1 - 0.04) = 100 53 ra s
As rotational losses are neglected, the mechamca. power
Pm = 60,000 W
<or 7 ^ = 60,000 W
_ 6 0 0 0 0 _ 59 6 g4 Nm
Ttfl * m s 100.53

Scanned by Cam Scanner


IA rt. 6,9
740 Electrical Machinery

(а) As stator resistance is neglected,


T, 2 2 _
~ smT _£l_ + 5 2
IT «»r 0 0 4 02
where —full-load slip —0.04

.’. Maximum torque, Tem = x 596.84 = 1551.8 Nm.

(б) Full-load rotor ohmic losses

= SlPg =
/ 0 .0 4 ^
x 60,000 = 2500 W
0.96
r2
(c) It is given that = smr = 0.2

New slip at maximum torque,

s. n = = 2 x 0.2 = 0.4

(d) As stator resistance is neglected, Tem = 1551.8 Nm


T
From Eq. (6.33), ■ b= --------------
M 1551.8 s2 0.4
0 .4 s2
where s 2 and T2 are new full-load slip and torque respectively.
1 5 5 1 .8 x 2 x 0 .4 s 2 1 2 4 1 .4 4 s2
s2 + 0.16 ~ s2 + 0.16
It is given that cor T2 = 60,000 W
60000
0r l104.72
f U 7 o (nl _- es \2) •••(»)

From (i ) and (ii),


60000 1241.44 s2
104.72 (1 - s2) ~ s2 + o .l6
or s2 + 0.16 = 2.167 (s2 - s2)
or3.167 s2 ~ 2.167 s2 + 0.16 = 0
Its solution gives s2 = 0.084 and 0.6
Full-load slip, s2 = 0.084, the other value being redundant
(e) 104.72 (1 - 0.084) T2 = 60,000

Full-load torque,To = §QjQ0Q cn c c xt


4 ’ 2 104.72 x 0.916 " 625,5 Nm -

t o r o ^ Z Z Z l r « 15' , F ° r ° 3 ' ^ a s e indu ction m axim u m torqu e is twice the


T a lZ 2 t T T S 2 Z l , “ meS ^ m i0 ad ‘° r<<Ue ln o rd er to get a fu ll load slip o f 5 *
calculate the percen tage reduction m rotor circuit resistance. N eglect sta tor im pedance

Scanned by Cam Scanner


— ---------------------------- Polyphase Induction Motors 741

e lu tio n . From Eq. (6.32),

lu- 1 6 T iff - 2
Tem 2 .0 T e fi smn ^ i
smTl
S«n " ^.5 SmTl + * “ 6
°r Its solution gives s,nTl = 0.5

From Eq. (6.26), for negligible stator impedance, = s mTl = 0.5 or r2 = 0.5 x2.

For a full-load slip of 0.05,


Tefi _ l _
?em 2 SmT2 0.05
0.05 smT2
smT22 —0-20 s mj 2 + 0.0025 = 0.
or
Its solution gives s mT2 = 0.1866 and from Eq. (6.26),

— = 0.1866
*2
Rotor-circuit resistance for obtaining a slip of 5%,
r 2' = 0.1866 x2
Thus, re d u ctio n in rotor circuit resistance
= 0.5 x 2 —0.1866 x2
.-. Percentage reduction in rotor-circuit resistance
_ 0.5 x2 - 0-1866 Xg ^ 1Q0 _ 62 68%.
0.5 x2
Exam ple 6 .1 6 . The rotor o f a 3-p h a se mducfwn P^P ^
0.2 n standstill reactan ce p er p h ase. W hatex ** ? N giect stat0r impedance.
order to get h a lf o f the m axim um torque at star ing g ?
By what percentage w ill this external resistance chang

Tt lt
St _
i r,„
\ '^em 22 _
Solution. ~xTem
Z ~~.smT
Tem = mi- +, _ L
1 SmT
<* s2 T - 4 » mT4 -l = 0
= 4 ± i g 2 I = 3.73 or 0.27.
or
SmT ‘ 2 , . , „ = 0.27. For negligible stator impedance,
Higher value of 3.73 is redundant. Therefor ,
fr°m Eq. (6.26),
r2 = x2
«- S: C 0.2X0.27 = 0.054 a
To

Scanned by Cam Scanner


(Art 6.9
742 E le ctrica l M ach in ery

External resistance that must be inserted in the rotor circuit


= r2' _ r2 = 0.054 - 0.04 = 0.014 Q.

Without external resistance,


j _ JV = 4.903 V
*' VO. 04 + 0.2
0.04 = 0.196.
pf V0.04 + 0.2^
With external resistance,
/ J--------4.827 V
*' Vo.0 5 4 + 0.2
pf= 2 ^ 1 — pp= 0 .2 6 1 .
P Vo.0 5 4 + 0 .2
Percentage reduction in starting current
4.903 - 4.827
x 100 = 1.55%.
~ 4.903
Percentage improvement in pf
0.261 - 0.196
x 100 = 33.16%.
“ 0.196

E xam p le 6.17. A 3-phase induction m otor with — = 0.5, h a s a starting torque o f 25.0 Nm.
x2
For negligible stator im pedance an d no-load current, determ ine the starting torque in case the
rotor-circuit resistance p er p h ase is (a) doubled (b) halved.
Solution. With negligible stator impedance and no-load current, the starting torque from
the induction motor equivalent circuit is given by
3V*
r e st ~ r2 = 25.00 Nm.
es‘ CO, r22 + x22
r9
With — = 0.5
II
o
* o

*2

3V\
T =~ r , = 25
est cos r\ + 4 r\
1 3 V?
or — l = 2 5 x 5 = 125.
r2

(a) When rotor-circuit resistance is doubled, the starting torque is given by


3V?
T
^a.c# = — ■
est Cl) (2r£)+x|
/oJ2\ . -2 (2r2)
But x2 = r2
3V?
*Te st =— • (2 r2)
4 r 2‘ + 4 r i

or (l 3V?j ( 1 ) 125
T
■* e st
- = 31.2 5 Nm.
4
\ /

Scanned by Cam Scanner


P olyp h ase In d u ction M o to rs 743
____
r otor-circuit resistance is halved, the starting torque is given by

{b) - 3V? fr2) 1 3Vf '


T e st ~ CO. /_ \ 2 2
v y v y
+ *2 T +44
V 3VJ1 2 _ 125x2
= 14.706 Nm.
, o)s r 2 17 17
v > .

le 6 18. A 3 p h a s e induction m otor has a starting torque o f 150% and a maximum


o f the fu ll-lo a d torque. N eglect stator resistance an d assum e constant rotor resis-
torquetf g ^ the snp a t m axim um torque (b) full-load slip and (c) the rotor current at
terms o f fu ll-lo a d rotor current.
S S o lu tio n , (a ) T ' “ = 1 5 T efi> T em = 2 . 5 T e f l

1.5 2smT
Test or
From Eq. (6.33), T em ®mT 1
"IT SmT

s l T - 3.333 s mT + 1 = 0
or
Its solution gives s mT = 0-3 3 3 -
[e fl_
(6)
C m Sfl _ s mT
SmT T S/I
J_ 2
or 2.5 s/j 0.333
0.333 s/7

or sj* - 1.665 Sfi + 0.111 = 0


Its solution gives Sfj = 0.0695

(c) From Eq. (6.13),

„ 1 r2 f l
T'fl " to , 2/7 Sfl

T est ‘ 2at

T e fl
\2 JJ5 0 _
(I
00695
^ j
or l u = 4.646 V t .mK (Ae res,s(on„

stator arcct rotor circuit*. fl ,on7W<? oj -


^ocfance referred to stator. The motor exCiting current.
^ starting and m axim um torques. Neg
X \~ x 2 = 4 f ! = 2
Solution. Here

Scanned by Cam Scanner


744 Electrical Machinery (A rt 6.9

At a slip of 0.04, the torque, from Eq. (6.24 b ), is


3V2 1 . ra
T. = 200 = 0.04
co.
+ (4 + 4) H
( 1 + 0.10 4
3V2
= 5920
co.
3V* = 5920 = 87.06 Nm
T est - M . ^2
[(1 + 1)* + (4 + 4)1 2+8

SmT ‘ Vr? + (x[ = 7 T 7 F = 0124


3V2 1 w 1 = 326.63 Nm
Tem = 0.124
CO.
1+ +8
0.124
/
Maximum torque can also be obtained from Eq. (6.27) directly without computing smT. From
this equation,
3V
° Tr2
11 1
1 = 326.63 Nm.
T*rn = —CO, 2 [i + Vlz + 82J = 5920 2 [1 + V65]
E xam p le 6.20. A 3-phase wound-rotor induction motor runs at a slip o f 5% at full load
torque an d with its slip rings short circuited. I f rotor-circuit resistance is now increased to four
times by inserting external non-inductive resistances in the rotor circuit; then determine, at full­
load torque, the follow ing param eters in terms o f their previous values with rotor resistance
r2-
(a) slip (b) rotor current, rotor ohm ic loss, rotor p.f. a n d (c) pow er output.
Solution. Here subscript A is used for the values when rotor resistance is r2 and B for the
values when rotor resistance is 4r2.
sK;
(a) From Eq. (6.30), Te =
r2
m 0.05 Kt
T * -
r2
SB K t
and T eB =
4r ,

—‘* _ i _ Sb K ( r2 SB
TgA * 4r2 0.05 K t 0.2
Sq —0.2
As sA = 0.05, this shows SB = 4sA

( b) From Eq. (6.29), T SV


*2 ------
r2
0.05 V
ha =

0.2 V
and I 2B ~
4ro

Scanned by Cam Scanner


Polyphase In d u ct’ jn M o to rs 745

IjB 0.2 V '2 .


. I u ~ 4r2 0.05 V ' 1
= hA
°r rotor current at 4r2 is the same as with rotor resistance r2.

Rotor ohmic loss =4 (4r2) = ( 4 4r2)


= 4 ( 4 r 2"! = 4 times the previous loss
4r2 4r2
Rotor p.f-» (C0S 0^fl ~ V(4r2)2 + (sfl x2f V(4r2)2 + (4 sAx2f

= T T - [:?------^ = (co s0)A


Vrl + (sAx 2)
This shows that rotor p.f. at full-load torque remains unchanged as rotor resistance is
changed from r2 to 4r2.
(c) Speed wA = (1 - sA) ws = 0.95 co,
Power output, PA = ^eA wa = 0 9 5 “ j TeA
o)s TeA = PA/0 .9 5
or
sB = 0.20 and (oB = (1 - s B) ws = 0.80 a), ,
Now
P b = T'B «,„ = TeA X 0.80 0)s ( T’eA = T eB)
0.8PA
= 0.80 (w/T*) = - ^ g -

or P fl = 0.842 PA
Thus power output with 4r2 is reduced from PAto 0.842 PA.
Example 6.21. A 3-phase, 50 Hz, 6-pole induction motor has a shaft output o f 10 kW at 930
r.p.m. Friction and windage loss amount to 1% o f output. Total stator losses are 600 W.
(a) Determine the rotor input and stator input.
<b) I f maximum torque is developed at 800 r.p.m.. compute the starting torque with rated-
voltage starting.
, , 1 0 0 0 - 9 3 0 _ nft7
Solution, (a) Full-load slip - ^qqq uu
I
Friction and windage loss = 10,000«x ^qq - 100 W

Mechanical power developed,


P = 10,000+ 100 = 10,100 W
•••Rotor input, P , - PmAl - *) “ 10100/0.93 = 10860.2 W
Stator input - Pe * total stator lo8SeS = 10S6° '2 + =
_ 1 0 0 0 ^ 8 0 0 = „ 20
[0) smT ~ 1000
P' 108602x^0 =1 0 3 7 lN m
T' f l = a i = 2tc x 1000

2 —
rm 0.2
_ —+— 0.07
~
0.07 0.2

S c a n n e ^ y C am scanner
746 Electrical Machinery

or Tem = 1.604 T #
Test 2
Tem 0 :2 + _ l
0.2
or Tts, = ^ x 1.604 x 103.71 = 63.981 Nm.

E x a m p le 6 .2 2 . A 7.5 kW, 420-V, 3-phase, star-connected, 50 Hz, 4-pole squ irrel cage indue
tion m otor develops fu ll-lo ad torque at a slip o f 4% un der rated voltage a n d frequ en cy conditions
Friction, w indage a n d core losses a re to be neglected. The m ach in e h a s the follow in g impedancp
data. '
ri = 1.2 a Xj = x 2 = 1.4 a xm= 38.6 n
D eterm ine the m axim um torque at rated voltage a n d frequen cy a n d the slip at which it
occurs. A lso calcu late the starting torque.
S o lu tio n . Thevenin’s equivalent circuit param eters, refer to Fig. 6.17 (a), are as under •
R 1 .2 x 3 8 .6
' 38.6 + 1.40 ~ 58 Q

= 1.351 n

As friction, windage and core losses are neglected, the mechanical power developed
P m = 7500 W

Now *2 r2
P ;„ = 3 ( l - s ) P ^ = 3 ( l - s )

From Eq. (6.29), // -------


sV*
'I r2
S2 V?e '_ 2
F,n = 3 (1 —s) „2
rj s
or sV2
7500 = 3 ( 1 - 5 )
r2
or 3 j0 j> 6 )(0 04) (234)2
7500 ~ 0 841 Q
The slip at which maximum torque occurs is given by Eq. (6.26) as
0.841
s mT =
V l.l5 8 z

From Eq. (6.27), Tem - 3 x 2342


50 n 1.158 + Vl.158^ + 2.7512 = 126.21 Nm.

From Eq. (6.28), Ttsl -~ 3 x 2342 0.841


50 7t ,qo2 . o = 76.05 Nm.
Exam p le
....... j - 6.23.■A .100
. kW,
. 3-phase
, , ' 420 V• °6-nnlp
Pole> tiz w . wound-rotor
, , . . motor, with
induction •.l
0 2 w ^ 7 o o e r a L 7 ° ? C7 T i a f, i t lo a d st‘P o f 0 04 a n d the slip at m aximum torque of
i s Find Z T ? 7 H f ° •f^Ouency. N eglect stator resistance an d rotational los­
ses.F in d (a) m axim um torque, (b) starting torque an d (c) fu ll-load rotor ohm ic loss.

by Cam Scanner
Polyphase Induction Motors 747

resistance is now dou bled by adding external series resistance. Determine (d) slip
fh e T°Y0utpnt (e) fu ll-load torque (f) slip at maximum torque.
(a) As rotational losses are neglected, 100 kW is the mechanical power developed
S°lu p = 100 kW
* ■*. r m
,ref°re’
fhe
P ,» P „ / (l-s * )-^ | k W

4 n f 4k x 50 100 k ,,
chronous speed, co. = —p = — g— = —- — rad/sec
Syn
Tefi
eft —~~J*~= n1QQ0; Q01°nX- -- = 994.72 Nm
efJ CO. 0.96 x 100 k

0.2 0.04
0.04 + 0.2
T = 2.6 Ten = 2 .6 x 9 9 4 .7 2 = 2586.272 Nm.
or
■es/

(M ■em
1 0.2 5.2
0.2 + 1

T = — x 2586.272 = 994.72 Nm.


.-. 1 est 5.2
. i _ o p = 0 04 x -^ 7 ^ : = 4.167 kW.
(c) Rotor ohmic loss -sfg u.u* 0 95

(d) From Eq. (6.30 a), full-load output is


3V*s n oA
Te • co, = —fT C1 “ s)
3V2 x 0.04 x 0.96
= H
With rotor resistance doubled, full-load output is
...(H)


From (i) and (ii), for the same full-load output,
- x 0.04 x 0.96 = g— « i (1 " ^
r2
or S 2 - S l + 0.0768 = 0

Its solution gives Sj = 0.0888


10020 0 0 j< 3 _ _ s 1042.27 Nm.
(e) T cft = - f f T a 0 8 3 8 p < l 0 0 n

(f) With rotor resistance r 2,

8 t = 5 = 0 -2 •'

With rotor resistance 2 r2,


25 - 2 x 0.2 = 0.4.
S/uTl- X

Scanned by Cam Scanner


IA rt 6.9
748 Electrical Machinery

E xam p le 6 .2 4 . The rotor o f a 3-phase slip-n n g inductw n m o t o r is r e w m n d w M J * “ “* «*


many tarns o f conductors having h a lf o f the cross-sectional a rea
space factor o f the old an d new windings are the sam e. D iscuss «,
different param eters, characteristics an d perform an ce o f the m otor. N egle mpedance.
pL
Solution. Rotor resistance, r2 =
= p (2 L ) = 4 p L = 4/>2
With new winding,
A
2
Leakage reactance (Number of turns)
*2 N il
With new winding, x2' &Nphr
x2
— = 4 or x2 = 4 x 2
%
Slip at which maximum torque occurs,
f2
smT —X2
4s,
With new winding, Sm H ~ “ SmT

Thus, the slip at which maximum torque occurs remains unchanged.


V
Starting current, ^ 2 st ~
VrT+^T
l2st
Starting current with new winding, =
4 Vr2" x2
Starting torque, Pest = l2st r 2
With new winding, Pest ~ < ! * / 4) 2 x 4 r2 = I T.est

From Eq. (6.27), maximum torque, T 3V2


=— —
2x 2

With new winding 3V2


*T
* em' =
2 (4 x 2) 4 em
It is seen from above that starting and maxi­
mum torques are reduced to 25% of their previous
values. For load torque profile TL as shown in Fig.
6.21, the operating speed drops from to n2. The
motor would now have reduced efficiency. The
motor rating is now reduced to about 25% of its
previous rating.
E x a m p le 6 .2 5 . A 3 -p h ase, 10 p o le , 50 Hz Test
wound-rotor induction m otor is to be used a s a fre- IT
quency changer. This induction m otor is driven by M ­
an adjustable-speed d.c. motor. The output pow er iT '
taken from the rotor slip rings is 48 kW a t 0.8 4 SPEE0 ~~
pow er factor lagging a n d it rem ains constant over Fie 6.21. Pertaining to Exam ple 6.24.

Scanned by Cam Scanner


Tolyphavf ludutlie.n M mom " 4 'I

,nut frMuency rangC Of! 20t ^ '? ? ° Hz\lf ? V the lhac}'"'<’ losses, leakage impedance dn.pt
tPu. ' currents are neglected, then calculate
t KriUn" , ,

^ l oIsy*
e ' d W W <>f d c- mmolor'.
\vA rating o f the induction-motor stator,
lb) . < ? w #
the d.c. motor rating and its maximum torque output.
C aximvm speed is lim ited to 2700 r.p.m., then compute
Id) the number o f poles for the induction machine and
te) the new speed range o f the d.c. motor.
Solution, (a) Speed of the stator field
120 f 1 2 0 x 5 0 _nn
= = — Yo— = 600 r P m
for obtaining rotor frequencies greater than the stator frequency, the ro*or should
f ablvbe driven against the direction of stator field. Hence, if rotor speed is S\ r p m . then
Relative speed between stator field and rotor conductors is (600 *„V,) r p m. In view of this, the
rotor frequency as per Eq. (6.6 a) is given by
P (600 + Nr)
120 -12
For /ij = 120 Hz,
10 (600 + Nr)
=120 Hz
120
. Nr = 840 r.p.m.
For f2 = ^ z'
10 ( 6 0 ° = 3QQ Hz
120
. Nr = 3000 r.p.m.
Therefore, D .C . motor s p e e d r a n g e is fr o m 8 4 0 to 3 0 0 0 r.p .m .

,6, ana
to; and icj.
(C. At
At stan d s*.., iew
stanasuu, ,et the
we rotor per rphase
------ voitage
- be V,- and rotor , „ ^
frequency changer be Since leakage impedance drops, exciting current =i n d ^ ™
neglected, per phase electrical output V, / 2 from the rotor is cqua «
in p u t fr o m ^ r t o r o t o r .i., stator
% just like a transformer. As the rotor starts rotating, its per pna. , g
«s per phase output in v o lt-a m p cre s becomes sV2 /,. ^ ^ ,h;u) , h c voU.
In the frequency changer, slip s is greater lan ^ ^ ^ mu$l bo suppliod bv
a^nperes supplied by stator. Naturally the rest o p *
d.c. motor.
Power handled by d.c. motor - (8^2 A2" ' co'
= (s - 1) V, /. cos 0 watts. ,
1; (Tu jc sunnlied in two ways, nauu .v
This shows that electrical power output from rotor shfi rmgs P
it) electrically, from stator to rotor by transform! r
W) mechanically, from the d.c. motor wlmd.
For N, - 840 r.p.m.,
H-in * 600 o
Hlip*ic noO

Scanned by Cam Scanner


750 Electrical Machinery (A rt. 6.9

and for Nr = 3000 r.p.m.,


,. 3000 + 600 „ n
slip S2 ™ ------ = 6.0

48 .
Rotor kVA output, 3sV1I 1 = 60 kVA
l).o
60 , , TA
.*. Stator kVA rating, S V Ji = — = ^
s 2.4 ° r M kVA
= 25 kVA or 10 kVA.
Since the stator sho'uld"b.e.able to handle higher kVA, the stator kVA rating is 25 kVA.
D.C. motor rating = (s - 1) V\ Ii cos 0 = (2.4 - 1) 25 x 0.8 kW ,
= (6 - 1) 10 x 0.8 kW = 28 kW or 40 kW.
/. D.C. motor rating is 40 kW.
„ „ . , _ 28,000 x 60. 40,000 x 60
D C. motor output torque = — — —r r r — or —--- - - -
^ H 27t X v840 27t x 3000
= 31^.3 Nm;or 127.3 Nm.
.-. Maximum torque output from d.c.vipotor is 318.3 Nm.
V. ^ V ^
(d ) When the maximum speed is limited to 2700 np.m. j then let the number of induction
% \' \
, . , . ~ 120x50 -
machine poles may be P v Then speed of stator field = r.p.m.
Pi
120 x 50 .\
Relative speed between stator field and rotor conductors is 2700 r.p.m.
Pi
„ (1200 x 50
•+ 2700
From Eq. (6.6 a), = 300 Hz.
120
p _ 30,000 1
1 2700 9
But P x can’t be a fraction, therefore, P x must be 12 (a higher even number).
(e) Maximum speed of d .c. motor
120x50
12 “ rl
12
= 300
120
A; Nrl = 2500 r.p.m.
Minimum speed of d.c. motor
120 x 50 \
12 + N.„r2
120
= 120
120
or Nr2 = 700 r.p.m.

Therefore, new speed range of d.c. motor is from 700 to 2500 r.p.m Note that the ratio of
maximum to minimum speeds is same f3000 2500 "i in both the cases.
840 700

Scanned by Cam Scanner


Polyphase Induction M otors 751

i 6.26. A 3 -phase, 50 Hz, 4-pole induction motor has rated output o f 10 kW at 1425
Qffit&P ifruirn torque is developed at 1200 rpm. Calculate the starting torque. Neglect
'"‘S ta n c e and rotational losses.
.. 1500 - 1 4 2 5 „
^lution- S1‘P’ s - 1500 0 05
output = 10,000 W = power developed,Pmas the rotational losses are neglected.

P0* Cf . . rrs p m
. Electromagnetic torque, Te = ^ (1 _ g)

1 0 ,0 0 0 x 6 0 r.7 n i TMm
... efl 2k x 1500 (1 - 0.05) 67013 Nm
Slip at which maximum torque occurs is given by
1500 - 1200 „
S^ = 1500 = °-2
Tefl 67.013 2
T a k i n g s as full-load slip, " 0,05 0.2
6m ' 0.2 + 0.05

or "em
T,e.m = 142.403 Nm
ining^tkrting torque, use the relation
For obtaining
Tc, t 2__________2 = 0.3846
Tem 1 SmT~ J _ 02
TZ+ 1 02 1
Te st = 0.3846 x 142.403 = 54.7682 Nm.
Example 6.27. An induction m otor runs at a slip frequency o f 2 Hz when supplied from a
3-phase, 400 V, 50 Hz supply. F or the sam e developed torque find t h e *hP / ^ g u e n c y a *"•
will run when supplied from a 3 -phase 340 V, 40 Hz system. Slip at which the ° r
maximum torque using 50 Hz supply is 0.1. Neglect the stator im pedance an d assum e lin ear
torque-slip characteristic between zero torque and maximum torque in e wor

Solution. With stator impedance ignored, the maximum torque in a 3 phase induction
motor is given by

T -A *.
iem (i)s 2x2
For 400 V supply, smT = 0.1

When /■= 50 Hz, ^ = 01 “ S


a2

For f= 40 Hz, s mTX oc


From these relations
40
5(
= “ x 0 , l = °.125
S . n

40
3 3P V
Maximum torque, Ttm = — = 4 t f ' 2 (2 n /!^

** ■ T
1 cm « V2//"5
/

S c a ^^c T B y Cam Scanner


A1
752 E le c tric a l M ach in ery [Art. 6.9

As torque-slip characteristic is linear between zero torque and maximum torque, the
developed torque Ted at a slip of 0.04 is

400.\2 0.04
Ted = x 0.04
s mT 50 O.i t
For 340 V, 50 Hz, si
0
340^
ed
40 0.125
For the same developed torque, from U) and (it),

'4 0 0 ' 2 0.04 340 _ s_


50 0.1 40 X o!l25
New value of slip, s = 0.0443
Slip frequency = s f= 0.0443 x 40 = 1.772 Hz.
E xam p le 6.28. A 50 Hz, 440 V, three-phase, 4-pole induction m otor develops h a lf the rated
torque at 1490 rpm. With the applied voltage m agnitude rem aining at the rated value, what
should be its frequency i f the motor has to develop the sam e torque at 1600 rpm ? Neglect stator
an d rotor winding resistances, leakage reactances and iron losses. (GATE, 1995)
c , nr 1 5 0 0 -1 4 9 0 1
Solution. Slip, s=
1500 150
™ 3 V2 ft
T= —
<os f s
r2
+ rl + (X X + X2)

With the neglect of resistances and leakage reactances,


T i if
T' “ 7 ' r 2 s
3_ {440^
...(i)
e 50 To 150

Also ...(it)
f r2
For the same torque, from (i) and (ii), we get
3 (440)^ (440) 1
f *2 S 50 r2 150
s
or ...(Hi)
7500 f'
120f
Synchronous speed, Ns = ——x = 30 f

„ 30 f - 1600
and
S|,P’ 5 = ' 30
Substituting the value of s in (iii ) above,
'3 0 f - 1 6 0 0 ^11 1
30 f f 7500

f - 7500 f - 400,000 = 0
Its solution gives f = 53.7 Hz.

Scanned by Cam Scanner


Polyphase Induction M otors 753

( t ^ p l c e - 2 9 - The follow ing data refers t0 a I 2 -P°k. 420 V, 50 3-phase mesh connected
£‘ a m o to r:
2 gtj Q %i - 6 82 r2 = 2.08 Q, x2' =4.11 Q per phase.
fl" load, the line value o f magnetizing current is 6.7 A and the total core loss is 269 W.
0n 0 tfoe p f input current, equivalent rotor current and torque developed by the motor at a
peierrIlin* uSing exact equivalent circuit. Determine the maximum torque developed and the
>lipten din g speed. (I.A.S., 1989)
^ g lution. The equivalent circuit on no load is as shown in Fig. 6.22 (a),
per-phase magnetizing current,
6.7
‘m=-^ g ■= 3.87 A
I„

Here Vi = £ 1 + J m(r1+ > 1)


I lags E x by 90° and with E x as reference,
Vx = E x - j3.87 (2.95 ->6.82) = E x- j l l A l l + 26.4

or V\ = (E x + 26.4 f + 11.4172

or 4202 = E\ + 52.8 E x + 697 + 130.35

or Its solution givesE\ + 52.8E xE=x -393.45


175572.65 = 0
V
:. Magnetizing reactance,
_ 393.45 = 1()167 Q
m 3.87
The exact equivalent circuit is now shown in Fig. 6.22 (6).
2-95 6-82 4-11
X| A
-dooo ■
—WW--- 'TTWC'- o- -VvVv---- 'UtitJtf'-
♦+ 2-95 6-82
' 11 - 3-87 A
101-67 : 2_08
42 0 V s
xm

(6)
(a)
Fig. 6.22. Pertaining to Example 6.29.

'2.08
+04.11 (/101.67)
0.03
From this circuit, Zf =
7" (69.33 + j 105.78)
69.452 /3 .4 ° x 1 0 1 .6 7 J 9 £ _ RR R3 ,36 64c
1o
or
= ~ rs
126.484 / 56.761
Rf +jXf = 44.80 + j 33.32
r i + j * l = 2-95 +J6.82
ZAB = 47.75 +040.14 = 62.38 fM S W

InPut current I - 420 = 6.733 /_IL40j)5


62.38 / 40.05 ''
p f = cos 40.05° = 0.765 lag

Scanned by Cam Scanner


754 Electrical Machinery

Equivalent rotor current,

/ ~
12 - /1l jX
- 7 + j (*2 + Xm)
O
i 101.68
= 6.733 / ~ 4 0.05 x g9 33 + . 1Q5 ?g
101.68 /9 0 °
= 6.733 4 0 0 5 x 126 484 / 5 6 . f ^ = 5-413 Z z ^ i i !

Torque =3^ g = x (6.733)2 x 44.80 = 116.364 Nm


o)s 2n x 500
Maximum torque can easily be obtained through the use of Thevenin’s equivalent circuit
X 1 = x 1 + Xm = 6.82 + 101.67 = 108.49 £1
2 .9 5 x 1 0 L 6 7 = 2 7 6 5 a
e X1 108.49
v * i Xm 6.82 x 101.67__ ______
X' = ~ x r = 108.49 = 6 -4 0 n
Slip a t which maximum torque occurs,
2.08__________
SmT ~ Vtf* + (x2 + X ef ~ [2.7652 + (4.11 + 6.40)2] 1/2 = ° ' 1914
Speed a t maximum torque = 500 (1 - 0.1914) = 404.3 rpm


Maximum ^ = 3V?
torque, Tem ------•— ■ ,■1
H ’ em 27m, 2 [Re + ylRt + (x2 + X f ]
_ 3 x 60 ___________ 3 9 3 .62________________
2 k x 500 2 [2.765 + V2.765* + (4.11 + 6.40)2] “ 325 55 Nm>
E x a m p le 6 .3 0 .A 10 kW, 50H z, 3 -phase induction m otor develops rated torque at 1440rpm.
In case lo a d torque is reduced to half, fin d the m otor speed a n d the p o w er output. Assume linear
torque-slip characteristics n ear the operatin g range.

S o lu tio n . Slip s = 15° 95~ q44Q = 0.04

For linear torque-slip characteristics, from Eq. (6.30 a),

T - 3V2 — '
* ' r2 -
As V, co, and r 2 are constant,
Te oc 0.04

and i T e~ s .

- T
2 6 1
" ~Y s ~ = 0 M or si = 2*0 .0 4 = 0.02

New motor speed = 1500 (1 - 0.02) = 1470 rpm

Rated torque = Nm
27t x 1460

Scanned by Cam Scanner


P oly p h ase In d u ctio n M o to rs 755

a‘ h alf of rated t0rqUe


'< lw er< )Utp 1 1 0 .0 0 0 :
P ^ r°
2 271 x 1460 60
6 31 A 3 -p h ase, 1455 rpm induction m otor drives a loa d requiring constant
gjtaiflP*6 ‘ Dpiy voltage d rop s to 90%, fin d w hat happens to m otor speed a n d losses. Take
I'I'*'l" slip ch aracteristic over the w orking range.
" > rq Slip Sl = 1500_^_1455 _ o os
Solution- ShpSl 1500
„ , T - 3 S
fro m E o - ( 6 '3 0 “ ’ ■ 0 ), r2

For the same load torque


3V2 0.03 _ 3(0.9V)2 s2
T
1 e = cos r2 cos r2
0-03 _ « «q7
. s * = o J i ~ °*037
M o to r speed = 1500 (1 - 0.037) = 1444.5 rpm
At small values of slips, rotor current from Eq. (6.29) is
s,V 0.03 V

Rotor current with reduced voltage,


s2 (0.9 V) _ 0.037 (0.9 V)
722“ Ti r2
h 2 0.037 (0.9 V)
1n
•• / 21“ 0.03 V '
f j v2
Ohmic loss with reduced voltage _ £22 = ( i n ) 2 = 1.2321
Ohmic loss with normal voltage V21J
Percentage reduction in speed
_ 1455 - 1444.5 _ Q722%
1455
Percentage increase in ohmic loss
1 .2 3 2 1 - 1 x 1Q0 = 23.21%
1 1 i. A Atinn in s u d d I v Voltage does not materially effect the
This example demonstrates that reducti temperature rise noticeably under the
motor speed but increases the losses and there
condition of constant load torque. sauirrel-cage induction m otor
Example 6.32. A 7.5 kW. 400 V. I applied. Assuming o lin ear
operates at 4% slip at fu ll load when rated ’ ? caiculate the no-load speed o f the
relationship between torque an d slip in the op g ^ no.ioad torque is 6 Nm.
m°tor when the supply voltage is reduced to h a lf its (IA .S., 1988)

Solution. When torque-slip relation is linear, from Eq. (6.30 a),


sV2
T OC ----------
e cog •r2

S ^innecl by Cam Scanner " ........... .....


[Art. 6(9

•755 Electrical Machinery


7500 x 60 _ 74.604 Nm
Full-load torque, Tefl = 2 n y. 1000 x 0.96
0.04 x 400
74.604 - w.,r2

s x 2002
6 06 ------
and «Vr2
g s x2002
or 74.604 0.04 x 400
f4001 * — 1 — = 0.01287
s = 0.04 x 6 x 200 * 74.604
or

^Exam ple 6.3 3 . A 6-pole 50 Hz, 3-Ph™e g t d i p . T he rotor resistan ce p e r phase is


torque o f 25 0p ercent o f fu ll-load torque fJL T t
1.0 D ot 50 Hz. The inputsupply vaItage-us ubj ^

full-load torque at starting at this reduced . S ., 1975)


"(/.A ................
Assume that the m agnetic circuit is unsaturated.
Solution. ' em = 2.5 Tefl, SmT = 018
1.0
Also smT^ - 0 - 18. *2 = 0.18 = 5.55 £1
0.36s
'ejl
2.5 0.18 s2 + 0.0324
1em
0.18
where s = full-load slip.
... s2 - 0.9s + 0.0324 = 0
Its solution gives s = 0.0376
V2 V2
T n= — •- 0.0376 co, 2 7 .7 5 4
le fl co. /
L ^ L _ + 5.5'
0.0376
Let the minimum voltage be x V where x < l . The motor can continue supplying its rated
torque until Tem at reduced voltage = Tefl.
_3 (xV)2 _3_ V2
co„ 2 x 5.55 co. 27.754

or : = V 27.754 = 0.6324 or 63.24% .


L et/? be the external resistance inserted in the rotor circuit. It is given th a t starting to r q u e
at reduced voltage = Tejj
A , (0.6324 V2
(/? + l ) 2 + 5.552 cus 27.754
or 11.1 (/? + 1) = /?2 + 2/? + 1 + 30.80
or R 2 - 9.1/? + 20.7 = 0
Its solution gives R = 4.55 £1.
.-. External resistance required in the rotor circuit = 4 .55 £1.

Scanned by Cam Scanner


Polyphase Induction M otors 757

amPle 6 ’3 4 ’ A S'p h ase inductlon motor is designed to operate on 50 Hz and V, volts. In


the motor is fed from 40 Hz, 0.9 V, voltage source, find the ratio o f 50 Hz to 40 Hz operation
C°Se starting currents, starting torques an d maximum torques.
f°r find the ratio o fV 2 at 40 Hz to Vx at 50 Hz for the same starting and maximum torques.
, t stator im pedance an d rotor resistance.
Solution. For x2 at 50 Hz, rotor leakage reactance x2 at 40 Hz is given by x2 - 0.8 x2.
Startin|icu_rre n t a t 50 Hz = Vi 0 .8 * 2 = 0,8 =
Starting cu rren t a t 40 Hz x2 0.9 Vj 0.9
Starting torque at 50 Hz _ _3_ V? 40 (0.8x2)2 _ f Q.sf
= 0.6321
Starting torque at 40 Hz 50 x\ 3 (0.9 V / ” ' j^O.9

Maximum torque at 50 Hz _ _3_ V\ 40 x 2 x 0.8 x2 _ ( p.sf


= 0.790
Maximum torque at 40 Hz 50 2 •x 2 3 (0.9 V!)2 1^0.9

For the same starting torque,


3^ 2 , 3 ^
40 ' (0.8 x 2)2 50 (X2)2

or ^ =ViVc.S)2 =0.755
For the same maximum torque
_3_ Vl _ 3 . Vf
40 ' 2 (0.8 x 2) 50 2 x 2

or TT = ’J(O.S) (0.8) = 0.8.


Vl
Example 6.35. A 3-phase, 4-pole, 60 kW, 50 Hz induction motor connected rate
milage and running without load consumes 3 kW. When prevented from rotating it draws rated
current al 30% rated supply an d takes a power input o f 4 kW. Assuming that under rated loa
conditions, the stator an d rotor copper losses are equal and that the mechanical losses are 3 M
of the no-load losses, determ ine (a) slip at rated load and (b) starting torque o f emx
rated applied voltage.
Solution. Mechanical loss

= io o x 3 0 0 0 = 9 0 0 W
Stator core loss = Power input at no-load - Mechanical loss - stator I*R loss at no load

Neglecting stator I 2R loss at no load, stator core loss


= 3000 - 900 = 2100 W
Power input during blocked rotor test
= stator P R loss + Rotor I 2R loss = 4000 W

Stator 12R loss = Rotor 12R loss = = 2000 W

(Q) At rated load, air-gap power, 2


P , = Output power + mechanical loss + rotor / R loss
*’* Pe = 60,000 + 900 + 2000 = 62900 W

Scanned by Cam Scanner


[Art.
758 E lectrical M ach in ery

But rotor I 2R loss = 2000 W - s Pg


. 2000 _ n ftQIB
/. Slip at rated load, s = 629oo -
(6) At rated voltage, power input to motor during blocked ro

'1 0 0 ') = 44.444 kW


30
I Ov On l
Air-gap power, P ,. = Power input
= ru w er inpixv - stator
— core t o s s - stator / R toss
= 44444 - 2100 - 2000 = 40344 W att

Synchronous speed, = 4,1 * 5- = 50 it rad/s

Start,no toraue =^ = 256 84 N m '

maximum torque

Solution.

At 50 Hz, leakage reactance of rotor = x2


At 45 Hz, leakage reactance of rotor = 0.9 x2
Subscripts 1 and 2 are used for Vlt 50 Hz operation and Vlf 45 Hz operation respectively.
Vx Vx
Starting currents. Istl = c r ~ 2 ' and J st2 = / 2 ^ m Qv .2
\r2 + x 2 \r2 + (\j.vx2)

I_st2 J r2 + x2 ~= J s 2mT+ 1 = f Q .2 -H =
ht\ + (0.9x2)2 s 2mT + 0.92 > 0.22 + 0.81 '
Starting to rq u es:

r$t-2 ^ r r2 r\ + x 2 s 2mT + 1 o .22 + 1


Tst.l r2 + (0.9x2)2 V fo f £ r + (0 .9 )2 0 .2 2 + 0.81
= 1.2235.

M aximum torques : = ~ •— —— •— — 1— l i n
Tnl to, 2 (0.9x2) 3 v? " 0.9 "

E x a m p le 6 .3 7 . A 2 0 kW, 400 V, 3-phase, 50-Hz, Y -connected, 4-p ole squ irrel-cag e induction
m otor h a s the follow in g p a ram eters in oh m s referred to sta to r :
= 0.2, Xj = x2 = 0.45, Xm = 18 .

l„ r Z hn L ‘kj ; mT n ! ? T , rg; Sed? ‘ rT d UOltage and f l u e n c y , it develops full-load internal


torque at a slip o f 0.04. Rotational and core losses are neglected. Calculate

a n d ^ maX‘mUm in tern al tor<>ue a n d in tern a l startin g torqu e a t r a te d voltage a n d frequency

(b) slip at m axim um torque.

Scanned by Cam Scanner


Polyphase Induction M otors 759

A " jlere rotational and core losses are neglected, therefore power output 20 kW =
S°1Vtifpower developed, P m.
^ p 20,000 W at a slip of s = 0.04.
:• m _ p m _ 20000 _ 20,000
. Air-gaP power, r g l _ s i _ 0.04 0.96
*' „ 0.04 ....................
f R loss = s Pa =
Total ro to r 019 6 X 20,000 = 833,33 W
3 l l r2 = 833.33 W

; " pe,p h a s e s ta to r v o lta g e ,V 1 = ^ = 2 3 0 .9 5 V

Vor Thevenin’s equivalent circuit,


r y _ V\ •Xm _ 2 3 0 .9 5 x 18 _
= 225.32 V
e Xm + xi 18.45
0.2 x 18 « vrvr- ^ v 0.45 x 18 ^
= 0.195 ft, Xe = - — — = 0.44 ft
Re 18.45 e 18.45
From Thevenin’s equivalent circuit, rotor current is given by

In -
12 ( R e + r 2 / s ) + 7 (X e + x 2)

225.322
3 I2 r2 — 3 x r 2 = 833.33 W
r*2
+ (0.44 + 0.45 )2
0.195 +
0.04
182.77 r2 = 0.038 + 625 r\ + 9.75 r 2 + 0.7921
or r\ - 0.2768 r 2 + 0.001328 = 0
Neglecting 0.001328, we get r 2 = 0.2768 ft.
x 50
(a) Synchronous speed, (Os = — —— = 50 •n rad/sec

Maximum internal torque, from Eqn. (6.27), is


vf

= 438.3 Nm

Initial starting torque, Te . st = — • r r2


[Re + r 2]2 + [Xe + x2l
3 225.322 x 0.2768 = 264.5 Nm
50 ic ' [0.195 + 0.2768]2 + 0.89'
Slip at which maximum torque occurs is

SmT=^Rf7 k ^ 2
0.2768 = 0.3038.
~ ff0.195)2 + (0.89)2]1/2

Scanned by Cam Scanner


760 E lectrical M achinery

E x a m p le 6 .3 8 . Electrom agnetic torque in a 3-phase induction m otor is given by

r c = | p 2 -<i) f 2 c o s Q2

Use this expression to obtain the torque-slip characteristic o f a 3-phase induction motor
S o lu tio n . In the torque expression for a 3-phase induction motor, P = number of po]
<{>= air-gap flux per pole, F 2 = rotor mmf per pole and cos 02 = rotor power factor. For obtain^’
the torque-slip characteristic, each term of the torque expression must be examined so far
their variation with slip (or speed) is concerned.
(i) P o le s (P). The number of poles is a constant parameter. Once the winding is designs
for a given speed, number of poles P is fixed.
(ii) F lu x p e r pole (<}>). For the stator of a 3-phase induction motor, per-phase applied volt­
age is given by
V1= E l + I 1 (rx+ j x i)
where E x = emf induced per phase in stator winding.
= V2 n f x •<t>■Nphi ■kw-L
With increase in load, stator current /j rises and E^ decreases slightly. Therefore, air-gap
flux also gets reduced accordingly with increase in load. However, the decrement in is quite
small, therefore air-gap flux <(>can be treated as almost constant with slip as shown in Fig. 6.23
(a).
Fl ux

(c)
Fig. 6.23. Variation of (a) air-gap flux <(>(6) rotor mm f F 2 (c) power factor cos 02 and
(rf) torque, with slip and speed in a 3-phase induction motor.

(Hi) R o to r m m f p e r pole (F2). It is seen from E q . (6.18) that as the slip decreases, rotor
current and therefore mmf F 2 decreases. At zero slip, I 2 = 0 and therefore F 2 = 0.
Variation of stator current is sketched in Fig. 6.19. As rotor mmf is balanced by s^a^or^jlU?i
the rotor mmf variation is similar to stator current or stator mmf variation of Fig. 6.19-
the variation of rotor mmf F 2 is as shown in Fig. 6.23 (6).

Scanned by Cam Scanner


Polyphase Induction Motors 761
-----------
' n o w e r-fa cto r (co s 02). At starting, rotor power factor is low, its value may be
(i„>Rotor P
Ac the rotor accelerates, rotor p.f. = 2 2 improves. The variation of rotor pf
n3to0-®' n r2'®) + x2
.hsliP sketched in Fig.
slip is sketcneu &• 6 .2
• 3 (c).
v /■
I 1< Tro rqaue.
u e - It
It isis seen
seen from
from electromagnetic
elec torque expression that torque is proportional
O' ^’) cos 0 02 Therefore, point-by-point
pomt'by*^vaa^vmultiplication
mwuviyuvuvuofwathe. vuw w4u***»wmof the
ordinates ———three
——- curves of
to^ ^ 3 (a)) for 0, <t>, 6.23 (6)
( 6 ) for F 2 and Fig. 6.23 (c) for cos 02
02 ; gives the torque-slip curve of a
Scr. 6.23 ’ _____ _____c 00 /Ji
^ 6‘ induction motor as shown in Fig. 6.23 (d).
fig
3|.phase 0 ,39 . A 3-phase delta-connected squirrel-cage induction m otor h as certain
circuit param eters. With the sam e stator core, the motor is rewound with star-con-
eqviv0' . ,- 0f sam e poles for operation at the same supply voltage and frequency to give
^ ra te d power output at the sam e slip. By what factors each o f the equivalent circuit
“ Z Z u r s would be changed I f A S - 1998\
P Solution. Let the subscripts d and y denote respectively the parameters of delta-connecte
. ?ar.connected windings of 3-phase induction motor.
Let the per-phase values of rated voltage and rated current be Vld and I ld for the stator in

delta.
Rating of 3-phase induction motor = 3 Vld I ld cos 6

When stator is rewound in star, rated stator voltage per phase, Vu and rated stator

current per phase, / ,y = V3 J u , so th at rating = 3 V „ 7 „ c o s 0 = 3 ^ jo /3 cos 0

=3 hd cos 0 remains the same as before.

Per-phase turns in star, N ly = NW) . Viy = ^J$ Vu

Cross-sectional area of conductor in star, a „ = V3 a ld, because I ly = <3' l ld.

. T 1 r
:. Length of conductor in star, L\y - r—^id

All parameters referred below are per-phase parameters.


e ■L id
(i) Stator resistance in delta, r ld -

e h ,. e W Lu
Stator resistance in star, ^iy - a d3 - a ^ 0 a ld 3

(ii) Stator leakage reactance. It is known that inductance - Re,

. ,
Leakage inductance in delta, lid ~ j^ej

i / _ ^Jy - 1 = -l\ d
leakage inductance in star, hy ~ r 6i 3 Rel 3
1
'• xly= 2 * u

Scanned by Cam Scanner


(A rt. 6.9
76 2 Electrical Machinery

(Hi) Magnetizing reactan ce. For delta connections, magnetizing


^-r)
N\d
Rel
For stator in star, magnetizing inductance,
»t2 _ .\N\d
_N\y ; 1

Magnetizing reactance, Xmy - ^ Xmd


in star remain the same as these
As no changes are made in cage rotor, ro or p when referred to stator in star, would
are for stator in delta. However, rotor circuit parameters, when relerren to ,
1
reduce to - as under :

r2d =r2 when referred to stator in delta -

Similarly

•i/a x i/j Xlh


-j— 'TRT'-

(a) (6)
Fig. 6.24. 3-phase induction motor equivalent circuit with stator winding in
(a)) ddelta
[a and
e lta a (b)) star,
n d (b s ta r , Example
E x a m p le 6.39.
b .3 9 .

Equivalent circuit of a 3-phase induction motor with stator winding in delta is shown in Fig.
6.24 (a). With stator rewound in star and for the conditions specified, the equivalent circuit is
given in Fig. 6.24 (6). All subscripts and primes are dropped for the sake of convenience.

Stator core loss (proportional to stator flux) in delta is proportional

Stator core loss in star a = - r^ ld V3" = —


iV i.,y W^ " Z
n Id n , ,
J-y\d
This shows that stator core loss remains the same. Stator ohmic loss in delta =3 l\d r 1(t

Stator ohmic loss in star = 3 l\y r ly = 3 (V3 I ld)2 ~ = Sl\d rld.


O
It is seen from above that stator losses for both delta and star connections remain the same.

For same power input, therefore air-gap power = 3 — is unchanged.


s
For stator in delta, stator m m f = rotor mmf
^1d ' N\d = I 2N 2

Scanned b y (JamScanner
Polyphase Induction M otors 763

j _ h d 'N ld
, Rotor current,
cux*— > h- N
iv2
Orator m sstar,
tai, I\y -N
*\y * iy
ly ^ z
f° rSt J Ily-Nly j 3 I ld Nu Iid ■Nld
. Rotor current, h ^ N% ^ ^

j^ows that rotor current is the same whether the stator winding is in delta or star. For

• can nower 3 /2 — , therefore operating slip s is the same in both types of stators.
thesa1*16 air s . .
• 6.40. A 3-phase induction motor has an air-gap length o f 2 mm. I f its air-gap
$%&m^creased to 2.5 mm, discuss its effect on the equivalent-circuit param eters as well as
M,tne'r’formance o f 3-phase induction motor.
' , , <i„n With increase in air-gap length from 2 mm to 2.5 mm, the new air-gap length
(2 5/2) = 1.25 times its previous gap length. Its effect on equivalent circuit parameters
^perform ance characteristics is examined below :
(a) Effect on equivalent-circuit param eters
(i) Stator resistance r, and rotor resistance r2 remain unaffected.
,-i l„ » 3 ohase induction motor, mutual flux remains practically constant. Therefore, with
air-gap length increased by 1.25 times, the magnetizing current must be 1,25 times ite previous

value so that constant flux is established. Therefore, new magnetizing reactance = = 0.8 I

(previous value of magnetizing reactance).


. X m' = 0 .8 X m
where prime is used to denote equivalent circuit parameters with increased air-gapengt ..
(in) With increased air-gap length, the stator winding the two
each other. The effect of this increased gap is to increase the: leakage ^ b e tw e e n
windings to 1.25 times its previous value. Therefore, Xi 1.2 i 2

Et Z : “ “ ength leads to poor power factor at no load and also on full load as

reactance results in lower va.ues of (a, maximum torque , . . . and (5,

starting torque Te st.


hii) Slip at which maximum torque occurs is reduced. ^
610. Operating (o r P erform an ce) C h aracteristics of ^ ‘ “"c a lly the varia-
Steady-state operating characteristics of an m u ghaft power output is varied
1011 of speed, power factor, stator current and he operating characteristics are
[ro® no-load to full load. For a given induction m o , and rotor slots_ The
°verned by its rotor resistance, air-gap lengt an , tate operating characteristics of
inf t of this article is to describe the nature o them. Fig. 6.25 illustrates typical
uction motors and to examine the factors a g 0f these curves is now explained
c^aracteristics of an induction motor.
, ’ .nnns <meed therefore, the no-load slip
i« v “! S p e c d - At no >°ad . rotor sPeed is T ^ n t to overcome the loss-torque required by fric-
C nr an, Als0' the n°-|oad t0rque’ S all no-load torque, the rotor current or the rotor
windage, is very low. As a result 01 s

_ IR 5C T S M 1M
Scanned bv CamScanner
IArt. 6. 10

764 E le c tric a l M a ch in ery _

0-25 050 0-75 1-25


Power output (p.u.)
Fig. 6.25. Operating characteristics of an induction motor.

m.m.f. F 2 is also quite small and the load angle, see Eq. (6.5), is very n e a r to 90°. As the applied
load torque is increased, electromagnetic torque Te m ust increase accordingly— this can happen
only if rotor m.m.f. F 2 and the load angle increase. In order th a t rotor m .m .f. F 2 or rotor current
/ 2 increase, the rotor e.m.f. sE 2 must increase or the slip m ust increase ; in other words the rotor
speed must decrease as the load torque is increased. Here E 2 is the rotor e.m.f. _per phase pha; a
11
standstill.
n » a ■■ *-« ■ ■

At no-load, rotor leakage reactance has little effect on the rotor leakage impedance, becaus<
rotor frequency and sx2 are very small and load angle 90 + tan ~ 1 — 2 is very n ear to 90°. Whei

' 1 ( \
the slip increases with an increase in load, the rotor power factor angle 02 U t a n ' 1 ^ in

3 “ aS 3 reSUlt f ‘: l0ad 3,1816 (90 + is 3150 tacreased - This Shows th a t as the apple,
load torque is increased, the rotor speed falls and both the rotor m m f F a i •
crease to supply the required load toraue T h p fa ll in a a a. ‘ ‘ ' 2 and *oad angle in
in the range of 2 to 5 per cent of rated speed In view of t h t ^ 0™ £ t0 ^ load is USuaU;
possess shunt characteristics. induction m otor can be said ti
(b ) P o w e r fa c to r. The stator current of a r,u • ^ .
following three components : -phase induction m otor is made up of thi

lossV S X Z Z Z r * * * COm~ » s ta to r iron ,„ s s and stator coppe,

^ d Z “ f ; r r r ^ th e r o t o r •
,POnent o f/" at no load is to supply f r ic tio n and vri ! ^ ftmction o fth e third. i.e. load, co.
F W „ ‘A h ird ' ““ Ponent is zero) The rotor no (F W ) '° 8S (F o r a transformer at i
w - ,o8s- * * this 1068 - o - t e s mi i , , ™ ; rpr: ; r ' d vth op^ a t ,n ° ioad- * « « « . «<uai
and therefore load component is very sroe

Scanned by Cam Scanner


Polyphase In d u ctio n Motors 765

uo)
^.0 the com bination of second and third com-
A
tJie ccurre>e
tbe :te small-
small B But
u t tne
the magnetizing
m agnetizing tu iie iiul mforms
current lurms a majur
major
5°0 e,nts
ptsis>s qUI
qU* - due
i . . « tn
to the Dresence
presence of air gap in an induction
induction motor,
^pone^
,0 0 °_load current
curre n t 10I 0 lags tne
the siaior
stator voltage bypy an angle 0O
esult- n°* 5<5 t0
^ a result’ tQ 85°.
g5o Consequently, the stator power factor at ino
* range ot
r ^ e • -- n^ ^ Q^i tn
tQ nQ 33 the
the lower
lowef values
V£dues being
being applicable f
applicable for
; a « n motors.
Wte
g tor is loaded, the third, i.e. load, component,
ntor component ui
of stator
^ the mo nodoac[ value. The increment of load com-
no-load
« ^ nt *r rfator current above its no load value is responsible for
P«»eIlt
P
<>nen load ttorque.
fI hpa load o r q u e . The
T h e sstator-load o m p o n e n t xI 27 given by Eq.
t a t o r - i o a o ccomponent
ipplying
su p P ^ dded t0 / , g i v e s t h e s t a t o r c u r r e n t OA at
gives the stator current at a power
a p o w e r factor
fa c to r

t6l5),0WWhh further increase of load on the motor, the rotor current - ^ Improvenlcnt „
ofcos 01- w stator-load component / 2' whep, lidded to I0 gives power factor with increase

loaded beyond this load, P ^ J ^ I ^ that bars over V* * , etc. to indicate them as
; t.r and rotor leakage r e a a n c e d r o p . Not ^ ^ Even Qtherwise lt ,mphes
phasors, are not shown in g. ^ ^ phasor diagram are phasors.
that the voltages, c u r r e n ts e tc ., dra m ach in es th e induction m achine losses a re m ade up
ir) Efficiency. As in other electrical ma » variable load losses. At no load, the
« core loss + friction and win age b >- " 1 ^ ^ ^ ^
shaft power output is zero, therefore, ,S ‘ therefore> low. As the load is increased, the
aremom as compared with the output, efficiencyus, ^ ^ varjable losses are equal,
efficiency also rises and b e c o m e s i m o » rated output, the higher vab esb ein g p
Maximum efficiency at about 80
o c c u r s ^ , oad resultm gm
plicable for larger motors. If the 08 raDidly than the output, consequen y
Sciency, the load losses increase more rapiQ y
decreases. r r B n t is a b o u t 3 0 t o 5 0 % o f r a t e d c u r r e n t , t h e
(d) S tator c u r r e n t. The no-load increase in load,
larger values being applicable to sm 1qCUS 0f the tips of stator ^ rre^ ^ 6 12
correspondingly. In the phasor diagr , induction motor circle diagr ,
loads, fo llo w s a s e m i - c i r c l e a n d t h i s ea h a v e (j) m o re ep ,
(e) Shape of s ta to r slo ts. For a given ab t a r e M h e ^ ^ ^ that slot leahage^ux
ith or (ii) less depth and more width^P induction m otorswithd P
iirectly proportional to f ‘ ^ I c e low values of T,.,„ Tm . s .r - Similar thought pr
ire leakage reactance and, theref . L<oka9.
applied
ap p lied tto
o induction motors
in d u c tio n m o w n * with wider s • — —v
’-closed
• or closed
-1 /\onn 1
Further, the slots may be open, se”11 g 27. For a
shown respectively in (a), (6) and (c ^ glots is
'en slot m.m.f., reluctance offered.% -x closed slots is
je (ii) semi-closed slots is less an ^ leakage
n less. Consequently the open slo ci0sed slots (o) <»> • W
L 6.27. (a) open (6) semi-open
stance than semiclosed slots, wherea . j osed slots.
and (c) closed slots.
^ more leakage reactance than t e s d jjnes.
% 6.27, the leakage flux is shown by dotted

Scanned by Cam Scanner


766 Electrical Machinery

(/) A ir gap. In induction motors, for constant supply voltage, the air-gap flux remains sub­
stantially constant. If the air-gap length is increased, then constant flux requires more mag­
netizing current. This reduces the no-load power factor as well as the full-load power factor of
the induction motor. Therefore, in order th at an induction motor operates at a better power
factor, the air-gap length is kept as small as is mechanically possible. Small air-gap clearance
in induction motors necessitates a heavier shaft and high-grade earm gs an are required for
other types of rotating machines of the same rating and speed.
Open slots in an induction motor has the effect of increasing the air-gap length (1.10 to 1.20
times the actual air-gap length) as a result of which more magnetizing current is required and
the operating power factor of the motor is worsened. But an induction motor with open slots has
less leakage reactance and, therefore, more Te.st> more Tem etc. Likewise an induction motor
with semi-closed slots or closed slots requires less magnetizing current and therefore better
operating power factor, hut its Te tl, Tem etc. are reduced. In view of this, at the design stage, a
compromise has to be made between Te sl, Tem and the operating power factor of the induction
motors.
6 .1 0 .1 . In d u ctio n m o to r sta b ility . Practically in all the motors the speed drops as the
load torque is increased. Mere intersection of motor torque Te and load torque TL charac­
teristics does not guarantee a stable operating point. Stable operation of the system consisting
of motor and load is possible if the motor speed decreases with an increase in load torque or
vice versa.
An induction motor is designed to operate at low values of full-load slips (from 0.02 to 005)
under normal operating conditions. In Fig. 6.28 (a), the normal operating region extending from
s = smT is shown by solid curve. The region from s = 1 to smT as shown by dotted curve in Fig.
6.28 (a) is rarely used.
In Fig 6 28 (b) motor starting torque OA is more than the load torque OB, the motor,
therefore, accelerates to the operating point C. If the load torque increases from curve TLl to
curve Tu , the operating point shifts from C to D and the motor speed drops from Oc to Od. It
can also be shown that if the load torque decreases, the motor speed increases. This leads to the
conclusion that point C is stable one.
For load characteristic TL as shown in Fig. 6.28 (c), the motor will not start, because start­
ing torque OA is less than load torque OB. Thus the question of any operating point does not
3F1S6. .
For load characteristic TLl of Fig. 6.28 (d), C is the stable operating point because an in­
crease of load torque from TLl to TL2 shifts the operating point from C to D and the speed is
thereby decreased. Such a load characteristic, however, would rarely be used under norma
circumstances, because operating point C would result in higher values of slip, high currents,
low power factor, more loss and very low efficiency.
For load characteristic TLX as shown in Fig. 6.28 (e), A is the stable operating point. , ^
to some reason or the other, the load torque decreases suddenly from TL1 to TL2, motor torque
exceeds TL2— the rotor gets accelerated till the operating point B is reached. After the pom ^
is arrived at, if TL2 again increases to TLl, the operating point C at reduced speed is obtaine^
Thus the normal operating region from s = 0 to smT is more stable than the region rom mr

s f lnfid torque
It can be concluded from above that stable operation depends on the nature ot o ,{
and motor torque characteristics and the way they intersect. It may also be se

*E k > *E± the system is stable. In case ^ , the system is unstable.


dn dn an an

Scanned by Cam Scanner


Torque
(d )
F ir 6 28 Pertaining to the induction motor stability.

Practically, the region between s = 0 and smT is used. This region is sometimes referred to
as stable one, because all types of load-characteristics in this region result in stable-operating
points Since the speed-torque characteristic in the normal operating region is like that of d.c.
shunt motor (low variation of speed from no load to full load), the induction motor is said to
possess shunt characteristics.
6.11. D eterm ination of Eq uivalent Circuit Param eters
The equivalent circu.t parameters of polyphase induction motors can be determined from
no load test, blocked-rotor test and stator winding d.c. resistance. The object of this article is to
describe the methods of determining the parameters from these tests.
la) No-load test (o r ru n n in g light test). The induction motor is made to run at no load
at rated voltage and frequency. Per phase values of applied stator voltage Vnl, input current
and input power Pnl are recorded.

The no-load -dip »„ is very small, therefore, £ in Fig. 6.14 is very large as compared to

K In view of this, the resultant of parallel branches and +y*2J .s almost equal to

35 illustrated in Fig. 6.29 (a). Thus the no-load reactance Xnl seen from the stator terminals is
^ ual to x 1 + X m i.e.
Y _y ...(6.39)
X„j - Xx + X m Xx
wW e Xx is the stator self-reactance.
From the instrument readings at no load, stator no-load impedance,
V*;
ZrJ =

rwwi’ m.

Scannea by Cam Scanner


768 E lectrical M achinery
I Art. 6.1)

*1
4 — <w/—W —r +9 W f— W T

Vnl lm
• m
"V I %

1 (a)
1 - ____________ I
(b)
Fig. 6.29. Induction motor equivalent circuits for (a) no-load test and (fe) blocked-rotor test.

nl
and stator no-load resistance, Rn[ =
hl
a Xn[ = ^ z2n l- R 2nl.
The rotational losses PR (friction, windage loss and core loss) are usually assumed constant
and can be obtained from the relation
=m fir ,)
where m is the number of stator phases and r : is the per phase stator resistance.
Thus the no-load test gives Xnl and the rotational losses PR.
(6) Blocked-rotor test. Blocked-rotor test, similar to the short-circuit test on a trans­
former, is performed on the induction motor to calculate its leakage impedance.
I For performing this test, the rotor shaft is blocked by external means, i.e. the rotor shaft is
held stationary by belt-pulley arrangement or by hand. Now balanced polyphase voltages at
rated frequency are applied to the stator terminals through a polyphase variac. This applied
voltage is adjusted till rated current flows in the stator winding. Per phase values of applied
voltage Vbr, input current (= rated current) Ibr and the input power Pbr are recorded. Current
I br may be affected by rotor position, in view of this the rotor should be held in a position that
gives I br equal to the mean of maximum and minimum current values. Measure the d.c. resis­
tance per phase of the stator winding soon after this test and multiply it by 1.1 to 1.3 in order
to obtain the per phase effective stator resistance r v
The equivalent circuit under blocked-rotor test is as shown in Fig. 6.29 (6). From the instru­
ment readings during blocked-rotor test, the parameters can be obtained as under :
y
The blocked-rotor impedance, Zbr = ——
hr
p
and the blocked-rotor resistance, R br = ~
hr
Blocked-rotor reactance, Xbr = 'Iz'br ~ Rbr-
An examination of Fig. 6.29 (6) reveals that the blocked-rotor impedance seen from the
stator terminals is given by

Z br = Rb r + jX br = T) + j x x + — m.(^2 ,
r2 + J (x2 + Xm)
= r, + ir x (rg + jx 2)[iz l + X m)
r 2 + j (*2 + X m) r2 -j (*2+ x m)
• J X ,m
= r) + ;x , + - ' h + j y 2 - > r 2 (% + X „ ) + X2 (X2 + f i„ ) l
'2 + (*2 + f i n)!

J
Scanned by Cam Scanner
Polyphase Induction Motors 769
17
r , rT _ r . h ...(6.40)
R br + jXbr - r \ + J X 1 + --------------- --------------------------
2 2
g.
or
^ s-x2 + -Xm, is the rotor self-reactance. Comparison of the imaginary components of
e-
^ i d e s o f E q . (6.40) gives
a

"m x « +X2 Vs
X m [r2 + x2Xd
Xbr = X\ + 2 ir2 = *1 +
rU X $
±
r + x 3r

r2
r2
is
Since X2 » ra, Y may be neglected. With this e,
i-
Xpi x2 Xmx2 *2
X 6r = x 1 + : = Xi + «*! +■
Xm+ x2 l-
1+

*2 .
Usually the magnetizing reactance Xm » rotor leakage reactance x2, therefore, ^ is neg
g
ligibly small and this gives ...(6.41) e
r
There is no practical methodof separating * , and * a. For wound-rotor induction motors, * ,
• 1 V
is assumed equal to x 2, i e - x \= x2 = 2 Afcr’ , , ,. . .. . aA
For squirrel cage induction motors, total leakage reactance Xbr (= * , + *2) can be distnbut
between stator and rotor as per the following table :
Table 6.1. E m p irica l D istribution of Leakage R eactan ce Xbr

Fraction o f Xj,r
s
Class o f motor *2
*1
0.5
0.5
1. Class A (norm al Te st and Ist)
0.6
0.4
2. Class B (norm al Te st and low l st)
0.7
0.3
3. Class C (high Te st and low Ilt)
0.5
0.5 -
4. Class D (high Te si and high slip)
« fa the stator magnetizing reactance is given by
Once is known, then from Eq. (6.39), tn
Xm - X ni *i- o n f vn (6 40) blocked-rotor resistance Rbr as
Now taking the real components of both sides of Eq. (6.40), bloc
8een from the stator terminals is
X2
R br —t j + r 2 ~2 . Y
o • v t'Jrherefore, in the denominator
For machines of normal design, X2 ^ r 2>
aa compared to X 2.
(X f
R bv = r x + r2
X2
V.

Scanned by CamScanner
770 E lectrical M achinery _________ ___________________________________ _

.*. Per phase rotor resistance,


( X 'f
r2 = (Xbr - r i) 3 T —(6.42)
Am
Thus Xm from Eq. (6.39), r2 from Eq. (6.42), r x from d.c. resistance per phase of stator wind,
ing and xv x2 from Eq. (6.41) and Table 6.1 can be determined from three tests. The equivalent
circuit can now be used for computing the motor performance.
During blocked-rotor test, the power input is almost entirely I R loss in both stator and
rotor windings. For low voltage during this test, the flux density is quite low and core loss is
therefore very small. For example, for 20% input voltage, the flux density is about one-tenth of
its normal value and the core loss is therefore about one percent of normal coreloss, which is
quite a small figure.
Note. For large motors (above 20 kW or so), if induction motor characteristics are required
near s = 1 {e.g., for starting torque e tc .); then since rotor frequency f 2 is equal to the line fre­
quency, the blocked-rotor test should be carried out at line frequency and with currents equal
to those encountered at the time of starting.
In case induction motor characteristics are required near synchronous speed {e.g. during
normal operation), then rotor frequency is equal to s fh therefore, the blocked-rotor test should
be carried out at reduced frequency and with normal currents.
Exam ple 6.41. A 10 kW, 400 V, 4 pole delta-connected squirrel cage induction motor gave
the following test results :
N o-load test : 400 V, 8 A, 250 watts.
Blocked-rotor test : 90 V, 35 A, 1350 watts. -
The d.c. resistance o f the stator winding p er p h ase m easured im m ediately after the blocked-
rotor test is 0.6 Cl. Calculate the rotational losses an d the equivalent circuit param eters.
Solution. No-load rotational losses are

PR = 2 5 0 - 3 x (0.6 x 1.20) = 203.92 watts.


V3
Note that the effective stator resistance per phase is taken equal to 1.2 times its d.c. value.
From no-load test,
400
Znt = = 86.6 Cl
(8/V3)
250
Rnl 3o (8/V3)
/Q /Jo \2 “ 3 91 n

••• Xnl = ' i z h - R h = V(86 .6 f - (3.91)2 = 86.51 Cl


From blocked-rotor test,
90
^br = Tnr /./rtx = 4.454 Cl
(35/V3)
n lOOU
1350 , „ ____
R br ~ ___ 1 2 “ 1-102 Q
3 (35/V3)
Xbr = V (4 .4 5 4 )" -(1.102)" = 4.316 Cl
„ i
x \ ~ x 2 ~ 2 ^ br = 2.158 Q.

and xm= Xn, - Xj = 86.51 - 2.158 = 84.352 Cl


X 2 =X,n + x 2 = 84.352 + 2.158 = 86.51 Cl.

Scanned by CamScanner
Polyphase In d u ction M o to rs 771
17
Eq (6.42), per phase rotor resistance is given by
^r°m ( y >2 5,
r2 = (R br ~ r x)

86.51 a
= ( 1.102- 0.6 x 1.2)
= 0.402 n.
84.352
Thus the parameters of the induction motor equivalent circuit are
rj = 0.72 £2, r 2 = 0.402 £2, = *2 = 2.158 £2
X m= 84.352 £2.
and
Example 6.42. (a) Explain how friction and windage loss can be computed from the no-load

test-
(b) Under norm al operation o f the induction motor, the rotor core loss is usually negligible.
Explain-
(a) The power input to the induction motor at no-load has to supply the stator
S o lu tio n ,

copper loss, core loss and friction and windage loss.


The d.c. resistance of the stator winding is measured and its per phase effective value rx is
calculated from the relation, .
= (1.1 to 1.3) (d.c. resistance of one phase).
For computing the friction and windage loss, the applied voltage to the unloaded induction
motor is varied from 1.25 times the rated voltage to about 20% of the rated voltage. The input
power, current and voltage are recorded so that a graph can be plotted. The speed, with reduc­
tion in voltage, will fall only slightly so that the friction and windage loss remains substantially
co n sta n t.
From each of the input-power readings, the corresponding stator ohmic loss is substracted
to obtain the core loss and friction and windage loss, i.e.
PR = m (P nl- I 2nlrl)
where P^ is the per phase power input, 1^ is the per phase stator current and r 1 is the effective
per phase stator resistance. The plot of rotational loss PR with variable stator voltage is shown
in Fig. 6.30. The intercept of the PR curve with APr
the ordinate gives the friction and windage
loss, because the core loss is zero for zero ap­
plied voltage. In order to get a more accurate Core loss at
value of mechanical loss (friction and windage ui fated voltage
loss), rotational loss PR should be plotted against g
(voltage)2. This plot of PR with (voltage)2 is almost -
linear and, therefore, the extrapolation is easier. c
(b) Core loss or iron loss consists of eddy-cur­
rent loss (oc B2f ) and hysteresis loss (« #*/)• At
standstill, the rotor frequency is equal to supply F.W. I oss Rated
frequency f, therefore, the rotor core loss is ap­ voltage
preciable. As the motor speeds up, the rotor fre­ JZ
quency decreases and with it the rotor core loss Voltage
le a s e s accordingly. Under normal running Fig. 6.30. Variation of rotational loss w ith voltage.
c°nditions,
'"tions, the slip is small (0.02 0.05), tne
( 0.02 to u.uoj, the
Jot<>r frequency is also very sm all; consequently the rotor core loss is very low and can, there-
ore. be neglected.

t5y“uam i)canner
772 E le ctrica l M ach in ery 1 ^ 6 ^

E xam p le 6.43. A 10 kW, 420 V. 3-phase, 4-pole, 50 Hz, delta connected squirrel cage induc
tion motor gave the following d ata for blocked rotor te s t .
210 V, 20 A, 5 kW
Stator core loss at rated voltage and frequency is 300 watts. The d.c. resistance measured
between any two stator term inals is 0.6 Cl. Determine the starting torque at rated voltage and
frequency.
S o lu tio n . With the stator winding in delta, the resistance per phase R is obtained from the
r'q^iftion

£ |u/l
| £ = 0.6 or * = 0 9 n.
Effective stator resistance per phase
= 1.2 x 0.9 = 1.08 Cl
Power input at rated voltage during blocked rotor test

= 5 x ( I o J = 20kw
Stator current at rated voltage during blocked rotor test

= 2 0 * | § = 40A

Air-gap power at rated voltage and frequency, with the rotor blocked, is given by

Pe = 20,000 - 3 ' 4 0 f x 1 . 0 8 - 3 0 0 = 17972 W


IE
Synchronous speed, (o. = = 471 * 50 = 50ti rad/sec
P 4
P 17972
.*. Starting torque =- f =- ■ = 114.413 Nm.
0)4 50n
E xam p le 6.44. B locked-rotor test on a 3-phase, 40 kW, 400 V, 50 Hz, 6-pole star-connected
induction motor gave the follow ing d a ta :
200 V, 110 A, p f = 0.4
Determine the starting torque for a 3-phase voltage o f 380 V at 45 Hz. N eglect magnetizing
current a n d assum e stator an d rotor ohm ic losses equal.

Solution. Dunng blocked-rotor test, per phase voltage, Vb = - ^ 2 = 115.5 V and per phase
current, 4 = 110 A
Total impedance per phase referred to stator,
7 _V b 115.5 ,
z ‘ - 7 r = n o - = io 5 £ i
(r i + r 2) = Zb COS e6 = 1.05 x 0.4 = 0.42 Q
(xi + x2) = Zb sin 06 = 1.05 sin (cos- 1 (0.4)] =0.9623 Cl
te that (/-j+ r2) and (jcj + x2) are referred to stator. Since stator and rotor ohmic losses are
equal, r x = r2 = - ~ = 0.21 Cl

(*i + x2) at 45 Hz = 0.9623 x ~ = 0.866 Cl


50

Scanned by CamScanner
Polyphase Induction M otors 773
17
1impedance
at 45 Hz = 0.42 + ; 0.866 ft
T 380 „ *g.
per.phase voltage at 45 Hz, V x = V
_ 380 re­
starting current V3 [0.42 + j 0.866] a
, 4n x 50 100 it , ,
Synchronous speed, wt = - = ^ rad/sec \s

• * - 3 Vj or
Starting orque ^ (impedance at 45 Hz)2 2
?e
= 3 x 3 x --------------------- x o .21 = 312.594 Nm. is
100 x rt 3 [0.42 + 0.866 ] e,
Example 6.45. A 3-phase, 400 V, 50 Hz star-connected induction motor gave the following 1-

test results:
No load : 400 V> 7-5 A > 0.135 power factor i-
Blocked rotor : 150 V, 35 A, 0.44 power factor
The ratio o f standstill leakage reactance o f stator and rotor is estimated as 2. I f the motor is
running at a speed o f 960 rpm, determine. g
(o) net m echanical pow er output (b) the net torque and e
(c) efficiency o f the motor. r
Assume stator an d rotor copper losses to be equal.

Solution. N o-load te s t. Znl = ^ 7 . 5 = 30 793 ° _


R„, = ZnI cos 0n/ = 30.793 x 0.135 = <4.157 ft _
Xnl = V30.7932 - 4.157^= 30.511 ft

B locked-rotor te st. z br = J f ^ 35 = 2 ,4 7 4 0
Rbr = Zbr cos %r = 2 4 7 4 x 0.44 = 1.09 ft
Xbr = J z Z ^ W r = ^2.474 2 - L 0F « 2.22 ft

Here ^ = 2, *i + *2 = X br or 3*2 = 2.22. This gives

2 = 0.74 a n d *, = 2 x 0.74 = 1.4811


Xm = x nl - xx = 30.511 - 1.48 = 29.031 ft
It is given that stator and rotor copper losses are eq
... r, = r2 = ^ = 0 545 0

The exact equivalent circuit for this induction E (6 22 ) and (6.23) are given
Thevenin’s equivalent circuit parameters obtained from hqs. t
below: .
V Y iJs . where x 1 + X „ = 1.48 .2 9 ,0 3 1 = 30.511 n
V‘ x ,+ X m

Per-phase stator voltage, V\ = = 230.95 V


930.95 x 29.031 = 219 75 y
V. = on *11

x j u u i ii i u u uy i i v j u u i ii
I A rt. 6 .U
774 Electrical Machinery
0 519/1 1-409/1 074A
0545A 1-49A 0-74 A

T
V,-230 95V 129-031 ^ I q-545
n rti .

Fig. 6 .3 L Induction motor equivalent circu its p ertainin g to E x am p le 6 .45

_ 0.545 x 29.031 = Q 5 1 9 f l
~ 30.511
_ 1 ,4 8 x 2 9 .0 3 1 _ j 408 n
30.511
Thevenin’s equivalent circuit is as shown in Fig. 6.31 (ft).
Rotational losses = V3 x 400 x 7.5 x 0.135 - 3 x 7.5 2 x 0.545 = 609.5 W
1000 - 960
= 1000 = 0.04
Synchronous speed = iw v rpm, slip *s -= 10oo

Rotor current at slip s = 0.04, from Fig. 6.31 (6 ), is given by


219.75
h =7 0.545
0.519 + +j 2.148
0.04

Mechanical power developed, Pm = 3 l\ r2

219.752
=3 x M r = 9258.48 W
(14.144)2 + 2.1482 0-04
(a) Net mechanical power output = P m - Rotational losses
or Psh = 9258.48 - 609.5 = 8648.98 W.
4n x 50 100 71
(6 ) Synchronous speed, cos = g rad /sec
3
P sH 8648.98
Met torque, Tsh = 0). = 86.033 Nm.
100 71
(1 - 0.04)

P m ■s 9258.48 x 0.04
(c) Rotor ohmic losses = 385.77 W
1-s 0.96
Stator ohmic losses = 385.77 W
Rotational losses = 609.5 W
Total losses in motor = 385.77 x 2 + 609.5 = 1381.04 W
Power input = 8648.98 + 1381.04 = 10030.02 W

••• Efficiency = Power


Ciinciency power outPut _- I1003() Q2 x inn
648 98 100 - 86 23%
86.23%.

E x a m p le 6 .4 6 . A 230 V, 20 h.p., 60 Hz, 6-pole, 3-phase induction m otor driving a constant


torque lo a d a t rated frequency, rated voltage a n d rated horse pow er, h a s a speed o f 1175 rpm
an d an efficiency o f 92.1%. D etermine the new operatin g sp eed i f a system disturbance causes
10% drop in voltage a n d 6% drop in frequency. A ssum e th at friction, w in dage an d stray
losses rem ain constant. [GATE, 2WV

Scanned by CamScanner
r
Polyphase Induction M otors 775

Solution. S ynchronous speed = M p O = mQ ^

s _ 1 2 0 0 - 1175 J _
SUP. 1 1200 “ 48
prom Eqn. (6 -3 0 ). ra te d to rQu e is given by

T, . i l S . l - .1 3 Vf 1
- - .A
ws r 2 48 co r,2
With reduced voltage and lowered frequency, we get
Qf(\ O T 7 \2 .
Ti r = s 2 ^ f ^ ± . l . A . ± 3V J i
0.94 o)s r2 48 ‘ co, ^
c 0.94 A
or 2 0 .8 1 x 4 8

New operating speed = — ° X 0g94 x 60 (i _ 0.0242) = 1100.70 rpm.

6.12. Circle D iagram


Induction motor circle diagram is a graphical representation of its equivalent circuit. This
means that whatever information can be obtained from the equivalent circuit, the same can also be
obtained from the circle diagram. The advantages of circle diagram are its simplicity and quick
estimation of the induction motor operating characteristics. Though a computer can rapidly provide
the performance characteristics of a 3-phase induction motor from its equivalent circuit, yet the
graphical solution through the use of circle diagram is even now popular in several organisations
It is because a circle diagram
ii) gives the results which are sufficiently accurate for practical purposes, despite the fact
that an approximate equivalent circuit is used in a circle diagTam,
(ii) provides a panoramic view of how operating characteristics are affected by changes in
the machine param eters, voltage, frequency etc.
A circle diagram shows that the extremities of stator current phasor follow a circular locus
as the slip varies from + °° to - » . In this article, first the development of circle diagram from
|s approximate equivalent circuit is presented. Then the use of this circle diagram for obtain­
Ing performance calculations is described.
(a) C u rre n t lo cu s fo r th e r o to r c irc u it. The rotor circuit voltage equation in phasor form
can be written, by referring to its equivalent circuit of Fig. 6.7, as

^ 2 = ^ 2o~ +J^ 2 x2 ....(6.43)


or E2 - - r2/ s
— = Io + lo ~
JX2 * 1 JX2
or Eo — — Vo
- jJ — = h - j h z r ...(6.44)
x2
I p ^ ^
9- (6.44), —. denotes a constant current ; - y — implies that this current lags E 2 by 90°. This
. x2 x2
0Wn ^ P h a s o r diagram in Fig. 6.32, where E 2 is taken along the vertical axis and - j — lags
P l a‘2
2by 9o° Tf ■ i /?o ,
•u is also seen from Eq, (6 .44 ) that current - j — (= OB) is made up of two components ;
x2
h

[A rt. 6. 12
i: 776 E le c tric a l M ach in ery

(i) / 2 (= OA) lagging E 2 by an angle 0 2 as per Eq. (6.43)

and (u) - j / 2 — (= OC) lagging / 2 by 90°, Fig. 6.32.

5X2 (s * 1
Here 0 2 = rotor power-factor angle = t a"H — I-

Now draw a line AB parallel and equal to OC. Since


angle OAB is equal to 90°, the point A must lie on a circle
with OB as diameter. This shows that the locus of the tip
of rotor-current phasor / 2 is a circle with diameter equal
E c
to —-. Since stator current is equal to the phasor sum of
x2
constant no-load current and referred rotor current, the
tip of the stator current phasor must also trace a circle. g ^ Current locU8 for the rotor circuit of a
This forms the basis for the induction motor circle induction motor,
diagram.
(b ) C ircle d iag ram from th e a p p ro x im a te e q u iv a le n t c i r c u it . The shunt branch co
sisting o f R c &ndXm in parallel, see i
A , A 2

/Tnflfi rmnr\
Fig. 6.13 (a), may be connected across + o— ------ -v\W-
the supply terminals, resulting in the f I-
approximate equivalent circuit of in­
V,
duction motor as shown in Fig. 6.33. R es i i *
J *<*>
This circuit neglects the stator leakage r t
C 1
impedance drop due to Ie and is used
* ------------------ 1 > — ■
mance calculations, because it leads to Fig g 33 Approximate equivalent circu it of an induction motor,
simplicity and reduced labour.
The per phase voltage Vx sends a cu rren t / 2 through the series circuit consisting of

r x x lt — and x2.
s
'
. r 2N+ j (*i + x2) ...(6 .4 5 )
y
The current I 2 lags Vx by an angle
« 1 x \+ *2
02 = tan ' 1 — -
r,
r' 7
Eq. (6.45) can be re-written as,

y rx+ —
-J X, -hX
= 72- J I 2 -
* l +*2

V r +
As before, - j — —l lags V, by 90° and ~ j l 2 — A lags j 2 by 9Qo and 7j lags by e2 where

02 = tan 1^ ~ T 7 This is ilIustrated in Fig. 6.34 (a), where Vj is taken along the vertical axis
ri + ~

Scanned by CamScanner
Polyphase Induction M o to rs 777

r \+ ~s_
is the diameter of the rotor-current locus, OA = I 2 and AB = OC = I 2
tfere OB = X i+ X2 * 1 +X2'
dstill, for V\ applied voltage, I 2 (= OD) becomes very large and angle 02 increases to
At st»ni 7* "4"
-1 This is also shown in Fig. 6.34 (a) in which DB = I2 —— -
tan rj + r2 xl + x2

(a) (b)
Fig. 6 .3 4 . Induction motor circle diagram (o) with shunt branch neglected and
(b) effect of shunt branch included.

In order to include the effect of shunt branch in the circle diagram, draw magnetizing current
V V\ . . .
Im = - r - = OQ' and core-loss component of current I c = Q'Q = ■Then OQ is the exciting current

m . C . . ( Vi ]
Ie. The rotor-current locus diagram of Fig. 6.34 (a) is repeated again with QB ~ x ^+ ^ the

diameter. In this manner circle diagram of Fig. 6.34 (6) is obtained from the approximate
equivalent circuit. Here QA is the rotor current I2, OA is the stator current I x and cos is the
operating power factor at stator terminals.
Note that QQ' x Vlt in Fig. 6.34 (b), represents the core loss only. In order to take into
account the core loss, friction and windage loss at no load operation of the induction motor,
length QP in Fig. 6.35 should be made equal to
No-load induction motor loss per phase
Per phase Vi
The circle diagram of Fig. 6.34 (6 ) is re-drawn in Fig. 6.35 on a larger scale for the sake of
clarity. From this circle diagram, the following observations can be made .
(0 On no-load, I 2 = QN = 0 , therefore, point Q indicates no-load operation.
(QP) (V2) = No-load input to stator. Note that QP x Vx is equal to the friction and windage
loss. core loss and negligible amount of I 2R loss. OP is the magnetizing current and 60 is the
load power factor angle, obtained from the relation
_ l No-load power input
0 “ cos V3 (No-load current)
Current I 2 = QN is the rotor current referred to stator and I\ is the total stator current.
Auction motor operating power factor is given by cos 6 i-

Scanned by CamScanner
778 E le c tric a l M a ch in e ry [A rt. 6.12

Fig. 6.35. Induction m otor circle diagram . S ta to r cu rrent-locu s shown by full-line (from point Q, N, D and
up to point S = « ) is for the induction motor and th at shown by dotted line (from point S = ° c , R ,M and up
to point Q) is for the induction generator.

OD is the short-circuit current for Vl applied voltage and with the rotor at standstill. Under
short-circuit at rated voltage, the power input is lost as I 2R loss in stator-rotor conductors and
core loss in stator-rotor irons. In Fig. 6.35, the power input under short circuit is
Vx •OD cos 0SC = Vl I sc cos 0SC = DE.
In this figure, E F = QP represents the friction, windage and core loss. At standstill E F may
be taken to be equal to short-circuit core loss, because the absence of friction and windage loss
un er short circuit is practically compensated by increased rotor core loss at standstill. In other
words, E F - Q P - (stator core loss + friction and windage loss) a t no load = (stator c o r e
oss + rotor core loss) at standstill Under this approximation, E F represents the per phase core
loss and DF gives the per phase I 2R loss in stator-rotor conductors at standstill.

Now divide D F so that = rotor ^ & 1°8S j • , „


OF stator j 2R 1oss • oint QO. For any operating point N, draw

phas„er T diCU' ar ‘ ° ^ h° riZOntal 11,16 ° X N° te that and ■ " Perpendicular to voltage

(u) S ta to r and ro to r I 2R losses. From similar triangles QKH and QFD we get
h k = QK ’
DF QF
In right-angled A K
Q
NQK
; = QN cos [N Q K and in righhangled A;= cos /DQF.
... HK QK QN cos [N Q K
^ QF = QD cos /D Q F .-(6.46)
From right-angled AQNR,

cos /N Q K = cos /N Q R = Q K
■ Q R

b t a n n e d "b"y” C a m S c a n n e r
r Polyphase Induction M o to rs 779

' N„w points ,R are jointed together as shown in Fig. 6.35. In the right-angled trian
D

qDR
cos [D Q F - cos [D Q R =
QR
Substitution of the values of cos /N Q K and cos /DQF in Eq. (6.46) gives,
QN)
QN
HK QR
_ = =

DF qD (Q » ) (QD)2 ( 4 )2
[ QR J
Multiplying the num erator and denominator by (r, + r2) of the right-hand side of above
expression, we get
H K _ (QN)2 (rt + r 2) _ (/2f (ri + r2)
(QO)2 (r 1 + r 2) " ( / sc)2 (r 1 + r2)
This shows that i f DF, on the basis of power scale, is the stator and rotor 12R loss at
standstill when the current is QD, then H K must represent, on the same scale, the stator and
rotor I2R loss when the current is QN.
Thus it can be concluded that if DF represents stator and rotor I 2R loss for a current of
QD, then H K must represent on the same scale the stator and rotor I 2R loss for a current of
QN. Further the triangles Q K J and QFG and also the triangles QJH and QGD are similar,
therefore, K J = stator I 2R loss at current QN and J H = rotor I 2R loss at current QN.
(iii) E fficien cy a n d slip. In Fig. 6.35, N L represents the total power input. Since KL rep­
resents friction, windage and core losses and KH the total stator and rotor ohmic losses, NH
must represent the power output, i.e. the shaft power Psh.

, Per unit efficiency = P ° Wer ° UtP f = ^ ...(6.47!


Power input NL
The line QD on which the power output point H lies, is called output line. In other words,
the locus of all power-output points such as H, lie on the line QD called the output line.
^ Rotor I 2R losses _ J H ...(6.48)
^ Rotor input JN
(iv) T orque. Torque,
Rotor input or air-gap power, Pg
T =
Now NL represents the total power input and J L the total stator losses. Ther
W (= NL - J L )represents the rotor input or air-gap power. Torque T, .s proport.onal to
power N J. The locus of point J as the operating point N moves around the circle, is t re me
(JG-this line is, therefore, called torque line. Starting torque is given by D a -
(")The maximum output power is obtained by drawing fromi ther“ nt^ ^ ^
Porpendicular CA to the output line QD. see Fig. 6.35 (a). Now A/1, drawn normal to CM, id
rescnts the maximum power output to scale. . , r norn„nfuCi,iar
CB torciue is ol>taino<l by drawmB fr0"j ‘gives tlio maximum torquo to scale.
CB‘o the torque line QG. Then flfl, drawn normal to OA, gives me mi h
obt normal operuting region of the motor is nonr l',nxia” la’ ■’ . j , , current cor-
°btained by drawing a tangent from 0 to the circle. In big. 6.36, / , is >
tfiP0nding to the maximum power factor cos 0,„.

Scanned by CamScanner
! I

ii
I.
I

'•I ,
Fig. 6.36 Circle diagram from no-load and blocked-rotor testa.

If the rotor speed is made above synchronous speed, the induction machine starts‘ working
as an induction generator for which the stator-current locus is showni dotted‘ ^ _6^ 5.
electrical power output is ML to scale, cos 0G is the operating power factor and I (- OM) is the
output current.
Power across the air gap = ML + Stator loss L J = M J.
Mechanical input from prime-mover MH.

rotor input (= 2>'G') is less than rotor ohmic loss Z G . Braking region m the circle diagram ot
Ff g 6 35, is indicated by part of the circle between points D and s = ~. Note that for s = ~, rotor
current is zero and, therefore, rotor I 2!? loss is zero.
(c) C ircle d ia g ra m from te st d ata. The circle diagram depicting the locus of stator cur­
rent, can be drawn by using the test data obtained from no-load and blocked-rotor tests and the
stator resistance.
No-load and blocked-rotor tests are conducted as explained in Art. 6.11. The rated votage
. , n ii nkoco vnlupR fnr no-load test. Current/,./ - OQ is drawn

...(6.49)
c°s e° VZl.
nl *nl
- 1 nl
0O= cos
V„,I.
nl nl
Note that OQ in Fig. 6.36 is drawn to a suitable scale.
In blocked-rotor test, the applied voltage V * is only a fraction of rated v , f S ^ d ’with
ed-rotor test data Vbr, I ir and Pbr is converted to values that would have been
rated applied per-phase voltage Vv

Xl
1 ac ~ ^ b r t r
'br
V il
P .c = P tr I Vbr
.(6.50)
„ p A$v
cos 8., - v i V7T.

Scanned by Cam scanner


Polyphase In d u ction M o to rs 781
Art. 6.12] ------------------------------- -

Now O D = I,c is drawn making an angle of 0SCwith V,. Points Q and D on the current loc
are therefore, determined. Lines QR and OX are drawn perpendicular to phasor V,. Kignt
I,
bisector of the line joining points Q, D meets QR in C which is the centre of locus-C]rcle_Wl
ifc:
c as centre and CQ as radius, the circle is drawn. Now draw D E perpendicular to OX. Divide

DFr at G so that 7Qp


^ = —, where is the effective per phase stator resistance and r 2 is the per
u f^
phase rotor resistance referred to stator. Now for any operating point P on the circle diagram,
the entire performance can be obtained by drawing a perpendicular dP on the line OX. Thus for
_ ^ a a ■ . n J __ . 71 m a A- a
1 s~ \ A 1 OO 1
the operating point P in Fig, 6.36, input current = OP, power output = a P , rotor-ohmic loss
= ab, stator ohmic loss =bc, friction-windage and core loss = cd, power input = dP , torque
= slip = power factor = ^ and efficiency =

If current scale is 1 cm = x amperes, then per phase power scale is 1 cm = xVl watts, where
Vj is the per phase voltage.
Example 6.47. A 15 kW, 400 V, 4-pole, 50 Hz, 3-phase star-connected induction m otor gave
the following test results :
No-load test B locked-rotor test
Line voltage 400 V 200 V
Line current . 9A 50 A
Powerinput - 1310 watts 7100 watts
Stator and rotor ohmic losses at standstill are assum ed equal.
Draw the induction motor circle diagram and calculate
(a) line current, power factor, slip, torque and efficiency at full l o a d ;
(b) maximum possible power factor and the corresponding line cu rren t;
(c) maximum power output and maximum power in pu t;
(d) slip at which maximum torque occurs, maximum torque and
(e) starting torque.
Solution. From no-load test, no-load power factor is

C0S 0“ = * 4 0 0 x 9 = 0 21
•• 0O= 77.88°
No-load current I0 = 9 A.
From blocked-rotor test, short-circuit power factor is
A 7100
C0S sc “ V3 x 200 x 50 41
•'* . 0SC= 65.8°.
Short-circuit current at rated stator voltage is

Is c = 200 * 5 0 = 100 A -

The circle diagram can now be drawn \yith the help of above data, as follows :
Scale 6 A = 1 cm.

Draw OQ = 9/6 cm at an angle 9„ = 77.88“ lagging Vh and draw = M cm at an ang[e


— “ *■ • " n at an angle
O f e„ = 65.8“ behind V,. Join Q with D to get the output line QD. The right-bisector of QD meets

■g j U I I I U U U I II I V
782 Electrical Machinery [A rt. 6.J 2

Q- O -e ^
Fig. 6 .3 7 . Induction m otor circle d iagram , E xam p le 6 .4 7 .

QR in C and this is the required centre of the circle diagram. Note th at OX and QR are drawn
perpendicular to phasor Vx. With C as centre, draw a circle with CQ as radius. From D, draw
DE perpendicular to OX. Since stator and rotor ohmic losses are equal, make FG = GD. Join
G with Q to obtain the torque line GQ, Fig. 6.37.
(a) Per phase power scale = xVx watts per cm.

Here Vx = volts
and x = 6 A/cm.

.*. Per phase power scale = x 6 = 1385.7 watts/cm.

Full-load power per-phase = = 5000 watts.


O
5000
In order to get the operating point P, extend FD to D'. Make DDf = = 3.61 cm. Now
1385.7
draw U P parallel to the output-line QD. Join P with O.
Full-load line current = OP = 6 x 6 .0 5 = 36.3 A
.*. Full-load power factor = cos ZOPd
= cos ZPOVx = cos 29.5° = 0.87 lag.
Full-load slip _ _ 2.5
= 0.065.
~ b P ~ 38.5
Full-load torque = — (bP ) (per phase power scale) x 3

(3.85) (1385.7) x 3 = 101.89 Nm.


5 0ti
_aP 3.6
Full-load efficiency = 0.809 or 80.9%.
dP ~ 4.45
(6 ) In this example, OP (per chance) turns out to be tangent to the circular-locus. Therefore,
maximum or best possible operating power factor is 0.87 lag and the line current is 36.3 A.
(c) For maximum power output, draw CA perpendicular to output-line QD. From A draw
AA' perpendicular to QR. Then maximum power output is given by
AA' x Per phase power scale x 3 = 5.3 x 1385.7 x 3 = 22.03 kW.

Scanned by CamScanner
A rt. 6.12]
Polyphase Induction M otors 783

For maximum power input draw C C i x ™ .


- r r + Friction-winriapp nnri ^ ’ 1 PerPendicular to QR. Then maximum power input
' „ g core-losses. But C C is equal to the radius of circle
Maximum power input = 8.45 x 1385.7 X 3 +1310 = 36.437 kW.
QB. The” maXlmum tor<lue' draw CB perpendicular to torque line QG. Draw BB' normal to

Maximum torque = A (BBO x (Per phase power scale) x 3

= 5 0 ^ (6-65>(1385-7)(3) = 176Nm.
Slip at which maximum torque occurs
_ fU_ _ 1.58
BB' ~ 6.65 ~ 238-

(e) Starting torque, T ,,, = i (DG) (Per phase power scale) x 3

= 5 0^ (3 -3° ) ( 1385.7)(3) = 87.33 Nm.

f o l l ^ r e Z f 45^ 400 V’ M *** — induction motor gooe the


No-load test (line values) ; 400 V, 4.2 A, 480 W.
Blocked-rotor test (line values) : 215 V, 15 A, 1080 W
Rotor resistance referred to stator is 1.2 £2 per phase.
standstill are assum ed equal and stator to rotor phase turns ratio is 2/1.

orderto o b t a in ^ aCt‘ rnal resistance that >>e inserted in series with the rotor circuit in

(a) maximum torque at starting and


(b) 1.25 times the full-load torque at starting. ,
For both parts (a) and (b), find the corresponding stator currents and power factors also.
I f the induction motor is m ade to work as an induction generator with output current equal
to motor full-load current, then find
(c) its pow er output, operating power factor and efficiency.
Solution. From no-load test, no-load power factor is
4 8 0 /3
cos 0„ = ; = 0.165 0O= 80.5°.
f4.2
400
V3

From blocked-rotor test, short-circuit power factor is


1080/3
cos 0SC= = 0.1933 0^ = 78.86°.
15
215

Per phase short-circuit current at rated voltage


400 15 .IC .
—— x~rz= 16.11 A.
215 V3

Scanned by CamScanner
[Art. 6.12
784 Electrical Machinery

4.2
Per phase no-load current = = 2-42 A.

Scale 1 A = 1 cm.
Draw the circle diagram as explained in Example 6.47.
(a) In order to get the maximum torque, draw CB perpendicular to the torque line QG.
Draw B B ’ perpendicular to OX. Then BB' gives the maximum torque, fig . b.d«.

Fig. 6.38. Induction motor and g enerator circle diagram . E xam p le 6 .4 8 .

The maximum torque at starting can be obtained only if the operating point at starting is
B (and not D). At this operating point B, the internal rotor resistance consumes a loss equal to
fB'. The remainder of the loss fB (at starting) must be consumed by the external resistance to
be inserted in the rotor circuit.
External resistance to be inserted _ Bf_ _ 4.6
Internal rotor resistance B 'f 1.25'
.*. External resistance to be inserted
4.6
x 1.2 = 4.416 Q.
1.25
External resistance referred to rotor (or actual value of the external resistance)
4.416
= i.i0 4 a .
(2 )
Corresponding to maximum torque at starting, the per phase starting current is OB = 11.24 A or
the starting line current is 11.24 x V3 = 19.47 A. The power factor is cos ZOBB' = cos 45.5° = 0.701
lag.
4500
(6 ) Per phase power scale -x V j - 1 x 400 = 400 W/cm. Full-load power per phase =
1500 watts.
In order to get the full-load operating point P and thus the full-load torque Te fi extend FD

to D’ so that DD' = = 3.75 cm. Now draw D’P parallel to the output line QD. The full-load
torque Ttfl is proportional to bP. Now extend bP to M so that 6Af = 1.25 bP. Draw MN parallel

Scanned by Cam scanner


A rt. 6.121 Polyphase Induction M otors 785

to the torque line QG. Draw NN’ normal to OX. Now N is the operating point at starting so that
1.25 times the full load torque is obtained. As in part (a), internal rotor resistance consumes a
loss equal to N'K and the remainder KN is consumed in the external resistance.
External resistance to be inserted
NK — .
= x Kotor resistance
2.9
= — x 1.2 = 1.657 Q.

External resistance referred to rotor


1.657
= 0.414 n.
(2)
Corresponding to operating point N, per phase starting current is ON = 14.35 A or the start­
ing line current is 14.35 V3 = 24.85 A. The operating power factor is cos ZONN’ = cos 58.3° =
0.525 lag.
(c) Per phase output current when working as an induction generator
= OH = 5.35 A.
Induction generator p.£= cos 0G= cos 41.3° = 0.75 leading.
.'. Induction generator output
= 3 x 400 x 5.35 x 0.75 = 4815 W = 4.815 kW.
Induction generator efficiency

=^P = = 0.80612 or 80.612%.


Ha 4.90
Exam ple 6.49. A 150 kW, 400 V, 8-pole, 50 Hz, star-connected induction motor has the
following impedance param eters in ohms per phase referred to stator.
r1 = 0.02 Q, r2 = 0.04 Q, x} = x2 = 0.2 O, Xm = 9.8 Q.
At rated shaft output, it runs with a slip o f 4 percent and with an efficiency o f 93%.
(a) Calculate the rotational and core losses at rated load from the data given above.
(b) I f this m otor is driven as a generator at a slip o f 4 percent, determine (i) electric power
output (ii) p f at the generator terminals and (iii) efficiency.
The induction generator is connected to a distribution system o f 400 V.
Solution, (a) From Fig. 6.14,
0.04
+ j 0.2 (79-8)
0.04 - 1.96 + j 9.8 1 ->10
Z,= i+ > io x i-jio
1 + j (9.8 + 0.2)

Rf + jXf = 96,° 41q / 2~ = ( ° 951 +J°-291) O


' T] + j x x = 0.02 +./0.2

Total input impedance,


Z = 0.971 + >0.491 = 1.088 726.824°
400
Stator current, = 212.27 A
, r f x 1.088
Air-gap power, P .= 3 1 % = 3 (212.27)2 x 0.951 = 128552.1 W

Scanned by CamScanner
786 E le c tric a l M ach in ery ( A r t 6.13

Total stator I 2R loss = 3 (212.27)2 x 0.02 = 2703.5 W


Total rotor I 2R loss = sPg = 0.04 x 128552.1 = 5142.1 W

Total losses output = - 1 150,000 = 11290.3 W


0.93

Total rotational and core losses


= 11290.3 - (2703.5 + 5142.1) = 3444.7 W
(b) When slip s = - 0.04, from Fig. 6.14,
0.04
+ j0.2 (j9.8)
0.04
- 1.96 -7 9 .8 - 1 —7*10
Z ,= " - 1 + jl0 - 1 -> 1 0
1 +710 - .

R f+ jX f = 9- 6 ° ' W 2 9 4 = - 0.951 +>0.291

r j + jx x = 0.02 + >0.2
Total impedance, Z = - 0.931 + >0.491 = 1.0525 /1 5 2 .2 °
. 400
/1 = -nr- x = 219.43 / - 152.2
11 ~ W X 1.0525 /1 5 2 .2
Stator output current = 219.43 /2 7 .8 °
Power factor at machine terminals = cos 27.8° = 0.8846
(i) Electric power output = Vjf x 400 x 219.43 x 0.8846 = 134478 W
(ii) Regarding 134478 W as output power, the induction generator works a t a leading pf of
0.8846.
(Hi) Air-gap power = 3 (219.43)2 x 0.951 = 137370.6 W
Total stator I 2R loss = 3 (219.43)2 x 0.02 = 2889 W
Total rotor I 2R loss = sPg = 0.04 x 137370.6 = 5494.8 W
Total rotational and core losses = 3444.7 W
Total losses = 11828.5 W
Mechanical power input to drive the induction generator
= Power output + total losses
= 134,478 -i-11828.5 = 146306.5 W
Efficiency of induction generator =— x 100 = 91.92% .

6.13. P o w er F a c to r C ontrol of T h ree P h a se In d u ction M otors

motors may be light-loaded for a p art of their operation timT rh g SmCe mduct,on
factor is very low. The reasons th at an induction m o to ^ ltaT O o I r J °P eratin« Power
for any value of the load, are as follows : always operates a t lagging power factors

appliedTvholtm
agfbey 9 0 " g CUrrent’ ^ Pr° duCti° " of m agnetic flux, lags the

reaC,tanCeS inCrease tha p— * « * ^


decrease the operating power factor. onsequently the effect of leakage reactances is to

Scanned by CamScanner
A rt. 6 .1 3 ]
— ------------ Polyphase Induction Motors 787

At present, major industrial and agricultural loads consist of polyphase induction motors '
operating at average low power factors. In order to utilize the distribution and feeder circuits .
be er an o ecrease e osses in these circuits, a larger operating power factor is essential.
The important methods of power factor control of polyphase induction motors are :
T ? ( stat*c c a Pac'f ° rs across the stator terminals of both the squirrel cage and
wound-rotor induction motors and
(6 ) by the use of auxiliary machines for wound-rotor induction motors.

sideredthiS artid e’ ^ firSt m6th° d °f P°Wer fact° r COntro1 of A u ctio n motors is con-

npross^ator t e r r n iS « &f *t0rS^ metllod>which involves the connection of static capacitors


T <?mallpr f **1, lon motor, does not alter the inherent motor characteristics,
economical controllmg the power factor by static capacitors is the simplest and most

fi qqfaVpS 6 SwM f static capacitors across the stator terminals, refer to Fig.
6.39 (a). Fig. 6.39 (6 ) shows that without the use of static capacitors, the stator current is 7, and the
motor operating power factor is cos Gj.
When capacitors are connected across Vl Jl
stator terminals as shown in Fig. 6.39
(c), the cu rre n t Ic through the
capacitors lead the voltage Vl by 90°
(assumed ideal capacitors). The phasor
sum of 11 and I c gives the current /j'
drawn from the supply. In Fig. 6.39 (6),
stator leakage impedance drop is ig­
nored for the sake of simplicity. The
power factor of the combination (a) (h)

(capacitors and induction motor) im­


proves from cos to cos and With c a pa cito rs
stator current decreases from 7Xto I { .
The degree of power factor cor­
rection at different loads can be ap­ W ithout
capacitors
preciated w ith th e help of circle
diagram. In Fig. 6.39 (c), the stator
current locus of the induction motor
is shifted to the left—this shift being
equal to the length of the current • . . (?}
Fig. 6.39. Pertaining to the induction m otor power-factor
phasor Ic. This means that the centre control by sta tic capacitors.
of current locus shifts from C to C
such that length CC' = 7C. It is seen from this current locus that with fixed value of capacitance
the degree of power factor correction is different at the various loads. Fig. 6.39 (c) reveals that
if full-load power factor is nearer to unity, the power factors at no load and half full-load are
leading. The leading power factor under certain condition may result in rise of the stator ter­
minal voltage, because the transformer supplying this induction motor may have negative volt­
age regulation at these loads.
Sometimes the switching on of induction motor and capacitor as a unit may result in self­
excitation which may cause serious stator over-voltages or excessive transient torques. The
self-excitation of the induction machine can be avoided by limiting the size of the parallel

Scanned Dy uam ^canner


m [Art ^
788 Electrical Machinery_________ _ _ ___________ _________ — --------- ----------------------------------
capacitors. A still bettor way of avoiding self-excitation is to adjust the capacitance with load
either manually or automatically. nhn** Ann v sn u
E xam p le 6.60. An induction m otor operoti"g f ™ * ' d i l a t e the p e r p h a se value o f
Z ^ ta T a n d [ Z Z I vaI ^ Z 3-phase ^ a c i t o r b a n k uthich, tvhen connected to the
motor terminals, would improve the line pow er factor to O.V ag.
Solution. It is not given whether the induction motor J L T u T ed m 's'ta^nd
the case with 3-phase capacitor bank. Here the indue ion m connected The above
the capacitor bank is taken to be first star-connected and t!hen de t < a b o v e
calculations are again repeated with the induction-motor sta o ri • >
to Fig. 6.39 (6 ).
(a) S ta to r in sta r. Per phase current,
J, = 10 A.
With phase voltage as reference, the current in phasor from is
7j = 10 ( 0 .8 - j 0 .6 ) = 8 - j 6 = oa - j ab.
When the power factor is improved to 0.9 lag, current is I lt but its real component oa is
again 8 A. Thus the magnitude of I f is given by
Real part o f /, 8
1 cos e x 0.9
In phasor form, I f = 8.89 (0.9 —j 0.436) = 8 —j 3.880 = oa —j ac.
Examination of Fig. 6.39 (6 ) reveals that reactive component of current to be neutralised by
capacitor is cb.
... cb = ab - ac = 6 - 3.88 = 2.12 A.
The capacitors must supply this current of 2.12 A.
(i ) If capacitor bank is star-connected, then
Ic = 2.12 A

id voltage across each capacitor = V.

Capacitive reactance = = >/3x^2 12 =

.or C- •— = 29.22 |iF.


2n x 50 x 108.94 ^
Total kVA rating of the 3-phase capacitor bank

= 3 /c x ^ = 3 x 2 . 1 2 x ^ = 1.469 kVA.

(ii) When capacitor bank is delta connected,


let C be its capacitance per phase. Problems
involving balanced star-load in parallel with delta-load should be solved by converting (i) star­
load into an equivalent delta or (ii) delta load into an equivalent star. Then the problem is
solved on per phase basis.
As stator is connected in star, it is convenient to convert delta-connected values to
equivalent star for this part.
Capacitive reactance for delta-connected bank,

X - - i - '
A 2 nfC

Scanned by CamScanner
A rt. 6 .1 3 ]
Polyphase Induction M o to rs 789
Star-equivalent of delta-connected capacitor bank

X Y= X 1
~a =
3 2nf(3C)

Per-phase voltage across 7-connected bank = - 2 2


V3
Per-phase current, / r = 2.12 A

Per-phase reactanceXv = - 400 _ 1


^ 3 V 2 J 2 ^ 5 0 7 3 C)
or c _ V3~x2.12__
400 x 2tc 50 (3) ~ 9‘74
Total kVA rating of 3-phase capacitor bank

—Qy O 1Qv 1
¥ X 1M0 = l-469 kVA
(6 ) S ta to r in d elta. Per phase current
r 10
m =^g = 5.77 A.
In phasor form, / , = 5 .77 (0.8 _ j 0 6 ) = 4 62 _ . 3 4(J
When power factor is 0.9 lag, then magnitude o f/,' is
r , 4.62
7l = " o T = 5 1 3 A -
In phasor form, /j ' = 5.13 (0.9 - 0.436) = 4.62 - j 2.24
As in part (a) cb = a b ~ ac = 3.46 - 2.24 = 1.22 A.
(i) For star-connected capacitor bank of capacitor C per phase
1
X r = 2nfC
Delta-equivalent of star-connected capacitor bank
3

Per phase voltage across delta-equivalent bank = 400 V


Per phase capacitor current = 1.22 A
400 3
X —
c 1.22 2 k x 50 x C

Total kVA rating of 3-phase capacitor bank

= 3 x 400 x 1.22 x = 1.464 kVA

(ii) For delta-connected capacitor bank, Ic = 1.22 A


and voltage across each capacitor bank = 400 V
X _ 400 _ 1
c 1.22 2 k x 50 x C

Scanned by CamScanner
__________________________ [Art. 6d 3
790 E lectrical M a c h i n e r y ______________________ __________ '

r - l 2 —---- = 9.708 |iF


or u 400 x 2k (50)
Total kVA rating of 3-phase capacitor bank
= 3 x 1.22 x 400 x y ^ = 1-464 kVA
f ftf nnwer factor of a given 3-phase induction
It is seen from above that for the improvement P ^ motor is ^ star or delta. Also
motor, the kVA rating of capacitor bank is the ^ capacit ance bank is one-third of that
the value of per-phase capacitance C delta-connected c p
f o r

required for star-connected caPac‘ta™e * „ star-connected induction motor works at



Exam ple 6.51. A 3-phase, 500 kW,3 .3 kV>5 ’ citor bank is used toraise thefull-load
a full-loadpowerfactor of0.7lag. Adel °n fthe bank If each capacitor is rated at 420
power factor to 0.9lag. Calculate the ^
h
r*T
V, 50 Hz, compute the capacitance o f each unit. ^ ^ saving in the
e n e ^ ^ Z ^ f T Z o i o e r f a c t o r is improve, from 0 7 to 0, lag.
Solution. Motor per phase input current
r _________ 206---------- -= 1 4 5 .3 2 A.
1 V3 x 3.3 x 0.7 x 0.86
With per phase stator voltage as the reference phasor, the current in phasor form is
7j = 145.32 (0 .7 - j 0.714) = 101.72 - j 103.76.
Magnitude of per phase motor current I f as the power factor is improved to 0.9 [refer to Fig.
6.39(6)],
Real component of I x 101.72 _ . ^ ^
7i/= 0^ " 0.9 ‘ ' ;l
In phasor form, I f = 113.02 (0.9 —j 0.436) = 101.72 —j 49.28. ;
.-. Reactive component of per phase current that capacitor must supply = Reactive com­
ponent of I 1 - Reactive component of I f
= 103.76 - 49.28 = 54.48 A.
Here the induction motor is star-connected and the capacitor bank is delta-connected. Con­
verting delta-connected bank of per-phase capacitance C to an equivalent star-connected bank,
we get

x L _ |
Y 3 27c f(3C)
r> phase
Per i u
voltage = 3300

Per phase current = 54.48 A

Per-phase reactance XY= |

n V3 x 54.48 „ I
0r 3300 x 2te x 50 x 3 ^ !
Now each capacitor is rated at 420 V, therefore, 8 =
330( such capacitors w ill have to be
420
connected in series.

Scanned by CamScanner
A rt. 6.14] _ _ ____________________________________Polyphase Induction M o to rs 791

.*. Capacitance of each capacitor = 30.34 x 8 = 242.72 |iF.


Let the resistance of the distribution circuit be R ohms per phase.

Then, power lost in the distribution circuit without capacitor bank = 3 (145.32)2 R watts.
Power lost when the capacitor bank is installed
= 3 (113.02)2 R watts.
.-. Percentage saving in the losses

_ 3_[145.32)2 R - 3 (113.02 )2 R |n ,
3 (145.32) 7? X 100 = 39'51%-
In case the capacitor bank is star-connected, then it can be computed that the capacitance
of each bank is 30.34 x 3 = 91.02 pF. Five such capacitors 3300
would have to be con-
~ y/3 x 420
nected in series and capacitance of each capacitor should be
91.02 x 5 = 455.10 pF.
6.14. Starting of Polyphase Induction Motors
It is seen from Fig. 6 18 that a 3-phase induction motor has positive finite starting torque
7^, when slip s - 1.0 . This means that 3-phase induction motor is a self-starting motor and
begins to rotate on its own when connected to a 3 -phase source
At the instant of starting, a 3-phase induction motor behaves like a transformer with short-
Clr<^ 1T 17' CoTnse(luently* a 3-phase induction motor takes high starting current if
started at full voltage In order to limit this high starting current to reasonable values, some
methods are adopted for the starting of 3 -phase induction motors.
In this article, various methods of starting polyphase induction motors are described. First
the methods of starting squirrel-cage motors are presented and then the methods of starting
wound-rotor motors. 6

6.14.1. S ta rtin g of sq u irrel-cag e m otors. For cage motors, the choice of any particular
method of starting depends on (i) size and design of the motor (ii) capacity of the power lines
and (lit) type of the driven load. There are primarily two methods of starting squirrel-cage
induction motors : (a) full-voltage starting and (6 ) reduced-voltage starting. The full-voltage
starting consists of DOL starting only. The reduced-voltage starting has the advantage of
reducing the starting current, but it produces an objectionable reduction in the starting torque,
on account of the fact that motor torque is proportional to the square of the applied voltage!
Despite this, reduced-voltage starting is the most popular method of starting three-phase
squirrel-cage induction motors and consists of stator resistor (or reactor) starting, auto-trans­
former starting and star-delta starting. The various methods are now described in what fol­
lows.
(a) D irect-on-line (across-the-line) starting. As the name suggests, this method invol­
ves the direct switching of polyphase stator on to the supply mains. The motor takes low-power
factor starting current of 5 to 7 times its full-load current, depending upon its size and design.
Such large currents of short duration don’t harm the rugged squirrel cage motor, but the high
currents may cause objetionable voltage drop in the power supply lines feeding the induction
motor. These large voltage drops cause undesirable dip in the supply line voltage; consequently
the operation of other equipments connected to the same supply line is effected considerably. A
common example is the momentary dimming of lamp and tube-lights in the home at the instant
a refrigerator motor starts.

canned oy uam bcanner


hi

792 E le c tric a l M a c h i n e r y l^ t_ k l4

If the supply system is of sufficient power capacity and the low-power factor starting-cur­
rent surges don’t cause objectionable voltage dips in the supply line voltage, then the direct-on­
line starting should be preferred.
The relation between starting torque Te gt and full-load torque Te.fi is now obtained. L et/rt
and Ifl be the per-phase stator currents drawn from the supply mains corresponding to starting
and frill-load conditions respectively. From Eq. (6.13),

-a
e o>, 2 s

n /4 f r \2
Te.st _ 1 l 2st ...(6.51 a)
T *fl j2 r2 h fl
l 2 fl —
S fi

Eq. (6.51 a) is valid in case rotor resistance remains constant. Actually, rotor resistance
varies with the frequency of rotor cu rren t; at starting rotor frequency is 50 Hz and at full load
it is only a few hertz.
Here I 2tt and I™ are the per-phase rotor currents at startin g and full-load conditions
respectively.
If no-load current is neglected, then
I tl x effective stator turns = I 2 st x effective rotor turns
or Itt = I?# (effective rotor to stator turns ratio)
I # (Effective rotor to stator turns ratio) I 2 st
or
Ifl (Effective rotor to stator turns ratio) I 2fl
ht Iz st
or
h h fi
\2
T e st _ [1 s t
From Eq. (6.51a), Sfi ...(6.51 b)
T e fi [ h

If Vi is the per phase stator voltage and Z$c is the standstill per phase leakage impedance
referred to stator, then per phase short-circuit current at standstill (or at starting) is,

/
^ 8C

where Zsc = (rx + r2) + j (Xl + x^.


Note that here shunt branch of the induction motor equivalent circuit of Fig. 6.13 (a) is
neglected.
Therefore, for direct switching,

/ - /1 SC
“ “ ry •
"» C

Eq. (6.51) can, therefore, be written as

T .„
\2
Sfi ...(6.52)
T < fl~
*

Scanned by CamScanner
*- - ^
Art. 6.14] Polyphase Induction M o to rs 793

(b) S ta to r re sisto r (or re a cto r) startin g. In this


method, a resistor or a reactor is inserted in between
motor terminals and the supply mains, as illustrated
I 3V,
in Fig- 6.40. At the time of starting, some^voltage drop t-rTnrjtor-*
occurs across the starting resistor or reactor and,
therefore, only a fraction x (less than 1) ofthe.supply
voltage appears across the stator terminals. This
reduces the per-phase starting current I st drawn by the Fig. 6.40. Reactor (or resistor) startin g of
squirrel cage induction motor.
motor from the supply mains. As the motor speeds up,
the reactor is cut out in steps and finally short-circuited when the motor speed is near to its
operating speed. Since per phase voltage is reduced to xVlt see Fig. 6.40, the per-phase starting
current Ist is given by
JcV,
Ist = =x L ...(6.53)
SC

■e st (I•*StN
As before, ...(6.54 a)
■efl
est 2 (I*sc '
or r~ =x ■Sfl ...(6.54 b)
•efl
In an induction motor, torque oc (voltage)2

Starting torque with Reactor starting _ (xV\


- x ...(6.55)
Starting torque with direct switching V1
v /
Series reactor is more costly than the series resistor, but the former has lower energy loss
and is more effective in reducing the voltage, because the induction-motor power factor at start­
ing is quite low.
(c) A u to -tra n sfo rm e r sta rtin g . A fraction x of the
supply voltage Vl is applied to the stator terminals at the
time of starting, by means of an auto-transformer as shown
in Fig. 6.41. This reduces the motor current and also the cur­
rent drawn from the supply. After the motor has accelerated
near to its operating speed, auto-transformer is disconnected
and full line voltage is applied to the induction motor by con­
Fig. 6 .4 1 . P ertain in g to
necting it directly across the supply mains. Note that here x au to-tran sform er sta rtin g .
is less than one. •
With auto-transforrner, per phase starting current in motor winding
_xV ±
= X I, ...(6.56)
Ztr
If no-load current of auto-transformer is neglected, then per phase output VA of auto-trans­
former must be equal to its per phase input VA. That is
hi • v i = xV l (per phase starting current in motor winding)
or 1st •^1 = ^ B c)
Per phase starting current from the supply mains,
...(6.57)
/ „ = X2 •/ «
794 E le c tric a l M a ch in ery .I
(A r t 6 4

Eq. (6.56) shows that motor starting current per phase is reduced only to x times the direct
switching current I sc; but the pe. phase starting current from the supply mains is reduced to
x 2 times the direct switching Isc. As per Eq. (6.51 b),
• _!• 2
Te sl (Per phase starting current in motor winding)
sfl
1 e fl (Per phase motor full-load current)"'
T e st (x I sc)
From Eq. (6.56) ■sfi
lefl 4
\2
) (T
ic
= X
Sfl ...(6.58)

Per phase starting current, Ist, from the supply mains can be included in Eq. (6.58) with the
help of Eq. (6.57). Substitution of the value of I sc in terms of Ist gives

Test _ 2 1 I
T e fl

l st
■Sfl ...(6.59)
v' /
The ratio of starting to full-load torque in terms of both I st and I 8Ccan be obtained from Eq.
(6.58) as follows.
Test (x I sc) I sc
T.e f l
1 st ‘ 1 sc
■Sfl ...(6.60)
$
Per phase motor starting current in terms of Ist, from Eq. (6.56), is given by

x I sc = x —2 I st
X

...(6.61)

For an induction motor, torque « (voltage)2

Test with an auto-transformer (xVy


-x ...(6.62)
Te $t with direct switching Vx
It is seen from above that with an auto-transformer, the starting current Ist from the m ains
and starting torque Te $t are reduced to x times their corresponding values with direct-on-line
starting, see Eqs. (6.57) and (6.62).
(d) S ta r-d e lta s ta rtin g . This method of starting is used for motors designed to operate
normally in delta. The six terminals from the three phases of the stator must be a v a ila b le as
shown in Fig. 6.42 (a). Here the terminals are marked a, A for phase A; b, B for phase B and
c , C for phase C. Commercially, the terminals are marked A lt A 2 for phase A ; B lt B 2 for phase
B and Clf C2 for phase C. Fig. 6.42 (6 ) illustrates a simple schematic diagram of a star-d elta
starter. As stated above, three stator terminals are marked A, B, C in Fig. 6.42 (b) and th e other
three stator terminals marked a, b, c are connected to the middle terminals of a triple pole
double throw ( TPDT) switch. The stator phases are first connected in star by throwing the

Scanned by CamScanner
A rt. 6.141 Polyphase Induction M otors 795

tripple pole double throw (TPDT) switch to position 1 and after steady-state speed has reached,
TPDT is thrown over to position 2, thereby connecting the stator winding in delta, see Fig. 6.42
(6).
In position 1, terminals a, b, c are short circuited. In position 2, terminal a is connected to
B ; b to C ; c to A.
Let VL be the line voltage. Then per phase motor starting current Isty with stator winding
in star is given by
V, ’
W zZ
With stator winding in star, I sty is also the starting line current with star-delta starter.
If the stator winding were delta-connected, then with direct switching, the per phase motor
starting current Istd would be given by
_ V,
^ s t d ~ ~y *s c -d
SC
VL
and starting line current with direct switching would be equal to V3 ^3 Ist d = ^3 Ix d . Note
^SC
that Ist y = ^ Ist d. Here the subscripts y and d denote star and delta connections of stator wind­

ing respectively.
Starting line current with star-delta starter
Starting line current with direct switching in delta
1 Yk
kV 1

Scanned by CamScanner
796 E le ctrica l M ach in ery [A rt. 6.14

Thus with star-delta starter, the starting current from the mains is one-third of that with
direct switching in delta. Also,

(V l
Is 1
, Starting torque with star-delta startei _ v, i
Starting torque with direct switching in delta v£ _3

This shows that star-delta starter also reduces the starting torque to one-third of that
produced by direct switching in delta.

In auto-transformer, if the ratio of output voltage to supply voltage is then both the starting
line current, Eq. (6.57), and motor starting torque, Eq. (6.62), are reduced to one-third of their
corresponding values with direct switching in delta. This shows that with star delta-starter, a
motor behaves as if it were started by an auto-transformer starter with* = ^ = 0.58, i.e ., with 58%
tapping.
Starting torque with star-delta starter, Test
Full-load torque with stator winding in delta, Tefld
x2
(1 T
V3 std

^5 Sfl

1 *scd
3 Lfld ...(6.63)

voltages exceeding1 6tC: ^ lin*


delta requires large turns making the motor more expensive winding in

starting is more than that obtained by s t a t ^ r J e a Z T L r r t m g ^ am pere WUh aut0

starting line current


hue current in cose o ,sfo to ,reacto r startin g

starting line current for an induction m otor with p 5 a n d m .lo a d slip equ a! (o 0 Q5

so lu tio n , (o, Starting torque with an auto-transformer starting = - L (per phase startlng

current in motor winding )2 ^ = - L (* f j r2 . see Eq (g 5g) *

Per-phase starting lin'e current from Eq. ( 6 .57 ), is I = x 2 . t


s* SC*

bcanned by uam bcanner


A rt. 6.14] Polyphase In d u ction M o to rs 797

/. Te st per line ampere with auto-transformer starting is given by


_1 C* •/«)a - r 2 _ i f r / j 2
ht w , , 2 . / s£

= — I sc
. , r'2 ...(6.64)
(i).

2
Test with stator-reactor starting - - L (per phase starting current in motor winding)2.
5 .

rJ . = ~ ■l l ■r2 = — (x Ix)2 r 2t see Eq. (6.53).


1 5

Per-phase starting line current, from Eq. (6.53), is Ist = x 7,


Test Per line ampere with stator-reactor starting

1_ (* h e) ' r2 _ 1 (* Ix)2 T 1

Id
ht ~ ^4 ~ ^ Mi. r r2~ ^ Js x s° r2
Test Per line ampere with auto-transformer starting
Test Per line ampere with stator-reactor starting

t 1
to, ic 2
,
— •x ■I.c r7
co, *c 2
1
x'

Since x is less than 1 , starting torque per line ampere with auto-transformer starting is
more than that obtained with stator-reactor starting.
(6) For auto-transformer starting, from Eq. (6.60),
Test ht ' he
l 'fl Ifl
sfl ...(6.6a)
For stator-reactor starting, from Eq. (6.54 a),

T e s, fh t
T e fl
SP ...(6.54 a)
I 7*
r or an induction motor ol given rating, 1 c^j, sp, Ifi and Is
are, therefore, constant. In view of this, Eq. (6.60) becomes

'T u l j
Test - - f t sf l ' 1SC lSt

or T
1 est = K Is i
^ 1 ...(6.65)
and Eq. (6.54 a) becomes

T — ~ eA o . 72
1e s t ~ ~ o“ Sfl *
l fl
= K Aj M
l ...( 6 .66 )
Eq. (6.65) shows that for an auto-transformer starting, variation of starting torque Tesi with
starting line current Ist is linear. For stator-reactor starting, Eq. (6 .66 ) shows that Te.st varies
n°n-linearly with Ist- • ' v
798 E le c tric a l M ach in ery lArt. 6.14

/ * T h t
For —■- 5 and st] = 0.05, various value of - - - for different values of , are given below in
In * e fl "

tabular form :
S tn rtin jj cu rre n t t ,i 2 3 4
0 1 5
Pull load cu rre n t Ip

A u to -tran s sta rtin g , Eq. (6.60) 0 0 .2 5 0 .5 0 0 .7 5 1.00 1.25


I* T .„
j I h fl S ta to r-re a c to r sta rtin g Eq. (6 .5 4 a ) 0 0.05 0 .2 0 0 .4 5 0 .8 0 1.25
.
A utotronslorm er
Note that when Isl = / gf, starting torque by both the starting

methods are equal. It is also seen from the table and Fig. j*
6.43 that stator-reactor starting is suitable only in those -v. *00 4
y /
applications, which require low-starting torques. J? 075
/ A sV
a fu ll-load slip o f 0.05. The motor starting current at / / starting
rated voltage is 6 times its full-load current. Find the /r
tapping on the auto-transform er starter which should 1 ■ i 5
give full-load torque at start. Also fin d the line current at r*Viu— ■
starting. Pig. 6.43. Pertaining to Example 6.52.

Solution. Here motor starting or short-circuit current /*. is 6/^. Now from Eq. (6.58),
n2

.*. 1 = x ‘ ( 6 r x 0.05

x= = ° -745 or 74.5% tapping.

From Eq. (6.57), starting line current is


h , = *2 I* = (0.745 )2 (61fl) = 3.33 Ifl.
E x a m p le 6.54. A 3-phase squirrel cage induction motor, with an a p p lied voltage o f 40%,
gives a blocked-rotor current o f 250% an d internal starting torque o f 30% o f their corresponding
rated values. I f an auto transform er lim its the starting line current to 150% o f the motor full­
load current, compute the percentage starting torque.
Solution. 40% of applied voltage Vlf or 0.4 Vx gives a blocked rotor current of 2.5 times the
full-load current.
Therefore, rated voltage Vx would give a blocked-rotor current or short-circuit current/*
equal to

/“ —51.
0.4 V

It is seen from Eq. (6.57) that per phase starting current from supply mains is, Ilt = *2 h
Since the starting line current Ist is limited to 1.5 Ifl, we have
f25
hi = 1-5 Ifl= x
4 M
2 6
or X = -----
25‘

/v_.ui ii iovj u 1 W III vVUUI II IWI


A rt. 6 .14)
Polyphase Induction M o to rs 799

Now for an induction motor,


Starting torque °c (Voltage)'
It is given that 0.3 Tcfl « (0.4 Vxf
Also ■c s /2 OC (X V ,)2

Y
^ s f2 = ( ° - 3 0 •Tc.n)
0.4 \\

= X 0 .3 T c „ = ^ X X 0 .3 T,.n = 0 .4 5 T,„
Starting torque is 45% of full-load torque.

720^ a;^ eres ^ A ssi^ in s^ h a^ a 2^ ^ V d,stnbut^n circuit is designed to supply not more than

■ c r l tu0 ™ w n 11 1 j , :nd,f
' 8 and tIlQt thc r mo,or
starting hasatarated
current {ull: !°voltage
at ef-
is 5 times the rated full-load current, what is the maximum permissible kW rating o f the motor,
(a) if it is to be started at full voltage ?
(b) if it is to be started using an auto-transformer stepping down the voltage to 80% ?
(c) if it is designed for use with a star-delta starter ? {I E S 1978)
c S?!Uti°n; ) ^ aximum Permissible linecurrent that the 3-phase induction motor can take
from the distribution circuit is 1200 A at the time of starting. It is given that the starting
current at rated voltage is 5 times the rated current of the induction motor. Therefore the rated
line current of 3-phase induction motor with full-voltage starting is = 240 A. Thus the
maximum permissible induction motor rating when started at full voltage
= V3 \\ l x cos 0! x Efficiency
= V3 (440) (240) (0.8) (0.85) watts = 124.371 kW.
(fe) From Eq. (6.57), maximum permissible starting current from supply mains,
Ist —1200 = x 2 Isc = x~ (5 Iff)
or 1200 = (0.8 )2 (5 Ip)
1200
= 375 A.
(0.8 )2 x 5
/. Maximum permissible induction motor rating
= V3 (440) (375) (0 .8) (0.85) watts = 194.33 kW.
(c) A star-delta starter is equivalent to auto-transformer starter with 57.8% tapping.

1200 = (0.578)2 (5 Ip) = ^ {5 Ip)

Ip = 720 A.
Maximum permissible induction motor rating
= V3 (440) (720) (0.8) (0.85) = 373.113 kW.
Exam ple 6.56. A 10 kW, 400 V, 3-phase induction motor has full-load efficiency o f 0.87 and
power-factor o f 0.85. At standstill and at rated voltage, the motor draws 5 times its full-load
current and develops a starting torque o f 1.5 times its full-load torque. An auto-transformer is

amScanner
[Art. 6.14
800 E le c tric a l M a c h in e r y ___________________________________________________ ____________________________________ _

installed to reduce the starting current an d to give fu ll-load torque at starting. Neglecting excit­
ing current o f auto-transformer, determ ine at the time of starting
(a) the voltage applied to the m otor term inals
(b) the current draw n by the motor an d
(c) the line current drawn from the supply mains.
Solution, (a) The electromagnetic torque Te is proportional to square o t e vo age.
... Te stl = 1.5 Te fl - (400)2
Also Test2 = Te.fi oc (V-!)2
gy.fl . 1 5 Te fl _ '400 f
^cs(2 Te.fi Vi
y
400
V1 = J ^ . = 326.6 volts.

(b ) For 400 V, the motor starting current = 5 Ifl


For 326.6 V, the motor starting current

= 5 // > w = 4 0 8 2 5 ^
Now motor full-load current,
r _ ______ 10,000_______ 19 50 a
f1 V3x 4 0 0 x 0 8 7 x 0 .8 5 ' '
.-. Motor starting current = 4.08 x 19.52 = 79.6904 A.
(c) On the auto-transformer secondary, the motor current is 79.64 A and the secondary
voltage is 326.6 V. Therefore the current on the auto transformer primary side o f400 V, is given
by

* 79.64 = 65.03 A.
400
Current drawn from the supply is 65.03 A.
Current drawn from the supply mains can also be obtained by using Eq. (6.57). This ex­
ample can also be solved by first calculating sft (= 0.06 here) and then using Eq. (6.58) for x etc.
E x a m p le 6.57. A 3 p h ase squirrel cage induction m otor has m axim um torque eq u al to twice
the fu ll-load torque. Determine the ratio o f m otor starting torque to its fu ll-lo ad torque, i f it is
started by
(а) direct-on-line starter,
( б ) star-delta starter,
(c) auto-transform er starter with 70% tapping .
The p er p h a se rotor resistance an d p er p h a se standstill reactance referred to stator are 0.2
Ci an d 2 Cl respectively. N eglect stator im pedance.
Solution, (a) With negligible stator impedance, slip at which maximum torque occurs is
given by
r2 02 ni
SmT=x~
m~ ~2~ ~
Te.st _ 2 2 2
Now
Te m s mT 1 01 JL 10-1
1 1 0.1

Scanned by CamScanner
1
A rt. 6 .1 4 ] P olyphase In d u ction M o to rs 801

T<» = J o X V T , n) = 0.39ST'fl.
(b ) With star-delta starter, starting torque is one-third of that obtained by direct-on-line
starter.
Tett = i (0.396 Tefl) = 0.132 Tefl.
(c) With auto-transformer starter, from Eq. (6.62), we get
Te.st = (0.70)2 x 0.396 Te fl = 0.194 Tefl.
Exam ple 6.58. A 40 kW induction motor with a star-delta starter is supplied through a
feeder from 400 V, 50 Hz main. Owing to the line drop, the starting torque is found to be the same
with star as with delta connections. Determine the resistance o f the feeder. From a short-circuit i
test on the motor, when delta-connected, the following data were obtained : V = 200 V ; I = 100
A ; power factor - 0.4 (all line values). I f a second feeder o f the sam e cross-section is run in
parallel with the original one, find the percentage increase in starting torque, obtained with each
connection. (I.A.S., 1986)
Solution. Per phase equivalent circuit parameters of the induction motor can be obtained
from short-circuit test data :

Zsc = ^ ^ = 3.464 £1
100

D Rsc = Zsc cos 0SC= 3.464 x 0.4 = 1.386 Q


Xsc = V3.4642 - 1.3862 = 3.175 a .
Let R be the resistance of each feeder line.
Starting torque with star connection
= ^_ (400 f 1
x 1.386
w, 1^3 j [(H + 1.386)2 + (3.175)'1
When the stator winding is connected in delta, balanced delta connection must be replaced
by an equivalent star connection so that each phase of the star is in series with each feeder line.
Therefore, starting torque with delta connection is given by
3 f 400 1.386
co. 1.386^1 f3.175']
R+
J
From (i) and (ii), for the same starting torque,
2 2 -i
1.386
(R + 1.386)2 +(3.175)2 = R +
j j
Its simplification gives R = 2 Cl.
Previous starting torque with star connection and with one feeder
x 1.386 = 0.0464 K
co. 3.386 + 3.175
^ = 3(231j ! x l 386
where co.
New starting torque with star connection and with two feeders in parallel
(231)" x 1.386 = 0.0634 K
cos 2.386 +3.175

Scanned by CamScanner
802 E le c tric a l M a ch in ery
(A rt. 6.14

/. Percentage increase in starting torque with star connection

=^
3 4 * - ° - ° 4 6 4 * x 100 - 36.64%
0.0464 K
Previous starting torque with delta connection and one feeder
(231)' xIM = o 1392 K
O). 1.386
\2
3.175
2+

where *i =- 1.386
co„
New starting torque with delta connection and two feeders in parallel
(231)'
co. x = 0.307 K v
\ ( 1.386 s) 3.175 ^

Percentage increase in starting torque with delta connection


= 0.307 K x - 0.1392 K x
0.1392 K x x 100 = 120.55%.

o „ ^ x t m pl^ f '59' A 40 k, W‘ :i'p h a se induction m otor h as stan dstill im pedan ce o f 1 2 + i 3 0


a p e r phase. The m otor ,s fed from 400 V, 50 Hz source through a feed er 500 m long Determine
s t u Z Z T J Z T ' c r o s s - s e c t i o n , 0 f e a c k '°°ndu" ° r ‘ in,orderZfsZti^orZ
t h r l T bel edT n y m° re than 30% due to voltaSe dr°P in the feeder. The m achine is started
ug a star-delta starter. Take resistivity o f feed er m aterial as 0.02 Ci / m/ m m 2.
S olution. Starting torque without feeder
3V2 1
co. 1. 2 ' -x x 1.2
...(i)
Let R be the resistance of each feeder line.
Starting torque when fed through feeder
3V2 i
co. (R + 1.2)2 + 32 x" *1.2
'* •(«)
Starting torque when fed through feeder should at „
torque as given by (i). equal to 0.7 times the starting
From ii) and (ii),

1 0.7
(R + 1.2)2 + 32 ( 1.2)2 + 32
or
R 2 + 2.4 R - 4.4743 = 0
or R = 1.232 n .
Now pL 0.02 x 500
A A ~ 1,232 n
A = 8.117 mm 2

E x Z f J r r ; " ; ; ab!e cr r sectir of cach conduct° r ° f the ^ «• ^ >■

rotor ohm ic loss during blocked rotor test. SS lS assu m ed e(l ual t0

Scanned by Cam scanner


A rt. 6.14| Polyphase Induction M otors

(a) Determine the starting torque i f DOL starting is used for this motor
(b) What mould be the starting torque i f phase windings are connected in star an d DOL
starting is used ?
Solution. During blocked-rotor test, power input to a 3 -phase induction motor appears
almost entirely as I R loss in both stator and rotor windings.
••• Stator 12R loss + rotor I2R loss = 120 kW
It is given that stator ohmic loss = rotor ohmic loss during blocked rotor test.

.*. Total rotor I2R loss = 3 l\ r2 = — = 60 kW.


z
(a) Synchronous speed, w, = = — ~Q50 = 20 •n rad/sec

.-. Starting torque, Te „ = J - l\ r2 = 6^ ° = 954.93 Nm

(b) With phase windings in star, starting phase current is reduced to ^ times the starting
current when in delta.
60
Total rotor ohmic loss, 3 l l r2 when in star = — = 20 kW
3

Starting torque, Te st = - ^ - 2 = 2^ °^ ° = 318.31 Nm.

Exam ple 6.61. (a) A 5 kW, 3-phase cage induction motor is fitted with an automatic star-
delta starter. On account o f faulty operation o f the starter, the motor continues running with its
stator winding in star. What is the power capacity o f the induction motor when running in star ?
Assume the motor losses to be negligible.
(b) What is the draw back o f star-delta starter ? How can it be overcome ?
(c)A3 p h ase squirrel cage induction motor is fitted with a star-delta starter. On sm all loads,
i f the stator winding is reconnected in star, its operating power factor improves. Explain.
In case a fully-loaded delta-connected induction motor is reconnected in star, explain what
happens to its operating power factor, line current and efficiency.
Solution, (q) Since the induction motor is fitted with star-delta starter, the motor is
designed to develop 5 kW with its stator winding in delta. Let the per phase value of voltage
and current for the delta-connected motor be V\ and 11so that 3 Vj/j cos 0i = 5 kW.
V,
When the motor winding is in star, the per phase value of voltage reduces to but the
rated phase current remains as /,. Thus the rating of induction motor when reconnected in star,

becomes 3 ^ cos 0! = ^ = 2.887 kW. It is assumed that the operating power factor of the

motor remains unchanged from delta connection to star connection.


(6 ) In an ordinary star-delta starter, the stator winding is disconnected from the supply
mains when changing from star to delta. During this small interval (0.1 to 0.3 sec), the motor
speed remains practically unchanged. As soon as the stator is disconnected from the supply the
rotating magnetic field is not allowed to collapse by the currents induced in the rotor bars This
rotating field now induces voltages in the temporarily open-circuited stator winding and the
frequency of these voltages is f ( 1 - s). When the star-delta switch connects the winding in delta
across the supply mains, the time phase angle between the supply voltage and the stator-in­

Scanned by CamScanner
804 E le c tric a l M ach in ery [Art. 6.14

duced voltages may be anywhere between 0° and 180°. In case the time phase angle turns out
to be 0 °, both the supply voltage and the stator induced voltages are added up algebraically and
a large current surge is observed in all the phases.
This drawback can be overcome by using a modified form of star-delta starter, which does
not interrupt the current during the change over from star to delta. Such a starter will natural­
ly be more complex and, therefore, more costly.
(c) In Fig. 6.44, OA is the no-load current with stator winding in delta, for a phase voltage
Vx
of Vj. When starter is reconnected in star, the phase voltage becomes ^ and the no-load cur­

rent reduces to OB. Had there been no saturation, OB would be equal to OA, but on account

of saturation OB is much less than OA. Blocked-rotor test gives point C with the winding in
delta and point D with the winding in star. The current locus passing through C has a diameter
. Vx V\
of and diameter of that passing through D is -j— For any particular power out-
X\+X% . 'Vo(Xi+X2)
put (less than that given by point E ), I x is the stator current with the winding in delta and I {
with the winding in star. Note that operating power factor cos 0X' with the winding in star is
better than the operating power factor cos 0 ! with the winding in delta. For power-outputs less
than that given by point E, the power factor is better with the winding in star. But for power
outputs greater than that given by point E, operating power factor is better with the winding
in delta. At the operating point E, power factor is same whether the winding is in star or in
delta, see Fig. 6.44 (6 ). On small loads (30 to 50% of full load) and with the winding in star,
various losses (because OB < OA and 7 / < I x) decrease, as a result its efficiency becomes better
than that obtained by the winding in delta. * . -

0-5 1-0
p.u.Output

Fig. 6.44. Pertaining to Example 6.53. Both the figures illu strateth e
effect of reconnecting the delta-connected winding in star.

With stator winding in delta, full-load power output is greater than that indicated by point
E. In/Fig. 6.44 (a), let this operating point be F. When the stator winding is reconnected in star,
the point F shifts to G, showing thereby that when operating in star at full-load the power
factor becomes poorer as compared to its value in delta and the current increases from OF to
OG. Consequently there are more losses and efficiency in star at full-load is less than its value
in delta at full load.
6.1 4 .2 . M ethods of startin g w ound-rotor m otors. The methods used for starting squirrel-
cage motors can also be employed for starting wound-rotor motors, but it is usually not done so be­
cause then the advantages of wound-rotor induction motors can’t be fully realized.

Scanned by CamScanner”
Art. 6.14| Polyphase Induction M o to rs 805

The simplest and cheapest method of starting wound-rotor induction motors is by means 01
added rotor resistance, with full-line voltage across the stator terminals. It has already been
discussed that at the time of start, the addition of external resistance in the rotor circuit of a
wound-rotor induction motor
(i) decreases its starting current
(ii) increases its starting torque (for a suitable external resistance) and
(iii) improves its starting power factor.
The rotor winding terminals of a wound-rotor motor are connected to three slip-rings
mounted on but insulated from the shaft. The
leads, from the three brushes pressing on these
Storting
slip rings, are taken to external resistances as resistan ce
shown in Fig. 6.45. At the time of start, the entire
external resistance is added in the rotor circuit.
As the rotor speeds up, the external resistance is
decreased in steps so that motor torque tends to Slip rings
remain maximum during the accelerating period.
Finally, under normal operation, the external
Fig. 6.45. Illustrating the addition of external
resistance is fully cut off and the slip rings are resistance in the rotor circuit of a wound-rotor
short-circuited so that motor now develops full­ induction motor.
load torque at low value of slip for which it is
designed.
Calculation of resistances of elements (or sections). Consider one phase of the rotor of a
wound-rotor induction motor, with resistance r2 and standstill leakage reactance x2. Let
R\, R2, R3 ... Rn be the resistances of the n resistance elements (or sections) and
R2 , R 3 , ... Rn , be the total resistances in each phase of the rotor circuit on 1st, 2nd, 3rd,
.... nth and (n + l)th stud respectively as shown in Fig. 6.46 (a), such that

Studs

Rn-i Rn
— —|-

Rn Rrm rj

...L ..L J L L
Slip rings-
(a)

_5j 54 55 Sn»i=5m
I, max

^min .

c
32
Q.
C 3
—u
Time.t-

(b)
Fig. 6.46. (a) Pertaining to the design of starter for wound-rotor induction motor.
(b) Variation of input current with time.
[A rt. 6.14
806 E lectrical M achinery -------------------------------------------------------------------------

R { = R X+ R2 + R 3 + #4 + -• + R n~ 1 + Rn + T2
R2’ = R 2 + R3 + R4 + ... + Rn- 1 + R n + r2
RS = i ? 3 + i?4 + - + R n- 1 + + r2

Note that Fig. 6.46 (a) illustrates n-element starter, n-section starter, (n + 1) stud starter
or n-step starter. #
For calculation of the section resistances, the following assumptions are raa e :
(i) During starting time, a constant load torque is assumed.
I (ii) The stator leakage impedance and its no-load current are neglected.
(iii ) Stator current is taken to fluctuate between fixed limits I\max (maximum value) and
H I lmin (minimum value) as shown in Fig. 6.46 (b ).
At the time of start, the movable handle is at stud 1 and the rotor-circuit resistance is R
When the supply is switched on to the stator, the input current shoots to I lmcjx and its value is
given by
T V,
...(6.67)
lmax V(R S / s t f + 4
Note that at the time of start, slip Sj = 1.00.
On first stud, R x' remains in circuit until the motor has started and the current has fallen
from 7lmar to I lmin. At the same time, the slip falls from sx to s 2
. F Vx
“ * min ~ 'l(R1'/s 2)* +*2 - (6,68)
As soon as Iimin is reached at stud 1, resistance R i is cut out by moving the handle from stud
1 to stud 2 . During the notching process (the process of moving the handle from one stud to the
next stud), the speed is assumed to remain constant, i.e., the slip remains as s 2 but current at
stud 2 becomes I ]in.IT as illustrated in Fig. 6.46 ( 6 ).

V s2f +4 .,.(6.69)

At stud 2, the speed rises so that the slip falls to s 3 and current decreases to
, - vi
• T w /n t+ 4 •■'(6TO)
During the next notching process, i.e.. at the third stud when J?, is cut out

' I '

and , - - Yj

and so on.

{
Scanned by CamScanner
A rt. 6 .1 4 ]
Polyphase Induction M o to rs 807

From Eqs. (6.67), (6.69), ( 6 . 7 !) etc., we get

1lmai ~ T — ^ Vi
V fE j'/s^ + x; V(i?2 / S2)2 V(/?3'/ s 3)2 + x2
From above it follows that

- ..... _ ^n_ _ R n + I _ r2
...(6.73)

rotor circuit is redirepH Under ncf mal °Perating conditions when all external resistance in
rotor circuit is reduced to zero and the input current is / ,
lmax*
From Eq. (6.73), we get
^2 _ S3 _ S4
^n+ l
S1 s 22 S
s3 Sn _ l

«i' Ri R 3 ’ = .........: Rn - 2 R,n - 1


_Rn + 1 r2
" Rn' = R j = a{s ay) ...(6.74)

ro ™ !nCe SUP Sl L’ thB t0tal resistance in rotor circuit on the first step (or first stud) from Eq.
(6.73), is
s,
R / = — r2= —
sr ...(6.75)
From Eq. (6.74),
R 2 - R\ a, R 3 = R2' a = R / a 2
R4t, = R 3, a = R 2, a 2 = R 1' a 3

+ I - R l a"
or r 2 = RS a n
.(6.76)
Substituting the value of Rj' from Eq. (6.75) in Eq. (6.76), we get

T ry = a n'

or a n = sr

0r OC = ( « * ) * ..(6.77)
Resistances of the sections are :
R\ = R\ ~ R 2= R \ (1 ~ oc) ..(6.78)
R2 = R2 - R 3'= R 2' (1 - a) = a R 3 (1 - a) = a R l
R 3 = R 2 ~ R i = R 3 (1 —oc) = a R2 (1 - a) = a 2Rj.
Similarly R4= a R1

Rn = a n~l R 1 ...(6.79)

Scanned by CamScanner
(A rt. 6.14
808 E lectrical M ach in ery -

.. •, i j - / Tn case is different from Ifj, slip sm should be cal-


The slip s = sn p r o v i d e d / _ In case i x 0 nce R f is determined by
culated accordingly and then from Eq. (6.77), a can De ouvdn i j
using Eq. (6.75), first the resistance element i?i is obtained by using q. . en
R2, R3, R4 ... can be calculated from Eq. (6.79).
Exam ple 6.62. Calculate the values o f resistance elements o f a 4-step starter for a 3-phase,
400 V, wound-rowr induction motor. The full-load slip is 3% an d the m axim um starting current
is limited to its full-load value. Rotor resistance p er p h ase is 0.015 LL
Solution. Fop a 4-step starter, there are 4 sections, i.e., n = 4. Here sm =s n = 3% = 0.03.
' A I
From Eq. (6 .77 ), ot = (sm)» = (0.03)4 = 0.416

From Eq. (6.75), Rf =~ ^ = 0.5 SI.

The resistances of the various elements are :


R x = R f (1 - a) = 0.5 (1 - 0.416) = 0.292 ft
R2 = a R x = 0.416 x 0.292 = 0.121 SI
R3 = a % = a R 2 = 0.416 x 0.121 = 0.051 ft
and i ?4 = a .% = a J ?3 = 0.416 x 0.051 = 0.021 ft.
As a check, sum of R x, R 2, R3, R 4 and r 2 should be equal to R f . Here R x+ R 2 + R 3 + R 4 + r2
= 0.292 + 0.121 + 0.051 + 0.021 + 0.015 = 0.50 ft = R x.
Exam ple 6.63. Design the 5 sections o f a 6-stud starter for a 3-phase slip-ring induction
motor. The full-load slip is 2% and the maximum starting current is lim ited to twice the full-load
current. Rotor resistance per phase is 0.03 ft.
Solution. In this example, note that slip sm is to be calculated corresponding to maximum
input current = 2 •(72 /i).

Rotor current, / 2=
+ x2
r_2
For small values of slips, — » x
S
f t
E
In view of this, / 2= — s
r2
r E
" 22 e2
or
=— *7, = - 2 (0 . 02 )
I~O2 sf J ~ Ir~o2

Since the starting current is limited to twice the full-load current, we have

2 0 (4/z) = T" (sm)


2k
e 2 E2
or (2 .0 ) ~ (0 . 02 ) = 7 2 («»)
r2 r2

sm = 0.04.
Here number of sections, n = 5
From E£l- a = (s j /n = (0.04)1/5 = 0.525.

Scanned by CamScanner
A rt. 6 .1 5 ] Polyphase Induction M o to rs 809

From Eq. (6.75), J? ' = M?L _ 9.75 q.


1 sm 0.04
.•. The resistance of the various elements are :
R i = R i (1 - a ) =0.75 (1 -0 .5 2 5 ) = 0.356 12
^2 = a R i = 0 .5 2 5 x 0 .3 5 6 = 0.187 12
Rz = a 2R l = a /?2 = 0 .5 2 5 x 0 .1 8 7 =0.09811
i ?4 = a 3R l = o(i?3 = 0 .5 2 5 x 0 .0 9 8 = 0.052 12
R s = a 4/?! = a /?4 =0.525 x0.052 =0.02711
Rotor resistance r 9 = 0.030 11
Total = 0.75 H
As a check, this total resistance is equal to R {.
Exam ple 6.64. A 200 kW, 3300 V, 6-pole, 50 Hz star-connected slip-ring induction motor

9
has a star connected rotor. Stator to rotor turns ratio is 3.2. Rotor resistance and leakage reac­
tance are 0.1 Cl an d 1 11 respectively. Neglect stator impedance and exciting current. Find (a)
current and torque at starting on rated voltage and with slip rings short circuited and (h) the
external resistance required to reduce the starting current to 50 A with across-the-line starting.
Compute also the starting torque under these conditions.
Solution. r2 when referred to stator
= (3.2)z x 0.1 = 1.02411
x2 when referred to stator = (3.2 )2 x 1 = 10.24 H
4 nf 4k x 50 100 k
cos = = — - — = — - — rad/sec.
r O O
/ X T 3300/V3 nc , J A
(а) Ict = 1 „ 9 , „ ^ i - 185.14 A
st V(1.024) ■ + (10.24)2
T . = — l l r 2 = 7 ^ - (185.14)2 x 1.024 = 1005.5 Nm.
64/ (l)s S t 100 K
*

(б ) Let R be the external resistance connected in each phase of the rotor circuit.

I , 3300/J 3 -— 50 A
st V(1.024 +R) + 10.24
or R = 35.681 Cl
, J 35.681 _ „
R when referred to rotor = 2 = 3-484 12
(3.2)'
T A . 72 (50)2 x (1 024 + 35 681) = 2628.8 Nm.
1 est St 2 100k V '

Note that the effect of external resistance is to decrease the starting current by a factor of
185.14/50 = 3.703 and increase the starting torque by (2628.8/1005.5) = 2.614.

6.15. P o la rity Test


U sually six term inals from the three phases of an induction motor (or a synchronous
machine) are available. I f their polarity markings (Aj, A2 \B B2 J Cj, C2) are not known, then
these can be found out by a polarity test on the induction motor. The object of this article is to
describe this polarity test which is, in general, applicable to all 3-phase machines whose six
terminals are available. Three methods of performing the polarity test are described here.
810 E lectrical M achinery IArt. 6.1 S

(a) F irst Method. In order to perform the polarity test on the induction motor, single­
phase supply is given to any one phase, say A, of the induction motor and see that the rated
current is not exceeded. Mark the terminals of phase A arbitrarily as A 1 and A 2 as shown in Fig.
6.47 (a). Now join one terminal of phase B with terminal A2 of phase A. Connect a voltmeter of
suitable rating across A! and the remaining terminal of Phase B as illustrated in Fig. 6.47 (a).
Since the magnetic axes of phases A and B are displaced from each other by a space angle of
120° electrical, the voltage induced in phase B by tansformer action is given by V1 cos 120°
(ideal case). If the phase B polarity, relative to phase A polarity, is as shown in Fig. 6.47 (b) then
voltmeter reading (traversing closed circuit clockwise) would be given by
Voltage drop (going from plus to minus) in phase B + Voltage rise in phase
A = - (Vj cos 120°) + Vj
= - (- 0.5 Vi) + Vx = 1.5 Vx.
In actual practice, the magnitude of voltage induced in phase B is less than 0.5 Vx because
of the existence of large leakage flux. From this, it can be concluded that if voltmeter reading
is more than the supply voltage Vx, then phase B polarity markings relative to phase A polarity
markings are as shown in Eicr fi 47 (h\

Fig. 6.47. First method of polarity test on a 3-phase induction motor.

In case the phase B polarity markings relative to phase A, are as shown in Fig. 6
the voltmeter reading would be less than the supply voltage V,. The same procedure can be
adopted for phase C of the induction motor.
With phase A excited from single-phase supply, the magnitude of voltage induced in phase
nnpn^ r°HUT indu<*“>n motor may be around 40% of V,, because its rotor winding is
open-circuited. In case of squirrel cage induction motor, the currents indnrnH in fh* f v,
oppose the pulsating flux produced by phase A *conseauentlv thn vnlfc • a J r° tor bars
C is much less (around 20% of that applied to phase A). induced in phase B or
(6 ) Second m ethod. In the second method all thp formic i i■
A, B and C arbitrarily as shown in Fig 6 48 (o') Phase I ? ™arklnSs are Put on phases
any two terminals are connected ^ f h e r and a v o ltm elr i ^ P, F ° r phaSeS 5 " d C'
terminals. In Fig. 6.48 (a) ■B , C are ™ „ « 4 j . lu placed across the remaining two
S « (O) , c , are connected together and a voltmeter is placed across

(a)

Scanned by CamScanner
Art. 6.15] Polyphase Induction Motors 811

B 2, C2. N ow phase A is energised by applying single-phase voltage as shown. The magnitude


of voltages induced in phases B and C by transformer action would be 0.5 V1. In case voltmeter
reads zero, the polarities of phases B and C would be as marked in Fig. 6.48 (a). In this figure,
from B 2 to B lt there is a voltage drop of 0.5 V1 and from C{ to C2, there is a voltage rise of 0.5
Vj and this leads to zero voltmeter reading in Fig. 6.48 (a). In case voltmeter reading is not zero
(usually much less than Vl) the polarity markings of phases B and C would be as shown in Fig.
6.48 (b). Same procedure can be repeated for other phases as well.
(c) Third m ethod. For this method, connect any two phases in series, say A and C, as
shown in Fig. 6.49 (a). For phases A and C, mark the polarities arbitrarily. Phase B is kept
isolated. Apply single-phase voltage Vj to phases A and C in series as shown. Measure voltage
across phase B. If this voltage is not zero (much less than Vj), the polarities of phases A and C
are as shown in Fig. 6.49 (a). With Vx as applied voltage, the voltage across each phase A or C
is Vj/2. With the polarities as shown in Fig. 6.49 (a), flux §A is set up by phase A along its
winding axis. Similarly, flux <])c is set up by phase C along its winding axis. Their resultant is
shown as §R = <t>. Since tyR is along the axis of phase B, emf induced in phase B is not zero.
Ideally, this emf in phase B should be equal to V\/2, but in practice it is much less than V\/2
because of the existance of large leakage flux. Thus if voltage across phase B is not zero, the
phase C polarity markings, with respect to phase A, are as shown in Fig. 6.49 (a).
If the polarity markings of phase C, with respect to phase A, are as shown in Fig. 6.49 (6 ),
then fluxes <J>A and <)>c are set up along their respective winding axis as shown. With 4>A = <})c
= <(), their resultant §R is equal to V3 (J>. As the axis of resultant flux tyR is perpendicular to phase
B axis, emf induced in phase B would be zero. Thus, in case voltmeter placed across phase B
reads zero, the polarity markings of phases A and C are as shown in Fig. 6.49 (6 ). Same se­
quence of steps can be adopted for other phases as well.
Aj

Of Php se B
axis

(a)

X
{b)
Fig. 6.49. Third method of polarity test on a 3-phase induction motor.

1. •

S c a n n e d by C a m S c a n n e r
812 E lectrical M achinery (Art. 6.16

6.16. INDUCTION GENERATOR


Fig. 6.50 shows the complete torque-slip (or torque-speed) characteristics of a 3-phase in­
duction motor over its entire slip range of - 1 to 2. During the motoring mode, the operating
point lies somewhere near point ‘a ’ between the points B and A. Here B indicates zero torque at
zero slip and A denotes maximum torque at slip smT.
Suppose that 3-phase induction motor is coupled to a prime-mover whose speed can be controlled
and that the operating point is ‘a’ in Fig.
6.50. If the prime-mover speed is in­
creased, the operating point begins to
travel from ‘a’ towards B. When
prime-mover attains synchronous
speed ns, slip is zero and operating
point is B. For prime-mover speed
above synchronous speed, slip becomes
negative. As a consequence, rotor emf
and rotor current as per Eqn. (6.17),
torque as per Eqn. (6.25) and power as
per Eqn. (6.34), all attain negative
values. This implies that for negative
slip, electric torque developed is nega­
tive, i.e., opposite to the prime-mover Fig. 6.50. Torque-slip characteristics of a 3-phase induction machine
torque. Under such a condition, showing plugging, motoring and generating modes.
machine must act as a generator and
delivers its generated power to the supply mains from which it was taking power when working
as a 3-phase induction motor. In other words, when rotor speed is made more than synchronous
speed, slip becomes negative and 3-phase induction machine begins to operate as a 3-phase
induction generator.
Just as point A denotes the maximum torque of a 3-phase induction motor, point C indi­
cates the maximum torque that can be applied to induction machine now working as an induc­
tion generator. If the prime-mover torque exceeds this maximum torque given by point C
lvanishes* g6nerat0r ° VerSPe<!d ^ S° me higher Va' Ue ° f slip' the B e r a t i n g effeci

For prime-mover speed above synchronous speed, the rotor is being driven at a speed faster
than the synchronously rotatmg magnetic field. The rotor conductors are now befng cut by the
d e ra te d e
generatedI m E
emf ^ 2,
3 rrotor
Z c™ T fI2* and hence dU"
current ng m°t0ring
its stator mode'
component U This shows
change theirthat rotor
signs as

^Z onool t ^ tStat° r r ? " 4 ° M iS" 0WC°mp0Sed° fcu rren t° « andcomPonent/ , ' in


ator one f 10n V° Uage F ig ' 6 '5 1 ' A s the ^ during induction-gener-
ator operation ,s not synchronous, it is also called an asynchronous generator.
In polyphase induction motors, rotating magnetic field is set up by the magnetizing current
drawn by the stator from the supply mains. When the speed of the machine is made more than
synchronous speed, even then this magnetizing current must he rf»iiv» I t. , .
erator by the supply mains so that rotating flu l Z s U b T s h e d £ “ l i t SnducUon £
orator is not a sdf-cxcted machine and must therefore continue to get its magnetizing current

.n d u cT o n 'ro to rT h , ,01? T l° Whid’ “ WaS connMt« l


oarallel Whb th P ,T lnduct'on K™o™tor cannot work in isolation ; it must work in
current I d th° buS' bar ° r 0thar S f'h ro n o u s generator/s, which can supply the magnetizing
current and reactive power needed by 3-phase induction generator.

Scanned by CamScanner
A rt. 6 .1 6 ]
P olyp h ase In d u ction M o to rs 813

The transition from motoring mode


to generating mode can easily be shown
on the circle diagram of a polyphase in­
duction machine. In Fig. 6.51, induction
> motor operation is from point Q, upper
part of semi-circle and up to point D
where s —1 .0 . Induction generator
operation is from point Q, lower part of
semicircle and up to point S •
—OOi
Suppose, 3-phase induction
machine is coupled with a loss-less
prime-mover (p.m.). With 3-phase in­
duction motor running at no load
from a 3-phase source, the operating
point is Q. The motor operates at a
p /’ small value of no-load slip. Under
this condition, ac source supplies (i)
quadrature magnetizing current OP
and (ii) a power component PQ to Fig. 6.51. Circle diagram for motoring as well as generating
supply friction, windage and core loss modes of a 3-phase induction machine.

in 3-phase induction motor. Now increase the speed of p.m. so that operating point P is ob­
tained. At this point P, speed is synchronous, all losses are supplied by p.m. and the bus-bar
supplies the magnetizing current OP as well as the corresponding reactive power (= Vr •OP) to
the 3-phase induction machine.
As the p.m. speed is increased, induction generator begins to share the active load of the
bus-bars or synchronous generator/s, but the reactive power is still withdrawn from the 3-phase
ac mams. When operating point M (at about s = - 0.03 to - 0.05) is reached in Fig. 6.51, then
MH = mechanical power input to induction generator
ML = electrical power output of induction generator
CM = stator current
OL = magnetizing current taken from the bus-bar which has increased from OP to OL
KL = constant losses
K J = stator ohmic loss
J H = rotor ohmic loss
cos eG= cos /V^ OM = leading pf of the induction generator.
6.16.1. Self-excited or isolated induction generator. This type of induction generator
does not require an existing ac supply system for obtaining its magnetizing reactive power. In
the self-excited induction generator, a capacitor bank is connected across its stator terminals
as shown in Fig. 6.52. The capacitor bank provides the lagging reactive power of both the in­
duction generator as well as the load. In Fig. 6.52,
Capacitive reactive power, Q = Qlt reactive power needed by 3-phase induction generator
+ Q2, reactive power needed by the load. '
In self-excited induction generator, stator terminal voltage depends upon its magnetization
curve as shown in Fig. 6.53 (a). The reactive current of a capacitor depends upon the voltage
across its terminals, i.e., capacitor voltage Vc = I ■Xc where Xc = ^ •In Fig. 6.53 (6 ), load lines
of capacitor banks having different capacitances C1> C2 > C3 > C 4 are drawn.

bcannea oy uam ^canner


if­
[Art. 6.16
814 E lectrical M achinery i

Fig. 6.52. A capacitor bank connected across stator terminals of a 3-phase induction generator.

(a) (b)
Fig. 6.53. Characteristics of (a) induction generator and (6) cap acitor bank.

In Fig. 6.52, capacitor bank is connected across the stator terminals of a 3-phase induction
machine. When the rotor of induction machine is run at the required speed, residual magnetism
present in the rotor iron generates a small terminal voltage oa across stator terminals, Fig. 6.54
(a). This voltage produces a capacitor current ob. This current ob creates a flux which aids the
residual flux, thus producing more flux and therefore more generated voltage be across stator
terminals. This voltage be sends a current od in the capacitor bank which eventually generates
voltage de. This cumulative process of voltage build up continues till the saturation curve of
induction generator intersects the capacitor load line at point/, thus giving a no-load generated
emf of g/for magnetizing current 7ml. Note that this voltage build up process is similar to that
in a dc shunt generator.
Polyphase Induction M otors 815
A r t. 6.16)

if the residual flux is absent in the rotor iron, the induction generator will not build up. This
problem can, however, be overcome by running the machine as a polyphase induction motor for
some time to create residual magnetism.
Note that the voltage build up depends upon the value of capacitor. Higher the value of
capacitance, greater is the voltage build up, see Fig. 6.54 (6). In case capacitor load line does not
intersect the magnetization curve of induction machine, there would be no voltage build up. In
Fig. 6.54 (6), voltage build up for capacitor C4 does not occur.
6.16.2. A pplications of induction generators
(а) E xtern ally -excited generators. This type of generators require a little auxiliary
equipment. These are run in parallel with an existing 3-phase system. Thus, voltage and fre­
quency of induction generators cannot be controlled. Only the active power can be regulated
through the speed control of prime-mover driving the 3-phase induction generator. These gen­
erators don’t hunt. The short-circuit current of these generators is limited. It is because a short-
circuit at once reduces the excitation and thus short-circuit current gets limited.
Externally-excited induction generators are also used in unattended small remote hydro
plants thereby interconnecting a small power station to a large power distribution network.
Their disadvantages are as under :
(i) They require considerable amount of reactive power from the existing supply system.
(ii) Efficiency is relatively poor.
(iii) It can work at leading power factor only.
Induction generator principle is used for regenerative braking of hoists or electric locomo­
tives driven by 3-phase induction motors.
(б ) Self-excited g en erators. With depletion of energy sources world wide, every effort is
made to convert other forms of unconventional energies into electrical energy. Therefore, ener­
gy recovery schemes are becoming an important aspect of present-day industrial processes. In
the coastal areas, wind energy is available in abundance. For the conversion of this wind energy
into electrical energy, an induction generator coupled with a wind-mill offers an ideal solution.
Exam ple 6.65. A 400 V, 3-phase, 6-pole, 50 Hz, star-connected induction motor has the
following per-phase param eters referred to sta tor:
rj = 0.2 O, r2 = 0.5 Q, Xj = x2 = 2 O, Xm =48 0.
This motor drives a hoist. During lowering o f the hoist, the load accelerates the motor to a
speed o f 1050 rpm. At this speed, determine (a) the line current (b) power returned to 3-phase
supply and (c) the efficiency in case its rotational and core losses are 600 W.
Solution. The induction motor equivalent circuit, for this example, is shown in Fig. 6.55.
400
Per-phase supply voltage, Vx = = 230.95 V

1 2 0 x 5 0 1ftnnrnm
Synchronous speed ------= 1000 rpm

Induction machine operating slip,


1 0 0 0 - 1050
s= = - 0.05
1000
0.5
(j 48)
0.05 -96-/480 Fig. 6.55. Pertaining to Example 6.65.
From Fig. 6.55, Zr _ 1Q 50 ‘ - 10 + j 50

Scanned by CamScanner
{Art. 6.17
816 E le c tric a l M a c h i n e r y -

- - l+ j 5 -1-7 5 26
Zf = R f + j X f = - 8.862 + j 3.692 Q
r\ + jx\ = 0.2 + j 2 fl
Total input impedance, Z = - 8,862 * j 3.692 + 0.2 + j 2
= - 8.662 + j 5.692 = 10.365 ^146^_
r _ Vi 230.95____ _ 22 282 / - 1 4 6 / P A
' 1_ Z “ 10.365 f 146.7°
Stator line current = 22.282 ( - 146/T A
Power factor at stator terminals = cos / r 146:Z! = " 0>83®
A negative value of pf means that power factor is 0.836 leading.
(6) Power at stator terminals = 3 ^ / ] cos Oj
= 3 x 230.95 x 22.282 x cos 146.7° = - 12906.24 W
As power at stator terminals is negative, the induction machine is acting as a 3-phase
induction generator and a power of 12906.24 W is being fed back to the 3-phase supply system.
(c) Air-gap power =3 l\ R ,=
3 x 22.282s x 8.862 = 1
Total stator 12R loss = 3 x 22.282s x 0.2 = 297.9 W
Total rotor I 2R loss = s Pg - 0.05 x 13199.62 = 659.98 W
Rotational and core losses = 600 W •
Total losses in induction generator = 297.9 + 659.98 + 600 = 1555.88 W
Mechanical power input to induction generator
= Power delivered to 3-phase source + total losses
= 12906.24 + 1555.88 = 14462.12 W

.-. Efficiency of induction generator = x 100 = 89.242%.

6.17. Applications of Polyphase Induction Motors


For loads requiring low starting torques and substantially constant speeds, squirrel-cage
induction motor is the best choice, because of its ruggedness, simplicity, low cost and reduced
maintenance charges. Squirrel cage motor may be designed with low rotor resistance or with
high rotor resistance. As stated before, a high rotor resistance gives better starting conditions
but poor running performance. On the other hand, a cage motor with low rotor resistance gives
poor starting conditions but better running performance. In view of this, the rotor-circuit resis­
tance should be chosen judiciously at the design stage so that there is a compromise between
its starting conditions and running performance.
Squirrel cage motors with relatively low rotor resistance (full-load slip 3 to 5%) are used for
fans, centrifugal pumps, most machinery tools, wood-working tools etc. Cage motors with rela­
tively high rotor resistance (full-load slip 3 to 7%) are used for com pressors, crushers,
reciprocating pumps. Squirrel cage motors with still higher values of rotor-circuit resistance
(full-load slip 7 to 16%) are used for intermittent loads like punching presses, shears, hoists,
elevators etc.
A wound-rotor induction motor is used for loads requiring severe starting conditions or for
loads requiring speed control. A wound rotor induction motor is more expensive than a s q u i r r e l
cage motor and also it requires more maintenance because of the brushes and slip rings. A
wound-rotor motor, also called slip-ring motor, may be used for hoists, cranes, elevators, com­
pressors etc.

Scanned by CamScanner
Prob. 6]
Pnlvphase In d u ctio n M o to r s
817

'".The relative sdvantfluoo „r __ .• „


are given below. ge motor over a wound-rotor motor of the same power rating,

mipntlv T2R lnoe • ec*u*res considerably less conductor material than a wound rotor, conse-
wound-rotor motor *S ^ SS' ^ ere^ore>ca8e motor is a little more efficient than a

a result of it?a w o u ^ - r o t a ! ^ 0n requir,es slip rin&s>brushes, short-circuiting devices etc. As


(Hi\ A • 1 motor is costlier than a cage induction motor.

overhang leakage AuT t Ms has^h'Tn-’T f ° f °Varhang' therefore' il h a s 'ow rotor


rotor than for a wound rotor Cool « °f,reSUltmg in low leakage reactance ** for a cage
greater than for a wound-rotor m o tT ™ 1. dla” eter of circ,e <*>ag>'a<" f° r a aaea motor 1S
PTpater maximum ™ motor. This shows that a cage motor has more pull-out torque,
rotor induction m o to ^ ^ ° U and better operating power factor as compared to a wound-

tenance'char^s are'low°re rUKed a" d re,;uires n0 sliP rings’ brushes etc., therefore, its main-

(") Cage rotor can be ca»la<i better because of its bare end-rings.
sadvantages of cage motor as compared to a wound rotor motor are its small starting
tntal tmortrtr s ^Ing. current and its poor starting power factor. In addition to it, the
to a energy lost during starting of cage motor is much more than with the wound-rotor motor
an is ac is very important where frequent starting of large number of motors is required.
It may be worthwhile here to compare the working of an induction motor with a synchronous
motor. J
S.No. Induction Motor
Synchronous M otor
1. It h as inherent self-starting torque
It has no inherent self-starting torque, therefore some e xtern al
means must be used to sta rt it.
Its speed falls with increase of load. It can It runs a t a constant synchronous speed a t all possible loads.
never run a t synchronous speed.
It requires no d.c. excitation, i.e., it is a It requires both a.c. and d.c., i.e. synchronous m achine is a
singly excited m achine. doubly excited machine.
It can operate only a t lagging power factors. It can operate both at leading and lagging power factors.
Speed control is possible. Under normal circumstances, synchronous m otor works w ith
no speed control.
It can be used to supply only mechanical It can be used for deliverin g m ech an ical loads and for
load. improving the system power factor.
Its m axim u m torque is proportional to Its maximum torque is proportional to supply voltage.
square of the supply voltage.
Induction motors with speeds above 500 Synchronous motors with speeds below 5 0 0 r.p.m . and ratin g s
r.p.m . and ratin gs below about 120 kW are above about 40 kW or with medium speeds from 5 0 0 to 1 0 0 0
cheap er th an synchronous motors. r.p.m. and ratings above about 5 0 0 kW, are less costly than
induction motors.

PROBLEM S

S.* ■ 6.1 . D escrib e con stru ction al featu res of both squirrel-cage induction motor and slip -ring induction m otor.
D iscuss th e m erits o f one over the other.

11 6 .2 . (a) D iscuss th e points of sim ila rities between a transform er and an induction m ach in e. H en ce, explain
why an induction m ach in e is called a generalized transform er.
(i>) E x p la in why a 3-p h ase induction motor, a t no-load, operates at a v eiy low power factor.

canned by CamScanner
818 E lectrical M achinery__________________________________________________________ [

(c) Two 3-ph ase induction m otors A , B a re identical in all re sp e cts e x ce p t t h a t m o to r A h a s a la rg e r air
gap th an m otor B. E xp lain which of th e two m otors will h ave
(i) m ore no-load cu rren t
(ii) poorer no-load power factor and
(Hi) b e tte r full-load power factor. lA ns. ( c ) : (i) and (ii) Motor A, (Hi) Motor B]

6 .3 . (a) Describe th e principle of operation of a 3-p h ase induction m otor. E x p la in w hy th e ro to r is forced


to ro ta te in th e direction of ro tatin g m agnetic field.
(6) D iscuss the differences betw een 3-ph ase induction m otors an d tra n sfo rm e rs.
6 .4 . A 3-ph ase induction m otor is som etim es called a generalized tra n s fo rm e r in so far as voltage and
frequency tran sfo rm atio n s a re concerned. As p er this sta te m e n ts , d iscu ss how a 3 -p h a se induction m otor
operates under the following conditions : 1
(a) rotor frequency f 2 = sta to r frequency f\

(b )f2 < f l

(O f2> h
(d) rotor g enerated voltage and rotor cu rre n t a re m axim u m
(e) rotor em f E2 and ro to r cu rre n t a re zero
(/) both E 2 and / 2 are minim um
(g ) both E 2 an d 72 are negative.

[A ns. (a ) A t stan d still (b) U n d er norm al run n in g conditions (c) R otor is d riven a g a in s t th e direction of
ro ta tin g m agnetic field (d ) A t stan d still (e) A t synchronous speed (f) A t no-load (g) W h en o p e ra tin g as a 3-phase
induction generator]
6 .5 . D iscuss the production of sta rtin g torque, th rou g h th e con cep t of in te ra ctio n of flux an d m m f waves
in a 3-ph ase slip-ring induction motor.
H ence show th a t th e ro to r is forced to ro tate in the direction of ro ta tin g flux w ave.
6 .6 . E xp lain the production of torque in a 3-p h ase slip -rin g induction m o to r w hen th e ro to r is ru n n in g with
a slip s. H ence introduce th e concept of load angle.
D iscuss th e conditions und er which optim um torq u e is developed in a 3 -p h a s e in d u ction m otor.
6 .7 . D escribe th e developm ent of electrom agn etic torque in a sq u irre l-ca g e in d u ction m o to r through the
in teraction of flux and m m f w aves, w hen the ro to r is ru n n in g a t a speed less th a n sy n ch ro n o u s speed.

.. c^ en.C* Sh. ™ th a t th e slip' ria g and sq u irrel-cage induction m o to rs a re id e n tica l in so fa r a s th e ir rotor-reac-


tions on the s ta to r a re concerned.

6 .8 . In a 3-p h ase induction m otor, electrom agn etic torq u e is given by

Tt = ^ ■P2 <t>F 2 cos 02 ,

T he above torq u e exp ression can also be exp ressed a s T = K in cos On I ,* ,., •
Kl a ♦ u* • n. i , e 2 2 E x p lain h °w th is torque expression
K I2 cos 62 can be used to obtain the torque-slip ch a ra cte ristics of a 3-p h ase in d u ction m otor.

- 6 .9 . (a) T h e speed of rotor field, w ith resp ect to sta to r, is alw avs eau al tn 5vnr(im T,n„o j * ui
speeds o f th e induction motor. E xplain . * q syn ch ron ou s speed a t all possible

( « E xp lain why th e rotor of a polyphase induction m otor can n ev er a tta in syn ch ron ou s speed.
(c) I he rotor o f a sh p -n n g induction m otor is connected to an n r « « , , , « „ v i 4 . ,. •
short-circuited. I f rota tin g m agnetic field produced by rotor w in d in g 1 ! , ’ , W a S 1 5 s ta to r wmding IS
which the rotor m ust revolve. g ro ta tes clockw ise, ex p lain th e direction in
_ n . . i [Ans. (c) Anti-clockwise]

4-Mes but « . ^ is

to p l d ’uce fh™ tam e3 n u m b e r * o f W i l l I n d u X n ’S X t a r t k l t o r a n ’ E x Z m " ' h ° WeVer' d“ iB" ed


M
A3-phase, 50 Hr induction motor has a full-load speed of 960 rpm. Calculate
(/) number of the poles
(ii) slip freq u en cy

Scanned by CamScanner
— ob‘ _____________________________________ Polyphase In

(m) speed o f rotor field w ith resp ect to rotor stru ctu re ; w ith resp ect to s ta to r stru c tu re and w ith resp ect
to sta to r field. (Ans (fl) Nq (ft) Yeg (c) g 2 Hz 4() ^ 1000 rpm flnd zero

6 .1 1 . (a) Show t h a t th e voltage g en erated in th e rotor circu it of a 3-p h ase in d u ction m otor a t a n y slip s is
equ al to s tu n es th e voltage g en erated a t stan d still.
(b ) W ith th e help of rotor eq u iv alen t circu it o f an induction m otor, show th a t th e pow er tra n sfe rre d

m ag n etica lly from sta to r to rotor is given by l\ — p er phase.


s
(c) E x p la in th e term s air-g ap power P in tern a l m ech an ical power developed Pm and sh a ft pow er P,h. How
a re th e se term s rela te d w ith each oth er ? H ence show th a t
Pg : rotor ohm ic loss : Plu = 1 ; s : (1 - s)
6 -1 2 . (a) Ju s tify the following sta tem en ts for a 3-ph ase induction m otor :
( ’) R otor leak ag e im pedance a t sta rtin g is different from its value a t norm al ru n n in g conditions.
( « ) R e la tiv e sp eed betw een s t a t o r field and ro to r field is zero.
(h i) S ta to r cu rre n t rises as the sh a ft load is increased.

m otor ^ v a n a b ^e *®sses m a 3-phase induction m otor ? G ive the power-flow d iagram for th is
m otor and d iscu ss th e various losses involved in it.

te rm in a ls ^ L ' a u e n c f n f M U ***’ 4 ^ m otor is connected to 5 0 H z supply. A t th e rotor


L X ra tio o f Shn r ♦Y u Fm d th e P° 8sible 8peeds a t w hich th e rotor m u st be ^ v e n W h at
is th e ra tio o f sh p -n n g em fs a t th e se speeds a t no load ? |Ans. 600 rpm, 2400 rpm. unityl

dir. 6 ' 1 4 ' Y 4 ‘P° le ’ 3 *p h a s®- 5 0 Hz synchronous m achine h as its rotor directly coupled to th a t o f a 3-p h ase
! i 3 Y r T 0 S ta to rs o f both m achines are connected to th e sam e 3-p h ase, 5 0 H z supply I t is
D eterm in e th e n u m b « 150 Hz across th e rotor term in als o f th e in d u ction m otor.
D eterm in e th e n u m b er of poles for w hich th e induction m achine should be wound. G ive a ll p o ssib ilities.
(7.A.S., 1987) (Ans. 8 poles or 16 poles)
a t t h t ’ n n p r a t i ^ Y 8' del^ ' C0Tm®cted - 4 'P ole<50 Hz induction motor h a s a s ta to r re sista n ce o f 0 .4 f i p er p h ase
? tem p eratu re. F o r a lu ie cu rren t of 2 0 A, th e total sta to r inpu t is 4 0 0 0 w atts. F o r n egligible
s ta to r core lo sses, find out th e in te rn a l torque. (Ang 2 4 45 Nml

6 ' i ? ‘ A. 3 ' P.haSe; 4 3 0 V ’ 5 0 Hz induction motor ta k es a power input of 35 kW a t its full-load speed of 9 8 0
r.p.m . T h e to tal s ta to r lo sses are 1 kW and the friction and windage lo sses are 1.5 kW . C a lcu la te (a ) slip ( b)
) rotor ohm ic lo sses (c) sh a ft power (d) sh a ft torque and (e) efficiency.

(A ns. (a) 0 .0 2 (6) 6 8 0 W (c) 3 1 .8 2 kW (d) 3 1 0 .0 6 Nm (e) 90.91% )

u 6 1 7 * ^ 4 0 0 V ’ 3 Ph a s e > 6 P °le * 5 0 H z induction m otor d raw s a pow er of 2 k W a t no load an d a t ra te d


V 1 /Y x Y x t Y Uen2Cy a fulM oad shp ° f 3 % ’ th e P ° w e r in Pu t t0 m o to r is 5 0 k W an d th e s t a t o r o hm ic loss
,S i k ^ , NCg eCt/ R ! T a t n 0 ,l0ad - I f the sta to r core loss and m ech an ical losses a re assu m ed eq u al th e n at
a slip o f 3% c a lc u la te (a) rotor ohm ic loss (6) sh a ft (or output) pow er (c) sh a ft torque (d) in te rn a l torq u e and
(e) efficiency. (Ans. (a) 1.425 kW (6) 45.075 kW (c) 443.75 Nm (d) 453.60 Nm (e) 9 ? l S i j
6 .1 8 . A 2 0 kW , 6 pole, 4 0 0 V , 50 Hz 3-ph ase induction m otor h as a full-load slip o f 0 0 2 I f th e torou e lost
in m ech a n ica l (fn c tio n and w indage) losses is 20 N m, find th e rotor ohm ic loss, m otor in p u t and e f L e n c v
S ta to r lo sses to ta l 9 0 0 w atts. F wm -iency.
^ 20,000 x 60......... .................
IH“ ‘ - T" = 2 . » 1 0 0 0 x 0 . 9 8 * 194 88 ^ N" ’-
M ech a n ica l torq u e developed = 1 9 4 .8 1 + 2 0 = 2 1 4 .8 8 Nm.
. D 2n x 1000 (0.98) ............ . . I4 „
” m~ go (214.88) = ...J [Ans. 450.04 w atts, 23402.1 w atts. 85.462%1

6 .1 9 . A 10 kW , 3 -p h a se, 5 0 Hz, 4 pole induction m otor h as a full-load slip of 0 .0 3 . M ech an ical and stra y
load lo sses a t full-load a re 3 .5 % o f output power. Com pute
(a ) pow er d elivered by s ta to r to rotor,
( b ) electro m a g n etic (in te rn a l) torque a t full load, and 1
(c) rotor ohm ic lo sses a t full load. |Ans. (a) 10.67 kW (6) 67.93 Nm (c) 320.10 W|

J ■II IVy■
(Prob . 6
820 Electrical M a ch in e ry _________________________________________ ____ ________________________________________

6 .2 0 . A 3-ph ase induction m otor has its s ta to r copper loss e q u a l to th e su m o f th e [® "d


losses. Its rotor copper loss is equal to one-third of s ta to r copper loss. I f its e icien y <■, ^ 0347)
Take m echanical loss equal to iron loss.
6 .2 1 . The power supplied to a 3-phase induction m otor is 4 0 kW an d th e co rresp on d in g s t a t o r losses a re
1.5 kW. C alculate the n et m echanical power developed and th e ro to r I R loss w|jen ® lf c m r h r n n l i l
W h at will be the net power developed if the speed of the above m otor is reduce o c ^
speed by m eans of extern al rotor resistors, assum ing th e torque and s ta to r losses to re m a in u n a e re ^ ^ jggQ\
and windage losses may be assum ed to be 0 .8 kW. ^ ^ kw ^ k — kW]

6 .2 2 . (a) Discuss why the speed of a 3-phase induction m otor falls as its load to rq u e is in cre a se d .
(6) Two w attm eters are connected to m easure the power input to a 3 -p h a se induction m o to r ru n n in g a t no
load. One of the two w attm eters gives negative reading. W hy ? E xp lain .
(c) Explain why a 3-phase induction motor, in gen eral sim ilar to a tra n sfo rm e r, ta k e s m ore m a g n etizin g
cu rren t as com pared to a transform er.
6 .2 3 . (a) Explain why slip in a 3-phase induction m otor is d irectly p roportion al to to rq u e w hen o p e ra tin g
n ear synchronous speed.
(ft) A 4-pole, 2 0 kW, 5 0 Hz, 4 0 0 V SCIM has a sta rtin g torque of 1 6 0 N m and a full-load to rq u e o f 1 2 0 Nm .
C alculate
(i) startin g torque for a s ta to r voltage of 3 0 0 V,
(ii) voltage so th a t m otor operates satisfactorily a t full load from a 6 0 -H z so u rce,
(lii) voltage applied to sta to r so th a t full-load torque is developed a t sta rtin g .
V'
[H int. (ft) Keep j constant) [A ns. (ft) 9 0 Nm , 4 8 0 V, 3 4 6 .1 4 V]

6 .2 4 . (a) A 3-phase, 5 0 Hz source feeds the sta to rs of both 3 -p h a se SR IM h a v in g 8 poles an d a synchronous


m otor having 2 poles. The synchronous m otor, coupled m ech an ically w ith in d u ction m o to r, ru n s clockwise
w hereas the ro tatin g field in 3-<J> induction m otor ro ta te s counterclockw ise.
C alcu late frequency of the voltages taken from th e slip rin gs of SR IM .
(6) Find the num ber of synchronous-m otor poles and th e conditions for o b tain in g a freq u en cy of 150 Hz
from the ro to r of SRIM of p art (a).

(c) R epeat p art (ft) for obtaining a slip-ring frequency of 16 - Hz.


3
[A ns. (a) 250 H z (ft) 4 poles, synchronous m otor ro ta tio n a g a in st th e d irectio n o f ro ta tin g field in SRIM (c)
6 poles, synchronous m otor rotation in the direction of ro ta tin g field in SRIM.
6 .2 5 . A 4-pole, 3-ph ase SRIM is coupled m ech an ically w ith a sv n rh rrm m ,c u , r™
synchronous m otor and sta to r of the induction m otor a re fed from n 7 ,n 8 P
frequency o f th e em fs a t the ro to r term in als if the sy n ch ro n ou s moTor k d r i v e n ^ S° UrCe' * * ‘ he
(a) in a direction opposite to the induction m otor s ta to r ro ta tin g field,
(ft) in the direction of ro tatin g field in SRIM .

If the frequency of th e ro to r voltage is req u ired to be 3 0 0 H z, th e n calculate


(c) th e n u m b er of poles th a t th e SR IM m u st h av e ,a , v, ,
c . IAns - (n ) 150 Hz (ft) 5 0 Hz (c) 10 poles)

sta to rs of both th e m achines a re given a S O -H ^ su p p lT w h a t f r e a ^ ^ 4 P° le ’ 3 ‘ p h a s e sy n ch ro n o u s m otor. If

(6) A 3-p h ase, 5 0 H z indue Uon m otor has a ^ 2 * ^ ^ ^ °f ^


m axim um torque w hich is 2 .5 tim es fuil-Ioad torq u e N e e l e d W S t 15 Um eS fu lM oad tor(lue and a
assu m in g co n sta n t ro to r re s is ta n c e , find : “ s t a t o r r e s is ta n c e a n d ro ta tio n a l losses and

(i) the slip a t full load (ii) th e slip a t m axim u m to rq u e and

(H
i)th e ro to r c u rr e n t a t s ta rtin g in p er u n it of full-lond ro to r c u r r e n t. , , . E .S .. W ?S|

n , , lA n s .(n ) 2 5 Hz or 125 Hz (ft) 0 . 0 5 6 , 0 . 2 6 8 . 4.7221

diagrum of aCtransformer ?80r ^ * * " P“'y,,1,“SC induCtil>'' » — •How does i, differ from the phasor

^s S tD
„Th?tprall ? : i ; t i : lt c “/eu irlyphasc induction ra°i°r-

Scanned by CamScanner
P ro b . 6]
Polyphase In d u ction M o to rs 821

circ u it p a ra m e te rs ? ' nt*u c t' on n iotor eq u iv alen t circu it, w h a t should be k ep t in m ind re g a rd in g th e e q u iv a le n t

W T h ^ s h u n t b r 1110 Cm uii ° f 0 0 mot°r- W h at do th e various p a ra m e te rs rep resen t ?


d u rin g th e a n a ly s is nf im *s t ' nS ° ^ c a n d X ,,, in p a ra lle l, is e ith e r o m itted or m oved to th e p rim a ry te rm in a ls

ind u ction m o to r e q u iv a le n t^ r c u T ^ E x p y a in '6111 C' rCU*t- T ^ is’ how ever, is n ° t p erm issib le in th e a n a ly sis o f

W> b" W,!en th e tr a n s f “r m c r “ d ta d “ «<>" " » t » r e q u iv a le n t c i r c u i t ,


I H i n t .( d ^ B e c a u s e o f t h ' 13" ' 63 * " “ * • ' * ™ l ™ ,h a " in ‘ " - “ fo rm ers. E x p la in ,
le a k a g e re a c ta n c e .) ^ ° Ce ° ^ a *r ®a P> induction m otor h a s m ore le ak ag e flux an d , th e re fo re , m ore

to rq u e p e r p h a se is g iven by Auction m o to r is n eglected , show from its e q u iv a le n t circu it t h a t m a x im u m

T - 1 ^
em 2 n n ,2 T 2

and hence show th a t T'


T..„
em s ^ S
smT
S smT
F o r sm a ll v a lu e o f slip o ccu rrin g in th e sta b le o p eratin g region prove th a t

r1' = -2 F„„ S
smT

m o d ified ' S k e ‘ Ch ‘ h e lyPiCa' * a r a c t e r i s , i c o f an induction m otor. How is th is c h a ra c te ris tic

(i) if its ro to r-circu it re s is ta n c e is in crea se d ,


(ii) if its ro to r circ u it re a c ta n c e is in crea se d

a t f„Sh,„“ „e e t rrg e " ? " " “ * d' » ° “ 'd * •

m a r i m u m t t u ? a U U r i t a g ° f “ ‘ e m a ' reS iS ,a" Ce ‘ h a l be i,1Serted “ *h ' » • « * « ■ » in o rd e r to obtain

^ [H in t. (W U) I f s t a t o r re s is ta n c e w ere con sid ered then th e slip a t w hich m a x im u m to rq u e o ccu rs, is given

s,nT=^ J 7 F
S in ce smT is red u ced w ith th e co n sid eratio n of s t a t o r re s is ta n c e , th e full-load slip w ould be sm a lle r.
r2 0 fl4
( « ) y = s,uT = 0 .2 o r X = = 0 .2 « etc.)

[A ns. (a) (,) See Fig. 6 .1 7 ( « ) All the three T „ „ Ttm and smT a re reduced. (6) « ) 0 .0 2 5 4 , sm aller, (ii) 0 .1 6 O).

6 .3 2 . (a ) S k e tc h th e torq u e-slip c h a r a c te r is tic o f an induction m o to r w ork in g a t ra te d v o lta re and


E x p la in an d d ra w th e s e c h a r a c te r is tic s , w ith re s p e c t to th e n orm al o- e, if th e follow ing ch a n g e s a re m a d e ^
(i) A pplied s t a t o r v o lta g e is red u ced to h a lf a t ra te d freq u en cy.

(ti) B o th th e ap p lied v o lta g e an d freq u en cy a r e red u ced to half.

(6 ) F o r a 3 -p h a s e in d u ction m o to r, th e ro to r o hm ic loss a t m a x im u m to rq u e is 16 tim e s t h a t a t full lnnd


torq u e. T h e slip a t fu ll-load to rq u e is 0 .0 3 . I f s t a t o r re s is ta n c e an d ro ta tio n a l losses a r e n eg le cte d , th e n c a lc u la te
(i) th e slip a t m a x im u m to rq u e,

(ii) th e m a x im u m to rq u e in te rm s o f full-load to rq u e an d

(iii) th e s t a r t i n g to rq u e in te rm s o f full load torq u e.

u u a i ii i c u vjy u a i i ixj^ ai 11 i c i
r
[Prob. 6
822 Electrical M achinery _______ "

T'f, 2
IHint- (6)
Sp s mT

I 21
T.n sfl S,nT- — SmT
Now -J-L
7 ? L= - — 75------------ 7Sn
r. = j2 7 ~ 16
16 S fl
i , 2 2 ‘ 2m T 2 '* '
. / r , . T --- ----------
/2'"r *,nr
1 s-nr 2 .e|c_
" . 16 Sfl SmT + Sfl
Sfl s mT
[Ans. (a) (i) Tttl and Ttm are reduced to ^, but n, and smT remain unchanged.
w Z- increase, T „ re ra -in , same but a . is reduced te half.

(b) (i) 0.167 (ii) 2.874 T ,n (Hi) 0.934 Tt fl.\


6 .3 3 . (a ) Show th a t th e m axim um in tern al torque developed by a polyphase induction m
depend on th e rotor circuit resistan ce.
(6) A 3-ph ase squirrel-cage induction m otor h as a rotor sta rtin g cu rre n t o f 6 tim es its full load value. The
m otor h as a full load slip of 5% . D eterm ine
(i) th e sta rtin g torque in term s of full-load torque ;
(ii) th e slip a t which m axim um torque occurs ; and
(Hi) m axim um torque in term s of full-load torque.
(H in t, (i) U se Eq. (6 .5 1 a). (ii) U se Eq. 6 .3 7 . lAns. (b) (i) 1.8 Tr „ (ii) 0 .3 1 (Hi) 3 .18 Ttfl]

6 .3 4 . (a) W ith s ta to r resistan ce neglected, th e torque-slip c h a ra c te ris tic can be obtained from th e expression

Tt
T eni s mT S
s s Tm

D erive this expression and show th a t

s = sm7 > ± VA:2 - 1]

where ^ = -sr-
1e
(U se n egative sign if smT > s, e.g., smT > and use positive sign in ca se s mT < s, e.g. a t s ta rtin g ).

(b ) The m axim u m torque of a 3-p h ase sq u irrel-cage induction m otor is 4 tim e s th e full-load torq u e and the
sta rtin g torque is 1.6 tim es th e full-load torque. N eglect s ta to r re s is ta n c e . C a lc u la te

(i) th e slip a t th e m axim u m torque


(ii) full-load slip and
(iii) th e ro to r cu rre n t a t sta rtin g in term s of full-load ro to r cu rre n t. (A ns. (i) 0 .2 1 , (it) 0 .0 2 6 7 , (Hi) 7.7591

6 .3 5 . (a ) S k etch th e torque-speed curve of a conventional in d u ction m o to r an d in d ica te how th is will change


w hen
(i) th e ro to r re s is ta n c e is doubled, keeping s ta to r v oltag e an d freq u en cy u n ch a n g e d ;

(ii) th e applied voltag e is halv ed , th e freq u en cy and ro to r re s is ta n c e re m a in u n ch a n g e d ;

(iii) both th e applied v oltag e and freq u en cy a re h alv ed , ro to r re s is ta n c e re m a in s u n ch an g ed .


(I.A.S.. 1987)

(b ) A 3-p h a se induction m o to r a t ra te d voltage an d freq u en cy h a s a s t a r t in g to rq u e of 1 5 0 % an d a m axim um


torque of 2 0 0 p e rce n t of full-load torq u e. N eglectin g s ta to r re s is ta n c e a n d ro ta tio n a l lo sse s, ca lcu la te the slip
a t full load and slip a t m a x im u m torq u e. (/.A .S ..1 9 5 4 )
[Ans. (6) 0.452, 0.1211

Scanned by CamScanner
Prob. 6]
Polyphase Induction Motors 823

N m a t a sp eed o f 1 2 o \ ) r n n ! d e l t a - c o n n e c t e d in d u ction m o to r d evelops a m a x im u m to rq u e o f 2 4 0


is 0 .2 fl p e r p h a se C a lc u la te t h J • a i^ e & r o ta tio n a l losses a r e a ssu m e d n egligib le. T h e ro to r r e s is ta n c e
(a ) 7 W r ^ alCUlate th e ad d itio n al ro to r re s is ta n c e re q u ire d to give a s t a r t i n g to rq u e o f :
(а ) 7 5 % o f m a x im u m to rq u e a t ra te d v o lta g e and

(б) 7 5 % o f m a x im u m to rq u e w hen th e su p p ly v o lta g e is red u ced to 3 6 0 v olts.

,Hint‘ smT = 0.2 ; x2 “ = 1 II.


0. s'"^
S in ce r t a n d x , a r e n egligib le,

T - i K
«». 2x2 ~ 2
or 2 40 = -
2
K = 480.

fa ) I f R is th e su m o f r o to r re s is ta n c e a n d e x te rn a l re s is ta n c e , th e n a t s t a t i n g

re sl ~- J - xX_ 3V? -
[«2 + *^l
or 0 . 7 5 x 2 4 0 = —^ — R etc
■ R 2 + 1 ,e tc -

<6 > 0 .7 5 x 2 4 0 = f M f _ i 8 0 _ R
1,400 J fl2 + i ’ ‘ IA n s. (a) 0 .2 5 1 4 Q, (6) 0 .4 7 2 fl]

t h a t to in c re a s e th e s t a r t i n g t o r q u ^ e x t S S ^ M t t e r o t o ! * ^ m 0 t° r Ke“ “ P r° V<i

P h a s e r ™ ts ^ V o P2hr c S . t X ^ l de ^ 'r 3 "17 * ^ l0">” at * • *«* °f W » ^ ™ «P-


e a ch r o , h fa p ro d u ce a ~ r £ e ^ ^ ^

[H in t, (a ) C on d ition for m a x im u m to rq u e is

r2
...(6 .2 5 )
T h e e x te r n a l r e s is ta n c e in ro to r c irc u it a t s ta r tin g is

(V ^ + J^ -r,).
(8) *- ^
2 smr, 1
*■f
1 s m T1

an H _ r 2 * added resistance
2 sistance
s m T, =
. [A ns. (6) 0 .3 3 6 fl)
6 .3 8 . ( a ) E x p la in th e d ifferen ces b etw een th e c h a r a c te ris tic s n f «lirx ,
in d u ction m o to rs. S k e tc h a ty p ica l c h a r a c te r is tic for e a ch . sq u irre l-c a g e polyphasi

(6 ) A 4-pole, 3-phase, 5 0 H z induction motor has a full-load sliD o f 3% »nH » mnar- . a


full-load to rq u e . C le a rly sp ecifyin g an y ap p ro x im a tio n s you m ay n eed to m ak e calcuTaTe^he s t a T ' th<
th e m o to r a s a p e rc e n ta g e of full-load to rq u e ca lc u la te th e s ta r tin g to rq u e o
(I.E.S.. 1975
co o t o u . , . [A ns. (6) 44 .2 4 %
b .d 9 . In a 3 -p h a s e in d u ctio n m oto r, th e s t a t o r r e a c ta n c e eq u als th e ro to r r e a c ta n c e a t et a „ j c n i u . i
r e s is ta n c e is o n e-fo u rth o f th is v alu e. I f th e m o to r develops 2 2 0 N m a t 3% slip, w h a t w ill b T i J f
(a ) s t a r t i n g an d

< W p u ll-o u t to rq u e s. Ig n o re no-load c u rr e n t. IA n .. 1 2 0 .6 7 N m . ( » 4 5 2 737 Nm

mot°r ha> a ro,M °f 0 2 0 P" ph“ « “ 0 * —


(a ) th e to rq u e for a fu ll-lo ad slip o f 4 % an d

Scanned by CamScanner
[Prob. 6
824 E lectrical M achinery _----------- !— :---------- ------------- - " 7 ..
------------ " . • , u* n of full-load torque a t starting.
(fe ) t h e resistance to b e added to the rotor circu it o o a |Ans. 9 2.89 Nm, 0.192 Q]

R otatio n al losses and s ta to r im pedance are neglected. ^ ^ ^ ^ ^ ^


6 .4 1 . A 4-pole, 3-p h ase, star-co n n ected slip-ring induction mo F o r negligib]e s ta to r impedance
m ains. Its rotor h as a stan d still leakage im pedance of 0 .4 + y2 ohm s p f

and rotation al losses, com pute :


(а) m axim um torque in new ton-m etres, g ^ m axim u m torq u e a t s ta rt.
(б) the resistan ce to be included in the rotor circu it to ae v • ^ ^ ^ ^ m (fc) Q6 Qj

j ■ tor h as s ta to r im p ed an ce of 0 .0 7 + j 0 .3 0 O and
6 .4 2 . A 420-V , 6-pole, 5 0 Hz s ta r ^ m agn etizin g c u rr e n t is n eglected . D eterm ine
stan d still rotor im pedance referred to s ta to r is 0 .0 8 + 7 0 . 3 7 U. i n g
(a) the m axim um internal power developed and the corresponding slip and

(b) the m axim um internal torque and the slip at ^ RW Q 1Q44 . (6) 108 8 .1 7 6 Nm, 0.1187]

. v d in a t normal voltage. Find th e slip o f th e induction motor


6 .4 3 . (a ) A 3-phase inductionp^ t ° r ^ s^ a sli,p ^ ^ ^ th e n o rm ai voltage.
w hen developm g the sam e torque bu g tnrmlP a n d a t n orm al voltage. T h e ro to r resistan ce
^ .(b ) A 3-ph ase induction m otor h as 5% slip a t full-loa q W h a t should be th e percentage
- 3 ,4 th of n o ™ , full-load speed.

N eglect s ta to r im pedance.
[Hint, (a) U se E q . (6 .3 1 )
V\ 0.10
(*>) T* P = T T T Iv x 0.05
r0.10] + ( 0 .6)2
0.05 J
(*V?) 0.10 [Ans. (a) 4.938% , (6) 20.356%]
X 0.2875 CtC‘
6 4 4 A 440-V 3-phase, 4-pole, 5 0 Hz slip-ring star-co n n ected induction m otor h a s a voltag e of 8 0 V between
slin Hnirs when full-voltage is applied to the sta to r and the slip-rings a re o p e n -c rc u ite d w ith the rotor
s a t i o n a i The sta to r cu rren t a t no-load is 2A a t a p f of 0 .2 lagging. T h e ro to r is sta r-co n n e cte d w ith a per
phase stan d still leakage im pedance of 0 .0 5 +j 0 .2 5 fi referred to rotor. F o r th e m o to r ru n n in g w ith th e slip-nngs
sh ort-circuited and a t a slip of 5% , calculate
(a) the torque developed in Nm,
{b ) the mechanical power developed,
(c) the rotor ohmic loss and •
(id) the stator current and power factor. -9
Neglect stator leakage impedance and rotational l’g sses.
[H in t, (d) R otor cu rre n t a t 0 .0 5 slip, / 2 = 4 4 .8 1 A.

Rotor pf at 0.05 slip, cos 02 = 0.970


Rotor current in phasor form, 72 = / 2 (cos 02 - j sin 02) = 4 3 .4 7 - j 10.89.
Stator current required to balance the rotor current is given by
" _1_T
5.5

T otal s ta to r cu rre n t, I] = ( / 1 ' + I q)

= - b h + ( 0 . 4 - 7 1 .96) = (8 .3 - j 3 .9 4 ) A e tc.].
5 .5 J
[Ans. (a) 3 8.35 Nm (6) 5 7 2 2 .8 w atts (c) 3 01.2 w atts (d) 9 .1 8 8 A at 0.903 pf lagging !

6 .4 5 . (a ) A 3 -p h ase induction m otor h as o p eratin g p f of 0 .8 5 a t full-load sp eed of 9 6 0 r.p .m . and a t 400 \


supply voltage. In case th e supply voltage falls to 3 5 0 V, find th e o p e ra tin g p f a t th e sa m e full-load torque.
(b ) A 3 -p h ase induction m o to r h as a ro to r re s ista n ce of 0 .5 D p er p h ase an d ro to r sta n d still leak ag e roa' ta " 2
of 1 5 O p er p h ase. I f th c ratio of m axim u m s ta rtin g torq u e to full-load to rq u e is 2 , find th e ratio o
s ta rtin g torque to full-load torque for d irect sta rtin g . N eglect s ta to r im p ed an ce a n d ro ta tio n a l losses.

Scanned by CamScanner
Prob. 6] Polyphase Induction Motors 825

[H in t, (a) 0 .8 5 = t= s ^ = y •
Vr22 + ( 0 .0 4 * 2)2
This gives r 2 = 0 .0 6 4 5 4 jc2.
Slip a t reduced voltage = 0 .0 5 2 2 etc.
(6) M axim um startin g torque can, at the most, be equal to maximum torque Tem. H ere smT = 1 /3 .

Te-it 2
etc. (A ns. (a) 0 .7 7 7 5 lag (6 ) 0 .6 1 4 3 )
T<m ' 1 /3 1
1 1 /3
.4 6 . A 3 -p h a s e , 5 0 H z, 4 0 0 -V w ound-rotor induction m otor ru n s a t 9 6 0 r.p .m . a t fu ll-load . T h e ro to r
resis a n ce an d sta n d still re a c ta n c e p er p h ase a re 0 .2 11 an d 1 £1 resp ectiv ely . I f a re s is ta n c e of 1 .8 £2 is ad d ed
o e a c h p h a se o f th e ro to r a t stan d still, w h a t would be th e ra tio o f s ta rtin g to rq u e w ith full v o lta g e an d th e
added re s is ta n c e to th e full-load torq u e u n d er n o rm al r u n n i n g conditions ? S ta te a ssu m p tio n s m a d e in y o u r
ca cu latio n s. C a n th e sa m e s ta r tin g torq u e be obtained w ith a n o th e r v alu e of th e a d d ition al r e s is ta n c e ? E x p la in .
It th e a n sw e r is y e s, find its v alu e. ( I E S 1979)

H in t . Te.,t = — • | v a ; r e/r = ^ - . - 5 - V 2 e tc
esl (os 5 ’ efl (og 2 6

[A n s. A ssu m p tio n s, (i) s ta to r im p edance and ro ta tio n a l losses ignored an d (ii) in d u ction m o to r p a ra m e te r s
re m a in co n sta n t. 2 .0 8 . Y e s , b u t w ith an additional re sista n ce of 0 .3 £1).

6 .4 7 . A 4 0 k W , 3 -p h a s e slip rin g induction m otor of negligible s ta to r im p edan ce ru n s a t a sp eed o f 0 .9 6


tim e s syn ch ron ou s speed a t ra te d torque. The slip a t m axim u m torq u e is 4 tim e s th e full load v alu e. I f th e
ro to r re s is ta n c e of th e m o to r is in creased by 5 tim e s, d eterm in e

(a ) th e sp eed, pow er o u tp u t and ro to r ohm ic loss a t ra te d torque


(b) th e sp eed co rresp on d in g to m axim u m torque.
N eglect m e ch a n ica l losses. (GATE 1993)
[Ans. (a) 0 .5 2 Ns, 2 1 .6 7 kW , 20 kW (b) 0 .0 4 Ns]
6 .4 8 . A 5 k W , 4 0 0 V , 5 0 H z, 4 poles d elta con n ected 3-p h ase induction m o to r is su p p lied by a cab le of
negligible in d u ctan ce. On s ta rtin g th e m otor u sing a s ta r-d e lta s ta r te r , it is found t h a t th e s ta r tin g to rq u e is
th e sa m e on s t a r a s w ell as d elta connection, due to th e v oltag e drop in th e feed er re s is ta n c e . T h e eq u iv alen t
circu it p a ra m e te rs of th e m o to r a re a s follows :

ri - 1 £1, * i = 4 .5 £1, r 2' = 1.4 Cl and x 2' = 4 .5 Cl


D eterm ine th e feeder resistance. (GATE 1991 )
[Ans. 5 .3 7 8 £1]
6 .4 9 . A 4 0 0 V, 5 0 H z, 3 -p h ase, d elta connected, 6-pole induction m o to r h a s th e follow ing sta n d still le a k a g e
im p ed an ces p er p h a se refe rre d to s ta to r:

z 1 = 1.2 +>2.1 £1, z2 = 1.32 + >2.2 £1

N eg lectin g m a g n etizin g im p ed an ce, d eterm in e


(a ) g ro s s o u tp u t p ow er a t a slip of 0 .0 4 ,

(b) th e ch a n g e in line c u rr e n t w hen th e s ta to r te rm in a ls are sw itch ed from s t a r to d e lta a t a slip of 0 4


(c) th e re s is ta n c e to be added in e a ch ro to r p h ase to obtain a s ta rtin g torq u e of 3 5 0 N m

[Ans. (a) 12 7 9 8 .6 W (b) 3 7 .1 0 5 A, 11 1 .3 0 9 A (c) 1.0 8 £1 or 6 .9 8 Cl]


6 .5 0 . A 1 0 k W , 6-pole, 5 0 H z, 3 -p h a se induction m otor h a s lin e a r torq u e-slip c h a ra c te r is tic s b etw een zero
torq u e an d m a x im u m to rq u e. Th e slip a t w hich m a xim u m torq u e of 5 2 0 N m o ccu rs is 0 .2 . F o r m e ch a n ica l losses
of 6 0 0 W , fm d th e sp eed a t w hich th e m o to r would ru n w hen d eliverin g ra te d sh a ft pow er. [Ans. 9 5 9 .5 rpm)

6 .5 1 . T h e n o rm a l full-load slip an d s h a ft torq u e o f a 3 7 3 kW , 5 0 H z, 3 -p h a s e ind u ction m o to r a re resp ectiv ely


1.9% a n d 2 4 0 0 N m . T h e ro to r w in ding h as a re s is ta n c e of 0 .2 5 £1 an d a sta n d still le a k a g e re a c ta n c e of 1 .5 £1
p er p h ase. E s tim a te th e slip an d th e pow er o u tp u t for th e sa m e full-load c u rre n t w hen e x te rn a l re s is to rs of
2 £1 p er p h a se a r e in s e rte d in th e ro to r circu it. N eg lect no-load cu rre n t. [/.A.S., 79971

[H in t. F o r so lu tion , re fe r to C h a p te r on P oly p h ase Induction m a ch in e s in th e book “G en eralised T h eory


of E le c tric a l M a ch in e s” by th e sa m e a u th o r.) [Ans. 17.1.% , 3 1 5 .2 0 6 kW)
826 Electrical M a c h i n e r y _____________________________________________________________________ [Prob . 6

6 .5 2 . A 3-p h a se indu ction m otor h as 4-pole sta r-co n n e cted s ta to r w in d in g an d ru n s on 4 0 0 V , 5 0 Hz supply.


T h e rotor re sista n c e is 0.1 Q and re a cta n c e 0 .9 O. T h e s ta to r to ro to r tu rn s ra tio is 1 .7 5 an d th e fu ll-lo ad slip
is 5% . C a lcu la te th e full-load pow er output. D ete rm in e also th e m a x im u m to rq u e an d th e co rresp o n d in g speed.
[7.A.S., 1998\
[A ns. 1 2 1 .0 8 0 6 kW , 184.791 Nm, 1333.5 rpm]

6 .5 3 . A 3-p h a se, 6 -pole, 5 0 H z, 4 0 0 V S C IM h a s p e r-p h a se ro to r le a k a g e im p e d a n ce = 0.8 + j 2 .4 Q a t


sta n d still. R oto r copp er b a rs o f th is m otor a re rep laced by a lu m in iu m b a rs . D e te rm in e how th e follow ing
perfo rm an ce p a ra m e te rs ch an g e w ith re sp ect to th e ir previou s v a lu es :
S lip a t m ax im u m to rq u e, s ta rtin g torque, m axim u m to rq u e, fu ll-lo ad slip .
T a k e re sistiv ity = 2.1 p fl cm fo r copper an d 3 .4 p fl cm for a lu m in iu m . [A ns. 1 .1 6 2 5 , 1 .2 267, 1.0, 1.1625]
6 .5 4 . A 3-p h a se, 5 kW , 4 0 0 V , 5 0 Hz S C IM h a s th e follow ing d a ta :
F lu x p er pole = 4 0 m W b, m ean conductor le n g th = 18 cm
C opper d en sity = 8 .9 4 gm /cm 3, cu rre n t d en sity = 4 A / m m 2.
F u ll-lo ad efficien cy = 0 .8 4 and pow er fa cto r = 0 .8 5 .
C om p are th e am o u n t o f copper requ ired for s ta to r w in d ing i f th e in d u ctio n m o to r is d esig n e d to o p erate
w ith s ta to r w ind ing in (a) d elta an d ( b ) sta r.
[A ns. ( a ) D elta : T o ta l tu rn s = 144, cond u ctor w eig h t = 0 .6 7 6 2 kg.
(b ) S t a r : T o ta l tu rn s = 8 4 , cond u ctor w eig h t = 0 .6 8 3 1 kg.]
6 .5 5 . A 3 -p h a se ind u ction m otor is designed to o p era te a t ra te d v o lta g e V a n d r a te d fre q u e n c y f . In ca se
both sou rce voltage an d freq u en cy a re re sp ectiv ely ch a n g ed to (a ) ~ , f / 2 (6 ) V , f / 2 (c) — , f
£ 2
(d ) V, 2f ; find th e m axim u m and s ta r tin g to rq u e s in te rm s o f th e ir ra te d v a lu e s . N e g le c t a ll s t a to r losses.

A n s . (a) 1, 2 ( 6 ) 4, 8 (c) i i (d ) J , |

6 .5 6 . (a ) W h a t a re th e fa cto rs th a t govern th e o p e ra tin g c h a r a c te r is tic s o f p o ly p h a s e in d u c tio n m otors


(W In ca se o f p o ly p h ase in d u ctio n m otors, ex p la in w hy th e ro to r sp e ed f a lls a s th e lo a d to rq u e is in creased .
(c ) R ow er fa c to r o f a p o ly p h ase in d u ctio n m o to r is low a t n o -lo ad b u t it im nrn v p s
is in crea sed . E x p la in . ’ im p ro v es a s th e lo a d on th e m otor

one offfie? Z £ £ Z Z S S tZ S S X ; *“ “ “ * —**


(a ) M o to r A h a s open s ta to r s lo ts b u t m o to r B h a s sem i-clo se d s t a t o r s lo ts .
Cb) M o to r A h a s sem i-clo sed ro to r slo ts b u t m o to r B h a s clo sed ro to r slo ts .
(c) M o to r A h a s lo n g er a ir-g a p th a n m o to r B.
F o r e a c h o f th e th r e e d iffe re n c e s lis te d ab o v e, d isc u ss w h ich o f t h e tw o m o to rs h a v e •
(i) b e tte r s t a r tin g to rq u e
( i i ) h ig h e r b reak d o w n to rq u e

( i i i ) b e tte r fu ll-lo ad pow er fa c to r and

( i v ) h ig h e r fu ll-lo ad speed . ..

6.58. fa) In case of induction motors exnlain „h„ ,h • , ^


possible. ’ P hy the air-gap length is kept as small as is mechanically
(b) Show that polyphase induction motors possess shunt ♦ .
W A 3-phase squirrel cape induction motor I d o lled “ o h * 7 “ '
aluminium is used for rotor bars, exnlain what hnrm.n. t •* [ copper bars. If, instead of using copper,
conditions. ’ ^ What haM>ens >«• speed, efficiency etc. under normal running

therete% ffic^ yda ^ l c ^ r main8*— •*— °*'mic 1— increase, output and,'

from no-load and blocked-rotor tests^Md* pw p L ^ ^ t O T w r n d ^ B ^ ^ i i t ^ c r 0" m°l°r be daten"ined


CMA 400 V. 50 Hz, 3-phase star-connected squirrel-cage induction motor^ave the following test results :

Scanned by CamScanner
P rob. 6] Polyphase Induction M o to rs 827

No-lond or o p en -circu it te s t (lin e v a lu es) : 4 0 0 V , 9 A, 5 6 0 W . B lo c k e d -ro to r o r s h o r t c ir c u it t e s t (lin e


v a l u e s ) : 2 1 0 V , 3 6 A, 4 8 2 0 W.

T h e effectiv e s ta to r re sista n ce is 0 .7 2 f l per p h ase. C a lcu la te th e eq u iv a le n t c irc u it p a ra m e te rs .


IA n s. r , = 0.72 fi, r 2 = 0 .59 fl, x , = x 2 = 1.567 Q , X ,n = 24 D|

6 .6 0 . (a) W h a t is re p resen ted by th e circle d iag ram o f an induction m otor? W h a t in fo rm a tio n ca n b e o b ta in e d


from it ?

D erive th e cu rren t-lo cu s for th e ro to r-circu it o f a polyphase in d u ction m otor.


(6 ) Show th a t th e d iam e te r o f current-locbs^circle o f a polyphase in d u ction m o to r is V j/ X j + x 2 , w h e re Xj
is th e per p h ase s ta to r le a k a g e re a cta n c e , x 2 is th e 's ta n d s till per p h a se ro to r le a k a g e re a c ta n c e re fe rre d to
s ta to r and V , is th e per p h ase s ta to r voltage.

6 .6 1 . (a) E x p la in how th e circle diagram for a polyphase induction m otor ca n be d raw n from its t e s t d a ta .
( 6 ) A 4 0 0 V , 3-p h a se, 8 pole, 5 0 Hz sta r-co n n ected induction m otor gave th e follow ing t e s t r e s u lts :
No-load test (line v a lu e s ): 400 V, 10 A, cos 0O= 0.2.
Blocked-rotor test (line v a lu e s ): 160 V, 30 A, cos 0,f = 0 . 35.

If, a t full load and rated voltage, th e power fa cto r is a t its m axim u m , th e n c a lc u la te fu ll-lo ad c u rre n t,
pow er factor, torq u e in n ew ton -m etres, speed, power output and efficiency. S ta to r and ro to r o h m ic lo sse s a re
equaL lA ns. 28.75 A, 0.806, 174.224 Nm, 6 98.7 r.p.m ., 12.644 kW, 78.834% 1

r n A 4 klV’ 400 Y’ 50 Hz‘ 3 ‘p h a se >4 -Pole d elta connected slip rin g ind u ction m otor h a s s ta to r r e s is ta n c e
o f 0 .3 6 D p er p h ase, rotor re sista n ce o f 0 .0 6 D per phase and per p h ase s ta to r to ro to r tu rn s ra tio o f 2 T h e
following d ata p e rta in s to th e line valu es d uring lig h t load te sts :
No load ; 400 V, 3.3 A, cos 0(1= 0.174
Locked rotor : 210 V, 16 A, cos 0 = 0.45

D raw th e circle d iag ram and com pute :


(a) lin e c u rre n t, pow er factor, slip, torque and efficiency a t full load,,

torque^ ^ ° p e ra tin g pow er fa cto r. m axim um power output, m axim u m to rq u e in N m an d slip a t m a x im u m

startin g 1116 eX te m a l re sista n c e th a t m u st be in se rted in series w ith ro to r circ u it to o b ta in m a x im u m to rq u e a t

(Ans. (a) 8.66 A, 0.8434, 0.057, 26.738 Nm. 77.65% (6 ) 0.852. 6.42 kW, 5 0.04 Nm, 0 .2 1 (c) 0 .2 2 6 4 f l referred to
1ULUI *J *

6 .6 3 . (a ) I f no-load pow er facto r a n gle 0O for a polyphase induction m otor is ta k e n to b e e q u a l to 9 0 °, show

w ith th e help o f circ le diagram th a t th e b est possible op eratin g pow er fa cto r is given by V*
V , + 2 / 0 (x , + x 2) ‘
(b) Tw o in d u ction m otors A an d B a re id en tical in a ll resp ects exceDt th a t m ntnr 4 c i

r : M t T p o t ^ r r win possess hie,,er v a J °f £ « r ^ x pow:sr “ dr


(H in t. (6 ) In d u ctio n m otor le a k a g e re a cta n ce is d irectly proportional to th e n u m b er o f poles.)
lA n s. ( b ) M otor B . C on sequ en tly slow -speed ind u ction m otors (m otor A h e re ) o p era te a t poor pow er fa cto rs I
6 .6 4 . (a ) W h a t a re th e cau ses o f an induction m otor o p eratin g alw ays a t lag g in g pow er fa cto rs ?
E x p la in how th e pow er facto r o f an induction m otor is controlled by s ta tic ca p a cito rs Show th a t for n
co n sta n t ca p a cita n ce , th e d egree o f pow er fa cto r co rrectio n is n ot th e sa m e a t d iffere n t load s.
(6 ) A d elta-co n n ected ind u ction m otor o p eratin g from a b alanced 3 -p h a se, 4 0 0 V 5 0 Hz suddIv tn k e« n
cu n-ent o f 5 0 A a t 0 .7 6 p.f. lag. C a lcu la te per p h ase v alu e o f th e ca p a cita n ce and to tal’ kV A ra tin g o f th e 3 D h o s o
d elta-connected ca p a cito r b an k w hich w hen connected to th e m otor te rm in a ls would im prove th e lin e pow er
factor to 0 .9 lag. | w 6 4 „ mF „ 768PkVA]

6 .6 5 . (a) D iscu ss why th e pow er fa cto r o f a 3 -p h a se ind u ction m otor is low a t (a ) no-load and ( b ) also u n d er
overloads. UCI

(6 ) A th re e -p h a se ind u ction m otor ra ted a t 4 0 0 V, 5 0 Hz, coupled to a pum p is ru n n in g a t a low pow


lacto r o f 0 .6 . T h e inp u t is 4 .5 kVA. It is proposed to im prove th e pow er fa cto r to 0 .8 by co n n ectin g a
d elta-conn ected ca p a cito r b an k . F in d th e v alu e o f th e ca p a cito r p er p h ase. (G A T E , 1 9 9 -i) (A ns 18 104 pFl

■■ ■iKJNNriiwnnroi ■■ wi
Scanned by CamScanner
828 Electrical Machinery ________________________ [Prob. 6

6 .6 6 . (a ) W h a t is th e o b je ctio n to th e re d u ce d -v o lta g e s t a r tin g o f p o ly p h a se in d u c tio n m o to rs ?


( b ) U n d e r w h a t co n d itio n s is th e d ire ct-o n -lin e s t a r tin g o f p o ly p h a se ca g e in d u c tio n m o to rs p re fe rre d ?

(c) T h e s ta r tin g period from zero to n o rm a l sp eed is m o re s e v e re in c a s e o f d .c. m o to rs th a n in in d u ctio n


m o to rs. E x p la in .
(d) A 3 -p h a s e in d u ctio n m o to r a t n o rm a l sp eed is c a r ry in g a b o u t h a l f th e r a t e d lo a d . I f o n e o f th e su p p ly -lin e
fu se s blow s off, w ill th e m o to r sto p or co n tin u e ru n n in g ? E x p la in .
lA n s. (c) L a rg e s ta r tin g c u r r e n ts in c a s e o f d.c. m o to rs g iv e r is e to c o m m u ta tio n d iffic u ltie s etc.
(d) M o to r w ill co n tin u e ru n n in g a s a sin g le -p h a s e in d u ctio n m o to r a t a s lig h tly re d u ce d speed .]
6 .6 7 . A 3 -p h a se in d u ctio n m otor h a s ro to r im p ed a n ce o f 0 .0 3 + ./ 0 .1 8 f i p e r p h a s e re fe rre d to sta to r.
F u ll-lo a d slip a t ra te d v o lta g e is 3% . E s tim a te th e p e rc e n ta g e re d u ctio n in s t a t o r v o lta g e to d evelop full-load
to rq u e a t h a lf o f th e fu ll-lo ad speed . A lso, d e te rm in e th e pow er fa cto r. N e g le c t s t a t o r im p e d a n c e an d m ech a n ica l
lo sse s. (A n s. 22.85% , 0.308)

6 . 6 8 . A 3 -p h a se , 4 -pole, 5 0 Hz in d u ctio n m o to r, d u rin g th e s h o r t-c ir c u it t e s t , to o k 1 0 0 A a n d 3 0 kW . In


c a s e s ta to r r e s is ta n c e is eq u al to e q u iv a le n t s ta n d s till ro to r r e s is ta n c e , co m p u te t h e s t a r t i n g to rq u e .
(A n s. 9 5 .4 9 3 Nm]

6 .6 9 . A 3 -p h a se sta r-c o n n e cte d , 4 4 0 V , 5 0 H z, 4-p o le in d u ctio n m o to r h a s th e fo llo w in g c o n s ta n ts in ohm s


p e r p h a se re fe rre d to s ta to r sid e :
r , = 0.2 9 4 , x , = 0 .503, r 2 = 0.1 4 4 , x 2 = 0 .2 0 9 , X m = 13.25

T h e s ta to r core lo sses a re n eg lig ib le.


T o ta l frictio n and o th e r lo sses (a ssu m ed co n s ta n t) = 1 4 0 0 W
F in d th e pow er o u tp u t in k W an d th e ra te d o u tp u t in N m i f th e m o to r is b e in g o p e ra te d a t ra te d voltage
and freq u en cy w ith 2 p e rce n t slip . ( I .A .S ., 1 9 7 9 ] |Ans. 2 1 .0 9 3 kW , 137.02 Nm]

6 .7 0 . A 2 0 kW , 4 0 0 V, 3 -p h a se in d u ctio n m o to r h a s fu ll-lo a d pow er fa c to r o f 0 .8 6 a n d fu ll-lo a d efficiency


o f 0 .8 8 . W ith s ta to r w ind ing in d e lta , s h o rt-c irc u it lin e c u r r e n t a t 2 0 0 V is 7 0 A. I f th is m o to r is fitte d w ith a
s ta r -d e lta s t a r te r , find
(a ) th e ra tio o f s ta r tin g to fu ll-lo ad lin e c u rre n t an d
( 6 ) th e s ta r tin g to rq u e in te rm s o f fu ll-lo ad to rq u e , for a fu ll-lo a d s lip o f 5 % .
(H in t. ( b ) U s e E q . (6.63)1. [A n s. ( a ) 1 .223 ( 6 ) 0.2244]

6 .7 1 . (a ) W h y is i t n e c e s s a r y to em ploy sp e c ia l s t a r tin g a r r a n g e m e n ts fo r in d u c tio n m o to rs ?


( b ) C a lc u la te th e re la tiv e v alu es o f s ta r tin g c u r r e n ts an d s t a r tin g to r q u e s o f a 3 -p h a s e squ irrel-cage
in d u ctio n m o to r, w h en i t is s ta rte d by
(i) d ire ct-o n -lin e s t a r t e r , •
(ii) s ta r -d e lta s t a r t e r an d
(tii) a u to -tr a n s fo r m e r s t a r t e r w ith 6 0 % ta p p in g . . ,
(A n s. (o) 1 : - : 0.361

I- A 7.2 ’ ^ 3 'P ^ a s e s q u ir r e l ca g e in d u ctio n m o to r h a s a s h o r t-c ir c u it c u r r e n t o f 5 tim e s th e fu ll-lo a d current.


It s fu ll-lo ad slip is 5% . C a lc u la te th e s t a r tin g to rq u e as a p e r c e n ta g e o f fu ll-lo a d to r q u e i f th e m otor is started

(а ) d ire c t-o n -lin e s t a r t e r ,


( б ) s t a r -d e lta s t a r t e r ,

find f r t " v lim it: r 6 th e " ? 0 , ° r « ™ n t to tw ic e t h e m o to r fu ll-lo a d cu rren t. Also

supp y’ m t o r m s o f m o t o r f u u -,o a d “ wha* - ihc

th e e u !o at ^ " ; se0 S" P Ply H" e S la r tin B fu ll-lo a d Fiad

(Hint. (6) and (c). Read auto-transformer starter between lines.]


(A n s. (a) 1 .25 ( 6 ) 0 .4 1 7 (c) 0 .2 0 , 0 .8 Ip . 40 % (d ) 0.5, 63.2%l

an d d r a t « f f r r tet t V° lta g ? ’ 8 p ° ly p h a s e in d u ctio ? m o to r d ev e lo p s a s t a r t i n g to r q u e o f tw ic e th e full-load torque


and d raw s fro m th e su p p ly a s t a r t in g c u r r e n t o f 6 tim e s th e fu ll-lo a d c u r r e n t.

D e t e ^ ™ t h T ™ . “ S.U w ed " $ i" B ‘ M m Pc n s a t o r ta u to -tr n n s fo r m e r ) w h ic h s t e p s dow n th e v o ltage to 7 0 * .


e te r m in e th e m o to r s t a r t in g c u r r e n t, s t a r t in g c u r r e n t d ra w n fro m th e s u p p ly a n d th e s t a r t in g torque.
—------------------------------------------------------------------------------------------Polyphase Induction Motors 829

a u ^° tra n sfo rm e r s ta r te r lim its the s ta rtin g cu rre n t from th e supply to tw ice th e fu ll-load c u rre n t,
d eterm in e th e s ta rtin g torque and au to -tran sfo rm er tapping.

H in t. From Eq. (6.51). s p = (Ans. (a) 4.2 Ip , 2.94 I p , 0.98 T , p (6 ) 0.667 T , p . 57 . 74 % tappingl

tim e * thn n . n T ' * ! 0/ Cage ‘nd uct‘on m °to r when sta rte d by sta r-d e lta s ta rte r, develops a s ta r tin g to rq u e o f 0 .4
th e su nnlv lin orque and l a ^es from th e supply a s ta rtin g cu rren t o f tw ice th e full-load c u rre n t. C a lc u la te
SUpply h ne cu rre n t and s ta rtin g torque if th is induction m otor is sta rte d by
(а) a cro ss-th e -lin e sta rte r,
( б ) a u to -tra n sfo rm e r s ta r te r w ith 80%. tapping.

H in t. Here sn = — ,. . . _.
. n 30 |Ans. (a ) 6 I p , \ .2 T t p (b ) 3.84 l p , 0 .7 6 8 T e p \

powe6r factor oPfO S ’ w S Y ’ J 0? V Squi.rre l‘ca e e induction m otor h a s a full-load efficien cy o f 0 .8 7 an d a fu ll-lo ad
ra ted voltage Its full lu d e lta * th e m ° t0 r ta k es a s ta r tin S c u rre n t o f 8 0 am p e re s a t
th a t s ta r tin g torq u e eo u al to h a lf th ' f* u i ; m m im um s ta r lin E cu rre n t to be ta k e n from th e su pply in o rd er
6 t0rqUe eq u al t0 h a lf th e full-load torque is developed w hen sta rte d by
(a ) a n a u to -tra n sfo rm e r sta rte r,
( b ) s ta to r re sisto r sta rte r.

C a lc u la te th e p e rce n ta g e tap p in g on a u to -tra n sfo rm er also. |Ana. (a) 47.432 A, 77% U pping <(,) 51 6 6 4 Al

r o t o n r la c U m e 'o f 2 ^ Sft'p e r T h e r a t io IT o to r,h a s • ,">*” e fO .3 n p e r p h a se an d a s ta n d s till

T
[H in t. W ith d ire ct sw itch in g , esl ^
3TeP ~ 0.12 + 1
2 0.12
W ith s ta r-d e lta s ta r tin g torq u e is reduced to o n e-th ird o f th a t d urin g d ire ct sw itch in g ]. [A n s. 0.2 3 6 6 ]

6 .7 7 . A 5 0 kVA 4 4 0 V , 3-p h a se, 5 0 Hz in d u ction m otor is provided w ith a 3 -p h a s e step -d ow n a u to tr-xn*


form er s t a r t e r w hich ste p s down th e v o ltage to 50 % o f th e input. G iven th a t th e s ta r tin g c u r re n t n f t l i
on ra te d v o ltag e ,s 6 tim e s th e ra te d full-load c u rre n t, e s tim a te th e c u rre n t d raw n hv r / a! u „ ♦ t h e fm o to r
from th e m a in s a t s ta r tin g . W h a t is th e s ta r tin g kV A draw n by th e a u to -tra n sfo rm e r ? u T s w Y )

IA n s . 9 8 .4 1 5 A, 75 kVA]
6 .7 8 . T h e s t a r tin g c u rre n t o f a d elta-co n n ected 3 -p h a se in d u ction m o to r a t ra te d v o lt * ™ ic * *• ‘
fu ll-load c u r r e n t and th e slip a t fu ll load is 5% . T h e no-load c u rre n t is n eg lig ib le . S
(а ) I f a n a u to -tra n s fo r m e r s t a r t e r is u sed to lim it th e s ta r tin g c u rre n t from m a in s to 9 r n i
c u rre n t, e s tim a te th e s ta r tin g to rq u e th e n o b ta in ed a s a p e rce n ta g e o f th e fu ll-lo ad to rq u e ° °3
<M N e g le c tin g s t a to r im p ed an ce, d e te rm in e th e slip a t w h ich m a x im u m to rq u e o ccu rs in th e m otor.

( I .A .S ., 1 9 9 1 ) [A n s. (a) 5 0 per ce n t (6 ) 0 .2 5 3 ]
6.79. A 3-phase, 4 0 0 V , 5 0 A, 4-pole, 1 4 4 0 r.p.m. induction motor ta k e s a hlnrkod rn in * ro •
it s fu ll-lo ad c u r r e n t a t 0 .4 p.f. la g a t ra te d v o lta g e an d d evelops a to rq u e o f 1 8 tim e s it s fu ll lo a d toroY
in d u ctio n m o to r is s ta r te d by a n a u to -tra n s fo r m e r w ith 6 0 % ta p p in g th e n a t th e tim e o f t i i
m o to r p o w er fa c to r ( 6 ) m o to r c u r r e n t (c) lin e c u rre n t « ) p o w e r 't a p * to m o to r “' f
o f fu ll-lo a d to rq u e . IA n, (a )P „ ,4 ,a g ^ m A

6 .8 0 . A SCIM has a starting current of six times the full-load current at a slip of 0.04 Calculate the lin e
current and starting torque in p.u. of full-load values for the following methods of starting :
ja ) Direct switching

(б ) Auto-transformer starting with motor current limited to 2.0 p.u.


(c) Star-delta starting.
.a , a. U.E.S.,20021
I ■la ) 6 Pu - 144 Pu (fc) 0 .6 7 p.u., 0 .1 6 pu (c) 2 pu, 0 .4 8 pul

caCe'nduclio)^otors ?°l ° dViSab'C^ S‘art W°“nd'r0l0r induction molors b* lhc employed for stnrtinC

" .o to ? m h e " ^ r « n B T CXtemal rCSiStnnC° lhC ° f " « « » * * — induction


3 830 Electrical M achinery____________________________________________________________ [P ro b ^

(c) How is th e induction m otor operation affected i f th e e x te rn a l re s is ta n c e in th e ro to r circ u it is n ot fully


cu t o ff ? ,
(d) T h e re sista n ce m easu red a t th e sta to r te rm in a ls o f a p olyphase in d u ctio n ^ ° to r 9 , or a g’ ven
lin e cu rre n t show th a t th e sta to r ohm ic losses would be sa m e w h eth er it is s a r or
6 .8 2 . C a lcu la te th e value o f th e re sista n ce elem en ts o f a 5 -step (o r 6 -stu d ) s t a r t e r for a 3 p h a se , 400l V
w ound-rotor ind u ction m otor. T h e full-load slip is 2.5 % and th e m ax im u m s t a r in g c u rr
load value. R otor re sista n ce per phase is 0 .0 2 12.
D erive an y form ula used sta tin g th e variou s assu m p tio n s m ade.
|Ana. 0 .4 1 8 12, 0 .2 0 G, 0 .0 9 6 12. 0 .0 4 6 G, 0.022 G|

6 .8 3 . D esign a 4-step s ta r te r for a 3 -ph ase wound rotor in d u ction m otor. T h e fu ll-lo a d slip is 2 .5 % and th e
m axim um sta rtin g cu rre n t is lim ited to 1.6 tim es its full load value. R oto r r e s is ta n c e p e r p h a se is 0 .0 2 12.
D erive th e form ula used for calcu latin g th e re sista n ce sectio n s an d s t a te th e v a rio u s a ssu m p tio n s m ade.
[A ns. 0 .2 7 7 12, 0 .1 2 4 12, 0 .0 5 5 12, 0 .025 111

6 .8 4 . (a) E x p la in an y two m ethods for perform ing th e p o la rity t e s t on a 3 -p h a s e in d u ctio n m otor.


(b ) A p o larity te s t is perform ed on a 3 -p h a se, 4 0 0 V w ound-rotor in d u ctio n m o to r. P h a s e A is g iven 100 V
and th e v o ltm eter connected across th e two te rm in a ls o f p h a se s A an d B is s e e n to reco rd a v o lta g e o f 141 volts,
w ith th e ro to r open circuited. W h at a re th e p o larity m a rk in g s o f p h a se B , r e la tiv e to p h a se A ?
[A n s. (6 ) T h e two te rm in a ls o f p h ases A and B , w hich a re co n n ected to g e th e r, h a v e th e s a m e p o larity
m arkin gs.)
6 .8 5 . ( a ) W h a t is an ind u ction g en era to r ? D escrib e, w ith th e h elp o f c irc le d ia g ra m , th e o p e ra tio n o f an
ex tern a lly -e x cited 3-p h ase induction g enerator.
(6 ) D iscu ss th e follow ing for a 3-p h ase in d u ction g e n e ra to r :
(i) I t o p erates alw ays a t a lead in g pow er factor.
(ii) C ontrol o f load on ind u ction g en era to r and its lim it.
(iii) A p p lication o f S C IM in th e g en e ra tin g mode.
(iv) It re q u ires reactiv e pow er for its op eration from a n e x is tin g su p p ly n etw o rk .
(o) C om p arison o f th e use o f induction g e n e ra to r w ith a sy n ch ro n o u s g e n e r a to r.
6 . 86 . (a) E x p la in th e p rin cip le o f op eration o f a self-e x cited 3 -p h a se in d u ctio n g e n e r a to r .
Give th e condition u nd er w hich th is g e n e ra to r m ay fa il to b u ild up.
(6 ) G ive th e ap p licatio n s o f both e x te rn a lly -e x c ite d an d se lf-e x c ite d 3 -p h a s e in d u c tio n g e n e r a to rs .
6 .8 7 . ( a ) C om p are th e re la tiv e ad v an tag es an d d isa d v a n ta g e s o f c a g e -ro to r a n d w o u n d -ro to r in d u ction
m otors o f th e sa m e pow er ratin g .

motor*? H°Wd° y°UCOmpare the operation of a P°lypbase induction motor with that of a polyphase synchronous

Scanned B yX a m b ca n n e r
'■■'I'.;.

SEVEN
Armature Windings
The windings used in rotating electnVal u- , .
windings and (6) distributed windings machines can be classified as (a) concentrated
In concentrated type of winding all
multi-turn coil. Examples of concentrator! g turns 816 wound together in series to form one
machines as well as for d.c. machines I n m n l T S f 6 ? 8111 windings for salient-pole synchronous
Primary and secondary windings of a tran<sf ** 1 C° ’ ^ the turns have the same magnetic axis.
In distributed of w S L ^ “ ^ r a l s o f o n n concentrated windings.
fractional-pitch coils. These coils" am tv, ' v. *ums are arranged in several full-pitch or
periphery to form phase or commut^or ^ n d ^ l L ” “ V " ? ^ “ the air^ P
armatures of both synchronous and d c r j . , r 0 r of induction machines, the
As stated before^, £ 3 2 the haved\Stributed windings.
ing flux. The armature winding is one in whichwlTfc-W “ E !s *hat whlch Produces the work-
object of this chapter is to S f e a s by the workinE flux- ^
d.c. and a.c. machines. introduction to the study of armature windings for both

A m ature wmdings. in general, are classified under two main heads, namely
(a) closed windings and (6) open windings. y’
1he closed windings are used only for commutator machines such as d c j
commutator machines. The open windings are used o X for a c ^machines \
machines' etc- in ci°sed windinEs- * • » “ • th e a tr e
Dnint f u any point on the winding and traverses it, one again reaches the starting
L rm " ^ M i°ne s t a r t e d - ° P en windings terminate at suitable number of slip-rings o r

circur-Hns'3 n h ^ C°™ eCted used for a"c' machines, forms a closed


rirwJ ' . 3'Phase delta can>however, be open-circuited and reconnected in star if desired
sed windings used for commutator machines can, in no case, be open-circuited.
Some of the terms common to both the closed and open armature windings are descrihpd
below. A reference to Art. 3.3 is helpful. g described
is a o n d u cto r- A length of wire which takes active part in the energy-conversion process
called a conductor. In Fig. 7.1(a), wire length AB is one conductor, wire length D E (or FD )
18 another conductor. ‘
(ii) T u rn . One turn consists of two conductors. In Fig. 7.1 (a), two conductors AB and DE
emf •>ne tUrn‘ two conductors of a turn are usually a pole-pitch apart, so that generated
* these conductors are additive in nature.
jv ^ Coil. One coil may consist of any number of turns; e.g. in Fig. 7.1 (a) one coil has one turn, in
V 6) one coil has two turns. The schematic diagram of a coil having more than one turn is as
W h* Fi°" 7‘1 (c)* The number of turns in any particular coil decides its designation, e.g. a coil
g one turn is called one-turn coil. Similarly, the coil may be designated as 2-tum coil, 3-tum coil
. •■
Jgjg" .... ■ * • --
Scanned by CamScanner
Art. ]
832 Electrical Machinery

or W-tum coil depending on whether it has two turns, three turns or W-turns respectively. A
multi-turn (or IV-tum) coil is one which has more than one turn.
(iv) Coil-side. One coil with any number of turns has two coil-sides, see Fig. 7.1 where
AB and DE are the two coil-sides.
The number of conductors in any coil-side is equal to the number of turns in that coil. For example,
each coil-side of one-tum-coil in Fig. 7.1 (a) has one conductor, each coil-side of N-tum coil in Fig. 7.1
(c) has N conductors. Note thatBCD in Fig. 7.1 is called the end-connection or overhang.
C Overhang C

B B

Coil - Coil- .
' sides’ sides

(a) (6 ) (c)
Fig. 7.1. Schem atic diagram of (a) one-turn coil, ( 6) two-turn coil and (c) m u lti-tu rn coil.

(u) Single-layer and double-layer windings. If the winding is so designed that one coil-side
occupies the total slot area, then it is called a single layer winding, see Fig. 7.2 (a). In case the slot
contains even number (may be 2,4 , 6 etc.) of coil-sides in two layers, the winding is referred to as a
two layer winding, see Fig. 7.2 (6) and (c). Single-layer winding is used only in small a.c. machines,
whereas double-layer winding is more common above about 5 kW machines.
Coil-sides
Top layer
i n [U H

w iir ir H i i ■ h ssr
(*)

Top layer

B o tto m layer

(c) .
Fig. 7.2. (a) One coil-side per slot ( b ) two-coil-sides per slot and (c) 4 coil-sides per slot.

The advantages of double-layer winding over single layer winding are (a) easier to house
the winding in slots during repairs, (6 ) lower-leakage reactance and, therefore, better perfor­
mance, (c) better e.m.f. waveform in case of generators and (d) more economical.
(vi) Pole-pitch. The term pitch indicates a particular method of measurement in terms of
coil-sides, teeth etc. A pole pitch is defined as the peripheral distance between identical points
on two adjacent poles, see Figs. 3.10 (a) and 3.11 (c). Pole-pitch is always equal to 180° electri-

(vii) Coi 1-span or coil-pitch. The distance between the two coil-sides of a coil is called
coil-span or coil-pitch. It is usually measured in terms of teeth, slots or electrical degrees.
(viii) Chorded-coil. If the coil-span (or coil-pitch) is equal to the pole-pitch as illustrated
in Fig. 7.3 (a), then the coil is termed a full-pitch coil. In case the coil-pitch is less than pole-

<• »•
o u aiiiieu uy i^ai i u u a i ii ici
*
[A rt. 7.1 lA
Armature Windings 833
pitch as shown in Fig. 7.3 (6 ), then it ic roii j l ,
p itch QTfractional-pitch coil. If there a reS slot's ( " t l t a T d IN I Is I
P poles, then pole pitch = | slots per pole. L tJ ■
Lr^J L?J
pitch Pole^
If coil-pitch =-p, it results in full-pitch winding. In case "pitch
1
coil-pitch < -p .it results in chorded, short-pitched or fraction-

_C oi(^
p ,hhCh Wi" ding' The C<>il' PitCh ^ rarely « " “ * * “ »■» Pole- s p on Coil -
"span '
7.1. Closed W indings
As has been stated before, closed windings are used for

ft! “ l f° r a'C' commulator machines.


It should be kept in mind at the outset that closed armature
windings are always double layer windings. Each coil in (a) (6 )
double layer winding has its one coil-side in top layer and its Fig. 7.3. (a) F ull pitch coil and
( b ) short-pitched or chorded coil.
other coil-side in the bottom layer. In drawing these wind­
ings, the top coil-side is shown with a solid linp and tho -j l j u
illustrated in Figs. 7.4 and 7 5 Upper part of F ie I d coi -side by a dotted line as
• .U * r 1 i. j *a , upper part oi rig. 7.4 (a) shows one lap coil with one coil-side
£ w e ro a rtZ 7 w t 0th: i C° \ Siie in the b0tt0ra 0f ° ther s'° ‘ ^ a pole pitch“ p a rt
segments 1 and 2 . schematic diagram of one lap-coil connected to commutator

Lap coil

Top co il-s id e s Bottom c o il-s id e s


Top
c o '1 -s id e .

B 5Z 5Z
Lap coil i
I % i
** (
'I
i
t
B ottom |
Top—* co il sid e |
c o i l sid e

. V
' C oil e n d
:.‘5 ’ •.

—^yc=tk- —Iv h -
(4)
I ■ Pie. 7.4. Up-coij mnnections, ,o) single mulU-tum lap coil and (4) three mnlti-turn lap coils.

Scanned by CamScanner
834 Electrical M achinery A rt. 7.1J

e simple closed windings are of two types, namely (i) simplex lap winding and (ii) simplex
wave win mg. From the construction point of view, the difference between lap and wave wind­
ings epends upon the manner of connecting the coil ends to the commutator segments.
n simplex lap winding (or lap winding), the two coil-ends of a coil are connected to the two
a jacent commutator segments as shown in Fig. 7.4 (a). Note that two coil-ends, one from top
coi -side and the other from bottom coil-side are connected to adjacent commutator segments.
examination of Fig. 7.4 (6) shows that bottom coil-side of coil 1 and top coil-side of coil 2 are
connected to segment 2 ; bottom coil-side of coil 2 and top coil-side of coil 3 are connected to
segment 3 and so on. In other words, for simplex lap winding each commutator segment has
two coil-ends connected to it-one coil-end is from the top coil-side of one coil and the other
coil-end is from the bottom coil-side of the adjacent coil.
In simplex wave winding, the two coil-ends of a coil are bent in opposite directions and
connected to commutator segments which are approximately two pole-pitches (i.e. 360° electri­
cal) apart as shown in Fig. 7.5. In wave winding also, each commutator segment has two coil-
ends connected to it-one from the top coil-side and the other from bottom coil side.
Examination of Fig. 7.4 (6) reveals that if lap coils are traversed, the movement is forward
and backward alternately ; whereas Fig. 7.5 (a) reveals that for wave winding, the movement
is forward only. In both the lap and wave windings, all the coils are traversed in series.

y Lpj y I4 J
! Coil-1 Coil -2

-yv»-yb*y,—*

=17-0-1 -V 2
- V ,0; 9
=9 26

(a)
Coil-1 Coil-2

1
l
26
I
l

X 1
f
ii
21
yc * w - ai
(b)
F ig . 7.5. Simplex wave winding showing (a) single-turn coils, and (5) multi-turn coils.

Scanned by CamScanner
[Art. 7.1 Armature Windings 835

• s i^usefal^or ^ knowledge of the following terms, pertaining to closed


windings, is useful for understanding the details of these windings.

, ^ 7^ n t ^ L °SeC* windings, the coil-sides for convenience are num-


^fr8K ttnm coil-sides nre rn K ^ ^ 8 top coi^sides are numbered odd ( 1, 3, 5 etc.) and
° A hnrk of nrmat Um 8 even (2>4. 6 etc.). Front of armature denotes commutator
end and back of arm ature means other side of commutator.

1 3 5

u 2 4 6
Fig . 7.6. Illu stra tin g the method o f num bering coil sides in com m utator m achines.

B ack P itc h . The distance between the top and bottom coil-sides of one coil, measured at
the back of the arm ature (or measured at the other side of the commutator), is called back pitch,
symbol It may be expressed in terms of teeth, slots or more conveniently in terms of coil-
sides.
In Fig. 7.4 ( 6 ), the top coil-sides are numbered 1,3 ,5 ... and bottom coil-sides are numbered,
say 8 ,1 0 ,1 2 ..., as per the num bering scheme adopted for commutator machines. For coil 1, the
top coil-side is numbered 1 and bottom coil-side is numbered 8 . Therefore, back pitch for coil 1
is 8 - 1 = 7 ; similarly for other coils, the back pitch isy 6 = 10 - 3 = 12 - 5 = 7. For wave winding
in Fig. 7.5, the numbers given to top and bottom coil-sides are 1 and 10 respectively, there back
pitch is therefore, yb = 10 - 1 = 9. Similarly, for coil 2 the back pitch isy 6 = 26 - 17 = 9 . Note that
back pitch y b is always odd, since it is equal to the difference between even and odd numbers
given to the two coil-sides of the same coil.
F ro n t p itch . The distance between the two coil-sides connected to the same commutator
segment, is called front pitch, symbol yf.

In Fig. 7.4 (b ), the two coil-sides connected to commutator segment, say 2, are numbered as
3 and 8 , therefore, front pitch is yf = 8 - 3 = 5. For segment 3, yf = 10 - 5 = 5. In Fig. 7.5, the
coil-sides connected to segment, say 11, are 10 and 17—therefore, front pitch yf = 17 - 10 = 7.
Note that a top coil-side numbered odd and a bottom coil-side numbered even, are con­
nected to the same commutator segment— this shows that the front pitchy^is always odd.
Winding pitch. The distance between the two consecutive and similar top, or bottom, coil-
sides, as the winding progresses, is called the winding pitch, symbol yw. It is expressed in terms
of coil-sides.
In Fig. 7.4 (6 ), the consecutive and similar top coil-sides are numbered 1, 3, 5 ... or similar
bottom coil-sides are numbered 8 , 10, 12 ... Therefore, for Fig. 7.4 (6 ), winding pitch yw = 3 - 1
= 5 - 3 = i o - 8 = 1 2 - 1 0 = 2 . An examination of Fig. 7.4 (6 ) reveals that for simplex lap wind-
^ - yb —yf-
In Fig. 7.5, the consecutive and similar top coil-sides, as the winding progresses, are num-
bfred 1, 17 or similar bottom coil sides are numbered 10, 26. Therefore, for Fig. 7.5, winding
= 17 - 1 = 26 - 10 = 16. An examination of Fig. 7.5 reveals that for simplex wave wind-
'ng>y,v = yb +yf.
Note that winding pitch yu, is always even, because it is equal to either the difference or the
Edition of two odd numbers yh and yf.

x j u u i ii i u u kj y u u i i iv-/Vy*ui ii iV/i
A rt. 7.2)
XJft Kleclrical M ach in en

Com m utator pitch. The distance between the two commutator segments, to which the two
ends of one coil are joined, is called the commutator pitch, symbol yc. It is always expressed in terms
of commutator segments.
For simplex lap winding, the two ends of coil 1 are joined to segments 2 and 1, therefore,
ye - 2 - 1 = 1, see Fig 7.4 For simplex wave winding, the two ends of coil 1 are joined to seg­
ments 11 and 1, therefore,yt = 11 —1 = 10. Fig. 7.5. For simplex lap winding, .y, is always equal
to 1. For simplex wave winding, yc is almost equal to 2 pole-pitches.
7.2. Sim plex Lap Winding
Let C be the number of armature coils and P the number of poles. Since each coil has two
coil-sides, total number of coil-sides is 2 C The back pitch yb, almost equal to pole pitch, must
be odd, therefore,
yb = coil-sides per pole ± K
or 2C
>'b = ±K ...(7 1)

where K is a number (integer or fraction* added to 2C /P to make yb an odd integer. Here


2C /P is equal to the pole-pitch in terms of coil-sides per pole.
An armature winding may be either progressive or retrogressive. In progressive lap wind­
ings, if coil 1 is placed first, then coil 2 is placed in slots to the right of slots occupied by coil 1,
coil 3 in the slots to the right of coil 2 and so on as show n in Fig. 7.7 (a). Thus the progressive
winding progresses to the right as seen from the commutator side. Therefore, for the progres­
sive simplex lap winding, the winding pitch is
y* = Vfc - >V= <8 - 1 >- (8 - 3) = + 2
and y, = ♦ 1
In retrogressive lap winding, coil 2 is placed in slots to the left of coil 1, coil 3 in slots to the
left of coil 2 and so on, see Fig 7 7 (b> Thus the winding retrogresses, i.e. progresses to the left
as seen from the commutator end Therefore, for simplex retrogressive lap winding the wind­
ing pitch is
V* =>'* - y, = 18 - 1) - |8 - ( - 1)) = 7 - 9 = - 2
and >V= - 1
Fig. 7.7 (b) reveals that coil-ends are cross­
ing in retrogressive lap winding, there are, Coil 1 Coil 3 Coil 2

therefore, more chances of fault and in view of


this, it is rarely used.
Hence for a simplex lap winding,
winding pitch, V(1 = ± 2
and commutator pitch. vf = ± 1
The positive sign is used for p r o g r e s s i v e wind­
ing and negative sign is used for retrogressive
winding.
In the term “simplex lap winding”, simplex
refers to the fact that coil 1 is placed in the
slots adjacent to coil 2, coil 2 is placed in slots
adjacent to coil 3 and so on ; lap refers to the 7o right ---
fact that coil 2 laps over coil 1, coil 3 laps over
• - (Q)
coil 2 and so on for one parallel path. The lap F ig . 7.7. S im p le x lap w inding w ith sing le tu rn coils
winding is also known as parallel winding 'a ) p ro g ressiv e type and (6 ) re tro g ressiv e type

Scanned by CamScanner
|A rt. 7 .2
' *~~~ -— —----- — _ _ _ _ ________ Armature Windings S37

lap w in d in g * ? ^ F o r Q com m utat° r m achine with € poles an d 40 coils, determ ine fo r a sim plex

(a) the num ber o f commutator segments,


<bi b ack pitch a n d front pitch an d
ic) com m utator pitch. .
Solu tion , (a) .Since one commntitnr i• . .
mutator segments = number of coils,C = jo “ requlred for cach coiI- thc number of com-

(b) Back pitch, yh - ^ ± K = 1 * 4 0 ± K _ 40 ± 1 '

The back pitch » must be odd. therefore.


*10 1 „
n * T ~3 3
But y h - yf a ± 2
” >'f * 13 - 2 = 11 for progressive winding.
*n<I * 13 ♦ 2 * 15 for retrogressive winding.
(r) Commutator pitch, y, = 1 1
hxnm ple r. 2 . h o r a com m utator m achine with 12 coils and 4 poles, design a progressive
simplex lap w inding with two coll u d es per slot Draw the winding diagram also.
Solution. Number of commutator segments » number of coils. C = 12. Twelve coils have 24
coil-sides For two coil-sides per slot, total number of slots
24
« ~ » 12
o(* ( J»4
Back pitch. y b ■ rn t A'« - ♦ 1 • 7 or 5.
/ •»
For progressive lap winding,
v„ * .v* - V/ » ♦ 2
Vr e» 5 for » * 7
or 3/ K 3 for yt, » 5.

It is desirable that ' should be as nearly equal to pole pitch as is possible. In v»rw of
7+5
this,y6 = 7 and 3y = 5 should be chosen for this commutator machine, because - y - = pole pitch

in terms of coil-sides I = — coil-sides per pole 1

Por progressive simplex lap winding, commutator pitch,


Vf =
3c = + 1.
+ L
For drawing the winding
Irawing the winding diagram
diagram in
in radial form 12 commutator segments 12 ^ots and 24
"x! tides an- fin;t drawn as illustrated in Fig. 7.S. Now segment 1 is connected to top coil-s.de
1“ d thou to bottom coil-side S (= 1 + * = 1 + 7> - that the 1 and sef ^ e"'
that the coils resemble a lap coil. S in ce ,, . 1, the co. -s.de .connected te s e g m e n t
* “ ♦ * - 1 - 1!. Bottom coil-side is now connected to top cod-s.de 3 ( 8 8
co.l-6ide 1 0 ,3 - 7) then to top coil-side 5 (10 - 5], then to bottom c o , ^ e l 2 (a + 7) and
53°'>' The winding table written below is helpful in drawmg this wmd.ng diagram.
/.-V .

Scanned by CamScanner
A rt. 7.2]
838 Electrical Machinery

+^6 -y f +yb -y f +Vb


(i — >8 ) ----> (3 ------ >10) ( 5 ___ ^ \2)----->(7 —> 14) —> (9 —> 16) —> (11 —> 18) —> (13 —>
20) ->(15 2 2 )-> (1 7 -> 2 4 )-> (1 9 26 or 2 6 - 2 4 , i.e. 2 )-> ( 2 6 -y^ or 21 -> 2 1 + y b,i.e. 4 )-*
(28 - y f. i.e. 23 -> 6 ) -> 1.
In the winding table (1 -> 8 ) means that the top coil-side 1 is connected to bottom coil-side
8 to form one coil. Same is true for all other coils. Note that winding table does not contain any
number greater than 24 (= total number of coil-sides). Each coil-side is used only once in the
winding table and the winding closes on itself (starting from 1 and ending on 1).
In this machine with 12 slots and 4 poles, 3 slots are assumed to be under the effect of each
pole. For example, 3 slots containing coil-sides 1 to 6 are taken under say N pole, next 3 slots
with coil-sides 7 to 12 under the adjacent S pole and so on. Directions of e.m.fs. and currents in
the coil-sides under N pole are indicated by, say, cross and those under S pole by dot. Now the
directions of currents can be marked by noting that current flows from segments to cross­
marked coil-sides and to the segments from dot-marked coil-sides.
A careful examination of winding diagram of Fig. 7.8 reveals that currents leave at the
commutator segments 1, 7 and the currents enter at the commutator segments 4 , 10 . Thus four
brushes A, B, C, D can be placed in contact with segments 1, 4, 7, 10 respectively as shown.
Brushes A and C have the same polarity, these can, therefore, be joined together, similarly
brushes C and D can be joined. Armature power can be handled at the terminals marked
E, F. If E and F are marked positive and negative respectively, the machine works as a gene­
rator as shown in Fig. 7.8 and the direction of rotation is clockwise. In case E and F are given
negative and positive polarities respectively, the machine works as a motor with the direction
of rotation anti-clockwise.
ro ; i
I I N

Fig. 7.8. Simplex lap winding for 4 poles. 12 slota and two coil-sides
per slot, pertaining to Example 7.2.
ocanneu uy uamocaMMfcM
[Art, 7.2
Armature Windings 839
From brush A, one parallel path is through *1 u ~ "
path is through coils I, k j to brush D From brn«i?r? ° t0 brUsh 3 and the second Parallel
brush B and the second parallel path through r ’I ^ parallel Path is through coils/, e, d to
a n y instant, are illustrated in Fig. 7.9 There are tb g ’r * 10 bmsh D- These ParalleI paths at
in general, for a simplex lap winding n u m W „ °,Ur paralIel Paths for a 4-pole machine—
D i.e.
P, pvnrpcoinn -§
• a —rd. expression ZnP reveals
z------- ' tkof u. & P 8 a is equal to the »number- ■ of poles
’ a reveals that the generated e.m f

‘I t ; " maChineS te<,Ual t0 ♦ * > is d e p e n d e n t of the

All examination of Fig. 7.9 reveals that current handled by


each brush is twice the current per parallel path. Iff “ arma I
S ! 6X ^ Windi- ~ P - “ nduc
I
:Ji
path z =~~ —
a P
and current per brush (or per brush-set or per brush-arm) ‘
= 2/ , w
- P I;
It is also seen from Fig. 7.8 th at brushes are physically
$ '■
Jk
placed just below the pole centres. Therefore, electrical angle be­
HSl'
tween adjacent brushes is 180°. Note that physical location of • '
brushes under the pole centres is because of the diamond shape ?'9' Pertainine to Example 7.2. •gpi
m1
of the coils Fig. 7.20. Brushes are actually connected electrically to the coil-sides lying close to
the mterpolar region This shows that brushes are displaced 90° electrical away from the main- •
pole axis. This fact of 90° displacement between main-pole axis and brushes (or brush-axis) is
used in the schematic representation of dc machines in Chapter 4 .
Exam ple 7.3. A progressive sim plex lap winding is to be designed for a 4-pole 14-slot 2
coil-sides p er slot d.c. arm atu re. O btain the follow in g :
m
ia) winding table, m
Si*
(b) winding d iag ram in r a d ia l form an d
imposition o f the bru shes on the commutator.

Solution, ia) Number of coils, C = j


^ = 14.

2C yt 2x14 , a „
Back pitch,
;± 0 = 7
por progressive lap winding,
ront Pitch, y/ = y 6 _ 2 = 7 - 2 = 5.
The winding table is as written below :
(1 8 ) ( 3 10) (5 12 ) -> (7 -> 14) -4 (9 -> 16)
^ (1 1 -> 18) -4 (13 -> 20) -> (15 -» 22) -> (17 -> 24) -i
26) (21 28) -> (23 -> 2) -> (25 ->4) -> (27 h 6 ) -> 1.

winding dia^ am in radiai f° rm sh° wn in Fig- 7 1 0 , is drawn as expiained in Ex*


\
k)
7 For
Por a 4-pole, 14-Slot d.c. machine, the slots per pole are 3 f Had there been 4 coif sides v
■; ,lot' * e n its two coil-sides (top and bottom) would be taken under the effect of one pole and

■/ ;
>canned by CamScanner
the remaining two coil-sides of the same slot (top and bottom) under the effect of neighbouring
pole. But in the present case, there are only two coil-sides per slot. Here if we take 7 slots under
the effect of two north poles and the remaining 7 slots under the effect of two south poles, the
position of brushes can be located. In Fig. 7.10, 4, 3, 3, 4 slots are taken under the effect of
N 1, S 1, N2, S 2 poles respectively. Crosses are shown under north poles and dots under south
poles.
Since currents are leaving segments 1 and 2, brush A is shown covering both the segments partly.
Same is true for segments 11 and 12 where the currents are entering. Brushes B and C touch seg­
ments 5 and 8 respectively. The machine works as a motor because E and F are given negative and
positive polarities respectively and the direction of rotation is counter-clockwise.
E xam p le 7.4. A d.c. m achine is designed to have
(a) 2 coil-sides p er slot
(b) 4 coil-sides p er slot
(c) 6 coil-sides p er slot and
(d) 8 coil-sides p er slot.
Indicate the num ber given to coil-sides p laced in 10th slot. Hence show that the number
given to the coil-side in nth slot is
(i) nu for thc last coil-side an d
(ii) (nu - u + 1) for the first coil-side.
Here sym bol u denotes the num ber o f coil-sides p er slot.

Scanned by Cam scanner


[A rt. 7.2
A rm a tu re W ind in gs 841

lnt from
slot from o^ to ^ nin third'slntP
to o, f
tnird slot from 9 *to^rst
12 anH fornnu =T'u-
slot’cr» 4 ’ are
ii. numbered
v. from
i 1j to *4, in second
n 11
where the coil-sides placed in 10th slot for u
&1UL’ IOr u -~94, a
4, a6 andj 8Vare lshown.
Pr0CGSS g‘ 1
19
37 39 55 57 59 73 75 77 79
O OO ooo oooo
O
20
Q Q
38 40
OOP,
56 58 60
oooo
74 7 6 78 80
(o) (1*) (d)
F>g. 7.11. N um bers given to coil-sides placed in 10th slot, Exam ple 7.4 .

1 s '- C0ll'slde® nu|nbered 20 (= 10 x 2 = nu), 40 (= 10 x 4 = nu), 60 (= 10 x 6 = nu)


and 80 (- 10 x 8 - nu) are the last coil-sides in the 10th slot. In general, therefore, the last
coil-side in nth slot is numbered as nu.
The coil-sides numbered 19 (= 10 x 2 - 2 + 1 = nu _«, + 1)f 37 (= 10 x 4 - 4 + 1 = nu - u + 1),
55 (- 10 x 6 - 6 + 1 - nu - u + 1) and 73 (= 10 x 8 - 8 + 1) are the first coil-sides in 10th slot. In
general, therefore, the first coil-side in nth slot is numbered as (nu - u + 1). .
7.2.1. Split coils. During thewinding design, it maybe foundthat somecoils with their
top coil-sides together in one slot, maynot havetheir bottomcoil-sidestogether in another
slot. Such cases arise only when there are more than two coil-sides per slot. These coils with
their top coil-sides in one slot and bottom coil-sides in two different slots, are called split coils.
So far as commutation is concerned, split coils are helpful but the labour charges increase
considerably. Example 7.5 illustrates the condition that must be satisfied if split coils are to
be avoided.
Exam ple 7.5. Work out a suitable arrangement for the simplex lap winding with no split
coils, in the follow ing two cases :
(a) 30 slots, 60 coils, 4 poles.
(b) 20 slots, 60 coils, 4 poles.
Solution, (a) Total coil-sides = 2C = 2 x 60 = 120.
12D
Coil-sides per slot, u= = 4.
30

Back pitch, yb = ^P - ± K = ~4 - ± 1 = 2 9 or 31.


For y b = 29, top coil-sides 1 and 3 in slot 1 should be connected to bottom coil sides num­
bered 1 + 29 and 3 + 29, i.e. 30 and 32 respectively. These coil-sides numbered 30 and 32 are in
the bottom layer of slot 8.
For yt = 31, top coil-sides 1 and 3 should be connected to bottom coil-sides 1 + 31 and
3 + 31 U 32 and 34 respectively as illustrated in Fig. 7.12 (a). Since bottom coil-side 32 is in
sl°t 8 and the other bottom coil-side 34 is in slot 9, yb = 31 results in split coils which have to be
avoided. Consequently y b = 29 is the only possible back pitch here.
(6) Total coil-sides —2C = 2 x 60 = 120

Coil-sides per slot, u = =6

B“ck pitch, yb =j - ±K
= ± 1 = 29 or 31.
4

3
Scanned by CamScanner
A r t . 7.3J
842 E lectrical M achinery

S lot 1

r m
(a) (ft)
F ig . 7.12. I llu s t r a t in g th e s p lit c o il. E x a m p le 7.5.

For yh = 29, top coil-sides 1, 3, 5 should be connected to bottom coil-sides 30 (1 + 29), 32 (3 + 29)
and 34 (5 + 29). Fig. 7.12(6) reveals that this back pitch results in split coils. Therefore,yb = 29 is ruled
out.
For v<>= 31, top coil-sides 1 ,3 ,5 should be connected to bottom coil-sides 32,34 and 36 which
arc in the bottom layer of slot 6 .
From parts (a) and (6 ), it may be concluded that split coils or split winding can be avoided
2 9 -1 is an integer whereas "~t~ is not
for some values of back pitches. In part (a), u
31 - 1 i yb - 1 is n whole number but (29 - l ) / 6 is not an integer. Thus
an integer. In part ( 6 ),
u
it can be concluded that split coils can be avoided only when ((yfc - \)/u) is an integer. Note that
for avoiding split coils, there is no restriction on the front pitch yf.
7 . 3 . Sim plex Wave Winding
In Fig. 7.5 (a), one end of coil 1 is connected to segment 1 and its other end is connected to
segment 11, which is approximately 360° electrical away from segment 1. One end of coil 2 is
connected to segment 11 and its other end, for a 4-pole machine, must be connected to a seg­
ment just ahead of segment 1 for progressive wave winding or to a segment just before segment
1 for retrogressive wave winding.
Fig. 7.5 (a) is reproduced in Fig. 7.13 in a radial form and for progressive wave winding.
This figure reveals that for a 4-pole machine, there are two commutator pitches yc along the
commutator periphery. As stated before, the number of commutator segments is equal to the
number of coils C. Since the last coil-end of coil 2 must be connected to a segment either ahead
(for progressive winding) or before (for retrogressive winding) segment 1, the following relation
for a 4-pole machine can be written as
2yc = C ± l

or |>-c = C ± l

For a 6 -pole machine, there are 3 commutator pitches,


.*. 3yc = C ± l

or | y e= C ± l .

Hence, for a P-pole machine there are P / 2 commutator pitches,

~yc= C ± l.

k _____
Scanned by CamScanner
Armature Windings 843

Consequently the commutator pitch for a simplex wave winding is given bv


V - C±1
* c P /2 ...(7.2)
Here C is the number of coils or the number of commutator segments.
An examination of Fig. 7.13 reveals that
for a 4-pole machine there are two winding
pitches 3V
Since the next coil-side connected to seg­
ment 2 must have its top coil-side numbered
three (2 more for progressive winding or 2
less for retrogressive winding), the relation
between winding pitch yw and coil-sides 2C
for a 4-pole machine, can be written as
2 y tv = 2C ± 2

or |y„ = 2C ±2.

For a 6 -pole machine, there are three


winding pitches,

| y (t, = 2 C ± 2 .

Consequently, for a P-pole machine,

22 y«> = 2C ± 2. F ig . 7.1 3. I llu s t r a t in g c o m m u ta to r p i t c h y a n d w in d in g


p itc h y w f o r p ro g re s s iv e s im p le x w a v e
w in d in g fo r a 4 -p o le m a c h in e .
Winding pitch for a simplex wave wind­
ing is
2C ±2
= 2yc ...(7.3)
P /2
Note that in Eqs (7.2) and (7.3), positive and negative signs should be used for progressive
311 retrogressive windings respectively.
As yc must be an integer, yw must always be even for wave winding. Alsoy,,, = yb +yf. Since
both
>6 andjy are their sum must always be even—this verifies the above statement.

is th^aVe is also known as series winding. The cause of designating it as wave winding
hat during the traverse of a parallel path, one moves through the winding in a wavelike
S e s s io n .
Example 7.6. Design an d draw a simplex wave winding with the following data :
Poles, l i slots, 2 coil-sides p er slot, progressive winding.
°lution. Number of coil-sides = 11 x 2 = 22

Number of coils = V = 11'


Or h • •
Progressive wave winding, the winding pitch is
2C + 2 22 + 2 = 1 2 .
y,v ~~ P /2 4 /2
yu>~yfc +yf= 12 . Since both yh and yf must be odd, choose y b = 7 a n d y ^ 5.

ScannecJ by CamScanner
A rt. 7 .3)
844 E lectrical M achinery

For progressive winding, the commutator pitch is


C+1 11 + 1
yc = P /2 = 6.
4 /2
The winding table for simplex wave winding is as follows :
+yb + yf +yb +>/ +yb +yf
(1 —>8 ) -> (13 -> 20) -> (20 + 5 - 22 = 3 -4 10) -+ (15 22) —»(5 —> 12) —> (17 —> 2) —>(7 —» 14)
-> (19 _» 4) -* (9 -» 16) (21 -> 6 ) -» (11 18) -> 1.
Note that there are 11 coils and 22 coil-sides. The winding diagram in radial form shown in
Fig. 7.14 is drawn as explained in Example 7.2.
In this example, number of slots are not divisible by the number of poles P . In view of this,
it may be assumed that 3 slots are under the influence of each of the three poles and the remain­
ing 2 slots are under the fourth pole. In Fig. 7.14, 2 slots are taken under the north pole N\. An
examination of Fig. 7.14 shows that at the instant shown, one brush A should touch the seg­
ment 3, because two currents are going towards this segment. The only ether point where the
two currents are going away from each other is at the other end of the commutator—this is
indicated by B x. Since the brushes must touch the commutator segment, the brush B can be
placed where the two coil-sides of coil k are connected. That is, brush B may be placed either at
segment 6 or at segment 11. In Fig. 7.14, it is shown at segment 11.
From brush A, one parallel path is through c, i, d.... to brush B and other parallel path is
through h, b,g.... to brush B as shown in Fig. 7.15. Therefore, in simplex wave winding, two

Fig. 7 .1 4 . P ro g ressiv e sim p lex w ave w inding for 4-p o les. 11 slo ts
and 2 co il-sid es per slot. E x a m p le 7 .6 .

Scanned by CamScanner
—------------------------------------------------------- _ _ _ _ ------------------------ Arnialure Windings #45

brushes are required and there are nnl„ f,., ,


in Fig. 7.15. Also the distance between t P 38 iI,ustrated
approximately equal to one pole pitch i n ^ r m T ^ 8 ^ ° f brushes) is
In general, for any number of poles a simnle commutator segments,
two parallel paths and a minimum of 2 sTts ^ * ? ,WayS'
larger currents are to be handled the numhJ r ? T However*,n case
equal to the number of poles. In such case* » ™s.h 'sets may be made
brushes are connected together respectively’ L .P°Sltlve aad a11 negative
rent handled by brush is then 21 / ? Hn m P Windin£s- Tbe cur_
winding is employed, only two b ashes In T ' k “ When
viewpoint of economy. r rusb'sets) are used from the

ture^lamii^tioruf ^ ^ i i s e ^ w ^ t f f e i ^ n ^ ^ t i n g ^ f t h * 811^31^ arm a*


machines. When simplex wave windings ^ u
^
Ln2 ?m ,‘ ' " I
fied. If this relation is not fulfilled then two pnd« ™ besat,s-

may be cut off, insulated and not connected to th ° n8 m° re) 0011


This unused and inactive cofi t h Z h o l l ed -n J T muta? )r S^ mentS'
mechanical balance is called a dead or dummv I ^ ar™a ure slots for
metrical armature winding a dummvcoil shn °r? e[ t0 have sym'
sibie However it mav ho f ’ , ™my CDl1 sh°nld be avoided as far as pos- Fig. 7. 15. Pertaining
armature core with existmg ^ d e Z ^ t l r e

on new punch, ng tools etc. If dummy coils are to be avoided, then ^ should not be an integer
this is revealed in Example 7.7. ’
Examples 7.7 and 7.8 illustrate the use of a dummy coil.

machine with ^

Solution. Number of coils, C = — — = 144.


3

Commutator pitch, * = ° ± ± . i ± L | i . 47| or 4g,

pJ m. T * Wmdingy‘ must be an intl* er' If all the 144 coils are used, wave winding is not

>« a na dthe
) h commutator
7 Z e J\ 'Stn0tpitch
a" r is now
8" - H° WeVer'
given by ^ ° " e COi' iS made dUmmy then actiTO are

y' = ^ 3 ^ = 48 0 r4 7 i
Winding pitch, ^ =^ = 2 x J | 3 ± 2 = 96or94|

Now°WS that ° nly pr0gressive simPlex wave winding with one dummy coil is possible.
y " =y b + y f= 96-
*»thy6 = 49 andy^= 47>hecause both need be odd. At the same time, split coils are avoided

y h - 1 _ 4JU 1 ’
189 Whrxl U 4
e number.
Scanned by CamScanner
A rt. 7.4]
846 E lectrical M ach in ery _ ---------------------------------------------------------------

Here = 24 is an integer and a dummy coil is essential. It shows that if dummy coils are
6/2 Q #
to be avoided in simplex wave winding, then should not be an integer.

Exam p le 7.8. Design the winding for a 4-pole armature with 2540 conductors, 127 com­
mutator sectors (or segments) and 32 slots.
I f an induced voltage o f 500 V is required, calculate its speed for a field flux o f 5 mWb per
pole.
Solution. Conductors per slot x Slots = zs x S = 2540
2540 nrk or7K
.’. Conductors per slot, zs - ^ ~

Sine zs must be a whole number, choose zs = 80.


This gives total conductors as 32 x 80 = 2560.
Total number of coils, C = number of commutator segments = 127.

.-. Turns per coil = 2^ fi27 = 10 08 s


Since the coil is a multi-turn coil, wave winding should be used.

Actual number of coils = = 128 co^s-

It is essential that actual coils are equal to the number of commutator segments 127. In
view of this, one coil must be made dummy, so that there are 127 active coils.

••
• y
yc
=C±l
P /2
= 127±JL = 64 o r 6 3
4 /2
= 2 C ± 2 = 2 x 1 2 7 4 2 = 128orl26
y“ P /2 4 /2
yb ~ 65 andy/-= 63 for progressive wave winding andyb = 63 andy^= 63 for retrogressive
wave winding.
Since the dummy coil is not in circuit, the number of active conductors is
127 x 1 0 x 2 = 2540

Now £„ = * /

5 x 1 0 _ 3 x 2540 x n x 4
or 500 = ---------------- ----------------
500 x 2
or n= ------------^---------------- r.p.s.
5 x l 0 _3x 2540x4
5x60
= -------------- 7 r.p.m. = 1181 r.p.m.
254 x l O " 3

7.4. M ultiplex Windings


In some cases, the number of parallel circuits may not be adequate enough from design
considerations. For example in a 10-pole commutator machine, simplex wave winding gives
only 2 parallel paths whereas simplex lap winding results in 10 parallel paths. In case the
parallel paths required are in between 2 and 10 or more than 10 , then multiplex windings are
employed.

Scanned by CamScanner
[Art. 7.5________________________________
Armature Windings H47
In duplex lap winding, the distance between th
two ends of one coil are connected is 2 , i e v = + J l Segments to wh>ch the Coil 1
of coil 1 is joined with segment 1, then for nrn~ example, if one end
its other coil-end is connected to segment 3 1 gre8S,Ve dM e x lap winding
between them, see Fig. 7.16. The coil starting f ^ Segment number 2 in
coil-end connected to segment 5 and so on ^ Segment 3 has its second
In triplex lap winding, the distance betwepn fk
two ends of one coil are connected is 3 and therefore88^ 80*8 t0 Which the
In duplex wave winding, the last coil H ft G’ ^ ~ J J
the commutator, is connected to two segment! klk* tJravellin&once around
ing point. g s behmd or ahead of the start - m m u n
C ±2
yc =
P /2 ...( 7 .4 ) F 'g 7.16. Illu stra tin g
duplex lap winding.
In triplex wave winding, the last mil or»ri a.
connected to 3 segments behind or ahead o f ar°Und the “ “ “ “tator. is
C ± 3
y° P/2

In genera!, if m denotes "plex" of the multip.ex windings, '


en yc = ± m for lap winding
and v _ C±m
P/ 2 wave Ending.

sign is f ^ t T e ^ e s X n^ndfngP° SltiVe ^ ^ pr0^ ssive winding and negative


The number of parallel paths a in lap winding is a = mP «„,» •
Multiplex wave windings are rarely used in L m a 'Jn e s ^ ‘ =^
7-5. Equalizer Rings

Showing reasons f l a p ‘ c o n n e c t e d ™ chines may be due to any one or all of the

faulty assemhi **** under different poles may not be the same, due to w e a r of th» h ■

^ an ^ °tber po*es and, therefore*^

re' different c m fs aZ
.* ? LUXbe different under th« various poles and. there-
(t'-) ^ s - are generated in the parallel circuits.

lrnpe^ectionfinTkT! re' uctances for each magnetic path may also be due to the impurities or
in e materials constituting the magnetic circuit.
Ordpr a._ •«!
!!nnected comm , J rate the detrimental efTects of circulating currents, consider a 4-pole lao-
ale-m.fs «en * ° j machine shown in Fi&- 7 1 7 - Suppose the full-load current is 200 A. For
5 erated in the parallel paths, the distribution of currents is ns illustrated in Fig

Scanned by CamScanner
i i
! A rt. 7 .5]
848 E lectrical M achinery

IX
100A
/
k /
\ N
\ • '* / 60 A

100A J S/50A VlOOA


"~B’\ 5 0 /a
\
' s' B k\
/
100A' \

1O0A 200 Ajl 100 A

(c)
F ig . 7.1 7. (a ) A r m a t u r e c u r r e n t d is tr ib u tio n fo r e q u a l e .m .fs . (6 ) C ir c u la t in g c u r r e n t d u e to u n e q u a l e .m .fs . a n d
(c) r e s u lt a n t a r m a tu r e c u r r e n t d is t r ib u t io n d u e to u n e q u a l e .m .fs .

\1 (a). All the four brushes are handling 100 A of current and each parallel path is carrying
50 A.
Now suppose that the air gap under the top pole-pair N h S x.becomes more than that under
the bottom pole-pair N2, S 2, due to wear of the bearings. As a result of this wear, air-gap flux
under N 4, S x decreases and that under poles N2, S 2, increases. On account of these different
air-gap fluxes, the emf. generated in the parallel paths under N lt S x becomes less and that
under N2, S 2 becomes more. Let these emfs generated in parallel paths B 3 B 2 and B 3 B 2 be 48
volts and that in parallel paths Bj B 4 and B 3 B 4 be 52 volts as shown in Fig. 7.17 (b). The poten­
tial of negative brushesBj, B 3 remains unchanged, because from B xto B 3 or from B 3 t o B x, there
is an equal voltage rise and equal voltage fall. From brush B 2 to brush B 4,

VD
u2B = ~ 48 + 52 = 4 V.
This shows that brush B 4 is at a higher potential than brush B 2 by 4 volts. If armature
resistance for each parallel path is 0.1 Q, then resistance between brushes B 2 B 4 is also
r 0 2^
0.10 = _2 [ In vieW ° f this’ a P°tential difference of 4 volts across brushes B 2 B 4 causes

4
q 2 —40 A to flow from brush B 4 to B 2 as shown in Fig. 7.17 (6 ). The resultant current distribu­

tion in various parallel paths, obtained by superimposing the current distribution of Figs 7.17
(a) and (6 ), is shown in Fig. 7.17 (c). This figure reveals that brushes B lt B 2, B 3 and B 4 handle
100 A, 60 A, 100 A, and 140 A respectively. Brush B 4 is thus overloaded and may cause spark­
ing. With uniform currents of Fig. 7.17 (a), the armature circuit losses are 4(50)2 x 0 1 = 1000
watts. With circulating currents present as shown in Fig. 7.17 (c), the arm ature circuit losses
are 2 (30) x 0.1 + 2 (70) x 0.1 = 1160 watts. This shows that the circulating currents result in
overheating of the armature also.

In order to avoid overloading and sparking at the brushes, it is essential to take some steps
so that circulating currents are not handled by the brushes. These steps consist in providing
equalizer rings or equalizer connections. An equalizer ring is a low resistance conductor wire,
which connects together the points in the armature winding which should be at the same poten­
tial. The function of equalizer ring is to cause the circulating currents to flow within the arma­
ture winding itself, without letting them pass through the brushes.
Armature Windings fr*9
[Art. 7.5

Refer to Fig. 7.18, where the coils connected to brushes Equaliser


ring
B2 and B* have potential difference of 4 V between them. If
an equalizer connection is joined to these coils, then cir­
culating current of 40 A will flow through the equalizer
a b c and then through the armature coils as shown in Fig.
7.18. Though the circulating currents are still present, the
brushes are saved from handling these currents. The volt­
age between brushes B 3B 2 and B xB 2 becomes 48 + 20 x 0.1
s 50 V and between brushes B l B 4 and B 3 B 4, the voltage is
52 _ 20 x 0.1 = 50 V, see Fig. 7.17 (6 ). Thus the tendency of
the circulating currents is to equalize the potential of all
parallel circuits—in other words, the circulating currents
have the effect of reducing the flux unbalance that was
responsible for the initial potential difference. Thus the Fig 7 18 Pertainin6 t0 e q u a liz e r ring,
functions of equalizer rings are
(t) to save the brushes from handling the circulating currents and
(ii) to reduce the magnetic flux unbalance that causes the potential difference in various
parallel paths.

Refer now to Fig. 7.19. Points 1 and 2 on the armature \ ^nngs^


winding are under the centre of poles N 3 and N2 respective­
ly. These points are two pole-pitches apart and have the
same induced emf, if there is no flux unbalance. If points 1
and 2 are short-circuited by an equalizer-ring as shown,
then this ring will carry no current and the armature wind­
ing remains unaffected. Similarly, points 3 and 4 can be
connected by another equalizer-ring as depicted in Fig. 7.19.
In case there is flux unbalance, then points 1, 2 or 3, 4
will be at different potentials and circulating current will
start flowing through the rings and armature coils, without
flowing through the brushes.
' An examination of Fig. 7.19 reveals that an equalizer
nog connects as many coils as there are pole-pairs. In Fig. F ig ..7.19. Illu stra tin g the m ethod o f
•19, the ring connects 2 coils because there are two pole- connecting equalizer rings.
Paus. It is also obvious from Fig. 7.19 that any two points
nu° Pole' pitches aPart, can be joined by an equalizer ring. There is, however, a limit to the
the 6r equabzer rings from the economy point of view. It is usual to employ 10 to 20 rings,
actual number depending on the size of the machine.
tial ^ *s seen distance between coils (or points) having the same poten-
118^pole-pitches, i.e.
„ 0 i .. , Total number of coils
yeQ= 2 pole-pitches = ----------- :------:----------
* Pole pairs
2C
coils ...(7.6)
P/2 " P
The -
di8tance 8^ mbo1 is called the equipotential pitch in terms of coils, because ycq indicates the
e etween two adjacent equipotential points.

Scanned by CamScanner
A rt. 7.5)
850 Electrical Machinery

In Fig. 7.19. it is shown that there are 4 ^


for a 6 -pole or 3 -pole-pair machine, there would be 6 taps ana c eq
may be concluded that
Total number of tapping points
rP ...(7.7)
= Number of equalizer rings (r) x Pole-pairs 2 2
v
Distance between adjacent tapping points,
Tntal number of coils _
yip ~ Total number of tapping points
c 2 c _y^ ...(7.8)
rP /2 rP r
Here y, may be called pitch of the tapping points in terms of coils.
Note from Eq. (7.8) that if equalizer rings are to be employed, 2 C /P must be a whole num-
ber.
It is usual to provide equalizer rings at the back of the armature, i.e. at the other side of the
commutator.
Equalizer rings are not required for simplex wave windings even i f there is ma^netlJ: ” n*
balance. This is due to the fact that coil-sides in two parallel paths are distributed under all the
poles. In view of this, both the parallel circuits are affected equally and there is, therefore, no
need of equalizer rings in simplex wave windings.
E xam p le 7.9. Work out a suitable arrangement o fte n equalizer rings for a commutator
m achine having 8 poles, 240 coils, simplex lap winding.
Solution. From Eq. (7.6), the equipotential pitch is
2C _ 2 x 240
yeq- p -
= 60 coils.
8
Pitch of the tapping points, from Eq. (7.8), is
= 2C = 24 M = 6coils.
JtD _P
rP -10
~ x 8
~

The arrangement of the equalizer rings and the coils connected to them is given below in
tabular form.

R ings
4 5 6 7 8 9 10
1 . 2 3
->

49 55
1 7 13 19 25 31 37 43

109 115
N um ber o f coils 61 67 73 79 85 91 97 103
connected to th e rings 175
121 127 133 139 145 151 157 163 169

235
181 187 193 199 205 211 217 223 229

Since there are 4 pole-pairs, 4 coils are connected to each equalizer ring. For example rnv
2 has coils connected to it which are numbered 7, 6 7 ,1 2 7 and 187 in the above table.

Scanned by CamScanner
lArt 7.5
ArmalureWindin
Summary. A review of the salient features o f l« n u . . .
7.1 below : 01 laP and wave winding is presented in Table

Table 7.1. Comparison between ■


---------------------------------------- P ex~lap and Simplex-wave Windings.

E m f. Sim plex-wave winding


nMde|* "d“>lt« "“mter ty Z n P
2 . depends upon th e num ber

2. Average coil-span
p coil-s.des where C = Number of 2c
-p coil-side
coils
Back pitch, y b
2C
± K , odd 0 ^. - y f) , odd
Front pitch, y f
>b ± 2 , odd <+ for retrogressive wdg
<yu. -y^, odd
and - for progressive wdr)
5. Commutator pitch, y c
±1
C 11
1______ JV 2 |
Winding pitch, y u ^ J b r ^ r o g r e s s i v e a ^ . for retrogressive w d ,
±2 ----
2C ± 2
L
P /2 r yb + yf
^ L P f g g l g j g i v e wdg and - for retrogressive wdg
Number o f parallel paths, a
Number o f brushes
2 (For large m achines, num ber o f
Dummy coils brushes may be equal to P )
Not needed
10. Equalizer rings May be required

may be needed ; then ^ must be Not required


an integer
Conductor cu rren t, I c = cu rre n t in
each path
12 .
For avoiding sp lit coils (m achines
pavin g 2 coil-sides per slot) •m ust be an integer
m ust be an integer

Low-voltage high-current machines


A . — ” H i g h -v o lta g e J o w - p 0w e r m a c h in e s

* * — * » .

if W e o r moTpnh m g um a y J b e f i t t e d w i t h 3 8 m a n y b r u s h e s a « t h e n u m b e r o f p o l e s I n s u c h »

t0" « ” - S~ U: t dr e.IOP, POOL C O n t a r t W U h t h e C° m m U t a t O T ’ t h a o o m m u t a t o r m a c h i r


«, Lop-connected m t b “ v " 0t ^ *" o n aP-“ >n“ «od machines
bcreas wave w in ? “ achlnes sq u ire equalizer rings for obtaining better commutatio
(#) g|n Ddings do not require any equalizer ring.
^ achin e 8 a nr e eJ ! P ' COni? e c t e d m a c h in e s re q u ire m o re b ru sh es an d e q u alize r rin g s the*
Lapw- j - r e c o s t ;^ a s c o m P a re d to w a v e -c o n n e c te d m a c h in e s .

i (50 t o 500 kW) a n d h i g h p o w e r ( a b o v


kW) ^ a c h i n L a w U S ed ° n , o w ' v o , t a &e m e d i u m p o w e r
Urn Power r« i^ aVe wind>ngs are used on high-voltage low-power (less than 50 kW) a n
machines.

Scanned by CamScanner
A rt. 7.51
852 E lectrical M ach inery --------- " '
. .• ■ a fart that wave windings do not require
The preferred choice of wave windings is duetc> are )ess expcnsive os compared to
equalizer-rings. As a consequence, wave-wound dc m - 400 A, lap winding is the
lap-wound dc machines. However, for armature per path in wave
only choice. It is because for ^niature currents^exceed,ng 4C inwave-
winding would be more than 200 A and such a curr
wound dc machines. „ - . 11a ___
In addition to lap and wave windings, there is another type of 3 ^ ^ “ “
winding or frog-leg winding. It is a combination of simplex lap and ^ P le^ a e J ngS
and requires no equalizer rings. It is beyond the scope of this book to describe frog leg winding.
This type of w in d in g is used mainly on high-power com m utator m ac m e .
Exam p le 7.10. (a) From on outside appearance o f the arm ature o f a com m utator machine,
how would you distinguish whether it is lap-or wave-wound.
(b) A simplex lap-wound armature has 12 coils, each o f resistance 0.1 £2.
(i) What is the resistance m easured between two adjacent com m utator segments ?
(ii) I f one coil gets open-circuited or short-circuited, explain how it can be located.
(c) A com m utator m achine has Z conductors, ‘a ’ p a ra llel p a th s an d P poles. I f each con­
ductor h as a length (including overhang) o f L metre, area o f cross-section o f A metre and a
resistivity o f pCl-m, fin d the resistance o f com m utator m achine. How are these expressions
m odified for lap and wave-connected m achines ?
Solution, (a) For a lap-wound armature, if the ends of the coils connected to commutator
segments are bent, say, to the left then its other coil-ends, on the other side of commutator, are
also bent to the left, see Fig. 7.4. In other words, lap-wound armatures have both their coil ends
bent in the same direction.
For a wave-wound armature, if the ends of coils connected to commutator segments are
bent, say, to the left, then its other coil-ends, on the other side of commutator, are bent to the
right, see Fig. 7.5. In other words, wave-wound armatures have their coil-ends bent in opposite
directions.
(b) (i) Resistance of one coil connected to adjacent commutator segments is 0.1 ft. This coil
is in parallel with the remaining 11 series-connected arm ature coils of total resistance
11 x 0.1 = 1.1 ft. In other words, one coil of resistance 0.1 ft becomes in parallel with 11 series-
connected coils of resistance 1.10 ft. This can be ascertained by referring to Fig. 7.8. Therefore,
the resistance measured between two adjacent commutator segments is

Q'* x = o 0917 ft
0 . 1 + 1.1 u u y i ' “ -
(ii) The resistance measured across two commutator segments to which one open-circuited
coil is connected, would be that given by the remaining 11 series-connected coils. In view of this,
the resistance measured across an open-circuited coil is
1 1 x 0 . 1 = 1.1 0 f t
With one coil short-circuited, the resistance measured across its two coil-ends (or segments)
would be (ideally) zero.
It may be concluded from above that an open-circuited coil is indicated by the highest pos­
sible resistance across its two segments ; whereas a short-circuited coil can be located by the
lowest possible resistance across its terminals.
- /

(c) Resistance of one conductor = ft.


A
%
. Each parallel path has — conductors.
a

Scanned"by CamScanner
[Art. 7.6 :
A rm a tu re W ind in gs 853 ...
■ ■
... Resistance of each parallel path = ^ . £ k q
a A I?'J'
Thus, resistance of the commutator
or machine with number of parallel paths ‘o', is given by ; jj
=± f e .e k V z .s k a ■$1

For lap-connected machine a = P itc


\
s.
j a
. , 7 nr
I
ii ■
’ ^ lts d is ta n c e is therefore, equal to — - ^ Q
p2 A • Iid
For wave-wound machine a - 9 it* w. • s. . v „r $
me, a - 2 , its resistance is, therefore, given by ^ ^ n. I
4 A 1
7.6. Open Windings o r A.C. A rm atu re Windings ill:
The machines which do not require a ♦
opened at one or more points. In c o m m u t a t o T m ^ - ^ haVG their armature windings
taining better commutation. Butin machinpC ,nes’ the turns Per coil are limited for ob- is
i ?A on the choice of number of turns per coil commutator, there is no such restriction Ki
If-
Closed or commutator windings arp a li i
windings may be single layer or double layer As stated before” " ^ ! whereas a c - "m a tu re !I
for small a.c. machines, whereas double layer windi™ Tr y6r Windings are used
u!
!‘
kW. 3yer wmdlngs are used for machines above about 5
I The main difference between single and dnnhio lo.m •j- .
i•■ their overhang. In single layer winding the coils arp a Windlngs 1S m the arrangement of l|j
i
overhang crossing the overhang of adjacent gronns TV ranSed *n groups. Each group has its if
coil groups of different sizes and shapes This means Jh ^ i8 by haVing individual
variety of sets of coils with each set differing bothTn si! hat a Sln^ layer winding requires a i\
adds to the cost of single-layer windings S W 1p.iT and shape from the other sets. All this ill
lr:‘
modern machines except in small sizes In sinelp law v!gS- therefore* rarely used in i; i
' have the same shape and size smgle-layer mush windings,however, all the coils
Ii

same shape and size. Each coil has its one


in
kink, see Fig 7 20 , 7 ^ 1 ° a"d other coil-side in the bottom half of some other slot A special $
1
vlf
I t o S ? A d“ub'e: layer winding as it is in single layer Slots
their fJk double‘layer winding has identical coils,
cost ncatl0n 1S eas*er and this leads to reduction in

Top coil Bottom


kver ud advanta&es °f double-layer winding over single­ side coil side i||
winding are as follows:
■ f'.'M 1

(6) f aS*0r t0 manu^acture and lower cost of the coils, •■'.j i


not an inracti0” al-sl°t winding (slot per pole per phase is
lnteger) can be used,
^ Cborded' wlnding is possible,
petformanle r leakaEe reactance and, therefore, better
(e)b 6 machine and
In g ^8r e'm ^ waveform in case of generators,
V;
cept
in high3 * dou^^e ' ayer Is preferred now-a-days, ex-
0peningR ana » tage induction motors with large slot
Fig, 7.20. Diamond shaped coil.
v;; nd short air-gap lengths. .

>canne( >canner
A rt. 7 .6 J
854 E le ctric a l M ach in ery --------------- --------------------- ------------------------------------ -
--------------------------------------- ” mnnpcted in several different man-
In a.c. armature windings, the separate cot s m ay
ners, but the two most common methods are lap and wave.
In polyphase windings it is essential that
(i) the generated emfs of all the phases are of equal magnitude,
(ii) the waveforms of the phase emfs are identical,
(iii) the frequency of the phase emfs are equal and
to ) the phase emfs have mutual time-phase displacement of |5= ^ electrical radians. Here

m is the number of phases of the a.c. machine. jw o n riin rr


P h ase sp read . A knowledge of the phase spread >sbener,cu,l for ml ^ ^
polyphase winding arrangements. For this purpose, re er g. > ^ouse(j jn 12 slots.
the rotor is shown to produce 2 poles and the stator carries under the centre of
Fig 3 19 (a) is reproduced in Fig. 7.21 (a) for convenience. Conductor 1 is under the centre ot
N pole, it has, therefore, maximum emf £ m induced in it. This emf can be represented by a

phasor of length E x = lying along the vertical. Conductor 4 is cutting zero lines of flux and
has, therefore, zero emf induced in it. This can be represented by phasor E Asince its vertical
projection is zero. The emf generated in conductor 7 is maximum, because it is cutting maxi­
mum lines of flux from S pole. The polarity of emf in conductor 7 will be opposite to that in
E . . 1 «.
conductor 1, it is, therefore, represented by £7 = ~^> opposite to E v Sim ilarly the emfs
generated in conductors 2 , 3 , 5 , 6 and in conductors 8 to 12 can be represented by phasors
£ 2, £ 3, £ 5, E 6 and £g to £ 12 as shown in Fig. 7.21 (a).
Eu

(a) (fr)
Fig. 7.21. P ertaining to 3-phase winding, (a) P h a se spread is 120° and
(fr) tim e phase angle is 120° betw een E A, E fl and E Q.

Here the slot-angular pitch is given by


180° 180°
= 30°.
Slots per pole 6
If back end of conductor 1 is connected to back end of conductor 2, front end of conductor 2
to front end of conductor 3, back end of conductor 3 to back end of conductor 4 , then all the four
e.m.fs. E h E 2, £3 and £ 4 are additive and their resultant appearing at the front ends of conduc­
tors 1 and 4 can be obtained by the phasor sum of E lt £ 2, £3 and E 4. This phasor sum is indi­
cated by E a in Fig. 7.21 (6 ). Similarly if the conductors 5, 6 , 7 and 8 are joined in series so that

na—kJ J - ^ 1411luuui
(Art 7.6
------------------------------- Armature Windings
/iiiiidiure n855
in u in p
their e.m.fs. are additive, then the resultant p m r uT ~ "
8 is obtained by taking the phasor sum of e.m T p v n the fr° nt ends of conduct° rs 5 and
by e.m.f. Eb in Fig. 7.21 ( 6 ). Similarly with S I ' 6’ &nd ^ 8' ^ bis Pbasor sum is indicated
sum of e.m.fs. £ 9, E * n and £ „ is ! ’ 10’ U “ d 12 in series, the phasor
three phases A, B, C. Note that the space an 1 ° manner, 12 conductors are grouped in
the time phase angle, Fig. 7.21 (6 ), between e m f V ' 2p ^ C°Vered by each phase is 120° and
3 and 4 in adjacent slots, belong to phase A—th • E ° iS a,S0 120°‘ The conductors 1, 2,
is called phase-band, phase-belt or p h a se-e rn n ^ Q0mblnation of 4 conductors in adjacent slots
second phase-belt and conductors 9, 10, l l and 19 rh !u-y JC° i; ductors 5 ’ 6 ’ 7 and 8 constitute
belt may be defined as the group of adjacent1 , 1 , 5 rd Phase belt. In general, the p W
The angle subtended by one phase-belt is called nfr bel° ngln&to ona phase under one pole-pair.
in Fig. 7.21 ( a ) is 120°, because 4 adjacent Symbo1 a ' Thus the Phase spread
pitches (= 4 x 30° = 120°). J3Cent slots Pertaining to any one phase belt cover 4 slot-
If conductors 1,2 are connected in series fh r
ly join 3' 4 - 5*6 >7 ‘8 . 9-10 and 11-12 r e s p e c tiv e ^ * Z l A ♦ ° btained’ see FiS- 7-2 2 lb). Similar-
shown in Fig. 7.22 ( 6 ). These 6 voltages form r I glVe voltages B, C, D, E and F as
them. Note that here the phase spread is 60° b e ^ a u ^ th ^ PhaSe &ngle of 60° between
consisting of two adjacent slots of any one phase's 60°, see Tig 7 2 2 ^ ? ^ * * ° ne phase-belt
^ ’A

(a)
Fig. 7.22. P e rta in in g t„ 6 -phase w inding, (a) phase spread i , 60* and (4) U n ip h a s e angle is 6 0 ”.

f ha1s e ’b eltf- The developed view of Fig. 7.22 (a) is drawn in Fig. 7.23 fa) f o r
hsvdtg a Phase spread of e T In FfaS 7C2 3 ( l US7d > ° btain.?hree-Phase single-layer winding
* 12/2 = 6 slots ,■ g' ' 3 (a)l coll'P,tch or “ d-span is taken equal to pole pitch
- 7.23 (a) p ‘
(a ) P n r V i1SfC0 j Po n ’ conductor
r T n 1 c1ondu.c t o r 1i should
snouid be connected to conductor 7 as shown in
fcrto . ' '• t o r 12 s lo t s a n d 9 n n lp s e ln f .o n r n t lo v n if n li u - ono rn ____ />___ - ,
' “•■two a d L . ? Sl0tS and 2 p0,es' ^ “t-nngnlar pitch 7 = 30”. Therefore, for phase spread
luctor 88 hhel"
Coriductor pl ,0tS,must
i!“ u a t belong
UC1U11S tow the
u i e ssame
am e p phase.
n a s e . Thus,
i n u s , sslot z cconductor
lo t 2 o n d u c t o r aand
n d iits
t s rreturn
etu rn
, COr 0 belong to nhase A In nfhpr words mnd,mfnrC 1 ond 9 „.......... return
7 1 + 6) a * a P A' In °ther words* cond«ctors 1 and 2 are connected to conductors
A c t o r s an Hi 6) * S S,h°Wn Fig- 7 2 3 (a)' ThnS' conductors 7 and 8 form the return
^W-belt rabelled A', A'. For the sake of convenience in drawing the winding diagrams,
te*UrP condC° nS*St*n^ conductors 2 is designated A whereas the phase-belt made up of
Wn the st Uf ° rs 7> 8 *s den°ted by - A. For a 3-phase winding, phase B must start 120° away
5>^0r cond °^pbase A ■Since the slot-angular pitch is 30°, the phase B must start from slot
^before nh ^ ^ a° d conductors 11 (= 5 + 6) and 12 (= 6 + 6) form the return conductors.
»P ase-belt made up of conductors 5, 6 is denoted by B and phase-band consisting of

Scanne amScanner
A rt. 7.7]
856 Electrical Machinery

, o• ^ hv R Phase C must start from slot 9. For conductors


return conductors 11, 12 is designated by F + g _ J2) ag ghown in Fig 7 23 (a)
9, 10 ; the return conductors are 3 (- 9 + o 1 ) , mac[e up 0f return conduc-
Here also, phase belt of conductors 9,10 ts .ndtcated by C a id th a t ^ ^
t o r s 3 , 4 b y - C . It c a n t h e r e f o r e b e c o n c lu d e d t h a t f o r A d q ^ C -B .
phase-spread 60°, the sequence ofphase-belts (or phase- an s , > > > >
. r \ z? j? r chown in Fis. V.22 (fl) csn be u s g u to
Star of conductor emfs (or slot emfs) E it E2, •••£12 OVQmi«nfinn nf Fitr
obtain resultant voltages across terminals Au A2 ; B it B 2 and Clf 2- •
7.23 (a) reveals that phasor sum of conductor emfs E v E 2, En and E q as 1 us ra g. .
(6 ) gives the resultant voltage A across terminals A 1( A2. Similarly, the resultant voUages 5and
C across the respective terminals B u B2 and C„ C2 are obtained as shown in Fig. 7.23 (6). The
three emfs A, B, C are time displaced by 120°. Two adjacent slots belonging to phase A, B or C
subtend an angle of 60°, the phase-spread is therefore 60° in Fig. 7.23.

V=3 0
✓ >" S
C ' V >
V
CL

<•1 (\ c! i 1b 0

Ai A2 B] B2 Cl C2
(a)

-E 12

X j ”, DU • a lo ts . 2 -p o le W in d in g a rra n g e m e n t.
(6) T im e -p h a s e d ia g ra m fo r th e e .m .fs. g e n e ra te d in (a).

7.7. Double Layer Windings


Synchronous machine armatures and induction-motor stators above a few kW are wound
with double layer windings. If the number of slots per pole per phase q, i.e., S /m P is an integer,

Scanned by CamScanner
[Art. 7.7
A rm a tu re W in d in g s 857

then the winding is called an integral-slot t


ie. q is not an integer, the winding is called fivw, 1° f 389 number of slots Per P°le Per Phase­
. °nal-slot winding. For example, a 3-phase winding
with 36 slots and 4 poles is an integral slot windina l 36
ding, because here q = — = 3 is an integer. A
3-phase winding with 30 slots and 4 poles is a ! . * . . 30 5
P 8818 a fractional-slot winding, because q = | is not
an integer. In both the integral and fractionaUW ^ , 3x4 2
to the number of slots, i.e. C = S. In order to nht • ^“lgs»the number of coils C is always equal
that the number of slots is a multiple of the number ofph^es^ polyphase Ending, it is essential

per pole. If the coil-pitch is taken tobe t^uT to hdTpltoh^the^b ” ofa Ending is 6 slots
in slot 1 should be connected to bottom coil-side (shown by

Since the* are 6 slots per pole of !80°, the slot angular pitch is T. i f . 30°. For a phase spread
of60°, slots 1 and 2 must contain coil-sidp<? + u a
connected to bottom coil-side in slot 8 (= 2 + 6 l Thp ^ pper co^‘s'de in slot 2 must be

r ^ r .T a ^ e ^ -
^itatedthat ^
N S N ^ s

12 3 6 5 6 7
I j 9 >0 11 12 13 U 15 16 17 18 19 20 21

H t t 1

A1°
Fig. 7.24. P ertain in g to the double-layer, full pitch integral slot winding.

Now consider a 3-phase machine with slot-angular pitch r = 30°, slots per pole = 6 and coil-
Span = full-pitch = 6 slots. For phase-spread a = 60°, 2 1 = ^ adjacent slots must belong to

slot8 7^ ^°r C0^ 'sPan = 6 * top coil sides in slots 1, 2 should be Connected to bottom coil-sides in
for h 1+ ® (= 2 + 6 ). As coil-sides 7, 8 form the return coil-sides for upper coil-sides 1, 2
£ m ase A>phase-belt 7, 8 is labelled - A and phase-belt 1, 2 as A, Fig. 7.25. Starting of phase
PhaUS» be Space disPlaced by 120° (= 4 slot-pitches) from the start of phase A. In view of this,
phaSe must start from slot 5 (= 1 + 4). For 60° phase spread, two slots 5 and 6 must belong to
Here6 ? ' The top co>l-sides in slots 5 , 6 should be connected to bottom coil-sides in slots 11, 12.
form- Phase-belt consisting of lower return coil-sides 11, 12 is designated - B and that
have ^,Upper c°il-sides 5, 6 as B, Fig. 7.25, slots 9, 10 belong to phase C. As thc various coils
then, span = full-pitch = 180° each slot contains coil-sides belonging to the same phase. If
coil ft;^0e phase winding having more than 2 poles is completed, slots 3 4 would contain return
th a t.S ' for Phase C, these phase-belts are therefore labelled - C in Fig. 7.25. It isthus seen
quenon, e 8Pread of 60° produces a phase-band sequence of A, C, B, - A, C, - B for a se-
Ce °f AflC for 3-phase voltage.

Scanned by CamScanner
A rt. 7.7]
858 E lectrical M achinery

Fig. 7.25. Developed winding diagram for a 3 -phase m achine w ith double layer,
slot-angular pitch = 30°. coil-span = 6 slots and phase spread - 6 0 .

Consider now a phase spread of 120° for the 3-phase machine with y = 30 , coil-span = full-
pitch = 6 slots. A phase spread of 120° requires 4 = — adjacent slots for each phase. Therefore,
upper coil-sides in slots 1, 2 ,3 , 4 pertain to phase A and its return coil-sides are in slots 7 , 8 , 9,
10. As before, the phase-belt consisting of upper coil-sides 1, 2, 3, 4 is labelled A and the phase-
belt formed by lower coil-sides 7, 8 , 9, 10 is designated - A. Similarly, the arrangements of
coil-sides pertaining to phases B and C is as shown in Fig. 7.26. An examination of this figure
reveals that for a phase spread o f 120°, sequence of phase-belts is A, B, C ; for a sequence of
ABC for its 3-phase voltages.
P o le - p ilc h Pole - p i l c h

• !
■SO—j y * 3 0oIh

5 6 I 10 12 13 1A 15 16
A A A !B B B
or
-B -B -c -C -C -A -A I -A -A
i IE i El I IE
-B -B
! El
-c -c-B -C

120° ■ 120

Fig. 7.26. Double-layer developed winding diagram for a 3-p h ase m ach in e w ith
slot-angular pitch = 30°, coil-span = 6 slots and phase spread = 120°.

In Fig_7.27 are shown the phase-belts for the first 12 slots, 2 poles of 3-phase double-layer
^ 7 3 f f ' sp,read 0f 60°- The Ph ase-belts shown here can be used to
bk n n r n Z U ftT “ h , f , " 1 T ? " 15 °an * * obtaiM‘l h? their interconnection. For
this purpose, left-hand terminal of phase-belt A forms its starting terminal A ,. The other end
of this phase-belt A is connected to right-hand terminal of phase-belt - A. The left-hand ter-
Phase -c B
belts -A -B
Slots 2 2 2
mm (W n 2
rw nmn n rm rm \

Scanned by CamScanner
(Art. 1^1
, , A rm a tu r e W in d in g s 859
minal of phase-belt - A forms the end terminal nf k , -------------------------
interconnect the phase-belts of phases B *P l*9 • same Procedure is adopted to
niinals B ,, B 2 and Clf C2 in Fig. 7.27. ’ 1 ° obta,n their respective start and end ter-
The winding arrangement can also be s h n k
method. In the dock diagram, the arrangement oU h . method, called clock-diagram
winding arrangement of Fig. 7.25 is drawn in Fie 7 o« 3 ln b°th layers is illustrated. The
the form of clock diagram. ln
Consider now another examnle witk qc i .
full-pitch winding with a phase spread of 6 0 °H trelhe
slot-angular pitch y = ^ . 20.. Therefore ^ #

spread of 60°, 3 slots f= fiha^sgreacn


. Y Would contain coil-
sides belonging to the same phase as shown in 1 1
diagram of Fig. 7.29. As before, the sequence of the Dh «
bC'th A B ' ~ '4 ’ C’ ~ B ‘ f° r ° ne polc Pair The
method of interconnection between the various phase
bells is shown in Fig. 7.30. Note that there arc 4 n h !!!
belts for each of the three phases. For the phase I the
terminals A„ A2 may be connected cither in series or in w
F ig . 7 .2 8 . C lo c k d ia g r a m fo r F ig . 7 .2 5 .

A-) ) r ,

cr-60° %
36
1 2 ^
b \~5
•b E l
L~ > t r /a X -c \ 5

P aralle i ■ 1.2% . C lock d iagram for 36 slots, 4 poles, phase spread 60°.

8eries c Wltb tbe terminals, A3,A a depending upon the operating voltage of the machii._.
inc. .For
See Pjg ^nec^i°n A2 and A 3 are joined together giving A l and A 4 as the two output terminals,
°utPutt 3° (ab Phases B and C are also connected similarly to give B h B Aand C u CAas the
erminals. Here A j, B h form either the starting or the finish of phases A, B , Crespec-

amScanner
A rt. 7.71
860 Electrical Machinery

P hase -c B A C -B
6 -A -B 3
b e lts ■A *C 3 3
3 3

n n nn nn nn
3 3
S lo ts ■ ■3 3 3 inn
ffT^ W

a3 a6
>A|
(a)

A -C B - a C -B
Phase C -B
A -C B -A
b e lts 3 3 3 3
S lo ts 3
ftfri
3

nn nn
3 3
inn nrci pre] pnr] nnn
nn
A2
Al

S ta rt of A F in is h of A

(6)
F in. 7 30. I llu s tr a tin g th e in te r-c o n n e c tio n s a m o n g s t th e p h a s e -b e lts
o f a 3-phase w in d in g w ith 36 slo ts, 4 poles and 60° phase s p re a d .

lively. For parallel connection. A„ A, are joined withAj, A, respectively as shown in Fig. 7.30
(b ). In this figure, the joints of A 1(A3 is taken arbitrarily as the start of phase A. For clarity
sake, the inter-connections of the various phase-belts is shown only for phase A. For phases
B and C, the same procedure is followed as adopted for phase A.
Coil pitch in polyphase machines is usually less than
In te g ra l slo t c h o rd e d w in d in g .
pole-pitch and such a winding arrangement is called short-pitch, fractional-pitch or chorded
winding. The advantages of using chorded coils are :
(t) to reduce the amount of copper required for the end-connections (or over hang) and
(ii) to suppress or reduce the magnitude of certain harmonics in the waveform of phase
e.m.fs. and mmfs.
In most a.c. machines, it is usually a standard practice to use chorded winding. The coil-
2 , , .
pitch generally varies from 2/3 pole-pitch to full pole-pitch. A coil-span less than - pole-pitch is
u
not used in practice. It is because a chording more than 1/3 pole-pitch would reduce the phase
emf noticeably. In order to compensate for this reduction, use of additional turns becomes
necessary and this offsets the saving made in the overhang copper. Consider a 3-phase machine
with 12 slots, 2 poles, 60° phase spread and with coil-pitch equal to 5/6 pole-pitch, i.e. I 50
electrical, 5 slots or 5 teeth. The winding details in developed and clock diagrams are illustrated
in Fig. 7.31. Upper coil-sides in slots 1 and 2 are connected to bottom coil-sides 6 (=1 + 5) and 7
(2 + 5). Phases B and C start from slots 5 and 9 as before. Note that some slots do not contain
coil-sides pertaining to the same phase. The interconnection between the phase belts of the
three-phase winding is done in a manner similar to that explained in Fig. 7.30.
A few examples given below, illustrate how 3-phase double-layer windings are design^
and drawn.

2*

Scanned by CamScanner
[Art. 7.7_____________________________ _
____________ A rm a lu r e W in d in g s H6 1

1.11
4 5 6
A -C --C
c aB B I -A
B - A c3 ™ V 12

U -C -c
snSJjlS
6B B |-A -A C C
I \ 1
—l*?=30l
\r l —
n

° ^ “A the
Total num ber o f slots = 24 n li i
Number o f p oles =4 Double-layer wtnd.
Coil-span fu ll.pUch

Name the upper coil-side a s n a n d low er coil-side as » ' for 24,

iiC n tZ tu p T ^ dlg^
in
W the connection o f

id)Calculate its distribu tion factor. IE(AMIE : S ’98). Parts (a), (6 ) & only]

Solution, (a) Slot angular pitch, y = — = 30 °


24
Phase spread, CT_ go0

Number of adjacent slots in one phase-belt = ^ = ^ = 2


y 30
. 24
Coil-span = full-pitch = — = 6 slots.
4
W in d ’ l
there ar ^ 8 f° r th° 3'Phase a r m a t u r e is p r e p a r e d b elo w . F o r d o u b le -la y e r w in d in g ,
W red 6 c o d 'S id e s p e r s lo t ; o n e u p p e r co il-sid e a n d th e o th e r lo w e r co il-sid e. A s re-

na m e d r Po ' e i \ COil' S i d e s a r e nam ed 1> 2 > 3 > ••• a n d th e co rre s p o n d in g low er co il-sid e s a r e
1 ’ 2 . 3', ... .

one con ^ ase^> upper coil-side 1 in slot 1 is connected to lower coil-side T (= 1 + 6) to form
M t- / ti,P P e r c o d ' s *d e s 1 . 2 form phase-belt A and their lower coil-sides 7 ' , 8 ' make phase-
^ » ese are called coil-group (C .G .) I and so on.
n toe winding table below, 5 and F denote start and finish of any one phase.

canned by CamScanner
bcai
A rt. 7.71
862 E le c tric a l M ach in ery

A-phase :
+6 +6 ...C.G.I.
S (1 -7 ')-> (2 -8 ')
^ + 6 = 1 pole pitch
(7 - 13') (8 - 14') - C-G - 11 Phase A
+6
(1(13 - 19') —> (14 - 20')
...C.G. Ill
+6
F (19 - 1') <- (20 - 2 ' (= 26 - 24))-* -.C.G. IV
C -phase: ...C.G.I.
F (3 - 9') -> (4 - 1 0 ')x
)+6 ...C.G. II
^ ( 9 - 1 5 ') <—(10 - 1 6 7 . Phase ( - C)
< (15 - 21') -> (16 - 22')---------
+6
...C.G. Ill

S (21 - 3') 4- (22 - 4' (= 28 - 24))- ...C.G. IV


ZJ-phase :
S (5 - 1 1 ') -> (6 -12')> ...C.G.I.

(11 - 1 7 ') < -(12 - 1 8 ')


v ...C.G. II
Phase B
+6
c ( 1 7 - 2 3 ') ...C.G. Ill
(18 —24')-
+6
...C.G. IV
F (23 - 5') <- (24 - 6 ' (= 30 - 24))—1 ...U.ii. iv
Above is the winding table for 3-phase armature winding having 24 slots, 4 poles, phase

SPre<6 ) B y referring to the above winding table, detailed winding diagram for phase A only is
drawn in Fig. 7.32 (a). As shown, phase A starts from top coil-side 1 in slot 1 and finishes P
coil-side 1 9 in slot 19. Note that the winding is traversed clockwise in coil - groups I and IIIand
anticlockwise in coil-groups II and IV. Terminals Al and A 2 are marked as the start and finish
respectively for phase A winding.
coil group I CG11 C G 111 CG IV
^ N S ^ 'Z '' N

Top coil- side) ] J


A Slot NO’ J

Lower ,
coil-side No

A1 * AAj
Fig. 7.3 2 . (a) D etailed d ouble-layer w inding d iagram for p h ase A for a 3 -phase
arm atu re having 24 slots, 4 poles, p h ase spread 60°. E x a m p le 7 .1 1 .

Scanned by CamScanner
V

[Art. 7.7
— ________________ A rm a tu r e W in d in g s 8 6 3
(C) The star of coil emfs can be drawn sjmilar
emfs. to the star of slot emfs or star of conductor

' ,

Scanned by CamScanner
H »
i i
» I1 A rt. 7.7]
864 E le c tr ic a l M a ch in e ry _________________________________ ______ ___________________ _ " ^
'— — “ , fUon slot 7 (= 1 + coil-span) would be
Suppose slot 1 is under the middle of north (WI po , ^ x _ r ig m aximUm, it is
under the middle of south (S) pole. Therefore, emf g FnvtheTt slot 13 (= 1 + 2 pole-pitches)
therefore shown by a vertical phasor 1 - 7 init ig. i (^ the mi(jdle of S pole. Thus,
is under the middle of N pole and slot 19 (= 13 + coll’®P fore emf in coil 13 - 19' is in phase
coil 13 - 19' has also maximum emf generated in n. 1 a v ertical phasor marked
w ith the em f in coil 1 - 7' and the two emfs are ^ ^ pitcheg) _ 20' are shown by a
1 - 7', 13 - 19' in Fig. 7.32 (6). Coi emfs 2 8 £ emf phasor 1 - 7', 13 - 19'. Same proce-
phasor displaced by slot-angular pitch y - 30 from the e y
dure is adopted for drawing other coil emfs in ig. • • showing phase voltages A, B, C
Phasor diagram for narrow-spread (or phase sp first obtain thg
can be drawn by referring the w .nd.ngjab^ made m part (a ,.^ g_ ^ ?
phasor sum of coil emfs 1 - 7 , 2 - 8 ot coil gr p Now ad(J C01j_emfs
coil-group 77 and add to oa giving o* as the resu t a n . e m ^ i f r ^ ^ and ^ _ y
13 - 19', 14 - 20' of coil-group 777 to ob to get oc. Furt . g Same ,
of coil-group TV and add to oc to get od as the net phase voltage A m * g V/
dure is adopted for obtaining phase voltages B and C in Fig. . ( )•
(d) Distribution factor, from Eqn. (3.46), is given by
o .
sm -
k d— v
2 sin

24
Here 0 = 60°, y = 30°, 7= ^ ^ =2

• k d= „Sin 3° ° - ■ 0.96593.
•• a 2 sin 15°
E xam p le 7.12. Repeat all the part o f Exam ple 7.11 in case p h a se spread is 120 .
[E.M.D. (AMIE : S’ 98)\

Solution, (a) Phase spread, c = 120°


, . , , a 120
Number of adjacent slots in one phase-belt = - = = 4.

Coil-span = 6 slots
Winding table for a = 120° is made below :
A-phase : S (1 - 7') - > (2 - 8') -4 (3 - 9') -4 (4 - 10') -4 ...C.G.I
i + 12 (= 2 Pole pitches) .
-4 ( 13 - 19') (14 - 2 0 , 4 (15 - 21') - 4 (1 6 - 22') F ...C.G. 11
5 -phase: S (5 - 11') -4 (6 - 12') v7 - 13') -> (8 - 14') -4
-> (17 - 23') -4 (18 - 24') -4 (19 - 1') -> (20 - 2 ' (= 26 - 24)) F - c G- 11
C-phase : S (9 - 15') -4 (10 - 16') -4 (11 - 17') -4 (12 - 18') -4 -C .G . }T
-4 (21 - 3') -4 (22 - 4') (23 - 5') -4 (24 - 6 ') F - CU
(6 ) Winding diagram for phase A is drawn in Fig. 7.33 (a). As shown, phase A starts fr°m
top coil-side 1 in slot 1 and finishes at bottom coil-side 22 ' in slot 22 .
(c) Star of the coil-emfs is the same as shown in Fig. 7.32 (6 ).
\ B Ccan
Phasor diagram for wide-spread, i.e. phase spread 120°, showing phase voltages A,
be drawn by referring to winding table prepared in part (a) above. For phase A, first obt, ^ nlfs
phasor sum oa of coil-emfs 1 - 7', 2 - 8 ', 3 - 9', 4 - 10' of coil-group 7. Now add coi -

Scanned by CamScanner
Armature Windings 865

Lower
coif-sid e N o . — *

16-22

Pjg y ^ — .— 0
• P h asor d ia g ra m sh o w in g the p h a so r sum o f coil em fs to obtain phase voltages
A and B . P h ase C is also shown, E xam p le 7.12.

icanneaDy CamScanner
A rt. 7.7)
866 E le c tr ic a l M a ch in ery _____________ _ _ _ _ _______ -

19 _ iq' 14 on' ik o r ifi _ 92' of coil-eroup U to oa to obtain ob as the resultant phase


Jolt'age A to Fig 7.33 (6). similarly, the p h ase voltages B and C can be obtained in Fig. 7.33 (6).
(d ) Distribution factor, from Eq. (3.46), is given by

S m 9 2 4 - 9
kd = \ Here a = 120°, y = 30 , q = j ^ - 2
. a
V Sm 2q
_ s i n 6 0 _ Qg66
R(l 120
. 2 sin ^

E xam p le 7.13.' A 3-phase machine has 4 poles, 9 slots per pole an d 4 condtwtors per slot
arranged in two-layer lap-connected winding with coil span closely equ al to 80 h o f the pole
pitch. There are to be two distinct circuits per phase arranged for p arallel connection. Draw up
a winding table, indicating clearly the start and finish o f e a c h p h a s e w indm g and a circuit
diagram for parallel connection. Also obtain effective turns per phase. [E.M.D. (AMIE : W98JJ
Solution. Slots per pole = 9
180
Slot-angular pitch, y = - j p = 20°
For a phase-spread of 60°, number of adjacent slots belonging to any one phase
_ a _ 60 _ „
•• ' "y 20
Pole pitch = 9 slots
80
Coil-span = 80% of pole pitch = -r^r x 9 = 7.2
n n l p n i t r h =

100
So take a coil-span of 7 slots.
From above, winding table for this machine is prepared as under. Here S and F denote start
and finish of any one phase. The top coil-sides are labelled 1, 2, 3, ... and bottom coil-sides
r , 2', 3 ' , . . . .
+7
A-phase: S (1 - 8 ') -> (2 - 9') -> (3 - 10')^”7\ ...C.G.I.
) + 9 (= 1 pole- pitch)
W 10 - 17') <- (11 - 18') f - (12 - 19') ...C.G. II
+ 9(
M 19 - 26') -> (20 - 27') -> (21 - 28')­ ...C.G. Ill
’ +9
F (28 - 35') 4 - (29 - 36') <- (30 - 1' (= 37 - 36 V ...C.G. IV

C -p h ase: F (4 - 11') -» (5 - 12') - » ( 6 - 13') ...C.G.I.


1+ 9
^ (13 - 20') <- (14 - 21') 4- (15 - 2 2 ')/ - C G‘ 11
+9( TTI
^ ( 2 2 - 29') -> (23 - 30') -> (24 - 3 1 ' ) ^ -G G- 111

S (31 - 2') 4- (32 - 3') 4- (33 - +9 - C-G' lV

Scanned by Cam scanner


(Art 7.7
Armature Windings 867
B-phase : S (7 - 14') - > (8 - 15-) (9 _
...C.G.I.
+ 9 ^ ( 1 6 - 2 3 ' ) - ( 1 7 - 2 4 ' , ^ (1 8 . 26, / 9
...C.G. II
(25 - 3 2 ' ) ( 2 6 - 33 ') _ (27 _ 34,
...C.G. Ill
F ( 34 - 5 ') <— (35 _ g.( )+ 9
As required, start and finish of PhaSe , r r tv
.able It is men from this winding table that each p h t e h ° a ‘ndicated in the above w iping

^ * c ,™ i - ^ / and « / f 0 rm' p h a H ^ ^ w and wn. ^ u p w ^

y s s s tt- *■ '« » s a r a s s s *
The various phase-belts are shown i p-
amongst phase-belts of phase A are s h o w n V r m i^ lt A ' 734 the interconnections
. , " Al arejomed together to form the
Coil ____
groups ~ I
III IV
PK“ " ‘ A -c e~' c -
cu>
Slots- _ a - t- B
-c e -a -B
nn
Stort of phase A ^ Ai
Finish of phase A
Coil
9foups
Ph. Belts
Slots

Start of phase B
Finish of
Co ft phase B
9'oups •
I!—

A Hi IV
Ph B e lts .
r­ —A —
-\ r
S lo lt( _ A ~C B -A
>3 3 -C B A A C -B

Finish of phase C ^ Start of phase C C,


(c)
P jg y f
two Ircuit ^ ‘fg rain for inter-connections amongst the phase-belts for obtaining
e * circuits for (a) phase-A (6) phase-fl and (c) phase-C windings.

Scanned by CamScanner
__________________ A n . 7.8J
868 Electrical Machinery -------------------------------
• i a ' 4 " together form the finish of phase A. Note that
starting point of phase A. T e r m i n a l s \A2 , 2 g . Fj 7 34 (ay One parallel path is
there are two distinct parallel paths for phase- „ a » Similarly, interconnections
from A,' to A,' and the second parallel path, ,s from 4 ^ * ^ 7 J 4 ((>) (c)
a m o n g s t p h a s e - b e l t s p e r t a i n i n g to p h a s e s \ B a h a s e s B a n d C in t h i s f ig u r e ,
ly . T w o d i s t i n c t p a r a l l e l p a t h s a r e a ls o in d i c a t e d f o r p h a s e s zj
9x4^4 ci
Total turns in the machine = ^ = 72
, _ _9 _ q y —20 °
Here phase spread a = 60°, slots per pole per phase, <7 - 3 »T
, _ sin g /2 _ sin 30° _ q 9593
.-. Distribution factor, nd - v “ 3 sin 10° *
q sin ^
Coil-span = 7 y = 7 x 20= 140°
Chording-angle, £ = 180” - 140” = 40”.
Coil-span factor, kp = cos —= cos 20° = 0.9397.
/. Winding factor, kw = kd x k p = 0.9598 x 0.9397 = 0.901924.
Number of effective turns per phase = [Turns/phase] x kw
= ^ x 0.901924 = 21.65.
o
7.8. Fractional Slot Windings
In fractional slot windings, the number of slots per pole per phase is not a whole number.
But from the view point of symmetry, the number of slots must be divisible by the number of
phases, i.e. 3.
The advantages of these windings are as given below :
(:) This winding reduces the high-frequency harmonics in the e.m.f. and m.m.f. waveforms.
(ii) This winding permits the use of already existing slotting numbers for the armature
laminations, because here the armature slots need not be a multiple of the number of poles.
This avoids additional expenditure on new punching tools etc.
(iii) This winding allows more freedom in the choice of coil-pitch. For example, a machine
4
with 48 slots and 10 poles has 4 - slots per pole and the coil-pitch for a fractional slot winding
can be taken as 4 or 3 (always less than the slots per pole).
The restrictions of the fractional-slot winding are th a t:
(i) it can be used only with double-layer windings and
(ii) the number of parallel circuits is limited (as explained at a later stage in this article).
Let S be the total number of slots (a multiple of 3 for a 3-phase winding) and P be the
number of poles. Then the slots per pole per phase, for a 3-phaseJSdUng a « W If * is
the highest common factor between 5 /3 and P, then slots per pole per phase can be written as
5 /3 _ k •Sh Sk
P kP k P,
3
where S k = 5 /3
k~
and p, = PP
k= k

Scanned by CamScanner
_ - ~— Armature W inding W9

The ratio S ./P , is called the seachitrl ratio of the fractional slot wind,no S, rep
per phase, distributed amone P. R„ e - „ e „ ; , *• * rrprr. cnu
t e n 5 /3 and P represents the number of re p e a ta b le ^ Th%h" :h''5' comm° " <*<'<" * •*-
_v be connected in series or in parallel k aUmriv ,u ,0" s Since thc repeatable sections
windin*. For example, wiih S 1 48 and P Mo™ pa,alW dreurt* •
_ 4 8 /3 16 2 x 8 8
Pk 10 " 10 2 x 5 „5
Here highest common factor k is 2, therefore v.i •
sible parallel circuits is 2 . The characteristic ratio 8 / ^ s W T \ 'T ,hp numb<' r of P0*'
distributed among 5 successive poles. In other words 8 coils b I ° ar<’ 8 C<" l!‘ ,K r pha,<--
distributed under 5 adjacent poles as follows : ’ 'longing to any one phase may be

lslP°lc 2nd pole 3rd pole .


. ■ ,
2 cods Icoil.. 2coj, 4,h r,th pole

• 1 co“ 2 c o ils

j nX. r.uh^v2e;t :^ s^ n r r st
* °f 8 per Phase
*• ~ '
J n " e d t i o w ■VCrC° mC thiS d' mCUlty’ aC0" ' KroUh table as shown Table 7 2 is prepared a,

1. Calculate the slot-angular pitch,


180 x P 180x10 0 l c
Y= * =37^
5 48 _
2

slot 2 ? 4 Cate241T -M i? h w a ? 8,° ‘ " umber 1 wi,lh “ ro dcercc- N °" fill in the table for the next
sn. , I f 1 °f 8 mt° aCC° Unt the si°t*angular pitch In case the angle exceeds
a particular pole! (° qUal ‘ ° ° ne TWs S" p « " of the locat.on I Z t

l o c a t ^ s u n d e r - ^ C° nSiderC<l pr0greSsiTOl* thc p h a - Broups for a phase-spread of 60 are-


Phase group A : 0° < angle < 60°
Phase group C : 60° < angle < 120°
Phase group B : 120° < angle < 180°
As proved before, the sequence of phase groups is A, - C t B , - A , C , - B

Poles can hT k!!P ° f f^ °VC thre° StepS’ thC distribution of 8 <= Sk) coils under 5 (= P k) adjacent
e obtained from Table 7.2 which is now prepared as under :
T a b l e 7 .2

1-oie-pucn 1-----------
-----------------------------------------------------------------
----------*1
1 2 3 5
*
0° 37! 75 112 5 ’ 150
2
A A -C . ............... . .
L »
1*— ----------- _
------------ I'ole-pitch 2 -----------------------
6 7 8 (1 .......................... 10
7 .5 " 45 ® 82 5 120 15? 5

_ -A - A c M B

Scanned by CamScanner
A rt. 7.8]
870 Electrical M achinery

\*---------- 15
14
12 13
Slot No. 11
1 27.5 165
52.5 90
Angle 15*
B B
A -C
Phase A
Pole-pitch 4 ■ ■*1
19 20
17 18
Slot No. 16
135 172.5
60 97.5
Angle 22.5
C - B -B
Phase - A C
T|
--------Pole-pitch 5
l<----------
22 23 24
Slot No. 21
67.5 105 142.5
Angle 30°

-C -C B
Phase A

Table 7.2 shows that 8 coils belonging to phase A are distributed under 5 successive or
adjacent poles as follows :

1st Pole 2nd Pole 3rd Pole 4 th Pole 5 th Pole

2 2 2 1 1

There is another method of determining the distribution of S h coils under Pk adjacent poles.
This method is quite simple and straight-forward. According to this method, a tabular construc­
tion, as explained below is adopted.
In this table, number of rows is made equal to the basic pole unit Pk (in this case 5). The
total number of columns is made equal to 3 S b (in the present case 24). The sequence of the
phase-belts is obtained by dividing 3Sh columns in three equal sections ACB for a 3-phase 60°
phase-spread winding. Start from the extreme left and top square with a cross. Now proceed to
the right and go on putting crosses separated from each other by P b (= 5 ). After first row, go to
the second row and so on. From Table 7.3, it is seen that S , (= 8 ) coils belonging to phased are
distributed under/* successive or adjacent poles as obtained from Table 7 2 In Table 7 3 coils
pertaining to phasert are indicated by 2 crosses in first row for first pole, 2 crosses in the second
row for pole number 2 and so on.
T a b l e 7 .3
T a b u la r m e th o d o f c o n s t r u c t i o n f o r f r a c t i o n a l - s l o t w in d i n g s

It is also seen from both the tables thaf thn knits


belonging to phase C coils is 2 1 1 2 2 and for phase s T s T ^ 2 T Sl<>t SCqUCnCC ° f P

ic a illie u Uy uanrouai n it:i


A rm atu re W indings 87J

The b a sic p ole u n i t o f Ph p o l e s f o r t h i s e x a m p l e m a y b e t a b u l a t e d a s f o ll o w s :

1 2 3 4 5
+ 2 -2 + 2 -1 + 1
-2 + 1 -1 + 2 -2
+ 1 -2 + 2 -2 -t-1

T h e c l o c k d i a g r a m l a y o u t f o r o n e l a y e r o f t h e w i n d i n g ( u s u a l l y u p p e r l a y e r ) is s h o w n in F i g .
7.35. T h e l o w e r l a y e r h a s t h e s a m e g r o u p i n g s b u t is d i s p l a c e d f r o m t h e u p p e r g r o u p s b y t h e
c o il-p itc h . S i n c e s l o t s p e r p o l e i s a f r a c t i o n a l n u m b e r , t h e coil p i t c h c a n ’t b e f u l l - p i t c h b u t i t m u s t

be same for all coils. With 48 slots and 10 poles, there are yjj = 4 ^ slots per pole and the coil-

P O iE -2

PDLE-3

P01E-4

Fig. 7.35. C lock d iag ram for 48 slots, 10 poles, 60» phase spread fractional-slot winding.

Requirement o f s y m m e try . Even in fractional slot windings, the time-phase angle be-
'* * n the 3-phase voltages must be 120° or 2n /3 ^ ians. between ^ sUrting of
Let Sab and Sac represent the number of slot-pitch dispia
Phase A and the starting of phases B and C respective y. ©n
O-re
c v = — or ^ - + XK
ab • 3 3

^ S
o r ~"T- + *xn71
y= — or
ac ' 3 3
i» “C 3, T; _ n o 4 .. the polarity of the starting phase belts
of i re x may have any integral value. If x - * * ’ ^elt 0f phase A. In case a. - 1, , .
tf ases B and C is the same as that of opposite to that of phase A These
pd]a 6?°'arity of the starting phase belts of phases : ter.connection of the phase e s.
es must be taken into consideration dur.ng the mter

M l
Scanned by CamScanner
w
1
A rt. 7.8)
872 E lectrical M achinery

Now slots per pole per phase are,


S S /3
q=
3P
or
P J_
S 3q
kP 71
Slot pitch, y= S ~ 3q
( 2n
S(lb =1 y +x7t
(2 7 1 ' = 3a + X7t = q (3x + 2)
Yv 71

= ^ ( 3 x + 2) ...(7.9)

Similarly, Sac = q ( 3 x + 4 ) = -^ (3 x + 4) ...(7.10)


rl
It may be seen from Eqs. (7.9) and (7.10) that if Pk is equal to 3 or a multiple of 3, Sah and
Sac can never be whole numbers and
therefore, fractional slot winding is not
possible. For example, 33 slots and 6 poles
give Sk/P k equal to 11/6, therefore, frac­
tional slot winding is not possible for these
slots and poles.
For illustrating the inter-connection
among the various phase belts, the layout
of the winding arrangement is as shown in -B § rr

Fig. 7.36. In the present example, ^ = - .


Pk 5
8
Sab ~ g ( 3 x + 2 )
8
For x = 1, Sab = 3 (3 + 2 ) = 8 .
Therefore, the starting of phase B is
displaced by 8 slot pitches from the start­
ing of phase A. Since x = 1, the polarity of *
-A B
the starting phase-belt of phase B must be Fig^ 7.36. P ertain in g to th e in ter-co n n ectio n o f phase belts
opposite to the polarity of starting phase or a fractional slot w inding w ith 4 8 slots, 10 poles.
belt of phase A, see Fig. 7.36. 60° ph ase spread .

Now 8
Sac = g (3x + 4)
For x = 2 8
Sac = - ( 3 x 2 + 4) = 16.

phase A. Since x = 2, the polarity of the starting oh h l / f 6 ^ pitches from the starting of
starting phase belt of phase A, see Fig 7 36 In thi^f ° f phase C is the same as that of the
only shown for the sake of clarity. However s ta r t i n ^ n k ,nte[*connection of p h a s e A belts is
Fmm tkn e f * x . . ’ of phases B and 0 has been indicated.
From the starting of phase A slot seouenm „r*k Z ' ” inaicaitu‘
10 poles. Examination of Fig, 7.35 reveals that the flM * M ‘ S Ls 22211' 22211 for “'! ‘If
m j : ____. .. . tnat thG s‘ot sequence of thn nkncnc n nml C belts
from their corresponding s t a ^ f p ” ^ Ph“ CS ° aWl ' bC"

* 6 - us
Scanned by CamScanner
(Art- 7.8
" ----- ----------------- ------- Armature Windings 873
7.14. (a) Develop a winding table for th--------------------------------- ----------------------
E x a m p le
the following data : * e for the armature of a 3-phase machine having
48 slots, 10 poles, narrow-spread, coil-span- 1 1 ,
Name the upper coil-side as n and lower coil ,
(b) Draw the coil-emfstar for phase A onl °S* the nth slot*
Solution, (a) A table giving the construct- r
already prepared in Table 7.3. This table is repeated a - - " WindinB of this « “»nple is
are written m the boxes instead of crosses. So now start ? “ . f ' 6 7'4 ' but here
(or and write 1 . In the next cross of Table 7^
b o x ) fr° m the and top square
row, wnte 3 and so on, till one unit of 5 poles is mm ’ 1 T f - and the next cross in the same
that for the first unit of 5 poles, completed in Table 7.4. It is seen from this table
(i) coils 1, 2, 6 , 7, 1 1 ,1 2 ,1 6 and 21 (total fit nDi . ,
(ii) coils 3, 4, 8 ,1 3 , 17, 18, 22 and 23 (total 81 bT® P ’
(iii)coils 5, 9 ,1 0 , 1 4 ,1 5 ,1 9 , 20 and 24 (total 8 >beto <_ C> a“ d
The winding table 7.5 for phases AB r a g ° phase B
coil-side 1 to bottom coil-side 4' (= 1 + 3); ^ c ! ! L ^ F ° r phase connect ‘ °P
Remember that coils under first unit of 5 poles r e a L t and , T C0,' ' Side 5' (= 2 + 3) and ™-
10-pole machine. P ePeat under the second unit of 5 poles for this
^ *i j * . .. T a b l e 7 .4
^ O o d d .,tn b u t io n f o r fr a c tio n a l s lo t „ io d in e , E x a n p U , u

T a b l e 7 .5
D e v e l o p m e n t o f w in d i n g t a b l e f o r f r a c t io n a l- s lo t w in d in g , E x . 7.1

Scanned by CamScanner
A rt. 7.9|
8 74 E lectrical M achinery___________________ _________ _________ _____
Vertical
The information obtained from Table 7.3 can also be
derived from Table 7.4 and 7.5.
, 10x180
(6 ) Slot-angular pitch y = ^ = 37.5°

Suppose emf in coil 1 is taken vertical. Then emf m coil


2 would be displaced by one slot-angular pitc Y~ _
from the emf in coil 1. This is shown in Fig. 7.37. Table /.
shows that emfs in coils 1, 2 under pole 1, in coils l i , — Horizontal
under pole 3 (= 1 + 2 pole-pitches) and in coil 21 under pol
5 are positive. Emfs in coils 6 , 7 under pole 2 and in coil lb
under pole 4 would be negative. Therefore, in Fig. • . coj
emf 6 is shown (7 .5 ° + 180°) away from vertical eml 1; coii-
em f 7, 187.5° away from coil emf 2. Em f in coil 11 is
7.5 x 2 = 15° away from emf 1, similarly emf in coil 12 is
15° away from emf 2 as shown in Fig. 7.37. Similarly, emf
in coil 16 is 15° away from emf 6 and that in coil 21 is ahead Fig. 7.37. C o il- e m f s t a r f o r ph a se A.
of emf 2 by 7.5°. All the emfs generated in one S-pole unit
shown in Fig. 7.37 constitute the coil-emf star for phase A.
7.9. Single-Layer Windings 4 .
As stated earlier single-layer windings (SLWs) are not commonly employed in practice
except for machines of a few kW rating. The three-phase SLWs are of two types, namely (i)
concentric windings and (ii) mush windings. In addition to the disadvantages enumerated ear­
lier, SLWs have the following limitations :
(i) Concentric SLW cannot have chorded coils.
(ii) In concentric SLW, effective coil span is equal to pole-pitch, even though coil-span of
individual coils in a coil-group differs from pole-pitch.
(iii) In mush windings, coil-span is constant. In other words, all the coils have the same
shape and size.
(iu) In mush windings, coil-span is always odd. This requirement may result in the coils to
be chorded. For example, in a 4-pole, 36-slot machine, the coil-span = pole-pitch = 9. In 4-pole,
24-slot machine, coil-span cannot be equal to pole-pitch = 6 slots. A coil-span has to be odd, so
a coil-span of 5 slots or 7 slots can be chosen. A coil-span of 5 slots results in chorded coils.
(v) Fractional-slot windings cannot be used for concentric as well as mush windings.
In single-layer windings, one coil-side occupies one slot completely, in view of this, number
of coils C is equal to half the number of slots S, i.e. C = ^ S. Now concentric and mush windings
are described briefly.
7.9.1. C oncen tric Windings. These windings are so called because the coils under one
pole pair are wound in such a manner as if these have one centre. The concentric winding can
further be sub-divided into (a) half-coil winding or unbifurcated winding and (6) whole-coi
winding or bifurcated winding.
(a) H alf coil w inding. Half-coil winding arrangement with 2 slots per pole per phase and
for 60° phase spread is as shown in Fig. 7.38 (a) for one phase only. A coil-group may be define
as the group of coils having the same centre. Thus Fig. 7.38 (a) reveals that there are two
= - coil-groups and each coil-group has 2 coils [= £l?.ase spread \ ^ ^ words, the numbed
ZJ i slot pitch J
of coils in each coil-group is equal to the number of coil-sides in each phase belt. It is als°

""Scanned Dy uam scanner


Armature Windings 875

f^ co u -g ro u p ^ J oil g r o

—* — I -------- --
-a
■b

c. (b)
d-

Fig. 7.38. Sin g le lay er co n cen tric w inding (0 ) half-coil *2

observed from this figure that the coils arP g6ment ^ ^ Wh°Ie coil arraneement.
direction in all the coil-groups, [clockwise in *S * * they Carry CUrrent in the same
(6) Whole-coil w inding The whole 1 *
phase is as illustrated in Fig." 7.38 (6 ) for'one X ^ o n W T ^ T " 1 With 4 sl°ts P<* Pole per

* ^ s a w s s a i a s :— - - • -“ =

Solution (n) •/
Z h Z
^
J lS 1™" *»**>•' o n u i
“ 60 P^ase spread,
on*(a) H alf-coil concentric winding.
Slots-angularpitch, y=—* *8Q-=30o
^11-pitchorpole-pitch =^ =6slots-pitches.
*"Hbe?,Pconnected
t e d toa21t°30(-76lThat
7Jr? Sf " beltmC0,US!1'pitch2isSlequal
0tS-“ toW*
6s|ot-1pwoul
itches.dbeCoil-
Joinsed.deto°
- or
nt of phase T ^ ' i : 6)and80t
PhaSe A,
, i.e. from slot number 5 shou,datart awayfromthe- £ i
“ l2o°
- 1 + 4 j Similarly phase C should start
frt"» * t 9 (= 5 +4).
The
a!^hich t S ‘ 0fT ,g 7,39 i[Justrates the coil distribution in all the 24 slots. For the instant
Pi? 7.39 Note tu / T 3" 1 ° f F l g - 7 39 is drawn>the Phasordia^ am is as shown in the right
°nbifurcated W‘ h current enters terminal Aj and leaves terminals B x and Ch The
(6) ^ wding shown in Fig. 7.39 requires two-plane overhang.
jUjber o f e ^ r c ° n c e n tr ic w inding. For slot-pitch y = 30° and phase spread o = 60°, the
' 8 ereth0 per Puase-belt are 2, therefore, the number of coils in each coil-pronn

Scanned by CamScanner
n p iiiiP ^ c
A ri
i ipo

17 18 19 29 21 22 23 2L - - - S l o t no
i p >i ii i p o c i

B B -A -A C C -B -B
of
phase belts

Second
plone

F i g . 7 . 3 9 . H a .f-c o U » i n « n 6 d i a ^ m fo r Si" 8 ,e -|ayer

I
— "---------------------------------------- A rm a ture W indings 877
r start from slots 5 and 9. For the instant at which th« „ ■ ^ '
,he voltage phasor diagram is the same as that of Fig 7 " g g d,agram of F'g- 7-40 is drawn
Note that the slots allotted to various phases are a s foil,
Phase A : 1 - 6 , 7 - 1 2 , 1 3 -1 8 , 1 9 -2 4 !
Phase B : 5—10, 11—16, 17—22, 23—4
Phase C : 9—14, 15—20, 21—2 , 3—8
For the sake of clarity, the inter-connections ^
pig. 7.40. The bifurcated winding shown in Fig 7 40 ref, C.u'S °f phase C arc "°t shown in
B* <,w requires three-plane overhang

F i , 7 ,0 . spread.

same for all the coils. B w 'i J 'm r t S J ? ' ' windings, the coil-pitch is not the
The single-layer mush windings are sometimes e m p l o y e T t o m a n s ^ ” * H°r i " the COils'

«*» the end connections „ f ? H i ‘ r T connections of the short coil-sides are bent to theleft
endconnections in adjacent S t C0ll'sldes are bent to the right. In other words the
ofSlots per pole per nhn« f ^ “ m °PPoslte directions. In this winding, the number
t°r 24 slots 4 D0L ? , T e 3 ™h°le number- Tbe coil-pitch is always odd. For example
must be odd ii caTbt takT ^ 1°'* PitCh iS 6 slot-Pitches- Si"ce the coil-
fCOl,s are preferred) the v a ™ ° T ChoosinS here a coil-pitch of 5 slot-pitches (chorded
forelarity sake th ’ ■ * Wlndin£ arrangement for the 3-phase is as shown in Fig. 7.41 where
. he inter-connections among phase B coils are not shown. Here y = 30° and for
_ S N e

COHfc
LENGTH

lg_ j 4i ' *
' •S in g le -la y e r m u sh w ind ing d iag ram for 24 slots, 4 poles nnd 60* phase spread

Scanne amScanner
I Prob. 7
878 Electrical Machinery---------------------------------------- ---------------------------------------------------------------------------
. Al U o c o EYir t h p instant shown in Fig. 7.41, the phasor
a = 60°, 2 slots must belong to thesame phase, ror
diagram is the same as in Fig. 7.39.
Note that the method of mush coil inter' < ; ^
sh o rtco T sid e ! “ u " a nand0So°o„8 The slots allotted to the 3-phases are the same
as given in Example 7.15 (6 ).
9

PROBLEMS
7 .1 . (a ) W h a t a re th e tw o g e n e ra l ty p es o f a rm a tu re w in d in g s an d w h e re a re th e s e u sed ?
(6) Explain the follow ing with respect to arm ature w in d ings :
C on d u ctor, tu r n . coil, co il-sid e. s ine U -»n d d o u b lo-lay cr w in d in g s, a d ’ " * "
s in g le -la y e r w in d in g s, p o le-p itch , co il-p itch , fu ll-p itch w in d in g s a n c or e -
. , , , .___. w inH inps so f a r a s t h e ir co n stru ctio n and
7 .2 . (a ) D istin g u is h b etw een sim p lex lap an d sim p lex w ave w in a g
n u m b er o f p a ra lle l p a th s a re concerned.
(6 ) E x p la in th e follow ing w ith regard to co m m u ta to r m a c h in e s .
N u m b erin g sc h e m e , b ack p itch , fro n t p itch , w in d ing p itch an d c o m m u ta to r p itch .
7 .3 . (a ) D istin g u is h betw een p ro g ressive and re tro g re ssiv e sim p lex lap w in d in g s. W h y is th e sim plex lap
w in d ing ca lle d so ?
(b) F o r a co m m u ta to r m a ch in e w ith 4 poles. 16 slo ts an d 2 co il-sid es p e r s lo t, o b ta in th e follow ing :
W in d in g ta b le , w in d ing d iag ram in rad ial form and position o f b ru s h e s on th e c o m m u ta to r In d ica te the
co ils p e rta in in g to th e p a ra lle l p ath s. T h e w in d ing is o f p ro g ressiv e sim p le x la p ty p e. | A n s.yft - 9 , y f - l , y e = I)

7.4. M ake a w inding table and winding diagram for a dc m achine a rm a tu re w ith th e follow ing specifications :

T o ta l n u m b er o f slo ts = 18, n u m b er o f poles = 6


N a tu re o f w in d ing : sim p lex p ro g ressiv e lap . C le a rly ex p la in how th e p o sitio n o f b ru s h e s is fixed and
in d ic a te th e n u m b e r o f p a ra lle l p ath s in you r w in d ing d iag ram .
7 .5 . (a ) W h a t a re s p lit coils ? How can th e se be avoided in co m m u ta to r m a c h in e s ?
(b ) F in d ou t th e s u ita b le v alu es o f b ack p itch for th e sim p le x lap w in d in g w ith n o s p lit co ils, in th e following
tw o ca s e s :
( j) 3 0 slo ts, 9 0 co ils, 6 poles and
(ii) 3 0 slo ts , 1 20 coils, 6 poles. |Ans. (b ) (i ) 31, (») 411

7 .6 . (a) D isc u ss th e m ain fe a tu re s o f dc m a ch in e w ind ings.


(6 ) A 4 -pole d c a rm a tu re w ind ing h av in g 4 0 slo ts an d 1 2 0 coils is to be provid ed w ith a sim p le x -la p winding.
W ork out a s u ita b le a rra n g e m e n t so th a t sp lit w in d ing is n o t used.
7 .7 . ( a ) F o r a sim p le x w ave w ind ing, d eriv e th e follow ing re la tio n s :

C om m utator pitch, yc =

2C ± 2
W in d in g p itch , y„, = f/ ~

(b ) D esign and draw a sim p le x w ave w in d ing w ith th e follow ing d a ta :


4 poles, 15 slots, 2 coil-sides per slot, progressive winding.
In d ic a te th e lo cation o f b ru sh es on th e w in d in g d ia g ra m . Sh ow th e co ils p e r ta in in g to th e p a ra lle l paths
a lso . [A n s. ( b ) y c = 8 ,y „ , = 16, y * = 9. J 7 "

7 .8 . (a ) E x p la in w h a t is m e a n t by a d u m m y coil. W h e n d oes a n a r m a tu r e h a v e a d u m m y coil and w h a t

pu rpose is served by it ?
(6 ) A sta n d a rd a rm a tu re w ith 3 7 slo ts an d 1 4 7 c o m m u ta to r s e g m e n ts is to b e u sed for a 4 -pole winding
w ith 1470 con d u ctors. D isc u ss w hich ty p e o f w in d in g sh o u ld b e em ployed fo r th is a r m a tu r e . T h e n workou
su ita b le a rra n g e m e n t for th is w ind ing.

^ ca n n e cn 5 ^ P a m ?ca n n e r
„ A rm ature W indings 879
prob^Jj—-——' " "
(b) Turns per coil = 5, A ctual coils = 148 Active coils = 147
^ S' y c = 74 or 73, y H, = 148 or 146. y h = 73 and y f = 75 for progressive
winding and y h = 7 3 and y f = 73 for retrogressive winding.|

7 9 , (a) E n u m e ra te th e m a in p o in ts o f d istin c tio n b etw een sim p lex lap an d sim p le x w ave w in d in g s ?
{b) A 20 kW , 2 5 0 V 4-p o le , la p -c o n n e cte d g e n e r a to r h a s 2 8 8 con d u ctors w ith s in g le -tu r n co ils. I f th e

8 t0 ° b ta ,n “ WaVe- " ta d i" e . c a lc u la te i t , e ffe c t on c u r r e n t. ■

" “’ S in t. lb) Wave-winding w ould h a v e one dummy c o i l j |Ans. 10 A 496 53 v le m 2 kw| 71:II
7. 10. (a) W h at a re th e ad van tages o f m ultiplex windings over simplex windings ?
Distinguish b etw een d u p lex an d tr ip le x w in d in g s o f both types. !i|;
(b) When are th e c ir c u la tin g c u r r e n ts p re s e n t in lap -co n n ected a rm a tu re s ? E x p la in th e ca u s e s o f th e ir
existence.
liij
w it h
, . U . to) In th e c a s e o f la p -c o n n e cte d a r m a tu r e s , illu s tr a te th e d e trim e n ta l e ffe c ts o f c ir c u la tin g c u rre n ts
suitable d iag ram s. fa I
lij {
(b) What is an e q u a liz e r rin g ? E x p la in how it perform s its two fu n ction s?

7.!2 (a)i A 2 0 0 kW , 5 0 0 V, 10-p o le d.c. m a c h in e is lap-w ound. F in d th e c u rre n t h an d led by e a c h b ru sh and


each parallel path. '

(W A 4-pole d.c m ach in e h a s g en era ted e m f o f 2 5 0 V. I f th is m achine is re-wound w ith 8 poles, o th er th in g s


remaining unchanged, th e m ag n itu d e o f g en erated e m f rem a in s a t 2 5 0 V. Is the m achine w ave or fep-w ound ’
ill
(c) Work out a s u ita b le a r r a n g e m e n t o f 10 e q u ilia e r rin g s for a com m u tator m a ch in e h a v in g 6 p oles 1 2 0
tjjV'M
ft
* ! ' mPIC,< lap wmd,n|!- IA” S- <»> 80 A, 40 A l b ) lap-wound (c) 4 0 coils, y , „ = 4 io ilsl
7.13. (a) A sim p lex w ave-w ou n d 4-p o le a rm a tu re h a s 11 coils, each o f re s ista n c e 0 .1 i l :
(i) What is th e re s is ta n c e m e a su re d b etw ee n two a d ja ce n t co m m u tato r seg m en ts ?
(ii) If one coil g ets o p en -circu ited or sh o rt-circ u ite d , ex p la in how it can be located.
I{ti^\»
Number o f poles, sp eed o f r o ta tio n , v o lta g e an d pow er ra tin g s a re th e sam e for two m a c h in e s one o f
(b)
!>ii
which is lap-wound an d th e o th e r is w ave-w ound . Sh ow th a t th e am oun t o f copper req u ired for th e a rm a tu re ■
winding of each m ach in e is th e sa m e . a rm a tu re

IHint. (b) S u b scrip ts L an d \V a re u sed to d en o te laD and w ave w indings resp ectiv ely.

W ' 1- X ,3
aid 7 _ 2 Eu 2 7

Area o f c o n d u c to r c ro s s -s e c tio n , A , = — • — = — ■—
L a 5 P 6
Also / I P
Vni
''Omrnp _____ , ■ ... = 2 8 2 L
^ ^ e ° w ppd -for w a v e - w in d in g = A ,„ Z,„ ...| |A n s . (a ) (/) 0.16364 tJ ( it ) 0.9 Q, 0 .0 9 tl|

to) Wh^ nuiT>b e r o f tu r n s per coil a re lim ited in case o f co m m u ta to r m a ch in e s.


U n 31 Wou^ h ap p en i f th e s p a c e o f d u m m y coil is le ft unoccupied ? ,
'*■* vOmrjaj-A j *.
i 715 f a d iscu ss th e u se o f s im p le x -la p an d sim p lex-w av e w ind ings. .
to)Un° We ac*v a n ta g e s o f d o u b le -la y e r w in d in g s over sin g le -la y er w ind ings,
i n^t o ♦Vv
j ij (c) Desc ^ e s s e n tia l co n d itio n s t h a t p o ly p h a se w ind ings m u st p o ssess ?
e*p of emf polygo^3 ^ te r m s p h a s e -b e lt an d p h a se -sp re a d as applied to a .c. a rm a tu re w in d in g s, w ith th e

°*c°ntains S a tr ^n te ET a ^ s l ° t w in d in g s ? Sh o w th a t for fu ll-p itch double la y e r in te g ra l s lo t w in d in g , ea ch


■ ; 4j S1 es b e lo n g in g to th e s a m e p h a se .
^®Se Voltarro^ 3 P ^a s e s p re a d o f 6 0 ° p ro d u ces a se q u e n ce o f A , - C, B , - A , C , - B in th e p h a se -b e lts, for

« ABC- ■
63'Phase vo\t P^ a se s P r e a d o f 1 2 0 ° p ro d u ces a seq u e n ce o f A B C in th e p h a se -b e lts for A B C p h a se seq u e n ce
/ •" *•

ScanneaDv CamScanner
880 E le ctrica l M a c h i n e r y ___________________________________________________ -— -------------------------------------- ^

7 .1 7 . (a) W h a t a r e t h e m e r it s o f u s in g c h o r d e d c o ils ?
(6 ) A 4 8 - s lo t s t a t o r is to be w o u n d f o r 4 -p o le s , 3 -p h a s e , d o u b le la y e r w i n d i n g a n d 6 0 " p h a s e s p re a d . D raw

a c lo c k d ia g r a m to s h o w t h e p o s it io n o f t h e p h a s e - b e lts in e a c h la y e r f o r ( i ) f u l l - p i t c h c o ils a n d (ii) ^ po le -p itch

c o ils .
S h o w t h e in t e r c o n n e c t io n s a m o n g s t t h e p h a s e - b e lts o f a n y p h a s e i n d i c a t i n g s e r ie s a n d p a r a lle l schem es
7 .1 8 . ( a ) W h a t is f r a c t io n a l - s lo t w i n d i n g ? W h y s h o u ld it s a r m a t u r e s lo ts be d i v is ib le b y 3 i n ca se o f 3-phase
m a c h in e s ?
( b ) G iv e t h e m e r i t s o f f r a c t io n a l- s lo t w in d in g s . A r e t h e r e a n y r e s t r ic t io n s o n t h i s w i n d i n g ?
(c ) W h a t is c h a r a c t e r i s t i c r a t io o f f r a c t io n a l- s lo t w in d in g s a n d w h a t d o e s i t s ig n if y ?
i d ) D e s ig n a n d d r a w a d o u b le la y e r w i n d i n g f o r 4 2 s lo ts , 10 p o le s , 3 p h a s e s a n d 6 0 p h a s e s p r e a d . In d ic a te
t h e s t a r t o f e a c h p h a s e . S h o w t h e in t e r - c o n n e c t io n a m o n g s t t h e c o ils o f a n y o n e p h a s e .
7 .1 9 . ( a ) E x p l a i n w h e t h e r c h o r d e d c o ils a n d f r a c t io n a l- s lo t w in d in g s a r e p o s s ib le in a s in g le - la y e r w in d in g .
( b ) D e s ig n a n d d r a w a d o u b le - la y e r w i n d i n g w i t h 9 0 s lo ts , 8 p o le s , 3 p h a s e s a n d 6 0 ° p h a s e s p re a d . In d ic a te
t h e s t a r t o f e a c h p h a s e . S h o w t h e in te r - c o n n e c t io n a m o n g s t t h e c o ils o f a n y o n e p h a s e .

7 .2 0 . J u s t i f y t h e f o llo w in g s t a t e m e n t s :
(a) A p h a s e s p r e a d o f 6 0 ° is m o r e c o m m o n ly u s e d t h a n a p h a s e s p r e a d o f 1 2 0 ° in 3 - p h a s e m a c h in e s .
( b ) T h e w i n d i n g s a r e u s u a l ly s h o r t- p it c h e d .
(c ) F r a c t io n a l - s lo t w in d in g s a r e q u it e c o m m o n in 3 - p h a s e a c m a c h in e s .
( d ) F r a c t io n a l - s lo t w i n d i n g is n o t p o s s ib le i f /% , in t h e c h a r a c t e r is t ic r a t i o S k/ P * , is a m u l t i p l e o f th re e .

7 .2 1 . (a) P re p a r e a w in d in g ta b le f o r th e a r m a tu r e o f a 3 -p h a s e m a c h in e w i t h t h e fo llo w in g s p e c ific a tio n s :


3 6 s lo t s , d o u b le - la y e r w i n d i n g , 4 p o le s , n a r r o w - s p r e a d a n d c o il- s p a n = p o le - p it c h .
In d ic a te th e u p p er co il-sid e by n an d low er co il-sid e by n ' fo r n th s lo t (1 < n < 3 6 ).
( b ) D r a w t h e d e t a ile d w i n d i n g d ia g r a m f o r o n e p h a s e o n ly c le a r ly s h o w in g t h e c o n n e c tio n o f d if f e r e n t coil
g ro u p s .
(c ) S h o w t h e s t a r o f c o il- e m fs . D r a w p h a s o r d ia g r a m f o r n a r r o w - s p r e a d c o n n e c tio n s o f t h e 3 -p h a s e w in d in g
s h o w in g c o il e m fs f o r p h a s e s A a n d C o n ly .
7 . 2 2 . ( a ) M a k e a w i n d i n g t a b le f o r t h e a r m a t u r e o f a 3 - p h a s e m a c h in e h a v in g t h e f o ll o w i n g d a ta :
5 4 s lo ts , 8 p o le s , n a rro w - s p r e a d , c o il-s p a n = 6.

N a m e t h e u p p e r c o il- s id e a s n a n d lo w e r c o il- s id e as n ' f o r t h e n t h s lo t.

(b ) D r a w t h e c o i l- e m f s t a r f o r p h a s e A o n ly .

7 . 2 3 . ( a ) D is tin g u is h b etw een sin g le -la y e r co n c e n tric an d m u sh w in d in g s.

( b ) W h y a r e th e co n c e n tric w in d ing s so ca lle d ? D is tin g u is h b etw ee n h a lf-co ile d a n d w h ole-coiled arm ature
w in d in g s.
( c ) F o r th e s a m e p h a s e v o ltag e, how m a n y tu rn s p er coil a r e th e r e for th e w h o le-co iled w in d in g w ith respect
to th e n u m b e r in a h a lf-co iled w inding.
( d ) D e fin e a co il-grou p . How a re th e co ils in su ch a g roup co n n e cte d ?
(e) E x p lain how th e n u m ber o f coils in a coil-group a re determ ined for half-coil a s w ell as whole-coil windings .
(/) E x p la in how th e su c c e ssiv e co il-g rou p s a r e co n n e cte d in th e u n b ifu rc a te d a n d b ifu rc a te d w indings.
[A n s . ( c ) N u m b e r o f coils in w hole-coil w in d in g a r e d o u b le o f t h a t in th e h a lf-c o il w in d ing . T h erefo re, for
th e s a m e v o lta g e and co n se q u e n tly for th e s a m e p h a se tu r n s , th e n u m b e r o f tu r n s per coil in a
w h o le-co il w in d in g is h a l f th e n u m b e r o f tu rn s in a h a lf-c o il w in d in g .!
7 .2 4 . ( a ) E x p la in how th e m u sh w in d in g is fitte d in th e a r m a t u r e s lo ts .
( b ) W h a t is th e s im ila r ity b etw ee n w h o le-co il an d m u sh w in d in g s.

( c ) D e sig n a n d d raw a m u sh w in d in g fo r a 3 -p h n se m a c h in e w ith 3 6 s lo ts , 4 p o le s, 6 0 ° p h a s e spread .

canned by Cam scanner


-"wak

O B J E C T IV E T Y P E Q U E S T IO N S
The multiple choice questions given in this appendix pertain to f t0pics
and in particular, to the electrical-machinery topics covered in this . q ns have been
taken from competitive examinations like GATE, IES, IAS etc.
Out of the various alternatives given, choose the most appropriate answer
ELECTRICAL ENGINEERING
1. The resistance between the opposite faces of 1 metre-cube is found to be 1 £2. If its length is
increased to 2 metres, its volume remaining unchanged, then resistance between the opposite
faces of its length is
(a) 2 £2 (tij 4 Q (c) 1 ft (d) 8 0.
2. Time constant for an RL series circuit is given by
(a) R /L (b) 1/R L (c) RL {&) L /R
3. Time constant for series RL circuit is defined as the time taken by the current to reach
(a) 36.8% of its final value (6) 36.8% of its initial value
« f 63.2% of its final value (d) 63.2% of its initial value
4. Time constant for RC series circuit is equal to
(a) 1/R C (b) R /C (c) C /R (8 ) RC
5. A series RC circuit is suddenly connected to a dc voltage of V volts. The current in the series
circuit, just after the switch is closed is equal to
(a) zero (6) V/R C (c) VC/R (d) V /R
6. A series RL circuit is suddenly connected to a dc voltage source of V volts. The current in this
circuit, soon after the switch is closed, is equal to
(a) zero (6) V /L (c) V /R (d) V L /R
7. A series LC circuit is suddenly connected to a dc voltage source of V volts. The current in this
circuit at t = 0+, is equal to
in) zero (6) V /L (c) V /C (d) V L /C
(ij.
8. In the network shown in Fig. C .l, the current through 11 Q resistor is '
1 ,)
(6) I A (c )f A (d )| A

90 12a

v t^ Fig. C.2.

9. In the network shown in Fig. C.2, the current through 4 Q resistor is


(a) 2 A (6) 1 A (c) | A (d) 1.5 A

10. In the circuit of Fig. C.3, the potential of terminal


(a) B is higher than A by 5 V (b) A is higher than B by 5 V
(c) B is higher than A by 13 V 0 ) A is higher than B by 13 V

: m m

Scanh e d ’ b’y 'C amS c a nner


* 2 0 V

A V \V
H i- io k y ~ v 5K
/ * v
■3H ' 5£;

4V ■=k
■i/1
A* 1— -\\\v-

Fig, C.3 Fig. C.4

11. The current through the galvanometer G shown in Fig. C.4 is zero. The current / 5 throush the
source is
(a) 1 mA (6) 2 m A (c) 3 m A (c/) 4 m A
12. The time constant of the network shown in Fig. C .5 is ^
(a) 2 RC (b) 3 RC (c) R C /2 '{ d f 2 R C /3
13. In the circuit shown in Fig. C.6 , the current through R L is
(a) 2 A (6) zero (e) - 2 A (cf) - 6 A
'.A
' ' -itr ' -•,‘V °

120
n

H^iov
420 v — 420 n

L
Fig. C.5. Fig. C.6.

14. The Thevenin’s impedance across the terminals AB of the network o f Fi^ C 7 is
« o fn ',n "
4
15. In the bridge given in Fig. C .8 , reading of the.high-impedance voltmeter is

~*T
V

Fig. C.8.
Fig. C.9.
(a) zero (6 ) 6 .6 6 V (c) 4.20 V ,33 V
16. Resistance between terminals A and B of Fig. C.9 is

(a) ~ R w | a id )R
5
17. Resistance between terminals A and B of Fig. C .1Q is

« ■ >5! *" « ! * W ! S (d )R
.
*>20 Electrical
^ M ii

R
—AAty\,—
A I R R I R R R
o - -* ■ -A W V — A \A V ■ - i - -A A A V ~ -oB A
o— -WV- -V W - -VVw—*—5
I .-AWV--------
R
Fig. C . l l .
Fig. C.10.

18. Resistance between terminuls A and B of Fig. C .ll R


IK
WW
R R R
(a) | R o— —AMA- — MW— <
(b)
A

(O R (d )\ R MW--------- — __
R
19. Resistance between terminals A and B of Fig. C. 12 Fig. C.12.
is
<6 > f « (c) R
< « > !*

20. The effective resistance between points A and B of Fig. C. 13 is


(o) 9 n (6) 12 n (c) 18 £i (d) 24 n

Fig. C.13. Fig. C.14.

21. The voltage VC1, vc2 and V^.3 across the capacitors in the circuit shown in Fig. C.14, under
steady-state, are respectively
(a) 80 V, 32 V, 48 V (6) 80 V, 48 V, 32 V
(c) 20 V, 8 V, 12 V (d) 20 V, 12 V, 8 V
22. Four resistances 80 £1, 50 £1, 25 £2 and R are connected in parallel. Current through 25 £1 resis-
£ tance is 4 A. Total current of the supply is 10 A. The value of R will be
(a) 66.66 Q (b) 40.25 Q Y ) 36.36 £1 (d) 76.56 £2
23. W1 n a resistor/? is connected to a current source, it consumes a power of 18 W. When the same
;i-o / | v b t M R is connected to a voltage source having the same magnitude
as the current source, the power absorbed by R is 4.5 W. The
magnitude of the current source and the value of R are
(a) VlfT A and 1 Q \U>T'3 A and 2 £1
(c) 1 A and 18 £2 (d) 6 A and 0.5 Q
24. In the circuit shown in Fig. C.15, the switch S is opened at
I = 0. Prior to that, switch was closed.
Current i (t) at t = 0+ is
Fig. C.15,

k !! A (6 ) f A (c) | a (d) 1 A

/ •- -• V .

Scanned by CarnScanner
htSsSa

921

{R jn Fig- C.16 *s adjusted so that power developed by the voltage source is zero.

i®12 ! value of R >s q (e) 6 Q (d) 0.667 £i

10)1,0 611
- 0 -

± -3 V

Fig. C.16.
n 17 the current through the ammeter is
j M n r w - 0 '1' ’ ______
(b) OA
<^7 4 A
C d lf^ A
(c)?A
9 • • f
, .nifhrm coil of inductance L henries and associated resistance R ohms is physically cut
27'•^w ^oexacth alves which are than rewound in parallel. The resistance and inductance of the

R a t i o n are (6) 2 B a n d 2 L

R aL
(c) — and —
'• 2 ““ “ 2
»t The current flowing through the resistor RL in the given circuit of Fig. C.18 is
10) 1 A (6, zero (c )3 A (d) 5

Fig. C.19.

Flg . _ ia nf o nhms each. The switch is initially open. What


29. All resistances in the circuit in Fig. C.19 ciOSed ?
happens to the lamp’s intensity when the swi •
(a)increases 2 ! " d e p e n d s on the value of K. [GATE, 19921
(c) remains the same W a resistor R is 1W when only source ‘1* is
30. In the circuit of Fig. C.20, the power dissip a)wer dissipated in the same resistor R is 4W
present and ‘2’ is replaced by short aircuit i p» ^ when both the sources <!
when only source ‘2’ is present and 1 is P
and ‘2’ are present, the power dissipate m J $ , f v4
m w («3W I

«• Under^steady-state conditions ; t h e t o t ^ L r d ic t a t e d , the energy stored in the inductor and

w S w S W . 1 2 .5 W s .2 5 W s
« 50 W, 25 Ws 25 W s H> « » W . 12 5 W s>25 W s

Scanned by Cam Scanner


ht

922

in 2n
-VWv- -VvVv-

ft . Source
■- -t
jtol V\ jta)t
|V,|® rce »R=in 2 Vjje
Volts
(u
O'.Volts

Fig. C.20. !L
32. A 0 - 10 mA PMMC ammeter reads 4 mA in a circuit. Its bottom control spring snaps suddenly,
The meter will now read nearly
(a) 10 mA (bJJJ mA
(c) 2 mA 4 <f)zero
33. The insulation resistance of a cable of length of 10 kM is 10 MQ. For a length of 100 ItM of the
same cable, the insulation resistance will be „V- M , ItjkUOVj
(a)^L00 MQ (ft) 10 MQ b O v. Tw 'r-!
\J n 1 MO (d) 0.1 MQ
c R
34. An ideal voltage source will charge an ideal capacitor Hh -AW .
(a)^in infinite time (6) exponentially
instantaneously (d) none of the above
35. In the series circuit shown in Fig. C.22, voltage across C starts increas­
ing when the dc source is switched on. The rate of increase of voltage
across C at the instant just after the switch is closed (i.e. at t - 0+) will IV
be
Fig. C.22,
(a) zero (b) infinity
(c) RC . ^ i/ rc
36. If v, w, q stand for voltage, energy and charge, then v can be expressed as
(a) v = ~R~
dw dq
, \ j dw
(c) du= — (<d) dv =
dq dw
37. A rectangular pulse of duration T and magnitude I has the Laplace transform
(a )- (b) - ■e~T s
s yS

yd) - [1 - exp ( - T ■s)l n


%>
a voltage waveform - (()= 1 2 (2 is applied across a 1 H inductor for i > 0, with initial current
through it being zero. The current through the inductor for.* S 0 is given bv
(a) 12 t (fe) 24 t
(c) 12 r Vv

39. A current input, 5 5 (t), is forced through a capacitor C. The -1


voltage vc (t), across the capacitor is given by
(°) 5 f (6) 5 g(0 .C

(«)§# Xd) p(i)

40. A current of the waveform shown in Fig. C.23 passes through


a pure inductance of 3 mH. The instantaneous power, in watts
during 0 < t < 2 ms, is ' '
(a) .25,000 t (6) 50,000 t
(c) 75,000 t (d) 100,000 /

v.: v ‘S'

Scanned by Cam Scanner


http'

di* —— ~ —"------- — — (^23


voilmeUsr i. c o n n e c t aero,, the . o i l , * , source ~---------------
it. ^ y It) * 5 + 10 co* (314 /t 4.
v (0 '40°.
+ 30°) untaneous value is
of the meter i*
(6 ) 5 V
1,1,0 T s v
S^v id) V75 V
a 9 capacitor is charged by a square-wave current nource, the voltage across th
4 >square wave ^A'b) triangular wave across the capacitor is
jjj step function id) zero
„ The current passing through a 10-ohm resistor of Fig. C.24 la) has th«. t •
^ % (^ T h e reading of the PMMC voltmeter connected across the w i Z r t ™ ^ “ F^

(“j 140 V id) 120 V

Kt)
18 A
i(t) 10^
-i-'W W V -
y 6A

PMMC
tiT;
(a) (b)
Fig. C.24.
44. A voltage source with an internal resistance Rs supplies power to load RL. The power delivered
to the load varies with RL as

Pi Pi

(o) (d)

45 A *
• unity feedback system with open-loop transfer function
fr-Ju»
G (s )= 9
8 (8 + 3)
has natural frequency = 9
‘ natural frequency = 3
6. damping ratio = 1 /2
4- damping ratio = 1 /6 v
Proi
111these, the correct answer is
<*) 1, 3 ltd 2 , 3
(c) 1, 4
(d) 2, 4 10A 50V
in .
volta6 Clrcuit ° f Fig. C.25, the inductor current and capacitor
Curage ? re given at a time instant t. A t this instant, the
(ai i an<* d i/d t are respectively
! : A. - 5 0 0 A / , (8 ) - 5 A , - 5 0 0 A /s
tcl 5 A, soo A /s Fig. C.25.
(d) - 5 A, 500 A /s . 1 f

Scanned by Cam Scanner


924 Electrical Ma

47. Match the waveforms on the left hand side with the correct mathematical description listed
on
the right-hand side :
W aveform s fit)
(P)t ■u ( t - 1)
(Q) it + 1) ■u it 1)
(R) t ■u (t)
iS) it + 1) •u (t)
(T) ( t - 1) u (t)
( U ) i t - l ) u (t - 1)

j\
viAA]“
0 -A
■A'l J.} * 0

5
0

\ \ i VN
I
48. The rms value of the periodic wave e (t) shown in Fig
C.26 is
u
(a)
v r- ib)
3
/ /
T --------- >
T t
iC) VV gT A. y[2 A
■• •» -
2

49. The output of a linear system for a unit step input iis
given by t2 - e *. The transfer function is given by Fig. C.26.

ia) ib) — 2g „ ( c ) -------- — ^ id)


is + l ) d (s + 1) s(s+ I f s ( s + 1)2
5 ° . A second order system with no zeros has it poles located at - 3 + j 4 and - 3 - 7 4 in the s-plane.
The undamped natural frequency and damping factor of the system are
(a) 4 rad/s and 0.75 (&) 3 rad /s and 0.60 Y
(c) 5 rad /s and 0.80 (d) 5 rad /s and 0.60

51* I c r o s s X ^ t e n n f th!e CUrre^ COil ° f a wattmeter is * = 1 + 2 sin at amperes and the voltage
(a) 8 00 V sin ^ volts. The wattmeter reading in watts would be
ro T, V . , ( 6 ) 5 ’05 ^ 2 -00 id) 0.00
° ’ actobs 2 n ’ 5 7a nnd 10 K are connected in series and a 20 V dc supply is applied
r> H I*.i i 1 Odbhmllon, A voltmeter of internal resistance 10 i f reads Vi volts across the
10 it ruMiHtor. o fth
onintor. Which of the* following statements
*& & & £ < «= true
is h, ?
W y ^
<") v , < & (d) v x > ~ ^ : ; . : . ;

Scanned by Cam Scanner


https :


— rCuit below resonant frequency is
x ^es ROC C1 (fa) capacitive (c) inductive (d) none of these
$S,AfreSisdV® .rcuit above resonant frequency is
er»eS • (b)Jcapacitive
(0 v(c) inductive
7 ----- —— *w (d) none ui
of i/iiese
these
(1) *eS^ ' ,eA'&oram for alternating quantities can be drawn if they have
hPphaS°r t (6) rectangular
(b) rectangularwaves
waves
tine waves (d) triangular waves
•'l
(c) isq treWaves • i off
nre is the reciprocal
(6) capacitive reactance
if
i-°:”id
„ d i ' ' ereac“ nce (d) impedance

'‘'"ttancci8 the reciprocal of


(6) capacitive reactance
S ) - ^ ’d„rtive_reaC“ nCe (id) impedance
(c) reSlS pcses resistance # and inductive reactance X in series. Its susceptance is given by
58.Aci^ 2P°SS (6) X /Z (c) R /Z 2 {d )R /Z
(0,X/ mih and I = c + jd , then the power is given by
59. If +J, <b)ac + bd (c ) b c - b d (d) be + ad
rt r series circuit, current at series resonance is
60. For an n iaffmng pf (b) maximum at leading pf
r! l £ Z S S y pf «*> — » at unity pf
. .n RLC parallel circuit, current at parallel resonance is
(1 for an KU. P f {b) maximum at lagging pf
<»> t unto Pf (#) - W " * ™ at pf
(e) ">axim unsymmetrical alternating quantity should be calculated over
* Mhalf cycle (M one cycle (c) two cycles (d) complete number of cycles
S3. A permanent-magnet moving coil irm trom ent^m easure

(a) ac only d pulsating quantities only


(c) dc only K Jv
64, A moving iron instrument can m e a s u r e ^ ^ ^ ^
(«) ac only (rf) pulgating quantities only

65.AdyLmmineter type of i n s t r u m e n t ac
W« ,rf) pulsating quantities only

US. Rectifier type instrument can measure ^ ^ ^ ^


g ac only Jd Jpulsating quantities only

« . An electrostatic type of instrument to used for measuring


(a) voltage in dc circuit
{b) current in dc circuit
(c) voltage in both dcand ac circuits
id) current in both dcand ac circuits
* Tfe internal resistance of milliammeter must be 1 ^ marimomvottage d r o p across the m
(®) higher accuracy . . . nce (d) high sensitivity
(c) minimum effect on the circuit rests ^ it possesses
69‘ A voltmeter must have very high into ltage range
(«> high accuracy * current through the meter.
(c) maximum loading effect i > .
7°. A material whose resistivity is 0.4 G (d) semiconductor
*«) conductor (6 ) resistor « «*u lator

Scanned by Cam Scanner


925
53. A series RLC circuit below resonant frequency is
(fl) resistive (6) capacitive (c) inductive
(d) none of these
54. A series RLC circuit above resonant frequency is
(a) resistive (6) capacitive (c) inductive {d) none of these

55' S coP" r m ^ can he drawn if t h e y L e


(c) square waves , ectanKular waves
(d) triangular waves
56. Conductance is the reciprocal of
(a)\ tnrlilftfcive
tn inductive ri'flptani'o
reactance ,
(c) resistance Co) capacitive reactance
(d) impedance
57. Admittance is the reciprocal of
(a) inductive reactance
(c) resistance (b) capacitive reactance
iiiipuuancc
(d)impedance
vv*,
58. A circuit possesses resistance Itand inductive reactance X
(a )X /Z a (6 ) X / Z (C)J?/Z2 cLanct^ in ^ 8eneu. its susceptance is given by
(d) I i /Z
59. If a +jb and / = c + jd , then the power is given
(a) ac + ad (6) ac + bd L .
60. For an RLC series circuit current nt cm • +
(a) maximum at lagging pf n c® resonance is
(c) maximum at unity pf . .. maximum at leading pf

« . For an RLC paraliei circuit, c u r r e n t “ “ Pf


(a) maximum at leading pf parallel resonance is
(c) maximum at unity pf , 2 max.lmum at lagging pf

62. The average value of u n s y m m e tr ij^ 3 “ ^ Pf


(«) half cycle (6 ) 0ne cycle (c) Sh° 7 ld be ™lculated over
63. A permanent-magnet moving coil instrument complete number of cycles
(a) ac only ument can measure
(c) dc only ^ both dc and ac

64‘ U 7 Z T inst~ - » ~ ulsatine quantities only


(c) dc only hoth dc and ac
65. A dynamometer tvne of i . W pulsatinS quantities only
(а) ac only W ° f ,nstr>™ent can measure
(c) dc only both dc and ac
66. Rectifier tvne (d) pulsatinR quantities only
(б) ac only^ mStrUment « > " “ * * * « « ‘y
(c) dc only both dc and ac
67. An electrosbiHe e• W puIsatin8 quantities only
(a) voltage in dc circuit “ ' StrUmcmt is used for measuring
to) currant in dc circuit
S ™ ltage in both dc and ac circuits

68 Th m b° th dC and 3C CirCuits

■w ' u S r S S S S T ° f mi" iammCter muat for

69. l l t i T ^ °n ^ CirCUit reSiStanCe « S K S S S T dr0P aWOaa tba meter

to) high a c c ^ ^ Ve,T WBh i“ et tan“ 80 ‘ hat k •>“ “ —


tolm u m ^ ^ to) ,

i Z * T \Wh0Se r iStivity is 0 4 n metre m ust be a


) conductor (6 ) resistor (c) insulator (d) semiconductor

‘S canned by Cam Scanner


n
Electrical
926

71. The function of a shunt used in the milhme


(a) extend the range and increase the me er
(b) extend the range and reduce the rests an
(c) decrease both the range and the me r
(d) decrease the range but increase the rests ^ choke is to
72. In a fluorescent tube circuit, the function £ 0£^jje cirCuit
(a) prevent flicker ‘® P^ e momentary h.v. to establish the main arc
(c) suppress the radio interference(rf) P checked because it has
73. A VTVM has negligible loading effect on the ctrc
(а) high resistance range
(б) low current range of 50 |iA or less
(c) high input resistance of 11 MQ or more
(d) low input resistance , . , ___. ,
74 A voltage source V with internal impedance R + j X is driving sue a oa ave maximum
power transfer. Power transfer to load is given by
V2
(a)
Yl
R (6) 2R

(c) (d) R
4R R +X M'
75. For a sinusoidal waveform, the ratio of average to rms value is '( *
a/2" •K
/ ^ 2 (b) n (d)
(0) i n (c
- )2^2
zvz 2
76. In the circuit shown in Fig. C.27, voltage V0
(a) leads current I by 90° (6) lags current I by 90°
(c) is in phase with / (of) leads V by some angle less than 90°
77. In the circuit of Fig. C.28, voltage Va
(а) leads current I by 90°
(б) lags current I by 90°
(c) lags current I by some angle less than 90°
(of) leads current I by some angle less than 90°
R
-Amr-

-*r 0

Fig. C.27. Fig. C.28.


78. For an ac circuit, v = Vm sin (tot + a) and i * Im sin (tot + p). The phase angle between voltage and
current waveforms is
(a) a + p or a - p (6 )a -p o rp -a
(c) P - a or p + a (d)a-p
79. In the figure shown below, <|>= power-factor angle, W = watts, V A = volt ampere and VAr-
volt-ampere reactive for an ac circuit. Tick mark the correct figure.

VAr VAr
.?

it

■vys-:; . vY* m
V>t'.
Scanned by Cam Scanner
1 ) (. „ 1« ' e l !
f ' J ’4 M , h„ , 1
: 2 Hi -A ’3-c1 ,
■' ^ ‘
<?
^
*
"-> a> 927

- ' ^ V , 100 W bulb has resistance fiA and 230 V, 200 W bulb has resistance R„. Here
0'A „ -,R„ 2. Rb > K a 3. Ra = 2 R b 4. R b„ =* 2 « , s d _ <c
these the correct answer is
W* i 3 ’ (6) 2, 4 (c) 1, 5 (d) 1 only
Jf a
0
In Fig- c -29’ the bulb Wil1 t*s
(6) not light
(d) light at regular intervals
(c )^ v®tflickering lig M r
<f v-
c

" 0>
n
-=-23° Bulb
R L
■AWV —TTQIT-

Fig. C.29. Fig. C.30.


82i At resonance, reading of voltmeter X in Fig. C.30 will be equal to

(o) V (&) 2 V (c) — V (d) zero

83. A coil of resistance R and inductance L is connected in parallel with a capacitor C across an ac
supply. At resonance,
1. the impedance of the circuit is maximum
2. the susceptance of the circuit is zero
3. the circuit behaves like a pure resistance of value R O'
4. the circuit behaves like a pure resistance of value L /R
Fromthese, the correct answer is
la) 2,4 (6) 1, 2 (C) 2, 3 (d) 1, 2, 4
84. The current in the circuit of Fig. C.31 is
(a) 5 A (6) 10 A (c) 15 A (d) 50 A

-j2 n
20 0

200V j§ zl=io^8
250Hz

'1
Fig. C.31. / Fig. C.32.
85. The
- impedance seen by the source in the circuit of Fig. C.32 is
ta) 14.54 - j 1.687) Q (6) (4.54 + j 2.313) Q
(c) (142.56 + j 78) Q (d) (142.56 - j 82) ft
If the ac voltage wave is corrupted with an arbitrary number of harmonics, then the overall
voltage waveform differs from its fundamental-frequency component in terms of
(Q) only the peak values (6 ) only the rms values
(C) only the average values (d) all the three measures
a*d V° lta?e Phasor of a circuit is 10 ^15° V and the current phasor ip 2 Z - 45° A. The active
Powers in the circuit are
(c 9A^[,and 17 32 vAr (6) 5 W and 8.66 VAr
i Jzuo W and 60 VAr (d) 2 0 V 2 W and 10 V2 VAr
Electrical Mad

928R _ ___ —
— d by the 5 n resistor is 10 W, then
88. In the circuit shown in Fig. C.33, if the power co
factor of the circuit is (d) zero
(a) 0.8 (6) 0.6 (<0 °-5 , an inductor as shown in Fig. C.34. At
89. A part of a circuit consists of a resistor, a rins value of the current through the
steady state, (t) =? 10 sin t and Vi (.t) - c
capacitor is ^
(d) 15 V2* A
(6) VI25 A
R
® ~

C
<•----- (M)- Hh

-(A 3)-------- ( ~ ) —
v(t)=100 sinoat
Fig. C.35
t^■ - Fig C.34.
^ Fig. C.33. g A and 13 A respectively,
, Jy 90. In Fig. C.3S ; A ^ a n d A g are ideal ammeters. If A , and 3 .
\ ' 1.*_Aof tirill
reading A% will be V
.

(a) 8 A (6) 12 A (c) 1® A _ , w are specified. V


(1
d) maewjiuiinatc
[a) indeterminate unless the actual values
—--------- , ^
91. In a series RLC circuit excited by a voltage = E sin <*, where LC < ^ .

(a) current lags the applied voltage


(b) current leads the applied voltage
(c) current is in phase with the applied voltage
(d)
(d) voltage across L
voltage across and C
L and C are equal
9 voltage and variable frequency source
92- S T c I d . Match lis t I with lis t II and select the e n a c t answer
using the codes given below the lists :
List I U stU

A. AA 1- current

BB 2. impedance
B.
CC 3. capacitive reactance
C.
DD 4. net reactance
D.
5. inductive reactance

A B C D
Codes
2 1 3 5
(a)

1 2 3 5
(b)
(c) 1 2 3 4

Id) 1 2 4 3

Vv/UI I IxJV/UI II IV^rI i !fWWBgWBMai


having an internal resistance of 20 Afl, is con-
20 HI resistors ucross a 200 V, 3-phase
OS- Fig. C.37.The reading shown by the
n8C. 8 as shown in
n',ItmetervWl11 the connections are similar to Scott-connection
(a) 115,5 v aS the connection are similar to Scott-connection

S& Y
i73 2 V
e the parallel circuit of Fig. C.38 constituted by
(<i>) 1
Altt !^S°n0^ed coil and a capacitor behaves like Fig. C.37.
an open circuit
^ aI«hort circuit
(/,) 8 nur^ resistor of valued
5}a pure resistor of value higher than R

Fig. C.38.
i
Q, ln the circuit of Fig. C.39, currents I x and I are respectively
16 A, 20 A (6) 32 A, 44 A ?z-
(e) 32 a! 34.2 A id) 16 A, 28 A
9t. A series RLC circuit has R = 10 £2, L = 0.01 H and C = 100 (|F. The Q factor of the circc
resonance is ,< [ \
44)1-0 W 0-316 K ! ; .-,„ v
(c) 0.1 • (d) 10 l-*.
97. Two loads of 10 kW each, are operating at a power factor 0.8 lagging (each). What is
combined power factor ? i.
(c) 0.4 lag (&) °-64 laS
(c) 0.8 lag (d) 1.00
98. Tw loads, one of 20 kW at pf 0.8 lag and other of 12 kW at f t lag, are connected in s,
Their combined power factor would be b , «.
(o)0.8 lag (6) 0.6 lag 7r % A '■*'* » *
(c) 0.7 lag (d) 0.711 lag
99. A water boiler at home is switched on to the ac mains supplying power at 230 V /5 0 Hz
frequency of the instantaneous power consumed by the boiler is
(a)OHz (6) 50 Hz
(<0 100 Hz (d) 150 Hz
I06-A constant voltage but variable frequency ac source feeds A
Jand C in parallel as shown in Fig. C.40. The impedance seen by KJ
«>e source is Z.
1■2 is zero when f = 0
•Z is zero when f = infinity
Fig. C.40.
•Z is infinite when f ~ 0
■ " is infinite when f —infinity tuVT“
resonant frequency, (Or -

O
-U

Scanned by Cam Scanner


930 Electrical Ms

80 a
-v \ w -
7. 0),
\J 20/1 250
From above, the correct answer is -------" A -'A -i -----
(a) 1, 2, 5 {b) 3, 4, 5 ^s.Ot ■ | - D
lc) 1, 2, 6 (d) 3 ,4 ,7 ^ ,5 ''U ') .
= _ 100V <
101. In the network of Fig. C.41, the current in the 25 12 resistof- F 10n
will be
____ __________ <----------------- - ..

(c)) 5 2.5
A A (5
(d)) 4 zero
A (I.A.i>., 1993)
, Fig. C.41.
102. Match List I withList IIand select thecorrect answer (o ■
using the codes givenbelowthe lists : V •

List I Ml'-•
(Transient current response)
X
(Condition)
« =0 1. Undamped oscillations
A.
B. R <2 XL/C 2. Damped oscillations

3. Critically damped response M ^ ’yl


C. R = 2 XLTC
D. R >2 VL/C 4. Non-oscillatory response

.\k'
j:

Codes : A B c D

(a) 1 2 3 4 S V
r* -O 1 4 3 2
(6)
(c) 3 2 1 4
V"
(d) 3 4 1 2 [I.A.S., 19931

ANSW ERS 0
1. (6) 2. (d) 3. (c) 4. (d) 5. (d)
6. (a) 7. (a) 8. (a) 9. (6) 10. (d)
11. (c) 12. (d) 13. (c) 14. (d) 15. {d)
16. (6) 17. (d) 18. (6) 19. (a) 2 0 . (a)
21. (6) 22. (c) 23. (6) 2 4 . (a) 25. (b)
26. (a) 27. (d) 28. (6) 29. (c) 30. (a)
31. (6) 32. (d) 33. (c) 34. (c) 35. (d)
36. (b) 37. (d) 38. (d) 39. (d) 40. (c)
41. (d) 42. (6) 43. (c) 44. (c) 45. (6)
46. (a) 47. A -> R , B - > U , C - > S , D - > P 48. (6) 49. (b)
50. (d) 51. (c) 52. (d) 53. (6) 54. (c)
55. (a) 56. (c) 57. (d) 58. (a) 59. (6)
60. (c) 61. (d) 62. (6) 63. (c) 64. (6)
65. (6) 66. (a) 67. (c) 68. (c) 69. (d)
70. (d) 71. (6) 72. (d) 73. (c) 74. (c)
75. (6) 76. (6) 77. (a) 78. (6) 79. {d)
80. (a) 81. (b) 82. (d) 83. (b) 84. (a)
85. (a) 86. (d) 87. (a) 88. (b) 89. (a)
90. (6) 91. (6) 92. (c) 93. (c) _94. (d) __
95. (a) 96. (a) 97. (c) 98. (d) 99. (c)
100. (a) 101. (d) 102. (a)

r r o a - i T r i - c f c r D y - v t c t i 'i u o a i n i c i
TRANSFORMERS
• dings of a transformer are designated us
i T"'0 'V1?orv and secondary windings
> ) P "” ry and h.v. windings
(W p condary and l.v. windings or primary and l.v. windings
(J) ^ and U windings '

L n transformers A andB, having identical ratings, are to be desienod v„;n n .


*•H r ani I-4 r espectively. The weight of transformer A per kVA would ho nsitics ° r
; ipes than that of transformer B '
m more than that of transformer B
f equal to that of transformer B ,
(j) may be less or more than that of transformer B depending upon other parameters
1 power transformers, core is made up of
3‘ [a) cast iron (6) silicon steel (c) ferrite (d) powdered alloy
4 Consider the following statements : "
' L Air-core transformers are used in gate-triggering circuits of thyristors
2, Cores made of soft-ferrite are used in pulse transformers
3, Air-core transformers are used in radio devices 1
4, Cores of powdered alloy are used in isolation transformers
From above, the correct answer is
(a) 2,3, 4 ( 6 ) 2 ,3 (c) 1, 3, 4 (d) 2, 4
5. For a given cross-sectional area of transformer core, stepped cores are used
(а) to reduce the core loss
(б) to provide more mechanical strength to the core
(c) to reduce the conductor material and therefore I2R loss
(cf) to reduce the magnetizing current '
6. In single-phase core-type transformers, LV and HV windings are arranged as under
(а) L.V. on one core-limb, H.V. on the other core-limb °
(б) half H.V. near the core and half L.V. outside the H.V. on each limb
(c) L.V. and H.V winding sections are sandwiched
(d) half L.V near the core and half H.V. outside the L.V. on each limb
7. A400/200 V, 50 Hz transformer operates at a flux density of 1.2 T when energised from its H.V.
side. For this transformer, linear dimensions of core are doubled while the number of turns are
halved on both its H.V. and L.V. sides. If this transformer is now connected to 800 V, 50 Hz on
its H.V. side, then its flux density would be
(a) 0.6 T (6) 1.2 T (c )3 .6 T (cf) 4.8 T
8. For core-type power transformers, both primary and secondary windings have circular coil
sections, because this section
(a) is easier to wind
m fequires minimum conductor material and, therefore, less I2R loss
fd! stron£est mechanical shape
) ' results in less core material and, therefore, less core loss
•Transformer action requires a
a) constant magnetic flux (6) increasing magnetic flux
c) alternating magnetic flux (d) alternating electric flux
le-phase transformer, with subscripts 1 and 2 for primary and secondary windings,
. } Z} N2 = E2Wi and I x N x = I2 N 2 (6) E x N x = E 2 N 2 and l x N x = I2 JV2
V) ExN2 = e 2 ATj and I x N 2 = I2 N x (d) E 1 N y = E 2 N 2 and I x No = I2 N x

^J6r^ ux inv°lved in the emf equation of a transformer has


8 value (6) average value(c) total value (d) maximuin value
^ .\ • - . ’ ’

Scanned by Cam Scanner


932 Electrical M

12. For understanding the behaviour of a transformer, the folkswing laws may be called for
-1. -Lenz’s
• ■law 2. Newton’s second law
3. Faraday’s law of electromagnetic induction 4. Ohm’s law
5. Fleming’s right-hand rule 6. Right-hand grip rule
From these, the correct answer is
(a) 1, 3, 4 (6) 2, 3, 4, 5 (e) 1, 3, 4, 5, 6 (d) 1, 3, 4, 6
13. In an ideal transformer, if K is some constant, then the supply voltage V, in terms of jtg
magnetizing current Im can be expressed as

(a)JKfln jK f (c) -jK fI,m


(6 ) lm
rn
14. In an ideal transformer, the impedance can be transformed from one side to the other
(o) in direct proportion to square of turns-ratio (b) in direct proportion totums-ratio
(c) in inverse proportion to square of turns-ratio (d) in inverse proportion to turns-ratio
15. Distribution transformers are designed to have
(a) core loss Pc > full-load ohmic loss Pah (b)P c <P 0h
(c)Pc = Poh (d) Pc negligible as compared, to Pok
16. A transformer has sometimes two or more ratings depending upon the use of
(a) the cooling techniques (6) the type of windings
(c) the type of core (d) the type of insulation
17. In a transformer, exciting current is made up of two components ; namely magnetizing ciirrent
Im and core-loss current lc. With negligible leakage impedance drop,
(а) both Im and Ic lag supply voltage V* by 90°
(б ) both Im and Ic are in phase with Vj
(c) l m lags Vi by 90° whereas lc is in phase with Vi
(d) Im is in phase with V1 but Ic lags V1 by 90°.

18. A transformer at no load is excited at rated voltage. Now a cut is made in the transformer yoke
thus creating a small air gap. With this, the transformer core flux
(a) will decrease and magnetizing current Im will increase
(b) will remain constant and Im will increase
(c) as well as Im both will increase
(d) as well as Im both will decrease

19. I f the secondary winding of the ideal transformer shown in the circuit of Fig. C.42 has 40 turns,
the number of turns in the primary winding for maximum power transfer to the 2 £2 resistor will
be
(a) 20 (b) 40 (c) 80 (d) 160 -

Ideal
transformer
8n
-yW W
~ i *j—
I
o —40 a o- -oc
I o Turns : 2X1

bo- -od
t
N ,: Nz

Fig. C.42. Fig. C.43.


20. In a single-phase transformer, polarities of terminals a and b at any instant are shown in tbe
Fig. C.43. A t the same instant,
(a) c is positive, d is positive and flux is clock-wise (cw)
(b) c is negative, d positive and flux is counterclock wise .
(c) c is negative, d is positive and flux is cw
933

gitive, d. is negative and flux is ccw.


$ c 181 filled transfonner, oil is provided for
. tn ao 011 (6) insulation
[“) cooling (d) preventing the accumulation of dust
, 3 . Moline and in8ulation
, ,-rters cores are laminated to reduce
8
^■^dTy-oirrentloss J {b) hysteresis loss
both eddy-current and hysteresis loss (,d) ohmic loss
^irO laminations in a transformer are used to minimise
23. Ch current loss (6) hysteresis loss
^oth eddy-current and hysteresis losses (d) ohmic loss
i case of a power transformer, the no-load currentin terms of rated current is
24’ (a) 10 to 20% (b) 2 to (c) 15 to 30% (d) 30 to 50%
25 Ifa transformer primary is energised from a square-wave voltage source, then its output voltage
will be .
,fl) zer0 (6) a sine wave
(C) a triangular wave (d) a pulsed wave
26. The primary (220 V side) of a 2 2 0 /6 V, 50 Hz transformer is connected to 110 V, 60 Hz source.
The secondary output voltage will be
(a) 3.6 V (6) 3.0 V (c) 2.5 V (d) 1.667 V
27. The no-load current in a transformer lags the applied voltage by
(a) 90“ (b) about 75° (c) 0° (d) about 110°
28. A transformer has N j and N 2 turns in primary and secondary windings respectively. Its secon­
dary-winding reactance x2 £2, when referred to primary, is

(N 2 ')2 KT
(a) *2 (6) *2 N,
Ni
\2
Nt
Hi
(c)*2
29. A 400/200 V transformer has total resistance of 0.02 pu on its l.v. side. This resistance when
referred to h.v. side would be
(a) 0.02 (6) 0.04 (c) 0.01 (d) 0.08
30. The leakage flux in a transformer depends upon
(a) the applied voltage (6) the frequency
(c) the load current (d) the mutual flux
31‘ useful flux of a transformer is 1 Wb. When it is loaded at 0.8 pf lag, then its mutual flux
(a) may decrease to 0.8 Wb (b) may increase to 1.01 Wb
(c) remains constant (d) may decrease to 0.99 Wb
32 If * , .
‘ u>m a transformer Pc = core loss and Psc = full-load ohmic loss, then maximum kVA delivered
the load at maximum efficiency is equal to rated kVA multiplied by

W p- f Pr
K (b) (c)
sc r C
38 fi\
de •*transformers of the same type, using the same grade of iron and conductor materials are
signed to work at the same flux and current densities ; but the linear dimensions of one are
0 »nae8 those of the other in all respects. The ratio of kVA ratings of the two transformer
cl°8ely equals
(0) 16
34 . (6) 8 (c) 4 (d) 2
t . ^, an8^orm'erhaia
The *^ hasuiijjcuautc
impedance uiofv-*
(1- "'+j j 5)*■
£2won the
** —— •l.v. side and (9 + j 45) £1 on the h.v. side.
(a) ] g A bivalent impedance at the h.v. terminals is
+ / 90 £1 (6) 2 + j 10 Q
(c) 10 +j 50 ft
(d) 8 + j 40 n

Scanne am Scanner
https://t.me/abcdelectrical

35. A 220/440 V, 50 Hz, 5 kVA, single-phase transformer operates on 220 V, 40 Hz supr


secondary winding open-circuited. Then
(а) both eddy-current and hysteresis losses decrease
(б) both eddy-current and hysteresis losses increase
(c) eddy-current loss remains the same but hysteresis loss increases
(rf) eddy-current loss increases but hysteresis loss remains the same
36. The hysteresis and eddy-current losses of a single-phase transformer working on 200 V, 50 u
supply are Ph and Pe respectively. The percentage decrease in these, when operated on
160 V, 40 Hz supply are 3
(a) 32, 36 (6) 20, 36 -
(c) 25, 50 (d) 40, 80
37. For a single-phase transformer, re = total equivalent resistance, xe - total equivalent leakage
reactance, Pc = core loss. The load current at which maximum efficiency occurs is

(b) ■\ZZjT

Pc Pc
(c) — (d )~

38. The maximum efficiency for a transformer occurs at 80% of full load. Its core loss is Pc and ohmic

loss is Poh. For this transformer, the ratio - is


■*oh
(a) 0.8 (b) 1.25 (c) 0.64 (d) 0.8944
39. Frequency of the supply voltage to a transformer at no load is increased but the supply voltaee
is held fixed. With this ‘
(1) eddy-current loss remains constant but hysteresis loss increases
(2) eddy-current loss remains constant but hysteresis loss decreases
(3) magnetizing current increases but core-loss current decreases
(4) both magnetizing and core-loss currents decrease.
From these, the correct answer is
(a) 2, 3 (b) 2, 4 (C) 1, 3 (d) 1, 4
40.
The voltage applied to a transformer primary is increased keeping — constant. With this, the
core loss will
(a) decrease and magnetizing current Im will increase
(b) increase and Im will also increase
(c) remain constant and Im will also remain constant

! ■■■ (d) increase and will remain constant

41. A 4 kVA, 400/200 V, 1-phase transformer has leakage impedance of 0.02 + j 0.04 per unit. This
leakage impedance in ohms, when referred to h.v. side is
(a) 0 . 8 + j 1.612 (b) 0.2 +j 0.4 Q
(c) 0.08 + j 0.16 Q (d) 1 + j 2 Q
42. As the load on a transformer is increased, the core losses
(a) decrease slightly Ideal
(b) increase slightly transformer
Ao-
(c) remain constant
(d) may decrease or increase slightly depending 1
I '■’ Pure
upon the nature of load. I inductive
o i i a itoad
»
o i
43. For the system shown in Fig. C.44, the phase rela­ I L.
U
tion of current I with respect to the voltage is Bo- T
(a) zero (6) 90° lead
(c) 90° lag (d) 180° Fig. C.44.

Scanned by Cam Scanner


httr

c
AP£5ndiV
,. -two transformers X and Y with identical r a t i m T Z l T —
J4' junctions respectively. I f * c andX m are the L g n e t t S S ^ ° ' 8 mm « * * « ■ »» thick
circuit «* . ^“ ‘ aiueiers in the equ
* * * -. in h n t h . . ? P~ ra >» ‘ » a « > - a l e nt
(a) Rcvalues in both are likely to be equal, but of X I .' i'l. ,
(W X„ values in both are likely to be equal, but R of X ! r f * t0 be hiSher than X m of
ol Y
(c|x „ values in both are likely to be equal, but R of X ! ^ * ‘ ° be higher tha" «c of
Dfy
(d) Rc values in both are likely to be equal, but X ‘ of X is t ° Y *° be ‘° Wer than Rc o
°nfV
45.. If the applied voltage of a certain h ..i
transformer .
is * *1 ee y to ba
be lower than X „ of 7

j . SO* (assuming that magnetic circuit remains “ rated “t h e ^ ^ frCIIUe" cy ia -duced

(а) change to three times the original value maX'mUm °°™ flUX ^
(б) change to 1.5 times the original value
(c) change to 0.5 times the original value
(d) remains the same as the original value
46. In a transformer, low-voltage winding j
aS‘ ° „ mdl” E lS ^ aed — the core m case of concentric windings so
1. reduce the leakage flux
2. reduce the insulation requirement
3. reduce the risk of voltage shnMr Jr, „ .
4. reduce the core loss “ “ ° f lnsala« “ > breakdown

5. reduce the total conductor material


From these, the correct answer is ,

47. c L Y s O H z tr a n sfe r e r t ’e L e d for 25 H z L f ’th ■ ^ '


rated value corresponding to 50 Hz ? ’ 6 mpUt VoltaSe 1S maintained constant at the

W N "n r 4 “ ugb U d X ^ n Ur nt l ! Vels wiU “ * chunge


(c) No. Owing to decreased reactance'of tran f C° re t° excessive ite ra tio n

48. v ? S' A‘ C° nStant V° ltag e' - “ l a« o n “ S eo v 5 1 T ' WiU be d° Ubled a t l0ad


Under t h i s ^ ^ -^ i o n f Pnmary of a transfb™ e r is kept constant but its frequency is decreased

W m a r r in g „ S T " " ^ “ deCrea“ * '


(C) magnetizing current f e^ eases1but core-loss current increases
(rf) m a m e t i W 5 core-loss current both decrease
49. In t current and core-loss current both increase

1. maLeti7Sf0rm8r’ lf primary leakage impedance is neglected, then


gnetizing current lags the applied voltage V1 by 90°
■core-loss current lags V x by 90°
•^citing current lags V x by 90°

g e loss current is in phase with V\


g Iting current lags V x by about 80°
' ma^etizing current lags V 1 by about 80° >■
^ni fL.
(a) 1 ' correct statements are
50. A tran4 .5 » 3 .4 ,6 ( c ) l,4 id) 1 , 2 , 6

sida will0™ " Secondary is connected to pure resistive load. The power factor on the primary

(0 ab° Ut ° ' 95 lead W » « r about 0.95 lag


(d) unity

Scanned by Cam Scanner


16 Electrical Machinery

51. If supply frequency in a transformer is doubled . . .


(a) hysteresis loss also doubles eddy-current loss doubles
(c) iron-losses double hysteresis loss decreases
52. A 50 Hz transformer having equal hysteresis and eddy-current losses at rated excitation is
operated at 45 Hz at 90% of rated voltage compared to rated operating point, the core loss under
this condition , , ,
(a) reduces by 10% educes y 0 ic a 'tv
(c) reduces by 14.5% * «*> remamS “ nchanged ^ B , 1998]
53. For the purpose of analysis, exact equivalent circuit of a transformer is usually replaced by an
• approximate equivalent circuit. In doing so, errors introduced ue o wm mg o mic oss and
core loss are of differential nature. Due to this, the analysis by approxima e equiva en circuit
gives fairly satisfactory results. Under the circumstances, which one o e o owing s a ments
is correct in respect of losses referred to approximate equivalent circui as compare to exact
equivalent circuit ? . , , , ,
(a) This accounts for somewhat greater primary winding ohmic loss and less core loss.
> (ib) This accounts for somewhat less primary-winding ohmic loss and more core loss.
(c) This accounts for somewhat greater secondary-winding ohmic loss and less core loss.
(d) This accounts for somewhat less secondary-winding ohmic loss and more core loss.
54. In a 1-phase transformer, the magnitude of leakage reactance is twice that of resistance of both
primary and secondary. With secondary short-circuited, the input pf is

(a) ^ (6) i t (c) w {d) 1


55. High leakage-impedance transformers are used for applications such as
(a) power distribution (6) electric toys
(c) fluorescent lamps (d) arc welding
56. In an ideal transformer shown in Fig. C.45,
(a) Vl = a V 2,I 2 = - a I l (6) V2 = a Vj, 72 = - a 7j

(c) V1 = a V 2, l 2 = ^ l 1 (d) V1 = a V 2,72 = - ^ 7 1

r O
o
o
o
o
a-.l

<
F
oo
o
o
o

!_
Fig. C.45. Fig. C.46.

57. In an ideal transformer shown in Fig. C.46


(a) Vl = a V2,l2 = - a h (6) V2 - a Vlt l 2 = - a l i

(c) V1 = a V 2, l 2 = ^ J 1 id) V2 = a V j ,I2 - /j


a 1
58. When compared with power transformer, a distribution transformer has
(а) low %age impedance and high I2R loss to core-loss ratio
(б ) high %age impedance and high / 2iZ loss to core-loss ratio
(c) high %age impedance and low I2R loss to core-loss ratio
(d) low %age impedance and low I2R loss to core-loss ratio
59. Following statements are made regarding the open-circuit test on a 1-phase transformer :
1. It is performed on l.v. side •
2. It is performed at rated current
3. It helps in the calculation of equivalent leakage impedance

mmrmmm-
>cannea oy ca m
m sScc a
c nner
937
. W orm ed on h.v. side ~
4 Jt I® erformed at rated voltage

5 1115 9 magnetizing current and core loss


6-11 & - io the determination of voltage regulation
1 It be»Ps
Tt gives turns ratio
8' *hpse the correct answer is
W * g 6) 8 (*») 2 . 4 * 6 > 8 (c) 3 > 4 > 5. 6, 8 (d) 1, 5> 7, 8
knowing statements are made regarding the short-circuit test on a 1-phase transformer •
60 H v. side is short circuited
2 It is performed at rated current
3 It gives the core loss
4 L.V. side is short-circuited
5 It is performed at rated voltage
6 It gives ohmic loss on the side instruments are connected
7 It helps in the calculation of voltage regulation
prom these, the correct answer is
(a) 1,2, 6, 7 (*>) 2, 3, 4, 7 (c) 4, 5, 6, 7 (d) 2, 4, 7
61. A 10 kVA, 400/200 V, single-phase transformer with 10% leakage impedance draws a steady
short-circuit line current of
(a) 50 A (b) 150 A (c) 250 A (d) 350 A
62. While performing the open-circuit and short-circuit tests on a transformer to determine its
parameters, the status of the low-voltage (L.V.) and high-voltage (H.V,) windings will be such
that
(а) in O.C., L.V. is open and in S.C., H.V. is shorted
(б) in O.C., H.V. is open and in S.C., L.V. is shorted
(c) in O.C., L.V. is open and in S.C., L.V. is shorted
(d) in O.C., H.V. is open and in S.C., H.V. is shorted
63. A multimeter, for measuring resistance, is connected to one terminal of primary and the other
terminal of secondary. The multimeter reading would be
(a) zero (6) infinity
(c) zero or infinity (d)equal to the resistance of the windings
64. For given base voltage and base volt-amperes, the per unit leakage impedance of a transformer
is x. What will be the per unit leakage impedance of this transformer when the voltage and
volt-ampere bases are both doubled ?
(a)0.5x (6) 2 x (c) 4 x id) x
At 50 Hz operation, a single-phase transformer has hysteresis loss of 200 W and eddy-current
loss of 100 W. Its core loss at 60 Hz operation will be
(a)432W (6) 408 W (c) 384 W (d) 3 6 0 W [I.E.S., 1995}
66’ The hysteresis and eddy-current losses of 1-phase transformer working on 200 V, 50 Hz supply
are Ph and Pe respectively. The percentage decrease in these losses when operated on a
160 V, 40 Hz supply would respectively be
1°) 32,36 (fc) 2 0 ,3 6 (c) 2 5 ,5 0 id) 4 0 ,8 0 1/.25.S., 2001]
67, Jj a transformer, eddy-current loss is 100 watts which is half of the total core loss. If both the
thickness of laminations and frequency are increased by 10%, the new core-loss would be
to) 256.41 W (6) 2 3 1 W (c) 267.41 W (d) 2 4 2 W
8’ * 220/U 5 V, 25 Hz, 1-phase transformer has eddy-current loss of 100 W which is half of the
no-load loss at rated applied voltage. If this transformer is used with primary connected to
Hz mains, the total no-load loss would be
to) 300 W (&) 600 W (c) 1000 W id) 400 W

Scanned by Cam Scanner


h

Electrical Mai
938
thick laminations has eddy-current loss of 12Q w
69. A 220 V, 50 Hz transformer with 0.35 mm tn transformer is built with 0.7 mm
which is two-third of the total loss at no-lo ■ nQ_load loss would be M
laminations and is worked from 110 V, 2 » ^ ^
(a) 150 W (6) 510 W (c) 50 Hz, has eddy-current loss of 50 W a ,
70. A transformer, fed from an alternator . ’ ver driving the alternator drops to 80*
hysteresis loss of 100 W. If the speed of the losses in the »*'
its previous speed, then eddy-current an y
transformer would respectively be , aq w , 64 W
«w 40 W, 80 W (6, 32 W, 80 W (c, 3 W « ^
71. Two windings of a transformer are ,, f l 0 0 V is applied
A S and CD as shown in Fig. C A V ^voltage of lOO V is^ P P ^
e n a

across AB with BD short-circuited, a voltage of 2UU v app


Fig. C.47.
•C. The turns ratio from CD to AB is
0d) 2 or 1
(a) 3 (6) 1 (O 3 or 1

72. In question 71, if voltage appearing across terminals AC is


100 V with input to AB as 100 V, then the turns ratio from
CD to AB is
(a) 2 (6) 1
(c) 2 or 1 (d) 3 or 1
73. A 400 V /2 0 0 V /200 V, 50 Hz, three-winding transformer is
connected as shown in Fig. C.48. The reading of the
voltmeter V will be
(a) 0 V (6) 400 V Fig. C.48.
(c) 600 V (d) 800 V
74. A transformer has leakage impedance of ze = re +jxe. Its maximum voltage regulation occurs at
a power factor of

(a) — leading (6) — lagging (c) — leading (d) — leading


xe ze ze ze
75. A transformer has leakage impedance of ze = re + j xe. Zero voltage regulation for this transformer
occurs at a pf of
r3 xe
e e
(a) — leading (6) — lagging (c)
(c) —— leading
leading (d) — leading
xe Ze zz„
e Zg

76. A 1-phase transformer has p.u. leakage impedance of 0.02 + j 0.04. Its regulation at pf 0.8 lagging
and 0.8 leading are respectively
(a) 4%, 0.8% (b) 4%, - 0.8% (c) 2.4%, - 0;8% (d) 4%, - 1%
77. A 10 kVA, 400 V /2 0 0 V, 1-phase transformer with a percentage resistance of 3% and percentage
reactance of 6% is supplying a current of 50 A to a resistive load. The value of the load voltage
is
(a) 194 V (6) 390 V (c) 192 V (d) 196 V ,
78. In a transformer, re2 + j xe2 = equivalent leakage impedance ze2. Its maximum voltage regulation
is equal to

(a) re2 at — lagging pf (6) xe2 at — lagging pf


z e2 z e2
re2 r e2
(c) ze2 at — leading pf (d) z 2 at — lagging pf
e^ Ze2 . near
79. Full-load voltage regulation of a power transformer is zero when power factor of the l°a'^ iS
(a) unity and leading (£>) zero and leading
(c) zero and lagging {d) unity and lagging

■ ..

>cannea by Cam Scanner


nd — -— " “ '----------------------------------- ---------------- -----------------------------
t a U :^|1
d“ nsf0nner has leaka®e imPedance of (0.02 + j 0.8) p.u.. Its val

^ in°0 2 + 7 0-8 ( b ) 0 .2 5 + j l (c) 0.02 + j 0.08 (d) 0.0625 + j 0.25


^ oltage regulation of a transformer is given by

9,' ThS 3 . ( o M m ?2=*i


(a) ~~V2 2 2 E2
M The voltage regulation of a transformer at M l-lo a d 0.8 pf lagging is 4%. Its voltage regulation
s2‘ t full-load °.8 pf Lsading e 5
ffl) will be positive (&) wjn be negative
(C) may P0S*Hve (d) may be negative
voltage regulation of a transformer depends on its
83’ i equivalent reactance 2. equivalent resistance 3. load power factor
4 transformer size 5. load current
From these, the correct answer is . .■ ..
(a) It 2- 3> 5 1} 2 ’ 3 ’ 4 ’ 5 (c) 1( 2 > 4 ’ 5 1’ 2> 3 . 4
g4. The voltage regulation of a transformer at full load and 0.8 pf lagging is 2.5%. The voltage
regulation at full load 0.8 p f leading will be
(a) - 2.5% (&) zero (c) - 0.9% (d) 2.5%
85. In Sumpner’s test on two identical transformers having rated frequency f,
(a) both the primaries and the regulating transformer should be fed from voltage sources of
frequency f
(b) both the primaries and the regulating transformer may be fed from voltage sources at
frequencies different from f
(c) both the primaries should be fed from voltage source at frequency /'but regulating transformer
may be fed at a frequency different from f
(d) both the primaries may be fed from voltage source at a frequency different from f but
regulating transformer should be fed at f
86. The efficiency of a transformer at full load 0.8 pf lag is 90%. Its efficiency at full load 0.8 pf lead
will be ;
(a) somewhat less than 90% (6) somewhat more than 90%
(c) 90% id) 91%
87. Transformer maximum efficiency, for a constant load current, occurs at
(a) at any pf (&) zero pf leading
(c) zero pf lagging id) unity pf
88. One transformer has leakage impedance of 1 + j 4 Q and 3 + j 11 a for its primary and secondary
windings respectively. This transformer has
(a) h.v. primary (&) medium voltage primary
(c) l.v. primary (<*) Lv- secondary .
89. Transformer at no-load behaves like ,
(a) a resistor p f= 0 an mductive reactor, pf = 0.2 lagging
(c) a capacitive reactor, p f = 0.2 leading «*) ™ inductive reactor, pf - 0.8 lagging
80. In a transformer, the tappings are provided on
to) h.v. side at one end of the winding Q>) -v- s|de at oae end ° f the winding
to) h.v. side at the middle . & Lv' Slde at the middle
81- In transformers, the windings are tapped in the middle
to) to avoid the radial forces on the windings
(b) to eliminate the axial forces on the windings
to) to reduce the insulation level of the windings
to) to provide a mechanical balance to the windings
In transformers, tappings are provided on the h.v. side because
it has larger number of turns which allow smoother control of output voltage
h-

Electrical MachiJ
940

2. it has to handle low value of currents


3. it is easily accessible for repairs
4. it requires less insulation
From these, the correct answer is ^ ^ 2, 3
(a) 1, 2, 3, 4 (6) 2, 3, 4 (c) 1, , the adjoining turns of the same winding
93. When a transformer winding suffers a sho >
experience
(a) an attractive force
(b) a repulsive force

( $ may ^attra ctive or repulsive depending upon the current directions ^


, , .marv (p) and secondary (S) ot a 1-phase trans-
94. The relative current directions through the Pnm y { / indicated in the various cross­
former connected to a resistive load on the secondary s.de are in
sectional views given in Fig. C.49. Which of these are correct representations^

(a) (b)
Fig. C.49.
95. Short-circuit test on a single-phase transformer gave the following data .
30 V at 50 Hz, 20 A ,p f = 0.2 lag
If s.c. test is performed on 30 V, 25 Hz, then short-circuit current
(a) decreases at a pf < 0.2 (&) increases at a p f < 0.2 .
(c) increases at a pf > 0.2 (d) decreases at a p f > 0.2
96. Open-circuit test on a single-phase transformer gave the following data :
230 V at 50 Hz, 2 A, pf = 0.2 lag
If open-circuit test is performed on 230 V, 45 Hz, then no-load current
(a) decreases at a p f > 0.2 (b) decreases at a p f < 0.2
0c) increases at a pf > 0.2 (d) increases at a p f < 0.2
97. When short-circuit test on a transformer is performed at 25 V, 50 Hz ; then drawn current T\ is
at a lagging pf-angle of <J>|. If the test is performed at 25 V, 25 Hz and the drawn current T 2 is
at a lagging pf-angle of <j»2, then
(a) I 2 > I\ and <J>2 < (&) ^2 < h anc* $2 < $1
(c) / 2 > / i and <j>2 ><f>, (d) I 2 < h and <t>2 > <t>i
98. Two transformers to be operated in parallel have their secondary no-load emfs E afor transformer
A and E b for transformer B. As Ea is somewhat more than E b, a circulating current Ic is
established at no load which tends to
E„ - Eu
(a) boost both Ea and E b with Ic =
’ea zeb
E —E
(6) boost E a and buck E b with Ic = ——------ -
z ea
E —E
(c) buck Ea and boost E b with Ic = a b
£ ea + z eb , . *•. ■
-'*■•
j-''/'••
- '*1 ■*- •: ■ '
i, - •

Eb -E ,
{d) buck both E a and E b with lc
z eb

Scanned by Cam Scanner


J .7ga-

941
.nd!s^:— -------------
tr a n s fo r m e r s of identical voltages but of different capacities are operating in parallel. For
99- T* ° f^tory load sharing,
must be equal
{<*) J impedances must be equal
* r-unit impedances and j| ratios must be equal
^ yr
J) imped®11068 ®nd ^ ratios must be equal

(jwn transformers of different kVA ratings working in parallel share the load in proportion to
1°°* their ratings when their
1 per unit leakage impedances on the same kVA base are equal
2 per unit leakage impedances based on their respective kVA ratings are equal
3. ohmic values of the leakage impedances are inversely proportional to their ratings
4 ohmic values of the magnetizing reactances are the same.
From these, the correct answer is
(a) 1, 3>4 2- 3 to) 2, 3 ,4 (d ) 1, 4 [GATE, 1992]
101. Transformer operating in parallel will share a common load in the best possible manner if
(a) the leakage impedances are proportional to their respective kVA ratings
(ib) the leakage impedances are equal
(c) per unit leakage impedances are equal
{rf) per unit leakage impedances are proportional to their respective kVA ratings
102. The necessary conditions for parallel operation of two 1-phase transformers is that these should
have the same
1. polarity 2. kVA rating 3. voltage regulation
4. efficiencies 5. voltage ratio 6. — ratio
R
From these, the correct answer is
(a) 1, 2, 3, 4, 5, 6 ^ 2, 5, 6
w 5>6 (d) 1, 3, 5, 6
103. Forsuccessful parallel operation of two single-phase transformers, the essential condition is that their
a percentage impedances should be equal (6) turns ratio shouldbe exactly equal
(c) polarities must be properly connected (d) kVA ratings should be equal

1M lT ad^“ ff M Cre 0Perating in Para“ el haVC different quality of their teakaf * impedances. For
(a) both would operate at pfe less than 0.8
(b) both would operate at pfe more than 0.8
(c) both would operate at the same pfe
Id) one would operate at pf > 0.8 and the other at pf < 0.8

W ^ + ; i Pm o L ^ f nSf° r!? erSJ ' ith e,U al turns rati0 have impedances of ( 0 .5 + j 3 ) Q and
SeCOmia,y- IftHey 0Parab! to h° “ share a Z l

w ra.2 k w ^ i 8 kw (6) 62 kw> 38 kw


IM. A . ^ , 1 f kW, W 8 5 5 kw- M .5 kW U.E.S.. 1992]
transformer, with kVA rating K, has voltage rating of V j /V * This transformer

be connected as an autotransformer to get two possible voltage ratings of + Y l nnri


Y1+ V2 v2 and
Fj 'Hie respective kVA ratings as an autotransformer are

vi t v2 v, + v2 v. + v,
vi r K --— k
2 1

ScanriecTby Cam Scanner


Electrical Maeli
942

V, + ^2 ^1 + ^2
W T v k - '- v T T .
. . ” r ..... {a reconnected as an autotransformer arm*
107. A .100 V /1 0 0 V, 10 kVA, two-winding transformer « » c()uld acr®»
n suitable voltage. Tho maximum rating of such a tin •
<«> 50 kVA (*, 15 kVA (c) 12.5 kVA « '7 j kVA
108. 'An ’-"VV.V.
nutotrnnsformur Imving a
MMA'IVUUIOI MOVI»»|; «» transformationratio
' of 0.8
1 aupplie.
■ a load of 10 kW. The p„Wer
transferred inductively from the primary to tho seconan y
(a )1 0 k W (6) 8 k W (c) 2 kW (d) zero
109. A single-phase transformer has a rating of 15 kVA, 6 ° ° V /1 2 0 V It ^ ^connected as an
autotransformer to supply at 720 V from a 600 V primary source. The maximum load it can
supply is :
(a) 90 kVA (6) 180 kVA (c) 15 kVA (4) 18 kVA

HO. in mi autotransformer of voltage ratio rj - and Vj > V%, the fraction of power transferred induc-
72 ’
tively is
Vl ^2 vl~ v2 V2

(<I) v'l + V2 (/>) V\ (C) V'l + ^2
111. A 10 kVA step down autotransformer has voltage ratio of 0.7. The transformed and conducted
kVA can be respectively
(a) 3, 7 (b) 7, 3 (c) 5, 5 (d) 3.5, 6.5
112. A supply of 100 V can be obtained from a source of 300 V, by means of a two-winding transformer
or an autotransformer. The ratio of weights of conductor material in the autotransformer with
respect to two-winding transformer is
(a) 1 : 1.5 (6) 1.5 : 1 (c) 3 : 1 (d) 1 : 3
113. In an autotransformer of voltage ratio Vx/V 2 with Vy > V2, the conducted power in terms of total
power is
Vy V2 V, -V o V, - Vo
W it :
f2 W
" " v,
i t -
■' v,
w V - 1 W>
w v2
114. A 20 kVA, 2300 V /230 V, two winding transfonner is to be used as an autotransformer to give
2300 V /2530 V. Its rating will be
(a) 200 kVA with conducted kVA = 20 (6) 200 kVA with conducted kVA = 180
(c) 220 kVA with conducted kVA = 20 (d) 220 kVA with conducted kVA = 200
115. Single-phase supply of 220 V, 50 Hz is to be obtained from 400 V, 50 Hz source The ratio of
weight of conductor material in a two-winding transformer to that required in an autotransformer
is
20 Q on
(«) T f (d) 2

116. A 400 V /200 V transformer has a full-load voltage regulation of x p.u. at 0.8 pf lagging If this
transformer is used as an autotransformer with voltage rating 400 V /6 0 0 V or 200 V /6 0 0 V then
its voltage regulation would be ’
, sx x , 2x x 2x 2x v 9v
( 0 ,3 '3 "3" ’ 3 w f ' T <d)f - f
117. An autotransfbrmer has V , . / , as input quantities nnd V2, / 2 as output quantises with
2 < 'l - The VA conducted from input to output is
(a) 72 (6) V2 ly (c) Vy Iy - V2 / 2 (d) (Vy - V2) Iy
118. An autotransformer has V „ 7, as input quantities and V2, / 2 as output quantities with V2 < V,.
ine VA transformed from primary to secondary is
(a) V\ h (b) V2 Iy (C) Vy Iy - V2 I2
h id) (Vy - V2) Iy

Scanned by Cam Scanner


h tr-V :

943
— - -
• rv of a 100 kVA transformer is 0.98 at full as well as half load. For this transformer
o Th« theohmic loss
I1 at fo11 ^ 3 0 core loss (£>) is equal to core loss
(fl) is than core loss (d)none of the above
lS VA transformer has a core loss of400 W and a full-load copper loss of 800 W. The proportion
i20. M °n load at maximum efficiency is
50# (b) 62'3% (C) 7° ‘7% 10° %
^ a the following statements about induction regulators :
nsid®1*1 4 4 J.
jjl. ipensating windings are needed in single-phase induction regulators (IR).
1 1 singlO'Pb®36 IR. secondary induced voltage E2 is in phase with the appliedvoltage Vv
^i single-phase IR, magnitude of E2 remains constant with the rotor movement.
ln 3-phase IR, magnitude of E 2 varies with rotor position.
5 In 3-phase IR, E2 is not in phase with Vx except when 6 = 0° or 180°.
these the correct statements are
( i ] W ( 0 1 ,2 ,4 1 ,2 ,5
In a tap changer, the voltage at consumers terminals is kept within the prescribed limits by

(ajTatfoo^turns between primary and secondary windings


(b) frequency
flux density in the core
(d angle between the magnetic axes of the primary and secondary windings
rn A single-phase induction regulator is a constant-voltage input transformer to obtain smooth
’ variation of the output voltage by varying the
(a) ratio of turns between primary and secondary windings
(b) frequency
(cl flux density in the core . . . ____
(d) angle between the magnetic axes of the primary and secondary windings
. ^ • a. u in a 1 nha«sp 200 V 5 kVA system to keep the voltage
,!4' t n w itZ l SiThe rating (in kVA, of the voitage reguiator is
(a) 0.05 (6 ) 0.5 (c) 5 (d) 50
125. A 3-phase induction voltage regulator needed to regulate 100 kVA between the voltage hunts
1200 and 800 V has a rating of kVA
(a) 10 kVA (6) 20 kVA (c) 5 kVA (d) 30 kVA , _
126. A 3-phase delta-star transformer has secondary to Pnaiary turns ratl° per p ase 0 '
primary voltage of 400 V, the secondary voltage wou e
(a) 2000 V (6) 80 V (c) 3464 V (d) 80 V3 V
m . A 3-phase star-delta transformer has secondary to primary turns ratio per phase of 5. For a
Primary voltage of400 V, the — ndary vo ^ g e wo ^ ^

A 3-phase delta-star transformer has rat,° ^ ° ' " '


primary line current of 10 A, the secondary 1.155 A
129 ^ 2A {b) 3,464 A ,(C) ' marv t0 secondary turns ratio per phase of 5. For a
' A 3-phase star-delta transformer has prim ry would be
Primary line current of 10 A, the secondary i
(“) 50 A (6) 86.6 A - ( 0 3.464 A W ^
In transformers, the noise or hum is produced prim y magnetostrjction
to) improper tightening of core laminations loose bolts etc.
to) tank walls

Scanned by Cam Scanner


https://t.me/abcdelectrical

— ■■■— i linn n m — a,

_____ I
131. Match the items on the left-hand side with the most appropriate item on the right-hand side •
Type \ Application
A. Power transformer 1. Thyristor firing circuits
B. Distribution transformer 2. Impedance matching
C. Pulse transformer 3-. At different city localities
D. Audio-frequency transformer 4. At generating stations

132. Which of the following statements are incorrect ?


1- Maximum voltage regulation of a transformer occurs at leading power factor
2. Voltage regulation of a transformer is the maximum when load power-factor (lagging) angle
has the same value as the angle of equivalent impedance
3. Voltage regulation of a transformer at zero power factor is always zero
4. Voltage regulation of a transformer can be negative at leading power factor
Select the correct answer using the codes given below :
Codes :
(a) 1 and 3 (6) 2 and 3 (c) 2 and 4 (d) 1 and 4 [/.AS., l&ft]
133. When a short-circuit test is conducted on a single-phase transformer, 30% of the rated voltage
is required to allow full-load current. The short-circuit power factor is found to be 0.2. The
percentage regulation at UPF is
30 (b) 29.5 (c) 15 (d) 6 [I.A.S., 1993]
134. A bank of three identical single-phase 250 kVA, 11 kV/230 V transformer is used to provide
400 V low-tension supply from a l l kV, 3-phase sub-station. The effective kVA rating of the bank
will be
(a) 250 (6) 250 V3~ <c) 500 (d) 750 [/A S ., 1994]
135. A transformer designed for operation of 60 Hz supply is worked on 50 Hz supply system without
changing its voltage and current ratings. When compared with full-load efficiency at 60 Hz the
transformer efficiency on full load at 50 Hz will
(a) increase marginally (6) increase by a factor of 1.2
(c) remain unaltered (d) decrease marginally
136. The voltage regulation of a large transformer is mainly influenced by
(а) no-load current and load power factor
(б) winding resistances and load power factor
(c) leakage fluxes and load power factor
(d) winding resistances and core loss
137. Fig. C.50 shows an ideal single-phase transformer. The primary and secondary coils are wound
on the core as shown. Turns ratio (N y/N2) = 2. The correct phasors of voltages E h E2, currents
Ilt 12 and core flux <j>are as shown in
(a) Fig. (a) (b) Fig. (b) (c) Fig. (c) (d) Fig. (d) [GATE, 2003]
, 1E1 El ’

i2a

"I
E2
III, *1 ih

Scanned by Cam Scanner


htt

(a) (-1 0 + y 10) A


(C) (io +> io) a - i ° : 7 10) A
• i v (d) (10 - j 1 °)A [GATE, 2003]
139. A single-phase transformer has a maximum efficiency of 90%
Efficiency at half load at the same power factor is full load and unity power factor.
(a) 86.7% (6) 88.26% (c) 88.9% (d) 87.8%
IGATE, 2003]

a n sw e r s
1. (d) 2. (6) 3. 0b) 4. (6)
6. (d) 7- (b) 5. (c)
8. (c) 9. (c)
11. (d) 12. (d) 10. (a)
13. (a) 14. (a)
16. (a) 17. (c) 15. (6)
18. ib) 19. (c)
21. (c) 22. (a) 20 . (c)
23. (c) 24. (6)
26. (b) 27. (6) 25. {d)
28. (c) 29. (a)
31. (d) 30. (c)
32. (c) 33. (a) 34. (a) 35. (c)
36. (b) 37. (a) 38. (c) 39. (6)
41. (a). 40. (d)
42. (d) 43. (&) 44. (c)
46, (d) 45. (a)
47. (b) 48. (d) 49. (a)
51. (d) 50. (6)
52. (c) 53. (6) 54. (6)
56. (a) 55. (d)
57. (d) 58. (a) 5 9 .(a)
6i. (C) 60. (d)
62. (6) 63. (&) 6 4 . (a)
66 .(6 ) 65. (c)
67. (a) 68. (6) 6 9 .(a) 70. (5)
71- (c) 72. (a) 73. (6) 74. (6) 75. (c)
76. (6) 77. (a) 78. (d) 79. (a) 80.(5)
81. (6)
82. (d) 83. (a) 84. (c) 85. (c)
86. (a)
87. (d) 88. (c) 89. (6) 90. (c)
91. ffi)
92. (d) 93. (a) 94. (a) & (&) 95. (c)
96. (d)
97. (a) 98. (c) 99. (c) 100. (6)
Ml. (c) 1 0 2 .(d) 103. (c) 104. (d) 105. (c)
106 . (a)
107. (a) 108. (e) 109. (a) 110. (d)
JU. (a)
112. (a) 113. (6) 114. (d) 115. (a)
1J6. (d)
117. (6) 118. (d) 119. (c) 120. (c)
!21- (d
122. (a) 123. (d) 124. (6) 125. (6)
126. {c;
127. (6) 128. (d) 129. (6) 130, (6)
131. A 4 d
, C - > l ,Z ) - > 2 132. (a) 133. (d) 1 3 4 .(d)
1®5. (d)
136. (c) 137. (d) 138. (d) 1 3 9 .(d)

Scanned by Cam Scanner


ELECTROMECHANICAL ENERGY CONVERSION PRINCIPLES
I. An tdtu'lromtH'Imnicnl onorgy conversion dovico iH shown in LiK- C.52. The instantaneous vah
id nrmntuni induced emf, bus bar voltage and armature current are respectively et u aryj *
l\ , l\, and ia( represent external torque, armature
developed torque end angular velocity of shall
respectively, Assume that the given directions of
parameters in the given figure are positive. If the
device is working in generating mode then
to) all parameters will have the positive direction
(M r\ e, / ’(, and u>r will have negative direction and
others will remain in positive direction
to) Tu ond I will hove negative direction and others
will remain in positive direction

id) ond u>r will have negntive direction and others will remain in positive direction.
^ Wfld - field energy stored, then magnetic force developed in a singly-excited linear magnetic
system is given by
, f d wnd (V. *)
*• h --------^ r ;-------, saturation may, or may not, be neglected

.) f d w{7d (<t>. *) .
fe * ^ . saturation is included

d X
, saturation is included

aw ,/7rf -r) ,
4. fe « + j . saturation is neglected.

From these, the correct answer is


(a) 1, 2, 3, 4 (6 ) 2, 3 (C) 2, 3, 4 (d) 1, 2, 4
3. A coupling magnetic field must react with
1. electrical system in order to extract energy from mechanical system
2 . mechanical system in order to extract energy from mechanical system
3. electrical system in order to extract energy from electrical system
4. mechanical system in order to extract energy from electrical system
5. both electrical and mechanical systems for electromechanical energy conversion
6 . electncal or mechanical system for electromechanical energy conversion
From these, the correct answer is
(a) 1. 2, 3, 4, 6 (fc) 1, 4, 5 (c) 2, 3, 6 (d) 2, 3, 5
4. In a magnetic system, if y = flux linkages, then

1. field energy stored, Wpd + co-energy, W ^ / = ^ y . j

2. Wju + Wfu' = v i
Wfld + Wfjd' - F <J>

4. = 4r Rl

5- Wfid + Wfu = | L i2

From these, the correct answer is


(°) 1. 3, 4 (6) 2, 3, 5 ( C l , 3. 4. 5 (d) 2 , 3 , 4

"ScahneS’ byTJamScanner
j ...

947
— -----------------
tomechanical energy conversion process, it is essential
that coupling magnetic field

reflCf ^ c a l system (6)


v~, mechanical system
(a)e ., electrical and mechanical systems l/f l electric
(d) alnn^wiM or
a . __ ■
mechanical system
(c) v tic circuit of Fig. C.53, the coil is excited by
in the choose the correct statement from the fol-
B® ,
0fthe magnetic energy will get stored in the! air
ai

fost of the magnetic energy will get stored in the iron


i

The magnetic energy stored will be stored almost


(c) ually in the iron core and the air gap
All the magnetic energy will be stored in the ironi core
to; *— Fig. C.53
nptic stored-energy density in iron is given by
Magnet 2
1 B2
ib) * w 2 7
' i.
The energy stored in a magnetic field is given by
* 2. \1 <f’ Rl- 3. | (mmf)2 Rl

From these, the correct answer is


,a) 1,2, 3, 4 (6) 1 ,2 , 3 (c) 1, 2, 4 w 1, »,
(d) 3, -4
a A linear magnetic circuit has flux linkages of 1.2 Wb-tums when a current of 10 A flows through
its coil The energy stored in the magnetic field of this coil, in Joules, is
la) 12 <&> 6 (c) 1 2 (d) 24 • u
ULOe electromagnetic force and/or torque, developed in any physical system, acto ,a such a
direcuon as to tend to ^
to) decrease the magnetic stored energy at constant mmf
(b) decrease the magnetic stored energy at constant flux
(c) increase the magnetic stored energy at constant flux
(d) increase the magnetic stored energy at constant curre h direction asto tend
11. The electromagnetic force, developed in any physical system, acts msuch a dnectmn as to
to -
(a) decrease the coenergy at constant mmi
{b) increase the coenergy at constant flux
(c) decrease the coenergy at constant flux
id) increase the coenergy at constant mmf _ m d Wlth magnetic saturation
12. The electromagnetic torque, developed m any physica y
neglected, acts in such a direction as to ten
(а) decrease both the reluctance and inductance
(б) increase both the reluctance and in<^u^ a® ^ . nce
to) decrease the reluctance and increase e inductance ,
(d) increase the reluctance and decrease e 1 i ovstem, acts in such a direction
* Electromagnetic force and/or toique, developed in any physical system,
fis to tend to ov at constant current
(«) increase both the field energy and co-energy ^ constant current
(&) increase the field energy and decrease t e constant current
(c) decrease both the field energy add co-energy rgy ^
ai. constant
— current. .
------------------------------------------
W) decrease the field energy and increase the electromechanical energy conversion sys-
-wtromechanical enei
*• developed electromagnetic force end/or torque
act in such a direction that tena»
tends
(a)to:- - stored* energy at* oonstan
increase the Mni
.XT**
J l / O I II I C U uy v^ai i lo o a i ii ici
r

94 8 __________________________________________ ________________________________________

ib) to decrease the stored energy at constant flux


Ic) to decrease the coenergy at constant mm [GATJP i
(d; to decrease the stored energy at constant mm , . , ’ i992)
15. The developed electromagnetic force and/or torque in electromech gy converaion %
tems acts in a direction that tends to
1. increase the coenergy at constant flux
2. increase the coenergy at constant mmf
3. decrease the stored energy at constant mmf
4. decrease the stored energy at constant flux
From these, the correct answer is .
(a) 2, 4 ib)1, 3 (c) 2, 3 («) x» 4
16. Electromagnetic force or torque developed in any physical system tends to
1. decrease both field energy and coenergy at constant current
2. increase both field energy and coenergy at constant current
3. increase the permeance
4. decrease the inductance
5. decrease current at constant flux linkages.
From these, the correct statements are
(a) 2, 3, 5 ib) 1, 3, 4 (c) 1, 3, 4, 5 (d) 1, 3, 5
17. A physical system of electromechanical energy conversion consists of a stationary part creating
a magnetic field with electric energy input and a moving part giving mechanical eneigy output.
If now the movable part is kept fixed, the entire electrical energy input will be
(a) stored in the magnetic field
ib) stored in the electric field
(c) divided equally between the magnetic and electric fields
id) zero
18. An electromagnetic relay has linear magnetic circuit. If the armature moves slowly to shorten
the air gap, then

1. mechanical work done, ^ c/j = | (electrical energy input, Welec)

2- Wmech = Welec

3. field energy stored, | Welec

4. Wfld = Wmech
From these, the correct answer is
(a) 1, 2, 3, 4 [b) 1, 2, 3
(c) 1, 2, 4 (d)1, 3, 4
19. A magnetic relay has linear magnetization curve at its open and closed positions If the armature
moves slowly from open to closed position, then electrical energy input to the relav
(а) gets stored in the magnetic field
(б) gets converted into mechanical work
ic) gets divided equally between the magnetic and electric fields
(d) gets divided equally between magnetic field and mechanical work
statements relating to the singly-excited electromechanical system of Fig-

from a areferenc^CUrr6nt ^ UX ^n^a^es C°A respectively, x is the position of the armature

1. Magnetic stored energy decreases with increased value of x

x ju u i ii iu u u y v^cii i ix ju u i ii itri
___


1
1
1
r
1
I 1
Y \ f°rce ° " . force is equal to the rate of 1
^ u o ^ e f ^ n e t i c field energy w.th
1
1 Moving
fw T h T r a tee roff fincrease of' coenergy 1t armature

i j■ ft <e ,s
^ equal
c o n s t„ a " « tna ntt current 0-----------

t f 1 ,!»n d * 8«« correct Fig. C.54


(«) ! ' 3
, »»d 3 are correct [ /A S ., ^ 9 5 ]
(W *' I 4 are correct
(<)?'5 3 and 4 are correct
3 3t l i r — suffers a short-circuit, the adjoining turns of the same winding
id) 1- A
# e n atran
2\7perience
^ a ttra ctiv e force

2; arePulsive force

3 ” ^ a c tiv e force proportional to currents


4 811 i force proportional to currents
5 a repulsive proportional to distance between adioimng turns
6. an attractive o proportion£l to distance between adjoining turns

7^ i v ’Jforce inversely proportional to distance between adjommg turns

r.
From these the
these, tne correct
^ ^ ^statements
? areg
(c) «/) 2, 5, 8

^ ^ Stati0naiT ParaUel ^ “ (Steady) "


, i8 inversely proportional to the separation of wires
2. is proportional to the magnitude of each current
3. satisfies Newton’s third law

Of these statements and 3 are correct


1 and
' )' 1
(a a 2 are correc . 1 22 and 3 are
and 3 are correct
correct
(c)
,c) 1 and 3 are correct W L an W ■]ong ,inear conductor carrying a
!3. Magnetic flux density at a point distant R d
current I is given by 1
U.J
/ n _ iL L = _A L (d)B =
(o)B=- {b)B = (c) B 2 k R*
2 prcK 2R v" 27tfl
M. The energy stored in the magnetic ®eld of a solenmd 30 Stator- pole
'a x is
cm long and 3 cm diameter wound with 1000 turns of
wire carrying a current of 10 A is
(o)0.015 joule (6) 0.15 joule
Rotor-pole
(c) 0.5 joule (d) 1.15 joule a x is
i5- In Fig. C.55, a free rotating permanent magnet R lies
between the poles of a stator. If dc supply is connected
to the terminals AS, the rotor will
(0) start rotating
\f>), oscillate
uouimie about
aoout the
tne stator-pole \axis
stator-,---------
Fig. C.55
(c)1 alien
align itself with the pole
nnle axis of( the stator
(d) have no effect
Electrical

86. In a simple magnetic relay, armature is in open j ^ nsity B and vary tht
yoke under the influence of magnetic force/;, air-gap '

(a) B , increases and f,decreases decreases and


(c) Both increase *
27. In an electric machine, if L = slut length and D - slot depth, then slot leakage flux is proportion,

(6) LD (e) D /L (d > §


(a)
LD
28. In an electric machine, if Z = conductors per slot and Vi = slot width, then the slot leakage a *
is proportional to
W id)
(a )Z /W [b)Z W (c) z -Z W

29. Singly- and doubly-excited magnetic systems are respectively .


(a) loud-speakers and tachometers
(b) synchronous motors and moving-iron instruments
(c) dc shunt machines and solenoids
(d) reluctance motors and synchronous motors
30. Reluctance torque in rotating machines is present when
1. air gap is not uniform ‘
2. reluctance seen by stator mmf varies with rotor movement
3. reluctance seen by rotor mmf varies with rotor movement
4. reluctance seen by the working mmf varies with rotor movement
From these, the correct answer is
(a) 1, 2, 3, 4 (b) 4 only (c) 2, 3 (d) 2, 3, 4
31. Electromagnetic torque in rotating electrical machineiy is present when
(а) air gap is uniform
(б) stator winding alone carries current
(c) rotor winding alone carries current
(d) both stator and rotor windings carry current

32’ i t l t S r l T 0 conversion devices make use o f the magnetic field rather


an tne electric field as the coupling medium. This is because
a) electric field systems present insulation difficulties
(o) electnc Field systems have more dielectric lose the.™ ™ „
rating of the machine magnetic loss for the same power

( S in the s t o r ^ energy density 1. high

33. The total electromagnetic torqUe develop^ T n Z Z ta - a * *


,2 may " e expressed in a general form as

dt dt
Which of the following are represented by the first
side of this equation ? ’ second and third terms on the right-hand
1. Accelerating torque
2. Damping torque
3. Friction torque
4. Load torque
Select the correct combination
using the following codes :
Codes:
First term Second term B — Third term [Cl
. dt2 . dt
(a) 3
2 1 and 4
(b) 3
4 1 and 2
(c) 1
4 2 and 3
(d) 1
2 3 and 4
951

_i pvnression for the instantaneous value of the torque of dynamo-electric machine i


The general exp .
given by
1 n dLi 1 n dLin dl
dM
Te = 2 {il) d 0 + 2 (l2) d 0 + ilt'2 dU

■ and 12are stator and rotor currents respectively


*here 11 ^ ^ are gtator and rotor windings’ self-inductances respectively
mutual inductance between stator and rotor windings and 0 is the angular displacement
jl| iS vile
0f the rotor.
For a synchronous machine (cylindrical rotor), the torque is given by only the third term of the
equation, because
(a) the average value of (ij) and (12) each is zero
(b) the first and second terms cancel each other
dL\ dLo . . . , dLi/dQ (ij)
— - and - r r have positive sign and , =— 2
(c) dQ dQ r "" dL2/dQ
(d) Lj and L2 do not vary with respect to 0
35 electromechanical energy conversion device has uniform air gap. If 5 is the space angle
A ro ta tin g
between the axes of stator field and rotor field, then the average torque developed is proportional
to (A and B are constants)
(a) A sin 8 (6) A sin 25
(c) A sin 8 + B sin 2 8 Id) 5
36. An electromechanical energy conversion device has cylindrical stator but salient-pole rotor. If
5is the angle between the stator field and rotor field, the average torque developed is proportional
to (A and B are constants )
(a) A sin 8 (6) A sin 28
(c) A sin 8 + B sin 25 Id) 8
37. An electromechanical energy conversion device has cylindrical stator but salient-pole rotor. Rotor
is not excited. If 8 is the angle between stator field and rotor-long axis, then average torque
developed is proportional to (A and B are constants)
(a) A sin 5 (8) A sin 25
(c) A sin 8 +B sin 25 (d) zero
38. In an electromechanical energy conversion device, both stator and rotor have salient ^ l e con
figuration. Stator carries a winding but rotor is made from ferromagnetic material. If 5 is the
space angle between the stator-field axis and long-rotor axis, then the average torque developed
is proportional to (A and B are constants):
(a) A sin 8 (5) A sm 25
(c) A sin 5 + B sin 25 5
39- Pour singly-excited electromagnetic structures are shown in Fig. C.56 and labelled I, II, III and
IV (R ; Rotor ; S : Stator).

Fig. C.56.

reluctance torque can be developed in


<Q) I and IV (fe) ij and III fr) HI and ^ ( )

ucii 1loudi 11 it:i


'WIWHl ... . ;;
h

Kl#< iriiiii

■40. Four electromagnetic rotating machines are shown in C.57, These are labelled J( 2,
4 /? :rotor ; S :stator)

Fig. C.57,
These machines raav produce electromagnetic torque (EMT), reluctance torque <ft f ; or both KMT
and RT. ' '
Use the following codes to choose the correct combination
Codes : EMT is produced in Figs. RT in produced in Figs.
(а) 1, 3,4 3,4
(б ) 1, 2, 3,4 1, 3,4
(ci 1,2,3 1,3,4
id' 1, 2, 3 1, 2,3
41. A parallel-plate capacitor has a capacitance of 10 |iF. If the linear dimensions of the plates arn
doubled and distance between them is also doubled, then new value of capacitance would Iks
l a) 1 0 uF (b) 20 pF
(c) 5 uF (c/)40pF
42. A parallel-plate capacitor is charged and then the dc supply is disconnected.Now the plate
separation is allowed to decrease due to force of attraction between the twoplates. As a
consequence
1. charge on the plates increases 2. charge on the plates remains constant
3. capacitance C increases 4. capacitance C remains constant
5. potential difference increases 6. potential difference decreases
7. energy stored decreases 8. energy stored increases
From these, the correct answer is
(a) 1, 3, 6 lb) 2, 4, 8 (c) 2, 3, 6, 7 id) 2,4, 5, 8
43. A parallel-plate capacitor is charged and then the dc supply is disconnected. The plate-separation
is then increased. Between the plates,
1. electric-field intensity is unchanged 2. flux density decreases
3. potential difference decreases 4. energy store', increases
From these, the correct answer is
(a) 1, 4 (6) 2, 4
<c) 2, 3, 4 id) 1, 3, 4
44. A parallel-plate capacitor is connected to a dc source. Now the plates are allowed to move a small
displacement under the influence of force of attraction between the two plates. As a result
1. charge on the platesincreases 2. charge on plates remains constant
3. energy stored increases 4. energy storedremains constant
5. electric-field intensity is unchanged 6. flux density increases
From these, the correct answer is
(Q) 2- 4. 5 ib) 2, 3, 6
W 1. 3- 5 (d) 1, 3, 6
45. Area of two parallel plates is doubled, distance between these plates is also doubled but capacitor
voltage is kept constant. Under these conditions, force between the plates of this capacitor
(a) decreases (b) increases

Scanned by Cam Scanner


' ap£ 953
reduces to half
Id) Rets doubled
, Match t o items on the .eft side with the most appropriate f,*ure on th„ ^ ^

Torque

Only reluctance torque is developed


A-

0. Electromagnetic torque alone is developed

C. No torque is developed

D. Both electromagnetic and reluctance torques


are developed

Ans.A-», C -> , D->


W. Two conductors are carrying forward ant
magnetic field intensity H at point P is

+1 -I
P
-►x © ©

h 4 .

Fig. C.58,

(“) ~~~:y
7id J (b)h x
(c) (id) ■x [GATE, 2003]
2nd

48‘ A Parallel plate capacitor has an electrode area of 100 mm2, with a spacing of 0.1 mm between
electrodes. The dielectric between the plates is air with a permittivity of 8.85 x 1 0 "12 F/m .
/he charge on the capacitor is 100 V. The stored energy in the capacitor is
W 8.85 p J (6) 440 p J
(c) 22.1 n J {d) 44.3 n J [GATE, 2003\

Scanned by Cam Scanner


ri <i iniftTrwi

Electrical Madji
954

ANSW ERS
4. (d) 5- («0
1. (a) 2. <o0 3. (c)
9. (6) 10. (6)
6. (a) 7. (c) 8. (c)
to (a) 14. (6) I5- <«)
11. (d) 12. (c)
16. (a) 1 7 .(a) Is £ 19 20' w
21. (a) 22. (d) S3 c) »• <» 25
26. (c) 27. (b) 28 a 29 <rf) 22’ '*
33 S 34. (<0 35. (a)
31. (rf) 32. (d)
38 f 39. 60 40. W
36. (c) 37. (6) 44 60 45.
41. (6) 42. (c) 43. (a)
46. A -> 3, B -> 4, C -> 1, D -> 2. 47. (a) 48< (d)

?>v

r -\ :

Scanned by Cam Scanner


https://t.me/abcdelectrical
::js

BASIC CONCEPTS o f ROTATING ELECTR\c I T Z --------


. The torque angle 8 is defined as the angle between MACHINES
,o) stator field axis and resultant field axis
,f>) rotor field axis and resultant field axis
stator field axis and rotor field axis
stator field axis and mutual field axis
j The interaction torque depends on
■ a lator field strength 2. „ tor fleld h
4. sin 0 5. cos 8 d- tor<Uie angle 8
From these, the correct answer is
(a) 1, 2, 4 (6) 1, 2, 5 (C) 1, 2, 3 j 2
3. Armature winding is one in which working
(a) flux is produced by field current , n j ,
ic) emf is produced by the workingflux ,j UX, !S p ucecl b-v the working emf
. . . «*> emf is produced by the leakage flux
4. Large synchronous machines are constructed with armature winHinr, ,, . , '
stationary armature winding nding on the stator because

I hiBher vol,“«es 2- be cooled morc efficiently


3. won d lead to reduced shp-nng losses 4. would have no sloT h "moo os
5. would have reduced armature reactance
From these, the correct answer is
(a) 1, 2, 3, 4 (6) 1, 2, 3, 5 (c) 1, 2, 3, 4, 5 (d) 1, 2, 4, 5
5. A pole pitch in electrical machines is
(a) = 180 electrical = 180" mechanical
(c) > 180° electrical (d) < 180° electrical
6. For a P-pole machine, the relationbetween electrical and mechanical degrees is given by

(a) 8<*c = p ^mech (b) Op/pr = p Onifc/i


p p
(C) ^mech ~ rjr ' ®elec (®0®Wrr = 2^‘ ®nic(7i
7. A coil consists of
(a) two conductors (b) two coil-sides
(c) two turns (d) four turns
8. One turn consists of
(a) two coil-sides (b) two conductors
(c) four conductors (d) four coil-sides
9- In ac rotating machines, the generated or speed emf
(a) is in phase with theworking flux <{> (b) leads $ by 90
(c) lags <j>by 90° (d) lags * by 180
10. A winding is distributed in the slots along the air-gap periphery
1. to add mechanical strength to the winding
2. to reduce the amount of conductor material required
3. to reduce the harmonics in the generated emf
4. to reduce the size o f the machine
5. for full utilization of iron and conductor materials

From these, the correct answer is id' 1 3 5


(o )l, 3, 4, 5 (d) 1,2 , 4, 5 (0 1 .2 ,3 .5 V * 1' * *
R . Consider the following statements alwutthecho *
1. chorded coils reduce the magnitude of output voltage
2. chorded coils lead to reduction l"J ® ™ f° " aveform
3. chorded coils give better generated

Scanned by Cam Scanner


956 * Electrical Machi

4. chorded coils require more conductor material


5. in chorded coils, coil span < 180° electrical
6. in chorded coils, coil span may be less or more than 180° electrical.
From these, the correct statement is _ _ _
(a) 1, 2, 3, 4, 5 (6) 1, 2, 3, 5 (c) 1, 2, 3, 4, 6 (d) 2, 3, 5
12. If Bp - peak value of sinusoidal flux-density wave, L = armature core length, D = armature
diameter, P = number of poles, then
1. Flux per pole, <j>= ^ Bp ■L D 2. Average flux-density, Bau = - ■ Bp

3 ! Bp L D 4

5 Q C D _ P
JtflL 6' B™ - n rl
From these, the correct answer is
(a) 1, 2, 5 (b) 2, 3, 5 (c) 2, 3, 6 (d)3, 4, 5
13. The belt factor is defined as the ratio of
(а) arithmetic sum of coil emfs to the phasor sum of coil emfs
(б) phasor sum of emf per coil to the arithmetic sum of emf per coil
(c) phasor sum of coil emfs to the arithmetic sum of coil emfs
(d) phasor sum of coil emfs to the per phase voltage
14. A fraction-pitch winding is used to reduce
1. amount of copper in the winding 2. size of the machine
3. harmonics in the generated emf 4. cost of the machine
From these, the correct answer is
(a) 1, 2, 3, 4 (6) 1, 2, 3 (c) 2, 3, 4 (d) 1, 3, 4
15. The pitch factor, in rotating electrical machinery, is defined as the ratio of resultant emf of a
(a) full-pitched coil to that of a chorded coil (6)full-pitched coil to the phase emf
(c) chorded coil to the phase emf (d) chorded coil to thatof a full-pitched coil
16. For a 3-phase uniformly distributed and narrow-spread winding, the distribution factor is
(а) 0.9 (6) 0.955 (c) 0.827 (d) 0.97
17. The armature winding of a 2-pole, 3-phase alternator for each phase is distributed in a number
of slots per phase. The rms value of the voltage per phase is less than the rms value of the voltage
per coil multiplied by the number of coils in series because the
(o) rms value of the voltage in different coils of the phase is different
(б) equal rms voltages in different coils of the phase has mutual phase difference
(c) maximum value of the induced voltage in different coils of the phase are different
(d) different coils of the phase pass through different saturated regions of the magnetic circuit.
18. A 3-pliase, 4-pole alternator has 48 stator slots carrying a 3-phase distributed winding. Each
coil of the winding is short chorded by one slot pitch. The winding factor is given by
cos 7.5 . c o t 7.5 1 ...c o t 7.5
(0) 16 {b> 8 (C)8^taT 5
19. The windings for a 3-phase alternator are :
1. 36 slots, 4 poles, span 1 to 8
2. 72 slots, 6 poles, span 1 to 10
3. 96 slots, 4 poles, span 1 to 21
The windings having pitch factor of more than 0.97 are
(a) 1 and 2 only (6) 2 and 3 only (c) 1 and 3 only (d) 1, 2 and 3
.
20 Consider the following statements regarding the design of distributed armature winding in 8
3-phase alternator :
1. It reduces the phase-belt harmonics
2. It increases the utilization of the armature iron and copper

?T%
*Y
J
______________________________________________957

^ increases rigidity and mechanical strength of the winding


3 It reduces copper in the overhang of the winding .
. irh of these statements are correct ?
! i 3 and 4 (6) 2, 3 and 4 (c) 1, 2 and 3 (of) 1, 2, 3 and 4
if the dimensions of all the parts of a synchronous generator, and the number of field and
2l' moture turns are doubled, then the generated voltage will change bya factor of
(b) 2 (c)4 «f) 8
rjy0 3-phase ac generators are such that one has twice the linear dimensions of the other. The
field windings of each are excited to give identical sinusoidal air-gap flux density waveform. Both
have the same number of stator slots and identical winding patterns. The conductors per slot in
big generator is K times that of the smaller one. The value of K to get equal no-load voltage at
same frequency is
«.) 8 4 <•> \ «> \
23. Two transformers of the same type, using the same grade of iron and conductor materials, are
' designed to work at the same flux and current densities ; but the linear dimensions of one are
two times those of the other in all respects. The ratio of kVA ratings of the two transformers
closely equals
(a) 16 (&) 8 (c) 4 {d) 2
24. For eliminating nth harmonic from the emf generated in the phase of a 3-phase alternator, the
chording angle should be
(a) n x full-pitch (6) — x full-pitch

(c) - x full-pitch (d) — x full-pitch


n n
25. For eliminating nth harmonic from the emf generated in one phase of a 3-phase alternator, the
coil-span (pitch of the coils) must be
(a) x full-pitch (b) — — - x full-pitch
n n
2 71
(c) x full-pitch (d) r X full-pitch
n+ 1 n+1
26. For eliminating 5th harmonic from the phase emf generated in an alternator, the coil-span in
terms of full-pitch (or pitch fraction) would be

(a) | (6) | W f
27. In 48 slot, 4-pole, 3-phase alternator, the coil-span is 10. Its distribution and pitch factors are
respectively
(o) 0.9717, 0.966 (&) 0-9822, 0.9814
(c) 0.9577, 0.9814 0.9577, 0.966
28. In an alternator, frequency per revolution is equal to
(a) number of poles I twice the ™ ^ ber ° f P°les
(c) speed in rps (d) number of pok-pairs
29- A 6-pole machine is rotating at a speed of 1200 rpm. This speed in mechanical rad/sec and
electrical radians per sec is respectively
(a) 40 7i, ^ - 2 120 n>40 71
(c) 20 n, 60 jt &) 40 n, 120 Jt
90. The short-pitch winding for an alternator gives
!• improved waveform of the generated emf per phase
2. reduced value of self-inductance of the winding
3- increased rating of machine
4‘ educed tooth ripples

Scanned by Cam Scanner


* -« -I.

958
Jllectri<-al Machir
5. increased total generated emf
6. saving in winding copper
From these, the correct answer is
(a) 1, 2, 4, 5, 6 (6) 1, 2, 6 (c) 1, 2, 4, 6 (d) 1, 3, 4, 6
31. A 3-phase machine has integral slot winding with fundamental distribution factor k^. p0r
>
harmonics, the distribution factor is
| (Q) less than kffi (6) morethan k^i
lc) equal to Arfl (rf)depends upon the number of slots and poles
32. Consider the following statements :
1. breadth factor for third hannonic is more than that for fundamental k^i
2. krfs < /trfl
3. may be less or more than k depending upon the number of slots and poles
4. coil-span factor for third harmonic kp3 > Apl (coil-span factor for fundamental)

5- Ap3<Api
6. /tp3 may be less or more than kpl depending upon the number of slots and poles.
From these, the correct answer is
(a) 2, 5 (6) 1, 4 (c) 3, 6 (d) 1, 3, 4, 6
33. In order to minimise both fifth and seventh harmonics, the coil-span in three-ptase ac machines
must be
(a) 144° (b) ^th of pels pitch
5
(c) -t h of pole pitch (rf) 120°

34. A synchronous machine has full-pitch coils having coil-span of 12 slots. For eliminating third
harmonic, the coil-span should be
■■ (a) 6 slots (6) 8 slots (c) 9 slots (d) 10 slots
35. Machine A has 60° phase spread and machine B has 120° phase spread. Both the machines have
1 uniformly distributed winding. The ratio of distribution factors of machine A to machine B is
(a) 0.866 (b) 1.1 (c.) 1.55 (d) 1.155
36. The emf generated in an alternator depends upon
1. speed 2. maximum flux per pole
3. series turnsper phase 4 . phase spread
5. coil-span 6 . type of alternator
From these, the correct answer is
(a) 1, 2, 3, 4, 5 and 6 (6) 1, 3, 4f 5
(c) 1, 2, 3, 5, 6 (d) I, 3, 4, 5, 6
37. A 3-phase alternator is wound with a 60 degree phase spread armature winding and devekps
300 kVA. If the armature is reconnected utilizing all the coils for single-phase operation with a
phase spread of 180 degree, the new rating of themaehine is.
(a) 100 kVA (6) 200 kVA (c) 250 kVA (d) 300 kVA
38. A 3-phase alternator is wound with a 60 degree phase-spread armature winding and develops
300 kVA. If the armature is reconnected utilising all the eoils for two-phase operation, the new
rating of the machine would be
(a) 282.8 kVA (b) 259.8 kVA (c) 200 kVA (d) 173.2 kVA
39. The armature of a single-phase alternator is completely wound with T single-turn cods di«'
tributed uniformly. The induced voltage in each turn is 2 V (rms). The emf of the whole wiadw£
is
(a) 2 T volts (6 ) 1.11 T volts (c) 1.414 T volts (d) 1.273 T volts [GATE,

Scanned by Cam Scanner


Appendix—C
959
40. The armature of a star-connont j " ~ ----------------------- ------------------- ------------------------
N full-pitched turns. The g "ne“ j ‘te™ ator 18 “ "tformly wound with T coils, each coil having
(u lfw T v o l* (6 ) 4 VT 6 P6r C r 181 V ,rmSl The Per Pha38 emf is
“ n ( C ) = O T volts ( d ^ w r volts
41. A 6-pole alternator with 36 slot
by ® t 15“ ^ Winding factor “ JiJen by'Pha8e distributed windin8 « d is short-pitched
|a) T 7 T <*> (c) c o t l £ cot 15­
42, The chortling angle for *!«_.- 2 6
(0) 30° 340 lnatmg 5th harmonics should be
43. I K ,. * , and B , are respectively t h e i r , (d) 3*°
generaW jneachphsss of a sfaw.J ! ^ ? 1? Woffundam4nt»l. and fifth harmonic voltages
{<*) 3 + £3 + alternator, then the line voltage is
(c) V3 (b) V3 ^ E 'i+ E $ + E (

44. In a 3-phase ac machine, phase emf • *- ^


emf ia e , and output is P2 for a w i d ^ p ^ H ^ V 1? “ “ ""w -sprcad, whereas phase
(a) E i - E2 , Pl = 1.15 p2 H •Mere E h E2, P x and P2 are related as under :
(c) E2 = 1.15 ; Pl = 1. 15p 2 (6) ® i = 1 .1 5 £ 2 ; P1 = 1.15P2

45. Mmf produced by one AT-turn coil carr™ f * ®* = ; = 1 1 5 Pi


(a) rectangular of amplitude — * * 118
(c) rectangular of amplitude Ni ^ trapezoidal of ^ P ^ u d e ^

4& , l k ™ ,tK ° f fi,nd™ Of mmf p m a u l «


4 „ °ne iV-turn cod carrying a current i
( a ) — • Ni /fc\ ± a t ; 1 ,
■ (b) a N ‘ C c )i iNw J L m

47-A winding of 20 full-pitched series ,,.J 1 a- . •. 2* '

(a) . . . 120 on
* (6) —Jt (c)
W V ? rrfx ^60
(d)

J|*» o f s i n u S ^ ^ i d ^ 116 eleCtriCSl madline has **** along q-axis. The peak
5 S . * * lt8e. ^ Peak ^
i I ortl I8*®"*® current peak
(c) 90 leading the current peak
amr *ag or lead depending
,4|j *---------------------------- * -------- *> upon the type of uiacmne
machine
distributed witting on the stator has three full-nifcrk«H 1
(a) sim vTT tUni carr7™S a cement i. The mmf produced by this wfod?’ having N
a sinusoidal m waveform with an amplitude 3 Ni V W,nd,ng »
1
/ICI sinusoidal
. rfflftAaAiJ
. in waveform with an amplitude 3 Ni/ 2
■.* " ‘ W _an
_ duipiainR OJVI/
m1 - __ ^ _
frf! ;rape20!daI in waveform with an amplitude 3 Ni
WJ trapezoidal in waveform with an amplitude 3 Ni/2
armature mmf wave in a dc machine is
(a) sinusoidal and depends on the speed (6) square and «

W t^ d a r n n d i^ e tX e e d
1 artaature mmf wave m a dc machine is triangular and
1- rotates with respect to stator
*• w stationary with respect to stator
3. rotates with respect to armature
18 stationary with respect to armature

W I I IV_yV/UI II IVyI
960

From these, the correct answer is


(a) 2 alone (6) 3 alone (c) 1, 4 (d) 2» 3
52. A 3-phase star-connected winding is fed from symmetrical 3-phase supply with their neutraU
connected together. If one of the three supply lines gets disconnected, then the revolving mmf
wave will have a
(а) constant amplitude, but subsynchronous speed
(б) varying amplitude, but synchronous speed
(c) constant amplitude, but supersynchronous speed
(d) varying speed and a speed fluctuating around the synchronous speed
53. IfFj is the constant amplitude of fundamental rotating mmf wave, then for the space harmonics
of order 6 A: + 1 {where k = 1, 2, 3 ...), the harmonic mmf wave is of
(а) constant amplitude and stationary in space
(б) constant amplitude and rotates along Fj
(c) varying amplitude and rotates along Fj
(d) coustant amplitude and rotates against Fx
54. The fifth harmonic mmf wave, produced by 3-phase currents flowing in 3-phase balanced wind­
ing, rotates with respect to the fundamental field {Ns = speed of fundamental mmf wave) at a
speed of
(Q) | Ns (6) <c)f H. (d )^ N s

55. The seventh harmonic mmf wave, produced by 3-phase currents flowing in 3-phase balanced
winding, rotates with respect to the fundamental field (Ns = speed of fundamental mmf wave)
at a speed of

G O ® *, (6) | W , (c )| w , (d )| w .
1
56. In a dc machine ; Z, P, Ia and a are respectively the number of conductors, number of poles,
armature current and number of parallel paths. The peak value of fundamental component of
armature mmf wave is
Z z
2P p
z a
2P a
57. In a rotating machine,
1. R ■sin (cot - a) is the backward rotating mmf
2. R ■sin a is the standing mmf
3. R ■sin (tot + a) is the backward rotating mmf
4. R sin tot cos a is stationary mmf
5. R cos cot cos a is stationary mmf
6. R sin (tot + a) is pulsating standing mmf
From these, the correct answer is
(a) 2, 3, 5 (b) 1, 2, 4, 5 (c) 2, 3, 4, 5 (d) 1, 4, 6
58. Chording and distribution of armature windings in rotating machines
1. result in reduction of air-gap mmf harmonics
2. need more number of turns for a given voltage
3. result in an increase in the number of effective turns
4. result in the requirement of less copper in the overhang
5. result in the reduction of slot harmonics
From these, the correct answer is
(a) 1, 2, 5 (6) 1, 3, 4 (C) 2, 3, 5 (d) 1, 2, 4

Scanned by Cam Scanner


https://t.me/abcdelectrical

50I. A 3-p h ?». 5° Hz squirrel cage induction motor has stator I,


stator ib cut and laid out flat, the speed of travelling field (tat o o ^ m e te r of 0.5 m. If its
)a
l 20 m /s im on / " - 3 -2) is
c 40 m /s * 30 m /s
{C‘ . (d)50 m /s
0. Consider the principle that the torque in a rotatine machine
60. .
mmf (F,), rotor mmf lFr) and the angle between them (5? m " ’” ' " ™ 1 t0 peak ° f stal°r
v
pftFr nd
andFe 8 in
5 in different
different : .
machine.
machines ” (6)' F° 11owing
<”« n g statements relate to
1. 5 is fixed in dc machine. ow e. j •
3. 5 is variable in induction machine a p ™ !D dc slJunt machine
5. Fr is variable in dc machine p •_ 8 , m fy^chronous machine
7. Resultant of Fs and Fr is fixed in synchronous motor " ” “ UCtl° n m° t0r
Of these statements :
(a) 1, 2, 3, 4 and 6 are correct /M i 9 q * Q„,i n
(C) 2, 3, 5, 6 and 7 are correct 5 ' I' I ’ I 7 are con-ect
' \a) l, l, 4, 5 and 7 are correct
61. Output rating of
1. induction motors is in h.p. 2. ac mottm js . k
3. ac generators is m kVA 4. dc motora js . Rw
5. synchronous motors is m kVA 6. dc generators is in kvA
From these, the correct statements are
(c) 1, 3, 4, 5 (6) 1, 2, 3, 4 (c) 2, 3, 4, 6 (d) 2, 3, 4
62. If 0 is the maximum value of flux due to any one phase, then resultant flux in 2-phase and
3-phase ac machines would respectively be given by
(Q) 0m and 1-5 <)>m; both rotating
(&) 0m and 1.5 0m ; both stand still
0m •standstill and 1.5 <|>m rotating
(d) 1.5 tpm and 2<pW
I ; both rotating

I t o t^ fie T d Z d tto r “ eldmif meS' eleCtriCi" torqUe “ whcn • * "« "


(a) zero
(fe) equal to rotor speed
(c) equal and opposite to rotor speed
(d) dependent upon the type of electrical machine
64. No-load rotational losses in electrical machines consist of
(a) Friction and windage (F and W) losses (fe) stator core, F and W losses
(c) rotor core, F and W losses (d) no-load core, F and W losses
65. Consider the following statements regarding efficiency of electrical machines :
1. Efficiency should be calculated by measuring output and input
2. Efficiency is maximum when constant losses = variable losses
3. Electrical machines are designed to have maximum efficiency at full load
4. Electrical machines are designed to have maximum efficiency at near about full load
5. Efficiency should be calculated by measuring the losses
6. Efficiency is maximum when constant losses =x (variable losses)
From these, the correct answer is
(g) 1, 3, 6 (6) 2, 4, 5 (c) 1, 2, 3 (d) 4, 5, 6
66. If P, be the iron loss and P0 be the ohmic loss on full load, then which of the following conditions
has to be satisfied to obtain maximum efficiency at 3 /4 full load ?
(a)P„ = 3 P ,/4 (fe) P„ = 4 P /3 (c)P0 = 16 P /9 (d) PQ= 9 P /1 6
67. For estimating the power rating of an electric derive, its losses can be considered to be proportional
to
(a) power (fe) (power)2 (c) (power)3 (d) torque

/ ' ■* ]

Scanned by Cam Scanner


962
. , • Jofimi temperature limit for class B type is
Electrical MacFI
68. In rotating electrical machines, the insulatio 180°C
. . ,(6)
i, 130°C (c) 150°C v 1
(a) 105DC

Rest
10 kW
Load : 20 kW
60 s
120 s
Duration : 120 s
c iuwuv* ------ *.
(b) 16.4 kW
(a) 20 kW (d) 15.81 kW
(c)
(c) 14.14
14.14 kW
kW temperature rise in one hour. Its heating
70. A machine attains a temperature rise of 60% ot its iina f

time constant is ^ 0.9163 hr


(a) 1.0914 hr {d) t 958i hr
(c) 0.513 hr v
71. .(l 0 3X e induction motor, if the anguiar s p e e d . o f , ^
speed
speed ofof the
the rotor is 0),
to,ot is and the
to,, and stator inauc
the stator ^ t b ^ p h -& ^
wi„ bg
be « J . ^
stator
turns per phase), then the rotor emf with A 2 as tne tolu f
winding factor and K „ - rotor winding factor), ;

(c) er = iV2 ((0 - cor) A„ir $ •sin (co - to,.) /


[I.A.S., 1997]

72. In hydrogen-cooled synchronous machines


1 pressure inside the machine is more than atmospheric pressu
2.' pressure inside the machine is less than atmospheric pressure
3 heat-storing capacity of hydrogen is more than that ot air

t f i ^ S T t h e m a c h i n e is minimised because hydrogen content is more than 70*.

From these, the correct answer is


( - > 2 , 3 , 4 and 5 rf ' 4 and 5
(c) 1, 3 and 5 (rf) 4’ an
73. Match List I with List II and select the correct answer from the codes given below

List I List II

(Applications)
(Type o f load torque on motor)

continuous loads 1. punch press


A.
2. sluice gates
B. pulsating loads

impact loads 3. centrifugal pumps


C.

short-time loads 4. reciprocating pumps


D. • , 7* • ; * . iy

Codes: A B C D ■•-'y
.. ■. ;• ’r< •v; -
(a) 1 2 3 4

(*) 3 4 2 1
'j ; ' •
(c) 3 4 1 2
•4 ; ' .
id) 3 1 4 2
-■
- ■■

Scanned D yuam S canner


Appendix C 963

74. The worst type of load on a supply system is


(o) rolling mill load (6) pumping load
(c) motors in paper mill (d) arc furnace load _
75. List I represents the different types of electric motors, whilst list II gives some of their charac­
teristics.

List I List II

A. DC series motor 1. constant speed

B. DC shunt motor 2. high-starting torque

C. 3-phase induction motor 3. fairly constant torque

D Synchronous motor 4. lower-starting torque

5. pulsating torque

6. very poor stability

Use the following codes to choose the correct answer :

Codes. A B C D
(a) 1 3 4 1
(b) 1 2 3 5
(c) 2 1 4 6
(d) 4 3 1 2
76. Match the items in the left-side column with t
Power losses
Dependent upon
A. No-load rotational loss
1. Square of load
B. Hysteresis and eddy-current losses
2. Rotor rotation
C. Brush contact loss
3. Value of flux
D. Stray load loss
4. Load
Ans. A B C -> , D —*
77. The sinusoidal distributed balanced 3-nhase
balanced 3-phase currents : shown m Fig. C.59 are supplied with
iA = / msin ((o f-y 6)
5jc '
rB = / in sin
v
ic ~ Im sin (co* + yfc)

M ^ angular in start o* = | the value ef angular position 9 of the rotating m m [F with respect
to phase A axis will be
(a)
(6) V i
(c) Ve
w (d)O - f
(
7B' Sinusoidal distributed balanced 3-phase winding shown in Fia r an • ,
3-phase currents : wmmng snown in fig. C.60 is supplied with balanced

iA = Im sin (to*+ *fc)

S c a n n e d Dy u a m bca n ne r
Lljft&ji-». 1;i.aJ:ia!iSaSS6IfIIIBTw
ifltlifiTliTl

Electrical Mathi
964

V c
N. r
•Zy
c.s '

Fig. C.60.
Fig. C.59.
iB = / msin (tot - %)
ic = lm sin (tot + 5)F6)
The rotating mmf would be directed along the axis of phase C when tot equals
, . 2n id) f
w -f (C)T
<■ >!
79. Following are some of the properties of rotating electnca mac 1 ■
P Stator winding current isdc, rotor winding current isac
Q Stator winding current isac, rotor winding current isdc
R Stator winding current isac, rotor winding current isac
S Stator has salient poles and rotor has commutator
T Rotor has salient poles and sliprings and stator is cyhndrica
U Both stator and rotor have polyphase windings
DC machines, synchronous machines and induction m achines exh ibit some of the above proper­
. « "w i • il ~ • n r m m ■h 1Cl S IP
j ■

DC machines Synchronous machines Induction machines

Q.T R, U
(a) P. s
Q. U P, T R, S
(b)
(c) PS R, U Q, T

R.S Q, U P.T
(d)
[GATE, 2003}

80 When stator and rotor windings of a 2-pole


rotating electrical machine are excited, each
would produce a sinusoidal mmf distribution
in the air-gap with peak values Fs andFr
respectively. The rotor mmf lags stator mmf by
a space angle 5 at any instant as shown in Fig.
C.61. Thus, half of stator and rotor surfaces
will form one pole with the other half forming
the second pole. Further, the direction of
torque acting on the rotor can be clockwise or
counter-clockwise
The following table gives four sets of state­
ments as regards poles and torque. Select the
correct corresponding to the mmf as shown in
Fig. C.61.

ScannetTby Cam Scanner


Stator surface ABC
forms

North pole_________
(a)
South pole_________
(M
North pole
(c)
Id) South pole

[GATE, 2003]

answ ers
1. (C) 2. (a)
6- (d) 3. (c)
7. (6) 4. (6) 5. (a)
11. (b) 8. [b)
12. (a) 9. (c) 10. id)
16. (b) 13. (c)
17. (6) 14. (d) 15. (d)
21. (d) 18. (6) 19. (c)
22. (d) 20. (c)
26. (a) 23. (a)
27. (d) 24. {b) 25. (a)
31. (c) 28. (d)
32. (a) 29. (d) 30. (6j
36. (6) 33. (c)
37. (6) 34. (fe) 35.
38. (a) (<d)
41. (a) 42. (C) 39. (d) 40.
43. (d) (6)
46. (6) 47. (a) 44.(6)
48. (C) 45. (a)
51. (d) 52. (d) 49. (d)
5 3 . {b) 50. [d)
56. (d) 57. (a) 54.(6)
58. (d) 55. (a)
61. (d) 62. (a) 59. (c) 60.
63. (a) (5)
66. (c) 67. (6) 64. (d) 65.
68 . ( 6) (5)
71. (c) 72. (d) 69. (c) 70.
73. (c) (a)
76. A - 2, B 74. (d) 75.
' 3, C -> 4, D 1. 77. (d) (c)
80. (c) 78. (6) 79. (a)
966 Electrical Machm^,

D.C. MACHINES
1. In dc machines, the armature windings are placed on ^ e r°t°r because of the necessity f0r
(a) electromechanical energy conversion development of torque
(c) commutation 1’
2. In dc machines, the fieldsystem has to be provided on stator unlike synchronous machine
wherein it could be on any member, because
(а) it reduces field structure iron losses
(б) it gives more uniform air-gap flux distribution
(c) commutator action is not possible otherwise
(d) dc machines are comparatively low rating machines. i . . - >96]
3. In dc machines, the space distribution of air-gap flux density wave at
(a) sinusoidal (c) co-sinusoidal (c) flat-topped (d) rectangular

4. A commutator in dc machines can Drovide full-wave rectification


1. provide half-waverectification ■^ dc tQ ac
3. convert ac to dc ’
5. provide controlled full-wave rectification
From these, the correct answer is ^ ^ ^ # ^ ^ 4 ,

5. In dc machines, the space waveform of the air-gap torque

$ b7 h f t e UtlTueandVv o S e «D the voltage nor the torque


6. In dc machines, the field-flux axis and a rm atu m -m m f ^ s ^ r ^ v e ^ n g , ^
(a) direct axis and indirect axis quadrature axis and interpolar axis
(c) quadrature axis and direct axis W qu«*ui«i
7. The brush-axis of^a bipolar
• n dc j motor
* is rotated kv
by qn°
yu . The
ine effect
enect of
ui this
u rotation on the back emf
Eb and the torque developed Td would be such that
(a)both E„ and Td are unchanged <» *6 » » " > . but T« “
<c, E„ is unchanged, but Td is zero t * both E„ and Td are zero ^

of 0 2 Q The current through the load will be


(a) 3.33 A (6)23.33 A (cj 5 A (d) 1 A
9. Consider the following statements regarding the magnetization characteristics of a dc generate.
1 Tt rpnresents the variation of field flux with field current. r ,
2. Variation of open-circuit terminal voltage with field current is independent of speed
3 It is non-linear from zero to rated field current
4'. It helps to determine the critical field resistance at a given speed
5. It always starts from the origin
Which of these statements are correct?
(a) 1 and 4 (6)1. 2 and 5 (c) 2, 3 and 4 (d) 3 and 5
10. A dc shunt generator builds up to a voltage of 220 V at no load while runnmg at its ra e ^
If the speed of the generator is raised by 25% keeping the circuit conditions unaltereu,
voltage to which the machine would build up will
(а) not changeand remain at 220 V
(б) increase to 1.25 times 220 V
(c) increase to value lying between 220 V and1.25 times 220 V 19951
[IA.S.
(d) increase to a value greater than 1.25times 220 V.
11. The armature core of a dc machine is laminated to minimise
(a) hysteresis loss (fe) eddy-current loss
(c) mechanical loss (d) temperature rise
— mjm A U tu iitu itS O tu m in'niiTift—i‘ ~4k^ “•1

19 The commutator segments nf j . , .


1 (fl) brass ° f dc macb*nes are made up of
(c) hard-drawn copper (b) copper

13. consider the following statement* regerdine a f ' ^ a" ° y


1. Possible resistance of shunt-fieM ; T g dc machines :
2. Possible resistance of r u ndm£ is 1 £2
3. Electrographite brU r ,i r inding 18 1 “
4. Copper-graphite b r u s h e s ^ ' ^ a V 2«m V’ 10 A machines
5. Possible armature-circuit re ■ f is 1
c Clrcuit resistance ? °£2V’ 10 A machines
From these, the correct stated *
W M (‘ ) 2 n tem~ 45 „
14. A 4 pole dc generator runs at isnn ’ mu a. *’ 3’ 5
(a) 25 Hz (6) 5() H HpaT]be frequency of current in the armature winding is
15. In normal dc machines ooeraf (d) 100 Hz
(а) field winding lng at fu»-load conditions, the most powerful electromagnet is
(б) armature winding °
(c) interpole winding

1. the field circuit is open to bmld up’ because


3. the residual magnetism is absent 2' th® armature circuit is open
From these, the correct answer is 4' ^ direction of rotation is wrong
(a) 3 only (&) j 3

17. A dc shunt generator builds up 230 V whe V " 1* 4 . W) 2 and 3

“ wise dirccti“- *<■ » f ; iven in


(■c) somewhat more than 230 V ^ somewhat less than 230 V
18. A separately-excited dc generator run„' . P° SSib'e
terminals with no field current When S a Speed’ has 6 volte across its armature
builds up to 250 V at no l o a d £ case'fiafd"cuZt 5 ^ ampare'
(a u fv armature terminal voltage would be ° £&° ’ W the speed remaining
(a) 6 V
(c) more than 6 V less tban 6 v
19 A Hr. (a); zero
1 zero

fa) f drCUit V° ltage 0f


If resis ance was higher than critical resistance ’ baPP®ned because
J .1 was no residual magnetism in the field circuit
winding was wrongly connected
(a) there was a break in the armature circuit

20* S,hUnt gGnerat0r having a shunt field of 50 £2 was generating normally at l nnn m
critical resistance of this machine was 80 £2. Due to some reason, the soeed^if tK •rpm' e
became such that the generator just failed to generate. 6 pnme' mover
The speed at that time must have been
(a) 1000 rpm (6) 800 rpm (c) 625 rpm [d) 500 rpm
21. A dc shunt generator, when commissioned after a major repair of the field system failed *■ u . j
up voltage. It was because ’
(a) field resistance was higher than critical resistance
(b) field system lost residual magnetism
(c) field winding was wrongly connected
(d) there was a break in the armature circuit

Scanned by Cam Scanner


https://t.me/abcdelectrical

w
Electrical Machin.^
i 968
i
4 • j ■ ifo rat-pH sDeed. The no-load voltage across its armah,
22. A 230-V dc series generator is driven at its rated speea armature
i
terminals would be s01Tiewhat more than 230 V
(a) 230 V ° 6V
(c) somewhat less than 230 V 1

23' generator,Mother things*

the same, it would build up ^ 220 V


?! 6V Oon v id) somewhat less than 220 V
(c) more than 220 V K’ t
24. A belt-driven cumulatively-compound dc generator is delivering power to the dc mains. If the
1 belt snaps then the machine will run as a
(а) cumulative compounded motor in the same direction
(б) differentially compounded motor in the same direction
(c) cumulative compounded motor in the opposite direction
(d) differentially compounded motor in the opposite direction
25. In a dc machine, ,
1. physically, the brush axis is along the direct axis
2. physically, the brush axis is along the interpolar axis
3. schematically, the brush axis is along the direct axis
4. schematically, the brush axis is along the quadrature axis
5. armature mmf is directed along the brush axis
From these, the correct statements are
(a) 1, 4, 5 (b) 1, 3, 5 (c) 2, 3, 5 (d) 2, 4, 5
26. The direction of generated emf in the armature coil of a dc machine is
1. the same as that of the current in a generator
2. the same as that of the current in a motor
3. opposite to that of the current in a generator
4. opposite to that of the current in a motor
From these, the correct answer is
(a) 1, 3 (b) 2, 3 (c) 2, 4 (d) 1, 4
27. The counter (or back) emf in a dc machine is
1. the same as generated emf in a dc motor
2. proportional to field flux and armature speed
3. not the same as generated emf in a dc motor
4. inversely proportional to field flux
5. proportional to number of armature conductors
6. constant from no load to full load
From these, the correct answer is
(a) 1, 2, 5 (b) 2, 4, 5, 6
(c) 2, 3, 6 (d) 3, 4, 5

28. A 220-V dc machine has an armature resistance of 1 £2.


If the full-load current is 20 A, the difference in the in­
duced voltages when the machine is running as a motor
and as a generator is
(a) 20 V (b) zero
- (c) 40 V id) 50 V
29. The external load characteristics of a dc generator at
rated speed is assumed to be linear and is as shown in
Fig. C.62. The load circuit comprises only resistance.
When the load current is 50 A, the value of the load 50A
circuit resistance will be Load c u rre n t in Am p'
(a) 0.9 £2 (b) 1.0 £2
(c) 1.9 £2 (d) 2.0 £2 Fig. C.62.

Scanned by Cam Scanner


Appen^r£__-------------------------------------------------------------------------------------------------------------------- ---------- -
30 A separately-excited dc motor has an armature resistance of 0.5 Q. It runs off a 250 V dc supply
’ drawing an armature current of 20 A at 1500 rpm. The torque developed for an armature current
of 10 A, for the same field current, will be
(a) 15.28 Nm (6) 15.92 Nm
(c) 15.6 Nm (dj 16.55 Nm
31. A bipolar dc machine with interpoles has a main-pole flux of (
j)per pole and an interpole flux of
0, per pole. The yoke of the machine is divided into four quadrants by the main-pole axis and
the commutation axis.
The flux-distribution in tbe quadrants will be

(a) - (0 + <J>() in all the\fourquadrants

(b) ~(<|>- 0() in all thefourquadrants

(c) g + ty) *n two diametrically opposite quadrants and ^ (tp - 0;) in the remaining two
quadrants

(^) 2 ^ + ^ ^wo adjacent quadrants and — (0 - 0() in the remaining two quadrants

[/.A S ., 1996 ]

32. The residual magnetism of a self-excited dc generator is lost. To build up its emf again
(а) the field winding must be replaced
(б) the armature connection must be reversed
(c) the field winding connection must be reversed
(d) field winding must be excited by low voltage dc supply [/AS 1993 ]

33. The armature mmf of a dc machine has


(а) triangular space distribution and rotates at the speed of armature
(б) trapezoidal space distribution and is stationary in space
(c) stepped distribution and rotates at the speed of armature
(d) triangular space distribution and is stationary in space
34. The flux is maximum in the following part of a dc motor :

?(c)! under
P° ! f C° leading
[e a- i tip
pole {d) under
under the interPole
trailing ^ tJp
35. The armature mmf in a dc machine is
1. stationary with respect to field poles
2. stationary with respect to armature
3. rotating with respect to field poles
4. rotating with respect to armature
From these, the correct answer is
(a) 1, 4 (6) 1, 3 (c) 2, 3 (d) 2, 4
36. The type of dc generator used for arc welding purposes is a
(а) series generator
(б) shunt generator
(c) cumulatively compounded generator
(d) differentially compounded generator
37. The magnetic circuit of a dc shunt generator with negligible armature resistance and brush
contact voltage drop remains unsaturated when it delivers full-load current at rated terminal
voltage. The brushes are kept in the interpolar axis and speed is maintained constant. When
the load is switched off, the terminal voltage would “
(a) remain the same (6) increase
(c) decrease (d) become zero [I.A.S., 1993]
38. Fig. C.63 shows the cross-section of a 2-pole dc machine. Corresponding flux density and mmf
waveforms will be

/
it:u u_y v^ai i lo u d i n itri
Kioctrica! Ma
970

Motion
‘ as Gen.

LEGEND
Flux density
Mmf

Fig. C.63.
U.E.S., 19951

39. In a loaded dc generator, if the brushes are given a shift from the interpolar axis in the direction
orotation, then the commutation will
f

(a) improve with fall of terminal voltage V, (b) deteriorate with fall of V,
(c) improve with rise in V, «*> deteriorate with rise in V,
40. In a loaded dc motor, if the brushes are given a shift from the interpolar axis in t h e direction o

" r i " ^ Wi" <*> deteriorate and the speed rises


(c) deteriorate and the speed falls id) improve and the speed rises
41. In a dc machine, the demagnetizing effect of armature reaction is due to
(a) component of armature mmf along field axis
(b) non-sinusoidal nature of armature mmf
(c) magnetic saturation in half of the field pole [I.A.S-, ^
(id) uneven air-gap length
42. Which of the following factors improve commutation in a dc machine ?
1 High contact resistance of brushes
2. High inductance of the coils undergoing commutation o p p o s i t e directs
3. Shifting of brushes in the direction of rotation for a generator and in the opp
for a motor
4. Narrow width for the commutation bars.
Select the correct answer using the codes given below :
Codes :
(a) 1, 2, and 3 (b) 1 and 3 (c) 2, 3 and 4 (cf) 1, 2 and 4 the
43. In a dc generator operating on load with its brushes on the geometrical neutra
magnetic neutral axis (MNA) is shifted in the direction of rotation.

Scanned by Cam Scanner


rl'

971

if the brushes are given a lead of 90° (electrical), then the MNA will
N - forward by 90° (6) shift forward by morethan 90r'
^ shift forward by less than 90° (d) coincide with the GNA II.A.S., 1996]
ml vnlt-ampere equation fora long-shunt compound motor is given by
* J ^ v ,= S . * I. r. (*) V, = + /„ (r„ +
(c) V, = Ea - I a ra (d)Vt = Ea + I ara + I L rs
Consider the following statements about commutating poles which are fitted on most large dc
shunt motors .
I The commutating poles are placed in the geometric neutral plane and their number is usually
equal to the number of main poles
2. The winding on the commutating poles is connected in series with the shunt-field winding
on the main poles
3. The polarity of the commutating pole must be that of the next pole further ahead
4. The commutating poles neutralize the reactance voltage produced in the coil undergoing
commutation.
Of these statements,
(a) 1, 2 and 3 are correct (b) 1 and 4are correct
(c) 2, 3 and 4 are correct (d) 1, 2 and 4 are correct
46. Consider the following statements :
Interpoles in dc machines
1. reduce armature reaction effects in the interpolar region
2. have the same dimensions as main poles
3. have their windings connected inseries with the armature
4. have same number of turns asthe armature
5. have the polarity same as that of the main pole ahead in a motor
From these, the correct statements are
(a) 1, 2 and 3 (6) 1, 3 and 5 (c) 1, 2, 3 and 5 (d) 1 and 3
47. Consider the following statements :
1. Interpole windings are connected in series with armature winding
2. Polarity of interpoles must be the same as that of the main poles in advance
3. Distortion of the main field under the pole shoes is not affected by the use of interpoles
From above, the correct answer is
(a) 1 and 2 (6) 2 and 3 (c) 1 and 3 (d) 1 alone
48. The introduction of interpoles in between the main poles improves the performance of a dc
machine, because
(а) the interpoles produce additional flux to augment the developed torque
(б) the flux waveform is improved with reduction in harmonics
(c) the inequality of air-gap fluxon the top and bottom halves of the armature is removed
(d) a counter-emf is induced in the coil undergoing commutation (/.A.S., 1997]
49. In dc machines,
1- reactance voltage is proportional to armature current
2. reactance voltage is proportional to core length
3. reactance voltage is due to self-flux of the coil undergoing commutation
4. armature-cross flux is neutralized by interpolar flux
5. interpolar mmf is more than armature mmf in the interpolar zone
From these, the correct answer is
(a) 1, 2, 3, 4 (6) 1, 2, 5 (c) 2, 3, 4 (d) 1, 3, 5
The interpoles in dc machines have a tapering shape in order to
(а) reduce the overall weight
(б) reduce the saturation in the interpole
(c) economise on the material required for interpoles and their windings
(d) increase the acceleration of commutation

Scanned by Cam Scanner


974 Electrical Machinery

70. A 440 V dc shunt motor has a no-load (/„ = 0) speed of 2000 rpm. It is running at 1000 rpm at
full load torque, reduced armature voltage and full field, Ifload to^ u® " v a l u e s th e ^ 3^
value with armature voltage and field voltage held constan P armature r e n ^
increases to 1050 rpm. The armature voltage drop at full loa g ion
effect) is „v
(a) 16 V (6) 18 V (c) 20 V («) v
71. A dc series motor drawing an armature current of Ia is operating un er saturate magnetic
conditions. The torque developed in the motor is proportional to

(fl) j - (b) \ (c) ft (d) 7“


ll
72. A dc shunt motor having unsaturated magnetic circuit runs at 1000 rpm g r a t e d voltage. If
the applied voltage is reduced to half of the rated voltage, t e mo r
(a) 2000 rpm <*> ^ rPm
(c) 750 rpm 500 rPm
73. A 240 V dc shunt motor with an armature resistance of 0.5 Q has a fulldoad current of 40 A.
Find the ratio of stalling torque to the full-load torque when a resistance of 1 P. is connected in
series with the armature ,r . T„ fnrtoI
(a) 4 (6 ) 1 2 (c) 6 id) S [GATE, 1998|
74. If speed of a dc shunt motor is increased above its rated speed, then its counter emf
(a) increases ib) decreases
(c) remains unchanged id) first increases and then decreases
75. A dc shunt motor drives a load torque at rated voltage and rated excitation. If the load torque
is doubled, the motor speed would
(a) increase slightly (6) decrease slightly
(c) become half (d) become double
76. A dc shunt motor is required to drive a constant power load at rated speed while drawing rated
armature current. Neglecting saturation and all machine losses, if both terminal voltage and the
field current of the machine are halved, then
(а) the speed becomes 2 pu but armature current la remains at 1 pu
(б) the speed remains 1 pu but la becomes 2 pu
(c) both speed and la become 2 pu
(d) both speed and la remain at 1 pu
77. A dc shunt motor runs at a no-load speed of 1140 rpm. At full load, armature reaction weakens
the main flux by 5% whereas the armature circuit voltage drops by 10%. The motor full-load
speed in rpm is
(a) 1080 (6) 1203
(c) 1000 (d) 1200
78. A permanent-magnet dc commutator motor has a no-load speed of 6000 rpm when connected to
a 120 V dc supply. The armature resistance is 2.5 C2 and other losses may be neglected. The speed
of the motor with supply voltage of 60 V developing a torque 0.5 Nm is
(a) 3000 rpm (6) 2673 rpm
(c) 2836 rpm (d) 5346 rpm
79. A dc shunt motor has external resistance Ra and Rf in the armature and field circuits respec­
tively. Armature current at starting can be reduced by keeping
(а) Rf maximum and Ra minimum
(б ) Ra maximum and Rf maximum
(c) Ra minimum and Rf minimum
(d) Ra maximum and Rf minimum

o u t f i n i e u u y u a m o u a i ii iei
Append* C_________________ __

80. While starting a dc shunt motor — -


(o) reduced armature voltage V„ and reduced field voltage V, should t>» i:
resistance R , should be included in the field circuit PPl' ed " nd “ ‘-regulator

M rated V “ ^ should be aPPlicd end R, should bezero


Idl rated V "",1 rat° d Vf Sh° uld be aPPlied and R, should be zero
° rated should be applied and should be maximum
81. Consider the following statements :
If the terminal voltapp nf „ L , .
of speed, then Shunt motor 1S halved with the load torque varying as the square
1. speed is halved
2. speed remains unaltered
3. armature current is doubled
4 armature current remains unaltered
ui these statements
(a) 1 and 3 are correct
(c) 2 and 3 are correct 1 and 4 are correct
82. Consider 2 and 4 are correct

1, w « d -L e o n a r d ™ L o datin u ltobiegf lrdine the Sp8ed Contro1 of dc motors :


2. Ward-Leonard e for “ nstant-torque drives

From these, the correct answer is constant-torque drives.


(a) 1 ,2 , 6 16) 1. 2 ,5 (c) 1 , 4 ,6 Wl l n n .. .
83. The lower limit of speed in a Ward Leonard *1, a r ’ ’ U.E.S., 2001)
1. losses in both the machines method of speed control is governed by
2. residual magnetism of the generator
4 Z Z T rA u TCUk resistance both the machines
4. speed of the generator-prime mover
From these, the correct answer is
(a) 2, 4
(c) 2, 3, 4 {b) 2 only
R4 a d u . {d ) l ’ 2 - 3 >4
. 84. A dc shunt motor is run under the following conditions •
1. Normal voltage with no additional resistance in armature or field circuits
2. 50% normal voltage with no additional resistance in armature nr film • •
3. Normal voltage with a small external resistance in the armature circuit^f f 1
the armature resistance rcuit ot a value equal to
4. Normal voltage with some additional resistance in the field circuit
Normal speed of the motor would increase in the sequence
(a) 1, 2, 3, 4 (5) 4t 3 ^ 2
(c) 2, 1 ,4 , 3 (d) 2, 3, 1 ,4
85. Consider the following statements :
To control the speed of a dc shunt motor above the base speed over a reasonahl m
motor m ust y Wlde ranSe- the
1. have compensating winding
2. have interpole winding
3. be started using a3-point starter
4. be started using a4-point starter
Of these statements
(a) 1, 2 and 4 are correct (6 ) 1, 2 and 3 are correct
(c) 1 and 3 are correct (d) 2 and 4 are correct
A

L
Scanned by Cam Scanner
h

976__________________________________________ __________________ ____________________________Electrical Machin ^

86. A dc shunt motor delivers rated load power at rated speed. If motor terminal voltage is reduced
to half (neglect saturation and armature reaction), then
(a) speed oom, armature current la and motor torque Te remain constant
(6j co„, remains constant, both la and Te are doubled
(c) both u)m and Te remain constant, la is doubled
(d) both and Ia are doubled, Te remains constant
87. The field windings of a dc compound motor are connected in short-shunt configuration. If
series and shunt field windings are interchanged by mistake and the motor is connected to rated
voltage,
(a) series winding gets overloaded
lb) shunt winding gets overloaded
(c) both windings get overloaded
id) neither of the two windings gets overloaded
88. Read the following statements regarding dc motors :
1. Direction of rotation of a shunt motor can be reversed by interchanging its supply terminals
2. Additional resistance in the field circuit of a shunt motor decreases its speed
3. Additional resistance in the field circuit of a shunt motor increases its armature current
a constant torque load
4. Direction of rotation of a shunt motor can also be reversed by interchanging its armature
terminals
5. A dc series motor should never be run at no load
From these, the correct answer is
(a) 2, 3, 4 (6) 1, 3, 4, 5
ic) 3, 4, 5 (d) 1, 3, 4
89. A separately-excited dc generator feeds a dc shunt motor. If the load torque on the motor is
halved approximately
[a) armature current of both motor and generator are halved
(b) armature current of motor is halved and that of generator is unaltered
ic) armature current of generator is halved and that of motor is halved
(d) armature currents of both motor and generator are halved
90. In a coal mine, Ward-Leonard method of speed control as shown in Fig. C.64 is used to lift the
coal from the pit through a crane and bucket arrangement. After lifting the coal to desired level
Z . T Z at
ne taken I Tthis ‘
I * 1stageduT nB gr0U"the
is to reverse d' the bucket has be lowered into the pit. The action to

Fig. C.64.
(a) direction of current in F2 and slowly increase it
ib) current in F3 and slowly decrease it
ic) supply terminal of
id) direction of current in F x and slowly increase it

speed squared kpora'Tn motor *s unsaturated and the load torque is proportional to
limTcun-ent
line current will he
be P d ° f 4° ° ^ motor line current is 20 A. For a speed of 800 rpm, the

ic) 40 A (4) 28.28 A


I J UA Id) 80 A

Scanned by Cam Scanner


>1*"

lix-
977
92. Take the normal values as unity, the approximate values of current and speed of a dc shunt
motor under the conditions of list 1 are given in list 2. Match the two lists and select the correct
answer using the codes given below the lists :
List 1 List 2
Armature Speed
Current
A. Terminal voltage halved, field current and load 1. 0.5 0.5
torque constant
B. Terminal voltage halved, field current and power 2. 1.0 0.5
output constant ‘
C. Field flux doubled, armature voltage and load 3. 2.0 0.5
torque constant
D. Field flux halved, armature voltage halved and 4. 2.0 1.0
power output constant
Codes:
A B C D A B C D
(a) 1 3 2 4 (b) 2 3 1 4
(c) 2 1 4 3 (d) 1 2 3 4
If the armature ot a ac motor is supplied from a constant current source, and its
constant voltage source, then the speed-torque characteristics of the motor will be

/
“s
“ r TD / T
3
"D Q>
C
D d)
a> a / C
L
a a
in in

/ 1
Torque Torque Torque T o rq u e
(b) (c) (d)
(a)

94. Match list I (dc motor) with List II (characteristics) and select the correct answer using the
codes given below the lists :
List II
List I
1. Fairly constant speed
A. Cumulatively compounded motor
2. It may start in reverse direction
B. Differentially compounded motor
3. Definite no load speed
C. Series motor Never started without load
4.
D. Shunt motor
C odes: A B C D
C D 2 4 1
A B 3
0b) 4
2 3 1 4 1 3 2
(a) (d)
2 4
43 f pycited dc motor is required to be
(c) 1
' - the speed of a separately-®* , f the fallowing methods
95. In machine-tool drive apphcation ofthe motor r l^ ° nt and h r ■field current
controlled both below and above the rated ^ . Jf. fleld currt„t and , r
ia heat suited for thie purpose ? <V„ . ar
at rated speed).
{a)If =If r ; V a variable
(6) Vafixed, If variable
(c) and v a r ia b le w .u .^ -f r
id) Vaand Vf variable wi f f

C-- •: .

Scanned by Cam Scanner


978______________________________________________ ________ __________________________

96. The following lists relate to speed control of dc motors. Match List I and List II, and select a
correct answer using the codes given below the lists . ?
List I List 11
A. Armature voltage control 1- SPeeds above base sPeed
B. Field current control 2. Speeds below base speed
C. Use of diverter resistance 3. Poor motor efficiency
D. Rheostatic control 4- Speed control of series motor
Codes :
A B C D A B C D
1 2 3 4 (&) 2 1 4
(a) 3
(c) 4 1 3 2 (rf) 2 3 1 4
97. A 2-pole series motor with its two field coils connected in series runs at a speed of 500 rpm, if
the field coils are re-connected in parallel and assuming that the torque is constant and the
magnetic circuit is unsaturated, the new speed will be
(a) 250 rpm (6) rpm (c) 500 VjT rpm (rf) 1000 rpm

98. The function of a dc motor starter is to


(а) start the dc motor (6) limit the starting current
(c) increase the starting torque (rf) avoid dips in the supply voltage
99. DC shunt motor should not be stopped by forcing the starter handle back to the OFF position
by hand to avoid **
(o) heavy sparking at all the studs as handle travels to OFF position
(б) dangerous sparking at the last stud as handle travels to OFF position
(c) dangerous to the operator due to heavy sparking
(rf) both (a) and (6).
100. Three-point starter for dc shunt motor is not used where wide speed control above rated speed
is required because
(а) the motor may stop at high speed
(б) the motor may stop at low speed
(c) hunting may occur in the motor
(rf) motor may attain dangerously high speed
J 101. Consider the following statements :
At starting, the fieldI exc.tat.on of a dc shunt motor is kept at its maximum value to reduce
3 Z Z t"/ . 2- accelerating time
3. starting current „ voltage dipB,n the supply
Of these statements,
(a) 1 and 2 are correct ,lv 9 «, j a .
i o j A n W d and 4 are correct
? '8re C0,TeCt W) 1, 2 ,3 and 4 are cmrect
’ « ,v ie fl0ld ° lrCUlt ° fd ,C1shunt motor>p i p p e d with 3-point starter, gets interrupted accidentally,
while running normally, then the
(a) starter arm will fly back to the off position
(o) speed of the motor will rise dangerously
(c) armature winding will draw heavy current
(rf) motor will continue to run normally. [I.A.S., 19^]
103. Consider the following statements :
A dc shunt-motor starter ensures that
1. armature current is under limit during starting
t. held flux is maintained at the maximum value
3. acceleration time is controlled
4. field failure is prevented

$ cann?ieT6y”Ca m Scanner
-V*' 'r\^< '
X

979
^ d j* ^ . . ------------
^ -^ ^ ta rtin g torque is always more than the load torque
rthese statements
^ 1 2 3 and ® are correct
(o) 2’ 3 and 4 are correct
(M , ’and 4 are correct
j 3 and 4 are correct
(d) *’ “ 1/.A.S., 1995]
Which one of the figures given here best reorespnto - . • , ,
l04- ”^ e(J # with time during the starting of a dc shunt m to Vanatl° n m armature current 1 and

N ,

105. A dc shunt motor is started at no load a d > j


After 5 hours of continuous no-load running S n0ted 38 1000 rpm during winter,
la) become somewhat more than 1000 mm P“ 4 d
(6) become somewhat less than 100 rpm
(c) remain 1000 rpm
(d) reduce to 950 rpm
106 In dc machines, constant loss is composed of
la) iron loss and mechanical loss
(6) friction, windage (FW) and iron loss and field circuit loss
(c) iron loss and field circuit loss
(d) FW and stray load loss
107. The efficiency of a dc shunt generator is maximum when
. ? af « t i c loss is equal to the mechanical loss
to) held ohmic loss is equal to the constant loss
(c) stray load loss is equal to the armature circuit loss
10R A )/ miatUre drCuit l0SS is e<*ual t0 the sum of no-load rotational loss and field circuit W
* rpRn gfenerat?r has an efficiency 90 % for an output voltage and current of 250 V a
spectwely. If this machine is made to operate at the same speed as a dc motor with 18 A
, ^ of 250 v and current of 18 A, the efficiency of the motor would be 30 mput
W y()% . Q, yy~
(C) 88.88% [d) 87.12%
• A 200 V dc shunt motor delivers an output of 17 kW with an input of 20 kW. The • a
resistance is 50 Q and the armature-circuit resistance is 0.04 £2. Maximum e ffic ie n c v '^ V ^
obtained when total armature ohmic losses are equal to 1
(a) 2632 W (b) 3000 W
(c) 3680 W (d) 5232 W

Scanned by Cam Scanner


\ 980
- , , . ____ Qr,j their functions are given below :
110 The windings of a large dc series motor and tne
Functions
Winding
1. to avoid field distortion under the
A. series fieldwinding Pole
2. to avoid sparking
B. shunt fieldwinding
3. to generate working flux
C. commutating-pole winding
4. to avoid runaway speeds at no load
D. compensating winding
Select correct answer using the following code .
A B C
A B C D D
0b) 2 3 4
{a) 1 2 3 4 1
id) 3 4 2 1
(c) 3 4 1 2
111. A dc series motor fed from rated supply vo\tege f characteristic
its magnetic circuit is saturated. The speed- orq curve
of this motor will be approximately represented y
of Fig. C.65.
(a) curve A (6) curve B
(c) curve C «*) curve D
112. Consider the followingstatements :
Permanent magnet dc motors used in casette-tape recorders have
Torque
1. magnets on stator and armature on the rotor Fig. C.65.
2. magnets on rotor and armature on the stator
3. electronic commutation and no brushes
4. mechanical commutation and brushes
5. automatic speed governors
Of these statements
(a) 1. 3 and 5 are correct w y ““ .
(c) 2, 3 and 5 are correct «*) 1and 4 are correct
113. In Swinburne’s method of testing dc machines, the shunt machine is run as a
(а) motor at no load at rated speed and rated voltage
(б) generator at no load at near rated speed andrated voltage
(c) generator at full load at rated speed and ratedvoltage
(d) generator at no load at rated speed and rated terminal voltage
114. In Hopkinson’s method of testing dc machines, the power taken from the mains has to supply
(a) the losses in the motor only
(b) the losses in both the machines
(c) the input to both the machines
(d) the input to motor and not to the generator
115. Two dc shunt machines are coupled mechanically and electrically. One dc machine is run
motor and the other as a generator. Their no-load rotational losses are equal because
1. their speeds are equal
2. their speeds and excitations are equal
3. generator iron losses are more than the motor iron losses
4. motor armature current is less than the generator armature current
5. motor has more stray load losses than those in the generator
From the above, the correctanswer is
(a) 1, 2, 3, 5 (b) 1, 2, 4,5 (c) 1, 2, 5 id) 1, 3, 5
116. In Hopkinson’s test on two dc shunt machines
1. armature currents of both the machines are equal
2. generator armature current is less than that in the motor
3. their speeds are equal
4. their field currents are equal

Scanned by Cam Scanner


II X

Appendix— C
981
5. stray load losses in motor are more than those in the generator
From these, the correct answer is
(a) 2, 3, 5 (i>) ! 3 g (c) 2 3 4 (</) j 3 4 5

117’ as^a penp" ? 8 m™ ? ines are mechanically coupled. One machine is run as a motor and the other
, . .,f\ r- ™ne iron and friction loss will be identical when
(a) their speeds are identical
ty.6'1" speeds ant* excitations are identical
td\ th *r spe^ sare eciual and back emfs are half of the supply voltage
e,r exatat-»°ns are equal and back emfs are half of the supply voltage
118. In Field's test on two series machines
generator field current is more than motor field current //„,
lfg = lfm
4 V ma*are v° ' taKe of generator Vtg is more than armature voltage of motor V,(lt
tg< fm
From these, the correct statements are
1, “ (e)2 ,4 (d) 2 ,3
MatCh LlSt 1 and Ust 11 and select the correct answer from the codes :
List I List 11
A. Voltage-drop test 1. Efficiency
B. Hopkinson’s test 2. Separation of iron and friction losses
C.
Swinburne’s test 3 . Open and short-circuited armature coils
D. Retardation test 4 Temperature rise
Codes
A B u D A B C n
(a) 3 1 4 2 ib) 2 4 1 3
(c) 3 4 1 2 (d) 2 1 4 3
A rotating am
(a) metadyne (6) amplidyne (c) regulax (d) rototrol
‘ 2 I - ^ =rapIidyne develops an output voltage of 100 volte at 1000 rpm. If its armature is now driven
at 2000 rpm, its output voltage would be
(a) 100 V (6) 200 V (c) 400 V (d) 50 V
122. The polarity of the output voltage in a cross-field generator can be altered by
(а) reversing the direction of rotation
(б) reversing both the direction of rotation and field current
(c) reversing the direction of field current
(d) reversing either the direction of rotation or the field current
123. In a 2-pole cross-field generator,
1. there are 4-polar projections
2. there are only 2-polar projection
3. there are four interpoles
4. there are only two interpoles
5. the number of brushes = 4
6. the number of brushes may be 2 or 4
From these, the correct answer is

(a) 3’ 5 2 ! ? ’ 1’ r
ic) 1, 4, 5 4’ 6
124. Typical characteristics of a dc series motor ° ^ ratinf
under loaded condition, are shown in g. • • ®
gives the identification of these characteristics, while

Scanned by Cam Scanner


982

, . T, . ‘™UCll
1S f,r|VCS 110 t,C8cnPtion of important load characteristics of dc motor working on l0ad
List I
List II
(see figure, curves labelled)
A.
A ... 1.
B. Efficiency Vs load
B ... 2.
C. Torque Vs load
C... 3. Current Vs load
,, 4.
*• Speed Vs load
Codes -J,S WlL 1 lKt 11 and SCl0Cl th° correct ansvver usinB the codes given below

A II C A B
(«) 4 C
2 3 ib) 2 3
(c) 3 1
2 1 id) 4 1 2
125. In a dc machine, reactance voltage

2 is th e tm n o f on f° hcl,f' incluct'anco o f thc coil undergoing commutation


finldnf n fs !nduced^ sulf and mutual fluxes
linking the coil undergoing commutation +j,
3. causes delayed commutation

' !Inter^l".flUutr liZed ty transforn' t!r cmf ' nduced by


5. can be compensated by voltage commutation
From these, the correct statements are

S 2 3 \ (b) 2) 3>4
( ) 2’ 3' 5 (d) 2, 3, 4, 5 r -
Commutation
126. Commutation characteristics of dc machines are shown time
in Tig C.67. Match List I with List II and select the
Fig. C.67.
correct answer by using the codes given below :
List I
List II
A. Resistance commutation 1. straight line 1
B. Delayed commutation 2. curve 2
C. Linear commutation 3. curve 3
D. Accelerated commutation 4. curve 4
C odes:

A B C D A B C D
(a) 3 2 1 4 (b) 3 2 4 1
ic) 4 1 3 2 0d) 4
2 3 1
127. The speed-torque regimes in a dc mot
respectively in Group II and Group I
Group / Group II
P. Field control 1. Below base speed
Q. Armature control 2. Above base speed
3. Above base torque
4.
4. Below
oeiow base
Dase torque
torque
The match between the control method and the speed/torque regime is as follows : .
(a) P-l ; Q-3 (b) P-2 ; Q -l (c) P-2 ; Q-3 (d) P-l ; Q-4 [GATE, 20031

Scanned by Cam Scanner


https://t .me/abcdelectrical

ipdix— 983
AP|*
l2g. To eondhLic1; load test on a dc shunt motor, it is coupled to a generator which is identical to the
^ ^ tu r e of the ° ^enerator is also connected to the same supply source as the motor. The
Armature reaction*161? 1* * is connected to a load resistance. The armature resistance is 0.02 pu.
the load resistance3 mechanical losses can be neglected. With rated voltage across the motor,
motor and generatorTT|SS the generator is adjusted to obtain rated armature current in both
jo <5 /0 qq p u- value of this load resistance is
129. A dc series motor drivi “ , « 096 <d> 0 94 IGATE. 2003\
speed and rated voltage If th ctnc train ^aces a constant power load. It is running at rated
be approximately broueht a speed has to be brought down to 0.25, the supply voltage has to
(a) 0.75 p.u. (fe)0g5 p uOWnto, '
(c) 0.25 p.u. (d) 0.125 p.u. bGATE, 2003]

1. c) a n sw e r s
2. c)
6. b) 7. 3. (c) 4. (a)
d) 5. {d)
11. b) 12. 8. (c)
c) 9- (a) 10. (6 )
16. c) 17. 13. (6)
d) 14. (6) 15, (a)
21. b) 18. (c)
22. d) 19. (c) 20. (c)
26. d) 23. (b)
27. a) 24. (b) 25. (a)
31. c) 28. (c)
32. d) 29. (c) 30. (a)
36. d) 33. (d)
37. a) 34. (c) 35. (a)
41. c) 38. (c)
42. b) 39. (a) 40. (c)
46. d) 43. (d)
47. c) 44. (6) 45. (b)
51. d) 48. (d)
52. a) 49. (fc)
56. 53. (a) 50. (b)
a) 57. b) 54. (6)
61. 58. (c) 55. (d)
c) 62. a) 59. (C)
66. 63. (d) 60. (d)
a) 67. b) 64. (c)
71. 68. (d) 65. (6)
d) 72. b) 69. (d)
76. 73. (a) 70. (d)
b) 77. a) 74. (c)
81. 78. lb) 75. (6)
c) 82. 79. (d)
a) 83. (c) 80. (b)
86. C) 87. 84. (d)
d) 88. (c) 8 5 .(a)
91. C) 92. 89. (a)
b) 93. (b) 90. (a)
96. b) 97. 94. (6)
d) 98. (a) 95. (c)
101. d) 99. (b)
102. a) 103. (a) 100. (a)
106. b) 107. 104. (6)
d) 108. (c) 10 5 .(a)
111. b) 109, (a)
112. d) 113. (d) HO. (d)
116. a) 114. (6)
117. b) 118. (c) 115. (d)
121. c) 119. (c) 120. (a)
122. c) 123. (a)
126. d) 124. (rf) 125. (c)
127. b) 128. (c) 129. (6)

i b
I .i '


VTSUM l

Scanned by Cam Scanner


POLYPHASE SYNCH RONOUS MACHINES

X. A syuctoouous muchbrn has #. " t ? r ^ ^ a " 8e "


the rotor. When running under steady-conditio ,
1.stationary with respect to stator 0pnfnr
2. rotating at synchronous speed Ns with respec
3. rotating at Ns in the direction ofrotor rotation
4 .rotating at double the Ns with respect to rotor
5. rotating at Ns with respect to rotor
6 .rotating in a direction opposite to rotor rotation
From these, the correct answer is „ _
(a) 2, 5 (6 ) 1, 4, 6 (c)2, 4, 6 (d) 1, 3,5
2. A synchronous machine has its 6 eld winding on the rotor and polyphase armature wmdmg on
the stator. When running under steady conditions, its air gap
1.stationary with respect to stator
2. rotating at synchronous speed Ns with respect to sta or
3. rotating at double the Ns with respect to rotor
4 .stationary with respect to rotor
5. rotating at Ns in the direction ofrotor rotation
From these, the correct answer is
(a) 2, 5 (b) 1, 4, 5 (c)2, 3, 4 (<d) 2, 4 ,5
3. The maximum possible speeds in rpm at which an alternator can be driven to generate voltages
at 60 Hz and 50 Hz are respectively:
(a) 2000, 2400 (b) 3000, 3600 (c)2400, 2000 (d)3600, 3000
4. Two mechanically-coupled alternators deliver power at 50 Hz and 60 Hz respectively. 'Hie
highest speed ofthe alternators is
(a) 3600 rpm (6 ) 3000 rpm (c)600 rpm (d) 500 rpm
5. Match List Iwith List II and select the correct answer using the codes given below the lists:
_ . - r 7 T
List I List II
[Parts of a turbogenerator used in [ Materials from which these parts are made ]
thermal power plants ]
A. Stator core 1. Copper
B. Rotor core 2. Copper alloy
C. Stator windings 3. Silicon steel
D. Slip rings 4. Mild steel
5. Aluminium
Codes:
A B C D
A B c D
ib) 4 5 1 2
(a) 3 5 2 1
id) 3 4 1 2
(c) 4 3 1 2
6. Consider the following statements :
The synchronous generators used in thermal power stations have
1. cylindrical rotors 2. slip rings and brushes
3. laminated rotor 4. stator slots in multip e o
5. 3-phase star-connected stator windings
Which of these statements are correct ?
(a) 1, 2, 3 and 4 (&) 2>3 and 5
(c) 3 ,4 and 5 id) 1 ,2 ,4 and 5 .pow er
7. The most appropriate speeds in rpm of generators used in thermal, nuclear an y
plants would respectively be
(a) 3000, 300 and 150 (b) 3000, 3000 and 300

Scanned by Cam Scanner


(c) 1500, 1500 and 3000 {d) 1000, 900 and 750
g in a synchronous generator operating at zero power factor lagging, the effect of armature reaction
is
(o) magnetizing (6) demagnetizing
(C) cross-magnetizing (d) both magnetizing and cross-magnetizing
[I.AS., 1995]
9. Match List I with List II and select the correct answer using the codes given below the lists :
List I List II
(Load pf) (Nature of armature flux in an alternator)
A. Zero leading
1. Demagnetizing
B. Unity
2. Magnetizing
C. Zero lagging
3. Cross-magnetizing
D. Zero with armature terminals suddenly 4. Initially constant, demagnetizing
short-circuited ’

C odes:

A B c D A B C D
(o) 1 2 4 3 ib) 2 3 1 4
(c) 4 3 1 2 (d) 2 1 3 4
[I.A.S., 1996]
10. Open circuit (oc) and short-circuit (sc) tests on an AC generator are conducted under which of
the following conditions ?
1. oc test at nominal flux and sc test at nominal current
2. oc test at reduced flux and sc test at reduced current
3. oc test at zero armature current and sc test at reduced flux
4. oc test at zero current and sc test at nominal flux
Select the correct answer using the codes given below :
C o d e s:
(a) 1 and 3 (6 ) 2 and 3 (c) 1 and 4 (d) 2 and 4
11. The zero-power-factor characteristic for the Potier diagram can be obtained by loading the
alternator using
(a) lamp load (6) synchronous motor
(c) water load (d) dc motor
12. Match List I (methods of full-load regulation of 25 MVA alternator at 0.8 pf lagging) with List
II (% regulation) and select the correct answer using the codes given below the lists:
List I List II
A. emf 1. 13%
B. mmf 2. 18%
C. zpf 3. 32%

C o d e s:
B C A B
A
(a) 1 2 (b) 1 3
3
1 1(d) 2
(c) 2 3
13. Read the following statements about a cylindrical-rotor alternator .
1. Emf generated by armature reaction mmf lags armature current by
2. Armature reaction -due to intermediate lagging pf is partly cross-magnetizing and partly
magnetizing
3. Air-gap voltage leads terminal voltage
4. Air-gap voltage lags the field flux by 90° ^
5. Armature reaction mmf lags the field flux by (90 internal pf angle)
9g6 Electrical Math
»1
From these, the correct answer is „ . c
(a) 1, 2, 3, 5 (6) 1, 3, 5 (c) 2, 3, 5 («) '
14. The power factor of an alternator under short circuit conditions is almost near
(а) zero leading
(б) zero lagging

(d) zero leading or zero lagging depending upon the type of alternator
15. Consider the following statements regarding synchronous machines .
1. In an alternator, armature mmf leads the field flux
2. In an alternator, airogap flux lags the field flux
3. In a synchronous motor, armature mmf leads the field flux
4. In a synchronous motor, air-gap mmf lags the armature m m f
5. In an alternator, field flux lags the armature current.
From these, the correct answer is
(a) 2, 3, 4 ib) 1, 3, 4 (c) 2, 3, 4, 5 (d) 1, 3, 4, 5
16. In a 3-phase cylindrical-rotor alternator,
(а) field mmf leads the air-gap flux and air-gap flux leads the armature
(б) the armature mmf leads the air-gap flux and the air-gap flux leads the field flux
(c) the armature mmf leads the air-gap flux and the air-gap flux lags the field flux
(d) field m m f leads the air-gap flux and air-gap flux lags the armature mmf.
17. Read the following statements about a cylindrical-rotor alternator .
1. Armature reaction mmf due to 0.8 lagging pf current is partly cross-magnetizing and partly
demagnetizing
2. the field poles lead the resultant air-gap flux
3. the field poles have a tendency to go away from the resultant air-gap flux
4. terminal voltage lags the field flux by 90+ 5.
From these, the correct answer is
(a) 1, 2, 4 (b) 1, 2, 3, 4 (c) 2, 3, 4 (d ) 1, 3, 4
18. Potier reactance of an alternator is almost the same as
(a) Field winding reactance (6) total armature reactance
(c) leakage reactance of field winding (d) armature leakage reactance
19. Which one of the following methods would give a higher than actual value of regulation of an
alternator:
(a) zpf method (b) m m f method
(c) E m f method (d ) A SA method
20. The four methods of calculating voltage regulation of a 3-phase alternator are :
1. E m f method
2. Saturated synchronous reactance method
3. New ASA method
4. M m f method
The correct sequence of the ascending order o f the values of regulation obtained by these methods
is
(a) 3, 4, 2 ,1 ( 6 ) 4 ,3 , 1 ,2 (c) 3, 4, 1, 2 (d) 4, 3, 2, 1
21. The flux per pole in a synchronous motor with the field circuit O N and the stator d i s c o n n e c t e d
* ,e 8uPPly is found to be 25 mWb. W hen the stator is connected to the rated supply w1
motor ic excit? tlon unchanged, the flux per pole in the machine is found to be 20 mWb w bie v
the motor^ronTthe^up ‘^ssum *n£ no*load losses to be zero, the no-load current drawn
(a) lags the supply voltaee f
.
. I■
(0 is in phase with the supply voltage ?2 ^ SUPPly ^
V ‘ r j vuiLage (d) 1S zero
P p ; ii iL/U
OCui ■ uy wui i ixJL-ui ii iVyi ’ "
ht
:-C
APF(cndix
- --------------------------------— - - vt
22 . A 3-phase alternator d e W ™ ! ~ ' ~
is observed that the open- ' ^.ower t° a balanced 3-phase load of power factor 0.707 lagging. It
15°. Neglecting the e f f ^e nc? 1^
effect t c mf Phasor
cm^ P^asor leads the corresponding terminal voltage phasor by
f? -.!a
the axis of armature m m r...M ,rmon^cs» the angle between the axis of the main-field m m f and
,,rmonics>
(o) 30° electrical W,U be
(c) 90° electrical (fe) 60° electrical

23. A 3-phase, 400 V, delta-conn t i 0d) 150°


(rf) 150° electrical
electrical [I.A .S .,1997]
phH ^ (je!1.vers a balanced has a synchronous impedance of (0 + j'20) ohms per
(a)- 5 0 on^ tho aC V Z * — power factor leading, then percentage
T OU , *----
(c) - 150
24. A 3-phase alternator has neglmihi r 150 (d) 150 [I.A.S.,1997]
:“ f tt ! P aaiCUlar ^ e d “ a t Z ^ T , A teSt is on this
rated armature c u r t n f * reducedt o h a l f f t T ‘ ^ ***** armatUre
(a) would be equal to I current required to m aintain
(fe) would be equal to 21
(c) would be equal to / / /
(d) cannot be predicted due to in*, «- •
25. In a 3-phase cylindrical-,- * nsufricieat data. ..........
1. effect of armature r e a c U o n T "?^ 0110113 machiae : ATE’ 1993]
2. synchronous reactance is ^ ° f by ma^ t i z i n g reactance

Prom these, the correct answer is g reactance and leakage reactance,

(c) 3’ 4 (fe) 2, 3
26. The full-load voltage regulation r i (rf) 1,
(rf) 1, 3,
3, 4
4

- - e d speed o f
(a) less than 6% (assuming negligible
^ 6% (&) more than 6%
27, A cylindrical mfnr u 5,5%

» ) load S & = S^ h t o ~ S S » ace L i e m“ m“ m ^ r output when


(c) load angle = 90°er Power'Pacl ° r angle
2 (d) lnPut Power factor is unity.

6XCitati0n i3 a* - t e d
fc) remain at unity (6) become laSging
29. Tho j (d) become zero.

of 11 kV6 S ^ M V A 11^ , 0^ 3 ge” erator IS 0 2 Pu on a base value


150 M V A is impedance value for a base of 22 kV,

$ o f PU (6) 0.20 pu
3D ' pu (d) 0.24 pu

tion Iff C ‘68i l the characteristic that corresponds to the varia-


the ®ynchronous reactance of a synchronous machine with
ie held current is

(c) p? 1*76 o curve B


^ ^ (rf) curve D

mat° Und' rotof alternator has a leakage reactance of 10%, ar-


are-reaction reactance of 90% and negligible armature
Fig. C.68.

Scanned by Cam Scanner


98 8 Metrical

resistance. With the machine initially running at rated speed and terminal voltage of i .q _
3-phase short circuit is applied. The sustained armature curren wi e »a
(a) 1.25 pu (b) 1.11 pu (c) 1.0 pu W 0 9 Pu
32. A synchronous generator with synchronous reactance of 1.3 pu is connected to an infinite bo*
whose voltage is 1 pu, through an equivalent reactance o . pu. or maximum output of
pu, the alternator emf must be _
(a) 1.5 pu (b) 1.56 pu (c) 1.8 pu W) 2 5 Pu
33. Consider the following statements for a 3-phase alternator designed to operate at pf 0.85 :
1. For pf > 0.85 lag, armature heating is the limiting factor.
2. For pf > 0.85 lag, rotor-field heating is the limiting factor.
3. For pf < 0.85 lag, armature heating is the limiting factor.
4. For pf < 0.85 lag, rotor-field heating is the limiting factor.
From these, the correct statements are
(a) 1 ,3 ( 6 ) 1 ,4 (0 2 ,3 (d) 2>4
34. Consider the following statements :
Modern alternators are designed for large air gap to have
1. large inherent regulation
2. very stable parallel operation
3. higher stability limit
4. sinusoidal mmf distribution
Which of these statements are correct ?
(a) 1, 2 and 3 (b) 2, 3 and 4 (c) 1, 2 and 4 id) 1, 3 and 4
35. A single-phase, 2000 V alternator has armature resistance and reactance of 0.8 £2 and 4 94Q
respectively. The voltage regulation of the alternator at 100 A load at 0.8 leading power factor

(a) 7% (b) - 8.9% (c) 14% id) 0%


36. A turbo-alternator set feeds power to a 3-phase constant-voltage, constant-frequency bus. If the
steam supply to the set is cut off, then the set will
(а) continue to run at rated speed in the same direction
(б) continue to run at a reduced speed in the same direction
(c) run at rated speed in the reversed direction
{d) come to a stop
37. Consider the following statements for lagging power-factor load :
1. In a cylindrical-rotor synchronous machine (CRSM), armature reaction flux <t>a is i P
with armature current Ia _ ...
2. In a salient-pole synchronous machine (SPSM), armature reaction mmf Fa is in phase

h
3. In a SPSM, (j)a is in phase with Ia
4. In a CRSM, «(>a lags Ia
5. In a SPSM, <|>a lags Fa
From these, the correct statements are
(o) 1 ,2 and 3 . ( « 2>4 an<J J
(c) 1, 2 and 4 W 1, 2 and 5 leakage
38. A 3-phase cylindrical-rotor synchronous generator, with its armature r e s is t a n c e ^ ^ kept
reactance being neglected, is synchronized to an infinite bus and its e ghaft®0
constant thereafter. New the machine ia loaded by supplying mechanical mp uid be
that the load angle 8 reaches a value of 60°. Under this condtion, the opera ing P
(a) 0.866 leading (6) 0.866 lagging
(c) 0.5 leading (d ) 0.5 lagging

f %' JIHAMByaunM^ u ... ________________ _ • -• ii■ niiiwn —i. ■i i « .tern: »■".. 'ii
Scanned by Cam Scanner
-------------- — _ 989
*9. A 100 MW water-turbiiip i«,„ i j .. . . ~ ..... .........
operation. The losses of the ° ? to 12^ MVA, 3-phase alternator rated for 0.8 lagging power-factor
factor lagging, consider ttha
h er ^no n)a^or
l f ^ ^ are negligible.
negligil For delivering a load of 90 MW at 0.5 power
, Lne following statement-?
efoUoW,"Bstatem ent,:• '
1. The set cannot deliver the load '

2. The alternator will be overloaded


3. The turbine will be overloaded

4. The turbine win not he overloaded


5. The alternator will net ,
6. The set can deliver the lo a d ^
‘uaa.
From these, the correct *t«t
: (a) 4, 5 and 6 => *a re
"

40. The phasor diagram of a svn h


synch, ^ ^ ^ and 3 ^ ^ 5 ^ 6
infinite bus
infinite bus is
is shown
shown in
in Fig ?c ? ? machine connected to an
to) generator and operating at « i
(а) machine is acting as a
(б) generator and operating a t » t ? ng pf
(c) motor and oneratintr 1 _ , ,adlng Pf
(d) motor and operating at a lagging pf
41. A star-connected svnrhmnn, Terminal vo ltage
pedance of 0 + j haS ^nchronous im-
Fig. C.69.
201A at 400 V. Its excitation emf is& pf leading of
(а) 400 + j ^3 x 100
(c) 400 - V3" x 100 (b) 400 - j V3 x 100
42, The torque angle of a ovnnV, ^ + x 100

^
(б) mt
(h\ t t ; f" a vcc dna^, stator
rotor Stat” =mmfr wave
“ -nf achine op6ratins &oma “
wave v°,tase tus' is usu°uy
to stator1mz " : z a: n ^ i T dr ity wave
« ) stator mmf wave and
41 a o u w**ve ana resultant mmf wave
A o-phase synchronous motor with neeliribl. i„ • KATE, 1
of tho
o7thC ™ b
e°m •rT inal ™Ultage
tantis ltage v«- Tae induced emf
V- mVS C0“ neCted t0 SU» ^ - t e d frequency
of the
the motor designated aa Bf
t? if 4-v. .
J motor
n , .s UCblgnatea
designated as Ef. If,he
If the motor is now , , "

excitation to o M a ta ‘ L T / ] 1” ” 61'' 7 iUSting “ S


then Efphasor would be^ ' agglng P f 'DpBration’
ai
fa) less than V, phasor and also lead K-phasor o □ 10
o
I eater than V, phasor and also lead Vr phasor > O
>
( less than Vr phasor and also lag ^-phasor 0 c
c o
^ eater than Vf-phasor
't andcuso
ailu alsomg
lag vr
Vrphasor
phasor E
a
. Ffcr P nn j *» _ <u
*-* u
* S te d 9 7°bdepiCf S the bad characteristics of an iso- X
D (U
Mat,h Th asr alternator’ " “ “ k g at constant speed. Cl

O
withTh sete of operating conditions
™ the given characteristics. Disregard the effects
i saliency, saturation and stator resistance 1-0
P o stator current
M ^ ° nstant excitation and non-zero leading pf
{ p 0nstant excitation and zero pf, leading Fig. C.70.
ic Constant terminal voltage and zero pf, leading
} Lonstant terminal voltage and non-zero leading pf
Ans. a ,b , d->
[GATE, 1992]

^C£
canned by Cam Scanner
990 ...___ _Eiwi»i«ni ^
— - ■—
45. A 3-phase synchronous generator, with negligible antm •ni < i <um iium,, opf.niies lit d
and delivers constant power to an infinite bus. If it* oxdtntlon >» Incm m d, then III* w w * *
and cos 0 = power factor)
(a) 5 decreases and cos 0 tends to become unity ^
(b ) 5 decreases and cos 0 tends to become mom legging
(c) 5 increases and cos 0 tends to become unity ^
(d) 8 increases and cos 0 tends to bocomo more uggmg
46 A synchronous motor with negligible armature resistance runs at n loin angle of 20" at r&t*}
f r e g u e n f l T s u p j d y f r e q u e n c y i a ^ n c r o i ,s o d by 10%, t a p i n g < * t a P » ™ m o l „ r » tb e J

load angle will be ,n


fa) 16° (6) 18“ W 20” <“ )
47. An alternator has rated Hold current of 2 A. It dcveb.pa 220 V -4' "'»»« >A « c l %
current. What will be the tenninal voltageot rated l ,M a ‘" ' M a l " " ?
fa) 220 V (6) 420 V (c) 440 V M )480V
48. A non-salient pole type of 3-phasc, 50 M,. 415 V, 4-polo
draws rated lino curront at 0.9 pf lead. Under those cond'tions the rat e pull-out torque p,
the developed torque on full-load is 2.5. The moasurnbio torque angle w.ll bo
fa) 23.56” (6)20° (c) 30° W) 25.84
49. A synchronous motor on load draws a curront at a lending pi anplo $. Ii tin, internal pf angle,
which is the phase angle between the excitation omf end the curront jn the l.ime-plianor diagram,
is vj/, then the air-gap excitation mmf lags tlic armature mini >y

(a)V (6 )| + V (e)f-v ‘b m w m s j
50. A 3-phase, 400 V, 50 Hz, synchronous motor is operating with a load angle 25", If load on the
motor is doubled, keeping other parameters constant, the now load angle will be
(a) 50° (b) 51.5° (c) 53.4° (cl) 57.7°
51. A cylindrical-rotor synchronous motor, with negligible armature resistance, operates at
1. unity pf if Ef cos 5 = V t 2. lagging pf if Ef cos o > Vt
3. lagging pf if ^ c o s 5 < Vt 4. zero pf leading if Ef > Vt
5. zero pf lagging if Ef> Vt
From these, the correct answer is
(a) 1, 3, 4 (b) 1, 2, 4 (c) 1, 3, 5 (d) 1, 2, 5
52. In a cylindrical-rotor synchronous motor, if Ef - excitation emf and Vt - terminal voltage, then
it is possible to have
I. Ef = Vt 2. Ef > Vt
3. E f< Vt just like a dc motor 4. Ef ahead of Vt
5. Ef behind Vt
From these, the correct answer is
(a) I, 2, 3, 4and 5 (b) 1, 3, 4 and 5
(c) 3 and 5 (d) 1, 2, 3 and 5
53. A synchronous motor operates at 0.8 pf lagging. If the field current of the motor is continuously
increased :
1. the pf deceases upto a certain value of field current and thereafter it increases
2. the armature current increases upto a certain value of field current and thereafter i
decreases
3. the power factor increases upto a certain value of field current and thereafter it decreases
4. the armature current decreases upto a certain value of field curront and thereafter
increases
From these, the correct answer is
(a) ly 2 (b) 3, 4 (c) 1, 3 (rf) 2, 4

Scanned by Cam Scanner


Appendix-C

54 it ^i^operatea^ (as1116 ^ fl° ating ° n an infinite bus at no load- If its excitation is varied, then

2 an ! ! f rnat0r Wltb Ef < v t at z pf leading


2. an alternator with Ef < Vt a t2 pf lagging
3. as a motor with F I g
4. as a motor with E I v “ 2 pf lcadine
5- as an nlternator .

f a ) T 3, 6 ^ ' the ®nswar is

56‘ A turbogenerator c o n n e d I . “ * V 1'


increased, then it would mte buS 18 operating at 0 8 Pf lagging. If its excitation is
l. teed more leading kVA
3. feed more real power to a c t e .!™ 2' fead m0re lagging kVAr t0 the bus
Prom these, the correct answer is 4' "*“ * 0Peratil18 at Pf < 0 8

**• A cyl'nclrica]-rotor SVnch 4 lC> *’ 4 {d) 2' 3' 4

faHSreinfi - ad aagle’ 6 ™ f 4 g i r aCnhj"a ‘Si , '! liVen" g 3 C° nStant load' When its agitation is
<« Er ™ * - £ £ ? and '• aia 9 = ™ S f Ure C,rCmt tba"
(r) E f cos 5 = constant ""n 008 0 = “ oMant

57. A cylindrical-rotor alt

~ 4 s t dertWs^
fa) 90 + v (6) " 6‘e between field axis and armature-
58- A synchronous machine i« (c) 9 0 - e M) 90 - ip

^ " ' t ^ ? , ~ r 81 “ l0ad “ “ " U" i t y ^ — ion

Fromthese, the correct answer is ’ ab8° rber »f reactive power


fa) 4, 4, 6 (6)1, 2, 5 fa) 2 , c
59. Which of the following limit th„ : ' Wl 3' “ ■6
1. Armature current 1 the reactive power output.of a synchronous generat ,
- 3. Load angle 2. Field current erator ?

Select the correct answer using the codes given below 4' Plim<!" ’MVer inPu‘
Codes:

60 A° 1 aUh 2 (6)2 and 3 (C) 3 and 4 1 and 4


A synchronous motor, fed from infinite bus, is delivering half fall ift ^ T, UA S > ^ 9 3 ]
current causes an mcrease in the armature current, then the motor ^11 ** lncrease in fiejd
a deliver reactive power and active power to the bus Wl11 d
. ) absorb reactive power and active power from the bus
(c) absorb reactive power from the bus and deliver active power to the bus
W deliver reactive power to the bus and absorb active power from the bus u
61. Which of the following will change in a 3-phase synchronous motor, as a consequence 2" 4]
variations? 4 ceof excitation
o' wUll~°Ut t0rqUe 2- Torque angle
3. Magnitude and power factor of stator current 4. Output power

V . , ■"
I
Scanned by Cam Scanner
Electrical Machl
992 ------------------------

Select the correct answer using the codes given below .

Codes : , x o o anA A (d) 1, 2 and 3 [7.A.S.,


(a) 1 and 3 (6) 2 and 4 (c) 2 , 3 and 4 v ' » ’
62. Power factor of a synchronous motor varies when ^ field excitation is varied
1, applied voltage is varied 4 supply frequency is changed
3. load is changed
From these, the correct answer is 2 3, 4
(a) 1, 2, 3, 4 (6 ) 2, 3 an infinite bus, is operating with normal excitation.
63. A 3-phase synchronous motor, connected to an
With decreases in load,
1. armature current decreases
2 . pf becomes lagging
3. pf becomes leading
4. load angle decreases
5. reactive power flows from motor to bus
6. reactive power flows from bus to the motor
From these, the correct answer is . „
(a) 1, 2, 4 (b) 3, 4, 5 (O 2, 4, 5 <tf> 1. 2, 4, 6

64. Consider the following statements : i&Qj i riCT ro n rtW nnwer to the hn<?
1. A grid-connected induction generator always supplies leading reactive power to the bus
2. An overexcited synchronous motor draws current at a laggmg p .
3. An underexcited synchronous generator connected to an infinite bus works at a leading pf
4. The torque angle of a synchronous machine is the angle between the excitation voltage and
the gap voltage •
O f these statements :
(a) 1 and 2 are correct «>> 3 and 4 are c0Irect
Ic) 1, 3 and 4 are correct (d) 1. 2, 3 and 4 are correct \
65. A cylindrical-rotor synchronous machine operates as a
1. generator at lagging pf if (Ef cos 5 + 70 r0 cos 0) > V*
2. generator at leading pf if (Ef cos 5 - l a ra cos 0) < Vt
3. motor at leading pf if (EfCos 8 + Iara cos 0) > Vt
4. motor at unity pf if (EfCos 8 - I a ra cos 0) = Vt

5. generator and also generating reactive power if {Ef cos 8 - I a ra cos 0) > Vt
From these, the correct answer is
(a) 1, 3, 4 {b) 2, 3, 5 (c) 2, 3, 4 {d) 1, 3, 5.
66. The two-reaction phasor diagram of a salient-pole synchronous generator is considered, where
7a = armature current per phase,
= armature leakage reactance per phase
Xd = cf-axis synchronous reactance, _
Xg = q-axis synchronous reactance ,
Vt = generator terminal voltage, taken as reference

The voltage in phase with quadrature axis will be the phasor sum of

te! u + !” r“ ‘ 4 X “' <A>


'W „ 'W „ A - ! (d) V , . / 0 r„ + / 0 (A ^ + X ,)
67. A synchronous motor is operating on no load at unity power factor. I f the field current i«.
increased, the pf will become
(a) leading and the current will decrease (6 ) lagging and the current will increase
geuig and the current will decrease (d) leading and the current will increase

■' ■' ' - • ' '


Scanned by Cam Scanner
1:. • ^ - - “■•"I
n,vi‘

Appendix-C________________ _________________________________________________ _________________________ 993

68. In a salient-pole synchronous machine,


(a) both Id and Iq are in phase with Ef
(b) Iq is in phase with S^and Id is at 90° to Ef
1 w u ? PhaSe With Ef andh is at 90° to Ef
>C
(rf) both/d and7, are at 90oto ^
1 j?8 Sfyient' P0^e synchronous machine with negligible armature resistance,
1. Ef= Vt cos 8 + Idx d ; generator at a lagging pf

2. Ef - V cos 8 - I dXd ; motor at a lagging pf

a co8 ^ + ! motor at a leading pf


4. Ef = V# cos fi —T v .
d d >generator at a leading pf.
Qffl above, the correct atat-Amn +.
ia) I, 3 (6) 2 4 S(car® 2 (d) 2 3

m°nic terms, because"80^ 8^ ° f 8 salient'Pole synchronous machine contains a second har

(h) the frequency^ha^atfto Second harra°nic component


(d) o f Z ,o t harmonics harm° niC C° mp° nent

71. In salient-pole machin el^ tan°e of air gap in the direct and quadrature axes.

t\ 3 3 : : i b K P * is not kept constant s° - to

72 *^ 1 1

3 Are" 4
1

4i A ^ " S e 7 i r ge r e X ^ d i0n
V f age reaCtanCe and armatUre reaCti0n mmf Ca“ be C0mputed from « *
"F ™
"y * " pf ’— «*
oni

5. M ° n f PfC ■ • 6 rCaCt,° ” mmf C“ ^ ° btained “ » and tw.


armature w f a i ” * * 1 r“ted armat“ re <wrent during short-circuit test ia equal t
■£ £ £ * * * l»ad on alternator, magnitude of tormina! voltage V , . magnitude f

From above, the correct statements are


fa) i , 2, 3, 5 ((,) 1, 3, 4, 6 (C) 1, 2, 3, 6 1, 3, 4, 5, 6
l ° r 3 ®ynlchr° n0US machine. the amplitudes of electromagnetic and rri, *
spectively P1 and P2.'The respective values of synchronous and synchronize P° WerS ar
rouml-rotor machine A and salient-pole machine B are as under : g P° Wers f<
a) A . Px sin 5, Pi cos 5 ; B : Pi + P2 sin 2 8, Px cos 8 + P2 cos 2 8
(6) A : Pi cos 5, Px sin 8 ; S : Px sin 8 + P2 sin 2 8, Px cos 8 + P2 cos 2 8
(c) A : Px cos 8, Pi sin 8 ; B : Px sin 8 + P2 sin 2 8, Px cos 8 + 2 P2 cos 2 8
(d) A : Px sin 8, Px cos 8 ; B : Px sin 8 + P2 sin 2 8, Px cos 8 + 2 P2 cos 2 8

74. p ie maximum power in cylindrical and salient-pole synchronous machines is obtained


tively at load angles of . 5pe
(a) 90°, 90° (6) < 90°, 90° (c)90°,<90° id) > 90°, 90°
75. A synchronous motor, fed from an infinite bus, is working at half-full load. If an increase in i
field current causes a reduction in the armature current, then the motor IS
i
(o) delivering reactive power to the bus at leading pf

Scanned by Cam Scanner


Electrical M
994 _____________________
(b) delivering reactive power to the bus at laS f ^ pf f
(c) absorbing reactive power from the bus a pf
(,d) absorbing reactive power from the u ^^ working at half-full load. Ifan
76. Asyncnronous
synchronousggenerator,
e n e connected
i arinatuIe current, then the generator is
- ~toan, armature
. •
in i.ts f
/i
-e ____ ______
iltd current _
causes a ro^nrhnn
n reduction 1m11Xil6
thes clfixi
(аa) delivering
deliveringreactive
reactivepower
poweruto
u Lithe
i<= bus iaffging. pff
(б )delivering reactive power to the bus S f
(c) absorbing reactive power from the bu pf
(cf) absorbing reactive power from e ^ angle 0f 2 0 = and with excitation voltage
77. A 3-phase synchronous motor is operatinga din terms 0fexcitation voltage willbe
' equal to applied voltage. The reactance d^p ^
s y n c h r o n o u s

(a) 69.46% (fe) 34.73% c ■ operates at a leading pf.For constant


78. A 3-phase synchronous motor, connected to an infinite bus, p
load torque, ifexcitation is increase , decrease
' (a) load angle 5 and power-factor angle 6 both decrease
(fe) 8 and 6 both increase
(c) 5 increases but 6 decreases
(d) 8 decreases but 0 increases infinite bus is overexcited. Considering only the
79. A synchronous generator connected svstem the machine acts as
reactive power, from the point ofview ofthe sys ,
(a) a capacitor (fe) an inductor (c) a resistor W as tu, cire j
, U inad anele 5 can be obtained from a knowiedge of[0 = pf
80. In a salient-pole synchronous machine, load ang
angle and ra = 0]
( \ \T J Y A
(a) VitIa,X d,Q j? T X B
<c> V„ Xd, Xq,0 W)«’ I
81. A salient-pole synchronous generator delivers about 20% ofrated power to an infinite bus. IfUs
field excitation fails,
(a) the machine will deliver reduced active power to the bus
(fe) the machine will deliver reduced active and reactive power to the bus .
(c) the machine will absorb reactive power from the bus but will deliver reduced active power
(d) the machine will run as an induction motor and will absorb both active and reactive power
from the bus
82. In a round-rotor alternator, reactive power is maximum at a load angle of
(a) 90° (fe) 180° (c) 0° (d) 45°
83. In a cylindrical-rotor alternator, maximum reactive power output is
3 y 3 V,
( a ) ^ \Er Vt) (6 ) (V j, - Ef)
As s
3Ef 3E e
(c) '-y * (Ef - Vt) Ud) - y 1 (Yt - Ef)
•fig s

84. Under the conditions ofmaximum active power output, in a round-rotor alternator, the reactive
power output is
3 V ,E f 3 Ef 3Vf , 3 V?
As (M -V
As- W - VA s- .A5
85. Under the conditions of maximum load on a cylindrical-rotor synchronous motor, the reach?
power input to the motor is
3 Vt Ef 3Ef 3 V? , , 3 if
(a ) f t (fe )--/ , {d)-==±-

86. In modern large sized synchronous machines, the synchronous impedance and operating
angle are respectively -
. (a) 0 .2,40° (fe) 0.6, 30° (c) 1 ,4 0 ° (d) 0 .5 ,4 0 ° . . ^

Scanned by Cam Scanner


Appendix-C
" ” I ‘ 995
87. Lists I and II pertain to th :----------------- --------------------------- -—
the two lists using the c o d w ^ e n ™ fa w a T u ste 16 aynChr°n0US machine at rated sP“ d- Match
List 1 - -
(from the phasor diagram) List 11
A. Vt cos 8 < Ef (Operating as)

B- Vt cos b > Ef L generator at lagging pf


2. motor at lagging pf

3. generator at leading pf
Codes : 4. motor at leading pf

112 3,4 A B
(C) 2’ 3 1, 4 1, 3 2, 4
88. A cylindrical-rotor svnchrnnA, {d) 2- 4 2 3
to) no“ S eS' U 18 SWitChed 0n 10 the its field windings shorted

(c) start as an?ndurr1 Synchronous speed

89. S eStT an<1 ^ SS a0s^ o n o ^ dmhoetorrUn * Synchronous Speed

*r i S t “ r suUd ^ p o s i^

(aT roS y86’ the ° ^ ect ooswer is g 3- phase swinging

90. During a disturbance^ a synch ^ 1 3. {d) 3 only

(c) torque aiding the developed torque S eddy' current torque


92. Damper winding is provided in a polyphase svnch “° * * * *
1. dampen out the noise of the machine m0t° r “ ordBr *°
2. prevent hunting
3. provide a starting torque
4. provide a cylindrical stmctnre to reduce wind friction
rom these, the correct answer is
(a) 2 only (b) 1, 2 and 4 (c) 2, 3 and 4 (<f) 2 and 3
• synchronous motor is running from an infinite bus of voltage V, in + *
of its rated load with a power angle 8Xbetween Vx and
ecreased to 25%. ^ a tta in s its new steady-state power angle 83 with Vx by initially “
(a) falling behind and making a complete rotation
Co) advancing and making a complete rotation
(c) falling behind, followed by oscillation about 82
(d) advancing, followed by oscillation about 82

®4. A 3-phase synchronous motor connected to an infinite bus is operating at half full-load with
normal excitation. When the load on synchronous motor is suddenly increased
(a) its speed will first decrease and then become synchronous

Scanned by Cam Scanner


(b) its speed will first increase and then becom e syn ^ en become synchronous
(c) its speed will fluctuate around synchronous sP
(d) its speed will remain unchanged ^ ^
95. In synchronous machines, damper windings are ^ run it as an induction motor
1. help in starting the motor 4 increase efficiency
3. help in reducing the hunting ^ subsynchronous speed
5. provide induction motor torque „ OT.SVnchronous speed
6. provide induction motor torque at sup
From these, the correct statements are (6) 1, 2, 3, 4, 5
(a) 1, 3, 5, 6 (d) 1, 3, 5
(c) 2’ 3- 6 v • fn
96. Function of damper bars in a s^ chf f a0Ul ^ C Jg gpeed 'Stator
(а) prevent rotor from running at subsyn speed
(б ) prevent rotor from running at supersynchronous , speea
(c) prevent rotor from running at sync^ n0US£fP®® noint
(d) reduce the rotor oscillations about the operating p Rotor

97. A synchronous motor h a v i n g ^


slows down to 1490 rpm ----------------- bar<? fz
Assuming a south pole on the stator near the damper Da
labelled 1, 2, 3 and 4 aa shown in Fig. C.71, the dtreotion of ■F3
\ J
induced force on conductor 3 is
i.d)FA
(a) F x (b) F2 (c) Fs
98. A salient-pole synchronous motor is running with normal ex­ Fig. C.71.
citation. If the excitation is reduced to zero . .
(a) it becomes an induction motor & »t becomes a reluctance motor
(c) it remains a synchronous motor (d) the motor would stop [I.A.S., 1995]
99. If the excitation is kept constant in an alternator, then the excitation voltage variation with
speed would be . .
(a) first linear and then curved upwards (b) first linear and then horizontal
(c) linear (cO first curved and then linear
100. A salient-pole synchronous motor is running clockwise at no load. I f its field current is reduced
to zero and then reversed, then
(a) the motor would stop
(&) the motor would run in the reversed direction
(c) the rotor slips by one pole-pitch and continues running at synchronous speed
(d) the rotor slips by one pole-pitch and speed would be less than synchronous speed
101. A salient-pole synchronous motor runs under steady-state conditions at no load with armature
current Ia. If the field circuit gets open-circuited, then
(а) the motor stops and Ia becomes very high
(б) it continues to run at the same speed and Ia increases
(c) it continues to run at the same speed and Ia decreases
(d) it runs at a slower speed as an induction motor and I„ increases.
102. A single-3-phase alternator in the laboratory is not run at synchronous speed. It would produce
no effect on its
(a) occ ,, v
/vi „ . (o) see
<c) occ and see , .v „ -
, oq ... W) z Pic
and the tranffn ? ° wf r MW 0.8 pf, ratings of alternator, its prime-mover, boiler
(I) inn Mu, f o rmershouIdresPectivelybe
(b) m 1? ^ MVA> 125 MA, 125 MVA
(o) 125 MVA, 100 MW, 100 M W 125 MVA

5 MVA’ 125 MVA, 100 MW, 125 MVA

-OOCtTii i-crcr~K^>
https://t.me/abcdelectrical

A P £2!!i£----------------_ _ _ _ _ _ 997
t 04. An isolated 3-phase altamof j ,■
a field current of 20 A. ^ 8^ Pf rated load at terminal voltaf?e Vf = 11 kV for
0.8 pf lag and 0 8 nf la r mamta:n*RE constant Vt, the field current required for rated loads of
(a) > 20 A, < 20 A WOuld resPectively be
(c) < 20 A, < 20 A (b) > 20 A, > 20 A
105. The name plate of an alte t * ^ < 2° A> > 20 A
(a) lagging so that exdtatio™emfFndir tea 3n °Perating Pf of 0.85. The nature of this pf is
(b) lagging so that £,-> v Q j . ^ Hninal voltage Vt and alternator absorbs reactive power Q
(0 leading a„ that fi'< V “ a ? ™ t° tdelivere «
fa) leading s0 that E, < v and ai*ernator absorbs Q
106. A synchronous motor and an ^ 9
Pfl If these machines are loaded trrJlwr1* arf delivering about 5% of their rated power at unity
fa decrease, motor operates a U CP acit* tiea armature currents
(W.increase, motor operates at a W P C r e a to r at leading pf

W) £ £ £ 2 , r° r atet at S ' ^ a“ emat0r St laBgi" B Pf


>»’ ■ £ alterar : ; pf a"d - - a t o r at .ending pf

fa)6/ MtCsatIOn em f£f ' 3 respectively givenbv sallent‘Pole *yPe are operating at leading pfs.

(ci v . “ 9 XdId, Vt cos 8 + Inr + y , /


VO V , COS 8 - r / _ V r T, «. *ara +Ad 7d
17
V
WJ V,
VjcosS
cP* 9 yd
9 d w COS
cos 8o_ + r 77«7++ Ad/
+ r Y
V Tr
cos 8 + r 7/ _- X r rr
\r * ° 1q+JLdid d
... „ ° <? Ad 4cf> V, cos 8 - r 7 -t- y r
fa)PC U to determine * ‘

S P7 Wv^ ’ aaare reec^ro Md n T T iS raMtance


W) sub-transient reactance negatlve sequence reactance
109. While conducting a ‘slin’ test f

(a) sf, sf, sf and f


s is k s - — —
fc> /» s/i 7 and 7 W s/i 7, s/and f
110. Squirrel-cage bars placed in th* mtn . r W) f' (1 ~ S) f’ (2 ~ s) f and f
hunting B P ther°torp0lefaCesofa 3 ^ alternator h e l p i n g , • .
1. above synchronous speed onlv 2 hain , Clng tke
3. at synchronous speed only W synchronous speed only
From above, the correct answer is
(a) 1 and 2 „
(c) 2 and 3 6) 1 and3
ii, . , . W) only 3

increment in load angle’ causes^chrenfring^owwtoXw from ° °ertain l0ad angle' A sudden


■i. innnite bus to motor o .
3. infinite bus to generator a ™ ^ f° ^ « bus
j, B ur 4- generator to infinite bus
rrom these, the correct answer is
^ d (b) 2 4
(c) 2 ^
KC) d (d) 1, 3

H2. In a slip test on a synchronous machine, match the following :


Ii!

Scanned by Cam Scanner


998
Electrical

Armature flux Voltage in field winding Armature current


Reacta
L Maximum Maximum Maximum
2- Maximum Maximum Minimum
Ki
3- Maximum Zero Minimum
4- Minimum Maximum Minimum
xd
5- Minimum Maximum Maximum

From these, the correct answer is


(a) 1, 4 (6) 2, 5 (c) 3, 4 (d) 3, 5
113. In a slip test on a salient-pole synchronous machine, the emf induced in the field winding
is
1. zero for X d 2. zero for Xn
3. maximum positive for Xa 4. maximum negative for X„
5. maximum positive for X d
From these, the correct answer is
(a) 1, 3, 4 (6) 1, 3, 5 (c) 1, 4, 5 (d) 2, 3, 4
114. A synchronous condenser is
(a) an over-excited synchronous motor driving a mechanical load
(b ) an ordinary capacitor bank
(c) an over-excited synchronous motor with no shaft extension
(d } an over-excited synchronous motor without mechanical load
115. A 3-phase overexcited synchronous motor is installed near a 3-phase induction motor (IM) with
a view to improve the power factor. With this installation,
(а) IM p f improves and its current decreases
(б) IM p f does not change but its current decreases
(c) IM p f improves but its current does not change
(d) IM p f and its current do not change, p f o f the combination improves.
116. A 3-phase induction motor draws 1000 kVA at a p f o f 0.8 lag. A synchronous condenser i
connected in parallel to draw an additional 750 kVA at 0.6 p f lead.The p f o f thetotalload
supplied by the mains is
(a) unity (6) 0.707 lag (c) 0.707 lead id) zero
117. A synchronous motor operating at rated voltage draws 1.0 pu current at 1.0 pf. The machine
param eters are : synchronous reactance 1.0 pu ; armature resistance, negligible.
Apart from supplying this rated power, i f the motor has to supply an additional leading reactive
power o f 0.8 pu, then the field current has to be increased by
(a) 42% (b) 46% (c ) 52% (d) 60%
118. While starting a 3-phase synchronous motor, its field winding should be
(a) kept open (6) short-circuited
(c) connected to a dc source (d ) none o f these
119. An ideal synchronous m otor has no starting torque because the
(а) rotor is made up o f salient poles
(б) relative velocity between the stator and rotor mmfs is zero
(c) relative velocity between the stator and rotor mmfs is not zero
id) rotor winding is highly inductive
120. During the starting o f a 3-phase synchronous m otor by dam per bars, the field winding is usually
short circuited so that starting torque is equal to
(а) induction m otor torque
(б) induction m otor torque plus an additional torque produced by short-circuited field winding
(c) electromagnetic torque produced by the interaction o f arm ature and field-winding mmfs
(d) reluctance torque due to rotor saliency
121. In an alternator, no-load rotational loss and short-circuit load loss are respectively made up
Ort F w n n’ Win? age (FW) plus <4>en-drcuit core loss (C L )]; (stray load loss (SLL) plus open-circuit lossj
(f>) (FW) loss ; (SLL plus direct load loss (DLL)]

>ocn ii i c u uy v ^ a i i l o o a i ii i ci
https://t.me/abcdelectrical

Appeodii£_

(c) IFW plus field circuit lossl ■ ra t i n , t , “


IFW plus CL) ; |SLL p,US DLLI
UJ. In synchronous mnchinos the . , amlatUre Wi" dingl
’ 1. eddy currents in the T ™ ? “ ““ d bV
2. . r mn t u r e . e , J flua™ a “ r; “ nd" « » -

3. skin effect in the armature L T u c “ tMth' C° rc’ 6" d PlaU!a ^


4. armature leakage flux as
From these, the correct answer l0SS in teeth’ core- and P,ates elc
(a) 1, 2, 4 (6) 1 ,3 4 •
123. Polyphase synchronous motor is nr«i 1 j ^ 4 2’ 3’ 4
a 600 kW at 3000 rpm G1Te 0ver P°lyphase induction motor for a load requiring
(c) 600 kW at 1000 rpm <&> 600 kW at 1500 rpnj
124. For driving a 750 few ^ 666 rpm

2 k wUl h e l f u l0Wer ^ '* eUU*


3- of the ^ s ib U tvno?°mPre8S0r Pressure faster
4 the sta^Hnl y f ° peratin^ a* better pf
e starting torque requirement is low
y t these statements
} a„nd 2 are correct
1, 2 and 3 are correct ^ 2 and 3 are correct
125. Match List I with I ;<=«■ it . (d) 3 and 4 are correct
Codes : 80 “ GCt the “ n *61 answer using the codes given below the lists :

List I
(Synchronous machines) List II
(Phasor diagrams)

A. Underexcited alternator

B. Underexcited synchronous motor

C. Overexcited alternator

D. Overexcited synchronous motor

Scanned by Cam Scanner


h

Electrical Mac
1000
A B C D
A Ii C D 3 4
lb) 2 I
1 2 3 4 4
(a) 3 2 1
Id)
2 3 1 4
(c) 2 3 \ 1 answer using the codes given below the lists
12G. Match List I with List II and select the corre

List 11
List I (Phasor diagrams)
(Synchronous machine Operation)

A. At the moment of synchronizing

B. As an alternator

C. As a synchronous motor

D. At the moment of losing synchronism

4.

C o d es C D
A B
B C D
A 1 2 3
2 1 (tb) 4
(a) 4 3 3 2
(d) 4 1
3 4 1 2
(c)
U 7 . Consider the M o w in g statements regarding the operation o f 3-phase synchronous mach.nes
depicted in Fig. C.72 (a), (b) and (c).

(c)
(6)
Fig. C.72.
o f Fig. (a) is operating as a synchronous motor at som e load
1. Machine
o f Fig. (a) is operating as an alternator at some load
2. Machine
o f Fig. (b) is operating as a synchronous m otor at som e load
3. Machine
4. Machine o f Fig. (b ) is operating as an alternator at some load
5. Machine o f Fig. (c) is operating as a synchronous motor at no loan
6. Machine o f Fig. (c) is operating as an alternator at no load

From these, the correct statements are


(a) 2, 3, 5, 6 (6) 1, 4, 5 (c) 1, 4, 6 (d) 2, 3, 6

Scanned by Cam Scanner


* if.'-i**•

________ tool
j«8. A stand alone engine driven e,™ u . .
now connected across the nous generator is feeding a partly inductive load. A capacitor is
(a) the field current . ^^Pletely nullify the inductive current. For this operating condition
it) the
(b) the field
field current
S “ e f5,,!i
and * input
“ P“ ‘ ,have to be
ve to be reduced
reduced
(c) the field current has to he18PUt to be increased
(d) the field current ho* tn i> lnc^eased and fuel input left unaltered
^ p„ y
v ,. e UCed and 1
u “Ha fuel input left
1161 mput unaltered
lelt ui [GATE, 2003)
Curves a and Y
129. CurvesXand in Fie n 7n .,
Fin
zero power factor (z Dn rh e open circuit and full-load
generator. Q is a point L t h l of a synchronous
vo tage. The vertical d i s ^ ; & characte*stic at 1.0 p.u.
voltage drop across ln *6' C.73 gives the
(a) synchronous reactance (M . .
' “ °her reactance . jnagnetizing reactance
W) leakage reactance
130. A round-rotor generator ■. ,GATS- 20031
® d X = 1,1 pu is co d to”. ™ ' V° ltae<> S l “ 2 0 pu
">°tor with internal velta» £ ° - l 7 “ " d T j Wohronous
reactance of the line eon™ . 2 " V8 pu and * = 12 pu. The
* 0.5 pu. When the generatogs,.uer enn at0r to the motor
rotor angle difference between tv, .. P-u- power, the
(a) 57.42° between the machines will be Field current
(c) 32.58° (6) 1° " Fig. C.73.
(d) 122.58°
[GATE, 2003]

1- (6) an sw er
2. (d)
6 .(d) 3. (d)
7. (6) 4. (c)
11. (6) 8. (6) 5- (d)
12. (“ ) 9. {b) 10. (a)
16. (a) 13. (b)
17. (a) 14. (6) 15. (a)
21 . (6) 18. (d)
22. (d) 19. (c) 20. (d)
26. (c) 23. (a)
27. (a) 24. (a)
31. (C) 28. (6) 25. (d)
32. (c) 29. (a) 30. (6)
36. (a) 33. (6)
37. (d) 34. (6)
41. (c) 38. (a) 35. (6)
42. (6) 39. (6)
43. {d) 40. (6)
46. (c) 47. 44. (d)
(6) 48. (a) 45. (6)
51. (a) 52. 49, (6)
(d) 53. (6) 50. (d)
56. (d) 57. 54. (b)
(d) 58. (c) 55. (a)
61. (d) 62. 59>- (a)
(a) 63. {b) 60. (d)
66. (c) 67. 64. (6)
(d) 68. (6) 65_(6)
71. (a) 72. 69. (d)
(6) 73. (d) 70.-(d)
76. (c) 74. (c)
77. (6) 78. (d) ?5. (d)
81. (c) 79. (a)
82. (c) 83. (a) 80. (6)
86. (c) 84. (c) 85. (d)
87. (d) 88 . ( 6)
91. (d) 89. (c) 90. (6)
92. (d) 93. (d)
96. (d) 94. (c) 95. (d)
97. (a) 98. (6) 99. (c) 100. (c)
101. (6 ) 102. (6) 103. (6) 104. (a)
106.(d) 10 5 .(fe)
107. (d) 108. (6) 109. (a)
111. (a) 110. (a)
112. (d) 113. (a) 114. (c)
116. (a) 115. {d)
117. (6) 118. (6) 119. (c) 120 . ( 6 )
121. (d) 122. (c) 1 2 3 .(d) 124. (c) 125.(b)
126. (d) 127. 128. (d) 129.(a)
(d) 130. (c)

Scanned by Cam Scanner


____________________ H e c t r i c a l MachiInc,.
1002 .
POLYPHASE INDUCTION MOTORS
• j 3-ohase balanced supply is given to the rotor and stator
1. In a 3-phase slip-nng induction motor, 3 p
winding is short-circuited. The rotor would ^ in t^ e direction of rotating fie]d
(a) not run . ~ ,j (d) run at half the synchronous speed
(c) run against the direction of:rotat.nlj M Btator and 2-pole rotor. With it, stator
2. A 3-phase slip-ring induction motor (bKl ) a no_ioad speed
energised from 50 Hz source, the rotor wo ^ SOmewhat less than 3000 rpm
(a) somewhat less than 1500 rpm ^ 0f zero rpm
somewhat less than 2000 rpm
( c ) . - hase induction motor (IM) is to
3. The effect of increasing the air-gap lengtn h v ^ improve the power factor
1. reduce the pulsation loss ^ worsen the power factor
3. reduce the air-gap flux q increase the efficiency
5. reduce the torque
From these, the correct statements^are g g g 1; 3, 4, 5

4. The rotor of an IM cannot run at synchronous speed, would be zero


1. rotor emf would be zero 4 rotor pf would be unity
3. rotor torque would be zero g ro^or core loss would bezero
5. stator core loss would be zero *
From these, the correct statements are
(a) 1, 2, 3, 4, 5 (6) 1, 2, 3, 4, 6 (c) 1, 2, 3, 5 (d) 1, 2, , ,
5. A 3-phase squirrel-cage induction motor (SCIM) has its statorrewound for 6 poles without any
alteration in the rotor. The motor would now run at a speed
(a) < 1000 rpm < 15UU rpm
I (c) < 1200 rpm zer0 rpm .
6. A 3-phase SCIM, designed to operate with stator in star, requires W k g of copper for its stator
winding. If this motor is rewound with stator in delta, with current and flux densities, kW and
voltage ratings remaining the same, then weight of stator copper would be
(a) V3 W kg

< » ;§ • Wkg
(c) Wkg
(id) more or less than W kg depending upon other design parameters
7. If the rotor of a 3-phase IM is assumed purely resistive, then electromagnetic torque in the motor
is ' 1
(a) minimum with load angle 0° (6) minimum with load angle 90°
(c) optimum with load angle 90° (d) optimum with load angle 0°
8. If the rotor of a 3-phase IM is assumed purely inductive, then the electromagnetic torque
developed in the motor is
(a) optimum with load angle 5 = 0° (b) optimum with 5 = 90°
(c) optimum with 8 = 0° (d) zero with 8 = 180°
9. A 3-phase, 4-pole, 50 Hz IM runs at a speed of 1440 rpm
1. Its slip is 0.04
2. Its rotor field rotates at 60 rpm with respect to rotor
3. Its rotor field rotates at 60 rpm with respect to stator field
4. Its rotor runs at a speed of 60 rpm with respect to stator field
5. Its rotor field rotates at a speed of 1500 rpm with respect to stator
From these, the correct statements are
(“ ) J. 2. 4, 5 (6) i , 2 , 3, 4, 5
M L 2. 5 (d) 1, 2, 4

Sea n n ed by~Ua m S ca n n er
10- £ £ w-n t S T ' * * * • m H U ** 60 “ * « " “ ** “ “<1 It* "to r fr a n 30 H« aource.
1. 1500 rpm a, jhh) rpm g Him rpm 4. '1400 rpm 5. 2100 rpm
From these, the correct answer in
(a) 3 only (6 )4 only (t) I g, 5 {ti) 3( 4
1J. The relative speed between the magnetic fields of ototor and rotor under steady-state operation
is zero for a
1. dc machine 2, 3-phase induction machine
3. 3-phase synchronous generator 4 . 3-phase synchronous motor
From these, the correct statements are
(a) 1, 2, 3 and 4 (b) 3 and 4 (c) 2. 3 and 4 (d) 1, 3 and 4
12. The m m f produced by the rotor currents of a 3-phase 1M
(a) rotates at the speed of rotor in the air gap
(b) is at standstill with respect to stator mmf
(c) rotates at slip speed with respect to stator mmf
(d) rotates at synchronous speed with respect to rotor
13. If the rotor pf of a 3-phase IM is 0.866, then spatial displacement between the stator magnetic
field and rotor magnetic field will be
(a) 30° (6 ) 90° (c)120° (d) 150°
14. The stator o f a 6-pole. 3-phase IM is fed from a 3-phase, 50 Hz supply which
pronounced 5th time harmonic. The speed of the 5th space harmonic field produced by the fifth
time harmonic in the stator supply will be _
(a) 1200 rpm (6 ) 1500 rpm (c) 1000 rpm (d) 5000 rpm
IS A 6 nole 50 Hz wound-rotor induction motor (WRIM) when supplied at the rated voltage and

fi-equency of voltage acrossslip rings will be


f/iTzero (6 ) 50 Hz (c)100 Hz (d) 200 Hz

16. A 6-pole, 3-phase WRIM i s i s in t h e ^ e


S S T n r r T I p u f r r n t m m t h e n the « of the etator v o it a , w„, he

17. The^absolute speed of the magnetic field in space of a 3-pha« mtojfcd m is


(а) synchronous speed, N s ^ ^ +^ [I.E.S., 19961

<C) ? " f '’ ves 120 oscillations per minute when c o n n e d to the mtnr of an IM. Urn stainr
18‘ ^ ° “ 50 Hz. The riip of the mofer is ^ K,B,S ,S93)

(0) 2% S ta te m e n t frem the following as the ioad on a 3-phase IM is memased from


19. Choose the correct statement fr
no-load to full-load , efficiency
p o ^ r factor ^ increases
. inCT increases
<*> mCreaSeS increases decreases
(б) decreases decreaflCS decreases
(C) decreases decreases
«D increases the a i r - g a p power, then
20. In a 3-phase IM, if Pg represents power developed

r o to r o h m ic lo s s (1 + s ) ^
g
to) s Pg
tpg
ib) (1 - S ) P g
(1 ~S)Pg
ic) Sp
g

o u a i ii i c u uy o a in o u a i 11i c i
1004 Electrical Machinery

C
rf) * pe P. /( l - s )
21. The variable resistance representing the mechanical load in the equivalent circuit of a 3-phaSe
IM ia given by
(a) r2 (s - 1) (b) r2 ( -1- 1 N (c) r2 1 - (d) r2 (1 - s)
s ' ' - $
22. If tos ss synchronous speed in mechanical rad/s, then torque in a 3-phase IM is given by
3 l\ r2 3 JS 3 / 2 r2 3 l\r2
(«) (6) ■(c) id) 2 2
P s- to. co*
*■ 2 ' ® -*
23. If stator impedance of a 3-phase IM is neglected, then the maximum torque (Vx = stator phase
voltage) is given by
V\ 3 Vf
(a) 3 * („MJ L . ¥t -t . 3_______
(c)
<0* * v“ y cos r2 us 2 r2 cos z x2
24. At low values of slips, the torque in a 3-phase IM is given by (V^ = stator phase voltage)
V? t/2 „ &
(a) r e a — s (b) Te a
r2
(c) Te a v f r2 (1 _ s) (d) Te a
. r<1
25. In a 3-phase SRIM, Tem = maximum torque, smT = slip at maximum torque and Te ,st = starting
torque. The following statements are made for this IM, when resistance is added in the rotor
Lircuiii a
1. Tem remains constant, smT > 1, Test < Tem
2. Tem remains constant, smT < 1, Test < Tem
3. Tem remains constant, smT= 1, Tcst = Tem
4. Tcm remains constant, smT> 1, Tcst > Tem
From these, the correct answer is
(a) all are correct
(6) 1, 2, 3
(c) 2, 3
id) 2, 3, 4
26. Three-phase induction motors with open slots have
1. more starting torque Test, more starting current Ist and improved pf
2. more Test, more Ist and worsened,pf
3. more Test, more breakdown torque and improved pf
4. more Testl more breakdown torque and worsened pf
5. more Test, more slip at which maximum torque occurs
From these, the correct answer is
*’ 3 (6 )1 ,3 , 5 (C) 2 ,4 ( d ) 2 ,4, 5

27. A SCIM drives a constant-torque load. If supply voltage reduces to times its previous stator
voltage, then slip and current get modified by factors of

$^P current
ia)
2 2
ib) <2 2
(c) 2 V2
(d) Vi2 l/<2

>canned by Cam Scanner


Appendix*C
-------- 1005
28. In a 3-phase IM if0tn- • , . ,
by keeping V/fm ne* * 1^ p€ ince 18 n^ected. If supply voltage and frequency are decreased
occurs « nnu v ’ maximum torque Tem and the slip smT at which maximum torque
" 7 ™ would change as under :
Ic\ T * ! " ^ eases and smT decreases (5) Tem decreases and emT increases
«m s constant and smT increases (<f) Tem remains constant and $mT decreases
29. In case of 3-nhaso m * .
the rotor ohmic losses’ are P° W6r “ 2700 W and mechanical losses are 180 W. At a slip of 4%,

so. & L !i d 2 w (6) 120 w (c) 108 w {d) 105 w


(а) improve ste r H n g t^ u cendOSe^ 3*ots are used in 3-phase induction motors, essentially to
(б) increase pull-out torque
(c) increase efficiency
(d) reduce magnetizing current and improve pf

” ■ ^ u rb T W' r OUtPUt 0f3‘Ph“ e IM “ 15 kW and the corresponding slip is 4%. The rotor ohmic
to) 800 W (6) 625 W (c) 6 5 0 w (d) 700w U.B. S . , m n

motor supply mains. When connected in delta, the


(a) 20 Vjf A (fc) i*2 . . 9n . °
y ’ 15 (c) 20 A (d) 10 A

S3. Breakdown torque of a 3-Phase IM of negligible stator impedance is


3. oc -L 4. « x2
. x2
1 R I i
2 x2 6 “ 7 T .-i 8

From these, the correct answer is


(a) 2, 5, 6, 8 (6) 2, 5, 6, 7 (c) 1, 3, 6, 7 (d) 2, 4, 6, 8

* * indicated in list I1^and


n d le lS f tthe
select h ^ correct
C° nf answer
qUent Chang6S indicated
using the in list
codes given 11 with
below thethe
listsparameters
:
List I
List II
A. Power factor L Increase
®' 2. Decrease
C, Effective rotor resistance
D. Magnetizing current
Codes:

A B C D A B C D
(a) 1 1 1 2 (6) 2 2 2 1
(c) 1 1 1 — id) — 1 1 -

35. A 3-phase SRIM is fed from the rotor side with stator winding short circuited. The frequency of
the currents flowing in the short-circuited stator is
(a) slip frequency (&) supply frequency
(c) zero (d) frequency corresponding to rotor speed
36. A 3-phase IM has rotor resistance R2, standstill rotor induced e m f# 2 and stator to rotor effective
turns ratio of m. In an equivalent circuit of this machine, the rotor circuit resistance is shown
a sm 2 - R 2/ s, where s is the slip. This implies that the value of the equivalent rotor circuit voltage
will be
ta)E2 (b)sE2 (c ) m E 2 {d )m sE 2 [I.E.S., 1993]

Scanned by Cam Scanner


1006 Electrical M a c W r}

the codes given below the lists :


37. Match List I with List II and select correct answer using
List II
List I
(Apparatus)

A. 3-phase IM on load

B. Synchronous generator

'a
-'WvV -W -

C. Single-phase transformer on load 3.

s.x,
-A W - -W P - -/VW\---- 'TTiST'—
D. D.C. series motor 4.

-+ o- -AVW
Codes :
5.
A B C D 6 e»
(a) 1 2 3 4
- o-
(b) 2 3 1 5 r1 X, r2/S *'2
■m a a ,------- Tnnp- ■'VWV-----Tftnp-
(c) 6 3 1 5 6.

(d) 6 3 4 2
'm

bs a o i Tlf . . . . [I.E.S., 1996]

1 pu at 6% siip- k has ^ °f
operation of the motor the minimum S^ T ” 6 1S assume^ to be linear. For the stable
W 0.25 “ ^ 5 0 m,mT ) 0.70S 7UPPly
A prime-mover drives a 6 nnlo o v. • j
Hz, 3;phaae supply £ , t L £ ^ ” 1 ^ I^ ^ ™ Terter- ^ in v e r te r is connected
W 12OPHzS160 H z'1” 118 fr° m the inverter are me' mover speed is 3000 rpm, the frequencies
. (C) 176 Hz’ 86 Hz (&) 90 Hz, 210 Hz
. . (d) 180 Hz, 210 Hz
hi-.I-,d r • / / 1. . mp / abrifipl p n t r i ria 1

AftpewtivC 1007

40. I ho torque-si ip characteristics of a polyphase induction motor becomes almost linear at small
values oi slips, because in this range of slips,
in) the effective rotor-circuit resistance is very large compared to the rotor reactance
(ft) the rotor resistance is equal to the stator resistance
(c) the rotor resistance is equal to the rotorreactance
(tI) the rotor reactance is equal to the stator reactance
41. The speed of a 3-phase IM is controlled by controlling its supply frequency. If the speed of the
machine is reduced by reducing the frequency by 50% of the rated frequency ; to keep the flux
in the machine constant, the motor voltage compared to rated voltage must be
(a) increased by 25% (ft) increased by50%
(c) decreased by 50% (rf) decreased by 25%
42. 1 ho effect of adding external resistance in the rotor circuit of 3-phase SRIM is to
1. increase the starting torque
2. decrease or increase the starting torque 3. reduce the starting current
4. reduce the maximum torque 5. improve the pf at starting
From these, the correct statements are
(a) 1, 3, 4. 5 (ft) 2, 3, 5 (C)2,3, 4, 5 (rf) 2, 4, 5
43. A 6-pole, 3-phase IM develops maximum torque at 1000 rpm when operated from a 60 Hz supply.
Rotor resistance per phase is 1.2 £5. Neglect stator impedance. The speed at which it will develop
maximum torque when operated from 50 Hz source is
(a) 1200 rpm (6) 1000 rpm (c) 800 rpm (rf) 960 rpm
44. A 6-pole, 3-phase IM develops the maximum starting torque at 1000 rpm when operated from a
60 Hz supply. Rotor resistance per phase is 1.2 Q. Neglect stator impedance. The external
resistance to be connected in series with each rotor phase, for developing maximum starting
torque on 50 Hz supply is
(a) 6 £3 (ft) 4.8 Q (c) 7.2 £2 (rf) 5 £1
45. A SCIM having a rated slip of 4% on full load has a starting torque the same as full-load torque.
The starting current is
(a) equal to full-load current (ft) twice the full-load current
(c) four times full-load current (d) five times full-load current
46. Approximate phasor diagrams of a 3-phase induction machines for different operating conditions
are indicated in List I (with usual notations). Match List I with the operating conditions given
in List II and select the correct answer using the codes given below the lists :

List I List II

1. Full-load generating

2. Blocked rotor

3. Full-load motoring

4. No-load

Scanned by Cam Scanner


Electrical
1008

Codes A B C /J
A B C D 2 3 4
(b) 1
2 4 1 3 4 2 3
(a) (d) 1
2 4 3 1
(c)
47. The slip of a 3-phase IM can be measured by
1.actually measuring the rotor speed
2 .stroboscopic method .^nries
3 .comparing rotor and stator supply frequenc ^
4. connecting a centre-zero galvanomet
From these, the correct statements are 2 3 ,4
(a) 1, 2, 3, 4 (b) 1, 2, 3 (c)-1,3, 4 ^ ’
48. Consider the following statements .
As a 3-phase IM is loaded from no-load to rated load
1 .there is an improvement in the power factor
2 .the torque increases almost in proportion to slip
3 .the air-gap flux falls sharply
Of these statements ^ ^ an(j 2 are correct
(a) 1, 2 and 3 are correct , ^ ancj3 are correct [I-A. S., 1997]
(c) 2 and 3 are correct w L ,
49. A 3-phase IM is connected to a 3 -phase supply. One ofthe line fuses is blown out when the motor
is running. Consequently,
1 the motor will come to a standstill , .
2 *the motor will continue to run at the same speed with line current unchanged
3.'the motor will continue to run at a slightly reduced speed with an increased current
4 .the rotor current will have both sf and (2 - s)/ component frequencies, where s is slip and f
is the supply frequency.
From these, the correct answer is
(a) 1 alone (b) 2, 4 (c) 3, 4 ([d)3 alone
50. Consider the following statements about the operation ofa three-phase IM running at hill load :
Ifone of the supply line fuses blows off, the motor will
1.stall
2 .continue to run with increased slip
3 .continue to run with normal supply current
4 .continue to run with excessive supply current
Of these statements
(a) 1 alone is correct (b) 3 alone is correct
(c) 2 and 3 are correct id) 2 and 4 are correct [LA.S., 1996J
51. The chart given below indicates four different conditions under which an IM may be called upon
to operate, with a constant shaft load.
Applied voltage Frequency Rotor resistance
1 Normal Normal Normal
2 80% Normal Normal
3 80% 80% Normal
4 Normal Normal 80% !
The operating conditions which lead to increasing values o f running slips will be in the sequence
(a) 1, 4, 3, 2 (6) 1, 4, 2, 3 (c) 4, 1, 3, 2 (d) 4, 1, 2, 3 U-A S -’ V ?
52. Which one of the following can be obtained by the equivalent circuit o f an electrical machine .
(a) Temperature rise in the cores
(h) Complete performance characteristic o f the machine

Scanned by Cam Scanner


hi- t-ps •./ /t. . mp/ abpdpl p r t r i pal

Appendix-C___________ 1009

(c) Type of protection used in the machine


(d) Design parameters of the windings [LAS., 1993]
53. If a 400 V, 50 Hz star-connected 3-phase SCIM is operated from a 400 V, 75 Hz supply, the torque
that the motor can now provide while drawing rated current from the supply
(a) reduces (6) increases (c) remains the same
( ) increases or reduces depending upon the rotor resistance
54. Neglecting the shunt branch from the equivalent circuit of a 3-phase IM, the slip at which
maximum torque occurs is given by

(°) — (h\ =
** ^rf + (x1 + x 2f

3-phas^ff/Hs^ Stat° r *mPe^ance> the condition for the development of maximum power in a
r2 ( l - s ) „
(o )— ~ r2
s z (b) : = x2
, v ^*2 ~ s) r~v jt _ /i' .
( c )— 7 — (rf)^ i z f ) =0

56* as under386 IM’ t° ique T* at any sUp s and maximum torque Temat slip smTare related

(„)£-. T, Zn„T
l m S+3^
, \ e 2S T s <? ™
W T ~ 2 2 (d) — — = m
Tem S + S I t W Tem s2 + s2 mT

57. A 3-phase SCIM is running at slip s with synchronous speed Ns clockwise and rotor speed Nr.
If its two supply leads are interchanged, then at that instant
1. slip is (2 - s)
2. speed of air-gap field with respect to stator is (Ns +N r) clockwise
3. speed of air-gap field with respect to rotor is (Ns +Nr) anticlockwise
4. speed of air-gap field with respect to stator is Ns anticlockwise
5. effective rotor resistance increases
6. stator current decreases
From these, the correct answer is
(a) 1, 3, 4, 5, 6 (6) 2, 3, 4, 5, 6 (c) 1, 3, 4, 5 (d) 1, 2, 4, 5
58. No-load current of a 3-phase IM in terms of its rated current is
(a) 10 to 20% (b) 2 to 6% (c) 20 to 30% (d) 30 to 50%
59. No-load and full-load pfe of a 3-phase IM are respectively of the order of
(a) 0.2, 0.85 (b) 0.5, 0.8 (c) 0.7, 0.9 (d) 0.3, 0.95
60. Two 3-phase SCIM shave efficiencies of 82% and 92% and their no-load pfs of 0.1 and 0.3. These
two motors A and B have the following values for their pfs and efficiencies :
(a) Big motor (BM) A : 0.1, 82% ; smaller motor (S M )B : 0.3, 92%
(b) B M A : 0 .1 ,9 2 % ; S M B : 0.3,82%
(c) B M A : 0.3, 9 2 % ; S M B : 0.1,82%
<d) B M A : 0 .3 ,8 2 % ; S M B : 0.1,92%
61. Motor A has deeper and narrow slots, whereas motor B has shallow and wider slots. Induction
motor A, as compared to motor B, has
1. less pull-out torque 2- ,le3s sturtmg torque
3. less operating speed er °Pera p

Scanned by Cam Scanner


Electrical Machine.,
1010 _ _ _ ---------------------— — “ ' — — I

From these, the correct answer is ^ 2 ,3, 4


(a) 1,2, 3, 4 (b) 1,2, 4 (<?) 1- **>4 0 -pole, 50 Hz SRIM makes 43 comply.
62. A centre-zero ammeter connected in the rotor circu MDeC(j0fstator field with respect to rotor
oscillations in one minute. In rpm, the rotor sp *
are respectively {,n985, 970
(a, 985, 15 (8 ,970, 1000 M N* » nurabor „f aten.tion8 pur m i„ulc
63. The no-load speed of a 3-phasc, 50 Hz IM m m f with respectto rotor would respectively
which the rotor emf will make and the speed ofrotor rnmt wi i ly
be (b) 30, zero rpm
(a) 0.5, 1485 rpm {d)30j 15 rpm
(c) 15, 15 rpm standstill is twice of its resistance. The
64. For a 3-phase, 50 Hz, SCIM rotor l e a k a g e obtaincd at utarting is
frequency of the supply at which m a x i m u m torq
(a) 50 Hz (5) 25 Hz (c) 100 z of H 4 0 rpm.Tota] ataLor
65. A 3-phase, 50 Hz IM takes a power input of30 kW at itsxun i
losses are 1 kW. The slip and rotor ohmic losses at fu 1 oao a
S o w m ow
(c) 0.04, 1160 W (d) °-04, 1
66 .A 3-phase SCIM has its copper bars replaced by aluminium bars. J ns c ange, mo r
characteristics would be as under: , «t mnvimmn
(a) starting torque T„, and maximum torque Tm would be same, sl.p »„r at maximum torque
would be more
(b) Tcst more, Tem same, smT more
(c) Test same, Tem more, smr same
(d) Tcst more, Tem less, slnT more
67. Match the correct values ofslips for the three operating regions of a 3-phase IM :
Generating region Motoring region Braking region
(a) (- 1) to0 0to 1 1 to 2
(b) 2 to 1 1 to 0 0 to (- 1)

(c) 0 to- 1 0 to I 0 to (- 1)

id) (- 1)to 0 1 to 0 0 to 1
68. A 3-phase SRIM with negligible stator impedance operates on a balanced 3-phase ac supply.
Consider the following statements in this regard :
The maximum torque developed by the motor
1. is independent o f the value o f the rotor-circuit resistance
2. is directly proportional to the square of the supply voltage
3. occurs at a slip whose value is independent of the value o f the rotor resistance.
Of these statements
(a) 1, 2 and 3 are correct (6) 1 and 2 are correct
(c) 2 and 3 are correct (d) 1 and 3 are correct. [7.A.S., 1993]
69. The stator of a 4-pole, 3-phase induction machine is supplied from 3-phase, 50 Hz supply and a
prime-mover drives its rotor at a speed of 750 rpm. The slip-rings o f the machine are open-
circuited. The frequency o f the voltage across any two slip rings is :
1. 50 Hz 2. 75 Hz 3. 100 Hz 4. 25 Hz
From these, the correct answer is
W l ,2 ' (6 )1 ,3 (c )2 ,3 (d) 2 ,4
TJ,a r ^ C"’ ^'Pbaf)e>50 Hz induction machine, the slip-rings o f the machine are open-circuitcd.
jne frequency of the voltage across slip rings is 75 Hz. The rotor is driven at a speed of
L 750 rpm 2. 1500 rpm 3. 2250 rpm 4 . 3750 rpm

Scanned by Cam Scanner


https://t.me/abcdelectrical

Appeadix-C_______ ■ |Q13

86. A 3-phase delta-connected SCIM has a starting current ld and a starting torque Td at rated
voltage. If the starting current and starting torque while the motor is started through star-delta
a er and auto-transformer (with 60% voltage) starter alternatively are ly, 7 , and T^.

rrape«i«ly. tho. i : :J .: ^ ^ w
ld ld *d
( a ) ^ : 0.6 ; ^ ; 0.6 (6) | : 0.6 : | : 0.36

t c ) | : 0.36: ^ 0.6 (d) ^ : 0.36: | : 0.36

t*ie rotor-resistance starting is preferred over reduced-voltage starting because it


1 t^ reases lts starting current 2. increases starting torque
. improves starting pf 4. decreases maximum torque
From above, the correct answer is
to) 1, 2, 3 (fc) 2, 3, 4 (c) 1, 2, 3, 4 {d) 1, 2, 4
***" jk'V mean5 ° f “ auto-transformer with 50% tapping, supply current at start of a
* 15 aui and when started by means of a star-delta starter, the supply current at start
ia 7V. The ratio of is
y
(a) 1.33 (61 0.813 (c) 1.5 (tf) 0.75
89. A 3-phase, 12 pole. 50 Hz wound-rotor induction motor (WRIM i is to be used as a frequency
converter. This IM is dnven by adjustable speed dc motor. Assume the field of IM to be rotating
clockwise. For an output frequency range of 100 to 300 Hz at the slop rings, speed range of dc
motor would be : r b
1. 1500 to 2500 rpm clockwise 2. 1500 to 3500 rpm clockwise
3. 500 to 2500 rpm anticlockwise 4 . 1000 to 3000 rpm clockwise
5. 1500 to 3o00 rpm anticlockwise
From these, the correct answer is
ta> 2, 3 (6 ) 1, 4 (C) 1, 5 (d) 2, 4
90. During the no-load test, the power input to a 3-phase IM has to supply
(a) core loss, friction and windage (FW) loss
(5) small ohmic loss and FW loss
(c) small ohmic loss, core loss and FW loss
id) small ohmic loss, FW loss and stray load loss
91. During blocked-rotor test on a 3-0 EM, the power input to 3-phase IM has to supply
(a) ohmic loss (b) ohmic loss and core loss (c) core loss
(d) ohmic loss, core loss and friction and winding loss
92. The diameter of the circle diagram for a 3-0 IM is given by (Vt = stator phase voltage
Vi Vj Vi „ Vj
(a )— ( 6 ) ------- — (c) ----- — ■ id) —
*i ri + r2 * i +*2 x2

93. The following tests are conducted on a 3-phase IM :


1. Running-light test 2. Blocked-rotor test 3. Stator-resistance test
The complete circle diagram of a 3-phase IM can be drawn with the help o f :
(a) test 2 alone (6) tests 2, 3 ic) tests 1, 2, 3 id) tests 1, 2
94. A star-delta starter is equivalent to an auto-transformer starter with a tapping of
(a) 86.6% ib) 57.73% (c) 57% id) 58%
95. Wattmeter readings during the no-load test on a 3-phase IM give .
1. stator !2R loss 2 stator «»*J”*?
3. rotor core loss 4 fhctlon “ d w,ndage loss
From above, the correct statements are
<a) 1, 2, 3, 4 ib) 1, 2, 4 (c) 2 , 3, 4 id) 1, 3, 4

Scanned by Cam Scanner


Electrical M achine
1014_____________________________ ______ -— ---------------------------

96. Wattmeter readings during blocked-rotor test on a 3 P 3 rotQr ^ ]oss


1, stator f'R loss 2. stator cor -
4. friction and winding loss 5. rotor / 03
From these, the correct statements are (cZ) 1, 5
(a) 1, 2, 5 (6) 1, 2, 3, 5 (c) l ’ ' 0 IM ninning at 975 rpm is 40 kW. The state
. 97. The
* 1*^ power input to
VI litpub aI -I
vw V <115 V, 50
' I—■- Hz(' 6-pole,
- . P re ^ KW» The
1 It efficiency of the motor is
losses are 1 kW and friction and windage osse.
(a) 92.5% (6)91% (c) 90.06%
. r tkmp-nhase IM running at a slip ‘s’is given by
98. The approximate value of efficiency of a t P
, . i
fo)_ (M_ .fe) 1+s ... „w i = » l » ( f l p ^ s - w a s :i

99. Consider the following statements :


The emf induced in the rotor of a 3 -pl.aae IM ia proportmnal to the
1.relative velocity between stator field and rotor conduct
2 .voltage applied to the stator
3. slip
of these statements ^ an(j 2 ai-c correct
(a) 1 , 2 and 3 are correct ■ . . j g are correct
(c) 2 and 3 are correct (a)
100 .A 3-phase IM requires a starter , _^ qtarHmr
(а) because there is no back emf induced in the stator winding dunng starting
(б )because the motor does not possess an inherent starting torque
(c) because the induced secondary voltage and hence the currents in the windings are abnormally
high during starting
(d) to increase the starting torque.
101. To improve the power factor ofa 3 -phase IM, the capacitor bank should be connected in deltato
rnstk.6
( a ) capacitance calculation easy (6 ) capacitance value small
(c) the connection elegant M) the pf correction more effective.
102. For the improvement ofpf in a 3-phase IM, the star-connected capacitor bank has a capacitance
ofCs and VA rating of VAS. Ifthe capacitor bank is delta-connected, then its capacitance Cd and
VA rating VAd will respectively be

(a)Cd = Cs, VAd = ^ V A s (b)Cd = ^ C s, VAd --^ V A s

(c) Cd = Cs, VAd = VAS (d) Cd = 3 Cs, V A (]= V A S

103. Induction generators deliver power at


(a) unity pf (&) zero pf lagging
(c) leading pf W) lagging pf
104. A 3-phase induction machine runs at supersynchronous speed. For self-excitation, the machine
(а) draws real power from the mains
(б) draws reactive from the mains
(c) feeds reactive power to the mains
(d )generates emf at the expense of residual magnetism
105. In a self-excited induction generator, to keep the frequency of generated voltage constant wit*1
the increase in load, the speed of the induction machine should be
(а) increased
(б) decreased
(c) maintained less than the rated synchronous speed
(d) maintained more than the rated synchronous speed

"Scanned by Cam Scanner


hf f nf f • / / f m p f a r 11- r i ra 1

Appendix-C __________________________ ___________________________________ ____________ !®?5

106. A S-phaae induction motor draws active power ‘F and reactive power ‘Q' from the glid. If it is
operated as a generator, P and Q will respectively be
(6) positive and negative (&) negative and negative
(c) positive and positive (d) negative and positive
107. Consider the following statements :
If a 3-phase SCIM operates at a slipof - 0.05 (i.e. minus 0.05), then the machine will
1. draw electrical powerfrom the mains
2. draw mechanical power through the shaft
3. deliver electrical power to the mains
Of these statements
(a) 1, 2 and 3 are correct (b) 1 and 2 are correct
(c) 2 and 3 are correct (d) 1 and 3 are correct
108. A 3-phase induction machine operates on 3-phase fixed frequency ac mains at a per unit slip of
1.5. Consider the following statements regarding the operating conditions of the machine .
1. It draws electrical power from the mains.
2. It draws mechanical power through the shaft
3. It delivers electrical power to the mains
4. It delivers mechanical power through the shaft
Of these statements
(a) 1 and 2 are correct (6) 1 and 4 are correct
(c) 2 and 3 are correct (d) 3 and 4 are correct. [7.A.S., 1994]
109. A 2.3 kV, 3-phase, 50 HzSCIM hasstarting current of 600 A and startingtorque of 640Nm.
The per unit tappingof auto-transformer to reduce the starting currentfrommains to150 A
and the corresponding starting torque would respectively be
(a) 0.5, 160 Nm (6)0.25, 40 Nm
(c) 0.6, 230.4 Nm (d)0.4, 102.4 Nm
110. A 3-phase SCIM with an applied voltage of 40% gives a blocked-rotor current of 240%. Its full-load
slip is 0.05. Per unit starting torques with direct-on-starter and auto-transformer with 50%
tapping are respectively given by
(a) 0.9, 0.45 (6) 0.9, 0.225 (c) 1.8, 0.9 (d) 1.8, 0.45
111. The power factor of a delta-connected 3-phase, 50 kW, IM is 0.4 when delivering 35% of its rated
load. If its stator is reconnected in star, than its
(а) pf is improved, stator current decreases
(б) pf is improved, stator current increases .
(c) pf is worsened, stator current increases
(d) pf remains unchanged, stator current decreases
112. No-load test on a 3-phase IM was conducted at different supply voltages and a plot of input
power versus voltage was drawn. This curve was extrapolated to intersect the y-axis. This
intersection point yields
(a) core loss (b) stator copper loss
(c) stray load loss (d)friction and windage loss. [GATE, 2003]
113. A 3-phase IM is driving a constant torque load at rated voltage and frequency. If both voltage
and frequency are halved, following statements relate to the new condition if stator resistance,
leakage reactance and core loss are ignored
P The difference between synchronous speed and actual speed remains same
Q The airgap flux remains same
R The stator current remains same
S The p.u. slip remains same
Among the above, correct statements are

S a s & s (V & S [GATE, 2003,

Scanned by Cam Scanner


•.• '.• " ......

Electrical Machinery
im
. .i . . , ■. „ . . . . . . . \ ...i,,,., star! ad hv direct switching. If an nuto-trans-
114. A d-phmte. SCIM taka# a linn current ol 100 A * / '‘ ” (.urr<mt and tho supply-line current would
limner with 50% tapping la used, the motor’ lino <■
respectively he {(i) 25 A, SO A
(<il SO A, 25 . ^ ^ 5Q j\
i?) no a, no H a w ’ 4 . . „
, . « ,aa v r.o My source. Stator to rotor oliective turns
115. A 3-ph jw , star-connected SIMM i* fed from 400 V * Qmf or phaso WQUid i,0 *
ratio ia 2, At a rotor speed o f 1>M0 rpm, tho retoi indi l
(in 3V
U (M JO.2 V (0) 8.0 V (<*> l)'2'1 V
118. Tnmun devotoiwl iK.., I j, co* Oj ore rotor circuit pnrmnotora nnd n>, = sync oonous spec ) m „
3-phaso IM is given hy
3 ,2 1- *
1 . n\
4 K'J•t*h•* Oa
4 ^ M •
_3
a, 4 - 4 3 4. s
to.
V ra
rj + — + (.Vj + .y2)

V? £a
<iv a
*’a
r, + -7 t tv |+ v*)

Prom those, the correct answer is


« t ) l , 2, 5 (b) 1. 3, 5 (c) 1, 2, 4 (d) 3, 5
117. With stator impedance neglected, starling torque of a 3-phaso IM is proportional to
ra -Vo
1. 3. 4. V”,
ra+o-a r’
J+t.ra
ra v2 /•a + ,1'o
2
5, V3 6. J . 7. a).
1

Prom t hese, the correct answer is


(0 ) 1, 4, 7 (/0 I, 5, (i (c) 3, 5, 7 (d) 2, 5, 6
118. An increase in the number of poles of a 3-phaso IM results in
(0 ) decrease in maximum pf
tb) increase in maximum pf
(e) no-chnnge in mnximum p f
(d) cannot ho depicted
H 9, Wlien the supply voltage to a 3-phase SCIM is reduced by 20%, the maximum torque will decrease

(n) 10% (b) 20% (c) 36% (d) 40%


120. If a 3-phase 350 V, 50 Hz SCIM is operated on 420 V, 60 Hz supply then
1. storting torque Tcst will increase ,J maximum
2. J------- - torque Tem will increase
3. Tttt will decrease
4. Te , m will remain constant
5. operating speod will bo more
6. operating slip will be less
Prom these, tho correct statements are
( o ) l , 2, 5, 6 (6) 3, 4, 5, 6 (e) 1 ,4 ,6 (d) 2, 3, 5, 6
121. Starting torque o f a 3-phase IM varies ns
(«)/' (b) j (c> ! W jS
r
122. Maximum torque in a 3-phase IM varies as
(a> f (b) -i , v 1
{ ) r ^ 7 w js

Scanned by Cam Scanner


1017

123. In » 3-phase SCIM, applied voltage is reduced to x times its rated value. Then, its starting line
current from mains and starting torque by auto-transformer starting would reduce respectively
from their corresponding direct switching values by a factor of
(a) x, x (b) x, x2
[C)x2,x 2 od )x2,x
124. Out of the several characteristics shown in Fig.
C.74, identify the appropriate ones to match
the following for a 3-phase induction motor :

x-axis y-axis

A. Load efficiency (■P) curve I


B. Speed current (Q) curve II
C. Speed power factor (R) curve III
(S) curve IV
(T) curve V

C o d e s:
A B C A B C
(a) R R P 0b) P R Q
(c) P Q T (d) Q P s
[GATE, 1996\
125. A 3-phase IM with stator in star draws a line current of 10 A, When connected in delta, it would
draw a line current of
(a) 10 A (c) 30 A o o f A

126. The most common application of 3-phase induction generator can be in a


(а) steam-power station
(б) hydro-power station
(c) wind-power station
(id) nuclear-power station
127. Two 3-phase SCIMs are identical in every respect except that the slot depths in machine A are
more than those in machine B. However, slot areas in both the machines are the same. Machine
B, as compared to machine A will have
(а) more pull-out torque but poor pf
(б) more pull-out torque and better pf
(c) less pull-out torque and poor pf
(rf) less pull-out torque but better pf
128. A 3-phase, 20 kW, 400 V, 1470 rpm, 50 Hz SCIM develops a torque of 100 Nm at a speed of 1400
rpm. If the motor is connected to a 30-Hz supply ; for keeping the same air-gap flux, the supply
voltage and the speed for the same load torque would respectively be
(a) 240 V, 840 rpm (6) 240 V, 800 rpm
(c) 240 V, 780 rpm (rf) 240 V, 750 rpm
129. A 3-phase SCIM has full-load efficiency of 0.8 and a maximum efficiency of 0.9. It is operated at
a slip of 0.6 by applying a reduced voltage. The efficiency of the motor at this operating point is
(o) less than 0.4 (6) greater than 0.6
(c) in the range o f 0,8 ± 0 .1 (rf) none o f the above [GATE, 19981

r*=.--- j --------------
1018 Electrical Machi
: O f
ANSW ERS
1- ic)
6, (c)
.
2 Ul) 3. (a) 4. ib)
9. (a)
5. (a)
10. id)
7. (c) 8. id)
14. (d) 15. (c)
11, in) 12. (5) 13, (c)
19, ia) 20. ic)
1 0 , ( /;) 17. (cj 18. </:)
21, (b) 24. ia) 25. lb)
22. (/;) 23. (fij
20. id) 29. ib) 30. id)
27, (c) 28, (c)
3 4 . (a) 35. (a)
3 1 . (/j) 32. (c) 33. (n)
30. (n 39. ib) 40. (flj
37, (c) 38. (c)
41. (c) 44. (6) 45. id)
42. ib) 43. (cj
40, (c) 49. (c) 50. (£/)
4 7 . (a) 48. ib)
54.(6) 55. (c)
51. (cj 52, ib) 53. ia)
50. (a) 58. id) 59. ia) 60. ib)
57. (c)
01. (I)) 63. id) 64. (6) 65. (c)
02. («)
00 . ( 6) 68. ib) 69, id) 70. (a)
67. f«)
72. («) 73. ib) 74. (c) 75. (c)
7 1 .( 0
70. (a) 77. 0/) 78. (a) 79. id) 80. (a)
HI. (c) 82. (c) 83. (6) 84. (a) 85. ic)
HO. (/>) 87. («) 88. (J) 89. (a) 90. (c)
01. (a) 02. (c) 93. (c) 94, ib) 95. ib)
0 0 . ( f /J 07. (cj 98. (c) 99.(6) 100. (c)
1 0 1 . ( /> ) 102. (cj 103. (cj 104. ib) 105. ia)
1 0 0 . { r /j 107. fc) 108. ib) 109. (a) 110. id)
111.(a) 112. «/) 113. ib) 114. ia) 115. ia)
110. (b) 117. (d) 118. (a) 119. ic) 120. (6)
121. id) 1 22. (c) 123. ic) 124. ib) 125. (a)
120. ic) 127. in) 128. ib) 129. (a)

Scanned by Cam Scanner


ARMATURE WINDINGS
j. Closed windings are used for
1. ac machines 2. dc machines
3. ac commutator machines 4. dc commutator machines
From these, the correct answer is
(a) 1, 3 (6) 2, 4 (c) 1, 4 id) 3, 4
2. A multi-turn coil has
(a) multi-coil-sides ib) one-coil-side
(c) two coil-sides id) four coil-sides
3. Open windings are used for
1. dc machines 2. ac machines
3. ac commutator machines 4. dc commutator machines
From these, the correct answer is
(a) 1 alone (6) 2 alone (c) 2, 4 (d) 3 alone
4. Closed windings may be
1. single-layer winding 2. double-layer winding 3. triple-layer winding
From these, the correct answer is
(a) 2 alone (6) 3 alone (c) 1 alone (d) 2, 3
5. Open windings may be
1. single-layer winding 2. double-layer winding 3. triple-layer winding
(a) 1 alone (6) 2alone ic) 1, 2 id) 2, 3
6. For C armature coils and P poles in a dc machine, the back pitch for simplex-lap winding is
ia )^ -+ K ib )^ f-K {c )™ ± K id )2 C P ± K

7. For a simplex-lap winding, the winding pitch is equal to


(a) + 2 (6) - 2 (c) ± 1 id) ± 2
8. For a simplex-lap winding, the commutator pitch is equal to
(a) + 1 (6) ± 1 (c) - 1 id) ± 2
9. A dc machine has 6 coil-sides per slot. The number given to the first coil-side in 8th slot is
ia) 43 (6) 37 (c) 48 (d) 47
10. If y b is the back pitch and u is the number of coil-sides per slot, then split coils can be avoided
if the following quantity is an integer
Vb + 1
(a) ib) iyb + 1) u ic) id) (yb - 1) u
u u
11. For C coils and P poles, the commutator pitch for simplex-wave winding is
. , C± 1 C+l M c - i ... C ± 2
(«) -r t t t ib) P /2 (c) i P
t /2
tt id) P /2
P /2
12. For C coils and P poles, the winding pitch for simplex-wave winding is
, , 2C + 2 2C±2 /.n 2 C - 2 , 2 C± 1
(a) -77777“ ib) vO id)
P /2 v_/ P /2 P /2 v ' P /2
13. For simplex wave and lap windings, the back pitch y b and front pitch yf are as follows
(a) y b is odd, y f is even ib) y b is even, yf is odd
(c) both y b andy^ are odd id) bothy6 andy^ are even

14. A dc machine with 6 poles and 4 parallel paths has


(a) simplex lap winding ib) simplex wave winding
(c) duplex lap winding' (d) duplex wave winding
15. The number of commutator segments in a dc machine is equal to the number of
(a) coil-sides (6) turns (c) coils id) slots

Scanned by Cam Scanner


......

Electrical Machinery
1020

16. In lap windings, the function of equalizer rings are


1. to save the armature winding from circulating curren
2. to save the brushes from carrying circulating currents ^
3 . to save the commutator from handling circulation curr
4 . to reduce the magnetic unbalance under different po
5 . to save the arm ature core from eddy currents
From these, the correct statements are ^ ^ 5
2>3- 4 (6) 2’ 4 (C) 1 ,3 ,0 . Xntr following should not be an integer:
17. If dummy coils are to be avoided in simplex-wave winding, ^

(a) S /P (6) P ±X

(C) —
** . ,
(d)f ±2
, . „ ti.pn following should be an integer ;
18. I f equalizer rings are to be provided in dc machine-,

w *£ ± ! ® T T . . , nnR

19. A 6-pole lap-connected dc generator has 480 things remaining unchanged,


ohm. If the conductors are reconnected to form wave wmding, otner 5
tbe value of the armature drcuit r e .s t a n c e ^ be ^ ^

20. A 4-pole lap-wound dc generator has a d e v e l o p e d S p m v e r ^ o f P m a c h i n e operates

- - " - -
machine are n . . r p /9
(c) £ , P (6) P /2 , P /2 (c) P, P /4 (d) E,
21. The dummy coil in a dc machine is used to
(a) eliminated reactance voltage
(b) reduce armature reaction
(cj provide mechanical balance to the armature
{1d) reduce harmonics generated in armature
22. In ac machine, double-layer winding is preferred over single-layer windings because
1. it needs less copper
2. it gives high induced emf for the same amount of copper
3. it requires one size of the coils
4. it is economical
5. it possesses lower leakage reactance
From these, the correct answer is
(a) 1, 3, 5 (6) 2, 4, 5 (c) 3, 4, 5 (d) 1, 4, 5
23. For C coils and P poles, the winding pitch for the two types of dc armature windings are :
Simplex-lap winding Simplex-wave winding
(а) +2 2 C± 2
P/2
(б) -2 2C+2
P/2
(c) ±2 4C± 4
P
id) ±2 2 C -2
P/2

bcannea oy u a m bca n ne r
https://t.me/abcdelectrical

^ppendi*£__ ------------------------------------------
24. For C coils and P poles, the commutator pitch for the two types of dc armature windings are :
Simplex-lap winding Simplex-wave winding
ia) ±1 2C±2
P
(b) +1 C± 1
P
ic) -1 2 C +2
P
id) ±1 C ±1

25. For C coils and P poles, the distance between the coils connected by an equalizer ring is
ia )C /P (6) C /2 (c) 2 C /P (d) C /2 P
26. In a P-pole dc machine with armature current Ia, the current per brush arm for a lap-connected
winding is

<0) P * > ir
27. An 8-pole dc machine has simplex-lap connected 1000 armature conductors, 4 turns per coil and
2 coil-sides per slot. The number of slots in this machine are
ia) 125, equalizers not possible (6) 125, equalizers possible
(c) 250, equalizers possible (£f) 250, equalizers not possible
28. Armature of an 8-pole dc machine has 15 slots with 4 coil-sides per slot. This machine
1. will have dummy coil/s if wave winding is used
2. will have equalizer rings if lap winding is used
3. has y b = 7 andjy= 5 for lap winding with no split coils
4. has y b = 9 and 7 for lap winding with no split coils
From these, the correct answer is
(a) 1, 2, 4 {b) 1, 2, 3 (c) 2, 3 (d) 1, 4

29' v o lt ^ O n e ^ n T o ^ l * g*nerator has ^developed power of P watts and brush voltage of £


volts. One brush of the machine is removed as it is worn out. If the machine operates with the
t e ) T p ng ! ^ 9eVp/°9Ped V0ltage and power that can be obtained from the machine are
ia) E, r ib) E /2 , P /2 ic) E, P /4 (d) E, P /2
3°. A 6-pole lap wound dc generator has a developed power of P watts and brush voltage of E volts
S h * !’ adjacent brushes of the machine are removed as they are worn out. If the machine operates
machine are” 81" 1" 8 6S’ developed voltage and power that can be obtained from the

ia )E ,P i b ) E ,^ f (C) | id) E, P /3

31. A 6-pole lap-wound dc generator has a developed power ofP watts and brush voltage of £ volts
rwo adjacent brushes of the machine are removed as they are worn out. If the machine operates
with the remaining brushes, the developed voltage and power that can be obtained from the
machine are

ia )E ,P (6) E, | ic) E, P /3 (d) E, 2P/3

32* ^ 6'p.°le laP wound dc generator has a developed power of P watts and brush voltage of E volts,
'■me brush of the machine is taken out as it is worn out. If the machine operates with the
remaining brushes, the developed voltage and power that can be obtained from the machine are

WE,P ( c) f ’ ^ id) E, P/ 3

Scanned by Cam Scanner


f \\’s ... -w-v''♦>.» >■-. -•*1'■'■-*i: i~m
S^VtfaVt!!' ~ "m '

Electrical Machinery

4. Phase spread of 60° produces a phase-belt se<P')t’" )c


5. Output with cr = GO"' is 15% less than with 0 = 1- bottom layers belonging to the same
G. In integrahslot chorded winding, each slot has t I
phase.
Of these statements j (J are correct
(n) 2, 3, 4 are correct . nro correct
(c) 1, 2, 3, 4 are correct _ * ’
34. A 3-phase machine has 39 slots and 6 poles. This jf^0t cliorded winding
(a) integral slot winding nQnc of thesc
(c) fractional-slot winding v ' . .
35. For a 3-phase fractional-slot winding with P poles, thein!umber o para e pn - -
( o) P (b) 2 P (OP/2 (rf) none of these
36. For a 3-phnsc machine with P poles, consider the following statement
1. The number of parallel paths for integral-slot winding is P / 2 ^
2. The number of parallel paths for integral-slot chorded winding is P/2,
3. The number of parallel paths for 54 slots and 10 poles is 2
4. The number of parallel path for 54 slots and 8 poles is 4.
From these, the correct statements are
(a) 1, 2, 4 (6) 1. 3 (0 1, 2, 3 (d) 1. 4
37. Fractional-slot winding is used in . . .
(a) turbogenerators W hydrogenerators
(e) fractional-kW motors «*) sqmrrel-cage induction motoi-s
38. A 3-phase, 10-pole low-speed water-turbine alternator has 144 slots. The spacing between the
starting of phase A and that of phase B, in terms of slot-pitches, are
(a) 15 (6) 24 (c) 48 (d) 14
39. A 3-phase, 16-pole alternator has 84 slots. Polarity of the starting of phase-belt of phase B with
respect to phase A and its displacement from phase-A starting are
(a) same polarity, 7 pitches (b) opposite polarity, 14 slot-pitches
(c) opposite polarity, 7 slot pitches (d) same polarity, 14 slot-pitches
40. For a 3-phase machine having double-layer winding, mark the correct statement
Full-pitched winding Chorded winding
(а) Each slot has coil-sides belonging to the sam e Each slot has coil-sides belon gin g to the same
phase phase
(б) Some slots have coil-sides belonging to different Each slot has coil-sides belonging to different
phases phases
(c) Each slot has coil-sides belonging to different Som e slots have coil-sides o f different phases
phases and som e from the sam e phase
(d) Each slot has coil-sides belonging to the sam e Som e slots have coil-sides o f the sam e phase
phase and som e from different phases

41. In single-layer and double-layer windings, the number of coils C is related to the number of slots
S by the following relations respectively
(a) C = S, 2 C s=S ( b ) 2 C = S, C = S
(c) 2 G = 3 S , C = S (d) 2 C = S, 3 C = S
42. For the use of mush windings in 3-phase induction motors,
1. the slots should be open
2. coil-pitch, in terms of slots, must be odd

Scanned by Cam Scanner


h t t n s • . l i t , , mfi / a b n d e l f i n t ' - i ' i n a l

1023

3. the slots should be semi-closed


4. coil-pitch, in terms of slots, may be odd
5. the maximum recommended voltage is 440 V
6. the coil-pitch, in terms of slots, may be even
7. the maximum recommended voltage is 600 V
From these, the correct answer is
(a) 2, 3, 5 ib) 3, 4, 5 fc) 1, 4, 6, 7 id) 3, 6, 7
43. Whole-coil and mush windings are similar because their
(a) coils are interconnected in the same manner (b) coil-pitches are the same
(c) phase spreads are equal (d) slot-pitches are equal
44. In most ac machines, it is usually a standard practice to use
(а) fractional-slot winding with ftill-pitch coils
(б ) integral-slot winding with chorded coils
(c) fractional-slot winding with chorded coils
id) integral-slot winding with full-pitch coils
45. Match List-I (type of winding) with List II (feature) and select the correct answer using the codes
given below the lists :
List-I List’ll
(a) Fractional-slot windings 1. Only two-third of the pole-pitch is occupied by the winding
(b) Chorded and distributed windings 2. Effect of slot harmonics are reduced or even eliminated
ic) Damper windings 3. Effect of phase-belt harmonics are reduced
id) Single-phase windings 4. Additional asynchronous torque capability in projected
pole motors
Codes
A B C D A B C D
(a) 2 3 4 1 2
ib) 3 1 4
ic) 3 2 1 4 id) 3 2
1 4
46. Match List I with List II and select the correct answer by using the codes given below the lists
List I (Windings) List II (Associated terms)
A. Closed windings 1. Dummy coil
B. Open windings 2. Fractional-slot winding
3. Mush winding
4. Split coils
5. Phase spread 180°
6. Parallel paths
Codes :

A B A B
<“) 1, 4, 5, 6 2, 3 ,6 ib) 1, 4,6 2,3, 5
ic) 1, 4 , 6 2, 3, 6 id) 1, 4,5 2, 3, 6

m r X-------------------- J m w w u w u w u i X V I M il Y V lb U

S0° phase spread. Coil span is 7 slot pitches. Number of slots in which top and bottom layers
belong to different phases is
*a> 24 (6) 18 (c) 12 id) 0 {GATE, 2003)

Scanned by Cam Scanner


'■mi ■IIIWHW ill ..

1024 Electrical Machinery

48. A 3-phase ac machine, with 54 stator slots and 10 poles, has t e 9 cmU, , ..n a m in g U. phase A
distributed under 5 adjacent poles as under (take double-layer windmg and phase spread
= 60°):

3rd Pole 4th Pole 5th Pole


Ist Pole 2nd Pole
2 2
(c) 1 2 2
2 2 2
(b) 2 1
1 2 2
(c) 2 2
2 1 2
id) 2 2

49. A 3-phase ac machine, with winding, has


d o u b l e - l a y e r ^ ”
slots are distributed under 5 adjacent poles as follows (take phase spre ■

3rd Pole 4th Pole 5th Pole


1st Pole 2nd Pole
5 5
(a) 6 6 5
5 5
ib) 6 5 6
6 6
ic) 5 5 5
5 6 6
id) 5 6

50. A 4-pole, 3-phase, double player winding is housed in a 36-slot stator for an ac machine with
60° phase spread. Coil span is 8 slot pitches. Number of slots m which top and bottom coil-sides
belong to different phases is
(a) 24 (b) 18 (c) 12 id) 0

ANSWERS
2. ic) 3. ib) 4. ia) 5. ic)
1. id)
7. id) 8. (6) 9. (a) 10. ic)
6. (c)
12. ib) 13. ic) 14. (d) 15. (c)
11. (a)
16. (b) 17. (c) 18. (d) 19. id) 20. ic)
21. ic) 22. (c) 23. (c) 24. (a) 25. (c)
26. (c) 27. (a) 28. id) 29. id) 30. id)
31. {b) 32. ib) 33. (a) 34. (d) 35. id)
36. (c) 37. (6) 38. (b) 39. id) 40. id)
41. ib) 42. ia) 43. (a) 44. (c) 45. (a)
46. ia) 47. (a) 48. (d) 49. ib) 50. (c)

Scanned by Cam Scanner


https://t.me/abcdelectrical

1025
Appendix-C

t h r e e -p h a s e c ir c u it s a n d m a g n e t ic c ir c u it s
!• A mild steel ringwith mean diameter of20 cm has core area of10 cm2.Fora relative permeability
of 500, the reluctance of the ring in AT/Wb is
(a) 1 x 106 (6 )0.1 x 106 (c) 0.01 x 105 (d) 10 x 106
2 . Magnetic flux, flux density and magnetic field intensity have the following units respectively .
(a) Wb, Wb/m2,Wb/m (&) Wb, Wb/m, Amp. turns
(c) Lines, Wb/m2,Amp.tums/m (d) Wb, Wb/m2,Amp.turns/m
3* A • • 2 j
•A magnetic circuit with relative permeability of 100 has a core cross-section of 5 cm and mean
core length of 25 cm. The coil has 120 turns withan mmf of 1000AT. The magnetic core flux is
(a) 0.5 m W b (6 )0.25 m W b (c) 0.75 m W b (d) 1 m W b
4. The magnetic flux in a homogeneous toroidal core excited by a coil with a given number ofturns
carrying a fixed current is
1.proportional to cross-sectional area ofthe toroid
2 .proportional to the diameter ofthe toroid
3. inversely proportional to cross-sectional area ofthe toroid
4. inversely proportional to diameter ofthe toroid
From these, the correct answer is
(°) L 3 (b) 2, 3 (c) 1, 4 (d) 2, 4
5. A toroidal ferromagnetic ring is uniformly woundwith copper wire ofcertain turns. A dc voltage
Vi is applied across the winding to cause a uniform flux density in the ring. If the linear
dimensions ofthe ring, number ofturns and length of mean turn are doubled without changing
® gauge of the wire, the dc-voltage required to be applied across the new winding to produce
the same flux density will be '
(a) 2 w(6) viVi (c) 2 Vx (d) 4 Vj
6 . In the magnetic circuitshown in Fig. C.75, the areasof cross-section of limbs B and C are,
respectively, 0.01m2 and 0.02 m 2.Air gaps of
lengths 1.0 m m and 2.00 mm, respectively, are
cut in limbs B and C.
Ifthe magnetic medium can be assumed to have
infinite permeability and the flux in limb B is
1.0 Wb, the flux in the limb A is 2 mm
(at) 3 Wb (b) 1.5 Wb
(c) 2 W b (d) 4 Wb [GATE, 1990]
7. The current in a coil wound over a ferromagnetic
core is gradually increased to 2 A and then
reduced to 1 A and then the current in its is Fig. c.75.

again increased to 2 A. The variation of magnetization ofthe core will be as shown in


(a) Fig. (a) (b) Fig. (b) (c) Fig. (c) (d) Fig. (d)

Scanned by Cam Scanner


1026 Electrical M

8. "In all caaea of akctromagnaUc induction. an indn^ed v o l t a g e » « . ojrrent to flo * in a


closed circuit in such a direction that the magnetic field whrch n»_careed b, that current w,l,

fo)P|
“ n2U")awa"eC"th“‘ P
stHe cum at'
x
w
d
o
r - ^Faraday
ic) Flemings la w of induction (tfi Ampere’s law u rn
8. The laws of electromagnetic induction (Faraday’s and lama's law) are summamad .n the follow,
ing equation ;
(a) e = iR <*)c = i f (r )r » -f (d, none of above. (GATE, ,993]
■fft A
10. A coil
-1 rotates at a constant speed m a uniform mnpnetic
niagnec field that
nfis oriented
fbp mil along the x-axis
Induced emf in the coii will be maximum when the axis o ro
(a) coincide s with the x-axis , * c
(b) lies in the x -y plane making an angle of 45' with hot x- an y
(c) lies in the z-x plane making an angle of 45' with both z- an x 3
{d) lies in tbey-z plane
11. Consider the following statements relating to a circular disc rotating
in a traverse magnetic field B Wb/m2 as shown in Fig. C.76.
The emf generated across outer rim A and centre O i3 proportional to J rzijs
1. angular velocity 2. flux density
3. radios of the disc 4. square of the radius of the disc
Of these statements
(a) 1, 2, 3 are correct 0 ) 1. 2. 4 are correct
(c) 2, 4 are correct (^) 1, 2 are correct
12. Iron-core is inserted in the air-cored coil so that now the coil becomes iron-cored coil. Inductance
of this iron-cored coil will
(a) increase
(b) decrease
(c) remain the same
(d) increase or decrease depending upon the coil configuration
13. A coil with GO turns w'ound over a ferromagnetic core having relative permeability 400 has an
inductance of 50 raH, If coil turns are doubled and the core is replaced by a new ferromagnetic
core having relative permeability 600, the new inductance would be
(a) 0.03 H (b) 0.3 H (c) 3 H (d) 0.15 H
14. The phase sequence of a 3-phase alternator will reverse if
(а) the field current is reversed keeping the direction of rotation same
(б) the field current remains the same but the direction of rotation is reversed
i (c) the field current is reversed and the number of poles is doubled
(d) the number of poles is doubled without reversing the field current
15. Three resistances, each of R 12, are connected in delta. Their equivalent star value for each
resistance is
(a) R [b] 3 R (c) K/3 (d) 2 R
16. Three capacitances, each of C farads, are connected in delta. Their equivalent star value for each
capacitance is
(a) C (b) 3 C (c) C/3 (d) 2 C
17. Jn a certain magnetic circuit, a current of 1 A produces a flux of 1 Wb. If linear dimensions of
the magnetic circuit are doubled,, then for producing the same flux, the current would be
(o)~A 0!> 1A (c) 2 A

18. An iron-cored choke, with 1 mm air-gap length, takes 1 A when fed from a constant-voltage
source o f230 V. If its air-gap length is increased to 10 mm, then the magnetic flux produced by
the choke would
(o)^decrease and the current would also decrease

Scanned by Cam Scanner


Appendlx-C_____________— _ _ _ _ _ 1027

(b) remain constant and the current would decrease


(c) decrease and the curront would increase
( ) remain constunt und the current would increase
10. For a solenoid shown in Fig. C.77,
Si
an* a ,
i JeCOm
e8 northP°ie if Chas positive polarity
ecomes nort^P°le ifD has positive polarity Voltage source
irl\ an a a . ecomes 80uth pole if C has positive polarity
Fig. C.77.
ecomes north pole if D has positive polarity

etch List I with List II and give the correct answer by using the codes given below the lists
List I
List II
(Motion of permanent magnet)
Motion qa
(Direction of emf in stationary coil)

A- fs"~ ”
JO 1. No emf in the coil

Motion

GE JJ 2. Cross in coil-side A and dot in coil-side B

3. Dot in coil-side A and cross in coil-side B


c. 31
M otion
D. i n (i" "Z D
E B
Codes:
A B C D
A B C
(o) 1 2 D
3 3 (b) 2 3
(C) 2 3 3 1
1 3 (d) 2 3 1 1
21. Match List I with List II and select the correct
answer by using the codes given below the lis
List I
(Magnetic circuit) List II
(Electric circuit)
A Magnetic flux density
1. current
B. Magnetic field intensity
2. Conductivity
C, Permeability
3. Electric field intensity
D- Magnetic flux 4. Current density
C o d e s:

A B C D A B C I)
(a) 4 3 2 1 ib) 1 2 3 4
tc) 4 2 1 3 id)
3 1 9 4
88. The iiutantansous value of currents in both pha.ee k and c of a 3-pha.e balanced system
7 *,n a pha0e 8equence abc’ the instantaneous value of current in phase ‘a’ is
W 1 7 .3 2 A (4)10 A (c) 20 A (rf) 34.64 A
38. The power in a 3-phaae ejretem is given by i/3V, /, cos 8. Here 8 is the pf angle between
(a) line voltage and line current (6) line voltage and phase current
(e) phase voltage and line current (d) phase voltage and phase current
Three equal resistances connected in star take a line current of 5 A when fed from 400 V 50
«M»ce. If the load resistances are reconnected in delta, the line current would now be
(®) 5 A (6) 5 VjTA (c) A (d) 15

ea Dy u a m ^c a n n e r
1028

25. A star-connected balanced load takes P watts from


o
Electrical Machinery

a balanced 3-phase supply. If the star-load is


reconnected in delta across the same 3 -phase
supply, the power consumed will be
(a) P (b)3 P
(c) V3 P (d)P /3 c
26. The inputvoltage to the 3-phase,50 Hz ac circuit orB
shown in Fig. C.78 is 100 V. For a phase sequence
ofabc (or RYB), the wattmeters would read B
(а) W j= 1732 W and W2 = 0 W or Y *
(б) Wj = 866 W and W2 = 866 W
(c) W1 = 1154.72 W and W2 = 577.36 W
(d) W1 = 577.36 W and W2 = 1154.74 W
27. Input to the 3-phase, 50 Hz ac circuit shown in Fig.
C.79 is 100 V. For a phase sequence of abc [or RYB],
the wattmeters would read
(a) lYj = 1732 W and \V2 = 0 W
(b) Wj - 866 W and W2 = 866 W
(c) Wx = 1154.72 W and W2 = 577.36 W
(d) Wx = 577.36 W and W2 = 1154.72 W
28. In a two-wattmeter method ofmeasuring power, one of Fig. C.79.
the wattmeters reading iszero watts. The power factor
of the circuit is
(a) zei'o (b) 1 (c) 0.5 (d) 0.866
29. The power delivered to a 3-phase loadican be measured by the use of two wattmeters only when
the ,( \ f
(а) load is unbalanced “
(б) load is balanced '( ~
(c) three-phase load is connected to the source through three wires
(d) three-phase load is connected to the source through four wires
30. The minimum number of wattmeters required to measure the real power in an n-phase system
with unbalanced load is
(a) n (b) n - 1 (c) n + 1 (d) n - 2
31. Three-phase power can be measured by two-wattmeter method in case of
1. balanced load and balanced source
2. balanced source with 3-phase, 3-wire unbalanced load
3. unbalanced source with 3-phase, 3-wire balanced load
4. balanced source with 3-phase, 4-wire unbalanced load
5. unbalanced load and unbalanced source
From these, the correct answer is
(a) A only (b) 2, 3 and 5
(c) I, 2, 3, 4 and 5 (d) 1, 2, 3 and 5
32. Measurement of power and power factor of a 3-phase system by two-wattmeter method can be
obtained in case of
1. balanced load and balanced source
2. balanced source with 3-phase, 3-wire unbalanced load
3. unbalanced source with 3-phase, 3-wire balanced load
4. balanced source with 3-phase, 4-wire unbalanced load
5. unbalanced load and unbalanced source .

Scanned by Cam Scanner


https://t.me/abcdelectrical

Appendlx-C
1029

From these, tho correct answer ia

£(*) °2 3 a
2, 3, , 5
4 and (6)
(d) 2'
1, 32,and 5 5
3 and

38 fesiator1«ft°f voltage acro8g the 100 ohm


3 nhn • r. switch S is closed in the

T ^ „ ? r „ rt r in * 080 * ,ure
£ ! SS* v v (6) 115475 v
3d 11 {d) 284 23 v [GATE<199® 400 V /3 -p h a s e
b a la n ce d source
’ nected ina? ? Cit0rS’ 08Ch ° f 10yP‘ are con‘ Fig. C.80.
sunnlv 1 aCr08S B 3'phase’ 400 V, 50 Hz
by two xvatt power consumed is measured
oy two-wattmeter method. Under the condition
ne of the wattmeters reads zero
both the wattmeters read zero
3. neither of the wattmeter reads zero
4. the total power consumed is zero

w ° ” f " * 1the X C f atCmentS(ca)r 3 4 d 2

a pure inductive two-wattmeter method in a balanced 3-phase system with

Ii S ? “ S :!!!£23: 3T" valuabutof°ppoaitesign


(d) one wattmete™winrinTilatedzCate ^ Valu6 and ° f the same sign
36. Two wattmeters used to iup ” n° n'Zer° value t/.AS., 1993)
reactive power drawn by the lo a T ° f 8 3'P W balanced load read w i and W2. The
(a) W, + IV y cne load 18
'
(c) V3 (1VX+ W2) (6) W1 - W2
(d) V3 (Wx - W2)

ANSWERS
1. (a) 2. (d)
.
6 (c) 7. (6)
3. (6)
8. (a)
4. (c) 5. (d)
11. (6) 12. (a) (c) 10. (d)
13.(6) 14. (6)
16. (6) 17. (a) 15. (c)
18. (d) 19. (d)
21. (a) 22. (c) 20. (d)
23. (d) 24. (d)
26. (c) 27. (d) 25. (6)
28. (c) 29. (c)
31. (d) 32. (a) 33.(6) 30. (6)
36. (d) 34. (c) 35. (a)

Scanned by Cam Scanner


A p p e«d *£___ —

prom these, the correct statements are


(a) 1, 4 (b) 1, 3 (C) 2, 3 (d) 2, 4
71. A 4-pole, 50 Hz, 3-phase synchronous motor and 8-pole, 50 Hz, 3-phase SRIM are coupled to each
other mechanically and operate on the same 3-phase, 50 Hz supply system. The slip rings of the
induction machine are left open-circuited. The frequency of the voltage across any two slip rings
would be
1. 50 Hz 2. 100 Hz 3. 150 Hz 4. 25 Hz
From these, the correct answer is
(«) 2>3 2, 4 (C) l, 3 (d) 1, 4
72. The stator of a 3-phase, 6-pole SRIM is connected to 50 Hz source but its rotor is energised from
20 Hz source. The rotor would run at a speed of
1. 1600rpm 2. 600 rpm 3. 1400 rpm 4. 400 rpm
Prom these, the correct answer is
(a) 2, 3 (6) 2, 4 (c) 1, 3 (d) 1, 4
73. If two IMs A and B are identical except that the air-gap of motor ‘A ’ is 50% greater than that of
motor 'B\ then
(а) the no-load pf of A will be better than that of B
(б) the no-load pf of A will be poorer than that of B
(c) the core losses of A will be more than those of B
(d) the operating flux of A will be smaller than that of B. [/.A.S., 1997]
74. Consider the following statements regarding the equivalent circuit parameters of an IM :
1. Leakage reactance is dependent on supply current
2. Magnetizing reactance is dependent on the air-gap flux
3. Core loss is dependent on the input voltage and frequency
4. Rotor resistance is dependent on the speed
Of these statements
(a) 2 and 3 are correct (6) 1, 3 and 4 are correct
(c) 1, 2, 3 and 4 are correct (d) 1, 2 and 4 are correct [7.A.S., J993]
75. The rotor of a 4-pole 3-phase SCIM is replaced by a 3-phase, 4-pole wound rotor. When fed with
normal supply, the machine will
(а) not run
(б) run at very low speed
(c) run at slightly lower than normal speed
(id) run at slightly higher than normal speed
76. A 3-phase stator winding of a SCIM is fed by 3-phase ac supply at a frequency f. If the rotor
speed is Nr and synchronous speed is Ns, then the absolute speed of resultant flux in space is
(a) Ns, irrespective of rotor speed Nr
(b) (Ns + Nr) if the flux rotates in the same direction as the rotor
(c) (Ns - Nr) if the flux rotates in the same direction as the rotor
(d) Nn irrespective of Ns.
77. Rotor of a 3-phase SRIM is fed from 3-phase balanced supply with its stator winding short-cir­
cuited. If Ns = synchronous speed, Nr = rotor speed, f = supply frequency and s is the slip, then
1. frequency of stator currents is s f
2. frequency of stator currents is (2 - s ) f
3. speed of stator field with respect to stator is Ns
4. speed of rotor field with respect to rotor is Ns
5. speed of rotor field with respect to rotor is + Nr)
6. speed of air-gap field with respect to stator is {Ns - Nr)
Prom these, the correct answer is
( « ) 2 . 5 .6 (6) 1 , 4 , 6 (0 1, 3, 5 (<(>1, 5, 6

S c a n n e d by Cam Scanner
r— .. v — — ' mmm

1012 Electrical MacElaaJ!

78. Rotor of a 3-phase SRIM is fed from 3-phase balanced supply with its stator *'"ldinp ‘sh° 11 Ci‘‘
cuited. Synchronous speed isNs and ifrotor rotates clockwise at a speec r,^
1. speed of air-gap field with respect to (w.r.t.) stator is (Ns - Nr) a n t ic lo c k
2 .speed of air-gap field w.r.t. stator is (Ns - N r) clockwise
3. speed of air-gap field w.r.t. rotor isNs anticlockwise
4. speed of rotor-produced field w.r.t. rotor is (Ns + Nr) anticlockwise
5. speed of air-gap field w.r.t. rotor isNs clockwise
From these, the correct statements are r
(a) 1, 3, 4 (6) 1, 2, 4 (c) 2, 3, 5 id) 1, 4 ,a
79. For SCIM, reactor starting is preferred over resistor starting because leac ors
1.improve the pf at starting 2 . increase starting toique voltaee
o. -incur
3 i lower energy1loss 4. are more
A
more e f.
emfi
e-cit.iivve
t in reducing the voltage
From these, the correct answer is
(a) 1, 3, 4 (6 ) 2, 3, 4 (c) 1, 3 id) 3, 4
80. Consider the following statements :
Star-delta starter is used in 3-pliase IM because it
1.mitigates (lessens) heating of the motor winding
2 .ensures permissible minimum starting current
3 .is regulated by electricity authority
4. ensures smooth run up to full load
Of these statements
(a) 1, 2 and 3 are correct 0) 2, 3 and 4 are correct
(c) 1,3 and 4 are correct id) 1 and 2 are correct
81. As compared to DOL starting, a cage IM with star-delta starting shall have
(a) more starting torque (b) more starting current
(c) reduced starting current (d) smooth acceleration
82. A 3-phase SCIM is started with a star-delta starter. The ratios
^ Line current at start with star-delta starter
- Line current at start with directswitching in delta
Starting torque with star-delta starter
'u Starting torque with directswitching in delta
are respectively,
1 .1 1 1
W 3 W .{ b) l 3 ' 3

m|, | «) f e , ic

83. Ifthe auto-transformer tapping K is less than 1, then the starting torque in a 3 -phase SCIM
. when started with this auto-transformer starter would he
(a) K x starting torque Test withdirect switching (6 )K2 x Tc s)

(c) ^ ' Te$t id) — •Tc . st


K
84. A starting torque of 100 N m is developed in a 3-phase SCIM by an auto-transformer starter with
a tapping 40%. Ifthe tapping ofthe auto-transformer ischanged to 80%, then the starting torque
would be
. (a) 400 N m (6 ) 200 N m (c) 50 N m (rf) 25 N m
85. The following starting method for a 3-phase SCIM is inferior in view of the poor stArtins torque
per ampere of the line-current drawn :
fcl S pH a« on line startmg (h) Auto-transformer method of switching
ic) Senes-mductor methodof starting (rf) star-delta starting

scanned Dy u a m ^c a n n e r
51. If two 8-pole dc machines of identical armatures are wound, one with lap winding and lhe ^
with wave winding, then rurrent and more voltage
(а) wave-wound machine will have more ratedc ^ mQre current
(б) lap-wound machine will have more rated vo g ^ ^ current
(c) lap-wound machine will have more ratedI volt g CUrrent
(rf) wave-wound machine will have more rated voltage aim
, machine
52. If the applied voltage to a dc i ■ is
■ 230
oun v then the back emf, tor maximum power devnin
V, ti elopcc|>

(a) 115 V (6) 200 V (c) 230 V «*) 460 V


53. A dc shunt generator has full-load voltage regulation o f 10%, at rated speed of 1000 rpm. Ifitj
now driven L 1250 rpm, then its voltage rcgu.atmn * nil load would ^
(а) be more than 10% be 12 5%
(c) remain unchanged
54. Two dc machines A and B, using the same conductor material, have armature circuit resistances
of 0.4 12 and 1.2 12 respectively. Of the two machines,
ta) A is bigger than B for the same current rating
(б) A is bigger than B for the same voltage rating
(c) B is bigger than A for the same current rating
(id) both are of the same size for the same current rating
55. A 10 kW, 230 V, dc shunt machine has four terminals brought out through four leads. For this
machine
(а) both the windings have thin leads
(б) both the windings have thick leads
(c) armature winding has thin leads whereas field winding has thick Je£ms
(rf) armature winding has thick leads whereas field winding has thin leads
56. There are two 2-pole dc machines. Main field axis of machine A is horizontal andthat of Bis
vertical. It is preferable to purchase
(a) machine A machineB
(c) any of the two machines (d) none of these
57. A compound dc generator is delivering full load current at a terminal voltage of230 V. Its
sengs-field winding gets short circuited. If its terminal voltage
1. becomes more than 230 V, it is overcompounded generator
2. becomes more than 230 V, it is differentially compounded generator
3. becomes less than230 V, itis overcompounded generator
4. becomes less than230 V, itis differentially compounded generator
5. becomes less than230 V, itis level compounded generator
From these, the correct answer is
(a) 1, 4, 5 (6) 2, 3, 5 (c) 1, 4 (d) 2, 3
58. DC generators are usually designed to develop armature voltages not exceeding 650 V because
of the limitations imposed by
(a) field winding (b) armature winding
(c) commutator (d) starters
59. In a dc series generator, the terminal voltage with increase in load will
(a) decrease
(b) increase gradually and then stay at rated voltage
(cj increase to rated voltage and then may decrease
(d) remain nearly constant
60. Consider the following statements : ^ muSt
For a level compounded dc generator to run at constant voltage, the series field m
effectively compensate
1. armature reaction mmf 2. armature resistance drop
3.. brush contact voltage drop

S canned by Cam Scanner


7- - - -J 2 ?
^hich of these statements is/arc correct ?
ta) 2 alone (6 ) 1 and 2 (c) 1 and 3 «/) 1,2 and 3
0 j; A soparutely-excited dc machine, having nn armature resistance of 2 ohms was working on a
220 V supply and drawing 10 A armature current from the source when the supply voltage
guddcnly dropped to 200 V. Assuming that the field circuit source voltage remained unaffected,
how will the armature current of tho machine react to the change V 1
(a)it will initially rise to 11 A and then settle down to 10 A
(b, Itwill fall momentarily to 9.09 A und then slowly attain 10 A
fCjIt will reduce to 7.ero first and then settle back to 10 A
(£i) Itwill remain unaffected by the change und continue to be 10 A [I.A.S., 1094J
§2. U n d er which ofthe following conditions isa dc motor provided with compensating winding used ?
1,Wide range of speed control above normal
2 ,Wide range of steady load variation with no speed control
3 ,Wide range of rapid variation in load
Select the correct answer using the codes given below :
Codes:
(a) 1,2 and 3 (b) 2 and 3 (c) 1 and 2 «/) 1 and 3 U.A.S., 7.9931
63. The following statements relate to compensating winding (CW) and interpole winding (/W) of a
dc machine :
1.C W neutralizes the armature reaction under a pole pitch
2. /W is connected in series with armature circuit
3 .IW produces mmf in the commutating zone only
4 .C W mmf is directed along the brush axis
5.Mmf produced by /W is equul to armature mmf
From these, the correct statements are
(a) 1, 2, 3, 4 and 5 (b) 1, 3, 4, 5
(c) 2, 3, 4, 5 (d) 2, 3, 4
64. Ifthe speed of a dc motor increases with load torque, then it is a
(a) series motor {b) permanent magnet motor
(c) differentially compounded motor [d) comulatively compounded motor
65. Ifthe field of a dc shunt motor gets opened while the motor is running, then the
(a) speed of motor will be reduced
(b) motor will attain dangerously high speed
(c) armature current will drop
(d) armature will oscillate about original speed as the mean speed
66.A dc cumulatively compounded motor delivers rated load torque at rated speed. Ifthe series field
is short-circuited, then the armature current and speed will
(a) both increase {b) both decease
(c) increase and decrease respectively (d) decrease and increase respectively.
[I.A.S., 1094]
67. A cumulatively compounded dc motor delivers rated load power at rated speed. Ifthe series field
isshort-circuited, then
(а) both armature current Ia and speed (0m increase
(б)Ia remains constant but com increase
(c) Ia increases and wm decreases
{d) both decrease
68.A 240 V dc series motor takes 40 A when giving its rated output at 1500 rpm. Us resistance is
0.3 £1.The value of resistance which must be added to obtain rated torque at 1000 rpm is
(a) 6 Q (b)5.7 £1 (c) 2.2 O (d) 1.9 £1
69. A dc shunt motor is running at 1200 rpm when excited with 220 V dc. Neglecting the losses and
saturation, the speed of the motor when connected to a 175 V dc supply is
(a) 750 rpm (b) 900 rpm (c) 1050 rpm (d) 1200 rpm

Scanned by Cam Scanner

Vous aimerez peut-être aussi